You are on page 1of 966

2012_ERM_Urology_Cover_v03_Urology Comp 7/3/12 1:27 PM Page 1

The American Urological Association


Educational Review Manual
in Urology
4th Edition • 2012

4th Edition Editors


2012
Daniel A. Shoskes, MD
Professor of Surgery, Cleveland Clinic Lerner College of Medicine

in Urology
Educational Review Manual
The American Urological Association
Director, The Novick Center for Clinical & Translational Research
Glickman Urological and Kidney Institute
The Cleveland Clinic
Cleveland, OH
Robert G. Uzzo, MD
Professor & Chairman Department ofUrology
Fox Chase Cancer Center
Philadelphia, PA

Robert G. Uzzo, MD
Daniel A. Shoskes, MD
2012_ERM_Urology_Cover_v03_Urology Comp 7/3/12 1:27 PM Page 2

SELF
SELF-STUDY
-S TUDY CME THE AMERICAN UR
UROLOGICAL
OL OGICAL AS
ASSOCIATION
S OCIATION

FROM
FROM THE AUA

The Update Series The JU Home Study Course


For the timeliest topics and the latest surgical Make the most of your Journal®
Journnal subscription! At Your
Your rresource
esource for
for urologic
urologic education.
education.
techniques in urology! The Updaate Series is the just $60 per yearr, The JU Home Study
Stud Course is the
AUA's most-popular self-study CME program. AUA's mostt afffor
ordable self-study CME program,
and is only available to subscribers of The
The JJournal
ournal
of Ur
of Urolog
ologgy
gy®!

2012
SASP
AAUA
UA Self-Assessment Study Program
Program

The Self-As
Assessment Visit
sit www
www.AUAnet.org
w..AUAnet.org
Study Program (SASP)
Preparing ffor
or an ABU
A examinaation? The SASP
SA is the for
for moree inf
o mor information.
forma
or tion.
orma Visit www
www.AUAnet.org
w..AUAnet.org to
to learn
learn more.
more.
AUA's best selling study tool ffor
or exam preparation.
2012_ERM_Urology_Cover_v03_Urology Comp 7/3/12 1:27 PM Page 2

SELF
SELF-STUDY
-S TUDY CME THE AMERICAN UR
UROLOGICAL
OL OGICAL AS
ASSOCIATION
S OCIATION

FROM
FROM THE AUA

The Update Series The JU Home Study Course


For the timeliest topics and the latest surgical Make the most of your Journal®
Journnal subscription! At Your
Your rresource
esource for
for urologic
urologic education.
education.
techniques in urology! The Updaate Series is the just $60 per yearr, The JU Home Study
Stud Course is the
AUA's most-popular self-study CME program. AUA's mostt afffor
ordable self-study CME program,
and is only available to subscribers of The
The JJournal
ournal
of Ur
of Urolog
ologgy
gy®!

2012
SASP
AAUA
UA Self-Assessment Study Program
Program

The Self-As
Assessment Visit
sit www
www.AUAnet.org
w..AUAnet.org
Study Program (SASP)
Preparing ffor
or an ABU
A examinaation? The SASP
SA is the for
for moree inf
o mor information.
forma
or tion.
orma Visit www
www.AUAnet.org
w..AUAnet.org to
to learn
learn more.
more.
AUA's best selling study tool ffor
or exam preparation.
The American
Urological Association
Educational Review
Manual in Urology
Fourth Edition • 2012

Editors-in-Chief

Daniel A. Shoskes, MD
Professor of Surgery, Cleveland Clinic Lerner College of Medicine
Director, The Novick Center for Clinical &Translational Research
Glickman Urological and Kidney Institute
The Cleveland Clinic
Cleveland, OH

Robert G. Uzzo, MD
Professor and Chairman, Department of Surgery
Temple University School of Medicine
G. Willing “Wing” Pepper Chair in Cancer Research
Fox Chase Cancer Center
Philadelphia, PA
Castle Connolly Graduate Medical Publishing, Ltd.
42 West 24th Street Flr 2 • NY NY 10010 • Tel: 212.644.9696 • Fax: 212.202.4972 • www.ccgmp.com • e-mail: info@ccgmp.com

Castle Connolly Graduate Medical Publishing, Ltd. publishes review manuals


to assist residents and fellows in preparing for board certification exams, and practicing
physicians in preparing for board recertification.

NOTICE

This book is intended for use by licensed Physicians, Nurse Practitioners, and Physician Assistants only. It is
not intended for use in the delivery of health care services, and cannot replace sound clinical judgment or indi-
vidualized patient care for such purposes.

The authors, publishers, distributors, and sponsors of this book expressly disclaim all warranties, express
or implied, with respect to this book and its contents, including any warranties as to the quality, accuracy,
completeness or suitability of the information contained in this book for any particular purpose. Without
limiting the foregoing, they caution the reader that use of this book does not guarantee passage of any
board certification exams, written or otherwise.

The authors, publishers, distributors and sponsors of this book expressly disclaim any liability to any per-
son or organization for any loss or damage caused by any errors or omissions in this book.

The content and images in this review manual and the interactive presentations are protected by U.S.
copyright and may not be reproduced or transmitted in any manner or medium without the advance writ-
ten permission of the copyright holders, The American Urological Association (“AUA”) and Castle
Connolly Graduate Medical Publishing, Ltd. (“CCGMP”).

The content and images in this review manual and the interactive presentations may not be uploaded to the
Internet or any intranet under any circumstances. The AUA and CCGMP will continuously monitor the web,
will pursue violations of these proscriptions, and will prosecute violators to the fullest extent of the law.

ISBN: 978-09707305-9-6

Our thanks to Copy Editor Sarah Herndon

© 2012. All Rights Reserved. Castle Connolly Graduate Medical Publishing, Ltd. and The American Urological Association
Contents

1. Applied Anatomy of the 13. Medical Management of Urolithiasis .....379


Genitourinary Tract ............................................1 Michael E. Lipkin, MD
Howard B. Goldman, MD, FACS Agnes J. Wang, MD
Glenn M. Preminger, MD
2. Pediatric Urology...............................................25
Hiep Thieu Nguyen, MD 14. Surgical Management of Stones ..............413
Hassan Razvi, MD
3. Pediatric Urological Oncology.....................57 Andrew Fuller, MBBS
Martin A. Koyle, MD
15. Renal Parenchymal and Upper Urinary
4. Genital Abnormalities .....................................73 Tract Urothelial Neoplasms ........................437
Warren T. Snodgrass, MD Robert G. Uzzo, MD

5. Essentials of Uroradiology............................95 16. Prostate Cancer...............................................479


Fergus V. Coakley, MD Judd W. Moul, MD
Rosaleen Parsons, M.D. Andrew J. Armstrong, MD
Joseph Lattanzi, MD
6. Cytology of the Urogenital Tract...............189
Donna E. Hansel, MD, PhD 17. Non-Muscle Invasive
Bladder Cancer ................................................519
7. Renal Physiology and Michael A. O’Donnell, MD
Pathophysiology. .............................................245
Daniel A. Shoskes, MD 18. Bladder Cancer ................................................549
Seth P. Lerner, MD
8. Renovascular Disease ..................................267
Daniel A. Shoskes, MD 19. Penile and Urethral Cancer ........................583
Kenneth W. Angermeier, MD
9. Renal Transplantation...................................285
Daniel A. Shoskes, MD 20. Testicular Cancer...........................................611
Lucas R.Wiegand,MD
10. Urodynamics .....................................................305 Jose J. Correa, MD
Victor W. Nitti, MD Wade J. Sexton, MD

11. Neuropathic Bladder: Voiding 21. Benign Prostatic Hyperplasia and


Dysfunctions Associated With Bladder Calculi.................................................661
Neurological Disease ....................................325
James C. Ulchaker, MD
Raymond R. Rackley, MD
Alexis E. Te, MD
12. Female Urology and
Urinary Incontinence.....................................347 22. Surgical Disease of the
Adrenal Gland ..................................................683
J. Christian Winters, MD
Revised 2012 by Harriette Scarpero, MD Jay T. Bishoff, MD
23a. Sexually Transmitted Diseases .................713 Companion DVD-ROM Table of Contents
Deborah R. Erickson, MD 1. Full book text

23b. Urinary Tract Infections ................................775 2. AUA Guidelines


John N. Krieger, MD 3. AUA Best Practices
4. Uroanatomy Slides-
24. Renal and Ureteral Trauma .........................795
Howard Goldman, MD
Allen F. Morey, MD

25. Bladder, Urethra and Genital Trauma .....809


Richard A. Santucci, MD

26. Physiology and Complications


of Laparoscopy .................................................843
Jeffrey A. Cadeddu, MD

27. Evaluation and Treatment of


Male Factor Infertility ....................................857
Paul J. Turek, MD

28. Erectile Dysfunction,


Peyronie’s Disease and Priapism .............885
Mikkel Fode, MD
Susanne A. Quallich, BSN, CUNP
Jens Sonksen, MD
Dana A. Ohl, MD

29. Interstitial Cystitis/Bladder Pain


Syndrome (IC/PBS)......................................923
Michel Pontari, MD

30. Pediatric Urological Incontinence


and Voiding Dysfunction............................935
Martin A. Koyle, MD
Faculty Disclosures

According to the American Urological Association’s Bishoff, Jay Todd: Pfizer: Scientific Study or Trial
(AUA) Disclosure policy, speakers and authors
involved in continuing medical education activities Cadeddu, Jeffrey Anthony: Ethicon Endosurgery,
are required to complete an online AUA disclosure. Inc: Consultant or Advisor, Scientific Study or Trial,
Although the American Urological Association Owner, Product Development; Applied Medical:
Educational Review Manual in Urology is not a con- Meeting Participant or Lecturer
tinuing medical education publication, we have
asked the contributing authors to disclose. This Erickson, Deborah R.: Trillium Therapeutics Inc.:
information is presented to the reader or participant Consultant or Advisor
so that they may make their own judgments about a
speaker’s presentation or an author’s chapter. Goldman, Howard B.: Johnson and Johnson:
Consultant or Advisor, Meeting Participant or
The AUA does not view the existence of these inter- Lecturer ; American Medical Systems: Consultant
ests or relationships as necessarily implying bias or or Advisor ; Allergan: Consultant or Advisor,
decreasing the value of the presentation or chapter. Meeting Participant or Lecturer ; Pfizer: Consultant
Prior to planning the program, directors review all or Advisor, Meeting Participant or Lecturer ;
speaker conflicts and implement a variety of mecha- Astellas: Meeting Participant or Lecturer ; TDoc:
nisms to resolve. Consultant or Advisor

Each faculty member presenting a chapter has sub- Hansel, Donna E.: Nothing to disclose
mitted a copy of his/her online disclosure to the
AUA. These copies are on file in the AUA Office of Koyle, Martin A.: Nothing to disclose
Education.
Krieger, John N.: I own a number of mutual funds
This review manual has been planned to be well bal- that invest in health care companies. I have no direct
anced, objective, and scientifically rigorous. control of specific decisions.
Information and opinions offered by the authors
represented their viewpoints. Conclusion drawn by Lerner, Seth Paul: Imalux: Consultant or Advisor,
the readers should be derived from careful consider- Scientific Study or Trial ; Celek Pharmaceuticals,
ation of all available scientific information. LLC: Consultant or Advisor, Scientific Study or
Trial ; GE Healthcare: Consultant or Advisor ; Endo
The following faculty members declare a relation- Pharmaceutical: Scientific Study or Trial ; Celgene:
ship with the commercial concerns listed below, Scientific Study or Trial ; Cephalon: Scientific
related directly or indirectly to this program. Study or Trial ; Tengion: Consultant or Advisor,
Readers may form their own judgments about the Scientific Study or Trial ; Dendreon: Consultant or
chapters in light of full disclosure of the facts. Advisor ; Photocure: Consultant or Advisor

Angermeier, Kenneth Wayne: American Medical Morey, Allen F.: AMS: Consultant or Advisor,
Systems: Consultant or Advisor, Meeting Meeting Participant or Lecturer ; Glaxo Smith
Participant or Lecturer Kline: Meeting Participant or Lecturer ; Coloplast:
Meeting Participant or Lecturer
Moul, Judd W.: Astra Zeneca: Scientific Study or Preminger, Glenn M.: Nothing to disclose
Trial ; Sanofi-Aventis: Health Publishing, Meeting
Participant or Lecturer ; Dendreon: Consultant or Razvi, Hassan: Allergan: Scientific Study or Trial ;
Advisor, Meeting Participant or Lecturer ; Kinsey Nash Corporation: Scientific Study or Trial
Theralogix: Consultant or Advisor ; Ferring ; Cook Urological: Owner, Product Development
Pharmaceuticals Inc: Consultant or Advisor,
Meeting Participant or Lecturer ; Amgen: Meeting Santucci, Richard A.: Nothing to disclose
Participant or Lecturer ; Medivation-Astellas:
Consultant or Advisor ; Janssen- J and J: Consultant Scarpero, Harriette Miles: Pfizer, Inc: Scientific
or Advisor, Meeting Participant or Lecturer Study or Trial ; American Medical Systems (AMS):
Consultant or Advisor ; Allergan: Consultant or
Nguyen, Hiep Thieu: Nothing to disclose Advisor, Meeting Participant or Lecturer ; Warner
Chilcott: Meeting Participant or Lecturer
Nitti, Victor William: Astellas: Health Publishing,
Consultant or Advisor, Scientific Study or Trial ; Sexton, Wade Jeffers: Endo Pharmaceuticals:
Allergan: Health Publishing, Consultant or Consultant or Advisor, Meeting Participant or
Advisor, Meeting Participant or Lecturer, Scientific Lecturer
Study or Trial ; Medtronic: Consultant or Advisor ;
Coloplast: Health Publishing, Consultant or Shoskes, Daniel Arthur: Triurol: Investment inter-
Advisor, Scientific Study or Trial ; Serenity est ; Farr Labs: Consultant or Advisor ; Astellas:
Pharmaceuticals: Consultant or Advisor, Consultant or Advisor
Investment Interest ; Uroplasty: Consultant or
Advisor ; American Medical Systems: Health Snodgrass, Warren Thomas: Nothing to disclose
Publishing, Consultant or Advisor, Scientific Study
or Trial Turek, Paul Jacob: MandalMed: Leadership
Position ; Medical Board of California: Consultant
O'Donnell, Michael A.: Viventia: Consultant or or Advisor ; HealthLoop.com: Consultant or
Advisor ; Spectrum Pharmaceuticals: Consultant or Advisor, Investment Interest ; Doximity, Inc:
Advisor ; Cytologics: Consultant or Advisor ; Consultant or Advisor, Investment Interest ;
Medical Enterprises: Consultant or Advisor ; Fertility Planit, Inc: Consultant or Advisor,
Sanofi-Aventis: Consultant or Advisor ; GE Investment Interest ; BioQuiddity, Inc: Consultant
Medical: Consultant or Advisor ; Photocure: or Advisor, Investment Interest
Consultant or Advisor ; Allergan: Consultant or
Advisor ; Endo Pharmaceuticals: Consultant or Ulchaker, James C.: Astellas: Consultant or
Advisor Advisor, Meeting Participant or Lecturer ; AMS:
Scientific Study or Trial ; Pfizer: Meeting
Ohl, Dana Alan: Coloplast: Consultant or Advisor, Participant or Lecturer ; GSK: Meeting Participant
Scientific Study or Trial ; Abbott: Meeting or Lecturer ; Fortec Medical: Consultant or Advisor
Participant or Lecturer ; Endo: Consultant or ; Urologix: Consultant or Advisor ; Metalase:
Advisor, Meeting Participant or Lecturer ; Eli Lilly: Investment Interest, Owner, Product Development
Consultant or Advisor, Meeting Participant or
Lecturer ; Pfizer: Meeting Participant or Lecturer Uzzo, Robert G.: Pfizer: Meeting Participant or
Lecturer ; Wilex: Scientific Study or Trial
Pontari, Michel Arthur: Pfizer: Scientific Study or
Trial ; Axcam Pharmaceutical: Consultant or
Advisor ; Lilly: Consultant or Advisor
Preface
Fourth Edition

Dear Colleague,

We are delighted to present to you the fourth edition of the Educational


Review Manual in Urology to be distributed as an additional educational
resource at the AUA Annual Review Course. While this manual is intended
to focus on the salient, practical material needed for test preparation, we
hope it may also be of value in your clinical practice. New in this edition are
both updated content and additional questions.

We applaud the heroic efforts of our authors, all of whom are nationally
recognized experts in their topic areas, who have updated their chapters or
written new ones. Some of the text duplicates information in the lectures
but much is additional and supplementary.

Many thanks once again to Michael D. Wolf, Ph.D. and his staff at Castle
Connolly Graduate Medical Publishing for their remarkable efforts in
bringing this project to completion.

We continue to look forward to your feedback. Authors were free to use


textbook style or point form outline, and we are eager to learn which you
found easier to study from. Best of luck in your exams and in all your
future endeavors.

Sincerely,

DanielA.Shoskes,MD
Glickman Urological and Kidney Institute
The Cleveland Clinic

RobertG.Uzzo,MD
Temple University School of Medicine
Fox Chase Cancer Center
Philadelphia, PA

© 2012. All Rights Reserved.


Castle Connolly Graduate Medical Publishing, Ltd. New York, NY and
The American Urological Association.
The American Urological Association Annual Review Course was established
with one simple mission: To provide the most up-to-date information regarding the
practice of Urology on an annual basis. The Course was originally designed to
provide a thorough review of pertinent urologic information for Chief Residents
who were about to take their written American Board of Urology Examination.
Since that time, the Course has grown to include a comprehensive review for those
Urologists who are participating in the Maintenance of Certification process and
also for those practicing Urologists who want a
general review in the field.

Now accommodating over 500 participants per year, this Annual Review Course
has become one of the most popular and perhaps, most important offerings from
the AUA Office of Education. By assembling the best and the brightest to present
this message, Co-Directors Dan Shoskes and Robert Uzzo have made this the pre-
mier Review Course in Urology. Moreover, they have added urologic pathology
and radiology modules, as well as an expansion of the course
syllabus, to this already comprehensive offering.

This volume was envisioned as an additional study aid to those preparing for the
American Board of Urology examinations, but it can also be used as a general
Review of the entire field of Urology. I believe that this mission has been accom-
plished with the production of the AUA EDUCATIONAL REVIEW MANUAL IN
UROLOGY. The fourth edition of the Review Manual, edited by Dan Shoskes and
Robert Uzzo, includes updated basic science and clinical information for all
Urologists.

Finally, thanks to Michael D. Wolf, Ph.D. and his excellent staff at Castle Connolly
Graduate Medical Publishing, Ltd., for their assistance in the effective design,
implementation and publication of this compendium.

The AUA Office of Education is proud to support this valuable endeavor.

ElspethM.McDougall, MD, FRCSC, MHPE


Chair, AUA Officer of Education
American Urological Association (AUA)
Educational Programs and Services

CME Educational Offerings offers regional seminars on coding and a two day
practice management symposium at the AUA
AUA Annual Meeting Annual Meeting each year. Visit the urology calen-
The American Urological Association’s Annual dar at www.AUAnet.org/calendar for details.
Meeting is the world’s largest meeting of urologic
professionals. With more than 10,000 urologists The Journal of Urology® Home Study Course
and health care professionals and more than 5,000 Earn credits while you read The Journal of
representatives from more than 350 companies in Urology® with the JU Home Study Course.
attendance, there is no better place to learn about Designed by a team of educators, academicians
the advances in urology. and clinicians, this program is the AUA’s most
convenient and affordable self-study program and
Year-round Subject-oriented and Surgical allows physicians to immediately test their knowl-
Learning Courses edge and apply this knowledge to improve the
The AUA offers many Subject-oriented and quality of patient care. This activity is available
Surgical Learning Center courses on a variety of only to subscribers of The Journal of Urology®.
topics essential to the urologist. Visit Journal subscribers will receive 12 individual five-
www.AUAnet.org/calendar to view a complete list question answer sheets or they can go on line to
of upcoming courses. receive instant credit. This activity is available to
JU subscribers for just $60 per year. Visit
The AUA Update Series www.auanet.org to order.
The AUA Update Series is the most professional
self-study product in urology. For more than 30 Self-Assessment Study Program
years, this invaluable tool continues the tradition The Self-Assessment Study Program (SASP) has
of providing up-to-date information that's relevant been a leading educational resource for practicing
to you. Improve your practice and patient care by urologists and urology residents since 1977. Each
staying abreast of the latest in treatments and sur- year, a new 150 multiple choice item examination
gical techniques. The AUA Update Series includes is developed and includes discussions and refer-
40 new lessons with content-related, five-question ences from current literature in clinical areas of
posttests. For CME credit, subscribers must interest to urologists. Sharpen your clinical skills
receive a minimum score of 80 percent for the post or prepare for an examination with this study pro-
tests. The AUA Update Series is available in print gram. The SASP is an excellent way to prepare for
and/or online formats (with four new lessons the American Board of Urology (ABU) Re-certifi-
posted as they are published for 10 months).This cation examination because 15-20 percent of the
innovative CME program helps urologists meet content for the exam comes from this product.
the core curriculum standards and CME required
by states and the American Board of Urology The SASP is now available on your smart phone
(ABU).Visit www.AUAnet.org/UpdateSeries for via the new, innovative Spaced Education format
more information. — the online platform that provides an easy, effec-
tive and enjoyable method to learn and retain urol-
Coding and Practice Management Seminars ogy content. CME earned from participation in this
By possessing the most up-to-date information on program will be recognized by the ABU in fulfill-
coding and practice management, urologists and ing MOC CME requirements. Visit
their staff members will manage their practices http://www.auanet.org/content/products/self-
more effectively, spend less time on paperwork assessment-study-program.cfm for more details.
and more time devoted to patient care. The AUA
AUA Online Education – Urologic education at at (as well as beyond) the bedside, as well as foster
your fingertips! an appreciation of the moral history of the profes-
The convenience, flexibility and power of Internet sion and of the current state of debate about a
technology combined with content developed by range of ethical issues--from assisted suicide and
expert clinicians, urologists and academia, pro- euthanasia to medical futility. In 2011, module 18
vide a learning environment that fosters life-long – expert witness, developed by the AUA Judicial
learning, maintenance of certification and premier and Ethics Committee, was added to educate urol-
patient care. ogists in providing quality expert witness testi-
http://www.auanet.org/content/education-and- mony regarding standard of care. Clinical Ethics
meetings/on-line-education.cfm for Urologists is free to AUA members and uro-
logic residents, fellows and trainees.
Urologic Robotic Surgery Online Course – Free http://www.auanet.org/content/education-and-
The AUAER Urologic Robotic Surgery Online meetings/on-line-education.cfm
Course was developed by the AUAER
Laparoscopic, Robotic Surgery and New Surgical The above activities have been approved for AMA
Technology Committee to address the need for a PRA Category 1 Credits™
national course/curriculum in urologic robotic
surgery. The online Urologic Robotic Surgery Non - CME Educational Offerings
Course consists of 9 modules covering topics in
Urologic Robotic Surgery including Urology Core Curriculum
Fundamentals, Basic and Advanced Surgical The AUA's Urology Core Curriculum is the most
Procedures. The Fundamentals module covers comprehensive reference guide available detailing
proper utilization of the daVinci robot in urologic the knowledge necessary to deliver quality uro-
surgery. Intuitive Surgical's Online Training logical care. Divided into more than 50 special-
System complements the Fundamentals section ized sections, the Core Curriculum covers the
and is a pre-requisite for the course. The Basic sec- depth and breadth of urology. Its on-line reference
tion includes transperitoneal radical prostatec- guide links to the most up-to-date and scientific
tomy, partial nephrectomy, pyeloplasty, and resources available, including clinical guidelines,
sacrocolpopexy surgical procedures. Advanced textbooks, scientific articles, videos, simulations,
procedures include LESS and microsurgery, interactive websites and more.
retroperitoneal radical prostatectomy, pediatrics,
and radical cystectomy surgeries. Urologic Each urological topic is divided into subject areas,
Robotic Surgery Course is free to AUA members and provides section objectives, learner objec-
and urologic residents, fellows and trainees. tives, related content, prerequisites, and refer-
http://www.auanet.org/content/education-and- ences. In outline format and through links to the
meetings/on-line-education.cfm latest scientific and clinical resources, each sec-
tion summarizes the minimum core knowledge a
CLINICAL ETHICS FOR UROLOGISTS resident should master about each subject area
Free, New Expert Witness Module! during residency.
Reformatted and updated, this online curriculum
introduces and challenges you to appreciate more All AUA members have free access to the AUA
fully and therefore, respond more effectively to Urology Core Curriculum. As a result of AUA
the ethical dimensions of medical practice in urol- membership, members will also have free access
ogy. The original 17-module Ethics curriculum, to references from The Journal of Urology® and
released in 2008, was developed in association other leading urology publications.
with the Society of University Urologists and
Society of Urology Chairs and Program Directors.
The instructional modules cover the legal aspects
of such topics as informed consent and the various
approaches to thinking through ethical dilemmas
Medical Journal • Allied – Nurse practitioners, physician assis-
tants, nurses, technicians and medical assistants.
The Journal of Urology®
The most widely read publication in the field, The For more information or to complete an applica-
Journal of Urology® brings solid coverage—all tion, visit www.AUAnet.org/join.
the clinically relevant information needed to stay
at the forefront of this dynamic field. This top- News and Information
ranking journal in the urology field presents inves-
tigative studies on critical areas of research and AUAnet.org
practice; survey articles providing short condensa- AUAnet.org is the AUA’s web site – designed for
tions of the best and most important urology liter- members and residents searching for information
ature worldwide; and practice-oriented reports on on our educational programs and services. The site
interesting clinical observations. This is the also features clinical guidelines, code of ethics,
Official Journal of the American Urological policy statements and membership information.
Association. Visit www.jurology.com for more
details. AUANews®
AUANews®, the Official News magazine of the
Membership American Urological Association, is written for
urologists by urologists and is provided to AUA
AUA membership offers unique benefits and members as a benefit. It features current, highly
experiences that enhance not only your career, but relevant, cutting-edge news of practical value to
also your personal growth. The AUA works to rep- urologists, while also serving as a vehicle for
resent its members on vital issues that impact your prompt and current communication between the
practice and your ability to offer the best possible AUA and its members. Visit
urologic care. In addition, the AUA provides you www.AUAnet.org/AUA News to view current
with unparalleled information and educational issues or visit www.AUAnet.org/catalog to order.
resources in a variety of formats. As the largest
Career Services
and most prestigious organization of urology pro-
AUA Jobfinder is the online source for careers in
fessionals in the world – the AUA has something
urology. This is a valuable and comprehensive
to offer you. Together We Are Urology – Join Us!
career networking service, and is available to both
employers and job seekers in the urological com-
The AUA offers the following categories of mem-
munity. Visit www.AUAnet.org/JobFinder for
bership:
more details.
• Active/Associate – Urologists and Doctors of Clinical Guidelines
Osteopathy practicing in the U.S., Canada, AUA evidence-based practice guidelines are
Mexico or Central America. developed through a partnership between guide-
lines department staff and clinician panel mem-
• Candidate – Residents and Osteopathy residents bers using an efficient nine-stage process. The
enrolled in a urology training program in the completed guidelines, as well as patient guides
U.S., Canada, Mexico or Central America. and webinars, are accessible to members and to
the public online at the AUA, National Guidelines
• International – Urologists practicing outside of Clearinghouse, and Guidelines International
the U.S., Canada, Mexico or Central America. Network websites. New guidelines are also pre-
sented at the AUA Annual Meeting plenary ses-
• International Residents-in-Training – Residents sions by the guidelines panel chairs. Executive
enrolled in a urology residency training pro- summaries of the guidelines are published in the
gram outside of the U.S., Canada, Mexico or Journal of Urology, and key findings are pub-
Central America. lished in other AUA publications including AUA
News and The Health Policy Brief. In addition, a
Guidelines at a Glance pocket guide, containing a Services
collection of summarized AUA guidelines, is
developed to serve as a quick reference tool for AUA Coding Hotline
practicing urologists. This product is available in Consider us a trusted advisor to your coding staff.
print, online PDF and Smartphone app format. An AUA coding specialists can be called on to answer
adaptation to the Guidelines at a Glance is also your toughest coding questions. Just dial 1-866-
developed for primary care practitioners, focusing RING-AUA (1-866-746-4282) and select option 2
on guidelines of interest to such an audience. or email codinghotline@auanet.org to receive
Webinars, wall charts, tri-folds and other dissemi- prompt answers to your tricky coding questions
nation products are developed for a number of from the certified professional coders on our staff.
guidelines to facilitate easy access to guideline Each call is $25, but unlimited calls are free to
statements and algorithms. AUA is currently in the subscribers to the AUA Practice Managers’
process of developing video and audio products to Network (PMN). See details below.
provide additional multimedia content to dissemi-
AUA CME Credit Registry
nate new guideline findings. AUA members can
The AUA’s free, online, self-reporting AUA CME
access an online topic submission form to nomi-
Credit Registry is a benefit for all AUA members,
nate new ideas for guideline development.
allowing you to self-report and track your AMA
Additional information can be found at
PRA Category 1 Credits™ online and print a CME
www.AUAnet.org/Guidelines.
transcript anytime. The AUA CME Credit
Coding Tips for the Urologist’s Office Registry offers an economical, fast and conve-
Important information that urologists, practice nient way to keep track of their Maintenance of
managers and coders need to know about urology Certification credits.
coding is featured online – from coding basics to
Practice Managers’ Network (PMN)
the Medicare fee schedule. Coding tips are fea-
Whether you are a solo practitioner running your
tured on our Web site at
own business or you employ a professional
www.AUAnet.org/CodingTips.
administrator, the challenges of running a urology
Health Policy Brief practice today requires access to expert advice and
The Health Policy Brief is an e-newsletter cover- updates on the ever-changing healthcare environ-
ing the latest topics in national and state legisla- ment. This network of urologists and practice
tion, clinical practice guidelines, emerging coding managers is a fee-based subscription program to
and reimbursement issues with both public and which every urology practice should belong. Visit
private insurers, quality measurement and www.AUAnet.org for more details.
improvement, regulation—with a focus on the
Medicare program, patient safety, and practice
management. This publication keeps urologists
abreast of issues vital to the everyday practice of
medicine and informs and engages them in the
policy arena at the local, state and national levels.
To receive this publication, call 1-866-RING-
AUA (1-866-746-4282) or read them at
www.AUAnet.org/HPBrief.
Chapter 1:
Applied Anatomy of the
Genitourinary Tract
Howard B. Goldman, MD, FACS

Acknowledgement
Sarah McAchran, MD for reviewing and making suggestions to the manuscript

Contents

Supporting Structures of the


Genitourinary System

1. Skeletal Structures

2. Muscles

3. Vasculature

4. Neuroanatomy
See also the presentation on
the Companion DVD-ROM 5. Inguinal Canal and Femoral Triangle
Urologic Organs

6. Adrenal Gland

7. Kidney

8. Ureter

9. Bladder

10. Prostate

11. Seminal Vesicles and Vasa Deferentia

12. Urethra

13. Female Pelvic Floor

14. External Genitalia

15. Testis and Scrotum

16. Questions

CHAPTER 1: APPLIED ANATOMY OF THE GENITOURINARY TRACT 1


Introduction 1. Skeletal Structures

A thorough knowledge of genitourinary anatomy is Ribs


the foundation upon which we base our understand-
ing of the pathophysiology of urologic disease. Fur- A number of ribs—specifically ribs 6 through 10
thermore, it is the surgeon’s roadmap, illuminating which are connected to the sternum by a cartilagi-
the arena in which they practice every day. At some nous costal arch—are important to urologists. When
point during medical school or residency, we all making a thoracoabdominal incision, the costal arch
memorized general anatomy – at least for a short may be cut. Ribs 11 and 12 are not joined anteriorly
time. As our interest in urology deepened, many of and are sometimes referred to as the ”floating” ribs.
these anatomic principles were reviewed. It is these ribs that are intimately associated with the
various flank incisions that are made to access the
Oftentimes, however, the integration of anatomic retroperitoneum. Each rib has a costal groove on the
knowledge regarding the genitourinary organs, the inner inferior surface within which the intercostal
supporting and surrounding structures, and their vessels and nerve travel. When making a flank inci-
physiologic function does not completely occur. sion, and especially when removing a portion of the
This review will focus on that integration by provid- rib, it is important to try to keep these structures
ing a generous overview of the genitourinary sup- intact to prevent abdominal wall or flank muscle
porting structures – specifically the bones, muscles, weakness. (Netter 186, 248)
vessels and nerves – and the urologic organs them-
selves. This review should be beneficial for those Vertebral Bodies
studying for exams as well as those interested in
refreshing their understanding of genitourinary A number of the vertebral bodies are significant as
anatomy. well. The thoracic 11th and 12th, the lumbar 1st
through 5th and the sacrococcygeal bones are par-
Anatomy requires illustration, and for that reason ticularly relevant to urologic anatomy. Most of the
this review is accompanied by references to the nerves that innervate the pelvis and urologic organs
Atlas of Human Anatomy (4th edition) by Frank travel through these vertebral bones. It is important
Netter and the Atlas of Urosurgical Anatomy by to recognize that the spinal nerves are named for the
Frank Hinman Jr. This chapter will be most benefi- vertebral body just above its foramen of exit and
cial if it is reviewed alongside a copy of Netter’s that while the spinal cord terminates at the lumbar
atlas. There are fewer references to Hinman’s, so if first and second level in the adult, the nerves con-
this book is not readily available that should not sig- tinue on as a structure called the cauda equina. For
nificantly impact the review. Ideally, it would be example, the S2 nerve leaves the sacral spinal cord
best to have all the figures reproduced alongside the just below the S2 vertebral body and then travels as
text; however, because of copyright issues and part of the cauda equina. Therefore, the area called
space, that is not possible. the sacral cord, which is the portion of the spinal
cord from which the spinal nerves emanate, is not
near the sacral vertebral bodies, but is more rostral,
at the termination of the cord at the L1 – L2 level.
Again, the important factor when naming the cord
level is the site of exit of the nerves emanating from
that portion of the cord. Similarly, the lumbar cord
runs through the T11 to L1 bodies, after which the
nerves that emanate from it continue on and exit
near the L1-L5 vertebral bodies. This becomes
important when classifying spinal cord injuries.
Though the actual fracture may be at the L1 level,
the spinal cord injury itself may be to the sacral
cord, since the sacral cord is at the L1 level. Simi-
larly, an injury to the T11 or T12 vertebral body can
produce a lumbar cord injury. (Netter 161)

2 EDUCATIONAL REVIEW MANUAL IN UROLOGY


Table 1 ogists as it is a thoroughfare for the passage of tro-
cars used in incontinence and prolapse surgery. The
Pelvic Foramina other pelvic foramina or openings within the pelvis
are formed by ligaments and bone. The sacrotuber-
Within bone ous ligament bridges the ischial tuberosity to the
1. Obturator foramen sacrum and coccyx, while the sacrospinous liga-
ment travels from the ischial spine to the sacrum.
Formed by ligaments Knowledge of the anatomy of the sacrospinous liga-
-Sacrotuberous (ischial tuberosity to ment is especially important for those utilizing it as
sacrum/coccyx) anchor during vaginal vault suspension procedures.
-Sacrospinous (ischial spine to lower sacrum) The greater sciatic foramen is bounded by bone and
the sacrospinous ligament inferiorly while the lesser
2. Greater sciatic foramen sciatic foramen is bounded by the sacrospinous liga-
ment superiorly, the sacrotuberous ligament inferi-
3. Lesser sciatic foramen orly and the ischium laterally. (Netter 353)

Pelvis

The bones that make up the pelvis help support and


protect many of the urologic organs. The innomi-
nates of the pelvis are 3 sets of paired bones: a right
and left ilium, right and left ischium, as well as right
and left pubic bones. Posteriorly, the sacrum and the
coccyx lay between the paired iliac bones and below
the L5 vertebral body. (Netter 248)

In the past, much of this anatomy was not clinically


relevant; however, neuromodulation techniques
which have recently entered the urologist’s arma-
mentarium utilize some of the foramina of the
sacrum and the knowledge of the sacral anatomy
has become important. For sacral nerve stimulation,
one must be able to identify the S3 foramen radio-
logically. When looking at a PA view, the S3 fora-
men sits at the level of a line drawn between the
lower edges of the sacroiliac joints approximately
1–2 cm from the midline bilaterally.

Within the pelvis there are a number of openings,


one of which is completely within and bordered by
pelvic skeletal structures and others which are
formed by portions of the bony structure as well as
various ligaments. The obturator foramen is
bounded anteriorly and anteromedially by portions
of the pubis, while posteriorly a portion of the
ischium surrounds it. The obturator vessels and
nerves travel through the superolateral portion of
the obturator foramen through the obturator canal.
The obturator foramen is clinically relevant to urol-

CHAPTER 1: APPLIED ANATOMY OF THE GENITOURINARY TRACT 3


2. Muscles

Subcutaneous Tissues Table 3

The abdominal wall is composed of muscle groups Abdominal Wall Musculature


covered by layers of subcutaneous tissue and skin.
Immediately beneath the skin is Camper’s fascia, Anterior
which is a layer of loose fatty tissue. Just deep to Rectus
Camper’s is Scarpa’s fascia, which is not real fascia Aponeurosis of oblique and transversus
but a thickening of tissue which superiorly and lat- muscles
erally blends with Camper’s fascia, inferiorly fuses
with the fascia of the thigh and medially is continu- Anterolateral
ous with Colle’s fascia. Colle’s fascia is continuous External oblique
with Scarpa’s superiorly and with the dartos fascia Internal oblique
overlying the penis and scrotum and runs to the pos- Transversus abdominis
terior edge of the urogenital diaphragm. (Netter Intercostals
380)
Posterior
Musculature Outer layer
- Latissimus dorsi
The abdominal wall musculature can be divided - External oblique
into 4 groups: anterior, anterolateral, posterior and
pelvic floor. Middle layer
- Internal oblique
- Serratus posterior inferior
Table 2 - Sacrospinalis

Skin and Subcutaneous Fascia Inner layer


-Quadratus lumborum
Camper’s fascia - Loose layer of fatty tissue -Iliacus
beneath skin
Innermost layer
Scarpa’s fascia - Deep to Camper’s -Psoas
- Superior/lateral―blends with -Diaphragm
Camper’s
- Inferior: fuses with fascia
of thigh
Anterior Abdominal Wall
- Medially: continuous
The most medial muscle which makes up part of the
with Colle’s
anterior abdominal wall is the rectus muscle, which
runs from ribs 5, 6 and 7 down to the pubic crest.
Colle’s fascia - Continuous with Scarpa’s
The other primary component of the anterior
and Dartos
abdominal wall is the fascial aponeurosis of the
- Till posterior edge of
external and internal oblique and transversus abdo-
UG diaphragm
minis muscles. There are anterior and posterior fas-
cial sheaths around the rectus from its origin in the
Dartos fascia - Superficial layer of penis and
upper abdominal wall to a point 1/3 of the way from
scrotum
the umbilicus to the pubis. Further than 1/3 of the
- Continuous with Colle’s
way down, the fascia is only anterior to the rectus
and Scarpa’s
muscles. This area of transition about 1/3 of the dis-
tance from the umbilicus to the pubis is called the
arcuate line. Rostral to the arcuate line, the external

4 EDUCATIONAL REVIEW MANUAL IN UROLOGY


and internal oblique fascia covers the rectus anteri- Layers Encountered During
orly, while a portion of the internal oblique and Surgical Incisions
transversus abdominis fascia cover it posteriorly.
Caudad to the arcuate line all of the various fascial When making a flank incision, many of these mus-
layers are anterior to the rectus muscle. (Netter 251) cle groups are encountered and incised. Posteriorly,
the latissimus dorsi may be incised and sometimes
Anterolateral Abdominal Wall portions of the serratus posterior inferior as well.
The anterolateral group of abdominal wall muscles Anterolaterally, the layers from out to in consist of
is made up of the external oblique, internal oblique, the external oblique, internal oblique, transversus
transversus abdominis and intercostal muscles. The abdominis layer (which is where one would find the
external oblique fibers run caudad and medially. rib), the transversalis fascia, thoracolumber fascia,
The free inferior border of the muscle is the inguinal the pararenal connective tissue, Gerota’s fascia, the
ligament and a portion of it continues as the external perirenal fat, and finally the kidney itself. The neu-
spermatic fascia. (Netter 249) The internal oblique rovascular bundle coming out of the intercostal
upper fibers run cranially, while the lower fibers run groove lies between the internal oblique and the
medially. Some of the lower fibers form part of the transversus abdominis. (Netter 237, Hinman 8.5b)
conjoined tendon and a portion continues as the cre-
master muscle. (Netter 250) The transversus abdo- When making an anterior subcostal or chevron inci-
minis fibers run medially and some of its lowest sion the various layers encountered include the
fibers form the conjoined tendon as well. The inter- skin, Camper’s fascia, external oblique, internal
costal muscles connect the ribs and are made up of oblique, rectus muscles, the transversus abdominis
both external and internal muscle layers. (Netter muscle, transversalis fascia, and finally the parietal
251) peritoneum. (Netter 252)

Posterior Abdominal Wall


The posterior abdominal wall muscles can be subdi- Table 4
vided into 4 layers: the outer, middle, inner and
innermost layers. The outer layer is made up of the Layers of a Flank Incision
latissimus dorsi—a large, triangular-shaped muscle
that covers most of the lower back—and the exter- Posteriorly -Latissimus dorsi
nal oblique. Petit’s triangle is an area bounded -Serratus posterior inferior
medially by the latissimus dorsi, laterally by the
Anterolaterally -External oblique
external oblique, and inferiorly by the iliac crest and
-Internal oblique
is an area with a relative dearth of musculature and Neurovascular bundle
has been used as an easy access route to the -Transversus abdominis (±rib)
retroperitoneum. (Netter 254) The middle layer of -Transversalis fascia
the posterior abdominal wall muscle group is made -Thoracolumber fascia
up of the internal oblique, the serratus posterior -Pararenal connective tissue fat
inferior and the sacrospinalis muscle. The inner -Gerota’s fascia
layer of the posterior abdominal wall is made up of -Perirenal fat
the quadratus lumborum muscle and the iliacus. The -Kidney
innermost layer is made up of the psoas and the
diaphragm. (Netter 254, 263) Layers of Anterior Subcostal or Chevron Incision

External oblique
Internal oblique and rectus
Transversus abdominis
Transversalis fascia
Parietal peritoneum

CHAPTER 1: APPLIED ANATOMY OF THE GENITOURINARY TRACT 5


Strata of Retroperitoneal Tissue which runs from the pubis to the ischial spine and is
readily palpable. For those more interested in the
The layers of retroperitoneal tissue are sometimes pelvic anatomy, there is actually the arcus tendineus
separated into different “strata” based on embry- fascia pelvis (ATFP) to which the various “fascial”
ologic origin. The tissues that are divided in this layers that are of most importance in female pelvic
way are the so-called “fascial” layers of the anatomy attach, as well as the arcus tendineus leva-
retroperitoneum. In truth, none of these are true fas- tor ani (ATLA) to which the levator muscles attach.
cia but instead are thin layers of tissue similar to The coccygeus muscle and the piriformis muscle
Scarpa’s fascia. The strata are divided into the outer, make up more of the posterior pelvic floor. On the
intermediate and inner layers. The important points inner surface of the obturator foramen is the obtura-
are as follows: tor internus muscle and more superficially, external
to the obturator foraman is the obturator externus
• The outer layer is made up of those “fascial” muscle. On the superolateral portion of the obtura-
layers, which invest the body wall muscles. tor foramen is the obturator canal through which the
Therefore, the endopelvic fascia and the obturator nerve and vessels travel. (Netter 356, 358)
transversalis fascia are part of the outer stra- Recently, the obturator foramen has become more
tum of the retroperitoneal tissue. important for urologists as a number of midurethral
sling and pelvic floor reconstructive procedures
• The components of the inner layer are those make use of the obturator foramen to pass various
tissues that invest the intestinal tract. For needles, trocars and mesh.
example, Denonvillier’s fascia is part of the
inner stratum. Perineum and Urogenital Diaphragm

• Finally, those layers that invest the urologic At first glance, there are obvious striking differ-
organs are part of the intermediate layer and ences between the structures of the male and female
thus Gerota’s fascia is part of the intermediate perineum. However, on closer inspection they are
stratum. really almost the same, the major difference being
that the proportions have changed. Most of the area
Pelvic Musculature of interest for urologists is within the urogenital tri-
angle, which is a triangle contained by the symph-
The pelvic musculature is made up of the psoas ysis pubis and the right and left ischial tuberosities.
muscle, which originates outside of the pelvis but
then passes through the pelvis, as well as the iliacus When evaluating the female perineum superficially,
which originates within the pelvis but likewise one can identify the superficial transverse perineal
passes through the pelvis and inserts on the proxi- muscles running from the ischial tuberosities to the
mal portion of the femur. Muscles that are entirely perineal body in the midline. The ischiocavernosus
contained within the pelvis include the obturator muscles run from the area of the ischial tuberosities
internus, the piriformis, the coccygeus, and the leva- towards the pubis, while the bulbospongiosus mus-
tor ani group—which is made up primarily of the cles run from the perineum towards the pubis pass-
puborectalis, pubococcygeus, and the iliococcygeus ing lateral to the vaginal wall. The perineal mem-
muscles. (Netter 263) The levator ani group pro- brane is the central point in the midline where many
vides support to the pelvic organs. The puborectalis of these muscles connect, and inferior to that is the
muscle originates from the pubis, is horseshoe- external anal sphincter. It is at this superficial level
shaped, wraps around the rectum and comes back to that the Bartholin’s glands lay at the inferior aspect
the pubic bone. The pubococcygeus originates lat- of the vagina. (Netter 379) In the male perineum we
eral to the puborectalis on the pubic bone and then find the same structures, differently proportioned.
travels to the coccyx where it ends. The iliococ- The superficial transverse perineal muscles and the
cygeus originates from the side wall of the pelvis external anal sphincter are relatively unchanged.
from the arcus tendineus and then runs to the coc- However, the ischiocavernosus muscles now wrap
cyx. The arcus tendineus is a condensation of tissue, around the corporal bodies while the bulbospongio-

6 EDUCATIONAL REVIEW MANUAL IN UROLOGY


3. Vasculature

sus muscles cover the corpus spongiosum and these The vascular anatomy will be divided into 3 sec-
muscles are part of the penis. (Netter 382) tions: the arterial, venous and lymphatic.

Deeper in the perineum is where the deep transverse Arterial Anatomy


perineal muscles lay as do the levator ani, and it is at
this level that the membranous urethra in the male The abdominal branches of the aorta as well as the
and what is sometimes referred to as the external pelvic arteries are of primary importance to the urol-
urethral sphincter in the female exist. In addition, ogist.
Cowper’s glands in the male are at this deep per-
ineal level. (Netter 383) The inferior phrenic arteries, which originate above
the celiac trunk, give off the superior suprarenal
(adrenal) arterial branches. This is one of the arterial
Table 5
supplies to the adrenal gland. The next branch of the
aorta is the celiac trunk, which gives off the com-
Pelvic Musculature
mon hepatic artery. The common hepatic artery sup-
plies the liver and gives off the gastroduodenal
Iliacus
branch, which gives off the right gastroepiploic
artery that supplies much of the stomach and omen-
Psoas
tum. For practical purposes, if one needs to utilize
an omental flap during surgery it is best to base the
Obturator internus
omentum on a pedicle originating from the right
gastroepiploic artery as that artery is of larger cal-
Piriformis
iber than the left gastroepiploic artery. Furthermore,
it originates more caudad and thus one is able to pull
Coccygeus
the omental flap further into the pelvis when the flap
is based on the right gastroepiploic artery. The

}
Levator ani - Puborectalis Arise from
splenic artery branches off of the celiac trunk and
Pubococcygeus arcus tendineus
gives off the left gastroepiploic artery, which sup-
Iliococcygeus and pubis
plies a portion of the stomach and omentum.
Finally, the left gastric artery comes from celiac
Table 6 trunk as well. Moving further along the aorta, the
middle suprarenal (adrenal) arteries come off the
Perineum aorta to supply the adrenal glands. In addition, along
the aorta at this point are 4 pairs of lumbar arteries
Superficial - Superficial transverse perineal which have the potential to cause troublesome
muscles bleeding, particularly during a retroperitoneal
- Ischiocavernosus muscles lymph node dissection. (Netter 264, 300)
- Bulbospongiosus muscles
- Perineal body The superior mesenteric artery arises from the ante-
- External anal sphincter rior portion of the aorta and supplies much of the
[Bartholin’s glands [F]) small and large bowel. The various details regarding
the arterial supply to the bowel are particularly
important when utilizing bowel segments for uri-
Deep - Deep transverse perineal muscles nary diversion. The superior mesenteric artery gives
- Levator ani rise to various intestinal arteries that supply the
- Perineal body jejunum and the ileum, as well as the ileocolic artery
- Membranous urethra (M)/ext. ure- that supplies the distal ileum, appendix and cecum,
thral sphincter (F) the right colic artery that supplies the ascending
(Cowper’s glands [M]) colon, and the middle colic artery that supplies the
transverse colon. Just beneath the take off of the

CHAPTER 1: APPLIED ANATOMY OF THE GENITOURINARY TRACT 7


superior mesenteric artery, the renal arteries leave bladder as well as the prostate. The internal puden-
the aorta bilaterally to supply the kidneys while the dal artery then comes off and leaves the pelvis
gonadal arteries leave the aorta just caudal to that. through Alcock’s canal and gives off penile and per-
Further down the aorta the inferior mesenteric ineal branches. (Netter 402, 403)
artery comes off the anterior portion of the aorta and
gives off a number of branches: the left colic artery Venous Anatomy
supplies the descending colon, the sigmoid arteries
supply the sigmoid, and various rectal arteries sup- In most cases, the venous structures parallel the
ply the rectum. (Netter 306, 307) arteries. Some of the exceptions include the left
gonadal and adrenal veins that empty into the left
The aorta then divides into the common iliacs, renal vein. In many cases, multiple veins or plexii
which themselves divide at the junction of L5 and accompany single arteries. (Netter 265)
the sacrum into the external iliac and the internal
iliac (hypogastric) arteries. (Netter 401) Lymphatic Anatomy

The external iliac artery gives off the inferior epi- The retroperitoneal lymph nodes are important
gastric medially and the deep circumflex artery lat- landing sites primarily for metastatic testis tumors.
erally. It then passes beneath the inguinal ligament The embryologic origin of the testicles is in the
and becomes the femoral artery. retroperitoneum where their vascular and lymphatic
structures are based and to where their tumors
The internal iliac (hypogastric) artery is responsible metastasize. It is important to recall that while left-
for most of the pelvic blood supply. It gives off a sided tumors stay on either the left side of the
posterior and an anterior branch. The posterior retroperitoneum or in the interaortocaval area, right-
branch gives off an iliolumbar artery, a lateral sacral sided tumors can cross over and occasionally
artery, and then continues as the superior gluteal involve even the left side of the retroperitoneum.
artery supplying the gluteal muscles. Loss of the That is why the templates for lymph node dissec-
superior gluteal blood supply can lead to cramping tions are different based on whether the tumor is
and pain in the gluteal muscles and buttocks. There- from the right or left testicle. The retroperitoneum
fore, if one were to attempt to tie off the hypogastric can also be a landing site for renal tumors as well.
arteries in an effort to stop or diminish pelvic bleed- The prostatic lymphatics drain through the internal
ing it is preferable to do that distal to the origin of and external iliac chains and many of the prostatic
the posterior branch, therefore sparing the blood metastases may be found within the obturator node
supply to the gluteal muscles. Currently it is much group. (Netter 408) The bladder drains through a
more common to utilize the skills of our interven- similar pathway. There are inguinal lymph nodes
tional radiology colleagues to selectively embolize which serve as landing sites for scrotal and penile
a particular artery in an effort to stop pelvic bleed- carcinoma. Superficial lymph nodes are external to
ing. The anterior division of the internal iliac artery the fascia lata while the deep chain lies along the
gives off the obturator artery followed by the umbil- femoral vessels, deep to the fascia lata. (Hinman
ical artery which itself becomes the obliterated 3.4, Netter 266)
umbilical and the superior vesical artery. The supe-
rior vesical artery is one of the primary blood sup-
plies to the pedicle of the bladder. During surgery
one way to easily find it is to follow the obliterated
umbilical to its origin which is where the superior
vesical artery originates as well. In women, the
vaginal and uterine arteries come off soon after the
umbilical artery. Distal to that is the origin of the
inferior vesical artery, which supplies much of the

8 EDUCATIONAL REVIEW MANUAL IN UROLOGY


4. Neuroanatomy

Table 7 The somatic system is composed of nerves respon-


sible for motor and sensory function. The auto-
Branches of Iliac Artery nomic system is responsible for involuntary
responses and control. We will review the thoracic,
External iliac lumbar and sacral portions of the somatic system
Inferior epigastric and then the autonomic system.
Deep circumflex (lateral)
Somatic System
Internal iliac (Hypogastric)
Posterior Division Thoracic nerves
Iliolumbar The thoracic nerves of interest to the urologist are
Lateral sacral the intercostal nerves numbers 6 through 12, which
Superior gluteal run beneath the ribs within the costal groove and
Anterior Division then travel between the internal oblique and
Obturator transversus abdominis muscles to innervate many of
Umbilical the abdominal wall muscles. These nerves are
- Obliterated umbilical responsible for motor and sensory function of these
- Superior vesical muscles and if injured, as can happen when a rib is
Vaginal/uterine resected during a flank incision, the result can be a
Inferior vesical bulge of the muscles in the area which that particu-
Internal pudendal lar nerve supplies. From a clinical standpoint, it is
- Penile important to be able to differentiate a flank bulge
- Perineal from a hernia, which is a correctable surgical com-
plication.

Lumbar Plexus

A number of important nerves originate from the


lumbar plexus. The iliohypogastric nerve provides
sensation for the lower abdominal wall, the pubic
area and motor innervation to the lower abdominal
wall. The ilioinguinal nerve has a primary sensory
function and supplies the upper thigh, base of the
penis and the upper scrotum. This nerve also travels
in the inguinal canal and care must be taken not to
damage it when operating in the inguinal area. The
lateral femoral cutaneous nerve is responsible for
sensation of the anterior and lateral portion of the
upper leg. The genitofemoral nerve provides sensa-
tion to the cord, the scrotum, and the anterior thigh
and motor function to the cremaster and the dartos
muscles. The femoral nerve provides sensation to
the anteromedial leg and motor function to the psoas
muscle, the iliac and the knee extensors. The obtura-
tor nerve is responsible for thigh adduction and the
sciatic nerve which takes its origin from both the
lumbar and the sciatic plexii providing both sensa-
tion and motor function to the posterior thigh and
lower leg. (Netter 497, 498, 499, Hinman 4.11,
4.12)

CHAPTER 1: APPLIED ANATOMY OF THE GENITOURINARY TRACT 9


Clinical Cautions Alcock’s canal. Therefore, it is not encountered dur-
ing routine pelvic surgery and in particular should
Lumbar plexus not be encountered and damaged during a radical
prostatectomy. That is the reason that even if the
A number of the nerves of the lumbar plexus can be neurovascular bundles near the prostate are dam-
injured during urologic surgery. The genitofemoral aged, causing erectile dysfunction, penile sensation
nerve lies directly atop the psoas muscle and can remains intact. The pudendal nerve also gives off a
occasionally be mistaken for the psoas minor ten- branch to the rhabdosphincter, which plays a role in
don and injured during a psoas hitch. If one is doing urinary continence. (Netter 410)
a psoas hitch, it is important to recognize that a third
of patients do not have a readily identifiable psoas The tibial nerve is a branch of the sciatic nerve and
minor tendon and care should be taken not to hitch originates from the sacral nerve roots. It runs down
the bladder to the genitofemoral nerve mistakenly. the lower extremity and then posterior to the medial
As noted earlier, the ilioinguinal nerve runs through malleolus. It is currently used as a site for needle
the inguinal canal and can be inadvertently dam- placement to facilitate neuromodulation for bladder
aged during an inguinal dissection with loss of sen- dysfunction.
sation to the upper thigh, base of the penis and upper
scrotum. The femoral nerve is hidden by the body of Autonomic System
the psoas muscle proximally and can be injured by
injudicious placement of retractors and lead to a loss The autonomic system is responsible for parasym-
of knee extension and anteromedial thigh sensation. pathetic and sympathetic functions of various uro-
The obturator nerve can be injured during a pelvic logic organs. When one looks at the image of the
lymphadenectomy, which can lead to loss of the sacral plexus, (Netter 410) the nervi erigentes or
ability to adduct the thigh. It can also be stimulated pelvic nerves are present. These are parasympa-
by electricity during a transurethral resection of a thetic nerves that go to the inferior hypogastric
bladder tumor causing thigh adduction or a sudden pelvic plexus where they combine with the sympa-
medial kick. When this occurs unexpectedly, an thetic nerves and then give off the autonomic supply
inadvertent bladder perforation can occur. Finally, to the bladder, prostate and penis. More cephalad
the sciatic nerve passes posterior to the are the celiac and superior mesenteric plexii, which
sacrospinous ligament and can be injured during a are responsible for autonomic control of the adrenal
sacrospinous vault suspension or if a patient is glands, kidneys and upper ureters. Along the aorta,
placed in an exaggerated lithotomy position, in a number of lumbar nerve roots that are largely sym-
which case there may be tension and stretching of pathetic travel along and on the aorta and then coa-
the sciatic nerve. (Netter 496) lesce just below the aortic bifurcation to form the
superior hypogastric plexus. These largely sympa-
Sacral plexus thetic nerves are responsible for seminal emission
and bladder neck closure. After a standard
The sciatic nerve, as noted above, originates both non–nerve-sparing retroperitoneal lymph node dis-
from lumbar and sacral roots. Also originating from section men may lose of the ability to ejaculate
the sacral roots is the posterior femoral cutaneous because of damage to these nerves. From the supe-
nerve, whose perineal branch provides sensation for rior hypogastric plexus, bilateral sets of nerves
the perineum and posterior scrotum. The pudendal called the hypogastric nerves travel to the inferior
nerve is of much importance to urologists and has hypogastric plexus where they combine with the
both sensory and motor functions. It gives off a per- parasympathetic pelvic nerves. (Netter 344, 416)
ineal branch, a posterior scrotal branch, and a geni-
tal branch that becomes the dorsal nerve of either The inferior hypogastric or pelvic plexus lies lateral
the penis or the clitoris. It is this branch that is to the rectum with its midpoint at the tip of the semi-
responsible for penile and clitoral sensation. Keep nal vesicles. There are multiple prostatic, rectal and
in mind that the pudendal nerve (like the internal vesicle blood vessel that penetrate this plexus.
pudendal artery) leaves the pelvis and runs through These nerves are responsible for bladder and pro-

10 EDUCATIONAL REVIEW MANUAL IN UROLOGY


Table 8

Lumbar Plexus

S = sensory
M = motor

Iliohypogastric S -Lower abdomen, pubis


M -Lower abdominal wall

Ilioinguinal S -Upper thigh, base of penis, upper scrotum


travels in inguinal canal

Lateral femoral cutaneous S -Anterior/lateral upper leg

Genitofemoral S -Spermatic cord, scrotum, anterior thigh


M -Cremaster and Dartos muscles

Femoral S -Anterior/medial leg


M -Psoas, iliac, knee extensors

Obturator M -Thigh adductors

Sciatic S/M -Posterior thigh and lower leg

Table 9

Autonomic

Celiac and Superior mesenteric plexus -Adrenal/kidney/upper ureter

Superior hypogastric plexus -Largely sympathetic - Lumbar roots


-Seminal emission
-Bladder neck closure

Continue as Hypogastric nerves (sympathetic)

Combine with Pelvic nerves & Nervi erigentes (parasympathetic)

Inferior hypogastric (pelvic) plexus - Lateral to rectum

CHAPTER 1: APPLIED ANATOMY OF THE GENITOURINARY TRACT 11


5. Inguinal Canal and
Femoral Triangle

static innervation and supply the neural component Inguinal Canal


of the neurovascular bundles that run alongside the
prostate until they continue within the corporal bod- The inguinal canal contains the spermatic cord in
ies of the penis as the cavernous nerves. These auto- males and the round ligament in females. There are
nomic nerves are responsible for penile erection and 3 sides to the canal: anterior made up primarily by
detumescence. It is the neurovascular bundles that the external oblique, posterior made up of the
can be injured during a radical prostatectomy and transversus abdominis and transversalis fascia, and
this can lead to a loss of erectile function. If one inferior made up of the inguinal ligament. Hessel-
looks at Netter’s plate 410, one can clearly see how bach’s triangle is bordered by the medial margin of
the somatic pudendal nerves run outside of the the inferior epigastric vessels, the lateral margin of
pelvis and thus penile sensation is maintained after the rectus muscle, and the inguinal ligament. When
prostatectomy, whereas the autonomic neurovascu- describing an inguinal hernia, those that are lateral
lar bundles travel alongside the prostate and can be to Hesselbach’s triangle are called indirect hernias
damaged during radical prostatectomy leading to whereas those that come directly through the trian-
erectile dysfunction. gle are called direct hernias. (Netter 259)

Femoral Triangle

The femoral triangle is bounded by the inguinal lig-


ament at the base, the sartorius muscle laterally, the
adductor longus muscle medially, and the floor is
made up of the iliopsoas and pectineal muscles.
Within the femoral triangle from lateral to medial
run the femoral nerve, femoral artery, and femoral
vein. The femoral triangle is covered by fascia lata
and there is an opening in the fascia lata, the fossa
ovalis, through which the superficial vessels and the
saphenous vein emerge. The node of Cabana is
found at the junction of the saphenous vein and the
superficial external pudendal vein and some feel
this is important to identify when doing a node dis-
section for penile cancer. The fascia lata divides the
deep and the superficial node groups. (Netter 262,
500)

Table 10

Femoral Triangle

Base - Inguinal ligament

Lateral - Sartorius

Medial - Adductor longus

Floor - Iliopsoas and pectineal


muscles

12 EDUCATIONAL REVIEW MANUAL IN UROLOGY


6. Adrenals 7. Kidney

The adrenal gland has a tripartite arterial supply. Gerota’s fascia


Superiorly, the superior suprarenal artery is derived
from the inferior phrenic artery. The middle Gerota’s fascia is part of the intermediate stratum
suprarenal artery comes directly off of the abdomi- and the perinephric fat is within it. The anterior
nal aorta while the inferior suprarenal artery is leaflet of Gerota’s fascia is sometimes referred to as
derived from the renal artery. On the left side, the Toldt’s fascia, while the posterior leaflet is some-
adrenal vein empties into the left renal vein whereas times referred to as Zuckerkandl’s fascia. Laterally
on the right the adrenal vein is a short vein that and superiorly, the fascia is fused while medially it
drains directly into the inferior vena cava. It enters is fused over the great vessels but contiguous with
the inferior vena cava almost posteriorly and can be the contralateral side and inferiorly it is an open
the site of troublesome bleeding during a radical potential space.
nephrectomy. The adrenal gland is separated into 2
distinct structures, both of which receive regulatory Kidneys
input from the nervous system. Nerves of the celiac
plexus and the renal plexus innervate the adrenal The kidneys lie in the retroperitoneum surrounded
glands. The adrenal medulla is at the center of the by perinephric fat and Gerota’s fascia. The anterior
adrenal gland and is the body’s main source of relations are as follows: the right kidney has the
epinephrine and norepinephrine. The adrenal cortex adrenal just above and anterior to it as well as the
which surrounds the medulla is separated into 3 liver, duodenum and the colon anterior to it. On the
zones: zona glomerulosa, zona fasciculata and zona left side, the adrenal, spleen, stomach, pancreas,
reticularis, which produce other important steroids. jejunum, and colon lie anterior to it. Posteriorly,
The adrenal gland lies within Gerota’s fascia supe- both the right and left kidney have the 12th rib,
rior to the kidney. (Netter 332, 347) diaphragm, transversus abdominis fascia, quadratus
lumborum, and psoas muscle posterior to it. The left
side has the 11th rib posterior to it as well.

The renal pedicle is at the level of the first through


third lumbar vertebrae. Anterior to posterior are the
renal vein, artery and then the renal pelvis. The
pedicle is just below the take off of the superior
mesenteric artery.

Renal Vasculature

Most people have 1 main renal artery but accessory


arteries are not uncommon. Five segmental arteries
come off of the main renal artery followed by inter-
lobar arteries, arcuate arteries, interlobular arteries
and finally the afferent arteriole. (Netter 335) The
arteries are end arteries without cross communica-
tion. The posterior segmental artery is usually the
first branch that comes off of the main renal artery
prior to the hilum. The other segmental arteries are
the apical, upper, middle and lower arteries. The
apical and lower arteries supply the uppermost and
lowermost portions of the kidney while the upper
and middle arteries supply the majority of the ante-
rior portion of the kidney. The posterior artery sup-
plies much of the posterior portion. An avascular
plane exists that separates the portion of renal

CHAPTER 1: APPLIED ANATOMY OF THE GENITOURINARY TRACT 13


Table 11

Kidneys - Anatomic Relations

Anterior

Right Left

Adrenal Adrenal

Liver Spleen

Duodenum Stomach

Colon Pancreas

Jejunum

Colon

Posterior

Right Right & Left Left

Rib 12, Diaphragm Rib 11

Transversus abdominis fascia

Quadratus lumborum, Psoas

Table 12

Renal Vasculature

Main renal artery

Segmental arteries (5)

Interlobar arteries

Arcuate arteries

Interlobular arteries

Afferent arteriole

14 EDUCATIONAL REVIEW MANUAL IN UROLOGY


8. Ureter

parenchyma supplied by the anterior branches from The renal pelvis drains into the ureters which are
that supplied by the posterior branch. This avascular generally 22–30 centimeters long, run along the
plane is not at the mid-portion of the kidney but in psoas muscle in the retroperitoneum, are usually
fact is posterior to the convex border of the kidney. adherent to the posterior peritoneum and cross the
This plane is important if one is doing an anatrophic iliac vessels near the iliac bifurcation. (Netter 341)
nephrolithotomy and seeks to bivalve the kidney In females they run behind the ovary, behind the
without damaging too much of it. In today’s era, this uterine artery, near the cervix, and beneath a portion
is done less frequently. (Netter 335, Hinman 12.34) of the vaginal wall. All of these are sites where the
ureter can be injured at the time of hysterectomy. In
The venous structures in the kidney accompany the males, the ureter runs behind the vas deferens.
arteries, but in contrast to the arteries there is cross There are multiple arterial branches that supply the
intercommunication between the venous structures. ureter. The upper ureter is supplied medially from
The right renal vein is a relatively short vein directly the renal artery, the gonadal artery, and from direct
to the inferior vena cava, while the left renal vein branches off of the aorta. The lower ureter supply is
receives the adrenal and gonadal veins and then lateral from the iliacs and vesical arteries and in
crosses anterior to the aorta and posterior to the women from branches off of the uterine artery. Once
superior mesenteric artery after which it drains into the arteries reach the ureter they run in longitudinal
the inferior vena cava. anastamosing plexii. The ureteral caliber is greatest
in the abdominal portion, where it is 10 millimeters
Within the kidney itself, the cortex is mainly made in diameter. There are 3 sites of narrowing: at the
up of nephrons, whereas the medulla contains pri- UPJ, over the iliacs and at the ureterovesical junc-
marily collecting ducts. Within the individual tion. In these areas, the ureteral diameter is from 2–4
nephron is the afferent arteriole which was derived millimeters.
from the interlobular artery. The afferent arteriole
enters the glomerulus where filtration takes place.
Whatever blood is left within the artery exits the
glomerulus via the efferent arteriole. Whatever was
filtered across the glomerular membrane travels
through the proximal convoluted tubule, then the
loop of Henle, the distal convoluted tubule, and then
the collecting tubule. (Netter 336) Multiple collect-
ing tubules make up a pyramid; the tip of which is
called a papilla. There are typically 7–9 papillae
within a kidney and each one is cupped by a minor
calyx. On occasion, one may see more than 1 papilla
per minor calyx. The minor calyces are connected
by necks or infundibulae to form 2 or 3 major
calyces which coalesce to form the renal pelvis.
(Netter 334)

CHAPTER 1: APPLIED ANATOMY OF THE GENITOURINARY TRACT 15


9. Bladder

The bladder and urethra receive innervation that is In the adult, the bladder is primarily a pelvic organ
both autonomic and somatic in nature. It is the while in children it is more abdominal in position.
parasympathetic system that is the primary driver of Superiorly, the bladder is covered by peritoneum,
normal bladder emptying. The parasympathetic posteriorly in males lies the rectum and Denonvil-
nerves are derived from the 2nd to 4th sacral portion liers' fascia and in females the vagina and anteriorly
of the spinal cord and are sometimes called the is a potential space to the pubis called the space of
pelvic nerves. The parasympathetic innervation Retzius. The detrusor muscle has 3 layers—outer
travels via the inferior hypogastric plexus to the longitudinal, middle circular and inner longitudinal.
bladder body and stimulation of these nerves is The trigone is made up of ureteric musculature that
responsible for bladder contraction. The sympa- runs from the ureteral orifices to the bladder neck.
thetic nerves are from the 10th thoracic to the sec- (Netter 365, 366)
ond lumbar portion of the spinal cord. They form
part of the hypogastric nerve and the impulses travel
via the inferior hypogastric plexus primarily to the
trigone and bladder neck. The somatic innervation
is primarily from S2 via the pudendal nerve to the
external urethral sphincter. (Hinman 13.35, 13.36,
Netter 417)

The blood supply to the bladder is derived from the


internal iliac artery via the superior and inferior
vesicle pedicles. The superior vesicle pedicle is pri-
marily supplied by the superior vesicle artery. The
superior vesicle artery and the obliterated umbilical
artery are derived from the same umbilical artery
branch. Thus, if one can trace the obliterated umbili-
cal artery down to its site of origin, it is relatively
easy to find the superior vesicle artery and gain
appropriate control over the superior vesicle pedicle
at the time of a cystectomy. The inferior vesicle
pedicle is supplied by the inferior vesical artery,
which also usually comes directly off of the internal
iliac artery but in some cases may be a branch of the
internal pudendal artery. (Hinman 13.33, Netter
403)

Table 13

Bladder - Innervation

Sympathetic T 10 -L 2 - Hypogastric nerve (trigone, bladder neck)

Parasympathetic S 2-4 - Pelvic nerve (bladder body)

Somatic S2 - Pudendal nerve (external sphincter)

16 EDUCATIONAL REVIEW MANUAL IN UROLOGY


10. Prostate 11. Seminal Vesicles and
Vasa Deferentia

The average prostate weighs about 18 grams, is 3–4 The seminal vesicles are approximately 5 centime-
centimeters long, 4 centimeters wide and 2 centime- ters in length and fuse with the ampulla of the vas to
ters deep. Anterior to the prostate lies the space of form the ejaculatory ducts. (Netter 384) Each vas
Retzius and the puboprostatic ligaments; anterolater- deferens arises from the tail of the epididymis, runs
ally lies the endopelvic fascia; laterally is fatty tis- posterior to the cord vessels, travels through the
sue, the levator ani and the neurovascular bundles; inguinal canal, passes through the internal ring and
posteriorly is Denonvilliers' fascia and the rectum; runs extraperitoneally where it passes above the
superiorly is the bladder and the seminal vesicles; ureter and where the dilated terminal vas, the
inferiorly is the urogenital diaphragm. (Netter 384) ampulla, fuses with the corresponding seminal vesi-
cle to form the ejaculatory duct which runs through
With the advent of transrectal ultrasound imaging the central zone of the prostate emptying into the
the zonal anatomy of the prostate has become much prostatic urethra. (Netter 390) The blood supply to
more readily apparent. The transition zone makes up the vas deferens is via the vesiculodeferential artery,
5%–10% of the normal volume of the prostate, gives which is a branch of the superior vesicle artery and
rise to benign prostatic hyperplasia and is where the innervation to the vas deferens is from the infe-
about 20% of prostate cancer originates. The central rior hypogastric plexus mostly via sympathetic
zone makes up 20% of the normal prostatic volume, nerves. Remember that the parasympathetic system
surrounds the ejaculatory ducts and is where a is primarily responsible for erectile function, while
minority of prostate cancer arises. The peripheral the sympathetic system is responsible for ejacula-
zone makes up 75% of the normal volume of the tion. Thus the vasa deferentia and the seminal vesi-
prostate and is where the majority of prostate cancer cles, which play an important role during ejaculatory
arises. function, are primarily subject to sympathetic con-
trol.
The vascular supply to the prostate is based on the
inferior vesicle artery which has a prostatic branch
that supplies the prostate. The prostatic branch gives
off urethral branches and capsular branches. The
urethral branches run perpendicular to the urethra
near the bladder neck and then turn parallel to the
urethra and supply the BPH adenoma. On the other
hand, the capsular branches run lateral to the prostate
as part of the neurovascular bundle and give off vari-
ous rami to the prostate. The venous return from the
prostate is via the prostatic plexus, sometimes called
Santorini’s plexus. The prostatic plexus receives
inflow from the deep dorsal vein of the penis as well
as from the prostate itself. It drains primarily to the
internal iliac veins and communicates via Batson’s
plexus with the vertebral veins. (Netter 403)

Innervation of the prostate is via the inferior


hypogastric plexus and there is both sympathetic
(hypogastric nerve) and parasympathetic (pelvic
nerve) supply. The lymphatic drainage is primarily
to the internal iliac chain of lymph nodes.

CHAPTER 1: APPLIED ANATOMY OF THE GENITOURINARY TRACT 17


12. Urethra 13. Female Pelvic Floor

Male The female pelvic floor has 3 primary responsibili-


ties; first, it prevents abdominal contents from
The male urethra is commonly divided into the pos- falling out; second, it controls the storage and evac-
terior and anterior portions and even further divided uation of feces, and; third, it allows for conception
into the prostatic and membranous urethra that are and parturition. There are 2 components to the
part of the posterior urethra and the bulbar, penile female pelvic floor: the viscerofascial layer, which
and glanular portions which are part of the anterior is made up of the various types of connective tissue
urethra. The anterior urethra runs within the corpus and fascia and the muscular layer, primarily made
spongiosum and there are multiple small glands that up of the various levator ani muscles. The compo-
empty into it. The blood supply to the urethra is via nents of the viscerofascial layer include structures
what is called a dual supply. It is based on the inter- such as the cardinal ligaments, the pubocervical fas-
nal pudendal artery that distally becomes the com- cia and the prerectal fascia. It is important to
mon penile artery and then gives off a number of remember that these fascial layers are not true fascia
branches. One important branch is the bulbourethral but instead are condensations of relatively loose
artery which directly supplies the corpus spongio- areolar tissue. The pubovisceral muscles are made
sum. Another branch of the common penile artery is up of various components of the levator ani and run
the dorsal artery which terminates at the glans. from the pubis to the coccyx in a sling-like configu-
Because the corpus spongiosum and glans are inti- ration supporting the pelvic organs. The pubococcy-
mately connected blood flow from the glans then geous runs from the pubis to the coccyx and as such
travels in a retrograde fashion through the corpus connects relatively immobile structures—parts of
spongiosum. This is why the corpus spongiosum the bony skeleton—and therefore has little contribu-
has what is called a dual blood supply, 1; directly in tion to overall pelvic floor support. The puborectalis
an antegrade fashion from the bulbourethral artery, runs from the pubis around the rectum and back to
and 2; in a retrograde fashion via the glans from the the pubis. It is attached to the lateral vaginal walls,
dorsal penile artery. This is also why if one is doing penetrates the rectal walls, and does play an impor-
a urethroplasty one can cut across the entire corpus tant role in pelvic floor support. The iliococcygeous
spongiosum and still expect blood flow in the distal which runs from the ilium to the coccyx is in more
portion. (Netter 385, 403) of a horizontal sheet configuration. Much of the
connection of the fascial layers to the pelvic side
Female wall is via the arcus tendineus, a condensation of tis-
sue on the ileum that runs from the pubis to the
The female urethra is approximately 4 centimeters ischial spine. (Netter 356)
long and has many small glands which are distally
grouped to form Skene’s glands that empty into the
urethra and the periurethral area. There is a rich
plexus of veins and smooth muscle that helps pro-
vide resistance to increase the closure pressure and
promote urinary continence. (Netter 356)

18 EDUCATIONAL REVIEW MANUAL IN UROLOGY


14. External Genitalia

Penis Buck’s and dartos fascia and there is minimal vascu-


lar communication between the corpora cavernosal
The penis is made up of the glans penis, the corpus and spongiosal bodies. (Netter 381)
spongiosum, and the paired corpora cavernosal bod-
ies. The corpora cavernosal bodies are attached The arterial supply to the penis has a similar origin
proximally via crura to the pubic arch. The distal ¾ to that of the urethra. It is from the internal pudendal
of each corporal body is closely apposed and dis- artery which terminally becomes the penile artery
tally evaginates the glans. The corpus spongiosum and then gives off the bulbourethral artery, the dor-
encloses the urethra and terminates to join the glans. sal artery of the penis and finally the cavernosal
(Netter 382) The corpora cavernosal bodies are artery. One cavernosal artery on each side supplies
enclosed within the tunica albuginea and are filled blood to its respective corpora cavernosal body and
with erectile tissue made up of smooth muscle with is thus responsible for blood inflow to the penis
intervening vascular spaces. They are divided by a leading to erection. There is also blood flow from
septum, although distally there is communication the femoral artery via the external pudendal arteries
between the 2 corporal bodies. This is why when to the superficial penile arteries and this is responsi-
intracavernosal injections are used for erectile dys- ble for supplying the shaft skin of the penis. (Netter
function only one corporal body needs to be injected 403)
as it affects both corporal bodies via the sites of
communication. Buck’s fascia surrounds the corpo- There are 3 levels of venous return of blood from
ral bodies and the dorsal nerves, arteries and vein the penis. The superficial skin is drained via a
lay under it. More superficially, a layer of loose are- superficial system where the superficial veins join
olar tissue, the dartos fascia, surrounds Buck’s and to form a superficial dorsal vein that drains via the
finally the penile skin is the most superficial layer. saphenous vein. The glans, the corpora spongiosum,
The corpus spongiosum as well is enclosed by and the distal 2/3 of the corpora cavernosal bodies

Table 14

Viscerofascial Layer

Cardinal ligaments apex

Uterosacral ligaments apex

Pubocervical fascia bladder/pelvic side walls

Prerectal fascia rectum/pelvic side walls

Pubovisceral Muscles

Levator ani – from pubis to coccyx – sling like

Pubococcygeous - Connects immobile structures


- Little contribution to support

Puborectalis - Attached to lateral vaginal walls


- Penetrates rectal wall

iliococcygeous - Horizontal sheet

CHAPTER 1: APPLIED ANATOMY OF THE GENITOURINARY TRACT 19


15. Testis and Scrotum

drain via the intermediate system. Blood from the Testis


glans drains via the retrocoronal plexus into the deep
dorsal vein whereas blood from the corporal bodies The normal testis is approximately 4–5 cm in length
drains via the circumflex veins into the deep dorsal with a volume of 30 mL and is covered by tunica
vein. The deep dorsal vein then empties into the pro- albuginea. The arterial supply to the testis is via the
static plexus. The deep system is responsible for testicular artery directly from the aorta as well as
draining the proximal 1/3 of the corpora cavernosal from a vasal artery from the superior vesicle artery.
bodies and the bulbar portion of the spongiosum. The venous return is via the pampiniform plexus,
The cavernosal veins drain to the internal pudendal which coalesces to form the gonadal vein which
veins, the bulbar veins drain to the prostatic plexus, empties on the right side directly into the inferior
and the crural veins drain to both the internal puden- vena cava and empties on the left side into the left
dal veins and the prostatic plexus. (Hinman 16.31) renal vein. (Netter 390)

The innervation to the penis is via both somatic and Spermatogenesis takes place in the seminiferous
autonomic nerves. The somatic nerves are responsi- tubules after which the spermatids travel via the rete
ble for sensory function. The dorsal nerve of the testis to the efferent ductules and then via the convo-
penis, a branch of the pudendal nerve which travels luted epididymal duct to the vas deferens.
via Alcock’s canal, is responsible for sensory func-
tion. The autonomic system is responsible for nor- Scrotum
mal erectile function. Both sympathetic and
parasympathetic nerves from the inferior hypogas- The layers of the scrotum are primarily derived from
tric plexus travel via the neurovascular bundle, lat- the various layers of the abdominal wall as the testes
eral to the prostate, and then into the corporal bodies descend into the scrotum. Most layers of the scro-
as the cavernous nerves. tum can be traced up to and are continuous with var-
ious layers of the abdominal wall. From inside out,
Thus, one can see how after a radical prostatectomy the various layers include the tunica albuginea sur-
where the neurovascular bundles are damaged one rounding the testis, the visceral tunica vaginalis, the
might lose erectile function however sensory func- parietal tunica vaginalis, the internal spermatic fas-
tion will be maintained as the sensory nerves leave cia, the cremasteric muscle and fascia, the external
the pelvis via a different route and are not damaged spermatic fascia, the dartos muscle, and finally the
during a radical prostatectomy. scrotal skin. (Netter 390)

Clitoris

The blood flow and innervation of the clitoris paral-


lels that of the penis. Sensory function is via the cli-
toral nerve which runs from the pudendal nerve. On
the other hand, clitoral engorgement is due to auto-
nomic stimulation via the cavernous nerves from the
inferior hypogastric plexus. Though clinically the
clitoris appears to be only 1–2 cm long, in reality it
is closer to 6–7 cm long, though the majority of that
is attached to the undersurface of the pubis and not
readily visible. It can be easily seen on MRI.

20 EDUCATIONAL REVIEW MANUAL IN UROLOGY


16. Questions

Table 15 1. Which of the following structures is not typi-


cally encountered in the course of renal surgery
Penis through a flank incision?

A. Internal oblique muscle


Arterial Supply
B. Transversalis fascia
Internal pudendal artery
C. Rectus muscle
Penile artery (via Alcock’s canal)
- Cavernosal arteries -corpora cavernosa D. Thoracolumbar fascia
- Bulbourethral – corpus spongiosum
- Dorsal artery -cavernosal branches E. Transversus abdominis
- Circumflex
- Glans
2. Regarding Gerota’s fascia, which of the follow-
Femoral artery ing are true?
- External pudendal arteries
- Superficial penile arteries (supplies shaft skin) A. It is part of the inner stratum of
retroperitoneal tissue
Venous Return
B. Inferiorly, there is an open potential space
Superficial - Superficial veins join
-Superficial dorsal vein C. Perinephric fat is outside of it
saphenous vein
D. Both A and C
Intermediate - Tlans, corpus spongiosum, distal
2/3 of corpora cavernosa E. None of the above
- Retrocoronal plexus
- Circumflex veins
- Deep dorsal vein 3. Which of the following is not typically a site of
- Prostatic plexus normal ureteral narrowing (where stones get
caught)?
Deep - proximal 1/3 of corpora cavernosa, bulb
- Cavernous vein internal pudendal A. UPJ
- Bulbar vein prostatic plexus
- Crural vein both of above B. Iliac vessels

C. Pelvic ureter

D. UVJ

CHAPTER 1: APPLIED ANATOMY OF THE GENITOURINARY TRACT 21


4. Normal voiding is dependent on all of the fol- C. Inferior vesical artery
lowing nerves except?
D. Obturator artery
A. Pelvic

B. Hypogastric 8. Which of the following nerves would one sus-


pect was damaged if a patient lost the ability to
C. Obturator adduct the thighs after pelvic surgery?

D. Pudendal A. Ilioinguinal

B. Femoral
5. Which of the following is true about the
prostate? C. Obturator

A. Most prostate cancer is from the D. Sciatic


transition zone

B. Most normal volume is in the 9. Which of the following structures do not make
peripheral zone up part of Hesselbach’s triangle?

C. The central zone is primarily distal A. Inferior epigastric vessels


to the verumontanum
B. Lateral margin of the rectus muscle
D. The primary blood supply is from
the superior vesicle artery C. Pectineal muscle

D. Inguinal ligament

6. The adrenal glands receive blood supply


from all of the following except? 10. In women, the ureter is in close proximity to
and can be damaged during gynecologic
A. Superior phrenic artery surgery on which of the following structures?

B. Inferior phrenic artery A. Ovary

C. A direct branch from the aorta B. Uterine artery

D. A branch off of the renal artery C. Cervix

D. Vaginal wall
7. If one needs to ligate the hypogastric arteries
for severe pelvic bleeding, it should be done E. All of the above
distal to which area?

A. Posterior division

B. Umbilical artery

22 EDUCATIONAL REVIEW MANUAL IN UROLOGY


Answers 9. C.
Hesselbach’s triangle is bordered medially by the
1. C. rectus muscle, laterally by the inferior epigastrics
The typical flank incision does not travel anterome- and inferiorly by the inguinal ligament.
dially to the border of the rectus. All other structures
noted are traversed. 10. E.
All of these structures can be damaged during gyne-
2. B. cological surgery.
It is part of the intermediate stratum and the per-
inephric fat is within it.

3. C.
Ureteral caliber is typically narrowest at the UPJ,
over the iliacs and at the UVJ. There is no particular
narrow point per se of the pelvic portion of the
ureter.

4. C.
Obturator nerves allow for thigh adduction. Pelvic
and hypogastric nerves carry the autonomic supply
to the bladder and innervation to the external
sphincter is via the pudendal.

5. B.
A, C and D are false—most cancer is in the periph-
eral zone, the central zone is proximal to the veru
and the primary blood supply is from the inferior
vesicle artery.

6. A.
The adrenal has a tripartite blood supply and
receives blood from all the options listed except the
superior phrenic artery.

7. A.
The posterior division includes the gluteal artery,
which supplies the gluteus. Ligating proximal to
this point may lead to pain in the buttocks.

8. C.
The obturator nerve is responsible for thigh adduc-
tion.

CHAPTER 1: APPLIED ANATOMY OF THE GENITOURINARY TRACT 23


24 EDUCATIONAL REVIEW MANUAL
Chapter 2:
Pediatric Urology
Hiep Thieu Nguyen, MD, FAAP

Dr. Barry A. Kogan (Albany Medical College)


Acknowledgment

kindly provided some of the figures.

Contents

1. Fetal Abnormalities

2. Ureteropelvic Junction Obstruction

3. Ureterovesical Junction Obstruction


(Obstructed Megaureter)

4. Ureteroceles

5. Renal Cystic Diseases

6. Posterior Urethral Valves

7. Prune Belly Syndrome (Triad Syndrome)

8. Urinary Tract Infections

9. Vesicoureteral Reflux

10. Further Reading

11. Questions

CHAPTER 2: PEDIATRIC UROLOGY 25


1. Fetal Abnormalities

With the advent of routine prenatal ultrasonography • Amniotic fluid:


(US), children with congenital anomalies are now * Early = transudate of amnion
being detected prior to developing signs and symp-
toms such as urinary tract infection, abdominal * Later = fetal urine + lung fluid-swallowing
mass, hematuria, kidney stones, and pain. These
children benefit from early diagnosis, with the goal * Amniotic fluid volume:
of preventing complications and to preserve renal • 380 cc at 20th week
function when possible. However, not all findings • 800 cc at 28th week
on prenatal US represent pathology; many have no • 800 cc at 40th week
clinical significance. The dilemma is to be able to
differentiate which children require intervention * Amniotic volume dependent on urine
from those who do not. Specific findings on prena- production at 16th week.
tal US can help to make this differentiation.
* Etiology of polyhydramnios (>1.5 L).
US of the Fetal GU Tract • Esophageal obstruction
• Multicystic kidney
• Fetal kidneys • Mesoblastic nephroma
* Ureteral bud formation at 5th week • Some obstructive processes
of gestation.
* Etiology of oligohydramnios (<0.5 L).
* Urine formation at 5th–8th week. • Amnion nodosum
• 5 cc/hr at 20th week • Amniotic fluid leak
• 50 cc/hr at 40th week • Urinary tract obstruction

* Can be visualized at 12th–13th week. * Consequences of oligohydramnios.


• Pulmonary hypoplasia
* Visualization of hydronephrosis • Potter’s syndrome: flat nose,
at 12th–18th week. recessed chin, low-set ears, bowed
legs, small chest, tales equinovares,
* Distinct renal architecture at 20th week. and hypoplastic hands
• Limb deformities
* Detailed examination is better in the
2nd and 3rd trimester. • Genitalia:
* Can be visualized by 24th week.
* Renal measurements: 12th–40th week. * In 40% of fetuses, the sex can be determined at
• AP diameter: 0.8–2.6 cm this time point
• Transverse diameter: 0.9–2.6 cm
• Length: 1.0–2.7 cm Findings on Prenatal US That May
Suggest GU Anomalies
• Fetal bladder:
* Can be visualized at 14th week • Hydronephrosis. Variable degree of dilation of
the renal pelvis (pelviectasis) and/or calyces
* Emptying of the fetal bladder can (caliectasis) caused by obstruction or reflux.
be seen at 15th week
• Renal pyramids. Echolucent structures within the
* Size: kidney suggest normal renal development and are
• 10 cc at 30th week sometimes misinterpreted as dilated calyces.
• 50 cc at term

26 EDUCATIONAL REVIEW MANUAL IN UROLOGY


• Echogenicity. Increased echogenicity similar to
the appearance of the spleen or liver suggests the
Figure 1

presence of renal parenchymal diseases such as


dysplasia, severe obstruction or reflux, and inheri-
Keyhole sign

table cystic diseases.

• Parenchymal thinning. Decreased thickness of


the renal parenchyma suggests the lost of function-
ing renal tissue caused by obstruction or reflux.

• Urothelial thickening. Increased thickness of the


renal pelvis lining suggests the presence of vari-
able dilation as seen with reflux or less often with
obstruction.

• Duplication. Separation of the renal pelvic sinus


echoes without hydronephrosis is possibly associ-
ated with reflux or obstruction, such as in ectopic
Keyhole sign observed on US suggesting the

ureter or ureterocele.
presence of posterior urethral valves.

• Renal cyst. Hypoechoic structures within the kid-


ney are seen in multicystic dysplastic kidney
(MCDK) or inheritable polycystic diseases.

• Intravesical cystic structures. Thin-walled cyst


Figure 2

in the bladder suggests the diagnosis of an uretero-


cele.
Prenatal US demonstrating hydronephrosis

• Urinoma. A fluid collection around the kidney is


seen in cases of severe obstruction.

• Bladder filling. Lack of bladder filling suggests


poor urine production or bladder exstrophy, while
lack of filling and emptying cycles suggests blad-
der outlet obstruction.

• Bladder wall thickness. Increased thickness of


the bladder wall suggests the presence of bladder
outlet obstruction or neurogenic dysfunction,
but it must be interpreted in the context of
bladder filling.

• Keyhole sign. Dilation of the posterior urethra


Prenatal US (cross-section) demonstrating

suggests the presence of posterior urethral valves


hydronephrosis greater on the left than right. Mea-

(Figure 1).
surement of the anterior-posterior diameter (between
the white arrows) can be performed on this view of the

• Oligohydramnios. Markedly reduced amniotic


kidneys.

fluid indicated by the absence of a pocket of fluid


>2 cm suggests poor urine output caused by
obstruction and/or renal dysfunction.

CHAPTER 2: PEDIATRIC UROLOGY 27


Hydronephrosis is the most common abnormality • Fetal US appearance of the major diagnoses:
detected on prenatal US. It is not a specific diagno-
sis, but a finding. * UPJ obstruction. Pelvicalyceal dilatation
without ureteral dilatation, usually unilateral.
• Defining pathologic hydronephrosis:
* UVJ obstruction. Dilatation of renal pelvis and
* Anterior-posterior diameter of the renal pelvis is ureter to the level of the bladder.
the simplest, most sensitive and reproducible
parameter in defining pathologic hydronephro- * VUR. Variable degree of hydronephrosis or
sis (Figure 2). hydroureteronephrosis, but there are no reliable
findings to predict reflux on fetal US.
* Dependent upon the gestation age.
* PUV. Bilateral hydroureteronephrosis with
bladder wall thickening and keyhole sign is
highly suggestive; however, variable findings
are seen due to the wide spectrum of severity of
Date of Pelvic

the disease.
Detection Degree Diameter

* Ureterocele. Upper pole hydroureteronephro-


2nd trimester Mild 4 – < 7 mm

sis with a thin-walled cystic structure in the base


Moderate 7–10mm

of the bladder.
Severe >10mm

* Ectopic ureter. Upper pole hydrouretero-


3rd trimester Mild 7 – < 9mm

nephrosis without an associated intravesical


Moderate 9 – 15mm

cystic structure.
Severe >15mm

* More likely to be pathological if there is:


• Calyceal dilatation * MCDK. Nonreniform structure with
• Ureteral dilation multiple noncommunicating, fluid-filled cystic
• Chromosomal abnormalities spaces, without a central large cyst and little
• Multiple malformations renal parenchyma; most often confused with
• Oligohydramnios severe hydronephrosis.

* Association with Down’s syndrome * Autosomal-recessive polycystic kidney disease


is low (0.3%–3.3%) (ARPKD). Bilateral markedly enlarged and
brightly echogenic kidneys.
• Differential diagnosis for hydronephrosis:
* Autosomal-dominant polycystic kidney
Transient/physiologic 41%–88% disease (ADPKD). Heterogeneous renal
UPJ obstruction 10%–30% cysts in size and location.
VUR 10%–20%
Megaureter 5%–10% * Prune belly syndrome. Bilateral
MCDK 4%–6% hydroureteronephrosis with an enlarged
PUV/urethral atresia 1%–2% bladder, often confused with PUV.
Ureterocele, 5%–7%
ectopic ureter, duplex system
Others: urethral atresia, prune Less common
belly, PCKD, renal cystic
disease, congenital ureteric
strictures and megalourethra

28 EDUCATIONAL REVIEW MANUAL IN UROLOGY


Fetal Intervention Postnatal Evaluation

• Purpose: • Immediate: renal/bladder (RUS) and voiding cys-


* Prevent pulmonary hypoplasia tourethrogram (VCUG) within 1–2 weeks
* Improve kidney function * Solitary kidney
* Improve bladder function * Moderate or severe bilateral hydronephrosis
* Ureterocele
• Types: * Thick-walled bladder or keyhole sign
* Early delivery
* Open vesicostomy • Delayed:
* Vesicoamniotic shunts—most common * Mild unilateral or bilateral hydronephrosis:
* Endoscopic relief of bladder outlet RUS at 2–3 months; VCUG controversial
obstruction * Moderate-to-severe unilateral hydronephrosis:
RUS and VCUG at 2–4 weeks
• Indications: * Severe unilateral hydronephrosis: RUS, VCUG,
* Evidence of severe bladder outlet obstruction and intravenous pyelogram or MAG-3 renal
* Normal karyotype scan at 1 month
* No other systemic anomalies
* Singleton and male fetus • Antibiotic prophylaxis
* Oligohydramnios * Moderate or severe unilateral/bilateral
* Noncystic kidneys hydronephrosis
* Favorable urinary indices by serial sampling * If planning to get VCUG, then antibiotic pro-
over 3 days phylaxis should be given
• Na <100 mg/dL * Amoxicillin (25 mg/kg/day) or cephalosporin
• Cl <110 mg/dL
• Osm <210 mOsm/L
• β2-microglobulin <4 mg/L
• Calcium <8 mg/dL
• Total protein <20 mg/dL
* Informed consent

• Complications:
* Shunt displacement
* Preterm labor and prematurity
* Chorioamnionitis
* Iatrogenic gastroschisis
* Urinary ascites
* Limited improvement in bladder or kidney
function

CHAPTER 2: PEDIATRIC UROLOGY 29


2. Ureteropelvic
Junction Obstruction

• More commonly seen in infants and young


children
Definition

An impairment of urine flow from the renal pelvis * Extrinsic obstruction (Figure 4)
into the proximal ureter, causing progressive dilata- • Caused by compression of the ureter by
tion of the renal pelvis and calyces, and potential anomalous (eg, lower pole) renal vasculature
renal injury. • An aberrant, accessory or early-branching
lower pole vessel is the most common cause
Etiology of extrinsic UPJ obstruction
• Intermittent obstruction
• Primary: • More commonly found in older children and
* Intrinsic obstruction (Figure 3) adults
• Results from luminal narrowing of the UPJ • Whether the aberrant vessel causes obstruc-
with or without kinking tion, or is a covariable that exists along with
• Usually characterized by excessive connec- an intrinsic narrowing, is unclear
tive tissue and decreased smooth muscle con-
tent in the ureteral wall
• Fixed obstruction

Figure 3

IVP demonstrating UPJ obstruction

IVP demonstrating (left arrow) UPJ obstruction and intra-operative imaging demonstrating (right arrow) the cor-
responding primary intrinsic obstruction.

30 EDUCATIONAL REVIEW MANUAL IN UROLOGY


• Most common cause of obstructive uropathy in
children
Figure 4

Associated Anomalies
Primary extrinsic UPJ obstruction

• UPJ obstruction in the contralateral kidney


(10%–40%)

• VUR (13%–42%)—usually low grade

• Renal dysplasia or MCDK (10%)

• Unilateral renal agenesis (5%)

• VATER syndrome (vertebral defects, imperforate


anus, tracheoesophageal fistula, and radial/renal
dysplasia)(9%–21%)

Symptoms

• Usually asymptomatic—diagnosed following


evaluation of prenatal hydronephrosis

• Intermittent pain/vomiting (Dietl’s crisis) in older


children

• Palpable abdominal mass in young children


Intra-operative image of a primary extrinsic UPJ
obstruction due to crossing vessels (arrow).

• Less common:
* Urinary tract infection (UTI)
• Secondary: * Hematuria or trauma
* Due to marked VUR or PUV. * Stones
* Iatrogenic: postoperative. * Failure to thrive or feeding difficulties
* Uncommon. * Hypertension
* Incidental findings on US, CT, MRI or bone scan
Epidemiology
Diagnosis
• Incidence = 1 in 1,500
• US: standard imaging test to identify
• Male to female ratio = 2:1 hydronephrosis
* Does not diagnose obstruction or
• Left to right ratio = 1.5:1 predict resolution
* Possible correlation with clinically relevant
• Bilateral in 10%–40% obstruction:
• Anterior-posterior diameter of the renal pelvis
• Diagnosed most commonly by prenatal US (>15 mm)
• Increased hydronephrosis on serial US exam-
• Second most common cause of antenatal ination
hydronephrosis • Increase in growth of the contralateral kidney
on serial US examination

CHAPTER 2: PEDIATRIC UROLOGY 31


• Resistive index (peak systolic velocity—low- * Pressure decay half-life reflects both efficiency
est diastolic velocity) ≥0.75 of urine transport and relative compliance and
volume of the collecting system
• Diuretic radionuclide renography: assess differen-
tial function and assessment of washout from each • Magnetic resonance imaging (MRI)
kidney * Able to define anatomy, calculate differential
* Agents: renal function, and assess obstruction
• DTPA (99m Tc-diethylenetriaminepen- * Comparable to radionuclide renography
taacetic acid): exclusively filtered by the * Disadvantages include cost, equipment limita-
glomeruli with an extraction fraction of 20%, tion, sensitivity to patient motion, and the need
providing an indirect means of measuring for sedation
glomerular filtration rate (GFR)
• MAG-3 (99m Tc-mercaptoacetyltriglycine): • Cystoscopy with retrograde pyelogram
secreted by the proximal tubular cells with an * Infrequently performed as an isolated diagnos-
extraction fraction of 50% and little back- tic modality
ground, providing an indirect means of mea- * Risk of introducing infection
suring estimated renal plasma flow (ERPF) * May be performed at the time of surgical repair to
* Not obstructed: rule out concomitant distal ureteral obstruction
• Symmetric uptake: differential renal function
(DRF) >40% • Biochemical markers
• Good washout: T1/2 <10 minutes * Elevated N-acetyl-β-D-glucosaminidase (NAG)
* Obstructed: in the urine from the obstructed kidney
• Decrease in DRF on serial examination * Elevated TGF-β1 in the urine from the bladder of
• Potential obstruction: patients with unilateral UPJ obstruction
* T1/2 >30 minutes on one exam
* Worsening T1/2 on serial examination Treatment
• False positive with immature kidney
• Indication for surgical repair:
• Radionuclide renography: assess differential renal * <40% function and poor washout parameters
function and scarring * Progressive hydronephrosis on serial US exami-
* Agents: DMSA (99m Tc- dimercaptosuccinic nation
acid) binds to functioning proximal * Progressive loss of renal function on serial
tubular cells radionuclide renograms
* Direct assessment of renal blood flow * Renal pelvic size >5 cm
* More accurate than DTPA or MAG-3 in assess- * Infection/stone.
ing renal function
* Gold standard for evaluation of renal scarring • Types of surgical repair:
* Dismembered pyeloplasty
• Intravenous pyelogram (open surgical repair)
* Less commonly used since radionuclide renog- • Most common and gold standard
raphy has become available • Advantages:
* Beneficial when anatomy is very complex * Broad applicability including preservation
of anomalous vessels
• Pressure flow studies (Whitaker test) * Excision of the pathologic UPJ and appro-
* Measurement of intrapelvic pressure during priate reposition of the UPJ
infusion of fluid into the renal pelvis * Ability to perform reduction pyeloplasty
* Calculate the decrease in pressure with time • >95% success rate
(pressure decay) • Stent/nephrostomy optional
• Not obstructed: rapid pressure decay • Penrose drain is recommended when the
• Obstructed: slow pressure decay approach is retroperitoneal

32 EDUCATIONAL REVIEW MANUAL IN UROLOGY


3. Ureterovesical
Junction Obstruction
(Obstructed Megaureter)

* Endopyelotomy (endoscopic incision)


• Can be performed antegrade or retrograde
Definition

• Contraindicated in patients with crossing ves- An impairment of urinary flow from the distal ureter
sels or with very large pelvis into the bladder, resulting in dilation of the entire
• 80%–85% success rate collecting system from the distal ureter to calyces.
• Stenting is required
• May be good for postoperative obstruction Etiology
* Laparoscopic pyeloplasty
• Reduced morbidity compared to open surgery • Primary: due to a deficiency of smooth muscle in
especially in older children and young adults the ureterovesical ureter, resulting in a dynamic
• Significant learning curve distal segment that impedes normal peristalsis of
• Robotic assistance significantly reduces the urine through the ureter (Figure 5)
difficulties associated with laparoscopic
suturing
• Success rate 90%–95%
• Stent/nephrostomy optional

• Complications:
Figure 5

* Early: uncommon
• UTI
Primary UVJ obstruction

• Prolonged urinary leakage from the


Penrose drain; treat with stent
* Late:
• Lack of improvement in renal function or
washout time
• Worsening hydronephrosis
• Pain or UTI
• Worsening renal function

Intraoperative image of a primary UVJ obstruction.


Note the degree of dilatation in the proximal ureter but
normal diameter at the distal segment.

CHAPTER 2: PEDIATRIC UROLOGY 33


• Secondary: due to extrinsic compression of the * Provides anatomic detail of the renal
ureter by a thick bladder wall in pathological states parenchyma, collecting system, and the bladder
such as posterior urethral valves or neurogenic * Baseline for the degree of hydroureteronephro-
bladder sis for serial examination

Epidemiology • VCUG:
* Need to rule out reflux as the etiology for the
• Accounts for approximately 8% of children that dilated ureter
presented with symptoms such as infection, hema- * Look for signs (such as thick-walled bladder or
turia, or pain, and had hydroureteronephrosis on dilated posterior urethra) of secondary causes of
imaging studies UVJ obstruction such as neurogenic bladder and
bladder outlet obstruction
• Accounts for 23% of newborns with prenatally
diagnosed hydronephrosis • Diuretic radionuclide renogram:
* Evaluate function and washout
• Third most common cause of antenatal * Need to define the region of interest around the
hydronephrosis distal ureter to accurately evaluate drainage
from the ureter (Figure 7)
• Male to female ratio = 2–4:1 * Defer study until after 3 months of age to allow
for tubular maturation
• Left to right ratio = 1.6–4.5:1
• Less often used imaging modalities: IVP,
• Bilaterally in 10%–25% of the cases MRI, and pressure perfusion study

Associated Anomalies

• Contralateral renal anomalies (renal agenesis or


MCDK)—9%

• Concomitant ipsilateral VUR


Figure 6

(obstructed reflux megaureter)—2% Dilated cystic structure

• Ureterocele and duplicated


systems—uncommon

• Concomitant ipsilateral UPJ obstruction—rare

Symptoms

• Usually asymptomatic—diagnosed following


evaluation of prenatal hydronephrosis

• Less common: UTI, stone, and hematuria

Diagnosis

• US:
* The presence of a dilated ureter distinguishes
US demonstrating a dilated cystic structure behind

UVJ obstruction from UPJ obstruction


the bladder, consistent with a dilated ureter.

(Figure 6)

34 EDUCATIONAL REVIEW MANUAL IN UROLOGY


* Progressive hydroureteronephrosis
* Stones
Figure 7

• Types of surgical repair


MAG-3 scan

* Reimplantation of affected ureter (intravesical


or extravesical)
• Need a tunnel length-to-ureteral diameter ratio
of 5:1
* Ureteral tailoring may be needed to decrease the
diameter of the ureter to allow for the appropri-
ately sized tunnel to be created

• Plication (Kalicinski or Starr)


* Infolding of the ureteral wall
* Advantages: preserves ureteral vascular-
ity and is reversible
* Disadvantages: tissue bulk and sponta-
neous reopening of the infolding seg-
ment, resulting in VUR
* Indicated for moderately dilated ureter
• Excisional tapering (Hendren) (Figure 8)
* Removal of lateral wall portion of the ureter
with preservation of the medial blood sup-
MAG-3 scan demonstrating a dilated left collecting ply to the ureter
system with much of dilatation occurring in the distal
ureter (arrow).

Figure 8

• Cystoscopy with retrograde pyelogram


* Infrequently performed as an isolated
Excisional technique of ureteral tapering

diagnostic modality
* Risk of introducing infection
* May be performed at the time of
surgical repair

Treatment

• Depending on the mode of presentation


* Asymptomatic—prenatal hydronephrosis
• Natural history: most improve spontaneously
• Significant risk for UTIs: need prophylactic
antibiotic during the first 6 months to first year
of life
* Secondary UVJ obstruction
• Correction of the underlying pathology Intraoperative image demonstrating the excisional

• Indication for surgical correction


technique of ureteral tapering.

* Recurrent UTIs while on prophylaxis


* Decrease in renal function

CHAPTER 2: PEDIATRIC UROLOGY 35


4. Ureteroceles

* Higher success rate


* Indicated for ureters >1.75 cm in diameter
Definition

or markedly thickened A cystic dilatation of the intravesical submucosal


ureter, usually associated with an obstructed orifice
• Complications: that impairs urinary flow into the bladder (Figure 9).
* Obstruction When associated with a duplicated system, the
• Related to postoperative edema ureterocele is associated with the upper pole collect-
* Resolve with time (up to 8 weeks) ing system (Weigert-Meyer Law).
* May require temporary diversion (stent or
nephrostomy tube) if patients have symp- • Intravesical. The ureterocele is entirely contained
toms (UTI or pain) in the bladder
• Related to ischemia of the distal ureter
* Persistent obstruction • Extravesical. Any portion of the ureterocele
* Required resection of the distal ureter and extends into the bladder neck or urethra
reimplantation
* VUR • Stenotic. The narrowed or pinpoint opening of the
• Mild cases resolve with time ureterocele is found inside the bladder
• When associated with UTI or persists, will
need surgical correction • Sphincteric. The orifice of the ureterocele is distal
• Need to rule out neurogenic or non-neurogenic to the bladder neck
bladder dysfunction as a contributory factor
• May consider subureteric injection therapy, • Sphincterostenotic. Combination of the above
ureteral reimplantation, or transureter-
oureterostomy • Cecoureterocele. A ureterocele that has an intrav-
esical orifice but a submucosal extension that goes
into the urethra

Etiology

Unknown, but it is hypothesized that it results either


from an incomplete breakdown of the ureteral
(Chwalla) membrane present at the time of the
ureteral bud arising from the mesonephric duct, or
from a delay in the establishment of the lumen of the
ureteral bud

Epidemiology

• Incidence: 1 in 500 to 1 in 4,000

• Male-to-female ratio = 1:5–7

• Much more common in Caucasians

• Bilateral in 10%

36 EDUCATIONAL REVIEW MANUAL IN UROLOGY


Associated Anomalies • UTI and sepsis due to urinary stasis

• Duplex system in 80% and single system in 20% • Abdominal mass due to obstruction of the upper
collecting system and/or bladder
• VUR:
* Ipsilateral lower pole—50%–70%. • Intralabial mass due to prolapse of the ureterocele
* Contralateral renal unit—10%–30% into the introitus
* Ureterocele—uncommon, need to consider
ectopic ureterocele with the orifice in the • Voiding dysfunction (urgency and incontinence)
urethra allowing reflux into the ureterocele associated with ureterocele
and its ureter
• Less commonly: hematuria, failure to thrive
Symptoms
Diagnosis
• Usually asymptomatic—diagnosed following
evaluation of prenatal hydronephrosis • US:
* Identification of a thin-walled cystic mass
within the bladder (Figure 10)
* Pitfalls:
• May be effaced and not visualized if the blad-
der is overdistended
Figure 9

• May resemble the bladder if the bladder is


empty
Ureterocele

• May be misidentified as a dilated or ectopic


ureter

Figure 10

Cystic, thin-walled structure

US demonstrating a cystic, thin-walled structure in the


Intraoperative image demonstrating bladder consistent with a ureterocele
a ureterocele (arrow)

CHAPTER 2: PEDIATRIC UROLOGY 37


* Evaluation for a duplex system (renal * Images should be obtained in the early phases of
parenchyma dividing central sinus echoes or filling, since the ureterocele may efface or evert
different degrees of hydronephrosis between the into the ureter appearing to be a diverticulum
upper and lower poles) (Figure 11) later in filling
* Identification of hydroureteronephrosis: * Evaluate for reflux into ipsilateral and contralat-
• If in a duplex system, hydroureteronephrosis eral renal unit
is typically seen in the upper pole
• If the ureterocele is not associated with upper • DMSA renal scan:
pole hydronephrosis, consider nonobstructive * Evaluate the function of the associated renal
ectopic ureterocele or ureterocele dispropor- unit to see if it is worth saving
tion (atrophic upper pole) * Can also use DTPA or MAG-3 to get renal func-
• Hydronephrosis associated with the lower tion as well as the degree of obstruction
pole is due to VUR
* Assessment of the amount and quality of the • IVP:
parenchyma of the ipsilateral renal unit * Commonly the function of the renal moiety
* Evaluation of the contralateral kidney for asso- associated with the ureterocele (upper pole) is
ciated anomalies poor, resulting in delayed excretion of contrast
or no excretion at all
• VCUG: * The associated renal moiety is deviated laterally
* Ureterocele appears as a smooth, broad-based from the spine due to the presence of
filling defect located near the trigone hydronephrosis, pushing the lower pole laterally
and inferiorly
* Due to the poor function of the upper pole, only
the lower pole calyces are visualized, creating
the “drooping lily” sign
Figure 11

* Due to the association with the dilated upper


pole ureter, the lower pole ureter is laterally
US demonstrating duplication

deviated, notched, and travels in a serpiginous


of right collecting system

manner
* A “cobra head” sign is seen when contrast fills
the ureterocele, creating a radiodense structure
outlined by the radiolucent ureterocele wall

• MRI, CT scan, or cystoscopy: if anatomy is


very complex

Treatment

• Goal:
* Preserve renal function if possible
* Minimize infection, obstruction, and reflux
* Maintenance of urinary continence

• Antibiotic prophylaxis: high risk of infection

• Surgical treatment:
* Endoscopic management: transurethral
US demonstrating duplication of right collecting sys-

incision of ureterocele
tem, with moderate hydronephrosis in the upper pole

• Effective in relief of the obstruction


(arrow) but no hydronephrosis in the lower pole

38 EDUCATIONAL REVIEW MANUAL IN UROLOGY


5. Renal Cystic Diseases

• Risk of developing reflux into the ureterocele


system
Definition

• Minimal morbidity and outpatient Microscopic or macroscopic fluid-filled sacs lined


procedure with epithelium.
• Effective for intravesical ureterocele but less
effective for ectopic ureterocele • Located at various sites along the nephron
• Its role as the definitive treatment for uretero-
cele is very controversial; most likely will • May or may not communicate with the glomeru-
need secondary procedure lus, tubule, collecting duct, or calyx
• Should be considered as first treatment of
patients who present with urosepsis • Located diffusely throughout the kidney; or only
* Open surgical repair in one segment, in one kidney, or both
• Options:
* Heminephrectomy and upper pole Etiology
ureterectomy with or without
ureterocele excision Most renal cystic conditions—except for MCDK
* Ureteroureterostomy and benign multilocular cysts—arise from nephrons
* Excision of the ureterocele with or collecting ducts after they have formed.
reimplantation
• Definitive treatment but higher morbidity • Genetic
• Complications: recurrent reflux and UTIs * Autosomal-recessive polycystic kidney
• Controversial as to whether upper tract disease (ARPKD)
surgery is better than lower tract surgery * Autosomal-dominant polycystic kidney
disease (ADPKD)
* Juvenile nephronophthisis (autosomal recessive)
* Medullary cystic disease (autosomal dominant)
* Congenital nephrosis (autosomal recessive)
* Familial hypoplastic glomerulocystic disease
(autosomal dominant)
* Syndromes with cysts (such as tuberous
sclerosis and von Hippel-Lindau disease)

• Nongenetic
* Multicystic dysplastic kidney (MCDK)
* Benign multilocular cyst (cystic nephroma)
* Simple cysts
* Medullary sponge kidney
* Sporadic glomerulocystic kidney disease
* Acquired renal cystic disease (such as end-stage
renal disease, ESRD, and dialysis)
* Calyceal diverticulum (pyelogenic cyst)

• Most common: simple renal cysts, MCDK,


ADPKD and ARPKD (Table 1)

CHAPTER 2: PEDIATRIC UROLOGY 39


Table 1

Epidemiology, Diagnosis, and Associated Anomalies

Radiologic Findings Associated Anomalies Incidence

ARPKD Large, homogenous, echogenic Hepatic fibrosis with 1:5000-40,000


kidneys on US; functioning kidneys biliary atresia
with radial streaking (sunburst pattern)
on IVP

ADPKD Large kidneys with scattered Diverticulitis: liver, spleen, 1:500-1000


macrocysts on US; may resemble and pancreatic cysts;
ARPKD on US and IVP intracranial aneurysms

MCKD Diffuse cysts of various size without 20%-40% with contralateral Most frequent
a larger central cyst or visible VUR; 3%-12% with contralateral cystic disease
communications between cysts on US; UPJ obstruction; rare in newborn
no function on DMSA or IVP hypertension or malignancy

Simple cysts Cysts with normal renal parenchyma None Very common;
and kidney size on US or CT; no increases with
septation, irregular margins, or age
calcifications; no enhancement
on CT or MRI

Symptoms Treatment

• Usually detected during routine prenatal US • ARPKD: requires treatment for portal and sys-
evaluation or from a family history temic hypertension, congestive heart failure, and
renal and hepatic failure
• For ARPKD, ADPKD and MCKD, may present
with abdominal mass • ADPKD: requires treatment of systemic hyperten-
sion, UTIs, loin/back pain, and renal failure
• For ARPKD and ADPKD, may present with
hypertension, proteinuria, or hematuria • MCKD: tends to regress spontaneously; surgical
removal if fails to regress, or hypertension
• Usual age at clinical presentation for ARPKD is
around the newborn period. While for ADPKD is • Simple cysts tend not to progress; surgical treat-
3rd to 5th decades ment (unroofing or percutaneous drainage) if
patient has pain or hypertension
• Large simple cysts can cause pyelocalyceal
obstruction or hypertension

40 EDUCATIONAL REVIEW MANUAL IN UROLOGY


6. Posterior Urethral Valves

• Bladder: poor sensation, hypercontractility, low


compliance with progression to myogenic failure,
Definition

A membrane that arises from the verumontanum on contributing to incontinence and incomplete emp-
the floor of the urethra, extending toward the bul- tying
bomembranous junction and attaching to the urethra
throughout its circumference. Symptoms

Has no active function, but is a simple barrier to • Newborns:


urine flow. * Detected on prenatal US
* In utero findings include: oligohydramnios,
Etiology hydroureteronephrosis, echogenic kidneys,
thickened bladder wall, and dilated posterior
It is hypothesized that the terminal ends of the urethra
Wolffian ducts mismigrate and are integrated into * Postnatal findings:
the urethral wall abnormally, resulting in obliquely • Respiratory distress due to pulmonary
oriented ridges that act as one-way valves, impeding hypoplasia
urine flow from the bladder. • Potter’s facies and bowed/deformed limbs
secondary to the oligohydramnios
Epidemiology • Renal insufficiency resulting in failure to
thrive, lethargy, and poor feeding
• Incidence: 1 in 5,000–8,000 males • Abdominal mass due to dilated bladder and
hydroureteronephrosis
• Accounts for up to 10% of obstructions • Ascites: urinary obstruction accounts for 40%
diagnosed prenatally of all cases of neonatal ascites

Associated Anomalies: • Older children: present much later in life


Due to the Consequences of PUV * UTI or voiding dysfunction (incontinence)
* Renal insufficiency in 35% while ESRD
• Lung-pulmonary hypoplasia in 10%

• Kidney: Diagnosis
* Obstructive uropathy: reversible renal injury
that improves with relief of obstruction; how- • US: findings include hydroureteronephrosis,
ever, may recur with bladder dysfunction echogenic kidneys, thickened bladder wall, and
* Dysplasia: dilated posterior urethra (Figure 12)
• Permanent level of injury that limits renal
growth and may lead to progressive renal fail- • VCUG: Primary modality for diagnosis
ure and hypertension (Figure 13)
• Associated with VUR (VURD syndrome); * Dilated elongated posterior urethra with an ele-
acts as pop-off mechanism protecting the con- vated bladder neck
tralateral kidney from the high bladder pres- * Thickened and trabeculated bladder
sure * The valve structure is often visible
* Tubular injury: resulting in inability to concen- * VUR—occurs in 50% of cases
trate, which may lead to diabetes insipidus
• Radionuclide renography: DMSA to evaluate
• Ureters: poor contractility that leads to chronic renal function and MAG-3 if secondary UPJ or
hydroureteronephrosis in some cases UVJ obstruction is suspected

• Cystoscopy: limited to those cases in which


VCUG is equivocal

CHAPTER 2: PEDIATRIC UROLOGY 41


Figure 12

US demonstrating a thick-walled bladder

US demonstrating a thick-walled bladder (A); dilated ureters (B); and severe hydronephrosis with renal
parenchymal thinning (C). These findings are suggestive of bladder outlet obstruction such as PUV.

• Laboratory evaluation:
* Need to obtain serum creatinine, blood urea
Figure 13

nitrogen, and electrolytes


* Serum creatinine after 48 hours reflects new-
VCUG demonstrating characteristic

born rather than maternal renal function


findings of PUV

* Nadir serum creatinine >1.0 predicts high


likelihood of developing ESRD
* Need to follow serum bicarbonate, sodium, and
potassium levels carefully due to the high inci-
dence of salt loss and acidosis

Treatment

• Initial management:
* Urinary catheter drainage
* VCUG for diagnosis
* Follow electrolytes and creatinine until stabi-
lized

• Endoscopic valve ablation:


* Disrupt the valves using Bugbee electrode, cold
knife, hooks, balloon catheters, valvulotomes,
or laser at the 5 and 7 o’clock positions (occa-
sionally at the 12 o’clock position)
* Complete resection leads to urethral and
VCUG demonstrating characteristic findings of PUV,

sphincter injuries (<5% with a limited


including a dilated elongated posterior urethra with

resection)
an elevated bladder neck, thickened and trabecu-
lated bladder and the valve structure (arrow).

42 EDUCATIONAL REVIEW MANUAL IN UROLOGY


* Often a catheter is left for drainage for 24 hours * Antibiotic prophylaxis while awaiting sponta-
after the procedure neous resolution
* Reimplantation in a valve bladder has a high
• Cutaneous vesicostomy: complication rate if the bladder is not well
* Indicated in infant too small for safe instrumen- rehabilitated
tation, or too ill * High-grade VUR associated with dysplastic
* Does not cause permanent loss of bladder renal units do not need to be removed unless
capacity patient has recurrent infection

• Upper tract diversion: • Management of nonrefluxing hydronephrosis:


* Ureterostomy or pyelostomy * 49% resolve after valve ablation
* Indicated in patients who fail to respond to * If persists after valve ablation, need to make
bladder-level drainage (persistent infection or sure that bladder function is normalized and
evidence of continued impairment of urinary drainage is adequate
drainage) * Most are due to chronic nonobstructive dilata-
* Will likely require further, more complicated tion of collecting system and high urine output
reconstructive surgery due to diabetes inspidus
* 80% of patients treated with upper tract diver- * Rare to have residual bladder outlet or UVJ
sion preserve their bladder capacity, while 69% obstruction
preserve their bladder compliance
• Management of bladder dysfunction:
• Management of secondary VUR: * Intrinsic bladder dysfunction may lead to deteri-
* 20%–32% resolve after valve ablation oration of the upper urinary tract and inconti-
* If persists after valve ablation, need to make nence (valve bladder syndrome)
sure that bladder function is normalized and * Three urodynamic patterns identified (Table 2)
drainage is adequate

Table 2

Urodynamic Patterns

Symptoms More Common In Treatment

Myogenic failure Overflow incontinence, Post-pubertal Time and double


incomplete bladder emptying voiding, ␣-blockers
or intermittent
catherization depending
on severity

Detrusor Frequency and urge incontinence Older children Anticholinergics


hyperreflexia

Decreased Frequency and urge incontinence Infants Anticholinergics


compliance/
small capacity

CHAPTER 2: PEDIATRIC UROLOGY 43


7. Prune Belly Syndrome
(Triad Syndrome)

• These patterns of bladder dysfunction overlap in


the majority of the patients and change
Definition

with age Variable in severity, constellation of anomalies con-


• Management of renal dysfunction sisting primarily of deficiency of the abdominal
* Dysplasia: not treatable wall musculature, bilateral undescended testis, and
* Obstructive uropathy: urinary tract dilatation
• Provide low-pressure storage by treating
bladder dysfunction • Kidney:
• Provide adequate drainage of urine by regular * Dilated, with preserved calyceal anatomy;
emptying of the bladder with time/double rarely UPJ obstruction
voiding and/or nighttime catheter drainage * 50% with dysplasia
* Prevent recurrent infection with management of
VUR and nonrefluxing hydronephrosis • Ureters:
* Control hypertension * Dilated, tortuous, and redundant
* High risk of renal failure during puberty due to * Proximal portion more normal muscle than the
the demands of rapid somatic growth distal portions
* Predictors of poor renal function * 75% with VUR; rarely UVJ obstruction
• Evidence of renal dysplasia on US
• Age of presentation <1 year of age • Bladder:
• Bilateral VUR * Massive, enlarged with pseudodiverticulum at
• Nadir creatinine >1.0 during the first year of the urachus
life * 25%–30% with a patent urachus
* Renal transplant: good graft survival if bladder * Large capacity with normal compliance and
dysfunction is treated decreased sensation
* 50% voids normally, with normal voiding pres-
sure, normal flow rates, and low postvoid residuals

• Urethra:
*Dilated prostatic urethra but no obstruction
*Occasionally, urethral atresia or megalourethra

• Accessory sex organs:


* Prostatic hypoplasia
* Atretic, dilated, or thickened vas and seminal
vesicles
* Epididymal, efferent ductules, and rete testis
disjunction

• Testes: usually intraabdominal

• Abdominal wall (Figure 14):


* Skin, subcutaneous fat, and a single fibrous
layer on the peritoneum
*Becomes less wrinkled and more pot-bellied as
the child gets older

44 EDUCATIONAL REVIEW MANUAL IN UROLOGY


Figure 14 Epidemiology

Prune belly syndrome • Incidence: 1 in 29,000–40,000

• 95% occurring in males; females have no genital


abnormalities

• Higher incidence in twins, African Americans, and


children born to younger mothers

• Incidence decreasing, possibly due to prenatal


diagnosis and elective termination

Associated Anomalies

• Cardiac: patent ductus arteriosus, ventricular sep-


tal defect, atrial septal defect, and tetralogy of Fallot

• Pulmonary: lobar atelectasis, pulmonary hypo-


plasia, pneumothorax, pneumomediastinum

• Gastrointestinal: intestinal malrotation, intestinal


atresia/stenosis, omphalocele, gastroschisis,
Hirschsprung’s disease, imperforate anus,
hepatobiliary anomalies

• Orthopedics: pectus excavatum or carinatum,


scoliosis, sacral agenesis, congenital hip subluxa-
tion or dislocation, genu valgum, talipes equino-
varus, severe leg maldevelopment
Typical abdominal finding in an infant with prune
belly syndrome.

• Splenic torsion and adrenal cystic dysplasia

Symptoms
Etiology
• Prenatal diagnosis
• Most cases are sporadic with normal karyotype * Similar US appearance to other causes of blad-
der outlet obstruction
• Association with Turner’s syndrome, monosomy * Irregular abdominal circumference may not be
16, trisomy 13, and trisomy 18 consistently seen until 30th weeks

• Exact mechanism is not known; possible theories: • Renal insufficiency due to renal dysplasia and
* Early in utero posterior urethral obstruction hypertension
* Primary defect in the lateral plate mesoderm
* Intrinsic defect of the urinary tract leading to • Recurrent pulmonary infection due to abdominal
ureteral dilatation and fetal ascites defect and associated pulmonary anomalies
* Yolk sac defect

CHAPTER 2: PEDIATRIC UROLOGY 45


8. Urinary Tract Infections

Diagnosis This is a common cause of fever in children and a


more common cause of renal parenchymal damage.
• VCUG to evaluate for reflux and rule out other Many anomalies of the genitourinary tract predis-
causes of bladder outlet obstruction pose to UTI, but there are also many other host and
bacterial factors. Recognition of these risk factors
• Radionuclide renogram: DMSA to evaluate renal will allow for prompt diagnosis and prevention of
function or MAG-3 to evaluate adequacy of uri- recurrent infections.
nary drainage
Definition
• Chest x-ray to evaluate for associated pulmonary
anomalies • Difficult to distinguish between contamination,
colonization, and active infection
Treatment
• Recurrent infection due to:
• Management of associated pulmonary and cardiac * Unresolved bacteriuria during therapy
issues * Bacterial persistence at an anatomic site
* Reinfections
• Avoid urinary tract infection with circumcision,
antibiotic prophylaxis, and avoiding instrumentation • Cystitis:
* Infection localized to the bladder
• Commonly observation is recommended since * Can lead to upper urinary tract infection
dilatation of the kidney, ureters, and bladder most
often improve with time • Acute and focal pyelonephritis:
* Infection localized to the renal parenchyma
• Surgical intervention (supravesical diversion, * Risks of renal scarring exponentially increase
cutaneous vesicostomy, reduction cystoplasty, or with the number of episodes
ureteral reconstruction) is indicated if there is poor * >17% of children with UTI acquire renal scarring
urinary drainage or recurrent infection
• Pyonephrosis:
• Orchidopexy at approximately 6 months of age * Accumulation of purulent debris and sediment
in the collecting system
• Reconstruction of the abdominal wall for cosmesis * High risk of developing urosepsis if instrumented
and possible improvement in bladder, bowel, and * Need to rule out obstruction as an etiology
pulmonary function
• Perinephric or renal abscess:
* Tissue necrosis leading to the formation of locu-
lated areas filled with purulent debris
* Less common since the development of broad-
spectrum and powerful antimicrobials

• Renal scarring:
* Loss of functioning renal parenchyma due to
focal ischemia and the release of bacteriocidal
compounds
* Leads to renal dysfunction/failure and hyperten-
sion (renin mediated)

46 EDUCATIONAL REVIEW MANUAL IN UROLOGY


Etiology and Associated Anomalies: Epidemiology

• Pathogenesis: • 51 per 100,000 children and 174 per 100,000


* Ascending infection most common infants are hospitalized with UTI annually
* Hematogenous spread (S aureus, Candida and
tuberculosis) • Accounts for 4.1%–7.5% of febrile illness
* Lymphatic from rectal, colonic, or periuterine
areas • Accounts for 2.4%–2.8% of physician visits for
* Direct from vesicovaginal or vesicointestinal children
fistula
• Male-to-female ratio changes with age:
• Bacterial factors:
* Surface structures, pili, and fimbriae
* Most commonly caused by Gram-negative
Age M/F Ratio

bacteria, especially E coli Neonate 2.2 to 1

• Host factors:
1–12 months 0.4 to 1

* Age
1–11 years 0.1 to 1

* Gender
11–16 years 0.33 to 1

* Colonization (periurethral, preputial,


vaginal, and bowel) • Only during the first year of life do males have
* Genetics (blood group antigens) more UTIs than females
* Prior renal scarring
* Native immunity • Uncircumcised male are at higher risk than cir-
* Sexual activity cumcised males (10–20x)
* Genitourinary anomalies:
• VUR and intrarenal reflux Symptoms
• Pregnancy
• Neurogenic anomalies • Infants: no reliable clinical symptom to rule UTI in
• UVJ, ureterocele, and other cause of urinary or out as a source of infection
obstruction
* Iatrogenic: foreign bodies, catheters, IUD • Children <2 years of age: vague symptoms (fever,
irritability, poor feeding, vomiting,
• Surgically correctible causes of bacterial persis- diarrhea, abdominal distension, or jaundice); none
tence in children: are reliable predictors of UTI
* Stones
* Nonfunctioning renal units or segments • Older children: signs better localized to the urinary
* Ureteral stump after nephrectomy tract (dysuria, incontinence; suprapubic, abdomi-
* Vesicointestinal or urethrorectal fistula nal or flank pain); many will still have nonspecific
* Vesicovaginal fistula symptoms
* Necrotic papillae in papillary necrosis
* Unilateral medullary sponge kidney • Adolescents: signs better localized to the urinary
* Urachal cyst tract; need to also consider concurrent sexually-
* Urethral diverticulum transmitted diseases (chlamydia and gonorrhea)

• Fever as a marker for pyelonephritis:


sensitivity = 53%–84% and
specificity = 44%–92%

CHAPTER 2: PEDIATRIC UROLOGY 47


Diagnosis * Radiologic exams:
• US:
• Urinary specimens: * Generalized enlarged kidneys (acute
* Reliability depends upon method of collection pyelonephritis) or focal hypoechoic or
(bag < voided < catherization < suprapubic hyperechoic areas (focal pyelonephritis)
bladder aspirate) * Fluid levels, debris, or air in the collect-
* Urinalysis: ing system (pyonephrosis)
* Hydronephrosis if associated with
obstruction or VUR
Tests Sensitivity (%) Specificity (%)

* Stones and other etiology for UTI


• DMSA: defects correlating with areas of
Esterase 83 (67–94) 78 (64–92)

infection
Nitrite 53 (15–82) 98 (90–100)

* Gold standard for diagnosis of


E or N 93 (90–100) 72 (58–91)

pyelonephritis
WBC 73 (32–100) 81 (45-98)

* Acute or chronic can be determined by


Bacteria 81 (16–99) 83 (11–100)

serial exams
Any above 99.8 (99–100) 70 (60–92)

• CT scan: areas of decreased perfusion corre-


lating with areas of infection
* Urine culture:
Collection CFU Prob. Treatment

• Antibiotic therapy
Infection

* Intravenous antibiotics and hospitalization:


(%)

• Infants <1 month of age


• Urosepsis
Suprapubic Gram neg., any > 99

• Pyelonephritis in infants and younger children


Gram pos. more >1000

• Complicated by renal abscess or obstruction


than a few thousand

* Oral antibiotics and outpatient management:


• Uncomplicated UTI
Catherization >106 95

• Cystitis
104-6 Likely

• Pyelonephritis in older children and


103-4 Repeat

adolescents
<103 Unlikely

* Selection of antimicrobial agents:


• Urinary tract sepsis
Clean Catch

* 3rd generation cephalosporin with or


Male >104 Likely

without aminoglycoside
Female 3 specimens: >106 95

* Ticarcillin/clavulanate
2 specimens: >106 90

* Imipenem or meropenem
1 specimen: >106 80

* Ampicillin with or without


5 x 104 – 106 Repeat

aminoglycosides
104 – 5 x 104 sympt. Repeat

• Acute uncomplicated pyelonephritis


104 – 5 x 104 nonsympt. Unlikely

* Fluoroquinolones (older patients)


<104 Unlikely

From American Academy of Pediatrics. Committee on Quality


Improvement. Subcommittee on Urinary Tract Infection: Prac-
* Amoxicillin/clavulanate
tice parameter: The diagnosis, treatment, and evaluation of the * 3rd generation cephalosporin
initial urinary tract infection in febrile infants and young chil- • Acute uncomplicated UTI
dren. Pediatrics. 1999;103:847.
* Trimethoprim-sulfamethoxazole
(TMX-Sulfa)
* Penicillins

48 EDUCATIONAL REVIEW MANUAL IN UROLOGY


* Cephalosporin • Indicated in young infants, all males with 1st
* Nalidixic acid febrile UTI and school-age females with 1st
* Fluoroquinolones febrile UTI; controversial with some studies
* Azithromycin not recommending evaluation unless patients
* Nitrofurantoin have US or DMSA abnormalities or recurrent
infection
* Risks for antimicrobial resistance: • Not likely to be positive for VUR in adoles-
• Diabetes mellitus cent females with febrile UTI or school-age
• Recent hospitalization females with cystitis
• Recent antibiotic usage (particularly TMX- • VCUG is preferable over RNC in cases in
Sulfa) which risk of urinary abnormalities is high
• Urinary tract malformations and urethral visualization is important
• Urethral catheters and other urinary tubes * DMSA: to evaluate for function and scarring if
• Antimicrobial prophylaxis there is a history of recurrent infection, high
* Prophylaxis: grade VUR or obstruction
• Indications:
* VUR
* Lower urinary tract obstruction (UVJ
obstruction, ectopic ureter, or uretero-
cele)
* Normal urinary tract but frequent
reinfections
* After acute UTI awaiting radiologic
work-up
* Urethral instrumentation
* Immunocompromised host
* Controversial: infants with UTI before
8–12 weeks of age and patients on clean
intermittent catheterization with VUR

• Agents:
Drug Daily Age
Dosage Limitations

Nitrofurantoin 1–2 mg/kg >1 month


TMX-Sulfa TMX-1–2 mg/kg >2 month
Cephalexin 2–3 mg/kg
Amoxicillin 25 mg/kg

• Follow-up imaging evaluation:


* US: to evaluate for stones, hydronephrosis, and
other urinary tract pathology
* VCUG or radionuclide cystogram (RNC)
• To rule out reflux or bladder outlet obstruction
• Performed when urine is sterile, bladder vol-
ume is normal, and bladder irritability is no
longer present

CHAPTER 2: PEDIATRIC UROLOGY 49


9. Vesicoureteral Reflux

* VUR is prevented by the compression of the


intravesical ureter against the bladder wall,
Definition

• Occurs when urine in the bladder flows retrograde which can only be accomplished with the appro-
into the upper urinary tracts priate ureterovesical junction (UVJ) anatomy

• Grading system • VUR:


* Primary: maldevelopment or delayed maturity
of the UVJ
* Secondary: results from distortion of the UVJ
Reflux Into Reflux Into

by changes in the bladder secondary to other


Grade the Ureter the Calices

pathologies such as posterior urethral valves or


neurogenic bladder
I + distal - None
segment only

• Role of bladder dysfunction in promoting VUR


II + without + without distention
tortuosity
III + with + with mild
minimal distention
tortuosity Figure 15
IV + with + with moderate
moderate distention VCUG demonstrating intrarenal reflux
tortuosity
V + with severe + with severe
tortuosity distention

• In the absence of UTI, it does not cause scarring


associated with pyelonephritis

• Can be associated with congenital renal dysplasia

• In the presence of UTI, can lead to renal scarring


* Primarily at the poles and at the site of
compound calyces (concave architecture)
* Incidence of scarring is related to the grade of
the VUR
* Young infants have the high risk of developing
renal scarring
* Less likely in older children and adolescents
* Intrarenal reflux increases the risk of develop-
ing renal scarring (Figure 15)

Etiology

• Normal anatomy:
* The ureter passes obliquely through the bladder
wall with an appropriate submucosal tunnel
length and opens onto the trigone of the bladder
in a correct location VCUG demonstrating intrarenal reflux (arrow).

50 EDUCATIONAL REVIEW MANUAL IN UROLOGY


Epidemiology • Megacystis-megaureter association:
* Massive bilateral VUR leads to dilatation of the
• Prevalence: bladder and ureters
* 0.4%–1.8% in the pediatric population * May result in poor drainage and emptying
* 30%–50% in children who present with a UTI
* 70% in infants who present with a UTI • Association with syndromes (VACTERL: verte-
* 25% in children with a history of prenatal bral, anal, cardiac, tracheo-esophageal, renal, and
hydronephrosis limb abnormalities; CHARGE: coloboma, heart
disease, atresia choanae, retarded development,
• Genetic: genital hypoplasia and ear anomalies, and imper-
* If a parent is affected, 50%–67% of the children forate anus)
will develop VUR
* If a sibling is affected, 33%–50% of the siblings • Associated with pregnancy due to effects of mater-
will develop VUR nal hormones on the smooth muscles in the urinary
* 80% of identical twins concordant tract

• More common in Caucasians than


African Americans
Figure 16

• Male-to-female ratio:
VCUG demonstrating a right

* Infants = 3–4: 1
periureteral diverticulum

* Older children = 1: 4–5

Associated Anomalies

• Renal abnormalities:
* Dysplasia: incidence increases with high grade
of VUR
* Scarring: associated with increased episodes of
UTI
* MCDK and renal agenesis

• Primary and secondary UPJ obstruction:


* Incidence of 9%–18%
* Primarily with high-grade VUR

• Ureteral duplication:
* Commonly associated with complete ureteral
duplication
* Most often in the lower pole
(Weigert-Meyer Law)

• Periureteral diverticulum (Figure 16):


* Distorts normal UVJ anatomy, leading to VUR
* Does not significantly alter the resolution rate
unless it is large in size VCUG demonstrating a right periureteral
diverticulum (arrow) with VUR.

CHAPTER 2: PEDIATRIC UROLOGY 51


Symptoms • Spontaneous resolution of VUR is common
• Sterile reflux is benign
• Following evaluation for UTI, prenatal • Extended use of prophylactic antibiotics is
hydronephrosis, or family history of reflux benign
• Prevention of UTI will prevent progressive
• Less common modes of presentation include renal damage and hypertension
hypertension, renal insufficiency/failure, inciden- * Antimicrobial prophylaxis may be discontinued
tal findings of a small or scarred kidney, or in children >7–8 years of age since the risk of
hydronephrosis observed on radiological imaging renal scarring in these children is low
tests such as US, CT, or MRI * Alternatively, low grade reflux (1–2) may be
observed without antibiotic prophylaxis, but
Diagnosis: need to make sure there is no voiding dysfunc-
tion or other risk factors for recurrent UTI
• VCUG or RNC:
* Gold standard for diagnosing VUR • Surgical management
* Also helps to identify other bladder and urethral * Indications:
anomalies that may cause 2nd VUR • Breakthrough UTI on prophylactic antibiotic
* VCUG provides better anatomic details and • Lack of compliance with medical manage-
more complete grading of the VUR ment
* RNC requires less radiation exposure and more • Failure to resolve with time
sensitivity in detecting VUR, but provides less • High-grade reflux with decreased renal func-
anatomic details (Figure 17) tion
* Obtain postvoid films to evaluate for concomi- • New renal scar formation while on medical
tant UPJ obstruction management
• Parental preference
• US: cannot rule in or out the presence of VUR * Rationale for management:
• High-grade reflux is less likely to resolve
• MSA renal scan to evaluate for renal scarring or spontaneously
dysplasia in high grade VUR (Figure 18) • The success rate with surgical correction is
high
Treatment * Types:
• Open surgery:
• Spontaneous resolution: * 98% success rate
* Dependent on grade: * Types:
• Grade 1: >90% • Intravesical (Politano-Leadbetter,
• Grade II: 60%–85% Cohen, Glenn-Anderson)
• Grade III: 50% • Extravesical (Modified Lich-Gre-
• Grade IV: 25%–40% goir/detrusorrhaphy technique)
• Grade V: <20% * Requires hospitalization and recovery
* Dependent on age and sex: time
• Infants and young children have the highest • Laparoscopic:
chance of resolving * Mimics open surgical techniques
• Males are more likely to resolve than females * Robotic assistance significantly facili-
* Heavily influenced by bladder/bowel function tates creation of the submucosal tunnel
and the ureteral anastomosis
• Medical management: * Reduces hospitalization and recovery
* First-line treatment with daily low-dose time
prophylaxis
* Rationale for management:

52 EDUCATIONAL REVIEW MANUAL IN UROLOGY


• Endoscopic:
* Success rate dependent on the grade of
Figure 17

reflux
* No hospitalization required and short
RNC demonstrating right VUR

recovery time
* Potential for recurrence of reflux and
UTI in follow-up following documented
successful endoscopic treatment

* Complications:
• Persistent reflux: need to rule out bladder
dysfunction as a cause
• Contralateral reflux:
* Incidence: 5%–10%
* Bilateral ureteral reimplantation for
unilateral VUR is not needed unless
there is a history of bilateral VUR
• Obstruction:
*Acute obstruction
• Due to edema
• Treatment with double J stent or
RNC demonstrating right VUR but cannot provide

nephrostomy tube
much other anatomic details compared to VCUG.

• The majority will resolve without


requiring additional surgery
* Late obstruction due to ischemia or
twisting of the ureter or stenosis of the
orifice
Figure 18

• Follow-up:
DMSA demonstrating kidneys

* Reflux nephropathy
* 90% will have diminished GFR
* 10% will develop ESRD
* During pregnancy, associated with increased
rates of pyelonephritis, toxemia, preterm deliv-
ery, fetal growth retardation, fetal loss, and
decreased maternal renal function
* Hypertension: in 10% of children and 50% of
adults with renal scarring

DMSA demonstrating a poorly functioning, small right


kidney and a left kidney with scarring (arrow) from a
patient with VUR.

CHAPTER 2: PEDIATRIC UROLOGY 53


10. Further Reading

Book Chapters

Caldamone AA, Woodard JR. Prune belly syn- Shortliffe LMD. Infection and inflammation of the
drome. In: Wein AJ, Kavoussi LR, Novick AC, pediatric genitourinary tract. In:Wein AJ, Kavous-
Partin AW, Peters CA, eds. Campbell-Walsh Urol- siLR, Novick AC, Partin AW, Peters CA, eds.
ogy. 10th ed. Philadelphia, PA:WB Saunders; Campbell-Walsh Urology. 10th ed. Philadelphia,
2011:3310-3324. PA:WB Saunders; 2011:3085-3122.

Casale AJ. Posterior urethral valves. In:Wein AJ,


Kavoussi LR, Novick AC, Partin AW, Peters CA,
eds. Campbell-Walsh Urology. 10th ed. Philadel-
phia, PA:WB Saunders;2011:3389-3410.

Khoury A, Bagli DJ. Vesicoureteral reflux. In: Wein


AJ, Kavoussi LR, Novick AC, Partin AW, Peters
CA, eds. Campbell-Walsh Urology. 10th ed.
Philadelphia, PA:WB Saunders; 2011:3267-3309.

Lee RS, Borer JG. Perinatal urology. In: Wein AJ,


Kavoussi LR, Novick AC, Partin AW, Peters CA,
eds. Campbell-Walsh Urology. 10th ed. Philadel-
phia, PA:WB Saunders; 2011:3048-3066.

Peters CA, Chevalier RL. Congenital urinary


obstruction: Pathophysiology and clinical evalua-
tion. In: Wein AJ, Kavoussi LR, Novick AC, Partin
AW, Peters CA, eds. Campbell-Walsh Urology.
10th ed. Philadelphia, PA:WB Saunders;
2011:3028-3048.

Peters CA, Schlussel RN, Mendelsohn C. Ectopic


ureter, ureterocele and ureteral anomalies. In: Wein
AJ, Kavoussi LR, Novick AC, Partin AW, Peters
CA, eds. Campbell-Walsh Urology. 10th ed.
Philadelphia, PA:WB Saunders; 2011:3236-3266.

Pope IV JC. Renal dysgenesis and cystic disease of


the kidney. In:Wein AJ, Kavoussi LR, Novick AC,
Partin AW, Peters CA, eds. Campbell-Walsh Urol-
ogy. 10th ed. Philadelphia, PA:WB Saunders;
2011:3161-3196.

Shapiro E, Bauer SB, Chow JS. Anomalies of the


upper urinary tract. In:Wein AJ, Kavoussi LR,
Novick AC, Partin AW, Peters CA, eds. Campbell-
Walsh Urology. 10th ed. Philadelphia, PA:WB
Saunders; 2011:3123-3160.

54 EDUCATIONAL REVIEW MANUAL IN UROLOGY


11. Questions

1. On prenatal US during the third trimester, 4. What is the most common abnormality in
what are the AP diameter criteria used to renal function associated with posterior
classify moderate hydronephrosis? urethral valves?

A. <7 mm A. Urine concentration defect

B. 7–<9 mm B. Increased active sodium absorption


from the descending limb of the loop of
C. 10–15 mm Henle

D. 15–20 mm C. Decreased calcium absorption from


ascending limb of the loop of Henle
E. >20 mm
D. The presence of heavy proteinuria
2. What is the reported incidence of vesi-
coureteral reflux in children with prena- E. None of the above
tally diagnosed hydronephrosis?
5. Which of the following is the most impor-
A. 50–70% tant in the treatment of VUR?

B. 40–60% A. Age of the patient

C. 5–20% B. Grade of reflux

D. <5 % C. Laterality

E. 1–2% D. Breakthrough infection

3. What is the most appropriate antibiotic E. All of the above


used for prophylaxis in a newborn with pre-
natal hydronephrosis? 6. At 30 weeks’ gestation, bilateral
hydroureteronephrosis, bladder distension,
A. Amoxicillin and oligohydramnios were detected.
Which of the following is the more likely
B. Suprax cause of this condition?

C. Bactrim A. Ureteropelvic junction obstruction

D. Nitrofuratoin B. Ureterocele

E. Ciprofloxacin C. Vesicoureteral reflux

D. Posterior urethral valve

E. Ectopic ureter

CHAPTER 2: PEDIATRIC UROLOGY 55


7. 38-week-gestation newborn with posterior
urethral valves has a serum creatinine of
1.6 mg/dl. That level:

A. Is an ominous predictor of future renal


function

B. Will decrease with completion of


nephrogenesis

C. Initially falls with a rapid rise in GFR

D. Will result in increased active sodium


absorption from the descending limb of
the loop of Henle

E. Is not reflective of the degree of renal


function impairment

Answers:

1. C.

2. C.

3. A.

4. A.

5. E.

6. D.

7. E.
The creatinine in a newborn is reflective of maternal
renal function and is not representative of the degree
of renal impairment or lack thereof due to the
obstruction.

Haycock G. Perinatal nephrourology. In: Gearhart


JP, Rink RC, Mouriquand PDE, eds. Pediatric Urol-
ogy. Philadelphia, PA: WB Saunders Co; 2001: 14-
26.

56 EDUCATIONAL REVIEW MANUAL IN UROLOGY


Chapter 3:
Pediatric Urological
Oncology
Martin A. Koyle, MD, FAAP, FACS

Contents

1. Wilms’ Tumor (WT)

2. Neuroblastoma

3. Rhabdomyosarcoma

4. Testes Tumors

5. Conclusion

6. References

7. Questions

CHAPTER 3: PEDIATRIC UROLOGICAL ONCOLOGY 57


Introduction: 1. Wilms’ Tumor (WT)

• Overall childhood cancer survival rate is >70% Etiology

• Price to pay for survival = increased risk of • This tumor is also called a nephroblastoma.
long-term sequelae
• First described by Max Wilms in 1889.
• 1 in 450 young American adults are childhood
cancer survivors, of whom 75% will have a • Although surgery and then radiation (1915) were
chronic health problem by age 40, and one-third of the predominant modalities utilized in therapy of
these health problems may be life-threatening this neoplasm for the first half of the 20th century,
Farber’s utilization of actinomycin ushered in the
• Children’s Oncology Group (COG) attempting to modern era of chemotherapy.
develop long-term follow-up guidelines
• The improved survival in WT reflects a success
• SMNs: 6x more common than general population story of multimodal therapy and multicenter,
with 3%–6% incidence within 20 years of original cooperative studies.
diagnosis
• Universally fatal prior to the 20th century,
but now with an expected 90% survival rate.

Epidemiology

• WT is the most common GU tumor of children


with an incidence of 6-7:1,000,000.

• More than 75% of the time, the age at diagnosis is


< 5 years.

• The ratio of male to female occurrence is equal,


but the tumor is more common in African
Americans than Caucasians.

• In 5% of cases, this tumor is bilateral and may be


multifocal in 12%.

• Although multicystic dysplastic kidney has been


surmised to be a risk factor, the National Wilms’
Tumor Study (NWTS) has suggested that >2,000
prophylactic nephrectomies would be required in
order to prevent a single case of WT.

Genetics

• Genetics are adding to our understanding of WT,


in particular with the syndromic patients and those
who are younger or with bilateral disease.

58 EDUCATIONAL REVIEW MANUAL IN UROLOGY


• In particular, the following loci have proved to be • Despite being present in only 5% of specimens,
of substantial importance: the anaplastic or unfavorable histological variant
(UH) is associated with >50% of the deaths.
* 11p13 Loci
* 11p15 Loci • Beckwith pointed out the importance of clusters of
* 16q Loci persistent nephrogenic blastemal cells termed

• Genetic influences so far are found in only 10%


nephrogenic rests.

of all cases of sporadic WT. • In clusters, they are histologically identical to WT


and have been termed nephroblastomatosis.
• Syndromes may be graded as high, moderate and
low risk of developing WT, as shown below: • These entities are found in <1% of routine infant
autopsies where WT is not found, but are found in
* High Risk (>20%) 45% of unilateral WT and 100% of bilateral
• WT1 deletions
(including WAGR syndrome)
cases.

• Truncating and pathogenic missense


WT1 mutations (including Denys-Drash
Clinical Presentation

syndrome) • Usually, the clinical presentation is that of a


• Familial Wilms’ Tumor healthy child with an abdominal mass.
• Perlman syndrome
• Mosaic variegated aneuploidy • Hematuria is found in <15% and usually is micro-
• Fanconi anaemia D1/Biallelic scopic only.
BRCA2 mutations
• Imaging has assumed an important role in the care
* Moderate Risk (5%-20%) of these patients and proper radiological evalua-
• WT1 intron 9 splice mutations tion is essential.
(Frasier syndrome)
• Beckwith-Wiedemann syndrome • Often, an ultrasound will accompany the child
• Simpson-Golabi-Behmel syndrome and demonstrate a solid or heterogeneous renal
caused by GPC3 mutations/deletions lesion.

* Low Risk (<5%) • It is particularly helpful if the ultrasonographer


• Isolated hemihypertrophy also evaluates the status of the renal veins and
• Bloom syndrome inferior vena cava if a WT is suspected.
• Li-Fraumeni syndrome
• CT scan has allowed excellent evaluation of the
contralateral kidney, liver and gross lym-
phadenopathy and will also include a baseline
Histopathology

• The pathology of WT reveals a classic triphasic evaluation of the chest. There is increased risk for
pattern composed of 3 elements in the favorable developing cancers in children who have received
histology (FH) variant: increased cumulative doses of radiation.

* Epithelial • IVPs are rarely used anymore. The role of the MRI
* Blastomal is still evolving but has the virtue of less ionizing
* Stromal radiation than a CT, and better vascular anatomical
visualization.

CHAPTER 3: PEDIATRIC UROLOGICAL ONCOLOGY 59


Surgical Staging Table 1

• In North America, staging is traditionally surgical


and has been done through a chevron transperi-
WT Staging

toneal incision (Table 1). It is now acceptable to


address resection via a flank incision. Current In America (NWTS) — surgical staging
COG guidelines are not antagonistic to
laparoscopy in selected cases. However, most In Europe (SIOP) — imaging
WT are very large, as they tend to grow rapidly
(with exceptions, of course) and hence may not be
amenable to laparoscopic resection.

• In the past, the contralateral kidney in presumed


I - Confined tumor, total resection

unilateral disease was first inspected and pal-


pated. With modern imaging, this is no longer
II - Outside capsule ( bx, spill), total resection

mandatory. III - Incomplete resection, massive spill, + LN

• Ipsilateral nephrectomy is then performed with


local lymph node sampling. The latter is impor-
IV - Distant mets ( hematogenous)

tant, as absence of lymph nodes in the specimen


will lead to an automatic upstaging to Stage III,
V - Bilateral tumors

with the presumption that the lymph nodes would


have been positive. • Rupture and/or tumor spillage increases the local
recurrence rate by a factor of 6, and this along with
• If there is contiguous organ involvement, one the biopsy upstages the disease to Stage III.
should biopsy and close rather than attempting
heroic surgery with removal of other organs. • It is recommended that presurgical downstaging
with chemotherapy be employed if tumors are
• With the presumption radiologically of bilateral bilateral, unresectable, solitary kidney or have
disease, chemotherapy can be initiated without a major vascular involvement.
preliminary biopsy, but will obviously lack pre-
treatment pathology. • Adjuvant chemotherapy with 2–3 drug
chemotherapy is employed in all patients, and
• Partial nephrectomy is not routinely advocated in the role of radiation is being utilized less often,
North America compared to SIOP protocols where primarily in higher risk patients (Table 2).
large tumors can often be reduced in size consider-
ably with up-front chemotherapy. • Today, survival for FH disease stages I–III is
>90% (Table 3).
• However, in cases where there are bilateral
tumors, renal insufficiency or where there is • As with all pediatric cancer survivors, secondary
reduced renal mass or function in the uninvolved tumors are more common, especially if patients
kidney, attempts at renal preservation and partial have received chemotherapy and radiation.
nephrectomy are recommended.
• In patients who have received doxorubicin, at
• Surgery has an associated 11%complication rate, 20 years >20% of survivors will have developed
with hemorrhage and bowel obstruction being the congestive heart failure.
2 most common complications reported.
• Still, this agent (doxorubicin) in particular has
been of particular importance in clear cell sarcoma
of the kidney, a non-WT neoplasm.

60 EDUCATIONAL REVIEW MANUAL IN UROLOGY


Table 2 Table 3

NWTS V Protocol NWTS III Outcomes

Stage XRT Chemo Stage Histology Survival (%)

I/II FH/UH - EE-4A I FH 97


II FH 92
III/IV FH + DD-4A III FH 84
IV FH 83
II-IV, focal anaplasia + DD-4A I-III UH 68
IV UH 55
II-IV, diffuse anap. + Regimen 1 All FH 89
CCSK -- 75
I-IV CCSK/rhabdoid + Regimen RTK Rhabdoid -- 26

EE-4A = dactinomycin and vincristine FH = favorable histology


DD-4A = dactinomycin, vincristine and doxorubicin UH = unfavorable histology
Regimen 1 = dactinomycin,vincristine,
doxorubicin, cyclophosphamide and etoposide
Regimen RTK = carboplatin, etoposide and
cyclophosphamide

Figure 1

Wilms’ tumor algorithm NWTS-5

Hx & Px
Suspect abd mass
RUS (IVC)
Suspect WT
CT (MRI);
(?MRI)Chest
;Chest

Unilateral tumor Confirm surgically;


Bilateral or solitary kidney
Place chemo line
Nephrectomy Goal - renal
unless not possible preservation!!!
- Bx Stage & Histology Bx. vs. Partial Nx

I & II FH III & IV FH; II-IV anaplastic I-IV RTK


I anaplastic I-IV CCSK

VCR XRT XRT CTX


ACT D VCR VCR Etoposide
ACT D ACT D Carboplatin
CTX
Etoposide
EE 4A DD 4A Regimen I Regimen RTK

CHAPTER 3: PEDIATRIC UROLOGICAL ONCOLOGY 61


2. Neuroblastoma

• Utilizing it in treatment protocols has led to


survival rates that approach those of WT.
Etiology

• Neuroblastoma, like pheochromocytoma, is a


• The same cannot be said for rhabdoid tumors of tumor of neural crest origin that is from the cells
the kidney, a tumor with propensity to metastasize that form the sympathetic nervous system.
early to the brain and with a dismal prognosis
(Figure 1). • Most common extracranial neoplasm: 750 new
cases/year.
• Renal cell carcinoma is also found in children but
interestingly is associated with a better survival • Most neuroblastomas tend to undergo sponta-
rate, stage for stage. In children >10 years, inci- neous regression.
dence of WT and RCC is equal.

• Congenital mesoblastic nephroma is the most


Epidemiology

common renal neoplasm in infants <3 months • It is thought that this entity may occur in as often
and is almost universally associated with a as 1:200 live births, and thus the majority of cases
benign course. are not of any significance clinically.

• Nephrectomy without chemotherapy is thus • Indeed, attempts to screen for neuroblastoma uti-
definitive in the bulk of patients with CMN. lizing VMA have led to a profound increase in
diagnosis but no change in long-term outcome.

Diagnostic Criteria

• Unlike in WT, these children appear sick and their


tumors, when abdominal, tend to cross the midline.

• Historically, calcifications were also found to


occur more often than in WT. Sadly, >70% present
with metastases at initial diagnosis.

• Unlike its sister lesion, pheochromocytoma, nore-


pinephrine is not converted to epinephrine.

• Since VMA is the metabolite of both neuroblas-


toma & pheochromocytoma, it serves as the pri-
mary tumor marker. It is easily measured in the
urine.

• Imaging is similar to that performed for WT,


although MIBG scanning can be used in equivocal
cases for diagnosis and is useful in staging as well.

• Bone marrow aspiration is performed in the diag-


nostic staging protocol, with positive aspirates
being documented in 70%. Neuroblastoma can
spread to both cortical bone and bone marrow.

62 EDUCATIONAL REVIEW MANUAL IN UROLOGY


• Prognosis is dependent on multiple factors includ-
ing: age, stage (INSS), histopathology, biological
Table 4

variables and INRG. It is worse when N-myc


amplification is present, serum ferritin is elevated
International Neuroblastoma

and when the DNA pattern is diploid. TRK A&B


Staging System

(tyrosine kinase) may be markers for angiogenesis


1. Localized tumor with complete resection and chemoresistance.

• Low-risk tumors (Stages 1 and 2 and asymp-


tomatic Stage 4S) that can be resected grossly and
2a. Localized tumor with/o complete gross resection; -

with negative N-myc, no diploidy and favorable


L.N.

2b. + Ipsilateral and - contralateral L.N. histology: >95% survival.

• Intermediate risk— in general, these are stage


2A/2B with 50% gross tumor resection and N-
3. Unresectable local tumor and/or + contralateral

myc negative. Also included are stage III who are


L.N.

<18 months old with UH or >18 months old with


FH, Stage IV <12 months old with UH or 12–18
4. Distant + hematogenous or L.N. mets

months old with diploidy and FH, and lastly, Stage


(excluding 4S)

4S who are symptomatic or who have UH or


diploidy.
4S. Stage 1, 2a, or 2b tumor with spread to skin, liver,

* Cisplatin, etoposide, doxorubicin,


and/or B.M. (limited to <1 y/o)

• As mentioned before, 70% of spot urine VMA cyclophosphamide with goal of >50%
assays will be positive, and 24-hour urines for all gross tumor resection: 70% survival
catecholamine are positive in almost all children
with this tumor. • High risk— Stage 2A/2B with N-myc amplifica-
tion, Stage III with UH, Stage IV with UHm
• Histopathology: small blue cells where MKI diploidy,18 months old, and/or N-myc amplifica-
(Mitosis-Karyorrhexus Index) is prognostically tion. Also included is Stage 4S with N-myc ampli-
important. fication.
* Higher dose chemo with more courses,
• Historically, the Evans Classification System has hope for >90% gross resection, XRT,
been replaced by the International Staging System stem cell transplant, retinoic acid:
(Table 4). 20%–40% survival

• Stage IVS—Unique. • This is still a tumor where surgical excision is of


* <1 year of age paramount importance to ultimate survival. There
* Local stage I or II but with widespread has been a push to more conservative surgery with
mets to liver, skin and bone marrow preservation of contiguous organs, including the
(<10% of cells) kidney. With intensive chemotherapy regimens, an
* Spontaneous regression is the norm but attempt to preserve all renal function is paramount.
can have high morbidity and even mortal-
ity from pulmonary issues

• Prognostically, those with adrenal tumors do better


than extraadrenal, as do children who are <1 year
of age.

CHAPTER 3: PEDIATRIC UROLOGICAL ONCOLOGY 63


3. Rhabdomyosarcoma

Etiology Imaging

• This is the most common soft tissue sarcoma • MRI (T2 images) is the best study to assess
with up to one-third arising from genitourinary pelvic tumors.
tract sites.
• Due to costs—and the necessity often times for
• It is derived from mesenchymal origin and sedation with MRI—CT and ultrasound are
demonstrates evidence of skeletal muscle sometimes used with greater frequency than MRI
differentiation. in monitoring therapy.

Epidemiology Treatment

• There is a bimodal distribution with peaks in the • Surgery with biopsy of the lesion and assessment
first 6 years of life and then after puberty. of local extension is the initial mode of therapy.

• Approximately 1:2,000,000 are diagnosed with • This is followed by VAC-based chemotherapy.


this tumor annually, with males and females being
affected equally. • VAC:

• Genetically, abnormalities of the 2q37 locus are *


thought to be important. *
Vincristine

*
Actinomycin D

• Spread tends to be local followed by lymphatic *


Cyclophosphamide

(20%) and least commonly by hematogenous


+ Ifosfamide

• VAC is used with/without radiation with re-imag-


ing and second-look surgeries, with the goal of pri-
(10%) routes.

• The embryonal (ERMS) pathological variant mary organ preservation.


predominates in >90%, with alveolar and pleo-
morphic variants comprising the remainder. • The latter is possible in two-thirds of cases but the
function of the preserved organ in the long run
may be far less desirable than hoped in the major-
ity.
Clinical Presentation

• The clinical presentation is dependent on the site


of origin and hence may range from bleeding to
stranguria, with or without hydronephrosis and
Staging

renal sequelae: • Staging is now based on the TNM system rather


than the Intergroup Rhabdomyosarcoma (IRS)
* Bladder system.
* Prostate
* Vagina
* Uterus
* Perineum

• It is also dependent on the size of the tumor.

64 EDUCATIONAL REVIEW MANUAL IN UROLOGY


• IRS was based on surgical resectability (Table 5). • Survival overall has improved from 25% to >70%
over the last quarter century, with >90% survival
• Controversy still surrounds the management of in low-stage, favorable histology patients (Table 6
small residual masses after therapy, especially if and Figure 2 ).
the pathology is mature rhabdomyoblasts.
Table 5

TMN-RMS

Stage Site T Size N M

I Non prost/bladder T1-T2 a or b 0, 1, x 0

II Blad/prost/others T1-T2 a 0, x 0

III Blad/prost/ T1-T2 a 1 0


others T1-T2 b 0,1, x 0

IV All T1-T2 a or b 0, 1 1

T1 = confined; T2 = extension
Size: a = <5 cm diam.; b = >5 cm diam.
Nodes: 0 = neg., 1 = pos., x = unknown
Mets: 0 = no distant mets, 1 = mets present

Figure 2 Table 6

Pelvic rhabdomyosarcoma algorithm Treatment Protocols and Outcomes

Clinical Group IRS I IRS II IRS III

I – Favorable 81 85 93

II – Favorable 73 88 54

III – Favorable 70 79 89

I/II – Unfavorable 57 71 80

IV 26 27 27

Overall 55 63 71

CHAPTER 3: PEDIATRIC UROLOGICAL ONCOLOGY 65


4. Testes Tumors

• With a normal AFP, enucleation is possible and is


the preferred treatment (Figure 3).
Etiology and Epidemiology

• Testes tumors in children occur in roughly


2–3:1,000,000 annually and represent 1%–2%
of pediatric neoplasms.
Figure 3

• Whereas historically the majority of such tumors


Surgical flowchart for prepubertal

were thought to be yolk sac tumors (YST), recent


testis tumors

data have lent doubt to this theory. Serum AFP

• Teratoma is now thought to be the most com-


Elevated Normal

mon variant, with YST incidence being similar to


No
Age >1 year? Ultrasound reveals

epidermoid cysts and testicular stromal tumors.


Yes salvageable testis

No Yes

• The bulk of YST are localized, especially in


Tumor excision

infants, and unlike adult germ cell neoplasms,


with frozen section

hematogenous spread is more common than the


lymphatic route.
Other
Yolk sac
Teratoma benign tumor
tumor

Adjacent
Tumor Markers

• Alpha-fetoprotein (AFP) is elevated in >90% of


parenchyma
prepubertal?

those diagnosed with YST, and beta hCG is not a


No Yes

useful marker to follow in prepubertal testicular


Leave remnant testicle

neoplasms.
Orchiectomy

• It should be remembered that in infant males, AFP


• Leydig cell tumors are the most common stro-
is elevated for up to 6–9 months postpartum and is
mal tumors of the testis in children.
also elevated in tumors such as hepatoblastoma.
• They should be suspected by the classic clinical
triad:
Surgical and Medical Treatment

• Since 85% of tumors are confined, radical


* Precocious puberty
orchiectomy is usually curative.
* Testis mass (may be small though)
* Elevated serum testosterone and urinary 17
• Routine lymph node dissection is not indicated.
ketosteroids
• Platinum-based chemotherapy is the mainstay of
• Like teratoma, these should be managed with enu-
therapy for metastatic disease, with survival rates
cleation whenever possible.
similar to those reported in the adult population.

• Teratoma should be suspected if an ultrasound of


the testis reveals a heterogeneous lesion with
internal echoes (calcification) and AFP is not ele-
vated for the patient age.

• In children <2 years, these tumors behave in a uni-


formly benign fashion.

66 EDUCATIONAL REVIEW MANUAL IN UROLOGY


Premalignant Tumors Recent Issues

• Gonadoblastoma is an interesting premalignant • Testicular microlithiasis is found to be present in


tumor that occurs primarily in patients with disor- 0.5%–1% of all men and is being discovered more
ders of sexual differentiation who have abnormal frequently in children. 80% of cases are bilateral.
intraabdominal gonads in patients with a Y Although microlithiasis may be discovered in
>50% of testes removed for cancer, there are no
compelling data that associate microlithiasis as an
genotype.

• More than 80% of such patients are phenotypi- etiology for tumor development. Hence, until fur-
cally female with a 46, XY chromosome makeup. ther data are available, at a minimum TSE must be
reinforced strongly.
• The gonads are streak in 22%, dysgenetic in 18%
and indeterminate in the remaining majority. • Testicular dysgenesis syndrome (TDS) has been
described by Skakkebæk. As a result of genetic
• As many as one-third of these tumors are bilateral. and environmental influences, there is a variable
effect on all cell lines in the testes (Leydig, Germ
• As germ cell elements outgrow the stromal com- & Sertoli), with an outcome in an increasing num-
ponent after puberty, dysgerminoma (malignant ber of cases of reduced semen quality, infertility,
seminoma) can develop in 10%–60%. CIS, overt CA of the testis, hypospadias and cryp-
torchidism.
• Hence, prepubertal gonadectomy will prevent this
tumor from developing. • Lastly, the issue of the undescended testis needs to
be addressed.
• ERMS of the spermatic cord may present with
lymph node involvement in almost one-third of • The more cephalad or undescended the testis prior
patients. to orchidopexy, the higher the likelihood of malig-
nancy developing.
• Still, routine lymph node dissection is not recom-
mended. • Cancer might be developed in the contralateral
descended gonad in up to 20% of instances.
• With chemotherapy alone, and no adjuvant radia-
tion, survival rates exceed 90%. • The risk of cancer developing overall is anywhere
from 10–35x that which occurs in the patient with
• Lymphoma involves the testes in 4% of cases, normally descended testes.
while up to 25% of leukemia patients will have
testicular disease. • Recent data have refuted our prior thoughts. It has
now been shown that orchidopexy priorto puberty
• Whereas routine biopsies were performed in the significantly reduces the risk of malignancy.
past, this is now done only if there is clinical evi-
dence of testicular involvement.

• Further chemotherapy, gonadal radiation and even


bone marrow transplant might be necessary in
such scenarios.

CHAPTER 3: PEDIATRIC UROLOGICAL ONCOLOGY 67


5. Conclusion 6. References

• Pediatric urological oncology represents an inter-


esting constellation of entities that involve the
General References

urologist, oncologists and radiation therapists. 1. Wein AJ, Kavoussi LR, Novick AC, Partin
AW, Peters CA, eds. Campbell-Walsh
• Understanding of these tumors by each specialist Urology. 9th ed. Philadelphia, PA: Saunders
and interaction with one another is of utmost Elsevier; 2007.
importance in assuring that the patient receives the
most appropriate therapy with the least potential 2. Gearhart JP, Rink RC, Mouriquand PDE.
for morbidity and the highest potential for quality, Pediatric Urology. Philadelphia, PA: Saunders
long-term survival. Elsevier; 2010.

• Due to the small numbers of cases in general, 3. Docimo SG, Canning DA, Khoury AE. Clini-
progress has been made because of multimodal cal Pediatric Urology. 5th ed. London, Eng-
therapy and cooperative groups such as the Chil- land: Informa Healthcare; 2007.
dren’s Oncology Group (COG). Descriptions of all
the tumors of childhood and current treatment pro- 4. Hinman F Jr, Baskin LS. Hinman’s Atlas of
tocols can be found on their Web site CureSearch: Pediatric Urologic Surgery. 2nd ed. Philadel-
http://childrensoncologygroup.org/ phia, PA: Saunders Elsevier; 2009.

5. Wilcox DT, Godbole PP, Koyle MA. Pediatric


Urology: Surgical Complications and Man-
agement. West Sussex, England: Wiley-Black-
well; 2007.

6. Thomas DFM, Duffy PG, Rickwood AMK.


Essentials of Paediatric Urology. 2nd ed. Lon-
don, England: Informa Healthcare; 2008.

Wilms’ Tumor

1. Ritchey ML, Shamberger RC, Haase G,


Horwitz J, Bergemann T, Breslow NE. Surgi-
cal complications after primary nephrectomy
for Wilms’ tumor: report from the National
Wilms’ Tumor Study Group. J Am Coll Surg.
2001;192:63-68.

2. Green DM. Wilms’ tumour.


Eur J Cancer. 1997;33:409-418.

3. D’Angio GJ, Breslow N, Beckwith JB, et al.


Treatment of Wilms’ tumor. Results of the
Third National Wilms’ Tumor Study. Cancer.
1989;64:349-360.

4. Shamberger RC, Guthrie KA, Ritchey ML,


et al. Surgery-related factors and local recur-
rence of Wilms tumor in National Wilms
Tumor Study 4. Ann Surg. 1999;229:292-297.

68 EDUCATIONAL REVIEW MANUAL IN UROLOGY


5. D’Angio GJ. Pre- or post-operative treatment 3. Ortega JA, Rowland J, Monforte H, Malo-
for Wilms tumor? Who, what, when, where, golowkin M, Triche T. Presence of well-differ-
how, why—and which. Med Pediatr Oncol. entiated rhabdomyoblasts at the end of therapy
2003;41:545-549. for pelvic rhabdomyosarcoma: implications
for the outcome. J Pediatr Hematol Oncol.
Neuroblastoma 2000;22:106-111.

1. Turkel SB, Itabashi HH. The natural history 4. Raney B Jr, Heyn R, Hays DM, et al. Sequelae
of neuroblastic cells in the fetal adrenal gland. of treatment in 109 patients followed for 5 to
Am J Pathol. 1974;76: 225-244. 15 years after diagnosis of sarcoma of the
bladder and prostate. A report from the Inter-
2. Peuchmaur M, d’Amore ES, Joshi VV, et al. group Rhabdomyosarcoma Study Committee.
Revision of the International Neuroblastoma Cancer. 1993;71:2387-2394.
Pathology Classification: confirmation of
favorable and unfavorable prognostic subsets 5. Ferrer FA, Isakoff M, Koyle MA. Blad-
in ganglioneuroblastoma, nodular. Cancer. der/prostate rhabdomyosarcoma: past, present
2003;98:2274-2281. and future. J Urol. 2006;176:1283-1291.

3. O’Neill JA, Littman P, Blitzer P, Soper K,


Chatten J, Shimada H. The role of surgery in
Testes Tumors

localized neuroblastoma. J Pediatr Surg. 1. Pohl HG, Shukla AR, Metcalf PD, et al.
1985;20:708-712. Prepubertal testis tumors: actual prevalence
rate of histological types. J Urol.
4. Matthay KK, Villablanca JG, Seeger RC, et al. 2004;172:2370-2372.
Treatment of high-risk neuroblastoma with
intensive chemotherapy, radiotherapy, autolo- 2. Olive D, Flamant F, Zucker JM, et al.
gous bone marrow transplantation, and Paraaortic lymphadenectomy is not necessary
13-cis-retinoic acid. Children's Cancer Group. in the treatment of localized paratesticular
N Engl J Med. 1999;341:1165-1173. rhabdomyosarcoma. Cancer.
1984;54:1283-1287.
5. Nickerson HJ, Matthay KK, Seeger RC, et al.
Favorable biology and outcome of stage IV-S 3. Ross JH, Rybicki L, Kay R. Clinical behavior
neuroblastoma with supportive care or mini- and a contemporary management algorithm
mal therapy: a Children’s Cancer Group study. for prepubertal testis tumors: a summary of the
J Clin Oncol. 2000;18:477-486. Prepubertal Testis Tumor Registry. J Urol.
2002;168:1675-1679.

4. Shukla AR, Woodard C, Carr MC, et al.


Rhabdomyosarcoma

1. Crist WM, Anderson JR, Meza JL, et al. Inter- Experience with testis sparing surgery for
group rhabdomyosarcoma study-IV: results testicular teratoma. J Urol. 2004;171:161-163.
for patients with nonmetastatic disease. J Clin
Oncol. 2001;19:3091-3102. 5. Thomas JC, Ross JH, Kay R. Stromal testis
tumors in children: a report from the prepuber-
2. Raney RB, Anderson JR, Barr FG, et al. tal testis tumor registry. J Urol.
Rhabdomyosarcoma and undifferentiated 2001;166:2338-2340.
sarcoma in the first two decades of life: a
selective review of intergroup rhabdomyosar- 6. Pettersson A, Richiardi L, Nordenskjold A,
coma study group experience and rationale for Kaijser M, Akre O. Age at surgery for unde-
Intergroup Rhabdomyosarcoma Study V. scended testis and risk of testicular cancer.
J Pediatr Hematol Oncol. 2001;23:215-220. N Eng J Med. 2007; 356:1835-1841

CHAPTER 3: PEDIATRIC UROLOGICAL ONCOLOGY 69


7. Questions

1. Wilms’ tumor prognosis is primarily dictated 4. A 7-month-old boy is found to have a firm
by testis mass. Alpha-fetoprotein is normal.
Ultrasound reveals calcific densities sur-
A. Stage rounded by cysts and heterogeneous solid tis-
sue surrounded by normal appearing
B. Patient age parenchyma. Next step is:

C. Resectability A. Repeat alpha-fetoprotein as this is a likely


yolk sac tumor
D. Familial variant
B. Obtain beta hCG as this test is diagnostic
E. Histology of childhood embryonal cancer

2. A 3-year-old female undergoes right nephrec- C. Evaluate with monthly ultrasounds for
tomy for Wilms’ tumor. The histology is microlithiasis
described as favorable and the tumor is sur-
rounded by intralobar nephrogenic rests. This D. Perform partial orchidectomy with frozen
suggests: section with presumptive diagnosis of ter-
atoma
A. Incomplete resection
E. Stage with chest and abdominal CT in
B. An increased risk of recurrence order to decide between radical orchidec-
tomy or chemotherapy
C. An increased risk of contralateral tumor
5. A 1-month-old male is being evaluated for
D. A likely variant associated with tuberous enlarged tongue, a large right thigh and a pal-
sclerosis pable liver. Your approach would be to:

E. Likely finding of positive lymph nodes A. Reassure that this is temporary and will
regress
3. A 2-week-old newborn with no prenatal
screening is found to have a firm right abdom- B. Screen siblings
inal mass.Ultrasound confirms this to be solid
but heterogeneous right renal mass and it C. Obtain MRI
crosses the midline. The next step is:
D. Suggest serial ultrasounds
A. Resection
E. Pursue genetic testing
B. Obtain VMA and HVA

C. Perform metastatic work-up

D. Consult ophthalmology because of likely


aniridia

E. Suggest downstaging with chemotherapy

70 EDUCATIONAL REVIEW MANUAL IN UROLOGY


6. A prepubertal 12-year-old male with a unilat- 9. In a 3-year-old male with newly diagnosed
eral, palpable, undescended testicle should be solid renal tumors, left encompassing the bulk
counseled that: of the entire kidney and the right a 2-cm exo-
phytic lesion off the lower pole, modern man-
A. Orchiectomy is mandatory due to high agement would include:
likelihood of cancer
A. Exploration with bilateral biopsy,
B. Infertility is very likely due to delay in then chemotherapy
treatment
B. Left nephrectomy, right partial
C. He likely has testicular dysgenesis syn- nephrectomy and then chemotherapy
drome and male siblings should be
screened C. Up-front chemotherapy

D. Orchidopexy at this age may reduce the D. Radiation and chemotherapy up front
risk of cancer development
E. Induction external beam radiation
E. Imaging to assess size of the testis should
be obtained prior to surgery in order to 10. The following are important prognostic risk
determine potential salvageability of the factors in patients with neuroblastoma:
testis
A. Family history of disease
7. The most common pediatric testicular
tumor is: B. Associated syndromes and genetics

A. Yolk sac tumor C. Initial VMA levels

B. Leydig cell tumor D. Age and stage

C. Secondary involvement from leukemia E. Tumor crosses the midline


or lymphoma

D. Sertoli cell tumor

E. Teratoma

8. Of the following syndromes, which one is


most likely to develop Wilms’ tumor?

A. Denys-Drash

B. Beckwith-Wiedemann

C. Bloom

D. Hemihypertrophy

E. Frasier

CHAPTER 3: PEDIATRIC UROLOGICAL ONCOLOGY 71


6. D.
Recent Scandinavian data have documented that
Answers:

1. E. orchidopexy for cryptorchidism still has the possi-


Unfavorable histology represents only 10% of all bility of reducing the risk of cancer by up to 50%.
Wilms’ tumor but >50% of the fatal cases
7. E.
2. C. Although historically yolk sac tumors have been
Nephrogenic rests are fetal blastemal remnants that considered the most common testicular neoplasm,
persist and are associated with syndromes but most contemporary data have now shown that teratoma is
importantly an increased risk of bilaterality. They more common. This influences treatment, as partial
may be intralobar and hence develop earlier in orchidectomy should be considered in such patients.
nephrogenesis, or may be perilobar and develop
later. 8. A.
Denys-Drash & WAGR syndrome along with Perla-
3. A. man syndrome have a >20% chance of developing
This patient in all likelihood has a congenital WT and thus close screening protocols are manda-
mesoblastic nephroma, the most common solid tory using ultrasound until they reach 8 years of age.
renal neoplasm of the first 3 months of life. Most
behave in a benign fashion and hence nephrectomy 9. C.
is the best choice. As bilateral solid lesions usually with favorable his-
tology in a child of this age are unlikely to be any-
4. D. thing other than Wilms' tumor, modern protocols
Newer data suggest that teratoma is the most com- now recommend up-front chemotherapy without
mon childhood testicular neoplasm. Because of the need for confirmatory biopsy.
their more benign nature, they may be managed
with a testicular salvage procedure. The keys in 10. D.
doing so are the normal alpha-fetoprotein and ultra- Neuroblastoma is associated with 5 primary risk
sound suspicion of this lesion. Beta hCG is not factors: age, stage, pathology, INRG and biological
important in prepubertal testicular neoplasm, variables.
whereas alpha-fetoprotein is especially helpful as
marker for yolk sac (embryonal) tumors.

5. D.
This patient has Beckwith-Wiedemann Syndrome
and is especially at risk of Wilms’ tumor (5%–20%)
in the first 6–7 years of life. Although overall sur-
vival is likely not impacted by ongoing screening,
the probability of diagnosis at an earlier stage or
smaller lesion is greater. This is particularly impor-
tant as bilateral tumors are more common, and ear-
lier diagnosis may allow a greater opportunity to
spare nephrons with appropriate chemotherapy and
surgery, thus impacting morbidity.

72 EDUCATIONAL REVIEW MANUAL IN UROLOGY


Chapter 4:
Genital Abnormalities

Warren T. Snodgrass, MD

Contents

1. Intersexuality

2. Hypospadias

3. Cryptorchidism

4. Testicular Torsion

5. Hernias and Hydroceles

6. Adolescent Varicocele

7. Epispadias/Exstrophy

8. References

9. Questions

CHAPTER 4: GENITAL ABNORMALITIES 73


1. Intersexuality

Normal Sexual Differentiation Molecular Basis of Testicular Development

Internal and external genitalia are identical in both Genes known to influence development of the indif-
sexes until the 6th gestational week, when mas- ferent gonad from the urogenital ridge include:
culinization normally begins in males. This process
is summarized in Figure 1. • WT1, the Wilms’ tumor gene

The sex-determining region on the Y chromosome • SF1, steroidogenic factor 1


(SRY gene) initiates the active process by which the
fetus is masculinized, by stimulating the indifferent Genes involved in the transformation of the indiffer-
gonadal primordia to develop a testis. Subsequent ent gonad into a testis include:
elaboration of testosterone by fetal Leydig cells and
Müllerian inhibition substance by fetal Sertoli cells • SRY, the sex-determining region found on the
result in changes to internal ducts and external geni- short arm of the Y chromosome
talia. MIS stimulates regression of the Müllerian
ducts, leaving a vestigial appendix testis and pro- • SOX 9, SRY-related OX genes
static utricle. Testosterone acts by local diffusion to
affect development of the epididymis, vas deferens, Those contributing to ovarian development include:
and seminal vesicle. External genitalia masculinize
from androgen activity beginning in the 8th gesta- • DAX 1, dosage-sensitive sex reversal region of
tional week. Normally, systemic testosterone is con- X chromosome
verted by 5-alpha reductase II to dihydrotestos-
terone, to result in elongation of the genital tubercle Abnormalities in WT1 may explain the association
and fusion of urethral folds to create a penis, fusion between Wilms’ tumor and genital abnormalities in
of labioscrotal folds into the scrotum, and develop- WAGR and Denys-Drash syndromes. Mutations in
ment of the prostate from the urogenital sinus. Both SRY occur in up to 20% of gonadal dysgenesis
passive and active androgen actions are mediated by cases in which 46,XY individuals have a female
the androgen receptor. In the absence of androgenic phenotype. Between 80% and 90% of 46, XX
stimulation, the internal and external genital pri- males have translocation of SRY from Y to an
mordia feminize, even without estrogen production. X chromosome.

Figure 1

Masculinization

74 EDUCATIONAL REVIEW MANUAL IN UROLOGY


Figure 2

Traditional Classification of Intersex Disorders

Gonadal Sex

Ovaries Ovary Streak Streaks Testes


Testis Testis
46,xx 46,xx/46,xy 45,xo/46,xy 45,xo 46,xy
46,xx 46,xx
46,xy 46,xy

Female True MGD Gonadal Male


Pseudoherm Hermaphrodite Dysgenesis Pseudoherm

Intersexuality: Gonadal Basis of 46,XX DSD

Female DSD may arise from exogenous exposure to


Classification

The traditional classification of intersex disorders androgens due to maternal ingestion or virilizing
(Figure 2) is based upon gonadal histology, outlined tumors, both of which are rare. Therefore, most 46,
above. New terminology labels intersexuality “dis- XX DSD results from endogenous androgen stimu-
orders of sexual development” (DSD) as follows: lation secondary to congenital adrenal hyperplasia
female pseudohermaphrodite (46,XX DSD), true (CAH).
hermaphrodite (ovotesticular DSD), and male pseu-
dohermaphrodite (46,XY DSD). CAH is an autosomal recessive defect in 1 of 3
enzymes needed for cortisol synthesis within the
adrenal cortex, and is the most common overall eti-
ology for DSD.
Genital Ambiguity

Most often, DSD are suspected from an abnormal


appearance of the external genitalia detected during
the newborn examination. Generally, this can be
21 Hydroxylase Deficiency

considered inappropriate masculinization of a • 90% of CAH cases


female, inadequate masculinization of a male, or
abnormal development due to dysmorphic gonads. • Diagnosed from elevated 17-hydroxyprogesterone
and androstenedione
Examples of abnormal external genital phenotypes
that should prompt consideration of DSD include: • Two-thirds salt wasting, due to impaired aldos-
terone production
• Clitoromegaly
• Variable genital ambiguity in females, but not in
• A “phallus” without palpable gonads affected males

• An apparent female with palpable gonads The condition is due to deficiency in CYP21, a pair
of genes including the active CYP21B and inactive
• Hypospadias with cryptorchidism CYP21A on chromosome 6. Recombination
between these homologous genes accounts for

CHAPTER 4: GENITAL ABNORMALITIES 75


Figure 3

Cortisol Synthesis
Cholesterol

Pregnenolone 17 OH Pregnenolone DHEA

3b OH steroid
dehydrogenase
Progesterone 17 OH Progesterone Androstenedione

21-hydroxylase

DOC Deoxycortisol
11b-hydroxylase

Corticosterone Cortisol Testosterone

Aldosterone

nearly all 21-hydroxylase deficiency cases. Variable • Ambiguity in males, due to impaired androstene-
reduction in 21-hydroxylase activity accounts for dione production
different degrees in genital ambiguity seen clini-
cally, ranging from minimal clitoromegaly to severe Evaluation of Suspected CAH
masculinization with a completely formed phallus.
• Ambiguous genitalia: clitoromegaly, urogenital
11-Beta-Hydroxylase Deficiency sinus, no palpable gonads

• 9% of CAH cases • Karyotype: 46,XX

• Diagnosed by elevated 17-hydroxyprogesterone • Pelvic ultrasonography: uterus


and deoxycorticosterone (DOC)
• Genitogram: urogenital sinus with vagina and
• Two-thirds eventually develop hypertension from cervix
salt retention, due to DOC
• Hormone assays
3-Beta-Hydroxysteroid Dehydrogenase Deficiency
Therapy for CAH
• 1% of CAH cases
Medical. Cortisol and minerocorticoid replacement
• Diagnosed by elevated 17-hydroxypregnenolone as indicated.
and dehydroepiandrosterone (DHEA)
Surgical. Clitoroplasty/vaginoplasty.
• Severe salt wasting typical

76 EDUCATIONAL REVIEW MANUAL IN UROLOGY


to play a role in determining the ultimate phenotype
in these cases, which ranges from partial to com-
46,XY DSD

In general, these disorders arise either from defec- plete resistance to androgen activity.
tive testosterone synthesis or, more commonly, from
diminished response by target tissues to androgen Complete androgen insensitivity. Previously
stimulation. Any step in the process of masculiniza- termed “testicular feminization,” individuals are
tion depicted in Figure 1 potentially results in inade- phenotypic females often diagnosed due to palpable
quate male development. gonads, sometimes presenting as hernias (1% of
females with hernias) or from amenorrhea at
Defects in Androgen Production puberty. MIS suppresses Müllerian ducts, although
The external genitalia vary from incompletely viril- the finding of a rudimentary uterus in one-third of
ized to feminized, but often with palpable gonads. cases implies some activity of the AR is needed for
Müllerian ducts are normally suppressed by MIS complete MIS action. Similarly, a rudimentary epi-
from Sertoli cells, and typically these individuals didymis and vas may be found. Retained testes have
are responsive to exogenous testosterone stimula- increased risk of malignancy, usually seminomas.
tion.
Partial androgen insensitivity. Ambiguous geni-
talia, with poor virilization at puberty in individuals
Diagnosed by low serum testosterone despite raised as males. A trial of testosterone therapy is
hCG/LH receptor defect of Leydig cells.

normal-to-elevated LH. usually recommended for penile stimulation; those


with good responses are raised as males.

Five enzymatic steps are involved in the conversion


Testosterone synthesis deficiencies.

of cholesterol to testosterone, and defects in each


MIS Deficiency

have been associated with incomplete virilization. This sex-linked autosomal recessive condition
arises from gene mutations affecting either the pro-
5-Alpha-Reductase II Deficiency duction of MIS by Sertoli cells or its action on
receptors to which MIS binds to induce apoptosis of
This autosomal recessive condition most often has Müllerian ducts. It is commonly referred to as her-
been described among inbred populations in which nia uteri inguinale.
the infant may be raised female, but progressively
masculinizes at puberty and assumes a male identity MIS deficiency is diagnosed by encountering fallop-
(“penis at puberty”). The enzyme converts testos- ian tubes and a uterus during hernia surgery or explo-
terone to dihydrotestosterone, which has greater ration for undescended testes. Occasionally, both
affinity for the androgen receptor. Normal viriliza- testes are found together within the hernia sac. The
tion of the external genitalia depends upon DHT. vas in an undescended testis runs within the wall of
the uterus, making orchiopexy more difficult.
• Elevated T:DHT ratio

• Ambiguous external genitalia (originally


Ovotesticular DSD

described as “pseudovaginal perineoscrotal True hermaphrodites have both ovarian and testicu-
hypospadias), suppressed Müllerian ducts from lar tissue, either as distinct ovaries and testes or as
MIS ovotestes. There is geographic variability in the
karyotype, which in Africa is usually 46,XX, while
in Europe and North America is usually mosaic
46,XX/46,XY or 47,XXX/46,XY. A smaller num-
Androgen Insensitivity Syndrome

The androgen receptor (AR) is located on the long ber of patients in Europe, North America, and Asia
arm of the X chromosome. Most mutations involve are 46,XY.
the steroid-binding region so that the AR does not
bind androgens, although additional factors appear

CHAPTER 4: GENITAL ABNORMALITIES 77


• Ambiguous genitalia that tends to be virilized • Sexual infantilism

• Diagnosis depends upon demonstration of both • Somatic anomalies—typically webbed neck,


ovarian and testicular tissue by gonadal biopsy widely spaced nipples, and increased incidence of
aortic coarctation and renal anomalies, especially
• Internal ducts vary according to ipsilateral gonadal horseshoe kidney
histology, but there is usually a vagina
Those with pure 45,XO do not appear to be at risk
• Management includes removal of tissues discor- for gonadal tumors, but a minority has Y chromoso-
dant with assigned sex of rearing mal material which may lead to development of
gonadoblastoma.
• Fertility documented in females, but less certain in
males, as there usually is a decrease in testosterone Mosaicism results in various karyotypes that
secretion requiring supplementation at puberty include:

Sex Chromosome DSD • 45,X/46,XX

Mixed Gonadal Dysgenesis • 45,X/47,XXX


Classically, this term refers to patients with one
testis and a streak gonad, the majority of whom are • 45,X/46,XX/47,XXX
mosaic 45,X0/46,XY. The fact that 95% of boys
found by prenatal amniocentesis with this karyotype There is wide variation in gonads ranging from nor-
have a normal phenotype demonstrates that expres- mal ovaries to streaks, and similarly somatic abnor-
sion of mosaicism varies within tissues. malities are also less predictable.

Patients have genital ambiguity related to the streak


gonad, which has impaired testosterone and MIS
Mosaicism also accounts for karyotypes with

secretion. Even the histologically normal,


Y chromosomes.

descended testis typically has diminished testos- • 45,X/46,XY (MGD)


terone production at puberty.
• 45,X/ 47,XYY
• Ambiguous genitalia occurs within a wide spec-
trum from predominantly feminized to masculin- • 45,X/46,XY/47,XYY
ized
Gonads range from streaks to normal testes,
• Fallopian tube and unicornate uterus and vagina accounting for variation in associated phenotype
result from streak with impaired MIS secretion (female, ambiguous, male) and somatic abnormali-
ties. Dysgenetic gonads with Y chromosome have
• Streaks have increased risk for gonadoblastoma greater risk for tumor development.

Turner Syndrome Pure Gonadal Dysgenesis


The karyotype typically is 45,XO and the absence Patients with bilateral streak gonads have female
of the 2nd sex chromosome is associated with: phenotype and karyotype of either 46,XX or 46,XY.

• Female phenotype They are sexually infantile, but have normal stature,
and lack somatic abnormalities associated with
• Bilateral streak gonads absence of the 2nd sex chromosome mentioned
above. Those with Y chromosome material are at
• Short stature increased risk for gonadal tumors.

78 EDUCATIONAL REVIEW MANUAL IN UROLOGY


2. Hypospadias

Gonadoblastoma
Streaks and dysgenetic gonads associated with Y
Etiology

chromosome material are at an increased risk to The genital tubercle appears during the 4th week of
develop gonadoblastoma, a tumor comprising germ gestation as anlage for development of either the cli-
cells, Sertoli cells, and stroma. These may be toris or penis. Endodermal cells from the cloaca
detected at any age. Gonadoblastomas are benign, migrate along its ventral midline surface to create
but premalignant with a propensity to transform into the urethral plate, while proliferating mesenchyme
dysgerminomas. Removal of streaks and dysgenetic to either side establish urogenital folds. Subsequent
gonads associated with Y chromosomes is therefore phallic development is dependent upon androgenic
advised following diagnosis. stimulation during a critical time period from the
9th to 12th weeks. Testosterone produced by fetal
Leydig cells within the testes is converted by 5-
alpha-reductase II in the genital tubercle to dihy-
drotestosterone. DHT then binds to androgen recep-
tors to initiate a cascade of downstream effects that
currently are only partially understood. Androgen
stimulation causes the genital tubercle to elongate
and the urogenital (urethral) folds to migrate
toward the midline and fuse, enclosing the urethral
groove and thereby creating the urethra proximally-
to-distally.

Hypospadias represents arrested penile develop-


ment and is characterized by:

• Proximal opening of the urethral meatus

• Open glans wings

• Ventral deficiency of the prepuce (a dorsal hooded


prepuce)

• Ventral penile curvature

Associated Anomalies

Non-syndromic hypospadias usually is an isolated


defect. Arrested development of the penis thought
to result in hypospadias occurs after 9 weeks, which
is well after renal formation. Therefore, it is not nec-
essary to image the urinary tract, even in boys with
proximal hypospadias. An enlarged prostatic utricle
may occur with proximal hypospadias, but its clini-
cal significance is almost always limited to intraop-
erative difficulty in catheter placement, and so
VCUG to visualize the utricle is unnecessary.

CHAPTER 4: GENITAL ABNORMALITIES 79


• Urethroplasty. Today, distal hypospadias is most
often corrected by tubularizing the urethral plate,
Reasons for Surgical Intervention

• The proximal meatus, even when distally located, usually with dorsal midline relaxing incision to
may be associated with a deflected, spraying widen the plate (TIP repair). Proximal hypospadias
stream that is difficult to aim options are, in part, determined by means needed
to straighten ventral curvature. When the urethral
• The minority with severe ventral curvature may plate is preserved, dorsal midline incision and
not be able to have sexual intercourse tubularization (TIP) can be done as for distal
hypospadias, or a preputial skin flap can be sewn to
• To correct the abnormal appearance of the penis the urethral plate (onlay flap) to complete the
and foreskin neourethra. When the urethral plate is transected
during straightening, the neourethra can be com-
pleted in one stage using tubularized preputial
flaps (transverse island repair) or grafts, or in two
Timing of Surgery

• >3 months of age for full-term, otherwise healthy stages using preputial flaps (Byar’s flaps) or grafts
infants from the inner prepuce.

• Preferably before 18 months, when genital aware- • Glansplasty. Glans wings are approximated over
ness begins the neourethra, taking care not to make the new
meatus too small, which would result in meatal
Preoperative Hormonal Stimulation stenosis.

• No randomized study compares outcomes in • Skin closure. Traditionally by circumcision, but


patients with vs without preoperative hormonal can also be done with foreskin reconstruction to
stimulation create a natural appearance. When the scrotum
extends lateral and superior to the penis, usually in
• Proponents claim improved vascularity of skin to proximal hypospadias, and/or has a deep cleft,
be used for flaps and increased size of the glans, scrotoplasty to correct these abnormalities can be
usually in proximal cases done at the time of hypospadias repair, since the
genital tubercle giving rise to the penis has a differ-
• Intramuscular or topical testosterone is used ent blood supply than the labioscrotal folds that
according to surgeon preference form the scrotum.

Surgical Repair Complications

Routine Steps Include: Fistula. This is the most common complication fol-
• Orthoplasty. Straightening ventral curvature. lowing urethroplasty. Steps to reduce its occurrence
Degloving penile skin often resolves suspected include subepithelial, two-layer closure of the
curvature, leaving ~15% of distal, but over 50% of neourethra turning all epithelium into the lumen,
proximal cases with persistent ventral bending. and barrier layers such as dartos or tunica vaginalis
Curvature <30°most often is corrected by midline flaps interposed between the neourethra and skin
dorsal plication, whereas that >30° is straightened stitches. A fistula occurring at the coronal margin
with ventral lengthening by corporotomy with or requires reoperative distal urethroplasty and
without grafting of the resultant defect. Transec- glansplasty for correction when only a thin skin
tion of the urethral plate is sometimes needed to bridge holds the glans wings together distally. When
achieve straightening, and multiple dorsal plica- the glans is well-formed, a coronal fistula can be
tions should not be done in patients with > 30° directly closed without reoperative urethro-
curvature to try to preserve the plate. plasty/glansplasty, as can more proximal fistulas.
After excision of the fistula tract and suture closure
of the urethra, a barrier flap is placed over the repair

80 EDUCATIONAL REVIEW MANUAL IN UROLOGY


3. Cryptorchidism

before overlying skin approximation. A catheter is


not needed after simple fistula closures, but is used
Incidence

when reoperative urethroplasty is required. • 3% term infant boys

Meatal stenosis. Although stenosis may occur from • 0.8% 1-year-old boys
ischemia with flaps, in tubularization procedures it
more likely is iatrogenic, indicating the urethral Normal descent occurs at approximately the 28th
plate was not incised deeply enough or was tubular- week, and so preterm infants have a higher inci-
ized too far distally. dence of cryptorchidism at birth. The presumed
stimulus for postnatal descent is the testosterone
Urethral stricture. Most commonly seen with surge that occurs from 6–12 weeks of age, briefly
tubularized flaps or grafts, especially at the proxi- elevating testosterone levels to near pubertal val-
mal anastomosis to the native urethra. Treatment ues. Therefore, postnatal descent occurs by 4
depends upon stricture length and method of prior months and surgery for undescended testis can be
urethroplasty. DVIU may be effective for strictures performed at any time subsequently. Data from tes-
less than 1 cm after onlay flap or tubularized plate ticular biopsies indicate initially normal germ cell
repairs. DVIU is not effective after tubularized flap counts in cryptorchid testes, but after 1 year of age
or graft procedures, when the stricture is >1 cm, or the counts diminish, which provides an additional
after prior DVIU. incentive to proceed with early correction.

Diverticulum. Prepuce and penile skin are elastic Diagnosis is best made by 6 months, because after
tissues and, when incorporated into the neourethra, that time testicular retraction may falsely simulate
can distend, especially if the meatus offers some undescended testis. Several studies indicate
resistance to flow. Produces a characteristic bal- orchiopexy rates approximately double the preva-
looning with voiding and post-void dribbling. lence of undescended testis, suggesting surgeons
mis-diagnosed some cases.
Dehiscence. The repair can partially or completely
dehisce, sometimes from tension on the wound, or
use of fast absorbing sutures such as chromic catgut.
Etiology

• Occurs in various syndromes


BXO. Delayed presentation of meatal stenosis more
than a year after repair, and a white discoloration • Occurs with defects in the hypothala-
around the meatus, suggest BXO. Topical treatment mic–pituitary–testis axis, most notably in Prader-
with tacrolimus can be used as first-line therapy. Willi syndrome
Persistent BXO requires excision of all involved tis-
sues and two-stage urethroplasty using oral mucosa • 3.5x increase if another family member is affected
(not skin grafts).
• Cause for sporadic UDT—the most common man-
Most complications require additional surgery for ifestation— remains unclear
correction. The finding of any complications
requires consideration that other problems may also
exist. For example, a fistula can occur alone or in
Reasons for Therapy

association with meatal stenosis and/or a diverticu-


lum. Typically, secondary repairs are delayed at • Approximately 7% of normal men are subfertile
Fertility

least six months to allow tissues to heal from prior


surgery. Symptomatic meatal stenosis or urethral • Approximately 10% of men with unilateral UDT
stricture requiring intervention more acutely is best are subfertile
treated by a temporary proximal urethrostomy until
definitive repair can be done. • Approximately 33% of men with bilateral UDT
are subfertile

CHAPTER 4: GENITAL ABNORMALITIES 81


Malignancy. Tumors develop in< 0.5% of cryp- LH, and testosterone levels are obtained, and FSH
torchid testes during teenage or adult years. There is >3x normal with low testosterone indicates
no increased risk for tumors in the contralateral, anorchia. Those with FSH and LH <3x normal can
normally descended testis. Orchiopexy before undergo hCG stimulation testing to detect a rise in
puberty reduces cancer risk (although overall risk is testosterone, indicating a testis. MIS can also be
very low). Untreated UDTs that form tumors most measured, with any value most likely indicating
often develop seminoma, whereas tumors in testes presence of a testis.
after orchiopexy are most often non-seminomas
(due to reduction in seminoma risk). Treatment Options

Cosmesis. Patients may be disturbed, especially at hCG. Not used as primary therapy to prompt
puberty, by the appearance of an empty hemiscro- descent; sometimes considered to distinguish a
tum. retractile from undescended testis.

Orchiopexy. Routine steps include: transection of


gubernaculum; dissection of hernia sac; placement
Differential Diagnosis

Retractile testis. A normally descended testis into a subdartos pouch.


pulled intermittently into the superficial pouch out-
side the external ring by cremasteric muscular con-
traction. Typically, birth exam and exams in the first
Additional maneuvers to gain length.

6 months of life were negative, with concern for • Intraabdominal dissection to free peritoneum from
possible UDT first raised at >6 months of age. Typi- the testicular vessels up to pelvic inlet.
cally, during physical examination the testis can be
brought into the scrotum and will remain in loca- • Prentiss maneuver: Opening transversalis fascia to
tion, at least briefly. move the testis medially under the inferior epigas-
tric vessels.
Nubbin. A nonpalpable testis can indicate either the
presence of a viable testis (equally likely to be intra- • Fowler-Stephens: Transection of the internal sper-
or extraabdominal), or a nubbin representing the matic vessels, relying upon collaterals from vasal
testicular remnant from presumed intrauterine tor- vessels to nourish the testis. Usually performed
sion during descent. The majority of nubbins are well proximally from the testis. Procedures can be
within the scrotum and most often associated with done in one stage or two, with the latter consisting
compensatory hypertrophy of the contralateral of a 1st stage vessel clipping followed by
descended testis (2 cm or larger). orchiopexy several months later after collaterals
are well-established. FS orchiopexy should not be
performed after the hernia sac has been removed
and the vessels extensively dissected.
Diagnostic Tests

Imaging studies. No test is sufficiently accurate to


detect an intraabdominal testis. Ultrasound can be Staged orchiopexy. A testis that does not reach the
used specifically to detect an extraabdominal, non- scrotum despite high dissection can be placed at the
palpable testis. Ultrasound cannot distinguish distal point it then reaches, followed by a secondary
between an undescended versus retractile testis, and procedure at least 6 months later to displace it fur-
surgery is not indicated by the simple finding of a ther into the scrotum. These secondary procedures
testicle in the groin on ultrasound. are more difficult due to scarring from the first oper-
ation, but usually are successful in bringing the
Laboratory tests. None are necessary for boys with testis to the scrotum.
palpable testes. The occasional boy with bilateral
nonpalpable testes may benefit from testing to
determine the presence of at least one viable testis
before exploration, although anorchia is rare. FSH,

82 EDUCATIONAL REVIEW MANUAL IN UROLOGY


4. Testicular Torsion

Complications Prenatal and in Newborns

Atrophy. Volume loss can occur with seemingly Extravaginal Torsion


uneventful orchiopexy, or more commonly after Testicular descent occurs at approximately 28
vessel ligation for either 1 or 2 stage Fowler-Stevens weeks, and during this process torsion can occur
orchiopexy. before the testicle becomes fixed within the scro-
tum. Unlike torsion in older children, this twisting
Retraction. Ideally the testis is placed in the scro- involves the entire spermatic cord and may extend
tum without tension, but remaining tension or scar back into the abdomen, creating the appearance of
contracture occasionally results in proximal dis- an intra-abdominal vanished testis.
placement.
Nubbins
The most common presentation of prenatal torsion
is unilateral nonpalpable testis, which must be dis-
Association of Intersex With Hypospadias

tinguished from cryptorchidism with a viable testis


and UDT

Congenital adrenal hyperplasia. Any virilized in the abdomen. The fact that the vast majority of
newborn with bilateral nonpalpable testes should be these testicular remnants (nubbins) are found within
considered to be a female with CAH, regardless of the scrotum indicates that prenatal torsion usually
the extent of phallic development. occurs after the testis has descended. Twisting of the
cord to within the abdomen results in apparent
Mixed gonadal dysgenesis. Typically presents as blind-ending vas and vessels above the internal ring
hypospadias with one undescended testis. when laparoscopy is performed, but if the scrotum
is explored a nubbin usually is found. There is an
Ovotesticular DSD. True hermaphrodites may pre- 8% incidence of finding germ cells within these
sent with hypospadias and UDT. A karyotype should remnants, prompting concern regarding their
be obtained in boys with UDT and any penile removal for the potential risk of malignancy,
anomaly. although there is no known report of a testis tumor
arising from a nubbin.

Postnatal Torsion
Torsion can occur shortly before or after birth, and
is detected by presence or development of a palpa-
ble hard mass and discoloration of the hemiscrotum.
These findings indicate response to ischemia, and so
urgent exploration would not be expected to salvage
the testis, although there have been anecdotal
reports of salvage. Because testicular salvage is not
likely, the risk of anesthesia is increased in new-
borns, and propensity for contralateral torsion may
be less than in intravaginal torsion in older children,
traditionally surgery has not been recommended to
pex the contralateral side. However, concern for
contralateral torsion, which results in anorchia and
may not be detected until there is evidence of necro-
sis, today leads many pediatric urologists to
promptly explore these patients to remove the ipsi-
lateral testis and pex the other.

CHAPTER 4: GENITAL ABNORMALITIES 83


Imaging Therapy. Urgent scrotal exploration is performed to
As with intravaginal torsion in older patients, lack reduce ipsilateral torsion and pex the contralateral
of arterial flow within the testis indicates torsion, side. Orchiopexy involves 3-point fixation—medi-
but Doppler ultrasound to demonstrate intratesticu- ally, laterally, and ventrally—to secure the tunica
lar arterial blood flow is challenging in newborns, albuginea of the testis to scrotal dartos using fine
and lack of experience with the test by general radi- permanent sutures. Obviously necrotic testes are
ologists reduces reliability of reported findings. removed. When testis viability is in doubt, the testis
is detorsed and set aside to re-perfuse while the
Differential Diagnosis other side is pexed and then reexamined. A small
Rarely, testis tumors will be detected in newborns as incision can be made into the tunica albuginea to
a mass, including yolk sac tumors, teratomas, and evaluate return of blood flow. Testes with question-
juvenile granulose cell tumors or other stromal able return of flow are often preserved, but may be
tumors. Of these, only yolk sac tumors are malig- noted in follow-up to have diminished volume.
nant.
Transfer of a teenager with testicular torsion to a
Surgery pediatric facility for care should not be done, since
The possible finding of a tumor has led to recom- the inevitable delay before de-torsion increases like-
mendations that surgery for suspected torsion in lihood for testicular damage.
newborns be performed through an inguinal inci-
sion. However, since tumors are very uncommon
and usually benign, a scrotal incision can be used.
Torsion of Testicular Appendix

The contralateral testis is exposed and fixed to scro- Testicular/Epididymal Appendices


tal dartos with 3 stitches using fine permanent These structures are vestigial remnants of Müllerian
suture. and Wolffian ducts, respectively. Each has small
vessels providing blood flow, and torsion results in
In Children and Teens ischemic necrosis of the appendix.

Intravaginal Torsion Presentation. Young children typically present


Twisting of the testis suspended within the tunica with signs of scrotal reaction, including edema and
vaginalis may occur anytime after the neonatal erythema. Older children and teens may complain
period, most commonly in teens during puberty. of an onset of pain and present before there is visible
reaction to the ischemic necrosis. A “blue dot” sign
Presentation. Children often do not present until representing the discolored ischemic appendix may
there is visible evidence of scrotal reaction to a be observed through the scrotal wall. Patients with
necrotic testis. Typical complaints in older boys and torsed appendixes typically have less severe pain
teens are abrupt onset of ipsilateral testicular pain, than those with testicular ischemia, and are comfort-
with variable nausea and vomiting. Sometimes able at rest but bothered by movement of the
there is a history of prior brief episodes of similar inflamed hemiscrotum.
pain that resolved spontaneously. If the patient pre-
sents within a few hours of onset, examination Imaging. Doppler ultrasound demonstrates pre-
should demonstrate no scrotal edema or erythema. served arterial blood flow within the testis, typically
The involved testis may be found riding high in with increased flow around the testis and inflamma-
comparison to the other side, and lying transversely. tory enlargement of the ipsilateral epididymis. The
radiologist may diagnose acute epididymitis based
Imaging. Doppler ultrasound shows greatly dimin- upon the appearance of the epididymis, but this
ished or absent arterial blood flow within the testis should not be confused with infection of the epi-
compared to the contralateral side, but as discussed didymis.
below, is not needed and may delay therapy when
the diagnosis is clear from history and physical
exam.

84 EDUCATIONAL REVIEW MANUAL IN UROLOGY


Therapy. There is no need to surgically remove a
torsed appendage, as the process resolves sponta-
Figure 4

neously within days. Non-narcotic analgesics usu-


ally suffice for pain relief.
Painful Testis

The Acute Scrotum

Differential Diagnosis
Testicular torsion, torsion of appendix testis or epi-
didymis, and acute epididymitis can each present
with pain and/or signs of scrotal inflammation (Fig-
ure 4 and Tables 1–2). Other considerations include:
hydroceles, insect bites to the scrotum, idiopathic
scrotal edema, or Schönlein-Henoch purpura
(which can be associated with scrotal pain and
edema). Torsion of an appendix testis is the most
common finding in children with acute scrotum,
while epididymitis is very uncommon and most
often distinguished by fever and UTI. The acute
Table 1

scrotum creates considerable angst for emergency


room physicians, urologists, and radiologists fearful
Differential Diagnosis:

of legal ramifications from missing testicular tor-


Testicular Torsion and Torsed Appendix

sion. The most important finding is the presence or


absence of scrotal reaction, as this indicates
response to prior ischemic necrosis in the case of
Testicular Torsion Torsed Appendix

torsed appendix or testicular torsion. The absence of


such reaction implies acute onset and potential abil-
• Acute onset of pain, • Gradual onset

ity to salvage a torsed testis, and so patients with


nausea and vomiting of pain

significant pain but without scrotal reaction should


• No scrotal reaction; high- • Scrotal reaction with

have emergent scrotal exploration without delays


riding, transverse testis edema, erythema

for imaging. Those males with obvious scrotal reac-


• UA normal • UA normal

tion may benefit from imaging primarily to distin-


• US: decreased/absent • US: intratesticular

guish a torsed appendix from a missed testicular tor-


intratesticular arterial arterial flow,

sion, as the latter will need surgery to pex the con-


flow “epididymitis”

tralateral side. The rare child with epididymitis typi-


cally presents with a febrile UTI, which should be
treated with appropriate antibiotics and then subse-
Table 2

quently evaluated with renal ultrasonography and


VCUG.
Epididymitis

Gradual onset of pain

Fever, scrotal edema/erythema

UA infected

US: increased testicular/paratesticular flow.


Inflammatory enlargement of epididymis

CHAPTER 4: GENITAL ABNORMALITIES 85


5. Hernias and Hydroceles

Imaging
Although rarely needed, ultrasound confirms fluid
Etiology

During testicular descent, peritoneum coating the or bowel within a sac.


ventral surface of the testis accompanies it to the
scrotum, leaving a communication to the abdomen
that subsequently closes. Persistence or later
Therapy

reopening of the sac results in hernia or hydrocele, Hydroceles in infants resolve spontaneously. Occa-
which by definition are indirect, extending lateral to sionally, an infant is referred with extremely large,
the inferior epigastric vessels down the inguinal tense hydroceles that distend the scrotum and
canal. Weakness of the floor of the canal medial to obscure the penis. In these cases, the processus vagi-
the inferior epigastrics, creating direct hernia that nalis is either closed or quite small, or else the fluid
occurs in adults, is rare in children. would drain back in the abdomen when the child is
resting. Despite their appearance, these large hydro-
celes also do not require intervention and resolve
within 1–2 years of life.
Definitions

Hydrocele. When the processus vaginalis in the


groin closes, some fluid typically is trapped around Communicating hydroceles. Infants with fluctuat-
the testis. ing hydroceles are also managed expectantly, antici-
pating spontaneous closure of the processus vagi-
Communicating hydrocele. The processus vagi- nalis within 1–2 years. Communicating hydroceles
nalis remains open sufficiently to allow intraperi- in older boys may also close without surgery,
toneal fluid down the groin to intermittently distend although it is routine to operate those detected after
the tunica vaginalis. the first 2 years of life.

Cord hydrocele. The neck of the tunica vaginalis is Hernias. When the bowel is within the sac, surgery
closed, but the processus vaginalis remains open, is planned for the near future because of risk for
allowing fluid to distend a sac above the testicle. incarceration, although that is low and diminishes
with age.
Hernia. Intraabdominal contents, such as bowel or
omentum, extend down the processus vaginalis. Surgery

Unlike adult hydroceles, pediatric hydroceles and


hernias are approached through inguinal incisions,
Diagnosis

Patients typically present with painless enlargement not the scrotum.


of the hemiscrotum and/or groin. Hydroceles are
stable in size, whereas communicating hydroceles The purpose of surgery is to close the neck of the
and hernias may vary with increases and decreases patent processus vaginalis near the internal ring.
in intraabdominal pressure, exhibiting increased
size when the child is active and reduced size during The sac around the testis does not require removal,
rest. and such excision may increase risk for injury to the
vas.
Physical examination demonstrates the enlarged but
nontender hemiscrotum. Fluids within a hydrocele Despite closure of the sac and the internal ring and
will transilluminate. When a hernia is suspected but drainage of fluid from around the testicle, some-
not seen on examination, the child can be stood up times fluid, possibly lymphatic in origin, reaccumu-
and/or stimulated to try to increase abdominal pres- lates within the tunica vaginalis and creates concern
sure to distend the sac. that the sac will remain open. However, this fluid
will spontaneously reabsorb over a period of a few
months.

86 EDUCATIONAL REVIEW MANUAL IN UROLOGY


6. Adolescent Varicocele

Management of the contralateral side with unilateral


presentation remains controversial. Despite nega-
Incidence

tive history and physical exam, there is a 7% risk the • Occasionally diagnosed in prepubertal boys
processus vaginalis is either currently open or will
re-open in the future, which is not influenced by age • Most often detected during routine school or sports
at initial repair, or gender. physicals in teenagers

• Approximately 15% of teens have varicoceles,


which is similar to reported adult incidence
Hernias in Females

Girls may present with distention of the groin or


labia, indicating a hernia. Presentation

A gonad palpated within the hernia most often is an • Most varicoceles in adolescents are asymptomatic
ovary.
• Occasionally a teen will note fullness, a mass, or
One percent of female hernias are found to represent testicular discomfort with exertion
males with complete androgen resistance, and the
gonad within the sac is a testis. This may be sus- • Most are Grade III (grossly visible)
pected during office examination if a Q-tip passed
into the introitus indicates an obviously short • The ipsilateral testis may be diminished in volume
vagina.
Treatment

The incidence of varicocele exceeds the incidence


of male infertility, meaning therapy for all detected
cases would result in overtreatment, while no inter-
vention is undertreatment. Since no test currently
predicts future fertility related to varicocele, the fol-
lowing recommendations are based upon consensus:

• Initial observation is routine, even when there is


ipsilateral testicular volume loss, since catch-up
growth has been reported to occur without surgery

• Varicocele ligation for persistent ipsilateral testicu-


lar volume loss >20%

• Varicocele ligation for abnormal semen analysis in


Tanner 5 teenagers

Catch-up growth is often observed when varicoce-


les associated with testicular hypotrophy are cor-
rected in teens before completion of pubertal devel-
opment.

CHAPTER 4: GENITAL ABNORMALITIES 87


7. Epispadias/Exstrophy

Complications Epispadias

Hydrocele. Incidence up to 15%, especially with • This rare condition occurs either alone or in associ-
ligation of lymphatics and the internal spermatic ation with exstrophy
artery. Usually noted more than 6 months postoper-
atively; most resolving spontaneously or with sim- • Males with distal epispadias involving the glans
ple aspiration and drainage, although some require are usually continent, while those with penopubic
scrotal hydrocelectomy. epispadias usually have stress incontinence from
an incompetent bladder neck. The penis is also
Recurrent varicocele. Reported more common short, especially with increasing width of the sym-
with testicular artery-sparing procedures. physis pubis separation

• Females with epispadias are incontinent, although


they can provide a misleading history of episodic
leakage rather than continuous dribbling that might
be expected from the associated bladder neck
anomaly

Bladder Exstrophy

• Almost always associated with epispadias, but oth-


erwise are isolated anomalies in most cases. The
diagnosis is obvious by inspection, and KUB
shows the typical separation of the symphysis
pubis

• Vesicoureteral reflux develops following bladder


closure, and there is an increased risk of inguinal
hernias

Cloacal Exstrophy

• In this condition, the cecum does not tubularize,


but is open in the lower abdominal midline. On
either side is an open hemibladder exstrophy; supe-
riorly the ileum typically prolapses, and distally a
small hindgut is encountered under the abdominal
wall. In males, the penis is completely separated
into two components, with corpora cavernosa and
a hemiglans

• Renal anomalies, including hypoplasia, pelvic kid-


ney, and reflux, are common

• There is a high incidence of associated anomalies,


including cardiac defects, myelomeningocele,
short gut syndrome due to inadequate small bowel
length, and limb malformations

88 EDUCATIONAL REVIEW MANUAL IN UROLOGY


8. References

1. Joseph D. Intersex, Parts I and II. AUA Update


Series, Lessons 5 and 6. Volume XXII; 2003.
Management Principles

Isolated epispadias in males is corrected in the first


6 months of age, and closure of the penopubic ure- 2. Hughes IA, Houk C, Ahmed SF, Lee PA. Con-
thral defect may improve bladder volume over time sensus statement on management of intersex
in both epispadias and bladder exstrophy. Bladder disorders. J Pediatric Urol. 2006;2:148-162.
neck surgery in either condition typically is deferred
until the bladder has a volume of at least 60 cc, and 3. Snodgrass W, Prieto JC. Straightening ventral
is usually done between ages 3–6 years. Bladder curvature while preserving the urethral plate in
exstrophy is closed soon after birth. Surgery in the proximal hypospadias repair. J Urol. 2009;
first 48 hours may allow approximation of the pubis 182:1720-1725.
symphysis without osteotomies, relying upon
maternal relaxin effects to bring the pubis together. 4. Snodgrass W, Bush N, Cost N. Tubularized
There has been much discussion in recent years incised plate proximal hypospadias repair:
regarding relative merits of traditional staged epis- Continued evolution and extended applica-
padias/exstrophy repair vs complete primary repair. tions. J Pediatr Urol. 2011;7(1):2-9.
With either approach, more than one operation is
required, and relative benefits of one vs the other 6. Manzoni G, Bracka A, Palminteri E, Marrocco
remain uncertain. G. Hypospadias surgery: when, what and by
whom? BJU Int. 2004;94:1188-1195.
Cloacal exstrophy repair begins with tubularization
of the cecal plate, reduction of ileal prolapse, and 7. Husmann DA, Rathbun SR. Long term follow-
colostomy using the end of the hindgut. The bladder up of visual internal urethrotomy for manage-
halves can be simultaneously closed or sewn ment of short (<1cm) penile urethral strictures
together in the midline to create an isolated bladder following hypospadias repair. J Urol. 2006;
exstrophy to be repaired at a later date. Males have 176:1738-1741.
been managed with genital reconstruction or sex
reassignment, depending in part on the size and sep- 8 Snodgrass W, Bush N, Holzer M, Zhang S.
aration of penile components. The greatest initial Current referral patterns and means to improve
problem in these children is short gut syndrome, accuracy in diagnosis of undescended testis.
requiring hyperalimentation in the majority. Pediatrics. 2011;127:e382.

9. Smith GHH, ed. Orchiopexy: tips and tech-


niques. Dialogues in Pediatric Urology.
2005;(26):5.

10. Jordan GH, Brady JD. Laparoscopic techniques


for the management of the impalpable unde-
scended testicle: parts I and II. AUA Update
Series. Lessons 30 and 31. Volume XX; 2001.

11. Kaefer M, Diamond D, Hendren WH, et al. The


incidence of intersexuality in children with
cryptorchidism and hypospadias: stratification
based on gonadal palpability and meatal posi-
tion. J Urol. 1999;162:1003-1007.

CHAPTER 4: GENITAL ABNORMALITIES 89


9. Questions

12. Wenzler DL, Bloom DA, Park JM. What is the 1. A 1-year-old boy undergoing laparoscopy for a
rate of spontaneous testicular descent in infants right non-palpable testis is found to have a nor-
with cryptorchidism? J Urol. 2004;171:849- mal-appearing testis just inside the internal
851. ring. As peritoneum distal to the vas is incised,
you notice the vas leads to and appears to join a
13. Miller KD, Coughlin MT, Lee PA. Fertility midline uterus. The next best step in manage-
after unilateral cryptorchidism. Horm Res. ment is to:
2001;55:249-253.
A. Stop surgery and draw serum for
14. Lee PA, Coughlin MT. Fertility after bilateral 17-hydroxyprogesterone levels
cryptorchidism. Horm Res. 2001;55:28-32.
B. Stop surgery and obtain a karyotype
15. Wood HM, Elder JS. Cryptorchidism and tes-
ticular cancer: separating fact from fiction. C. Stop surgery and biopsy the contralateral
J Urol. 2009;181:452-461. descended gonad for ovarian tissue

16. Yerkes EB, Robertson FM, Gitlin J, et al. Man- D. Remove the uterus and perform right
agement of perinatal torsion. J Urol. 2005; orchiopexy
174:1579-1582.
E. Proceed with right orchiopexy, splitting the
17. Eaton SH, Cendron MA, Estrada CR, et al. uterus sagitally if the vas is tethered
Intermittent testicular torsion. J Urol. 2005;
174:1532-1535. 2. You are consulted to evaluate a full-term new-
born male with proximal hypospadias. Your
18. Belman AB, Rushton HG. Is the vanished exam confirms the urethral defect and ventral
testis always a scrotal event? BJU Int. 2001; penile curvature. Genital examination also
87:480-483. shows a well-developed scrotum with a mid-
line cleft, and bilateral nonpalpable testes. The
19. Baker LA, Sigman D, Mathews R. An analysis next step is to:
of clinical outcomes using color Doppler ultra-
sonography for testicular torsion. Pediatrics. A. Obtain FISH to detect a Y chromosome
2000;105:604-607.
B. Draw serum LH and testosterone levels
20. Johnson CW, Fisch H, Hensle TW. The adoles-
cent varicocele. AUA Update Series. Lesson. C. Order retrograde genitography to detect
33. Volume XXIII; 2004. a utricle

D. Schedule repeat examination for testicular


descent at age 6 months

E. Recommend laparoscopic orchiopexy at


age 6 months and hypospadias repair at age
1 year

90 EDUCATIONAL REVIEW MANUAL IN UROLOGY


3. A 6-year-old female presents with bilateral 5. An 8-year-old girl is referred after a febrile
groin masses noted mostly when she is play- UTI. She brings a CD with a renal ultrasound
ing. During herniorrhaphy, bilateral testes are that shows a horseshoe kidney and a 2-cm
found. You should next: heterogeneous mass in the region of the right
ovary. Notes from the pediatrician’s office
A. Immediately remove both testes because indicate she has been healthy, although her
of the increased risk for childhood germ height is more than 2 standard deviations
cell tumors below normal for her age. During examination
you notice her nipples appear wider apart than
B. Repair the hernias usual. Which of the following statements is
most likely true?
C. Perform laparoscopy to assess the uterus
A. She is beginning puberty and has a follicular
D. Draw serum for Müllerian inhibition ovarian cyst
substance levels
B. A karyotype would show a Y chromosome
E. Obtain a cerebral MRI to rule out an
empty sella C. She has an unresolved tubo-ovarian abscess
misdiagnosed as a febrile UTI
4. A 2-year-old boy had complete wound dehis-
cence after a proximal hypospadias repair with D. A VCUG would show high-grade right
preputial flap urethroplasty. During reopera- reflux
tion you open the right hemiscrotum to obtain
tunica vaginalis to cover over the neourethra, E. Vaginoscopy would show an obstructed
and encounter a dumbbell-shaped ovotestis. right hemivagina
The next step is to:
6. A 5-year-old boy presents after his mother
A. Replace the ovotestis and use fibrin glue noticed he was straining to urinate. His uro-
instead of tunica vaginalis to seal over the logic history is significant for a distal TIP
neourethra hypospadias repair done in another city at age 8
months. The family moved shortly after
B. Complete the hypospadias repair and then surgery and so he had no follow-up. He toilet
perform laparoscopic hysterectomy trained over a year ago, but mother is certain
his stream has slowed since then. On examina-
C. Remove the ovarian portion of the ovotestis tion, his penis is circumcised and overall looks
and explore the contralateral gonad normal, except that the meatus appears small
and has a faint white discoloration. Uro-
D. Remove the entire dysgenic gonad and flowmetry shows 3 cc/sec peak flow while
complete the hypospadias repair voiding 35 cc. The best long-term solution for
his problem is:
E. Remove the ovarian portion and biopsy the
testis for gonadoblastoma A. Meatotomy

B. Optical urethrotomy

C. Reoperative TIP repair

D. Reoperation with a flip-flap urethroplasty

E. Neourethral excision with 2-stage buccal


graft urethroplasty
CHAPTER 4: GENITAL ABNORMALITIES 91
7. A 6-year-old boy is referred for a left unde- C. Oral cefixime for 10 days
scended testis. Although he has seen the same
pediatrician his entire life, this problem was D. Oral cefixime with renal ultrasound and
first diagnosed on a routine exam one month VCUG when the acute inflammation sub-
ago. A testicular ultrasound was obtained sides
before referral, reporting both testes are the
same size and located in the inguinal canal. E. Oral analgesics as needed
During the exam, you notice his scrotum is
symmetric, and the right testis is easily manip- 9. A 7-year-old boy presents with urinary inconti-
ulated into the scrotum. The left testes seems nence since toilet training. He wears a pull-up
higher in the groin, but you can also manipu- that he changes twice a day. His parents report
late it into the scrotum, where it remains a few he can be dry during the night, but wets consis-
seconds before reascending. tently during the day, possibly when he is play-
ing too hard and does not go to the bathroom
The next step in management is to: when he should. His urologic history is other-
wise remarkable only for repair of penile epis-
A. Schedule testosterone injections padias at age 6 months. Examination shows the
2 mg/kg once a month for 3 months penis with a glanular meatus and some upward
and then reexamine him curvature. The bladder is not palpably dis-
tended and there is no cutaneous back lesion.
B. Schedule left orchiopexy The treatment most likely needed to correct his
incontinence is:
C. Schedule bilateral orchiopexy
A. Timed voiding
D. Reassure his parents testicular retraction is
common in this age group B. Laxatives for occult constipation

E. Repeat ultrasonography since the right testi- C. Oral anticholinergics


cle seems descended on examination
D. Meatotomy
8. An 8-year-old boy presents to the emergency
department with a red and swollen scrotum, E. Bladder neck repair
which his mother found after observing him
walking bow-legged. He is afebrile and seems 10. A 15-year-old teenager was found to have a
comfortable lying on the exam table, but palpa- scrotal mass during a sports physical. He has
tion of the scrotum causes discomfort. The ED no symptoms. Your exam confirms Tanner 3
physician obtained a testicular sonogram pubertal development with a visible left varic-
before calling you, reporting increased left tes- ocele and symmetric testes, confirmed by tes-
ticular blood flow with a swollen left epi- ticular ultrasound. You inform his parents:
didymis consistent with epididymo-orchitis. A
urinalysis is normal. You should manage this A. He can play sports and should have another
problem by: testicular ultrasound in one year

A. Urethral swab and cefixime for 7 days B. He should not participate in contact sports
because of increased risk for scrotal
B. Intravenous ceftriaxone until the erythema hematoma
improves, followed by oral cefixime for a
total of 10 days’ therapy C. He needs a semen analysis to rule out varic-
ocele effect on sperm function

92 EDUCATIONAL REVIEW MANUAL IN UROLOGY


D. He should undergo left varicocele ligation
for a Grade 3 varicocele
Answers:

1. E.
E. He should have bilateral varicocele ligation, The patient has failure of Müllerian inhibition sub-
since most varicoceles are bilateral stance, resulting in a male with a uterus. There is no
gender identify issue, and orchiopexy is needed to
11. A 6-month-old male is referred for possible her- relocate the intraabdominal testis. The ipsilateral
nia. Mother reports right scrotum periodically vas most often fuses into the Müllerian structures,
enlarges and appears blue. She is not certain making it necessary at times to split the uterus to
when this began, but states he cries when it is gain additional length for the testis to reach the scro-
swollen. The right scrotum is visibly enlarged tum. Theoretically, the patient could have an
and transilluminates. The next step is to: ovotesticular disorder of sexual differentiation,
although a normal appearing ipsilateral testis and a
A. Obtain scrotal ultrasound to visualize the contralateral descended gonad make that diagnosis
processus vaginalis unlikely and does not influence need to proceed
with orchiopexy.
B. Recommend inguinal surgery since the
hernia is symptomatic 2. A.
Although the baby appears virilized, bilateral non-
C. Recommend inguinal surgery because palpable testes, especially with hypospadias, man-
of the high risk for bowel incarceration date evaluation for congenital adrenal hyperplasia
in a genetic female. FISH detecting a Y chromo-
D. Reassure mother the problem is likely some would rapidly exclude that potentially life-
to resolve threatening diagnosis, since CAH females have a
46,XX karyotype.
E. Recommend scrotal hydrocele repair
3. B.
This phenotypic female has complete androgen
insensitivity. The testes are at risk for germ cell
tumor development during or after puberty, and so
will eventually need to be removed. However, they
can be maintained during childhood to assist with
secondary sexual development when puberty
begins. Therefore, herniorrhaphy can be completed
with or without simultaneous orchiectomy.
Although a minority of patients have a rudimentary
uterus, laparoscopy is not needed.

4. C.
There is no issue regarding gender identity, but the
ovarian portion of the ovotestis should be removed
to prevent breast development during puberty. The
testicular portion does not have increased risk for
gonadoblastoma as it is not dysgenetic. The con-
tralateral gonad can be easily exposed to rule out
bilateral ovotestes.

CHAPTER 4: GENITAL ABNORMALITIES 93


5. B. 9. E.
Short stature, widely spaced nipples, and a horse- Patients with epispadias often also have bladder
shoe kidney indicate a likely diagnosis of Turner’s neck incompetency, requiring surgical repair.
syndrome. The streak gonads have the potential to
develop gonadoblastoma, suggested by the apparent 10. A.
gonadal mass on ultrasound, when there is Y chro- Indications for varicocele ligation in teenagers
mosomal material. include decreased ipsilateral testicular size or pain.
Semen analysis is not considered useful until puber-
6. E. tal development is completed, typically around age
Both meatal stenosis and neourethral stricture are 17 years.
unusual after TIP hypospadias repair. The history of
progressive stranguria and finding of white discol- 11. D
oration around the meatus indicates BXO. BXO in The child has a communicating hydrocele, which is
the urethra after circumcision requires total excision likely to spontaneously close. Imaging is not
of affected tissues and buccal graft urethroplasty, needed. It almost certainly is not symptomatic, but
since the condition may recur if skin flap or graft rather becomes more prominent when the child is
urethroplasty is done. crying. Communicating hydroceles almost never
become hernias, and the risk for incarceration is
7. D. very low. If surgery is needed, an inguinal incision
Undescended testes are most often unilateral, and is most often used to close the processus vaginalis at
are most accurately diagnosed during examinations the internal ring. Scrotal hydrocele repair as is done
as a newborn and in the first 6 months of life. After in adults is not done in children with hydroceles.
that time, cremasteric muscular activity may retract
the testis into the upper scrotum, a normal finding
that may continue until puberty. Ultrasound cannot
distinguish between undescended and retractile
testes, since the cremaster muscle contracts and ele-
vates the testis when the gel is applied. In this case,
the scrotum appears symmetrical and the left testis
can be moved into the correct position, where it
remains a brief time – all typical findings of a retrac-
tile testis. This diagnosis is also supported by the
negative history of undescended testis at birth and in
the first months of life.

8. E.
The history and examination are most consistent
with a torsed appendage testis, which causes edema
of the epididymis that is often reported erroneously
as epididymitis or epididymo-orchitis. Epididymitis
is rare in otherwise normal, prepubertal males, and
most often presents with fever and urinary infection.
A torsed appendage resolves spontaneously, and so
only supportive measures are needed, such as anal-
gesics for discomfort.

94 EDUCATIONAL REVIEW MANUAL IN UROLOGY


Chapter 5:
Essentials of
Uroradiology
Fergus V. Coakley, MD
Rosaleen Parsons, MD

Contents

1. Imaging Modalities for Uroradiology

2. Adrenal Masses

All figures for this chapter


3. Renal Masses

appear starting on page 142


4. Renal Calcification and Urinary Stones

5. Renal Infection and Infarction

6. Renal Trauma

7. Ureteral Disease

8. Bladder Disease

9. Prostatic Disease

10. Urethral Disease

11. Testicular Masses

12. Pediatric Uroradiology

13. References

14. Questions

15. Figures

CHAPTER 5: ESSENTIALS OF URORADIOLOGY 95


1. Imaging Modalities for
Uroradiology

from -1000 to +1000. The scale is anchored by the


attenuation of air (set as -1000 Hounsfield units)
Overview

Imaging is integral to the modern practice of urol- and pure water (set as 0 Hounsfield units). In gen-
ogy, as it is used for diagnosis, therapeutic guidance eral, simple fluid, such as in renal cysts, has a den-
and follow-up of a wide variety of urological condi- sity -20 to +20 HU. High density or hemorrhagic
tions. The range of imaging modalities that are cysts contain fluid of higher attenuation, typically
available includes: plain radiographs, intravenous 60–100 HU. Macroscopic fat, as in a renal angiomy-
urography, fluoroscopy, ultrasound, CT, MRI, olipoma or an adrenal myelolipoma, is of lower
nuclear medicine and interventional radiology pro- density, typically -40 to -100 HU.
cedures. Optimal use of imaging in urologic prac-
tice requires matching the appropriate modality to
the clinical setting and question. The purpose of this CT was invented in 1972 by Godfrey Hounsfield in
Conventional, Spiral and Multidetector CT

chapter is to provide an overview of uroradiology his laboratory at EMI in England, with initial clini-
with an emphasis on clinical applications, in order cal installations in 1974. The first CT scanner
to allow an informed use of the available modalities required several hours to acquire the data for a sin-
in an expeditious fashion that optimizes patient care gle slice, and took several days to reconstruct the
while also providing a strong foundation in uroradi- corresponding image. Data acquisition and image
ology for urology residents reaching the end of their reconstruction became progressively faster during
training. From the latter viewpoint, it is important to the 1970s and 1980s, although the speed of the scan-
know not only the primary indications for the vari- ners remained limited by the need for “stop-start”
ous imaging modalities, but also those diagnoses slice-by-slice acquisition. That is, in conventional
that have distinctive features on a given modality; CT, an axial slice is generated by rotating an x-ray
both of these aspects are summarized in Table 1. It tube and detector array in a 360° circle around the
should be noted that this chapter provides an patient. After a 360° rotation, the rotating gantry
overview of critical aspects of uroradiology, but reverses direction to prevent disruption of the teth-
should not be regarded as a comprehensive review. ered cables that transfer the data from the detector
array to the computer. Such sequential slice acquisi-
tion limits the speed of conventional CT, prevents
volumetric data acquisition, results in slice misreg-
CT Technique and Terminology

istration (due to inconsistent patient breath holding


CT is the backbone of uroradiology, having virtu- from slice to slice), and limits temporal resolution
Basics of CT and CT Density

ally replaced IVU for the evaluation of urinary cal- so that multiphase volumetric scanning is not possi-
culi and being critical for the detection and charac- ble. The development of spiral (or helical) CT in the
terization of adrenal, renal and upper tract masses, late 1980s represented a technologic breakthrough.
and other urologic diseases. As such, a basic under- In spiral CT, data is carried from the rotating gantry
standing of CT technique and terminology is help- to the computer by slip rings, which allow continu-
ful. In CT, an x-ray source (tube) rotates around the ous gantry rotation and data transfer. Scanning can
patient. The x-ray beam passes through the patient be performed while the patient is moved slowly but
and strikes a detector system. A computer collects continuously through the gantry. The ability to con-
the signal from the detector system, and uses the tinuously scan allows for “non-stop” volumetric
large amount of data collected to form axial images data acquisition. Data are gathered on a 3-dimen-
of the patient, where the density of each picture ele- sional volume in a spiral fashion. Images are recon-
ment (or pixel, generally 512 x 512 pixels per slice) structed from the data volume. Physical and clinical
is proportional to the attenuation of the correspond- performance studies of spiral CT were first reported
ing volume of tissue (voxel). Tissue that highly in 1989, and these scanners rapidly entered clinical
attenuates the x-ray beam (e.g., bone or urinary practice. Spiral CT was an important technological
stones) is white, while tissue that causes less attenu- advance, but many limitations and compromises
ation of the x-ray beam is relatively black (e.g., fat remained. The rotational speed of the gantry was
or air). The attenuation of a given tissue type is mea- generally 1 rotation per second, and clinical results
sured using Hounsfield Units (HU), a scale that runs suggested that a pitch (ratio of longitudinal distance

96 EDUCATIONAL REVIEW MANUAL IN UROLOGY


Table 1

Primary Indications and Diagnoses with Particularly Distinctive Features on Each Modality
for the Major Imaging Modalities Relevant to Uroradiology.

Modality/Procedure Primary Indication Distinctive (“Spot”) Diagnoses

Plain radiography Follow-up of urinary stone Staghorn calculus


disease (non-contrast CT Nephrocalcinosis (cortical vs. medullary)
generally preferable at Emphysematous pyelonephritis
baseline) Oxalosis
Horseshoe kidney

Intravenous Largely replaced by non- Congenital ureteropelvic junction obstruction


urography contrast CT for flank pain Cobra-head of ureterocele

Retrograde Suspected upper tract Goblet sign of ureteral transitional cell carcinoma
pyelography TCC Fibroepithelial polyp

Cystography Bladder leak/rupture Blader leak (intra- vs. extraperitoneal)


Pine cone appearance of spastic neurogenic bladder

Voiding Bladder outlet Posterior urethral valves


cystourethrography obstruction Reflux

Retrograde Bladder outlet Urethral stricture


urethrography obstruction Urethral tear

Ultrasound Upper tract pathology Angiomyolipoma


Testicular mass Testicular cancer
Tubular ectasia of rete testis

CT Upper tract Adrenal mass with macroscopic fat — myelolipoma


pathology Adrenal mass with venous invasion —adrenocortical
carcinoma
Renal mass with macroscopic fat — angiomyolipoma
Renal mass with venous invasion — renal cell
carcinoma
Lucent filling defect — Transitional cell carcinoma,
hematoma, fungus-ball

MRI Problem-solving, As for CT, plus;


contrast allergy or “Light bulb” bright adrenal mass on T2W image —
renal impairmenta pheochromocytoma

*The appropriate use of MRI in renal impairment is currently evolving due to the recent association of gadolinium with
nephrogenic systemic fibrosis.

CHAPTER 5: ESSENTIALS OF URORADIOLOGY 97


moved by the tabletop during 1 gantry rotation to a relatively long time period, beginning approxi-
beam collimation) > 2 was undesirable. As a result, mately 2 minutes after the start of the contrast bolus,
either large volumes could be covered or thin sec- and persisting for a variable period of time there-
tions acquired, but not both. The next major after, on the order of hours. Upper tract opacifica-
advance was the development of systems with more tion images obtained during this late or delayed
than 1 row of detectors, allowing the x-ray beam to period (known as the excretory phase), and images
pass through a thicker slab of tissue, while preserv- acquired during this time period form the basis of
ing high spatial resolution. The earliest multidetec- CT urography (CTU). Of note, CTU may be per-
tor CT system had 2 detector rows, and in 1998 a formed with technical modifications aimed at opti-
scanner with 4 detector rows became commercially mizing distention and opacification of the collecting
available. Modern multislice (or multidetector) CT system, such as hydration, diuretic administration
using up to 256 detector rows essentially abolishes and changes in position, although such modifica-
the remaining obstacles to rapid isotropic (same res- tions remain under investigation and are not essen-
olution in x, y and z directions) volumetric imaging tial for the term CT urography to be appropriate.
by utilizing multiple side-by-side detectors simulta- Images obtained without the administration of intra-
neously. Such multislice CT systems can recon- venous contrast may be referred to as non-contrast,
struct a 512 x 512 matrix image from millions of non-enhanced or pre-contrast images. Strictly, such
data points in less than a second. An entire body images are not acquired during a “phase of enhance-
cavity (brain, chest, or abdomen) can be scanned in ment,” although the term non-contrast phase is fre-
5–10 seconds using the most advanced multislice quently employed in daily practice. Using such ter-
CT systems. Faster imaging with modern scanners minology, 5 enhancement phases of urologic inter-
is due not only to multiple detector rows but also to est can be identified (Figure 1):
increased gantry rotational speed. Many commer-
cial scanners are now capable of a complete gantry • Non-contrast phase. This is the optimal set of
rotation in 0.5 seconds or less. images for detection of calcification, particularly
the detection of urinary calculi. These images
also provide a baseline for the assessment of
After the peripheral intravenous administration of enhancement.
Phases of Enhancement at CT

iodinated contrast, the contrast material follows the


venous drainage to the right heart, and then passes • Arteriographic or early arterial phase. These
through the lungs to the left heart, from where it is images are primarily acquired for the evaluation
ejected into the systemic circulation. Within the of arterial pathology, such as fibromuscular
abdomen and pelvis, the first structures to show dysplasia or renal artery stenosis.
enhancement are the arteries. Strong arterial
enhancement is seen beginning approximately 20 • Late arterial or corticomedullary phase. These
seconds after the commencement of the intravenous images also provide good evaluation of arterial
contrast bolus (typically a volume of 100–50 cc is pathology and, in addition, occasionally allow for
injected at a rate of 2–5 cc per second). This is the improved detection of renal masses that
arteriographic or early arterial phase of enhance- demonstrate very rapid enhancement, either
ment. From a urologic perspective, the next tissue of wholly or in part. Renal cell carcinomas are
interest to demonstrate enhancement is the renal sometimes hypervascular and better detected in
cortex, with strong cortical enhancement seen at this phase (Figure 2).
40–45 seconds after the initiation of the contrast
bolus. This is the corticomedullary or late arterial • Nephrographic phase. This is the standard phase
phase of enhancement. At approximately 80–100 with the assessment of the abdominal and pelvic
seconds after the start of the bolus, the cortex and viscera, including the urinary tract. While hyper-
the medulla are both well-enhanced, and this is the vascular masses are occasionally better seen
nephrographic phase. Finally, contrast is excreted during the corticomedullary phase, overall more
by the kidneys and begins to opacify the collecting renal masses are visible in the nephrographic
systems and bladder. Such opacification is seen over phase than the corticomedullary phase (Figure 3).

98 EDUCATIONAL REVIEW MANUAL IN UROLOGY


• Excretory phase. This is the optimal phase for bolus (e.g., 110 cc) is administered. Corti-
evaluation of the collecting system. In general, comedullary and nephrographic phases are then
both the non-contrast and excretory phase images acquired based on a time delay from the start of the
are required from the diaphragm to the symphysis main bolus. The trick of this technique is that simul-
pubis, in order to provide complete visualization taneous with the corticomedullary and nephro-
of the entire urinary tract. graphic phase images, excretory phase images are
also required, because the contrast administered for
the “mini bolus” of 40 cc has already passed into the
collecting systems. The need for a separate acquisi-
Split Bolus Protocol

Based on the discussion above, there are 5 different tion of excretory phase images is thus obviated.
phases during which the urinary tract can be imaged
during a CT examination. The obvious question is
which of these phases are critical in routine practice,
Safe Use of Intravascular Iodinated

as in the evaluation of a patient with hematuria. The


Contrast

non-contrast phase is vital for the detection of


stones, and also very useful as a baseline for Renal impairment is one of the major side effects of
Nephrotoxicity

enhancement. Early arterial phase images are gener- intravenous iodinated contrast administration.
ally not critical unless highly selective visualization The true frequency of contrast nephropathy is diffi-
of the arterial system is of particular interest (late cult to establish, because there are no standard diag-
arterial phase images also provide good evaluation nostic criteria, but it is clear that the primary risk
of arterial pathology). The corticomedullary or late factor is baseline renal impairment, especially with
arterial phase images are of a somewhat question- coexistent diabetes. Routine creatinine testing prior
able utility, but provide arterial evaluation and also to contrast administration is generally not consid-
will occasionally better demonstrate a hypervascu- ered necessary in all patients; the major indications
lar renal cell carcinoma that might be missed on the are age >70 years and diabetes. The decision to pro-
nephrographic phase. As such, its institutional prac- ceed with contrast administration in patients with a
tice is to routinely acquire such images. The nephro- creatinine >1.5 mg/dL should always be a matter of
graphic phase is the standard phase for evaluation of clinical judgment, based on the individual circum-
the abdominal and pelvic viscera, and is also rou- stances of the patient and following consultation
tinely acquired. Finally, evaluation of the collecting between the radiologist and requesting physician.
system is obviously critical in a patient with hema- There is no general agreed upon threshold of serum
turia, and these images are also routinely acquired. creatinine elevation or level of renal dysfunction
beyond which iodinated contrast media should not
The need to acquire images in 4 distinct phases of be administered. There is no data to support the use
enhancement (non-contrast, corticomedullary, of mannitol, furoseminde, endothelin-1 and infu-
nephrographic and excretory) raises a number of sion of fenoldopam or theophylline in the preven-
practical problems, including a relatively large radi- tion of contrast-induced nephropathy. If contrast
ation dose to the patient and the need to acquire, administration is considered essential, the following
store and analyze a large number of images. In addi- options should be considered.
tion, such a protocol places greater demand on the
scanner itself, ultimately reducing the useful lifes- • Oral acetylcysteine. Administration of oral acetyl-
pan of the expensive x-ray tubes. For such reasons, cysteine reduces serum creatinine in normal
several institutions have adopted a technical modifi- patients but data supporting administration in
cation known as the split bolus protocol, that is patients at risk for contrast-induced nephropathy
essentially a trick that allows for a reduced number is inconclusive. A frequently cited study claimed
of images (Figure 4). In the split bolus protocol, the a 9-fold reduction in contrast-induced nephropa-
non-contrast images are acquired in the usual fash- thy in chronic renal insufficiency patients receiv-
ion. Then a small amount of the bolus (e.g, 40 cc) is ing 600 mg acetylcysteine (Mucomyst®) orally
administered. Nothing is done for the subsequent twice daily on the day before and the day of a con-
4–5 minutes, when the remaining major part of the trast-enhanced study, when compared to controls.

CHAPTER 5: ESSENTIALS OF URORADIOLOGY 99


In the study, patients were randomly assigned to • Increase the amount of time between contrast-
receive acetylcysteine and 0.45% saline intra- enhanced studies.
venously or to receive placebo and saline. Only 1 of
the 41 (2%) patients in the acetylcysteine group had • Discontinue other nephrotoxic drugs.
an increase of at least 0.5 mg in serum creatinine at
48 hours after administration of contrast compared While these options may be helpful, it should be
to 9 of 42 (21%) patients in the control group remembered that contrast nephropathy is uncom-
(P=0.01). Since that original study, there have been mon and usually transient. A critical diagnostic
multiple randomized controlled trials and meta- study should not be delayed because of excessive
analyses which have shown conflicting results, concern regarding possible contrast nephropathy. In
some supporting the use of acetylcysteine and oth- addition, it should be noted that there is little data on
ers not. Many of the controlled trials have been the combined use of these approaches, although the
hampered by poor study design, including small clinical setting may dictate which regimen is more
patient numbers. Given the heterogeneity of data, it practical in a patient with elevated creatinine.
is difficult to support the administration of acetyl-
cysteine as a proven and effective means by which
to prevent contrast nephropathy.
Prior Contrast Reaction

Premedication is generally reserved for patients


• Sodium bicarbonate infusion. A recent trial of with a history of a significant prior contrast reac-
patients with a baseline creatinine of at least 1.1 tion. The risk of a repeat reaction in a patient with a
mg/dL found a significant reduction (P=0.02) in history of prior severe reaction is 18.5%, even with
the frequency of nephrotoxicity (defined as an non-ionic contrast media. The use of premedication
increase of 25% or more in serum creatinine to prevent reactions to intravascular non-ionic con-
within 2 days of contrast) in those randomized to trast media is controversial. The most supportive
sodium bicarbonate infusion (1 of 60) compared study states premedication reduces the incidence of
to sodium chloride (8 of 59). Patients received 154 all reactions by approximately 60%, but it is unclear
mEq/L of either sodium chloride or sodium bicar- whether the statistical power and methodology of
bonate, as a bolus of 3 mL/kg per hour for 1 hour the study allows extrapolation of this risk reduction
before iopamidol contrast, followed by an infu- to patients developing moderate and severe reac-
sion of 1 mL/kg per hour for 6 hours after the pro- tions (i.e., those of most concern). Corticosteroids
cedure. The advantage of this regime is that it can are the critical component of any premedication
be implemented rapidly, facilitating the early regime, and should be given at least 6 hours before
scanning of patients from the Emergency Depart- the test. For simplicity, an oral regime is recom-
ment, for example. mended, and one regimen endorsed by the Ameri-
can College of Radiology is 32 mg methylpred-
• Hydration. This is one of the most important nisolone (Medrol®) given orally 12 and 2 hours
methods for decreasing the incidence of contrast before the study, supplemented by 50 mg diphenhy-
nephropathy; however, the exact hydration proto- dramine (Benadryl®) given orally one hour before
col is unclear. The regimen which is considered the study.
most ideal is 1/2 normal saline at 1 mL/kg per
hour for 12 hours before and 12 hours after con-
trast administration. This regimen, however, is
Safe Use of Intravascular Gadolinium

very impractical in an outpatient setting. Addi- Gadolinium-containing chelates perform essentially


tionally, there is little data to either support or the same function at MRI as iodinated intravascular
refute the effectiveness of oral compared to intra- contrast media do at CT. Unlike iodinated contrast,
venous hydration regimens. The exact hydration gadolinium was traditionally considered extremely
protocol is probably not nearly as important as the safe, with very minor risks of contrast reaction and
fact that some form hydration is administered nephrotoxicity. As such, renal impairment was often
quoted as one of the primary reasons for performing
• Decrease total amount of contrast administered. MRI. This situation changed rather dramatically in

100 EDUCATIONAL REVIEW MANUAL IN UROLOGY


2. Adrenal Masses

2006, when reports began to emerge that high-dose


gadolinium studies in patients with severe renal
Normal Appearances

impairment were associated with the development The normal adrenal glands are small, measuring
of nephrogenic systemic fibrosis (NSF). NSF is a approximately 4 x 2 x 0.5 cm and weighing 3–6
chronic condition in which patients experience grams each. They have a V- or Y-shaped appear-
swelling and tightening of the skin that develops ance, because each has a medial and lateral limb.
over a period of days to several weeks. NSF was The adrenal glands are of a fixed anatomic relation-
first observed in 1997 and described in 2000, and ship to vessels, which can be useful for localization,
has only been seen in individuals with renal impair- particularly when other anatomy is distorted, such
ment. In many cases, the thickening of the skin as after nephrectomy (Figure 5). The right adrenal
inhibits joint movement, and in severe cases, gland lies posterior to the upper inferior vena cava,
affected patients may be unable to walk. NSF can while the left lies posterior to the splenic vessels. No
also affect the liver, lungs, muscles and heart. There strict definition of an adrenal mass or enlargement
is no specific treatment, although restoration or has been established, but, in general, a nodule is
improvement of renal function may help. At first, considered present if there is a focal contour defor-
NSF was thought to be perhaps an extremely rare mity, and enlargement is generally considered
complication, seen only with high-dose studies and present if the thickness of the adrenal gland exceeds
certain formulations of gadolinium. While informa- 7–10 mm. The adrenal glands have 2 layers, an
tion on this topic continues to emerge, it now outer cortex and inner medulla. The cortex makes
appears that NSF can develop after any type of up about 10% of the adrenal gland, and is derived
gadolinium and the condition may develop even from the urogenital ridge. The cortex secretes vari-
after standard doses of gadolinium in as many as ous steroid hormones, including aldosterone, corti-
3%–5% of patients with severe renal impairment costeroids and trace amount of sex hormones.
who receive gadolinium. The current approach of Remembering that steroid hormones are based on
most institutions is to screen patients for renal the backbone of the cholesterol molecule helps to
impairment prior to an MRI scan, including calcula- explain why adrenal adenomas (benign tumors of
tion of eGFR as needed, and to administer gadolin- the adrenal cortex) are lipid-rich. This lipid richness
ium in patients on dialysis or with an eGFR under can be exploited at imaging to help characterize
30. adrenal adenomas, since it results in low density at
non-enhanced CT and signal loss on opposed-phase
gradient-echo MR imaging. The inner medulla
makes up approximately 90% of the adrenal gland,
and has a separate embryological origin, being
derived from the neural crest. The medulla secretes
catecholamine, primarily epinephrine and nore-
pinephrine.

Overview of Adrenal Masses

Various classification terms for adrenal masses


exist, such as benign or malignant, functional or
non-functional. A more practical classification is
presented in Table 2, which is based on the fre-
quency with which adrenal masses are encountered
in clinical practice. While this table is not compre-
hensive, it easily covers 99% of the adrenal masses
seen in clinical practice. Each of the entities in
Table 2 is discussed in more detail below, but it
should be noted that the non-functional adenoma is
by far the most common adrenal mass seen on a
day-to-day basis.
CHAPTER 5: ESSENTIALS OF URORADIOLOGY 101
which CT density measurements may be used to
characterize adrenal adenomas, the non-contrast
Table 2

images and delayed phase images (Figure 6). With


respect to non-contrast CT images, 0, 10 and 18
Practical Classification of Adrenal Masses

Hounsfield units have all been proposed as potential


attenuation thresholds, below which an adrenal
Common Non-Functional adenoma

mass can be confidently characterized as an adrenal


Metastasis

adenoma (Table 3). In general, it is safe to use a


threshold of 18 Hounsfield units, particularly if
Uncommon Myelolipoma

other secondary characteristics, such as small size


Pheochromocytoma

and homogeneity, are also present. With respect to


Functional adenoma

delayed phase CT thresholds, the situation is some-


what more complex, since presumably there will be
dependency both on the time delay and the volume
Very Rare Adrenocortical carcinoma
Adrenal cyst

and concentration of contrast administered. That


said, a density of <52 Hounsfield units or a wash-
out of >60% at 15 minutes both appear to be useful
criteria (Table 4). It is particularly noteworthy that
Non-functional Adrenal Adenoma

Non-functional adrenal adenomas are very com- contrast wash-out appears to be a feature of adrenal
mon, as indicated by autopsy frequency of 3% for adenomas, even if they are not lipid rich, and may
adenomas of 3 mm or greater in size and 1% for therefore help characterize some adrenal masses as
adrenal adenomas of 5 mm or greater in size. Given adenomas that could not be so classified using only
the current resolution of CT and MRI, the latter fig- non-enhanced CT image density (Figure 7).
ure of 1% is likely a fair reflection of the frequency Opposed-phase gradient-echo MR imaging is a spe-
with which patients may be expected to have an cial type of T-1 weighted sequence that is sensitive
incidental adrenal adenoma during imaging with to the presence of microscopic fat. Two sets of
these modalities. Of note, the frequency of non- images are acquired, 1 in phase and 1 out of phase
functional adrenal adenomas increases with age, (opposed-phase). Voxels that contain pure fat or
being 5% for patients over 60 years. In general, non- pure water have the same intensity both in and out
functional adrenal adenomas are believed to be of of phase images. Voxels that contain a mixture of fat
little to no clinical significance. Various associa- and water demonstrate signal loss. This gives rise to
tions have been proposed, such as with hypertension a distinctive etching or “India ink” artifact on
and diabetes, although it is conceivable that such opposed-phased imaging. Because the boundaries
associations are spurious and simply reflect closer of muscles and organs that are surrounded by fat
examination of the adrenal glands in such patients. contain voxels that represent mixed fat and water,
More recently, there has been some interest in the they turn black on the opposed-phase image. If the
finding that perhaps 5%–25% of patients with tradi- parenchyma (as opposed to the boundary) of an
tionally defined non-functional adrenal adenomas adrenal mass demonstrates clear cut darkening on
may in fact have subclinical hormonal dysfunction, opposed phase imaging, then a diagnosis of adrenal
although the true relevance of this finding remains adenoma can be made with confidence (Figure 8). A
to be established. quantitative study at Memorial Sloan-Kettering
Cancer Center (MSKCC) showed that a signal loss
At imaging, adrenal adenomas are typically small, on opposed vs in-phase imaging of > 45% was 80%
well-defined and homogenous. In addition, because sensitive and 100% specific in the diagnosis of
of their high lipid content, they are of low density on adrenal adenoma. While such quantitative criteria
non-enhanced CT and demonstrate signal loss at can be used, a qualitative study at Massachusetts
opposed phase gradient echo MR imaging. Finally, General Hospital showed that simple visual evalua-
it is an empirical observation that they demonstrate tion of clear cut signal loss was also highly accurate
rapid wash-out of intravenous contrast. Based on and just as good as quantitative evaluation.
these considerations, there are 2 time points at

102 EDUCATIONAL REVIEW MANUAL IN UROLOGY


Table 3

Sensitivity and Specificity of Different Attenuation Thresholds for the Characterization of


Adrenal Adenomas at Non-enhanced CT
Cut-off Sensitivity Specificity n

<0 HU 47 100 66

<10 HU 79 96 66

<18 HU 85 100 61

Table 4

Sensitivity and Specificity of Different Attenuation and Washout Thresholds for the Charac-
terization of Adrenal Adenomas at Delayed Contrast-enhanced CT
Delay Cut-off Sensitivity Specificity n

60 min <30 HU 95 100 51

15 min <37 HU 96 96 76

10 min <52 HU 92 95 135

15 min >60% washout 86 92 71a

*Only lipid-poor (>10 HU on NECT) adenomas included.

• Conn’s syndrome, or primary hyperaldosteronism,


is increasingly diagnosed with better testing and
Functional Adrenal Adenomas

These are rare tumors that present clinically with found in 5%–13% of patients with hypertension. It
hormonal features related to excess corticosteroids is due with approximately equal frequency to
(Cushing’s syndrome), mineralocorticoid (Conn’s either idiopathic bilateral adrenal hyperplasia or
syndrome) or sex hormones (various adrenogenital an aldosterone-producing adrenal adenoma
syndromes). With respect to imaging, non-func- (aldosteronoma).
tional and functional adrenal adenomas are indistin-
guishable and clinical or biochemical correlation is • When evaluating patients with Conn’s syndrome,
critical. 3 other points should be remembered when even a small unifocal nodule is likely to be an
imaging suspected functional adenomas: aldosteronoma (Figure 10). Adrenalectomy is usu-
ally indicated for such a finding. In the absence of
• Cushing’s syndrome, when not due to exogenous a clear-cut unilateral nodule, or if CT findings are
steroids, is usually secondary to a pituitary tumor equivocal, adrenal vein sampling can be under-
producing excess ACTH that then induces adrenal taken in an attempt to determine if there is a later-
hyperplasia. Accordingly, imaging abnormalities alized source of excess aldosterone from the
of the adrenal glands in Cushing’s syndrome, even adrenal glands.
unifocal enlargement, often reflect secondary hyper-
plasia rather than an autonomous primary functional
nodule tumor of the adrenal gland (Figure 9).

CHAPTER 5: ESSENTIALS OF URORADIOLOGY 103


myelolipoma. While case reports of macroscopic fat
in adrenal adenomas and carcinomas have been
Adrenal Metastases

Metastases to the adrenal glands are found in 27% described, examination of these cases show that, in
of cancer patients at autopsy, with common primary general, only a tiny speck of fat was present in an
sites including breast, lung, melanoma and renal otherwise non-fatty mass, and real-life confusion
cell carcinoma. In particular, it should be noted that between myelolipoma and these other entities is
renal cell carcinoma can involve the adrenal gland unlikely.
not just by direct upward invasion, but also by
hematogenous spread. Thus, it is possible to have a
renal cell carcinoma on one side of the body, and
Pheochromocytoma

have hematogenous metastases to the adrenal on the Pheochromocytoma are functional tumors of
contralateral side (Figure 11). Overall, given the fre- medullary origin that secrete excess catecholamine,
quency of adrenal adenomas in the general popula- resulting in the characteristic syndrome of episodic
tion, it is clear that the primary consideration to an hypertension. Approximately 1 in 200 cases of
adrenal mass in a cancer patient is incidental ade- hypertension are due to pheochromocytoma. These
noma vs metastasis. Overall, approximately 44% tumors are characterized by the so-called 10% rule,
of adrenal masses in cancer patients are metastatic, because approximately 10% of these tumors are
although this number will clearly vary with the malignant, 10% are multiple, 10% occur outside
nature and stage of the primary malignancy. the adrenal gland and 10% are familial pheochro-
mocytomas.
The imaging findings of adrenal metastases are
variable. They may be small and homogeneous, At imaging, pheochromocytomas are typically
mimicking adenomas (except that they are not of 2–5 cm, well-defined ovoid masses of high T2 sig-
low density on non-contrast CT), or large and irreg- nal intensity (Figure 15). The latter finding has been
ular (Figure 12). They may be unilateral or bilateral referred to as the “light bulb” sign, although in prac-
(note that even if bilateral, adrenal insufficiency is tice, this is not a frequently encountered finding.
extremely rare). Metastases to the adrenal glands More commonly, these tumors are prone to internal
hardly ever invade the adrenal vein, which is a find- hemorrhage, necrosis and sometimes calcification
ing seen almost exclusively with primary adreno- and these latter findings can be useful in suggesting
cortical carcinoma. At MR imaging, adrenal metas- the diagnosis (Figure 16). Overall, pheochromocy-
tases are of variable T2 signal intensity, so that tomas have quite a variable imaging appearance,
while high T2 signal intensity may favor a diagnosis and clinical correlation is often critical in suggest-
of metastases, low T2 signal intensity can be seen ing the correct diagnosis. That said, it is possible for
with both metastases and adenomas. Adrenal metas- a pheochromocytoma to be unsuspected clinically,
tases are usually hypermetabolic on PET scans, and raising the question as to the likelihood of cate-
this may be a useful test for an indeterminate cholamine crisis if such patients were inadvertently
adrenal mass in an oncologic patient (Figure 13). given intravenous contrast or underwent biopsy.
Limited studies suggest that intravascular contrast is
unlikely to precipitate a catecholamine crisis even
in an unblocked pheochromocytoma, but that
Adrenal Myelolipoma

Adrenal myelolipomas are rare sporadic benign biopsy is quite likely to result in such an event. If
tumors of the adrenal gland containing fat and bone pheochromocytoma is a consideration for an
marrow (myelo) elements, with an autopsy fre- adrenal mass, it may be worthwhile to perform an
quency of 1–2 per thousand. Hemorrhage has been MIBG scan, which has a sensitivity of 76%–100%
reported, and the occasional finding of calcification and near 100% positive predictive value for the
within these masses may be related to prior subclini- diagnosis of pheochromocytoma (Figure 17).
cal episodes of bleeding. At imaging, the fat ele-
ments of the tumor result in a characteristic finding
of macroscopic fat in an adrenal mass (Figure 14).
This appearance is virtually diagnostic of

104 EDUCATIONAL REVIEW MANUAL IN UROLOGY


Adrenocortical Carcinoma Practical Imaging Approach to an

Primary adrenocortical carcinoma is an extremely


Adrenal Mass

rare malignancy, with an incidence of 1 per million When an adrenal mass is encountered at imaging,
per year. The imaging findings are those of a large a logical stepwise approach will usually result in a
irregular heterogeneous mass, often with necrosis or noninvasive diagnosis.
calcification. These findings, while certainly
suggestive of malignancy, do potentially overlap Are distinctive imaging findings present? This is
with those of a large atypical adenoma, pheochro- usually not the case, but the finding of macroscopic
mocytoma or metastasis. Two imaging findings that fat (myelolipoma), fluid (adrenal cyst) or venous
might be helpful in suggesting the correct diagnosis invasion (adrenocortical carcinoma) leads rapidly to
are venous invasion (Figure 18) (an adrenal mass specific diagnosis; however, keep in mind that
with venous invasion to be considered an adreno- adrenal cysts are occasionally malignant and should
cortical carcinoma until proven otherwise) and be examined for complex features.
metastases (35% of patients have metastatic spread
at diagnosis).
adrenal malignancy? If the answer is yes, then the
Does the patient have a known primary extra-

primary differential consideration is between an


incidental adenoma and a metastasis. CT, possibly
Adrenal Cysts

Adrenal cysts are rare, with an estimated frequency with delayed images, or in- and out-of-phase imag-
of 1–2 per 10,000 with a female predominance. ing will usually lead to the diagnosis of adenoma, if
Adrenal cysts are classified as: this is the pathology present. Remember, however,
that the inability to make a diagnosis of adenoma
• Endothelial – 45% —benign simple cyst. based on the absence of low density at CT or the
absence of sufficient signal loss at out-of-phase MR
• Hemorrhagic – 40%—these are important imaging does not indicate the mass is a metastasis,
because very occasionally they are malignant since lipid-poor adenoma is still a consideration. In
(Figure 19). such a scenario, biopsy or PET scanning may be
helpful, assuming that the distinction is not aca-
• Paracystic – 10%—they are usually echinococcal demic, such as in a patient with widely disseminated
(hydatid disease). metastatic disease.

• Epithelial – 5% —benign simple cyst. Does the patient have clinical or biochemical signs

Because of the occasional occurrence of malig-


of an endocrinopathy, namely Conn’s syndrome,

nancy within an adrenal cyst, 1 traditional approach catecholamine excess? Such findings will lead to a
Cushing’s syndrome, adrenogenital syndrome or

to these lesions has been to recommend surgical diagnosis of functional adenoma (but remembering
resection in all cases. This is probably somewhat that adrenal nodularity in the setting of Cushing’s
aggressive and conservative management may be syndrome may reflect hyperplasia rather than an
appropriate for those adrenal cysts with a clearly autonomous functional adenoma), pheochromocy-
benign simple appearance, that is thin-wall toma or, sometimes, adrenocortical carcinoma
(<3 mm), small (<6 cm), with lesions of homoge- (since this tumor can be functional).
neous fluid density (-20 to +20 Hounsfield units),
with or without thin hairline calcifications or septa. If none of the above questions have a positive
If the cystic mass is not clearly simple, then invasive answer, the mass is truly incidental, and the vast
resection or biopsy may be appropriate. majority of these are non-functional adenomas.

CHAPTER 5: ESSENTIALS OF URORADIOLOGY 105


3. Renal Masses

Occasionally, an incidental mass will be an adreno-


cortical carcinoma and appropriate work-up for this
Overview

consideration should be undertaken. One question Contrast-enhanced CT is the gold standard for the
that arises from the approach outlined above is detection of renal masses. Using this modality as the
whether an adrenal “incidentaloma” can ever be a standard of reference, IVP is markedly limited in the
metastasis from an occult primary malignancy. An detection of renal masses, particularly at smaller
important study from MD Anderson Cancer Center sizes, and even ultrasound demonstrates substantial
strongly suggests that the answer to this question, limitations. It should be noted that detection and
for all practical purposes, is no. In a review of 1639 characterization are different, though often related,
patients with metastatic disease of unknown pri- imaging endpoints, and there are times when CT
mary origin, 95 (5.8%) of the patients had adrenal may need to be supplemented by other modalities,
metastases. Only 4 (0.2%) of these patients had the such as ultrasound or MRI to fully characterize a
adrenals as the only site of metastatic disease. In CT-detected renal mass. From a practical stand-
these 4 patients, all the tumors were > 6 cm, and point, renal masses can be classified as solid, cystic
bilateral in 3 patients. The conclusion of this study or indeterminate (Table 5). Given that the distinc-
is that a small solitary incidental adrenal mass is tion of solid from cystic is a critical first step in
highly unlikely to be the initial manifestation of an classifying renal masses, it is important to note the
occult primary malignancy. following points.

Table 5

Practical classification of renal masses

Solid Non- (macroscopic) fat-contain-


ing (renal cell carcinoma – rarely,
fat-poor angiomyolipoma,
oncocytoma, lymphoma, metas-
tases to the kidney or upper
tract transitional cell carcinoma)
(Macroscopic) fat-containing
(angiomyolipoma)

Cystic Benign simple cyst (Bosniak 1)


Benign (appearing) high density
cyst (Bosniak 2)
Benign (appearing) mildly
complex cyst (Bosniak 2)
Possibly malignant moderately
complex cyst (Bosniak 3)
Probably malignant markedly
complex cyst (Bosniak 4)

Indeterminate Too small to characterize


Unclear if solid or cystic
Bosniak 2F cyst
Unclear if fat-containing or not

106 EDUCATIONAL REVIEW MANUAL IN UROLOGY


down), and reported studies suggest a drop in CT
density of 15 HU on delayed images is indicative of
Benign Simple Cysts

Irrespective of modality, benign simple cysts are a solid lesion. De-enhancement can be helpful in 2
ovoid lesions with thin-to-imperceptible walls that settings: first, when a suspected renal mass is
demonstrate a sharp parenchymal interface. At detected on a post-contrast study and delayed
ultrasound, simple cysts are anechoic with posterior images are obtained, but no pre-contrast images are
acoustic enhancement. At CT, simple cysts are of available; second, when a lesion is not well seen on
uniform fluid attenuation (-20 to +20 HU) and non- non-contrast images, precluding confident place-
enhancing. At MR imaging, simple cysts are of low ment of a region of interest for evaluation of
T1 signal intensity, extremely high T2 signal inten- enhancement (Figures 22 and 23).
sity (comparable to fluid in the spinal canal, gall
bladder or bowel loops) and non-enhancing after the
administration of gadolinium (Figure 20). More
Pseudo-enhancement

complex cysts are characterized by the presence of Pseudo-enhancement is another phenomenon that
proteinaceous fluid, walls, septa, calcification or has been described in recent years. This refers to an
nodules. For practical purposes, the identification of artifactual increase in density that can be seen in
walls or septa in a lesion implies that it is cystic. small (10–15 mm or less) cysts after contrast admin-
istration at CT, irrespective of the thickness of the
slices acquired. The basis of this phenomenon is not
completely understood, but conceptually the prob-
Enhancement

Enhancement of a lesion refers to a demonstrable lem can be considered “bleeding” of high density
uptake of intravascular contrast (which could be at from adjacent renal parenchyma into the measure-
ultrasound, CT or MRI) and indicates that a lesion is ment performed on the small cyst. Artifactual
solid, since solid tissue includes blood vessels and increases of density of up to 28 HU have been
will contain the contrast agent, while cysts do not described due to pseudo-enhancement (Figure 24),
have blood vessels fluid and do not contain the con- and could potentially result in a mischaracterization
trast agent. At CT, enhancement is generally defined of a small cyst as a solid lesion.
as an increase in attenuation of 10 Hounsfield units
or more when compared with the non-contrast base-
line. At MRI, enhancement is defined as an increase
Subtraction Images

in signal intensity of 15% or more when compared The term “subtraction images” may be encountered
to the baseline. While these thresholds are empiri- when discussing the presence or absence of
cally established, it is important not to relax these enhancement at MRI. This refers to the process of
criteria, because papillary renal cell carcinomas can electronically subtracting the pre-gadolinium
be of limited vascularity and enhancement, and images from post-gadolinium images, such that, at
even a small increase in attenuation or intensity may least in theory, the resulting images should only
be the only indicator of their true solid nature show enhancing structures in varying shades of
(Figure 21). In cases of equivocal enhancement, gray, with non-enhancing tissue appearing black.
ultrasound can usually distinguish solid from This technique is sometimes useful when a lesion is
cystic masses. of high signal intensity before gadolinium is admin-
istered, since it is then visually difficult to determine
if the lesion has become even brighter after gadolin-
ium administration (Figure 25). The technique does
De-enhancement

De-enhancement is a newer concept, but is simply require good registration of the pre- and post-
the converse and logical phenomenon of enhance- gadolinium images, since otherwise subtraction will
ment. De-enhancement refers to a decrease in the not be successful, and objective criteria for
presence of contrast within a solid lesion when enhancement on subtraction images (what “shade of
delayed post-contrast images are compared to early gray” is solid?) have not been defined and the
post-contrast images (what goes up, must come assessment of enhancement is therefore subjective.

CHAPTER 5: ESSENTIALS OF URORADIOLOGY 107


of renal cell carcinoma is rising but it may well be
that this reflects the increasing use of imaging in the
Characterization of Solid Renal Masses

Solid renal masses can be subdivided into those that population, resulting in the detection of more small
do not contain macroscopic fat and those that do. renal cell carcinomas of questionable clinical
This is a critical distinction, because the characteri- importance. For example, the clinical frequency of
zation of solid renal masses can be summarized by renal cell carcinoma in patients over 70 is 0.3%, but
2 rules: renal cell carcinomas are found at gross inspection
in 1.5% of autopsies and at microscopy in 22% of
• A solid renal mass that does not contain macro- autopsies. Furthermore, in multivariate analysis of
scopic fat is a renal cell carcinoma, until proven patient outcome, the mode of presentation (inciden-
otherwise. There are a small number of other tal vs symptomatic) is an independent significant
solid, non-fat-containing renal masses (that are predictor, in addition to tumor size and tumor histol-
usually distinguishable based on clinical or ogy. These data suggest that renal cell carcinoma is
radiological features); these are fat-poor angiomy- analogous to prostate cancer, in that indolent or
olipoma, oncocytoma, renal transitional cell microscopic disease that is unlikely to threaten the
carcinoma, renal lymphoma and metastasis to long-term health or survival of the patient is com-
the kidney. mon and that overdiagnosis of subclinical disease is
a real entity. That said, when a renal cell carcinoma
• A solid renal mass that contains macroscopic fat is is diagnosed in an individual patient, it is clearly
an angiomyolipoma. incumbent to offer appropriate therapy.

Macroscopic fat is best assessed on CT, and this In North America, renal cell carcinoma is frequently
determination can often be made by the naked eye, staged using the Robson system (Table 6). Inspec-
looking for density within the renal mass that is sim- tion of this table shows that 2 critical local observa-
ilar to nearby subcutaneous or perinephric fat (Fig- tions from an imaging viewpoint are venous invasion
ure 26). Sometimes, a small amount of fat will be and locoregional nodal metastases. Venous invasion
difficult to establish visually, but the presence of a can be seen in up to 23% of renal cell carcinomas,
CT attenuation value under -20 HU is generally with invasion into the inferior vena cava in up to
indicative of adipose tissue. Small amounts of fat 10% of cases. Venous invasion can be identified at
may require thin section, non-contrast images for ultrasound, CT, MRI or conventional venography.
optical detection (Figure 27). At MRI, macroscopic
fat can be identified because it will be of the same
signal intensity as nearby subcutaneous or per-
inephric fat on all sequences, and will also demon-
Table 6

strate suppression of signal intensity with fat satura-


tion techniques (Figure 28). At ultrasound, fat is
Robson Staging System for Renal Cell

classically very echogenic (Figure 29), although


Carcinoma

this is not a reliable sign, because renal cell cancers


can sometimes be quite echogenic as well.
Stage Description

I Organ confined (intracapsular)


Renal Cell Carcinoma

The typical renal cell carcinoma is a solid well-cir-


cumscribed mass of variable size within the renal
II Invasion of perirenal fat only

parenchyma that does not contain macroscopic fat


and demonstrates enhancement with contrast (Fig-
III A: Venous invasion

ure 30). Occasionally, renal cell carcinomas have an


B: Local nodal metastases

infiltrative appearance (Figure 31). Up to 61% of


C: Both

renal cell carcinomas are now found incidentally at


imaging performed for other reasons. The incidence
IV Distant metastases

108 EDUCATIONAL REVIEW MANUAL IN UROLOGY


In most cases, modern spiral CT performed with small quantities of fat within fat-poor angiomy-
multi-phase imaging and multi-planar reformation olipomas (Figure 34). This remains to be validated
can adequately demonstrate the presence and extent in larger studies.
of tumor thrombus, particularly extension into the
right heart which may necessitate cardio-thoracic Ultrasound features: The relatively characteristic
surgical involvement at the time of resection (Figure marked echogenicity of angiomyolipomas on ultra-
32). In problematic cases, both MRI and conven- sound (Figure 29) is only partially due to fat con-
tional venography are somewhat more accurate, tent, and may be related to the vascular or muscular
although MRI has the advantage of being less inva- component. This suggests that marked echogenicity
sive. Locoregional adenopathy (defined in the usual might be useful even in fat-poor angiomyolipomas,
fashion as a short axis > 1 cm) can be seen in but unfortunately too much overlap exists with
approximately 25% of patients with renal cell carci- echogenic renal cell carcinomas for this to be use-
noma, but about half the time pathology shows no ful. That said, some findings may be helpful; for
metastatic disease in the nodes, suggesting nodal example, shadowing is seen only with angiomy-
enlargement in renal cell carcinoma is often reactive olipomas, while hypoechoic rings and intratumoral
in nature. cysts are seen only in renal cell carcinoma. Unfortu-
nately, these findings are seen in only a minority of
Fat-poor Angiomyolipoma cases, and are therefore not particularly helpful.

An estimated 5% of angiomyolipomas have insuffi- Opposed-phase MRI: It has been suggested that
cient fat to be demonstrably fat-containing at imag- opposed-phase MRI might be helpful to detect
ing. These masses mimic renal cell carcinoma, and small amounts of fat in angiomyolipomas, but this is
occasionally a resection will be performed for a pre- a dangerous suggestion because microscopic fat can
sumed renal cell carcinoma and the final pathology be present in clear cell renal carcinomas and these
demonstrates fat-poor angiomyolipoma (Figure 33). tumors can show significant signal loss on opposed-
Arguably, this remains appropriate management, phase MRI. (Remember that the clear cell variant of
since the preoperative probability of such a lesion renal cell carcinoma is called thusly because of the
being a renal cell carcinoma is extremely high and it presence of intracellular lipid and/or glycogen.)
is difficult to see how surgery could be avoided.
That said, there is increasing interest in better character-
ization of small, solid, non-fatty renal masses, since
Oncocytoma

it is increasingly recognized that many of these Oncocytoma is a rare benign tumor that appears as a
lesions turn out to be fat-poor angiomyolipomas or solid, non-fat-containing renal mass. The tumors
oncocytomas, and a variety of secondary signs of mimic renal cell carcinoma not only radiologically,
fat-poor angiomyolipoma have been studied: but also histopathologically—to the point where
some pathologists believe oncocytoma is not a dis-
Enhancement characteristics: Fat-poor angiomy- tinct benign tumor, but rather a low-grade variant of
olipomas demonstrate more uniform and prolonged renal cell carcinoma. Two radiological features have
enhancement than renal cell carcinomas, but the been described and are said to be distinctive, namely
existing data would suggest such findings are not the presence of a central stellate scar or of a spoke-
sufficiently accurate to allow confident non-surgical wheel pattern on angiography (Figure 35). In real-
management, although conceivably might be used ity, it is unlikely that detection of either feature
to justify surveillance in a patient with multiple would be sufficiently compelling to obviate surgery.
comorbidities.

Histogram pixel analysis: CT histogram analysis, in


Renal Transitional Cell Carcinoma

which the CT densities of the individual pixels Most upper tract transitional cell carcinomas can be
within a region of interest are displayed as a his- identified based on clinical or imaging findings.
togram of number of pixels vs attenuation, has been Clinically, the majority of these patients will have a
reported as helpful in the demonstration of very history of bladder cancer, and transitional cell carci-

CHAPTER 5: ESSENTIALS OF URORADIOLOGY 109


noma should be a leading consideration for any Metastases to the kidney are often small and multi-
upper tract abnormality in a patient with such a his- focal or perinephric in distribution. Unifocal
tory. Radiologically, upper tract transitional cell car- involvement can occur, in which case the findings
cinoma is often primarily located in the pelvica- may mimic renal cell carcinoma, and this pattern is
lyceal system (Figure 36), rather than the renal said to be more common in patients with colorectal
parenchyma, and also may result in pelvicalyceal cancer.
dilatation due to obstruction. That said, occasion-
ally, an upper tract transitional cell carcinoma will
exist largely or entirely within the renal parenchyma
Cystic Renal Masses

(Figure 37), in which case the distinction from renal


(Bosniak Classification)

cell carcinoma may be difficult. It should be noted


that a renal mass with pelvicalyceal invasion is sug- Benign simple cysts are very common, occurring
Benign simple cyst (Bosniak category 1)

gestive of a transitional cell carcinoma, but that this in 30% of patients over 50 years. The accuracy
feature can also be seen with renal cell carcinoma of imaging approaches 100% when all of the well-
(while pelvicalyceal invasion is reported known morphologic features are present. These
histopathologically in up to 14% of renal cell carci- include an ovoid uniocular lesion with a thin-
nomas, it is rarely seen at imaging). to-imperceptible wall that is well-circumscribed
with a sharp parenchymal interface. At CT, the
lesion is of uniform fluid density (-20 to +20 H)
(Figure 20).
Renal Lymphoma

At autopsy, 30%–60% of lymphoma patients


demonstrate renal involvement, although renal
involvement is seen in only 3%–8% of lymphoma These are benign cysts that meet all of the criteria
Benign high density cyst (Bosniak category 2)

patients undergoing CT. Primary lymphoma of the for a benign simple cyst, except that the cyst fluid is
kidneys is rare, and most patients with lymphoma- > 20 HU in density (often the range of
tous involvement of the kidneys will also have 50–90 HU), and this is believed to reflect hemor-
extrarenal disease, which serves as a useful imaging rhagic or proteinaceous content. At ultrasound, such
clue to the correct diagnosis (Figure 38). Certain internal hemorrhage or protein may be recognized
patterns of varying distinctiveness may be seen by low level echogenicity within the cystic lesion,
when the kidneys are involved. In particular, multi- while at MRI, hemorrhagic or proteinaceous con-
ple small masses, renal invasion by spread from the tent may result in high T1 signal intensity that does
retroperitoneum, diffuse infiltration of the kidneys, not suppress with fat saturation (Figure 41). Such
perinephric encasement or a single homogeneous lesions can still be classified as benign, provided
mass (the latter may mimic renal cell carcinoma) non-contrast images are present to confirm that the
have all been described. Perinephric encasement is lesion is truly non-enhancing.
probably the most suggestive pattern with respect to
the specific diagnosis of lymphoma, although it is
not the most common (Figure 39). This is the second type of Bosniak category 2 cyst,
Benign mildly complex cyst (Bosniak category 2)

and this designation is appropriate for a cyst that


otherwise has the features of a simple cyst but
contains hairline-thin septa or fine curvilinear
Metastasis to the Kidney

At autopsy, 2%–20% of patients dying of cancer calcification (Figure 42).


have metastases to the kidney, with common pri-
mary sites being lung, breast, stomach, pancreas,
colon and melanoma. In practice, metastases to the
Possibly malignant complex cyst (Bosniak cate-

kidney are rarely recognized at CT and, when seen, Features that constitute signs of possible malig-
gory 3)

usually occur in the setting of widely disseminated nancy include thick or irregular walls or septa, irreg-
disease (Figure 40). For example, in one series, the ular thick calcification or non-enhancing nodules
kidneys were the sole site of involvement in only (Figure 43). The risk of malignancy in a Bosniak 3
3 of 27 patients with metastases to the kidney. cyst is approximately 50%, and accordingly these
lesions are generally managed by resection.
110 EDUCATIONAL REVIEW MANUAL IN UROLOGY
up), which Bosniak himself has described as “a
Features that result in a Bosniak 4 categorization for group of cystic lesions that are not complex enough
Malignant complex cysts (Bosniak category 4)

a cystic lesion include irregular enhancing thick to be characterized as category 3 but that are more
wall, enhancing or large nodules, or the presence of complex than category 2 lesions.” The features that
solid components (Figure 44). Most Bosniak 4 result in a Bosniak 2F categorization are an
lesions are cystic renal cell carcinomas, although increased number of hairline-thin septa, minimal
occasionally an upper tract transitional cell carci- thickening of septa or wall, minimal enhancement
noma may be predominantly cystic. It should be of hairline-thin septa or wall, thick or nodular calci-
noted that cavitary necrosis does not truly constitute fication, or high-density cysts > 3 cm. The problem
a cystic lesion, and when such cases are excluded, with these characteristics, as acknowledged by
approximately 3.5% of renal cell carcinomas Bosniak himself, is that they are largely subjective.
are cystic. Nonetheless, when a mildly complex cyst is seen
that is on the borderline between Bosniak 2 and 3,
an assignment of Bosniak 2F maybe useful, since
surveillance may result in a more confident diagno-
Bosniak Classification of Renal Cysts:

sis at follow-up imaging. The good news is that


FAQs

most such lesions appear to be benign, and in 1 ret-


rospective study of 42 cases, with a mean follow-up
Can Bosniak 2 lesions be malignant, despite

There are 2 types of Bosniak 2 cysts, the high den- of 6 years, only 1 cystic renal cell carcinoma was
having a benign appearance?

sity cyst and the mildly complex cyst. One case ultimately diagnosed.
report described renal cell cancer in a high density
cyst, but the focus of malignancy was microscopic
and on the cyst wall. Given the frequency of micro-
Is there a scientific (i.e., not medicolegal) basis for

scopic renal cell carcinoma at autopsy, it is arguable While this question may also be relevant to solid
suggesting follow-up of renal masses?

that this was a coincidence, and certainly this single renal masses, it follows logically from discussion of
case report is hardly sufficient evidence to change Bosniak 2F lesions. Unfortunately, while follow-up
the traditional approach to high density cysts. A few is often suggested as an appropriate method of dis-
case reports have described cancer in mildly com- tinguishing benign from malignant renal lesions, the
plex Bosniak 2 cysts, though there is only 1 good scientific basis for this approach is somewhat lim-
series. In this series, 23 cystic renal cancers were ited. For example, the growth rate of oncocytomas
resected and their preoperative Bosniak appearance does not differ significantly from that of Fuhrman
examined. The study claims that 1 of the lesions was grade 1 renal cell carcinomas. More disturbingly, it
a mildly complex (Bosniak 2) cyst, although the has been shown that small presumed renal cell car-
legitimacy of this characterization is difficult to cinomas can remain relatively stable in size over
confirm, since images were not provided in the pub- several years, and then grow quite dramatically.
lication. Again, this seems insufficient evidence to
change the common assumption that mildly com-
plex cysts are benign. While the Bosniak system is widely used, it is not
Is the Bosniak system objective?

clear that it is sufficiently objective and repro-


ducible. In an interesting study where 70 cystic
The distinction of Bosniak 2 from Bosniak 3 lesions masses were classified by 3 readers, major discrep-
What are Bosniak category 2F lesions?

is critical, because the former will be dismissed as ancies (i.e., Bosniak category 1 or 2 vs 3 or 4) were
benign, while the latter will likely undergo resection seen in 11 cases (16%). The authors of this study
because of the high risk of malignancy. As such, any suggested that interobserver variation in distin-
reader making a Bosniak 2 assignment clearly guishing Bosniak 2 and 3 lesions may present
requires a high degree of confidence in this catego- difficulties in recommending surgical vs conserva-
rization, but there are times where cysts have some tive management.
degree of complexity that introduces doubt and
uncertainty. In such cases, there is the additional
classification of Bosniak 2F (F stands for follow-

CHAPTER 5: ESSENTIALS OF URORADIOLOGY 111


probably safer to remove an indeterminate lesion
While cystic renal cell carcinomas tend to be of than to miss a potential cancer.
Do cystic renal cell carcinomas matter?

lower grade and stage than control groups of


matched solid renal carcinomas, there is still an
appreciable mortality associated with these cancers,
Angiomyolipoma

and appropriate treatment is still indicated. In 1 Angiomyolipomas are rare, benign, hamartomatous
study, there was an 82% 4-year disease-specific sur- tumors of the kidney that are composed of blood
vival for cystic renal cell carcinoma. vessels (angio), smooth muscle (myo) and fat (lipo).
Most are sporadic, but some are associated with
tuberous sclerosis. The latter is a particular consid-
eration if the tumors are multiple or associated with
Indeterminate Renal Masses

Too Small To Characterize (TSTC): Hypodense other manifestations, such as pulmonary lymphan-
lesions under <1–1.5 cm in the kidney are very com- gioleiomyomatosis.. The lipo component of
mon, and too small to characterize using density angiomyolipomas is critical for imaging characteri-
measurement. Such measurements are unreliable in zation, since it is estimated that 95% contain
small lesions, due to both partial voluming in the Z demonstrable fat, and so the imaging rule is a solid
axis and also the phenomenon of pseudo-enhance- renal mass that contains macroscopic fat is an
ment. The available evidence suggests that the vast angiomyolipoma. The typical angiomyolipoma is a
majority of such lesions are small cysts and they can well-defined mass in the renal cortex, measuring up
generally be dismissed, unless there is a visual con- to 5 cm that is composed largely of fat density adi-
cern or possibly a clinical concern, such as a high- pose tissue (Figure 26). The lesions are often exo-
risk patient with prior renal cell carcinoma or von phytic, in which case the finding of a parenchymal
Hippel-Lindau disease. notch or a vascular pedicle extending from the kid-
ney into the lesion can help confirm that it is of renal
Unclear if the lesion is solid or cystic: Occasionally, origin, and therefore an exophytic angiomyolipoma
the criteria described above for distinguishing solid (Figure 46). Two obvious questions arise with
and cystic masses may conflict within or between respect to angiomyolipoma. First, can renal cell car-
modalities. This is uncommon, and the situation can cinoma contain fat? If so, can fat-containing renal
usually be resolved by additional imaging, although cell carcinoma be mistaken for angiomyolipoma?
management should be tailored to each patient. Renal cell carcinomas can, on very rare occasions,
contain macroscopic fat. Three mechanisms have
Bosniak 2F Lesion: These are cysts in which suffi- been described, namely entrapment of adjacent fat
cient complexity is present to result in some disquiet (either perinephric fat, sinus fat or even tumorous
in assigning a Bosniak 2 categorization, but the fat in an adjacent angiomyolipoma), bone marrow
lesion is not considered sufficiently complex to formation in osseous metaplasia within the tumor or
merit resection. As noted above, these lesions can fat-producing necrosis. From a practical point of
be followed, although the logic for this approach is view, all these mechanisms are likely to be associ-
debatable. ated with an obvious heterogeneous or necrotic
tumor mass, so in real life it is highly unlikely that a
Unclear if fat-containing or not: This is probably fat-containing renal cell carcinoma would truly
the least common type of indeterminate renal lesion, mimic an angiomyolipoma.
but occasionally a solid mass will be seen with some
fat at the periphery, and it may be unclear whether
this is tumoral fat (indicating angiomyolipoma) or
entrapped fat adjacent to a renal cell carcinoma
(Figure 45). Alternatively, a tiny speck of fat within
a large solid lesion may be considered insufficient
for a confident diagnosis of angiomyolipoma.
Again, management should be tailored to the indi-
vidual patient, but with the general caution that it is

112 EDUCATIONAL REVIEW MANUAL IN UROLOGY


4. Renal Calcification and Urinary
Stones

the standard of reference for renal colic evaluation,


because it combines both functional and anatomic
Overview

Renal calcification is usually focal and within the assessment, and because direct opacification of the
pelvicalyceal system, due to stone formation urinary system facilitates identification of radiolu-
(nephrolithiasis), but occasionally diffuse calcifica- cent stones. The main disadvantages of IVU are the
tion can be seen in the parenchyma (nephrocalci- need to administer IV contrast and the duration of
nosis). These topics will be discussed separately. the study, which can be particularly prolonged when
delayed films are required in high-grade obstruc-
tion. However, while IVU reliably demonstrates
obstruction, the cause is not always clearly shown.
Nephrolithiasis

In a series of 46 patients with calculous ureteral


Non-enhanced CT is much more sensitive for uri- obstruction, the stone was not visible at IVP in 6
Imaging of Flank Pain

nary stones than intravenous urography, and has (13%). The 6 missed stones were confirmed by ret-
largely replaced IVU in patients with suspected rograde pyelography, ureteroscopy or spontaneous
ureteral colic (Figure 47). The sensitivity of non- passage. In 2 other series, obstructing stones were
enhanced spiral CT for depicting a renal or ureteral missed by IVU in 12 of 28 and 6 of 11 patients. The
stone is 94%–98%, while that of US is approxi- emergence of non-enhanced CT probably accounts
mately 20% and IVU is 52%–59%. Non-enhanced for the apparent drop in IVU sensitivity for stone
CT is also useful for identifying causes of flank pain identification. In the past, an IVU showing mild
other than nephrolithiasis, such as appendicitis and hydroureteronephrosis but no stone was often
diverticulitis. In a study of 93 patients referred for reported as “possible recent stone passage.” It is
clinically suspected renal colic, a change in treat- possible that many of these cases were due to small
ment plan was made in 57 patients (61%) based on urographically occult stones.
findings at non-enhanced CT. In theory, 90%–95%
of stones are sufficiently radiopaque to be visible by
plain radiography. In practice, the plain radiograph
Stone Characterization by Imaging

is very limited in the evaluation of renal colic, Imaging features that reflect stone composition and
because multiple radiodensities can mimic stones formation include morphology, internal structure
(e.g., gallstones, costochondral cacifications, bone and density.
islands and fecal densities) and because stones may
be easily missed (e.g., radiolucent stones and stones Stone morphology: Most stones are small and
obscured by bowel content or bone). Ultrasound can round, which is relatively nonspecific. Pure calcium
be used to identify obstruction in a patient with oxalate stones are often homogenous, dense and
flank pain, but the endpoint is demonstrative of smooth. Mixed calcium oxalate stones may be irreg-
hydronephrosis rather than direct visualization of a ular in shape, inhomogeneous and may have den-
stone (although sometimes an obstructing calculus dritic projections. Staghorn calculi are so called
may be seen high in the ureter through the acoustic because they develop in the pelvicalyceal system,
window for the kidney, or in the distal ureter and in advanced cases have a branching configura-
through the bladder). Bowel and bone limit acoustic tion that resembles the antlers of a stag. Staghorn
access to the rest of the ureter in the retroperi- calculi are typically composed of magnesium
toneum. The main drawback of US is that not all ammonium phosphate (struvite), which forms in
obstructed kidneys have dilated pelvicalyceal sys- urine that has abnormally high pH (> 7.2). This high
tems, either because dilatation has not yet devel- pH usually develops because of recurrent urinary
oped, the upper tract has decompressed by forniceal tract infection with Gram-positive micro organisms,
rupture or the upper tract is noncompliant. In addi- such as Proteus mirabilis. However, large urate acid
tion, the diuretic effect of intravenous contrast and cystine stones can also have a staghorn configu-
likely explains why dilatation is occasionally absent ration, and staghorn calculi in children or young
at ultrasound but present at IVU. The reported false- adults with no history of infection are frequently
negative rate for ultrasound in acute flank pain composed of cystine. Radiographically, struvite
ranges from 6%–38%. Traditionally, IVU has been stones are relatively low density. Low-density stru-

CHAPTER 5: ESSENTIALS OF URORADIOLOGY 113


vite stones may not be appreciated on plain radio- structure is seen leading up to a calculus without an
graphs, but can be readily detected by US or CT. associated rim of soft tissue around the calculus
IVP or retrograde pyelography may also be used to (Figure 50), is often described as a characteristic of
demonstrate the typical branching appearance of a phlebolith rather than a ureteral stone. However,
staghorn calculi. Faceted stones are usually multiple the comet-tail sign is neither sensitive nor specific
and have arisen in a small cavity, such as a renal for phleboliths and, when observed in patients with
pelvis above an ureteropelvic junction obstruction or ureteral obstruction, was shown to actually repre-
in a calyceal diverticulum, where the stones are sent a ureteral calculus in 50%–67% of cases.
minimally mobile but in contact with each other. Use of a computer workstation or PACS, which
Lithotripsy may be inappropriate for such stones, allows dynamic scrolling of axial CT images as well
because residual small fragments may not pass. as 3-dimensional reformations, is also useful to
define the path of a ureter and determine whether a
Internal structure: Most stones are homogeneously calcification is within or external to a ureter. In the
dense. Struvite stones may have a laminated appear- rare case when a CT examination is equivocal for
ance, with alternating dense and relative lucent layers. location of a calcification, intravenous contrast
material may be administered to opacify the renal
Density: In the IVP era, stones were traditionally collecting system, allowing more precise depiction
divided into radiopaque (calcium and struvite of the ureter relative to the calcification.
stones) and radiolucent (pure cystine, urate, and
matrix or mucoprotein stones). This division was
not precise, because so-called lucent stones could
Prognosis of Ureterolithiasis

become secondarily calcified and become at least Both the size and location of ureteral stones are pre-
partially opaque. More importantly, the distinction dictive of whether the ureteral stones will pass with
between radiopaque and radiolucent stones has conservative medical management, or whether
become obsolete in the CT era. Due to the exquisite surgical intervention will be required. At CT, a stone
sensitivity of CT to even small amounts of calcium, which ≤4 mm in axial diameter will have a > 75%
all stones appear opaque (i.e., white) on CT. The likelihood of passing with conservative manage-
only common exception to this statement is the ment, while stones ≥9 mm are <50% likely to pass
occurrence of indinavir stones in HIV patients (indi- without intervention. Similarly, stones which are
navir is a protease inhibitor that is poorly soluble in identified in the mid-ureter or more distally have a
urine, and may form obstructive stone-like precipi- >60% likelihood of passing without need for inva-
tates after excretion into the collecting system). sive intervention, while more proximal stones are
These lucent stones may still be detectable due to <50% likely to pass without intervention.
secondary calcification or to signs of ureteral
obstruction in the setting of an HIV patient with
flank pain (Figure 48).
Nephrocalcinosis

Nephrocalcinosis refers to radiologically demon-


strable, diffuse fine parenchymal (non-calyceal)
renal calcification. The calcification may be pre-
Differentiation of Stones from Phleboliths

A problem with non-enhanced CT is that stones, dominantly medullary or cortical. Common causes
particularly in the pelvis near the ureterovesical of medullary nephrocalcinosis (Figure 51) are
junction, must be differentiated from phleboliths. hyperparathyroidism, renal tubular acidosis and
The soft tissue rim around the stone is presumed to medullary sponge kidney. The most common cause
be the edematous ureteral wall. In a study of CT in of cortical nephrocalcinosis is chronic glomeru-
442 patients, of whom 136 had ureteral calculi, it lonephritis (Figure 52). Other causes include acute
was found that a thin rim of soft tissue surrounding a cortical necrosis, Alport syndrome and renal trans-
calcification (the soft tissue rim sign, Figure 49) plant rejection. Diffuse renal calcification can occur
was seen in 105 of 136 ureteral calculi and in only in children with oxalosis, resulting in a characteris-
20 of 259 phleboliths. The converse sign, the tic radiological appearance (Figure 53).
“comet-tail sign,” where a thin linear soft tissue

114 EDUCATIONAL REVIEW MANUAL IN UROLOGY


5. Renal Infection and Infarction

Pyonephrosis: When an obstructed collecting sys-


tem becomes infected pus fills the distended renal
Pyelonephritis

Acute pyelonephritis is an inflammation of the renal pelvis. The diagnosis should be considered when a
parenchyma caused by an ascending infection. Pre- patient with known urinary tract obstruction devel-
disposing conditions include urinary retention, his- ops flank pain and fever. Septic shock and decline of
tory of urinary tract infections, neurogenic bladder, renal function can rapidly develop if left untreated.
reflux, immunocompromised host, congenital
anomalies of the kidneys and ureters, prolonged Imaging: Ultrasound provides the greatest sensitiv-
catheterization and pregnancy. Vesicoureteral ity, specificity and accuracy in differentiating sim-
reflux is seen in 30%–50% of affected children. ple hydronephrosis from pyonephrosis. The collect-
While E. coli is the most common pathogen, Pro- ing system is distended with echogenic debris. Gas
teus miranilis is frequently the causative agent in can occasionally be detected within the collecting
elderly patients following instrumentation. With system. CT and MRI are not as accurate as ultra-
ascending pyelonephritis the process begins in the sound for establishing the diagnosis.
renal pelvis and progresses through the medulla and
finally the cortex. The opposite pattern of involve-
ment is found with hematogenous infection
Emphysematous Pyelonephritis

although radiographically it can be difficult to dis- Emphysematous pyelonephritis refers to


tinguish between the two. pyelonephritis with gas formation, and is usually
due to infection by a gas-forming organism
Hematogenous infection is less common and usu- (E. coli, most commonly) in diabetics or immuno-
ally produces microabscess, often with negative compromised patients. Imaging (particularly CT)
urine cultures. Imaging findings are quite variable, shows air in the kidney, perirenal space or collecting
and are related to the degree and duration of infec- system (Figure 55). Emphysematous pyelonephritis
tion. IVU is normal in about 75% of cases. Findings is a potentially life-threatening necrotizing infec-
characteristic of pyelonephritis include a decreased tion.
or delayed nephrogram which may be patchy or stri-
ated, enlargement of the involved kidney (due to On CT the normal renal parenchyma is replaced by
edema) and pelvicalyceal effacement due to com- mottled gas with renal or perirneal fluid collections.
pression by the edematous parenchyma. Ultrasound Historically treatment has been nephrectomy
is usually normal. Nonspecific findings that may be because of a mortality rate that approached 50%.
seen include renal enlargement, focal wedge-shaped A recent shift in the treatment paradigm with percu-
decreased echogenicity and loss of pyramidal taneous drainage and aggressive medical treatment
visualization. Focal decreased perfusion may be has reduced the mortality rate to 8%. If this is not
seen with Doppler techniques. CT findings extend curative partial or complete nephrectomy might still
from striation due to scattered areas of wedge- be necessary.
shaped decreased perfusion, through renal enlarge-
ment. There may be fascial thickening and blurring
of the perinephric fat. If acute pyelonephritis is not
Xanthogranulomatous Pyelonephritis

recognized or the treatment is inadequate a renal Xanthogranulomatous pyelonephritis is a rare,


abscess can develop and maybe intra- or extra- chronic renal inflammatory disease characterized
parenchymal but is typically contained within Gero- by destruction and replacement of renal
ta’s fascia. The majority of renal abscess occur in parenchyma with lipid-laden macrophages, which
diabetic patients. On CT an abscess is usually well are also known as xanthoma or foam cells. The most
circumscribed with an enhancing pseudocapsule common bacteria associated with XGP include :
surrounding a low attenuation non enhancing cen- P. mirabilis and E. coli. Most patients have a history
tral collection. CT is the imaging modality of of obstructing stones, often staghorn, complicated
choice for the evaluation of pyelonephritis and its by multiple urinary tract infections. The condition is
complications (Figure 54). most common in middle-aged women, although any
age and sex can be affected. Clinical features are

CHAPTER 5: ESSENTIALS OF URORADIOLOGY 115


nonspecific, and include fever, malaise, flank pain On imaging, fungus balls are seen as multiple small
and weight loss. Ultimately, chronic inflammatory filling defects in the collecting systems (Figure 57).
changes extend outside the kidney into neighboring Other findings include diminished excretion of
tissues, including the psoas muscle, diaphragm, contrast, papillary necrosis and hydronephrosis. CT
bowel and lung. The diffuse inflammation results in is the preferred imaging modality for evaluating
parenchymal distortion which may be diffuse or fungal balls and for fungal pyelonephritis in gen-
focal. The focal form seen in approximately 10% of eral. It can accurately depict morphologic lesions,
cases is found in duplicated collecting systems and functional abnormalities in contrast uptake and
can mimic both renal abscess and neoplasm. Fea- excretion, as well as associated complications, such
tures that suggest the diagnosis of xanthogranulo- as perinephric abscess. Treatment includes systemic
matous pyelonephritis at imaging include antifungal therapy. Instrumentation of the urinary
tract with direct removal of the fungus balls may be
(Figure 56):
helpful as adjuvant therapy.
• Presence of a staghorn calculus

• Obstructive hydronephrosis with calyceal


Renal Infarction

Renal infarction may result from embolic or throm-


dilatation and decreased or absent renal function:
botic disease. Thrombotic disease usually affects
Irregularity of the calices may be present the larger vessels, including the main renal artery,
due to superimposed papillary necrosis while emboli may affect vessels of various sizes.
Renal artery emboli usually come from the heart,
• Replacement of the renal parenchyma by and typically result in the sudden onset of flank
multiple small abscess cavities pain, hematuria, proteinuria, fever and leukocytosis.
In contrast, patients with thrombotic disease usually
• Marked perinephric inflammatory changes present with hypertension or renal insufficiency.
Thrombotic disease usually results from atheroscle-
Distinction from renal malignancy may be difficult rosis, though blunt abdominal trauma may cause
in the rare focal form of xanthogranulomatous intimal tears with subsequent dissection and throm-
pyelonephritis. Treatment is nephrectomy, and par- bosis. Excretory urography shows absent enhance-
tial nephrectomy can be done for the focal type. ment of the affected renal segment. On ultrasound,
the infarcted area may be visibly enlarged and
hypoechoic. Doppler may show absent intrarenal
arterial signal, or a tardus-parvus waveform if there
Fungal Infection

Fungal infection of the kidney is usually an oppor- is incomplete occlusion or collateral supply.
tunistic infection in diabetic or immunocompro- Subtle renal infarcts are best demonstrated on CT,
mised patients, or patients with indwelling where they appear as wedge-shaped, cortically
catheters. The most common organism is Candida based, hypodense areas (Figure 58). Renal swelling
albicans or other candidal species. Candidal may also be seen. The rim sign may be present if the
infection of the kidneys may occur as the result of entire kidney is involved. That is, the entire kidney
systemic or primary renal candidiasis. Primary renal is non-enhancing except for the outer 2–4 mm of
candidiasis is most commonly seen in diabetic cortex, which is perfused by capsular branches.
women. Pathologically, fungi which are filtered by Nuclear imaging shows a photopenic area corre-
the glomeruli become lodged in the distal tubules, sponding to the region of ischemia or infarction.
where they proliferate and produce microabscesses. Arteriography provides the definitive diagnosis,
Papillary necrosis ensues as the fungi infiltrate the showing abrupt termination of vessels or filling
tips of the renal papillae. The fungi are then defects. Cortical atrophy and irregular scarring are
extruded into the renal collecting system, resulting seen as late sequelae. In the case of end-stage renal
in fungus balls (mycetomas). The fungus balls may artery thrombosis, a small kidney with a smooth
cause obstruction, resulting in hydronephrosis, pain contour is seen unless multiple small infarcts had
and even renal failure, if severe and long-standing. independently occurred. Treatment of renal
Fungal infection of the kidneys can be diagnosed ischemia consists of anticoagulation, intraarterial
through demonstrating fungal hyphae in urine. thrombolytic therapy and surgical revasculariza-
tion.
116 EDUCATIONAL REVIEW MANUAL IN UROLOGY
6. Renal Trauma

Subcapsular hematoma appears as a well-defined,


peripheral, high-density fluid collection adjacent to
Overview

Renal injury occurs in 8%–10% of abdominal the kidney, which is often crescentic or elliptical in
trauma patients, particularly those with blunt (rather shape and may deform the kidney.
than penetrating) trauma. From a clinical perspec-
tive, the management of renal injury is increasingly Perinephric hematoma, unlike subcapsular
conservative, with a higher threshold for surgical hematoma, is ill-defined peripherally, not associ-
intervention. From a radiological perspective, con- ated with renal deformation and may be associated
trast-enhanced CT is the preferred imaging option, with a visible laceration.
although 1-shot IVU and ultrasound are alternatives
(Figures 59 and 60), particularly in an unstable Renal contusion appears as an ill-defined area of
patient with multiple other injuries where transfer to reduced enhancement and excretion.
a CT scanner may be too risky or time-consuming.
Renal trauma is graded based on the imaging Renal infarction appears as a well-defined and
appearances using the American Association for the wedge-shaped area of absent enhancement.
Surgery of Trauma (AAST) System (Table 7).
Active hemorrhage is characterized by linear or
flame-like intense contrast enhancement within a
laceration or hematoma during the early phase of
CT Appearances

Since the AAST grading system is heavily depen- enhancement, with later washout.
dent on imaging, it is important to have a good under-
standing of CT terminology and distinctions in renal
trauma (Figures 61-63).

Table 7

American Association for the Surgery of Trauma System for Grading Renal Trauma

Grade American Association for the Surgery of Trauma System


(Moore et al - J Trauma 1989; 29: 1664-6)

I Hematuria with normal imaging studies


Contusion
Nonexpanding subcapsular hematoma

II Nonexpanding perinephric hematoma confined to retroperitoneum


Superficial laceration <1 cm deep without collecting system injury

III Renal laceration >1 cm deep without collecting system injury

IV Renal laceration into the collecting system


Main renal artery or vein injury with contained hemorrhage
Segmental infarction without associated laceration
Expanding subcapsular hematoma compressing kidney

V Shattered or devascularized kidney


Ureteropelvic avulsion
Complete laceration or thrombus of the main renal artery or vein

CHAPTER 5: ESSENTIALS OF URORADIOLOGY 117


7. Ureteral Disease

Urinary extravasation is also characterized by


extraluminal contrast, but unlike active hemor-
Congenital Ureteropelvic Junction

rhage, the contrast accumulates progressively over a


Obstruction

multiphase CT study, rather than being primarily Congenital ureteropelvic junction obstruction is the
transient and in the arterial phase. In addition, a uri- most common congenital anomaly of the urinary
nary leak is generally contiguous with the upper tract, and 10%–40% of cases are bilateral. The dis-
track at some point. order is due to functional narrowing of the uretero-
pelvic junction, producing variable degrees of
Pseudoaneurysm appears as an intense focus of con- hydronephrosis that may be intermittent. The condi-
trast in a laceration or hematoma during the early tion may be due to intrinsic abnormal motility of the
phase of enhancement but, unlike active hemor- ureteropelvic junction or a crossing anomalous ves-
rhage, the area of enhancement is focal and rounded sel, and some controversy exists as to the relative
rather than linear or flame-like. frequency of these etiologies (although it should be
noted that these mechanisms are not mutually
Renal laceration appears as a jagged or linear exclusive and, for example, a congenital anomalous
parenchymal disruption which may contain fresh or crossing vessel might cause disordered motility as a
clotted blood. pressure effect on the developing ureteropelvic
junction). Congenital ureteropelvic junction
obstruction may present as an abdominal mass (i.e.,
the hydronephrotic kidney) in the neonate. The dis-
Traumatic Renal Devascularization

A devascularized kidney after trauma is a grade order is being increasingly discovered prenatally,
5 injury. CT findings of a non-enhancing kidney are due to the increasing use of obstetric ultrasound. In
distinctive (Figure 64). These patients often have many cases, the abnormality is clinically silent until
little to no hematuria or hematoma, and the devas- adulthood when symptoms of flank pain, fever or,
cularization is usually the result of a partial dissect- rarely, hypertension cause the patient to seek medi-
ing tear with thrombus in the renal artery. Success- cal attention.
ful surgical salvage is rare, but perhaps worth
attempting if the other kidney is absent or injured. At imaging, the condition results in hydronephrosis
Nephrectomy may be appropriate for an unstable with a non-dilated ureter, in the absence of a
patient, or for the development of hypertension as a detectable stone or stricture at the ureteropelvic
delayed complication. junction (Figure 65). Milder or intermittent forms
may be difficult to distinguish from a prominent
extrarenal pelvis, and imaging during acute symp-
toms or the use of a diuretic renogram may then be
Indications for Surgery

Probably the only absolute indication for surgery helpful. Conversely, with longstanding or very high
after renal trauma is life-threatening renal hemor- grade obstruction, an essentially nonfunctioning
rhage. Relative indications include: the presence of kidney may be present. CT or MR angiography may
extensively devitalized tissue (> 50% of the renal be used to depict an associated crossing vessel.
parenchyma), urinary extravasation that cannot be
controlled by stenting or nephrostomy, and arterial
thrombosis. Eighty-seven percent of urinary leaks
Ureteral Carcinoma

can be managed without surgery. Intermediate grade Ureteral carcinoma is a rare malignancy and
injuries can also usually be handled without surgery, accounts for only about 1% of all urinary tract
and aggressive monitoring, percutaneous drainage tumors. Most are transitional cell carcinomas. At
and angiographic embolization can help reduce the IVU or CT urography, ureteral carcinomas are typi-
laparotomy rate to about 10%. cally seen as a solitary non-calcified filling defect in
the upper tract, often with upstream dilatation. The
ureter below the filling defect is often dilated for a
short distance, presumably because it is “held open”
by the firm tumor. This results in the classic “gob-

118 EDUCATIONAL REVIEW MANUAL IN UROLOGY


8. Bladder Disease

let” sign (Figure 66). CT or MRI may be helpful in


demonstrating periureteral extension or more dis-
Bladder Cancer

tant spread of disease. Most patients have a history


of bladder transitional cell carcinoma, and any Bladder cancer is relatively uncommon, but does
Overview

upper tract filling defect in a patient with this history account for 2% of cancer deaths. Ninety percent of
should be considered recurrent transitional cell car- bladder cancers are transitional cell cancers, and
cinoma until proven otherwise (Figure 67). these are associated with smoking, aniline dyes and
increasing age. Eight percent are squamous cell car-
cinomas, for which the risk factors are stone and
chronic inflammation. Two percent are adenocarci-
Ureteral Fibroepithelial Polyp

Ureteral fibroepithelial polyp is a rare, benign nomas, and these are associated with congenital
ureteral tumor composed of a fibrovascular core anomalies and chronic inflammation. With respect
covered by transitional epithelium. Flank pain and to the latter, it should be noted that a bladder mass
hematuria are the most common presenting symp- arising in the midline anteriorly should raise the
toms. Ultrasound demonstrates a variable degree of possibility of urachal adenocarcinoma (Figure 69).
hydronephrosis. On IVU, fibroepithelial polyps
appear as a pedunculated filling defect of varying
length. Polyps of up to 13 cm in length have been
Imaging

described, and the finding of a long linear filling The imaging appearances of bladder cancer are
defect in the ureter should suggest the diagnosis somewhat variable, with low-grade tumors appear-
(Figure 68). Polyps are usually solitary, but may be ing as polypoid papillary masses within the bladder
multiple, and typically smooth, cylindrical and lumen (Figure 70), while higher grade tumors tend
mobile. Fibroepithelial polyps are most common in to result in more focal infiltrative wall thickening
the upper third of the ureter, and in the 20–40-year- (Figure 71). Surface calcification or encrustation on
old age group. polypoid masses is occasionally seen, and is rela-
tively distinctive (Figure 70). The detection rate of
bladder cancer by imaging, whether ultrasound,
IVU or CT, is approximately 70%–90%. All modal-
ities can miss smaller or flat tumors, and conse-
quently cystoscopy remains the standard of refer-
ence for the detection of bladder cancer.

Staging

For simplicity, bladder cancer staging can be


divided into tumors that are superficial, invasive
(into the bladder muscle layer, perivesical fat or
regional nodes) or metastatic (Table 8). Staging is
important because it allows prediction of prognosis
and treatment planning. Examination of Table 7
shows that the critical imaging features that might
be amenable to imaging detection are muscle inva-
sion, perivesical fat invasion, and lymph node
metastasis. Unfortunately, published results show
that evaluation of these features is of relatively lim-
ited accuracy for all modalities, although there are
times when unequivocal radiological findings may
be present (Figure 72). It should be noted that not
every soft density mass seen in the bladder at CT is a
bladder cancer, and the radiological differential

CHAPTER 5: ESSENTIALS OF URORADIOLOGY 119


Table 8

Simplified Staging System for Bladder Cancer, Highlighting the Important Findings that
May be Detected by Imaging (muscle layer invasion, extension into perivesical fat,
adenopathy and distant metastases)

Stage % Survival Main Rx options

Superficial 70% 82%–100% at 5 years TURBT, fulguration,


± intravesical Rx

Invasive 10% Muscle layer: 63%–83% at 5 years Radical cystectomy,


10% Perivesical fat: 45%–55% at 5 years radiotherapy
5% Regional nodes: 25% at 5 years (if nonsurgical)

Metastatic 5% 5% at 2 years Chemotherapy, RT,


palliative surgery

includes hematoma, pheochromocytoma and tying. Conditions which can cause neurogenic blad-
leiomyoma (Figure 73). der include: cerebrovascular disease, multiple scle-
rosis, Parkinson disease, motor neuron disease,
spinal cord injury, spina bifida, diabetes, pelvic
surgery or intervertebral disc herniation. Modern
Bladder Rupture

Traumatic bladder rupture may be extraperitoneal assessment of neurogenic bladder may require uro-
(50%–71%), intraperitoneal (25%–14%). dynamic and electrophysiological assessment for
Extraperitoneal rupture is generally due to perfora- complete evaluation. A variety of detailed classifi-
tion by a bony fragment (pelvic fractures are seen in cations have been developed to describe neurogenic
89%–100% of cases) or an avulsion tear at the fixa- bladder; however, from the imaging viewpoint,
tion points of the puboprostatic ligaments. It usually there are essentially 2 recognizable types—the spas-
occurs close to the bladder base anterolaterally. tic bladder and the atonic bladder.
Intraperitoneal rupture is typically a “burst” injury
where a blow to the abdomen perforates the dome of Spastic neurogenic bladder results in urination, is
a full bladder. At conventional or CT cystography, involuntary and frequent, and a true sensation of
extraperitoneal ruptures results in flame-shaped, fullness is lacking. At imaging, the bladder is small
starburst or feather-like perivesical contrast extrava- and trabeculated with a typical “pine cone” or
sation while intraperitoneal ruptures results in “Christmas tree” appearance (Figure 76).
intraperitoneal pools of contrast that outline bowel
loops (Figures 74-75). The distinction of the 2 types Atonic (or flaccid) neurogenic bladder typically
of bladder rupture is important, because 85% of results in retention with overflow incontinence.
extraperitoneal ruptures will heal with catheteriza- At imaging, the bladder is large, capacious and
tion alone, while most intraperitoneal ruptures smooth (Figure 77). Radiological distinction from a
require surgical repair. bladder that is physiologically large can sometimes
be difficult, but coexistent fecal loading of the
large bowel may help suggest an underlying neuro-
logical problem.
Neurogenic Bladder

Neurogenic bladder refers to bladder dysfunction


secondary to disturbance of the complex neurologi-
cal pathways that regulate bladder filling and emp-

120 EDUCATIONAL REVIEW MANUAL IN UROLOGY


9. Prostatic Disease

Zonal Anatomy Benign Prostatic Hyperplasia

Awareness of the often poorly understood zonal Imaging can be used in benign prostatic hyperplasia
anatomy of the prostate is important for interpreting to detect hydronephrosis, determine the post-void
imaging of both benign prostatic hyperplasia (a dis- residual urinary volume and determine the size of
ease of the central gland) and prostate cancer the prostate prior to treatment. These observations
(mainly a disease of the peripheral zone). The sim- can all be well-evaluated using ultrasound. Findings
plest conceptual approach to the zonal anatomy of on IVU that may be caused by prostatic enlargement
the prostate is the 2-compartment model, where the and bladder outlet obstruction include: a horizontal
prostate is likened to a cone containing a scoop of orientation to the normally downward course of the
ice cream. The “cone” is the peripheral zone, and distal ureter (so-called “J-hooking”),
makes up 70% of the prostate gland by volume in hydroureteronephrosis, ureteral tortuosity, bladder
young men. The ducts of the peripheral zone glands trabeculation, diverticula, elevation of the bladder
drain to the distal prostatic urethra. The “scoop of base and an increased post-void residual urinary
ice cream” is the central zone, and makes up 25% of volume (Figure 79). At cross-sectional imaging
the prostate gland volume in young men. The ejacu- (ultrasound, CT, or MRI), benign prostatic hyper-
latory ducts traverse the central zone, and the ducts plasia results in a heterogeneous nodular appear-
of the central zone drain to the region of the veru- ance to the transition zone of the prostate, some-
montanum clustered around the entry of the ejacula- times with small focal calcifications. The prostate
tory ducts. The final 5% of the prostate consists of may become extremely large and protrude into the
the transition zone, which is composed of 2 small bladder base, although there is only a loose relation-
bulges of tissue that surround the verumontanum in ship between symptoms of outlet obstruction and
a horseshoe-like fashion. This 2-compartment prostate volume.
model is deficient anteriorly, where the peripheral
zone is interrupted by the anterior fibromuscular
stroma, a band of smooth muscle mixed with
Prostate Cancer

fibrous tissue that forms a thick shield over the ante- Prostate cancer is the second most common fatal
rior aspect of the gland. As a result, the peripheral malignancy in men. The greatest dilemma in the
zone lies predominantly lateral and posterior to the care of prostate cancer patients is the limited ability
central zone. of all available methods (including imaging) to
predict tumor behavior, and allow informed treat-
Confusion regarding the zonal anatomy of the ment stratification that aligns active surveillance
prostate arises because the zonal anatomy changes with indolent tumors and definitive treatment (by
with age (Figure 78). The transition zone is the por- surgery or radiation) with aggressive tumors. As
tion of the prostate that develops benign prostatic such, the use of imaging in the evaluation of
hyperplasia. As a result, the transition zone prostate cancer is a subject of controversy, with the
becomes progressively bigger with age and com- exception of CT or bone scintigraphy for the detec-
presses the surrounding central zone. The latter tion of metastatic disease (Figures 80-81). Typi-
becomes the surgical pseudocapsule. One approach cally, prostate cancer appears as a focus of reduced
to incorporate and simplify this age-related com- echogenicity in the peripheral zone of the prostate at
plexity is to refer to the transition and central zones transrectal ultrasound, sometimes with increased
collectively as the central gland. Using this termi- vascularity at Doppler interrogation (Figure 82).
nology, the prostate is composed of the peripheral Transrectal ultrasound is undoubtedly useful in the
zone and central gland, such that the central gland is localization of the prostate for needle biopsies, but
composed mainly of central zone tissue in young appears of limited accuracy in the localization or
men and mainly of transition zone tissue in older staging of tumor, even with the addition of Doppler
men. Given that prostate cancer is largely a disease techniques. At endorectal MRI, prostate cancer is
of older men, in the population that comes to MRI, it typically seen as an ovoid or crescentic subcapsular
is reasonable to regard the terms central gland and focus of reduced T2 signal intensity (Figure 83). In
transition zone as essentially synonymous. single institutional studies, endorectal MRI has

CHAPTER 5: ESSENTIALS OF URORADIOLOGY 121


10. Uretheral Disease

shown considerable promise in the diagnosis,


staging and post-treatment surveillance of prostate
Urethral Stricture

cancer, particularly when combined with the newer Inflammation and trauma are the leading causes of
techniques of spectroscopic, perfusion and diffusion acquired urethral strictures, including prior gonor-
imaging, but the results of multi-institutional rhea and catheterization. Post-traumatic strictures
studies have been somewhat disappointing. Some of result from partial or complete urethral rupture, usu-
the controversy with respect to the role of imaging ally as a result of pelvic fracture. Imaging plays a
in local evaluation of prostate cancer arises from the major role in the diagnostic evaluation of urethral
variability of published results, which raises con- stricture. The study of choice is a combination of
cerns about interobserver variability and lack of retrograde urethrogram (which distends the anterior
reproducibility. For example, the reported staging urethra) and voiding cystourethrogram (which dis-
accuracy of transrectal ultrasound ranges from tends the posterior urethra). In addition to distend-
58%–90%, and the reported staging accuracy of ing different portions of the urethra, such a com-
endorectal MRI ranges from 54%–90%. Nodal bined approach allows better evaluation of the
evaluation is equally limited, with very variable upper end of a stricture, may reveal a stricture
published results. obscured by obliquity on the retrograde urethro-
gram and allows assessment of upstream dilatation
(indicating functionally significant narrowing)
(Figure 84). High-frequency ultrasound can also be
used for evaluation of the stricture length (Figure
85), particularly as an adjunct to surgical planning.

Urethral Disruption

The prostatic urethra, membranous urethra and uro-


genital diaphragm form a unit during trauma, so the
usual site of disruption is the bulbomembranous
junction. Posterior urethral trauma occurs in
10%–15% of pelvic fractures, almost always in
men. Retrograde urethrography is the most common
imaging test used to evaluate posterior urethral
trauma, and is indicated in a man with a pelvic frac-
ture who has gross hematuria, blood at the meatus,
perineal swelling, an impalpable prostate or an
inability to urinate. The radiographic findings can
be used to classify the urethral injury as types I, II
and III. In type V, the urethra is intact, but is
stretched and narrowed due to elevation of the blad-
der by hematoma. In type II, there is a tear at the
prostatomembranous junction (previously
described as the “classic” injury), and contrast
extravasates into the true pelvis above the urogeni-
tal diaphragm. In type III, the urethra is torn below
the urogenital diaphragm and contrast extravasates
into the perineum and scrotum. Type II or III
injuries may be subclassified as partial or complete.
A tear is indicated by the extravasation of contrast
(Figure 86). If the bladder also fills with contrast,
the tear is partial (30%), but if the bladder does not
fill, the tear is complete (70%). If the urethrogram is

122 EDUCATIONAL REVIEW MANUAL IN UROLOGY


11. Testicular Masses

normal, the catheter should be advanced into the


bladder and a cystogram performed to evaluate for
Testicular Cancer

possible bladder rupture. While urethrography has Testicular cancer accounts for 1% of all malignan-
been the traditional modality used to evaluate ure- cies in men, and is the most common cancer
thral injuries, the length of urethral injury and extent between the ages of 15–34 years. Most testicular
of associated soft tissue damage can be difficult to cancers present as a palpable painless testicular
evaluate. MRI can be used to complement conven- mass that is hypoechoic at ultrasound (Figure 89). If
tional urethrography in this setting. By accurately performed, MRI demonstrates a mass which is rela-
defining the pelvic anatomy, it provides valuable tively isointense to the surrounding normal testicu-
information to guide the surgical plan and determine lar parenchyma on T1-weighted images and hypo-
whether a transperineal or suprapubic approach intense on T2-weighted images, and which shows
should be used. brisk and early enhancement after intravenous
gadolinium. This combination of findings in a
young man is virtually diagnostic of testicular can-
cer, and warrants surgical exploration and radical
Urethral Diverticulum

Urethral diverticula occur in as many as 1%–6% of orchiectomy. Percutaneous biopsy is rarely per-
all women. They typically arise from the posterior formed for preoperative confirmation, because it
wall of the urethra, bulging into the vagina or vagi- may cause malignant spread along the needle
nal introitus. Occasionally, they arise laterally or tract, and because the probability of malignancy
anteriorly. The origin is unknown. Most are proba- exceeds 90%.
bly acquired, though some may be congenital. Pos-
sible causes include: infection and obstruction of
periurethral glands, trauma from childbirth or
Simple Testicular Cysts

repeated catheterization. Symptoms are often non- Simple testicular cysts are recognized with increas-
specific, consisting of urgency, frequency, dysuria, ing frequency due to increasing use of ultrasound
dyspareunia, recurrent urinary tract infection, drib- and improvements in ultrasound technology. There
bling and incontinence. Careful examination may are 2 types: cysts of the tunica albuginea and intrat-
demonstrate a palpable tender suburethral mass. esticular cysts. Tunica albuginea cysts may be inci-
Occasionally, pus may be expressed from such a dental findings, or present with pain, swelling and a
mass, or it may contain a palpable store or even a firm, pinhead-sized mass. The incidence is said to
hard nodule, due to complicating carcinoma. Ure- increase with age. The etiology is unknown; they
thral diverticula in women may be difficult to visu- may be due to postinflammatory cystic dilatation of
alize radiographically. They may be visible on a individual ductules. At ultrasound, they are small,
voiding cystourethrogram, but only if they fill and subcapsular, sharply demarcated, uni- or multilocu-
distend with contrast. If the diagnosis is strongly lar cysts, with no solid portions and no disturbance
suspected clinically, then special techniques to gen- of the adjacent parenchyma. Intratesticular cysts
erate high pressure in the urethra should be used to represent incidental findings and are usually impal-
force contrast into the diverticulum. This can be pable. They are typically solitary and adjacent to the
achieved by occluding the meatal opening with a rete testis. Intratesticular cysts may contain sperma-
finger during voiding, or by using a special double- tozoa and are frequently associated with spermato-
balloon catheter technique that essentially seals the celes, suggesting they may be due to postinflamma-
urethra at both ends and forces contrast into the tory or posttraumatic obstruction of the testicular
diverticulum from a hole in the catheter between the tubule system. In practice, difficulties arise when a
balloons (Figure 87). Transvaginal sonography and very small cyst cannot be definitely characterized,
MRI have also been used to image urethral divertic- or when a cyst deep in the testis raises concern of a
ula in women, particularly when standard imaging small cystic teratoma. Such lesions may require
has failed to demonstrate a suspected diverticulum imaging surveillance.
(Figure 88).

CHAPTER 5: ESSENTIALS OF URORADIOLOGY 123


may represent a benign monodermal form of ter-
atoma. However, carcinoma in situ is not found in
Tubular Ectasia of the Rete Testis

Tubular ectasia of the rete testis is a benign condi- adjacent seminiferous tubules, unlike typical ter-
tion, often bilateral, which is usually seen in elderly atoma. They are considered benign tumors which
men. It is commonly associated with epididymal can be safely treated by local excision, with organ
abnormalities (spermatoceles, epididymal cysts and preservation.
epididymitis). Ultrasound shows numerous small
tubular cystic structures within the rete testis
(Figure 90) —a characteristic finding. MRI shows
Dermoid Cyst

nonenhancing serpiginous tubular structures of low These differ histologically from epidermoid cysts in
T1 and high T2 signal intensity in the vicinity of the that they contain skin appendages. They are rare
rete testis. No specific treatment is required. tumors of the testis, though the rarity may be par-
tially due to misclassification of these tumors as epi-
dermoid cysts, or to classification of the tumors as
benign monodermal mature teratomas. They present
Epidermoid Cyst

Epidermoid cysts account for<1% of all testicular as painless testicular masses. No case of metastasis
neoplasms. The tumor consists histologically of arising from a testicular tumor regarded as being a
cystic cavities which contain desquamated kera- dermoid cyst has been well-documented, and these
tinized epithelium and are lined by stratified squa- cysts can be considered benign, differentiated ter-
mous epithelium. Skin appendages, such as seba- atomas. To our knowledge, imaging features have
ceous glands and hair follicles, do not occur. Price not been specifically described. Based on the histo-
defined the following criteria for the pathological logical similarity, overlap with the imaging findings
diagnosis of testicular epidermoid cyst: of epidermoid cysts appears likely.

• The lesion is within the testicular parenchyma. Leydig Cell Tumor

• The cyst contains keratinized debris or amorphous Leydig cell tumors arise from the Leydig cells of the
material with cleft-like spaces. testicular stroma. They are the most common of the
non-germ cell testicular tumors, and account for
• The cyst wall consists of fibrous tissue with an 2%–3% of all testicular neoplasms. The age distri-
inner lining of squamous epithelium. bution is bimodal, occurring in children between
5–10 years and in adults between 30–35 years. Only
• Teratomatous elements are not present in either 5%–10% of Leydig cell tumors are malignant, and
the cyst wall or the adjacent parenchyma. most of these occur in the elderly. Both testes are
involved in 5%–9% of patients. The tumors may
• The cyst wall is discrete and separate from the secrete estrogens, androgens, progesterone or
tunica albuginea. corticosteroids, and consequently 30%–40% of
patients have an endocrinopathy. Patients may pre-
Epidermoid cysts present as painless, palpable, firm sent with gynecomastia or precocious puberty, with-
solitary masses. Bilateral epidermoid cysts are very out a palpable testicular mass. Leydig cell tumors
rare. At ultrasound, epidermoid cysts are well-cir- appear at ultrasound as well-demarcated, homoge-
cumscribed ovoid lesions with variable echogenic- neous hypoechoic masses, a nonspecific appearance
ity. The cyst wall may be hypo- or hyperechoic. The indistinguishable from germ cell cancer. A hypere-
cyst content may be anechoic, uniformly hypoe- choic pattern has been described in 2 cases. The
choic, heterogeneously hypoechoic, or consist of tumors are usually small and slow-growing.
concentric rings of hypo- and hyperechogenicity. Orchiectomy is the treatment of choice, although
The latter can give rise to a typical “target” or partial orchiectomy has been described in a case of
“onion” appearance. This laminated appearance bilateral tumor. The prognosis is good, except for
should suggest the diagnosis. Calcification of the malignant Leydig cell tumors, which are generally
cyst wall or cyst content can occur. These lesions unresponsive to treatment.

124 EDUCATIONAL REVIEW MANUAL IN UROLOGY


Sertoli Cell Tumor Testicular Mass Due to Disseminated

Sertoli cell tumors arise from the testicular stroma,


Malignancy

like Leydig cell tumors. They usually occur in Lymphoma and leukemia are the most common
childhood. Even in this population they are rare; extratesticular malignancies to cause testicular
Sertoli cell tumors accounted for <4% of a series of involvement. Lymphoma accounts for 1%–7% of
556 pediatric testicular tumors. They typically pre- all testicular tumors, and is a frequent cause of tes-
sent as a unilateral testicular mass under 1 year of ticular enlargement in men over 50 years (Figure
age, though bilateral cases have been described. 91). The testis is usually involved secondary to sys-
Echogenicity is variable, but the dense collagenous temic disease, it is sometimes a site of post-
stroma of the tumor may result in a hyperechoic pat- chemotherapy recurrence and rarely is the primary
tern. Endocrinopathy is rare, but can result in organ. Involvement is bilateral, either simultaneous
gynecomastia, virilization or feminization. Ten per- or successive, in 50% of cases. Leukemic infiltra-
cent of adult cases are malignant, but pediatric cases tion is usually seen in children, and the testis is the
are nearly always benign and adequately treated by most common site of relapse of acute leukemia.
simple orchiectomy. Large cell calcifying Sertoli About 50% of patients have bilateral involvement.
cell tumor is a subtype characterized microscopi- Gray-scale sonography of both lymphomatous and
cally by the presence of large Sertoli cell with abun- leukemic testicular involvement typically shows
dant eosinophilic cytoplasm, and macroscopically diffuse or multifocal decreased echogenicity,
by the presence of calcifications of various sizes. which can be more subtle and ill-defined than the
This results in a characteristic sonographic picture typical well-circumscribed hypoechoic mass of pri-
of multiple nodular calcifications of varying sizes, mary testicular malignancy. Lymphomatous and
in otherwise normal-appearing testes. The calcifica- leukemic testicular involvement is generally hyper-
tions are macroscopic, and are not to be confused vascular on color Doppler sonography, irrespective
with testicular microlithiasis. This tumor type con- of tumor size or extent. This is in contrast to pri-
stitutes one of the components of Carney’s complex, mary tumors, which are usually hypovascular on
and should alert the physician to the possibility of color Doppler when <1.6 cm, and hypervascular
cardiac myxomas in the patient and family mem- >1.6 cm. These findings may help suggest the diag-
bers. nosis in the appropriate clinical setting.

Nonlymphomatous metastases to the testes are


rare, and usually incidental autopsy findings.
Adenomatoid Tumor

Adenomatoid tumors usually occur in the epi- Reported autopsy rates vary from 0.02%–2.5%. A
didymis, but can occur in the testis and are said to more recent autopsy study of 1,715 solid extrates-
account for up to 5% of testicular tumors. They are ticular malignancies showed metastases to the
thought to arise from mesothelial cells. They arise testes in 16 (0.9%). Of these, only 6 (38%) were
from the tunica albuginea and therefore are always macroscopically evident. The common primary
found in the periphery of the testis. Most present as sites are lung, prostate, stomach and large bowel—
a slow-growing mass in the 3rd to 5th decade. accounting for 75% of cases (Figure 92). Rare but
On imaging they appear as well-circumscribed reported primary tumors are melanoma, renal cell
masses which are of variable echogenicity, but gen- carcinoma, pancreatic carcinoma, adrenocortical
erally iso- to hyperechoic. This appearance should carcinoma, transitional cell carcinoma and cholan-
prompt intraoperative frozen section biopsy, since if giocarcinoma. Neuroblastoma and Wilms’ tumor
the diagnosis is confirmed radical orchiectomy can are possible primary sites in children. Imaging
be averted. findings of metastases to the testes have not been
specifically described.

CHAPTER 5: ESSENTIALS OF URORADIOLOGY 125


12. Pediatric Uroradiology

abnormal renal histogenesis and are precursor of


Wilms’ tumor. It is characterized by the presence of
Neuroblastoma

Neuroblastoma is a primitive neuroectodermal multiple, well-defined round or ovoid foci in the


tumor arising from the sympathetic nervous system. periphery of the renal cortex (Figure 96). The tumor
It may occur anywhere in the neck, thorax, abdomen may spread to paraaortic and renal hilar nodes, lung,
or pelvis, with approximately one-third occurring in bones (less common) and liver. Hepatic or pul-
the adrenal glands. Neuroblastoma is the second monary metastases are found in 30% of patients at
most common solid tumor of childhood. There is a presentation. The lesion may also invade vascular
slight male predominance, and incidence is structures. Syndromes associated with an increased
increased in neurofibromatosis. Twenty-five per- incidence of Wilms’ include: Beckwith-Wiedemann
cent occur in the first year of life, and up to sixty syndrome, hemihypertrophy, sporadic aniridia,
percent occur in the first two years of life. The usual Drash syndrome and chromosome 11 deletion.
presentation is an incidentally discovered abdomi-
nal mass, often by a parent. Plain radiographs are Radiological evaluation demonstrates an intrarenal
often helpful in diagnosis as at least half of these mass, and the initial study is generally an abdominal
tumors contain calcium, which is usually stippled ultrasound (Figure 97). The tumor can appear
but may be linear or ring-shaped (Figure 93). This is evenly echogenic, but more echolucent areas repre-
in distinction to Wilms’ tumor, which rarely calci- senting necrosis may be seen. US can also be used to
fies (≤5%). Other plain film findings include par- evaluate the renal vein or inferior vena cava for
avertebral widening, bony destruction or widening tumor extension. When a tumor is identified within
of the interpedicular distances. Ultrasound can usu- the inferior vena cava, the proximal extent of the
ally distinguish neuroblastoma from other solid thrombus must be established prior to operation.
abdominal masses. IVU has some value in differen- Cardiac extension of such a tumor thrombus may
tiating neuroblastoma from Wilms’ tumor when this produce few or no signs, but it can, on rare occasion,
distinction is unclear. Neuroblastoma classically be responsible for sudden death following manipu-
displaces the kidney without distorting the renal lation and embolization of the tumor at laparotomy.
collecting system, while the collecting system is CT of the abdomen usually shows the tumor to be a
usually deformed by Wilms’ tumor. In addition, the large, complex renal mass, which is typically
kidney may appear displaced laterally with neurob- sharply marginated (Figure 98).
lastoma. CT is the imaging procedure of choice in
evaluating patients with neuroblastoma (Figure 94).
CT is able to depict the full extent of the lesion,
Renal Ectopia

including vascular encasement and metastases. Up As the fetal kidneys ascend from their pelvic posi-
to 85% of tumors are calcified on CT. While helpful, tion to meet the adrenal glands, each kidney
the role of MRI in the evaluation of patients with acquires blood supply from the neighboring vessels.
neroblastoma has yet to be determined (Figure 95). The initial supply from the external and internal
iliac vessels is lost during this process, and blood
supply directly from the aorta is acquired in the 8th
week of development. Any abnormality in acquir-
Nephroblastoma (Wilms’ tumor)

Wilms’ tumor is the most common renal neoplasm ing such blood supply or an abnormality of the spine
in children with a peak incidence at 2 years of age. may prevent cephalic migration from occurring.
Nearly 50% of cases occur prior to 3 years of age This will result in a renal ectopia or an abnormal
and 75% before 5 years of age (the tumor is rare in position of the kidney. The findings on urography
newborns). Patients present with an abdominal depend on the degree of renal function in the pelvic
mass (90%), hypertension (50%), pain and/or gross kidney and the presence of associated abnormali-
hematuria. The tumor is bilateral in 5%–10% of ties. On sonography, the reniform mass will be pre-
cases. Bilateral tumors are virtually always associ- sent in the pelvis with a characteristic pattern of
ated with nephroblastomatosis. Nephroblastomato- renal sinus echoes. A functioning mass of renal
sis is the presence of multiple or diffuse nephro- parenchyma can be usually identified on CT.
genic rests, embryonal tumors that result from Ectopic kidneys are more susceptible to infection,

126 EDUCATIONAL REVIEW MANUAL IN UROLOGY


obstruction, blunt trauma and inadvertent intraoper- projected more medially than the ureter. The diag-
ative injury. The most common renal ectopia is the nosis may be difficult to make on ultrasound. Both
pelvic kidney (Figure 99), which is often associated CT (Figure 102) and MRI demonstrate the posi-
with conditions like ureteropelvic junction obstruc- tional abnormality of the kidneys and the connect-
tion, reflux, infection and urolithiasis. Vascular sup- ing band.
ply to the kidney is from a low-lying renal artery
either from the aorta or iliac vessels. Multiple renal
vessels are common. Bilateral pelvic kidneys are
Reflux

rare. Due to its characteristic central sinus complex Vesicoureteral reflux is the retrograde passage of
and corticomedullary differentiation, it is easily dis- urine from the bladder into the ureter. This is abnor-
tinguished from a primary pelvic tumor on ultra- mal, because the normal vesicoureteral reflux junc-
sound. Crossed renal ectopia is the term used when tion acts as a 1-way valve, allowing the normal ante-
both kidneys lie on the same side and can be fused grade passage of urine into the bladder, but prevent-
or nonfused, fused being more common (Figure ing flow in the reverse direction. Vesicoureteral
100). The vascular supply is often anomalous and reflux may be primary or secondary. Primary vesi-
varying degrees of malrotation are seen. Thoracic coureteral reflux is common in childhood, and is
kidney is an uncommon anomaly where the kidney believed to be due to a developmental deficiency in
lies above the diaphragm, having herniated through the muscle layer of the ureterotrigonal region.
the foramen of Bochdalek. Vessels may come off Familial cases have been described. Other congeni-
from the high abdominal or thoracic aorta. tal causes of vesicoureteral reflux include: complete
ureteral duplication (reflux typically occurs into the
ureter of the lower pole moiety), ectopic ureter,
prune belly syndrome and congenital (Hutch) peri-
Horseshoe Kidney

Horseshoe kidney is a congenital anomaly in which ureteral diverticulum. Acquired causes include
the kidneys are connected by an isthmus, usually in bladder wall edema or fibrosis, prostatectomy, blad-
the lower poles. The band of parenchymal tissue has der neck incision and ureteral reimplantation. Vesi-
its own blood supply. In some cases, the connecting coureteral reflux is a potentially serious condition,
band consists purely of fibrous tissue. It is thought because it may lead to renal damage by allowing
to occur due to an abnormal position of the umbili- reflux of infected urine from the bladder to the kid-
cal artery which disturbs the normal cephalad ney, which results in pyelonephritis, or by allowing
migration of the kidneys, so that it stops at the infe- transmission of bladder voiding pressures to the
rior mesenteric artery. The blood supply is usually kidneys, causing hydronephrosis. Resulting renal
from multiple arteries including the aorta, the infe- disease is known as reflux nephropathy. Patients
rior vena cava, the common external and internal may present with pyelonephritis, cystitis or uremic
iliac arteries and the inferior mesenteric artery. The symptoms. Asymptomatic pyelonephritis may be
isthmus is usually in front of the aorta and inferior discovered as an incidental finding on routine uri-
vena cava. Horseshoe kidney is the most common nalysis. The incidence of vesicoureteral reflux in
congenital anomaly of renal form, occurring in 1 in healthy children is under 1%, but is 20%–50% in
400 births with a 2:1 male predominance, without children with urinary tract infection.
clear evidence of hereditary transmission. One-third
of patients remain asymptomatic, while the remain- The definitive test for the diagnosis of reflux is con-
der may have calculi (75% metabolic, 25% stru- ventional contrast cystogram (Figure 103). Films
vite), infection or obstruction. One-third may have are taken during bladder filling, during voiding and
an associated ureteropelvic junction obstruction. after voiding. An international grading system has
Patients are also more prone to injury from blunt come into general use for the assessment of the
abdominal trauma. There is an increased incidence severity of reflux, which allows objective compari-
of malignancy in horseshoe kidneys. On intra- son of therapeutic modalities. In grade I reflux, con-
venous urography, the kidneys are malrotated with trast refluxes into the ureter only, opacifying part or
the lower poles rotated medially (Figure 101). The all of the ureter. In grade II reflux, contrast reaches
pelvis is rotated anteriorly, with the lower calcyces the renal pelvis, which is not dilated. In grade III

CHAPTER 5: ESSENTIALS OF URORADIOLOGY 127


reflux, contrast reaches the renal pelvis, with mild
dilatation of the ureter and pelvicalyceal system. In
Posterior Urethral Valves

grade IV reflux, there is moderate pelviureteroecta- Posterior urethral valves are anomalous develop-
sis, with obliteration of the forniceal angles but mental mucosal folds in the prostatic urethra, and
preservation of the papillary impressions. In grade are the embryological equivalent of the hymen in
V reflux, there is moderate-to-severe pelvi- females. Posterior urethral valves are the most com-
ureteroectasis, with near complete or complete mon cause of urethral obstruction in children, with
obliteration of the papillary impressions. (Reflux an incidence of 1 in 5,000–8,000 male births. The
may also be demonstrated by voiding radionuclide clinical presentation of posterior urethral valves is
cystography; it is sometimes detected by ultra- highly variable, and related to the degree of obstruc-
sound.) Vesicoureteral reflux may be unilateral, tion. Severe cases may cause intrauterine or postna-
bilateral or intermittent. Children with lower grades tal death. Antenatal ultrasound may suggest the
of primary vesicoureteral reflux can often be suc- diagnosis by demonstrating bilateral
cessfully managed with medical treatment, with hydroureteronephrosis, an enlarged bladder and
spontaneous resolution as they grow up. Other chil- oligohydramnios. After birth, children may present
dren may require surgery. with mild-to-severe symptoms of outflow obstruc-
tion. The urinary stream is often poor and dribbling.
Hydronephrotic kidneys may be palpable, as may
the enlarged bladder. Occasionally, the only clinical
Ureterocele

Ureterocele is a dilatation of the distal end of the feature may be failure to thrive. Urinary tract infec-
ureter where it enters the bladder. It may be uni- or tions may occur, as may renal impairment or failure.
bilateral. This is usually thought to be caused by Voiding cystourethrography is the definitive radio-
narrowing of the ureteral orifice. The back pressure graphic study (Figure 106). The posterior urethra is
causes the bulging of the lower ureter into the blad- elongated and dilated, with a prominent bladder
der. This is commonly associated with duplex kid- neck. The valves may be seen as linear radiolucent
neys, in which case this is always associated with defects. Typically, retrograde urethrography is nor-
the upper element of the duplex. It can be seen, mal, because the retrograde flow compresses the
however, at the lower end of a singleton ureter valves against the urethral wall. In posterior urethral
(orthotopic ureterocele). Ureterocele is the primary valves, the dilated prostatic urethra has a cylindrical
consideration for a filling defect in the pediatric outline with a rounded inferior margin, whereas in
bladder, and the diagnosis can be easily confirmed bladder neck dysfunction, the dilated urethra has a
by ultrasound (Figure 104). Ureteroceles may be triangular configuration with a sharp inferior beak.
detected as an incidental “cobra-head” appearance However, endoscopy may be required for definitive
of the distal ureter at IVU (Figure 105). These vary distinction between these 2 conditions.
in size with ureteral peristalsis. When small they
may not be clearly visible, unless observed for a
period of time when ureteral peristalsis will cause
distension of the distal ureter and the ureterocele
within the bladder. When renal function is good,
these can also be demonstrated by excretory urogra-
phy as a thin-walled structure within the bladder.
When large and poorly functioning, they can be
seen as a negative filling defect on micturating cys-
tourethrography but can easily be masked by the
dense contrast during this examination.

128 EDUCATIONAL REVIEW MANUAL IN UROLOGY


13. References

10. Dawson P, Sidhu PS. Is there a role for corticos-


teroid prophylaxis in patients at increased risk
Imaging Modalities for Uroradiology

1. Cohan RH, Sherman LS, Korobkin M, Bass JC, of adverse reactions to intravascular contrast
Francis IR. Renal masses: assessment of corti- agents? Clin Radiol. 1993;48:225-226.
comedullary-phase and nephrographic-phase
CT scans. Radiology. 1995;196:445-451. 11. Katayama H, Yamaguchi K, Kozuka T,
Takashima T, Seez P, Matsuura K. Adverse
2. Chow LC, Sommer FG. Multidetector CT urog- reactions to ionic and nonionic contrast media.
raphy with abdominal compression and three- A report from the Japanese Committee on the
dimensional reconstruction. AJR Am J Safety of Contrast Media. Radiology.
Roentgenol. 2001;177:849-855. 1990;175:621-628.

3. Tippins RB, Torres WE, Baumgartner BR, 12. Lasser EC, Berry CC, Mishkin MM,
Baumgarten DA. Are screening serum creati- Williamson B, Zheutlin N, Silverman JM. Pre-
nine levels necessary prior to outpatient CT treatment with corticosteroids to prevent
examinations? Radiology. 2000;216:481-484. adverse reactions to nonionic contrast media.
AJR Am J Roentgenol. 1994;162:523-526.
4. American College of Radiology. Manual on
Contrast Media. Edition 4.1, 1998 (revised 13. Public Health Advisory—Gadolinium-
2001). containing Contrast Agents for Magnetic
Resonance Imaging (MRI): Omniscan,
5. Tepel M, van der Giet M, Schwarzfeld C, OptiMARK, Magnevist, ProHance, and
Laufer U, Liermann D, Zidek W. Prevention MultiHance. Available at:
of radiographic-contrast-agent-induced reduc- http://www.fda.gov/Drugs/DrugSafety/Postma
tions in renal function by acetylcysteine. rketDrugSafetyInformationforPatientsand-
N Engl J Med. 2000;343:180-184. Providers/ucm142882.htm. Accessed
June 7, 2009.
6. Bagshaw SM, McAlister FA, Manns BJ, Ghali
WA. Acetylcysteine in the prevention of con- 14. Public Health Advisory—Update on Magnetic
trast-induced nephropathy: a case study of the Resonance Imaging (MRI) Contrast Agents
pitfalls in the evolution of evidence. Arch Containing Gadolinium and Nephrogenic
Intern Med. 2006;166:161-166. Fibrosing Dermopathy. Available at:
http://www.fda.gov/Drugs/DrugSafety/Postma
7. Solomon R, Werner C, Mann D, D'Ella J, Silva rketDrugSafetyInformationforPatientsand-
P. Effects of saline, mannitol, and furosemide to Providers/DrugSafetyInformationforHeathcare
prevent acute decreases in renal function Professionals/PublicHealthAdvisories/ucm124
induced by radiocontrast agents. N Engl J Med. 344.htm. Accessed June 7, 2009.
1994;331:1416-1420.
15. Sadowski EA, Bennett LK, Chan MR, et al.
8. Merten GJ, Burgess WP, Gray LV, et al. Preven- Nephrogenic systemic fibrosis: risk factors and
tion of contrast-induced nephropathy with incidence estimation. Radiology.
sodium bicarbonate: a randomized controlled 2007;243(1):148-157.
trial. JAMA. 2004;291:2328-2334.
16. Broome DR, Girguis MS, Baron PW, Cottrell
9. Siegle RL, Halvorsen RA, Dillon J, Gavant AC, Kjellin I, Kirk GA. Gadodiamide-
ML, Halpern E. The use of iohexol in patients associated nephrogenic systemic fibrosis: why
with previous reactions to ionic contrast mate- radiologists should be concerned. AJR Am J
rial. A multicenter clinical trial. Invest Radiol. Roentgenol. 2007;188:586-592.
1991;26:411-416.

CHAPTER 5: ESSENTIALS OF URORADIOLOGY 129


17. FDA Drug Safety Newsletter. Updated 8. Schwartz LH, Panicek DM, Koutcher JA, et al.
8/13/2009. Available at: http://www.fda.gov/ Adrenal masses in patients with malignancy:
Drugs/DrugSafety/DrugSafetyNewsletter/ prospective comparison of echo-planar, fast
ucm142889.htm. Acessed March 20, 2011. spin-echo, and chemical shift MR imaging.
Radiology. 1995;197:421-425.
18. ACR Manual on Contrast Media Version 7,
2010. ACR Committee on Drugs and Contrast 9. Young WF Jr. Minireview: primary
Media. aldosteronism - changing concepts in
diagnosis and treatment. Endocrinology.
Adrenal Masses 2003;144:2208-2213

1. Lee MJ, Hahn PF, Papanicolaou N, et al. 10. Berkman WA, Bernardino ME, Sewell CW,
Benign and malignant adrenal masses: CT Price RB, Sones PJ Jr. The computed tomogra-
distinction with attenuation coefficients, size, phy-guided adrenal biopsy. An alternative to
and observer analysis. Radiology. surgery in adrenal mass diagnosis. Cancer.
1991;179:415-418. 1984;53:2098-2103.

2. Korobkin M, Brodeur FJ, Yutzy GG, et al. Dif- 11. Gollub MJ, Bosniak MA, Schlossberg P,
ferentiation of adrenal adenomas from nonade- Chachoua A. Extension of a secondary adrenal
nomas using CT attenuation values. AJR Am J neoplasm into the inferior vena cava. Abdom
Roentgenol. 1996;166:531-536. Imaging. 1994;19:359-360.

3. Korobkin M, Brodeur FJ, Francis IR, Quint LE, 12. Mukherjee JJ, Peppercorn PD, Reznek RH,
Dunnick NR, Goodsitt M. Delayed enhanced et al. Pheochromocytoma: effect of
CT for differentiation of benign from malignant nonionic contrast medium in CT on circulating
adrenal masses. Radiology. 1996;200:737-742. catecholamine levels. Radiology. 1997;202:
227-231.
4. Korobkin M, Brodeur FJ, Francis IR, Quint
LE, Dunnick NR, Londy F. CT time-attenuation 13. Casola G, Nicolet V, vanSonnenberg E, et al.
washout curves of adrenal adenomas and Unsuspected pheochromocytoma: risk of
nonadenomas. AJR Am J Roentgenol. blood-pressure alterations during percutaneous
1998;170:747-752. adrenal biopsy. Radiology. 1986;159:733-755.

5. Szolar DH, Kammerhuber FH. Adrenal adeno- 14. Ferrozzi F, Bova D. CT and MR demonstration
mas and nonadenomas: assessment of washout of fat within an adrenal cortical carcinoma.
at delayed contrast-enhanced CT. Radiology. Abdom Imaging. 1995;20:272-274.
1998; 207:369-375.
15. Schlund JF, Kenney PJ, Brown ED, Ascher SM,
6. Caoili EM, Korobkin M, Francis IR, et al. Brown JJ, Semelka RC. Adrenocortical carci-
Adrenal masses: characterization with com- noma: MR imaging appearance with current
bined unenhanced and delayed enhanced CT. techniques. J Magn Reson Imaging. 1995;5:
Radiology. 2002;222:629-633. 171-174.

7. Mayo-Smith WW, Lee MJ, McNicholas MM, 16. Lumachi F, Tregnaghi A, Zucchetta P, et al.
Hahn PF, Boland GW, Saini S. Characterization Sensitivity and positive predictive value of CT,
of adrenal masses (< 5 cm) by use of chemical MRI and 123I-MIBG scintigraphy in localizing
shift MR imaging: observer performance pheochromocytomas: a prospective study.
versus quantitative measures. AJR Am J Nucl Med Commun. 2006;27:583-587.
Roentgenol. 1995;165:91-95.

130 EDUCATIONAL REVIEW MANUAL IN UROLOGY


17. Koizumi M, Endo K, Sakahara H, et al. Com- ment: evaluation with an anthropomorphic body CT
puted tomography and 131I-MIBG scintigra- phantom. Radiology. 2002;225:83-90.
phy in the diagnosis of pheochromocytoma.
Acta Radiol Diagn (Stockh). 1986;27:305-309. 3. Jayson M, Sanders H. Increased incidence of
serendipitously discovered renal cell carci-
18. Lee JE, Evans DB, Hickey RC, et al. Unknown noma. Urology. 1998;51:203-205.
primary cancer presenting as an adrenal mass:
frequency and implications for diagnostic eval- 4. Stanley RJ. Inherent dangers in radiologic
uation of adrenal incidentalomas. Surgery. screening. AJR Am J Roentgenol. 2001;177:
1998;124:1115-1122. 989-992.

19. Warshauer DM, McCarthy SM, Street L, et al. 5. Lee CT, Katz J, Fearn PA, Russo P. Mode of
Detection of renal masses: sensitivities and presentation of renal cell carcinoma provides
specificities of excretory urography/linear prognostic information. Urol Oncol.
tomography, US, and CT. Radiology. 2002;7:135-140.
1988;169:363-365.
6. Robson CJ, Churchill BM, Anderson W. The
20. Birnbaum BA, Jacobs JE, Ramchandani P. results of radical nephrectomy for renal cell car-
Multiphasic renal CT: comparison of renal cinoma. J Urol. 1969;101:297-301.
mass enhancement during the corticomedullary
and nephrographic phases. Radiology. 7. Tongaonkar HB, Dandekar NP, Dalal AV,
1996;200:753-758. Kulkarni JN, Kamat MR. Renal cell carcinoma
extending to the renal vein and inferior vena
21. Davidson AJ, Hartman DS, Choyke PL, cava: results of surgical treatment and prognos-
Wagner BJ. Radiologic assessment of renal tic factors. J Surg Oncol. 1995;59:94-100.
masses: implications for patient care.
Radiology. 1997;202:297-305. 8. Kallman DA, King BF, Hattery RR, et al. Renal
vein and inferior vena cava tumor thrombus in
22. Macari M, Bosniak MA. Delayed CT to evalu- renal cell carcinoma: CT, US, MRI and
ate renal masses incidentally discovered at venacavography. J Comput Assist Tomogr.
contrast-enhanced CT: demonstration of 1992;16:240-247.
vascularity with deenhancement. Radiology.
1999;213:674-680. 9. Studer UE, Scherz S, Scheidegger J, et al.
Enlargement of regional lymph nodes in renal
23. Ho VB, Allen SF, Hood MN, Choyke PL. Renal cell carcinoma is often not due to metastases.
masses: quantitative assessment of enhance- J Urol. 1990;144:243-245.
ment with dynamic MR imaging. Radiology.
2002;224:695-700. 10. Kim JK, Park SY, Shon JH, Cho KS. Angiomy-
olipoma with minimal fat: differentiation from
renal cell carcinoma at biphasic helical CT.
Radiology. 2004;230:677-684.
Renal Masses

1. Macari M, Bosniak MA. Delayed CT to


evaluate renal masses incidentally discovered 11. Päivänsalo M, Lähde S, Hyvärinen S,
at contrast-enhanced CT: demonstration of Kallioinen M, Jalovaara P. Renal angiomy-
vascularity with deenhancement. Radiology. olipoma. Ultrasonographic, CT, angiographic,
1999;213:674-680. and histologic correlation. Acta Radiol.
1991;32:239-243.
2. Birnbaum BA, Maki DD, Chakraborty DP,
Jacobs JE, Babb JS. Renal cyst pseudoenhance-

CHAPTER 5: ESSENTIALS OF URORADIOLOGY 131


12. Siegel CL, Middleton WD, Teefey SA, 22. Tada S, Yamagishi J, Kobayashi H, Hata Y,
McClennan BL. Angiomyolipoma and renal Kobari T. The incidence of simple renal cyst by
cell carcinoma: US differentiation. Radiology. computed tomography. Clin Radiol.
1996;198:789-793. 1983;34:437-439.

13. Outwater EK, Bhatia M, Siegelman ES, 23. Davidson AJ, Hartman DS, Choyke PL, Wag-
Burke MA, Mitchell DG. Lipid in renal clear ner BJ. Radiologic assessment of renal masses:
cell carcinoma: detection on opposed-phase implications for patient care. Radiology.
gradient-echo MR images. Radiology. 1997;202:297-305.
1997;205:103-107.
24. Curry NS, Bissada NK. Radiologic evaluation
14. Newhouse JH, Wagner BJ. Renal oncocytomas. of small and indeterminant renal masses. Urol
Abdom Imaging. 1998;23:249-255. Clin North Am. 1997;24:493-505.

15. Urban BA, Buckley J, Soyer P, Scherrer A, 25. Davidson AJ, Hartman DS, Choyke PL, Wag-
Fishman EK. CT appearance of transitional ner BJ. Radiologic assessment of renal masses:
cell carcinoma of the renal pelvis: Part 1. implications for patient care. Radiology. 1997;
Early-stage disease. AJR Am J Roentgenol. 202:297-305.
1997;169:157-161.
26. Rosenberg ER, Korobkin M, Foster W,
16. Urban BA, Buckley J, Soyer P, Scherrer A, Silverman PM, Bowie JD, Dunnick NR.
Fishman EK. CT appearance of transitional cell The significance of septations in a renal cyst.
carcinoma of the renal pelvis: Part 2. AJR Am J Roentgenol. 1985;144:593-595.
Advanced-stage disease. AJR Am J Roentgenol.
1997;169:163-168. 27. Wolf JS Jr. Evaluation and management of
solid and cystic renal masses. J Urol.
17. Uzzo RG, Cherullo EE, Myles J, Novick AC. 1998;159:1120-1133.
Renal cell carcinoma invading the urinary col-
lecting system: implications for staging. J Urol. 28. Curry NS, Cochran ST, Bissada NK. Cystic
2002;167:2392-2396. renal masses: accurate Bosniak classification
requires adequate renal CT. AJR Am J
18. Palapattu GS, Pantuck AJ, Dorey F, Said JW, Roentgenol. 2000; 175:339-342.
Figlin RA, Belldegrun AS. Collecting system
invasion in renal cell carcinoma: impact on 29. Davidson AJ, Hartman DS, Choyke PL, Wag-
prognosis and future staging strategies. J Urol. ner BJ. Radiologic assessment of renal masses:
2003;170:768-772. implications for patient care. Radiology. 1997;
202:297-305.
19. Urban BA, Fishman EK. Renal lymphoma:
CT patterns with emphasis on helical CT. 30. Rosenberg ER, Korobkin M, Foster W,
Radiographics. 2000;20:197-212. Silverman PM, Bowie JD, Dunnick NR.
The significance of septations in a renal cyst.
20. Choyke PL, White EM, Zeman RK, Jaffe MH, AJR Am J Roentgenol. 1985;144:593-595.
Clark LR. Renal metastases: clinicopathologic
and radiologic correlation. Radiology. 31. Onishi T, Oishi Y, Goto H, Tomita M, Abe K,
1987;162:359-363. Sugaya S. Cyst-associated renal cell carcinoma:
clinicopathologic characteristics and evaluation
21. Hélénon O, Merran S, Paraf F, et al. Unusual of prognosis in 27 cases. Int J Urol.
fat-containing tumors of the kidney: a diagnos- 2001;8:268-274.
tic dilemma. Radiographics. 1997;17:129-144.

132 EDUCATIONAL REVIEW MANUAL IN UROLOGY


32. Hartman DS, Weatherby E 3rd, Laskin WB, 3. Svedström E, Alanen A, Nurmi M. Radiologic
Brody JM, Corse W, Baluch JD. Cystic renal diagnosis of renal colic: the role of plain films,
cell carcinoma: CT findings simulating a excretory urography and sonography. Eur J
benign hyperdense cyst. AJR Am J Roentgenol. Radiol.1990;11:180-183.
1992;159:1235-1237.
4. Laing FC, Jeffrey RB Jr, Wing VW. Ultrasound
33. Koga S, Nishikido M, Inuzuka S, et al. versus excretory urography in evaluating acute
An evaluation of Bosniak's radiological classi- flank pain. Radiology. 1985;154:613-616.
fication of cystic renal masses. BJU Int.
2000;86:607-609. 5. Juul N, Brøns J, Torp-Pedersen S, Fredfeldt KE.
Ultrasound versus intravenous urography in the
34. Israel GM, Bosniak MA. Follow-up CT of initial evaluation of patients with suspected
moderately complex cystic lesions of the kid- obstructing urinary calculi. Scand J Urol
ney (Bosniak category IIF). AJR Am J Nephrol Suppl. 1991;137:45-47.
Roentgenol. 2003; 181:627-633.
6. Rodgers PM, Bates JA, Irving HC. Intrarenal
35. Bosniak MA, Birnbaum BA, Krinsky GA, Doppler ultrasound studies in normal and
Waisman J. Small renal parenchymal neo- acutely obstructed kidneys. Br J Radiol.
plasms: further observations on growth. 1992;65:207-212.
Radiology. 1995; 197:589-597.
7. Niall O, Russell J, MacGregor R, Duncan H,
36. Rendon RA, Stanietzky N, Panzarella T, Mullins J. A comparison of noncontrast
et al. The natural history of small renal masses. computerized tomography with excretory
J Urol. 2000;164:1143-1147. urography in the assessment of acute flank pain.
J Urol. 1999;161:534-537.
37. Siegel CL, McFarland EG, Brink JA, Fisher AJ,
Humphrey P, Heiken JP. CT of cystic renal 8. Smith RC, Rosenfield AT, Choe KA, et al.
masses: analysis of diagnostic performance and Acute flank pain: comparison of non-contrast-
interobserver variation. AJR Am J Roentgenol. enhanced CT and intravenous urography.
1997;169:813-818. Radiology. 1995;194:789-794.

38. Han KR, Janzen NK, McWhorter VC, et al. 9. Coll DM, Varanelli MJ, Smith RC. Relation-
Cystic renal cell carcinoma: biology and clini- ship of spontaneous passage of ureteral calculi
cal behavior. Urol Oncol. 2004;22:410-414. to stone size and location as revealed by unen-
hanced helical CT. AJR Am J Roentgenol.
Renal Calcification and Urinary Stones 2002;178:101-103.

1. Abramson S, Walders N, Applegate KE, Gilke- 10. Fielding JR, Silverman SG, Samuel S, Zou KH,
son RC, Robbin MR. Impact in the emergency Loughlin KR. Unenhanced helical CT of
department of unenhanced CT on diagnostic ureteral stones: a replacement for excretory
confidence and therapeutic efficacy in patients urography in planning treatment. AJR Am J
with suspected renal colic: a prospective sur- Roentgenol. 1998;171:1051-1053.
vey. 2000 ARRS President's Award. American
Roentgen Ray Society. AJR Am J Roentgenol. 11. Sourtzis S, Thibeau JF, Damry N, Raslan A,
2000;175:1689-1695. Vandendris M, Bellemans M. Radiologic
investigation of renal colic: unenhanced
2. Thornbury JR, Parker TW. Ureteral calculi. helical CT compared with excretory urography.
Semin Roentgenol. 1982;17:133-139. AJR Am J Roentgenol 1999;172:1491-1494.

CHAPTER 5: ESSENTIALS OF URORADIOLOGY 133


12. Smith RC, Verga M, McCarthy S, Rosenfield
AT. Diagnosis of acute flank pain: value of
Renal Infection and Infarction

unenhanced helical CT. AJR Am J Roentgenol. 1. Morehouse HT, Weiner SN, Hoffman JC.
1996; 166:97-101. Imaging in inflammatory disease of the kidney.
AJR Am J Roentgenol. 1984;143:135-141.
13. Reiter WJ, Schön-Pernerstorfer H, Dorfinger
K, Hofbauer J, Marberger M. Frequency of 2. Piccirillo M, Rigsby C, Rosenfield AT.
urolithiasis in individuals seropositive for Contemporary imaging of renal inflammatory
human immunodeficiency virus treated with disease. Infect Dis Clin North Am.
indinavir is higher than previously assumed. 1987;1:927-964.
J Urol. 1999;161:1082-1084.
3. Browne RF, Zwirewich C, Torreggiani WC.
14. Schwartz BF, Schenkman N, Armenakas NA, Imaging of urinary tract infection in the adult.
Stoller ML. Imaging characteristics of indinavir Eur Radiol. 2004;14:E168-E183.
calculi. J Urol. 1999;161:1085-1087.
4. Grayson DE, Abbott RM, Levy AD,
15. Bruce RG, Munch LC, Hoven AD, et al. Sherman PM. Emphysematous infections
Urolithiasis associated with the protease of the abdomen and pelvis: a pictorial review.
inhibitor indinavir. Urology. 1997;50:513-518. Radiographics. 2002;22:543-561.

16. Blake SP, McNicholas MM, Raptopoulos V. 5. Roy C, Pfleger DD, Tuchmann CM, Lang HH,
Nonopaque crystal deposition causing ureteric Saussine CC, Jacqmin D. Emphysematous
obstruction in patients with HIV undergoing pyelitis: findings in five patients. Radiology.
indinavir therapy. AJR Am J Roentgenol. 2001;218:647-650.
1998;171:717-720.
6. Joseph RC, Amendola MA, Artze ME, et al.
17. LeRoy AJ. Diagnosis and treatment of Genitourinary tract gas: imaging evaluation.
nephrolithiasis: current perspectives. Radiographics. 1996;16:295-308.
AJR Am J Roentgenol. 1994;163:1309-1313.
7. Wan YL, Lee TY, Bullard MJ, Tsai CC. Acute
18. Amis ES, Newhouse JH. Urinary stone disease. gas-producing bacterial renal infection: correla-
In Amises, Newhouse JH (eds): Essentials of tion between imaging findings and clinical out-
Uroradiology. Boston, MA: Little Brown; come. Radiology. 1996;198:433-438.
1991: 213-231.
8. Zorzos I, Moutzouris V, Korakianitis G, Katsou
19. Heneghan JP, Dalrymple NC, Verga M, G. Analysis of 39 cases of xanthogranuloma-
Rosenfield AT, Smith RC. Soft-tissue "rim" tous pyelonephritis with emphasis on CT find-
sign in the diagnosis of ureteral calculi with ings. Scand J Urol Nephrol. 2003;37:342-347.
use of unenhanced helical CT. Radiology.
1997;202:709-711. 9. Goldman SM, Hartman DS, Fishman EK,
Finizio JP, Gatewood OM, Siegelman SS. CT
20. Guest AR, Cohan RH, Korobkin M, et al. of xanthogranulomatous pyelonephritis: radio-
Assessment of the clinical utility of the rim logic-pathologic correlation. AJR Am J
and comet-tail signs in differentiating ureteral Roentgenol. 1984;142:963-969.
stones from phleboliths. AJR Am J Roentgenol.
2001;177:1285-1291. 10. Hayes WS, Hartman DS, Sesterbenn IA. From
the Archives of the AFIP. Xanthogranuloma-
tous pyelonephritis. Radiographics.
1991;11:485-498.

134 EDUCATIONAL REVIEW MANUAL IN UROLOGY


11. Erden A, Fitoz S, Karagülle T, Tükel S, Akyar
S. Radiological findings in the diagnosis of
Renal Trauma

genitourinary candidiasis. Pediatr Radiol. 1. Moore EE, Shackford SR, Pachter HL, et al.
2000;30:875-877. Organ injury scaling: spleen, liver, and kidney.
J Trauma. 1989;29:1664-1666.
12. Morris BS, Chudgar PD, Manejwala O. Pri-
mary renal candidiasis: fungal mycetomas in 2. Haas CA, Dinchman KH, Nasrallah PF, Spirnak
the kidney. Australas Radiol. 2002;46:57-59. JP. Traumatic renal artery occlusion: a 15-year
review. J Trauma. 1998;45:557-561.
13. Dembner AG, Pfister RC. Fungal infection of
the urinary tract: demonstration by antegrade 3. Knudson MM, Harrison PB, Hoyt DB, et al.
pyelography and drainage by percutaneous Outcome after major renovascular injuries: a
nephrostomy. AJR Am J Roentgenol. 1977;129: Western trauma association multicenter report.
415-418. J Trauma. 2000;49:1116-1122.

14. Kale H, Narlawar RS, Rathod K. Renal fungal 4. Bertini JE Jr, Flechner SM, Miller P, Ben-Men-
ball: an unusual sonographic finding. J Clin achem Y, Fischer RP. The natural history of
Ultrasound. 2002;30:178-180. traumatic branch renal artery injury.
J Urol. 1986;135:228-230.
15. Suzer O, Shirkhoda A, Jafri SZ, Madrazo BL,
Bis KG, Mastromatteo JF. CT features of renal 5. Boone TB, Gilling PJ, Husmann DA. Uretero-
infarction. Eur J Radiol. 2002;44:59-64. pelvic junction disruption following blunt
abdominal trauma. J Urol. 1993; 150:33-36.
16. Harris RD, Dorros S. Computed tomographic
diagnosis of renal infarction. Urology. 6. Bretan PN Jr, McAninch JW, Federle MP, Jef-
1981;17:287-289. frey RB Jr. Computerized tomographic staging
of renal trauma: 85 consecutive cases. J Urol.
17. Glazer GM, Francis IR, Brady TM, Teng SS. 1986;136:561-565.
Computed tomography of renal infarction:
clinical and experimental observations. 7. Carroll PR, McAninch JW, Klosterman P,
AJR Am J Roentgenol. 1983;140:721-727. Greenblatt M. Renovascular trauma: risk
assessment, surgical management, and out-
18. Harris RD, Dorros S, Scheible FW. Appearance come. J Trauma. 1990;30:547-552.
of renal infarction on computed tomography.
Comput Radiol. 1983;7:181-187. 8. Cass AS, Luxenberg M, Gleich P, Smith CS.
Clinical indications for radiographic evaluation
19. Kamel IR, Berkowitz JF. Assessment of the of blunt renal trauma. J Urol. 1986;136:370-
cortical rim sign in posttraumatic renal 371.
infarction. J Comput Assist Tomogr.
1996;20:803-806. 9. Corr P, Hacking G. Embolization in traumatic
intrarenal vascular injuries. Clin Radiol.
20. Wong WS, Moss AA, Federle MP, Cochran ST, 1991;43:262-264.
London SS. Renal infarction: CT diagnosis and
correlation between CT findings and etiologies. 10. Dinkel HP, Danuser H, Triller J. Blunt renal
Radiology. 1984;150:201-205. trauma: minimally invasive management with
microcatheter embolization experience in nine
21. Yücel C, Ozdemir H, Akpek S, Gürel K, patients. Radiology. 2002;223:723-730.
Kapucu LO, Arac M. Renal infarct: contrast-
enhanced power Doppler sonographic findings.
J Clin Ultrasound. 2001;29:237-242.

CHAPTER 5: ESSENTIALS OF URORADIOLOGY 135


11. Federle MP, Kaiser JA, McAninch JW, Jeffrey 21. McGahan JP, Richards JR, Jones CD, Ger-
RB, Mall JC. The role of computed tomography scovich EO. Use of ultrasonography in the
in renal trauma. Radiology. 1981;141:455-460. patient with acute renal trauma. J Ultrasound
Med. 1999;18:207-213; quiz 215-216.
12. Hagiwara A, Sakaki S, Goto H, et al. The role
of interventional radiology in the management 22. Morey AF, McAninch JW, Tiller BK, Duckett
of blunt renal injury: a practical protocol. CP, Carroll PR. Single shot intraoperative
J Trauma. 2001;51:526-531. excretory urography for the immediate evalua-
tion of renal trauma. J Urol. 1999; 161: 1088-
13. Harris AC, Zwirewich CV, Lyburn ID, 92.
Torreggiani WC, Marchinkow LO. Ct
findings in blunt renal trauma. Radiographics. 23. Moudouni SM, Patard JJ, Manunta A, Guiraud
2001;21:S201-S214. P, Guille F, Lobel B. A conservative approach to
major blunt renal lacerations with urinary
14. Jeffrey RB Jr, Cardoza JD, Olcott EW. Detec- extravasation and devitalized renal segments.
tion of active intraabdominal arterial hemor- BJU Int. 2001; 87: 290-4.
rhage: value of dynamic contrast-enhanced CT.
AJR Am J Roentgenol. 1991;156:725-729. 24. Nicolaisen GS, McAninch JW, Marshall GA,
Bluth RF Jr, Carroll PR. Renal trauma: re-eval-
15. Kamel IR, Berkowitz JF. Assessment of uation of the indications for radiographic
the cortical rim sign in posttraumatic renal assessment. J Urol. 1985; 133: 183-7.
infarction. J Comput Assist Tomogr.
1996;20:803-806. 25. Santucci RA, McAninch JM. Grade IV renal
injuries: evaluation, treatment, and outcome.
16. Kawashima A, Sandler CM, Corriere JN Jr, World J Surg. 2001; 25: 1565-72.
Rodgers BM, Goldman SM. Ureteropelvic
junction injuries secondary to blunt abdominal
trauma. Radiology. 1997;205:487-492.
Ureteral Disease

1. Lawler LP, Jarret TW, Corl FM, Fishman EK.


17. Kenney PJ, Panicek DM, Witanowski LS. Adult ureteropelvic junction obstruction:
Computed tomography of ureteral disruption. insights with three-dimensional multi-detector
J Comput Assist Tomogr. 1987;11:480-484. row CT. Radiographics. 2005;25:121-134.

18. Lane MJ, Katz DS, Shah RA, Rubin GD, Jef- 2. Khaira HS, Platt JF, Cohan RH, Wolf JS, Faer-
frey RB Jr. Active arterial contrast extravasa- ber GJ. Helical computed tomography for iden-
tion on helical CT of the abdomen, pelvis, tification of crossing vessels in ureteropelvic
and chest. AJR Am J Roentgenol. 1998; junction obstruction-comparison with operative
171:679-685. findings. Urology. 2003;62:35-39.

19. McAndrew JD, Corriere JN Jr. Radiographic 3. Mitsumori A, Yasui K, Akaki S, et al. Evalua-
evaluation of renal trauma: evaluation of tion of crossing vessels in patients with uretero-
1103 consecutive patients. Br J Urol. pelvic junction obstruction by means of helical
1994;73:352-354. CT. Radiographics. 2000;20:1383-1393.

20. McAninch JW, Federle MP. Evaluation of 4. Dyer RB, Chen MY, Zagoria RJ. Classic signs
renal injuries with computerized tomography. in uroradiology. Radiographics. 2004;24:S247-
J Urol. 1982;128:456-460. S280.

136 EDUCATIONAL REVIEW MANUAL IN UROLOGY


5. Fritz GA, Schoellnast H, Deutschmann HA, 4. Amendola MA, Glazer GM, Grossman HB,
Quehenberger F, Tillich M. Multiphasic multi- Aisen AM, Francis IR. Staging of bladder car-
detector-row CT (MDCT) in detection and stag- cinoma: MRI-CT-surgical correlation. AJR Am
ing of transitional cell carcinomas of the upper J Roentgenol. 1986;146:1179-1183.
urinary tract. Eur Radiol. 2006;16:1244-1252.
5. Fisher MR, Hricak H, Tanagho EA. Urinary
6. Browne RF, Meehan CP, Colville J, Power R, bladder MR imaging. Part II. Neoplasm.
Torreggiani WC. Transitional cell carcinoma of Radiology. 1985;157:471-477.
the upper urinary tract: spectrum of imaging
findings. Radiographics. 2005;25:1609-1627. Prostatic Disease

7. Baron RL, McClennan BL, Lee JK, Lawson 1. Coakley FV, Hricak H. Radiologic anatomy of
TL. Computed tomography of transitional-cell the prostate gland: a clinical approach. Radiol
carcinoma of the renal pelvis and ureter. Clin North Am. 2000;38:15-30.
Radiology. 1982;144:125-130.
2. Scheckowitz EM, Resnick MI. Imaging of the
8. Harvin HJ. Ureteral fibroepithelial polyp on prostate. Benign prostatic hyperplasia. Urol
MDCT urography. AJR Am J Roentgenol. 2006; Clin North Am. 1995;22:321-332.
187:W434-W435.
3. Grossfeld GD, Coakley FV. Benign prostatic
9. Bellin MF, Springer O, Mourey-Gerosa I, et al. hyperplasia: clinical overview and value of
CT diagnosis of ureteral fibroepithelial polyps. diagnostic imaging. Radiol Clin North Am.
Eur Radiol. 2002;12:125-128. 2000;38:31-47.

10. Stukart RA, van Haga JJ, Bruijnes E. 4. Boczko J, Messing E, Dogra V. Transrectal
Fibroepithelial ureteral polyps. Urol Int. sonography in prostate evaluation. Radiol Clin
1984;39:336-338. North Am. 2006;44:679-687.

5. Littrup PJ, Bailey SE. Prostate cancer: the role


of transrectal ultrasound and its impact on can-
Bladder Disease

1. Caskurlu T, Tasci AL, Sevin G, Cek M, Car- cer detection and management. Radiol Clin
bone A, Gezeroglu H. The role of trans-rectal North Am. 2000;38:87-113.
echography (TRE) in the evaluation and staging
of bladder tumors: comparison with suprapubic 6. McClennan BL. Diagnostic imaging evaluation
echography and computerized axial tomogra- of benign prostatic hyperplasia. Urol Clin
phy. Arch Ital Urol Androl. 1998;70:1-6. North Am. 1990;17:517-536.

2. Ravi R, Rao RC, Ahlawat R, Berry M. Carci- 7. Rajesh A, Coakley FV. MR imaging and MR
noma bladder: comparative evaluation of uri- spectroscopic imaging of prostate cancer. Magn
nary cytology, excretory urography and ultra- Reson Imaging Clin N Am. 2004;12:557-79.
sonography. Indian J Cancer. 1990;27:55-61.
8. Coakley FV, Qayyum A, Kurhanewicz J.
3. Voges GE, Tauschke E, Stöckle M, Alken P, Magnetic resonance imaging and spectroscopic
Hohenfellner R. Computerized tomography: an imaging of prostate cancer. J Urol.
unreliable method for accurate staging of blad- 2003;170:S69-S75.
der tumors in patients who are candidates for
radical cystectomy. J Urol. 1989;142:972-974.

CHAPTER 5: ESSENTIALS OF URORADIOLOGY 137


10. McAninch JW, Laing FC, Jeffrey RB Jr.
Sonourethrography in the evaluation of urethral
Urethral Disease

1. Sung DJ, Kim YH, Cho SB, et al. Obliterative strictures: a preliminary report. J Urol.
urethral stricture: MR urethrography versus 1988;139: 294-297.
conventional retrograde urethrography with
voiding cystourethrography. Radiology. 11. Golomb J, Leibovitch I, Mor Y, Morag B,
2006;240:842-848. Ramon J. Comparison of voiding cys-
tourethrography and double-balloon urethrog-
2. Kawashima A, Sandler CM, Wasserman NF, raphy in the diagnosis of complex female ure-
LeRoy AJ, King BF Jr, Goldman SM. thral diverticula. Eur Radiol. 2003;13:536-542.
Imaging of urethral disease: a pictorial review.
Radiographics. 2004;24:S195-S216. 12. Heidenreich A, Derschum W, Bonfig R,
Wilbert DM. Ultrasound in the evaluation of
3. Gallentine ML, Morey AF. Imaging of the male urethral stricture disease: a prospective study in
urethra for stricture disease. Urol Clin North 175 patients. Br J Urol. 1994;74:93-98.
Am. 2002;29:361-372.
13. Gupta N, Dubey D, Mandhani A, Srivastava A,
4. Kim B, Hricak H, Tanagho EA. Diagnosis of Kapoor R, Kumar A. Urethral stricture assess-
urethral diverticula in women: value of MR ment: a prospective study evaluating urethral
imaging. AJR Am J Roentgenol. 1993;161:809- ultrasonography and conventional radiological
815. studies. BJU Int. 2006;98:149-153.

5. Neitlich JD, Foster HE Jr, Glickman MG, Smith 14. Narumi Y, Hricak H, Armenakas NA, Dixon
RC. Detection of urethral diverticula in women: CM, McAninch JW. MR imaging of traumatic
comparison of a high resolution fast spin echo posterior urethral injury. Radiology.
technique with double balloon urethrography. 1993;188:439-443.
J Urol. 1998;159:408-410.
15. Dixon CM, Hricak H, McAninch JW. Magnetic
6. Hricak H, Secaf E, Buckley DW, Brown JJ, resonance imaging of traumatic posterior ure-
Tanagho EA, McAninch JW. Female urethra: thral defects and pelvic crush injuries. J Urol.
MR imaging. Radiology. 1991;178:527-535. 1992;148:1162-1165.

7. Jacoby K, Rowbotham RK. Double balloon


positive pressure urethrography is a more
Testicular Masses

sensitive test than voiding cystourethrography 1. Silverberg E. Cancer in young adults


for diagnosing urethral diverticulum in women. (ages 15 to 34). CA Cancer J Clin. 1982;32:
J Urol. 1999;162:2066-2069. 32-42.

8. Ryu J, Kim B. MR imaging of the male and 2. Bree RL, Hoang DT. Scrotal ultrasound.
female urethra. Radiographics. 2001;21:1169- Radiol Clin North Am. 1996;34:1183-1205.
1185.
3. Hricak H, Hamm B, Kim BH, eds. Imaging of
9. Choudhary S, Singh P, Sundar E, Kumar S, the Scrotum: Textbook and Atlas. New York,
Sahai A. A comparison of sonourethrography NY: Raven Press; 1995: 49-93.
and retrograde urethrography in evaluation of
anterior urethral strictures. Clin Radiol. 4. Yalowitz BR, Eble JN, Wilks DC. Spermato-
2004;59:736-742. zoa-containing simple cysts of the rete testis.
J Urol. 1989;142:1572-1573.

138 EDUCATIONAL REVIEW MANUAL IN UROLOGY


5. Brown DL, Benson CB, Doherty FJ, et al. Cys- 16. Kressel K, Schnell D, Thon WF, Heymer B,
tic testicular mass caused by dilated rete testis: Hartmann M, Altwein JE. Benign testicular
sonographic findings in 31 cases. AJR Am J tumors: a case for testis preservation? Eur Urol.
Roentgenol. 1992; 158:1257-1259. 1988;15:200-204.

6. Tartar VM, Trambert MA, Balsara ZN, Mattrey 17. Burt AD, Cooper G, MacKay C, Boyd JF. Der-
RF. Tubular ectasia of the testicle: sonographic moid cyst of the testis. Scott Med J. 1987;32:
and MR imaging appearance. AJR Am J 146-148.
Roentgenol. 1993;160:539-542.
18. Avery GR, Peakman DJ, Young JR. Unusual
7. Thomas RD, Dewbury KC. Ultrasound hyperechoic ultrasound appearance of
appearance of the rete testis. Clin Radiol. testicular Leydig cell tumour. Clin Radiol.
1993;47:121-124. 1991;43:260-261.

8. Price EB Jr. Epidermoid cysts of the testis: a 19. Brosman SA. Testicular tumors in prepubertal
clinical and pathologic analysis of 69 cases children. Urology. 1979;13:581-588.
from the testicular tumor registry. J Urol.
1969;102:708-713. 20. Liu P, Thorner P. Sonographic appearance of
Sertoli cell tumour: with pathologic correlation.
9. Sanderson AJ, Birch BR, Dewbury KC. Case Pediatr Radiol. 1993;23:127-128.
report: multiple epidermoid cysts of the testes -
the ultrasound appearances. Clin Radiol. 21. Noszian IM, Balon R, Eitelberger FG, Schmid
1995;50:414-415. N. Bilateral testicular large-cell calcifying Ser-
toli cell tumor and recurrent cardiac myxoma in
10. Maxwell AJ, Mamtora H. Sonographic appear- a patient with Carney’s complex. Pediatr
ance of epidermoid cyst of the testis. J Clin Radiol. 1995;25:S236-S237.
Ultrasound. 1990;18:188-190.
22. Mackay B, Bennington JL, Skoglund RW. The
11. Malvica RP. Epidermoid cyst of the testicle: adenomatoid tumor: fine structural evidence for
an unusual sonographic finding. AJR Am J a mesothelial origin. Cancer. 1971;27:109-115.
Roentgenol. 1993;160:1047-1048.
23. Feuer A, Dewire DM, Foley WD. Ultrasono-
12. Meiches MD, Nurenberg P. Sonographic graphic characteristics of testicular adenoma-
appearance of a calcified simple epidermoid toid tumors. J Urol. 1996;155:174-175.
cyst of the testis. J Clin Ultrasound.
1991;19:498-500. 24. Horstman WG, Sands JP, Hooper DG. Adeno-
matoid tumor of the testicle. Urology. 1992;40:
13. Bahnson RR, Slasky BS, Ernstoff MD, Banner 359-361.
BF. Sonographic characteristics of epidermoid
cyst of the testicle. Urology. 1990;35:508-510. 25. Wheeler JE. Testicular tumors. In: Hill GS,
ed. Uropathology. New York, NY: Churchill
14. Carr L, Feigin G, Dikman S. Epidermoid cyst of Livingstone; 1989:1085-1088.
the testicle: ultrasonographic characteristics.
Urology. 1984;24:79-81. 26. Ritzén EM. Testicular relapse of acute lym-
phoblastic leukemia (ALL). J Reprod Immunol.
15. Heidenreich A, Engelmann UH, Vietsch HV, 1990;18:117-121.
Derschum W. Organ preserving surgery in tes-
ticular epidermoid cysts. J Urol.
1995;153:1147-1150.

CHAPTER 5: ESSENTIALS OF URORADIOLOGY 139


27. Musmanno MC, White JM. Scrotal ultrasonog- 6. Riccabona M. Imaging of renal tumours in
raphy as adjunct to testis biopsy in leukemia. infancy and childhood. Eur Radiol.
Urology. 1990;35:239-241. 2003;13:L116-L129.

28. Mazzu D, Jeffrey RB Jr, Ralls PW. Lymphoma 7. Goske MJ, Mitchell C, Reslan WA. Imaging of
and leukemia involving the testicles: findings patients with Wilms' tumor. Semin Urol Oncol.
on gray-scale and color Doppler sonography. 1999;17:11-20.
AJR Am J Roentgenol. 1995;164:645-647.
8. Bellin MF, Maidenberg M, Raveau V, et al. MR
29. Horstman WG, Melson GL, Middleton WD, imaging of adult Wilms’ tumor: correlation
Andriole GL. Testicular tumors: findings with with US, CT, and pathology. Urol Radiol.
color Doppler US. Radiology. 1992;185:733- 1990;12:148-150.
737.
9. Montgomery P, Kuhn JP, Berger PE, Fisher J.
30. Hanash KA, Carney JA, Kelalis PP. Metastatic Multifocal nephroblastomatosis: clinical
tumors to testicles: routes of metastasis. significance and imaging. Pediatr Radiol.
J Urol. 1969;102:465-468. 1984;14:392-395.

31. Tiltman AJ. Metastatic tumours in the testis. 10. Fernbach SK, Feinstein KA, Donaldson JS,
Histopathology. 1979;3:31-37. Baum ES. Nephroblastomatosis: comparison of
CT with US and urography. Radiology.
32. Grignon DJ, Shum DT, Hayman WP. 1988;166:153-156.
Metastatic tumours of the testes. Can J Surg.
1986;29:359-361. 11. Birmole BJ, Borwankar SS, Vaidya AS,
Kulkarni BK. Crossed renal ectopia. J Postgrad
Pediatric Uroradiology Med. 1993;39:149-151.

1. Kirks DR, Merten DF, Grossman H, Bowie JD. 12. McCarthy S, Rosenfield AT. Ultrasonography
Diagnostic imaging of pediatric abdominal in crossed renal ectopia. J Ultrasound Med.
masses: an overview. Radiol Clin North Am. 1984;3:107-112.
1981;19:527-45.
13. Strauss S, Dushnitsky T, Peer A, Manor H,
2. Alexander F. Neuroblastoma. Urol Clin North Libson E, Lebensart PD. Sonographic features
Am. 2000;27:383-92. of horseshoe kidney: review of 34 patients.
J Ultrasound Med. 2000;19:27-31.
3. Lonergan GJ, Schwab CM, Suarez ES, Carlson
CL. Neuroblastoma, ganglioneuroblastoma, 14. Siegfried MS, Rochester D. Computed tomog-
and ganglioneuroma: radiologic-pathologic raphy appearance of fused (horseshoe) kidney.
correlation. Radiographics. 2002;22:911-34. J Comput Tomogr. 1983;7:301-304.

4. Mehta K, Haller JO, Legasto AC. Imaging neu- 15. Fazio L, Razvi H, Chin JL. Malignancy in
roblastoma in children. Crit Rev Comput horseshoe kidneys: review and discussion of
Tomogr. 2003;44:47-61. surgical implications. Can J Urol.
2003;10:1899-1904.
5. Hugosson C, Nyman R, Jorulf H, et al. Imaging
of abdominal neuroblastoma in children. Acta 16. Dacher JN, Hitzel A, Avni FE, Vera P. Imaging
Radiol. 1999;40:534-542. strategies in pediatric urinary tract infection.
Eur Radiol. 2005;15:1283-1288.

140 EDUCATIONAL REVIEW MANUAL IN UROLOGY


17. Riccabona M, Fotter R. Urinary tract infection 5. Subramanyam BR, Raghavendra BN, Bosniak
in infants and children: an update with special MA, Lefleur RS, Rosen RJ, Horii SC. Sonogra-
regard to the changing role of reflux. Eur phy of pyonephrosis: a prospective study. AJR
Radiol. 2004;14:L78-L88. Am J Roentgenol. 1983; 140: 991–993.

18. Agrawalla S, Pearce R, Goodman TR. How to


perform the perfect voiding cystourethrogram.
Pediatr Radiol. 2004;34:114-119.

19. Jequier S, Jequier JC. Reliability of voiding


cystourethrography to detect reflux. AJR Am J
Roentgenol. 1989;153:807-810.

20. Kroovand RL. Ureterocele. Urol Clin North


Am. 1983;10:445-449.

21. Nussbaum AR, Dorst JP, Jeffs RD, Gearhart JP,


Sanders RC. Ectopic ureter and ureterocele:
their varied sonographic manifestations.
Radiology. 1986;159:227-235.

22. Griffin J, Jennings C, MacErlean D. Ultrasonic


evaluation of simple and ectopic ureteroceles.
Clin Radiol. 1983;34:55-57.

23. Macpherson RI, Leithiser RE, Gordon L,


Turner WR. Posterior urethral valves: an update
and review. Radiographics. 1986;6:753-791.

Renal infection

1. Akbar SA, Jafri S Z.H, Amendola M, Wister B.


Renal infections: An update. Applied
Radiology. 2009: 38:3.

2. Craig WD, Wagner BJ, Travis MD.


Pyelonephritis: radiologic-pathologic review.
January 2008. RadioGraphics. January
2008;28:255-277.

3. Kawashima A, Sandler CM, Goldman SM.


Imaging in acute renal infection. BJU Int. 2000;
86(suppl 1):70–79.

4. Chen MT, Huang CN, Chou YH, Huang CH,


Chiang CP, Liu GC. Percutaneous drainage in
the treatment of emphysematous pyelonephri-
tis: 10-year experience. J Urol. 1997;
157:1569–1573.

CHAPTER 5: ESSENTIALS OF URORADIOLOGY 141


14. Questions

Case 1 Case 2

66-year-old male with increasing abdominal disten- A 38-year-old s/p complicated rt inguinal hernia
sion. The best diagnosis is: repair now with small painless rt testis. The ultra-
sound findings show?
A. Renal angiomyolipoma
A. Normal flow in the right testicle
B. Pancreatic pseudocyst
B. Increased flow in the left testicle
C. Retroperitoneal liposarcoma
C. No flow in the right testicle
D. Lymphoma
D. Normal flow in the left testicle

Rt Testicle

Lt Testicle

142 EDUCATIONAL REVIEW MANUAL IN UROLOGY


Case 3 Case 4

A 60-year-old male with palpable abnormality in the A 42-year-old male with 11 month history of back
right testis. Regarding the finding in the rt testis, pain. Differential considerations include all of the
which of the following is false? following except?

A. Often associated with spermatoceles A. Lymphoma

B. Frequently bilateral B. Primary germ cell tumor

C. Annual surveillance is suggested C. Metastatic seminoma

D. Common in elderly men D. Retroperitoneal fibrosis

CHAPTER 5: ESSENTIALS OF URORADIOLOGY 143


Case 5 Case 6

An 80-year-old man with pelvic pain. Bilateral A 75-year-old male had cardiac cath 48 hrs earlier,
ureteral stents in place(arrows). Based on the imag- no other interim studies were obtained. Based on the
ing findings which statement is false? imaging findings all of the following are true
except?
A. Requires immediate surgical management .
A. Incidence in the general population is
B. Medical management is appropriate less than 2%

C. Diabetic patients at risk for development B. NSAIDS use is associated with increased risk

D. Can be seen after bladder instrumentation C. High volume and repetitive contrast
administration over short time period is
a risk factor

D. Risk is same for both arterial and intravenous


injection of contrast

Soft Tissue Window

Lung Window

144 EDUCATIONAL REVIEW MANUAL IN UROLOGY


Case 7 Case 8

Based on the imaging findings in the left kidney, CT scans done eight months apart on a 52-year-old
which statement is false? female surveillance CT. History of stage III
NSCLA. Which statement is true?
A. Abnormality can be seen in patient with
Crohn’s disease and prior small A. FNA can not reliably distinguish a primary
bowel resection renal cell carcinoma from metastatic disease

B. Blood clot is in the differential B. Breast and lung carcinomas metastases


with equal frequency to the kidney
C. High association with analgesic abuse
C. Oligometastasis to the kidneys are rare
D. This is the most common site for this
entity in the ureter D. A renal mass detected on FDG PET is more
likely to represent metastatic disease than
primary renal cell carcinoma.

CHAPTER 5: ESSENTIALS OF URORADIOLOGY 145


Case 9 Case 10

A 75-year-old man with left back pain. Based on the A 70-year-old 1 week s/p left partial nephrectomy
bone scan findings, which statement is false? now with anemia and hematuria. Given the imaging
findings which statement is false?
A. An obstructing calculus is a consideration
A. Is not associated with blunt trauma
B. Transitional cell carcinoma is a consideration
B. Spontaneous resolution can occur
C. Renal cell carcinoma is a consideration
C. Hematuria can be a delayed finding
D. Renal ultrasound would be the next study
of choice if there is a history of severe D. Angiographic assisted embolization is
contrast allergy the treatment of choice

146 EDUCATIONAL REVIEW MANUAL IN UROLOGY


6. D.
Classic example of contrast-induced nephropathy.
Answers

1. C. Persistent excretion of contrast 72 hrs after a cardiac


The demonstration of fat as indicated by the - 77 catheterization. Contrast media is more nephrotoxic
Hounsfield units excludes pancreatic pseudocyst when instilled intra-arterial probably because of a
and lymphoma. A Pseudocyst would have fluid higher arterial renal concentration. Contrast
attenuation HU between 1-20HU. Lymphoma nephropathy is uncommon in the general population
would measure as soft tissue greater than 40 HU. (1) is true. NSAIDS and other drugs which inhibit
Renal angiomyolipoma can have fat components vasodilatation increase the risk of renal failure from
but in this case the mass is displacing the kidney not contrast administration. High volume administra-
arising from it, thus excluding this as a diagnostic tion over a short period of time is a risk factor for
possibility. Therefore a retroperitoneal liposarcoma development of contrast nephropathy.
is the correct diagnosis.
7. D.
2. C. Classic example of an upper tract TCC. The lower
Pt had infarction of the rt testis following inguinal tract is the most common site for TCC (70%), with
hernia repair. The right testis does not show any the remainder equally divided between the upper
intratesticular flow. There is peripheral flow but no and middle third of the ureter. Patients with Crohn’s
flow is demonstrated centrally. The flow in the left disease can develop urate stones which are not
testicle is normal. radiodense on CT and could account for the filling
defect seen here. Matrix stones can have similar fea-
3.C. tures. Blood clots and fungus balls should also be
Dilated rete testis is a benign condition frequently considered in the differential of this finding. Anal-
seen in elderly men and usually in association with gesic abusers have an 8-fold increase for develop-
epididymal abnormalities. No further work-up is ment of transitional cell carcinoma.
required.
8. D.
4. D. FDG PET is poor for the detection of primary renal
The larger retroperitoneal soft tissue mass displaces cell cancers but can be superior to CT in the detec-
and encases the aorta. Scrotal ultrasound in this tion of often radiographically occult metastases to
patient showed a left testicular seminoma which had the kidney. FNA has been shown to be diagnostic in
metastasized to the retroperitoneal. Lymphoma and differentiating primary from metastatic disease to
primary germ cell tumors are appropriate considera- the kidney. Lung carcinoma followed by lymphoma
tions in this patient. Retroperitoneal fibrosis is usu- are the most frequent metastases to the kidneys and
ally seen in association with an atherosclerotic are rarely isolated.
aorta. The aorta and often the ureters are adherent to
the retroperitoneal and are not displaced. 9. D.
The left kidney is partially obstructed, secondary to
5.A. a large centrally located left renal cell carcinoma
The images show emphysematous cystitis. The causing partial obstruction of the left upper pole. On
presence of gas is confirmed on the lung windows. a contrast enhanced study the left nephrogram
E Cystitis is an uncommon bladder infection that is would be significantly delayed. A similar appear-
associated with DM, urinary obstruction and ance could be seen with an obstructing calculus or a
chronic infections. Medical management with proximal urothelial neoplasm. The cross-sectional
antibiotics and bladder decompression represents imaging study of choice would be a non-contrast CT
the appropriate management. Gas can occasionally or MR. Renal ultrasound is not accurate for the
be seen in the wall following bladder instrumenta- detection of urethral lesions. Both non-contrast CT
tion and requires no further follow-up. Emphysema- and MR would be able to detect an obstructing cal-
tous cystitis is not a surgical emergency and answer culus and in most instances distinguish a renal cell
# 1 is the false statement. carcinoma from a urothelial neoplasm.

CHAPTER 5: ESSENTIALS OF URORADIOLOGY 147


10. A.
The imaging findings show a large left renal artery
pseudoaneurysm which was treated with coil
embolization and is the treatment of choice. Pseu-
doaneurysms usually develop from parenchymal
penetrating injuries associated with renal biopsies
and knife wounds. While less common, pseudoa-
neurysms can also develop from blunt trauma asso-
ciated with deceleration injuries. Treatment should
be embolization but there are reports of spontaneous
resolution. In the absence of significant hematuria,
conservative management with close imaging
follow-up isn't inappropriate.

148 EDUCATIONAL REVIEW MANUAL IN UROLOGY


15. Figures

Figure 1A Figure 1B

Non-contrast CT image Arteriographic (early arterial phase) CT


image

These images are used for detection of urinary stones In this phase, contrast is predominantly within the
and other calcifications, and also serve as a baseline arterial system, as seen in the well-opacified right
for assessment of enhancement when compared to renal artery (arrow) in this patient.
post-contrast images.

Figure 1C Figure 1D

Corticomedullary or late arterial phase Nephrographic phase CT image


CT image

In this phase, the cortex shows marked enhance- In this phase, the cortex and medulla are isodense.
ment, while the medulla remains relatively unen- This is the standard phase for the assessment for the
hanced. Arterial opacification remains prominent. abdominal viscera.

CHAPTER 5: ESSENTIALS OF URORADIOLOGY 149


Figure 1E Figure 2A

Excretory phase CT image Corticomedullary phase image

These images are obtained several minutes after the


injection of intravenous contrast, and excreted con-
trast is seen within the collecting systems, as in the left
renal pelvis (arrow) of this patient. Shows a small renal cell carcinoma (arrow) with a
hypervascular rim.

Figure 2B Figure 3A

Nephrographic phase CT image in the same Axial CT image in the corticomedullary


patient phase

The carcinoma is less appreciable, being visible Shows a barely appreciable hypodense lesion
simply as a contour deformity (arrow). While it is (arrow).
uncommon, a wholly or partially hypervascular renal
cell carcinoma is occasionally better seen during
the corticomedullary than the nephrographic phase,
as in this case.

150 EDUCATIONAL REVIEW MANUAL IN UROLOGY


Figure 3B Figure 4A

3B Axial CT image in the nephrographic Schematic illustration demonstrating the


phase traditional approach to a renal protocol CT

Non-contrast images are acquired followed by


administration of intravenous contrast bolus. Subse-
quently, corticomedullary, nephrographic, and excre-
Shows the small hypodense lesion to be more appre-
tory phase images are sequentially acquired.
ciable. In general, renal lesions are better detected
on the nephrographic phase than the corti-
comedullary phase as in this case.

Figure 4B Figure 5

Schematic illustration demonstration of the Axial contrast-enhanced CT image demon-


split bolus protocol strating the normal vascular relationships
of the adrenal glands

As in the standard protocol, non-contrast images are


acquired. Then a small component of the contrast
bolus is administered. No images are acquired for the Such images are useful in localizing the adrenal
subsequent 4-5 minutes, when the remaining portion glands in difficult cases or when the anatomy is dis-
of the bolus is administered. Corticomedullary and torted. Both adrenal glands are seen as V- or Y-
nephrographic phase images are acquired after the shaped structures, the right adrenal gland lying pos-
major part of the bolus. These images also include an terior to the inferior vena cava (horizontal arrow) and
excretory phase, due to excretion of the initially the left adrenal gland seen posterior to the splenic
administered smaller part of the contrast bolus. The vessels (vertical arrows).
use of such a split bolus protocol is essentially a trick
that reduces the number of images and radiation
dose.

CHAPTER 5: ESSENTIALS OF URORADIOLOGY 151


Figure 6a Figure 6b

Axial non-contrast CT image of a left Axial contrast-enhanced CT image in the


adrenal adenoma (arrow) nephrographic phase


+8 
+8

The low density (9 Hounsfield units) of the lesion con- Shows the adenoma (arrow) enhanced, rising to a
firms that it is an adenoma. density of 76 Hounsfield units. In this phase of
enhancement, the CT density measurements of ade-
nomas overlap those of nonadenomas, and do not
help in the distinction of these two pathologies.

Figure 6c Figure 7A

15-minute delayed post-contrast-enhanced Non-contrast axial CT image


image


+8

+8

Shows the adenoma (arrows) has decreased in den- Shows a left adrenal nodule (arrow) that is not of low
sity to 46 Hounsfield units. Such a decrease in density enough density to be classified as an adenoma.
is typical of adenomas, and other adrenal masses
such as metastases to the adrenal glands will typi-
cally not de-enhance to this extent. A density of under
52 Hounsfield units at 15 minutes is suggestive of an
adenoma, as in this case (46 Hounsfield units).

152 EDUCATIONAL REVIEW MANUAL IN UROLOGY


Figure 7B Figure 7C

Axial post-contrast-enhanced CT image 15-minute delayed post-contrast image


+8 
+8

Shows the nodule (arrow) enhancing to a density of Shows the nodule has decreased to a density of 42
86 Hounsfield units. Hounsfield units. Such a decrease in density is
indicative of an adenoma, despite the fact that the
lesion was not of sufficiently low density on non-con-
trast images. The washout phenomenon can be used
to characterize adenomas that cannot be so charac-
terized on non-contrast images (presumably
because they are lipid poor).

Figure 8A Figure 8B

Coronal in-phase gradient echo T1- Coronal opposed-phase gradient echo T1-
weighted MR image of a right adrenal mass weighted image
(arrow)

Demonstrates marked signal loss in the mass (arrow),


consistent with an adenoma. Such signal loss is char-
acteristic of tissue that contains a substantial amount
of microscopic fat, as in adrenal adenomas that are
lipid-rich due to their derivation from the adrenal cor-
tex. The characteristic India ink or etching artifact
seen at the boundary of the viscera on opposed-
phase imaging is distinctive of this sequence, and
due to signal loss in boundary voxels that are mix-
tures of fat (in the retroperitoneum) and water (in the
visceral organ).

HAPTER 5: ESSENTIALS OF URORADIOLOGY 153


Figure 9 Figure 10

Axial contrast-enhanced CT image in a Figure 10 Axial contrast-enhanced CT


patient with Cushing’s syndrome image in a patient with Conn’s syndrome

Shows a small unifocal right adrenal nodule (arrow). In a patient with Conn’s syndrome, even a very small
In the setting of Cushing’s syndrome, such a nodule is unifocal adrenal nodule (arrow) is likely to be a func-
more likely a focus of hyperplasia induced by exces- tional adenoma producing excessive aldosterone
sive ACTH from the pituitary gland. (aldosteronoma).

Figure 11 Figure 12A

Coronal reformatted post-contrast CT Axial contrast-enhanced CT image


image in a patient with a renal cell carci-
noma in the upper pole of the left kidney

A metastasis (arrow) is visible in the right adrenal Shows a relatively small uniform adrenal metastases
gland. (arrow).

154 EDUCATIONAL REVIEW MANUAL IN UROLOGY


Figure 12B Figure 13A

Axial contrast-enhanced CT image in a Axial contrast-enhanced CT image showing


different patient showing large heteroge- a small non-specific adrenal nodule (arrow)
neous bilateral adrenal metastases in a patient with lung cancer.
(arrows)

Adrenal metastases have a variable appearance.

Figure 14

Axial contrast-enhanced CT image in a


patient with a left adrenal myelolipoma
(arrow)

Figure 13B

Axial PET image

The nodule is clearly of macroscopic fat density, being


similar in CT appearance to the adjacent retroperitoneal
fat.

Shows intense uptake within the adrenal nodule (arrow),


consistent with a metastatic deposit.

CHAPTER 5: ESSENTIALS OF URORADIOLOGY 155


Figure 15 Figure 16

Axial T2-weighted image showing a right Axial T2-weighted MRI image in a patient
adrenal pheochromocytoma (arrow) with a left adrenal pheochromocytoma

Internal cystic change and hemorrhage is a useful sign


that an adrenal mass may be a pheochromocytoma. Note
the fluid level (arrow) in this lesion, due to cyst formation
with internal hemorrhage.

Note that the lesion is of extremely high T2 signal,


approaching that of cerebrospinal fluid. This “lightbulb”
sign is said to be characteristic of pheochromocytoma,
although in practice it is infrequently observed.
Figure 17B

Coronal image from a MIBG scan in the


same patient
Figure 17A

Axial T1-weighted post-contrast MRI image


showing a relatively non-specific heteroge-
neously enhancing left adrenal mass
(arrow)

Shows increased uptake within the adrenal lesion (arrow),


confirming it is a pheochromocytoma.

156 EDUCATIONAL REVIEW MANUAL IN UROLOGY


Figure 18 Figure 19A

Coronal T2-weighted MRI image showing a Coronal reformat CT image showing a large
large heterogeneous mass superior to the cystic lesion superior to the left kidney
left kidney

Figure 20A
Tumor invasion (arrow) is seen into the inferior vena cava.
The finding of venous invasion in association with an
Ultrasound image demonstrating a well-cir-
adrenal mass is strongly suggestive of an adrenocortical
carcinoma, as in this case. cumscribed anechoic lesion (arrow) in the
lower pole of the right kidney with posterior
acoustic enhancement

Figure 19B

Axial T2-weighted image showing a mass


has marked internal complexity

This is a typical simple cyst.

This lesion was resected and the final pathology demon-


strated a benign hemorrhagic adrenal pseudocyst.
Resection is an appropriate management for such lesions,
because a small number of these hemorrhagic masses
harbor malignancy.

CHAPTER 5: ESSENTIALS OF URORADIOLOGY 157


Figure 20B Figure 20C

Axial contrast-enhanced CT image Coronal T2-weighted MRI image

Shows the lesion (arrow) is of uniform water density (8 Shows the lesion (arrow) is of extremely high T2 signal
Hounsfield units) with a imperceptible wall and a well-cir- intensity, consistent with fluid content.
cumscribed interface with the adjacent renal parenchyma.

Figure 20D Figure 21A

Axial post-contrast T1-weighted image Axial non-contrast image

Confirms the lesion (arrow) is non-enhancing and cystic.

Shows a non-specific lesion (arrow) arising from the poste-


rior aspect of the left kidney. The lesion is of intermediate
density (49 Hounsfield units).

158 EDUCATIONAL REVIEW MANUAL IN UROLOGY


Figure 21B Figure 22A

Axial contrast-enhanced CT image Axial contrast-enhanced CT image

Shows a mass (arrow) in the left kidney. Non-contrast


images were not obtained in this case, so based on this
image alone, it is not possible to determine whether these
are high density cysts or a solid mass.
Shows the lesion (arrow) increases in density to 61
Hounsfield units. A diagnosis of papillary renal cell carci-
noma was confirmed at surgery. An increase in density of
over 10 Hounsfield units is indicative of enhancement, and
suggests the lesion is solid. In particular, papillary renal
cell carcinoma can demonstrate minimal enhancement, as
in this case. In cases of doubtful enhancements by CT,
ultrasound may be helpful to document the solid vs. cystic
nature of a mass.

Figure 22B Figure 23A

Delayed contrast-enhanced CT image Axial contrast-enhanced CT image

Shows the mass (arrow) has decreased in density to 46


Hounsfield units. This de-enhancement confirms that the
lesion is solid, and resection documented the presence of
a renal cell carcinoma. Shows a small high density lesion (arrow) in the left kidney.

CHAPTER 5: ESSENTIALS OF URORADIOLOGY 159


Figure 23B Figure 23C

Corresponding axial non-contrast CT image Delayed contrast-enhanced CT image

Shows that the lesion is difficult to localize, precluding


confident placement of a region of interest to confirm Shows the lesion (arrow) has decreased in density to 54
enhancement. Hounsfield units. Resection confirms that this was a solid
renal cell carcinoma. De-enhancement can be useful to
confirm the solid nature of a lesion that is not visible on
non-contrast images.

Figure 24A Figure 24B

Axial non-contrast-enhanced CT image Axial contrast-enhanced CT image

Shows a small hypodense lesion (arrow) in the left kidney. Shows the lesion (arrow) has increased from 10
Hounsfield units to 35 Hounsfield units. However, MRI con-
firmed that this lesion was a cyst, and the increase in CT
density was artifactual. Such spurious increases in CT
density can be seen in small cysts at CT, and is known as
pseudoenhancement.

160 EDUCATIONAL REVIEW MANUAL IN UROLOGY


Figure 25A Figure 25B

Axial T1-weighted MR image showing a lesion Axial post-contrast MRI image


(arrow) of high T1 signal in the left kidney

Shows the lesion is now isointense to the adjacent to the


renal parenchyma. It is difficult to assess visually whether
the lesion (arrow) has increased in signal intensity or not.

Figure 25C Figure 26

Subtraction image generated by electronically Axial contrast-enhanced CT image


subtracting the pre-contrast image from the
post-contrast image

Shows that the lesion (arrow) is non-enhancing (black).


Subtraction images can be helpful in the assessment of
Shows a typical angiomyolipoma (arrow) arising from the
enhancement, particularly when the lesion is of high T1
left kidney. The mass is clearly of similar CT density to the
signal intensity prior to gadolinium administration. In such
adjacent retroperitoneal fat. For all practical purposes, a
a setting, it is difficult to visually appreciate whether there
renal mass that contains macroscopic fat is an angiomy-
is further increase in T1 signal after contrast.
olipoma.

CHAPTER 5: ESSENTIALS OF URORADIOLOGY 161


Figure 27A Figure 27B

Axial contrast-enhanced 7mm thick CT image Axial 2.5mm thick non-contrast CT image

Shows a hypodense lesion (arrow) in the left kidney that is Shows that the lesion (arrow) is of fat density (-40
not of sufficiently low density (-5 Hounsfield units) to con- Hounsfield units), confirming the presence of macro-
firm the presence of macroscopic fat. scopic fat and the diagnosis of angiomyolipoma.

Figure 28A Figure 28B

Axial T1-weighted MR image Axial fat saturated T1-weighted MRI image.

Shows a lesion (arrow) of high T1 signal intensity in the left


Shows that the lesion (arrow) has lost signal intensity, con-
kidney.
firming it contains macroscopic fat and is therefore an
angiomyolipoma.

162 EDUCATIONAL REVIEW MANUAL IN UROLOGY


Figure 29 Figure 30A

Ultrasound image Axial non-contrast CT image

Shows the typical marked echogenicity of an angiomy-


olipoma (arrow) in the upper pole of the right kidney.

Shows a contour bulge in the right kidney, without visible


macroscopic fat.

Figure 30B Figure 31

Axial contrast-enhanced CT image Axial contrast-enhanced CT image

Shows an infiltrative mass in the left kidney, associated


with left renal vein invasion (arrow). While renal cell carci-
Shows a heterogeneously enhancing mass in the right kid- noma is usually well-circumscribed, it is occasionally more
ney. The appearance is typical of a renal cell carcinoma, infiltrative, as in this case.
and density measurements are not required to confirm
enhancement because the visual appearances are so dis-
tinctive.

CHAPTER 5: ESSENTIALS OF URORADIOLOGY 163


Figure 32 Figure 33

Coronal reformative CT image Axial contrast-enhanced CT image

Shows almost complete replacement of the right kidney by


a mass (horizontal arrow) with tumor thrombus extending Shows a uniformly enhancing mass (arrow) in the left kid-
into and expanding the inferior vena cava, reaching with ney, without macroscopically visible fat. Resection docu-
the upper end of the tumor thrombus (vertical arrow) being mented that this lesion was a fat poor angiomyolipoma,
at the level of the diaphragm. although the presumptive preoperative diagnosis was a
renal cell carcinoma based on the imaging appearance.

Figure 34A Figure 34B

Axial contrast-enhanced CT image Axial non-contrast-enhanced CT image

Shows a small nonspecific lesion (arrow) in the left kidney. Does not show appreciable macroscopic fat in the lesion.
Histogram analysis was performed on the region of inter-
est indicated by the white rectangle.

164 EDUCATIONAL REVIEW MANUAL IN UROLOGY


Figure 34C Figure 35A

Histogram analysis from the region of inter- Axial contrast-enhanced CT image


est in Figure 34B

Shows that several voxels have a density under -20


Hounsfield units. A final diagnosis of fat-poor angiomy-
olipoma was established. While the use of histogram anal- Shows bilateral oncocytomas (arrows). The lesion in the
ysis was helpful in this case, the criteria for establishing the right kidney has a central stellate scar, but this finding is
diagnosis of a fat-poor angiomyolipoma using this analyti- not seen in the lesion in the left kidney. While a central stel-
cal approach has not been well-established. late scar may suggest the diagnosis of oncocytoma, it is
difficult to distinguish such a central scar from cavitary
necrosis in a renal cell carcinoma.

Figure 35B Figure 36

Arteriogram (in a different patient) showing Axial contrast-enhanced CT image


the typical spoke-wheel appearance of an
oncocytoma in the right kidney

Shows upper tract transitional cell carcinoma in the left


kidney. Infiltrative tumor is seen along the wall of the pelvis
(arrow), with similar infiltrative tumors seen within the
dilated cavities.

While such an appearance is suggestive, it is difficult to


see how the findings could be considered sufficiently con-
vincing to obviate the need for surgical confirmation.

CHAPTER 5: ESSENTIALS OF URORADIOLOGY 165


Figure 37 Figure 38

Axial contrast-enhanced CT image Axial contrast-enhanced CT image

Shows an intraparenchymal hypodense mass within the


right mass (arrow) within the right kidney. This was a
proven transitional cell carcinoma, although the radiologic Shows a unifocal mass (horizontal arrow) in the left kidney.
appearances overlap with that of renal cell carcinoma. The appearance is suggestive of a renal cell carcinoma,
but an additional hypodense mass (vertical arrow) is pre-
sent in the spleen. The final diagnosis was renal lym-
phoma. The presence of extra renal disease can be a use-
ful clue to the diagnosis of lymphoma.

Figure 39 Figure 40

Axial contrast-enhanced CT image Axial contrast-enhanced CT image in a


patient with widely disseminated
metastatic breast cancer

Shows metastases (arrows) within both kidneys. Multiple


liver metastases are also visible.
Shows infiltrative soft perinephric soft tissue encasing the
left kidney, and also similar infiltrative soft tissue in the left
periaortic space and surrounding the left renal artery. Per-
inephric encasement is typical of lymphomatous involve-
ment of the kidneys, although it is not the commonest pat-
tern.

166 EDUCATIONAL REVIEW MANUAL IN UROLOGY


Figure 41A Figure 41B

Axial non-contrast CT image Axial contrast-enhanced CT image

Shows a lesion (arrow) of intermediate density (54 Shows that the lesion (arrow) is non-enhancing (the
Hounsfield units) in the right kidney. observed increase in 3 Hounsfield units from 54 to 57 does
not constitute enhancement). The lesion is also of uniform
density and well-circumscribed, and is characteristic of a
hyperdense Bosniak II cyst.

Figure 42 Figure 43

Axial contrast-enhanced CT image Axial contrast-enhanced CT image

Shows a cystic lesion in the left kidney that contains a very


thin septum (arrow). This is a mildly complex Bosniak II
cyst. Such mildly complex features are benign.
Shows a heavily and thickly calcified cystic mass (arrow)
in the right kidney. Thickly calcified walls and septa are
indicative of a Bosniak III lesion, and such lesions have
approximately 50% chance of malignancy. This lesion was
resected and found to be a calcified cystic renal cell carci-
noma.

CHAPTER 5: ESSENTIALS OF URORADIOLOGY 167


Figure 44 Figure 45

Axial contrast-enhanced CT image Axial contrast-enhanced CT image

Shows a markedly complex cyst in the left kidney, with a


thickly enhancing and nodular wall. Such markedly com-
plex features of those of a Bosniak IV cyst, and this lesion Shows a heterogeneously-enhancing mass in the right kid-
were found to be a cystic renal cell carcinoma at resection. ney. A small sliver of fat (arrow) is seen at the periphery of
the lesion. While it is difficult to know whether this fat is
within the mass or represents entrapped fat peripherally,
the primary consideration should remain renal cell carci-
noma, and this diagnosis was confirmed surgically.

Figure 46 Figure 47

Axial contrast-enhanced CT image Coronal reformatted non-contrast CT image

Shows a fat density mass arising from the left kidney. The Shows left hydroureteronephrosis that can be traced to the
mass is largely exophytic, but a parenchymal notch is visi- level of an obstructing stone (arrow) in the upper to mid left
ble in the kidney, as a vascular pedicle (arrow), hoping to ureter. Non-contrast CT as demonstrated excellent accu-
confirm the diagnosis of an exophytic angiomyolipoma. racy in the diagnosis of obstructive calculi, and has essen-
tially replaced IVU in the evaluation of suspected renal
colic.

168 EDUCATIONAL REVIEW MANUAL IN UROLOGY


Figure 48 Figure 49

Coronal reformatted non-contrast CT CT image


images in a patient with HIV infection
receiving indinavir (Crixivan)

Shows a small stone (arrow) in the distal left ureter. A thin


surrounding rim of tissue is visible, representing edema-
Left hydroureteronephrosis can be traced at the level of tous ureteral wall. This so-called ureteral rim sign helps dif-
upper to mid-left ureter (arrow) but there is no visible stone ferentiate a ureteral calculus from a phlebolith.
at the point of transition from dilated to non-dilated ureter.
For practical purposes, indinavir stones are the only ure-
thral calculi that are radiolucent at CT, although secondary
signs may help establish the diagnosis, as in this case.

Figure 50 Figure 51

Non-enhanced CT image Bilateral nephrocalcinosis in a patient with


medullary sponge kidney

Note that the calcifications are relatively coarse and corre-


Shows a pelvic phlebolith (white arrow). A linear strand of spond to the locations of the medullary pyramids, where
tissue (black arrow) is seen extending from the phlebolith. the calcifications form.
This tissue strand is believed to represent the atretic ves-
sel within which the phlebolith formed, and this so-called
“comet-tail” sign is another finding that may help distin-
guish stones and phleboliths.

CHAPTER 5: ESSENTIALS OF URORADIOLOGY 169


Figure 52 Figure 53

Plain abdominal radiograph demonstrating Plain abdominal radiograph in a child


bilateral cortical nephrocalcinosis, with a
thin rim of calcification visible around the
periphery of the kidneys

Demonstrates the characteristic nephrocalcinosis


(arrows) seen in children with primary hyperoxaluria
Such an appearance is most commonly due to chronic (oxalosis).
glomerulonephritis.

Figure 54A Figure 54B

Axial contrast-enhanced CT image Axial contrast-enhanced CT image in a dif-


ferent patient

Shows wedge-shaped striations in the right kidney, due to


pyelonephritis.

Shows formation of an abscess (arrow) in addition to the


clinical scenario of sepsis; this lesion can be distinguished
from a simple cyst by the presence of a peripheral irregu-
lar rim of low density edematous renal parenchyma.

170 EDUCATIONAL REVIEW MANUAL IN UROLOGY


Figure 55 Figure 56

Plain abdominal radiograph Axial contrast-enhanced CT image

Shows the typical appearances of xanthogranulomatous


pyelonephritis in the right kidney. The kidney shows cal-
iceal dilatation due to the presence of a staghorn calculus
(vertical arrow) and reduced enhancement in the kidney.
In addition, inflammatory changes (horizontal arrows) can
be seen in the perinephric space extending to the poste-
rior abdominal wall.

Shows air (arrow) surrounding the right kidney due to


emphysematous pyelonephritis.

Figure 57 Figure 58

Axial contrast-enhanced CT image in a Axial contrast-enhanced CT image in a


patient with fungal pyelonephritis patient with systemic lupus erythematosus

Wedge-shaped areas (arrow) of irregular hypodensity are


A lucent filling defect (arrow) is seen in a left-sided calyx, seen in the right kidney, consistent with renal infarcts.
due to a fungus ball. The differential for such a lucent filling
defect in the upper tract include hematoma and transi-
tional cell carcinoma.

CHAPTER 5: ESSENTIALS OF URORADIOLOGY 171


Figure 59 Figure 60

One-shot IVU in a patient after a motor vehi- Ultrasound image showing a post-trau-
cle accident matic perinephric hematoma (between
arrows)

The nephrogram of the left upper kidney is virtually absent


above a visible fracture line (arrow).

Figure 61A Figure 61B

Axial contrast-enhanced CT image Axial contrast-enhanced CT image in a dif-


ferent patient

Shows a post-traumatic fracture (arrow) in the right kidney.


Ill-defined low density surrounding the right kidney with an
Shows a subcapsular hematoma (arrow) in the right kid-
irregular outer margin is consistent with associated per-
ney. Note that the subcapsular hematoma has a smooth
inephric hematoma.
outer margin and deforms the underlying kidney.

172 EDUCATIONAL REVIEW MANUAL IN UROLOGY


Figure 62A Figure 62B

Axial contrast-enhanced CT image Axial contrast-enhanced CT image

Showing a wedge-shaped area of non-enhancement


(arrow) in the left kidney, consistent with infarction.

Shows an ill-defined hypodense area (arrow) in the right


kidney, consistent with a contusion.

Figure 63A Figure 63B

Photomontage showing the appearance of Photomontage showing the appearance of


active extravasation on early and late post- urinary extravasation on early and late
contrast CT images after trauma post-contrast images

The active extravasation is seen as an irregular focus of The extravasation appears as a non-specific collection of
brisk enhancement (arrow) within a large perinephric fluid (arrow) on the early image. On the delayed phase
hematoma. The extravasated contrast has dispersed on image, extravasated contrast (arrow) is seen anterior to
the later image. the left kidney, and also is contiguous with the left renal
pelvis.

CHAPTER 5: ESSENTIALS OF URORADIOLOGY 173


Figure 63C Figure 64

Photomontage showing the appearances of Axial contrast-enhanced CT image


a pseudoaneurysm on early and late post-
contrast images

The pseudoaneurysm is seen as a focus of intense


rounded smooth enhancement (arrow) on the early image, Shows a devascularized left kidney in a patient after a
which gradually fades on the delayed phase image motor vehicle accident. Note that the left renal artery has a
(arrow). blind-ending (arrow). This is likely due to an occlusive inti-
mal tear, the commonest etiology of devascularization
after trauma.

Figure 65A Figure 65B

MAG3 renogram in a patient with right Retrograde pyelogram showing marked


ureteropelvic junction obstruction right hydronephrosis to the level of the
ureteropelvic junction, below which the
ureter is of normal caliber

Shows retention of the isotope within the right kidney with- The appearance is typical of ureteropelvic junction
out visible excreted activity within the right renal pelvis. obstruction. Lucent filling defects within the right mid
ureter are due to air bubbles.

174 EDUCATIONAL REVIEW MANUAL IN UROLOGY


Figure 65C Figure 66

Coronal reformatted CT image in the same Retrograde pyelogram showing a lucent fill-
patient ing defect (arrow) in the left ureter

Shows contrast within the dilated pelvis (black arrow) and


an underlying crossing anomalous left renal artery (white
arrow).

Note that the ureter below the filling defect is slightly


Figure 67 dilated. This “goblet” side is relatively characteristic of
ureteral transitional cell carcinoma, because the stiff tumor
Axial contrast-enhanced CT image in a holds open the ureter.
84-year-old man with a history of bladder
cancer
Figure 68A

Axial contrast-enhanced CT image in a


56-year-old women with a 5-year history of
reccurrent hematuria

A hypodense mass (arrow) in the left kidney was found to


be a transitional cell carcinoma at resection. Transitional
Shows a lucent filling defect in the distal right ureter
cell carcinoma should be considered in the differential for
(arrow).
a solid renal mass in a patient with a history of bladder
cancer.

CHAPTER 5: ESSENTIALS OF URORADIOLOGY 175


Figure 68B Figure 69

Coronal reformatted CT image Axial contrast-enhanced CT image in a


patient with a urachal adenocarcinoma

Demonstrates that the filling defect (arrow) has a linear


configuration in the distal ureter. Final histopathology con-
firmed the diagnosis of a fibroepithelial polyp. The finding of a midline anterior bladder mass (arrow)
should raise consideration of a urachal cancer.

Figure 70 Figure 71

Axial contrast-enhanced CT image Sagittal T2-weighted post-gadolinium


image

Shows an infiltrative enhancing lesion in the (arrow) poste-


Shows a polypoid filling defect in the bladder with surface rior wall of the bladder. Such an infiltrative appearance is
calcification (arrow). This appearance is typical of transi- typical of higher grade transitional cell carcinoma.
tional cell carcinoma.

176 EDUCATIONAL REVIEW MANUAL IN UROLOGY


Figure 72A Figure 72B

Coronal T2-weighted MRI image Axial contrast-enhanced CT image in a


patient with an infiltrative transitional cell
carcinoma encasing most of the bladder
wall

Shows a polypoid transitional cell carcinoma (arrow) in the


bladder. The muscular wall of the bladder can be seen as
a low signal layer deep to the tumor, and there is no sign of
tumor extension into this muscle layer.

A focus of extravesical extension (arrow) is visible.

Figure 72C

Axial contrast-enhanced CT image in a


patient with an infiltrative transitional cell
carcinoma of the bladder

Figure 73A

Photomontage showing the appearance of


hematoma within the bladder on prone and
supine post-contrast CT images

Metastatic adenopathy (arrow) is visible abutting the left Note that the hematoma (arrow) changes position
pelvic side wall. between prone and supine scans.

CHAPTER 5: ESSENTIALS OF URORADIOLOGY 177


Figure 73B Figure 73C

Axial T2-weighted MRI image in a patient Axial T2-weighted MRI image in a patient
with a pheochromocytoma (arrow) of the with a leiomyoma (arrow) of the bladder
bladder wall. wall.

The finding of low T2 intensity would be unusual for a


malignant mass, and other diagnosis should be consid-
ered.

Figure 74A Figure 74B

Cystogram in a patient after a motor vehi- Cystogram demonstrating the typical


cle accident appearance of intraperitoneal bladder rup-
ture, with extravasated contrast outlining
bowel loops

The typical flame shaped appearance (arrow) of extraperi-


toneal extravasation is evident, consistent with extraperi-
toneal bladder rupture. Note there is also wide post-trau-
matic diastasis of the symphysis pubis.

178 EDUCATIONAL REVIEW MANUAL IN UROLOGY


Figure 75A Figure 75B

CT cystogram showing perivesical contrast Axial CT cystogram image showing


extravasation (arrow) due to extraperi- extravasated contrast outlining bowel
toneal bladder rupture loops within the pelvis and lower abdomen
due to intraperitoneal bladder rupture

Figure 76 Figure 77

Cystogram showing the typical “pine cone” Axial contrast-enhanced CT image in a


or “Christmas tree” appearance of a spastic patient with a flaccid neurogenic bladder
neurogenic bladder

In this case, the findings were due to sacral agenesis. The bladder seemed to be of large volume, although the
distinction from physiologic distention is difficult. However,
note fecal loading within the rectum. This coexistent find-
ing should raise consideration of neurological problems.

CHAPTER 5: ESSENTIALS OF URORADIOLOGY 179


Figure 78A Figure 78B

Coronal T2-weighted MR image of the Coronal T2-weighted MR image of the


prostate in a healthy 26-year-old man prostate in a healthy 50-year-old man

The prostate is small, of uniform intermediate T2 signal


intensity, and lacks visible zonal differentiation.

The prostate is of intermediate size due to early changes


of benign prostatic hyperplasia, and the hyperplastic tran-
sition zone (arrows) can be seen as a central globular
Figure 78C structure, surrounded by the compressed central zone.

Figure 79
Coronal T2-weighted MR image of the
prostate in a healthy 78-year-old man
IVU in a patient with marked benign pro-
static hyperplasia

The prostate is enlarged due to marked hyperplastic


changes in the transition zone. The peripheral zone is
compressed to a rim of thin tissue and the central zone is Shows a large filling defect (asterisk) indenting and lifting
barely visible as thin “pseudocapsule” between the transi- the bladder base. This filling defect is the markedly large
tion and peripheral zones. prostate. Note the bladder is also trabeculated.

180 EDUCATIONAL REVIEW MANUAL IN UROLOGY


Figure 80A Figure 80B

Axial contrast-enhanced CT image in a Axial contrast-enhanced CT image in the


patient with metastatic prostate cancer, same patient at a higher level demonstrat-
demonstrating adenopathy (arrow) along ing retroperitoneal adenopathy (arrow) due
the pelvic side wall and external iliac to metastatic prostate cancer
chains

Figure 81 Figure 82A

Bone scan showing the typical appear- Transrectal ultrasound showing a hypoe-
ances of widely metastatic prostate cancer choic area (arrow) in the left peripheral
with multifocal areas of increased uptake zone of the prostate due to prostate cancer
in the pelvis and spine

CHAPTER 5: ESSENTIALS OF URORADIOLOGY 181


Figure 82B Figure 83

The addition of Doppler interrogation Axial T2-weighted MRI image in a patient


shows increased vascularity within the with prostate cancer
lesion (arrow), again typical of prostate
carcinoma

Shows a large irregular focus of low T2 signal intensity in


the left peripheral zone of the prostate. The lesion has an
irregular interface with the adjacent periprostatic fat and
extracapsular extension is evident (arrow).

Figure 84A Figure 84B

Retrograde urethrogram in a patient with a Voiding cystourethrogram in the same


stricture of the bulbar urethra patient

The stricture (arrow) is not well appreciated due to obliq- This image better delineates the stricture (arrow), and also
uity. documents upstream dilatation of the urethra, confirming
functionally significant obstruction.

182 EDUCATIONAL REVIEW MANUAL IN UROLOGY


Figure 85 Figure 86

High frequency ultrasound image of the ure- Retrograde urethrogram in a patient with
thra showing irregular wall thickening traumatic urethral disruption
(arrow) of a urethral stricture

Extravasated contrast is evident (vertical arrow) within the


soft tissues of the pelvis. A fracture line can also be seen in
the left superior pubic ramus (oblique arrow).

Figure 87 Figure 88

Cystogram performed with a double-bal- Axial T2-weighted MRI image showing a


loon technique shows a large urethral urethral diverticulum (arrow)
diverticulum (arrow)

CHAPTER 5: ESSENTIALS OF URORADIOLOGY 183


Figure 89 Figure 90

Ultrasound image shows a large heteroge- Testicular ultrasound showing the typical
neously hypoechoic mass (between appearance of tubular ectasia of the rete
arrows) in the upper pole of the testes due testis (arrow)
to testicular cancer

Figure 91 Figure 92

Photomontage of bilateral testicular ultra- Testicular ultrasound showing a heteroge-


sound with Doppler showing a hypervascu- neous and enlarged testis
lar mass (arrow) in 1 testis

This mass was due to lymphomatous spread in an elderly


male patient.

In this case, it was due to metastatic prostate cancer.

184 EDUCATIONAL REVIEW MANUAL IN UROLOGY


Figure 93 Figure 94

IVU demonstrating the typical stippled cal- Axial contrast-enhanced CT image


cification of neuroblastoma

Shows a large heterogeneous mass (asterisk) above the


right kidney, with leftward displacement of the aorta and
obliteration of the inferior vena cava. This was a proven
neuroblastoma.

Figure 95 Figure 96

Coronal T1-weighted post-contrast MRI Axial contrast-enhanced CT image of


image nephroblastomatosis in a 9-month-old boy

Shows bilaterally enlarged kidneys due to presence


of cortical hypodense, non-enhancing, soft tissue mass
(arrows). Note distorted remaining renal parenchyma
centrally.
Shows a heterogeneously enhancing neuroblastoma
(between arrows) above the left kidney.

CHAPTER 5: ESSENTIALS OF URORADIOLOGY 185


Figure 97 Figure 98

Ultrasound image showing a Wilms’ Tumor Axial contrast-enhanced CT image showing


(between calipers) in the kidney a Wilms’ tumor (arrow) in the right kidney

Figure 99 Figure 100

Axial contrast-enhanced CT image showing Coronal T2-weighted MRI image of crossed


a pelvic kidney (arrow) anterior to the unfused ectopia
sacrum

Note that the right kidney (white arrow) is located normally


while the left kidney (black arrow) is located abnormally in
the right lower quadrant.

186 EDUCATIONAL REVIEW MANUAL IN UROLOGY


Figure 101 Figure 102

IVU of horseshoe kidney, showing the pelvi- Coronal reformat CT image showing a
caliceal systems to be malrotated with horseshoe kidney
medial deviation inferiorly

The bridging tissue (arrow) between the lower folds of both


kidneys is clearly documented.

Figure 103 Figure 104A

Voiding cystourethrogram showing marked Cystogram showing a filling defect (arrow)


bilateral vesicoureteral reflux in the bladder of a child.

CHAPTER 5: ESSENTIALS OF URORADIOLOGY 187


Figure 104B Figure 105

Ultrasound image confirms that the lesion IVP showing the typical cobra-head appear-
(arrow) is fluid-containing, consistent with ance (arrow) of the distal right ureter due to
a ureterocele a ureterocele

A ureterocele is the primary consideration for a filling


defect in the bladder of a child.

Figure 106

Voiding cystourethrogram in a patient with


posterior urethral valves

Note that the prostatic urethra (P) is dilated and elon-


gated, with a horizontal filling defect (arrow) inferiorly due
to the valve leaflets.

188 EDUCATIONAL REVIEW MANUAL IN UROLOGY


Chapter 6:
Cytology of the
Urogenital Tract
Donna E. Hansel, MD, PhD

Contents
Cytology Renal Tumors III:
1. Types of Epithelial Cells Miscellaneous Renal Lesions

2. Use of Cytology 1. Epithelial Tumors

3. Inflammatory and Other 2. Mesenchymal Tumors


Non-neoplastic Conditions
4. Tumors of the Urinary Bladder Renal Cystic Diseases
5. Effects of Therapy 1. Renal Dysplasia
2. Polycystic Kidney Diseases
Renal Tumors I: 3. Cysts (without dysplasia)
Pediatric Renal Tumors in Hereditary Syndromes
1. Wilms’ Tumor (Nephroblastoma) 4. Miscellaneous Conditions
2. Mesoblastic Nephroma
3. Clear Cell Sarcoma Adrenal Gland
4. Rhabdoid Tumor of Kidney 1. Normal Histology
2. Lesions of the Adrenal Cortex
Renal Tumors II: 3. Lesions of the Adrenal Medulla
Renal Carcinoma 4. Miscellaneous Adrenal Lesions
1. General Features
2. Associations and Syndromes Penis and Scrotum I:
3. The Fuhrman Grading System Non-Neoplastic Lesions
for Renal Cell Carcinoma 1. Normal Anatomy and Histology
4. Clear Cell Renal Cell Carcinoma 2. Non-neoplastic Diseases of
5. Papillary Renal Cell Carcinoma the Penis
6. Chromophobe Renal Cell Carcinoma 3. Non-neoplastic Disease of
7. Collecting Duct Carcinoma the Scrotum
(Bellini Duct Carcinoma)
8. Medullary Carcinoma Penis and Scrotum II:
9. Mucinous Tubular and Spindle Tumors and Tumor-like Lesions
Cell Neoplasm 1. Condyloma acuminatum
10. Translocation Carcinoma 2. Pre-malignant Lesions
3. Malignant Neoplasms of the Penis
4. Malignant Lesions of the Scrotum

CHAPTER 6: CYTOLOGY OF THE UROGENITAL TRACT 189


Bladder I: Testis I:
Non-Neoplastic Lesions Non-Neoplastic Disorders
1. Normal Histology 1. Cryptorchidism
2. Normal/ Metaplastic Lesions 2. Infertility
3. Cystitis 3. Inflammation and Infection
4. Miscellaneous Lesions 4. Testicular Infarcts

Bladder II: Testis II:


Tumors of the Urinary Bladder Germ Cell Tumors
1. Benign Epithelial Neoplasms 1. General
2. Malignant Epithelial Neoplasms 2. Intratubular Germ Cell Neoplasia
3. Paraganglioma 3. Classic Seminoma
4. Benign Mesenchymal Neoplasms 4. Spermatocytic Seminoma
5. Malignant Soft Tissue Neoplasms 5. Embryonal Carcinoma
6. Yolk Sac Tumor
Prostate I: Non-Neoplastic Lesions of the 7. Teratoma
Prostate 8. Choriocarcinoma
1. Normal Histology 9. Mixed Germ Cell Tumors
2. Prostatic Hyperplasia 10. Special Stains
3. Infarcts 11. Regression of Germ Cell Tumors
4. Prostatitis
5. Basal Cell Hyperplasia Testis III:
6. Clear Cell Cribriform Hyperplasia Non-Germ Cell Tumors
7. Atrophy 1. Leydig Cell Tumor
8. Squamous Metaplasia 2. Sertoli Cell Lesions
3. Other Testicular Tumors
Prostate II:
Tumors and Tumor-Like Lesions Testicular Adnexae
of the Prostate
1. Epididymal Tumors
1. Polyps of the Prostatic Urethra
2. Rete Testis: Adenocarcinoma
2. Prostatic Intraepithelial
Neoplasia (PIN)
3. Prostatatic (Acinar) Adenocarcinoma
4. Urothelial Carcinoma of the Prostate
5. Squamous Cell Carcinoma of
the Prostate
6. Treatment Effects

190 EDUCATIONAL REVIEW MANUAL IN UROLOGY


Cytology

f. Nucleoli are inconspicuous or absent


in normal cells
1. Types of Epithelial Cells

Urothelium – bladder, ureters, renal pelvis, g. Multinucleation (as in >2, <50) and
proximal urethra small red nucleoli can be seen after
catheterization, as a reactive change
Squamous cells – bladder trigone (40% of women), h. In voided urines, urothelial cells are
distal urethra shed singly: in washings/ brushings,
the cells tend to be aggregated in
Glandular cells – caused by foci of glandular meta- sheets. If there are no fibrovascular
plasia in the GU tract cores, however, these sheets do not
represent papillary clusters
i. Increased numbers of urothelial cells
can be seen with:
2. Use of Cytology

A. Diagnosis and follow-up of patients with • Instrumentation/ catheterization


neoplasms of the urinary tract • Calculi
B. Screening of high-risk patients (e.g., chemi- • Inflammatory and neoplastic
cal industry workers) conditions
2. Squamous Cells
1. Voided urine -- catheterization can dis- a. Normally seen in voided urine as a
C. Sampling Techniques:

lodge clusters of cells and induce mild contaminant from the distal ure-
atypia thra/vagina/ vulva; also may be seen
2. Washings and brushings arising from the bladder trigone in
a. Produce abundant well-preserved 40% of women and in men on estro-
cells for evaluation gen therapy for prostate cancer
b. Risk of false-positive as instrumen- b. Also increased in cases of chronic
tation may result in papillary” irritation and inflammation
clusters as well as red blood cells 3. Columnar cells
a. Due to glandular metaplasia in the
1. Urothelial cells bladder and ureters (in men, may
D. Cytology of Normal Cells

a. Superficial/umbrella cells -- form also originate from the prostate or


the majority of cells in specimens seminal vesicles after prostatic mas-
• Vary greatly in size and shape, but sage)
generally are large, flat, polygo- b. Prostate cells
nal cells with abundant thin • Form small aggregates that
(transparent) cytoplasm maintain a ball-like or glandular
b. Deep urothelial cells -- tend to have configuration
a smaller size and a denser cyto- • Cells are small and may be
plasm columnar or round with
c. Cytoplasm: cyanophilic (blue) slightly pink cytoplasm;
and vacuolated cytoplasm also is often foamy
d. Nuclei: tend to have round/ oval or vacuolated
nuclei with smooth contours, finely • Nucleus: single, small, round
granular chromatin and well-defined or oval, with finely granular
nuclear membranes chromatin
e. N:C ratio may be high, but never c. Seminal vesicle cells
exceeds 1:2

CHAPTER 6: CYTOLOGY OF THE UROGENITAL TRACT 191


• Usually smaller than b. Has been associated with interstitial
transitional cells and are almost nephritis and obstruction of the
always degenerated ureteral lumen and may be important
• Nuclei are enlarged and hyper- in renal transplantation
chromatic with little or no chro- c. Decoy cells: huge cells with large
matin structure (glassy appear- nuclei having interrupted nuclear
ance) membranes and glassy basophilic
• Cytoplasm contains golden-yel- inclusions that push chromatin to the
low lipochrome pigment and edges
vacuoles d. Because of their high N:C ratio, cells
may mimic carcinoma (hence the
name decoy cells) but the key to
their benign nature is the glassy
3. Inflammatory and Other

chromatin pattern and the relative


Non-neoplastic Conditions

paucity of affected cells


1. Nuclei become enlarged and hyperchro-
A. Reactive urothelial cells

matic with apparent nucleoli, but main-


tain their smooth borders and low N:C
ratio
4. Tumors of the Urinary Bladder

2. Binucleation/ multinucleation common A. Urothelial papilloma and papillary

1. Usually associated with lots of neu- potential (PUNLMP) -- generally not


B. Bacterial/ fungal/ parasitic infections urothelial neoplasm of low malignant

trophils, red cells, and lymphocytes in the diagnosed on urine cytology


urine (a few lymphocytes and some scat-
tered red cells can be seen in normal peo- 1. Papillary fragments or clusters of cells
B. Low-grade papillary urothelial carcinoma

ple) 2. Cells have an increased N:C ratio


2. The usual suspects in these situations 3. Nuclei vary in size and shape, but often
include: appear indented or grooved
a. Bacteria – especially 4. Variability of nuclear staining within a
Gram-negative rods given group
b. Candida – see budding yeast and 5. These findings are most significant in a
pseudohyphae voided urine sample (since similar fea-
c. Trichomonas – most common tures can be induced by instrumentation)
parasite in the US; in Egypt, 6. Urothelial carcinoma cells are usually
look for Schistosomiasis larger than normal urothelial cells and
3. Remember: if you see a lot of bugs and have eccentric nuclei, but chromatin is
yeast, but no inflammation, the most still fairly evenly distributed
likely explanation is contamination of the 7. Nucleoli are absent or small
specimen C. High-grade papillary urothelial carci-

1. HSV: look for multinucleated transitional


C. Viral infections noma and flat urothelial carcinoma in

cells with “ground glass” nuclei and large 1. Large numbers of poorly cohesive cells
situ

eosinophilic intranuclear inclusions sur- that shed singly or in small clusters


rounded by a halo 2. High N:C ratio
2. CMV: classically associated with an 3. Nuclei are large, hyperchromatic, pleo-
enlarged nucleus which contains a single morphic, with prominent red nucleoli
irregular blue-ish bird’s eye inclusion. 4. Chromatin is coarsely clumped

a. Seen most often in


3. Polyoma virus:

immunocompromised hosts

192 EDUCATIONAL REVIEW MANUAL IN UROLOGY


Renal Tumors I:
Pediatric Renal Tumors

5. Look at the background to distinguish


between in situ and invasive disease: a
1. Wilms’ Tumor (Nephroblastoma)

dirty background with cellular debris and


numerous neutrophils suggests invasion, 1. Comprises >80% of renal tumors in
A. General:

while a clean background suggests that childhood


the tumor may still be in situ (papillary or 2. Occurs most frequently in children 2–4
flat) years of age; extremely uncommon in
children <6 months and >6 years
1. Represents 5% of bladder tumors, and is 3. Slight preponderance of females; bilat-
D. Squamous cell carcinoma

associated with chronic inflammatory eral in 4.4% of cases.


conditions (calculi, diverticulum, cord 4. May be associated with other
bladder, schistosomiasis) congenital anomalies:
2. Urine specimens show abundant kera- a. AGR syndrome
tinized cells (“hard” peach/pink cyto- b. Beckwith-Wiedemann
plasm) with large hyperchromatic nuclei, syndrome -- 5% of patients develop
and often have a dirty background Wilms’ tumor
because the lesions are usually advanced c. Denys-Drash syndrome
(invasive) 5. Clinical presentation:
a. Large abdominal mass that may
1. Represent <5% of bladder tumors cross the midline
E. Adenocarcinoma

2. If arising in the dome, often originate b. Hematuria, abdominal pain, intesti-


from urachal remnants, whereas those in nal obstruction, hypertension
the bladder base are derived from islands c. Laboratory: may have increased ery-
of glandular metaplasia thropoietin level without poly-
3. Tend to shed as small clusters (balls) cythemia
rather than as single cells d. Radiology: calcification uncommon
4. Cytoplasm is often vacuolated and nuclei (in contrast to neuroblastoma)
have prominent nucleoli
5. Poorly differentiated adenocarcinoma 1. Usually large (>5 cm, >500 g)
B. Gross pathology:

can be indistinguishable from high-grade 2. Cut surface is typically solid, soft, and
UCC gray or pink, resembling brain tissue
6. Whenever the diagnosis of adenocarci- 3. Foci of hemorrhage and necrosis are
noma is made, must rule out direct spread often present
or metastatic disease (especially spread 4. Cysts are common
from colon) 5. May appear well circumscribed
C. Pertinent histopathologic

1. Nephrogenic rests: foci of persistent


5. Effects of Therapy definitions:

A. TURP: See nuclear enlargement and dying nephrogenic cells resembling those of the
cells for days or weeks post-procedure developing kidney
B. Irradiation/ Chemotherapy: Cellular a. Perilobar nephrogenic rests –
enlargement and hyperchromasia – see larger located at the edge of the lobe
nuclei but N:C ratio is typically preserved • Have well-defined smooth bor-
(one key to the reactive nature of these cells). ders and show a predominance of
Also: should see degenerative changes, with blastema
poorly preserved (smudged) chromatin • Often are numerous or diffuse
C. BCG treatment: Granulomas; lymphocytes • May be present in approximately
and histiocytes in the background; cellular 1% of infants < 3 months of age
enlargement/ nuclear hyperchromasia

CHAPTER 6: CYTOLOGY OF THE UROGENITAL TRACT 193


b. Intralobar nephrogenic rests -- • Myxoid and fibroblastic stroma
located in the cortex or medulla types are the most common
within the renal lobe where they • Smooth muscle, skeletal muscle,
mingle irregularly with the renal fat, cartilage, bone, and neural
parenchyma: components may also be present
• Show a predominance of stroma 2. Anaplasia
• Usually are single a. Defined as the combination of cells
• Almost never seen except with with very large hyperchromatic
Wilms’ tumor nuclei and multipolar mitotic fig-
c. Nephroblastomatosis: the diffuse
or multifocal presence of nephro- b. The enlarged nuclei must be at least
ures

genic rests, or multicentric or bilat- three times the size of typical


eral Wilms’ tumor blastema nuclei in both axes, and
2. Nephrogenic rests are present in approxi- hyperchromasia must be obvious
mately 41% of patients with unilateral c. Hyperdiploid mitotic figures must
Wilms’ tumor, but are present in 95% of be present
patients with bilateral Wilms’ tumors. d. Mitotic figures should be enlarged
Therefore, careful examination of the as well
grossly uninvolved renal tissue is impor- e. Anaplasia is significant, as it
tant, since the presence of nephrogenic
rests indicates a greater probability of
predicts resistance to treatment

synchronous or metachronous bilateral- 1. Most important prognostic indicator


E. Clinical Behavior and Prognosis

ity is stage
2. Other prognostically significant
1. Wilms’ tumors are typically composed of features:
D. Microscopic pathology:

3 elements: a. Age <2 years


a. Blastema: consists of sheets of ran- b. Size (weight)
domly arranged, densely packed c. Anaplasia (bad)
small cells with darkly staining d. Extensive tubular
nuclei, frequent mitotic figures, and differentiation (good)
inconspicuous cytoplasm, resem- e. Extensive skeletal muscle
bling other “small blue cell tumors” differentiation (good)
of childhood f. Mucin production (bad)
b. Epithelial component: consists of g. DNA ploidy
small tubules or cysts lined by primi- 3. Most common metastatic sites: peri-
tive columnar or cuboidal cells toneum, liver and lung (metastasis to
• May form structures resembling bone is rare, in contrast to neuroblas-
glomeruli, or may display muci- toma)
nous, squamous, neural, or 4. Now 90% long-term survival using 3
endocrine differentiation modalities (surgery, chemotherapy, radi-
• Predominantly cystic Wilms’ ation therapy)
tumors that contain blastema and 5. Patients with anaplastic features in the
other Wilms’ tumor tissues in the tumors tend to be more resistant to treat-
septa, are designated cystic par- ment: they are often older than 2 years of
tially differentiated nephroblas- age, and typically present with more
toma advanced disease
c. Stromal component: may differen-
tiate along the lines of any type of
soft tissue

194 EDUCATIONAL REVIEW MANUAL IN UROLOGY


4. Propensity to metastasize to bone is
marked: it is at least 10 times more likely
2. Mesoblastic Nephroma

to metastasize to bone than other pedi-


1. Most common renal neoplasm in infants atric renal tumors – in fact, it was origi-
A. General:

<3 months; uncommon after 6 months nally known as the bone metastasizing
2. May be associated with polyhydramnios renal tumor of childhood
and prematurity
3. Usually presents with abdominal mass 1. Variable appearance: homogeneous gray
B. Gross pathology:

and lobular, or variegated with firm gray


1. Typically large with a firm, gray-white, whorled tissue and light pink, soft areas;
B. Gross pathology:

trabeculated (whorled) cut surface 30% have cystic areas


2. Often unencapsulated; may extend into 2. Most appear to be well circumscribed
surrounding tissues 3. Often weighs more than 500 g
4. Almost always unilateral
1. Classical mesoblastic nephroma
C. Microscopic pathology:

a. Moderately cellular proliferation of 1. Monotonous sheet of cells with pale (but


C. Microscopic pathology

thick interlacing bundles of spindle not truly CLEAR) cytoplasm


cells with elongated nuclei that usu- 2. At higher magnification, cells appear to
ally infiltrate renal and perirenal tis- be arranged in cords separated by vascu-
sues lar septa having a “chicken wire” pattern
b. Entrapment of glomeruli and renal 3. Neoplastic cells have pale or vacuolated
tubules is common cytoplasm and indistinct borders
c. Mitotic figures are rare 4. Nuclei contain finely dispersed chro-
(0–1 per 10 hpf) matin and small nucleoli
2. Cellular mesoblastic nephroma: 5. One helpful feature is the infiltrative bor-
a. Densely cellular proliferation of der between the clear cell sarcoma and
polygonal cells with pushing borders the surrounding renal parenchyma
b. Mitotic figures in the range of
8–30 per 10 hpf 1. Death rate has been reduced to 30%–40%
D. Clinical Behavior and Prognosis

c. Cysts are common in this pattern by modern chemotherapy


2. May have late recurrences, so outcome is
1. Good prognosis: surgical resection is ultimately iffy
D. Clinical Behavior and Prognosis:

almost invariably curative; considered


benign in most cases
2. Adverse prognostic indicators:
4. Rhabdoid Tumor of Kidney

a. Age >3 months


b. Incomplete resection 1. The most malignant renal neoplasm of
A. General

c. Cellular histological pattern childhood, this tumor usually metasta-


sizes widely and causes death within 12
months of diagnosis; metastasis to the
brain is common
3. Clear Cell Sarcoma

2. Patients are typically < 1 year, rarely


1. Highly malignant neoplasm comprising older than 3 years of age
A. General

6% of pediatric renal tumors 3. Male > female (1.5: 1)


2. Most diagnosed in patients between 4. May be associated with atypical teratoid
12 and 36 months of age tumors of CNS and paraneoplastic hyper-
3. 66% of patients are male calcemia
5. EM shows characteristic concentric
whorls of microfilaments in the cyto-
plasm

CHAPTER 6: CYTOLOGY OF THE UROGENITAL TRACT 195


Renal Tumors II: Renal Carcinoma

1. No evidence of encapsulation, (in con-


B. Gross pathology: 1. General Features

trast to Wilms’ tumor or clear cell sar- A. Comprises 80%–90% of adult primary renal
coma) tumors, 3% of all adult malignancies
2. Often located medially in the kidney, and 1. Approximately 23,000 new cases/ year in
the renal sinus and pelvis are almost the U.S.
always infiltrated 2. Peak incidence in 6th and 7th decades;
3. Yellow-gray or light tan friable (mushy) rarely seen in children
tumors with hemorrhage, necrosis 3. Male > female (2:1)
4. Treatment: Surgery is the principal ther-
1. Diffuse “sheet-like” growth pattern apy for RCC
C. Microscopic pathology:

2. Neoplastic cells are large and polygonal 5. Clinically occult RCC may present at dis-
with abundant eosinophilic cytoplasm – tant sites with unknown primary, or may
superficially resemble skeletal muscle recur years after an apparently successful
cells, but do not stain for desmin radical nephrectomy
3. Round nuclei with prominent membranes
and large nucleoli
4. Cytoplasm often contains an eosinophilic 1. Originally thought to arise from hetero-
B. Etiology and Pathogenesis

globular inclusion that displaces the topic adrenal rests within the kidney
nucleus (hence: “hypernephroma”)
5. Infiltrative borders 2. Now known to have several distinct cyto-
6. Vascular invasion common genetic abnormalities, many of which
involve loss of the tumor suppressor
Extremely malignant, aggressive behavior: genes at 3p25
D. Clinical behavior and prognosis:

75% of patients die within 12 months of 3. Other etiologic agents/risk factors


diagnosis a. Smoking – major risk factor,
accounting for up to 30% of tumors
b. Obesity – particularly in women;
accounts for approximately 25% of
tumors
c. Long-term phenacetin and
acetaminophen exposure as well as
exposure to other chemicals
(petroleum products, industrial
chemicals, cadmium)
d. In most cases, the carcinogenic
influence is not known

1. “Classic” triad of symptoms: hematuria,


C. Signs and Symptoms

pain, and a flank mass (only seen in 10%


of patients)
2. Other symptoms include:
a. Abdominal pain, weight loss,

b. Fever of unknown origin


anorexia

c. Elevation of the ESR


d. Sudden appearance of a scrotal
mass, usually left-sided – due to
obstruction of testicular vein by
tumor

196 EDUCATIONAL REVIEW MANUAL IN UROLOGY


e. Hypochromic anemia – secondary
to bone marrow suppression, not
3. The Fuhrman Grading System for

hematuria
Renal Cell Carcinoma (Table 2)

f. Liver changes (Stauffer’s A. Used primarily for clear cell and papillary
syndrome) – elevated alkaline phos- renal cell carcinoma; may be applied to
phatase, ALT, AST with hep- unclassified renal cell carcinoma as well
atosplenomegaly, coagulopathy, and
elevated alpha-globulin concentra-
B. The entire tumor receives the grade of its

tions C. Note that mitotic figures are NOT a part of


worst component, regardless of extent

g. Systemic amyloidosis this system


h. Paraneoplastic syndromes: D. 5-year survival rates: 86% for Grade 1; 24%
• Hypercalcemia of malignancy for Grade 4
(pseudohyperparathyroidism) E. Sarcomatoid histology is also significant:
• Erythrocytosis – secondary to patients with even small foci of sarcomatoid
erythropoietin production by carcinoma have a much worse prognosis
tumor than patients without
• Hypertension – secondary to
renin production by the tumor
• Gynecomastia – secondary to
4. Clear Cell Renal Cell Carcinoma

prolactin or gonadotropin produc- A. Most common form of RCC – represents


tion by tumor approximately 70% of RCC tumors
i. RCC is also notorious for presenting
as metastatic tumor with unknown 1. Loss of genetic material (thought to be a
B. Genetics:

primary – and to complicate matters, tumor suppressor gene) on the short arm
is one of the “great mimickers” of chromosome 3 (3p) is the most fre-
quent and consistent abnormality
2. May also see gain of 5q
3. c-MYC activation may also play a role
2. Associations and Syndromes (Table 1)

disease – association with RCC is contro- 1. Classically demonstrates a variegated cut


A. Autosomal dominant polycystic kidney C. Gross pathology:

versial surface with golden-yellow regions alter-


1. Can include any form of renal cell carci- nating with areas of hemorrhage, necro-
noma and may be multifocal tumors sis, and cystic degeneration
B. Acquired renal cystic disease – strong asso- 2. Usually well circumscribed with bulging
ciation with RCC borders
1. Has a characteristic type of carcinoma 3. May have central stellate area of edema-
that is called “acquired cystic disease- tous gray connective tissue
associated renal cell carcinoma” 4. May be multicentric in 14% of cases, and
2. Arises from end-stage kidney disease in bilateral in 1% of cases
the setting of dialysis
3. Microcystic pattern and oxalate crystal 1. Cells have abundant clear cytoplasm
D. Microscopic pathology:

deposition characteristic features (result of artifact in which the cytoplas-


mic lipid/ glycogen is lost during fixa-
1. Distinct lesions associated with these tion, leaving behind clear “holes” where
C. Familial RCC

entities the lipid was)


2. Often younger patients with multifo-
cal/bilateral disease

CHAPTER 6: CYTOLOGY OF THE UROGENITAL TRACT 197


Table 1

Syndrome Gene Protein Renal (and other) Lesions

Von Hippel Lindau VHL (3p25) pVHL Clear cell RCC, CNS and retinal
hemangioblastomas, renal cysts,
pancreatic cysts and neuroendocrine
tumors, pheochromocytoma

Hereditary FH (1q42.3-q43) Fumarate hydrase Papillary RCC, type II, leiomyomas of


leiomyomatosis and the skin, uterine leiomyomas and
renal cell cancer leiomyosarcomas
(HLRCC)

Birt-Hogg- BHD (17p11.2) Folliculin Hybrid oncocytic tumors, multiple types


Dubé (BHD) RCC, fibrofolliculomas, acrochordons,
pneumothorax, colorectal polyps

Hereditary papillary MET (7q31) MET Papillary RCC, type I


renal cell carcinoma
(HPRCC)

Tuberous TSC1 (9q34) Hamartin (TSC1) Clear cell RCC, angiomyolipoma, facial
sclerosis (TS) and and angiofibromas, periungual fibromas,
TSC2 (16p13.3) Tuberin (TSC2) cortical tubers, cardiac rhabdomyomas,
retinal hamartomas

Table 2

Grade Nuclear Size Nuclear Shape Chromatin Nucleoli

1 <10 µm Round Dense Inconspicuous

2 15 µm Round Finely granular Small, not visible with


10X objective

3 20 µm Round/oval Coarsely granular Prominent

4 >20 µm Pleomorphic, multilobated Open, hyperchromatic Macronucleoli

198 EDUCATIONAL REVIEW MANUAL IN UROLOGY


2. Some cells may have granular
eosinophilic (pink) cytoplasm as well – 1. Typically light-tan, well circumscribed
E. Gross pathology:

and these cells may even predominate. 2. Cut surface may have a granular “fri-
But as long as there are clear cells pre- able” appearance, reflecting papillary
sent, the tumor is classified as a clear cell architecture
RCC 3. Dense fibrous pseudocapsule
3. Growth pattern: nests, tubules, cysts.
Often the tubular and cystic patterns 1. Typically have predominantly papillary
F. Microscopic pathology:

demonstrate central hemorrhage – so- or tubulopapillary architecture


called “blood lakes” 2. Papillae consist of delicate fibrovascular
4. Delicate interconnecting vasculature cores covered by a single layer of neo-
(“chicken wire vasculature”) plastic cells
5. Nuclei of the clear cells are central and 3. Papillae may be expanded by numerous
round, ranging from small and hyper- foamy histiocytes
chromatic with indistinct nucleoli to 4. Subdivided into Type I (low nuclear
large and pleomorphic with prominent grade) and Type II (very pink cells with
nucleoli prominent nucleoli); it is thought that
6. Sarcomatoid growth pattern may be Type II has a worse outcome, although
seen in 5% of tumors, and carries a worse this may be generally a reflection of
prognosis Fuhrman grade alone
7. Variants: multilocular cystic renal cell
carcinoma
a. Grossly appears to be a multilocular
6. Chromophobe Renal Cell Carcinoma

cyst A. Comprises approximately 5% of RCC


b. Histologically, the septa demon- tumors
strate focal areas of clear cells, and B. Affects men and women equally
clear cells often line the cysts C. May have a better prognosis than clear cell
c. Better prognosis, as these have a RCC
very low potential for recurrence or
metastasis 1. Loss of multiple chromosomes is
D. Genetics:

common: 1,2,10,13,6,21 and 17


(90% of cases)
2. Loss of 3p (as in clear cell RCC) is
5. Papillary Renal Cell Carcinoma

A. Second most common type of RCC – NOT seen


3. Trisomy/ tetrasomy 17, 17 and loss of
B. 16% 10-year mortality Y (as is seen in chromophil RCC) is
represents 10%–15% of RCC tumors

C. Sarcomatoid variant probably has same poor NOT seen


prognostic implication as it does in clear cell E. Gross pathology: solitary, solid beige or
RCC light brown, circumscribed

1. Typically associated with trisomy (or 1. Usually solid growth pattern, but may
D. Genetics: F. Microscopic Pathology:

tetrasomy) 17 and 7 and loss of Y chro- show some tubules/nests


mosome 2. Cells are large, polygonal and pink
2. Loss of 3p and gain of 5q (as seen in clear 3. Cytoplasm appears to condense at the
cell RCC) are NOT found in chromophil edges, so the cytoplasmic membranes
RCC
4. Nuclei are wrinkled/raisinoid and a
are very thick and distinct

perinuclear halo is common

CHAPTER 6: CYTOLOGY OF THE UROGENITAL TRACT 199


5. Cells may be binucleated
6. Hale’s colloidal iron stain is positive
9. Mucinous Tubular and Spindle

(stains chromophobe cells vivid blue); as


Cell Neoplasm

opposed to a negative reaction in oncocy- A. Uncommon tumor


toma 1. Strong female predominance (M:F 1:4)
2. Favorable outcomes, except in cases
with sarcomatoid change
3. Most discovered incidentally
7. Collecting Duct Carcinoma
(Bellini Duct Carcinoma)

A. Represents about 1% of renal tumors 1. Well-circumscribed, homogeneous


B. Gross pathology:

1. Often aggressive course with bony gray-tan cut surface


metastasis 2. Minimal hemorrhage or necrosis
3. Cut surface may be shiny and mucoid
1. Tumor often arises in the medullary
B. Gross pathology:

region with extension into the cortex or 1. Elongated cords and tubules of cells
C. Microscopic pathology:

hilar tissues 2. Background of pale mucin


2. Gray-white cut surfaces with central
necrosis and infiltrative borders 10. Translocation Carcinoma

1. Consists of irregular duct-like structures, A. Commonly affects young adults, although


C. Microscopic pathology:

nests and cords of cells in an abundant, broad age range


loose, slightly basophilic stroma 1. Caused by translocations of Xp11 that
2. Nuclei are pleomorphic with thick result in gene fusions involving the TFE3
nuclear membranes gene and by translocation
3. “Hobnail” (tombstone) appearance of t(6:11)(p11.2;q12) that involves the
the cells lining the duct lumens is a TFEB gene
helpful feature 2. Gene fusion results in protein overex-
pression
8. Medullary Carcinoma 3. Can present with high-stage disease

A. Rare tumor, associated with sickle cell trait 1. Somewhat specific for translation
B. Gross pathology:

1. Affects teenage to young adults subtype


2. Poor prognosis
1. Has a variety of appearances, including
C. Microscopic pathology:

1. Irregular large tumor arising from the papillary architecture lined by clear cells
B. Gross pathology:

central portion of the kidney or nested, pink appearance


2. Hemorrhage and necrosis common 2. Immunostains and FISH for the TFE
translocation is needed for the diagnosis
1. Complex tubular/cribiform structures
C. Microscopic pathology:

and sheets of cells


2. Variable compositions of pleomorphic
cells
3. Open vesicular nuclei with prominent
nucleoli

200 EDUCATIONAL REVIEW MANUAL IN UROLOGY


Renal Tumors III:
Miscellaneous Renal Lesions

6) Gross vascular invasion


7. Ultrastructural (EM) features:
1. Epithelial Tumors

a. Cytoplasm is filled with mitochon-


1. Originates from the renal cortex
A. Oncocytoma

2. Generally discovered incidentally during b. Chromophobe RCC has cytoplasm


dria

radiologic examination of the kidneys for filled with round/ oval vesicles by
other reasons EM
3. At one time, oncocytomas were thought c. Clear cell RCC has cytoplasm filled
to be “granular cell” RCC but these with vacuoles and lots of rough ER
tumors have distinctive properties that by EM
set them apart from RCCs:
a. Staining characteristics 1. Usually small lesions discovered inci-
B. Metanephric adenoma

b. Benign course dentally, solitary in most cases


c. Males > females (2:1) 2. Appear grey-tan on gross evaluation
d. Age > 50 years 3. Microscopically, have small
4. Classically associated with a “spoke- tubules/acini that are highly cellular and
and-wheel” appearance on radiographs appear blue due to a low amount of cyto-
5. Gross pathology: plasm
a. Classically described as mahogany 4. Nuclei are bland and small
5. Generally considered benign lesions
(contrasting with bright golden-yel-
brown with a central stellate scar

low appearance of RCC)


b. Hemorrhage and necrosis are rare
2. Mesenchymal Tumors

(as opposed to RCC) – in fact, the


presence of gross necrosis or hemor- 1. Generally benign tumor composed of fat,
A. Angiomyolipoma

rhage militates against a diagnosis of muscle and thick-walled blood vessels


oncocytoma 2. Probably more of a hamartoma than a
c. Bilaterality/ multicentricity noted in true neoplasm
3.6% of cases 3. Occurs in two distinct settings:
d. Rare: oncocytomatosis – numerous a. Those associated with tuberous
tumors of cortex
6. Microscopic pathology: 1) Approximately 50% of patients
sclerosis

a. Cells are arranged in diffuse sheets with TS develop these tumors


or as cellular “islands” in a back- 2) Tend to be asymptomatic, multi-
ground of loose edematous connec- ple, bilateral, and small
tive tissue (“archipelaginous archi- 3) Strong female predominance
b. Those not associated with tuberous
b. Cytoplasm is intensely sclerosis (sporadic)
tecture”)

eosinophilic and finely granular 1) Tumors tend to be large,


c. Nuclei are characteristically round symptomatic, and solitary
4. Gross pathology:
d. Mitotic figures are absent or rare a. Range in size from 1–20 cm
with inconspicuous nucleoli

e. Features impermissible in a renal (average: 9 cm)


b. Typically golden-yellow, but color
1) Mitotic figures depends on proportion of fat/muscle/
oncocytoma

2) Papillary architecture vessel


3) Clear or spindle cells c. Well-demarcated but not encapsu-
4) Positive colloidal iron stain lated; may show local invasion
5) Chromophobe-type vesicles d. Cut surface may resemble a lipoma
seen by electron microscopy

CHAPTER 6: CYTOLOGY OF THE UROGENITAL TRACT 201


5. Microscopic pathology: d. Renal and perirenal infiltration are
a. Contains varying amounts of common
4. Microscopic pathology:
a. Spindle cell neoplasm with
smooth muscle, fat, and thick-

b. Frequent finding is radial arrays of fascicular growth


walled vessels

b. Necrosis, nuclear pleomorphism,


c. Blood vessels are abnormal, with
smooth muscle around the vessels

thick walls like arteries, but eccen-


and mitoses are common

tric small lumens. Lack elastic tissue


so rupture easily
d. Nuclear pleomorphism may be
pronounced but has no prognostic
significance
e. Mitotic figures may be present –

f. May see local invasion and vascular


no prognostic significance

invasion of the vena cava – no prog-

g. The smooth muscle cells react with


nostic significance

HMB-45 (typically a melanoma


marker), which helps distinguish
this neoplasm from other “spindle
cell” neoplasms of the kidney
h. Epithelioid features are present
when the cells show abundant
cytoplasm and limited spindled
appearance
6. Clinical behavior and treatment:
a. Hemorrhage is the most common
serious complication (more likely if
size > 4 cm)
b. Epithelioid variants may demon-
strate a more aggressive course, with
metastases reported in many
instances; in contrast, classic
angiomyolipomas tend to follow a
benign course

1. Most common primary renal sarcoma,


B. Leiomyosarcoma

but not nearly as common as sarcomatoid


RCC
2. Usually in patients > 40 years; women >
men (2:1)
3. Gross pathology:
a. Firm solid tumor with well-circum-
scribed margins
b. Whorled cut surface but may have

c. May arise from renal capsule or


hemorrhage and necrosis

renal vein

202 EDUCATIONAL REVIEW MANUAL IN UROLOGY


Renal Cystic Diseases

reniform kidneys with normal


ureters and pelves
1. Renal Dysplasia

A. A dysplastic kidney is a kidney with aber- • Biopsy shows a focal form of


rant differentiation of the nephron. It is dysplasia characterized by an
not: admixture of normal nephrons
1. Pre-malignant (i.e., “dysplasia” does not and aberrantly formed nephrons
suggest “neoplasia”) usually with microcysts and car-
2. A hypoplastic kidney (which is small but tilage or dysplastic ducts
normally developed) • The renal prognosis is bleak, and
3. A polycystic kidney (which are cystic, the infant usually develops pro-
but don’t contain dysplastic elements) gressive renal failures as it grows
b. Segmental Dysplasia
1. Urinary tract obstruction in utero—may • Occurs only in kidneys with
B. Two theories of dysplasia

or may not play a role duplication of the collecting sys-


2. Fundamental defect in the inducer tissue tem (duplex kidney)
(ureteric bud) or responding tissue • The upper pole is affected, and
(metanephric blastema): in other words, histologically shows the same
range of aberrant nephrogenesis
branching as it normally should, or the encountered in aplastic and mul-
the ureteric bud is inhibited from

ticystic dysplasia
to the organizing influence of the ureteric c. Dysplasia associated with lower
metanephric blastema fails to respond

bud tract obstruction


• Bilateral renal dysplasia may be
1. Clinical presentation: associated with urethral steno-
C. General features:

a. Multicystic dysplasia is the most sis, posterior urethral valves, or


common cause of a renal mass in a
child • Gross appearance: kidneys are
bladder neck obstruction

b. Usually unilateral, but bilateral dis- typically reniform, may be large


ease can occur (Potter’s syndrome), or small, but show distinct corti-
in which case death generally occurs comedullary differentiation
in infancy due to pulmonary • The bladder is either hyper-
hypoplasia trophic or greatly dilated, and the
2. Gross appearance: highly variable ureters are dilated and tortuous
appearance, ranging from the large mul- • There may be a severe degree of
ticystic kidney to the small aplastic kid- dysplasia with scant nephronic
ney elements or only a peripheral
3. Histological appearance: also very zone of dysplastic elements with
variable, with 2 characteristic elements: normal deeper nephrons
a. Primitive or dysplastic ducts -- lined d. Hereditary renal adysplasia and uro-
by columnar epithelium and sur- genital adysplasia
rounded by a collar of spindle cells • Renal agenesis, aplastic dyspla-
b. Fetal or immature appearing carti- sia, and multicystic dysplasia are
lage -- thought to be derived from the most severe forms of
metanephric kidney maldevelop-
c. Renal cortex is usually thinned; may ment, and are usually sporadic
the metanephric blastema

contain immature nephrons unless associated with multiple


4. Variations on the theme: malformation syndromes
a. Focal Dysplasia • Rarely may be hereditary in an
• Tend to present in infancy with autosomal dominant fashion
renal insufficiency and small (hereditary renal adysplasia)

CHAPTER 6: CYTOLOGY OF THE UROGENITAL TRACT 203


2. Clinical features:
a. Most present in 4th to 6th decade
2. Polycystic Kidney Diseases

with 100% penetrance at the age of


1. Clinical features: 80
A. Infantile Polycystic Kidney Disease

a. Autosomal recessive (Potter type I) b. 25% of patients lack a family history


b. 4 types: perinatal, neonatal, infan- c. Chronic flank pain is the most com-
tile, and juvenile mon complaint followed by hema-
c. Most cases result in stillbirth or turia and acute flank pain (due to
hemorrhage into a cyst)
d. Affected neonates have massively 3. Complications:
early neonatal death

enlarged, diffusely cystic kidneys a. Hypertension from compression of


that produce abdominal distension the intrarenal vasculature by expand-
and compress thoracic organs; death ing cysts
usually ensues due to pulmonary b. Perinephric abscess (60% mortality
hypoplasia rate) following UTI
e. If the kidneys are non-functional, c. Berry aneurysms of the Circle of
Potter’s phenotype may develop Willis (5%–15% of cases)
f. May be accompanied by a liver d. Kidney stones (10% of cases)
lesion that looks like congenital hep- e. Renal cell carcinomas
atic fibrosis (1%–5% of cases)
2. Gross: cysts extend through the renal 4. Gross features: cysts progressively
enlarge in size and number, resulting in
pendicular to the cortex imparting a massive renal enlargement, although
parenchyma in “rays” oriented per-

spongy quality the normal reniform shape is preserved


3. Histology: 5. Histologic features: fairly nondescript
a. Cysts consist of dilated collecting a. Most of the cysts are lined with a
ducts lined with uniform cuboidal single layer of flattened to cuboidal
cells epithelium
b. Nephrons between the collecting b. Remaining parenchyma may show
ducts appear normal interstitial inflammation, glomeru-
c. In older patients, cysts may be less losclerosis
prominent with more interstitial
scarring and glomerulosclerosis,
resulting in a superficial resem-
3. Cysts (without dysplasia)

blance to APKD
in Hereditary Syndromes

1. General:
B. Adult Polycystic Kidney Disease A. Medullary cystic disease/ Juvenile

a. Autosomal dominant 1. Hereditary tubulointerstitial disease that


nephronophthisis

is usually classified as a cystic disease


b. Most common cystic renal disease, because 75% of cases are accompanied
(Potter type II)

and 3rd or 4th leading cause of renal by medullary cysts


failure 2. Clinical features: affected individuals
c. Most common genetically transmit- present with
ted disease (1: 500–1,000): the prin- a. Polyuria and polydipsia due to salt
cipal gene has been localized to wasting
chromosome 16 (90% of cases); a b. Anemia disproportionately severe
gene on chromosome 4 has also for the level of renal insufficiency
recently been implicated c. Growth retardation

204 EDUCATIONAL REVIEW MANUAL IN UROLOGY


3. Four types have been described: e. Renal angiomyolipomas
a. Familial juvenile nephronophthi- f. Renal cysts -- actually uncommon,
sis—autosomal recessive; presents but can (particularly in children)
in infancy/ childhood become quite large, mimicking
b. Renal-retinal disease—autosomal APKD
recessive; presents in adulthood 3. Microscopic features: the cysts are lined
c. Medullary cystic disease—autoso- by large eosinophilic cells with large
mal dominant (15% of cases); pre- hyperchromatic nuclei
sents in adulthood 4. Complications: renal cell carcinoma may
d. Sporadic medullary cystic develop, but far less likely than in Von
disease—nonfamilial (15% of Hippel-Lindau disease
cases)
4. Gross appearance:
a. The kidneys are usually small due to
4. Miscellaneous Conditions

b. Cysts measure up to 1 cm and con- 1. Development of multiple and bilateral


cortical atrophy A. Acquired cystic kidney disease

gregate at the corticomedullary junc- renal cysts in patients whose chronic


tion renal failure cannot be attributed to a
5. Microscopic features: hereditary cystic disease
a. Nonspecific: chronic inflammation, 2. Clinical features:
tubular atrophy, interstitial fibrosis a. Bilateral and asymptomatic in its
b. Prominent lymphoid infiltrate is early stages
usually present b. Cysts are present in 8% of patients at
c. Cyst formation seems to remain the time dialysis is started and
fairly stable, but the interstitial increase in incidence, number, and
inflammation progresses, resulting size proportional to the duration of
in glomerulosclerosis and renal fail- dialysis
ure 3. Gross features:
a. The cysts initially form in the proxi-
1. Autosomal dominant disorder with: mal tubules of end-stage kidneys
B. Von Hippel-Lindau Disease

a. Retinal/ cerebellar/ spinal b. Most cysts are less than 0.5 cm but
may enlarge to 2–3 cm in diameter
b. Pheochromocytoma c. Initially the cysts are cortical, but in
hemangioblastomas

c. Epididymal and pancreatic advanced cases medullary cysts


form and the entire kidney may be
d. Multiple and bilateral renal cysts replaced by cysts and resemble a
cystadenomas

(70% of patients) smaller version of APKD


e. Renal cell carcinoma, often bilat- 4. Histology: the cysts are lined with flat-
eral and multiple (50% of patients) tened, cuboidal, or columnar epithelium
2. Microscopic features: renal cysts are and may contain a proteinaceous to hem-
lined by single or multiple layers of clear orrhagic fluid
cells, mimicking low-grade renal cell 5. Complications: intrarenal and retroperi-
carcinoma toneal hemorrhage, cyst infection, and
renal cell carcinoma
1. Autosomal dominant disorder
C. Tuberous Sclerosis

characterized by: 1. Characterized by cysts in the papillary


B. Medullary sponge kidneys

a. Mental retardation collecting ducts of one or more renal


b. Epilepsy pyramids
c. Angiofibromas
d. Cardiac rhabdomyomas

CHAPTER 6: CYTOLOGY OF THE UROGENITAL TRACT 205


2. Clinical features: 5. Gross features:
a. Usually bilateral and more common a. Typically large, well-circumscribed
in males mass that protrudes into the pelvis
b. Usually detected radiographically in b. Fibrous pseudocapsule and delicate
adults being evaluated for fibrous trabeculae
c. No calcification, hemorrhage, or
c. Patients tend to have normal renal necrosis
nephrolithiasis

6. Histologic features: locules separated by


3. Gross: kidneys are not enlarged a variably cellular connective tissue and
function

4. Histology: lined by cuboidal or flattened epithelium


a. Papillary collecting ducts are
dilated and lined with
cuboidal/flattened epithelium
b. Can be distinguished from
medullary cystic disease and IPKD
by patient age and by cyst locations:
in medullary cystic disease, the cysts
are located at the corticomedullary
junction, while in the juvenile form
of IPKD the cysts are in the cortex
and medulla and do not congregate
at the papilla

1. Most common cystic renal lesion


C. Simple cortical cysts

2. Rare before the age of 40, and increase in


frequency with increasing age
3. Gross: in older patients, may be multiple
and large
4. Histology: lined with a flattened layer of
cells, or may lack an epithelial lining

1. Uncommon nonfamilial lesion felt to be


D. Multilocular cyst

a benign neoplasm
2. Peak presentation is biphasic: seen in
children (often females) < 2 years of age
and in patients 40–70 years of age
3. Radiology: avascular mass causing
calyceal distortion
4. Diagnostic criteria:
a. Unilateral, solitary, and multilocular
b. Cyst does not communicate with

c. Septations consist of fibrous tissue


the renal pelvis or between locules

only (no parenchyma)


d. Residual renal tissue is normal

206 EDUCATIONAL REVIEW MANUAL IN UROLOGY


Adrenal Gland

1. Defined as adrenal gland weight > 6g


1. Normal Histology B. Acquired Adrenal Hyperplasia

A. The adrenal gland is composed of 2 2. Always bilateral


endocrine organs: the cortex (derived from 3. Microscopically: increased thickness of
mesoderm) and the medulla (derived from zona reticularis and fasciculata with lipid
neuroectoderm) depletion of the fasciculata cells
B. Cortex is divided into 3 layers, all of which 4. May have nodular or diffuse pattern
respond to ACTH stimulation: a. Diffuse pattern
1. Zona Glomerulosa (outer layer) – • ACTH-dependent (“pituitary
aldosterone production dependent”) because typically
2. Zona Fasciculata (middle layer) – results from excessive ACTH
glucocorticoid production production by pituitary or other
3. Zona Reticularis (inner layer) – neoplasm
sex hormones b. Nodular pattern
C. Medulla contains 2 cell types • ACTH-independent
1. Pheochromocytes
a. Contain epinephrine and • May represent progression of
(“adrenal dependent”)

norepinephrine diffuse pattern


b. Large polygonal cells with poorly • Includes primary pigmented
defined cell borders
c. Abundant cytoplasm is usually plasia, as seen with Carney’s syn-
nodular adrenal cortical hyper-

granular and faintly blue drome


2. Sustentacular cells -- “supporting”
cells that surround the nests of 1. Often incidental findings; can be found in
C. Adrenocortical Adenoma

pheochromocytes any age group


2. Gross findings:
a. Usually solitary lesions,
well-encapsulated
2. Lesions of the Adrenal Cortex

b. Cut surface is solid,


1. Autosomal recessive defect in the homogeneous yellow
A. Congenital Adrenal Hyperplasia

biosynthesis of cortisol c. By definition, generally <5 cm


2. Responsible for the large majority of or <50 g
cases of adrenogenital syndrome devel- 3. Microscopic findings:
oping in the first year of life (although a. Recapitulate the zona glomerulosa
may occasionally have initial presenta- and zona fasciculata (pink to clear
tion in adulthood) cells with foamy cytoplasm)
3. Various enzyme deficiencies may result b. Bizarre nuclei may be seen, but
in CAH, but most common ones are: mitoses are exceptionally rare or
a. 21-hydroxylase deficiency absent
(>90% of cases): virilization c. May contain foci of myelolipoma
and salt wasting (see below)
b. 11-beta-hydroxylase deficiency 4. Variations on the theme:
(5% of cases): virilization and a. “Aldosterone-omas” --
hypertension golden-yellow color; contain
4. Pathologic change is the same, regard- “spironolactone bodies”
less of the defect, and consists of diffuse
cortical hyperplasia, especially of zona
reticularis

CHAPTER 6: CYTOLOGY OF THE UROGENITAL TRACT 207


b. “Black adenomas” -- dark brown
due to the presence of lipofuscin
3. Lesions of the Adrenal Medulla

pigment; generally nonfunctioning,


but may occasionally be associated 1. Clinical characteristics:
A. Neuroblastoma

with primary aldosteronism or a. Patients are almost always < 4 years


Cushing’s syndrome of age (median age 21 months) who
c. Adrenocortical oncocytomas: present with abdominal mass
highly eosinophilic cytoplasm due to b. May be associated with Beckwith-
numerous mitochondria Wiedemann syndrome, neurofibro-
matosis, or as complication of fetal
1. Clinical characteristics: hydantoin syndrome
D. Adrenocortical Carcinoma

a. Males and females affected equally c. 70% occur in the retroperitoneum,


b. Average age is 50 years the majority of which involve the
c. 50% accompanied by hormonal adrenal gland
manifestations 2. Gross findings:
2. Gross findings: a. Large, soft, gray, relatively
a. Weight usually >100g well-circumscribed
b. Cut surface frequently demonstrates b. Hemorrhage, necrosis, and
necrosis and hemorrhage calcifications are common
3. Histologic findings: 3. Microscopic findings:
a. Giant cells with abundant cytoplasm a. “Small round blue cell tumor” with
and bizarre nuclei may be present very little evidence of differentiation
b. Potential criteria of malignancy: b. Classically characteristic feature is
• Nuclear grade III or IV the presence of Homer-Wright
• Mitoses > 5/ 50 HPF rosettes: collections of tumor cells
• Atypical mitoses arranged around a central area filled
• Paucity or absence of clear cells with pink fibrillary material (which
• Diffuse architecture represents tangled neuritis). There is
• Necrosis no central lumen
• Capsular or vascular invasion 4. Immunohistochemistry: positive for
• Spindled tumor cells neuroendocrine markers like NSE, neu-
c. However, nothing short of detect- rofilament, chromogranin, synapto-

5. Clinical staging of neuroblastoma:


ing metastases firmly discrimi- physin

4. Immunohistochemical profile: Stage I: Tumor confined to the organ or


nates benign from malignant

a. Often positive for calretinin and structure of origin


inhibin, frequently keratin negative Stage II: Tumors extending in continuity
5. Generally demonstrates hematogenous beyond the organ or structure of origin,
spread to retroperitoneum, kidney, liver but not crossing the midline; regional
(60%), lymph nodes (40%), peritoneum, lymph nodes in the ipsilateral side may
bone, etc. be involved
6. Prognosis: 50% 2-year survival rate; Stage III: Tumors extending in continu-
20%–35% 5-year-survival rate ity beyond the midline; regional lymph
nodes may be involved bilaterally
Stage IV: Remote disease involving the
skeleton, organs, soft tissues, distant
lymph node groups, or other structures

208 EDUCATIONAL REVIEW MANUAL IN UROLOGY


Stage IV-S: Patients who would other- b. Neuropil (pink fibrillary matrix) is
wise be stage I or II but who have remote much more apparent
disease confined to one or more of the c. Two microscopic varieties:
following sites: liver, skin, or bone mar- • Imperfect (better prognosis) and
row (but no bony metastases by radio- composite (worse prognosis)
graphic skeletal survey)
6. Prognostic factors in neuroblastoma: 1. Clinical characteristics:
C. Ganglioneuroma

a. Shimada classification looks at the a. Invariably benign


quality of the stroma, the degree of b. Only rarely in adrenal, usually
differentiation within the neurob- in retroperitoneum or posterior
lasts, the age of the patient, and the
MKI (mitotic-karyorrhectic index) c. Almost always associated with
mediastinum

to come up with “favorable” and


“unfavorable” histology subgroups 2. Gross: large encapsulated masses of
catecholamine synthesis

b. N-myc amplification, loss of chro- gray-white firm tissue


mosome 1p, and extra copies of 17q 3. Microscopic findings: looks like neu-
are significantly associated with rofibroma (fibrous tissue and schwann
unfavorable outcome cells), but has abnormal but mature
7. Clinical course:
a. May occasionally undergo matura-
ganglion cells (multinucleated)

tion and spontaneous regression 1. Clinical characteristics:


D. Adrenal Medullary Hyperplasia

b. May locally invade surrounding a. Can cause symptoms akin to


tissues: intraspinal extension and pheochromocytoma and can actually
kidney give rise to benign/ malignant
c. May metastasize, most commonly pheochromocytoma
to: b. May be associated with von Reck-
• Liver (Pepper’s syndrome) linghausen’s disease, MEN-IIb
• Skeletal system, especially skull (Gorlin’s syndrome) and somato-
and orbit (Hutchison’s statin-rich duodenal carcinoid
2. Gross findings: always bilateral
• Lymph node 3. Microscopic findings: nodular or
syndrome)

• Ovary diffuse pattern


8. Treatment: surgical excision,
multi-agent chemotherapy, bone 1. Clinical characteristics:
E. Pheochromocytoma

marrow transplant a. Classic triad of sweating attacks,


9. Prognosis: 30% 3-year survival headaches, and tachycardia
b. Hypertension may be intermittent
or sustained
B. Ganglioneuroblastoma

1. Clinical characteristics: found most c. Increased urine vanillylmandelic


(malignant ganglioneuroma)

commonly in young children but may acid (VMA) and metanephrines in


present in early adulthood; usually occur up to 90% of cases
in retroperitoneum d. “10% tumor”: 10% bilateral, 10%
2. Gross findings: more homogeneous with in children, 10% extraadrenal, 10%
firmer consistency than neuroblastoma malignant
3. Microscopic findings: e. May secrete:
a. Intermediate degree of differentia- • Epinephrine
tion between neuroblastoma and
ganglioneuromas

CHAPTER 6: CYTOLOGY OF THE UROGENITAL TRACT 209


• Norepinephrine (if extraadrenal,
only secrete norepinephrine since
4. Miscellaneous Adrenal Lesions

adrenal cortex is necessary for


methylation of norepinephrine to 1. Especially common in infants during the
A. Adrenal Hemorrhage

epinephrine) first few weeks of life


• ACTH (occasionally), which can 2. May be caused by neuroblastoma,
result in Cushing’s syndrome hypoxia, septicemia, decreased platelets,
• Parathormone (occasionally), DIC, etc.
which can result in hypercal- 3. Waterhouse-Friedrichsen syndrome:
cemia acute bilateral hemorrhagic infarcts of
f. Extraadrenal sites: retroperitoneum, the adrenal cortex, most commonly sec-
mediastinum, bladder ondary to meningococcal or pseu-
g. May be associated with neurofibro- domonas septicemia (DIC-related)
matosis, renal artery stenosis, other
paragangliomas, adrenal cortical 1. Usually detected in infancy – part of
B. Adrenal Cytomegaly

tumors, or Von Hippel-Lindau dis- Beckwith-Wiedemann syndrome


ease 2. Consists of giant cells with large
h. Familial cases may be seen hyperchromatic nuclei containing
as a component of MEN IIa pseudoinclusions
(Sipple’s Syndrome) or MEN
IIb (Gorlin’s syndrome) 1. One of the classic causes of Addison’s
C. Tuberculosis

2. Gross findings: encapsulated, yellow- disease


white to red-brown, soft 2. Glands are enlarged, calcified, and oblit-
3. Microscopic findings: erated by granulomatous inflammation
a. Tumor cells are arranged in
well-defined nests (“zellballen”) 1. Usually incidental findings in obese
D. Myelolipoma

bound by delicate spindled S-100 adults with otherwise normally function-


sustentacular cells ing bone marrow. Typically hormonally
b. Cells vary in size and shape consid- inactive
erably and have finely granular cyto- 2. Consist of mature adipose tissue with
plasm active bone marrow elements
c. Huge/ hyperchromatic nuclei do
not indicate malignancy; in fact,
the only reliable evidence of

4. Hematogenous spread is common,


malignancy is metastases

usually to bone (ribs, spine)


5. Treatment: surgical excision
6. Prognosis: excellent (if benign);
otherwise die within a year (if malignant)

210 EDUCATIONAL REVIEW MANUAL IN UROLOGY


Penis and Scrotum I:
Non-Neoplastic Lesions

c. Gross: Presents as a single large (>2


cm) bright red moist patch on the
1. Normal Anatomy and Histology

A. Penis: composed of paired corpora caver- glans or inner prepuce. Biopsy is


nosa and a single corpus spongiosum mandatory because the clinical
1. The 2 cavernous bodies lie on the dor- appearance mimics erythroplasia of
sum of the penis and are surrounded by a Queyrat/ Bowen’s disease as well as
double layer of dense fibrous connective candidal balanitis
tissue called Buck’s fascia (tunica albug- e. Histology:
inea) • Distinct upper dermal band-like
2. The corpus spongiosum lies in the ven- infiltrate containing plasma
tral aspect of the penis and surrounds the
urethra in its center • Dermis also contains a lot of
cells

3. The glans represents the distal expansion dilated capillaries


of the corpus spongiosum • Overlying epidermis may be thin
or partially absent
1. Nonkeratinizing squamous epithelium • Keratinocytes appear “diamond-
B. Foreskin:

in uncircumcised males; keratinizing like” or rhomboid-shaped and are


squamous epithelium in circumcised separated by clear spaces (intra-
males. Composed of 3 layers: cellular edema)
a. Epidermis 3. Balanitis Xerotica Obliterans (BXO):
b. Dermis a. Male equivalent to lichen sclerosus
c. Dartos et atrophicus in females
2. Dermis contains sweat glands and b. Usually affects older men
sebaceous glands but no hair follicles c. Gross:
• Well-defined white patch on the
1. Composed of 3 major layers: glans penis or prepuce that
C. Scrotum:

a. Skin envelopes or involves the ure-


b. Dartos muscle thral meatus
c. Fascia (external spermatic, • Lesion may be firm in long-
cremasteric, and internal spermatic) standing cases, due to underlying
2. Dermis contains hair follicles as well as fibrosis
apocrine, eccrine and sebaceous glands d. Histology: 3 layers
3. Left side is usually lower due to the • Epidermis, which is atrophic
longer length of the left spermatic cord and hyperkeratotic
• Beneath the thinned epidermis is
a homogenized collagen layer
• Beneath the collagen layer is a
2. Non-neoplastic Diseases of the Penis

1. Phimosis may demonstrate chronic


A. Inflammatory processes band-like lymphoplasmacytic

inflammation, fibrosis, edema, vascular e. Etiology: usually idiopathic but may


infiltrate

congestion; or may be histologically be associated with carcinoma


normal f. Treatment: laser therapy, circumci-
2. Plasma cell balanitis (Zoon’s balanitis sion, steroids, antifungal agents, and
or balanitis circumscripta plasmacellu- retinoids
laris) 4. Reiter’s syndrome:
a. Benign disorder, etiology unknown a. Characterized by the triad of arthri-
b. Affects primarily uncircumcised tis, urethritis and conjunctivitis
b. Most patients also have mucocuta-
neous lesions as well
males

CHAPTER 6: CYTOLOGY OF THE UROGENITAL TRACT 211


c. Clinical: • Present with painful erection
• Male patients in 3rd or
4th decade
accompanied by distortion,

• Endemic form: sexually


bending or constriction of the

transmitted – follows urethritis • Time course varies from a few


erect penis

• Epidemic form: secondary to months to a few years


enteric infections (may also • Usually have palpable plaques
have urethritis) or nodules on the dorsal surface
d. Causative agents include: of the erect penis; examination
• Chlamydia trachomatis (most of the flaccid penis may be unre-
common agent in sexually trans- markable
mitted cases) c. Histology:
• Ureaplasma urealyticum • Fibrosis of the tunica albuginea
• Shigella flexneri without involvement of the cor-
• Salmonella species pora cavernosa; fibrosis may
• Campylobacter species have a “proliferative” look simi-
• Yersinia enterocolitica lar to fibromatosis
• Neisseria gonorrhea • Calcification and ossification
e. Genetic susceptibility: 60%–80% of (os penis) may occur, especially
patients are HLA-B27 positive in older men
f. Genital involvement: • Early lesions demonstrate
• Balanitis circinata perivascular inflammation in the
* More common form (85%) loose connective tissue between
* Painless lesion beginning as the tunica albuginea and the sinu-
small red papules that soids of the corpora cavernosa
enlarge centrifugally to form d. Clinical course and treatment:
a circular or ring-like con- • Disease resolves spontaneously
figuration in <1/3 of patients
• Disease progression occurs in up
* Predominantly cutaneous to 40% of patients
• Keratodermia blennorrhagica

form affecting the palms and • The disease remains stable in


soles the remainder
* Characterized by erythema- • Treatment includes surgical exci-
tous macules that enlarge to sion of the plaques, radiotherapy,
form hyperkeratotic papules and steroid injections
with red halos
* Clinically and histologically 1. Gonorrhea:
B. Infections

similar to psoriasis a. Agent: Neisseria gonorrhea


g. Histology:
• Epidermis becomes thickened
(Gram-negative “biscuit-shaped”

with broad rete pegs b. Gonorrhea means “flow of semen”


diplococci)

• Hyperkeratosis, parakeratosis referring to the exudate of gonor-


• Neutrophilic aggregates in the rheal urethritis
uppermost layer of the epidermis c. Clinical: profuse purulent or scant
5. Peyronie’s disease: “gleet” urethral discharge with burn-
a. Clinical features ing micturition
• Affects men between the ages of d. Transmission: sexual
20 and 80 years (mean = 53 e. Chief complication:
years) urethral stricture

212 EDUCATIONAL REVIEW MANUAL IN UROLOGY


f. Mimicked by—and often accompa- • Tertiary stage: granulomatous
nied by—chlamydial infection so inflammation (gummas) with
need lab tests to distinguish “hard” granulomas that eventu-
g. Treatment: antibiotics ally demonstrate central caseous
2. Syphilis: necrosis
a. Agent: Treponema pallidum e. Therapy: dependent on stage; con-
b. Gram-negative spirochete sists of antibiotic treatment
c. 3 clinical stages: 3. Herpes Simplex:
• Primary: tiny papule on the a. Agent: Herpes simplex virus,
penis, usually at site of trauma
that progresses to the ulcerated b. Clinical: pain, itching, urethral dis-
type 2

chancre, a round painless ulcer charge, dysuria, and tender lym-


with sharp margins and a clean phadenopathy
indurated base. With or without c. Histology:
therapy, the primary chancre • Vesicles with adjacent multinu-
heals within 6–8 weeks
• Secondary: the spirochetal
cleated giant cells containing

organisms proliferate in the lym-


characteristic intranuclear

phatic system and blood for 6 d. Treatment: acyclovir and phospho-


inclusions

weeks to 6 months, producing noformate trisodi decrease duration


symmetric skin lesions and gen- and intensity of primary and recur-
eralized lymphadenopathy rent episodes, but are not curative
• Tertiary: characterized by granu- 4. Candidiasis:
lomas (“gummas”); may also a. Agent: Candida albicans
demonstrate CNS involvement in b. Causes balanitis or balanoposthitis,
this stage characterized by bright red patches
d. Histologic findings: of inflammation with numerous
• Primary stage: minute pustules and erosions
* Spirochetal organisms can c. Most commonly found in
be identified in scrapings
from the chancre (usually d. Predisposing conditions: diabetes
uncircumcised males

seen best using dark-field mellitus, antibiotic treatment, and


microscopy) immunosuppression
* Histologically, the charac- e. Diagnosis confirmed by KOH prep
teristic change is a dense f. Treatment: antifungal agents
lymphoplasmacytic infil- 5. Molluscum contagiosum:
trate around vessels (espe- a. Agent: Molluscum contagiosum
cially plasma cells) virus (a pox virus)
* Antibody test: negative b. Incubation: 2–7 weeks
• Secondary stage: organisms c. Clinical: multiple discrete dome-
may or may not be visualized by shaped 3–6 mm papules with central
special stains (Warthin-Starry umbilications through which milky-
stain) on tissue, or by dark-field white fluid may be expressed under
microscopy (scrapings) pressure
* Antibody test: positive d. Affects all age groups
e. Histology:
• Cup-shaped invagination of
acantholytic epidermis into the
dermis

CHAPTER 6: CYTOLOGY OF THE UROGENITAL TRACT 213


• Cytoplasmic inclusions (“mol- 4. Histology: lesions are located in the der-
luscum bodies” or “Henderson- mis and are characterized by globules of
Patterson bodies”) are identifi- cyanophilic (bluish) calcific material
able within the stratum with or without accompanying granulo-
malpighii. The inclusions pro- matous inflammation
gressively enlarge as cells reach 5. Treatment: surgery if symptomatic
the surface, and also change from
being eosinophilic to being 1. Common in the scrotum
C. Epidermal cyst:

basophilic and granular as they 2. Single or multiple rubbery nodules filled


enlarge and displace the nucleus with gray-white cheesy material
g. Treatment: podophyllin or silver 3. Histologically, they are lined by
nitrate or laser vaporization keratinizing squamous epithelium

3. Non-neoplastic Disease of the Scrotum

1. Idiopathic form of necrotizing fasciitis of


A. Fournier’s gangrene:

the subcutaneous tissue and skeletal


muscle of the genitals and perineum,
especially the scrotum
2. Lesions begin as reddish plaques with
necrosis and are accompanied by severe
systemic symptoms including pain and
fever
3. Causative agents are most commonly

species, which may or may not be mixed


staphylococcal or streptococcal

with Gram-negative bacilli and anaero-


bic bacteria
4. Predisposing factors include: diabetes,
immunosuppression, alcoholism, recent
surgical intervention, trauma, and mor-
bid obesity
5. This is a serious, life-threatening condi-
tion requiring surgical intervention, and
broad spectrum antibiotics

1. Occurs in 2 settings:
B. Idiopathic scrotal calcinosis:

a. Calcification of preexisting
epidermal or pilar cysts
b. Calcification of dermal connective
tissue in the absence of cysts (idio-
pathic)
2. Usually young men, present with
multiple long-standing firm-to hard
nodules measuring up to 3 cm in
diameter
3. Overlying skin is generally intact,
although it may ulcerate and extrude
cheesy material

214 EDUCATIONAL REVIEW MANUAL IN UROLOGY


Penis and Scrotum II:
Tumors and Tumor-like Lesions

5. Differential diagnosis (clinical): Zoon’s


balanitis, drug eruptions, psoriasis,
1. Condyloma acuminatum

A. Etiologic agent: human papilloma virus lichen planus, and other inflammatory
B. Clinical features: usually sexually transmit- processes
ted; variable incubation period
C. Gross features: flat, warty or papillary 1. This is essentially the same lesion as ery-
B. Bowen’s Disease

lesions, typically involving glans or urethral throplasia of Queyrat, in the sense that it
meatus represents squamous cell carcinoma in
D. Histologic features: situ; however, unlike erythroplasia of
1. Proliferation of squamous epithelium Queyrat, Bowen’s disease does not have
with papillomatous architecture (warty) the velvety red patch, and it more often
showing orderly epithelial maturation involves the shaft of the penis rather than
2. Hyperkeratosis (thickened keratin layer), the glans/prepuce area
parakeratosis (pyknotic nuclei within the 2. Clinical: presents slightly earlier
keratin layer) and koilocytes (raisinoid than erythroplasia of Queyrat
cells within a clear “halo”) are character- (4th –5th decades)
istic features 3. Gross features: sharply demarcated
3. Penile condylomas are usually cytologi- hyperkeratotic plaque, usually on
cally benign, although treatment with shaft of penis
podophyllin or lasers may cause marked 4. Histologic features:
cytologic atypia – a history of prior treat- a. Squamous cell carcinoma in situ
ment is therefore always appreciated b. Hyperkeratosis is more common
4. Viral strains 6 and 11 are common in than in erythroplasia
typical condyloma without dysplasia, 5. Prognosis: approximately 5%–10%
while strains 16, 18, 31 and 33 are progress to invasive squamous cell
more common in the dysplastic forms carcinoma
of condyloma
E. Prognosis and treatment: may sponta- 1. Clinical: multiple papules in young men
C. Bowenoid Papulosis

neously regress in 50% of cases, but are oth- that histologically resemble squamous
erwise treated with podophyllin or laser cell carcinoma in situ
2. Gross: usually occur on penile shaft;
papules range from 0.2–1.0 cm; some-
times coalesce to form plaques like
2. Pre-malignant Lesions

condyloma acuminata
1. Clinical: usually uncircumcised men 3. Histologic features:
A. Erythroplasia of Queyrat

in 5th–6th decades • Hyperkeratosis, parakeratosis and


2. Gross features: bright red, glistening, papillomatosis
velvety plaque on the glans and the • May show a little more maturation
prepuce; may also involve urethral evident in the epithelium than is seen
meatus, frenulum , or neck of the penis in Bowen’s or Erythroplasia, but you
3. Histologic features: squamous may still see mitotic figures in the
carcinoma in situ (full thickness upper layers of the epithelium
involvement by neoplastic cells with 4. Prognosis: spontaneous regression
loss of maturation, etc.) occasionally occurs
4. Prognosis: 10% of patients progress to
invasive squamous cell carcinoma, and
2% develop distant metastases

CHAPTER 6: CYTOLOGY OF THE UROGENITAL TRACT 215


c. Hematogenous spread is uncommon,
despite the rich vascularity of the
3. Malignant Neoplasms of the Penis

corpora cavernosum
1. Cancer of the penis is uncommon, affect- 6. Staging: the most commonly used
A. General features:

ing approximately 1/100,000 men in systems are the Jackson system and the
North America; other areas (Haiti, AJCC system
Uganda, Brazil, etc.) have a much higher 7. Treatment and prognosis:
rate of penile carcinoma (10%–12%) • Grade and stage are the best predic-
2. 95% of penile carcinomas are squamous tors of clinical behavior, with stage
cell type
• Well-differentiated tumors typically
being the most important factor

1. Predisposing factors: present earlier, involve the prepuce,


B. Squamous cell carcinoma

a. Lack of circumcision combined and have a better long-term survival


with poor hygiene (80%), while poorly-differentiated
b. Phimosis: smegma may play a role tumors tend to present later, involve
in both phimosis and SCCA of penis the shaft, and are more often associ-
c. Viral (human papilloma virus ated with inguinal or distant metas-
infection) – questionable tases
2. Clinical features: men > 40 years;
blacks > whites (2:1)
C. Squamous cell carcinoma:

3. Gross features: 1. Verrucous Carcinoma:


Variations on the theme

a. May be exophytic and fungating or a. Clinical: accounts for 5%–16% of


ulcerated and infiltrative penile malignancies
b. Typical location is glans penis or b. Gross: large, fungating,
prepuce; rare sites include shaft and exophytic/warty mass arising in
urethral meatus coronal sulcus with spread to
4. Histologic features: glans and prepuce
a. Well-differentiated SCCA shows c. Histologic features:
finger-like downward invasion with • Well-differentiated squamous
extensive keratin formation and cell carcinoma with broad push-
prominent intercellular bridges ing borders rather than the finger-
b. Poorly-differentiated SCCA form like infiltrative borders of a typi-
little or no keratin, have numerous cal squamous cell carcinoma
mitotic figures and marked nuclear • Cytologic atypia is minimal and
pleomorphism mitotic figures are rare
c. Moderately-differentiated SCCA d. Behavior: locally invasive but typi-
lies somewhere between the 2 cally does not metastasize; may
extremes recur if inadequately excised
d. In general: exophytic/ papillary e. Differential diagnosis: condyloma
lesions tend to be well-differenti- acuminatum (giant condyloma of
ated, while the ulcerating/ infiltra- Buschke and Löwenstein) and the
tive lesions are moderately- to more typical form of SCCA. (Actu-
poorly-differentiated ally, some people feel that the Giant
5. Clinical behavior: condyloma of B and L may be the
a. Locally invasive, destroying the same entity as verrucous carcinoma;
prepuce and penile shaft however, verrucous carcinoma typi-
b. Metastases can go to ipsilateral or cally is not associated with HPV,
contralateral inguinal lymph nodes whereas the giant condyloma is)

216 EDUCATIONAL REVIEW MANUAL IN UROLOGY


f. Treatment: radical or partial penec- 4. Clinical: slow-growing warty lesion,
tomy reduces the risk of recurrence usually single, located on anterolateral
to 33%, so this is the recommended aspect of scrotum; with time the lesions
treatment. (80% recurrence with ulcerate and may be associated with
local excision) Importantly: radia- rolled edges and purulent exudate; usu-
ally affects men in 6th–7th decades; left
> right (probably related to carcinogen
tion should be avoided as it may

exposure); 50% have inguinal metas-


induce dedifferentiation into a

tases at the time of presentation


highly aggressive spindle cell squa-

2. Spindle cell (sarcomatoid) squamous 5. Histologic features: looks like SCCA


mous carcinoma

cell carcinoma: of the penis


a. Rarely reported on the penis 6. Staging: Lowe’s system is the most
b. Gross: often forms polypoid mass common staging system used
c. Histologic features: spindle cell 7. Treatment and prognosis: poor prognosis
neoplasm with focal squamous (30%–52% survival at 5 years)
differentiation
d. Behavior: aggressive clinical course 1. Essentially represents an adenocarci-
B. Paget’s Disease:

noma in situ
1. Less than 100 reported cases 2. Clinical: 6th –7th decades, fairly rare,
D. Malignant Melanoma

2. Usually in white males, 5th–6th decades but most cases are associated with an
3. Gross: black, brown, or blue variegated underlying carcinoma (especially blad-
papules with irregular borders and der, urethra or prostate)
irregular pigmentation 3. Gross: scaly eczematous lesion
4. Histologic features: 4. Histologic features:
a. Nests of melanocytes at the dermal- a. Intraepithelial proliferation of atypi-
epidermal junction cal cells with vacuolated cytoplasm
b. Atypical melanocytes may and large vesicular nuclei
“migrate” to the upper layers b. Atypical cells tend to cluster at the
of the epidermis tips of the rete pegs
c. Infiltrative component shows c. Intracytoplasmic mucin vacuoles
prominent nucleoli and none of the (highlighted by PAS or muci-
maturation that is often seen in nevi carmine)
d. Hyperkeratosis, parakeratosis, and
4. Malignant Lesions of the Scrotum papillomatosis are common

1. Sarcoma of the scrotum is very unusual,


C. Leiomyosarcoma

1. Scrotal squamous cell carcinoma was the but the most common form is
A. Squamous cell carcinoma:

first cancer linked to occupational expo- leiomyosarcoma


sure: Potts first described the higher fre- 2. Tumor arises from the Dartos muscle;
quency among men exposed to high lev- <20 cases have been described
els of dust and soot (chimney sweeps,
cotton factory workers) – hence, scrotal
SCCA was at one time called “Pott’s can-
cer” or “chimney sweep’s cancer”
2. The offending agent has subsequently
been identified as 3,4-benzpyrene
3. SCCA is the most common malignant
neoplasm of the scrotum, but is still
much less common than SCCA of the
penis

CHAPTER 6: CYTOLOGY OF THE UROGENITAL TRACT 217


Bladder I: Non-Neoplastic Lesions

2. Not a “normal” finding and should


be reported
1. Normal Histology

A. Urothelium: 5–7 cells thick, with umbrella, 3. May be seen in association with
intermediate, and basal cells. The urothe- adenocarcinoma or other cancers of
lium renews approximately once per year the bladder
B. Lamina propria: loose fibrovascular tissue
with highly variable, delicate smooth muscle 1. Nonkeratinizing: stratified squamous
E. Squamous metaplasia:

fibers (muscularis mucosae) epithelium usually at the trigone and


C. Muscularis propria/detrusor muscle: bladder neck
several layers of thick muscle tissue a. Seen in 85% of reproductive
D. Urachus: remnant is often found at the age women and 75% of
dome and appears as a central lumen lined menopausal women
by epithelium; surrounded by fibrous tissue b. May also be seen in men receiving
and smooth muscle estrogen therapy for prostate cancer
c. No clinical significance, other than
the fact that patients may have
urinary urgency and frequency
2. Normal/ Metaplastic Lesions

(“pseudomembranous trigonitis”)
1. Well-circumscribed nests of benign 2. Keratinizing (leukoplakia): stratified
A. Von Brunn’s nests:

urothelial cells in the lamina propria squamous epithelium with keratin layer
without obvious attachment to the over- on top; looks like a flat gray-white
lying epithelium plaque, and typically involves
2. Thought to be a “normal” feature of anterior wall
the bladder mucosa, most commonly a. 20% of patients will have
in trigone
b. 20% of patients will develop
synchronous bladder cancer

1. Essentially von Brunn’s nests bladder cancer at a later date


B. Cystitis cystica:

which have undergone central cystic c. Commonly precedes squamous


degeneration
2. Usually seen as a submucosal nodule
cell carcinoma in areas where

3. Contains pink luminal secretions


schistosomiasis is endemic

1. In adults, more common in males (2:1);


F. Nephrogenic adenoma

1. Found in 71% of bladders, usually in children, more common in females


C. Cystitis glandularis:

microscopic but may be grossly apparent 2. 75% occur in the bladder, but usually
as raised, red nodular lesions; particu- does not involve the anterior wall
larly common in the trigone; likely a 3. Clinical features: typically solitary and
variant of cystitis cystica may be associated with previous GU
2. Typical type: similar to cystitis cystica surgery (60%), calculi (15%), trauma
except have (10%), renal transplant (8%)
a. Cuboidal or columnar epithelium 4. Histologic features: small simple
b. One or more surrounding layers of tubules lined by a single layer of
bland cuboidal to low-columnar
3. Intestinal type: less frequently there is a cells (“hobnail” cells) with clear or
urothelial cells

a. Tall columnar epithelial lining with eosinophilic cytoplasm


obvious mucin production and a. May also have cystic, polypoid,
papillary, and diffuse patterns
b. No surrounding layer of urothelium b. No mitoses or atypia
goblet cells

5. Likely due to shed renal tubular cells


1. Similar in appearance to goblet/secretory implanting in the wall of the bladder
D. Intestinal metaplasia:

cells of the intestine

218 EDUCATIONAL REVIEW MANUAL IN UROLOGY


f. Typically seen in females (90%)
ages 30–50
3. Cystitis

g. Etiology unknown (but it is not


1. Polypoid (papillary) cystitis:
A. Non-specific cystitis:

a. Characterized by chronic inflam- 3. Post-surgical necrobiotic granulomas:


bacterial)

mation and edema of the lamina a. Characterized by granulomas with


propria
b. Most often seen with indwelling
serpiginous borders and central

catheters (80%) and fistulas b. Typically seen after surgery


acellular eosinophilic material

c. Urothelium is usually normal thick- (hence the name)


ness with umbrella cells present 4. BCG granulomas:
2. Follicular cystitis: a. See marked chronic inflammation
a. Characterized by lymphoid follicles with giant cells in the lamina propria
in lamina propria, typically with b. Epithelioid granulomas may be
germinal centers seen which lack central necrosis
b. Often occur in patients with bladder c. Typically seen after BCG treatment
cancer (40%) and in patients with (again, hence the name)
urinary tract infections (35%) 5. Radiation cystitis:
3. Giant cell cystitis: a. See loss of urothelium with the
a. Characterized by atypical mes- remaining urothelial cells showing
enchymal cells with enlarged multi-
nucleated nuclei in the lamina pro- chromatin, but normal N/C ratio
nuclear atypia, smudgy

pria b. Blood vessels become hyalinized


4. Hemorrhagic cystitis: c. Cytoplasm becomes very vacuolated
a. Characterized by congested vascu- d. Typically seen 3–6 weeks
lature and hemorrhage in the lam- post-radiation
ina propria 6. Reaction to topical chemotherapy:
b. Etiologies include: chemical toxins a. Induces denudation of the
(cyclophosphamide, busulfan, urothelium; similar changes to
thiotepa, aniline dyes, and insecti- radiation including nuclear
cides), radiation, viral infection, and pleomorphism and hyperchromasia
idiopathic
1. Bacterial cystitis: most common cause of
C. Infectious cystitis:

1. Eosinophilic cystitis: cystitis, usually due to coliform bacteria


B. Specific forms of cystitis:

a. Characterized by dense infiltrate of a. Histology: ulcerated urothelium


eosinophils in the lamina propria with neutrophils in lamina propria
and muscularis 2. Encrusted cystitis: may arise when urea-
b. Rarely may represent allergic
cystitis and inorganic salts are deposited in the
splitting bacteria alkalinize the urine

2. Interstitial cystitis (Hunner’s ulcer): damaged mucosa


a. May or may not be associated a. Most commonly seen in women
with an ulcer b. Histology: lesions are covered by a
b. Grossly appears as hemorrhagic coat of fibrin (pink stuff) mixed with
spots or linear cracks calcified necrotic debris (dark purple
c. Characterized by lymphoplasma- stuff) and inflammatory cells
cytic infiltrate in lamina 3. Emphysematous cystitis: usually occurs
d. May see increased mast cells in women and diabetics, with the
e. No microscopic findings are causative agents being E. coli or Aer-
pathognomonic: must use clinical obacter aerogenes
correlation

CHAPTER 6: CYTOLOGY OF THE UROGENITAL TRACT 219


a. Histology: empty cavities lined
with flattened cells and surrounded
4. Miscellaneous Lesions

by thin septa
4. Malakoplakia: occurs due to impair- 1. Normal bladder surface is quickly
A. Exstrophy:

inflamed and ulcerated, and undergoes


phagocytosed bacteria (E. coli but also squamous and intestinal metaplasia
ment of mononuclear cells in killing

Proteus vulgaris, Aerobacter aerogenes, 2. Untreated patients have an increased


and Klebsiella pneumoniae); more com- risk of cancer, but surgical closure
mon in women prevents this
a. Gross: multiple soft yellow plaques
b. Histology: accumulation of histio- 1. Saccular evaginations of the bladder
B. Diverticula:

cytes with eosinophilic cytoplasm mucosa through the detrusor muscle


(von Hansemann histiocytes) in the 2. Usually seen in males with outlet
superficial lamina propria beneath obstruction (prostatic hyperplasia)
an intact urothelium 3. May be associated with squamous or
c. The histiocytes contain laminated intestinal metaplasia
calcified cytoplasmic inclusions 4. There is an increased risk of carcinoma
(Michaelis-Gutmann bodies) with arising within these diverticula
a bull’s-eye appearance
d. Must see M-G bodies for diagnosis; 1. When it occurs in the bladder, it is usu-
C. Amyloidosis:

these stain for calcium and iron ally primary rather than systemic
5. Tuberculous cystitis: almost always 2. Adults generally affected, men = women
follows renal infection by M. tuberculo- 3. Typically present with hematuria
sis 4. Histologically characterized by deposi-
a. Histology: granulomatous inflam- tion of eosinophilic glassy material in
lamina and muscularis propria (Congo
red positive with “apple-green” birefrin-
mation with epithelioid histiocytes

gence)
and multinucleated giant cells sur-

6. Schistosomiasis: usually due to


rounding caseous necrosis

Schistosoma haematobium 1. Microscopic remnants of the urachus


D. Patent urachus:

a. May be characterized by polyps, are common


ulcers, squamous metaplasia, 2. Remember: urachus connects bladder
chronic inflammation and the pres- and the allantois
ence of ova in the lamina propria 3. The discontinuous epithelium-lined duct
b. Ulcers are more common in the can give rise to cysts, sinuses, or neo-
chronic stage when there are a large plasms, particularly adenocarcinomas
number of eggs mimicking colonic type

1. Typically seen in menstruating women


E. Endometriosis:

2. Most women will have a history of


previous pelvic surgery (50%)
3. Bladder is the most common site of
urinary tract involvement, although only
1% of patients with endometriosis will
have bladder involvement
4. See endometrial glands, stroma, and
hemosiderin-laden macrophages

220 EDUCATIONAL REVIEW MANUAL IN UROLOGY


Bladder II:
Tumors of the Urinary Bladder

• Thought to be related
to nitrosamines and
1. Benign Epithelial Neoplasms

2-naphthylamine
1. Comprises 2%–3% of papillary b. Occupational exposure:
A. Urothelial papilloma:

urothelial tumors 33% of cases


2. Small, usually unifocal, and consists of • Especially: dye industry
delicate fibrovascular stalks covered by (benzidine and 2-naphthylamine)
cytologically and architecturally normal c. Phenacetin: significant risk only
urothelium when high cumulative doses (2 kg)
3. Tend to occur in younger patients are reached
(<50 years) d. Cyclophosphamide:
4. May occasionally recur, although • 10.7% increased risk at 12 years;
generally considered a benign neoplasm latency of 65–141 months
• Thought to be due to accumulation
1. Uncertain etiology: some authors of acrolein (degradation product)
B. Inverted urothelial papilloma:

consider it a neoplasm, others feel it • Majority of tumors are UCC, but


is reactive (à la cystitis glandularis/ can also result in squamous cell
cystitis cystica) carcinoma, adenocarcinoma, and
2. Most commonly occurs in middle-aged leiomyosarcoma
men presenting with hematuria or irrita- e. Radiation: risk of UCC is increased
tive symptoms 2-4x in women who have received
3. Most are small, single, and appear as pelvic radiation for cervical cancer
smooth domed masses at the trigone f. Molecular:
4. Microscopic: • Includes frequent loss of
a. Anastomosing cords or sheets of p53 and RB
urothelium arising from the • Additional mutations include
overlying mucosal surface PTEN, TSC1, PI3KCA and
b. The central epithelium appears to many others
have a swirling appearance g. Heredity: does not play a big
c. Generally, bland-appearing cells role in UCC
with minimal atypia and rare to 3. Clinical features: 90% present with pain-
no mitoses less hematuria
4. Diagnosis:
a. Urine cytology: good for following
high-risk population, but not good for
2. Malignant Epithelial Neoplasms

a screening procedure
1. Epidemiology: b. Cystoscopy and biopsy: currently
A. Urothelial Carcinoma: general facts

a. Whites > blacks; males > females the primary diagnostic tool
b. Median age: 65 years c. Cystograms and excretory urogra-
2. Etiology and pathogenesis: risk factors phy: it is important to screen for
include: upper tract disease as the cause of the
a. Tobacco smoke: 33% of UCCs presenting symptom (hematuria) and
occur in cigarette smokers to exclude synchronous upper tract
• Cigarettes >> pipes and cigars >> cancer in patients with bladder TCC
smokeless tobacco (essentially 5. Staging is important for clinical manage-
no increased risk) ment and outcomes: currently, biopsies
• Cigarette smokers have 2–4x and TUR specimens should describe
increased risk, which decreases whether or not muscularis propria (detru-
gradually once they quit smoking sor muscle) was obtained during sampling

CHAPTER 6: CYTOLOGY OF THE UROGENITAL TRACT 221


6. NO LONGER USE G1, 2 or 3 in the b. Appearance is similar to high-grade
United States papillary urothelial carcinoma,
except that the lesion is flat rather
than on a fibrovascular core
7. Papillary urothelial carcinoma:

c. Commonly involve the prostatic ure-


a. Papillary urothelial neoplasm

thra (67%), prostate gland (up to


of low malignant potential

• Defined as a papillary neoplasm 40%) and ureters (up to 57%)


(PUNLMP)

lined by thickened, but otherwise d. Prognosis: 32%–83% of patients


normal, urothelium with CIS will develop invasive can-
• May progress to high-grade cer, and up to 34% already have
and/or invasive disease in a sub- microinvasion elsewhere at the time
set of cases of diagnosis
• Some argue that this is a form of
low-grade papillary urothelial a. May be polypoid, sessile, ulcerated
10. Invasive urothelial carcinoma (UCC)

carcinoma and infiltrative


b. Most invasive UCCs consist of
cohesive nests of cells with large
b. Low-grade papillary urothelial

• Fibrovascular cores hyperchromatic nuclei, moderate


carcinoma

• Epithelium may be thick or thin amount of cytoplasm, and some-


• Contains occasional dark times demonstrating peripheral pal-
(hyperchromatic) nuclei isading
• Occasional mitotic figures are c. The invasive nests may cause a
present but not atypical desmoplastic reaction (fibrotic)
• Overall, the lesion maintains which can be helpful in diagnosing
polarity and cellular organization early lesions
throughout d. Approximately 15 or so variants of
UCC exist – most are rare and their
significance/molecular background
8. High-grade papillary urothelial

a. Fibrovascular cores are unclear. Some key subtypes


carcinoma

b. Characterized cells with markedly include micropapillary carcinoma


enlarged nuclei (>4x size of a lym- (appears to have an aggressive
phocyte) that are hyperchromatic; course), nested urothelial carcinoma
cells may also have prominent and many others
nucleoli and numerous (atypical) e. Mixed morphology tumors are a
mitoses in the mid to upper portion common variant and include squa-
of the urothelium mous differentiation (most common)
c. Cells have a random orientation, and glandular differentiation
giving overall disorganized f. Sarcomatoid variant: contains
appearance to the mucosa epithelial and spindle cell compo-
d. There is often striking discohesion nents → tends to occur in older
resulting in extensive loss of the patients (60s–70s) and is highly
epithelial cells (“denuding cystitis”), malignant (25% survival at 2 years)
which is why cytology can be so g. Critical pathologic staging rests
helpful in these lesions upon the presence or absence of

a. CIS is usually associated with prior


9. Flat urothelial carcinoma in situ (CIS): muscularis propria invasion

or synchronous bladder tumors

222 EDUCATIONAL REVIEW MANUAL IN UROLOGY


2. Urachal adenocarcinomas:
1. Clinical features: a. Diagnosis requires
B. Squamous Cell Carcinoma:

a. In areas where schistosomiasis is • Location in the dome with pri-


endemic, SCCA accounts for up to mary involvement of the muscle
73% of bladder cancers, but in the or deeper structures,
• Presence of a suprapelvic mass
• Tumor growth in the bladder
US and UK, this tumor comprises

b. Typically seen in males (M:F = wall extending into the space of


only 3%–6% of all bladder cancers

1.7:1); with a mean age of 65.5 years Retzius


c. Presenting complaint: hematuria or • Sharp demarcation between
irritative symptoms
d. Many will have advanced disease at
tumor and adjacent normal

the time of diagnosis • Absence of cystitis cystica/


urothelium

2. Etiology and pathogenesis: glandularis elsewhere in


a. Schistosomiasis: 3 types (S. man- the bladder
soni, S. japonicum and S. haemato- • Presence of urachal remnants
bium) but only S. haematobium b. Urachal adenoca. Typically occur in
causes bladder cancer adult males, mean age 50.6 years
b. Chronic bladder irritation c. Thought to occur due to intestinal
(catheters, urinary retention, recur- metaplasia of the urachal remnant
rent infections, stones) d. Generally form discrete masses in
c. Bladder diverticula the dome of the bladder and
3. Gross: bulky necrotic masses that
usually fill the bladder the bladder rather than based in the
appear to be centered in the wall of

4. Histology: the diagnosis of squamous mucosa


e. Microscopic: usually colloid carci-
noma (cells floating in mucin), but
carcinoma of the bladder is reserved

histology (i.e., if there is any urothelial may have signet ring or columnar
for those tumors with pure squamous

cancer, the diagnosis should be UCC morphology


with squamous differentiation) f. Staging: same as used for UCC;
5. Grading: assessed according to the other systems have been proposed
degree of keratinization (lots = well-dif- but have not been widely adopted
ferentiated; none = poorly-differenti- g. Treatment and natural history:
ated) partial or radical cystectomy with
6. Staging: same system as is used for UCC en bloc resection of urachal remnant
7. Prognosis: appears to be relatively simi- and umbilicus
lar to UCC when adjusted for stage; h. Survival is poor: 37% and 17% at 5
however, as these tumors often present in and 10 years, respectively
an advanced stage many patients have 4. Nonurachal adenocarcinomas:
poor outcomes a. Clinical features:
• Accounts for the majority of
1. Primary adenocarcinoma of the bladder
C. Adenocarcinoma:

accounts for <1% of all bladder tumors • Males > females; mean age
bladder adenocarcinomas

and generally falls into 2 groups 59.4 years


(although it may be hard to distinguish • Hematuria is the most common
site of origin in large tumors for all prac- presenting complaint
tical purposes): b. Etiology and pathogenesis:
a. Those occurring in the urachus • Most cases arise from intestinal
b. Those occurring in the bladder
proper plasia of the urothelium
metaplasia and glandular dys-

CHAPTER 6: CYTOLOGY OF THE UROGENITAL TRACT 223


• Most commonly occur in patients
with non-functioning bladder,
3. Paraganglioma

1. Found equally in males and females


chronic irritation, obstruction A. Clinical features

• May also occur in the setting 2. Thought to arise from the paraganglionic
and cystocele

of exstrophy tissue within the bladder wall


c. Histology: generally divided into 3. Hematuria is the common presenting
6 categories complaint, but the classic scenario is
• Adenocarcinoma NOS paroxysmal hypertension with micturi-
• Enteric (looks like intestinal tion
adenoca)
• Signet ring 1. Intramural lesions with predilection for
B. Histology:

• Mucinous (colloid – cells the trigone, anterior wall, and dome


floating in mucin) 2. Generally pink-tan or yellowish, well-
• Clear cell circumscribed, lobulated appearance
• Mixed – 2 or more of the 3. Histology: nesting growth pattern (zell-
above patterns ballen) – sort of like pheochromocytoma
d. Grading: no currently accepted 4. Mitotic figures are rare, and necrosis
grading standard is absent
e. Staging: use the same system 5. No definite pathologic feature that
as UCC predicts benign or malignant course
f. Treatment and natural history: C. Treatment and natural history: may recur
• Radical cystectomy/ cystoprosta- if not completely excised; metastases
tectomy with pelvic lymph node occurred in <15% of cases (only 2 cases
dissection is the recommended showed distant mets)
treatment
• Chemo and radiation play
uncertain roles
4. Benign Mesenchymal Neoplasms

• Poor prognosis overall:


5- and 10-year survivals are 1. More common in women
A. Leiomyoma:

17% and 11%, respectively 2. Obstructive symptoms are most common


(ball-valve effect of tumor)
1. <1% of bladder tumors 3. Gross: submucosal mass
D. Small Cell Carcinoma:

2. Males > females; 60–70 years old 4. Micro: spindle cell proliferation with
3. Hematuria is the common presenting cigar-shaped nuclei and no mitoses
symptoms
4. Paraneoplastic syndromes are rare 1. Typically benign, but some reports of
B. Inflammatory myofibroblastic tumor

5. Gross: highly variable malignant behavior are in the literature


6. Histology: sheets of small round blue 2. Clinical: often associated with recent
cells with numerous mitoses, crush biopsy/TUR but not always the case
artifact, nuclear molding and necrosis 3. Hematuria is the most common
7. Will stain for neuroendocrine presenting complaint
markers (NSE, chromogranin, 4. Gross: solitary nodule
and synaptophysin) 5. Micro: spindle cells in a myxoid matrix
8. Prognosis: dismal with prominent vasculature; mitoses
common; lots of eosinophils
6. Immunostains for ALK-1 may help in
the diagnosis

224 EDUCATIONAL REVIEW MANUAL IN UROLOGY


Prostate I: Non-Neoplastic
Lesions of the Prostate

5. Malignant Soft Tissue Neoplasms: 1. Normal Histology

A. Rhabdomyosarcoma: A. Ducts:
1. Rare in adults, more common in 1. Elongated branching, tubular structures
children (boys > girls) with a ruffled, undulating luminal sur-
2. Clinical: generally present with face
obstruction and hematuria 2. Some of the larger ducts connect to the
3. Gross: forms polypoid masses surface urothelium (so UCC can arise
(sarcoma botryoides) within the prostate)
4. Trigone is the most common place B. Acini: small, rounded simpler glands with
5. Histologically, most are embryonal lobular arrangement around the ducts
(small round blue cell tumor with C. Cell types:
focal strap cells) 1. Epithelial:
B. Leiomyosarcoma: a. Columnar secretory epithelial cells
1. The most common sarcoma arising line the smaller ducts and the acini
in the bladder • Have abundant clear cytoplasm
2. Mean age 52 years; several have • Stain for PSA and PAP
occurred following cyclophosphamide b. Basal cells are the second layer of
treatment cells located between the secretory
3. Often lobulated grossly, with or cells and the basement membrane
without ulceration (markers for basal cells include p63
4. Micro: spindle cells with cigar-shaped and CK903)
nuclei, numerous mitoses c. Scant cytoplasm, small round nuclei
5. Differential dx: spindle cell variant of • Stain for cytokeratin, NOT for
TCC (sarcomatoid carcinoma) PSA and PAP
• Check for areas that look like typical 2. Stromal
TCC (epithelial component) 3. Neuroendocrine: The prostate has the
• Do stains: LMS will stain for actin highest number of endocrine-paracrine
and desmin (muscle markers); sarco- cells of any GU organ
matoid carcinoma will stain for D. Seminal vesicles:
cytokeratin (epithelial marker) 1. Form large complex glands with
papillary tufts
2. Look for “monster cells” (highly
pleomorphic and enlarged) and
lipofuscin (golden-brown pigment)

2. Prostatic Hyperplasia

A. Can first be seen at 20–30 years; increases


steadily after age 40

1. Typically results in smoothly


B. Clinical:

enlarged prostate
2. Symptoms due to obstruction
(urgency, frequency, nocturia, etc.)
3. Etiology: unknown, but most likely
related to androgen stimulation with
estrogen synergism

CHAPTER 6: CYTOLOGY OF THE UROGENITAL TRACT 225


3. Micro: Lymphocytic inflammation
1. Both glands and stroma can become around the acini with neutrophils in the
C. Histologic features:

hyperplastic lumina
2. Hyperplastic glands typically are large 4. Chronic prostatitis is generally a clinical
with complex papillary in foldings – rather than a histologic –diagnosis,
3. Cytologically benign, with 2 cell layers, since the features we use to diagnose
bland nuclei, and abundant pale cyto- chronic prostatitis are quite similar to the
plasm inflammatory changes seen in BPH
4. Will often see corpora amylacea
(bright pink laminated concretions) in
C. Non-bacterial prostatitis

these areas 1. Inflammatory condition of unknown


(clinical diagnosis):

5. Also often associated with chronic lym- cause


phocytic infiltrate – very nonspecific 2. Most common prostatitis syndrome
finding 3. Diagnosis is based upon negative
cultures and a negative history of UTIs

1. Mycobacterial:
3. Infarcts D. Granulomatous prostatitis:

A. 20%–25% of BPH specimens have infarcts a. May occur in patients with systemic
(due to inadequate blood flow from large TB (3%–12%), but now is more
size) commonly seen with BCG treat-
B. Grossly appear mottled and yellowish with ment for TCC
hemorrhagic rim b. Most cases of tuberculous prostatitis
C. Microscopically, demonstrate zoning effect arise from hematogenous spread
1. Central coagulative necrosis c. Histologically, the features of BCG
2. Surrounded by hemorrhage and prostatitis are indistinguishable from
inflammation with reactive glands those of tuberculous prostatitis:
3. Surrounded by a zone in which the • Small noncaseating granulo-
glands show squamous metaplasia mas that are well-demarcated
from the surrounding stroma
• Some larger granulomas may
show central caseation (consists
4. Prostatitis

of grummous fine granular


debris, as opposed to coagulative
A. Acute bacterial prostatitis (clinical diag-

1. Clinically associated with UTIs, necrosis which is amorphous fib-


nosis):

increased numbers of inflammatory cells rinoid material with ghost-like


in the prostatic secretions, and growth of outlines of vessels, cells, etc.)
bacterial organisms from prostatic secre- • Stains for acid-fast bacilli are
tions often performed to identify
2. Majority are caused by Gram-negative organisms
organisms (E. coli) e. Treatment: BCG granulomas
3. Micro: sheets of neutrophils in and resolve on their own; TB needs
around the acini treatment
2. Malakoplakia:
a. As in malakoplakia of the bladder,
B. Chronic bacterial prostatitis (clinical diag-

1. Clinically characterized by recurrent this reflects an inability to digest


nosis):

UTIs caused by the same pathogens bacteria following phagocytosis


2. Mostly caused by Gram-negative organ- b. Symptoms include urinary retention,
isms; NOT by chlamydia or trichomonas urgency, frequency, etc.
(contrary to popular belief)

226 EDUCATIONAL REVIEW MANUAL IN UROLOGY


c. Clinically can mimic cancer because
causes an indurated prostate on
5. Basal Cell Hyperplasia

exam and may appear as a hypoe- A. May occur in association with BPH or atro-
choic lesion on transrectal ultra- phy
sound B. Histologically, the glands are small and
d. Histologically: von Hansemann angular, and appear blue at low power due to
the multilayering of basal cells that have
bodies (see bladder section for more scant cytoplasm
histiocytes + Michaelis-Gutmann

details) C. Hyperplasia ranges from a few layers to solid


3. Non-specific granulomatous prostatitis: nests; may occasionally show cribriform
a. Most commonly diagnosed architecture
granulomatous prostatitis D. Typically have bland cytologic features
b. Etiology is thought to be due to (no nucleoli, etc.)
inflammatory reaction to bacterial E. Can do keratin immunostains to confirm
toxins, cell debris and secretions cell type
spilling into the stroma from

c. Major significance is that it can


blocked ducts 6. Clear Cell Cribriform Hyperplasia

mimic carcinoma on rectal exam, A. Benign lesion that may be confused with
ultrasound and give an elevated PSA PIN or adenocarcinoma
d. Histologically, the granulomas are B. Composed of numerous cribriform glands
less well-formed, and are associated separated from one another by a modest
with dense stromal inflammatory amount of stroma in a pattern of nodular
infiltrate with scattered giant cells hyperplasia
e. Have more mixed inflammatory C. The epithelial cells have distinctive clear
infiltrate than is seen in infectious cytoplasm and small bland nuclei with
granulomatous prostatitis (plasma inconspicuous or small nucleoli
cells, neutrophils, lymphocytes) D. Often surrounded by a striking basal
f. Associated with ruptured acini cell layer
(as opposed to infectious, which will E. Cribriform “bridges” are delicate and
have intact acini) demonstrate “streaming” of the nuclei, rather
g. Treatment: warm sitz baths, fluids than rigid perpendicular alignment
and antibiotics if a UTI is docu-
mented
4. Postsurgical granulomas:
7. Atrophy

a. Appears to be a reaction to A. Can give rise to firm gland with hypoechoic


lesions on transrectal ultrasound
b. Can occur anywhere from 9 days to B. Histologically characterized by well-formed
cauterized tissue

52 months post-TURP glands with very basophilic appearance


c. Histologically have the same due to scanty cytoplasm rather than nuclear
appearance as postsurgical enlargement (as is seen in PIN);in contrast,
granulomas elsewhere: adenocarcinomas have abundant cytoplasm
• Serpiginous border C. Glands are often angulated and surround a
(geographic pattern) central dilated duct (adenocarcinoma has
• Central fibrinoid necrosis small round glands that do not maintain lob-
• Surrounded by palisaded ular architecture)
histiocytes D. No prominent nucleoli
d. Treatment: None

CHAPTER 6: CYTOLOGY OF THE UROGENITAL TRACT 227


Prostate II: Tumors and Tumor-
Like Lesions of the Prostate

8. Squamous Metaplasia 1. Polyps of the Prostatic Urethra

A. Common adjacent to infarcts or following A. Clinical presentation: hematuria


antiandrogen therapy and in neonates B. Papillae lined by normal prostate tissue
(response to maternal hormones) that is PSA and PAP positive
B. In general, the cells lack reactivity to pro- C. Benign finding. It is important to distinguish
static markers (PSA, PAP) this entity from ductal adenocarcinoma of
the prostate, which may also present with a
polyp-like appearance in the urethra

2. Prostatic Intraepithelial Neoplasia (PIN)

A. Generally accepted as the probable


precursor lesion for many cases of
prostatic carcinoma
B. Only high-grade PIN is currently diagnosed
C. On low power, glands maintain normal
architecture but appear very basophilic
(blue), due to larger nuclei, higher N:C
ration, nuclear crowding and pseudostratifi-
cation
D. Prominent nucleoli are the mainstay
of the diagnosis
E. Basal cells are present (p63 and
CK903 stain)

3. Prostatatic (Acinar) Adenocarcinoma

1. Most common cancer in adult men; 2nd


A. Clinical:

in males
leading cause of cancer-related death

2. Microscopic foci of cancer may begin in


the 30s; present in 70% of men by age 70
3. Metastases most commonly occur to
bone (osteoblastic), lung and pelvic
(obturator) lymph nodes

1. 80% occur in the peripheral zone;


B. Gross features:

10%–20% occur in the transition zone


2. Often difficult to detect grossly: said to
have yellow color, but more often appear
gray-white

1. Main criteria include:


C. Microscopic features:

• Small back-to-back glands


• Infiltrative appearance
• Prominent nucleoli
• Lack of basal cells, generally
demonstrated by staining

228 EDUCATIONAL REVIEW MANUAL IN UROLOGY


2. Helpful clues are the presence of sharp
crystalloids and/or blue mucin in the 1. Grade 1:
D. The Gleason Grading System

gland lumina a. Circumscribed nodules of uniform,


3. Neoplastic glands like to track along the single, separate, closely-packed
nerves glands
4. The Gleason system classifies prostate b. Glands tend to be larger than those
cancer into 5 grades based upon low- seen in the higher-grade
power architectural features c. Usually found in the transition zone
a. On biopsy, the score is the most 2. Grade 2:
common + worst (minimum score on a. Again, a nodular pattern of tumor;
biopsy is 3+3=6 since architecture however, the edge of the tumor
cannot be appreciated) nodule may show minimal
b. On prostatectomy, the score is the
most common + second most com-
extension of glands into the

mon pattern
surrounding non-neoplastic

b. Glands are more loosely


prostate

1. Recent work has identified ERG overex- arranged and not quite as uniform
D. Molecular:

pression (caused by fusion of members as in Grade 1


of the ETS nuclear transcription factor 3. Grade 3:
family with androgen receptor-regulated a. Tumor infiltrates in and among the
genes, such as TMPRSS2) in the major- non-neoplastic prostate
ity of prostate cancer cases b. Glands have marked variation in
2. Ongoing work is underway to define the size and shape
function of this proto-oncogene in pre- c. Glands tend to be smaller than in
cancerous and cancerous conditions of grade 1 or 2
the prostate d. In contrast to pattern 4, the glands in
Gleason pattern 3 are discrete units
(If one can mentally draw a circle
around well-formed individual
Figure 1

glands)
4. Grade 4:
Gleason’s scoring system for

a. The main distinction between Grade


prostatic adenocarcinoma

3 and Grade 4 is fusion of the


glands: the glands are no longer sin-
gle and separate as seen in patterns
1–3; they are fused and ill-defined,
even at low power
b. Lumina are lost (i.e., glands are
being poorly formed)
5. Grade 5:
a. No gland formation is evident
b. Tumor infiltrates as solid mass or
single cells
E. Other variations on the adenocarcinoma

1. Ductal adenocarcinoma
theme:

a. May present as a polypoid/papillary


mass in the urethra
b. Slit-like lumens and columnar
nuclei with prominent nucleoli

CHAPTER 6: CYTOLOGY OF THE UROGENITAL TRACT 229


C. Differential diagnosis: prostatic adenocarci-
2. Sarcomatoid carcinoma/carcinosarcoma noma
c. Typically behave as a 4+4=8 tumor

a. Adenoca. having a malignant a. UCC is PSA negative (prostate


adenocarcinoma is PSA positive)
b. Usually seen postradiation therapy b. UCC has more atypia
spindle-cell component

(50% of cases)
c. Highly aggressive tumors
3. Mucinous variant:
5. Squamous Cell Carcinoma of

a. Tumor cells floating in a sea of


the Prostate

A. Very rare in North America


b. Extracellular mucin must comprise B. Seen in association with schistosomiasis
mucin

C. Squamous metaplasia may occur in adeno-


c. Gleason grade 4 carcinomas treated with radiation or hor-
at least 25% of the tumor

4. Signet ring cell variant: mones (particularly estrogens)


a. Tumor cells contain intracytoplas-
mic mucin droplets that compress
the nucleus against the cell mem-
6. Treatment Effects

brane
b. At least 25% of the tumor must be 1. Changes in the normal prostate include
A. Hormone therapy

composed of cells having this mor- a. Atrophy with a decrease in number


phology and size of glands
c. Gleason grade 5 b. Prominent basal cells
5. Small cell variant: c. Pale cytoplasm
a. Tumor composed of sheets of cells 2. Changes in tumor cells include
with small nuclei containing finely a. Small hyperchromatic nuclei
b. Inconspicuous nucleoli
b. Nucleoli are inconspicuous and c. Abundant clear cytoplasm
stippled chromatin

d. Occasionally only single cells will


c. Nuclear molding is often identified remain that look like histiocytes
cytoplasm is scant

d. PSA and PAP stains are negative 3. Bizarre stromal cells may be present
e. Highly aggressive lesions often pre-
sent with metastatic disease to solid 1. Changes in the normal prostate include
B. Radiation therapy

organs a. Marked atrophy of normal glands


f. Serum PSA may only be slightly with squamoid differentiation
elevated b. Nuclei may become enlarged and
g. 50% are associated with typical
adenocarcinomatous component 2. Changes in tumor cells are variable but
pleomorphic

may make tumors more difficult to rec-


4. Urothelial Carcinoma of the Prostate ognize (as in hormone therapy)

A. Primary UCC of the prostate is very rare:


arises from the prostatic urethra and spreads
into the prostatic ducts
B. Secondary UCC represents spread from a
bladder primary through the bladder wall
(new staging system restricts this definition
to pT4 tumors)

230 EDUCATIONAL REVIEW MANUAL IN UROLOGY


Testis I: Non-Neoplastic Disorders

f. Sperm excretory duct anomalies,


including:
I. Cryptorchidism

• Ductal fusion anomalies (25%) –


1. Causes include: fusion of epididymis to testis or
A. True cryptorchidism:

a. Anatomical anomalies of vas deferens, usually with


the gubernaculum intraabdominal or high scrotal
b. Hormonal dysfunction cryptorchidism
(hypogonadotropic hypogonadism) • Ductal suspension anomalies
c. Mechanical impairment (short (59%) – caput epididymis
spermatic cord, underdeveloped attached to testis while corpus
vaginal process) and cauda are separated by
d. Dysgenesis (primary testicular mesentery
anomaly) • Anomalies associated with
e. Heredity absent or vanishing testes (16%)
2. Decreased spermatogenesis in
cryptorchid testes may be multifactorial: 1. Testicular cancer:
C. Complications of cryptorchidism:

a. Transient hypogonadotropic a. Risk of testicular cancer is 5%–10%


hypogonadism higher in cryptorchid than in non-
b. Primary testicular anomaly cryptorchid patients
c. Injury caused by increased b. 10% of patients with testicular can-
temperature cer have cryptorchid testes
c. Most frequent tumor in cryptorchid
a. Atrophic and sclerotic seminiferous testis is seminoma
3. Histology:

tubules with thickened basement 2. Infertility: more commonly seen in bilat-


membranes eral cryptorchidism, although
b. Tubules with Sertoli cells and orchiopexy performed <4 years of age
spermatogonia only seems to preserve fertility
c. Mixed tubular atrophy with relative
Leydig cell hyperplasia
d. Hypoplastic rete testis
2. Infertility

1. Spermatogenic arrest: rare because germ


B. Congenital anomalies associated A. Basic definitions:

1. Cryptorchidism may be associated with cell maturation usually does not arrest at
with cryptorchidism:

Klinefelter’s, Noonan’s, Kallmann’s, the level of a defined germ cell type


Prader-Willi and prune belly syndromes (therefore, “irregular hypospermatogen-
2. Undescended testes may also be seen in esis” is more often appropriate in cases
trisomies 13, 18, 21, and the Aarskog- where the germ cell arrest occurs at vary-
Scott and Rubinstein-Taybi syndromes ing degrees of maturation)
3. Most cryptorchid patients have 2. Spermatocytic arrest (meiotic arrest):
persistent processus vaginalis and incidence varies from 12%–30%
inguinal hernia 3. “Disorganization of the seminiferous
4. Other urologic anomalies epithelium”: a relatively worthless diag-
(10.5% of patients) include: nosis that has not been supported by any
a. Hypospadias light microscopic or ultrastructural eval-
b. Complete duplication of uation. Just what is disorganized epithe-
urinary tract lium?
c. Kidney malrotation
d. Nonobstructive ureteral dilatation
e. Posterior urethral valves

CHAPTER 6: CYTOLOGY OF THE UROGENITAL TRACT 231


4. Tubular blockage: when at least 50% of b. Lesions in the adluminal compart-
the tubules lack central lumens; found in
28% of infertile men, so may represent a are classified according to the most
ment of the seminiferous tubules

real lesion (likely some variant of germ- abundant type of sloughed germ
cell sloughing) cell: young spermatids, later
primary spermatocytes, and early
primary spermatocytes --
B. Common lesions seen on testicular

1. Sertoli cell-only syndrome: Etiology of adluminal defects is


biopsies:

a. Seminiferous epithelium consists most likely obstruction:


only of Sertoli cells and rare sper- • Rete testis obstruction – usually
matogonia due to varicocele
b. Adult Sertoli cells have characteris- • Seminiferous tubule obstruction
tically pale, triangular nuclei with – usually dysgenetic or postor-
irregular indented outlines chitic
c. Clinical: normal external genitalia, c. Lesions in the basal and adluminal
well-developed secondary male
characteristics, azoospermia, ele- tubules are the most frequent find-
compartments of the seminiferous

vated FSH, normal or elevated LH, ings on testicular biopsy and can be
and normal or slightly low testos- classified into 2 major groups:
terone • Hypospermatogenesis: defined
d. Histology: characteristic as reduced numbers of both
“windswept” appearance to the spermatogonia and primary
seminiferous tubules spermatocytes with primary
2. Tubular hyalinization:
a. Nonspecific catch-all category in
spermatocytes outnumbering

which azoospermic patients demon- * May be pure (proportionate


spermatogonia

strate diffuse hyalinization of semi- decrease in all types of germ


niferous tubules cells) or be associated with
b. Because hyalinization is the end-
stage of atrophy, the causes of tubu-
primary spermatocyte

lar hyalinization are numerous, and * Etiology: hormonal dys-


sloughing

include: dysgenesis, hormonal function, congenital germ


deficit, ischemia, obstruction, cell deficiency, Sertoli cell
inflammation, physical or chemical dysfunction, Leydig cell
agents, and autoimmunity dysfunction, androgen
3. Diffuse lesions in spermatogenesis: insensitivity, exposure to
a. Lesions in the basal compartment chemical or physical agents
of the seminiferous tubules include and vascular malfunction
all infertile testes with normal num- * Usually patients have
bers of spermatogonia per tubular deficits in FSH, LH, or
profile, but decreased numbers of hyperprolactinemia or
other types of germ cells (aka, imma- adrenal/ thyroid dysfunction
ture germ cell sloughing). Diagnosis • Spermatogonia maturation arrest:
is based on finding >4% of ejacu- defined as <17 spermatogonia
lated cells are spermatogonia per tubular profile, with fewer
primary spermatocytes than sper-
matogonia
* Spermatids are absent

232 EDUCATIONAL REVIEW MANUAL IN UROLOGY


* Most frequent causes 2. 46 XX males:
include: cryptorchidism, a. Clinical features similar to Kline-
alcoholism or exposure to felter’s (small testes, azoospermia,
other toxic substances, normal or small penis, gynecomas-
chemotherapy and hypogo- tia, and minimal secondary sex char-
nadotropic hypogonadism acteristics) but stature is normal

b. Incidence: 1/9,000 live births


C. Infertility associated with chromosomal and intelligence is normal

1. Klinefelter’s syndrome: c. These patients have hyperg-


anomalies:

a. Abnormal number of X chromo- onadotropic hypogonadism with


somes with primary gonadal insuffi- elevated levels of FSH and LH, and
ciency normal or low testosterone
b. Frequency: 1/1,000–14,000 d. Biopsy reveals decreased numbers
surviving newborns; 1/100 patients of germ cells with or without Sertoli
in mental institutions, and 3.4/100 cell-only pattern. Leydig cells are
infertile men not quite as “hyperplastic” as in
c. Karyotype: 47 XXY (80%); Klinefelter’s
remaining 20% have chromosomal e. Etiology is controversial
mosaicism with at least 2 X chromo- 3. 47 XYY syndrome:
somes a. Patients tend to be tall and may dis-
d. Most common clinical findings are: play learning and behavioral abnor-
• Eunuchoid phenotype with malities
increased stature (dispropor- b. Incidence: 1.5/1,000 live births;
tionate lengthening of the lower 3/100 men in mental institutions and
extremities) prisons
• Incomplete virilization c. Normal external genitalia and sec-
• Gynecomastia, often bilateral ondary sex characteristics, but
• Mental retardation decreased fertility
e. External genitalia are normally d. Biopsy reveals mixed tubular
developed with small to normal atrophy (Sertoli cell-only tubules
testes admixed with hypospermatogenic
f. Histology: classically, see small tubules)
e. Serum testosterone levels are
normal, as are LH levels
hyalinized seminiferous tubules

of Leydig cells (the Leydig cells only 4. Down’s syndrome (trisomy 21)
with pseudoadenomatous clusters

appear to be increased due to the a. Patients usually have cryp-


decreased testicular volume) torchidism, small testes, and
g. Increased incidence of extrago- hypoplasia of the penis and scrotum
nadal germ cell tumors, while tes- b. Adults have azoospermia or
ticular germ cell tumors are rare oligospermia secondary to a primary
• Most occur in the mediastinum testicular deficiency
(70%) followed by the pineal c. FSH and LH levels are elevated
gland, CNS and retroperitoneum while testosterone is normal or low
• Most frequent types: teratoma 5. Prader-Willi syndrome
and choriocarcinoma; embry- (chromosome 15p11-12):
onal and seminomas are rare a. Characterized by hypogonadism,
h. Often associated with panhypopitu- obesity, muscular hypotonia, mental
itarism or incomplete hypopitu- and physical retardation, and
itarism acromicria

CHAPTER 6: CYTOLOGY OF THE UROGENITAL TRACT 233


b. Incidence: 1/25,000 live births b. Complications: testicular infarct,
(60% more common in males) scrotal pyocele and chronic draining
c. FSH, LH, testosterone and estradiol scrotal sinus
levels are low 3. Granulomatous
d. Penis and testes are hypoplastic and
most patients have cryptorchidism • The incidence of tuberculous
a. Tuberculosis:

e. Biopsy of adults with this syndrome orchoepididymitis has recently


demonstrates infantile morphology increased with the advent of
AIDS
• Usually associated with involve-
D. Infertility resulting from physical and

1. Occupational exposure: carbon ment of the GU tract elsewhere,


chemical agents:

disulfide (production of rayon); especially prostate involve-


dibromochloropropane (soil fumigant); ment (which, in turn, is usually
lead (inorganic); oral contraceptive secondary to renal tuberculo-
manufacture; radiation; heat; etc. sis)
2. Cancer therapy: radiotherapy • In children, involvement of the
(especially prepubertal), testis usually represents
chemotherapy (cyclophosphamide, hematogenous spread from pul-
procarbazine), surgery monary infection
• Tuberculous orchoepididymitis
occurs predominantly in adults
(72% are >35 years, 18% are >65
3. Inflammation and Infection

years of age)
1. Viral -- usually due to mumps virus and • Symptoms: scrotal pain and tes-
A. Orchitis:

Coxsackie B virus. Mumps orchidoepi- ticular enlargement; fever and


didymitis: constitutional symptoms may be
a. Complicates 14%–35% of adult infrequent or absent altogether
mumps cases, and is bilateral in • Histology: caseating and non-
20%–25% caseating granulomas with acid-
b. Epididymal involvement alone fast bacilli
is rare
c. Clinical symptoms appear • Syphilitic orchitis may be con-
b. Syphilis:

4–6 days after parotiditis genital or acquired


symptoms, but may also occur • In congenital, both testes are
without parotid involvement enlarged at birth (secondary
d. Histology: multifocal acute syphilis); in adults, acquired
inflammation of interstitium and syphilitic orchitis represents ter-
seminiferous tubules tiary stage of disease
e. With time, testes shrink, and bilat- • Early in disease, painless testicu-
eral involvement usually results in lar enlargement occurs
infertility • Histology: early lesions consist
f. Incidence is low in childhood of predominantly plasmacytic
2. Bacterial -- usually associated with inflammation beginning in the
E. coli epididymitis mediastinum testis and testicular
a. Histology: tubules covered up with septa, and later extending into the
intense acute inflammation with or seminiferous tubules
without abscess formation (usually * Tubules slough their epithe-
seen with E. coli, strep, staph, pneu- lium and become sclerotic
mococci, Salmonella enteritidis, and * Vessels demonstrate endar-
Actinomyces israelii) teritis obliterans

234 EDUCATIONAL REVIEW MANUAL IN UROLOGY


Testis II: Germ Cell Tumors

* Small gummas may


be present
1. General

• Gummatous orchitis is character- A. 95% of testicular tumors are of germ cell


ized by the presence of one or derivation and usually occur in young men
multiple well-delineated gray- B. Rarely, supporting and interstitial cells can
yellow necrotic lesions that histo- give rise to sex cord-stromal tumors and
logically show ghosts of seminif- hamartomatous lesions (discussed in next
erous tubules with inflammatory section)
infiltrate of lymphocytes, plasma
cells, and a few giant cells 1. Testicular neoplasms generally
C. Patterns of Metastases

• Special stains (Warthin-Starry) metastasize to the retroperitoneal


may demonstrate spirochetes lymph nodes first
2. Tumors may metastasize via lymphatics
• Preferentially affects young (seminomas), hematogenously
c. Sarcoidosis:

black adults (choriocarcinoma), or both (embryonal)


• GU involvement is unusual but
typically asymptomatic 1. Occur almost exclusively in young men
D. Epidemiology:

• Testicular sarcoidosis is usually (peak age = 30 years), with the one


unilateral and nodular exception being spermatocytic semi-
• Histology: noncaseating noma
granulomas – but diagnosis of 2. Incidence is highest among Caucasian
exclusion so must do bug stains males; higher frequency in upper
• Important differential diagnosis socioeconomic class
is seminoma, which may show an 3. Risk factors:
intense sarcoid-like reaction;
also: spermatocytic granulomas (3.5–5x increased risk)
a. Cryptorchidism

may look like sarcoid • If unilateral, the contralateral


testis is also at increased risk but
1. Testicular involvement occurs in 12% of not to as great a degree as the
B. Malakoplakia:

cases of malakoplakia of the GU tract actual cryptorchid testis


2. Grossly: testis enlarged with yellow- • 2%–8% of patients with cryp-
brown parenchyma, often with abscesses torchid testes intratubular germ
3. Histology: as always, look for von cell neoplasia of the unclassified
Hansemann histiocytes and Michaelis- type (IGCNU), and at least 50%
Gutmann bodies of those men go on to develop
germ cell tumors within 5 years

(5-10x increased risk)


4. Testicular Infarcts b. Prior testicular germ cell tumor

A. Torsion of the spermatic cord is the most • 2%–5% of patients with one
frequent cause of testicular infarct germ cell tumor will develop a
B. Trauma and lesions of the spermatic cord tumor in the contralateral testis
may also result in infarction (especially if the second testis
C. Clinical symptoms mimic testicular tumor has biopsy-proven IGCNU)
D. Histology: ghost-like appearance of • 50% develops within 3–5 years
parenchyma – outlines of tubules and sup- of the first tumor, but delays of
porting structures, but has a generalized over 10 years have been reported
washed-out or diffusely eosinophilic appear- • The second tumor may have the
ance (coagulative necrosis) same or different histology from
the first tumor

CHAPTER 6: CYTOLOGY OF THE UROGENITAL TRACT 235


• Chemotherapy given for the first 1. Neoplastic cells usually appear at the
tumor decreases the risk of tumor basilar aspect of the tubules
development in the second testis 2. Clear cytoplasm, large cells, prominent
nucleoli, and hyperchromatic nuclei
relative) (5.5x increased risk) 3. Often spread to rete testis in a pagetoid
c. Family history (first-degree

• These patients also have a greater fashion, intermixing with and usually
incidence of bilateral tumors lifting up nonneoplastic epithelium
(8%–14% frequency vs 2%–5% 4. The cells are PLAP (placental alkaline
frequency in patients without a phosphatase) positive
family history of germ cell D. Prognosis: 50% proceed to invasive germ
tumors) cell tumors within 5 years

mosome (50x increased risk)


d. Gonadal dysgenesis with a Y chro-

• Especially 46XY gonadal dysge-


3. Classic Seminoma

nesis (Swyer’s syndrome), mixed A. Most common form of pure testicular germ
gonadal dysgenesis, and dysge- cell tumor, accounting for approximately
netic male pseudo- 50% of germ cell tumors
hermaphroditism
• 25%–30% of these patients 1. Average age is 40 years (10 years later
B. Clinical features:

develop gonadoblastoma than other germ cell neoplasms)


• Gonadectomy is recommended 2. Present with painless testicular mass
in any patient with these syn- 3. Gynecomastia may occur as a result of
dromes since invasive tumors can elevated HCG (usually seen in tumors
develop in childhood with syncytiotrophoblastic giant cells)
4. 30% have metastases at the time of diag-
nosis
e. Androgen insensitivity syndrome

• 5-10% of patients with androgen


(15X increased risk)

insensitivity also develop germ 1. AFP normal


C. Laboratory values:

cell tumors so prophylactic 2. HCG elevated in 10%-–20% of stage I


gonadectomy should be per- patients, and in >25% of patients with
formed by puberty (22% risk of disseminated disease
germ cell tumor in patients >30 3. Levels of HCG >40 IU/L are poor
yrs) prognostic indicator
4. Alcohol and tobacco use, prior vasec-
tomy, and radiation do NOT increase the 1. Gross: tan or cream-colored,
D. Pathologic findings:

risk of germ cell tumors multinodular tumor with bulging


cut surface
2. Micro:
a. Cells are large with clear or lightly
2. Intratubular Germ Cell Neoplasia

A. All forms of testicular germ cell tumor – eosinophilic cytoplasm and well-
except for spermatocytic seminoma – arise
from a common precursor known as vesicular nuclei generally contain 1
defined cytoplasmic membranes;

intratubular germ cell neoplasia or 2 prominent nucleoli


B. Gross: testis may be unremarkable or may b. Sheet-like growth pattern inter-
be atrophic and fibrotic rupted with branching fibrous septa
C. Micro: proliferation of malignant germ
cells, which may be undifferentiated or may
containing inflammatory (lym-

resemble specific neoplastic types (embry-


phocytic) infiltrate

onal, etc.)

236 EDUCATIONAL REVIEW MANUAL IN UROLOGY


c. Mitoses may be numerous → used 3. Histologically: cord-like or nested
to be interpreted as an anaplastic fea- growth pattern with edematous cystic
ture if there were > 3/hpf, but now areas
that term is no longer used (no prog- 4. A lymphocytic infiltrate is usually
nostic difference) not seen
d. Septa may render a nesting or cord- 5. 3 major cell types:
like appearance to the tumor at low a. Small, lymphocyte-like cell
power (6–8 µm)
e. Syncytiotrophoblasts (large, multi- • Smudged, degenerated appear-
nucleated cells) may be scattered ance with minimal cytoplasm
through some tumors, and usually b. Intermediate-size cell (15–20 µm)
correlate with elevated HCG levels • Round nucleus with granular or
f. Intratubular spread into the rete testis filamentous (spireme) chromatin
occurs in pagetoid fashion and scant cytoplasm
3. Special studies: c. Giant cells (50–100 µm)
a. Cells are PLAP and CD117 • May be multinucleated or
(c-kit) positive uninucleate, and often have
spireme chromatin as well
1. Early seminoma (stage I or II):
E. Treatment and Prognosis:

orchiectomy and radiation to paraaortic 1. PLAP negative


C. Special Studies:

and paracaval nodes; cure rate >95%


for stage I 1. Only 1 credible case of a metastasizing
D. Treatment and Prognosis:

2. Most patients with non-bulky retroperi- spermatocytic seminoma (other cases


toneal disease are also cured (87%) are most likely misdiagnosed testicular
3. Chemotherapy is recommended for lymphomas)
patients with “bulky” retroperitoneal 2. Orchiectomy alone is generally curable
disease (defined as mets >5–10 cm in 3. Adjuvant therapy is not warranted, and
diameter) → cure rate is still high in this may even be harmful
group (80%)
4. Prominent lymphocytic reaction in the
tumor is good prognostic feature
5. Embryonal Carcinoma

A. Usually identified as a component of mixed


germ cell tumors (87%) rather than in pure
form
4. Spermatocytic Seminoma

1. Represents 1%–2% of testicular germ 1. Average age: 30 years; extremely


A. Clinical Features: B. Clinical Features:

cell tumors rare in childhood


2. Occurs only in the testis, usually on the 2. Mets are clinically evident in 40% of
right side patients at the time of diagnosis, and an
3. Not associated with cryptorchidism additional 20%–30% of patients will
4. Average age is 52–58 years (vs 40 years have evidence of metastatic disease at
for classic seminoma) staging
5. Whites > African Americans and Asians C. Laboratory values: AFP usually not ele-
6. All serum markers (AFP, HCG, LD) are vated in pure embryonal carcinoma, but may
negative be elevated if there is an element yolk sac
tumor; HCG may be elevated in 60% of
1. Multinodular fleshy white tumor with cases (again, correlates with the presence of
B. Pathologic features:

mucoid and cystic areas, ± hemorrhage syncytiotrophoblasts in the tumor); LDH and
2. Paratesticular extension may uncom- PLAP can also be elevated
monly occur

CHAPTER 6: CYTOLOGY OF THE UROGENITAL TRACT 237


B. Laboratory values: significantly elevated
1. Poorly circumscribed gray-white tumor AFP (hundreds to thousands of ng/ml) as
D. Pathologic features:

with hemorrhage and necrosis opposed to minor elevations that may be


2. May appear solid or with gland-like seen in embryonal or teratoma
areas
3. Prominent areas of necrosis 1. Children: solid gray-white tumor
C. Pathologic Features:

4. Cells have abundant eosinophilic cyto- nodules with myxoid or gelatinous


plasm and pleomorphic nuclei with cut surface
prominent nucleoli 2. Adults: heterogeneous appearance with
hemorrhage and necrosis, because usu-
1. CD30 and cytokeratin positive ally part of a mixed germ cell tumor
E. Special studies:

F. Treatment and Prognosis: (MGCT)


1. Stage I disease: 40% relapse, but most 3. Eleven patterns have been described but
can be salvaged with chemotherapy the most common are:
a. Treatment: orchiectomy ± lymph a. Microcystic (honeycomb, reticular,
node dissection vs careful surveil- vacuolated) – most common -- intra-
lance cellular vacuoles create attenuated
b. Survival of 97% lengths of cytoplasm that imparts a
2. Stage II disease with non-bulky spiderweb-like appearance; often
retroperitoneal involvement seen in conjunction with the myxo-
a. Treatment: orchiectomy and lymph matous and solid patterns
node dissection ± adjuvant therapy b. Endodermal sinus (perivascular):
b. >95% survival consists of a central vessel rimmed
3. Stage II disease with bulky disease or by fibrous tissue which is, in turn,
more advanced stage surrounded by malignant epithelium
a. Treatment: as above, only with (Schiller-Duval bodies or glomeru-
chemotherapy added
b. Survival is 70%–80% c. Myxomatous -- stellate or spindle
loid bodies)

4. Prognostic factors in patients with non- cells dispersed in a myxomatous


seminomatous germ cell tumors: tumor stroma that stains very palely with
stage, extent of tumor marker elevation, H&E; often seen in conjunction
age of patient (older is worse), the pres- with microcystic pattern
ence of choriocarcinoma, and the prolif- d. Solid: may resemble seminoma
erative index by flow cytometry because the cells have pale cyto-
plasm and well-defined borders;
however, yolk sac won’t have the
fibrovascular stroma with the lym-
6. Yolk Sac Tumor

phoid infiltrates
1. Most common tumor in prepubertal • Other solid patterns may resem-
A. Clinical Features:

children (average age = 18 months) ble blastema (small round blue


2. Postpubertal tumors occur at average cells in solid sheets), but these
age of 25–30 years areas generally are admixed with
3. Childhood yolk sac tumors are not asso- more typical areas of yolk sac
ciated with cryptorchidism tumor
4. Present with painless testicular mass 4. A general feature of yolk sac tumor is the
5. Evidence of metastases is very rare at deposition of extracellular basement
diagnosis membrane (92% of cases), which consist
of eosinophilic bands of matrix
deposited between the neoplastic cells

238 EDUCATIONAL REVIEW MANUAL IN UROLOGY


5. Another helpful (but not diagnostic) fea-
ture is the presence of hyaline globules 1. Histologically, teratomas consist of a
B. Pathologic Features:

that are PAS positive, and may occasion- variety of somatic-type tissues, most
ally be AFP positive commonly including cartilage, smooth
and skeletal muscle, neuroglia, enteric-
1. Majority are positive for AFP type glands, squamous or respiratory
D. Special Studies:

epithelium, and urothelial islands


1. Adults: treated like embryonal (see pre- 2. Gray-white cartilaginous areas may also
E. Treatment and Prognosis:

vious section for guidelines) be present


2. Children: 80% are Stage I, and are 3. Malignancy may arise within a teratoma,
treated by orchiectomy with close although the phrase “malignant transfor-
surveillance (no retroperitoneal lymph mation” may be somewhat misleading,
node dissection); 5-year survival = 91% since it implies that teratomas are not, by
3. Presence of metastatic yolk sac tumor themselves, malignant
bodes poorly for prognosis, as the a. An overtly malignant component in
metastatic lesions are typically resistant a teratoma may have an immature or
to chemotherapy mature appearance – i.e., invasive
adenocarcinoma of GI type, squa-
mous cell carcinoma and undifferen-
tiated carcinoma are examples of
7. Teratoma

mature types; PNET, embryonal


1. Children: rhabdomyosarcoma, or Wilm’s
A. Clinical Features:

a. Second most common form of tes- tumor-like elements would be


“immature” types
(yolk sac is the most common), b. Diagnosis pretty much depends on
ticular germ cell tumor in children

accounting for 14%–18% of cases finding overgrowth of the remainder


b. Mean age = 20 months; >4 years is of the teratoma by the malignant
unusual elements (carcinoma, sarcoma, etc.)
c. Associated with a variety of c. The prognostic significance of
congenital anomalies: an overtly malignant element in
• Hemihypertrophy teratoma is still uncertain
• Spina bifida
• Retrocaval ureter 1. Staining is usually as would be expected
C. Special Studies

• Hernia for the given somatic types of cells


2. Adults: usually occurs as component of (cytokeratin positive in squamous
a MGCT and are present in more than epithelium, etc.)
half of all MGCT and over 25% of all 2. AFP may be present within enteric or
nonseminomatous germ cell tumors respiratory-type epithelium; thus, pure
3. Metastatic potential increases with adult teratoma may be associated clinically
population even when the histology is of with mildly elevated AFP levels
a pure mature teratoma (probably
because the tumor developed along the 1. Dermoid and epidermoid cysts (benign)
D. Differential Diagnosis

IGCNU → seminoma → teratoma → must be distinguished from mature ter-


mature teratoma pathway) atoma (malignant)
a. Dermoid cyst: epidermal-lined cyst
with adnexal structures in the wall
b. Epidermoid cyst: epidermal-lined
cyst lined with keratinizing squa-
mous epithelium but lacking associ-
ated adnexal structures

CHAPTER 6: CYTOLOGY OF THE UROGENITAL TRACT 239


2. These cysts show no atypia, are not asso- 5. Metastases particularly like the lung,
ciated with immature elements and are brain, and GI tract, and spread
generally not associated with IGCNU hematogenously
6. Patient age: 2nd to 3rd decades
1. Prepubertal patients are cured by B. Laboratory values: markedly elevated HCG
E. Treatment and Prognosis:

orchiectomy → secondary hormonal manifestations, such


2. Postpubertal patients with mature ter- as gynecomastia and thyrotoxicosis (because
atoma have a guarded prognosis – 70% HCG cross-reacts with TSH)
survival (with or without seminomatous
component) 1. Testis may show hemorrhagic nodule or
C. Pathologic features:

3. Patients may die of metastatic disease have residual scarring from regression of
having nonteratomatous components the original lesion
2. Histologically: classical combination of
syncytiotrophoblasts (multinucleated
cells with smudged, degenerate-appear-
8. Choriocarcinoma

ing nuclei) admixed with cytotro-


1. Uncommon component of mixed germ phoblasts (mononuclear cells with pale-
A. Clinical features:

cell tumors (15% of cases) staining cytoplasm)


2. Pure choriocarcinoma is extremely rare 3. Usually has hemorrhage surrounding the
(0.3% of testicular tumors) tumor and the central portion of the neo-
3. Most patients present with symptoms of plasm is invariably necrotic
metastatic disease, unlike the other
tumors in which a palpable testicular 1. HCG stains are strongly positive in the
D. Special Studies:

mass is the most common presenting syncytiotrophoblasts and less so in the


complaint cytotrophoblasts
4. Testicular tumor may remain occult even E. Treatment and Prognosis: Has a fairly
after the metastasis has been identified aggressive course, although patients may
achieve substantial tumor-free survival with
chemotherapy

Table 3

Cytokeratin PLAP CD117/ckit CD30 AFP b-HCG Oct 3/4

Seminoma - + + - - - +

Spermatocytic - -, rare -, rare - - - -


seminoma

Yolk sac tumor + - - - + - -

Embryonal carcinoma + - - + - - +

Choriocarcinoma - - - - - + -

Teratoma Variable - - - - - -

240 EDUCATIONAL REVIEW MANUAL IN UROLOGY


Testis III: Non-Germ Cell Tumors

9. Mixed Germ Cell Tumors 1. Leydig Cell Tumor

A. Classified as nonseminomatous tumors, A. Sex cord-stromal tumor that comprises


even if seminoma is one of the components 1%–3% of testicular tumors; some seen in
B. Comprise 30% of all germ cell tumors, and association with Klinefelter’s and with cryp-
69% of all nonseminomatous germ cell torchid testes
tumors of the testis B. 3% are bilateral
C. May produce endocrine symptoms due to
1. Patients have the same features as those increased production of androgens and/or
C. Clinical Features:

with nonseminomatous germ cell tumor, estrogens


and most present with a testicular mass D. Presenting complaint in adults: testicular
2. Average age: 28 years (slightly younger mass and gynecomastia
than pure seminomas) E. Presenting complaint in children: preco-
3. AFP and HCG may be elevated in 60% cious pseudopuberty (“pseudo” because they
and 55%, respectively, of these patients get growth of pubic hair and penis but no
D. Gross features: MGCT often have variegated spermatocytic maturation in the non-neo-
cut surface with hemorrhagic and necrotic plastic testis) – symptoms typically regress
areas with tumor removal
E. Histologic features: areas demonstrate F. Gross features: Solid brown nodules
embryonal, seminomatous, teratomatous, within the testicular parenchyma; average
yolk sac or choriocarcinomatous differentia- size 3.0 cm
tion
F. Treatment and Prognosis: MGCT patients are 1. Tumor cells have well-defined outlines
G. Microscopic features:

managed like non-seminomatous patients and abundant eosinophilic cytoplasm


2. May have numerous eosinophilic
crystalloids (crystals of Reinke)
3. Tumor cells may have marked
10. Special Stains (Table 3)

A. Immunostains are commonly used to con- pleomorphism with giant atypical forms
firm the diagnosis of a germ cell component 4. Growth pattern is generally solid
and include the following: H. Clinical behavior: 10% are malignant –
malignancy may be suggested by high
mitotic rate, necrosis, large size (average
7.5 cm), and blood vessel invasion.
11. Regression of Germ Cell Tumors

A. Primary germ cell tumors were, at one time, I. Treatment: orchiectomy (+ lymph node dis-
thought to arise in the retroperitoneum; section if suspect malignant tumor)
however, increasing evidence seems to sug-
gest that most likely these retroperitoneal
tumors represent metastatic spread of pri-
2. Sertoli Cell Lesions

mary testicular lesions that have regressed


by the time of diagnosis. 1. Seen in 50% of patients with cryptorchid
A. Sertoli cell hyperplasia

B. Examination of the testis generally reveals a testes but may also be seen in normal
scar-like focus where the tumor once was testes
C. Choriocarcinoma is particularly likely to 2. Frequency decreases with age
produce metastatic spread in the face of a 3. May be a manifestation of abnormal
“burned out” testicular tumor, although sexual maturation.
regression may be seen in any of the germ
cell types. 1. Often seen in patients with testicular
B. Sertoli cell adenomas (“Pick’s adenoma”)

feminization
2. Microscopic features: composed of elon-
gated tubules lined by Sertoli-like cells

CHAPTER 6: CYTOLOGY OF THE UROGENITAL TRACT 241


1. May arise in the normal testes and are 1. May be seen as a component of a germ
C. Sertoli cell tumors C. Carcinoid tumors

associated with gynecomastia in 30% cell tumor, as a primary neoplasm arising


of cases from autochthonous neuroendocrine
2. Gross: well-circumscribed gray-white cells, or as a metastasis from a GI pri-
nodules with firm and focally cystic mary
areas 2. Rarely occur in children
3. Microscopic features: tubules lined by 3. Gross: firm, yellow-brown nodules
Sertoli-like cells 4. Micro: ribbon and festoon-like growth
• Solid areas may be confused pattern of argyrophil or argentaffin cells
histologically with seminomas 5. Most cured by orchiectomy
• Positive staining for vimentin, 6. Usually NOT associated with carcinoid
keratin, inhibin syndrome
4. 10% malignant with metastatic involve-
ment of paraaortic and iliac lymph nodes
(suspect with high mitotic rate, pleomor-
phic cells, large tumor size, and necrosis)
5. Treatment: orchiectomy (+ excision of
metastatic lesions, as indicated)
6. Adjuvant chemo and radiation has been
helpful in malignant tumors

3. Other Testicular Tumors

1. 5% of all testicular lesions, but is the


A. Lymphoma

most common testicular tumor in elderly


patients
2. 50% of patients with bilateral testicular
masses will have lymphoma
3. Most cases are large non-cleaved or
immunoblastic B-cell lymphomas
4. Most patients have systemic disease by
the time they develop testicular involve-
ment, and the prognosis is poor
5. Treatment: orchiectomy for stage I and
II disease
6. Micro: tubules are normal but the inter-
stitium is infiltrated by sheets of lympho-
cytes (small round blue cells) →
LCA/CD45 positive if in doubt

1. Not uncommonly seen in children,


B. Leukemia:

particularly as the first site of relapse


following bone marrow remission
2. Microscopic rate of involvement is
approximately 20%
3. Radiation may help control testicular
involvement, but most patients will go
on to develop bone marrow relapse

242 EDUCATIONAL REVIEW MANUAL IN UROLOGY


Testicular Adnexae

4. Special stains: positive for soybean


agglutinin
1. Epididymal Tumors

5. DDX: renal cell carcinoma (which is


1. Most common neoplasm of the negative for soybean agglutinin)
A. Adenomatoid tumor

epididymis
2. Presents in patients in 3rd or 4th decades
3. Clinically presents as a mass, sometimes
2. Rete Testis: Adenocarcinoma

associated with pain A. Very rare


4. Gross: small, firm, grayish-white nodule B. May be difficult to distinguish from malig-
± small cysts nant mesothelioma of the tunica vaginalis
5. Micro: unencapsulated lesion that may
involve adjacent testis 1. Involvement centered on hilum of testis
C. Diagnostic criteria:

a. Proliferation of cells ranging from 2. Lack of direct extension through the


cuboidal to flattened, which form parietal tunica
solid cords with an epithelial appear- 3. Transitions from the tumor to normal
ance admixed with channels having rete testis epithelium
dilated lumina-like vascular struc- 4. No evidence of teratoma
tures 5. Lack of any other primary tumor
b. May have abundant smooth muscle,
desmoplastic response, or be infil-
trated by inflammatory cells
c. Special stains; (+) for keratin and
calretinin
6. May also arise in the spermatic cord,
ejaculatory duct, fallopian tubes and
uterus
7. Felt to arise from mesothelial cells
8. Often seen in association with chronic
inflammation and fibrosis, so may repre-
sent a peculiar form of nodular mesothe-
lial hyperplasia
9. Behavior: benign

1. Arises from tunica vaginalis


B. Mesothelioma

2. Most cases are malignant; may occasion-


ally be associated with asbestos exposure
3. Behave aggressively with potential for
late recurrence and metastasis

1. May be seen as part of the Von Hippel-


C. Papillary cystadenoma

Lindau syndrome, in which circum-


stances these tumors are more likely to
be bilateral
2. Gross: measures 1–5 cm and is well cir-
cumscribed, either cystic or solid
3. Micro: papillary infoldings lined by
columnar cells with abundant clear cyto-
plasm

CHAPTER 6: CYTOLOGY OF THE UROGENITAL TRACT 243


244 EDUCATIONAL REVIEW MANUAL IN UROLOGY
Chapter 7:
Renal Physiology and
Pathophysiology
Daniel A. Shoskes, MD, FRCS(C)

Contents

1. Objectives

2. Basic Renal Anatomy

3. Renal Hemodynamic Definitions

4. Control of Vascular Tone

5. Tubular Function

6. Diuretic Effects

7. Disorders of Water and Sodium

8. Tubular Handling of Water

9. Electrolyte Disorders

10. Acid-Base Disorders

11. Disorders of Potassium

12. Renal Tubular Acidosis

13. Effects of Urinary Diversion

14. Postobstructive Diuresis

15. References

16. Questions

CHAPTER 7: RENAL PHYSIOLOGY AND PATHOPHYSIOLOGY 245


1. Objectives 2. Basic Renal Anatomy

The purpose of this chapter is to provide a review of Arteries are end arteries. Veins communicate.
renal physiology and pathophysiology focused on
material that may be covered on Urology Board • Blood enters the kidney through the renal arteries,
exams. For Part I of the qualifying exam, renal and divides into progressively smaller arteries:
physiology is often a fertile testing area because the
principles rarely change over time and most
answers are not controversial. For those taking Part
* Interlobar artery

I, pretty much everything in this chapter is fair


game; untestable minutiae have been omitted. For
* Arcuate artery

urologists taking the re-certifying exam, it is highly


unlikely that any renal physiology questions will be
* Interlobular artery

asked, although clinically relevant pathophysiology • Until it enters the glomerular capillary through
may be covered if it directly impacts urologic care. the afferent arteriole
As a minimum, I would recommend reviewing GFR
(as it may relate to further nephron loss from Basic unit of the kidney is the nephron.
nephrectomy), emergency treatment of hyper-
kalemia, post-obstructive diuresis, and effects of • Blood enters the glomerular capillary through
bowel segments in the urinary tract. the afferent arteriole

• The glomerulus is a tuft of capillaries interposed


between two arterioles (afferent and efferent);
located in the renal cortex

• Blood exits the glomerular capillary through the


efferent arteriole

• Figure 1 shows renal architecture demonstrating


processes of filtration, reabsorption, secretion,
and excretion in the nephron

Figure 1

Renal architecture demonstrating


processes of filtration, reabsorption,
secretion, and excretion in the nephron

Adapted from Costanzo LS. BRS Physiology. 3rd ed.


Baltimore, MD: Williams & Wilkins; 2003:171.

246 EDUCATIONAL REVIEW MANUAL IN UROLOGY


3. Renal Hemodynamic Definitions

Renal Blood Flow (RBF) • Balance of hydrostatic and oncotic pressures

• 20%–25% of cardiac output • Hydrostatic pressure controlled by relative

• Blood to kidneys per minute (1200 mL/min)


tonicity of pre- and postglomerular arterioles

• Hyper-renin state will maintain GFR by


• Directly proportional to the pressure difference postglomerular arteriolar constriction
between the renal artery and the renal vein, and
inversely proportional to the resistance of the • GFR may be approximated by creatinine clear-
renal vasculature ance, since creatinine is filtered, not resorbed,
and is minimally secreted
Renal Plasma Flow (RPF)
• Estimates of GFR with blood urea nitrogen
• Plasma flow to kidneys per minute (BUN) and serum creatinine
(670 mL/min)
• When GFR decreases, both BUN and serum
• Varies with hematocrit (RPF=RBF x [1-Hct]) creatinine increase

• Clearance of para-aminohippuric acid (PAH) is • In prerenal azotemia (hypovolemia), BUN


used to measure effective RPF. PAH can also be increases more than serum creatinine and there is a
used to measure RBF resultant increased BUN/creatinine ratio (>20:1)

Glomerular Filtration Rate (GFR) • ACE inhibitors cause vasodilation of both the
afferent and efferent arterioles, and proportion-
• Volume of plasma filtered per minute by the ally more dilation of the efferent arteriole. This
glomeruli (125 mL/min males, 100 mL/min results in a drop in glomerular hydrostatic pres-
females). Note the units! sure and decreased GFR

Filtration Fraction (FF) Clearance

• Filtration of RPF filtered across the glomerular • Indicates the volume of plasma cleared of a
capillaries substance per unit time

• GFR:RPF (about .18–.22) • Units of clearance are mL/min and mL/24 hour

• Thus, about 20% of the RPF is filtered. The • Clearance = [urine concentration x (urine vol-
remaining 80% leaves the glomerular capillar- ume/time)]/plasma concentration
ies by efferent arterioles and becomes the
peritubuar capillary circulation • Most widely used estimate of GFR is the

• See Table 1 for effect of afferent and efferent


24-hour creatinine clearance (CrCl)

arteriole constriction on RPF, GFR, and filtra- * CrCl overestimates GFR by 10%–20% due to
tion fraction tubular secretion of creatinine

Glomerular Filtration Rate (GFR): * See Figure 2 for Cockcroft-Gault formula


Reflects Overall Renal Function for CrCl calculation, and Figure 3 for MDRD
calculation.
• Important function of the kidney is the process
of glomerular filtration

CHAPTER 7: RENAL PHYSIOLOGY AND PATHOPHYSIOLOGY 247


Table 1

Effect of Renal Plasma Flow on Filtration Fraction

Effect RPF GFR FF (=GFR/RPF)

Afferent arteriole ↓ ↓ Remains constant


constriction (no change)

Efferent arteriole ↓ ↑ ↑
constriction

Figure 2

Cockcroft-Gault formula for creatinine clearance

[140 - age] x [Ideal body weight in kg]


CrCl (mL/min) = x 0.85 (women)
Plasma creatinine [mg/dL x 72]

urine Cr % urine vol


CrCl (mL/min) =
Serum creatinine

(total creatinine should be 1 mg/kg/hr)

Figure 3

MDRD Formula: more accurate in patients with renal impairment. One obviously cannot be
expected to calculate by hand in an exam, but one should know the variables used.

Glomerular Filtration Rate (ml/min/1.73m2) =

186 x (PCr)-1.154 x (age)-0.203 x (0.742 if female) x (1.210 if African American)

248 EDUCATIONAL REVIEW MANUAL IN UROLOGY


4. Control of Vascular Tone

• Vascular tone is the net result of vasoconstrictive • Overall action of angiotensin II: increases
and vasodilatory forces, which are essential in
maintaining RBF, GFR, tubular renal function, and
intravascular volume and increases BP via:

systemic blood pressure * Potent vasoconstriction


(efferent arteriole constriction)
Vasoconstrictors
* Release of aldosterone from adrenal cortex
• Endothelin (most potent vasoconstrictor)
* Release of ADH from posterior pituitary
• Angiotensin II
* Stimulates the hypothalamus to increase thirst
• Atrial natriuretic peptide (ANP)

• Vasopressin

• Norepinephrine

Vasodilators

• Prostaglandins (PGE-2)

• Acetylcholine

• Serotonin/bradykinin (NO mediated)

• Glucocorticoids

• Nitric oxide (potent vasodilator)

• Carbon monoxide (potent vasodilator)

Renin-Angiotensin System

• Mechanism

1. Macula densa cells sense decreased →


BP renin is secreted

2. Angiotensinogen cleaved to angiotensin I

3. Angiotensin I cleaved by angiotensin-


converting enzyme (ACE) (from lung) to
angiotensin II

CHAPTER 7: RENAL PHYSIOLOGY AND PATHOPHYSIOLOGY 249


5. Tubular Function

• Maintain appropriate water, acid, and electrolyte • Majority of sodium, chloride, and water is
balance using passive and active mechanisms reabsorbed in the proximal tubule driven
by active forces
• Reabsorb selectively up to 99% of the glomeru-
lar filtrate • HCO3 generated in cell and absorbed with Na+

• Respond to endocrine signals to make necessary • Secretes ammonia, which acts as a buffer for
changes to the fluid prior to excretion (eg, secreted H+
excrete more acid, conserve more water)
Loop of Henle
• See Figure 3 for diagram of renal tubular
organization • Early water and urea permeability, filtrate
becomes hypertonic
Proximal Tubule
• Later Na+-Cl permeability
• Reabsorbs 100% of glucose and amino acids,
90% of bicarbonate, and 80%–90% of inorganic • Final, Na+-Cl actively transported, filtrate
phosphate and water hypotonic

• Solutes active, water passive: water reabsorp- • Creates high interstitial osmolality (highly
tion in the proximal convoluted tubule is a pas-
sive process, driven by the reabsorption of other
hypertonic) which permits maximal urinary

solutes and the subsequent osmotic gradient that


concentration

develops between the lumen and intercellular Thick Ascending Limb


space
• The diluting segment
• Na reabsorption through Na+-H+ and Na+-
solute active transporters • Active transport of NH4 and Na

• Aldosterone and ADH augment Na+


reabsorption
Figure 4

• PGE-2 reduces Na+ reabsorption


Renal tubular organization

Collecting Duct
Osmolality gradient in loop of Henle allows
concentration of urine in the collecting duct

• Provides final touches to Na+, HCO3- and K+

• Na absorbed, K+ secreted (stimulated by


aldosterone)

• H+ secreted based on blood pH

• NH3 secreted into lumen and can trap H+ to


make NH4

• Reabsorption of water is regulated by vaso-


pressin (ADH)

250 EDUCATIONAL REVIEW MANUAL IN UROLOGY


6. Diuretic Effects

• ADH acts on receptors located on the basolateral Thiazide Diuretics


membrane of the principal cells of the collecting
ducts. Induces migration and insertion of water- • HCTZ, chlorthalidone, metolazone
permeable protein channels to the luminal mem-
brane • Mechanism: inhibit Na+ and Cl reabsorption

• If ADH is present, the tubule is permeable to


in distal convoluted tubule, reducing the dilut-

water and water is drawn by hypertonic medulla


ing capacity of the nephron

• Reduce GFR and RBF


• Osmolality of medulla can reach 1200 mOsm
• Decreases urinary calcium
Countercurrent Multiplier
• In diabetes insipidus, they have an
• Ultimate goal is to produce high osmolality in antidiuretic effect

• Hyponatremia may occur with thiazide diuretics


the medulla which is later used for urinary
concentration

* Loop structure generates a longitudinal Loop Diuretics


gradient
• Furosemide, ethacrynic acid, bumetanide
* Thin ascending limb contains active sodium
transporters and is water permeable • Mechanism: inhibit Na/K/Cl cotransporter on

* Thick ascending limb has solute pump and is


the lumina membrane of the thick ascending

water impermeable
portion of loop of Henle

• Increased diuresis and increased excretion of


* Collecting duct is water permeable in the
presence of circulating ADH
Na, K, Cl, Ca and Mg

• Reduce medullary solute content and impair


urinary concentrating and diluting capacity

CHAPTER 7: RENAL PHYSIOLOGY AND PATHOPHYSIOLOGY 251


7. Disorders of Water and Sodium 8. Tubular Handling of Water

Primary Goals Osmolality

• Maintain blood pressure • POsm = 2 x plasma [Na] + [glucose]/18 +


[BUN]/2.8
• Excrete wastes
• The osmolality of plasma is tightly regulated
Mechanisms around a mean of 290 mOsm/kg (+ 5mOsm/kg)

• Thirst • Therefore, kidney is able to adjust the rate of


water excretion over a wide range, generating
• ADH dilute urine when water is abundant and a concen-
trated urine when water is scarce
• Aldosterone
Osmolality Changes

• Loop of Henle creates an osmolality gradient in


the medullary interstitium

• Collecting duct traverses medulla: urine concen-


trated by osmotic water removal when duct wall
made permeable by ADH

Response to Water Loss

• In response to increased plasma osmolality or


decreased arterial circulating volume:

* Increased thirst and increased water intake

* Increased ADH and decreased water excretion

• Leads to water retention

How Water Disorders Develop

• For any electrolyte disorder, you need


two components:

1. Something must change the concentra-


tion (eg, loss of sodium through an NG
tube)

2. Something must interfere with the nor-


mal physiologic response

252 EDUCATIONAL REVIEW MANUAL IN UROLOGY


9. Electrolyte Disorders

Hyponatremia and Hypernatremia Evaluation • Sodium deficit = volume of distribution x body


and Treatment weight (kg) x (125–plasma [Na])

• Two key factors are volume status and urinary • Volume of distribution for men=0.5 and
sodium, which will lead you to the diagnosis women=0.6

1. Determine the volume (water) status of the Hypernatremia


patient
• Disorder of urine concentration with inadequate
2. Determine why the normal compensatory water intake
mechanisms have failed
• Water balance is altered, and total body sodium
3. Therapy is directed at both the cause of the can be high, normal, or low
condition and the water imbalance itself
• In contrast to hyponatremia, which usually
Hyponatremia results from a defect in renal water handling, the

• Water excess relative to extracellular sodium


primary defect in hypernatremia is impaired
water intake

• More commonly, hypernatremia results from


that has not been handled by normal compen-

inadequate fluid intake in the setting of


satory mechanisms of thirst suppression and

increased free water losses


decreased ADH

• Serum sodium >125 mmol/L → Mild hypona-


tremia, and is usually asymptomatic • Geriatric patients are at increased risk because
renal concentrating ability and thirst decline
• Serum sodium <120 mmol/L → With more with age, although the osmoreceptors’ response
severe acute hyponatremia, nausea, headache, to hypernatremia is maintained
confusion, agitation, and incontinence may
develop. Seizures, coma, respiratory arrest and • In acute hypernatremia, the water deficit can be
death can occur with profound acute hypona- replaced relatively rapidly
tremia
• In chronic hypernatremia (>24–48 hours dura-
• Difficult to distinguish between volume deple- tion), plasma sodium should be decreased by
tion and euvolemic hyponatremia on clinical 1–2 mmol/L/hour until symptoms resolve, and
examination. In these cases, measurement of the then the rate of correction slowed so that the
sodium is normalized over the ensuing 24–48
hours
urine sodium concentration can be helpful

• A low urine sodium concentration suggests


decreased effective arterial volume, whereas • Free water deficit = 0.6 x body weight in kg x
in SIADH, the urine sodium is usually [(plasma sodium/140) -1]
>30 mmol/L
Secretion of Acid
• Central pontine myelinolysis can be caused by
rapid correction of hyponatremia • Active Na-H pump

Sodium Deficit • Glomerular filtration of buffers

• Hypovolemic patient has a sodium deficit • HCO3, HPO4


greater than their water deficit
• Ammonia (NH3) secreted in tubule combines to
form nondiffusible NH4 (ammonium ion)
CHAPTER 7: RENAL PHYSIOLOGY AND PATHOPHYSIOLOGY 253
10. Acid-Base Disorders

• Normal arterial blood pH ranges from * Third, check compensation formulas for
7.37–7.43, maintained by lungs (pCO2) coexisting (mixed acid-base) disorders
and kidneys (HCO3)
• If compensation not appropriate, suspect
• Sudden changes tempered by buffers in blood mixed disorder

* First, check the pH for acidosis or alkalosis * When you see a metabolic acidosis,
(see Figure 4)
(Na-[Cl+HCO3]) looking for “extra” anions
always calculate the anion gap

* Second, check the pCO2 to determine if (normal is 10–12)


it is a primary respiratory disorder (vs
metabolic) Simple Acid-Base Disorders

• Metabolic acidosis—a primary process • Metabolic Acidosis


that causes [HCO3] to fall
* Expected pCO2 = 1.5 x [HCO3-] + 8 ± 2
• Metabolic alkalosis—a primary process (Winter’s formula)
that causes [HCO3] to rise
• Metabolic Alkalosis
• Respiratory acidosis—a primary process
that causes pCO2 to rise * Expected pCO2 = 6 mm Hg per 10 mEq/L
in HCO3
• Respiratory acidosis—a primary process
that causes pCO2 to fall • Respiratory Acidosis

* Acute Expected HCO3 =1 mEq/L for each


10 mm pCO2

Figure 5

Acid-base evaluation algorithm

Check arterial pH

pH<7.4 pH >7.4
Acidosis Alkalosis

PCO2>40 mmHg PCO2 <40 mmHg PCO2 <40 mmHg PCO2 > 40 mmHg
Respiratory acidosis Metabolic acidosis Respiratory alkalosis Metabolic acidosis
with compensation with compensation
Hypoventilation Hyperventilation
Vomiting, diuretic use,
Check anion gap
antacid use,
Anion gap=Na+ -
hyperaldosteronism
(Cl- + HCO3-)

anion gap Normal anion


gap (8-12 mEq/L)

254 EDUCATIONAL REVIEW MANUAL IN UROLOGY


* Chronic Expected HCO3 = 3.5 mEq/L for • See Table 2 for summary of metabolic disor-
each 10 mm Hg pCO2 ders, and Table 3 for summary of acid-base pri-
mary disorders and compensatory responses
• Respiratory Alkalosis
Note that presently, everyone uses nomograms
* Acute Expected HCO3 = 2 mEq/L for each rather than calculating expected compensation by
10 mm Hg pCO2 hand, and so these compensation formulas are prob-
ably of historic interest only (See Figure below)
* Chronic Expected HCO3- = 5 mEq/L for each
10 mm Hg pCO2

Metabolic Disorders

• When normal metabolism is impaired, acid


forms (eg, poor blood supply stops oxidative
metabolism and lactic acid forms)

• Metabolic disorder has a raised H+ level with a


normal pCO2. To maintain the equilibrium, the
high H+ would cause a reciprocal fall in the
HCO3-. However, in practice, respiratory com-
pensation occurs and lowers the pCO2, which
reduces both the H+ and the HCO3- (ie,
metabolic acidosis lowers the bicarbonate level
and respiratory compensation lowers it further).

Table 2

Summary of Metabolic Disorders

Metabolic Acidosis Metabolic Alkalosis

With elevated anion gap Chloride-responsive


Ketoacidosis (diabetes, chronic alcoholism) Vomiting or nasogastric drainage
Lactic acidosis Surreptitious laxative abuse
Renal failure Diuretics
Intoxication Posthypercapnic states

With normal anion gap Chloride-resistant


GI alkali loss (diarrhea, ileostomy, colostomy) Severe Mg or K deficiency
Renal tubular acidosis Diuretics (thiazides or loop diuretics)
Interstitial renal disease Hypermineralocorticoidism
Ureterosigmoidostomy, ureteroileal conduit Licorice, chewing tobacco
Acetazolamide therapy Inherited disorders
Ingestion of ammonium chloride

CHAPTER 7: RENAL PHYSIOLOGY AND PATHOPHYSIOLOGY 255


Table 3

Summary of Acid-base Primary Disorders and Compensatory Responses

pH PCO2 [HCO3-] Compensatory


Response

Metabolic acidosis ↓ ↓ ↓ (primary disorder) Hyperventilation

Metabolic alkalosis ↑ ↑ ↑ (primary disorder) Hypoventilation

Respiratory acidosis ↓ ↑(primary disorder) ↑ Renal [HCO3-]


reabsorption

Respiratory alkalosis ↑ ↓(primary disorder) ↓ Renal [HCO3-]


secretion

Primary Disorder * Pure respiratory acidosis (high pCO2) causes


molecules of CO2 and water to form carbonic
• pH = 7.18, pCO2 = 14, HCO3 = 14, acid which ionizes to increase both HCO3-
Na = 140, Cl = 104 and H+. The H+ changes slightly due to
buffering of H+ by hemoglobin. At this raised
* pH is low → therefore acidosis pCO2, the kidney compensates by eliminat-
ing H+. To maintain chemical equilibrium the
* Bicarb is low → therefore metabolic acidosis HCO3- rises further (ie, respiratory acidosis
raises the bicarbonate level and metabolic
* pCO2 is low→ expected compensation compensation raises it further)

* Predicted pCO2 compensation is 1.5*4+8 = 14 • Respiratory alkalosis

* Anion gap 140-(104+14) = 22 → * Due to hyperventilation


(elevated anion gap)
* Resulting from anxiety, fever, pain,
• Therefore, anion gap metabolic acidosis with septicemia, iatrogenic (ventilator settings)
appropriate compensation
* May have tetany, paresthesia
Respiratory Disorders
Mixed Disorder
• Respiratory acidosis
• pH = 7.42, pCO2 = 65, HCO3 = 41,
* When breathing is inadequate, carbon dioxide Na 143, K 3.1, Cl 88
(a respiratory acid) accumulates. The extra
CO2 molecules combine with water to form * pH is normal
carbonic acid (H2CO3) which contributes to
an acidic pH * Bicarb is 17 too high → metabolic alkalosis

* Due to insufficient respiration, so consider * pCO2 should be 1.7*6+40 = 50.2


central, mechanical and obstructive causes
* Measured pCO2 is 65
* Headache and drowsiness lead to coma and
death * Too much pCO2 retained, so also has
respiratory acidosis
256 EDUCATIONAL REVIEW MANUAL IN UROLOGY
11. Disorders of Potassium

• Mostly intracellular * T-wave flattening

• Serum levels do not reflect total body content in * Appearance of U waves


disease states
Hyperkalemia
• K+ driven into cells by insulin, bicarbonate,
beta-agonists • Usually reflects decreased renal excretion or

• Changes in dietary intake handled by intracellu-


shift out of cells (acidosis)

lar stores and urinary excretion • GI bleed or hemolysis will often exacerbate

• Excretion promoted by aldosterone, high distal • Most common causes include: renal failure,
Na+ load, chronic acidosis drugs (K+ sparing diuretics, ACE inhibitors,

Hypokalemia
beta blockers), chronic acidosis (RTA type 4)
and hyperaldosteronism

• Usually increased loss (GI, urine) or • Usually asymptomatic until cardiac changes

* Short QT, peaked T waves, ventricular


intracellular shift

• Alkylosis = Low K+ arrhythmia

• Iatrogenic causes common • Mild increase→ reverse predisposing cause

• Diuretics, laxatives, amphotericin, • Moderate→ use binding agents (Kayexalate®)

• Severe→ give calcium/insulin/glucose drip;


theophylline, postobstructive diuresis

• Metabolic causes include: hyperaldosterone dialysis


states, Cushing syndrome
EKG Changes in Hyperkalemia
• Tachycardia, heart block, ST depression
• EKG changes have a sequential progression
• Treat underlying cause, replace of effects, which roughly correlate with the
K+ (40 mEq/hr IV) potassium level

• Magnesium depletion promotes potassium wast- • EKG findings:


ing; important to correct Mg if repleting K+
* Early changes of hyperkalemia include
EKG Changes in Hypokalemia peaked T waves, shortened QT interval, and

• The EKG in hypokalemia may appear normal or


ST segment depression

may have only subtle findings immediately * These changes are followed by bundle branch
before clinically significant dysrhythmias blocks causing a widening of the QRS com-

* Ventricular dysrhythmia
plex, increases in the PR interval, and
decreased amplitude of the P wave

* Prolongation of QT interval * These changes reverse with appropriate


treatment
* ST-segment depression

CHAPTER 7: RENAL PHYSIOLOGY AND PATHOPHYSIOLOGY 257


Treatment of Hyperkalemia • Acts quickly and can be lifesaving, thus
they are the first-line treatment for severe
• Institute treatment based on clinical severity of hyperkalemia when the ECG shows sig-
hyperkalemia nificant abnormalities (eg, widening of
QRS interval, loss of P wave, cardiac
• Direct treatment at stabilizing the myocardium, arrhythmias)
shifting potassium from the extracellular envi-
ronment to the intracellular compartment, and • Not indicated when the ECG shows only
promoting the renal excretion and GI loss of peaked T waves
potassium
• Calcium gluconate dose: 10 mL of 10%
* Calcium gluconate solution IV over 2 minutes

• Used to reduce the risk of ventricular * Sodium Bicarbonate


fibrillation caused by hyperkalemia
• Bicarbonate ion neutralizes hydrogen
• Does not lower potassium, only reduces ions and raises urinary and blood pH
effect on cardiac rhythm

Table 4

Clinical Manifestations of Electrolyte Abnormalities

Electrolyte Low Serum Concentration High Serum Concentration

Na+ Nausea, malaise, headache, lethargy, Lethargy, weakness, irritability, twitching,


obtundation, disorientation, stupor, seizures, coma
seizures, coma

Cl- Due to metabolic alkalosis Due to nonanion gap acidosis

K+ Muscle weakness, ileus, cramps, Muscle weakness or paralysis,


paresthesias, ST segment depression, cardiac conduction abnormality,
U waves, flattened T waves peaked T waves, widened QRS,
arrhythmias

Ca2+ Tetany/neuromuscular irritability, Fatigue, weakness, anorexia, depression,


fatigue, anxiety, depression, papilledema, abdominal pain, constipation,
seizures nephrolithiasis, renal tubular dysfunction
(decreased concentrating ability),
and acute and chronic renal insufficiency

Mg2+ Weakness, anorexia, tetany, Neuromuscular toxicity: somnolence, loss


delirium, coma of deep tendon reflexes, muscle paralysis,
delirium, cardiopulmonary arrest

PO42- Bone loss, metabolic encephalopathy, Tumor lysis syndrome, rhabdomyolysis,


muscle dysfunction renal failure

258 EDUCATIONAL REVIEW MANUAL IN UROLOGY


• Onset of action within minutes, lasts • Fine-tuning of Ca2+ reabsorption takes place
approximately 15–30 minutes along the distal convoluted tubule and the con-
necting tubule, where the remaining 15% of
• Dose: 1 mEq/kg slow IV push or continu- filtered Ca2+ is transcellularly reabsorbed
ous IV drip; not to exceed 50–100 mEq.
Use 8.4% solution in adults and children, Role of Vitamin D and PTH in
4.2% solution in infants Calcium Homeostasis

* Glucose and insulin • Vitamin D is involved in normal bone mineral-


ization by maintaining normal serum calcium
• Glucose and insulin temporarily shift and phosphorus levels through increased
K+ into cells; effects occur within first
30 minutes of administration
intestinal absorption of calcium and phospho-
rus and increased renal absorption of calcium

• Administer glucose along with insulin • PTH maintains normal serum calcium and phos-
to prevent hypoglycemia phorus by increasing bone resorption, increas-

• Adult dose: 5–10 U regular insulin and


ing renal reabsorption of calcium and excre-

1–2 amps D50W IV bolus


tion of phosphorus, and stimulating calcitriol
production

* Sodium polystyrene sulfonate (Kayexalate) • Increased PTH→ increase urinary phosphate

• Exchanges Na+ for K+ and binds it in gut,


excretion by reducing proximal tubule reab-

primarily in large intestine, decreasing


sorption of phosphate

total body potassium Response to Hypocalcemia

• Onset of action after PO ranges • Increased intestinal reabsorption of calcium


from 2–12 hours
• Increased PTH
• Adult dose: 25–50 g mixed with 100 mL
of 20% sorbitol PO/PR. Multiple doses * Decreased renal calcium excretion
may be necessary
* Increased bone resorption
* Dialysis

• Immediate onset of action.

• Requires dialysis access

Tubular Handling of Calcium

• Only 1%–3% of Ca2+ that is filtered by the


kidney is excreted

• The majority of Ca2+ reabsorption occurs pas-


sively along the proximal tubule and the thick
ascending limb of Henle’s loop

CHAPTER 7: RENAL PHYSIOLOGY AND PATHOPHYSIOLOGY 259


12. Renal Tubular Acidosis

• A family of syndromes of metabolic acidosis • Diagnose by urine pH>5.5; if no metabolic aci-


resulting from defects in tubular H+ secretion dosis, provoke with ammonium chloride test
and urinary acidification. See Table 5 for a sum-
mary of RTA types • Treat by oral alkalinization and citrate, moni-
tor for low potassium during therapy
RTA Type 1
RTA Type 2
• Most common form
• Failure of bicarbonate reabsorption in the
• Failure of H+ secretion in the distal nephron proximal tubule

• Stones are usually calcium phosphate • More common in children

* High urine pH, Ca2+ and low urine citrate • Normal citrate, no stones

• Hyperchloremic metabolic acidosis with high • Growth retardation, metabolic bone disease
urinary pH (think of Charles Dickens’ Tiny Tim character)

• Treat with NaHCO3

Table 5

Summary of Types of Renal Tubular Acidosis

Type 1 Type 2 Type 4

Location Distal Proximal Distal

Problem Impaired distal acidification Reduced proximal Aldosterone deficiency or


bile reabsorption, resistance
same features but can
acidify urine

Serum K+ ↓ ↓ ↑

Urinary pH ↑ ↓ ↓
>6.0 (inappropriate)

Management K+, bicarb K+, bicarb Mineralcorticoids,


low K+ diets

Notes Hypo K, hyper Cl, nonanion HCO3 wasting from Hyper K, hyper Cl
gap metabolic acidosis; inability to absorb
75% of these patients
get stones

260 EDUCATIONAL REVIEW MANUAL IN UROLOGY


13. Effects of Urinary Diversion

RTA Type 4 • Dependent on bowel segment, length used,

• Impaired cation exchange in the distal tubule


time of exposure, solute concentration, renal
function, urine pH

Stomach
with reduced secretion of H+ and K+

• Hyperkalemia (only RTA type with high K+)


• Hypochloremic, hypokalemic metabolic
• Often have azotemia and hypertension alkalosis

• Treatment aimed to reduce potassium • More significant problem if patient has existing
renal insufficiency

• Treatment includes: proton pump blockers and


acidification with dilute hydrochloric acid

Jejunum

• Hyponatremic, hypochloremic, hyperkalemic


metabolic acidosis

• Rehydrate with NaCl and NaHCO3

• Sodium chloride replacement and thiazides


are treatments of choice

Ileum and Colon Diversions

• Hyperchloremic metabolic acidosis

• Predisposed by impaired renal function

• Ammonium absorption with chloride is


exchanged for Na and HCO3

• Inability to secrete acid as ammonium depletes


buffers

• Treat with Na bicarbonate and/or nicotinic acid


(chloride transport inhibitor)

• Urea and creatinine are reabsorbed by both


the ileum and colon

* Therefore, serum concentrations of urea and


creatinine are less accurate measures of renal
function after urinary diversion

CHAPTER 7: RENAL PHYSIOLOGY AND PATHOPHYSIOLOGY 261


14. Postobstructive Diuresis

* Affects bone metabolism through a decrease • Requires bilateral obstruction or obstruction of


in calcium reabsorption and impaired vitamin solitary unit
D metabolism
• Mechanisms
• The resulting chronic acidosis may pro-
* Na leak, urea osmotic diuretic, loss of
concentrating ability from urea washout in
duce osteoporosis or osteomalacia

Consequences of Malabsorption medulla

• The ileum is the site of vitamin B12 and of bile • Mild form can be corrected by oral intake

• Severe cases require partial IV replacement and


acid absorption

• Resection of up to 60 cm of ileum in patients electrolyte monitoring


who retain their terminal ileum and ileocecal
valve is generally well tolerated

• If more than 100 cm of ileum is resected, fat-sol-


uble vitamin (A, D, E and K) malabsorption
results

• Vitamin B12 deficiency results in anemia and


neurological degeneration

• Oxalate absorption is increased, because the


unabsorbed fat saponifies with calcium, which is
no longer available to form a chelate with
oxalate. This raises serum and urinary oxalate
levels, and urinary stones may result

262 EDUCATIONAL REVIEW MANUAL IN UROLOGY


15. References

1. Vikas B, Le T, Chandwani R, Ozturk A. 13. Shoskes DA, McMahon A. Renal physiology


First Aid for the USLME Step 1 2006. New and pathophysiology. In: Wein AJ, Kavoussi
York, NY: McGraw-Hill; 2006. LR, Novick AC, et al, eds. Campbell-Walsh
Urology. 9th ed. Philadelphia, PA: Elsevier;
2. Burkhard FC, Kessler TM, Mills R, Studer UE. 2007:1129-1155.
Continent urinary diversion. Crit Rev Oncol
Hematol. 2006;57(3):255-264. 14. Verive M, Jaimovich D, et al. Hypokalemia.
Emedicine.com. March 30, 2006.
3. Costanzo LS. Renal and acid-base physiology.
In: BRS Physiology. 3rd ed. Baltimore, MD:
Williams & Wilkins; 2003:161-213.

4. Fall PJ. Hyponatremia and hypernatremia. A


systematic approach to causes and their correc-
tion. Postgrad Med. 2000;107(5):75-82.

5. Field M, Pollock C, Harris D. The Renal Sys-


tem: Basic Science and Clinical Conditions.
New York, NY: Harcourt Publishers; 2001.

6. Fried LF, Palevsky PM. Hyponatremia and


hypernatremia. Med Clin North Am.
1997;81(3):585-609.

7. Garth D, et al. Hyperkalemia. Emedicine.com.


April 25, 2006.

8. Levy David A. Urology Pearls of Wisdom.


Boston, MA: Boston Medical Publishing;
2001.

9. Mandal AK. Hypokalemia and hyperkalemia.


Med Clin North Am. 1997;81(3): 611-639

10. Mattu A, Brady WJ, Robinson DA. Electrocar-


diographic manifestations of hyperkalemia.
Am J Emerg Med. 2000 Oct;18(6):721-729.

11. Mensenkamp AR, Hoenderop JG, Bindels RJ.


Recent advances in renal tubular calcium reab-
sorption. Curr Opin Nephrol Hypertens. 2006
Sep;15(5):524-529.

12. Shayman JA. Renal pathophysiology. In:


Shayman JA, ed. Lippincott’s Pathophysiology
Series. Philadelphia, PA: JB Lippincott Co;
1995.

CHAPTER 7: RENAL PHYSIOLOGY AND PATHOPHYSIOLOGY 263


16. Questions

1. Which of the following is true about sodium C. Lactic acidosis usually presents as a non-
and the kidney? anion gap metabolic acidosis

A. By definition, hypernatremia is always asso- D. Appropriate respiratory compensation for a


ciated with elevated total body sodium content metabolic acidosis is decreased respiration
with an increased pCO2
B. Normal compensation for hyponatremia is
decreased ADH secretion and thirst E. It is not possible to have both a respiratory
suppression and metabolic acidosis at the same time

C. Abnormal elevation of serum lipids can lead


to a measured false elevation of serum 4. All of the following can increase total GFR
sodium except:

D. If asymptomatic hyponatremia does not A. Increased RBF


improve within 24 hours, intravenous hyper-
tonic saline should be started B. Increased intraglomerular (hydraulic) pressure

E. In therapy of symptomatic hyponatremia, the C. Increased glomerular permeability


goal should be a normal serum sodium of 135
mEq/L within 48 hours D. Increased efferent arteriolar resistance

E. Increased functioning nephron number


2. Which of the following is NOT true about
potassium?

A. ACE inhibitors may be a cause of 5. Which of the following metabolic effects of


hypokalemia intestinal segments in the urinary tract is
TRUE?
B. Potassium is primarily an intracellular ion
A. The effects are independent of renal function
C. Acidosis drives potassium out of the cell into
the circulation B. Jejunum produces a hypernatremic
metabolic alkalosis
D. A high sodium load in the distal tubule pro-
motes potassium excretion C. Stomach produces an anion gap metabolic
acidosis
E. The upper limit for safe intravenous potas-
sium infusion is 40 mEq/hr D. Ileum produces a non-anion gap metabolic
acidosis

3. Which of the following is true about acidosis? E. A conduit will be more likely to lead to a
metabolic disorder than a pouch
A. Increasing the blood HCO3 level increases
the anion gap

B. Direct bicarbonate loss from the kidney


would lead to metabolic acidosis and a
normal anion gap

264 EDUCATIONAL REVIEW MANUAL IN UROLOGY


6. Which of the following is true about renal tubu-
lar acidosis?
Answers

1. B.
A. Patients with type 4 usually require potas- The physiologic response to hyponatremia is
sium supplements decreased ADH secretion and thirst suppression.

B. If urinary pH is high but there is no 2. A.


metabolic acidosis, it can be provoked with a ACE inhibitors may cause hyperkalemia.
sodium chloride infusion test
3. B.
C. Type 1 is the most common form seen in Direct bicarbonate loss is measured in the anion gap
children and therefore leads to metabolic acidosis with a nor-
mal anion gap.
D. Type 2 patients commonly develop renal
stones 4. C.
Glomerular permeability is already maximal under
E. Type 1 patients commonly have low urinary normal conditions for water and small solutes, so
citrate GFR will not increase significantly with increased
glomerular permeability. Rather, one sees increased
filtration of larger substances such as albumin.

Effects are more pronounced in the face of poor


5. D.

renal function and increased urinary contact time as


would be seen in a pouch rather than a conduit.

Urinary citrate is low in type 1 which predisposes to


6. E.

renal stones. Type 2 is the most common form in


children and the provocative infusion test is done
with ammonium chloride.

CHAPTER 7: RENAL PHYSIOLOGY AND PATHOPHYSIOLOGY 265


266 EDUCATIONAL REVIEW MANUAL IN NEPHROLOGY
Chapter 8:
Renovascular Disease
Daniel A. Shoskes, MD FRCS(C)

Contents

1. Definitions

2. Pathophysiology

3. Classification of Lesions

4. Clinical Evaluation

5. Medical and Surgical Management

6. Further Reading

7. Questions

CHAPTER 8: RENOVASCULAR DISEASE 267


Introduction 1. Definitions

Surgery for renovascular disease has become a very


uncommon niche within Urology. Nevertheless, the
A. Hypertension

pathophysiology, pathology, diagnostic options and 1. Definition


surgical options remain a feature on the Part I certi-
fication exam. This chapter presents the information
Table 1

in point form with an attempt to focus on the key


testable areas. For Urologists studying for the recer-
tifying exam, the only possible scenarios that might
Blood Pressure (BP) Systolic BP Diastolic BP

be asked would be renal artery aneurysms and per-


Classification mm Hg mm Hg

haps the handling of a renal artery stenosis in a kid-


ney that you were planning other surgery on (e.g.
Normal <120 And <80

donor nephrectomy or partial nephrectomy for


tumor)
“Prehypertension”* 120-139 Or 80-89

Stage I hypertension 140-159 Or 90-99

Stage II hypertension ≥160 Or ≥100

* “Prehypertension” should be treated aggressively in dia-


betics and those with kidney disease to a goal of 130/80

2. Incidence
a. Age 60–69, 50%; age >70, 75%

b. Lifetime risk: 86%–90% female; 81%–83%


male

3. Scope of Problem
a. 30% adults unaware of BP

b. >40% with hypertension are not on treatment

c. Two-thirds of patients with hypertension are not


being controlled to <140/90 mm Hg

d. End-stage renal disease (ESRD) is on the rise;


diabetes and hypertension are the most com-
mon causes of ESRD

B. Renovascular Hypertension (RVHT)

1. Renal Duplex Criteria

a. Renal artery peak systolic velocity >1.8 m/s,


>60% stenosis, or no Doppler signal

2. Angiographic/Magnetic Resonance (MR)/


Computed Tomography (CT) Criteria

a. No clear-cut definition

268 EDUCATIONAL REVIEW MANUAL IN UROLOGY


2. Pathophysiology

b. Some say 75% stenosis, while others quote


50% stenosis (Vasbinder, 2001)
Figure 1

3. Incidence: Poorly Characterized


Hypertension, kidney disease, and renal
artery demonstrate significant overlap

a. <1%–3% of all hypertensive patients have


RVHT 18%–20% among patients undergoing
cardiac angiography

b. 35%–50% among patients undergoing angiog-


raphy for aortic occlusive disease (Olin, 1990)
Nephropathy

C. Ischemic Nephropathy

1. Renal failure due to hemodynamically significant


atherosclerotic renal artery stenosis and related
renal parenchymal disease Hypertension Stenosis

2. Estimates of 8%–10% of new cases of ESRD


caused by ischemic nephropathy (Novick, 2006)

A. RVD Associated With:

1. Ischemic nephropathy

2. Hypertension

B. Mechanism of Ischemic Nephropathy


(Figure 1)

1. Critical stenosis necessary

2. Injury is initially reversible, but then progresses


to permanent damage

3. At end-stage, interstitial fibrosis and ischemic


nephropathy result

4. Glomerulosclerosis and atheroembolism seen on


biopsy (Figure 2 and Figure 3) suggest that the
kidney is unlikely to be salvaged with revascular-
ization, and 5-year survival (predominantly from
cardio- and cerebrovascular events) drops to only
around 50% (Krishnamurti, 1999)

CHAPTER 8: RENOVASCULAR DISEASE 269


Figure 2

Relationship between renal artery stenosis and renal blood flow

100

80
High-grade
60
stenosis
% Flow
40

20

0
0 20 40 60 80 100
% Stenosis

Figure 3

Renal histopathology in ischemic nephropathy

Normal histopathology Ischemic but viable glomeruli

Glomerulosclerosis Atheroembolism

270 EDUCATIONAL REVIEW MANUAL IN UROLOGY


5. Vascular hypertrophy/remodeling
(ATI receptor-mediated)
C. Unilateral RVD

1. Equivalent to the “2-kidney-1-clip” model,


whereby increased BP is accompanied by 6. Sympathetic activation
increased plasma renin activity (PRA)
7. Left ventricular hypertrophy
a. Nonstenotic kidney is subjected to higher perfu-
sion pressures leading to pressure natriuresis
(excretion of sodium, lowering pressure) caus-
G. The RAS System Generates an

ing increased aldosterone and renovascular


Appropriate Response to Hypovolemia

vasoconstriction
or Hypotension, but Pathologic Cycle
in RVD

b. Subsequent decrease in perfusion pressure to


the stenotic kidney causes release of renin
H. Difficult to Determine Whether a Hyper-
tensive Patient Will Respond to Renal
Revascularization or Stenting

1. A lot of overlap with essential hypertension, dia-


D. Bilateral RVD, or Solitary Kidney RVD

1. Equivalent to the “1-kidney-1-clip” model, betes, obesity and smoking, which are common
whereby PRA remains normal, and BP is non- sources of irreversible renal disease
responsive to angiotensin blockade (see Figure 5)

a. In the absence of a normally perfused kidney, 2. Data suggest renovascular hypertension is asso-
excessive sodium cannot be excreted in ciated with increased sympathetic input, mani-
response to higher perfusion pressures fested by greater variability in BP throughout the
day. This can be documented with 24-hour ambu-
b. Sodium and water retention occur, leading to vol- latory BP monitoring
ume expansion, which is the primary mechanism
of elevated BP in this model (see Figure 4) 3. May demonstrate higher nocturnal BP than
essential hypertensives

4. May demonstrate hypokalemia and hypona-


E. Renin – Increased by:

1. Reduced sodium delivery to distal tubule tremia

2. Hypoperfusion of juxtaglomerular cells 5. More likely to have reduced glomerular filtration


rate (GFR)
3. Beta-adrenergic stimulation

4. Prostaglandin E2 (PGE2) & prostacyclin (PGI2)

F. Angiotensin II (ATII) –
Powerful Effector of Multiple Actions
Downstream, Including:

1. Efferent arteriolar vasoconstriction

2. Peripheral vasoconstriction

3. Renal sodium retention

4. Aldosterone secretion

CHAPTER 8: RENOVASCULAR DISEASE 271


Figure 4

Goldblatt (1930s) demonstrated that reduced perfusion to the kidney can cause increased
arterial blood pressure

Two-kidney hypertension One-kidney hypertension

Blood Blood
pressure Renin Volume pressure Renin Volume
↓ High Normal ↓ Normal High
ARB/ACE inhibitors help Only help when Na depleted

Figure 5

Renin-angiotensin system (RAS) primarily responsible for observations of Goldblatt

272 EDUCATIONAL REVIEW MANUAL IN UROLOGY


3. Classification of Lesions

A. Atherosclerotic (70%–90%) • Commonly bilateral

1. Usually involves ostium and proximal one-third • Progression less common


of main renal artery (see Figure 6)
• “String of beads” appearance on angiography
2. Systemic disease
• Mid to distal portion of artery
3. Progression common, with 18% occlusion rate
over 5 years • Associated with Ehlers-Danlos Syndrome
(type IV)

b. Perimedial fibroplasia (see Figure 8)


B. Fibromuscular Disease (10%–30%)

1. Primarily localizes to distal half of renal artery or


branched renal vessels • Women aged 15–30

2. Typically affects women 15–50 years of age • Collateral circulation common

3. Classified by arterial layer--intima, media, or • Progression common


adventitia--in which the lesion predominates
• Homogeneous collar of elastic tissue at junc-
a. Medial Fibroplasia (see Figure 7) tion of media and adventitia

• Most common • More focal than medial hyperplasia, but there


may be multiple constricted areas
• Women 25–50
Figure 6

Atherosclerotic renovascular disease

High-grade stenosis in solitary kidney High-grade bilateral stenosis

CHAPTER 8: RENOVASCULAR DISEASE 273


Figure 7 Figure 9

Medial fibroplasia Intimal fibroplasia

• Most common fibrous


• Women 25-50
• Commonly bilateral • Children and adolescents
• “String of beads” • Collagen inside elastic membrane
• Involves distal two-thirds and “branches” • May dissect
• Progression less common • Progression common

c. Intimal fibroplasia (see Figure 9)

• Focal, concentric, long stenosis


Figure 8

• Appearance of vasculitis, distinguished by


Perimedial fibroplasia

lack of markers of inflammation and isolation


to renal vessels only

• Children and adolescents

• Collagen inside elastic membrane

• May dissect

• Progression common

d. Adventitial (periarterial or fibromuscular)

• Rarest

• Women aged 15-30 • Children and adolescents

• Frequently progressive
• Collateral circulation common
• Tight stenosis and progression common

• Sharply demarcated areas of stenosis


274 EDUCATIONAL REVIEW MANUAL IN UROLOGY
4. Clinical Evaluation

A. When to Consider Evaluation for RVH

6. Azotemia of uncertain etiology (especially if


(Chobanian, 2003) (Figure 10)

1. Onset of hypertension <30 years old or no proteinuria)


>55 years old
7. Acute renal failure precipitated by ace inhibitor
2. Abdominal bruit, especially if a diastolic (ACEI) or angiotensin receptor blocker (ARB)
component is present
8. Lack of family history
3. Accelerated hypertension/malignant crisis
9. Disseminated atherosclerotic disease
4. Hypertension that had been easy to control
becomes resistant 10. Renal size disparity

5. Recurrent flash pulmonary edema

Figure 10

RVH diagnostic algorithm

CHAPTER 8: RENOVASCULAR DISEASE 275


procedure. At the time of intervention, an arteri-
ogram will be performed using limited contrast to
B. Screening

1. Recommendations from Seventh Report of confirm the stenosis and identify the anatomy of
the Joint National Committee on Prevention, the renal artery.”
Detection, Evaluation, and Treatment of High
Blood Pressure 2. Noninvasive tests—How good are they?

a. Urinalysis, serum studies (Cr, glucose, K, uric a. ACE-enhanced renal scan


acid, lipids), electrocardiogram (EKG), and
+ ambulatory BP monitoring • Limited value for anatomy but excellent func-
tional study and can help predict response to
b. Recommended noninvasive options include: surgery
Captopril-enhanced renal scan, duplex Doppler
flow studies, and magnetic angiography • Poorer sensitivity and specificity
than MR or CT
c. “While renal artery angiography remains the
gold standard for identifying the anatomy of the b. Duplex ultrasound (U/S)
renal artery, it is not recommended for diagno-
sis alone because of the risk associated with the

Figure 11

RVD treatment algorithm: medical and surgical management

Confirmation of RVD in noninvasive testing

Will revascularization benefit patient?

Possible Unlikely
Resistive index <0.80 on duplex Resistive Index> 0.80 on duplex
Scr <4.0 mg/dL Scr >4.0 mg/dL
Lateralization of renin on renal vein sampling
No lateralization on selective renal vein sampling
GFR cannot be maintained by angiotensin
Nonfunction on isotope scan
blockade or pt does not tolerate BP meds
Solitary kidney or significant bilateral disease Unfavorable renal biopsy
Loss of renal volume, size ≥7cm Medial fibroplasia
Recurrent pulmonary edema Atherosclerotic stenosis without nephropathy

Select method of revascularization Continue medical therapy, then


dialysis or transplant

PTRA Stent Surgery


Ostial plaque or Needs
FMD atherosclerotic surgical aortic repair
Bilateral dz or solitary renal artery aneurysm
Limited disease kidney and unable to Failed multiple
tolerate surgery endovascular repairs

276 EDUCATIONAL REVIEW MANUAL IN UROLOGY


5. Medical and
Surgical Management

• Diastolic blood flow velocity is measured by


resistive index (RI) and suggests small vessel
A. Medical

disease and parenchymal damage. 1. Most patients are already on multiple-drug ther-
apy when they are referred
• If RI >0.80, revascularization unlikely to
improve renal function 2. Select medical management based on risk of
ischemic nephropathy and lesion progression
• Sensitivity 80%–90%; specificity 80%–90%
a. Medial fibroplasia and atherosclerotic (without
c. 3D contrast-enhanced MR arteriogram ischemic nephropathy) are best for medical
management
• Poor visualization of distal arteries, but sensi-
tivity and specificity of >90% for proximal 3. ATII and aldosterone play a direct role in end
arterial segments; thus better for atheroscle- organ damage to kidney, peripheral vessels and
rotic lesions vs fibromuscular dysplasia heart; therefore, drug treatment can be targeted at
interrupting this cycle—ARBs, ACE-I and beta
• Sensitivity 88%–100%; specificity blockade are important first-line drugs
75%–100%
a. If goal BPs can be achieved with ARB/ACEI,
• Uses gadolinium – now contraindicated in outcomes are comparable to surgical revascu-
patients with renal impairment larization

d. Spiral CT Arteriogram 4. Aggressive lifestyle modification and health


management necessary: smoking cessation, lipid
• Sensitivity 94%–100%; specificity 92%–99% management, and diabetes control

• Contrast media—risk of nephrotoxicity 5. Potential regression of atherosclerotic disease with


aggressive lipid management and smoking cessa-
3. Invasive Tests—How good are they? tion

a. Angiography B. Surgical: PRTA, Stenting,

• Significant interobserver variability in deter-


Revascularization

mining what is a “clinically significant” lesion 1. Reserved for situations where:

b. Selective renal vein sampling for renin: a. BP is not controlled with meds or patient does
not tolerate meds
• If renal vein (RV) renin lateralizes to one kid-
ney, the likelihood of favorable blood pressure b. ARBs/ACE-I significantly reduce GFR (by
improvement after revascularization is >90%, >30% or increased Cr by >0.5mg/dL) by loss of
but if RV renin fails to lateralize, the likeli- compensatory filtration pressure
hood of benefit is still near 50% (see Figure
11) c. Significant bilateral, or solitary kidney,
renal stenosis

d. Recurrent, difficult to control, congestive


heart failure

e. Loss of renal volume over time, but kidney size


still at least 7 cm

CHAPTER 8: RENOVASCULAR DISEASE 277


f. Function on isotope scan 4. Surgical Revascularization

g. Favorable renal biopsy a. Major indications:

h. Collateral circulation • Patients undergoing surgical aortic repair

i. SCr<4.0 mg/dL • Patients with renal artery aneurysm—


- Risk of rupture
2. Percutaneous Transluminal Renal - Absent/incomplete calcification
Artery Angioplasty (PTRA) - >2 cm diameter
- Expanding
a. Poor outcome for ostial atherosclerotic - Hypertension
lesions (29%) - Pregnancy

b. Little benefit over medical treatment alone • Multiple failed endovascular repairs
in prospective trials for atherosclerotic RVD
b. In most series of patients with impaired renal
c. First-line approach with fibromuscular function before revascularization, little or no
dysplasia recovery of GFR follows either surgical or
endovascular restoration of blood flow
d. Restenosis rates 7%–27% except for
atherosclerotic lesions (near 50% restenosis c. Signs of reversibility
rate)
• Progressive occlusion
3. Stenting
• Collaterals
a. Better outcomes for ostial lesions vs PTRA
alone (75%) • Retrograde arterial filling

b. Expected improvements in BPs in well-selected • Size >7 cm


patients about 25/10 mm Hg
• Cr <4.0
c. CORAL trial (Cardiovascular Outcomes in
Renal Artery Lesions)—compares medical • Preservation of glomeruli on biopsy
management with and without stenting;
accrual underway. d. Technical success 89%–97%

d. Restenosis rates 10%–15%, coated stents may e. Hypertension control for patients with
perform better fibromuscular dysplasia undergoing surgical
repair:
e. Post-stent acute decline in renal function in • Cured = 33%–63%;
10%–20% of patients likely from cholesterol • Improved = 24%–57%;
atheroembolism • Failure to improve = 3%–33%

f. For patients with significant bilateral disease or f. Retrospective studies suggest similar moderate
solitary kidneys, stenting associated with non- and long-term outcomes between endovascular
significant improvements in GFR, nearly 10% and open revascularization
restenosis, 5% dissection, but significant
decreases in BP

278 EDUCATIONAL REVIEW MANUAL IN UROLOGY


g. Graft thrombosis rate ~4%, perioperative • Aortorenal
mortality ~3% (Fergany, 1995)
• Thoracic aorta—renal
h. Surgical approaches (Figure 12)
5. Other Surgical Interventions
• Hepatorenal
a. Nephrectomy if small and nonfunctioning
• Splenorenal
b. Autotransplantation an option in young patients
• Ileorenal with complicated anatomy (Sevmis, 2006)

• Arteriotomy

Figure 12

Techniques for surgical revascularization in ARAS

Abdominal Thoracic
aortorenal bypass aortorenal bypass

Hepatorenal Splenorenal
bypass bypass

CHAPTER 8: RENOVASCULAR DISEASE 279


6. Further Reading

Anderson CA, Hansen KJ, Benjamin ME, Keith Fergany A, Kolettis P, Novick AC. The contempo-
DR. Dysplasia: results of current surgical therapy. rary role of extra-anatomical surgical renal revascu-
J Vasc Surg. 1995;22:207-216. larization in patients with atherosclerotic renal
artery disease. J Urol. 1995;153(6):1798-1802.
Anderson GH, Blakemen N, Streeten DH. The
effect of age on prevalence of secondary forms of Forbes JM, Hewitson TD, Becker GJ, Jones CL.
hypertension in 4429 consecutively referred Simultaneous blockade of endothelin A and B recep-
patients. J Hypertens. 1994;12:609-615. tors in ischemic acute renal failure is detrimental to
long-term kidney function. Kidney Int.
Atkinson AB, Davies DL, Leckie B, et al. Hypona- 2001;59:1333-1341.
tremic hypertensive syndrome with renal-artery
occlusion corrected by captopril. Lancet. Galaria II, Surowiec SM, Rhodes JM, et al. Percuta-
1979;2:606-609. neous and open renal revascularizations have equiv-
alent long-term functional outcomes. Ann Vasc
Basso N, Tarragno NA. History about the discovery Surgery. 2005;19(2):218-228.
of the renin-angiotensin system. Hypertension.
2001;38:1246-1249. Gavras H, Brunner HR, Thurston H, Laragh JH.
Reciprocation of renin dependency with sodium vol-
Berglund G. Secondary hypertension in the commu- ume dependency in renal hypertension. Science.
nity. In: Birkenhager WH, Reid JL, eds. Handbook 1975;188:1316-1317.
of Hypertension. Amsterdam, Netherlands: Else-
vier; 1985:249-254. Gavras H, Brunner HR, Vaughan ED, Laragh JH.
Angiotensin-sodium interaction in blood pressure
Brunner HR, Kirshmann JD, Sealey JE, Laragh JH. maintenance of renal hypertensive and normoten-
Hypertension of renal origin: evidence for two dif- sive rats. Science. 1973;180:1369-1371.
ferent mechanisms. Science. 1971;174:1344-1346.
Goldblatt H, Lynch J, Hanzal RE, Summerville
Chabova V, Schirger A, Stanson AW, McKusick M, WW. Studies on experimental hypertension: I. The
Textor SC. Outcomes of atherosclerotic renal artery production of persistent elevation of systolic blood
stenosis managed without revascularization. Mayo pressure by means of renal ischemia. J Exp Med.
Clin Proc. 2000;75:437-444. 1934;59:347-379.

Chobanian AV, Bakris GL, Black HR, et al. and the Iantorno M, Pola R, Schinzari F, et al. Association
National High Blood Pressure Education Program between altered circadian blood pressure profile and
Coordinating Committee. Seventh Report of the cardiac end-organ damage in patients with renovas-
Joint National Committee on Prevention, Detection: cular hypertension. Cardiology. 2003;100:114-119.
Evaluation, and Treatment of High Blood Pressure.
Hypertension. 2003;42;1206-1252. Isles CG, Robertson S, Hill D. Management of reno-
vascular disease: a review of renal artery stenting in
DeForrest JM, Knappenberger RC, Antonaccio MJ, ten studies. QJM. 1999;92:159-167.
Ferrone RA, Creekmore JS. Angiotensin II is a nec-
essary component for the development of hyperten- Kashyap VS, Sepulveda RN, Bena JF, et al. The
sion in the two kidney, one clip rat. Am J Cardiol. management of renal artery atherosclerosis for renal
1982;49:1515-1517. savage: does stenting help? J Vasc Surg. 2007;
45(1):101-109.
Edwards MS, Craven TE, Burke GL, Dean RH,
Hansen KJ. Renovascular disease and the risk of Kennedy DJ, Colyer WR, Brewster PS, et al. Renal
adverse coronary events in the elderly: a prospec- insufficiency as a predictor of adverse events and
tive, population-based study. Arch Intern Med. mortality after renal artery stent placement. Am J
2005;165:207-213. Kidney Dis. 2003;42:926-935.

280 EDUCATIONAL REVIEW MANUAL IN UROLOGY


Krishnamurthi V, Novick AC, Myles JL. Atheroem- Rihal CS, Textor SC, Breen JF, et al. Incidental
bolic disease: effect on morbidity and survival afte renal artery stenosis among a prospective cohort of
revascularization for atherosclerotic renal artery hypertensive patients undergoing coronary angiog-
stenosis. J Urol. 1999;161:1093-1096. raphy. Mayo Clin Proc. 2002;77:309-1316.

Leiner T, de Haan MW, Nelemans PJ, van Rossi GP, Cesari M, Chiesura-Corona M, Miotto D,
Engelshoven JM, Vasbinder GB. Contemporary Semplicini A, Pessina AC. Renal vein renin mea-
imaging techniques for the diagnosis of renal artery surements accurately identify renovascular hyper-
stenosis. Eur Radiol. 2005;15:2219-2229. tension caused by total occlusion of the renal artery.
J Hypertens. 2002;20:975-984.
Leung DA, Hagspiel KD, Angle JF, Spinsoa DJ,
Matsumoto AH, Butty S. MR angiography of the Safian RD, Textor SC. Renal-artery stenosis. N Engl
renal arteries. Radiol Clin North Am. 2002;40:847- J Med. 2001;344:431-442.
865.
Sevmis S, Karakayali H, Boyvat F, Aytekin C,
Martinez AG, Novick AC, Hayes JM. Surgical Haberal M. Renal autotransplantation for the treat-
treatment of renal artery stenosis after failed percu- ment of complex renovascular hypertension. Trans-
taneous transluminal angioplasty. J Urol. 1990; plant Proc. 2006;38(10):3412-3415.
144:1094-1096.
Slovut DP, Olin JW. Fibromuscular dysplasia. N
Mounier-Vehier C, Cocheteux B, Haulon S, et al. Engl J Med. 2004;350:1862-1871.
Changes in renal blood flow reserve after angio-
plasty of renal artery stenosis in hypertensive Stanley JC. Surgical treatment of renovascular
patients. Kidney Int. 2004;65:245-250. hypertension. Am J Surg. 1997;174:102-110.

Novick AC, Ziegelbaum M, Vidt DG, Gifford RW Strong CG, Hunt JC, Sheps SG, Tucker RM,
Jr, Pohl MA, Goormastic M. Trends in surgical Bernatz PE. Renal venous renin activity. Enhance-
revascularization for renal artery disease. Ten years’ ment of sensitivity of lateralization by sodium
experience. JAMA. 1987;257:498-501. depletion. Am J Cardiol. 1971:27:602-611.

Olin JW, Melia M, Young JR, Graor RA, Risius B. Textor SC, Wilcox CS. Renal artery stenosis: a com-
Prevalence of atherosclerotic Renal Artery Stenosis mon, treatable cause of renal failure? Ann Rev Med.
in patients with atherosclerosis elsewhere. Am J 2001;52:421-422.
Med. 1990; 88:46N-51N.
Tollefson DF, Ernst CB. Natural history of
Plouin PF, Chatellier G, Darné B, Raynaud A. atherosclerotic renal artery stenosis associated with
Blood pressure outcome of angioplasty in aortic disease. J Vasc Surg. 1991;14:327-331.
atherosclerotic renal artery stenosis: a randomized
trial. Hypertension. 1998;31:823-829. Uzzo RG, Novick AC, Goormastic M, Mascha E,
Pohl M. Medical versus surgical management of
Postma CT, Joosten FB, Rosenbusch G, Thien T. atherosclerotic renal artery stenosis. Transplant
Magnetic resonance angiography has a high relia- Proc. 2002;34:723-725.
bility in the detection of renal artery stenosis. Am J
Hypertens. 1997;10:957-963. van de Ven PJ, Kaatee R, et al. Arterial stenting and
balloon angioplasty in ostial atherosclerotic reno-
Radermacher J, Chavan A, Bleck J, et al. Use of vascular disease: a randomised trial. Lancet.
Doppler ultrasonography to predict the outcome of 1999;353: 282-286.
therapy for renal-artery stenosis. N Engl J Med.
2001;344:410-417.

CHAPTER 8: RENOVASCULAR DISEASE 281


7. Questions

van Jaarsveld BC, Krijnen P, Pieterman H, et al. The 1. Which of the following is more likely to be
effect of balloon angioplasty on hypertension in associated with renovascular hypertension?
atherosclerotic renal-artery stenosis. Dutch Renal
Artery Stenosis Intervention Cooperative Study A. Positive family history
Group. N Engl J Med. 2000; 342: 1007-1014.
B. Mild hypertension
van Jaarsveld BC, Pieterman H, van Dijk LC, et al.
Inter-observer variability in the angiographic C. Age of onset of 22 for hypertension
assessment of renal artery stenosis. DRASTIC
study group. Dutch Renal Artery Stenosis Interven- D. BP well controlled with a diuretic alone
tion Cooperative. J Hypertens. 1999;17:1731-1736.
E. Kidneys equal size by ultrasound
Vasbinder GB, Nelemans PJ, Kessels AG, Kroon
AA, de Leeuw PW, van Engelshoven JM. Diagnos-
tic tests for renal artery stenosis in patients sus- 2. Which of the following does NOT increase the
pected of having renovascular hypertension: a meta- risk of renal artery aneurysm rupture?
analysis. Ann Intern Med. 2001;135:401-411.
A. Pregnancy
Webster J, Marshall F, Abdalla M, et al. Ran-
domised comparison of percutaneous angioplasty B. < 1.5-cm diameter
vs continued medical therapy for hypertensive
patients with atheromatous renal artery stenosis. C. Incomplete calcification
Scottish and Newcastle Renal Artery Stenosis Col-
laborative Group. J Hum Hypertens 1998;12: D. Size increased from 3 months ago
329–335.
E. Untreated hypertension
Detection, evaluation, and treatment of renovascu-
lar hypertension. Final report. Working Group on
Renovascular Hypertension. Arch Intern Med. 3. Which of the following increases the likelihood
1987;147:820–829. that renal revascularization for ischemic
nephropathy will improve renal function?
Zoccali C, Mallamaci F, Finocchiaro P. Atheroscle-
rotic renal artery stenosis: epidemiology, cardiovas- A. Unilateral disease
cular outcomes, and clinical prediction rules. J Am
Soc Nephrol. 2002;13:S179-S183. B. Use of a drug-eluting stent

C. Kidney size <7 cm

D. Cr >5.0 mg/dL

E. Retrograde arterial filling on angiogram

282 EDUCATIONAL REVIEW MANUAL IN UROLOGY


4. Which of the following is TRUE about the renin- 6. Which of the following is TRUE about imaging
angiotensin system? studies for renovascular hypertension?

A. Angiotensin II raises systemic vascular resis- A. Administration of captopril would be expected


tance to increase GFR in patients with renovascular
hypertension
B. Angiotensinogen is produced by the kidney
B. Duplex ultrasound is dependent on the degree
C. Aldosterone increases urinary sodium concen- of remaining renal function
trations
C. MRA is ideal for imaging branch vessel disease
D. The ACE enzymes convert angiotensinogen to
angiotensin I D. A positive captopril renal scan is the best pre-
dictor of surgical cure
E. Angiotensin II inhibits aldosterone secretion
E. Renal vein renin sampling should be done
before any surgical repair

5. Which of the following is TRUE about renal


artery stenosis?
7. Which is the following is true regarding surgical
A. Atherosclerosis is the most common mecha- repair of renal artery stenosis?
nism in children
A. Artificial grafts are superior to autologous tis-
B. Intimal fibroplasia produces the classic “string sue because they have lower failure rates
of beads” appearance
B. In cases where the abdominal aorta and all its
C. Medial fibroplasia is the most common form of major branches are heavily diseased with
fibromuscular disease atherosclerosis, the thoracic aorta is often
spared and can be used for a left renal artery
D. Progression of atherosclerotic renal artery repair
stenosis is rare
C. An added risk of autotransplant is the need for
E. Perimedial fibroplasia is more common in lifelong immunosuppression
elderly men
D. Hepatorenal bypass is a good choice for repair
of a left renal artery stenosis

E. Carotid stenosis should be treated after renal


artery repair to allow normalization of blood
pressure

CHAPTER 8: RENOVASCULAR DISEASE 283


Answers

1. C.

2. B.

3. E.

4. A

5. C

6. E

7. B

284 EDUCATIONAL REVIEW MANUAL IN UROLOGY


Chapter 9:
Renal Transplantation
Daniel A. Shoskes, MD, FRCS(C)
Ahmed Aboumohamed, MD

Contents

1. Introduction

2. Basic Transplantation Immunology

3. Immunosuppressive Drugs

4. Recipient Evaluation

5. Living Donors

6. Cadaveric Donor

7. Ischemia-Reperfusion and
Delayed Graft Function

8. Surgical Technique and Complications

9. Surgical Complications

10. Early and Late Patient Management Issues

11. Questions

CHAPTER 9: RENAL TRANSPLANTATION 285


1. Introduction 2. Basic Transplantation
Immunology

While not a significant part of most urology prac- A. The immune system has evolved mechanisms
tices, non-transplant specialty urologists may be to recognize potential threats:
asked to consult on these patients, and transplant
questions are a regular feature of certifying and • Recognition of determinants that do not belong to
recertifying exams. For Part 1 of the Boards, every- the host (self vs nonself theory)
thing in this chapter would be considered testable,
with the exception of specific immunosuppressive • Recognition of situation associated with potential
drug dosing (which has large inter-center variability threats to the host, such as inflammation (safe vs
in any case). For the recertifying exam, it should be danger theory)
safe to assume that questions will focus on urologic
issues in transplant patients that might generate a B. Based on animal skin grafts, the laws of
consultation, such as ureteral obstruction, posttrans- transplantation were discovered
plant urologic malignancy or management of an
abnormal lower urinary tract. • Isografts (tissue from the same individual or from
an identical twin) succeed
A. Renal transplantation is usually the preferred
form of renal replacement therapy, as it • Xenografts (tissue from different species) fail
significantly improves life expectancy immediately

• Relative risk of mortality • Allografts (different individual from within the


*All dialysis patients 1.00 same species) fail over variable time
*Patients on waiting list 0.4
*Cadaveric kidney transplant 0.32 C. The 2 immune distinct ways to recognize and
*Living kidney transplant 0.21 to respond to danger that may threaten the
host:
B. The surgical procedure and its associated
medical management (immunosuppression) • Innate immunity
are cost effective if the transplanted organ * Recognizes inflammation and
lasts more than 2 years tissue destruction
* Can recognize and target specific structures
C. The immunosuppressive agents used to on certain organisms (bacteria, tumor cells)
prevent transplant rejection introduce com- * Not associated with immune memory: the
plications on their own: next encounter with the same organisms will
not be more rapid or more effective
• Infections * Cells and mechanisms:
• Macrophages
• Cancer • Monocytes
• Polymorphonuclear cells (PMN)
• Diabetes • Complement
• NK (natural killer) cells
• Hypertension
• Acquired immunity
• Coronary artery disease * Recognizes and responds to specific targets
(antigens)
* Activation generates effector cells (cytotoxic
T-lymphocytes and plasma cells) together
with memory cells
* Associated with long-lasting memory: the
next encounter with similar organisms will be
faster and more effective

286 EDUCATIONAL REVIEW MANUAL IN UROLOGY


* Consist of:
• T-cells (cellular arm)
Figure 1

• B-cells and antibodies (humoral arm) T-cell – APC interaction

D. Antigen Recognition

• Antigen recognition is central to the generation of


both specific effector and memory cells

• By T-cells
* Larger molecule (antigen) must be broken
down into smaller peptide fragments that
T- cells can recognize specifically and then
respond to:

• The receptor at the surface of the T-cells


is called the T-cell receptor (TCR)
(Figure 1)

• The peptides must be displayed by spe-


cialized antigen-presenting cells (APCs)

• The APCs will also express other molecules • Plasma cell generation: terminally differ-
required for T-cell activation (so-called “second- entiated cells that produce large quantities
signal”) of antigen-specific antibodies

• After successful antigen recognition: E. Human Leukocyte Antigens (HLA)


* Clonal proliferation of T-cells and generation
of memory cells • The HLA molecules are encoded by a group of
* Cytotoxic T-lymphocytes (usually CD8 genes on chromosome 6, also known as the
molecule on their surface) can kill target cells human major histocompatibility complex(MHC)
via cell-surface acting molecules (perforins) (Figure 2)
* Helper T-lymphocytes (usually bearing the
CD4 molecule on their surface) secrete • Highly polymorphic
cytokines required for both CD8 T-cell and
B-cell activation • Class I
* The major antigens are HLA-A, B and C and
• By B-cells are made of more than 1,200 alleles
* Receptor is a surface immunoglobulin (sIg) * Found on the surface of virtually all
* sIg, unlike the TCR, can “see” the whole nucleated cells
antigen by recognizing its tertiary structure * Has a binding site for CD8 molecules on
* There is no need for antigen processing for T-cells (usually cytotoxic)
recognition
* Cytokines from helper T-cells are required for
B-cell activation, together with antigen
recognition by the B-cells
*After successful antigen recognition
• Clonal proliferation of B-cells and
generation of memory cells

CHAPTER 9: RENAL TRANSPLANTATION 287


Figure 2
G
Polymorphism of HLA locus

Class II Class III Class I


Glyoxylase
DP DQ DR B C A

DZ DO Cyp21 TNF

Short arm of chromosome 6: HLA complex

• Class II • In periphery
* The major antigens are HLA-DP, DQ and DR * The thymus can obviously not display every
and are made of more than 700 alleles antigen present in the organism: other mech-
* Found usually on professional antigen-pre- anisms must be present to prevent autoim-
senting cells (macrophages, dendritic cells mune cells
and B-cells) * When a T-cell recognizes an autoantigen
* Has a binding site for CD4 molecules on without a second signal, it is turned off
T-cells (usually helper) (anergy) or dies (deletion)

• APCs are further divided into professional (den- G. Immune Cell Activation
dritic cells, macrophages and B-cells) or nonpro-
fessional (fibroblasts, vascular endothelial cells) • Macrophages, monocytes and dendritic
depending on whether or not they need activation * Professional APCs (see above)
to present antigen. In area of tissue injury, more * They are among the first cells to respond to
cells can function as APCs injury by localizing to sites of trauma
* They release chemoattractant molecules and
F. T-cell Education increase the level of responsiveness of the
immune system through cytokines such as
• The TCR is randomly generated, including tumor necrosis factor (TNF) and interleukin-1
potentially autoreacting cells (IL-1)

• Mechanisms are in place to limit the presence of • After CD4 T-cells recognize antigen, they release
such autoreactive cells IL-2 and interferon-gamma (INF gamma), leading
to clonal expansion and activation of both B- and
• In the thymus T-cells
* There is positive selection for T-cells
expressing a TCR that binds to self MHC • T-cell activation will generate antigen-specific
(so they can all function properly) cytotoxic T-lymphocytes, which can kill foreign
* There is negative selection for T-cells cells
expressing a TCR that very tightly binds to
self MHC (would auto-react)

288 EDUCATIONAL REVIEW MANUAL IN UROLOGY


3. Immunosuppressive Drugs

• Activated B-cells become plasma cells (antibody A. Immunosuppression


factories) and memory B-cells (for more rapid
response to the same antigen later on) • Required for the life of the transplanted kidney to
prevent rejection
• Molecular mechanisms for CD4 T-cell activation
* Proper CD4-MHC engagement, together with • Modern protocols combining several agents acting
second signal (CD28), will lead to increased at different sites of the immune response have
intracellular ionized calcium from both intra- dropped rates of acute rejection to 10% and
cellular and extracellular stores. The CD3 increased 1-year graft survival to 90%
complex, associated to the TCR, is central to
those early events • Figure 3 presents the activation of the immune sys-
* The increase in intracellular calcium will tem, together with the proposed site of action of
cause activation of calcineurin, a calcium- each drug
dependent phosphatase (calcineurin)
* The activated calcineurin will then dephos- • Total immunosuppression dose increases the risk
phorylate a transcription factor (NFAT: of infections and certain malignancies
Nuclear Factor of Activated T-cells)
sequestered in the cytoplasm, exposing its • Each immunosuppressive drug has its specific
nuclear translocation signal and allowing its nonimmune side effects
export to the nucleus of the cell
* NFAT in the nucleus then increases the tran- B. Antibody Agents
scription of a number of genes required for T-
cell activation, among which includes IL-2 • Anti-IL-2 receptor antagonists (non depleting,
because the cells are inactivated but not killed)
* Daclizumab (Zenapax®), basiliximab
(Simulect®)
* Monoclonal antibody that binds to part of the
IL-2 receptor surface of T-cell, thus prevent-
ing IL-2 from activating T-cells
* Given at time of transplant and for 1–5
additional doses to prevent (but not treat)
rejection
* These agents have minimal drug-specific side
effects

• Depleting agents (kill cells)


*OKT3 (Muromonab)
• Murine monoclonal antibody against
human CD3
• Prevents the function of the CD3
complex, blocking T-cell activation
• Used to prevent and treat acute
rejection episodes
• Side effects:
* Anaphylactic reactions
* Cytokine release syndrome: fever,
chills, malaise, hypotension
* Infection (cytomegalovirus, CMV)
* Malignancies (posttransplant lym-
phoproliferative disorders, [PTLD])

CHAPTER 9: RENAL TRANSPLANTATION 289


Figure 3

Immune system activation and action sites of specific immunosuppressants

Alloantigen Immunosuppressants
1 1 OKT3, polyclonal antibodies
(Thymoglobulin)
T-cell Receptor
2 2 Tacrolimus , Cyclosporine

Activated Calcineurin

Dephosphorylation
of NFAT

3 3 Glucocorticoids
IL-2 Gene Promotion
4 4 Daclizumab, Basiliximab

IL-2

IL-2 Receptor
5 5 Sirolimus

Progression into
Cell Cycle
6 6 Azathioprine, Mycophenolate
Mofetil
Cell Proliferation

290 EDUCATIONAL REVIEW MANUAL IN UROLOGY


D. Pharmacology and Pharmacokinetic
• Polyclonal antibodies made by immuniz-
*Thymoglobulin

ing rabbits with human thymocytes • Use of these drugs require close monitoring of
• Mixture of antibodies against various cell blood level in order to minimize side effects
surface antigens
• Cause profound depletion of T-cells, • Best correlation between dose and side effect pro-
affecting the functioning of the immune file is through precise monitoring of the area under
system for weeks the curve (AUC)
• Premedication with corticosteroid,
acetaminophen and antihistamine is rec- • Best approximation of AUC for cyclosporine is
ommended and may reduce the incidence through the use of C2 blood level (level found in
and intensity of fever and chills (cytokine blood 2 hours after PO dose)
release) side effects
• Can be used for both induction and to treat • Approximation of AUC with trough level (level
acute rejection episodes found in blood just before next dose) is usually
• Side effects: used for tacrolimus
* Myelosuppression: leukopenia,
thrombocytopenia • Metabolism is largely hepatic
* Cytokine release syndrome: fever,
chills, malaise • Cyclosporine
* Infection (CMV) * Oral dose: 5–10 mg/kg/day divided BID
* Malignancies (PTLD) * Target trough 250–350 ng/mL for the first 3
months, tapered to about 150–250 ng/mL up
C. Calcineurin Inhibitors (CNI) to 6 months, after which levels are maintained
at about 100 mg/mL
• Foundation of modern immunosuppression to
prevent allograft rejection • Prograf
*Oral dose: 0.1–0.15 mg/kg/day divided BID
• Two molecules: cyclosporine (Neoral®, Gengraf®) * Target trough 10–15 ng/ml, drop to 6–10 after
and tacrolimus (Prograf®) 6 months

• Interfere with IL-2 gene transcription, by prevent- • IV dose is generally one-quarter to one-third of
ing NFAT activation, thus preventing T-cell activa- total daily oral dose
tion early on
• Significant drug interaction, from extensive hep-
• Side effects atic metabolism (CYP3A4 enzyme systems)
* Overall relatively similar: both are nephro- * Drugs that may increase blood concentration
toxic and can cause hypertension, reversible * Calcium channel blockers: diltiazem, vera-
neurotoxicity and metabolic abnormalities pamil, nifedipine, nicardipine
* Cosmetic changes (hirsutism, gingival hyper- * Azole antifungal agents: clotrimazole,
plasia and coarsening of facial features) may fluconazole, itraconazole, ketoconazole,
be more pronounced with cyclosporine voriconazole
* Higher incidence of diabetes may be associ- * Macrolide antibiotics: clarithromycin,
ated with tacrolimus erythromycin
* Other: grapefruit juice, metoclopramide,
• Tacrolimus associated with better graft survival in allopurinol
high-risk patients

CHAPTER 9: RENAL TRANSPLANTATION 291


• Drugs that may decrease blood concentration * Femoral head aseptic necrosis
* Anticonvulsant: carbamazepine, * Glaucoma
phenobarbital, phenytoin * Glucose intolerance/diabetes mellitus
* Antibiotics: rifampin, rifabutin, nafcillin * Growth retardation (children)
* Hirsutism
E. Antimetabolites * Hyperlipidemia
* Hypertension
• These agents interfere with both T- and B-cell prolif- * Increased appetite
eration after cellular activation by interfering with * Osteoporosis
nucleotide synthesis required for DNA replication * Peptic ulcers
* Water and salt retention
• Mycophenolate mofetil (CellCept®, Myfortic®) has
largely replaced azathioprine (Imuran®) as agent of • Dosage:
choice * High initial dose tapered over weeks or
* Mycophenolate mofetil is a reversible months
inhibitor of Inosine Monophosphate dehydro- * Solumedrol (250 mg IV every 12 hours for 3
genase (IMPDH); the rate-limiting enzyme in doses) is frequently used perioperatively, then
de novo purine synthesis, blocking B- and T- PO prednisone is gradually tapered to 0.1
cell proliferation mg/kd/day over a few months
* Azathioprine is a purine analogue, incorpo-
rated into the cellular DNA, inhibiting the G. Rapamycin (Sirolimus)
synthesis and metabolism of RNA (blocking
transcription) • Most recent immunosuppressive agent introduced

• Commonly used in combination to prevent acute • Distinct mechanism of action: interferes with a
cellular rejection. cell-cycle specific regulatory kinase (mTOR:
mammalian target of rapamycin)
• Side effects
*Leukopenia and anemia • Dosage: 5–10 mg PO daily
*Diarrhea
• Blood level
• Dosage: 1–2 g PO/IV per day * Initially 12–15 ng/mL during the first 4
months after transplant
F. Corticosteroids *4–12 ng/mL thereafter

• Known to prolong allograft survival since 1951 • Side effects


* Anemia, leukopenia
• Mechanism of action is complex and likely * Hypercholesterolemia
involves blockade of multiple inflammatory * Wound complications
cytokines at many stages of the activation of the
immune system H. Targeting B-cells and Plasma Cells: (anecdo-
tal evidence for benefit, not FDA approved for
• Low dose blocks IL-1; high dose blocks IL-2 this indication and almost certainly not testable
as well in the upcoming year)

• Side effects
* Acne
* Adrenal cortex suppression
* Cataracts
* Dermal atrophy

292 EDUCATIONAL REVIEW MANUAL IN UROLOGY


• Rituximab: Monoclonal antibody against CD 20 • Typical regimen:
on B-lymphocytes and mediates depletion of these * 500 mg–1g IV methylprednisolone
cells. Approved for use in B-cell malignancies and for 3 days
used off in transplantation. Use in attempts to * 120–250 mg IV methylprednisolone
reduce the high levels of preformed HLA antibod- or PO prednisone for 3–5 days
ies, treatment of humoral rejection, treatment of * Antibody treatment
PTLD, and facilitation of living donor transplant • OKT3 or thymoglobulin will reverse
with positive cross match or ABO incompatibility about 90% of first acute rejection: greater
effectiveness at the expense of more side
• Bortezomib: Proteasome inhibitor resulting in cell effects
apoptosis. Approved for treatment of multiple • Generally reserved for more severe rejec-
myeloma. Recently introduced in the treatment of tion (Banff II or Banff III) or Banff I that
rejection, mainly antibody-mediated does not improve with steroids

I. Acute Rejection • Antibody-mediated acute rejection


* Immunofluorescence will reveal IgG, some-
• Suspect rejection after other causes of transplant times IgM and C3 in the wall of arteries
dysfunction have been ruled out: * Diffuse peritubular capillary staining for C4d
* Dehydration * Treatment should be aggressive
* Bacterial or viral infection • OKT3/thymoglobulin
* Drug toxicity • Intravenous immunoglobulin (IVIg)
* Obstruction • Plasmapheresis
* ATN • Anti–B-cell antibodies
* Bortezomib
• Diagnosis of acute rejection requires a kidney
transplant biopsy, usually performed with local
anesthesia under ultrasound guidance

• The Banff 97 classification is used to grade the


histopathological severity of rejection
* Grade I: Tubulointerstitial rejection; severe
interstitial inflammation (>25% of
parenchyma) and foci of moderate tubulitis
(>4 lymphocytes/tubular cross-section)
* Grade II: Vascular rejection; severe intersti-
tial inflammation (>25% of parenchyma) and
foci of severe tubulitis (>10 lympho-
cytes/tubular cross section); ± mild or moder-
ate intimal arteritis
* Grade III: Severe transmural arteritis, arterial
fibrinoid necrosis, ± focal infarction or inter-
stitial hemorrhages

• Treatment of acute rejection


* Steroid-pulse
• Will reverse about 75% of first acute
rejection, especially if Banff I

CHAPTER 9: RENAL TRANSPLANTATION 293


4. Recipient Evaluation

A. Contraindications to Kidney Transplant a pathognomonic feature. So diagnosis of


FSGS at this early stage is a diagnosis of
• Malignancy exclusion
* Immunosuppressive drugs may unfavorably * Polycystic kidney disease
influence natural history of the malignancy • Will not recur unless donor also had
* Recommended tumor-free waiting periods for disease (e.g., sibling)
common malignancies • Recipients with headaches, other central
• Kidney nervous system symptoms or familial his-
* Incidental: no waiting tory of aneurysms should be screened for
* Large, infiltrating lesion: intracranial aneurysms
at least 2 years • May require native nephrectomy for pain,
* Wilms’ tumor: at least 2 years size, infection or suspicious mass
* May be wise to remove complex * Hyperoxaluria
cystic mass before transplant • Type 1 hyperoxaluria is an autosomal-
• Bladder recessive inborn error of metabolism:
* In situ: no waiting missing hepatic alanine-glyoxylate
* Invasive: at least 2 years aminotransferase
* Beware the need for BCG in • Missing enzyme leads to oxalate deposi-
immunosuppressed patients tion in the kidneys and eventually ESRD
• Testis, prostate: at least 2 years • Kidney transplant failure will rapidly
• Breast, skin melanoma, invasive occur despite very aggressive manage-
cervical neoplasia: at least 5 years ment designed to minimize oxalate
deposition
• Infection • Require combined liver-kidney transplant
* Chronic infection precludes transplantation * Goodpasture’s syndrome/disease
* Osteomyelitis and diabetic foot ulcers must • Autoimmune disorder characterized by
be addressed before transplantation glomerulonephritis, circulating anti-
* Tuberculosis: preferable to wait 1 year after glomerular basement membrane (GBM)
completion of treatment antibodies (Goodpasture’s disease).
* HIV If diffuse pulmonary hemorrhage is
• A previous absolute contraindication also present, it is referred to as
• Now being reconsidered with the success Goodpasture’s syndrome
of antiretroviral therapy • Will frequently recur (histopathological
evidence deposition of IgG along the
• Recurrence of primary disease glomeruli), but without functional dam-
* Diabetes mellitus age to the kidney
• May recur but low risk (5%–10%) of * IgA nephropathy (Berger’s disease)
graft failure • IgA deposition in the glomerular
* Focal segmental glomerulosclerosis (FSGS) mesangium
• May recur rapidly and aggressively in up • High rate of recurrence in transplanted kid-
to 25% of patients after kidney transplant ney (40%–60%), but slowly progressive,
• 10%–65% risk of graft failure causing graft failure in 10% of patients
• The diagnosis of FSGS in the early post
transplant period is problematic, since • Noncompliance
significant proteinuria may precede diag- * Past history of noncompliance with medical
nostic histopathologic features by weeks. therapy: at high risk for graft loss
Only EM shows foot process fusion * Wise to demand period of acceptable compli-
retraction, which is characteristic but not ance before transplant

294 EDUCATIONAL REVIEW MANUAL IN UROLOGY


• Psychiatric illness • Prostatic enlargement in older men
* Any condition (organic mental syndromes, * Prostate may be resected before transplant if
psychosis and mental retardation) that may the patient produces sufficient volume of
impair the patient’s capacity to understand the urine
transplantation, its demands and complica- * Anuric patients should not be resected:
tions may be a contraindication to transplant dry TURP syndrome (strictures
and contractures)
• Severe extrarenal disease * Clean intermittent catheterization (CIC) is
* Chronic liver disease and advanced uncor- safe and effective in transplant patients
rectable heart disease are contraindications to
kidney transplant, but patients may benefit
from combined organ transplant
* Chronic lung disease may prevent safe
general anesthesia
* Severe peripheral vascular disease may pre-
clude arterial anastomosis and endanger limb

B. Urological Evaluation

• Ideally: sterile, continent and compliant urinary


tract before transplant

• Obtain voiding cystourethrogram in patients with


suspected abnormality

• Even very small bladder may develop normal


capacity and compliance after transplantation,
unless extensive fibrosis is present

• Transplant is possible with similar outcome in


patients with urinary diversion or augments

• Indications for native nephrectomy:


* Chronic or recurrent acute bacterial
pyelonephritis
* Infected stones
* Heavy proteinuria
* Intractable hypertension
* Polycystic kidney disease: massive kidneys,
recurrent infection or bleeding
* Infected reflux
* Acquired renal cystic disease with suspicion
of adenocarcinoma

CHAPTER 9: RENAL TRANSPLANTATION 295


5. Living Donors

A. Living donors represent about 45% of kidney • Abdominal/pelvic CT scan with contrast (renal
transplants performed in the United States anatomy) or angiogram

B. HLA-matched cadaveric kidney fares worse • ABO, HLA testing and crossmatch
than poorly matched living donor kidney
• Donors with high risk for cardiovascular disease
C. Potential advantages of living vs cadaveric “males >45 years old, females >55 years old, EKG
kidney donation abnormalities, hyperlipidemia, hypertension,
smoker” consider cardiac stress testing
• Better short- and long-term results * Chest x-ray. Consider pulmonary function
tests and chest CT for smokers
• More consistent early function (less delayed graft * Mammogram for females >40 years old,
function) and easier management Pap smear for females <65 years old
* Colonoscopy >50 years old, earlier if
• No brain death stress on transplanted kidneys higher risk
* PSA for Caucasian males >50 or African
• Reduced waiting time for recipients American >40 years old

• Ability to time transplant for convenience or F. Exclusion Criteria


medical reasons
• Age <18 or >65
D. Potential Disadvantages of
Living Kidney Donation • Blood pressure >140/90
(or antihypertensive medication)
• Immediate risk to the donor
* Mortality: 0.02% • Diabetes or abnormal glucose tolerance test
* Major morbidity: 0.2%
* Minor morbidity: 10% • Proteinuria >250 mg/day

• Long-term morbidity • Measured glomerular filtration rate


* Possible mild hypertension and proteinuria <80 mL/min/1.73 m2
but rates similar to other family members
* Risk of traumatic injury to remaining kidney • Adult polycystic kidney disease families:
* Risk of unrecognized renal disease abnormal CT scan in adult >30 years old

E. Living Donor Evaluation • Hematuria

• History and physical examination • Recurrent urolithiasis

• Comprehensive laboratory panel plus: • Significant medical history: malignancy, chronic


*Viral serology lung or liver disease
*Urine analysis and culture
*24-h urine collection for protein and creati- • History of thrombosis or thromboembolism
nine and/or random albumin:creatinine ratio
• Abnormal psychological evaluation
• Psychological and social evaluation

296 EDUCATIONAL REVIEW MANUAL IN UROLOGY


6. Cadaveric Donors

G. Donor Nephrectomy A. Brain Death

• The better kidney stays with the donor: always • Since 1968, brain death criteria has been used to
transplant the kidney with minor anomaly identify organ donors

• Consider right donor nephrectomy in female of B. Expanded Criteria Donors (ECD)


childbearing age (right-sided hydronephrosis of
pregnancy) • In an attempt to reduce the discrepancy between
available organs and waiting list patients,
• Can be performed open or with laparoscopy expanded criteria for cadaveric donation have
been used
• Institutions with long experience of laparoscopic
techniques have excellent outcomes, comparable • An expanded criteria donor is:
to open technique * Age >60
* Age 50–60 and at least 2 of the following:
• Open donor nephrectomy • History of hypertension
* Pros • Serum creatinine >1.5
• Established technique, safe and effective • Cause of death: cerebrovascular accident
• Relatively short operating time
• Minimal warm ischemia • These organs may not benefit all transplant candi-
• Excellent graft function dates and are often allocated to older recipients
* Cons and/or those with urgent need
• Postoperative pain
• Longer convalescence C. Donation After Cardiac Death (DCD)
• Surgical scar, with potential hernia
• Death is declared on the basis of cardiopulmonary
• Laparoscopic kidney donation criteria “irreversible cessation of circulatory and
* Pros respiratory function” rather than the neurologic
• Possibly smaller incision criteria used to declare (brain death) “irreversible
• Shorter hospital stay loss of function of the entire brain including the
• Less pain medication brain stem”
• Faster return to work
• Increased donor acceptance • DCD donor death occurs when respiration and cir-
• Excellent graft function culation have ceased with no cardiopulmonary
* Cons function and do not resume spontaneously. EKG
• Longer operative time silence is not required for death determination, the
• Learning curve may be associated with criterion is the absence of circulation; pulseless-
surgical damage to the kidney ness
• Shorter vessels
• Right kidney laparoscopic nephrectomy
may be more challenging
• Pneumoperitoneum may compromise
renal blood flow, with unknown long-
term effects

CHAPTER 9: RENAL TRANSPLANTATION 297


D. Organ Allocation * Negative T-cell and positive B-cell
cytotoxic crossmatch: first transplant
• Waiting list possible (antibody induction), but re-
* National list for “zero mismatch” kidneys (no do transplant at very high risk, espe-
HLA antigens different between donor and cially if DSA against HLA-DR are
recipient) present
* Regional list for local donors * Positive B-cell flow crossmatch only:
* For each donor, a list is generated of patients second transplant with high titer DSA
with the same blood group according to not recommended
points for: * Better matching associated with bet-
• Waiting time ter long-term results when all else
• High antibody levels equal but donor quality (living donor)
• Degree of HLA antigen match trumps HLA matching
(especially HLA-B and HLA-DR) * First 2 healthy recipients with nega-
• Other local variances (e.g., children) tive crossmatch get the kidneys

• Tissue typing
* Panel reactive antibody (PRA): statistical risk
of positive crossmatch in a given population
* Crossmatch
• Assesses the presence of donor-specific
antibodies (DSA) in a specific donor-
recipient patient
• Cytotoxic crossmatch is performed by
mixing donor cells with serum from the
recipient and complement, then assessing
viability of donor cells
• Flow cytometry crossmatch also assesses
the presence of DSA, using HLA antigens
immobilized on very small latex beads
• Flow crossmatch is more sensitive, but
less specific because it may identify anti-
bodies that won’t destroy donor cells
• Reminder:
* T-cell crossmatch assesses
HLA class I Donor Specific Antibody
(DSA)
* B-cell crossmatch assesses
HLA class II DSA
• Potential crossmatch results
* Positive cytotoxic T-cell crossmatch:
never transplant
* Positive T-cell flow crossmatch only:
first transplant possible (antibody
induction), but do not perform re-do

298 EDUCATIONAL REVIEW MANUAL IN UROLOGY


7. Ischemia-reperfusion Injury and 8. Surgical Technique
Delayed Graft Function and Complications

A. The delayed graft function (DGF) is the failure of A. Intraoperative Decisions


the kidney to function immediately, usually
defined as need for dialysis in the first week. • Always attempt to preserve lower pole arteries as
Ischemia-reperfusion injury (IR) and rejection they often supply blood to the ureter
contribute
• It is acceptable to tie off small upper pole arteries
B. IR can promote local inflammation (upregulation
of cell adhesion molecules, costimulatory • Small venous branches can be ligated without con-
molecules and histocompatibility antigens) that sequence
can lead to rejection
• In cadaveric donors, a short right renal vein can be
C. The resulting DGF is a strong predictor of short- elongated with the use of the supplied inferior
and long-term graft survival, independent of vena cava (almost routinely done for right cadav-
rejection eric transplant)

D. Differential Diagnosis of Delayed • Cadaveric iliac vessels can be used as extenders


Graft Function for both vein or artery

• Acute tubular necrosis • It is possible to conjoin arterial branches end-side


or end-end (Wallace-like) on the bench table, on
• Prerenal renal failure (hemorrhage) ice before implantation. Alternatively, 2 separate
anastomoses can be performed for arterial
• Vascular stenosis or thrombosis branches that are far apart

• Postrenal obstruction • Small lower pole arterial branches can be anasto-


(ureteral or Foley obstruction) mosed end-end with the inferior epigastric artery

• Renal parenchymal injury (nephrotoxic drugs, • It is possible to use the ipsilateral native ureter
recurrence of primary renal disease) end-end without performing a native nephrectomy

• Acute rejection • If 2 ureters are present, they can be implanted


together (conjoined) or separately
E. Pulsatile perfusion may improve early func-
tion, especially with long cold ischemia times B. Short Ureter

F. Perioperative maneuvers to reduce • Sometimes, the transplant ureter will not reach the
the impact of IR bladder safely

• Adequate hydration • How to close a ureteral gap


* Psoas hitch
• Mannitol * Boari flap
* Native ureteroureterostomy
• Furosemide * Native ureteropyelostomy
* Pyelocystostomy
• Intraarterial verapamil * Ileal ureter

• Low-dose dopamine

CHAPTER 9: RENAL TRANSPLANTATION 299


9. Surgical Complications 10. Early and Late Patient
Management Issues

A. Vascular Leak or Thrombosis A. Early Graft Dysfunction

• Incidence: 1%–2% • Differential diagnosis


* Acute tubular necrosis (ATN)
• Re-operation, incurs a high risk to lose kidney * Acute rejection
* Drug toxicity
B. Urine Leak or Ureteral Obstruction * Vascular (arterial or venous) compromise
* Prerenal renal failure (hemorrhage, volume
• Incidence: 4%–5% contraction)
* Postrenal renal failure (ureteral or bladder
• May require revision of the ureteral anastomosis catheter obstruction)
* Renal parenchymal injury (nephrotoxic
• Many endoscopic approaches are available drugs, recurrence of primary renal disease)

• Surgery may be preferable for early leaks, necrotic • Management


ureter or failure or endoscopic management * Check bladder catheter for patency, irrigate as
needed
C. Lymphocele * Fluid bolus
* IV furosemide
• Incidence * Check drug levels
* 5%–10% * Renal ultrasound with Doppler or nuclear
* May be reduced by minimal iliac vessel dis- medicine renal scan to assess patency of vas-
section and ligation of visible lymphatics, as cular anastomosis
opposed to electrocoagulation * Transplant biopsy to rule out rejection

• Presentation B. Long-term Issues


* Ureteral obstruction: rising serum creatinine
* Iliac vein compression: ipsilateral leg edema, • Chronic allograft nephropathy
deep-vein thrombosis * Acute changes superimposed on a
background of chronic kidney disease
• Small lymphocele may be asymptomatic and does associated with a slow progressive decline
not require treatment in renal function
* The histologic constellation of arteriosclero-
• The larger the lymphocele, the more likely it will sis, arteriolosclerosis, glomerulosclerosis,
be symptomatic and require treatment tubular atrophy and interstitial fibrosis with
chronic inflammation
• Treatment options * The pathogenesis could be immune-mediated
* Percutaneous drainage chronic rejection and/or non–immune-medi-
* Sclerosis (fibrin glue) ated as original disease recurrence and drug
* Peritoneal window (can be performed toxicities
open or laparoscopically)
• Hypertension
D. Wound Infection
• Diabetes
E. Hernia
• Heart disease (it is the most common cause of
long-term graft loss)

300 EDUCATIONAL REVIEW MANUAL IN UROLOGY


• Infections • Management
* Cytomegalovirus (CMV) infection * Optimal management has yet to be
• Previously referred to as the “42-day determined
fever” from its frequent occurrence * Reduce immunosuppression
around 42 days after transplant * Stop mycophenolate mofetil and
• May be transmission from donor or reac- introduce leflunomide (pyrimidine
tivation of latent infection synthesis inhibitor)
• Donor and recipient CMV status used to * Antiviral agents: cidofovir
stratify risk: highest risk when donor is (nephrotoxic)
positive and recipient is negative
• Prophylaxis • Erectile dysfunction
* Acyclovir *Possible etiology
* Ganciclovir • Drug-induced
* Valganciclovir • Widespread atherosclerosis
* Cytogam (IV Ig from CMV antibody • Iatrogenic, if internal iliac artery is used
positive blood donors) for the arterial anastomosis
• Target organ affected * Likely more than 1 etiology present
* GI tract (oesophagus, stomach) at a given time
* Liver * Phosphodiesterase type 5 inhibitors can be
* Glomerulopathy used in transplant patients
* Retinitis

• Non-enveloped, double-stranded DNA


*BK virus infection

polyomavirus
• Latent in immunocompetent adults, with
seroprevalence ranging from 60%–80%
• Viruria and viremia can be found in 35%
and 10% of transplant patients, respec-
tively
• BK virus nephropathy (BKVN) is found
in 5%–10% of transplant patients and can
cause renal allograft loss in 50% of those
patients
• Presentation
* Increased serum creatinine
* Hemorrhagic cystitis
* Ureteral stenosis
* Sterile pyuria
• Diagnostic
* Presence of virus in blood and urine
(quantified by PCR)
* DNA hybridization on renal allograft
biopsy

CHAPTER 9: RENAL TRANSPLANTATION 301


11. Questions

1. Which of the following factors do not con- C. Place a suprapubic tube before the kidney
tribute significantly to erectile dysfunction in transplant
men after a kidney transplant?
D. Start an alpha blocker posttransplantation,
A. Use of both internal iliac arteries with mul- and teach self-catheterization if necessary
tiple kidney transplantations
E. Place a prophylactic prostatic stent, then
B. Prolonged hypertension proceed with transplantation

C. Diabetes mellitus

D. Elevated prolactin levels 4. When treating urolithiasis of a transplanted kid-


ney, which of the following treatments is often
E. Side effects of hypertension drugs more difficult in the transplanted kidney?

A. Retrograde ureteroscopy

2. A 54-year-old male presents to his 4-year status B. Antegrade ureteroscopy


postrenal transplantation with erectile dysfunc-
tion. The graft is functioning well with a serum C. Percutaneous nephrolithotomy
creatinine of 1.2. Which of the following rec-
ommendations for his ED would not be appro- D. Laser lithotripsy
priate?
E. Extracorporeal shockwave lithotripsy
A. Sildenafil

B. Tadalafil
5. The placement of prophylactic ureteric stents
C. Intracorporeal prostaglandin injections with kidney transplantation has been associated
with which of the following?
D. Placement of penile prosthesis
A. High risk of stent encrustation unless
E. Most ED treatments are unsafe in kidney removed within 3 months of kidney trans-
Transplant patients plant

B. Increased ureteric complications

3. A 60-year-old male with bladder outlet obstruc- C. Higher overall cost


tion due to an enlarged prostate is preparing to
undergo a living-related kidney transplant. He D. More urinary tract infections unless micro-
is anuric currently, but worried that he will be bial prophylaxis is added
unable to void after the transplant. Which of the
following would be an appropriate manage- E. Improved patient survival
ment strategy?

A. Perform a prophylactic TURP, then pro-


ceed with transplantation

B. Perform a TURP at the same time as the


kidney transplant

302 EDUCATIONAL REVIEW MANUAL IN UROLOGY


6. A 37-year-old female, 3 days status post-kidney A. High terminal creatinine
transplantation, presents with a ureteric leak.
During open exploration, the urologist notes the B. Donor age
transplant ureter is entirely necrotic and that
there is a mobile bladder and a healthy, well-per- C. 2 ureters
fused native ureter available. Which of the fol-
lowing options would be an appropriate manage- D. 30% glomerulosclerosis
ment strategy?
E. 2 renal arteries
A. Cutaneous ureterostomy

B. Creation of a colon conduit 9. Which of the following is least likely to cause


an elevation of serum creatinine in a transplant
C. Native ureteropyelostomy recipient?

D. Creation of ileal ureter A. High sirolimus level

E. Percutaneous nephrostomy for 6 weeks and B. High cyclosporine level


then re-explore
C. Acute rejection

D. BK virus infection
7. A 42-year-old man who received a kidney trans-
plant 12 years ago is referred because of new E. Ureteral necrosis
transplant hydronephrosis on an ultrasound. The
bladder was empty. His serum creatinine is 1.5
mg/dl and has been stable at this level for years. 10. A transplant patient has a baseline serum Cr of
What would be the next most appropriate study? 1.8 mg/dL and takes tacrolimus, MMF and
steroids. Because of persistent hypertension,
A. Noncontrast CT scan of abdomen and pelvis he is started on an ACE inhibitor. One week
later the Cr is 3.1 mg/dL. The most likely
B. Diuretic MAG-3 renogram explanation is:

C. Surgical exploration A. Acute rejection

D. Transrectal ultrasound B. Renal artery stenosis

E. Antegrade nephrostogram C. Hypotension and acute tubular necrosis

D. Acute renal vein thrombosis

8. A transplant center is offered a cadaveric kidney E. Tacrolimus toxicity since the ACE inhibitor
from a 53-year-old donor who died from head raised the blood levels
trauma. Terminal creatinine was 1.6 mg/dL.
Patient had a history of hypertension well con-
trolled with 1 drug for 2 years. It is a left kidney
with 2 arteries and 2 ureters. Biopsy shows 30%
glomerulosclerosis. The most likely reason to
turn down this kidney is:

CHAPTER 9: RENAL TRANSPLANTATION 303


11. Which of the following is used to both prevent
and treat acute rejection?
Answers

1. D.
A. Azathioprine
2. E.
B. Tacrolimus
3. D.
C. Cyclosporine
4. A.
D. Basiliximab
5. D.
E. Thymoglobulin
6. C.

7. B.
12. Which of the following would be a contraindi-
cation to receiving a kidney transplant? 8. D.

A. Diabetes 9. A.

B. Primary ureteral reflux 10. B.

C. Untreated tuberculosis 11. E


(the other drugs listed only prevent rejec-
D. Ileal conduit tion).

E. Bladder augmentation 12. C.

304 EDUCATIONAL REVIEW MANUAL IN UROLOGY


Chapter 10:
Urodynamics
Victor W. Nitti, MD

Contents

1. Introduction

2. The Role of Urodynamics in Clinical


Practice

3. Functional Classification of Voiding


Dysfunction

4. Simple Tests

5. The Urodynamics Test

6. Videourodynamics

7. Further Reading

8. Questions

CHAPTER 10: URODYNAMICS 305


1. Introduction 2. The Role of Urodynamics in
Clinical Practice

The lower urinary tract has two essential functions: UDS has been used for decades, yet clear-cut, level
the storage of urine at low pressure and the vol- 1, evidence-based indications for UDS are lacking.
untary evacuation of urine. Low pressures are This is not surprising when one considers how diffi-
essential to protect kidneys and assure continence, cult it is to collect level 1 evidence for the use of
while voluntary evacuation allows for the elimina- UDS. It is difficult to conduct proper randomized
tion of urine in socially acceptable situations with- controlled trials on UDS for conditions where lesser
out fear of leakage or overdistension. It is clear that levels of evidence and expert opinion strongly sug-
a number of diseases affect the lower urinary tract gest clinical utility and where empiric treatment is
and disrupt the storage and/or evacuation of urine. potentially harmful or even life-threatening (e.g.,
This can lead to bothersome symptoms (e.g., uri- neurogenic voiding dysfunction). Furthermore,
nary incontinence or pain from failure to empty) or symptoms can be caused by a number of different
in some cases potentially harmful sequela. In many conditions and it is difficult to study pure or homo-
cases, a precise assessment of storage and emptying geneous patient populations. Given the current state
is necessary to optimally treat patients. Urodynam- of evidence for UDS studies, what is most important
ics (UDS) is the dynamic study of the transport, is that the clinician has clear-cut reasons for per-
storage and evacuation of urine. It is comprised of a forming the study and that the information obtained
number of tests which individually or collectively will be used to guide treatment of the patient. There-
can be used to gain information about urine storage fore, it is probably more useful to describe the role
and evacuation. of UDS in clinical practice rather than precise indi-
cations for its use.
UDS is most useful when history, physical exam
and simple tests are not sufficient to make an accu- In practical terms, UDS is most useful when history,
rate diagnosis and/or institute treatment. In some physical exam and simple tests are not sufficient to
cases, this may be to obtain an accurate diagnosis make an accurate diagnosis and/or institute treat-
for what condition is causing symptoms (e.g., lower ment. This has clinical applicability in 2 general
urinary tract symptoms or incontinence). In others, scenarios:
it may be to determine the impact of a disease that
has the potential to cause serious and irreversible 1. To obtain information needed to make an accurate
damage to the upper and lower urinary tract (e.g., diagnosis for what condition(s) is causing symp-
neurological diseases like spinal cord injury, multi- toms (e.g., lower urinary tract symptoms or
ple sclerosis or radiation cystitis). Sometimes pro- incontinence).
found abnormalities can be found in the relative
absence of symptoms. 2. To determine the impact of a disease that has the
potential to cause serious and irreversible damage
As mentioned above, one of the most important to the upper and lower urinary tract (e.g., neuro-
parts of UDS is its proper performance with careful logical diseases like spinal cord injury, multiple
attention to technical details so that accurate inter- sclerosis or radiation cystitis). Sometimes pro-
pretation is possible. It is beyond the scope of this found abnormalities can be found in the relative
chapter to describe the proper performance of UDS; absence of symptoms.
however, the reader is referred to Schaefer et al for
good urodynamic practices and Abrams et al for ter-
minology (see Further Reading).

306 EDUCATIONAL REVIEW MANUAL IN UROLOGY


Rather than refer to a list of indications for UDS that detrusor overactivity in a patient with urgency
often are not evidence-based at all, it is more useful incontinence). Finally, not all UDS observations are
for clinicians to think of how UDS should be used in clinically significant. Therefore, it is important to
a broader clinical perspective. In keeping with that interpret UDS studies in the context of the patient’s
theme, the role of UDS in clinical practice has been history, including symptoms and concomitant dis-
nicely summarized by Hosker et al for the following eases/conditions, and other information like
situations: postvoid residual volumes and frequency volume
charts (voiding and intake diaries) when clinically
1. To identify or rule out factors contributing to applicable.
lower urinary tract dysfunction (e.g., urinary
incontinence) and assess their relative importance

2. To obtain information about other aspects of


lower urinary tract function or dysfunction

3. To predict the consequences of lower urinary tract


dysfunction on the upper urinary tract

4. To predict the outcome, including undesirable


side effects, of a contemplated treatment

5. To confirm the effects of intervention or under-


stand the mode of action of a particular type of
treatment (especially a new one)

6. To understand the reasons for failure of previous


treatments for symptoms (e.g., urinary inconti-
nence) or for lower urinary tract function in gen-
eral

In order to use UDS in a practical and effective way,


it is important that the clinician has the proper
expertise to know when and why to perform a UDS
study. Despite many technical advances in the
recording, processing and printing of UDS studies,
careful attention to technical details to assure accu-
rate collection of data remains the cornerstone of a
good study. Since not all patients undergo UDS for
the same reasons, the clinician should customize
UDS to the patient’s symptoms and condition. That
means deciding on the questions to be answered
before starting each study and designing that study
to obtain the answers to those questions. It is impor-
tant to remember that UDS is performed in an
unnatural setting and does not always duplicate real
life. A UDS study that does not duplicate com-
plaints or symptoms when an abnormality is
recorded is not necessarily diagnostic. In addition,
failure to record an abnormality does not always
rule out its existence (e.g., failure to demonstrate

CHAPTER 10: URODYNAMICS 307


3. Functional Classification of
Voiding Dysfunction

In order to formulate a set of questions to be intricate neurophysiologic aspects of voiding and


answered by a urodynamic test, an understanding of storage dysfunction and fail to think in practical
the possible causes of symptoms and the possible terms. One should always focus on the possible uro-
urodynamic manifestations of a preexisting condi- dynamic findings in a given case and how each of
tion is necessary. In order to accomplish this, a prac- the findings may ultimately affect treatment. Obvi-
tical classification of voiding dysfunction is invalu- ously, these potential findings will be determined by
able. The system proposed and popularized by Wein symptoms and/or underlying disease.
(1982) is simple and allows one to classify voiding
dysfunction according to urodynamic findings. In
addition, this functional classification system helps
to clarify treatment options for a given patient.

Functional abnormalities of the lower urinary tract


can be divided into:

1. Storage dysfunction (failure to properly store


urine)

2. Emptying dysfunction (failure to empty the blad-


der normally)

3. Combined dysfunction (failure to store and


empty)

Anatomically, storage and emptying abnormalities


can be caused by:

a. Overactive (causing failure to store)


Bladder dysfunction

b. Underactive (causing failure to empty)

a. Overactive (causing failure to empty)


Bladder outlet dysfunction

b. Underactive (causing failure to store)

Combined bladder and bladder


outlet dysfunction

The urodynamic evaluation should help to deter-


mine if there is bladder or bladder outlet dysfunc-
tion (or both) and whether there is a storage and/or
emptying problem. By providing answers to these
simple questions, the urodynamic test can lead to a
correct diagnosis and, equally as important, institu-
tion of appropriate treatment. Obviously, an under-
standing of the physiology of urine storage and
voiding, and the pathophysiology of voiding dys-
function, is required to formulate appropriate ques-
tions to be answered by a urodynamic study. How-
ever, all too often clinicians become caught up in the

308 EDUCATIONAL REVIEW MANUAL IN UROLOGY


4. Simple Tests 5. The Urodynamics Test

There are several simple, noninvasive tests that can The micturition cycle can be divided into 2 phases:
be used to evaluate voiding function and perhaps the filling (and storage) phase and the emptying
prompt further testing. phase. Similarly, the UDS can be divided into the
same 2 components.
Post-void Residual (PVR)

PVR is an excellent assessment of bladder empty-


Urodynamics: Filling and Storage

ing. It can be performed by ultrasound (bladder The cystometrogram (CMG) assesses the bladder’s
scan) or catheterization. Elevation of PVR indicates response to filling. The CMG can measure filling
a problem with emptying, but does not tell why. pressure, sensation, involuntary contractions, com-
pliance and capacity. Sensation is the part of cys-
tometry that is truly subjective and therefore
requires an alert and attentive patient and clinician.
Uroflowmetry

This is also an assessment of bladder emptying. There are several subjective parameters that can be
Normal uroflow is a bell-shaped curve. When the recorded during filling that are recognized by the
flow rate is reduced or the pattern is altered, this International Continence Society (ICS):
could indicate bladder (underactive) or bladder out-
let (obstruction) dysfunction. First sensation of bladder filling. First aware of
bladder filling.

First desire to void. Feeling that would lead patient


to pass urine at next convenient moment, but void-
ing can be delayed if necessary.

Strong desire to void. Persistent desire to void with-


out fear of leakage.

Urgency. Sudden compelling desire to void.

Normally, the bladder should store urine at low


pressure and not contract involuntarily. Once capac-
ity is reached or voluntary voiding is desired, intrav-
esical pressure will increase (voluntary detrusor
contraction). In actuality, this is preceded by a relax-
ation of the external sphincter. A normalized adult
CMG is depicted on the next page (Figure 1).

During UDS, bladder pressure is measured by an


intravesical catheter (usually transurethral). It is
well known that increases in intraabdominal pres-
sure are transmitted to the bladder. Therefore, when
intravesical pressure is monitored during cystomet-
ric studies, it must be realized that this pressure is
actually the sum of intraabdominal pressure and the
pressure generated by the detrusor itself, either
through a contraction or wall tension with bladder
filling. The simultaneous measurement of abdomi-
nal pressure (Pabd), usually by a rectal or vaginal
catheter and vesical pressure (Pves) during urody-
namics, provides a means of calculating the true

CHAPTER 10: URODYNAMICS 309


detrusor pressure (Pdet) by subtracting abdominal
Pabd from Pves. The ability to calculate subtracted
Figure 1

detrusor pressure allows one to distinguish between


a true rise in detrusor pressure (either via a contrac-
A normalized adult CMG

tion or loss of compliance) and the effect of


increased abdominal pressure (e.g., straining, Val-
salva). This is especially important when rises in
F illing V oid ing

detrusor pressure are small or when they are accom-


P h ase P h ase

panied by changes in abdominal pressure (Figure 2).

Adding intraabdominal pressure monitoring gives a


better representation of the true detrusor pressure.
P ressure
cm H 2 O

In Figure 2A the total vesical pressure tracing is pre-


sent (CMG)—note the multiple spikes and rises in
pressure. Without having simultaneous monitoring
of intraabdominal pressure, it is impossible to dis-
tinguish if these pressure spikes are due to a rise in
V o lum e
mL

detrusor or abdominal pressure.

In Figure 2B the same tracing is present with


intraabdominal pressure monitoring added (multi-
channel UDS). One can clearly see now that the
changes in pressure were due to the changes in
abdominal pressure. When one looks at the Pdet
curve, it is noted to be flat and without any rises in
pressure.
Figure 2

Total vesical pressure and intraabdominal pressure

A CMG

M ulticha n nel U ro dy n am ics


B

310 EDUCATIONAL REVIEW MANUAL IN UROLOGY


sured excluding any detrusor contractions. Various
definitions of impaired compliance have been used
Abnormalities of Bladder Filling: Detrusor

(e.g., between 10 and 20 mL/cm H2O); however,


Overactivity and Impaired Compliance

During filling involuntary detrusor contractions there is not a consistent definition based on mL/cm
(IDC) can occur. These are often associated with H2O. In practical terms absolute pressure is proba-
urgency and even urge incontinence. Detrusor over- bly more useful than a “compliance number” or
activity (DO) is a urodynamic observation charac- value. It is well known that storage pressures >40
terized by involuntary IDCs during the filling phase cm H2O are known to be harmful to the upper tracts.
which may be spontaneous or provoked. According Impaired compliance is seen in a variety of neuro-
to the ICS, DO may be further characterized as neu- logical conditions, especially lower motor neuron
rogenic DO which means it is associated with a rele- lesions such as spina bifida and cauda equina syn-
vant neurological condition (e.g., spinal cord injury, drome. It can result from outlet obstruction
multiple sclerosis) or idiopathic DO which means (anatomical or functional) or structural changes like
that there is “no defined cause” (non-neurogenic). radiation cystitis or TB. Impaired compliance with
The term idiopathic is a bit of a misnomer, in that elevated storage pressures is a urodynamic risk fac-
the cause of DO in a non-neurogenic patient may be tor and usually needs to be treated to prevent renal
readily apparent (e.g., bladder outlet obstruction, damage (Figure 4).
inflammatory process) or may be truly “unknown.”
Thus, from a practical standpoint, the terms neuro-
genic and non-neurogenic DO make sense, but do
Leak Point Pressures

not fit the ICS definitions. (It should be noted that There are 2 distinct types of leak point pressures that
the term neurogenic DO replaced the term detrusor can be measured in the incontinent patient. The two
hyperreflexia and the term idiopathic DO replaced are independent of each other and conceptually
the term detrusor instability in the last ICS terminol- measure completely different things. The first is the
ogy.) abdominal leak point pressure (ALPP) which is a
measure of sphincter strength or the ability of the
From a practical standpoint during UDS, DO can sphincter to resist changes in abdominal pressure.
further be observed as a single event or multiple ALPP is defined as the intravesical pressure at
IDCs. It can be phasic, or terminal (occurring at the which urine leakage occurs due to increased abdom-
end of filling). It can also be suppressed or inal pressure in the absence of a detrusor contrac-
unabortable and may lead to leakage or precipitant
micturition. Classifying DO in such a way can be
valuable in certain circumstances. For example,
Figure 3

overactivity bladder symptoms associated with


obstruction have a higher likelihood of resolving
Detrusor overactivity

with intervention (e.g., TURP) when DO is terminal


rather than phasic.

The viscoelastic properties of the bladder, based on


its composition of smooth muscle, collagen, and
elastin, normally produces a highly compliant struc-
ture. Therefore as the bladder fills there is little
change in pressure (Figure 3). Compliance relation-
ship between change in bladder volume and change
in detrusor pressure (∆ volume / ∆ pressure) is mea-
sured in mL/cm H2O. The ICS recommends 2 stan-
dard points: the Pdet at start of bladder filling (usu-
ally zero) and the Pdet at cystometric capacity or
before the start of any detrusor contraction that
This UDS tracing depicts DO. Note the 2 IDCs (arrows).

results in significant leakage. Both points are mea-


There is a rise in Pves with no associated rise in Pabd and
therefore the subtracted Pdet looks identical to the Pves.

CHAPTER 10: URODYNAMICS 311


patients had high grade incontinence with 81% hav-
ing continuous leakage and 75% having a fixed ure-
Figure 4

thra (no urethral hypermobility). When ALPP was


between 61-89 cm H2O, 80% had pronounced ure-
UDS representation of impaired compliance

thral hypermobility and grade moderate to high


grade incontinence. When ALPP was >90 cm H2O,
patients had lesser grades of incontinence and mini-
mal to gross urethral hypermobility. The inference
is that:

ALPP <60 cm H2O signifies ISD

ALPP between 60-90 cm H2O is equivocal


(there is a component of ISD)

ALPP >90 cm H2O indicates little or no ISD

However, from a practical standpoint, if there is no


Note the rise in Pves (and Pdet) with bladder filling. The

hypermobility, SUI must be caused by ISD,


Pdet at the end of filling is approximately 45 cm H2O,
which is a potentially dangerous situation.

regardless of the ALPP. Current technology does


not permit a method to distinguish between ISD and
Figure 5

hypermobility. Therefore, although these ALPP val-


ues are often used, they should be interpreted with
Leak point pressure

caution.

The second type of leak point pressure is the detru-


sor leak point pressure (DLPP), which is a mea-
sure of detrusor pressure in patients with
decreased bladder compliance. It is defined as the
lowest detrusor pressure at which urine leakage
occurs in the absence of either a detrusor contrac-
tion or increased abdominal pressure.
Figure 6

Pressure in the bladder will continue to


increase as the bladder fills

tion. This measure of intrinsic urethral function is


Leakage at arrow = DLPP = 45 cmH2O

applicable to patients with stress incontinence. An


ALPP can only be demonstrated in a patient with
stress urinary incontinence. Conceptually, the
lower the ALPP, the weaker the sphincter. There
is no normal ALPP, as patients without stress incon-
tinence will not leak at any physiologic abdominal
pressure (Figure 5).

Attempts have been made to quantify intrinsic


sphincter deficiency (ISD) using ALPP. In 1993,
McGuire and associates used ALPP to measure 125
women with SUI. When the ALPP <60 cm H2O, all

312 EDUCATIONAL REVIEW MANUAL IN UROLOGY


The higher the urethral resistance, the higher the The urethral closure pressure profile (UCP) is
DLPP. One can imagine that in a poorly compliant given by the subtraction of intravesical pressure
bladder, if outlet resistance is low, incontinence will from urethral pressure.
occur at a relatively low or “safe pressure”. How-
ever, if outlet resistance is high, the pressure in the Maximum urethral pressure (MUP) is the highest
bladder will continue to increase as the bladder fills pressure measured along the UPP.
(Figure 6). There is potentially less incontinence,
but eventually the pressure is transmitted to the Maximum urethral closure pressure (MUCP) is
upper tracts. the maximum difference between the urethral pres-
sure and the intravesical pressure.
From a clinical perspective DLPP is most useful in
patients with lower motor neuron disease affect- Functional profile length is the length of the ure-
ing the bladder (spina bifida, spinal cord tumors, thra along which the urethral pressure exceeds
after abdominal-perineal resection of the rectum or intravesical pressure.
radical hysterectomy) and in non-neurogenic
patients with low bladder compliance (after multi- In most continent women, the functional urethral
ple bladder surgeries, radiation and tuberculous cys- length is approximately 3 cm and the MUCP is
titis). The higher the DLPP, the more likely is upper 40–60 cm H2O but normal values vary widely.
tract damage as intravesical pressure is transferred to
the kidneys. McGuire documented the deleterious One caveat of UPP is that its measurement does not
effects that a high leak point pressure has on the diagnose stress incontinence. For example, there is
upper urinary tracts; leak point pressures greater a difference between the urethra of an incontinent
than 40 cm H2O result in hydronephrosis or vesi- patient whose MUCP = 38 cm H2O and that of a
coureteral reflux in 85% of myelodysplastic patients. continent women with the same MUCP.

In summary, ALPP and DLPP, although both called In 2002, the ICS standardization subcommittee con-
“leak point pressure” are completely different. cluded that the clinical utility of urethral pressure
The ALPP measures the sphincter response to measurement is unclear. Furthermore, there are no
increased abdominal pressure. The lower the urethral pressure measurements that: 1.) discrimi-
ALPP, the “weaker” the sphincter. The DLPP nate urethral incompetence from other disorders; 2.)
measures the injured bladder response to increased provide a measure of the severity of the condition;
outlet resistance. The higher the resistance (e.g., 3.) provide a reliable indicator to surgical success,
detrusor-external sphincter dyssynergia), the higher and return to normal after surgical intervention.
the DLPP, which is potentially dangerous to the
upper tracts. Stress-induced Detrusor Overactivity

Sometimes detrusor overactivity can be triggered by


a rise in abdominal pressure. Thus the symptom
Urethral Pressure Profilometry

Despite an abundant literature on urethral profilom- may appear to be stress incontinence, but the condi-
etry, its clinical relevance is controversial. Many tion causing the symptom is actually an involuntary
urologists do not routinely perform urethral pro- contraction, not sphincteric weakness.
filometry. Urethral pressure is defined as the fluid
pressure needed to just open a loose urethra. The
urethral pressure profile (UPP) represents the
Occult Stress Incontinence

intraluminal pressure along the length of the urethra Stress incontinence is demonstrated in a clinically
in graphic form. Several parameters can be obtained continent woman with pelvic prolapse only when
from the UPP: the prolapse is reduced.

CHAPTER 10: URODYNAMICS 313


flow rate. A healthy bladder is able to overcome
obstruction by contracting more forcefully and,
Urodynamics:

although flow may be slower, the bladder is able to


Emptying – The Voiding Phase

Normal voiding accomplished by activation of mic- empty itself. Over time, the detrusor may decompen-
turition reflex, which involves: sate and may no longer be able to generate the neces-
sary pressure to overcome obstruction. When this
1. Relaxation of striated urethral sphincter occurs, retention of urine results. Thus, the urody-

2. Contraction of detrusor muscle (BOO) is high pressure and low flow (or more practi-
namic manifestation of bladder outlet obstruction

cally speaking, increased pressure and reduced flow).


3. Opening of vesical neck and urethra Over time, if bladder decompensation occurs, detru-
sor underactivity or impaired contractility can result.
4. Onset of urine flow
In males, obstruction has been defined based on
This occurs as a result of coordination between pon- the model of benign prostatic obstruction (BPO).
tine and sacral micturition centers, with suprapon- Using clinically obstructed and unobstructed (after
tine input which allows for voluntary control of the TURP) males, several different formulas or nomo-
micturition reflex. grams have been created. The ICS nomogram is
the one most commonly used today (Figure 7).
UDS can evaluate several parameters during the Using this nomogram and the bladder outlet
voiding phase including: obstruction index (BOOI) derived from it, men
can be divided into 3 groups: obstructed, equivocal
1. Detrusor contractility and unobstructed.

2. Relaxation of the bladder outlet Based on the slope of the curves, the BOOI is
defined as the detrusor pressure at maximum flow
3. Coordination of sphincters rate (PdetQmax) minus 2 times the maximum flow
rate (Qmax):
According to the ICS, normal detrusor function is
characterized by a voluntarily initiated continuous
contraction that leads to complete bladder emptying
BOOI = PdetQmax – 2(Qmax)

within a normal time span, and in the absence of A BOOI >40 is obstructed; BOOI <20 is unob-
obstruction. Detrusor underactivity is when there structed; and BOOI 20-40 is equivocal for obstruc-
is a contraction of reduced strength and/or duration, tion. This formula and nomogram applies only to
resulting in prolonged bladder emptying and/or a males, as female voiding dynamics are different.
failure to achieve complete bladder emptying
within a normal time span. Finally, an acontractile BOO is associated with abnormalities of storage as
detrusor is when there is no demonstrable contrac- well. This is presumably due to changes in ultra-
tion during UDS. However, when evaluating detru- structure that occur with obstruction. Detrusor over-
sor function urodynamically, one should correlate activity and impaired compliance occur in conjunc-
UDS with clinical findings. For example, if a patient tion with obstruction. For example, about two-thirds
who normally voids is unable to void during a UDS of men with symptomatic BPO have DO and one-
study, one cannot make a definitive diagnosis of half to two-thirds of the time DO resolves with treat-
acontractile detrusor. ment of obstruction. Reduced compliance is also
associated with obstruction and has been shown to
The Pressure Flow Relation improve with treatment of obstruction (TURP).
Detrusor pressure during voiding is a function of out-
let resistance. For a normal detrusor, the greater the Obstruction in women cannot be defined by the ICS
nomogram or the BOOI as these will grossly under-
during voiding. This is accompanied by a reduced estimate female BOO. This is because normally
outlet resistance, the higher the detrusor pressure

314 EDUCATIONAL REVIEW MANUAL IN UROLOGY


Figure 7

ICS nomogram

women void at much lower pressures than men and


therefore the obstructed female bladder outlet may
Figure 8

not respond as dramatically (or at least with the


same pressures) as in the male. Unfortunately there
is no condition in women that causes BOO as com-
monly as BPO in men and therefore creating a con-
sistent standard is difficult. Thus concepts are the
same (higher pressure and lower flow), but the val-
ues are different and less well defined. Those who
are interested are referred to the Further Reading
section.

Sphincter Coordination
Normal voiding requires external sphincter relax-
ation followed by contraction of the detrusor. The
external sphincter (and internal sphincter) should
remain relaxed until voiding is complete. This is
shown in Figure 8.

Note that the rise in detrusor pressure is preceded by


a fall in urethral pressure and relaxation of the exter-
nal sphincter as measured by electromyography
(EMG). The sphincter and urethral pressure remain
low during voiding and then increase when voiding
is completed.

CHAPTER 10: URODYNAMICS 315


The signal source for measurement of the EMG Normally, EMG activity decreases before a volun-
activity of the striated external sphincter is the elec- tary bladder contraction; however, it is not abnormal
trical activity of the external urethral sphincter, the for EMG activity to increase with an involuntary
external anal sphincter, or the pelvic floor muscles. contraction as part of a guarding reflex to inhibit the
Accurate EMG measurement of this relatively low IDC (Figure 9).
level of electrical activity can be somewhat diffi-
cult; however, performance of EMG studies as part In this case, the patient experienced an IDC, which
of routine urodynamics can be accurate and easily led to increased external sphincter and an increase
done. The 2 most commonly used electrodes are in EMG activity. However, shortly after, when the
patient voluntarily voids, the sphincter relaxes and
trodes. Most experts feel that the concentric needle EMG activity decreases.
surface electrodes and concentric needle elec-

electrode is the superior technique for obtaining a


signal source of EMG activity of the striated exter-
nal sphincter muscles but has the distinct disadvan- occurs when there is an involuntary increase in
Detrusor-external sphincter dyssynergia (DESD)

tage of being uncomfortable for the patient, espe- external sphincter associated with DO and also with
cially if more than one attempt at placement of the voiding. It is caused by a neurological lesion in the
electrode is required to obtain an adequate signal. suprasacral spinal cord. DESD can produce pro-
The surface electrodes have a significant advantage found changes as the detrusor involuntarily con-
compared with the needle electrode regarding tracts against a relatively closed sphincter. This will
patient convenience and comfort. However, the sur- result in high pressures and can even cause impaired
face electrodes are considered to provide an inferior bladder compliance over time. DESD is considered
signal source by some clinicians who routinely per- a urodynamic risk factor for upper tract deteriora-
form urodynamic studies, but surface electrodes can tion (Figure 10). If there is no neurological lesion,
provide very good signal source quality if properly then the dyssynergia is considered a learned behav-
used. It is important that the intensity of the EMG ior (dysfunctional voiding) and can often be
signal can determine what the tracing looks like. unlearned. True DESD cannot be unlearned and
One should observe relative changes in EMG activ- must be treated by bypassing the voiding phase
ity as opposed to raw numbers. (For a more detailed (e.g., with intermittent catheterization) or by chemi-
explanation of EMG, see O’Donnell 1998 in the cal or surgical ablation of the sphincter. The latter
Further Reading section.) will lead to incontinence.

Figure 9

Not abnormal for EMG activity to increase with involuntary contraction


as part of guarding reflex inhibiting IDC

ID C V o id

316 EDUCATIONAL REVIEW MANUAL IN UROLOGY


Figure 10

UDS tracing of a patient with myelodysplasia and neurogenic DO and DESD

DO DO V o id

Note the initial IDC associated with DESD and incontinence (measured on the flow channel). With refilling, the UDS is
again DO with DESD and then the patient is told to voluntarily void and there is persistent increased EMG activity. As a
result there is high pressure, low flow voiding (obstruction from the dyssynergic sphincter).

CHAPTER 10: URODYNAMICS 317


6. Videourodynamics

Videourodynamics (VUDS) consists of the simul-


taneous measurement of UDS parameters and imag-
Urodynamic Risk Factors

ing of the lower urinary tract. It provides the most


precise evaluation of voiding function and dysfunc-
It cannot be emphasized enough that certain

tion. VUDS is particularly useful when an anatomic


UDS findings are potentially dangerous and usu-

picture is desired, for example:


ally require intervention. These include:

• In cases of known or suspected anatomical


• Impaired compliance

abnormality • DESD

• Failure to demonstrate incontinence by


conventional methods
• DISD

• Evaluation of bladder neck (internal sphincter)


• High pressure DO present throughout filling

synergy • Elevated DLPP (>40 cm H2O)

• Neurological diseases (or other potentially danger-


ous causes of voiding dysfunction) where there is
• Poor emptying with high storage pressures

associated vesicoureteral reflux. In such cases,


reflux volumes and pressures can be measured

VUDS is also useful in the evaluation of suspected


female BOO.

VUDS Examples

Figure 11 is a VUDS study of a 45-year-old male


with LUTS including frequency, urgency and
decreased force of stream. The UDS shows high
pressure and low flow consistent with BOO, but in
this young man, BPO cannot be assumed. The
simultaneous fluoroscopic view of the bladder and
outlet taken during voiding shows an incomplete
opening of the bladder neck and the diagnosis of pri-
mary bladder neck obstruction can be made.

Figure 12 is a VUDS study of a male with a C1-2


spinal cord injury with suspected DESD and reten-
tion of urine being considered for external sphinc-
terotomy. The UDS shows 2 high pressure IDCs
with significant increase in EMG activity, consistent
with neurogenic DO with DESD. However, the flu-
oroscopic picture taken during the second IDC also
shows internal sphincter dyssynergia as the bladder
neck remains relatively closed. This phenomenon is
known to happen in high spinal cord lesions (above
the lower thoracic cord). An external sphinctero-
tomy would not be enough to relieve obstruction in
this case. An internal sphincterotomy (bladder neck
incision) is also needed.

318 EDUCATIONAL REVIEW MANUAL IN UROLOGY


Figure 11

VUDS study of a 45-year-old male with LUTS including frequency,


urgency and decreased force of stream

Figure 12

VUDS study of a male with a C1-2 spinal cord injury with suspected DESD and retention of
urine, being considered for external sphincterotomy

CHAPTER 10: URODYNAMICS 319


7. Further Reading

1. Abrams PH, Griffiths DJ. The assessment of 11. Chassagne S, Bernier PA, Haab F, Roehrborn
prostatic obstruction from urodynamic mea- CG, Reisch JS, Zimmern PE. Proposed cutoff
surements and from residual urine. Br J Urol. values to define bladder outlet obstruction in
1979;51:129-134. women. Urology. 1998;51:408-411.

2. Abrams PH, Farrar DJ, Turner-Warwick RT, 12. Defreitas GA, Zimmern PE, Lemack GE,
Whiteside CG, Feneley RC. The results of Shariat SF. Refining diagnosis of anatomic
prostatectomy: a symptomatic and urody- female bladder outlet obstruction: comparison
namic analysis of 152 patients. J Urol. of pressure-flow study parameters in clinically
1979;121:640-642. obstructed women with those of normal con-
trols. Urology. 2004;64:675-679.
3. Abrams P. Bladder outlet obstruction index,
bladder contractility index and bladder void- 13. Fleischmann N, Flisser AJ, Blaivas JG,
ing efficiency: three simple indices to define Panagopoulos G. Sphincteric urinary inconti-
bladder voiding function. BJU Int. 1999; nence: relationship of vesical leak point pres-
84:14-15. sure, urethral mobility and severity of inconti-
nence. J Urol. 2003;169:999-1002.
4. Abrams P, Cardozo L, Fall M, et al. The stan-
dardisation of terminology in lower urinary 14. Griffiths D, Höfner K, van Mastrigt R,
tract function: report from the standardisation Rollema HJ, Spångberg A, Gleason D.
sub-committee of the International Conti- Standardization of terminology of lower uri-
nence Society. Urology. 2003;61:37-49. nary tract function: pressure-flow studies of
voiding, urethral resistance, and urethral
5. Akikwala TV, Fleischman N, Nitti VW. Com- obstruction. International Continence Society
parison of diagnostic criteria for female blad- Subcommittee on standardization of Termi-
der outlet obstruction. J Urol. 2006; nology of Pressure-Flow Studies. Neurourol
176:2093-2097. Urodyn. 1997;16:1-18.

6. Bass JS, Leach GE. Bladder outlet obstruction 15. Griffiths DJ. The mechanics of the urethra and
in women. Prob Urol. 1991;5:141-154. of micturition. Br J Urol. 1973;45:497-507.

7. Blaivas JG, Groutz A. Bladder outlet obstruc- 16. Hosker G, Rosier P, Gajewski J, et al.
tion nomogram for women with lower urinary Dynamic testing. In: Abrams P, Cardozo L,
tract symptomatology. Neurourol Urodyn. Khoury S, Wein A, eds. Incontinence: 4th
2000;19:553-564. International Consultation on Incontinence.
London, United Kingdom: Health Publica-
8. Blaivas JG. Videourodynamic studies. In: tions; 2009: 413-552.
Nitti VW, ed. Practical Urodynamics.
Philadelphia, PA: WB Saunders; 1998:78-93. 17. Huckabay C, Twiss C, Berger A, Nitti VW. A
urodynamics protocol to optimally assess men
9. Boone TB, Kim YH. Uroflowmetry. In: Nitti with post-prostatectomy incontinence. Neu-
VW, ed. Practical Urodynamics. Philadelphia, rourol Urodyn. 2005;24:622-626.
PA: WB Saunders; 1998:28-37.
18. Lemack GE, Zimmern PE. Pressure flow anal-
10. Cespedes RD, McGuire EJ. Leak point pres- ysis may aid in identifying women with out-
sures. In: Nitti VW, ed. Practical Urodynam- flow obstruction. J Urol. 2000;163:1823-
ics. Philadelphia, PA: WB Saunders; 1998:94- 1828.
107.

320 EDUCATIONAL REVIEW MANUAL IN UROLOGY


19. Leng WW, McGuire EJ. Obstructive uropathy 29. Schäfer W. Principles and clinical application
induced bladder dysfunction can be of advanced urodynamic analysis of voiding
reversible: bladder compliance measures function. Urol Clin North Am. 1990;17:553-
before and after treatment. J Urol. 2003; 566.
169:563-566.
30. Schäfer W, Sterling AM, Liao L, et al. Good
20. Lose G, Griffiths D, Hosker G, et al. Standard- urodynamic practice: report from the Stan-
isation of urethral pressure measurement: dardisation Sub-Committee of the Interna-
report from the Standardisation Sub-Commit- tional Continence Society. Neurourol Urodyn.
tee of the International Continence Society. 2002;21:261-274.
Neurourol Urodyn. 2002;21:258-260.
31. Wein AJ, English WS, Whitmore KE. Office
21. Massey JA, Abrams PH. Obstructed voiding urodynamics. Urol Clin North Am. 1988;
in the female. Br J Urol. 1988;61:36-39. 15:609-623.

22. McGuire EJ. Urodynamic findings in patients 32. Wein AJ. Classification of neurogenic voiding
after failure of stress incontinence operations. dysfunction. J Urol. 1981;125:605-609.
Prog Clin Biol Res. 1981;78:351-360.

23. McGuire EJ, Woodside JR, Borden TA, Weiss


RM. Prognostic value of urodynamic testing
in myelodysplastic patients. J Urol. 1981;
126:205-209.

24. McGuire EJ, Fitzpatrick CC, Wan J, et al.


Clinical assessment of urethral sphincter func-
tion. J Urol. 1993;150:1452-1454.

25. Nitti VW, Combs AJ. Urodynamics: when,


why and how. In: Nitti VW, ed. Practical Uro-
dynamics. Philadelphia, PA: WB Saun-
ders;1998:15-26.

26. Nitti VW, Tu LM, Gitlin J. Diagnosing blad-


der outlet obstruction in women. J Urol.
1999;161:1535-1540.

27. Nitti VW. Cystometry and abdominal pressure


monitoring. In: Nitti VW, ed. Practical Uro-
dynamics. Philadelphia, PA: WB Saunders;
1998:38-51.

28. O’Donnell PD. Electromyography. In: Nitti


VW, ed. Practical Urodynamics. Philadelphia,
PA: WB Saunders; 1998:65-71.

CHAPTER 10: URODYNAMICS 321


8. Questions

1. The indications for performing UDS 4. Detrusor pressure_____________.

A. Are supported by high quality, level 1 evi- A. Can be measured directly via a
dence for most conditions transurethral catheter

B. Are better defined for men vs women B. Should remain low (near zero) during blad-
der filling
C. Are best defined by the clinician who has
clear-cut reasons for performing the study C. Rises abruptly and returns to baseline with
and will use the information obtained to impaired compliance
guide treatment
D. Rises before the external sphincter relaxes
D. Are clearly defined for women with stress in normal voluntary micturition
urinary incontinence
E. Both a and b
E. Both a and b
5. Which of the following measures the ability of
2. Before performing a UDS study, the clinician the urethral sphincter complex to resist
should: changes in abdominal pressure?

A. Decide on questions to be answered for a A. Abdominal leak point pressure


particular patient
B. Detrusor leak point pressure
B. For consistency, be prepared to perform the
study the same way, no matter what the cir- C. Maximum urethral closure pressure
cumstances
D. All of the above
C. Customize the study depending on a patien-
t’s symptoms and condition E. Both a and c

D. Both a and b 6. Which of the following is not a UDS risk fac-


tor for upper tract damage?
E. Both a and c
A. Impaired compliance
3. Which of the following is not true regarding
detrusor overactivity? B. Detrusor-external sphincter dyssynergia

A. It can only be diagnosed by UDS C. Poor emptying with high storage pressures

B. It is often associated with urinary urgency D. A high detrusor leak point pressure (>40
cm H2O)
C. It is synonymous with the term “overactive
ladder” E. A high abdominal leak point pressure
(>100 cm H2O)
D. It is classified by whether or not the patient
has a known neurological disease

E. It can be provoked by a cough or Valsalva


maneuver

322 EDUCATIONAL REVIEW MANUAL IN UROLOGY


7. Videourodynamics_________________. 2. E.
All patients are not alike and therefore each urody-
A. Is the most precise measure of lower uri- namic evaluation may be different depending upon
nary tract function and ideally should be the information needed to answer the questions rele-
used in all cases where UDS is to be per- vant to a particular patient. Therefore, in many
formed cases, the study must be customized to answer spe-
cific questions for a given patient.
B. Is the only way to assess obstruction in a
man 3. C.
Detrusor overactivity is an involuntary bladder con-
C. Is the procedure of choice for documenting traction seen on UDS testing which can be either
bladder neck dysfunction in men and neurogenic or idiopathic. It is commonly associated
women with the symptom of urgency or even urgency
incontinence. It can be provoked by a cough or Val-
D. Is of limited value in patients with neuro- salva maneuver (stress-induced detrusor overactiv-
logical disease, such as spinal cord injury, ity). It is not the same as overactive bladder (OAB),
because of difficulties with patient posi- which is a term that describes the syndrome of uri-
tioning nary urgency usually accompanied by frequency
and nocturia, with or without urgency urinary
E. Both c and d incontinence in the absence of UTI or other obvious
pathology. OAB is a symptom complex that does
Answers: not require UDS to make its diagnosis.

1. C. 4. B.
UDS has been used for decades, yet clear-cut, level Detrusor pressure normally remains low during fill-
1, evidence-based indications for its use are surpris- ing as the bladder is highly compliant. It cannot be
ing lacking. There are a number of reasons for this. measured directly with a transurethral catheter, but
It is difficult to conduct proper randomized con- must be obtained via subtraction of abdominal pres-
trolled trials on UDS for conditions where lesser sure from vesical pressure. With impaired compli-
levels of evidence and expert opinion strongly sug- ance, pressure increases with increasing bladder
gest clinical utility and where empiric treatment is volume, but does not return to baseline (compliance
potentially harmful or even life-threatening (e.g., = change in pressure/change in volume).
neurogenic voiding dysfunction). Additionally,
symptoms can be caused by a number of different 5. E.
conditions and it is difficult to study pure or homo- ALPP and MUCP are measures of urethral function
geneous patient populations. Given the current state against stress. The DLPP is a measure of bladder
of evidence for UDS studies, what is most important function against increased sphincteric resistance.
is that the clinician has clear-cut reasons for per-
forming the study and that the information obtained 6. E.
will be used to guide treatment of the patient. Upper tract damage occurs as a result of high intrav-
Despite having established nomograms for BOO in esical pressures during storage. Abdominal leak
men, the indications for UDS in men are no more point pressure measures outlet resistance and cannot
clear-cut than they are in women. UDS probably has be demonstrated in continent patients (i.e., it is well
its most important role in the diagnosis and manage- over 100 cm H2O).
ment of patients with neuropathic voiding dysfunc-
tion.

CHAPTER 10: URODYNAMICS 323


7. C.
Although VUDS provides the most precise evalua-
tion of voiding function and dysfunction and is par-
ticularly useful when anatomic structure and func-
tion are important, it is not practical or necessary for
all centers to have VUDS capabilities. VUDS is
useful for a number of conditions when an accurate
diagnosis cannot otherwise be obtained (e.g., by
conventional UDS), including complicated voiding
dysfunction or known or suspected neuropathic
voiding dysfunction (adults and children), unex-
plained urinary retention in women, prior radical
pelvic surgery, urinary diversion, pre- or postrenal
transplant, or prior pelvic radiation. VUDS is the
procedure of choice for documenting bladder neck
dysfunction in men and women.

324 EDUCATIONAL REVIEW MANUAL IN UROLOGY


Chapter 11:
Neuropathic Bladder:
Voiding Dysfunctions
Associated With
Neurological Disease
Raymond R. Rackley, MD

Contents

1. Lexicon of Terms and Key Concepts

2. Overview of Normal Voiding Function

3. General Patterns and Classifications of


Neurogenic Voiding Dysfunction

4. Evaluation and Consequences of


Neurogenic Voiding Dysfunction Progression

5. Clinical Presentations of Neurogenic


Voiding Dysfunction

6. Overview of Treatments for Neurogenic


Voiding Dysfunction

7. Further Reading

8. Questions

CHAPTER 11: NEUROPATHIC BLADDER: VOIDING DYSFUNCTIONS ASSOCIATED WITH NEUROLOGICAL DISEASE 325
1. Lexicon of Terms and
Key Concepts

Bladder Compliance Leak Point Pressures

Bladder compliance (C) is defined as the change in Two pressures obtained during urodynamics mea-
Volume (V) relative to the corresponding change in sure different aspects of lower urinary tract function:
intravesical or detrusor pressure (Pdet):
• Detrusor leak point pressure (DLPP):
C = change V/change Pdet
(expressed as mL/cm H2O) DLPP is defined by the ICS as the lowest detrusor
pressure at which urine leakage occurs in the
• Compliance is generally calculated between absence of either a detrusor contraction or
two points: the Pdet with the bladder empty at the increased abdominal pressure
start of urodynamic filling and the Pdet at either
the maximal cystometric capacity or the start of a • Abdominal leak point pressure (ALPP):
detrusor contraction

• Normal bladder compliance should be less than


ALPP is the intravesical pressure at which urine

12.5 mL/cm H2O


leakage occurs because of increased abdominal
pressure in the absence of a detrusor contraction

• Compliance arises from the neuromuscular and


biomechanical (collagenous and elastic) compo-
Smooth vs. Striated Sphincter

nents of the bladder wall The smooth sphincter refers to the smooth muscula-
ture of the bladder neck and proximal urethra. This is
• Three variables (all of which may be affected by a physiologic but not an anatomic sphincter and one
multiple factors) as defined by Laplace’s equation that is not under voluntary control. The striated
come into play: sphincter refers to the striated musculature that is a
part of the outer wall of the proximal urethra in both
• Laplace’s equation: the male and the female (this portion is often referred
T ~ Pdet x R(radius of the bladder) to as the intrinsic or intramural striated sphincter)
and the bulky skeletal muscle group that closely sur-
1. Wall tension (T) rounds the urethra at the level of the membranous
portion in the male and primarily the middle seg-
2. Detrusor pressure ment in the female (often referred to as the extrinsic
or extramural striated sphincter). The extramural
3. Bladder volume portion is the classically described external urethral
sphincter and is under voluntary control.
• A decrease in bladder compliance (neuromuscular
and/or biomechanical properties) can alter bladder
wall tension, cause afferent nerve activation, and
Guarding Reflex

thereby change bladder sensations and the volume The guarding reflex constitutes the efferent limb of
threshold for micturition a spinal somatic reflex, which results in a gradual
increase in striated sphincter activity during normal
bladder filling and storage. The gradual increase in
urethral pressure during bladder filling is due in part
to the striated sphincteric element and perhaps in
part to the smooth sphincteric element as well. The
rise in urethral pressure seen during the filling/
storage phase of micturition can be correlated with
an increase in efferent pudendal nerve impulse fre-
quency and in EMG activity of the periurethral stri-
ated musculature.

326 EDUCATIONAL REVIEW MANUAL IN UROLOGY


• In the absence of neurologic disease, one cannot
use the term detrusor sphincter dyssynergia.
Autonomic Hyperreflexia (Dysreflexia)

Autonomic hyperreflexia represents an acute mas- Instead, the term pelvic floor hyperactivity or
sive disordered autonomic (primarily sympathetic) dysfunctional voiding is used
response to specific stimuli in patients with SCI
above the cord level of T6–T8 (the sympathetic out- • Patients with Parkinson’s disease may demon-
flow). strate a similar EMG picture that has been termed
sphincter bradykinesia. There is a delay in the
• It is more common in cervical (60%) relaxation of the sphincter at the onset of voiding
than thoracic (20%) injuries owing to skeletal muscle hypertonicity

• Onset after injury is variable—usually soon after


spinal shock but may be up to years after injury
Pseudodyssynergia

Pseudodyssynergia refers to an EMG sphincter


• Distal cord viability is a prerequisite “flare” during filling cystometry that is secondary to
attempted inhibition of an involuntary bladder con-
• Symptomatically, autonomic hyperreflexia is a traction by voluntary contraction of the striated
syndrome of exaggerated sympathetic activity in sphincter. If a patient has an unstable bladder con-
response to stimuli below the level of the lesion. traction during filling cystometry, the normal
The symptoms are pounding headache, hyperten- response of the external sphincter is to contract in an
sion, and flushing of the face and body above the attempt to prevent incontinence. This has the same
level of the lesion with sweating. Bradycardia is a appearance as detrusor sphincter dyssynergia on an
usual accompaniment, although tachycardia or EMG study but is of different significance because
arrhythmia may be present. Hypertension may be it is a voluntary guarding event.
of varying severity
Spinal Cord vs. Spinal Column
Detrusor Sphincter Dyssynergia in Spinal Cord Injury

Sphincter EMG is used to evaluate the striated Spinal column (bone) segments are numbered by
sphincter complex and the activity of the pelvic the vertebral level, and these have a different rela-
floor during bladder filling, storage and voiding. tionship to the spinal cord segmental level at dif-
Clinically, the most important information obtained ferent locations. One must be careful to specify
from sphincter EMG is whether there is coordina- cord vs column level when discussing spinal cord
tion or discoordination between the external sphinc- injury. The sacral spinal cord begins at about spinal
ter and the bladder. column level T12 to L1. The spinal cord terminates
in the cauda equina at approximately the spinal col-
• Failure of the sphincter to relax or stay completely umn level of L2.
relaxed during micturition is abnormal. When it
occurs in patients with neurologic disease, it is
termed detrusor sphincter dyssynergia; this typi-
Spinal Cord Shock

cally occurs in patients with suprasacral spinal Following significant spinal cord injury, a period of
cord injury in which there is an interruption of the decreased excitability of spinal cord segments at
spinobulbar-spinal pathways that normally coordi- and below where the level of injury occurs is known
nate the detrusor and the sphincter as spinal cord shock. It includes a suppression of
autonomic activity, as well as somatic activity, and
the bladder is acontractile and areflexic.

CHAPTER 11: NEUROPATHIC BLADDER: VOIDING DYSFUNCTIONS ASSOCIATED WITH NEUROLOGICAL DISEASE 327
2. Overview of Normal
Voiding Function

Cauda Equina Syndrome Innervation of the Lower Urinary Tract

Cauda equina syndrome is a term applied to the Parasympathetic Nervous System


clinical picture of perineal sensory loss with loss of
voluntary control of both anal and urethral sphincter • Neurotransmitter:
and of sexual responsiveness. This can occur not
only secondary to disc disease (severe central poste- *Acetylcholine (ganglia and effector sites)
rior disc protrusion) but to other spinal canal patho-
logic processes as well. • Receptors:
*Nicotinic (ganglia and motor endplates)

*Muscarinic (autonomic effector sites,


i.e., bladder contractions)

Sympathetic Nervous System

• Neurotransmitter:

*Acetylcholine (ganglia)

*Norepinephrine (effector sites)

• Receptors:

*Alpha (bladder base and neck)—


closure of the outlet

*Betas (bladder body)—


detrusor muscle relaxation

Function of the Lower Urinary Tract:


2 Basic Functions

• Bladder filling/urine storage: involves the bladder


muscle and bladder outlet

• Bladder/detrusor muscle: there are 3 processes


active during bladder filling:

1. Accommodation of increasing volumes of


urine at low pressures with appropriate sen-
sations through 3 activities:
a. The afferent activity sending sensations
to the CNS

b. The tonic inhibition from the brain


inhibits parasympathetic activity

328 EDUCATIONAL REVIEW MANUAL IN UROLOGY


3. General Patterns and
Classifications of Neurogenic
Voiding Dysfunction

c. The inhibition of the parasympathetic


system also provided by sympathetic and
General Comments

somatic reflex activity • Neurologic disease generally affects lower


urinary tract function in a consistent fashion
2. The prevention of involuntary detrusor
contractions • Variability of clinical presentation occurs when the
following is present:
3. The prevention of parasympathetic activity
*The injury is irritative vs. destructive
• Bladder outlet: must remain closed by spinal reflex
activity that activates sympathetic and somatic *The injury is partial vs. complete
nerve pathways (the guarding reflex)
*The injury is muticentric vs. focal
• Bladder emptying/urine voiding
*The injury is specific vs. nonspecific
• Bladder/detrusor muscle:
Contraction provided via the process of: General Prevalences of Neurogenic

*Afferent activity: the primary stimulus of mic-


Voiding Dysfunction

turition reflex is bladder distention Central Nervous System Conditions


that provides afferent activity from stretch
receptors • Spinal cord injury: vast majority

*Parasympathetic activity: gives rise to the • Cerebrovascular accidents: 32%–79%


smooth muscle contraction and inhibits sym-
pathetic reflex • Dementia: 30%–100%

*Sympathetic and somatic nerve activity: • Parkinson’s disease: 40%–70%


becomes inactive as voluntary relaxation
of the external sphincter initiates the micturi- • Cerebral palsy: 36%
tion reflex
• Severe learning disabilities: 12%–65%
*The Pons/CNS coordinates the bladder and
outlet activity • Normal pressure hydrocephalus: case reports

• Bladder outlet: opens via: • Cerebral tumors: case reports

*Lowering of resistance at the outlet

*Absence of anatomic obstruction

CHAPTER 11: NEUROPATHIC BLADDER: VOIDING DYSFUNCTIONS ASSOCIATED WITH NEUROLOGICAL DISEASE 329
Central and Peripheral Nervous * It should attempt to include conclusions
System Conditions reached from urodynamic testing

• Multiple system atrophy The Functional Classification (Table 1)


(MSA or Shy-Drager syndrome): 73% Classification of voiding dysfunction can be formu-
lated on a simple functional basis, describing the
• Multiple sclerosis: 52%–97% dysfunction in terms of whether the deficit produced
is primarily one of the filling/storage or the empty-
• Spinal stenosis: 62% ing/voiding phase of micturition. There are indeed
some types of voiding dysfunction that represent
• Spine surgery: 38%–60% combinations of filling/storage and empty-
ing/voiding abnormalities.
• Disc disease: 28%–87%
• Storage failure results because of either bladder
• Diabetes mellitus: 25%–87% or outlet abnormalities or a combination

Peripheral Nervous System Conditions *Bladder abnormalities include involuntary


bladder contractions, low compliance, and
• Complex pelvic surgery: 20%–79% heightened or altered sensation

• Radical hysterectomy: 8%–57% *Outlet abnormalities include intermittent or


continuous decrease in outlet resistance
• Guillain-Barré: 25% in case series
• Emptying failure results because of bladder or
• HIV: 12% in advanced cases outlet abnormalities or a combination

• Herpes: 0.4% *Bladder abnormality includes an inadequate


or unsustained bladder contractility
• SLE: 0.01%
* Outlet abnormalities include anatomic
Classification Systems of obstruction and sphincter dyssynergia

The ICS Classification System (Table 2)


Neurogenic Voiding Dysfunction

General Comments on Classifications Systems The storage and voiding phases of micturition are
• Due to the diversity of neurological injuries, it has described separately, and within each, various des-
been difficult to have one general system ignations are applied to describe bladder and ure-
for the entire population of patients with neuro- thral function, as well as bladder sensation.
genic voiding dysfunction; i.e., no one system is
perfect In general:

• In general, the classification system should do the • Detrusor activity: Normal (N), hyperactive (+),
following: or hypoactive/areflexic (-)

* It should localize the site and type of • Bladder capacity or compliance: Normal (N),
neurological lesions high (+), or low (-)

*It should attempt to outline expected clinical • Urethral function: Normal (N), overactivity (+),
symptoms and presentations or incompetent (-)

• Bladder sensation: Normal (N),


hypersensitive (+), or hyposensitive/absent (-)
330 EDUCATIONAL REVIEW MANUAL IN UROLOGY
Table 1

The Functional Classification

I. Failure to Fill/Store B. Due to Outlet Dysfunction


1. Combination (GSI and ISD)
A. Due to Bladder Dysfunction 2. Genuine stress urinary incontinence (GSI)
1. Overactivity a. Lack of suburethral support
a. Involuntary contractions b. Pelvic floor laxity, hypermobility
Neurologic disease or injury 3. Intrinsic sphincter deficiency (ISD)
Bladder outlet obstruction (myogenic) a. Neurologic disease or injury
Inflammation b. Fibrosis
Idiopathic
b. Decreased compliance C. Due to a Combination of Bladder and Outlet Dysfunctions
Neurologic disease or injury
Fibrosis II. Failure to Void/Empty
Idiopathic
c. Combination A. Due to Bladder Dysfunction
2. Hypersensitivity 1. Neurogenic
a. Inflammatory/infectious 2. Myogenic
b. Neurologic 3. Psychogenic
c. Psychological 4. Idiopathic
d. Idiopathic
3. Decreased pelvic floor activity (?) B. Due to Outlet Dysfunction
4. Combination 1. Anatomic
a. Prostatic obstruction
b. Bladder neck contracture
c. Urethral stricture in the male
d. Urethral compression, fibrosis in the female
2. Functional
a. Smooth sphincter dyssynergia
b. Striated sphincter dyssynergia
c. Dysfunctional voiding

C. Due to a Combination of Bladder and Outlet Dysfunctions

Adapted from: Wein AJ. Pathophysiology and classification of voiding dysfunction.


In: Wein, Kavoussi, Novick, Partin, Peters, eds. Campbell-Walsh Urology. Philadelphia, PA: Saunders/Elsevier; 2007:
1973-1985.

CHAPTER 11: NEUROPATHIC BLADDER: VOIDING DYSFUNCTIONS ASSOCIATED WITH NEUROLOGICAL DISEASE 331
4. Evaluation and Consequences
of Neurogenic Voiding
Dysfunction Progression

Table 2 Overview

Evaluation of voiding dysfunction in neurogenic


causes should be undertaken to prevent conse-
The International Continence Society

quences and to determine the goals of treatment:


Classification

Storage Phase Voiding Phase • Prevent upper urinary tract deterioration

• Restore hygienic and socially acceptable


filling and emptying functions of the neurogenic
Bladder Function: Bladder Function:

Detrusor activity: Detrusor activity: bladder

• Avoidance of infections
Normal or stable Normal
Overactive: Underactive:

• Considerations of bowel and other pelvic organ


Neurogenic Acontractile

functions
Idiopathic Areflexic

• Clinical evaluations for neurogenic conditions


Bladder Sensation: Urethral

(other than spinal cord injury) in general:


Function:
Normal

1. Tend to correlate poorly with dysfunctions


Increased or hypersensitive Normal

noted on urodynamics
Reduced or hyposensitive Abnormal:
Absent Mechanical

2. Tend to correlate poorly with clinical


obstruction

symptoms
Bladder Capacity/
Compliance: Overactivity

• Localization of neurogenic injury as a means of


Dysfunctional

predicting voiding dysfunction and planning for


Normal voiding,

evaluation:
High Detrusor sphincter
Low dyssynergia,

• Lesions above the brainstem (CVA, Parkinson’s


Nonrelaxing

disease, MS, CVA, etc.) between the


Urethral Function urethral sphincter

cortex and the pontine micturition center


dysfunction

(PMC) typically result in the following ICS


Normal closure mechanism
Incompetent closure mechanism

Adapted from Abrams P, Cardozo L, Fall M, et al.The stan- Classification:

*Detrusor = (+); involuntary bladder contrac-


dardisation of terminology in lower urinary tract function:

tions (CNS inhibition is lost); rarely is the


report from the standardisation sub-committee of the

detrusor acontractile (-)


International Continence Society. Urology. 2003;61:37-49.

*Sphincter = N; Detrusor – sphincter synergy


(smooth and striated sphincter synergy)

*Sensation = N; usually normal

332 EDUCATIONAL REVIEW MANUAL IN UROLOGY


*Special notes: • UTI management: In a consensus reached
by the National Institute on Disability and
• Detrusor areflexia may occur either ini- Rehabilitation Research Group (1992),
tially or as a permanent dysfunction bacteriuria should be treated only when
signs or symptoms present
• Urinary incontinence may occur due to
the detrusor overactivity Follow-up: In the American Paraplegic Society
(APS) Guidelines (1999) for urologic care of SCI,
• Spinal cord lesions (myelodysplasia, MS, spinal annual follow-up is recommended for the first
cord injury, stenosis, or disc disease, etc.) with 5–10 years after injury. If the patient is doing
injuries between the pontine micturition center and well, then follow-up every other year is advised.
S2; i.e., above S2 typically result in the following Upper and lower tract evaluation should be done
ICS Classification: initially and yearly for 5–10 years, then every
other year. Urodynamic evaluation was recom-
*Detrusor = (+); involuntary contractions mended by the APS at the same intervals as
upper and lower tract screening. Cystoscopy was
*Sphincter = (+); dyssynergia recommended annually in those with an
indwelling catheter.
*Sensation = (-); absent
• Disease at S2 and below; and peripheral lesions
*Special notes: (Myelodysplasia, MS, tethered cord, spinal steno-
sis, polio, etc.) typically result in the following ICS
• Those with lesions above T6-8 may Classification:
develop autonomic dysreflexia
* Detrusor = (-); acontractile and poor detrusor
• Incontinence may occur due to detrusor compliance typically develops
overactivity, but the outlet obstruction
can also cause urinary retention * Sphincter = (+); open smooth muscle sphinc-
ter, but fixed striated sphincter with loss of
• A careful initial evaluation and periodic voluntary control is the usual finding
follow-up evaluation must be performed
to identify and correct the following risk * Sensation = (-)
factors and potential complications:
bladder overdistention, high-pressure * Special note:
storage, high detrusor leak point pres- True peripheral neuropathy can be
sure, vesicoureteral reflux (VUR), stone motor or sensory
formation (lower and upper tracts), and
complicating infection, especially in asso- • Evaluation of upper and lower urinary tracts
ciation with reflux
• BMP, serum creatinine or creatinine clearance in
• Reflux management: The best initial treat- select cases
ment for reflux in a patient with voiding
dysfunction secondary to neurologic dis- • Renal ultrasound
ease or injury is to normalize lower uri-
nary tract urodynamics as much as possi- • Selective use of renal scans for evaluation of renal
ble scarring, function and obstruction

CHAPTER 11: NEUROPATHIC BLADDER: VOIDING DYSFUNCTIONS ASSOCIATED WITH NEUROLOGICAL DISEASE 333
• Urodynamics (UDS): Determining the predomi- *Bethanechol supersensitivity testing with
nate bladder pressure against which the kidneys UDS: based on the assumption that dener-
drain; while no RCTs had documented beneficial vated/neurogenic bladder will be more sensi-
outcomes achieved with UDS information, the tive to acetylcholine stimulation than a non-
overwhelming bulk of evidence supports the use of neurogenic acontractile bladder; a positive
this testing in neurogenic patients with voiding test reveals a rise in detrusor pressure >15 cm
dysfunction H2O during the filling after giving sq bethane-
chol chloride
*Determine bladder capacity
* Electrosensitivity testing involves placement
*Determine bladder/detrusor compliance of bladder catheter electrode to test for abnor-
mal sensitivities as found in DM, MS, etc.
*Determine detrusor over/under/normal
activity

*Determine the detrusor leak point pressure


(DLPP):

• The DLPP was first introduced by


McGuire and associates for the evaluation
of patients with low bladder compliance
secondary to myelodysplasia. McGuire
found that in myelodysplastic patients
with an elevated outlet resistance from a
fixed external sphincter, those with a
DLPP >40 cm H2O were at significantly
higher risk for upper tract deteriora-
tion (hydronephrosis, reflux) than those
with DLPPs <40 cm H2O. However,
while pressures >40 cm H2O are impor-
tant, the time over which an elevated pres-
sure is exerted onto the system is also sig-
nificant

• Ancillary neurogenic bladder testing: impractical


(ice water and bethanechol) or evolving (electro-
sensitivity):

*Ice water UDS testing: reveals detrusor over-


activity in pathologic conditions; normal blad-
ders or sacral neurogenic lesions have no
inducible detrusor contractions, but
suprasacral spinal cord lesions will reveal a
positive test with inducible involuntary detru-
sor contractions when using ice cold water
during the filling phase of UDS testing

334 EDUCATIONAL REVIEW MANUAL IN UROLOGY


5. Clinical Presentations of
Neurogenic Voiding Dysfunction

*Benign prostatic obstruction with Parkinson’s


disease is often noted:
Disease at or Above the Brain Stem

• Neurogenic voiding dysfunction secondary to


cerebrovascular injury (stroke): Note: It is now thought that in older series of PD,
many cases may have actually been multisystem
*Following the acute CVA event or cerebral atrophy (MSA). Previous reports put postprostate-
shock, acute detrusor areflexia or retention ctomy incontinence (PPI) from TURP – 20% vs.
may be present. Once this resolves, the most 1% in non-neurogenic patients. Current literature
common finding is detrusor overactivity, but support that TURP should not be contraindicated
normal detrusor compliance in patients with PD as external sphincter acontrac-
tility is rare.
*Common features of chronic impairment
include: *Sensation is usually intact
Detrusor = (+), Sphincter = N,
Bladder sensation = N • Neurogenic voiding dysfunction secondary to
multiple system atrophy (MSA); See neurologic
*True detrusor dyssynergia does not occur as lesions consisting of cell loss and gliosis in
the lesion is above the pons; pseudodssyner- widespread areas
gia may occur
*Voiding dysfunction symptoms precede the
*Variable sensation; usually intact and often diagnosis of MSA (4 years on average) or they
with OAB symptoms present with other symptoms of MSA

*Always consider preexisting pathology, *73% of patients have detrusor overactivity;


such as BPO and a hypocontractile function Detrusor = (+)

*Always consider using urodynamics to docu- *19% have urinary frequency and urgency
ment and reveal voiding dysfunction before without urinary incontinence
invasive procedures
*66% have significant emptying dysfunction
• Neurogenic voiding dysfunction secondary to with elevated PVRs
Parkinson’s disease:
Sphincter: may see open bladder neck from intrinsic
*Duration and severity of Parkinson's disease sphincter deficiency. Also see striated sphincter
does not correlate with urodynamic findings denervation.

*Voiding dysfunction develops in 35%–70% of *93% of men have erectile dysfunction


cases; detrusor overactivity is the most com-
mon finding; Detrusor = (+). In some cases, Late-stage MSA: also known as Shy-Drager syn-
impaired contractility may cause hesitancy; drome. Clinically see orthostatic hypotension,
Detrusor = (-) anhidrosis, and varying degrees of cerebellar and
Parkinson-like dysfunction. See both voiding and
*Sphincter bradykinesia and pseudodyssyner- erectile dysfunction.
gia complicate urodynamics
• Cerebral Palsy: most commonly see urinary con-
*Poorly sustained contractions with obstruc- trol with normal filling storage
tion is problematic (DHIC – detrusor hyper-
reflexia and impaired contractility) In patients with dysfunction, see detrusor overactiv-
ity with coordinated sphincters.

CHAPTER 11: NEUROPATHIC BLADDER: VOIDING DYSFUNCTIONS ASSOCIATED WITH NEUROLOGICAL DISEASE 335
• Cerebellar ataxia: voiding dysfunction is charac- * Most important parameters predisposing
terized by detrusor overactivity and sphincter syn- patients with MS to significant urologic com-
nergia plications:

o Can see poor emptying from detrusor are- 1. Striated sphincter dyssynergia in men
flexia (less worrisome in females)

o May also see striated sphincter dyssynergia, 2. Detrusor filling pressure >40 cm H2O
as the pathologic degeneration can extend to
the spinal cord 3. An indwelling catheter

Disease Primarily Involving the Overall, MS rarely causes upper tract changes.

• Spinal cord injury


Spinal Cord Above S2

• Neurogenic voiding dysfunction secondary to


multiple sclerosis * Voiding dysfunction is typically: Detrusor =
(+), Sphincter = (+), and Sensation = (-):
• 2%–10% of patients with MS present with voiding
dysfunction as part of their initial * Involuntary detrusor contractions
presentation; 80% of MS patients develop voiding
dysfunction as part of the condition. Voiding pat- * Detrusor dyssynergia
tern is dependent on location of plaques
* Smooth sphincter: dyssynergic above T7–8,
*Urodynamic patterns can change in synergic below
15%–35% of cases when symptoms change
* Absent sensation
*3 basic patterns of voiding dysfunction:
* Major concepts in treating SCI:
1. Detrusor = (+), overactive; Sphincter =
N, normal synergy; Sensation = N, nor- 1. Spinal shock: see transient absent reflex
mal; this pattern represents 26%–50% activity and autonomic activity, i.e.,
of cases bladder acontractility. Sphincter retains
some pressure. This is why there may be
2. Detrusor = (+), overactive; Sphincter = initial detrusor acontractility with low
(+), dyssynergy; Sensation = N, normal; bladder pressures and then one may see
this pattern represents 24%–46% signs of neurogenic detrusor activity sev-
of cases eral months later

3. Detrusor = (-), overactive; Sphincter =


(?), dyssynergy; Sensation = ?;
2. Autonomic hyperreflexia

this pattern represents 19%–40% with lesions. Treat with terazosin (a


(autonomic dysreflexia):

of cases selective 1 adrenergic blocker) for long-


term management (3-month study) and
* Pelvic floor and sphincter flaccidity can also prophylaxis. A nightly dose varying
occur between 1–10 mg will reduce severity,
whereas erectile function and blood pres-
* In most all cases, bladder sensation remains sure will remain unchanged. Acutely use
intact beta and/or alpha blockers

* Smooth sphincter is synergic

336 EDUCATIONAL REVIEW MANUAL IN UROLOGY


Note: any endoscopic procedure in a patient with *Usually presents with low back pain with
SCI at risk for dysreflexia should be done under radiation to the legs, relieved with rest
spinal anesthesia or carefully monitored general
anesthesia. *Variety of bladder and sphincter dysfunctions

3. Detrusor leak point pressure: *Urodynamics and comorbid conditions dic-


An important concept in urodynamics is tate diagnosis and treatment
the fact that bladder outlet resistance is
the main determinant of detrusor pres- • Tethered cord
sure and urethral outlet resistance at a
moment of flow determines Pdet. If the *Primary or secondary (3%–5% of surgical
outlet resistance is high, a higher bladder closures for myelodysplasia)
pressure is needed to overcome this
resistance and cause leakage. This high *May present in early childhood and up to 80
pressure can be transmitted to the upper years of age
tracts, causing reflux and hydronephro-
sis. However, while pressures >40 cm *Symptoms: urinary incontinence, leg weak-
H2O are important, the time over which ness, and backache
an elevated pressure is exerted onto the
system is also significant *Progressive decline without treatment

• Myelodysplasia Disease Distal to the Spinal Cord:

*Voiding dysfunction is more variable than SCI


Injury Below S2; i.e., Sacral Injury

above S2: Detrusor (+,-), Sphincter Sacral spinal cord injury:


(+, N, -), Sensation (+, N, -)
See depression of deep tendon reflexes below the
*At risk for: level of a complete lesion along with paralysis.

• Upper tract deterioration, reflux See detrusor areflexia with high or normal compli-
ance.
• Infection
Outlet: competent but nonrelaxing smooth and stri-
• Incontinence ated sphincters. Some loss of closure pressure.

• Carcinoma • Radical pelvic surgery: prevalence of voiding


dysfunction
• Sexual dysfunction
*Post APR: 20%–60%
• Bowel dysfunction
*Post-radical hysterectomy:16%–80%
• Spinal stenosis
*15%–20% of cases are permanent
*Spinal canal narrowing (congenital or dysfunctions
acquired)
*Allow 6 months for apraxia to resolve
*Nerve root or cord damage and/or reinnervation to occur

*Ischemia, swelling *Voiding dysfunction: Detrusor = (-),


Sphincter = (+), Sensation = N, (-)

CHAPTER 11: NEUROPATHIC BLADDER: VOIDING DYSFUNCTIONS ASSOCIATED WITH NEUROLOGICAL DISEASE 337
• Usually areflexic or hypocontractile • Disk disease
detrusor
o Most common site of disk compression is
• Increased risk of detrusor compliance loss L4-L5 and L5-S1 interspaces. See low back
pain radiating along the involved spinal roots
• Bladder neck often open due to sympathetic dener-
vation of compliance loss o Nerve root compression will cause reflex loss
and sensory loss. Can see difficulty voiding
• Fixed external sphincter resistance and straining to void

*Mixed storage and emptying symptoms o Voiding dysfunction occurs in 27%–92%


of cases
*Avoid TURP or outlet surgeries in males after
APR unless carefully studied with UDS o Can see cauda equina syndrome

Miscellaneous Conditions Herpesvirus infections

• Diabetes mellitus Herpes zoster infection: causes detrusor areflexia


and urinary retention. Occurs days to weeks after
*5%–59% of patients with diabetes report the other primary viral manifestations.
symptoms of voiding dysfunction
Generally see resolution in 1–2 months.
*Diabetic cystopathy is a term used to describe
the involvement of the lower urinary tract by Herpes simplex infection: can see urinary retention
this disease with anogenital herpes simplex infection.

* The classic description of voiding dysfunc- Guillain-Barré syndrome: inflammatory demyeli-


tion secondary to diabetes is that of a periph- nating disorder of peripheral somatic and autonomic
eral and autonomic neuropathy that first nervous system.
affects sensory afferent pathways, causing the
insidious onset of impaired bladder sensation. Autonomic neuropathy is common complication.
As the classic description continues, a gradual
increase in the time interval between voiding Prevalence of lower urinary tract dysfunction is
results, which may progress to the point at reported at 25%–80%. Varies from urinary retention
which the patient voids only once or twice a to detrusor overactivity.
day without ever sensing any real urgency. If
this continues, detrusor distention, overdis- Since symptoms may resolve, voiding dysfunction
tention and decompensation ultimately occur. should be managed by reversible therapy.
Detrusor contractility, therefore, is classically
described as being decreased in the end-stage
diabetic bladder

It has recently been reported that the classic


diabetic cystopathy may be less common than
detrusor overactivity.

338 EDUCATIONAL REVIEW MANUAL IN UROLOGY


6. Overview of Treatments for
Neurogenic Voiding Dysfunction

Voiding Dysfunction: Prioritizing the Patient Factors to Consider When Choosing


Medical Goals of Management Therapies Outlined in Tables 3 and 4

• Upper urinary tract preservation or • Prognosis of underlying disease, especially if


improvement progressive or malignant

• Absence or control of infection • General health

• Adequate storage at low intravesical pressure Limiting factors: inability to perform certain tasks
(e.g., hand dexterity, ability to transfer, body habi-
• Adequate emptying at low intravesical pres- tus)
sure
• Mental status
• Adequate control
• Motivation
• No catheter or stoma
• Desire to remain catheter or appliance free
• Social acceptability and adaptability
• Desire to avoid surgery
• Vocational acceptability and adaptability
• Sexual activity status

• Reliability
Reasons to Change or Augment a
Given Management Plan

• Upper urinary tract deterioration • Educability

• Recurrent sepsis or fever of urinary • Psychosocial environment, interest, reliability


tract origin and cooperation of family

• Lower urinary tract deterioration • Economic resources

• Inadequate storage

• Inadequate emptying

• Inadequate control

• Unacceptable side effects

• Skin changes secondary to incontinence or


collecting device

CHAPTER 11: NEUROPATHIC BLADDER: VOIDING DYSFUNCTIONS ASSOCIATED WITH NEUROLOGICAL DISEASE 339
Table 3

Therapy to Facilitate Urine Storage/Bladder Filling

Bladder Related (Inhibiting Bladder Outlet Related (Increasing Outlet Resistance)


Contractility, Decreasing Sensory Input
and/or Increasing Bladder Capacity) Behavioral Therapy
Education
Behavioral Therapy, Including Bladder training
Any or All of the Following: Timed bladder emptying or prompted voiding
Education Fluid restriction
Bladder training Pelvic floor physiotherapy ± biofeedback
Timed bladder emptying or prompted voiding Electrical stimulation
Fluid restriction Pharmacologic Therapy
Pelvic floor physiotherapy ± biofeedback ␣-Adrenergic agonists
Tricyclic antidepressants; serotonin and
Pharmacologic Therapy norepinephrine reuptake inhibitors
(Oral, Intravesical, Intradetrusor) ␤-Adrenergic antagonists, agonists
Anticholinergic agents Vaginal and perineal occlusive and/or supportive
Drugs with mixed actions devices; urethral plugs
Calcium antagonists Nonsurgical periurethral bulking
Potassium channel openers Collagen, synthetics, cell transfer
Prostaglandin inhibitors Vesicourethral suspension ± prolapse repair
␤-Adrenergic agonists (female)
␣-Adrenergic antagonists Sling procedures ± prolapse repair (female)
Tricyclic antidepressants; serotonin and Closure of the bladder outlet
norepinephrine reuptake inhibitors Artificial urinary sphincter
Dimethyl sulfoxide (DMSO) Bladder outlet reconstruction
Polysynaptic inhibitors Myoplasty (muscle transposition)
Capsaicin, resiniferatoxin, and like agents
Botulinum toxin Circumventing the Problem
Bladder overdistention
Electrical neuromodulation Absorbent products
Acupuncture and electroacupuncture External collecting devices
Interruption of innervation Antidiuretic hormone-like agents
Very central (subarachnoid block) Short-acting diuretics
Less central (sacral rhizotomy, Intermittent catheterization
selective sacral rhizotomy) Continuous catheterization
Peripheral motor or/and sensory Urinary diversion
Augmentation cystoplasty
(Auto, bowel, tissue engineering)

Adapted from: Wein AJ. Pathophysiology and classification of voiding dysfunction. In: Wein, Kavoussi, Novick, Partin,
Peters, eds. Campbell-Walsh Urology. Philadelphia, PA: Saunders/Elsevier; 2007:1973-1985.

340 EDUCATIONAL REVIEW MANUAL IN UROLOGY


Table 4

Therapy to Facilitate Bladder Emptying/Voiding

Bladder Related (Increasing Outlet Related (Increasing Outlet Resistance)


Intravesical Pressure or
Facilitating Bladder Contractility) Prostatectomy, prostatotomy
(diathermy, heat, laser)
External compression, Valsalva Bladder neck incision or resection
Promotion or initiating of reflex contraction Urethral stricture repair or dilation
Trigger zones or maneuvers Intraurethral stent
Bladder “training”; tidal drainage Balloon dilatation of stricture/contracture
Pharmacologic therapy (oral, intravesical)
Parasympathomimetic agents At level of smooth sphincter
Prostaglandins Pharmacologic Therapy
Blockers of inhibition ␣-Adrenergic antagonists
␣-Adrenergic antagonists ␤-Adrenergic agonists
Opioid antagonists Transurethral resection or incision
Electrical stimulation Y-V plasty
Directly to the bladder or spinal cord At level of striated sphincter
Directly to the nerve roots Behavioral therapy ± biofeedback
Intravesical (transurethral) Pharmacologic therapy
Neuromodulation Benzodiazepines
Reduction cystoplasty Baclofen
Bladder myoplasty (muscle wrap) Dantrolene
␣-Adrenergic antagonists
Outlet Related (Increasing Outlet Resistance) Botulinum toxin (injection)
Urethral overdilation
At a site of anatomic obstruction Surgical sphincterotomy
Pharmacologic Therapy— Urethral stent
decrease prostate size or tone Pudendal nerve interruption
␣-Adrenergic antagonists
5␣-Reductase inhibitors Circumventing the Problem
Luteinizing hormone-releasing
hormone agonists/antagonists Intermittent catheterization
Antiandrogens Continuous catheterization
Urinary diversion (conduit)

Adapted from: Wein AJ. Pathophysiology and classification of voiding dysfunction. In: Wein, Kavoussi, Novick, Partin,
Peters, eds. Campbell-Walsh Urology. Philadelphia, PA: Saunders/Elsevier; 2007:1973-1985.

CHAPTER 11: NEUROPATHIC BLADDER: VOIDING DYSFUNCTIONS ASSOCIATED WITH NEUROLOGICAL DISEASE 341
7. Further Reading 8. Questions

1. Abrams P, Cardozo L, Fall M, et al. The 1. All of the following statements regarding
standardisation of terminology in lower bladder compliance are true, except:
urinary tract function: report from the stan-
dardisation sub-committee of the Interna- A. Bladder compliance is defined as the
tional Continence Society. Urology. change in intravesical or detrusor pres-
2003;61:37-49. sure (Pdet) relative to the corresponding
change in volume.
2. Peterson AC, Webster GD. Urodynamic
and videourodynamic evaluation of void- B. Normal bladder compliance is 12.5
ing dysfunction. In: Wein AJ, Kavoussi mL/cm H2O.
LR, Novick AC, Partin AW, Peters CA,
eds. Campbell-Walsh Urology. Philadel- C. Is calculated between 2 points: the Pdet
phia, PA: Saunders/Elsevier; 2007: 1986- with the bladder empty at the start of uro-
2010. dynamic filling and the Pdet at either the
maximal cystometric capacity or the start
3. Rackley RR, Appell RA. Evaluation and of a detrusor contraction (involuntary or
management of lower urinary tract disor- voluntary).
ders in women with multiple sclerosis. Int
Urogynecol J Pelvic Floor Dysfunct. 1999; D. Compliance arises from the neuromus-
10(2):139-143. cular and biomechanical (collagenous
and elastic) components of the bladder
4. Wein AJ, Rackley RR. Overactive bladder: wall.
a better understanding of pathophysiology,
diagnosis and management. J Urol. 2006 2. The difference between the detrusor leak
Mar;175(3 Pt 2):S5-S10. point pressure (DLPP) and the abdominal
leak point pressure (ALPP), two pressures
5. Wein AJ. Pathophysiology and classifica- obtained during urodynamics that measure
tion of voiding dysfunction. In: Wein AJ, different aspects of lower urinary tract
Kavoussi LR, Novick AC, Partin AW, function, is the following:
Peters CA, eds. Campbell-Walsh Urology.
Philadelphia, PA: Saunders/Elsevier; A. How much fluid is in the bladder when
2007:1973-1985. the measurements are obtained.

6. Wein AJ, Dmochowski RR. Neuromuscu- B. When the Pdet is measured during the
lar dysfunction of the lower urinary tract. filling phase of the urodynamic study in
Wein AJ, Kavoussi LR, Novick AC, Partin the presence of increased abdominal
AW, Peters CA, eds. Campbell-Walsh pressure.
Urology. Philadelphia, PA: Saun-
ders/Elsevier; 2007:2011-2045. C. When the Pdet is measured during the
filling phase of the urodynamic study in
7. Roth B, Studer UE, Fowler CJ, Kessler the presence of a detrusor contraction.
TM. Benign prostatic obstruction and
parkinson's disease--should transurethral D. The rate of urodynamic filling of the
resection of the prostate be avoided? bladder.
J Urol. 2009;181:2209-2213.

342 EDUCATIONAL REVIEW MANUAL IN UROLOGY


3. The following statements regarding the D. Distal cord viability is a prerequisite.
smooth and striated sphincter muscle of the
bladder outlet and urethra are true, except: E. A. and C.

A. The smooth sphincter refers to the F. B. and D.


smooth musculature of the bladder neck
and proximal urethra. G. D. only.

B. The smooth muscle is a physiologic and H. All the above.


an anatomic sphincter and one that is not
under voluntary control. 5. To differentiate detrusor sphincter dyssynergia
from pelvic floor hyperactivity or dysfunctional
C. The striated sphincter refers to the stri- voiding, which one of the following statements
ated musculature that is a part of the must be true:
outer wall of the proximal urethra in
both the male and the female is often A. Failure of the sphincter to relax or stay
referred to as the intrinsic or intramural completely relaxed during micturition
striated sphincter. must be present.

D. The bulky striated skeletal muscle group B. Uninhibited contractions on the filling
that closely surrounds the urethra at the part of the urodynamics must be present.
level of the membranous portion in the
male and primarily the middle segment C. Neurologic disease must be present.
in the female is often referred to as the
extrinsic or extramural striated sphinc- D. Bladder sensation must be absent.
ter.
E. Bowel dysfunction must be present.
E. The extramural portion is the classically
described external urethral sphincter and F. A. and B.
is under voluntary control.
G. C. and D.
4. Autonomic hyperreflexia represents which
one of the following? H. E. only.

A. An acute massive disordered autonomic I. All of the above.


(primarily sympathetic) response to spe-
cific stimuli in patients with SCI above 6. Cauda equina syndrome is a term applied to
the cord level of T6 to T8 (the sympa- the clinical picture which typically includes
thetic outflow). which of the following criteria:

B. Onset after injury is variable—usually A. Loss of voluntary control of


soon after spinal shock but may be up to anal sphincter.
years after injury.
B. Perineal sensory loss.
C. It is more common in cervical (60%)
than thoracic (20%) injuries. C. Loss of voluntary control of the
urethral sphincter.

D. Loss of sexual responsiveness.

CHAPTER 11: NEUROPATHIC BLADDER: VOIDING DYSFUNCTIONS ASSOCIATED WITH NEUROLOGICAL DISEASE 343
E. Loss of motor function of the legs. C. Overactive obstructive urethral
function.
F. A. and C.
D. Low bladder capacity.
G. B. and D.
E. Normal bladder compliance.
H. E. only.
F. A. and C.
I. All of the above.
G. B. and D.
7. Spinal cord shock may be characterized by
which of the following features: H. E. only.

A. It represents a period of decreased I. All of the above.


excitability of spinal cord segments at
and below where the level of injury
occurs. 9. The voiding dysfunction of a stroke patient
with urgency incontinence would best be
B. It may be short term or chronic. described as follows:

C. It includes suppression of autonomic A. Overactive neurogenic detrusor


activity. function.

D. It includes a suppression of somatic B. Normal bladder sensation.


activity.
C. Normal urethral function.
E. The bladder is acontractile and areflexic.
D. Low bladder capacity.
F. A. and C.
E. Normal bladder compliance.
G. B. and D.
F. A. and C.
H. E. only.
G. B. and D.
I. All of the above.
H. E. only.
8. Lower urinary tract dysfunction in a classic
T10 spinal cord level paraplegic patient I. All of the above.
after spinal shock has passed would be
described as follows:

A. Overactive neurogenic detrusor


function.

B. Absent bladder sensation.

344 EDUCATIONAL REVIEW MANUAL IN UROLOGY


10. Which of the following comments regard-
ing vesicoureteral reflux in spinal cord
Answers

injury (SCI) patients are true: 1. A.


Compliance = change Volume / change Pdet
A. More common in suprasacral injuries. (expressed as mL/cm H2O)

B. Infections are a contributing factor. 2. B.


DLPP is defined by the ICS as the lowest detrusor
C. Elevated intravesical pressure during pressure at which urine leakage occurs in the
filling and emptying is a contributing absence of either a detrusor contraction or increased
factor. abdominal pressure. ALPP is defined as the intrav-
esical pressure at which urine leakage occurs
D. Persistent reflux can lead to chronic because of increased abdominal pressure in the
renal damage. absence of a detrusor contraction.

E. Risk factor for decreased long term sur- 3. B.


vival in SCI patients. The smooth sphincter refers to the smooth muscula-
ture of the bladder neck and proximal urethra. This
F. A. and C. is a physiologic but not an anatomic sphincter and
one that is not under voluntary control.
G. B. and D.
4. H. All of the above.
H. E. only. Distal spinal cord viability (incomplete or partial)
has to be intact for somatic and sensory stimuli to
I. All of the above. enter CNS system to trigger the sympathetic out-
flow.

5. F.
Failure of the sphincter to relax or stay completely
relaxed during micturition is abnormal. When it
occurs in patients with neurologic disease, it is
termed detrusor sphincter dyssynergia; this typi-
cally occurs in patients with suprasacral spinal cord
injury in which there is an interruption of the
spinobulbar-spinal pathways that normally coordi-
nate the detrusor and the sphincter. In the absence of
neurologic disease, one cannot use the term detrusor
sphincter dyssynergia. Instead, the term pelvic floor
hyperactivity or dysfunctional voiding is used.

6. I. All of the above.


Loss of leg motor function is not typically seen in
cauda equina syndrome. In addition to all of the
above findings, cauda equina syndrome occurs sec-
ondary to disk disease (severe central posterior disc
protrusion) and other spinal canal pathologic pro-
cesses as well.

CHAPTER 11: NEUROPATHIC BLADDER: VOIDING DYSFUNCTIONS ASSOCIATED WITH NEUROLOGICAL DISEASE 345
7. I. All of the above.

8. I. All of the above.


Generally, complete spinal cord lesions above the
sacral spinal cord, but below the area of the sympa-
thetic outflow, result in detrusor overactivity, absent
bladder sensation, and striated sphincter dyssyner-
gia. While normal bladder compliance may be
maintained, reduced bladder capacity is typically
noted.

9. I. All of the above.


The most common type of voiding dysfunction after
stroke would then be characterized as a failure to
store secondary to bladder overactivity, specifically
involuntary bladder contractions. The dysfunction
would most likely be classified as overactive neuro-
genic detrusor function, normal sensation, low
capacity, normal compliance, and normal urethral
closure function during storage; during voiding the
description would be normal detrusor activity and
normal urethral function assuming that no anatomic
obstruction existed. Treatment, in the absence of
coexisting significant bladder obstruction or signifi-
cantly impaired contractility, is directed at decreas-
ing bladder contractility and increasing bladder
capacity.

10. I. All of the above.


Surprisingly little is written about vesicoureteral
reflux (VUR) in the SCI patient. The reported inci-
dence varies between 17% and 25% of such patients
and is more common in those with suprasacral SCI.
Contributing factors include: 1) elevated intravesi-
cal pressure during filling and emptying; and 2)
infection. Persistent reflux can lead to chronic renal
damage and may be an important factor in the long-
term survival of SCI patients. In the series of SCI
patients, persistent reflux was present in 60% of
patients of those dying of renal disease. In patients
with only transient reflux over a 5- to 15-year
period, the urogram was normal in 83%, or calyceal
changes were only minimal. It should be noted that
high storage and voiding pressures without reflux
can be responsible for renal damage.

The best initial treatment for reflux in a patient with


voiding dysfunction secondary to neurologic dis-
ease or injury is to normalize lower urinary tract
urodynamics as much as possible.

346 EDUCATIONAL REVIEW MANUAL IN UROLOGY


Chapter 12:
Female Urology and
Urinary Incontinence
J. Christian Winters, MD, FACS
Revised 2012 by Harriette Scarpero, MD

Contents

1. Urinary Incontinence: Pathophysiology,


Evaluation and Treatment

2. Pelvic Organ Prolapse

3. Vesicovaginal Fistulae

4. Urethra Diverticulum and


Periurethral Masses

5. Overactive Bladder

6. Questions

CHAPTER 12: FEMALE UROLOGY AND URINARY INCONTINENCE 347


1. Urinary Incontinence:
Pathophysiology, Evaluation
and Treatment

Urinary incontinence is defined as the involuntary a. Extremely important when evaluating


loss of urine per urethra. Stress urinary incontinence incontinence in elderly or patients with
is the involuntary leakage of urine on effort or exer- significant comorbidity.
tion or on sneezing or coughing. The International
Continence Society (ICS) defines incontinence in B. Functionally Defined Classification:
various domains: The symptom, where a patient or
caregiver describes involuntary urine loss; the sign, 1. Failure to Empty
which is an objective demonstration of urine loss;
the urodynamic (UDS) observation, which is incon- a. Bladder Underactivity
tinence demonstrated during UDS testing; and the
condition, which is the lower urinary tract patho- b. Urethral Obstruction
physiology causing incontinence as demonstrated
during evaluation. 2. Failure to Store (Urinary Incontinence)

One must remember that incontinence is a symptom a. Urethral Incompetence


of an underlying disorder, not a disease process.
This disorder has significant socioeconomic impact b. Bladder Overactivity
on the quality of life of our patients. More than 20
million Americans are incontinent and urinary
incontinence accounts for at least 30% of nursing
Other Causes of Incontinence

home admissions. It is estimated that 650,000 senior Overflow incontinence: associated with urinary
citizens restrict activity as a result of incontinence, retention and increased PVR. The etiology is vari-
and as many as 50% of females over the age of 40 able, either from a reflex urethral relaxation or an
experience some episodes of wetting. unstable contraction of the bladder. This may
occur as a result of: outlet obstruction (cystocele/
prostate), diabetes and neurogenic retention. There
are also anatomic causes of incontinence, which
Classification of Incontinence

Urinary incontinence is classified in many ways; the include: fistula, urethral diverticulum, vaginal void-
most commonly used method to classify this condi- ing and ectopic ureters (predominately pediatric).
tion is by way of a symptomatic or functional classi- Evaluation of incontinent patients must include
fication. methods to rule out these conditions.

A. Symptomatically Defined Classification: Transient/Reversible Causes of Urinary

1. Stress Urinary Incontinence (SUI)


Incontinence (DIAPPERS)

Delirium
2. Urge Incontinence Infection (UTI)
Atrophic changes of the genital tract
3. Overflow Incontinence Pharmacologic agents
Psychologic/psychiatric factors
a. Retention of large urine volumes Excess urine production
precipitates reflex urethral relaxation Restricted mobility
or unstable bladder contraction. Stool impaction

4. Total Incontinence

5. Transient Incontinence

348 EDUCATIONAL REVIEW MANUAL IN UROLOGY


On physical examination, it is of paramount impor-
tance to assess the degree of urethral hypermobility.
Drugs that Affect the Lower Urinary Tract

Potentiate urinary retention A Q-tip test can be utilized to quantify the degree of
Psychotropics urethral hypermobility. The vaginal examination
antidepressants should assess the presence of: cystocele, enterocele,
antipsychotics rectocele, vault or uterine prolapse and perineal
sedative/hypnotics body integrity. The quality of the vaginal mucosa
Calcium channel blockers should be assessed. A neurourologic examination
Anticholinergics (bulbocavernosus reflex, cremasteric reflex, per-
Alpha-adrenergic agonists ineal sensation) should be considered.
Beta-agonists
Narcotics A post-void residual (PVR) and stress test are also
adjuncts to the physical exam.
Potentiate urinary incontinence
ACE inhibitors A urinalysis should be performed on all patients. In
Alpha-adrenergic blockers patients with any degree of hematuria, a workup
Alcohol should be considered as incontinence may be a
Caffeine symptom.
Diuretics
Radiographic imaging may be obtained (cys-
togram/defecography) in complex cases of inconti-
nence. Cystograms with straining views are useful
The Evaluation of Incontinence

As with all conditions, the evaluation of inconti- in complex cases of incontinence with pelvic organ
nence begins with a thorough history and physical prolapse.
examination. One should assess the following
symptomatology: Does leakage occur with exertion Urodynamic studies investigate the interaction of
(cough, exercise, etc.)? Does leakage occur with the detrusor activity and urethral sphincter activity.
impending sensation to void? Is there supine leak- This is done by measurement of pressure changes
age or bedwetting (usually in complicated cases)? A within the bladder and urethral sphincter. A cys-
very important aspect of the history is: How has tometrogram measures bladder pressure during fill-
incontinence changed the quality of life? How much ing and voiding. From these measurements, one
is the patient bothered by these symptoms? Empty- should identify: bladder capacity, bladder sensation,
ing symptoms such as hesitation, straining to void, bladder contractility, bladder compliance and the
intermittent or slow stream, and a feeling of incom- presence (or absence) of involuntary bladder activ-
plete emptying are essential to document in the his- ity. Urethral pressure profilometry or abdominal
tory. Impaired voiding may be the source of storage leak point pressure measurement assesses urethral
symptoms, particularly urinary incontinence. sphincter function. These studies quantify the
intrinsic closing pressures of the urethra. Concomi-
In addition, one should ask if there is pelvic or per- tant EMG activity (patch or needle) is used to assess
ineal pain or pressure related to prolapse and/or sex- striated voluntary external sphincter function. How-
ual dysfunction. One should also assess bowel func- ever, one must appreciate the tremendous anatomic
tion: constipation, splinting or fecal incontinence. detail that fluoroscopy provides when assessing
The past history assessment should include the sphincter function. Videourodynamics, which is the
number of pregnancies and vaginal deliveries, and if combination of multichannel urodynamics and fluo-
there were any complications associated with them. roscopy, provides the most comprehensive assess-
Also, was there any history of gynecologic surgery? ment of anatomy and function of the lower urinary
One should obtain a detailed history of any neuro- tract.
logic disorder, previous bladder suspension or vagi-
nal repair.

CHAPTER 12: FEMALE UROLOGY AND URINARY INCONTINENCE 349


symptoms in up to 90% of women. Dropout rates
are high and physicians need to continually remind
Nonsurgical Treatment of Stress

patients. One should also instruct patients on the


Urinary Incontinence

As with most conditions, an initial conservative avoidance of bladder irritants. This is crucial in
approach is appropriate. When one selects therapy patients with urge incontinence. Patients should be
for urinary incontinence, it is essential to consider taught to avoid: acidic foods, citrus juices and fruits,
the degree of patient bother and also what the caffeinated and alcoholic beverages, and spicy
patient expectations are. If a patient’s major goal is foods. Pelvic floor muscle training (PFMT) is actu-
to eliminate nocturia, a sling for coexistent stress ally comprised of a comprehensive program of
urinary incontinence is probably not the best initial pelvic floor physiotherapy and not just Kegel exer-
approach for her. Whereas if a woman cannot exer- cises. The goal is to strengthen the levator support
cise because of SUI and her expectation is to be able system and enhancing reflex contraction of the
to resume those activities, a sling may be a perfect pelvic muscles at the time of stress. Many studies
choice. report improvement in high proportions of women,
although few become dry. Greater improvement is
Conservative therapy is behavioral modification, achieved in women who are physically instructed
which may be effective in stress and urge inconti- on proper technique and with the addition of
nence. Bladder diaries are an inexpensive and easy biofeedback. This is a most useful modality in
way of obtaining baseline data on a patient’s void- women with pelvic pain, dysfunctional voiding and
ing patterns in her own environment. The informa- interstitial cystitis.
tion obtained may provide insights into behavioral
factors that could be responsible for urinary inconti- Stress urinary incontinence is not treated pharmaco-
nence. No standardization exists for what informa- logically. Alpha-agonists which increase the
tion should be recorded, and patient compliance is sphincteric tone were previously utilized:
an issue with any diary. In most cases, the bladder ephedrine, pseudoephedrine, phenylephrine or
diary should include the time of micturition, time phenylpropanolamine (Entex® LA, Ornade). These
and type of incontinence, and voided volume in a agents had limited effect and results were usually
24-hour period. In addition, the patient should quan- transient. One must be aware of the adverse events
tify her fluid intake as closely as possible and record associated with these medications, including hyper-
the time at which she is drinking. Recording the tension and stroke. In fact, the FDA has eliminated
type of beverage can help point out if she is a heavy the use of many alpha-agonist medications. It is for
consumer of bladder irritants, such as caffeine and these reasons that this treatment modality cannot be
alcohol. Review of the bladder diary provides an recommended at this time in women with stress uri-
assessment of the patient’s day and night time fre- nary incontinence. Estrogens have primary usage
quency, a quantification of how much she drinks a for menopausal atrophic urethral and vaginal symp-
day and an estimate of her functional bladder capac- toms. Thus, topical estrogen may be useful in
ity, which is the largest amount of urine her bladder chronic cystitis/UTI or atrophic urethritis. Several
will comfortably hold. From these data, one can large clinical trials using estrogen and a meta-analy-
institute measures such as fluid management (moni- sis of the literature have demonstrated that estrogen
toring intake) and timed voids. Comparing bladder is not effective primary or monotherapy for SUI or
diaries before and after initiation of treatment may urge incontinence. In fact, the Women’s Health Ini-
be used to assess response. tiative Project found that estrogen actually wors-
ened incontinence symptoms. These findings have
The principle of timed voids is to establish voiding led many to use estrogen only in cases of urethritis
by the clock (by time) as opposed to voiding in or recurrent UTI in association with vaginal atro-
response to urge. The bladder volumes remain phy.
lower. This facilitates bladder emptying before an
involuntary contraction may occur and compensates
for the lack of warning time in OAB patients.
Expected results include initial improvement in

350 EDUCATIONAL REVIEW MANUAL IN UROLOGY


Several FDA-approved bulking agents are available
for use today: Contigen®, Durasphere™, Coaptite
Surgical Management of Incontinence

NOTE: Be familiar with Guideline for the and Macroplastique. Contigen® (Bard, Inc. - Cov-
Surgical Management of Female Stress Urinary ington, GA) is a glutaraldehyde cross-linked bovine
Incontinence: 2009 Update collagen that causes no inflammatory reaction and
becomes colonized by host fibroblasts after injec-
When conservative therapy fails (or the patient tion. Patients must be skin tested 30 days prior to
chooses surgery as treatment of choice), there are a injection to prove the patient non-allergic. Injection
number of surgical options for stress urinary incon- of Contigen® is easy in experienced hands and well
tinence. There are 4 main categories of surgical SUI tolerated by the patient. Its effect is not permanent
treatments: retropubic suspensions, slings, bulking and repeat injections will be necessary. Durasphere™
agents and artificial urinary sphincters. (Carbon Medical Technologies, Inc. – St. Paul, MN)
is composed of nonabsorbable pyrolytic zirconium
Standard from Guideline: oxide beads in a carrier gel. It is a reasonable first
option bulking agent and also offers an option for
a. History: identify impact on quality of life the patient with an allergy to Contigen®. An advan-
tage of this material is that no skin test is needed
b. Physical Exam: positive stress test before its administration. Coaptite is synthetic cal-
cium hydroxyapatite. Macroplastique is silicone
c. Urinalysis microparticles.

d. Other diagnostic tests to evaluate cause of Success rates of bulking agents vary widely from
incontinence study to study, and our ability to interpret the results
is hampered by differences in the definitions of cure
e. Cystoscopy should be performed during all and dry used and lack of long-term results. Overall,
sling procedures (2007) short and intermediate term results with both agents
support dry rates of 25%, improvement rates of 50%
f. Synthetic midurethral sling procedures and failure rates of 25%. Patient satisfaction with
should not be done at same time as urethral bulking agents is highly dependent on them having
reconstruction or urethral diverticulectomy the correct expectations of improvement rather than
(2007) cure.

The Panel found equivalent results for retropubic Retropubic suspensions and autologous fascial sling
midurethral slings to Burch and AFS. procedures are the most effective procedures for
long-term success. Both procedures show cure rates
Bulking agents are minimally invasive, temporary of approximately 85% at 4 years. Slings differ in
options for the treatment of SUI. Originally material, suspension technique and length. Numer-
described for the treatment of ISD, use has been ous biologic materials (allografts and xenografts)
expanded to include patients with hypermobility are commercially available and obviate the need to
and to salvage a recurrent incontinence after sling harvest autologous fascia. Use of these tissues poses
procedures or urethrolysis. These materials are new questions and concerns regarding biocompati-
injected with transurethral or periurethral method bility, reaction or integration with host tissue, and
during cystourethroscopy to coapt the mucosa of the disease transmission. Allografts and xenografts are
proximal urethra and increase outlet resistance. The meant to function as scaffolding for the ingrowth of
risk of permanent urinary retention is almost negli- native tissue that ultimately will replace the graft,
gible and rates of de novo detrusor overactivity are but recent data questions the permanence of some
less than or equal to that with surgery. Disadvan- materials.
tages are that one injection may not be enough to
obtain dryness. Bulking agents need to be reinjected
over time.

CHAPTER 12: FEMALE UROLOGY AND URINARY INCONTINENCE 351


Autologous fascia is an attractive sling material Synthetic midurethral slings such as the tension-free
because it is cost-effective, available and biocom- vaginal tape (TVT) reconstitute tension in the pub-
patible by definition. Rectus fascia and fascia lata ourethral ligaments and increase resistance in the
are the autologous sling materials of choice. urethra. Placed under no tension, the sling works by
Regardless of the material used, the pubovaginal physically kinking the urethra during strain.
sling attempts to restore sufficient outlet resistance Midurethral slings have now become the most com-
to prevent stress urinary incontinence without com- mon surgical procedure performed for SUI. Long-
promising normal voiding or producing voiding term outcomes in stress urinary incontinence
dysfunction. Historically the pubovaginal sling was demonstrate 84.7% of women are objectively and
reserved for SUI due to intrinsic sphincteric defi- subjectively cured of stress leakage and another
ciency or prior surgical failure, but its use has 10.6% are improved. In patients with mixed symp-
expanded to include all forms of SUI. The evolu- toms, 85% are cured of stress and urge symptoms.
tion of our theories of the pathophysiology of SUI
has extended the use of pubovaginal slings to all NOTE: AUA Best Practice Statement on
types of SUI. The pubovaginal sling therefore, may Urologic Surgery: Antimicrobial Prophylaxis
be applied universally in SUI. The choice of what recommends antimicrobial prophylaxis in all
anti-incontinence procedure to perform and by what vaginal surgery (including urethral sling
technique is still based on a variety of factors: the procedures) with 1st/2nd generation
patient wishes, patient characteristics, surgeon’s cephalosporins for ≤24 hours.
experience and surgeon’s comfort level with a par-
ticular technique.

Complications unique to autologous rectus slings


include harvest site infection, seroma or hematoma
formation, herniation or pain at the site. Transient
obstructive symptoms are quite common. Urinary
retention requiring urethrolysis, which is the surgi-
cal loosening of a prior suspension procedure if it
considered too tight, occurs in 1%–2% of cases. No
cases of rejection have been reported with autolo-
gous materials, and the few reported cases of ero-
sion were likely due to excessive sling tension or
overly aggressive periurethral dissection. Sling
surgery with most alternative biologic materials
produces short-term success rates comparable to
autologous fascia. Rates of postoperative voiding
dysfunction and urinary retention appear similar as
well.

Allograft and xenograft slings shorten operative


times and obviate the morbidity of fascial harvest.
Current literature points to higher risks of early fail-
ure and immunogenicity leading to rejection and
poor tissue healing. Patients must be counseled pre-
operatively and informed consent must include
information regarding the sling material to be used.

352 EDUCATIONAL REVIEW MANUAL IN UROLOGY


2. Pelvic Organ Prolapse

cygeus muscle runs atop the sacrospinous ligament.


Coccygeal pain can develop after SSLF because
Normal Female Pelvic Anatomy

Understanding anatomic and functional defects of sutures penetrate this muscle as well as the liga-
the female pelvis that contribute to female pelvic ment. The levator ani muscles include the puborec-
floor dysfunction requires knowledge of normal talis, pubococcygeus and iliococcygeus.
anatomy and their dynamic interrelated nature. The
pelvic floor is a term that refers to the bony pelvis, The pelvic diaphragm attaches to the pelvic side-
muscles and fascia of the pelvis, and pelvic organs. wall at the arcus tendineus fasciae pelvis (ATFP),
which is a condensation of pelvic fascia overlying
Bony Pelvis the obturator internus from the ischial spine to the
pubic symphysis. During transvaginal prolapse
The bony pelvis is composed of the sacrum, coccyx surgery, the ATFP may be palpated as a band-like
and 2 innominate bones formed by the fusion of the structure on the pelvic sidewall. The ATFP is the
iliac, ischial and pubic bones. The opening formed major support of the anterior pelvic diaphragm to
between the pubic and ischial bones medial to the the bony pelvis.
acetabulum is the obturator foramen. The ischiopu-
bic ramus is medial to the fusion of the pubic and Openings in the midline LAG provide passage of
ischial bones. the urethra, vagina and rectum and are together
referred to as the levator hiatus.
Clinical correlation: transobturator slings traverse
the obturator foramen and are passed beneath the
ischiopubic ramus.
Fascia of the Pelvic Floor

The LAG and associated fascial attachments pro-


If examining from above, the pelvis is diamond- vide support. Any weakness of these structures con-
shaped with the pubic symphysis anteriorly, the tributes to functional disorders of urinary or fecal
sacrum posteriorly, and ischial spines laterally as incontinence and pelvic organ prolapse. The
the apices. These are crucial anchoring points for endopelvic fascia lies superior to the LAG just
supporting structures of the pelvic floor. beneath the peritoneum and covers adjacent organs.
Condensations of endopelvic fascia are separately
Cooper’s Ligament (pectineal ligament)—overlies named according to their discrete function, but they
the pectineal line on the pubic bone. The Burch col- are in fact in continuity.
posuspension suspends the BN to Cooper’s liga-
ment bilaterally. Although retropubic procedures Pubocervical fascia— runs from the pubis to the
are less common, the Burch is still upheld as an effi- cervix and lies anteriorly around the bladder.
cacious incontinence procedure.
Cardinal and uterosacral ligaments—fascia that
Sacrospinous Ligament—runs from the ischial lies posteriorly around the cervix.
spines to the anterolateral sacrum and coccyx. They
create the greater and lesser sciatic foramen. These Pubourethral ligament—supports the urethra to
are often the anchoring points for transvaginal api- the inferior pubis just proximal to the external
cal suspensions known as sacrospinous ligament sphincter.
fixation (SSLF).
Urethropelvic ligament—fusion of endopelvic and
periurethral fascia and is continuous with the pubo-
cervical fascia laterally. They attach laterally to the
Muscles of the Pelvic Floor

The muscles of the pelvic floor provide dynamic ATFP and are therefore important support for the
support for the pelvic viscera and abdominopelvic urethra and anterior vaginal wall.
cavity. What is known as the pelvic diaphragm is
composed of the levator ani group (LAG) of mus-
cles, coccygeus muscles and their fascia. The coc-

CHAPTER 12: FEMALE UROLOGY AND URINARY INCONTINENCE 353


inserting laterally to the pubourethral ligament and
the arcus tendineus and to the perineal body posteri-
Pelvic Nerves and Vessels

Obturator neurovascular bundle—runs in the orly.


obturator canal, over the obturator internus muscle
about 5 cm superolateral to the midpoint of the
ischiopubic ramus. It must be avoided as any
Types of Prolapse

trochar device is passed around the ischiopubic Cystocele


ramus. There are 4 different areas within the connective tis-
sue support of the bladder that predispose one to
Internal pudendal neurovascular bundle—ves- cystocele if the connective tissue support fails.
sels and the pudendal nerve (S2-4) exit the pelvis There are 4 anatomic defects that can account for
through the greater sciatic foramen, and wrap the development of cystourethrocele (see Figure 1):
around the ischial spine and sacrospinous ligament
laterally. Care must be taken to avoid these when a. Paravaginal defect (lateral defect)
placing sutures in the SSL. They then pass through
the lesser sciatic foramen and run alongside the lat- b. Transverse defect
eral aspect of the ischiorectal fossa to enter the
pudendal canal (Alcock’s) in the posterior per- c. Midline defect (central defect)
ineum. The pudendal nerves branch into the inferior
rectal, perineal and dorsal clitoral nerves. d. Distal defect (urethrocele)

The paravaginal defect occurs when a separation of


the pubocervical fascia from its lateral attachment
Levels of Anatomic Pelvic Support

Muscular and fascial pelvic support is also com- to the fascia over the obturator internus muscle
monly described as 3 levels of vaginal support occurs at the level of the arcus tendineus fascia
(Level I, II and III). The upper third of the vagina pelvis. This represents a break of the pubocervical
(level I) and uterus are supported by fibers from the fascia from the white line. This loss of lateral attach-
uterosacral and cardinal ligaments. This supports ments can occur both unilaterally or bilaterally.
the upper vagina above the pelvic diaphragm. These Usually, a cystourethrocele is seen when this lateral
fibers mainly merge into the pericervical ring of loss of support occurs, and this defect usually pre-
connective tissue and into the upper vagina. The disposes one to symptoms of stress urinary inconti-
middle third of the vagina is attached by the mid- nence.
portion of the endopelvic fascia (level II). The ante-
rior wall of the vagina in this location is held in The transverse defect is a separation of the pubocer-
place by the lateral attachments of the pubocervical vical fascia from its attachment to the pericervical
fascia to the fascia over the obturator internus mus- ring of tissue at the apex of the vagina (level I). This
cle at the arcus tendineus fascia pelvis (ATFP) (Fig- will allow the base of the bladder to herniate into the
ure 1). anterior vagina.

The posterior vaginal wall is supported laterally by The midline defect is any break in the central por-
the lateral attachments of the rectovaginal fascia to tion of the hammock-like sling of pubocervical fas-
the fascia overlying the iliococcygeus muscle. At cia upon which the bladder is resting (Figure 2).
the lower third of the vagina (level III), the vagina
merges with the fascia of the endopelvic fascia and Commonly, this condition can create stress inconti-
pubourethral ligaments anteriorly to the medial nence as well, because the hammock-like break in
margins of the pubococcygeus. The lower extent of the pubocervical fascia does involve the area under-
the pubocervical fascia merges into the urogenital neath the bladder neck. This occurs commonly in
diaphragm and the rectovaginal fascia merges into patients with paravaginal and lateral defects. How-
the perineal body. Therefore, in the distal third of ever, they are rare in isolation. The distal defect is an
the vagina, the endopelvic fascia structures are avulsion of the urethral attachment to the urogenital

354 EDUCATIONAL REVIEW MANUAL IN UROLOGY


Figure 1

Insertion of lateral vaginal attachments on the arcus tendineus fascia pelvis

Reprinted with permission: AUA Update Series. Lesson 23, Volume 21; 2002.

Figure 2

Break in pubocervical fascia creating central cystocele

Reprinted with permission: AUA Update Series. Volume 25, Lesson 25; 2006.

CHAPTER 12: FEMALE UROLOGY AND URINARY INCONTINENCE 355


diaphragm as it passes under the pubic symphysis.
Essentially, these patients lose the lateral attach-
Rectocele

ment to the urethra, to the arcus tendineus and pub- The anatomic defect creating a rectocele is similar
ourethral ligament. to the midline defect anteriorly creating a cystocele.
As the hammock of rectovaginal fascia overlying
In addition, these patients lose the anterior attach- the rectum breaks, a bulge of the rectum into the
ment of the urethra to the pubic symphysis. This vaginal canal occurs. A transverse defect rectocele
would account for an urethrocele. This condition occurs simply by a detachment of the perineal body
does predispose one to SUI. from the rectovaginal fascia. The hammock of rec-
tovaginal fascia supporting the rectum remains
intact but separates from the perineal body. A mid-
line vertical defect is created by a midline separa-
Uterine or Vaginal Vault Prolapse

In patients with a loss of the level I support of the tion of the rectovaginal fascia, and a separation of
uterosacral ligaments and cardinal ligaments, the the rectovaginal fascia can occur from its lateral
apex of the vagina (cervix or vaginal cuff) loses its attachments. Rectoceles are more commonly situ-
attachment. Therefore, the apex of the vagina herni- ated in the mid- to distal aspect of the posterior vagi-
ates downward as a result of the increased intraab- nal wall. These specific anatomic defects are the
dominal pressure. This condition will lead to pro- principles behind site-specific reconstruction of
lapse of the vaginal cuff and/or prolapse of the posterior defects.
uterus. Uterine descensus is the result of, and not the
cause of, vaginal vault prolapse. It is a result of the
loss of level I fascia support that creates uterine or
Epidemiology of Pelvic Floor Disorders

vaginal vault prolapse.


(Urinary Incontinence and Pelvic
Organ Prolapse)

Prevalence differs by the population studied and


definition used. Among adult women in the commu-
Enterocele

An enterocele is a herniation of the cul-de-sac peri- nity, the prevalence of any urinary incontinence
toneum with or without intraperitoneal contents into ranges widely from 9%–69%. When defined as
the fascial layers between the vagina and rectal daily or most of the time, urinary incontinence
walls. An enterocele may range from a small bulge prevalence is 3%–17%.
posteriorly in the upper part of the vagina to a large
defect which protrudes beyond the introitus with Since the symptoms of pelvic organ prolapse (POP)
visible small bowel internally. The upper aspect of are nonspecific, determining prevalence is harder
the posterior vaginal wall is where enteroceles by questionnaire alone without a confirmatory
occur as a result of a separation of the rectovaginal exam; therefore, determination of the prevalence of
septum from the level I complex of support. Iatro- POP is difficult. There is no consensus as to what
genic enteroceles develop after surgical procedures level of physical findings define clinically signifi-
that distort the normal horizontal axis toward the cant prolapse.
vertical, such as retropubic urethropexy. In approxi-
mately 30% of the patients following a Marshall- Common risk factors for pelvic floor disorders
Marchetti-Krantz bladder neck procedure or a include sex, age, race, parity, estrogen status, obe-
Burch bladder neck suspension, enteroceles have sity and smoking. Other risk factors for bladder dys-
been reported. function include: neurologic disease, previous
pelvic irradiation, pelvic surgery or pelvic trauma.
Urinary incontinence has a 3:1 predilection toward
women. The prevalence of incontinence increases
with age. Whether difference in prevalence accord-
ing to race has a biologic or sociocultural cause is
not clear. Parity, however, is a well-established risk
factor for urinary incontinence and prolapse in

356 EDUCATIONAL REVIEW MANUAL IN UROLOGY


young women. The effect of childbirth decreases previous bladder suspension may indicate the possi-
with age. Vaginal childbirth has a higher risk for bility of an anatomic abnormality.
pelvic floor dysfunction than C-section. An epi-
siotomy at the time of vaginal delivery predisposes Symptoms related to pelvic organ prolapse should
to fecal incontinence. be assessed. Vaginal pressure or protrusion is the
most common symptom associated with prolapse.
Other symptoms include vaginal discharge, back
pain, sexual difficulty, dysfunctional voiding or
Diagnosis of Pelvic Support Defects

Grading of Prolapse bowel dysfunction. Bowel habits should also be


Many grading systems evaluate the severity of assessed. Patients may have difficulty emptying the
pelvic organ prolapse; however, these systems are rectum, requiring pressure on the posterior vaginal
fairly subjective, and cross comparison is difficult. wall and perineum (known as splinting). Constipa-
Baden and Walker introduced the “half way” sys- tion and fecal incontinence are other bowel symp-
tem, classifying pelvic organ relaxation, allowing toms that should be elicited.
one to assess each area of prolapse in relation to the
hymen while the patient is straining. The Pelvic Medication usage, including over-the-counter
Organ Prolapse Quantification examination (OTC) medication, should be documented. Hor-
(POPQ) is a reproducible method to evaluate all monal status and the use of estrogens are important.
quadrants of pelvic floor support. Distinct measure- The obstetrical history, including parity, route of
ments of various sites are obtained in relation to the deliveries, previous episiotomy and obstetrical
hymenal ring in order to define the stage of pro- complications should be determined.
lapse. These examination techniques should be
emphasized to more adequately standardize pelvic The use of a questionnaire can facilitate obtaining
examinations. an accurate history. This ensures that questions con-
cerning the types and severity of incontinence, void-
Diagnostic Testing ing dysfunction, quality-of-life (QOL) impact and
The evaluation begins with a thorough history. It is risk factors can be answered by the patient prior to
important to determine the extent and bother of a meeting with the clinician. There are incontinence
vaginal bulge, urinary symptoms, bowel symptoms severity scales which are validated instruments to
and sexual symptoms. Patients may complain that objectively estimate the adverse impact inconti-
they can see a bulge or feel a bulge. They may nence has on QOL. These scales can be compared
describe a feeling of “sitting on a ball.” Women with before and after treatment to assist in outcome
prolapse may or may not be incontinent. If inconti- assessment. The most commonly used instruments
nent, the frequency and characteristics of inconti- are the Incontinence Impact Questionnaire (IIQ-7)
nent episodes should be described. One must deter- and the Urogenital Distress Inventory Scale (UDI-
mine the type(s) of incontinence present by eliciting 6).
information regarding precipitants of the inconti-
nent episode. Did leakage occur with coughing or Physical examination
did it occur with a sensation of needing to void? Are A thorough physical examination is essential to
protective pads necessary? If so, what types of pads establish the etiology of urinary incontinence and
and how saturated are they? The clinician should the presence of concomitant pelvic floor pathology.
understand that it is often very difficult for patients During the physical examination, every segment of
to discuss these symptoms. the pelvic floor should be evaluated. The anterior
vaginal wall, vaginal apex (or uterus), the upper
A voiding history should be obtained. Day and night portion of the posterior vaginal wall, the distal por-
time urinary frequency and any abnormality in urine tion of the posterior vaginal wall and the perineum
flow (weak or interrupted) or difficulty initiating the should be separately evaluated. The pelvic exami-
urine stream requiring straining should be assessed. nation begins with the patient in the lithotomy posi-
The presence of nocturnal frequency and enuresis is tion with straining. If all defects of pelvic floor sup-
important. A history of recurrent UTI, hematuria or port can be assessed in this position, the exam is

CHAPTER 12: FEMALE UROLOGY AND URINARY INCONTINENCE 357


then completed. A lower half of a vaginal speculum scar and “dimples” at the lateral aspects of the
is utilized to examine for these defects. During vagina are landmarks that are used to localize the
examination of the anterior vaginal wall, the ure- apex. These dimples usually represent the attach-
thral support should be established. A Q-tip test may ments of the uterosacral ligaments. When these
be employed with a well-lubricated Q-tip placed at landmarks are noted, and the apex of the vagina
the level of the bladder neck. During straining, a extends halfway to the hymenal ring with straining,
deflection of the Q-tip >30° signifies urethral hyper- these patients have a significant prolapse of the
mobility. In many cases, the lack of support and vaginal vault. This is a difficult aspect of the pelvic
hypermobility of the urethra are obvious with examination that warrants attention to detail as api-
straining; therefore, the Q-tip examination is cal prolapse can be hidden by large anterior or pos-
reserved for equivocal examinations only. Addition- terior compartment prolapse or partially reduced by
ally, the urethra should be inspected for characteris- the examining speculum. In most cases, these
tics of scarring, kinking, overcorrection or fistula. patients should be evaluated standing with the
The characteristic “stove pipe” urethra is still examiner’s fingers placed at the vaginal apex. If the
encountered, but usually more subtle findings on examiner’s fingers descend halfway to the hymenal
examination may indicate postoperative changes ring or more with straining, vaginal vault prolapse is
that adversely effect urethral function. A stress test present.
is performed in an attempt to elicit SUI. This test
should be performed in the lithotomy and standing After the pelvic examination is completed, lower
positions. If loss of urine is seen during the straining urinary tract function should be assessed. It is
maneuver, stress incontinence is likely present. known that stress incontinence may develop after
When a cystocele is present, it should be determined the correction of some types of female pelvic organ
if the loss is in the midline (central defect) or if the prolapse. This probably occurs as a result of
loss of support is caused by a lateral defect separa- unmasking occult stress urinary incontinence after
tion of the vagina from its attachments to the pelvic the bladder support is restored. Therefore, in all
sidewall at the arcus tendineus. patients with severe prolapse, despite a lack of com-
plaints, lower urinary tract function should be
With a speculum then displacing the anterior vagi- assessed. A urinalysis and residual urine are rou-
nal wall, the posterior wall of the vagina is assessed. tinely obtained. A stress test with the reduction of
It is important to distinguish the etiology of prolapse the vaginal prolapse by a pessary or vaginal packing
high in the posterior wall as either enterocele or high should be performed. Care should be taken not to
rectocele. This is accomplished by bimanual recto- occlude the urethra during packing as a false-nega-
vaginal evaluation in the supine and/or standing tive result may occur. Many of these women may
position to detect enterocele. If a sac or bowel can present with concomitant urethral sphincteric defi-
be palpated during this exam between the rectum ciency; therefore, the authors prefer urodynamics
and the examiner’s finger, this indicates the pres- with vaginal packing to assess abdominal leak point
ence of an enterocele. If no enterocele sac or bowel pressures to evaluate urethral function. This is an
is palpated between the rectum and the vagina, this accurate way to assess abdominal leak point pres-
defect is most likely a high rectocele. sures in patients with significant prolapse and
allows selection of the most appropriate stress
Perhaps the most critical aspect of the exam is to incontinence procedure. All patients should be care-
assess the uterine or vaginal apical support. This is fully screened for colorectal dysfunction. Although
usually easier when the cervix is present, as it is a the vast majority of patients complain of constipa-
well-defined landmark of the vaginal apex. With tion, symptoms of fecal incontinence and/or rectal
straining, the descent of a well-supported uterus prolapse should be evaluated.
should not occur and is noted by an absence of cer-
vical descent. The descent of the cervix halfway to Urodynamic testing. Urodynamic testing should
the hymenal ring indicates a moderate loss of uter- be considered to rule out occult SUI. Occult inconti-
ine support. In posthysterectomy patients, defining nence is urinary leakage demonstrated with
the true vaginal apex can be more difficult. The cuff increased intraabdominal pressure when vaginal

358 EDUCATIONAL REVIEW MANUAL IN UROLOGY


prolapse is reduced in a woman who is otherwise provide functional information regarding the effi-
continent. Patients with a loss of level I support gen- ciency of rectal emptying. However, correlation of
erally have greater degrees of prolapse warranting patient symptoms and clinical outcomes has not
preoperative urodynamic assessment. The urody- been correlated to these studies.
namic studies should be performed with a vaginal
packing or pessary. In addition to leak point pres- In very severe prolapse or recurrent cases, it may be
sure assessment, these studies may provide useful desirable to understand clearly what structures are
information regarding the voiding characteristics in within the prolapsing bulge. A dynamic pelvic floor
these women. MRI can provide static and straining views of the
pelvic organs in several planes for better distinction
Cystoscopy. Although not mandatory in the evalua- of key support defects.
tion of prolapse and incontinence, cystoscopy
should be considered in women who have had pre-
vious bladder suspension or prolapse repair. Cys-
Surgical Correction of Pelvic

toscopy should be performed in women with signif-


Organ Prolapse

icant irritative storage symptoms, hematuria or Anterior Defects (Cystocele)


recurrent urinary tract infections. A central defect cystocele is surgically repaired by
the reduction of the prolapsing bladder and reap-
Radiographic Imaging. Imaging is not a standard proximation of the attenuated pubocervical fascia
part of the evaluation of pelvic support, but it can be using plicating sutures (Figure 3).
a helpful adjunct. A voiding cystourethrogram may
be helpful to differentiate apical herniation as blad- This operation, the anterior colporrhaphy, is the
der and/or enterocele. Also, the degree of bladder most frequently utilized procedure in the correction
descensus may be determined. Defecography (defe- of cystocele, and is associated with success rates
cating proctography) can aid in detection and quan- from 30%–75%. The anterior colporrhaphy only
tification of rectoceles. Additionally, this study can corrects central defects, and the failure to identify
Figure 3

Reapproximation of pubocervical fascia: anterior colporrhaphy –


repair of central cystocele

Reprinted with permission: AUA Update Series. Volume 25, Lesson 29; 2006.

CHAPTER 12: FEMALE UROLOGY AND URINARY INCONTINENCE 359


and correct concomitant pelvic organ prolapse, par- the rectovaginal fascia (which is intact) to the per-
ticularly apical or level I prolapse, that commonly ineal body. This detachment can be responsible for
present with cystocele is probably a main risk factor large distal rectoceles that can become symp-
in failures of anterior repairs. tomatic. This can be accomplished through a small,
distal transverse or diamond-type incision. Success
A lateral defect cystocele is corrected by the reap- rates of 66%–75% have been recorded, with the
proximation of the vagina to the pelvic sidewalls. major complications being pain and/or sexual dys-
This operation is completed by placing a row of function. Rates of dyspareunia are lower with a site-
interrupted sutures from the vagina into the pelvic specific repair in comparison to a full plication of
sidewall at the arcus tendineus, extending all the rectovaginal septum.
way to the ischial spine (Figure 4). This repair is
known as a paravaginal repair and can be performed Enterocele
abdominally, laparoscopically or transvaginally. The most common form of enterocele repair is that
When one performs this repair through the vagina, a of transvaginal sac isolation and closure. The ente-
graft material is usually secured to each pelvic side- rocele sac is isolated after making an incision near
wall at the ATFP. The graft traverses the pelvis (or through) the cuff of the vagina. After the sac is
under the bladder. This provides support to the cen- dissected out, the bowel contents are reduced, and
tral cystocele component as well. Graft material the sac is closed proximally near the “neck” of the
commonly used is porcine dermis or polypropylene sac. Following this the remaining sac is dissected
mesh. Utilizing these techniques, the success rate of out and discarded. The enterocele can also be
these procedures is reported from 80%–95%. There repaired transabdominally. The bowel is manually
is a higher incidence of complications from the lifted out of the pelvis, and a culdoplasty is per-
vaginal paravaginal repair when compared to the formed by closing of the cul-de-sac with permanent,
anterior colporrhaphy. interrupted sutures. The success rates of these pro-
cedures are approximately 85%–95%.
A transverse cystocele occurs when the pubocervi-
cal attachments separate from the level I (cardinal Apical Prolapse of the Vaginal Cuff
and uterosacral) support which stabilizes the apex There are a wide number of procedures to correct
of the vagina. In an isolated transverse defect, the apical vaginal prolapse. The importance of recog-
cystocele is repaired simply by restoring this sup- nizing and correcting apical defects is becoming
port. This is most commonly performed by reestab- increasingly stressed by vaginal surgeons. This is
lishing cuff support. This explains why many the means by which the upper vagina is stabilized
women with cystocele may have correction after an proximally, and this protects against widening of the
abdominal colpopexy or uterosacral cuff suspen- genital hiatus. There are a number of abdominal,
sion. vaginal and laparoscopic approaches to the correc-
tion of apical prolapse, and the surgeon should be
Posterior Defects (Rectocele) well versed to perform several of these procedures.
A rectocele is repaired by reapproximating the rec- The sacrospinous ligament fixation (SSLF)
tovaginal fascia, usually with interrupted sutures in achieves a functional vagina and vault prolapse cure
an operation called a posterior colporrhaphy (Figure in 67%–79% of patients. One should place the
5). The operation extends distally toward the per- sutures directly into the body of the sacrospinous
ineal body, and involves incorporating levator fas- ligament as there is a potential for significant vascu-
cia into the repair distally. In fact, many posterior lar or neurologic injury. There is a posterior dis-
repairs involve plication and reinforcement of the placement and elongation of the vaginal apex after
perineal body. One must take great care not to nar- SSLF, which may predispose one to anterior com-
row the vagina excessively, as this may cause sexual partment defects. The apex can be sutured directly
dysfunction. Graft materials can be incorporated in to the iliococcygeus fascia, or to the uterosacral lig-
this repair, and many have adopted the use of grafts, ament remnants usually isolated via a transvaginal,
particularly for repeat repairs. A site-specific recto- intraperitoneal approach. Ureteral patency must be
cele repair is accomplished by reapproximation of assured after these procedures, and this can be

360 EDUCATIONAL REVIEW MANUAL IN UROLOGY


accomplished simply by examining for efflux after
the IV administration of indigo carmine.
Figure 4

The abdominal sacral colpopexy is one of the most


Reapproximation of lateral defect:

successful ways to correct apical prolapse—it is the


correction of paravaginal defect

gold standard vault prolapse repair. This procedure


is accomplished via abdominal, laparoscopic or
robotic approaches. A graft is utilized to secure the
apex of the vagina to the sacrum. Most authors now
fix the graft material to the sacral promontory. A
synthetic mesh should be utilized, as there is level I
evidence demonstrating the superiority of perma-
nent mesh materials to biologic graft materials in
women undergoing colpopexy. Success rates as
high as 95% have been reported by multiple
authors, with an acceptably low graft erosion rate.
Complications can include serious bleeding
encountered from the presacral venous complex.

In women who are no longer sexually active and too


frail or ill to undergo a larger reconstruction,
colpocleisis is a reasonable option. In colpocleisis,
the vagina is closed precluding future intercourse.
This can be accomplished even with a uterus in
Reprinted with permission: AUA Update Series.

place. It is a fairly quick procedure with fewer risks


Lesson 23, Volume 21; 2002.

involved than with the larger reconstructions. Figure 5

Plication of pararectal fascia,


posterior colporrhaphy

Reprinted with permission: AUA Update Series.


Lesson 23, Volume 21; 2002.

CHAPTER 12: FEMALE UROLOGY AND URINARY INCONTINENCE 361


3. Vesicovaginal Fistulae

cuff may not allow access to the fistula transvagi-


nally. It is also important to assess tissue quality for
Etiology

In underdeveloped countries, birth trauma from pro- the presence of inflammation or edema. This is
longed engagement of the fetal head during pro- vitally important when timing repair.
tracted labor is the most common cause of fistula. In
developed countries, fistula formation after hys-
terectomy is the most common cause, occurring in
Cystogram

approximately 1 per 1,800 cases. This may occur as A cystogram may be helpful in revealing an unrec-
a result of suture placement into the bladder with ognized bladder injury or fistula. A cystogram
resultant tissue necrosis, or an undetected bladder offers little value in determination of fistula location
injury with resultant urine leak through the vaginal or size. This study usually does not provide addi-
cuff establishing a fistulous connection. Less com- tional information to other methods of diagnosis;
mon causes of VVF include: pelvic irradiation, therefore, it is not usually done unless complex
vaginal surgery, foreign body, trauma and urethral reconstruction is planned.
diverticulectomy.
Cystoscopy

Cystoscopy is essential in the diagnosis of VVF. A


Symptoms

The clinical presentation of VVF may vary depend- fistula should be suspected if mucosal changes, such
ing on the size and location of the fistula. Continu- as a dimple, band of scar tissue, or inflammation are
ous urine leakage or watery vaginal discharge 7–14 present. Placing a small ureteral catheter into the
days after a potentially causative surgical procedure fistula may assist in determining where the fistula is
is the most common presentation. The amount of located in the vagina. The proximity of the fistula to
leakage can be so severe that all of the bladder con- the ureteral orifices and the presence of other sus-
tents drain through the fistula, or minimal enough to pected fistula should be assessed cystoscopically.
drain only when the bladder is full or in certain An assessment of the tissue surrounding the fistula
patient positions. In order to make a timely diagno- should be examined to ensure that it is acceptable to
sis, one must be aware of the potential varying pre- incorporate in a repair. Serial cystoscopic examina-
senting symptoms. tions may be needed to assess tissue quality prior to
embarking on fistula repair.
Diagnosis

The essential step in diagnosis is suspicion. One


Dye Testing

must suspect the presence of fistula in order to If the diagnosis of VVF remains in doubt after phys-
detect it. A fistula should be suspected in the pres- ical examination and cystoscopy, various dye tests
ence of a watery vaginal discharge in any patient can be used to determine if a fistula is present. A
presenting for an incontinence evaluation. “double dye” test can be performed at the bedside.
The vagina is packed with 4 moist gauze pads— one
in the right vaginal fornix, and one in the left vaginal
fornix. A 3rd pad is placed in the mid-vagina, while
Physical Examination

A physical examination will detect the fistula in the 4th pad is placed at the vaginal opening. The
most cases. During the physical examination, sev- bladder is filled with 1% carmine solution (red), and
eral associated factors are important to address 10 mL of indigo carmine is injected intravenously.
when planning correction. Fistula size and location The swabs are removed 15 minutes after injection.
are important. Smaller fistulas may be approached A red stain on the mid-vaginal or upper forniceal
using more conservative measures. The location of packs is suspicious for VVF. A blue stain on the
the fistula in the vagina is imperative to identify. Is pack is suspicious for a ureterovaginal fistula, and a
the fistula located where transvaginal access may be red-stained pad only at the vaginal opening is
difficult? Fistulas are most commonly in the cuff of indicative of urethral leakage. A more simple diag-
the vagina. A small narrowed vaginal canal or fixed nosis can be obtained using oral phenazopyridine

362 EDUCATIONAL REVIEW MANUAL IN UROLOGY


until the urine is stained orange. The patient then The timing of the surgical repair should be individu-
inserts a tampon. The bladder is then filled with a alized. Each patient should be examined at frequent
saline/methylene blue solution. After 10 minutes, intervals to assess tissue integrity. The repair should
examination of the tampon is performed. An orange be performed only when the edema has subsided,
stain at the top of the tampon indicates a necrotic tissue disappeared, and the inflammatory
ureterovaginal fistula, and a blue stain suggests the response has resolved. It is important to communi-
presence of a VVF. cate these principles to the patient. This will restore
confidence in the patient that the surgery will be per-
formed at a time that optimizes the chance of a suc-
cessful outcome. The best chance of success is the
Intravenous Pyelogram or Retrograde

first repair—it is much better to do the repair when


Pyelography

This is a mandatory test when evaluating VVF. The the tissue quality is best.
presence of a ureterovaginal fistula or obstruction
may be found in as many as 10%–15% of cases of Catheter Drainage
VVF. Before correction, the ureters must be cleared Small fistula or fistulae associated with a foreign
of any abnormality. In cases where the IVP is inde- body (suture) may respond well to a trial of catheter
terminate or the VVF is in close proximity to the drainage. If the foreign body can be successfully
ureter, retrograde pyelography should be consid- removed and a small fistulous tract persists, a trial
ered. of catheter drainage may result in resolution of the
fistula. For catheter drainage to be successful the
Urodynamics fistula should be small (few millimeters) and the tis-
Urodynamic studies are not routinely needed in the sue quality should be excellent. If a fistula will heal
evaluation of VVF unless the presence of detrusor with a catheter, there should be no leakage with the
instability or the competence of the urethral sphinc- catheter in place. If leakage across the fistula stops,
ter is in question. Urodynamic studies are not the fistulous tract may heal with a 4-week trial of
needed prior to fistula repair. continuous catheter drainage. If continuous
drainage is chosen, anticholinergic medication
CT or MRI should be given to decrease bladder spasm. If leak-
There are only a small number of case reports evalu- age does not stop within 1 week of the institution of
ating the use of these imaging modalities in the catheter drainage, it is unlikely that the fistula will
diagnosis and management of VVF. The usefulness close with drainage alone.
of these studies appears limited after physical and
endoscopic examination. At present, the authors
have not utilized these imaging modalities unless Fulguration of Fistulous Tract
other pelvic pathology is suspected. The theory behind fulguration of the fistulous tract
is that the epithelium covering the tract is denuded,
promoting closure of the tract. The fistula must be
quite small, and a low current is employed which is
Treatment of Vesicovaginal Fistulae

Timing of Repair carried out with a pediatric Bugbee electrode. A low


Traditionally, repair of fistula was delayed 3–6 current is utilized, and the fulguration is usually per-
months after the offending surgery to allow the formed through the bladder. There are several small
edema and inflammation to subside. This can be series documenting successful closure of fistula
quite difficult due to the anxiety of the patients. A with fulguration. One should not attempt fulgura-
number of studies have demonstrated that early tion of larger fistula. A potential adverse effect of
repair of VVF may be carried out. Early repair of fulguration is the destruction of the tissue around
VVF has been reported to have success rates from the fistula by cautery. This has the potential to actu-
90%–100% via abdominal or vaginal approaches. ally create a larger fistula defect.

CHAPTER 12: FEMALE UROLOGY AND URINARY INCONTINENCE 363


Vaginal Repair Abdominal Repair
The vaginal approach is preferred as the morbidity An abdominal approach should be considered in the
of this approach is much less. An abdominal inci- following circumstances: high retracted fistula, con-
sion is avoided along with opening (bivalving) the comitant ureteral pathology or proximity, multiple
bladder. Reduced bladder dissection results in less fistulae or postradiation fistulae. If the surgeon who
postoperative instability, and potentially shorter performs does not repair fistulae frequently, they
catheter time. may favor an abdominal approach. This is important
when choosing the route of repair. The surgeon
The vaginal repair is usually carried out in the dorsal should perform the procedure where he/she is likely
lithotomy position. Lateral vaginal incisions can to achieve a successful outcome. The principles of
provide better exposure if the vaginal capacity is the abdominal repair include closure of the fistula
small. An inverted lithotomy position may also be with watertight, multiple, non-overlapping suture
utilized to improve exposure of the vaginal cuff. lines under no tension. Additionally, the abdominal
The principles of transvaginal repair are to isolate approach affords access to the omentum which is
the fistula, and close the fistula with multiple non- used as interposition tissue. For large (several cen-
overlapping suture lines. Vaginal flaps are created timeters) fistulae or hard-to reach cuff fistulae, the
and the fistula is closed without excision of the fis- author has had significant success utilizing a trans-
tula tract. The sutures utilized to close the fistula abdominal approach.
should invert the fistula. A second layer, which is
placed perpendicular to the fistula, is then used to The abdominal approach begins with an incision in
further invert the closed fistula (Figure 6). The vagi- the midline of the bladder. This incision is carried
nal flap is then advanced over the area of fistula all the way posteriorly around the fistula, isolating it
repair and closed with a running suture. A Martius from the bladder. The bladder flaps are then mobi-
fat pad flap may then be incorporated to add an lized in order to minimize tension on the bladder
additional layer to cover the repair. These tech- closure. The fistula is closed in 2 layers. After fis-
niques are associated with 80%–100% success tula closure, omentum is tacked into position cover-
rates. ing the fistula, and the bladder is closed in 2 layers.
A suprapubic cystostomy provides large-bore
catheter drainage for 3–4 weeks (Figure 7). This

Figure 6

Transvaginal VVF fistula repair

Note: inverting fistula closure covered by vaginal flap.

364 EDUCATIONAL REVIEW MANUAL IN UROLOGY


Figure 7

Abdominal approach

Circumscribing the fistula.

Fistula closure with


omental interposition.

CHAPTER 12: FEMALE UROLOGY AND URINARY INCONTINENCE 365


4. Urethral and Periurethral
Masses

usually provides more than adequate exposure. Uti-


lizing abdominal approaches, success rates of
Differential Diagnosis

90%–100% are routinely achieved. • Vaginal leiomyoma

Partial Colpocleisis (Vaginal Cuff Closure) • Skene’s duct cyst


In many instances, the fistula is located in the cuff of
the vagina. Many times the tissue around the cuff is • Vaginal inclusion cyst
scarred and friable. Other small fistula may be pre-
sent or at risk to form after surgery. In some • Ectopic ureter/ureterocele
instances the fistula is so small that dye can be seen
leaking from a fold of scar in the cuff, but the defi- • Malignancy
nite fistula is not identified. It is for these reasons
that a partial colpocleisis (Latzko procedure) may • Urethral prolapse
be an attractive alternative. The vaginal epithelium
is denuded off of the cuff extending 3–4 cm on the • Urethral caruncle
anterior and posterior wall of the vagina. Linear
sutures are placed 1 cm apart to plicate the soft tis- • Urethral diverticulum
sue of the vaginal cuff together, thus closing off the
proximal 3–4 cm of the vagina. A second layer is • Vaginal wall cysts
used to reinforce the first suture line. The mucosal
margins are re-approximated with a running suture.
A catheter is left in for 3 weeks. One potential fear
Vaginal Leiomyoma

of this procedure is vaginal foreshortening, and Vaginal leiomyoma is a benign mesenchymal tumor
patients should be warned of this possibility. of the vaginal wall that arises from smooth muscle
elements. It presents usually as a smooth, firm mass
on the anterior vaginal wall. About 300 cases have
been reported—therefore, true incidence data are
Conclusion

Prompt diagnosis of vesicovaginal fistula is essen- unknown.


tial. The decision on timing of the surgical repair is
individualized, based on the quality of the tissues. These masses may be asymptomatic or cause local
Transvaginal approaches are most commonly symptoms related to the mass effect, such as
employed achieving excellent success rates, but obstruction, pain and dyspareunia.
similar success rates are achieved using abdominal
approaches. The abdominal approach should be These masses are estrogen-dependent and docu-
considered on all fistulae high in the vagina, which mentation of postmenopausal shrinkage has
may be difficult to reach via a vaginal approach. occurred.
The major factor determining the route of repair
should be the experience of the surgeon. This will Excision via a transvaginal approach is curative.
ensure selection of the technique most likely to Local recurrences do not seem to be a problem.
achieve the best outcome. Additionally, a benign diagnosis can be assured by
way of pathologic confirmation.

Skene’s Gland Cysts

Skene’s gland cysts present as small, cystic masses


lateral or inferolateral to the urethral meatus. These
lesions do not communicate with the urethral
lumen. A distal, lateral location which displaces the
urethral lumen should make one suspicious for a
Skene’s gland cyst. These are usually quite distal,

366 EDUCATIONAL REVIEW MANUAL IN UROLOGY


and displace the urethral meatus, in contrast to ure-
thral diverticula which are located in the mid- to
Urethral Mucosal Prolapse

proximal urethra. Urethral prolapse presents as a circumferential her-


niation or eversion of the urethral mucosa through
Symptoms can consist of local symptoms due to a the urethral meatus. This presents a red, engorged,
mass effect, or commonly as a result of abscess for- doughnut-shaped lesion that completely surrounds
mation, which is a common occurrence with larger the urethral meatus. The mass may be asymp-
Skene’s gland masses. These can become tomatic, or it may present with bleeding, pain or
exquisitely tender and actually may require marsu- local urinary symptoms.
pialization, aspiration or incision and drainage due
to extensive infection. In less complicated cases, These occur in 2 populations: postmenopausal
excision is the usual treatment. Although these women and prepubertal girls. It may occur in chil-
masses don’t communicate with the urethra, entry dren as a result of a deficiency of the muscular
into the distal urethra is not uncommon during exci- attachments of the urethral smooth muscle. This can
sion. be aggravated by Valsalva maneuvers or chronic
constipation. In postmenopausal women, an estro-
Adenocarcinoma has been reported, but is gen deficiency may aggravate this condition.
extremely rare.
Treatment may consist of topical application of
estrogen cream and warm baths to reduce inflam-
mation. Removal of urethral prolapse has been
Vaginal Wall Cysts

Vaginal wall cysts usually present as small asymp- accomplished by way of cauterization techniques,
tomatic masses on the anterior vaginal wall. These ligation around catheters and other means. How-
masses can enlarge and cause local symptoms as a ever, the preferred method is simple excision. Cir-
result of a mass effect. cumferential excision with suture reapproximation
of the urethral mucosa is usually easily accom-
Vaginal wall cysts can arise from multiple cell type plished, with or without an indwelling catheter.
origins:

• Mesonephric (Gartner’s duct cysts)—Found on


Urethral Caruncle

anterolateral vaginal wall as mesonephric rem- Urethral caruncle is an inflammatory condition of


nants. These patients should have upper tract eval- the distal urethra, which is most commonly diag-
uation. Up to 50% may have Müllerian duct nosed in postmenopausal women. The presentation
obstruction is usually that of a red, engorged, exophytic mass at
the urethral meatus, which is covered with mucosa.
• Paramesonephric (Müllerian)
These lesions may be etiologically related to ure-
• Endometriotic thral prolapse; however, this does not represent a
true prolapse of the distal urethra. Chronic inflam-
• Urothelial mation can be associated with hemorrhage, swelling
and edema, and this inflamed tissue may expand
• Epidermoid (inclusion cyst) from the urethral meatus due to already weakened
muscular and connective tissue.
Diagnosis is made after excision and pathologic
confirmation. Pathologic analysis of these lesions is usually con-
sistent with the chronic inflammatory nature.
Inflammation, hemorrhage and localized necrosis
are commonly seen.

CHAPTER 12: FEMALE UROLOGY AND URINARY INCONTINENCE 367


Treatment is usually conservative. Small asymp- images to plan for the surgical repair. For many, the
tomatic lesions may be observed, or treated with VCUG remains the initial imaging modality. The
topical creams, usually estrogen. Larger, symp- VCUG done under fluoroscopy is a generally reli-
tomatic lesions can be easily excised. Usually trac- able way to diagnose and localize diverticula. The
tion on the caruncle will expose the base, below sensitivity of this modality is reported between
which it can be excised. Reapproximation of the 49%–90%. This is an excellent study to screen for
mucosa can then follow with a fine, absorbable coexistent bladder pathology. One should remember
suture. to include straining films to assess the bladder neck.
An advantage of the VCUG is that it may give a
Urethral Diverticula visual confirmation as to the level of urethral origin.

A urethral diverticulum is an epithelial-lined, sac- Positive pressure urethrography is another imaging


like outpouching that arises from the urethral wall. modality with greater sensitivity (85%–100%). The
These diverticula are usually acquired, but in some advantages of this study such as excellent detail of
instances may be congenital. The reported inci- the urethra and diverticulum without the need for a
dence is between 1.4% and 5%, but the true inci- patient void seem to be outweighed by the fact that
dence is not known due to the fact that many small this is a cumbersome procedure for many to per-
diverticula are probably not detected. Most cases form, and it can be considerably uncomfortable for
are thought to be acquired. An infection or obstruc- the patient. Cystoscopy is usually performed in an
tion of the periurethral glands can lead to the forma- attempt to localize the ostium of the fistula, and also
tion of a periurethral cyst. These cysts gradually to look for other causes of LUTS. The ostia are usu-
enlarge, and then eventually rupture into the ure- ally seen in about two-thirds of cases. Transvaginal
thral wall, allowing urine to enter the cyst and pool ultrasonography has a reported >90% sensitivity in
within its confines. Epithelialization then follows, the detection of diverticula. These images provide
completing the transformation into a diverticulum. anatomic detail regarding the diverticulum and in
Stagnation of the urine in the diverticulum can lead many cases identification of the ostium. This is a
to an infectious process which can cause recurrent highly operator dependent imaging modality. MRI
UTI, stone formation, fistula development and, has taken on a considerable role in the imaging of
rarely, malignancy. diverticula. This permits a noninvasive, high resolu-
tion and multiplanar imaging study of diverticula.
The presentation of diverticula is variable. The clas- Diverticula appear as areas of decreased signal
sic triad of dysuria, dyspareunia and dribbling may intensity on T1 images when compared to the sur-
occur, but any female with a history of recurrent rounding tissues. They have high signal intensity on
UTI or dyspareunia should be suspected of having a T2 images. This modality has the best diagnostic
diverticulum. Other symptoms of a vaginal mass, accuracy, providing the best anatomic detail regard-
vaginal pain or even persistent LUTS may be asso- ing the relationship to the urethra, and whether there
ciated with a diverticulum. Many (20%–30%) of are any complicating factors: multiple, horseshoe
these women are asymptomatic. configuration, multiple ostia, etc. This can be a cost-
effective option if a limited urethral protocol is fol-
Most diverticula can be felt on physical examina- lowed.
tion. This is usually a suburethral mass which may
be tender. On compression of this mass, urine or Urodynamics should be strongly considered in
purulent fluid may be seen exiting the urethral mea- women with diverticula. When doing urodynamics,
tus. However, this finding is not present on many a fluoroscopic assessment should be considered to
examinations of diverticula. assist in localizing the source of leakage. Urody-
namics are indicated in women with incontinence,
Imaging can assist in the detection of diverticula. pelvic pain, urinary urgency and incomplete empty-
Today, it is important also to use imaging to localize ing. Ganabathi et al reported that only 37% of
and stage diverticula. As imaging modalities have women with diverticula had normal urodynamics,
become more sensitive, one should utilize these 49% had stress incontinence and 10% had detrusor

368 EDUCATIONAL REVIEW MANUAL IN UROLOGY


overactivity. These findings are important, as sev- ultrasonography has a reported >90% sensitivity in
eral authors have documented the safety and effi- the detection of diverticula. These images provide
cacy of performing a simultaneous anti-inconti- anatomic detail regarding the diverticulum and in
nence procedure with a diverticulectomy. many cases identification of the ostium. This is a
highly operator-dependent imaging modality. MRI
The operative approach is a transvaginal urethral has taken on a considerable role in the imaging of
diverticulectomy. This is generally done in 4 steps. diverticula. This permits a noninvasive, high resolu-
First, one creates an inverted, U-shaped vaginal tion and multiplanar imaging study of diverticula.
incision, creating a broad-based flap. Second, the Diverticula appear as areas of decreased signal
periurethral fascia is incised transversely. Proximal intensity on T1 images when compared to the sur-
and distal flaps of periurethral fascia are raised from rounding tissues. They have high signal intensity on
the diverticulum. The third step is isolation and T2 images. This modality has the best diagnostic
excision of the diverticulum. When possible, the accuracy, providing the best anatomic detail regard-
diverticulum should be isolated to the ostial urethral ing the relationship to the urethra, and whether there
attachment to ensure complete excision. Fourth, the are any complicating factors: multiple, horseshoe
urethra is closed longitudinally with a 4.0 configuration, multiple ostia, etc. This can be a cost-
absorbable suture, followed by a transverse reap- effective option if a limited urethral protocol is fol-
proximation of the periurethral fascia in order to lowed.
avoid overlapping suture lines. The vaginal flap is
advanced over the repair and closed. One may Urodynamics should be strongly considered in
safely perform an indicated anti-incontinence pro- women with diverticula. When doing urodynamics,
cedure in conjunction with a urethral diverticulec- a fluoroscopic assessment should be considered to
tomy. In this population, a synthetic sling is con- assist in localizing the source of leakage. Urody-
traindicated, and autologous pubovaginal slings are namics are indicated in women with incontinence,
preferred. After the fascia is harvested and a supra- pelvic pain, urinary urgency and incomplete empty-
pubic tube is placed, the diverticulectomy proceeds ing. Ganabathi et al reported that only 37% of
in the manner as described above. After dissecting women with diverticula had normal urodynamics,
out the diverticular sac, the retropubic space is 49% had stress incontinence and 10% had detrusor
entered after removing any infectious material. The overactivity. These findings are important, as sev-
retropubic space is entered, and the sling sutures are eral authors have documented the safety and effi-
placed into the retropubic space prior to closing the cacy of performing a simultaneous anti-inconti-
urethral defect. After closing the urethral defect, the nence procedure with a diverticulectomy.
periurethral fascia is then closed. The sling is then
loosely positioned under the proximal urethra after
the periurethral fascia is closed. Placement of a
labial fat pad can be done in cases of diverticulec-
tomy where the tissues are thin, and poor healing is
of concern.

Positive pressure urethrography is another imaging


modality with greater sensitivity (85%–100%). The
advantages of this study such as excellent detail of
the urethra and diverticulum without the need for a
patient void seem to be outweighed by the fact that
this is a cumbersome procedure for many to per-
form, and it can be considerably uncomfortable for
the patient. Cystoscopy is usually performed in an
attempt to localize the ostium of the fistula, and also
to look for other causes of LUTS. The ostia are usu-
ally seen in about two-thirds of cases. Transvaginal

CHAPTER 12: FEMALE UROLOGY AND URINARY INCONTINENCE 369


5. Overactive Bladder

smooth muscle. In this theory, OAB and UI result


from changes within the smooth muscle of the blad-
Definition

OAB is urgency, with or without urgency inconti- der wall.


nence, usually with increased daytime frequency
and nocturia. The definition is based on a clinical The rationale for this theory is that there is some
diagnosis only. The majority of patients with OAB spontaneous contractile activity in strips of detrusor
and urge incontinence have no identifiable underly- in all species that originates from the muscle itself.
ing etiology. In fact, most cases of OAB are idio- It is uncoordinated, so this does not lead to a coordi-
pathic. nated detrusor contraction and it is not perceptible
by the patient or urodynamically. In the neurogenic
Urgency is a sudden compelling desire to pass urine theory OAB and UI are secondary to changes within
that is difficult to defer; urge urinary incontinence, the central nervous system (CNS) and peripheral
involuntary leakage accompanied by or immedi- nervous system (PNS). CNS mechanisms control-
ately preceded by urgency; frequency: patient con- ling lower urinary tract function are organized in
siders that he /she voids too often in the day brain and SC as simple on-off switches under volun-
(>8times in a 24-hour period); nocturia, where the tary control. Any damage causing loss of peripheral
patient wakes up >2 times per night to void. OAB is inhibition or increased lower urinary tract afferent
not synonymous with the urodynamic observation input can enhance excitatory neurotransmission in
of detrusor overactivity (DO), but may be present the micturition pathway. For example, Parkinson’s,
on cystometry. During urodynamic studies overac- MS, or CVA can reduce suprapontine inhibition.
tive bladder activity may demonstrate several dif-
ferent manifestations: involuntary detrusor contrac- Spinal cord injury can damage axonal pathways and
tions, abnormal compliance, sensory urgency and a allow emergence of primitive spinal bladder path-
reduction in the cystometric capacity. ways.

There are a number of references to detrusor overac-


tivity with which one must be familiar. Neurogenic
Neuroanatomy Background

detrusor overactivity (formerly detrusor hyper- Bladder


reflexia) is the presence of overactive bladder in The bladder consists of smooth muscle, which
patients with known neurologic conditions. Idio- accommodates increasing volumes of urine without
pathic detrusor overactivity (formerly detrusor increasing its storage pressure. This process of blad-
instability) is the presence of overactive bladder der compliance is a unique ability of the urinary
activity in a neurologically intact individual without bladder facilitating storage of urine. The bladder has
a reversible identifying cause. Wet overactive blad- a uniform contraction of sufficient magnitude after
der is the presence of urinary incontinence as a sphincter relaxation, which is responsible for
result of overactive bladder. Dry overactive bladder expelling urine. The bladder smooth muscle is
consists of OAB symptoms (urgency and/or fre- innervated by the pelvic nerve arising from the
quency) in absence of incontinence. This represents parasympathetic nervous system arising from sacral
approximately two-thirds of OAB cases. segments S2-S4. This nerve releases the neurotrans-
mitter acetylcholine. Stimulation of this nerve leads
There are identifiable, reversible conditions which to contraction of bladder via promoting contraction
can cause symptoms of OAB: infection, stones, of the smooth muscle. (Thus, anticholinergic medi-
tumor (CIS) and neurologic disease. Treatment of cations promote bladder relaxation by inhibiting
these conditions is usually met with improvement of this process.)
symptoms.
Normal bladder function depends on the parasym-
Although in idiopathic OAB, the etiology is not pathetic nervous system and Ach stimulation of
known, the mechanism is likely either neurogenic or muscarinic receptors. The M3 muscarinic receptor
myogenic. The myogenic theory of the etiology of subtype is directly responsible for detrusor contrac-
OAB is supported by observations in bladder tion. Many pathologic conditions can affect bladder

370 EDUCATIONAL REVIEW MANUAL IN UROLOGY


function by altering normal muscarinic receptor implies a potential significance to urothelial (affer-
number and function. Normal bladder function ent) activity in pathologic conditions of the blad-
depends on the parasympathetic nervous system and der—OAB, IC, NGB, etc. This is a major emphasis
Ach stimulation of muscarinic receptors. The M3 of current research.
muscarinic receptor subtype is directly responsible
for detrusor contraction. Many pathologic condi-
tions can affect bladder function by altering normal
Bladder Neck/Proximal Urethra

muscarinic receptor number and function. The bladder neck and proximal urethra consist of
smooth muscle abundantly innervated by the sym-
There are 2 types of cholinergic receptors: nicotinic pathetic nervous system. The hypogastric nerve
and muscarinic. Within the bladder, these receptors which arises from the sympathetic nervous system
have been located in detrusor smooth muscle, in the arising from segments T10- L2 releases nore-
urothelium, and on the parasympathetic and sympa- pinephrine, causing stimulation of the alpha recep-
thetic nerve terminals. Five muscarinic receptors, tors. This activity promotes contraction of these
glycoproteins designated M1–M5, have been muscles, causing bladder neck closure and closure
molecularly cloned and described. The large num- of the proximal urethra. This is the major contribu-
ber of M2 receptors begs the question of their pur- tor of passive continence and is commonly referred
pose. Thus far, M2 receptors are thought to play to as the internal sphincter. The pelvic floor striated
only a minor role in smooth muscle contraction complex which surrounds the urethra as it exits the
despite their prevalence, but there is evidence to pelvic floor comprises the external sphincter com-
suggest that M2 may be an active participant in plex. This is a voluntary striated muscle, and relax-
detrusor contraction in certain pathologic states. ation of the pelvic floor is essential for efficient
Muscarinic receptors also have been identified on bladder emptying. Dysfunction of the striated mus-
parasympathetic and sympathetic nerve endings, cle complex manifests commonly as dysfunctional
regulating Ach and noradrenaline release, respec- voiding or detrusor sphincter dyssynergia (in
tively. In the bladder, inhibition and facilitation by patients with neurogenic bladder). The pudendal
presynaptic muscarinic receptors are observed. M1 nerve, a somatic nerve arising from sacral segments
receptors at prejunctional sites on cholinergic nerve S2-S4 stimulates activity of the pelvic floor striated
terminals in the bladder are facilitatory. Activation muscle.
of these receptors facilitates Ach release during pro-
longed high-frequency nerve firing (e.g., during Thus, during urine storage, the sympathetic activity
voiding). Conversely, inhibition resulting in a promotes increased resistance of the bladder neck
reduction in the release of Ach occurs through M2 and proximal urethra via the hypogastric nerve, and
or M4 receptors. Stimulation of muscarinic recep- the external sphincter activity increases via puden-
tors outside the urinary tract is responsible for many dal nerve activity. Increased EMG activity near the
of the bothersome side effects of antimuscarinic end of urinary storage is normal and considered the
medications. Ach stimulation of muscarinic recep- guarding reflex.
tors in submandibular acinar cells activates cal-
cium-dependent ion transport pathways responsible
for the secretion of saliva. Central muscarinic recep-
Micturition

tors, particularly M1, are involved in higher cogni- Urine storage is a largely sympathetic function. The
tive processes of learning and memory. Central M1 voiding cycle is dependent on synergistic outlet
antagonism may lead to cognitive dysfunction and reaction and bladder contraction. This activity is
other CNS-related adverse events. regulated via the pontine micturition center located
in the pons. Upper cortical centers in the brain con-
The bladder also contains beta receptors in bladder trols when voiding will occur, mainly by the
body, which, when stimulated, promotes bladder inhibitory effect on the pontine micturition center.
relaxation. In addition, a number of nonadrenergic, The pontine micturition center serves as the “on-
noncholinergic receptors (NANC) have been identi- off” switch to activate the voiding cycle. During
fied within the urothelial lining of the bladder. This storage, sympathetic activity promotes urinary stor-

CHAPTER 12: FEMALE UROLOGY AND URINARY INCONTINENCE 371


age by contracting the bladder neck and proximal There does not appear to be a clear efficacy differ-
urethra via the hypogastric nerve. Additionally, ence when comparing these medications. One must
stimulation of the beta receptors in the bladder body be aware of the common anticholinergic adverse
also promotes urine storage. When inhibition events (dry mouth and constipation are the most
ceases, the pontine micturition center promotes an common) and warn patients of these. There are data
inhibition of this sympathetic activity and stimula- to suggest over time many patients stop taking these
tion of the pelvic nerve (parasympathetic nervous medications. Whether it is a lack of efficacy, side
system). This results in bladder contraction in the effects, expense or a combination of all 3 is not
presence of relaxed sphincter complex. known. The concept of muscarinic selectivity
improving tolerability is unproven in humans.
In combination with the conservative measures Antimuscarinics for the treatment of UUI are stan-
above, medications are also efficacious in the man- dard first-line treatment in patients with significant
agement of urinary incontinence. When treating bother from the condition. The rationale for their
incontinence (failure to store), drugs which promote use is that muscarinic receptor subtype M3 is
bladder storage or increase urethral resistance are responsible for detrusor contraction and M2 may
desirable. The most commonly utilized drugs are also play a role, particularly in some pathologic
antimuscarinic medications which promote urinary conditions. All currently available antimuscarinics
storage by blocking the uptake of acetylcholine at have a significant effect at the M3 receptor and var-
the muscarinic receptor level in the bladder. This ied degrees of effect at other muscarinic receptors
has been shown to decrease tone and contractility of subtypes. None of the available agents are com-
the bladder. There are a number of widely utilized pletely uroselective; therefore, these medications
antimuscarinic (or anticholinergic) medications: have side effects caused by their action on mus-
carinic receptors outside of the urinary tract. The
Oxybutynin gel 10% (GelniqueTM) most common side effects of dry mouth, constipa-
1 packet applied transdermal QD tion, and blurry vision are due to the effect of the
drug at the salivary gland, bowel and ciliary muscle,
Oxybutynin IR (gen Ditropan®) respectively.
2.5, 5.0 mg BID QID
NOTE: Anticipate release of AUA Guidelines
Oxybutynin ER (Ditropan XL®) for the Treatment of Idiopathic OAB at annual
5, 30 mg QD meeting 2012

Oxybutynin TDS (Oxytrol®) Surgical Treatment of Overactive Bladder


1 patch twice weekly
A. Neuromodulation. Stimulation of S3 nerve roots
Tolterodine IR/ER (Detrol® LA) modulates neural activity to bladder (Interstim)
2, 4 mg QD
1. Indicated for refractory OAB (urge
Trospium (Sanctura®, Sanctura® XR) incontinence or frequency)
20 mg BID, 60 mg QD
2. Indicated for nonobstructed urinary
Fesoterodine (ToviazTM) retention.
4 mg QD/, 8 mg QD
3. No current FDA indication in neurogenic
Solifenacin (Vesicare )®
bladder condition
5, 10 mg QD
4. Current routes of stimulation
Darifenacin (Enablex®)
7.5, 15 mg QD a. Sacral Nerve Stimulation, InterStim®

372 EDUCATIONAL REVIEW MANUAL IN UROLOGY


5. Suggested Reading

b. Posterior Tibial, Urgent® PC 1. AUA Guideline for the Surgical Management


of Female Stress Urinary Incontinence: 2009
B. Botulinum toxin: FDA approved for neurogenic Update.
detrusor overactivity
2. DeLancey JO, Richardson AC. Anatomy of
1. Prevents release and fusion of acetylcholine genital support. In: Urogynecologic Surgery.
Gaithersburg, MD: Aspen; 1992.
2. The PI describes 30 injections into detrusor
muscle: 200 units 3. Herbst AL, Mishell DR, Stenchever MA,
Droegemueller W. Disorders of the abdominal
3. Excellent results in randomized controlled wall and pelvic support. In: Stenchever MA, ed.
trial in neurogenic detrusor overactivity Comprehensive Gynecology. 2nd ed. Philadel-
phia, PA: Mosby Yearbook; 1992:594-612.
4. 30% risk of urinary retention in neurogenic
detrusor overactivity 4. Iselin CE, Aslan P, Webster GD. Transvaginal
repair of vesicovaginal fistulas after hysterec-
5. 42–48 week durability in studies tomy by vaginal cuff excision. J Urol.
1998;160:728-730.
C. Augmentation cystoplasty
5. Karram MM, Miklos JR, Sze EH. Pelvic pro-
1. Bowel augmentation lapse and lower urinary tract dysfunction in
females. In: O’Donnell P, ed. Urinary Inconti-
a. ≥20% of patients who void preop will nence. 1st ed. Philadelphia, PA: Mosby Year-
need CIC postop book; 1997:302.

2. Detrusor myomectomy 6. Käser O. The Latzko operation for vesico-vagi-


nal fistulae. Acta Obstet Gynecol Scand.
D. Denervation: neurectomy, rhizotomy or phenol 1977;56:427-429.
injections
7. Leach GE, Trockman BA. Surgery for fistulas
1. Rarely indicated and diverticulum. In: Walsh PC, Retik AB,
Vaughan ED Jr, Wein AJ, eds. Campbell’s
E. Future technologies in the treatment of Urology. 7th ed. Philadelphia, PA: WB Saun-
overactive bladder ders Co; 1998:1146-1147.

1. New drugs working on different sites of neu- 8. Miller EA, Webster GD. Current management
rohormonal axis of vesicovaginal fistulae. Curr Opin Urol.
2001;11:417-421.
2. Intravesical delivery systems
9. Rashid H, Cos L. Urethral diverticula in the
3. Bladder instillations adult female. In: Resnick E, Elder J, Spirnak J,
eds. Critical Decisions in Urology. 3rd ed.
4. Peripheral neuromodulation London, England. BC Decker; 2004:302-307.

10. Leach G, Raz S. Vaginal flap technique: A


method of transvaginal vesicovaginal fistula
repair. In: Raz S, ed. Female Urology. Philadel-
phia, PA: WB Saunders; 1983:372-377.

CHAPTER 12: FEMALE UROLOGY AND URINARY INCONTINENCE 373


6. Questions

11. Rovner ES. Bladder and urethral diverticula. In: 1. Four days after laparoscopic-assisted vaginal
Wein A, Kavoussi L, Novick A, Partin A, hysterectomy, a 55-year-old presents to the ER
Peters C, eds. Campbell Walsh Urology. 9th ed. with fever, abdominal pain, hematuria and
Vol 3. Philadelphia, PA: WB Saunders; 2007: decreased urine output. A noncontrast CT scan
2361-2391. reveals an intraabdominal fluid collection and a
new finding of hydronephrosis. As the urologic
12. Winters JC, Rackley RR, Appell RA, Nitti V. consultant, the next step should be:
Miscellaneous female urologic conditions. In:
Nitti V, ed. Practical Urodynamics. Philadel- A. Nephrostomy tube
phia, PA: WB Saunders; 1998:219-230.
B. Cystoscopy with retrograde pyelo-
gram and possible stent

C. Foley placement

D. Paracentesis

E. Tampon test

2. A 45-year-old woman with SUI undergoes a


midurethral sling. At the time of surgery, a
trochar is found to have penetrated the urethra.
The best management is :

A. Place the sling and leave a catheter


for 1 week

B. Place a biologic graft between the


urethra and the sling

C. Repair the defect, leave a Foley and


abandon the sling in this setting

D. Change the plan to an autologous sling

E. Place a suprapubic tube for diversion


of the urine

374 EDUCATIONAL REVIEW MANUAL IN UROLOGY


3. A 55-year-old woman underwent a midurethral D. Rectal injury
sling for stress incontinence 2 weeks ago. She
now has urgency, frequency and urge inconti- E. Pelvic abscess
nence. Her urinalysis is normal. She has no
obstructive symptoms and pelvic US reveals a 6. All of the following are important surgical prin-
30cc PVR. The best next step is: ciples in fistula repair, except:

A. Reassurance A. Adequate bladder drainage


after surgery
B. Anticholinergics
B. Unidirectional attention to alignment
C. Sling lysis of all suture lines

D. Injection of a bulking agent C. Tension-free anastomosis

E. Cystoscopy D. Watertight closure and


adequate drainage
4. A previously continent 33-year-old woman
develops urinary incontinence after the birth of E. Consideration of interposition grafting
her second child 4 months ago. She does not
plan to have any more children. The most 7. A 65-year-old woman who underwent a
appropriate first management is: midurethral sling 2 years ago, complains of uri-
nary frequency, urgency and positional voiding.
A. Pelvic floor muscle exercises with or She must stand to urinate. Her symptoms began
without physical therapy after her surgery. She has tried anticholinergics
without success. Her PVR is 225cc by bladder
B. Midurethral sling ultrasound. Her urinalysis is normal. The best
next step is:
C. Trial of imipramine
A. Neuromodulation
D. Injection of a bulking agent
B. Intravesical Botox
E. Cystogram
C. Cystoscopy
5. On postop day 1 after sacrospinous ligament
fixation surgery and a sling, the patient com- D. Urodynamics
plains of bilateral perirectal pain. It is worse
when sitting but feels better when she is stand- E. MRI of the urethra
ing or walking. The most likely explanation for
this pain is:

A. Entrapment of sciatic nerve fibers

B. Inflammation in the coccygeus


muscles

C. Obturator nerve injury

CHAPTER 12: FEMALE UROLOGY AND URINARY INCONTINENCE 375


8. A 77-year-old woman with dementia and uri- 10. A 50-year-old woman underwent a transvagi-
nary frequency, urgency and urge incontinence nal mesh prolapse repair and synthetic sling 8
is prescribed oxybutynin. After 3 weeks on the months ago. She now complains of dysuria,
medication, she returns to the office with her urge incontinence and occasional gross hema-
daughter who complains that her mother seems turia. Urodynamics were performed and show
more confused. The most likely explanation: high-amplitude detrusor overactivity. The best
next step is:
A. Allergy to oxybutynin
A. Trial of anticholinergic
B. Drug interaction between oxybutynin
and other medications B. Suppressive antibiotics and
vaginal estrogen cream
C. The patient may have suffered a stroke
C. Pelvic MRI
D. Oxybutynin effect on muscarinic
receptors in the brain D. Cystoscopy

9. A 75-year-old woman complains of a vaginal E. Urethrolysis


bulge. She had a hysterectomy 30 years ago. On
examination, the bulge extends beyond the
introitus when she is examined standing. All are
appropriate surgical options to address the vault
prolapse except:

A. Colpocleisis

B. Sacrospinous ligament fixation

C. Abdominal sacrocolpopexy

D. Iliococcygeus fixation

E. Anterior colporrhaphy

376 EDUCATIONAL REVIEW MANUAL IN UROLOGY


8. D.
Oxybutynin’s effect on muscarinic receptors in the
Answers

1. B. brain is a well-known cause of confusion and mem-


This patient’s presentation suggests a right ureteral ory loss particularly in the elderly.
injury. The most direct approach for diagnosis and
potential treatment is a cystoscopy, retrograde pyel- 9. E.
ogram and ureteral stent. Given the description of her vaginal exam, this
patient has a component of vault prolapse. An ante-
2. C. rior colporrhaphy alone will not address the vault
In the case of an urethral injury, a synthetic sling prolapse and recurrence will be likely.
should not be placed at the time of the repair due to
the higher risk of erosion. The sling should be aban- 10. D.
doned until after a suitable time of healing. The urologist must have a high index of suspicion
for a foreign body in the urinary tract given her his-
3. A. tory of surgery and hematuria. A cystoscopy is indi-
De novo bladder overactivity is a bothersome com- cated at this time.
plication of sling surgery. The incidence varies from
5%–10% but the majority of these cases will resolve
without intervention within the first 3 months after
surgery. Since this patient is only 2 weeks out from
surgery, reassurance and education is the best next
step.

4. A.
Since this patient is only 4 months postpartum, her
SUI may continue to improve. Surgical intervention
this early after childbirth is not the best step. The
patient should be encouraged to do pelvic floor
muscle exercises before committing to surgery.

5. B.
The sacrospinous ligament sits beneath the coc-
cygeus muscle. Sutures placed in the ligament often
cause inflammation and tenderness in the coccygeus
muscles causing perirectal/perianal pain for several
weeks.

6. B.
Overlapping suture lines are to be avoided in fistula
surgery.

7. C.
Imipramine has both a strong inhibitory action on
bladder smooth muscle and a stimulant effect on the
bladder outlet. The net result is it promotes urinary
storage by preventing DI and increasing urethral
resistance.

CHAPTER 12: FEMALE UROLOGY AND URINARY INCONTINENCE 377


378 EDUCATIONAL REVIEW MANUAL IN UROLOGY
Chapter 13:
Medical Management
of Urolithiasis
Michael E. Lipkin, MD
Agnes J. Wang, MD
Glenn M. Preminger, MD

Contents

1. Goals of Medical Management of Urolithiasis

2. Types of Urolithiasis and Clinical Associations

3. Diagnoses/Pathophysiology

4. Mechanisms of Stone Formation

a. Promotor Excess

b. Inhibitor Deficiency

c. General Urine Parameters

5. Metabolic Evaluation

6. Management Strategies

7. Pitfalls of Medical Stone Management

8. Questions

CHAPTER 13: MEDICAL MANAGEMENT OF UROLITHIASIS 379


1. Goals of Medical 2. Types of Urolithiasis and
Management of Urolithiasis Clinical Associations

A. Prevent new stone disease Table 1

B. Stabilize existing urolithiasis Types of Urolithiasis Based


on Stone Composition

a. Urinary tract obstruction


C. Prevent urologic complications of urolithiasis

b. Urinary tract infection


c. Renal compromise/end-stage renal disease
Calcium-based Non-calcium-based

i. Primary hyperoxaluria
ii. Hyperuricemia
Calcium oxalate Uric acid

iii. Staghorn renal calculi


monohydrate
Calcium oxalate Ammonium acid urate
dihydrate
Sodium urate
D. Prevent nonurologic complications Dihydroadenine

a. Osteodystrophy
of urolithiasis Xanthine

i. Absorptive hypercalciuria
ii. Renal tubular acidosis
Calcium phosphate Cystine

iii. Primary hyperoxaluria


b. Cardiomyopathy
Struvite

i. Primary hyperoxaluria
c. Retinopathy
Drug-induced

i. Primary hyperoxaluria
(indinavir, ephedrine,

d. Gouty arthritis
triamterene)

i. Hyperuricemia

E. Identify nonurologic conditions predisposing

a. Hyperparathyroidism
to urolithiasis

b. Gout
c. Sarcoidosis
d. GI malabsorptive conditions
e. Crohn’s disease/ulcerative colitis
f. Biliary disease
g. Pancreatitis
h. Bariatric surgery

380 EDUCATIONAL REVIEW MANUAL IN UROLOGY


3. Diagnoses/Pathophysiology

a. Anatomic
Table 2 A. Contributors to urolithiasis

i. Obstruction (UPJO, stricture, BPH)


ii. Renal anatomy
Stone Composition and

(medullary sponge kidney)


Possible Clinical Associations

iii. Foreign body (suture, staple, stent)


iv. Diverticulum (calyceal, bladder)
v. Hydronephrosis
Stone Analysis Possible Associations

vi. Urinary diversion/intestinal substitution


b. Functional
Calcium oxalate Hypercalciuria

i. Voiding dysfunction (neurogenic)


Hypercalcemia
Hyperoxaluria
Hypocitraturia
Gouty diathesis B. Overall approach to classification

a. Type of stones
Low urine volumes of urolithiasis

i. Calcium-based
ii. Non–calcium-based
Ca phosphate Distal renal

b. Predisposing factors
tubular acidosis

i. Promoter excess
Hyperparathyroidism

ii. Inhibitor deficiency


Low urine volumes

iii. General urine parameters


UTI

Uric acid Gouty diathesis

a. Promoter excess
Ammonium acid urate Gouty arthritis C. Mechanisms of stone formation

i. Hypercalciuria
Sodium urate Gouty nephropathy

ii. Hyperuricosuria
Dihydroadenine Gouty tophi

iii. Hyperoxaluria
Xanthine Hyperuricosuria

iv. Cystinuria
Hyperuricemia

v. Drug-induced stones
Obesity

b. Inhibitor deficiency
Inborn errors of

i. Hypocitraturia
metabolism (e.g., Lesch-

ii. Hypomagnesuria
Nyhan disease)

c. General urine parameters


Myeloproliferative disorder

i. Urine acidity (gouty diathesis)


Tumor lysis syndrome

ii. Low urine volumes


ETOH abuse

Cystine Cystinuria iii. Urinary tract infections

Struvite Urinary tract infection

CHAPTER 13: MEDICAL MANAGEMENT OF UROLITHIASIS 381


4a. Mechanisms of Stone Forma-
tion – Promotor Excess

a. Pathophysiology
a. Classification i. Increased absorption of Ca
A. Hypercalciuria

i. Hypercalcemic from GI tract


ii. Normocalcemic ii. Independent of dietary Ca
b. Pathophysiology content
i. Hypercalcemic iii. Decreased dietary Ca →
1. Increased serum calcium → negligible effect on urine
increased filtered calcium → hyper- Ca
calciuria iv. Increased absorption of Ca
ii. Normocalcemic → negative feedback →
c. Most common cause decreased serum PTH
i. Hypercalcemic: primary 2. Type II
hyperparathyroidism a. Pathophysiology
ii. Normocalcemic: absorptive i. Increased absorption of Ca
hypercalciuria from GI tract
d. Hypercalcemic hypercalciuria ii. Dependent on dietary
i. Most common cause —1° hyperparathy- Ca content
roidism iii. Decreased dietary Ca →
1. Parathyroid adenoma normalization of urine Ca
2. Parathyroid hyperplasia iv. A less severe form of
ii. Effects of increased PTH AH Type I
1. Increase in Ca absorption from v. Because of the increased
GI tract absorption of Ca → nega-
2. Increase in Ca absorption tive feedback → decreased
from kidney serum PTH
3. Increase in Ca absorption from bone ii. Renal hypercalciuria
4. Increase 1,25 OH-Vitamin D pro- 1. Pathophysiology
duction from kidney → increased a. Increased loss of Ca from kid-
absorption from GI tract ney (distal tubule)
5. Decreased phosphate resorption b. Loss of Ca → increased
from kidney serum PTH
e. Normocalcemic hypercalciuria c. Increased serum PTH →
i. Absorptive increased GI Ca absorption and
1. Increased absorption/reabsorption of increased bone Ca resorption (to
Ca from GI tract/bone/kidney → keep serum Ca near normal)
increased filtered calcium (to keep 2. Net effect
serum Ca near normal) → hypercal- a. Normal serum Ca
ciuria b. Increased urine Ca
2. Types I, II, III 3. Most common form of hypercalci-
ii. Renal uria in pediatric population
1. Distal renal tubule “leaks” calcium iii. Resorptive hypercalciuria
into urine → hypercalciuria 1. Pathophysiology
iii. Resorptive a. Increased serum PTH (mild)
1. Increased bone demineralization → i. → increased GI Ca
increased filtered Ca (to keep serum absorption
Ca near normal) → hypercalciuria ii. → increased bone Ca
iv. Idiopathic resorption
f. Absorptive hypercalciuria (AH) b. PTH levels not high enough to
i. Classification cause hypercalcemia
1. Type I

382 EDUCATIONAL REVIEW MANUAL IN UROLOGY


c. Most of the increased Ca in the 2. Hypoxanthine → xanthine → uric
serum is filtered into the urine acid
i. → serum Ca near normal 3. Same enzyme catalyzes both steps
d. Net effect iv. pKa uric acid = 5.5
i. Near normal serum Ca 1. Thus the insoluble uric acid tends to
ii. Increased urine Ca precipitate out of solution in acidic
urine
B. Hyperuricosuria 2. Low urine pH is the primary risk
(Hyperuricosuric CaOx nephrolithiasis) factor for uric acid stones
a. How does high urinary uric acid contribute to 3. Urine pH above 5.5 favors formation
calcium-based urolithiasis? of CaOx stones, even in the presence
i. Formation of small uric acid crystals act of significant hyperuricosuria
as nidus for calcium-based stones—so- d. Risk factors for hyperuricosuria
called heterogenous nucleation i. High dietary purine intake,e.g.,
ii. Sodium acid urate may inhibit the effec- animal protein
tiveness of naturally occurring inhibitors 1. Includes red meat, chicken and fish
of stone formation, e.g., glycosamino- ii. Heritable disorders of purine synthesis
glycans 1. Lesch-Nyhan disease
b. Classification 2. Overactivity of PRPP (= PRS =
i. Hyperuricemic hyperuricosuria phosphoribosyl pyrophosphate syn-
1. Serum uric acid increased thetase)
ii. Normouricemic hyperuricosuria 3. Adenosine phosphoribosyl
1. Serum uric acid normal transferase (APRT) deficiency
c. Uric acid iii. Myeloproliferative disorders
i. Byproduct of purine metabolism 1. Secondary to high cellular turnover
ii. Precursors: hypoxanthine, xanthine → increased purine metabolism
1. Both more soluble than uric acid iv. Tumor lysis syndrome
iii. Enzyme involved 1. Significant tumor cell death during
1. Xanthine oxidase therapy, usually chemotherapy

Table 3

Comparison of Metabolic Findings in Absorptive Hypercalciuria Types I – III

Absorptive Renal Resorptive

Serum calcium Normal Normal High


Normal/increased

PTH levels Decreased (secondarily) Increased (primarily) Increased

Fasting urinary calcium Type I: Increased Increased Increased


Type II: Normal

Intestinal calcium Increased (primarily) Increased (secondarily) Increased (secondarily)


absorption

CHAPTER 13: MEDICAL MANAGEMENT OF UROLITHIASIS 383


2. 2° to high cellular turnover → i. Glyoxylate reductase
increased purine metabolism ii. D-glycerate dehydrogenase
v. Medications d. Cofactor
1. Probenecid i. NOT pyridoxine
2. Sulfinpyrazone v. Classic findings – 24-hour urine collec-
tion
C. Hyperoxaluria 1. Urinary oxalate >100 mg/day
a. Classification vi. Clinical manifestations: urologic
i. Primary 1. Urolithiasis
ii. Secondary a. Early onset
iii. Enteric b. Recurrent
iv. Idiopathic 2. Renal insufficiency
b. Primary a. Can be silent in 33%
i. Pathophysiology 3. End-stage renal disease
1. Increased hepatic production → vii. Clinical manifestations: non-urologic
increased serum oxalate → 1. Pathophysiology
increased filtered oxalate → hyper- a. Secondary to oxalate deposition
oxaluria in tissues
2. = 1° oxalosis 2. Heart
ii. Subtypes a. Cardiomyopathy
1. Type I b. Arrhythmia
2. Type II 3. Bones
iii. Type I a. Osteodystrophy
1. More common b. Joint pain
2. Pathophysiology 4. Eyes
a. Excess glyoxylate oxidized a. Retinopathy
to oxalate b. Maculopathy
b. Enzyme defect c. Secondary (metabolic hyperoxaluria)
i. AGT (alanine-glyoxylate i. Pathophysiology
aminotransferase) 1. Increased hepatic conversion of
c. Co-factor for AGT enzyme metabolites to oxalate → increased
i. Pyridoxine (= vitamin B6) serum oxalate → increased filtered
ii. One of the limiting factors oxalate → hyperoxaluria
of the reaction 2. Hepatic enzymes normal
3. Autosomal recessive ii. Risk factors
a. 3 subtypes 1. Pyridoxine/vitamin B6 deficiency
b. Chromosome 2 (for type I); a. Almost like a secondary cause
Chromosome 19 (for types II of 1° hyperoxaluria type I
and III) 2. Excessive intake of high oxalate-
4. Present EARLY in life (mean age 5 containing food
years old) a. Spinach
iv. Type 2 b. Tea
1. VERY rare c. Chocolate
2. Pathophysiology d. Nuts
a. 2 enzyme deficiencies → e. Beets
increased urine oxalate and f. Soy
glyceric acid 3. Ethylene glycol ingestion
b. Condition also called L-glyceric a. Rare cause of recurrent
aciduria urolithiasis
c. Enzyme defect

384 EDUCATIONAL REVIEW MANUAL IN UROLOGY


d. Enteric D. Cystinuria
i. Pathophysiology a. Pathophysiology
1. Increased GI absorption of oxalate i. Inborn error of metabolism
→ increased filtered oxalate → ii. Proximal convoluted tubule defect
hyperoxaluria 1. Defect in transport system for 4
a. Small bowel pathology (espe- dibasic amino acids
cially ileum) → decreased bile a. Cystine, ornithine, lysine, argi-
salt reabsorption → increased nine (COLA)
chelation of Ca in GI tract → iii. Why cystine stones rather than other
increased free oxalate → dibasic amino acid stones?
increased GI absorption of 1. Cystine (pKa 8.3) is relatively insol-
oxalate uble at typical urine pH (5-7)
b. Increased bile salts in colon → iv. Autosomal recessive
increased permeability of colon v. Heterogenous nucleation can occur
to oxalate 1. Can have associated calcium-based
c. Diarrhea → loss of Mg → urolithiasis (often calcium oxalate)
hypomagnesuria b. Very few significant nonurinary tract
d. Decrease in oxalobacter manifestations
formigines
i. This bacterium metabolizes E. Drug-induced calculi
oxalate in GI tract a. Pathophysiology
ii. Common conditions associated with i. Drug itself or a metabolite is filtered in
enteric hyperoxaluria the kidney and precipitates out of solu-
1. Inflammatory bowel disease tion in the urine
(Crohn’s/UC) ii. Usually requires long-term use of drug
2. Biliary disease b. Stone types
3. Chronic diarrheal syndromes i. Indinavir
4. Pancreatitis/pancreatectomy 1. Protease inhibitor
5. Small bowel resection 2. Used in treatment of HIV infection
6. Short gut syndrome ii. Triamterene
7. Small bowel bypass procedures 1. Potassium-sparing diuretic
(bariatric surgery) iii. Ephedrine
8. Bacterial overgrowth syndromes 1. Synthetic alpha-adrenergic agonist
iii. Classic findings: 24-hour urine collec-
tion
1. Increased urine oxalate
2. Decreased urine Ca
a. Significant loss in GI tract →
less available for renal excretion
b. Often very low (<100 mg/day)
3. Decreased urine pH
a. Secondary to HCO3 loss in
GI tract
e. Idiopathic
i. Pathophysiology – theories
1. Abnormal GI tract receptor →
increased oxalate absorption
2. Decreased intestinal bacteria, which
metabolize oxalate

CHAPTER 13: MEDICAL MANAGEMENT OF UROLITHIASIS 385


4b. Mechanisms of Stone Disease
– Inhibitor Deficiency

A. Hypocitraturia ii. Autosomal-recessive form


a. Pathophysiology – Southeast Asians
i. Decreased citrate → decreased chelation iii. Encodes for basolateral
of Ca in urine → increased chelation of Cl-/HCO3- exchanger on
Ca in urine with other components → distal tubule’s type I inter-
urolithiasis calated cells
b. Pathophysiology b. H+/ATPase Transporter
i. Metabolic Acidosis mutation
1. Pathophysiology i. Autosomal recessive
a. Acidosis → increased chelation ii. Transporter has multiple
of H+ with citrate in mitochon- subunits, different muta-
dria → decreased citrate secre- tions can involve different
tion into urine subunits
b. Acidosis → increased absorp- iii. Sensorineural deafness
tion of citrate into renal tubular c. Carbonic anhydrase II mutation
cells → decreased citrate secre- i. Autosomal recessive –
tion into urine often in consanguineous
2. Conditions marriages
a. Distal renal tubular acidosis = ii. Mixed form of distal (Type
renal tubular acidosis type I I) and proximal (Type II)
b. Chronic diarrheal states RTA
c. Medications: thiazide (causes iii. Osteopetrosis and conduc-
intracellular acidosis) tive deafness
d. High dietary protein intake (= 2. Secondary
increased acid-ash load) a. Pyelonephritis
e. Excessive exercise (lactic acido- b. Urinary tract obstruction
sis) c. Sarcoidosis
f. Idiopathic d. Renal transplant
ii. Intestinal citrate malabsorption e. ATN
iii. UTI (formation of enzyme citrate lyase) f. Analgesic nephropathy
c. Renal tubular acidosis type I g. Primary hyperparathyroidism
i. Pathophysiology h. Idiopathic hypercalciuria
1. Inability of distal nephron to iii. Findings
secrete H+ 1. Acidosis (systemic)
a. Complete 2. Hypocitraturia
i. More severe form a. Usually severe (<200 mg/day)
ii. Have systemic acidosis and b. Pathophysiology
high urinary pH i. Acidosis → increased chela-
b. Incomplete tion of H+ with citrate in
i. Less severe form mitochondria → decreased
ii. No systemic acidosis citrate secretion into urine
iii. Urine pH may be normal, ii. Acidosis → increased
but cannot acidify urine absorption of citrate into
with acid load renal tubular cells →
ii. Associations decreased citrate secretion
1. Primary into urine
a. AE1 gene mutation c. Sequelae
i. Autosomal-dominant form i. Urolithiasis (main
– Caucasians contributor)

386 EDUCATIONAL REVIEW MANUAL IN UROLOGY


ii. Nephrocalcinosis b. Sequelae
iii. Bone disease i. Osteomalacia
3. Hypercalciuria ii. Growth retardation
a. Pathophysiology iv. Concept of incomplete distal renal tubu-
i. Acidosis → increased disso- lar acidosis
ciation of Ca from serum 1. Clinical findings
proteins → increased Ca a. Mild/absent acidosis
available for renal filtration b. Normal/near-normal serum
ii. Acidosis → increased dem- potassium
ineralization of bone → c. Previously known as RTA
increased Ca available for Type III
renal filtration
b. Sequelae B. Hypomagnesuria
i. Urolithiasis a. Pathophysiology
ii. Nephrocalcinosis i. Decreased Mg → decreased chelation of
4. Hyperchloremia oxalate in urine to form soluble
a. Pathophysiology Mg-oxalate complex → increased free
i. Dehydration → increased oxalate in urine → increased Ca chelation
serum aldosterone → reab- with oxalate
sorption of H2O via Na b. Etiology
reabsorption → concomi- i. Often associated with hypocitraturia
tant Cl reabsorption ii. GI malabsorptive conditions
b. Sequelae 1. Inflammatory bowel disease
i. Can be an early sign of pos- (Crohn’s/UC)
sible metabolic acidosis iii. UTIs
5. Hypokalemia
a. Pathophysiology
i. Dehydration → increased
serum aldosterone → reab-
sorption of H2O via Na+
reabsorption → concomi-
tant K+ secretion into urine
ii. Dehydration → increased
serum aldosterone →
decreased renal K+/H+
antiporter activity → Loss
of K+ in urine
b. Sequelae
i. Cardiac: arrhythmia
ii. Musculoskeletal: weakness
iii. GI: ileus
6. Hyperphosphaturia
a. Pathophysiology
i. Acidosis → increased dem-
ineralization of bone →
increased Phos available for
renal filtration

CHAPTER 13: MEDICAL MANAGEMENT OF UROLITHIASIS 387


Table 4

Comparison of Renal Tubular Acidosis Types I, II and IV

RTA I RTA II RTA IV

Nephron segment Distal Proximal Depends on disease


involved

Pathophysiology Inability to secrete H+ Inability to reabsorb Decreased GFR


HCO3 (and multiple
other species)

Association with urolithiasis Yes No No

Acidosis Non-anion gap Non-anion Gap Early: Non-anion gap.


Hyperchloremic Hyperchloremic Hyperchloremic
Late: (GFR <15-20
mL/min): Anion gap
2° to uremia

Serum K Low Low High

Urine pH > 5.5 Usu >5.5 but can Usu >5.5 but can
be <5.5 be <5.5

Urine citrate Low High Slightly Low

Urine Ca High High Low

Urine HCO3 Increased

Special tests Ammonium chloride 1. FeNa (usu >15%) 1. Lasix infusion


loading test 2. Bicarbonate (→ increased urine
loading test acidification)

Associations 1. Multiple etiologies Fanconi syndrome Causes of end-stage


2. Hypophosphatemia renal disease

Treatment Treat underlying 1. HCO3 – supplementation


cause if possible. 2. K Citrate
Urinary alkalinization
(K-Citrate, NaHCO3)

388 EDUCATIONAL REVIEW MANUAL IN UROLOGY


4c. Mechanisms of Stone
Formation – General
Urine Parameters

A. Low urine volume a. Abnormality of purine


a. Important contributor to all forms of urolithi- metabolism associated with
asis raised but variable serum uric
b. Can contribute to urine acidity acid levels secondary arthritis
i. Dehydration → increased renal Na+ ii. Gouty arthritis
reabsorption → increased Na+/H+ 1. Often used as a synonym for gout
exchanger activity→ increased secretion 2. Secondary to deposition of sodium
of H+ urate crystals in joints
iii. Gouty nephropathy
B. Gouty diathesis 1. Definition
a. Definition a. Interstitial nephropathy sec-
i. Urine pH <5.5 ondary to uric acid crystal depo-
b. Risk factors for gouty diathesis sition
i. High dietary purine intake iv. Gouty tophi
1. Increased purines → increased 1. Definition
metabolism of purines → increased a. Subcutaneous depositions of
uric acid production → increased fil- uric acid in the periarticular tis-
tered uric acid sues giving rise to raised, pain-
ii. High dietary protein intake less subcutaneous lesions
1. Amino acid load = acid-ash load
iii. Obesity C. Infection-based
1. Overweight associated with low a. Pathophysiology
urine pH i. UTI → Urinary alkalinization → urolithi-
iv. Metabolic syndrome and insulin asis
resistance ii. Alkalinizing effect more significant if
1. Decreased ammonium excretion in bacteria contains urease
the proximal tubule iii. E. coli does not produce urease
v. Heritable disorders of purine synthesis b. Urease
1. Lesch-Nyhan disease i. Metabolizes urea to ammonia and carbon
2. Overactivity of PRPP (= PRS = dioxide
phosphoribosyl pyrophosphate syn- ii. Ammonia → urinary alkalinization →
thetase) precipitation of urinary calculi with high
3. Adenosine phosphoribosyl pKa values
transferase (APRT) deficiency 1. Struvite (magnesium ammonium
vi. Myeloproliferative disorders phosphate)
1. Secondary to high cellular turnover 2. Calcium phosphate
→ increased purine metabolism c. Bacteria reside in the interstices of the
vii. Tumor lysis syndrome calculus
1. Significant tumor cell death during i. → Difficult (if not impossible) to truly
therapy, usually chemotherapy sterilize urine with antibiotic therapy
2. 2° to high cellular turnover → alone
increased purine metabolism ii. → Urinary sterilization necessitates
viii. Medications removal of all stone material
1. Probenecid d. Other possible contributors include factors
2. Sulfinpyrazone which increased the risk of UTI
ix. ETOH abuse i. Urinary tract obstruction
1. Metabolism to organic acids ii. Chronic nonobstructive hydronephrosis
c. Clinical associations iii. Neurogenic voiding dysfunction
i. Gout iv. Urinary diversion/intestinal substitution
1. Definition

CHAPTER 13: MEDICAL MANAGEMENT OF UROLITHIASIS 389


5. Metabolic Evaluation

A. Approaches 11. Urinary diversion/intestinal substi-


a. Abbreviated evaluation tution
i. When to perform? iii. High-risk populations
1. Low suspicion of significant 1. History of recurrent urolithiasis
metabolic abnormality or contribut- 2. Family history of stones
ing pathophysiology 3. Pediatrics
2. Few/no obvious risk factors for 4. Nephrocalcinosis
recurrence 5. Solitary kidney
ii. Indications for abbreviated metabolic c. Abbreviated metabolic evaluation: details
evaluation i. History
All of the following should be true: 1. Dietary excesses
1. First stone episode a. Purine
2. Single urinary calculus b. Protein
3. Calcium oxalate calculus c. Oxalate
4. Non-staghorn calculus d. Sodium
5. Uncomplicated clinical course 2. Medical conditions which might
6. No anatomic GU anomalies contribute to urolithiasis
7. Struvite stones—if pure struvite a. Urologic
b. Comprehensive evaluation b. Nonurologic
i. When to perform 3. Identification of high-risk popula-
1. High suspicion of significant tions
metabolic abnormality or contribut- a. History of recurrent urolithiasis
ing pathophysiology b. Family history of stones
2. Risk factors for recurrence present c. Pediatrics
ii. Indications for extensive metabolic eval- d. Nephrocalcinosis
uation e. Solitary kidney
1. Any of the requirements for abbrevi- 4. Medications
ated metabolic evaluation not met a. Indinavir
2. Non–calcium-based urolithiasis b. Triamterene
a. Uric acid c. Ephedrine
b. Cystine d. Probenecid
c. Ca Phos e. Sulfapyrazole
3. Medical conditions with increased f. Chemotherapy
risk ii. Physical examination
a. Gout 1. Head and neck
b. Inborn errors of metabolism of a. Palpable mass
purines i. Parathyroid adenoma
i. Lesch-Nyhan syndrome ii. Parathyroid hyperplasia
ii. Overactivity of PRS 2. Abdomen
iii. APRT deficiency a. Palpable kidney
4. Myeloproliferative disease i. Hydronephrosis
5. Tumor lysis syndrome b. Palpable bladder
6. ETOH abuse c. Voiding dysfunction
7. GI malabsorptive/diarrheal states 3. Musculoskeletal
8. H/O GI surgery a. Lumbosacral abnormality
9. Distal renal tubular acidosis i. Neural tube defect → void-
10. Hypercalcemia ing dysfunction
a. Hyperparathyroidism b. Painful joint movements
b. Granulomatous disease i. Gouty arthritis

390 EDUCATIONAL REVIEW MANUAL IN UROLOGY


ii. Primary hyperoxaluria- vi. Radiologic imaging
related 1. KUB
4. Neurologic a. Low sensitivity (small stones,
a. Risk of voiding dysfunction radiolucent stones, ++ bowel
5. Cardiovascular contents)
a. Arrhythmia b. Tomograms help to increase
i. Hypokalemia → distal RTA sensitivity
ii. Hyperoxaluria 2. IVP
6. Skin a. Functional component to study
a. Gouty tophi 3. U/S
iii. Urine tests a. No radiation
1. Urinalysis b. Less sensitive for ureteral
a. pH <5.5 and renal calculi
i. Gouty diathesis c. Can assess for signs of
b. pH >7.5 obstruction
i. UTI i. Doppler ultrasound
2. Urine culture (resistive index) may
3. Urine microscopy increase sensitivity for
a. Crystals obstruction
i. Big one to remember: ben- d. May overestimate stone size
zene-ring appearance = e. Can assess bladder emptying
cystine 4. CT
iv. Stone analysis a. High sensitivity and specificity
1. If not calcium oxalate, proceed with b. Consider low-dose CT to reduce
extensive metabolic evaluation radiation exposure
v. Bloodwork 5. Nuclear renogram
1. Electrolytes a. If obstruction in absence of cal-
a. Decreased HCO3 culus suspected
i. Acidosis, e.g., distal RTA d. Extensive metabolic evaluation – details
b. Decreased K i. Just as with the abbreviated medical
i. Distal RTA exam,
ii. Rx 1. History
c. Increased Cl 2. Physical exam
i. Metabolic acidosis, e.g., 3. Bloodwork
distal RTA 4. Radiologic imaging
2. Ca 5. Urine tests including culture and
a. High microscopy
i. Hyperparathyroidism → ii. 24-hour urine collection
check PTH 1. Do 2 separate collections initially
3. Phos a. > 1 collection helps to cor-
a. Low rect for daily variations in
i. Renal hypercalciuria diet and fluid intake
b. High 2.Components measured (goal values)
i. Resorptive hypercalciuria a. Volume (>2–3L/day)
4. Uric acid b. pH (5.5–7.0)
a. High c. Ca (<250 mg/day)
i. Gout d. Mg (>60 mg/day)
e. Phos (<1,100 mg/day)
f. Na (<200 mEq/day)
g. K (19–135 mEq/day)

CHAPTER 13: MEDICAL MANAGEMENT OF UROLITHIASIS 391


h. Allows assessment of com- iv. Special tests
pliance on potassium citrate 1. Pak test (calcium fast and loading
therapy (value should test)
increase from baseline) a. Allows differentiation of the dif-
i. Creatinine (600–1,800 ferent types of hypercalciuria
mg/day) b. Rarely used
i. Value depends on i. Since absorptive and renal
patient characteristics hypercalciuria are currently
(weight, renal function) managed with thiazides
1. Females ii. Other tests can be used to
10-15 mg/kg determine presence of
2. Males resorptive hypercalciuria
15-20 mg/kg (serum phosphorus)
ii. Allows for assessment c. Uses calcium fasting and load-
of completeness of col- ing to exacerbate the differences
lection between the types of hypercalci-
iii. If too low (<500 uria
mg/day), consider d. Urine cAMP used as surrogate
incomplete collection marker of serum PTH
or dilution with fluid i. It changes more quickly and
j. Uric acid (<700 mg/day) reliably with changes in
k. Oxalate (<45 mg/day) serum Ca
l. Citrate (>320 mg/day) e. Procedure
m. SO4 (<30 mmol/day) i. 7 days low-Ca and low-Na
n. Cystine (<250 mg/L/day) diet
i. Have to ask SPECIFI- ii. Fast from 9 PM prior to day
CALLY for cystine in of test
some labs iii. Distilled water between 9
3. Method of collection PM and midnight
a. Discard first voided urine iv. At 7 AM, empty bladder
on day of collection and discard urine
b. Collect all urine for that day v. Collect urine from
and the first voided urine of 7 AM–9 AM=
the NEXT day fasting sample
c. Most containers do not need vi. 1 gm Ca PO
to be refrigerated/cooled (if vii. Collect urine from
preservatives are included 9 AM– 1 PM =
in the collection contain- post-load sample
ers) viii. Each sample checked for
iii. Stone analysis Ca, cAMP and creatinine
1. Calcium oxalate – 2 forms f. Results
a. Dihydrate i. Absorptive hypercalciuria
b. Monohydrate Type I
2. Calcium phosphate – 2 forms 1. Fasting urine Ca: high
a. Brushite a. Remember – diet
b. Apatite independent
3. Uric acid 2. Fasting urine cAMP =
4. Cystine fasting serum PTH:
5. Struvite Low

392 EDUCATIONAL REVIEW MANUAL IN UROLOGY


a. The increased Ca 4. Post-load urine cAMP
absorption sup- = post-load serum
presses serum PTH PTH: normal
3. Post-load urine Ca: a. The increased Ca
high available from GI
a. GI absorption of tract normalizes
Ca still high and the serum PTH
independent of diet iv. Resorptive hypercalciuria
4. Post-load urine cAMP 1. Fasting urine Ca: high
= post-load serum a. Not related to diet
PTH: low 2. Fasting urine cAMP =
a. GI absorption of fasting serum PTH:
Ca still high and high
independent of diet a. Primary problem is
ii. Absorptive hypercalciuria decreased Phos
Type II causing increased
1. Fasting urine Ca: low serum PTH
a. Remember – diet 3. Post-load urine Ca:
dependent high
2. Fasting urine cAMP = a. Not related to diet
fasting serum PTH: 4. Post-load urine cAMP
normal = post-load serum
a. Because now Ca PTH: high
absorption is nor- a. Again, not related
mal to diet
3. Post-load urine Ca: 2. Distal RTA
high a. Blood gases
a. Dietary dependent i. Acidosis
increase in Ca 1. Consider distal RTA
absorption b. Ammonium chloride loading
4. Post-load urine cAMP test
= post-load serum i. Allows diagnosis of incom-
PTH: low plete distal RTA
a. 2° to the increased (mild/absent systemic aci-
dietary Ca dosis, normal/near normal
iii. Renal hypercalciuria serum K)
1. Fasting urine Ca: high ii. Premise
a. Not related to diet 1. Primary problem is
but to distal renal decreased ability to
tubule abnormality renally eliminate
2. Fasting urine cAMP = excess acid → urine pH
fasting serum PTH: remains high even
high when significant acid
a. 2° to the Ca loss in load delivered
urine iii. Procedure
3. Post-load urine Ca: 1. Baseline venous pH
higher and bicarbonate level
a. More Ca available are drawn
for renal filtration

CHAPTER 13: MEDICAL MANAGEMENT OF UROLITHIASIS 393


2. Ammonium chloride iv. Results
capsules (5 mg or 0.1 1. Urine pH >5.5 after
mg/kg) PO given and acid load: incomplete
100 ml of water given distal RTA
every hour 3. Cystinuria
3. Urine collected over 6- a. Sodium nitroprusside colorimet-
hour period and pH ric test
measured i. Qualitative rather than
4. Venous pH and bicar- quantitative test
bonate levels mea- ii. Results
sured to confirm sys- 1. Urine turns magenta
temic acidosis color at urine cystine
values >75mg/L
Table 5

Results of Calcium Fast and Loading (Pak) Test for Differentiating Types of Hypercalciuria

Type of Fasting Fasting Post-load Post-load


Hypercalciuria Urine Ca Urine cAMP Urine Ca Urine cAMP

Absorptive Type I High Low High Low

Absorptive Type II Normal Normal High Lower

Renal High High Higher Normal

Resorptive High High High High

394 EDUCATIONAL REVIEW MANUAL IN UROLOGY


6. Management Strategies

A. Absorptive hypercalciuria 2. Side effects


a. Goals – absorptive hypercalciuria type I a. Lassitude/sleepiness
i. Decrease renal filtration of Ca b. Hypokalemia
1. Decreased dietary sodium i. Often administered with K-
2. Thiazides (chlorthalidone, inda- citrate or utilize K-sparing
pamide, hydrochlorothiazide) diuretic (e.g., moduretic =
ii. Increase urine inhibitors of stone HCTZ + amiloride)
formation c. Acidosis (mild)
1. Potassium citrate i. Secondary to mild hyper-
iii. Change general urine parameters uricemia
1. Increased urine output ii. Often administered with K-
2. Urinary alkalization citrate to address this issue
a. Potassium citrate d. Can unmask subtle hyper-
b. Diet parathyroidism
i. Moderate calcium intake e. Glucose intolerance
ii. Decreased purine intake f. Erectile dysfunction
iii. Decreased animal protein iii. Sodium cellulose phosphate (no longer
intake available)
iv. Increased fiber intake 1. Mechanism of action
b. Goals – absorptive hypercalciuria type II a. Binds Ca in GI tract
i. Decreased renal filtration of Ca 2. Side effects
1. Decreased dietary sodium a. GI: nausea, diarrhea
2. Thiazides b. Hypomagnesuria
ii. Decreased GI absorption of Ca i. Binds Mg in GI tract →
1. Sodium cellulose phosphate (no decreased GI Mg absorption
longer available) → decreased renal Mg fil-
2. Orthophosphates tration
iii. Increased urine inhibitors of stone c. Hyperoxaluria
formation i. Binds Ca and Mg in GI tract
1. Potassium citrate → increased free oxalate in
iv. Change general urine parameters GI tract → increased GI
1. Increased urine output oxalate absorption →
2. Urinary alkalinization increased renal oxalate fil-
a. Potassium citrate tration
b. Diet d. Negative calcium balance
i. Moderate calcium intake i. Secondary increased PTH
ii. Decreased purine intake → bone resorption
iii. Decreased animal protein iv. Orthophosphates
intake 1. Mechanism of action
iv. Increased fiber intake a. Binds Ca in GI tract
c. Specifics 2. Side Effects
i. Decreased dietary sodium a. GI: nausea, diarrhea
1. Renal sodium excretion correlates b . Hypomagnesuria
with renal Ca excretion i. Binds Mg in GI tract →
ii. Thiazide diuretics (chlorthalidone, inda- decreased GI Mg absorption
pamide) → decreased renal Mg fil-
1. Mechanism of action tration
a. Inhibits distal tubular Na secre-
tion → concomitant decreased
renal Ca excretion

CHAPTER 13: MEDICAL MANAGEMENT OF UROLITHIASIS 395


c. Hyperoxaluria load → decreased consumption of cellular citrate →
i. Binds Ca and Mg in GI tract more citrate available for filtration into urine →
→ increased free oxalate in decreased hypocitraturia
GI tract → increased GI iii. Decreased amino acid load
oxalate absorption → → decreased metabolism
increased renal oxalate fil- to oxalate → decreased
tration risk of associated calcium
d. Options oxalate urolithiasis
i. K-Phos neutral 3. Decreased sodium intake
ii. Neutra-Phos K a. Renal sodium excretion corre-
iii. Uro-KP-Neutral lates with renal Ca excretion
v. Potassium citrate i. Reduction of urinary
1. Mechanism of action sodium by 100 mEq/day
a. Increases urinary citrate reduces urinary calcium by
b. Urinary alkalization 50 mg/day
i. Citrate binds to H+ 4. Increased fiber intake
c. Counteracts the K-wasting and a. Mechanism of action
acidotic effects of thiazides i. Binds Ca in GI tract
2. Side effects 5. Increased urine output
a. GI intolerance a. Increased fluid intake to keep
i. Reduced when taken with urine output >2,000 mL/day
food/meals 6. Neutral phosphates
vi. Diet a. Mechanism of action
1. Decreased purine intake i. Increased GI absorption of
a. Mechanism of action phosphate → increased
i. Decreased purine load → serum phosphate → nega-
decreased purine tive feedback → decreased
metabolism → decreased vitamin D → decreased GI
uric acid production → absorption of Ca
decreased renal filtration of b. Side Effects
uric acid → decreased urine i. GI: nausea, diarrhea
acidity → decreased risk of
calcium oxalate urolithiasis B. Renal hypercalciuria
b. What to avoid a. Goals
i. Excessive animal protein i. Decreased renal excretion of Ca
intake (high purine levels) 1. Thiazides (chlorthalidone, inda-
2. Decreased animal protein intake pamide, hydrochlorothiazide)
a. Mechanism of action 2. Decreased dietary sodium
i. Decreased amino acid load ii. Increased stone inhibitors
→ decreased serum acid 1. Potassium citrate
load → decreased urine iii. Change general urine parameters
acidity 1. Increased urine output
ii. Decreased amino acid load b. Specifics
→ decreased serum acid i. Thiazides
1. Mainstay of therapy of renal hyper-
calciuria
2. Will correct renal leak of Ca
3. Will maintain positive calcium
balance
ii. Decreased sodium intake

396 EDUCATIONAL REVIEW MANUAL IN UROLOGY


1. Renal sodium excretion correlates the drug
with renal Ca excretion 2. Side effects
iii. Potassium citrate a. GI: nausea, diarrhea
1. Used in combination with thiazides iii. Potassium citrate
to prevent hypokalemia with subse- 1. Especially if hypercalciuria not well
quent hypocitraturia controlled on orthophosphate ther-
iv. Increased urine output apy
1. Increased fluid intake to keep urine iv. Increased urine output
output >2,000 mL/day 1. Increased fluid intake to keep urine
output >2,000 mL/day
C. Resorptive hypercalciuria
a. Goals D. Hyperuricosuria
i. Decreased Ca resorption from bone a. Goals
1. Bisphosphonates i. Decreased stone promoters
2. Surgical removal of parathyroid ade- 1. Diet
noma or parathyroid hyperplasia a. Decrease dietary purine intake
ii. Increased stone inhibitors b. Decrease dietary animal protein
1. Potassium citrate intake
iii. Change general urine parameters c. Decrease dietary sodium intake
1. Increased urine output 2. Allopurinol
2. Potassium citrate ii. Increase stone inhibitors
b. Specifics 1. Potassium citrate
i. Surgical removal of parathyroid adenoma iii. Change general urine parameters
or hyperplastic parathyroid tissue best 1. Increase urine output
treatment 2. Urinary alkalization
ii. Orthophosphate a. Potassium citrate
1. Mechanism of action b. Specifics
a. Inhibition of osteoclast activity i. Diet
→ counters effect of increased 1. Decrease purine intake
PTH → Decreased bone resorp- a. Mechanism of action
tion i. Decreased purine load →
b. Possible augmentation of tubu- decreased purine
lar Ca reabsorption → metabolism → decreased
decreased hypercalciuria uric acid production
c. This may result in negative feed- ii. Decreased uric acid pro-
back on PTH production → duction → decreased renal
decreased PTH → decreased filtration of uric acid →
bone resorption of Ca decreased urine acidity
d. Increased urinary citrate excre- 2. Decrease animal protein intake
tion a. Mechanism of action
i. Likely secondary to the i. Decreased amino acid load
alkaline nature of → decreased serum acid
orthophosphate (sequesters load → decreased urine
H+) acidity
e. Increases urinary pyrophosphate
excretion → increased urine
stone inhibitor
i. Likely from metabolism of

CHAPTER 13: MEDICAL MANAGEMENT OF UROLITHIASIS 397


ii. Decreased amino acid load ii. Decreased GI oxalate absorption
→ decreased serum acid 1. Low oxalate diet
load → decreased con- 2. Neutral phosphates
sumption of cellular citrate iii. Decreased renal oxalate excretion
→ more citrate available 1. Thiazide
for filtration into urine → 2. Avoid excessive vitamin C
decreased hypocitraturia iv. Increased stone inhibitors
iii. Decreased amino acid load 1. Potassium citrate
→ decreased metabolism v. Change general urine parameters
to oxalate → decreased 1. Increase urine output
risk of associated calcium 2. Urinary alkalization
oxalate urolithiasis a. Potassium citrate
3. Decrease sodium intake vi. Fix the primary problem
a. Renal sodium excretion corre- 1. Liver transplant ± renal
lates with renal Ca excretion transplant
ii. Allopurinol b. Specifics
1. Mechanism of action i. Pyridoxine (= vitamin B6)
a. Inhibitor of xanthine oxidase → 1. Mechanism of action
decreased uric acid, increased a. Cofactor for AGT → maximizes
xanthine (more soluble) the effect of the enzyme
2. Side effects b. Even if the enzyme is not defec-
a. Rash tive (i.e., type 2 primary hyper-
i. Can progress to an exfolia- oxaluria), it can help to maxi-
tive hemorrhagic dermatitis mize enzyme activity → mini-
with systemic vasculitis mize oxalate production
(rare) ii. Low oxalate diet
b. Flare of gouty arthritis 1. Avoid foods with high oxalate con-
c. Xanthine urolithiasis (rare) tent
iii. Febuxostat a. Spinach
1. Xanthine oxidase inhibitor b. Nuts
2. Approved for patients with gout c. Chocolate
3. Studies on application for d. Beets
hyperuricosuria are pending e. Tea
iv. Potassium citrate f. Soy
1. Mechanism of action iii. Neutral phosphates
a. Urinary alkalinization 1. Bind oxalate in GI tract
i. Increased urinary citrate → iv. Thiazide
increased chelation of urine 1. Hypocalciuric effect → decreased
Ca → decreased chance of Ca available for stone formation
associated Ca-oxalate v. Avoid excessive vitamin C
urolithiasis 1. Excess ascorbic acid → converted to
v. Increase urine output oxalate
1. Increased fluid intake to keep urine vi. Potassium citrate
output >2,000 mL/day 1. Mechanism of action
a. Urinary alkalinization
E. Primary hyperoxaluria b. Decreased chance of associated
a. Goals calcium oxalate urolithiasis
i. Increased conversion of metabolic inter- vii. Increased urine output
mediates to nonoxalate end products 1. Increased fluid intake to keep urine
1. Pyridoxine (= vitamin B6) output >2,000 mL/day

398 EDUCATIONAL REVIEW MANUAL IN UROLOGY


F. Enteric hyperoxaluria vi. Sodium bicarbonate
a. Goals 1. Many of these conditions associated
i. Decrease GI oxalate absorption with increased HCO3 loss in GI tract
1. Increased GI chelation of oxalate
a. Low-fat diet G. Hypocitraturia
b. Bile salt resins a. Goals
c. Oral calcium supplementation i. Increase stone inhibitors
d. Low-oxalate diet 1. Potassium citrate
e. Reversal of GI bypass if ii. Change general urine parameters
possible 1. Increase urine output
ii. Increase stone inhibitors 2. Urinary alkalinization
1. Potassium citrate a. Potassium citrate
iii. Change general urine parameters b. Sodium bicarbonate
1. Increase urine output c. Citrus juice
2. Urinary alkalinization i. Lemon juice
a. Potassium citrate ii. Orange juice – higher
b. Sodium bicarbonate calories than lemonade
b. Specifics
i. Low-fat diet H. Hypomagnesuria
1. Decreases the chelation of Ca by fats a. Goals
→ increased chelation of oxalate by i. Increase stone inhibitors
Ca in GI tract 1. Magnesium citrate
ii. Bile salt resins 2. Magnesium gluconate
1. Decreases the bile salts in GI tract → 3. Potassium magnesium citrate (not
decreased chelation of Ca with bile yet FDA-approved)
salts → increased chelation of b. Change general urine parameters
oxalate by Ca i. Increased urine output
iii. Low-oxalate diet ii. Treat concomitant hypocitraturia
1. Avoid foods with high oxalate con- 1. Potassium citrate
tent
a. Spinach I. Low urine volume
b. Nuts a. Goals
c. Chocolate i. Change general urine parameters
d. Beets 1. Increase urine output
e. Soy b. Specifics
f. Tea i. Ideally >3,000 mL/day
iv. Oral calcium supplementation ii. Realistically >2,000 mL/day
1. Usually as calcium citrate iii. Usually requires >100 oz fluid/day
2. Increased chelation of oxalate in GI iv. Need to be cognizant of situations where
tract with extra Ca increased fluids required
3. The extra Ca does not significantly 1. Exercise
increase the risk of stone formation 2. Heat
since these patients have low urine 3. Illness
Ca 2° to Ca loss in GI tract
v. Potassium citrate J. Gouty diathesis
1. Alkalinizes urine a. Goals
a. Decreased chance of associated i. Change general urine parameters
Ca-based urolithiasis 1. Increase urine output
2. Urinary alkalization

CHAPTER 13: MEDICAL MANAGEMENT OF UROLITHIASIS 399


a. Potassium citrate iii. Change general urine parameters
b. Sodium bicarbonate 1. Increase urine output
3. Citrus juices will not work for 2. Urinary alkalization
gouty diathesis (no significant a. Potassium citrate
change in urine pH) b. Sodium bicarbonate
a. Decrease acid load c. Acetazolamide
i. Low-purine diet b. Specifics
ii. Low-sodium diet i. Low-methionine diet
iii. Low-sugar diet 1. Methionine
iv. High-fiber diet a. Essential amino acid metabo-
v. Allopurinol lized to cystine
b. Specifics b. Fairly unpalatable → low com-
i. Potassium citrate pliance
1. Urinary alkalization ii. Thiola
2. Decreased acid load 1. alpha-MPG stands for:
3. Goal is urine pH 6.5–7.0 alpha-mercaptopropionylglycine
ii. Sodium bicarbonate 2. Better side effect profile than peni-
1. Urinary alkalization cillamine
2. Decreased acid load 3. Usually first-line treatment for
3. Goal is urine pH 6.5–7.0 cystinuria
4. Problem is increase sodium load iii. Penicillamine (specifically d-penicil-
iii. Low-purine diet lamine)
1. Decreased acid (especially uric acid) 1. Forms penicillamine-cystine disul-
iv. Low-sodium diet fide: 50x more soluble than cystine
1. Decreased urine Ca 2. Side effects
v. Low-sugar diet a. Hematologic: agranulocytosis,
1. Decreased ketoacidosis thrombocytopenia
vi. High-fiber diet b. Dermatologic: pemphigus
1. Binds Ca in GI tract → decreased Ca c. Renal: nephrotic syndrome
absorption d. Rheumatologic: polymyositis
2. Decreased intestinal transit time → e. B6 Deficiency → usually add
decreased Ca absorption pyridoxine 50 mg PO QD
vii. Allopurinol 3. Cheaper than thiola
1. Indications iv. Bucillamine
a. Profound hyperuricosuria (urine 1. Better side effect profile than peni-
uric acid >900–1,200 g/day) cillamine
b. Hyperuricosemia 2. Medication is still investigational
v. Captopril
K. Cystinuria 1. Captopril-cysteine 200x more
a. Goals soluble than cystine
i. Decreased stone promoters 2. Effect not as reliable as other thera-
1. Low methionine diet (rarely pies
effective) 3. Rarely used
ii. Increase urinary inhibitors vi. Potassium citrate
1. Urinary cystine-chelating agents 1. Urinary alkalization
a. Penicillamine 2. Goal: urine pH approximately 7.0
b. Thiola (alpha-mercaptopropi- a. Greater than this → paradoxical
onylglycine) increased risk of CaPhos stone
c. Bucillamine (investigational) formation
d. Captopril

400 EDUCATIONAL REVIEW MANUAL IN UROLOGY


vii. Sodium bicarbonate a. Neurogenic voiding dysfunction
1. Urinary alkalization b. Chronic nonobstructive
2. Usually added when K-citrate maxi- hydronephrosis
mized iii. Acetohydroxamic acid
viii. Increased urine output 1. Mechanism of action
1. Solubility of cystine approximately a. Urease inhibitor
250 mg/L of urine 2. Especially where contributing fac-
a. Thus the goal for urinary cystine tors difficult to control
levels on therapy is <250 mg a. Neurogenic voiding dysfunction
cystine/L urine b. Chronic nonobstructive
hydronephrosis
L. Infection-based urolithiasis 3. Side effects
a. Goals a. Hematologic: anemia, reticulo-
i. Decrease stone promoters cytosis
1. Surgical removal of as much stone b. Dermatologic: rash
material as possible c. GI: hepatotoxicity, diarrhea,
a. Ideally, complete stone removal abdominal pain, nausea, vomit-
2. Correct anatomic/functional abnor- ing
malities d. Teratogenic (avoid use in
a. UPJO women of childbearing age)
b. UVJO e. Other: palpitations, H/A, deep
c. Calyceal diverticulum venous thrombosis
d. Voiding dysfunction iv. Treat associated metabolic abnormali-
3. Suppressive antibiotics ties
a. Especially where contributing 1. Approximately 50% will have addi-
factors difficult to control, e.g., tional metabolic abnormalities
neurogenic voiding dysfunction
ii. Increase stone inhibitors M. Medication-based urolithiasis
1. Urease inhibitors a. Goals
a. Acetohydroxamic acid i. Decrease stone promoters
iii. Change general urine parameters 1. Stop/change medication
1. Increase urine output ii. Change general urine parameters
iv. Treat associated metabolic abnormali- 1. Increase urine output
ties
1. Metabolic evaluation warranted if
stone is mixed (i.e., struvite + cal-
cium)
2. Metabolic abnormalities rarely
found if stone composition is pure
infection
b. Specifics
i. Surgical removal of as much stone mate-
rial as possible
1. Decreased bacterial load
2. Decreased urine stasis
ii. Suppressive antibiotics
1. Especially where contributing fac-
tors difficult to control

CHAPTER 13: MEDICAL MANAGEMENT OF UROLITHIASIS 401


7. Pitfalls of Medical
Stone Management

A. Ensure complete 24-hour urine collection


a. 24-hour creatinine as measure of “complete-
ness” of collection

B. New stones/stone growth with 24-hour urine


collection volumes → 2,000 mL and “normal”
parameters
a. Check 24-hour creatinine to ensure patient is
not adding water/fluid to 24-hour urine col-
lections

C. Increased 24-hour uric acid after initiating


K-citrate therapy
a. Decreased urine acidity → increased solubil-
ity of uric acid → more uric acid analyzed

D. Citrate level not responding to K-citrate


therapy
a. 24-hour urine K should increase with K-cit-
rate therapy
b. Is patient compliant?

E. Urinary diversions/intestinal substitutions


often absorb some urinary components
a. 24-hour urine collections often unreliable
b. Often treat presumptively with K-citrate

402 EDUCATIONAL REVIEW MANUAL IN UROLOGY


Table 6

Stone Management Medication Summary

Medication Indications Side Effects Dose Other Notes

Thiazide Hypercalciuria Hypokalemia Chlorthalidone High sodium intake


(absorptive Type I, 25-50 mg will offset effect
II; renal) PO QD of thiazide; often
Hyperoxaluria Indapamide: administered with
(primary, idiopathic) 1.25 mg K-citrate or amiloride
PO QD to counter hypokalemic
effects; often administ-
ered with K-citrate to
counter acidotic effects

Orthophosphate Hypercalciuria GI intolerance Each tab has Side-effect profile


(absorptive) 250 mg Phos makes this 2nd line
1-2 tabs if thiazides ineffective
PO QID

Bisphosphonate Hypercalciuria Hypocalcemia Alendronate Ensure PTH normal


(resorptive) Rash 5-10 mg PO QID before starting
GI intolerance Risedronate medical therapy
5 mg PO qd

Potassium citrate Hypercalciuria GI intolerance 40-90 mEq Take with meals


Hyperoxaluria Hyperkalemia PO divided to avoid GI upset
Gouty diathesis BID/TID
Cystinuria

Sodium Hyperoxaluria (enteric) GI intolerance 650-1300 mg


bicarbonate Gouty diathesis Alkalosis PO up to QID
Distal renal
tubular acidosis
Cystinuria

(continued)

CHAPTER 13: MEDICAL MANAGEMENT OF UROLITHIASIS 403


Table 6 (continued)

Stone Management Medication Summary

Medication Indications Side Effects Dose Other Notes

Allopurinol Hyperuricosuria Rash 300-600 mg Wait until after acute


Hyperuricosemia Xanthine PO QD gouty arthritis attack
urolithiasis subsided before starting

d-Penicillamine Cystinuria Pyridoxine 500-1000 mg Side effect profile


deficiency, PO QID makes this more
Rash of a second-line agent;
Agranulocytosis cheaper than Thiola
Leucopenia
Nephrotic syndrome
Arthralgia

Thiola Cystinuria Similar to 200-1200 mg 1st line agent


penicillamine but PO divided
less common BID-QID
and less severe

Captopril Cystinuria Cough 75-150 mg Usually 3rd


Renal PO QD line agent
insufficiency

Acetohydroxamic Infection-based Anemia 250 mg F/U q 3-4


acid nephrolithiasis Rash PO TID/QID mo required on
Hepatotoxicity this medication
GI upset to monitor
DVT for side effects

Pyridoxine Hyperoxaluria Primary:


(primary) 200-400
Cystinuria PO QD
(if penicillamine Adjunct:
being used) 50 mg PO QD

Ca Hyperoxaluria Constipation 1-4 gm Follow-up urine


supplementation (enteric) PO TID collections to avoid
with meals hypercalciuria
Use calcium citrate
preparation

Mg gluconate Hyperoxaluria GI intolerance Start at Side effects usually


(primary) Hyporeflexia 400 or 420 mg seen at very high doses
Hypotonia/ PO QD
Muscle paralysis and tailor to
Hypotension response
Cardiac arrhythmia

Bile salt resin Hyperoxaluria GI intolerance Cholestyramine


(enteric) 1-4 gm PO TID
with meals

404 EDUCATIONAL REVIEW MANUAL IN UROLOGY


8. Questions

Case 1 1. This condition is best described as:

A 65-year-old African American female with an 8- A. Primary hyperoxaluria


year history of recurrent stone formation reports
having spontaneously passed 35 stones. She has B. Gouty diathesis
required 3 ureteroscopic laser stone fragmentations
and 1 percutaneous nephrolithotripsy. Current C. Distal renal tubular acidosis
radiographs reveal 3 small stones in the left
intrarenal collecting system and 2 in the right. This D. Enteric hyperoxaluria
patient’s medical history is significant for inflamma-
tory bowel disease. There is a family history of E. Hyperuricosuria
nephrolithiasis in one sibling.
2. The risk factor most associated with recur-
24-hour Urine Collections rent stone formation secondary to this con-
dition is:
Normal Range Initial Visit
Vol >2,000 mL/d 1,320 A. Hyperabsorption of oxalate in the
pH 5.5–6.7 5.51 jejunum
Calcium <200 mg/d 85
Sodium <200 mg/d 95 B. Hyperexcretion of calcium from the dis-
Potassium <60 meg/d 45 tal tubule
Uric acid <600 mg/d 375
Oxalate <45 mg/d 78 C. Diminished citrate absorption in the ter-
Citrate >600 mg/d <20 minal ileum
Magnesium >60 mg/d 50
SO4 <20 mg/d 8 D. Hyperabsorption of calcium in the
Cystine 0 mg/L 0 small bowel

Serum Values E. Increased colonic absorption of free


Sodium 135–145 mEq/L 138 oxalate
Potassium 0.2–4.8 mEq/L 3.5
Chloride 98–108 mEq/L 107 3. The optimum treatment for patients with
Bicarbonate 21–30 mEq/L 20 this disorder would include:
Creatinine 0.7–1.4 mg/dL 1.1
Calcium 8.7–10.2 mg/dL 9.2 A. Calcium supplements, potassium cit-
Phosphorus 2.3–4.3 mg/dL 3.1 rate, increased oral fluid intake
Uric acid 2.5–8.0 4.5
PTH 13–64 ng/mL 48 B. Dietary restriction of oxalate

C. Thiazides and potassium citrate

D. Allopurinol

E. Pyridoxine

CHAPTER 13: MEDICAL MANAGEMENT OF UROLITHIASIS 405


4. The most important factor predisposing
patients to this metabolic disorder is:
Case 2

A 50-year-old obese Caucasian male is evaluated


for a 2-day history of left flank pain without fevers, A. Hypercalciuria
nausea or emesis. He has a prior history of stone dis-
ease for 6 years and has spontaneously passed 4–5 B. Low urinary pH
stones. He’s had no prior surgeries for calculus dis-
ease and family history is negative for nephrolithia- C. Hypocitraturia
sis, but positive for gout. No stones are seen on the
plain abdominal radiographs. Prior stone composi- D. Low urine volumes
tion contains mixed calcium oxalate.
E. Hyperuricosuria
24-hour Urine Collections
5. The most appropriate medical treatment of
Normal Range Initial Visit this condition is:
Vol >2,000 mL/d 1,300
pH 5.5–6.7 5.12 A. Allopurinol
Calcium <200 mg/d 140
Sodium <200 mg/d 170 B. Thiazides
Potassium <60 mEq/d 35
Uric acid <600 mg/d 285 C. Increased fluids
Oxalate <45 mg/d 35
Citrate >600 mg/d 220 D. Dietary calcium restriction
Magnesium >60 mg/d 70
SO4 <20 mg/d 24 E. Potassium citrate
Cystine 0 mg/L 0
Serum Values
Sodium 135–145 meg/L 140
Potassium 3.2–4.8 mEq/L 4.2
Chloride 98–108 mEq/L 102
Bicarbonate 21–30 mEq/L 27
Creatinine 0.7–1.4 mg/dL 0.9
Calcium 8.7–10.2 mg/dL 2.8
Phosphorus 2.3–4.3 mg/dL 2.8
Uric acid 2.5–8.0 mg/dL 7.7
PTH 13–64 ng/mL 43

406 EDUCATIONAL REVIEW MANUAL IN UROLOGY


6. The stone composition of this patient is
most likely:
Case 3

A 58-year-old Hispanic female is seen by her family


physician with a history of recurrent urinary tract A. Calcium oxalate
infections treated 3–4 times in the last 18 months.
At present, she is asymptomatic. She denies a his- B. Uric acid
tory of nephrolithiasis. Renal ultrasound demon-
strates moderate left hydronephrosis and a large C. Magnesium ammonium phosphate
density within the renal pelvis with posterior shad-
owing. KUB with tomography reveals a poorly D. Cystine
opacified stone involving the renal pelvis and lower
pole calyces. Prior urine cultures have grown Pro- E. Hydroxyapatite
teus and Klebsiella species. Following uncompli-
cated left percutaneous nephrolithotomy, she 7. The most common cause of recurrent stone
returns with the following metabolic results: disease in a patient having undergone
“sandwich” therapy (PNL followed by
24-hour Urine Collections SWL) for a staghorn calculus is:

Normal Range Initial Visit A. Hypomagnesuria


Vol >2,000 mL/d 1,600
pH 5.5–6.7 6.9 B. Hyperoxaluria
Calcium <200 mg/d 210
Sodium <200 mg/d 120 C. Retained stone fragments
Potassium <60 mEq/d 40
Uric acid <600 mg/d 360 D. Renal tubular acidosis
Oxalate <45 mg/d 20
Citrate >600 mg/d 110 E. Hypercalciuria
Magnesium >60 mg/d 80
SO4 <20 mg/d 10 8. Acetohydroxamic acid contributes to
Cystine 0 mg/L 0 reducing infection stone formation by:

Serum Values A. Reversing associated metabolic defects


Sodium 135–145 mEq/L 136
Potassium 3.2–4.8 mEq/L 3.8 B. Preventing recurrent urinary tract infec-
Chloride 98–108 mEq/L 98 tions
Bicarbonate 21–30 mEq/L 22
Creatinine 0.7–1.4 mg/dL 1.6 C. Alkalinization of the urine
Calcium 8.7–10.2 mg/dL 9.5
Phosphorus 2.3–4.3 mg/dL 3.1 D. Irreversibly inhibiting urease
Uric acid 2.5–8.0 mg/dL 6.6
PTH 13–64 ng/mL 28 E. All of the above

CHAPTER 13: MEDICAL MANAGEMENT OF UROLITHIASIS 407


9. The likely metabolic diagnosis contribut-
ing to this patient’s recurrent stone forma-
Case 4

A 12-year-old male is seen for evaluation of recur- tion is:


rent nephrolithiasis. He has spontaneously passed 3
stones over the previous 4 years, and has recently A. Hypocitraturia
undergone shock wave lithotripsy twice without
success. He has been treated in the past with an B. Hyperoxaluria
unknown medication, but was discontinued because
the parents felt it was of no benefit. Family history C. Low urine volumes
is negative for stone disease.
D. Gouty diathesis
24-hour Urine Collections
E. Cystinuria
Normal Range Initial Visit
Vol >2,000 mL/d 550 10. Alpha-mercaptopropionylglycine
pH 5.5–6.7 5.4 (Thiola®) may be helpful in the manage-
Calcium <200 mg/d 110 ment of cystinuria, since it:
Sodium <200 mg/d 117
Potassium <60 mEq/d 39 A. Acts as a diuretic, further decreasing uri-
Uric acid <600 mg/d 215 nary cystine concentration
Oxalate <45 mg/d 22
Citrate >600 mg/d 260 B. Is significantly more effective than
Magnesium >60 mg/d 80 D-penicillamine
SO4 <20 mg/d 7
Cystine 0 mg/L/day 1,345 C. Can be used as both an oral and
intrarenal chemolytic agent
Serum Values
Sodium 135–145 mEq/L 140 D. Has equivalent efficacy at increasing
Potassium 3.2–4.8 mEq/L 4.1 solubility with reduced toxicity as com-
Chloride 98–108 mEq/L 108 pared to D-penicillamine
Bicarbonate 21–30 mEq/L 23
Creatinine 0.7–1.4 mg/dL 0.8 E. Adequately alkalinizes the urine, obviat-
Calcium 8.7–10.2 mg/dL 9.0 ing the need for potassium citrate
Phosphorus 2.3–4.3 mg/dL 4.0
Uric acid 2.5–8.0 mg/dL 5.9
PTH 13–64 ng/mL 43

408 EDUCATIONAL REVIEW MANUAL IN UROLOGY


11. The most definitive test to identify this dis-
order would demonstrate:
Case 5

A 19-year-old Caucasian female with a 6-year his-


tory of recurrent stone disease is found to have mul- A. Decreased serum parathyroid hormone
tiple bilateral renal calculi by renal ultrasound dur- levels
ing an evaluation for recurrent flank pain. She
reports having passed >10 stones in the previous 2 B. Persistently elevated urine calcium
years. Review of the renal ultrasound indicates no
evidence of hydronephrosis. KUB and tomograms C. Inability to reduce the urine pH below
demonstrate 5 stones on the left and 8 stones on the 5.5
right, all <4 mm in size. She has a strong family his-
tory of stones with 3 first-degree relatives and 2 D. Normalization of hypercalciuria
cousins with nephrolithiasis. Stone compositions
have been mixed calcium phosphate and calcium E. Marked increase in urinary uric acid lev-
oxalate. els with initiation of treatment

24-hour Urine Collections

Normal Range Initial Visit


Vol >2,000 mL/d 1,425
pH 5.5–6.7 6.94
Calcium <200 mg/d 260
Sodium <200 mg/d 160
Potassium <60 mEq/d 28
Uric acid <600 mg/d 410
Oxalate <45 mg/d 32
Citrate >600 mg/d 140
Magnesium >60 mg/d 66
SO4 <20 mg/d 13
Cystine 0 mg/L 0

Serum Values
Sodium 135–145 mEq/L 142
Potassium 3.2–4.8 mEq/L 3.3
Chloride 98–108 mEq/L 107
Bicarbonate 21–30 mEq/L 21
Creatinine 0.7–1.4 mg/dL 0.9
Calcium 8.7–10.2 mg/dL 9.2
Phosphorus 2.3–4.3 mg/dL 2.2
Uric acid 2.5–8.0 mg/dL 4.1
PTH 13–64 ng/mL 58

CHAPTER 13: MEDICAL MANAGEMENT OF UROLITHIASIS 409


12. The primary defect in this condition is con-
sidered to be:
Case 6

A 49-year-old Caucasian female with a 4-year his-


tory of stone disease has passed 6 stones sponta- A. Primary hyperabsorption of intestinal
neously, 3 in the last year. Noncontrast renal CT calcium
demonstrates a 2-mm calcification in each kidney
without secondary signs of obstruction. Previous B. Hypersecretion of parathyroid hormone
stone analysis has revealed a mixed composition of
calcium phosphate and calcium oxalate. She had C. Renal leak of calcium
been treated with hydrochlorothiazide in the past,
but this medication was discontinued after 3 months D. Bone disease
of therapy. Her family history is significant for a
brother and grandmother with stones. She is other- E. Excessive dietary intake of calcium-con-
wise healthy and has prior surgical history. taining foods

24-hour Urine Collections

Normal Range Initial Visit


Vol >2,000 mL/d 1,800
pH 5.5–6.7 5.8
Calcium <200 mg/d 335
Sodium <200 mg/d 230
Potassium <60 mEq/d 38
Uric acid <600 mg/d 472
Oxalate <45 mg/d 29
Citrate >600 mg/d 680
Magnesium >60 mg/d 70
SO4 <20 mg/d 15
Cystine 0 mg/L 0

Serum Values
Sodium 135–145 mEq/L 138
Potassium 3.2–4.8 mEq/L 4.3
Chloride 98–108 mEq/L 102
Bicarbonate 21–30 mEq/L 25
Creatinine 0.7–1.4 mg/dL 1.3
Calcium 8.7–10.2 mg/dL 9.1
Phosphorus 2.3–4.3 mg/dL 2.8
Uric acid 2.5–8.0 mg/dL 5.1
PTH 13–64 ng/mL 18

410 EDUCATIONAL REVIEW MANUAL IN UROLOGY


worsen metabolic acidosis and hypokalemia
through its diuretic effects and renal potassium
Answers

losses. Colon resection may be of benefit in those


patients refractory to medical management, as the
Case 1

1. D. primary site of intestinal absorption of oxalate is the


Enteric hyperoxaluria is a disorder most commonly large bowel.
affecting patients with inflammatory bowel disease,
particularly intestinal segments involving the small
intestine. Fat malabsorption is the hallmark condi-
Case 2

tion predisposing to saponification and sequestering 4. B.


of calcium to be passed in the stool. Less calcium is Although low urine volumes and hyperuricosuria
available in the GI tract to bind oxalate, thereby contribute to the possibility of uric acid stone for-
allowing more oxalate to be absorbed with a relative mation, the most critical determinant of the crystal-
increase in urinary oxalate. Primary hyperoxaluria lization of uric acid remains urinary pH. In addition,
is an autosomal-recessive disorder which manifests uric acid stones may be formed in patients with pri-
itself only in the homozygous state. Unless effec- mary gout with associated severe hyperuricosuria
tively treated early, primary hyperoxaluria typically and other secondary causes of purine overproduc-
runs a malignant course with early death from renal tion, such as myeloproliferative states, glycogen
failure. storage disease and malignancy.

2. E. 5. E.
Intestinal hyperabsorption of oxalate in patients Allopurinol will decrease the production of uric
with enteric hyperoxaluria is the most significant acid by inhibiting xanthine oxidase in the purine
risk factor leading to recurrent calculus formation. metabolic pathway, but is most effective in patients
Intestinal transport of oxalate is primarily increased with extremely elevated levels of uric acid (urinary
because of the effects of bile salts and fatty acids on uric acid >1,500 mg/day). In addition, increasing
the permeability of colonic intestinal mucosa to total urine volume will decrease the concentration
oxalate. The total amount of oxalate absorbed may of uric acid to assist in preventing stone formation.
also be increased because of an enlarged intralumi- However, raising the urinary pH above the dissocia-
nal pool of oxalate available for absorption. Intesti- tion constant of uric acid is the key to preventing
nal fat malabsorption characteristic of ileal disease recurrent uric acid stone formation and correcting
will exaggerate calcium soap formation, limit the gouty diathesis. The urine pH should be maintained
amount of free calcium to complex to oxalate, and between 6.0 and 6.5. Thiazides and calcium restric-
thereby raise the oxalate pool available for absorp- tion have a limited role in the medical treatment of
tion. uric acid stone patients.

3. A.
The initial goals of medical management are to
Case 3

rehydrate and reverse metabolic acidosis. Hydra- 6. C.


tion is at times difficult in some patients as an Ascending urinary tract infections with urea-split-
increase in oral fluids may exacerbate diarrhea. ting organisms, such as Proteus species, will metab-
Hydration and potassium citrate will contribute to olize urea to ammonia. Ammoniuria, in conjunction
the reversal of the metabolic acidosis, as well as with a matrix composed of organic compounds, car-
enhance the excretion of citrate to increase its bonate apatite, inflammatory cells and bacteria,
inhibitory effects on stone formation. Calcium sup- results in the rapid formation of an “infection” cal-
plements will bind excess oxalate within the intes- culus, eventually progressing into a mineralized,
tine thereby reducing intestinal oxalate absorption. dense stone. Bacteria trapped within the stone per-
Calcium citrate may offer an ideal calcium supple- petuate the recurrent urinary tract infections and
ment in this condition as it should reduce urinary further stone formation, eventually developing into
oxalate and increase urinary citrate. Thiazides may the classic staghorn calculus.

CHAPTER 13: MEDICAL MANAGEMENT OF UROLITHIASIS 411


7. C.
After removal of an infected struvite calculus, the
Case 5

most common cause of recurrent stone formation is 11. C.


failure to completely eradicate the urinary tract Renal tubular acidosis is a clinical syndrome of
infection. Surgical therapy may leave retained frag- chronic metabolic acidosis resulting from renal
ments of infected stone within calyces, thus allow- tubular abnormalities, while glomerular filtration is
ing infection to persist. Underlying metabolic disor- relatively well preserved. Although patients may
ders may also contribute to recurrent stone forma- present with many different symptoms and physical
tion, but persistent infection remains the most findings, renal stone formation is a well-recognized
important risk factor. manifestation of distal renal tubular acidosis
(dRTA). Patients with the incomplete form of dRTA
8. D. are not persistently acidemic despite their inability
Acetohydroxamic acid (AHA), a competitive to lower urinary pH with an acid load. These
inhibitor of the bacterial enzyme urease, will reduce patients are able to compensate for their acidifica-
the urinary saturation of struvite and retard stone tion defect and remain in acid-base balance by
formation. When given at a dose of 250 mg orally increasing ammonia synthesis and ammonium
TID, this medication can prevent the recurrence of excretion as a buffering mechanism. The initial
new stones and inhibit the growth of existing stones identification of incomplete dRTA is often a chance
in patients with chronic urea-splitting infections. finding. Many of these patients will present with
AHA can also cause dissolution of small stones. recurrent nephrolithiasis or may be referred for
However, up to 30% of patients will experience evaluation after the discovery of nephrocalcinosis
minor side effects, including headache, nausea, after routine abdominal radiographs. Most patients
vomiting, anemia, rash or alopecia. In addition, will have normal serum electrolytes, yet they will
15% of patients have developed deep venous throm- have a high-normal urine pH along with significant
bosis while on long-term treatment. Therefore, care- hypocitraturia. The diagnosis of incomplete dRTA
ful monitoring is required when using this medica- can be confirmed by inadequate urinary acidifica-
tion. tion after an ammonium chloride loading test.

Case 4 Case 6

9. E. 12. A.
Cystinuria is a complex autosomal-recessive disor- The basic abnormality in absorptive hypercalciuria
der of amino acid transport involving cystine, type I is the intestinal hyperabsorption of calcium.
ornithine, lysine and arginine. Supersaturation of The consequent increase in the circulating concen-
the urine will occur in patients with the homozygous tration of calcium enhances the renal filtered load
state. Therefore, it is unusual to see a family history and suppresses parathyroid function. Hypercalci-
with cystine stones. The age of onset is often in the uria results from the combination of increased fil-
1st or 2nd decade of life. tered load and reduced renal tubular reabsorption of
calcium, a function of parathyroid suppression. The
10. D. excessive renal loss of calcium compensates for the
D-penicillamine and alpha-mercaptopropionyl- high calcium absorption from the intestinal tract and
glycine are equally effective in their ability to helps to maintain serum calcium in the normal
decrease urinary cystine levels. However, studies range.
have demonstrated that alpha-MPG is significantly
less toxic than D-penicillamine. Moreover, the side
effects that may occur with alpha-MPG are also less
severe. However, if a patient has been doing well on
D-penicillamine with no significant complications,
there is no need to switch medications.

412 EDUCATIONAL REVIEW MANUAL IN UROLOGY


Chapter 14:
Surgical Management
of Stones
Hassan Razvi, MD, FRCSC
Andrew Fuller, MBBS, FRACS

Contents

1. Shock Wave Lithotripsy

2. Ureteroscopy

3. Percutaneous Stone Treatment

4. References

5. Questions

CHAPTER 14: SURGICAL MANAGEMENT OF STONES 413


1. Shock Wave Lithotripsy (SWL)

described in this chapter depends on a number of


factors, including careful preoperative planning,
Introduction

Despite an improved understanding of the patho- interpretation of radiological investigations and the
physiological mechanisms underlying urinary stone availability of specialized instrumentation to effec-
disease, the prevalence of this condition continues tively delineate and negotiate the urinary tract.
to increase.1 The lifetime risk of stone disease for Many cases are likely to require improvization and
individuals residing in the USA has been estimated the ability to safely apply a range of techniques in
at 10%–15%.2,3 In 2000, direct treatment costs were combination to achieve the desired outcome.
estimated to exceed $2.1 billion.4 This reflects both
an increase in the prevalence of the condition as
well as the emergence of novel minimally invasive
Shock Wave Lithotripsy

treatment options. After its initial introduction in the early 1980s, SWL
continues to be widely utilized in the management
Practice patterns within the USA have been cap- of urolithiasis. It represents an effective, minimally
tured by several databases, including Healthcare invasive management option in well-selected
Cost and Utilization Project (HCUP), Centers for patients. In most cases, SWL can be performed
Medicare and Medicaid Services (CMS), Centre for under neurolept anesthesia as an outpatient proce-
Health Care Policy and Evaluation (CHCPE) and dure with a low risk of major complications.
National Hospital Ambulatory Medical Care Sur-
vey (NAMCS). A total of 617,647 individuals pre- All lithotripters have 4 features in common:
sented to an emergency room with a listed primary
diagnosis of urolithiasis in 2000.4 This amounts to 1. An energy source
an estimated rate of 226 cases per 100,000 individu-
als. The increased availability and acceptance of 2. A coupling medium
minimally invasive techniques such as shock wave
lithotripsy (SWL) and ureteroscopy led to an 3. A mechanism to focus the generated
increase in ambulatory surgery between 1994 and shock wave
1998 with rates of 123/100,000 and 199/100,000,
respectively. Accordingly, the mean length of hospi- 4. An imaging system for stone
tal stay decreased for upper tract stones from 2.6 to localization (fluoroscopy/ultrasound)
2.3 days during the same time period.4

A reduction in open surgical procedures corre-


Mechanism of Action

sponded with a 60% increase in ureteroscopic man- There are many lithotripters available with different
agement for the period between 1992 and 2000. The mechanisms of shock wave generation (electromag-
rates of Percutaneous Nephrolithotomy (PCNL) netic, electrohydraulic and piezoelectric), coupling
(3%–6%) and SWL (49%–54%) remained stable.5 and stone localization. Each has in common the
Although more recent data are lacking, it is likely ability to extracorporeally produce a shock wave at
that the proportion of cases performed ureteroscopi- the F2 focal point with a characteristic waveform
cally has continued to increase with the advent and consisting of a sharp peak in positive pressure fol-
dissemination of improved optical systems and lowed by a negative (tensile) wave (Figure 1). The
ancillary devices. precise mechanisms by which this results in stone
fragmentation remain under investigation, but may
This chapter focuses on the three most commonly be generally classified as direct stress (compressive
utilized treatment modalities used in the surgical and shear-induced) and cavitation. Direct stress is
management of urolithiasis. As with any form of induced by the positive component of the wave-
surgical management, definitive treatment for stone form. In comparison, cavitation relies on the nega-
disease should be achieved in an efficient manner, tive pressure (tensile) component of the shock
whilst minimizing invasiveness and morbidity. Suc- wave, which causes microbubbles to form at the
cessful treatment using each of the modalities stone/fluid interface.6 Collapse of these bubbles

414 EDUCATIONAL REVIEW MANUAL IN UROLOGY


results in the release of energy and stone fragmenta-
tion. Although cavitation is necessary to achieve
Contraindications

stone fragmentation, the formation of microbubbles There are a range of patient and stone-related vari-
in renal parenchyma and blood vessels has been ables that impact upon the success of SWL. Many of
proposed as an important mediator of SWL-induced these factors (e.g., obesity, stone composition/size,
tissue injury.7 anomalous anatomy) represent relative contraindi-
cations to SWL. Table 1 outlines the absolute con-
Indications traindications to treatment

The American Urological Association (AUA) pub- Table 1


lished guidelines in 2007 with regard to the manage-
ment of ureteral calculi.8 SWL was recommended as Absolute Contraindications to SWL
an acceptable first-line treatment option for ureteral
stones which failed or were deemed unsuitable for
medical expulsive therapy, albeit with a signifi- Pregnancy
cantly inferior stone-free rate (SFR) compared with
ureteroscopy (URS). The decision of whether to Bleeding diathesis
proceed with SWL or URS in this context should be
made by an informed patient who is aware of the Distal obstruction
SFR, reintervention rates and expected complica-
tions of each treatment modality. In general terms, Calcified renal artery aneurysm
the benefit of URS in this context in terms of higher
SFR needs to be balanced against the risk of a more Untreated urinary tract infection
invasive procedure with slightly higher complica-
tion rates.

Figure 1 Peri-Procedural Assessment

A thorough history and physical examination should


be completed prior to SWL. In particular, one
SWL waveform characterized by a steep

should exclude the presence of any absolute con-


rise in positive pressure followed by a

traindication to treatment. A complete list of pre-


negative (tensile) component

scribed and over-the-counter medications should be


reviewed to identify anticoagulant and antiplatelet
agents that may increase the risk of hemorrhagic
complications.

Uncorrected coagulopathy is generally considered


an absolute contraindication to SWL. Life-threaten-
ing hemorrhages have been reported in patients
undergoing SWL who continued their antiplatelet
agent.9 The management of patients on antiplatelet
agents who require shock wave lithotripsy has
become more pertinent than ever with the increasing
use of these agents. A recent review of SWL in
patients requiring anticoagulation or antiplatelet
Reprinted with permission from Coleman AJ et al. Ultra- agents attempted to provide clarity on this issue.10
sound Med Bio1987;13:651-7
Unfortunately, there are no prospective randomized
trials assessing patients undergoing SWL requiring
the concomitant use of anticoagulants. Guidelines

CHAPTER 14: SURGICAL MANAGEMENT OF STONES 415


on the optimal time for cessation of these agents contrast enhanced computed tomography (CT) by
specifically as it relates to lithotripsy at this point in measuring 3 distances from the stone to the skin at
time are based on expert opinion. Where possible, 0o, 45o and 90o 14 (Figure 2). The mean of these val-
such agents should be discontinued prior to treat- ues represents the calculated SSD. More recent
ment. In patients at high risk of thromboembolic studies have confirmed these findings for renal cal-
events, URS and laser lithotripsy may be a more culi independent of location within the renal collect-
appropriate treatment modality as it is associated ing system.12,15 The negative impact of high body
with a lower bleeding risk.10 mass index (BMI) appears to persist for ureteric cal-
culi. For both renal and ureteric calculi, BMI is an
Investigations should include urine microscopy and independent predictor of SWL failure.11,16
culture, complete blood picture, coagulation stud-
ies, electrolytes and creatinine. A kidney, ureter and
bladder (KUB) x-ray should ideally be performed
Figure 2

on the day of the procedure given the possibility of


stone migration and passage. An x-ray will also
Method of SSD calculation based on an

confirm the radiopaque nature of calculi, which


axial CT image – mean distances of mea-

allows for fluoroscopic localization at the time of


surements taken at 0°, 45° and 90°

therapy.

Techniques to Optimize Outcome

As for any form of surgical intervention, outcomes


for SWL are to a large extent dependent on appro-
priate case selection. One should aim to maximize
SFR whilst minimizing renal damage and the poten-
tial for acute and longer term complications.
Although there are few absolute contraindications
to SWL, a number of anatomical, technical and
stone-related factors have been shown to be useful Reprinted with permission from Pareek G et al. Urology
in predicting the outcome and have been incorpo-
Stone-Related Factors
2005;66:941-4

rated into clinical nomograms to assist with deci-


sion making prior to SWL.11 Such nomograms may A number of stone-related factors including size,
help to avoid unnecessary exposure to ionizing radi- number, location, composition and density are
ation and shock waves where the likelihood of suc- known to influence the outcome of SWL.17 SWL is
cess is very low. not recommended for renal stones >20 mm, given
the low SFR and higher rates of reintervention rela-
tive to PCNL in this context.18,19 For ureteral stones
>10 mm, both SWL and URS are considered accept-
Patient Selection

Obesity able treatment options; however, it should be recog-


Attenuation of shock wave energy by adipose tissue nized that URS is associated with significantly
in the context of obesity has been associated with higher SFR and lower rates of reintervention at the
inferior outcomes with SWL.12 The utility of SWL expense of slightly higher complication rates.8 The
in obese individuals is also limited by table weight presence of multiple calculi also predicts an inferior
restrictions and difficulty with stone localization.13 outcome with SWL.18
The concept of skin-to-stone distance (SSD) was
first proposed in the context of lower pole calculi.14 The likelihood of treatment success with SWL can
This study demonstrated a significantly increased be further classified by stone location. In a prospec-
likelihood of residual calculi with a SSD in excess tive, randomized trial comparing SWL and PCNL
of 10 cm. As outlined by Nakada and colleagues, for treatment of lower pole renal stones, Albala and
SSD may be calculated from axial images of a non- associates demonstrated a SFR of only 21% with

416 EDUCATIONAL REVIEW MANUAL IN UROLOGY


SWL for stones larger than 1 cm.19 For lower pole ated with a higher incidence of stone formation due
stones <1 cm in size, Pearle and colleagues20 com- to urinary stasis. In both cases, impaired drainage
pared flexible URS and SWL. No statistically sig- limits the clearance of fragments following SWL.
nificant difference was shown between the 2 treat- For small calculi, SFR of between 50%–79% have
ment modalities. Despite the smaller stone size in been reported in horseshoe kidneys.28-30 For larger
this study, the SFR for SWL remained low at 35%. (>2 cm) stones, PCNL is the preferred modality of
With increased availability and utility of CT in the treatment with SFR of up to 89%.31 PCNL is also the
assessment of urolithiasis, stone attenuation has preferred approach for CD with SFR of 77%–89%
been identified as an important predictor of success and low complication rates.32 PCNL also provides
with SWL.12,21-23 While it is well recognized that access to the diverticulum to allow ablation of the
stones such as calcium oxalate monohydrate, diverticular sac.
brushite and cystine stones fragment less with SWL,
the absence of a stone analysis in most index cases
of stone disease limit the extent to which stone com-
Outcomes

position can be used to guide therapeutic decision Stone Free Rates


making. CT attenuation (measured as Hounsefeld The outcomes of SWL are affected by a combina-
units) may act as a surrogate marker of stone com- tion of patient, stone and anatomical factors as out-
position. Stones with an attenuation value exceed- lined above. In a meta-analysis which considered
ing 900–1,000 HU are associated with significantly outcomes following SWL for stones in the proxi-
inferior SFR and higher rates of reintervention mal, mid and distal ureter, SFR were 82%, 73% and
when treated with SWL.12,21 It has been suggested 74%. Predictably, SFR were lower for stones >10
that patients should be counseled toward an alterna- mm in size. As outlined above, the results show
tive therapeutic modality in this context. more variability for renal calculi, with large stones
located in the lower pole calyx associated with the
Anatomical Variables poorest outcomes and smaller renal pelvic and
Several investigators have evaluated the impact of upper pole calyceal stones responding most favor-
variations in the lower pole infundibulopelvic ably to SWL.19
anatomy (infundibulopelvic angle, infundibular
length, infundibular width) on outcomes following Complications
SWL with mixed results.24-26 An accurate assessment In experienced hands and with appropriate attention
of calyceal anatomy requires the administration of to patient selection, the risk of major complications
contrast which is otherwise not part of routine imag- in association with SWL is low. Complications may
ing protocols prior to SWL. It has been proposed be classified as:
that an infundibulopelvic angle >70o, infundibular
length <3 cm and infundibular width >5 mm may be 1. Stone related
associated with superior outcomes.27 Given the dis-
crepancy in the studies performed to date, no clear 2. Infectious
guidelines currently exist to direct therapy based on
these anatomical factors. 3. Hemorrhagic

In the context of hydronephrosis caused by an When stone fragmentation is insufficient to allow


obstruction distal to the level of the stone, SWL is spontaneous passage, there is a risk of both Stein-
contraindicated. Treatment in this setting is unlikely strasse (4%–7%) and retained fragments. The risk
to result in stone clearance due to the inability for of these complications is related to stone size and
stone fragments to pass. Additionally, both URS number, stone composition, location, renal mor-
and PCNL afford the opportunity to directly visual- phology as well as shock wave rate and energy.17,33
ize and if necessary relieve the point of obstruction. Small residual fragments (<5 mm) are often conser-
vatively managed following SWL on the assump-
Common anatomical variants such as horseshoe tion that they will pass spontaneously. While this is
kidney and calyceal diverticulum (CD) are associ- true in many cases, such fragments may act as a

CHAPTER 14: SURGICAL MANAGEMENT OF STONES 417


nidus for both infection and stone growth. As the improvements in SFR relative to controls. Despite
number and size of residual fragments increase, the these findings, this technique is not in widespread
risk of associated complications also rises.34 In use.
patients for whom conservative management is rec-
ommended, careful follow-up is mandatory to allow Medical Expulsive Therapy
for timely adjuvant therapy as required. The risk of Recognition that alpha blockers may augment the
requiring intervention for such patients has been passage of ureteral stones has lead to a number of
estimated to be as high as 59%.35 studies in the context of SWL. Zhu and colleagues45
conducted a meta-analysis regarding the role of
The risk of infectious complications following SWL alpha blockers following SWL. Unfortunately, the
is increased by a number of factors including posi- studies included in this analysis were characterized
tive preoperative urine cultures, history of recurrent by considerable heterogeneity and the lack of a
urinary tract infections (UTI), struvite stones and placebo control group. Despite this, medical expul-
the presence of foreign bodies such as ureteric stents sive therapy was associated with increased SFR,
and indwelling urinary catheters.17 Bacterial and reduced time to stone passage and reduced anal-
fungal infections limited to the urinary system at the gesic requirements.
time of therapy may cause systemic sepsis due to
hematogenous spread facilitated by tissue damage Ureteric Stents
and endothelial disruption induced by SWL.17 The There appears to be no role for routine ureteric stent
risk of such complications may be minimized by placement prior to SWL for ureteral calculi. Mid-
ensuring the urine is sterile prior to SWL. dela and associates46 described a series of 342
patients with ureteric calculi <1 cm assigned to
Symptomatic perinephric hematoma (PNH) is either nephrostomy tube, ureteric stent or no decom-
uncommon following SWL. Fortunately, PNH can pression tube prior to SWL. No significant differ-
be managed conservatively in most cases.33 In a ence was demonstrated between the 3 groups. The
recent prospective comparison of the modified indications to stent prior to SWL include the pres-
HM3 and the Storz Modulith SLX-F2, the per- ence of high-grade obstruction and large stone bur-
inephric hematoma rates were 1% and 3%, respec- dens where steinstrasse is a significant risk.
tively.36 A number of potential risk factors for bleed-
ing post-SWL have been proposed, including hyper- Forced Diuresis
tension, obesity, diabetes mellitus, coagulative dis- It has been proposed that inducing a diuresis prior to
orders, shock wave frequency, number and SWL may create a fluid space around the calculi and
energy.33,37-39 allow better transmission of shock wave energy.
Tiselius and colleagues47 described the administra-
The association between SWL and an increased risk tion of intravenous fluid and furosemide prior to
of both hypertension (HTN) and diabetes mellitus SWL. This study demonstrated no significant bene-
(DM) remains controversial, with several studies fit in terms of SFR, retreatment rate or number of
providing conflicting results.40-42 In a recent retro- shock waves administered.
spective cohort study by Chew and colleagues,43 the
20-year prevalence of both HTN and DM was not Shock Wave Rate
increased after treatment with the Dornier HM-3 Several investigators have evaluated the outcomes
device. related to differences in shock wave rate.48,49 While
most treatments are administered at a rate of 120
shocks per minute, it has been shown that reducing
the rate to 60 per minute is associated with a higher
Techniques to Optimize Outcome

Mechanical Percussion and Inversion rate of stone clearance and a reduced retreatment
The role of mechanical percussion and inversion rate at the expense of a longer duration of treatment.
therapy (MPI) has been described as a useful It appears that the benefit of this technical modifica-
adjunct to SWL in the management of lower pole tion is most apparent for larger stones.50
calculi.44 MPI is associated with significant

418 EDUCATIONAL REVIEW MANUAL IN UROLOGY


2. Ureteroscopy

Voltage Escalation The proportion of stones managed with


Protocols incorporating a gradual escalation in ureteroscopy in North America continues to
shock wave energy have been suggested to improve increase with the advent and dissemination of
stone fragmentation and reduce renal injury. Data to improved optical systems, intracorporeal lithotrip-
support the routine use of voltage escalation are tors and ancillary devices. With appropriate training
conflicting. Honey and colleagues51 found immedi- and equipment, URS can be used to safely access
ate escalation to be associated with more rapid stone the entire ureter and intrarenal collecting system. In
fragmentation with no increase in morbidity. recognition of this rapid evolution in ureteroscopic
Retreatment rates and SFR were not significantly equipment and technique, the combined EAU/AUA
different relative to gradual voltage escalation nephrolithiasis panel now recognizes both URS and
strategies. Conversely, Lambert52 demonstrated SWL as first-line treatment options for ureteral
superior SFR in association with gradual voltage stones.8
escalation and some evidence of reduced renal
injury. Indications

With the advent of flexible instrumentation, the use


of ureteroscopy has expanded from the treatment of
distal ureteric calculi to incorporate stone disease
throughout the entire intrarenal collecting system
and ureter. Relative to SWL, URS is associated with
higher SFR and reduced rates of reintervention.
These improvements in stone-related outcomes
come at the cost of increased invasiveness and a
higher risk of complications.

For ureteric calculi, both SWL and URS are accept-


able treatment options. Stone localization for SWL
in the distal ureter may be challenging due to the
limits imposed by the bony pelvis. URS should be
favored where there has been previous SWL failure,
stones >1 cm, obesity, radiopaque stones, and for
individuals who are employed in occupations where
the certainty of the final results is important. Many
of these factors apply equally to stones within the
intrarenal collecting system. Additionally, one may
favor URS over SWL for renal calculi where the
stone is located within the lower pole calyx as well
as in the context of comorbidities, such as morbid
obesity, musculoskeletal deformity, renal artery
aneurysm and bleeding diathesis. The likely stone
composition based on CT attenuation values and
previous stone analysis should also be considered.

CHAPTER 14: SURGICAL MANAGEMENT OF STONES 419


Peri-Procedural Assessment Figure 3

Patients should be thoroughly evaluated with his-


tory and physical examination prior to URS. Urine
Deflection of a flexible ureteroscope to

microscopy and culture should be sent to ensure the


access and fragment stones in the lower

urine is sterile. Culture-specific antibiotic therapy


pole calyx

should be administered in the event that a UTI is


demonstrated.

CT has a high sensitivity and specificity and is the


favored modality for preoperative imaging. It
allows an assessment of stone burden, location and
attenuation. It is imperative that the CT is assessed
thoroughly, particularly with regard to anatomical
anomalies such as ureteral duplication, renal
ectopia, horseshoe kidney and calyceal diverticu-
lum. Failure to appreciate such factors preopera-
tively may lead to the erroneous conclusion that a
stone has passed or is inaccessible with potential
clinical consequences.

Equipment Reprinted with permission from Beiko DT et al. Urol Clin


North Am 2007;34:397-408.

Both semirigid and flexible ureteroscopes continue Fortunately, improvements in the mechanics and
to evolve rapidly, particularly with regard to optics. optics of ureteroscopes have corresponded with
Charged coupled devices (CCD) have recently been advances in devices available for intracorporeal
incorporated onto the distal end of both semirigid lithotripsy. One of the most significant recent
and flexible ureteroscopes. When used in combina- advances in endourology has been the widespread
tion with a digital control box, digital ureteroscopes adoption of the holmium:yttrium-aluminium-garnet
obviate the need for a separate camera and light (Ho:YAG) laser. This is now recognized as the gold
source and produce a large, high resolution image standard means of achieving stone fragmentation
free of the honeycomb effect characteristically seen with both rigid and flexible URS and, where avail-
with the early fiberoptic devices. able, has replaced electrohydraulic and pneumatic
devices in this context.
Access to all areas within the intrarenal collecting
system has improved with developments in the Where there is a reliance on rigid ureteroscopy and
deflection mechanism of flexible scopes. This has pneumatic lithotripsy due to limited resources,
taken the form of both improved primary deflection stone retropulsion may negatively affect SFR.
and the incorporation of secondary deflection mech- Ureteral occlusion devices such as the Stone Cone
anisms. In particular, stones within the lower pole (Boston Scientific Corp) (Figure 4) and N-Trap
calyx may be visualized and fragmented (Figure 3). (Cook Urological) (Figure 5) have been designed
In many cases, repositioning of lower pole stones for use in this context and have been shown to
into the upper pole or renal pelvis may simplify increase SFR, reduce ureteral wall trauma and the
access and fragmentation. need for reintervention.53

420 EDUCATIONAL REVIEW MANUAL IN UROLOGY


Ureteral access sheaths (Figure 6) have been
described as a means of facilitating repeated upper
Figure 4

tract access, reducing intrarenal pressures and


improving visibility. In a retrospective series of 256
Stone cone ureteral occlusion device

patients, L’Esperance and associates demonstrated


significantly improved SFR for renal stones man-
aged by flexible URS using an access sheath.54

Stent placement is not always necessary after rou-


tine URS.55 A number of factors may necessitate
stent insertion, including the need for balloon dilata-
tion, ureteral perforation, residual calculi, bleeding,
Image courtesy of Boston Scientific Corporation. Opinions ureteric stricture and solitary kidney.
expressed are those of the author alone and not of Boston
Scientific
Outcomes

Complications
Improvements in ureteroscope design and ancillary
devices have corresponded with significant reduc-
Figure 5

tions in the rates of both intraoperative and postop-


erative complications. Ureteroscopy is associated
N-trap ureteral occlusion device

with a ureteral perforation rate of <5% and a long-


term stricture rate of <2%.8,56

Serious intraoperative complications, such as


ureteral perforation and avulsion, can be prevented
in most cases by adhering to the principles of safe
ureteroscopy, including the establishment and main-
tenance of guidewire access and careful, non-force-
ful passage of instrumentation under direct vision.
Blind basketing techniques have no role whatso-
Permission for use granted by Cook Medical Incorporated,

ever.
Bloomington, Indiana

In cases where difficulty is experienced obtaining


Figure 6
access to the targeted stone, one should consider
placement of a ureteric stent as a means of achieving
Navigator access sheath
passive ureteral dilatation and facilitating future
access.

Stone-free Rates
Stone-free rates after URS depend on a range of
patient and stone related factors. For ureteral calculi
81%–94% of patients are rendered stone free.8 In the
context of multiple renal calculi, flexible URS is
associated with single session stone-free rates of
approximately 65%, with 92% of patients stone-free
after 2 treatment sessions.57
Image courtesy of Boston Scientific Corporation. Opinions
expressed are those of the author alone and not of Boston
Scientific

CHAPTER 14: SURGICAL MANAGEMENT OF STONES 421


3. Percutaneous Stone Treatment

Additionally, one may assess the relationship of the


planned site of access to surrounding structures
Percutaneous Nephrolithotomy (PCNL)

Fernstrom and Johansson67 were the first to describe including colon, liver, spleen and pleura. This is
the technique of establishing percutaneous access to particularly important in the context of previous
the renal collecting system for the purposes of stone retroperitoneal surgery, where the risk of retrorenal
removal. Since this initial description in 1976, the colon is higher (Figure 7).
technique has evolved to represent the gold standard
for treatment of renal stones >2 cm in size. When
performed by an experienced urologist, PCNL is
Technique of PCNL

associated with high SFR and low rates of major Anatomical considerations
complications.68,69 A thorough appreciation of renal collecting system
and vascular anatomy is essential in determining the
most appropriate site of access. The main renal
artery divides into anterior and posterior divisions.
Indications

Although a range of management options including The anterior segmental branch supplies the anterior
SWL, URS, open and laparoscopic pyelolithotomy and polar areas. The posterior branch supplies the
have been described in the context of large renal cal- remainder of the posterior surface. In this knowl-
culi, PCNL is the most appropriate first-line therapy edge, one can appreciate that medial puncture risks
in most cases.8 Table 2 outlines the common indica- damage to the posterior segmental artery. In addi-
tions for PCNL. tion, entering the calyx in the correct orientation
(end on rather than side on) avoids the interlobar
arteries which cross the infundibula. One must enter
via a posteriorly oriented calyx to allow adequate
Peri-Procedural Assessment

Patient assessment prior to PCNL should incorpo- access to the collecting system.
rate history and physical examination as well as lab-
oratory and radiological investigations. History and Equipment
examination should be directed to highlight factors To achieve fluoroscopic-guided access, a catheter
such as bleeding diathesis, anticoagulant therapy, must be inserted in a retrograde fashion through
recurrent UTI, chronic obstructive pulmonary dis- which contrast can be injected to opacify the col-
ease and morbid obesity, all of which may signifi- lecting system. C-arm fluoroscopy is essential. Ini-
cantly increase the risk of perioperative complica- tial access is achieved with an 18-gauge needle
tions. One should also assess for the presence of uri- through which a guidewire may be inserted once the
nary tract anatomical anomalies. Musculoskeletal needle has entered the desired calyx. The guidewire
disorders resulting in contractures and scolio- may be directed down the ureter with the assistance
sis/kyphosis may present challenges in achieving of an angled angiographic catheter. Tract dilation
appropriate patient positioning to facilitate access. should be performed over an extra stiff wire using
either a sequential technique with Amplatz dilators
Laboratory investigations should include complete or a balloon device. Once dilation has occurred, a
blood count, group/reserve, electrolytes, creatinine working sheath is advanced through which rigid and
and urinalysis/culture. Even in the context of a neg- flexible nephroscopy can be performed.
ative preoperative urine culture, there is evidence
that the use of prophylactic fluoroquinolones Patient Position
reduces the risk of septic complications following PCNL is traditionally performed in a prone position,
surgery.70 although more recent studies have suggested that
supine positioning may offer some benefit, particu-
Cross-sectional imaging in the form of non-contrast larly in patients at high anesthetic risk or where con-
CT has largely replaced intravenous pyelogram in current retrograde ureteral access is required. Val-
the assessment of urolithiasis. CT affords the oppor- divia71 considered outcomes related to prone and
tunity to assess the renal collecting system and plan supine positioning in a large series of 5,803 patients.
appropriate sites of access prior to the procedure. Prone positioning was associated with shorter oper-

422 EDUCATIONAL REVIEW MANUAL IN UROLOGY


Table 2 Figure 7

Indications for PCNL CT demonstrating retrorenal colon

Staghorn calculi

Stones >2cm in size

Lower pole stones > 1 cm

Cystine stones

Patients who must be stone free (Pilots)

Failure of other treatments


Reprinted with permission from Ko R et al. BJU Int
2008;101:535-9
Associated anatomical anomalies
(UPJ obstruction, calyceal diverticulum, horseshoe anticoagulant medications.75
kidney)
Stone Fragmentation and Retrieval

A number of intracorporeal lithotripters are avail-


able, including pneumatic, ultrasound, electrohy-
ative duration, higher SFR and lower rates of draulic (EHL) and laser. While each of these
failed access. devices is effective in achieving stone fragmenta-
tion, ultrasound and pneumatic devices should be
Access used where possible due to their efficiency. The
The technique of collecting system access has ultrasound device has the added benefit of aspirat-
been well described previously.72 Once the renal ing small fragments during fragmentation. Irrespec-
collecting system has been opacified with con- tive of the device used, one should aim to fragment
trast, the most appropriate calyx is chosen for the stone sufficiently to allow removal of stone frag-
puncture. Particularly in the case of staghorn cal- ments with a rigid nephroscope and graspers where
culi and large calculi at the ureteropelvic junction possible to maximize efficiency. When multiple
(UPJ), an upper pole access is often desirable, stones are located peripherally within the renal col-
despite the higher risk of pulmonary complica- lecting system, flexible nephroscopy in combina-
tions associated with this approach.73 tion with EHL or laser lithotripsy and basket
retrieval may become necessary.
Tract dilation can be safely achieved with either
Amplatz serial dilators or a balloon device. One-
stage balloon dilation is quicker than the use of
Postoperative Drainage

Amplatz dilators, although it has been suggested The indications for placement of a nephrostomy
by Lopes and colleagues74 that this technique may tube are listed in Table 3. A nephrostomy tube in
pose a higher risk of hemorrhagic complications. each of these scenarios aids in the healing of the
In a follow-up study conducted on behalf of the nephrostomy tract, promotes hemostasis, prevents
Clinical Research Office of the Endourological urinary extravasation, drains purulent material and
Society (CROES), no such association was found allows re-entry in the event that second-look
on multivariate analysis taking into consideration nephroscopy is indicated. Several investigators
factors such as previous surgery, stone location, have considered the feasibility and safety of omit-
stone size, patient comorbidities and the use of ting the placement of a drainage tube (totally tube-

CHAPTER 14: SURGICAL MANAGEMENT OF STONES 423


less PCNL) or placing an internal ureteric stent
when none of the above mentioned indications are
Special Scenarios

present (tubeless PCNL).76,77 In a meta-analysis Staghorn Calculi


comparing tubeless and standard techniques, Wang PCNL is the preferred management option for man-
and associates analyzed 7 studies incorporating aging both partial and complete staghorn calculi.
results from 1365 procedures and demonstrated that The successful percutaneous management of such
tubeless PCNL is associated with reduced length of calculi depends on sound technique, surgeon expe-
stay and analgesic use with no significant increase rience and the availability of a range of endourolog-
in hemorrhagic complications in appropriately ical equipment.79 In the largest published single sur-
selected patients.76 geon series to date, Desai described outcomes
related to 834 procedures over a 17-year period. The
overall SFR in this series was 86%. With the evolu-
tion of both technique and equipment, improve-
Outcomes

PCNL has been demonstrated to be an effective and ments were noted throughout the series in terms of
safe procedure in multiple studies. It is associated operative duration, blood loss, reintervention rate,
with high SFR and low rates of major complica- number of tracts, complications, length of hospital
tions.68,69 stay and SFR.

Complications Obesity
In a review of 5,803 PCNL procedures performed at In 2010, 80.5% of males and 76.5% of females in
96 centers worldwide between November 2007 and the USA were overweight or obese.80
December 2009, de la Rosette and colleagues69
found an overall complication rate of 14.5%. Only In addition to increasing the difficulty and limiting
4.1% of patients developed a major complication the therapeutic options for existing stones, obesity is
(Clavien III, IV, V). The most common complica- closely associated with comorbid health conditions
tions included bleeding (7.8%), perforation (3.4%) such as diabetes mellitus, hypertension and the
and hydrothorax (1.8%). Although the transfusion metabolic syndrome which have been implicated in
rate was 5.7% in this series, other investigators have the increased prevalence of urolithiasis.81-84 Obesity
demonstrated rates as low as 0.8%.68 has been shown to negatively impact upon urinary
parameters, with an increase in the excretion of
When bleeding persists after PCNL despite conser- lithogenic substances including calcium, oxalate,
vative measures, one should have a low threshold to sodium and uric acid.85,86
further investigate with angiography. Selective
embolization can be performed concurrently when a A number of anesthetic concerns may arise in obese
pseudoaneurysm is identified (Figure 8). patients undergoing PCNL in the prone position,
including the potential for reduced total lung capac-
Stone-free Rate ity and functional residual capacity with abdominal
SFR following PCNL are influenced by a range of compression as well as IVC compression with
factors, including stone size, location, multiplicity, resultant reduced preload and impaired oxygena-
patient comorbidities and anatomical anomalies. tion. A number of strategies have been proposed to
Several large studies have evaluated stone-related overcome such difficulties, including lateral decubi-
outcomes following PCNL, with overall SFR tus and supine positioning, awake endotracheal
between 75% and 89%.68,69,78 In a series of 5,803 pro- intubation with patient self-positioning and the per-
cedures, 85% of patients required no further surgi- formance of PCNL under local anesthesia and seda-
cal intervention following primary PCNL. Second- tion.87-89
look nephroscopy was required in 6.9%. Adjuvant
therapy in the form of SWL or URS was utilized in In a series of 3,709 patients stratified by BMI, oper-
8.9% of cases.69 ative time was significantly longer for obese
patients. SFR declined with obesity and a signifi-
cantly higher retreatment rate was seen.90 No differ-

424 EDUCATIONAL REVIEW MANUAL IN UROLOGY


Table 3 Figure 8

Absolute indications for insertion of a Digital renal arteriogram demonstrating a


nephrostomy tube post-PCNL 1.2-cm lobulated pseudoaneurysm arising
from a lower pole segmental artery

Significant collecting system injury

Excessive hemorrhage

Residual calculi

Multiple tracts

Pyonephrosis necessitating reliable


external drainage

ence was demonstrated for length of stay or transfu- puncture into the diverticulum may facilitate cannu-
sion rate. There was no significant difference lation and dilation of the diverticular neck at PCNL.
between the groups with regard to intraoperative or Where cannulation of the diverticular neck cannot
postoperative complications. be achieved, transdiverticular access with creation
of a neoinfundibulum is a useful salvage technique
Horseshoe Kidney (Figure 9).32 PCNL has been associated with SFR of
As a result of an altered configuration of the UPJ up to 89% in this context with no significant differ-
and calyces, horseshoe kidneys predispose to stone ence in perioperative complications.32 Patients with
formation. Both SWL and flexible URS are associ- stones in anterior located diverticuli are not appro-
ated with inferior SFR in this context. Although priate candidates for PCNL. In these cases, open or
SFR as high as 88%91 have been reported, success laparoscopic nephrostomy and marsupialization
depends on the ability to adjust the surgical tech- have been described.92,93
nique to allow for abnormal renal position, calyceal
orientation, vasculature, relationship to surrounding Urinary Diversion
organs and altered UPJ configuration. The calyces The risk of urolithiasis in the context of urinary
are more posteriorly oriented. In many cases, an diversion is increased due to a combination of
upper pole access is preferable to facilitate access to chronic UTI, urinary stasis, foreign bodies and
the collecting system, UPJ and proximal ureter. hyperchloremic metabolic acidosis. PCNL may be
Contrast-enhanced cross-sectional imaging should performed in this context with equivalent SFR,
be reviewed thoroughly prior to any contemplated complications and length of hospital stay compared
intervention to precisely define vascular and vis- to patients with normal lower tract anatomy. Uri-
ceral relations. nary diversion is associated with a higher likelihood
of requiring ultrasound or CT-guided access and
Calyceal Diverticulum increased utility of second-look nephroscopy.94
Calyceal diverticula (CD) are cavities within the
renal parenchyma lined with non-secretory transi-
tional epithelium. Urinary stones may complicate
Special Scenarios

this abnormality in up to 50% of cases. PCNL is


considered the treatment of choice for CD. Direct

CHAPTER 14: SURGICAL MANAGEMENT OF STONES 425


Pregnancy children, albeit with a high retreatment rate of
The optimal approach to the diagnosis and manage- 55%.66
ment of urolithiasis during pregnancy remains con-
troversial. The risk of ionizing radiation, general With the miniaturization of ureteroscopic instru-
anesthesia and surgical intervention to the develop- ments, ureteroscopy has emerged as an effective
ing fetus needs to be balanced against the risks asso- means of ureteral and intrarenal stone management
ciated with a missed diagnosis or complications where appropriate equipment is available. Preopera-
arising due to conservative management. tive stent placement may be required to induce pas-
sive dilation and allow retrograde passage of instru-
There has been a recent shift away from temporiz- mentation.
ing measures, such as ureteral stent or nephrostomy
insertion, toward definitive management such as
ureteroscopic stone extraction. Several studies have
advocated for the use of ureteroscopy during preg-
nancy as a safe, effective treatment option.58,59 Wat-
terson and colleagues demonstrated the feasibility
of URS and laser lithotripsy in pregnant women
with no associated obstetric or urological complica-
tions.60 Conversely, the use of ureteric stents and
nephrostomy tube drainage has been associated
with high rates of repeat interventions due to pain
and encrustation.61,62

Obesity
For obese individuals with small (<2 cm) ureteric
and renal calculi, SWL is often not feasible due to
table weight restrictions, high skin-to-stone dis-
tances and difficulty with stone localization.
Ureteroscopy in this context has been associated
with SFR and complication profiles similar to those
seen in non-obese individuals and should be
employed as the preferred modality of treatment.63-65

Pediatrics
SWL, URS and PCNL have all been described in a
pediatric context. As in adult patients, PCNL is
reserved for large stones or stones which prove
resistant to minimally invasive management
options. The improved SFR with PCNL needs to be
balanced against the increased invasiveness of the
procedure, longer recovery and higher morbidity.

Although not approved by the Food and Drug


Administration (FDA) for management of pediatric
stone disease, SWL is frequently utilized in this
context, as it represents an effective outpatient pro-
cedure with minimal risk and short recovery. In a
study assessing the utility of SWL for stones <2 cm
in size, SWL was associated with a SFR of 85% in

426 EDUCATIONAL REVIEW MANUAL IN UROLOGY


4. References

Figure 9 1. Saigal CS, Joyce G, Timilsina AR, Uro-


logic Diseases in America Project. Direct
When transdiverticular access cannot be and indirect costs of nephrolithiasis in an
employed population: opportunity for dis-
ease management? Kidney Int. 2005;68:
achieved, a Neff set needle may be
advanced through wall of CD into renal col-
lecting system to create an infundibulum 1808-1814.
and allow antegrade passage of a
guidewire 2. Lieske JC, Peña de la Vega LS, Slezak JM,
et al. Renal stone epidemiology in
Rochester, Minnesota: an update. Kidney
Int. 2006;69:760-764.

3. Hesse A, Brändle E, Wilbert D, Köhrmann


KU, Alken P. Study on the prevalence and
incidence of urolithiasis in Germany com-
paring the years 1979 vs. 2000. Eur Urol.
2003;44:709-713.

4. Pearle MS, Calhoun EA, Curhan GC, Uro-


logic Diseases of America Project. Uro-
logic diseases in America project: urolithi-
asis. J Urol. 2005;173:848-857.

5. Kerbl K, Rehman J, Landman J, Lee D,


Reprinted with permission from Méndez-Probst CE et al. J

Sundaram C, Clayman RV. Current man-


Endourol 2011;25:1741-5

agement of urolithiasis; progress or


regress? J Endourol. 2002;16:281-288.

6. Crum LA. Cavitation microjets as a con-


tributory mechanism for renal calculi disin-
tegration in ESWL. J Urol. 1988;140:
1587-1590.

7. Evan AP, Willis LR, McAteer JA, et al.


Kidney damage and renal functional
changes are minimized by waveform con-
trol that suppresses cavitation in shock
wave lithotripsy. J Urol. 2002;168:1556-
1562.

8. Management of ureteral calculi:


EAU/AUA nephrolithiasis panel (2007).
Available at: http://www.auanet.org/
content/clinical-practice-guide-
lines/clinical-guidelines/main-
reports/uretcal07/chapter1.pdf. Accessed
February 10, 2012.

CHAPTER 14: SURGICAL MANAGEMENT OF STONES 427


9. Sare GM, Lloyd FR, Stower MJ. Life- 18. Al-Ansari A, As-Sadiq K, Al-Said S, You-
threatening haemorrhage after extracorpo- nis N, Jaleel OA, Shokeir AA. Prognostic
real shockwave lithotripsy in a patient tak- factors of success of extracorporeal shock
ing clopidogrel. BJU Int. 2002;90:469. wave lithotripsy (ESWL) in the treatment
of renal stones. Int Urol Nephrol.
10. Alsaikhan B, Andonian S. Shock wave 2006;38:63-67.
lithotripsy in patients requiring anticoagu-
lation or antiplatelet agents. Can Urol 19. Albala DM, Assimos DG, Clayman RV, et
Assoc J. 2011;5:53-57. al. Lower pole I: a prospective randomized
trial of extracorporeal shock wave
11. Wiesenthal JD, Ghiculete D, Ray AA, lithotripsy and percutaneous nephros-
Honey RJ, Pace KT. A clinical nomogram tolithotomy for lower pole nephrolithiasis-
to predict the successful shock wave initial results. J Urol. 2001;166:2072-
lithotripsy of renal and ureteral calculi. J 2080.
Urol. 2011;186:556-562.
20. Pearle MS, Lingeman JE, Leveillee R, et al.
12. Perks AE, Schuler TD, Lee J, et al. Stone Prospective randomized trial comparing
attenuation and skin-to-stone distance on shock wave lithotripsy and ureteroscopy
computed tomography predicts for stone for lower pole caliceal calculi 1 cm or less.
fragmentation by shock wave lithotripsy. J Urol. 2008;179:S69-73.
Urology. 2008;72:765-769.
21. El-Nahas AR, El-Assmy AM, Mansour O,
13. Thomas R, Cass AS. Extracorporeal shock Sheir KZ. A prospective multivariate anal-
wave lithotripsy in morbidly obese ysis of factors predicting stone disintegra-
patients. J Urol. 1993;150:30-32. tion by extracorporeal shock wave
lithotripsy: the value of high-resolution
14. Pareek G, Hedican SP, Lee FT Jr, Nakada noncontrast computed tomography. Eur
SY. Shock wave lithotripsy success deter- Urol. 2007;51:1688-1694.
mined by skin-to-stone distance on com-
puted tomography. Urology. 2005;66:941- 22. Joseph P, Mandal AK, Singh SK, Mandal P,
944. Sankhwar SN, Sharma SK. Computerized
tomography attenuation value of renal cal-
15. Patel T, Kozakowski K, Hruby G, Gupta culus: can it predict successful fragmenta-
M. Skin to stone distance is an independent tion of the calculus by extracorporeal shock
predictor of stone-free status following wave lithotripsy? A preliminary study. J
shockwave lithotripsy. J Endourol. Urol. 2002;167:1968-1971.
2009;23:1383-1385.
23. Gupta NP, Ansari MS, Kesarvani P, Kapoor
16. Delakas D, Karyotis I, Daskalopoulos G, A, Mukhopadhyay S. Role of computed
Lianos E, Mavromanolakis E. Independent tomography with no contrast medium
predictors of failure of shockwave enhancement in predicting the outcome of
lithotripsy for ureteral stones employing a extracorporeal shock wave lithotripsy for
second-generation lithotripter. J Endourol. urinary calculi. BJU Int. 2005;95:1285-
2003;17:201-205. 1288.

17. Bach C, Buchholz N. Shock wave 24. Sorensen CM, Chandhoke PS. Is lower
lithotripsy for renal and ureteric stones. pole caliceal anatomy predictive of extra-
Eur Urol. 2011;10:423-432. corporeal shock wave lithotripsy success
for primary lower pole kidney stones?
J Urol. 2002;168:2377-2382.

428 EDUCATIONAL REVIEW MANUAL IN UROLOGY


25. Elbahnasy AM, Shalhav AL, Hoenig DM, 34. Khaitan A, Gupta NP, Hemal AK, Dogra
et al. Lower caliceal stone clearance after PN, Seth A, Aron M. Post-ESWL, clini-
shock wave lithotripsy or ureteroscopy: the cally insignificant residual stones: reality
impact of lower pole radiographic or myth? Urology. 2002;59:20-24.
anatomy. J Urol. 1998;159:676-682.
35. Türk C, Knoll T, Petrik A, Sarica K, Straub
26. Sampaio FJ, Aragao AH. Inferior pole col- M, Seitz C. Guidelines on urolithiasis.
lecting system anatomy: its probable role in European Association of Urology Web site.
extracorporeal shock wave lithotripsy. Available at: http://www.uroweb.org/
J Urol. 1992;147:322-324. gls/pdf/18_Urolithiasis.pdf. Accessed
February 10, 2012.
27. Elbahnasy AM, Clayman RV, Shalhav AL,
et al. Lower-pole caliceal stone clearance 36. Zehnder P, Roth B, Birkhäuser F, et al. A
after shockwave lithotripsy, percutaneous prospective randomised trial comparing the
nephrolithotomy, and flexible modified HM3 with the MODULITH
ureteroscopy: impact of radiographic spa- SLX-F2 lithotripter. Eur Urol.
tial anatomy. J Endourol. 1998;12:113- 2011;59:637-644.
119.
37. McAteer JA, Evan AP. The acute and long-
28. Sheir KZ, Madbouly K, Elsobky E, term adverse effects of shock wave
Abdelkhalek M. Extracorporeal shock lithotripsy. Semin Nephrol. 2008;28:200-
wave lithotripsy in anomalous kidneys: 11- 213.
year experience with two second-genera-
tion lithotripters. Urology. 2003;62:10-16. 38. Micali S, Sighinolfi MC, Grande M,
Rivalta M, De Stefani S, Bianchi G.
29. Serrate R, Regué R, Prats J, Rius G. ESWL Dornier Lithotripter S 220 F EMSE: the
as the treatment for lithiasis in horseshoe first report of over 1000 treatments. Urol-
kidney. Eur Urol. 1991;20:122-125. ogy. 2009;74:1211-1214.

30. Kirkali Z, Esen AA, Mungan MU. Effec- 39. Zanetti G, Kartalas-Goumas I, Montanari
tiveness of extracorporeal shockwave E, et al. Extracorporeal shockwave
lithotripsy in the management of stone- lithotripsy in patients treated with
bearing horseshoe kidneys. J Endourol. antithrombotic agents. J Endourol.
1996;10:13-15. 2001;15:237-241.

31. Shokeir AA, El-Nahas AR, Shoma AM, et 40. Makhlouf AA, Thorner D, Ugarte R,
al. Percutaneous nephrolithotomy in treat- Monga M. Shock wave lithotripsy not
ment of large stones within horseshoe kid- associated with development of diabetes
neys. Urology. 2004;64:426-429. mellitus at 6 years of follow-up. Urology.
2009;73:4-8.
32. Méndez-Probst CE, Fuller A, Nott L, Den-
stedt JD, Razvi H. Percutaneous 41. Sato Y, Tanda H, Kato S, et al. Shock wave
nephrolithotomy of caliceal diverticular lithotripsy for renal stones is not associated
calculi: a single center experience. J with hypertension and diabetes mellitus.
Endourol. 2011;25:1741-1745. Urology. 2008;71:586-592.

33. Skolarikos A, Alivizatos G, de la Rosette J.


Extracorporeal shock wave lithotripsy 25
years later: complications and their preven-
tion. Eur Urol. 2006;50:981-990.

CHAPTER 14: SURGICAL MANAGEMENT OF STONES 429


42. Krambeck AE, Gettman MT, Rohlinger 50. Pace KT, Ghiculete D, Harju M, Honey RJ,
AL, Lohse CM, Patterson DE, Segura JW. University of Toronto Lithotripsy Associ-
Diabetes mellitus and hypertension associ- ates. Shock wave lithotripsy at 60 or 120
ated with shock wave lithotripsy of renal shocks per minute: a randomized, double-
and proximal ureteral stones at 19 years of blind trial. J Urol. 2005;174:595-599.
followup. J Urol. 2006;175:1742-1747.
51. Honey RJ, Ray AA, Ghiculete D, Univer-
43. Chew BH, Zavaglia B, Sutton C, et al. sity of Toronto Lithotripsy Associates,
Twenty-year prevalence of diabetes melli- Pace KT. Shock wave lithotripsy: a ran-
tus and hypertension in patients receiving domized, double-blind trial to compare
shock-wave lithotripsy for urolithiasis. immediate versus delayed voltage escala-
BJU Int. 2011;109:444-449. tion. Urology. 2010;75:38-43.

44. Pace KT, Tariq N, Dyer SJ, Weir MJ, D'A 52. Lambert EH, Walsh R, Moreno MW, Gupta
Honey RJ. Mechanical percussion, inver- M. Effect of escalating versus fixed voltage
sion and diuresis for residual lower pole treatment on stone comminution and renal
fragments after shock wave lithotripsy: a injury during extracorporeal shock wave
prospective, single blind, randomized con- lithotripsy: a prospective randomized trial.
trolled trial. J Urol. 2001;166:2065-2071. J Urol. 2010;183:580-584.

45. Zhu Y, Duijvesz D, Rovers MM, Lock TM. 53. Farahat YA, Elbahnasy AE, Elashry OM.
alpha-Blockers to assist stone clearance A randomized prospective controlled study
after extracorporeal shock wave lithotripsy: for assessment of different ureteral occlu-
a meta-analysis. BJU Int. 2010;106:256- sion devices in prevention of stone migra-
261. tion during pneumatic lithotripsy. Urology.
2011;77:30-35.
46. Middela S, Papadopoulos G, Srirangam S,
Rao P. Extracorporeal shock wave 54. L’esperance JO, Ekeruo WO, Scales CD Jr,
lithotripsy for ureteral stones: do decom- et al. Effect of ureteral access sheath on
pression tubes matter? Urology. stone-free rates in patients undergoing
2010;76:821-825. ureteroscopic management of renal calculi.
Urology. 2005;66:252-255.
47. Tiselius HG, Aronsen T, Bohgard S, et al.
Is high diuresis an important prerequisite 55. Denstedt JD, Wollin TA, Sofer M, Nott L,
for successful SWL-disintegration of Weir M, D'A Honey RJ. A prospective ran-
ureteral stones? Urol Res. 2010;38:143- domized controlled trial comparing non-
146. stented versus stented ureteroscopic
lithotripsy. J Urol. 2001;165:
48. Semins MJ, Trock BJ, Matlaga BR. The 1419-1422.
effect of shock wave rate on the outcome of
shock wave lithotripsy: a meta-analysis. 56. Johnson DB, Pearle MS. Complications of
J Urol. 2008;179:194-197. ureteroscopy. Urol Clin North Am.
2004;31:157-171.
49. Honey RJ, Schuler TD, Ghiculete D, Pace
KT, Canadian Endourology Group. A ran- 57. Breda A, Ogunyemi O, Leppert JT, Schu-
domized, double-blind trial to compare lam PG. Flexible ureteroscopy and laser
shock wave frequencies of 60 and 120 lithotripsy for multiple unilateral intrarenal
shocks per minute for upper ureteral stones. stones. Eur Urol. 2009;55:1190-1196.
J Urol. 2009;182:1418-1423.

430 EDUCATIONAL REVIEW MANUAL IN UROLOGY


58. Semins MJ, Matlaga BR. Management of 67. Fernström I, Johansson B. Percutaneous
stone disease in pregnancy. Curr Opin pyelolithotomy. A new extraction tech-
Urol. 2010;20:174-177. nique. Scand J Urol Nephrol. 1976;10:257-
259.
59. Burgess KL, Gettman MT, Rangel LJ,
Krambeck AE. Diagnosis of urolithiasis 68. Duvdevani M, Razvi H, Sofer M, et al.
and rate of spontaneous passage during Third prize: contemporary percutaneous
pregnancy. J Urol. 2011;186:2280-2284. nephrolithotripsy: 1585 procedures in 1338
consecutive patients. J Endourol.
60. Watterson JD, Girvan AR, Beiko DT, et al. 2007;21:824-829.
Ureteroscopy and holmium: YAG laser
lithotripsy: an emerging definitive manage- 69. de la Rosette J, Assimos D, Desai M, et al.
ment strategy for symptomatic ureteral cal- The Clinical Research Office of the
culi in pregnancy. Urology. 2002;60: Endourological Society Percutaneous
383-387. Nephrolithotomy Global Study: indica-
tions, complications, and outcomes in 5803
61. Kavoussi LR, Albala DM, Basler JW, Apte patients. J Endourol. 2011;25:11-17.
S, Clayman RV. Percutaneous management
of urolithiasis during pregnancy. J Urol. 70. Mariappan P, Smith G, Moussa SA, Tolley
1992;148:1069-1071. DA. One week of ciprofloxacin before per-
cutaneous nephrolithotomy significantly
62. Khoo L, Anson K, Patel U. Success and reduces upper tract infection and urosepsis:
short-term complication rates of percuta- a prospective controlled study. BJU Int.
neous nephrostomy during pregnancy. 2006;98:1075-1079.
J Vasc Interv Radiol. 2004;15:1469-1473.
71. Valdivia JG, Scarpa RM, Duvdevani M, et
63. Nguyen TA, Belis JA. Endoscopic man- al. Supine versus prone position during
agement of urolithiasis in the morbidly percutaneous nephrolithotomy: a report
obese patient. J Endourol. 1998;12:33-35. from the clinical research office of the
endourological society percutaneous
64. Andreoni C, Afane J, Olweny E, Clayman nephrolithotomy global study. J Endourol.
RV. Flexible ureteroscopic lithotripsy: 2011;25:1619-1625.
first-line therapy for proximal ureteral and
renal calculi in the morbidly obese and 72. Ko R, Soucy F, Denstedt JD, Razvi H. Per-
superobese patient. J Endourol. cutaneous nephrolithotomy made easier: a
2001;15:493-498. practical guide, tips and tricks. BJU Int.
2008;101:535-539.
65. Dash A, Schuster TG, Hollenbeck BK,
Faerber GJ, Wolf JS Jr. Ureteroscopic treat- 73. Kim SC, Kuo RL, Lingeman JE. Percuta-
ment of renal calculi in morbidly obese neous nephrolithotomy: an update. Curr
patients: a stone-matched comparison. Opin Urol. 2003;13:235-241.
Urology. 2002;60:393-397.
74. Lopes T, Sangam K, Alken P, et al. The
66. Shokeir AA, Sheir KZ, El-Nahas AR, El- Clinical Research Office of the Endouro-
Assmy AM, Eassa W, El-Kappany HA. logical Society Percutaneous Nephrolitho-
Treatment of renal stones in children: a tomy Global Study: tract dilation compar-
comparison between percutaneous isons in 5537 patients. J Endourol.
nephrolithotomy and shock wave 2011;25:755-762.
lithotripsy. J Urol. 2006;176:706-710.

CHAPTER 14: SURGICAL MANAGEMENT OF STONES 431


75. Yamaguchi A, Skolarikos A, Buchholz NP, 83. Taylor EN, Stampfer MJ, Curhan GC. Obe-
et al. Operating times and bleeding com- sity, weight gain, and the risk of kidney
plications in percutaneous nephrolitho- stones. JAMA. 2005;293:455-462.
tomy: a comparison of tract dilation meth-
ods in 5,537 patients in the Clinical 84. Cappuccio FP, Strazzullo P, Mancini M.
Research Office of the Endourological Kidney stones and hypertension: popula-
Society Percutaneous Nephrolithotomy tion based study of an independent clinical
Global Study. association. BMJ. 1990;12:1234-1236.
J Endourol. 2011;25:933-939.
85. Taylor EN, Curhan GC. Body size and 24-
76. Wang J, Zhao C, Zhang C, Fan X, Lin Y, hour urine composition. Am J Kidney Dis.
Jiang Q. Tubeless vs standard percutaneous 2006;48:905-915.
nephrolithotomy: a meta-analysis. BJU
Int. 2012; 109:918-924. 86. Ekeruo WO, Tan YH, Young MD, et al.
Metabolic risk factors and the impact of
77. Istanbulluoglu MO, Cicek T, Ozturk B, medical therapy on the management of
Gonen M, Ozkardes H. Percutaneous nephrolithiasis in obese patients. J Urol.
nephrolithotomy: nephrostomy or tubeless 2004;172:159-163.
or totally tubeless? Urology.
2010;75:1043-1046. 87. Gofrit ON, Shapiro A, Donchin Y, et al.
Lateral decubitus position for percutaneous
78. AUA guideline on the management of nephrolithotripsy in the morbidly obese or
staghorn calculi. Available at: kyphotic patient. J Endourol. 2002;16:383-
http://www.auanet.org/content/clinical- 386.
practice-guidelines/clinical-guide
lines.cfm?sub=sc Accessed February 10, 88. Kanaroglou A, Razvi H. Percutaneous
2012. nephrolithotomy under conscious sedation
in morbidly obese patients. Can J Urol.
79. Desai M, Jain P, Ganpule A, Sabnis R, 2006;13:3153-3155.
Patel S, Shrivastav P. Developments in
technique and technology: the effect on the 89. Wu SD, Yilmaz M, Tamul PC, Meeks JJ,
results of percutaneous nephrolithotomy Nadler RB. Awake endotracheal intubation
for staghorn calculi. BJU Int. 2009; and prone patient self-positioning: anes-
104:542-548. thetic and positioning considerations dur-
ing percutaneous nephrolithotomy in obese
80. WHO: Fact Sheet No.311 – Obesity and patients. J Endourol. 2009;23:1599-1602.
overweight, March 2011. Available at:
http://www.who.int/mediacentre/factsheet 90. Fuller A, Razvi H, Denstedt JD, et al. The
s/fs311/en/index.html. Accessed August 9 , Clinical Research Office of the Endouro-
2011. logical Society Percutaneous Nephrolitho-
tomy Global Study: The Influence of Body
81. Taylor EN, Stampfer MJ, Curhan GC. Mass Index on Outcomes. J Urol. 2012. In
Dietary factors and the risk of incident kid- press.
ney stones in men: new insights after 14
years of follow-up. J Am Soc Nephrol. 91. Symons SJ, Ramachandran A, Kurien A,
2004;15:3225-3232. Baiysha R, Desai MR. Urolithiasis in the
horseshoe kidney: a single-centre experi-
82. Taylor EN, Stampfer MJ, Curhan GC. Dia- ence. BJU Int. 2008;102:1676-1680.
betes mellitus and the risk of nephrolithia-
sis. Kidney Int. 2005;68:1230-1235.

432 EDUCATIONAL REVIEW MANUAL IN UROLOGY


5. Questions

92. Miller SD, Ng CS, Streem SB, Gill IS. 1. Which of the following factors is not an
Laparoscopic management of caliceal absolute contraindication to SWL therapy?
diverticular calculi. J Urol.
2002;167:1248-1252. A. Pregnancy

93. Wyler SF, Bachmann A, Jayet C, Casella B. Bleeding diathesis


R, Gasser TC, Sulser T. Retroperitoneo-
scopic management of caliceal diverticular C. Distal obstruction
calculi. Urology. 2005;65:380-383.
D. Skin to stone distance >10 cm
94. Fernandez A, Foell K, Nott L, Denstedt
JD, Razvi H. Percutaneous nephro- E. Calcified renal artery aneurysm
lithotripsy in patients with urinary diver-
sions: a case-control comparison of periop-
erative outcomes. J Endourol. 2011;25:
1615-1618. 2. Skin-to-stone distance may be calculated
based on:

A. The mean of measurements taken on


axial CT from the stone to the skin at
0o, 45o and 90o

B. A single measurement on axial CT


from the stone to the nearest point at
skin level

C. The median of measurements taken on


axial CT from the stone to the skin at
0o, 30o and 60o

D. Coronal CT images

E. The mean of measurements taken on


axial CT from the stone to the skin at
0o, 30o and 90o

CHAPTER 14: SURGICAL MANAGEMENT OF STONES 433


3. Which of the following factors is not asso- 5. When performing semirigid and flexible
ciated with improved stone-free rates in ureteroscopy, where available, the safest
association with SWL? and most effective first-line intracorporeal
lithotripsy device is:
A. Alpha antagonists
A. Electrohydraulic lithotripsy (EHL)
B. Mechanical percussion and
inversion therapy B. Pneumatic lithoclast

C. Ureteric stent placement C. Holmium:yttrium-aluminium-garnet


laser
D. Forced diuresis
D. Neodymium: yttrium-aluminum-gar-
E. Shock wave rate of 60 shocks/minute net laser

E. CO2 laser

4. A 42-year-old pilot presents with a


radiopaque 11-mm calculus in the
lower pole of the right kidney. The most 6. You are performing semirigid ureteroscopy
appropriate initial form of management is: on a 32-year-old woman who presents with
a 8-mm stone in the distal left ureter. Which
A. Medical expulsive therapy of the following factors is not an absolute
indication to place a ureteric stent at the
B. SWL monotherapy completion of the procedure?

C. Percutaneous nephrolithotomy A. Balloon dilation of the ureteric orifice

D. Flexible ureterorenoscopy and B. Stone fragmentation with the Ho:


laser lithotripsy YAG laser

E. Dissolution therapy C. Concomitant ureteric stricture

D. Solitary kidney

E. Intraoperative ureteral perforation

434 EDUCATIONAL REVIEW MANUAL IN UROLOGY


7. A 25-year-old patient presents in the sec- 9. When performing fluoroscopic-guided
ond trimester of pregnancy with right flank puncture of the collecting system during
pain, fever and hematuria. Ultrasound PCNL, one should enter the tip of the cho-
demonstrates bilateral hydronephrosis, sen calyx. Medial puncture and dilation
more severe on the right side. No calculus into the infundibulum or renal pelvis is
is visualized. The most appropriate form of associated with:
management is:
A. Damage to the anterior segmental
A. Conservative management with branch of the renal artery
oral antibiotics and analgesia
B. Damage to the posterior segmental
B. Insertion of a right percutaneous and interlobar arteries
nephrostomy tube
C. Injury to surrounding structures
C. Cystoscopy and insertion of a right including colon, spleen and liver
ureteric stent
D. Inadvertent injury to the main renal
D. Magnetic resonance urogram pedicle

E. Low-dose non-contrast CT E. Injury to the apical branch of the


anterior division of the renal artery

8. You perform percutaneous nephrolitho-


tomy on a 67-year-old man who presents 10. A 68-year-old man presents with macro-
with a single 2.3-cm stone in the left renal scopic hematuria and flank pain 3 days
pelvis. Which of the following factors is after an uncomplicated PCNL. The
not an absolute indication for nephrostomy nephrostomy tube was removed 4 hours
tube placement? prior to the onset of these symptoms. He
looks pale. Vitals demonstrate tachycardia
A. Collecting system injury involving (115/min) and hypotension (BP 90/60 mm
renal pelvis Hg). The most appropriate initial manage-
ment is:
B. Hemorrhage with intraoperative
hypotension A. Resuscitation followed by an
immediate non-contrast CT scan
C. Multiple tracts
B. Conservative management with fluid
D. Pyonephrosis resuscitation and blood transfusion

E. Bilateral renal calculi C. Resuscitation followed by a renal


artery arteriogram and embolization
if required

D. Immediate open surgical exploration


and nephrectomy if required

E. Immediate re-insertion of a percuta-


neous nephrostomy tube

CHAPTER 14: SURGICAL MANAGEMENT OF STONES 435


sidered the preferred first-line imaging investiga-
tion, CT is useful when the findings are inconclu-
Answers

1. D. sive to direct further surgical intervention as


Although skin-to-stone distance has been shown to required. In some centers, MR urogram may be used
adversely affect stone-free rates following SWL for instead of CT and is a reasonable imaging option,
renal and ureteric calculi, it should be considered a albeit with inferior sensitivity and specificity for
relative rather than absolute contraindication to detection of stone disease.
therapy.
8.E.
2. A. The presence of calculi in the contralateral kidney at
As originally described by Nakada and colleagues, the time of PCNL does not necessitate placement of
SSD may help to predict success in overweight and a percutaneous tube at the completion of the proce-
obese individuals. SSD may be calculated as the dure.
mean of measurements taken on axial CT from the
stone to the skin at 0o, 45o and 90o 9.B.
Medial puncture risks injury to the posterior seg-
3. C. mental artery. Additionally, entering the calyx in the
Although placement of a stent is indicated prior to correct orientation (end on rather than side on)
SWL in the context of high-grade obstruction and avoids the interlobar arteries which cross the
high stone burden, studies have demonstrated no infundibula.
benefit for routine stent placement prior to SWL.
10. C.
4. D. Sudden hemodynamic instability day 3 post-PCNL
Although SWL has been described in the context of in association with flank pain and hematuria is most
lower pole calculi, SFR following treatment may be likely to represent bleeding from a pseudoa-
as low as 21%. This patient requires complete stone neurysm. This should be diagnosed and managed
clearance and would benefit from flexible URS, with angioembolization where appropriate inter-
laser lithotripsy and basket removal of stone frag- ventional radiology expertise is available.
ments. There is no role for medical expulsive or dis-
solution therapy in this scenario.

5. C.
The Holmium:YAG laser now represents the gold
standard means of achieving stone fragmentation. It
is effective for all stone types. Fibres are available to
pass through the working channel of all modern
ureteroscopes.

6. B.
Holmium laser energy is rapidly absorbed by water
and usually results in no adjacent tissue injury. The
remainder of the factors indicate placement of a
ureteric stent to facilitate drainage.

7. E.
Ultra low-dose CT protocols have been developed
for the definitive diagnosis of urolithiasis in preg-
nancy. The dose of fetal radiation delivered has been
shown to be low and not associated with fetal or
maternal harm. Although ultrasound should be con-

436 EDUCATIONAL REVIEW MANUAL IN UROLOGY


Chapter 15:
Renal Parenchymal and
Upper Urinary Tract
Urothelial Neoplasms
Robert G. Uzzo, MD

Contents

1. Renal Parenchymal Neoplasmsa 2. Upper Urinary Tract


a. Epidemiology Urothelial Carcinoma
b. Molecular Biology a. Epidemiology and Basic Biology
c. Kidney Cancer Syndromes b. Pathology
d. Pathology and Classification c. Evaluation, Diagnosis and
e. Evaluation and Imaging Imaging
f. Prognosis Factors d. Staging and Prognosis
g. Management e. Management
• Local disease • Laparoscopy and
• Open or laparoscopic open approaches
radical nephrectomy • Endoscopic
• Open/lap/robotic NSS • Topical and
• Ablation systemic therapies
• Active Surveillance
• Follow-up following 3. Suggested Reading
treatment
• Locally advanced and 4. Questions
metastatic disease
• Cytoreductive
nephrectomy
• Metastasectomy
• Systemic therapies
• Special circumstances
• Other malignant tumors
of the kidney
• Benign renal neoplasms
• Paraneoplastic syndromes

Chapter 15: renal parenChymal and Upper Urinary traCt Urothelial neoplasms 437
1. Renal Parenchymal Neoplasms

a. Epidemiology • Kidney cancer is the most lethal of all GU neo-


plasms, as a function of percentage of patients
• Cancer is the 2nd overall leading cause of death in diagnosed with the disease who ultimately die of
the United States (following heart disease) and disease.
accounts for approximately 1 in 4 deaths. Other
leading causes of death include: o Prostate cancer
o Cerebrovascular disease (#3) Approximately 14% (27,000 deaths/year)
o Chronic pulmonary diseases (excluding of the 192,000 cases diagnosed each year.
cancer) #4
o Unintentional accidents (#5) o Bladder cancer
o Diabetes (#6) Approximately 20% (14,000 deaths/year)
of the 71,000 cases diagnosed each year.
• Over the last 50 years, the death rate due to cancer
has decreased marginally while deaths from: o Kidney cancer
o heart disease have dropped more than 2-fold Approximately 23% (13,000 deaths/year)
o cerebrovascular disease have dropped more of the 58,000 cases diagnosed each year
than 3-fold (excluding renal pelvic cancers)

• In 2003, the number of net deaths from cancer o Testicular cancer


dropped for the first time since 1930. There has Approximately 5% (380 deaths/year) of
been a slow steady decline since. the 8,400 cases diagnosed each year.

• In 2010, approximately 767,000 men and 679,000 • Male to female ratio for malignant renal
women were diagnosed with cancer. The rates of parenchymal lesions is approximately 1.7 to 1.
genitourinary (GU) cancer were as follows:
• Primarily a disease of the elderly with peak
o In adult men, GU cancers accounted for more incidence in the 6th and 7th decades of life.
than 40% of all cancer diagnoses and 17% of
all cancer deaths. • 10%–20% higher incidence in African Americans
for unknown reasons.
#1 Prostate cancer (33%)
• Accounts for 9% of all cancer deaths • 96% of cases are sporadic while 4% are
in men. associated with familial syndromes.

#4 Bladder cancer (6%) • The incidence of kidney cancer has increased by a


• Accounts for 3% of all cancer deaths rate of approximately 2%–4% per year over the
in men. last 3 decades. This may be explained by:
o Increased incidental detection.
6 Kidney cancer (3%) o Unidentified environmental influences.
• Accounts for 3% of all cancer deaths
in men. • There has been a steady rise in the reported mortal-
ity rates attributed to kidney cancer over the last 3
o In adult women, GU cancer accounted for decades. This may be steadying off or perhaps
approximately 3% of all cancer diagnoses. even beginning to fall with the advent of new sys-
temic (primarily antiangiogenic) therapies.
9 Bladder cancer (2%)

#10 kidney cancer (1%)

438 edUCational reVieW manUal in UroloGy


• A slow stage migration has occurred with kidney A small number of established genetic
cancer over the last 3 decades. Current rates of mechanisms are responsible for the
stage at the time of presentation are: majority of known mutations in tumor
o Localized disease – 65%–70% suppressors.
o Regionally advanced disease – 15%–20% • Loss of heterozygosity (LOH) with or
o Metastatic disease – 15%–20% without homozygous deletion.
o Involves the loss of specific
• The only generally accepted environmental risk chromosomal regions that con-
factor is tobacco exposure: tain known or presumptive tumor
o Increases relative risk by 1.4 to 2.5. suppressor genes. LOH may be
o All forms of tobacco have been implicated. achieved due to a deletion, gene
o Risk increases with cumulative exposure. conversion, mitotic recombina-
tion, translocation, chromosome
b. Molecular Biology breakage and loss, chromosomal
fusion or telomeric end-to-end
• Relevant kidney cancer genes: fusions, or whole chromosome
loss. This may result in asymmet-
o 3p25 – the VHL gene (clear cell RCC) ric loss of a gene or homozygous
encodes the VHL protein (pVHL) deletions whereby both copies of
the gene are affected.
A tumor suppressor gene – o LOH is a useful diagnostic
marker because of its high level
Tumor suppressor genes negatively regu- of specificity for transformed
late growth. Their essential function is to cells.
prevent uncontrolled proliferative pro- • Point mutations
cesses including cellular division. Each o Occur at the level of individual
gene has a corresponding copy of the nucleotides where substitutions,
tumor suppressor gene. Knudsens’ two-hit insertions and deletions may
hypothesis predicts that it only takes 1 result in frame shift, missense
functioning copy to regulate growth; and nonsense mRNA transcripts
therefore, both copies or alleles must be and ultimately truncated or non-
mutated or silenced in order to promote functional proteins.
carcinogenesis. In the case of a germ-line • Promoter hypermethylation
mutation, the offspring inherits a mutated o In most somatic cells, the CpG
copy of the tumor suppressor from one of dinucleotide islands in the con-
their parents. In this scenario, only the sec- trolling or promoter region of a
ond copy of the gene need undergo gene are normally protected from
somatic mutation to induce transforma- methylation. Although not a dis-
tion. This is the case with inherited forms tinct genetic event, promoter
of clear cell kidney cancer associated with hypermethylation is an epigenetic
the VHL syndrome. In cases of sporadic event capable of silencing expres-
clear cell carcinoma, the patient is born sion.
with 2 normal copies of the VHL gene,
both of which need undergo a somatic One or more of these events
mutation to induce transformation. Since is responsible for nearly all
this is a statistically less common event, inherited (germ-line) and
clear cell RCC is less frequent in nonmu- 40%–70% of acquired
tant VHL carriers. (somatic) mutations in VHL
responsible for cases of clear
cell RCC.

Chapter 15: renal parenChymal and Upper Urinary traCt Urothelial neoplasms 439
o 7q31 – the cMet gene (papillary Type I RCC) c. Kidney Cancer Syndromes
(encodes met protein [receptor tyrosine
kinase family]) • There are 4 described hereditary clinical kidney
cancer syndromes (see Table 1)
An oncogene
• Responsible for gain of function o All are autosomal-dominant:
molecular events leading to increased Von Hippel-Lindau (VHL)
cellular growth, differentiation or pro- Hereditary Papillary Renal Carcinoma
liferation through the accumulation of (HPRC)
normal signaling proteins or creation Birt-Hogg-Dubé (BHD)
of a mutant-activating protein. Hereditary Leiomyoma Renal Cell
• Unlike tumor suppressor genes, onco- Carcinoma (HLRCC)
genes may only require a single
mutating event in 1 (not both) allele to o 2 have cutaneous manifestations:
promote malignant transformation. BHD and HLRCC.
• The most common mutating genetic
event is point mutation. o Epidemiological but no definitive increased
genetic risk of RCC with:
o 17p11.2 – The Birt Hogg Dubé gene Acquired renal cystic disease associated
with dialysis.
Tumor suppressor gene. Autosomal-dominant polycystic kidneys.
Responsible for cases of oncocytoma and Tuberous sclerosis.
chromophobe carcinoma.
Encodes for the folliculin protein (func- • Von Hippel-Lindau
tion unknown).
o Syndrome includes (see Table 2):
o 1q42.3-q43 – The Hereditary Leiomyoma Retinal angiomas
Renal Cell Carcinoma gene (HLRCC) associ- (among the earliest finding).
ated with Papillary Type II RCC. Endolymphatic sac tumors.
Hemangioblastomas of the CNS
A tumor suppressor. (benign lesions).
Encodes for the Kreb’s cycle protein Pancreatic cysts and islet tumors.
fumarate hydratase. Epididymal cystadenomas.
Renal solid and cystic clear
o Tuberous sclerosis genes (associated with cell carcinoma.
angiomyolipomas). Pheochromocytomas.
• Most frequently associated with
9q34 – TSC1. missense mutations of the VHL
16p13 – TSC2. gene (encode for an amino acid
Defects in TSC genes have not been substitution).
definitively linked to RCC.
o Relevant molecular pathway
A mutated VHL gene encodes for a
mutated pVHL.
The main action of the VHL protein is
thought to be its E3 ubiquitin ligase activ-
ity that results in the HIF complex being
“marked” for degradation.
pVHL is coregulated by its interactions
with elongin and cul family of proteins.

440 edUCational reVieW manUal in UroloGy


The VHL protein complex in the cyto- • Hereditary Leiomyoma Renal Cell Carcinoma
plasm regulates activity of Hypoxia
Inducible Factors (HIF). o Manifestations include:
HIF-α marks the VHL protein complex in Painful cutaneous leiomyomas, which can
the cytoplasm for ubiquitination and occur anywhere on the body.
degradation. Uterine leiomyomas (fibroids) –
If pVHL is mutated, it cannot bind HIF-α, early age of onset, multiple and painful.
which then translocates into the nucleus. Type 2 papillary RCC
In RCC, the primary HIF isoform is • Typically aggressive tumors.
HIF2α.
• Nuclear accumulation of HIF upregu- d. Pathology and Classification
lates VEGF, PDGF, EGF, glucose (see Table 3)
transport proteins, TGF, erythropoi-
etin, CAIX (important for acid base • Renal tumors can be classified by pathological
balance), CXCR4 (chemokine) and and/or radiographic features.
other proteins which regulate cellular o Pathological classification of renal tumors
growth, development and angiogene- The most common benign renal tumors
sis. include:
• As much as 1%–5% of the human • Angiomyolipomas, oncocytomas,
genome may be under some degree of cortical adenomas, cystic nephromas,
HIF regulation. reninomas (JG apparatus tumors),
leimyomas, capsular fibromas, and
• Hereditary Papillary Renal Cell Carcinoma vascular tumors such as arteriovenous
malformations and pseudotumors.
o The rarest form of hereditary RCC. • Benign inflammatory tumors of the
o All manifestations are confined to the kidney include renal abscesses and
kidney. There are no extrarenal findings. granulomatous diseases such as XGP.
o Characterized by bilateral and multifocal The most common malignant renal
type I papillary RCC. parenchymal tumors include conventional
o Histological appearance of all tumors is (clear cell), papillary, chromophobe,
relatively identical. translocation, medullary and collecting
duct carcinomas.
• Birt Hogg Dubé Sarcomas of kidney include leiomyo, lipo,
angio, osteogenic, clear cell and rhab-
o Manifestations include: domyo sarcomas.
Other tumors of the renal parenchyma
Fibrofolliculomas of the head and neck. include Wilms’ tumors, neuroectodermal
These lesions are usually painless and tumors, carcinoids, lymphoma or metas-
develop after the age of 30. tases to the kidney.
Pulmonary cysts and spontaneous pneu-
mothorax. o Most renal cell carcinomas originate
Multifocal and bilateral renal tumors. from the cells of the proximal collect-
• Most commonly oncocytoma and ing tubule with the exception of chro-
chromophobe carcinoma. mophobes, oncocytomas and papil-
• Clear cell and papillary tumors lary carcinomas, which are thought to
can coexist. be derived from the distal tubule.
Other possible associations include nevi,
parathyroid adenomas, lipomas, oral
mucosal papules, colonic polyps or
tumors.

Chapter 15: renal parenChymal and Upper Urinary traCt Urothelial neoplasms 441
• Components of RCC pathology include: Chromophobe RCC
o Histology. The most common histologic types • Accounts for about 5% of all RCC.
are: • Derived from the cortical portion of
Conventional (clear cell) carcinoma the collecting duct or distal tubule.
• Accounts for 70%–80% of all RCC • Grossly solid brown or tan. Histologi-
• Yellow when bivalved, hypervascular cally with large pale or pink cells in
with clear or vacuolated cytoplasm. sheets or nests.
Clear cells contain abundant glycogen • Wrinkled, raisinoid nuclei with a per-
and phospholipids. inuclear halo is a typical and a distinc-
• Some cells may contain granular tive finding. It is due to microvesicles
eosinophilic (pink) cytoplasm. that stain positive for Hale’s colloidal
• Exhibit nested architecture with iron (blue), indicating the presence of
intervening fibrovascular stalks. a mucopolysaccharide unique to
• 2%–5% contain sarcomatoid features RCC. This stain is negative in onco-
with more aggressive clinical behav- cytoma.
ior compared with other histologies. • Studies suggest that the prognosis
• Most likely to respond to may be better than for conventional
targeted/immunotherapies. RCC.
Papillary (chromophilic RCC) Collecting Duct (Bellini) Carcinomas
• Second most common histological • A rare subtype of RCC accounting for
type representing 10%–15% of all <1% of RCC.
RCCs. • May occur in a younger population of
• Contain basophilic or eosinophilic patients but age range is broad (ie, can
cells arranged in papillary patterns occur in older populations as well).
lined by carcinoma cells; however, • Are derived from the medullary
solid variants are possible. collecting system but often extend
• Fibrovascular core of the papillae into the cortex.
often have foamy macrophages and • Most cases are symptomatic and
psammoma bodies (calcifications). present at a later stage, often with
• 2 subtypes have been described, osteoblastic bony metastases.
including type 1 (more basophilic • Grossly large, irregular tumor arising
with scant cytoplasm) and type II in central portion of kidney with irreg-
(more eosinophilic and more aggres- ular angulated tubules with marked
sive). cytologic atypia. Also characteristic is
o Type I: carcinoma cells are small, a desmoplastic stroma with associated
limited nuclear atypia. neutrophils.
o Type II: carcinoma cells are • Immunohistochemical and molecular
larger, pink and have prominent analysis suggest that these tumors
nucleoli; more aggressive. resemble urothelial carcinomas.
• Tends to be multicentric in as many as • Systemic treatment often includes
40% of cases. bladder-like chemotherapy regimens.
• Grade for grade and stage for stage, Renal Medullary Carcinomas
prognosis is likely not much different • Occur almost exclusively in associa-
than conventional RCC. tion with sickle cell trait and is there-
fore most common in African Ameri-
cans.
• Typically diagnosed in the 3rd
decade of life.
• Most cases are locally advanced and
metastatic at diagnosis.

442 edUCational reVieW manUal in UroloGy


• Highly aggressive. Mean survival is * Fuhrman grade 1 = no atypia,
12–15 months. small nuclei.
• Grossly, irregular large tumors arising * Fuhrman grade 2 = variable
from the central portion of the kidney. atypia.
Histology notes complex tubular/crib- * Fuhrman grade 3 = prominent
iform structures and sheets of cells nucleoli.
with open vesicular nuclei with * Fuhrman grade 4 = marked
prominent nucleoli. atypia.
Translocation carcinoma of the kidney * Grade 1 and 2 are considered low
• Most commonly affects young adults. grade while grades 3 and 4 are
• Xp11 translocations the result in gene considered high grade.
fusions involving the TFE3 gene and • Papillary (chromophilic) RCC are
by translocation t(6:11)(p11.2;q12) classified as type 1 (low grade,
that involves the TFEB gene, result- basophilic) or type II (high grade,
ing in protein overexpression. This eosinophilic).
can be identified by immunohisto- Tumor grade is recognized as an impor-
chemistry with TFE antibody. tant prognostic variable in RCC.
• TFE3 carcinomas have papillary or
nested architecture with clear cells. o Stage
• TFEB carcinomas have nested archi- Refers to the local, regional and systemic
tecture only with clear cells and a extent of disease.
biphasic cell population. Historically used the Robson staging sys-
• Frequently present as high-stage tem.
disease. Currently uses the 2010 v7 AJCC TNM
staging system (see Table 4).
Sarcomatoid changes have been described The TNM system defines the anatomic
of most histologic types of RCC. It is extent of disease explicitly and can be
therefore no longer considered a distinct used to facilitate comparisons of data
cell type. They are: from centers worldwide.
• Poorly differentiated regions of other Changes in the v7 of TNM for kidney
histological types. include the addition of T2 substages (a
• Found in 1%–5% of RCCs, most and b) based on tumor size, changes in the
commonly with conventional (clear substaging of T3 tumors such that pT3a is
cell) or papillary RCC but may be for tumors involving the fat or segmental
seen in chromophobes as well. venous branches and pT3b/c depend on
• Median survival is poor (depending height/invasion of IVC involvement, and
on stage). changes in T4 to include ipsilateral
adrenal involvement.
o Grade
Nuclear features can be highly variable e. Evaluation and Imaging
in RCC.
Grading systems are based on • Clinical staging for RCC begins with a thorough
nuclear size, shape and presence or history, physical exam and judicious use of labora-
absence of nucleoli. tory tests:
• Fuhrman’s system is the most com- o Symptomatic presentation, significant weight
mon classification for conventional loss (>10% of total body weight), bone pain
RCC (performed at 10x). and poor performance status all suggest
advanced disease, as do physical exam find-
ings of a palpable mass or lymphadenopathy.

Chapter 15: renal parenChymal and Upper Urinary traCt Urothelial neoplasms 443
o Nonreducing varicocele or lower extremity o Metastatic evaluation in all cases should
edema suggests possible venous involvement. include a routine chest x-ray, careful review of
o Absence of findings on history and physical the CT findings of the abdomen and pelvis,
exam does not rule out advanced disease. and liver function tests.
o Abnormal liver function tests, elevated alka- Bone scan, CNS imaging and chest CT
line phosphatase, sedimentation rate or ane- can all be reserved for symptomatic
mia suggest possible advanced disease. patients, abnormalities on CXR, abdomi-
o Clinical clues will often be helpful in distin- nal imaging or laboratory studies (ie, ele-
guishing a malignant renal mass from other vated alkaline phosphatase), or patients
causes (see Table 5). with extensive disease identified by rou-
tine studies.
• Imaging of the renal mass PET has shown poor sensitivity for rou-
o Radiographic staging of RCC requires a con- tine use in the evaluation of RCC. Newer
trast-based study demonstrating enhancement immunoPET scans like the G250 scan
(blood flow) of the mass. may be available soon and are specific for
3-phase CT of the abdomen and pelvis CAIX expression in clear cell RCC.
with all 3 phases being done at the same o Renal cysts identified by CT are classified
setting is imperative. This is the single according to the Bosniak system (see Table 6).
best test. Should not rely on a noncontrast Bosniak I and II lesions are considered
CT or a contrast CT performed at different most likely benign and can be followed.
settings, especially on different machines. Bosniak IIF lesions require closer
Enhancement (>15 to 20 units by CT) of follow-up.
the mass is required. Bosniak III and IV lesions should be
MRI pre and postgadolinium should be considered malignant and treated
reserved for patients with suspected accordingly.
venous involvement, those with an iodine o Ultrasound findings
contrast allergy and those with renal A simple cyst on US should be anechoic,
insufficiency. Risk of NSF low in patients have good through transmission and pos-
with eGFR >30. terior wall enhancement.
Lymphadenopathy >2 cm generally har- • Complex or hyperdense cysts on US
bor malignancy. Smaller nodes may be may appear to be a solid lesion and
inflammatory and if suspicious should be read incorrectly as RCC.
removed at time of initial surgery. If posi- Fatty lesions such as an angiomyolipoma
tive, a more extensive lymphadenectomy (AML) appear hyperechoic (white) on
is generally warranted. For questionable US.
nodal enlargement outside of the typical o MRI findings
landing zone in the retroperitoneum, A simple cyst on MRI will appear bright
serial radiographic evaluation recom- on T2 and dark on T1. It will not enhance.
mended. A hyperdense cyst may appear bright on
Venous involvement is best imaged by T1 and dark on T2 depending on the age
MRI. The sensitivity of CT for renal vein and composition of its contents. It will not
and IVC involvement is 78%–96%. enhance.
Venography is reserved for patients with Fat (ie, from an AML) is generally bright
equivocal MRI findings or those with on MRI but depends on technique. Look
contraindications to MRI. at subcutaneous fat for an idea as to how
Transesophageal echo is an accurate the fat looks on that particular MRI
method of determining the cephalad sequence.
extent of the tumor and may be used intra-
operatively. It has no real advantage over
MRI in the preoperative setting.

444 edUCational reVieW manUal in UroloGy


o Percutaneous biopsy stage, grade, histology, symptoms at presentation
Indications for percutaneous biopsy or and performance status.
aspiration remain relatively limited due in o Several investigators have combined various
part to sampling error, difficulties inter- prognostic factors into predictive algorithms
preting limited amount of tissue obtained, or nomograms which may help predict recur-
difficulties with differential diagnosis (ie, rence or response to therapy. These have been
eosinophilic variants of RCC vs oncocy- operationalized on
toma) and grading. www.cancernomograms.com for point of care
Biopsies should be considered in patients use (see Table 7).
with suspected lymphoma, abscess or • 5-year risks of recurrence for local or regional RCC
metastatic disease to the kidney. In most fully excised are approximately:
cases of patients with a primary malig- o 5%–9% for low-risk disease.
nancy elsewhere, metastases to the kidney o 20%–25% for intermediate-risk disease.
are late and are usually accompanied by o 60%–80% for high-risk disease.
extensive disease elsewhere. Solitary • Metastases from RCC have been reported in nearly
renal metastases are uncommon. every site in the body. Most metastases
Biopsies increasingly used in “vulnera- (50%–75%) are asymptomatic.
ble” patient populations (elderly, infirm, o More than 70% of metastases are identified by
large/multifocal or bilateral disease). abdominal imaging and CXR.
Risk stratification tools including the risk o The most common sites of recurrence are
of RCC (vs a benign lesion) and grade can lung>lymph nodes>bone>liver>adrenal>CNS
be calculated using published nomograms (see Table 8).
(www.cancernomograms.com). o Most recurrences occur within the first 24
months after resection. The risk of recurrence
f. Prognosis decreases as disease-free interval increases.

• Prognostic factors for RCC can be divided g. Management


into anatomic, histologic, clinical and molecular
categories: • Recent guidelines have been released from the
o Anatomic factors include tumor size, exten- AUA on the management of T1 RCC
sion, adrenal, venous or lymphatic involve- (www.auanet.org/guidelines)
ment OR radiographically identifiable o Primary goals of management in this order:
metastatic disease. • Cure the cancer
o Histologic factors include histologic type, • Save the kidney
nuclear grade, presence of necrosis, sarcoma- • Perform surgery using MIS technique if
toid features and invasion of the collecting possible
system. • Surgery remains the mainstay of treatment for
o Clinical features include performance status, RCC. The objective is to resect all tumor with an
presence of symptoms related to primary or adequate negative surgical margin.
metastases, and weight loss. Also included are o Local disease
possible laboratory factors such as anemia, • Open or laparoscopic radical nephrectomy
polycythemia, thrombocytosis, hypercalcemia • The classic description of the radical
and/or elevated alkaline phosphatase nephrectomy includes early ligation of
(MSKCC criteria). the vessels, removal of the kidney out-
o Numerous molecular prognostic factors have side Gerota’s fascia, excision of the
been investigated. Currently, none are used ipsilateral adrenal and complete
routinely in clinical practice. regional lymphadenectomy.
o The most important prognostic factors remain

Chapter 15: renal parenChymal and Upper Urinary traCt Urothelial neoplasms 445
• A completed phase III randomized o Large population-based studies suggest
European Trial (EORTC 30881) that NSS is underutilized nationally.
resection of clinically negative nodes o NSS entails complete local excision
does not improve PFS or OS. Unsus- of the renal tumors leaving the largest
pected node positivity was identified possible amount of normal function-
in only 4% of clinically N0 patients. ing parenchyma. Enucleation of the
o Cases of clinically involved lym- tumor implies incomplete resection
phadenopathy require resection and is not a preferred term for
of all involved nodes where feasi- nephron-sparing surgeries for local-
ble, particularly if there are no ized kidney tumors. Indications are
metastases. classified as:
• Excision of the ipsilateral adrenal Absolute – bilateral tumors or a
gland is not routinely necessary in the tumor in a anatomically or func-
absence of radiographic involvement. tional solitary kidney.
• The operative approach to the kidney Relative – a renal tumor involv-
depends on the size and location of the ing kidneys compromised or
tumor as well as the body habitus of potentially compromised by sys-
the patients. temic disorders which decrease
o Open approaches include flank global renal function (diabetes,
(subcostal, intercostals or with hypertension, glomerulopathy,
excision of portion of the rib), etc).
chevron, or thoracoabdominal Elective – an ipsilateral renal
(especially useful on the right tumor in an otherwise healthy
side with large upper pole individual with 2 anatomically
tumors, IVC involvement or and functionally normal kidneys.
adjacent organ involvement). The accepted tumor size cut-
o Laparoscopic approaches include off for elective partial
transabdominal, retroperitoneal nephrectomy is 4 cm; how-
or hand assisted. ever, lesions 7+ cm may also
• Laparoscopic nephrectomy has be amenable to safe elective
emerged as a less morbid and safe partial nephrectomy with
option for most cases of localized excellent outcomes.
RCC, including large tumors (<20 o Pre, peri and postoperative implica-
cm), patients with significant prior tions of NSS
surgical histories, the morbidly obese Evaluation for a patient prior to
and even in cases with early venous NSS should include all staging
involvement. Multiple studies have considerations as outlined above.
demonstrated the safety and efficacy In addition, specific renal imag-
of laparoscopic nephrectomy in these ing, such as video 3D volume-
settings. rendered CT, may be useful to
• Nephron Sparing Surgery (NSS) delineate the relationship of the
o Interest in NSS has grown as a result tumor to the intrarenal vascular
of stage migration due to improved supply and collecting system.
imaging, a greater experience with Additionally, a functional assess-
renovascular surgery, improvements ment of the contralateral kidney,
in surgical methods including means such as a contrast-based CT or
to prevent renal ischemic injury, and MRI is imperative. Nuclear renal
excellent long-term published out- scan adds little in the absence of a
comes. well-done, contrast-based
CT/MR.

446 edUCational reVieW manUal in UroloGy


Intraoperative considerations Bilateral synchronous sporadic
should mimic that of a donor RCC occurs in 0%–5% of cases
nephrectomy with excellent and may occur in African Ameri-
exposure and hemostasis, com- can men more commonly. In the
plete vascular control, and pro- case of bilateral RCC, surgeries
motion of a brisk intraoperative are usually staged with the less
diuresis. Once these factors are involved size treated first. This
assured, hilar clamping, full exci- provides more options to the sur-
sion with a negative margin and geon and may reduce the need for
detailed reconstruction of the col- perioperative dialysis.
lecting system and renal remnant Cancer-specific survival rates
are essential. Warm ischemic depend on the pathological fea-
times should be <30 minutes and tures of the tumor and are
cold ischemic times should be reported to be 85%–100% in most
<60 minutes whenever possible. series. The risk of local recur-
Clamping and re-clamping rence is 5%–10% and may repre-
should be avoided and is associ- sent residual disease or de novo
ated with reperfusion injury. tumors secondary to microscopic
Recent data in 660 partial multifocality.
nephrectomies in solitary kidneys Patients who undergo NSS, par-
suggest that percent of ticularly those with a small
parenchyma preserved and pre- amount of residual functioning
operative GFR are most predic- nephrons, may be at increased
tors of postoperative renal func- risk for hyperfiltration renal
tion. injury. This places patients at
Patients undergoing NSS for increased risk for proteinuria,
absolute or relative indications, focal segmental glomerulosclero-
particularly in the case of a soli- sis and progressive renal failure.
tary kidney, should be told there Efforts to ameliorate the damag-
is a 5%–10% risk of temporary or ing effects of hyperfiltration
permanent dialysis associated injury have focused on dietary
with NSS depending on preopera- and pharmacological interven-
tive variables. tions, including dietary restriction
Urinary fistula rates are 3%–20% of protein (animal studies) and
depending on the size of the the use of an angiotensin-convert-
lesion, the location of the lesion, ing enzyme inhibitor.
the ischemic time and the experi- o Laparoscopic/robotic vs open NSS:
ence of the surgeon. Most fistulae Open NSS remains the standard
will close after a time with ade- of care.
quate drainage. Some may Per the AUA guidelines, laparo-
require a ureteral stent and/or scopic radical nephrectomy with
Foley catheter to promote heal- loss of nephrons is not an ade-
ing. quate tradeoff for nephron preser-
vation.

Chapter 15: renal parenChymal and Upper Urinary traCt Urothelial neoplasms 447
Laparoscopic NSS presents a o Ablation
technical challenge given the con- Thermal ablative techniques include
cerns about hemostasis, margin renal cryosurgery and radiofrequency
status and reconstruction of the ablation (RFA).
renal remnant. Nevertheless, most Both can be administered percuta-
series demonstrate good results in neously or through laparoscopic expo-
selected patients, although the sure, thus decreasing morbidity and
reported rates of complications allowing a more rapid recovery.
are higher than those for open Data suggest that these therapies are
NSS per the AUA guidelines. effective but the length of follow-up in
Laparoscopic NSS is generally most studies is quite limited (on aver-
performed via a transabdominal age 15–20 months) and long-term
approach given the larger work- efficacy is not established.
ing space. Intraoperative ultra- Acceptable candidates for thermal
sound may facilitate the proce- ablations are patients with advanced
dure. Retroperitoneoscopic NSS age, patients with significant comor-
is feasible in well-selected bidities and those with ipsilateral rem-
patients. nant recurrence after prior NSS.
Increased experience with robotic Tumors most amenable to ablation are
NSS suggests the 3D optics and small (<3.5 cm), peripheral, solid,
intracorporeal suturing are a ben- exophytic lesions remote from major
efit to MIS NSS. vascular and collecting system struc-
o NSS for patients with VHL represents tures. These are the same characteris-
an absolute indication. Adequate sur- tics of the ideal tumor for NSS.
gical treatment requires excision of all Heat sinking refers to the dissipation
solid and cystic renal lesions. Unfor- of thermal energy by nearby vascular
tunately, patients with VHL have mul- structures. This is a greater concern
tifocal disease, which can rarely be with RFA than with cryosurgery. Ani-
excised fully without radical nephrec- mal data also suggest there is a greater
tomy. Several studies demonstrate a risk of urinary collecting system
3-cm threshold to intervene in patients injury and fistula formation with RFA
with VHL. In these patients, lesions than with cryosurgery. Therefore, the
>3 cm have been shown to have a type of energy ablation used should be
greater propensity toward the devel- carefully considered for lesions close
opment of metastases; although there to the collecting system and larger
have been reports of smaller lesions intrarenal vessels.
associated with metastases. There- Experience with cryosurgery is more
fore, NSS may be withheld until the extensive than with RFA in the uro-
dominant lesion reaches 3 cm. logic literature.
Surgery should then be aimed at reset- Both ablative techniques cause tumor
ting the biological clock, such that all death by immediate cellular damage
dominant lesions are excised and the and delayed microcirculatory failure.
renal remnant is watched closely until The ideal treatment parameters are yet
a dominant lesion reaches 3 cm. to be fully determined; although most
Options in this setting include radical will double ablate (2 cycles) for a mar-
nephrectomy with transplantation, gin of safety.
attempted NSS or ablative technolo-
gies.

448 edUCational reVieW manUal in UroloGy


Imaging concerns remain one of the Active surveillance of a small enhanc-
greatest obstacles to ablative tech- ing renal mass requires periodic con-
nologies. Real-time imaging is best trast–based imaging, which is often
achieved with intraoperative ultra- more frequent initially (every 3–6
sound; however, most ablations will months in the first 12 years) and less
error on overtreatment of the tumor to frequent once stability or slow growth
attain an adequate margin. For this kinetics are established.
reason, more nephrons may be lost Active surveillance, like any interven-
during ablation than excision. tion, is a calculated risk.
Reported recurrence and/or persis-
tence rates following ablative tech- o Follow-up after definitive treatment for
niques are between 5% and 12% per localized RCC:
the AUA guidelines. Surveillance for recurrence is impor-
tant after surgical excision of RCC.
o Active surveillance Stage-specific surveillance means that
The natural history of small enhancing follow-up intervals are based upon the
renal masses presumed to be RCC is likelihood of recurrence as a reflection
only recently becoming well defined, of initial tumor stage and size.
whereas previously all tumors were All patients should be evaluated peri-
promptly excised. odically with a history, physical exam-
Elderly patients or those with signifi- ination and selected serum studies
cant medical risks are offered the (calcium, hemoglobin, liver and renal
opportunity for serial radiographic profiles) yearly or twice yearly.
surveillance to define the growth Radiographic studies including CXR,
kinetics of untreated RCC. CT or MRI are tailored to risk based
Collectively, there are approximately on stage (see Tables 10a-c), grade and
900 small renal masses reported in the histology. A surveillance calculator is
literature which have undergone a available on line at www.cancernomo-
period of active surveillance for an grams.com following NCCN guide-
average of nearly 3 years. In over half lines, although new NCCN guidelines
of these, pathology is available, and in suggest even less intense radiographic
over 80%–90%, RCC was present. follow-up may be needed.
Although the growth kinetics may be • Asynchronous renal recurrences in
somewhat variable, the median rate of the contralateral kidney occur in
radiographic growth in most series is 0%–5% of cases and may be
between 0.08 and 0.58 cm/year (see higher in African American men.
Table 9). The preferred treatment for a con-
As many as 30% of small renal masses tralateral recurrence is NSS,
show no radiographic growth when whereas they are ordinarily identi-
followed over a median of 3 years. fied early and there is often a rela-
The risk of metastatic progression in tive or absolute indication.
active surveillance series appears to
be quite low: about 1%–2% at a
median follow-up of 3 years.
Most series are limited by their retro-
spective nature and selection biases.

Chapter 15: renal parenChymal and Upper Urinary traCt Urothelial neoplasms 449
• Locally advanced and metastatic disease • For cases of level 1 thrombus,
o Surgery for locally advanced RCC: simple milking back of the clot
The principles of surgery for locally into the renal vein with open or
advanced RCC remain the same: exci- laparoscopic ligation proximally
sion of all localized disease. This ordi- is often sufficient.
narily involves radical nephrectomy, • For cases of level 2 thrombus,
adrenalectomy, regional lym- control and sequential clamping
phadenectomy and thrombectomy of the IVC above and below the
where appropriate. clot and the contralateral renal
In selective cases, NSS may be appro- vein after full exposure and liga-
priate and feasible in the setting of tion of lumbar veins provides a
node-positive disease and/or venous bloodless field.
involvement. • For cases of radiographic level 3
Principles of surgery for venous thrombus, it may be possible to
involvement include complete exci- get above the clot by ligating short
sion with full vascular control. hepatics to the caudate lobe of the
• Involvement of the IVC with liver, isolating and selectively
RCC occurs in 4%–10% of clamping the porta hepatis and
patients. proceeding as with a level 2 clot.
• In cases of suspected venous In cases where the hepatic veins
involvement, it is imperative to are clearly involved, full mobi-
determine the highest level of lization of the right lobe of the
involvement preoperatively. This liver with selective veno-veno
is often best staged with an MRV. bypass has been used success-
• Staging includes: fully.
o level 1 – into the renal vein • For cases with level 4 thrombus,
and ostium. cardiopulmonary bypass ad
o level 2 – extending up the hypothermic circulatory arrest is
IVC to below the hepatics. the preferred approach.
o level 3 – involving the intra- • Bland thrombus without active
hepatic portion of the IVC. RCC may be seen in the IVC
o level 4 – extending above the below the level of the renal veins.
diaphragm. If extensive, it need not be fully
• The prognostic significance of the resected but consideration should
level of involvement has been be made for placement of an IVC
controversial but unlikely mean- clip or filter or for IVC ligation
ingful. (with GIA) to prevent subsequent
• 5-year survival for fully resected embolization.
Mo RCC with IVC involvement is • Mortality rates associated with
reported to be 50%–70%. radical nephrectomy and IVC
• Preoperative angio infarction may thrombus have ranged as high as
be useful in cases of arterializa- 5%–10%.
tion of the thrombus or in patients
with excessive hilar adenopathy
in whom ligating the artery first
may be problematic.

450 edUCational reVieW manUal in UroloGy


• Cytoreductive nephrectomy o Patients in whom metastasectomy is
o Removal of the kidney in patients most often beneficial are those that
with metastatic RCC is termed cytore- have favorable metastatic biology
ductive. including solitary lesions, pulmonary
o In the immunotherapy era, some lesions, and long disease-free inter-
patients were unable to proceed to vals in the case of metachronous dis-
systemic therapy, calling into question ease.
the benefit of cytoreduction. o Those with multiple, hepatic and/or
o In randomized prospective trials osseous metastases presenting with
sponsored by SWOG and EORTC, the synchronous metastatic disease
role of cytoreductive nephrectomy and/or a short disease-free interval are
was shown to be associated with a sta- poorer candidates for metastasec-
tistically significant improved overall tomy.
survival in the immunotherapy era. o The MSKCC criteria may be used to
o Selection is key to identifying those stratify the risk of metastasectomy or
most likely to benefit from cytoreduc- systemic therapy in patients with
tive nephrectomy. Patients should mRCC and include performance sta-
have a good performance status, tus, weight loss, serum calcium, LDH,
lesser metastatic disease (ie, bulk of alkaline phosphatase, hemoglobin,
disease represented by the primary platelets and sedimentation rates
tumor), and no/minimal hepatic or o Local recurrence following radical
osseous metastases. CNS metastases nephrectomy is uncommon and
are considered a contraindication with occurs in about 2%. Only about 40%
rare exception. of local recurrences are isolated and
o Cytoreduction has been successfully the majority also has systemic dis-
performed laparoscopically; however, ease, such that an extensive disease
a laparoscopic approach should not evaluation should be pursued.
compromise the primary goal, which Many local recurrences are in
remains excision of all localized dis- adjacent nodes which were not
ease including involved regional removed at the time of surgery.
lymph nodes. This emphasizes the importance
o For locally advanced RCC involving of adequate preoperative evalua-
adjacent organs such as colon or tion and surgical resection.
spleen, cytoreduction is often neces- • Systemic therapies for mRCC:
sary to control local symptoms. En o Multiple adjuvant therapies have been
bloc resection is the primary aim. In assessed to reduce the risk of recur-
selective patients such as those with rence following resection of high-risk
solitary regional metastases, long- disease. Of these, only 1 positive trial
term survival is possible with wide has been published using autologous
excision. tumor vaccine (Jacham et al). Unfor-
o The role of cytoreductive nephrec- tunately, there were several design
tomy in the setting of TKIs is unclear and enrollment concerns that limit this
but under investigation. study’s clinical relevance. Therefore,
• Metastasectomy at the current time, the only role for
o Resection of synchronous or adjuvant treatment of high-risk RCC
metachronous metastatic RCC is in the context of a clinical trial.
(mRCC) is termed metastasectomy. o There is no convincing role for hor-
o Aggressive surgical resection may be monal or radiation therapies in mRCC
associated with favorable long-term other than for palliation.
survival.

Chapter 15: renal parenChymal and Upper Urinary traCt Urothelial neoplasms 451
• RCC is a chemoresistant tumor. Multi- Cytokine combination ther-
ple studies have demonstrated a low apy with IL-2 and IFN alpha
(2%–6%) response rate to multimodal suggest response rates of
chemotherapeutic approaches. approximately 19% but with
• Immunologic therapies have been the no improvement in overall
preferred treatment for mRCC for survival.
much of the last 2–3 decades. These o Adoptive immunotherapy
strategies include cytokines, adoptive This strategy involves the
immunotherapies and vaccines transfer of autologous lym-
o Cytokines: phocytes for the treatment of
Interferon (IFN) alpha – a mRCC including lym-
member of a group of phokine-activated killer
pleiotropic proteins with (LAK) cells, T cells and
antiviral, immunomodulatory tumor-infiltrating lympho-
and antiproliferative activities cytes (TIL).
related to modulation of gene Randomized trials combining
expression in selected cell cytokines with adoptive
populations. Overall response immunotherapy have not
rates in mRCC to single agent shown any improvement in
IFN alpha are approximately survival. Adoptive
12% with only 1.8% complete immunotherapy remains
responses and a median over- experimental.
all survival of 5–15 months. o Vaccines
Interleukin-2 (IL-2) – a T-cell The use of active specific
growth factor produced by immunotherapy to indirectly
activated T cells which medi- enhance the immune
ates its effect by binding to the response.
IL-2 receptor which causes Vaccines used in RCC include
expansion of cytotoxic T autologous tumor lysate com-
cells. The overall response binations, tumor fused with
rate to IL-2 is approximately dendritic cells (an antigen-
15% with durable complete presenting cell), dendritic cell
responses in 6-8%. IL-2 can vaccines and heat shock pro-
be administered in a variety of teins.
ways including as a bolus or No vaccine trial has demon-
continuous infusion, or subcu- strated improvement in sur-
taneously. In the US, IL-2 was vival and therefore these
approved on the basis of data remain investigational.
from a high-dose infusion • Targeted therapies
protocol. The potential side o With the identification of the
effect includes vascular leak VHL gene and subsequent delin-
syndrome (hypotension, olig- eation of portions of its pathways,
uria, multiorgan failure). various agents have been investi-
Median overall survival for gated which seek to perturb the
mRCC treated with IL-2 is VEGF pathway for therapeutic
12–18 months. Patients with benefit.
ccRCC respond the best to IL-
2.

452 edUCational reVieW manUal in UroloGy


o Currently, specific targeted thera- Sorafenib – an oral tyrosine
pies with activity in RCC carci- and Raf kinase receptor
noma exist. These have rapidly inhibitor approved for
reached first-line status in the mRCC. Has demonstrated
treatment of this tumor. The tar- improvements in progression-
geted therapies include: free survival, tumor regres-
Bevacizumab – an IgG1 mon- sion and stabilization of dis-
oclonal antibody that binds all ease in phase II/III trials. Side
isoforms of VEGF thereby effects include nausea, vomit-
inhibiting its signaling by ing, diarrhea, fatigue, hyper-
sequestering the circulating tension, and hand and foot
ligand. High-dose beva- syndrome.
cizumab has been shown to Temsirolimus – an intra-
increase time to progression venous inhibitor of mTOR
compared to placebo, (mammalian target of
although it did not improve rapamycin). This agent
overall survival. This was blocks translation of HIF1. It
among the first proof of prin- has been shown in a phase III
ciples that demonstrated that study of poor risk patients
therapies targeting the VEGF with mRCC to improve over-
pathway could be used for all survival vs IFN. It is not
therapeutic benefit. In combi- yet approved for use in RCC.
nation with IFN, beva- Other recently approved med-
cizumab has been noted to ications for mRCC include
increase survival (AVOREN pazopanib and axitinib (TKI)
trial). Its use as a single agent and evirolimus (oral mTOR
is not currently FDA inhibitor).
approved in mRCC. • Multimodal therapy
Sunitinib – an oral tyrosine o Most cases of advanced RCC are not
kinase receptor inhibitor treated by any one modality alone. The
approved for mRCC that combination of integrated surgery of
blocks multiple kinases the primary tumor with selective
including VEGFR-2, PDGF, surgery on metastases and combina-
cKit and Flt-3. In phase III tri- tion or sequential targeted systemic
als, sunitinib has been shown therapies is aimed at prolonging sur-
to have a higher response rate vival in patients with mRCC. Ran-
as measured by tumor regres- domized trials are continuing to vali-
sion (30%–40%) when com- date and refine these strategies.
pared directly to IFN with a
statistically significant • Special circumstances
improvement in progression- o Other malignant neoplasms of
free survival. Side effects the kidney
include fatigue, hypertension, - Sarcomas of the kidney
rash, diarrhea and potentially 1. Represent 1%–2% of all
thyroid and left ventricular malignant renal tumors in
dysfunction. adults with a peak incidence
in the 5th decade.

Chapter 15: renal parenChymal and Upper Urinary traCt Urothelial neoplasms 453
2. Renal sarcoma is less com- 3. Often asymptomatic.
mon but more lethal than 4. More common with non-
other genitourinary sarco- Hodgkin’s disease than with
mas. Hodgkin’s variants.
3. Differentiation of a renal sar- 5. Histologically diffuse forms
coma from a sarcomatoid predominate over nodular
variant of RCC may be diffi- forms.
cult. True renal sarcomas are 6. Primary renal lymphoma is
derived from mesenchymal very rare given the paucity of
elements of the kidney and lymphoid tissue in the normal
are typically surrounded by a kidney.
pseudocapsule, which is 7. CT characteristics of renal
often infiltrated by tumor lymphoma include adjacent
cells. adenopathy (particularly in
4. Often present with metastatic uncommon locations for
disease (most often to lung). advanced kidney cancer such
5. Median survival is months as in the mesentery),
for a high-grade renal sar- splenomegaly and diffuse
coma but may be longer for enlargement of the kidney
more indolent low-grade sar- which maintains its reniform
comas. appearance.
6. Most important prognostic 8. If renal lymphoma is sus-
variables in renal sarcomas pected, biopsy should be per-
are their histology, grade and formed with flow cytometry.
margin status. Leiomyosar- Treatment is systemic and the
coma (50%–60%) is the kidney should not be
most common type and may removed.
have a female preponder- 9. Other clinical clues include
ance. fever, weight loss, fatigue and
7. Liposarcoma is the 2nd most “B” symptoms of lymphoma.
common histology and must - Metastatic tumors to the kidney
be distinguished from 1. Metastatic tumors are the
angiomyolipoma (AML). most common kidney tumor.
8. Retroperitoneal sarcomas Autopsy studies reveal 12%
often involve but do not arise of patients dying from cancer
from the kidney. have metastases to their kid-
9. Wide and complete excisions neys.
are mandatory. 2. Nearly all represent
- Renal lymphoma and leukemia hematogenous spread.
1. Renal involvement in 3. Most common primaries
patients with hematologic include lung (most common
malignancies is commonly solid metastasis to the kid-
found in 34% of patients ney), breast, colon and
dying of progressive lym- melanoma.
phoma or leukemia at
autopsy.
2. Usually a late manifestation
of disease and represents
hematogenous spread.

454 edUCational reVieW manUal in UroloGy


4. Most often metastatic disease 5. Denys-Drash syndrome =
to the kidney is multifocal pseudohermaphroditism,
with small nodules and is mesangial sclerosis, majority
associated with metastatic with Wilms’ tumor.
disease elsewhere. Solitary 6. Beckwith-Wiedemann syn-
metastases to the kidney are drome = macroglossia, hemi-
fairly uncommon. hypertrophy, omphalocele,
5. If there is uncertainty as to the visceromegaly.
nature of a renal mass in a 7. Triphasic tumor consisting of
patient with a high-risk tumor variable proportions of undif-
elsewhere, a percutaneous ferentiated blastemal cells,
biopsy should be considered. epithelial component making
- Carcinoids of the kidney primitive rosettes or papillary
1. Arise from neuroendocrine structures, and stromal compo-
cells which are not normally nent.
present in the kidney. 8. Anaplasia is associated with
2. This is therefore an excep- worse outcomes.
tionally rare renal tumor. 9. 3% of Wilms’ tumors are seen
3. Correlations have been made in adults
to horseshoe kidneys 10. Most adult Wilms’ in young
(increased relative risk multi- adults (15–20 years old).
fold over normal kidneys). 11. Treatment of adult Wilms’
4. Most patients are asymp- tumor is similar to that in a
tomatic, a minority will pre- pediatric population and is
sent with carcinoid syndrome multimodal.
(flushing, wheezing, diar- 12. Favorable and unfavorable
rhea). histology.
5. Should consider evaluation - Other rare renal malignancies
for carcinoid elsewhere, such include PNET tumors and small cell
as with colonoscopy or upper carcinomas of the kidney.
GI endoscopy.
- Wilms’ tumor • Benign renal neoplasms
1. Most common abdominal o Oncocytoma
malignancy in children. 1. Represents 3%–7% of all solid
Derived from nephrogenic renal masses. As with RCC, demo-
blastemal cells; attempts to graphic is in adults in their 60s.
recapitulate developing kid- Male predominance 2:1.
ney. 2. Grossly light brown or mahogany
2. Mean age at diagnosis 3–4 in color, often with a stellate cen-
years; most occur <10 years tral scar.
of age, although rare adult 3. Microscopically uniform round
forms occur. eosinophilic (pink) cells containing
3. May be associated with abundant mitochondria, scant
WAGR, Denys-Drash syn- loose stroma and rare
drome, Beckwith-Wiede- mitoses/necrosis.
mann syndrome, and hemihy- 4. >10% may extend into perinephric
pertrophy, among others. fat and/or blood vessels.
4. WAGR = Wilms’ tumor, 5. Derived from the distal convoluted
aniridia, GU malformation, tubule.
mental retardation.

Chapter 15: renal parenChymal and Upper Urinary traCt Urothelial neoplasms 455
6. May be part of the Birt-Hogg- 10. Management depends on size
Dubé syndrome and be part of the and symptoms. Most symp-
spectrum of chromophobe carci- tomatic AMLs are large and
noma continuum. should be resected with NSS
7. Cannot be distinguished clini- approaches. Even very large
cally. radiographically or some- AMLs can safely be excised
times even on percutaneous while sparing the kidney.
biopsy from RCC. 11. A 4-cm cutoff is recognized as
8. Requires treatment as if it were one above which as many as
an RCC (excision, ablation as 80% of patients develop symp-
appropriate). toms, including spontaneous
o Angiomyolipoma (AML) bleeding in 10%.
1. Benign tumor containing varying 12. Selective embolization for
amounts of mature adipose tis- bleeding lesions is often effec-
sue, smooth muscle and thick- tive at stopping the hemorrhage.
walled vessels. Triphasic lesion Primary therapy with emboliza-
consisting of fat cells, spindled or tion has been used but may
epithelioid smooth muscle cells require repeat treatment.
and abnormal thick-walled blood 13. Epithelioid features may indi-
vessels. cate a more aggressive course.
2. Some lesions may demonstrate a 14. Expresses melanocytic markers
predominance of 1 or 2 elements. (HMB45, Melan A, MART) and
3. Found in 0.3% of all autopsies. smooth muscle markers
4. Female preponderance, with a (smooth muscle actin/SMA,
potential hormonal dependency desmin).
(rare before puberty). 15. Lacks epithelial markers/
5. Approximately 20%–30% of cytokeratins.
AMLs are found in patients with o Cystic nephroma (multiloculated
tuberous sclerosis (TS) which cystic nephroma – MLCN)
also has associated mental retar- 1. A benign lesion with a bimodal
dation, epilepsy, and adenoma age distribution. Occurs in young
sebaceum. Those associated with (2–3 years old) boys and middle
TS occur in younger women aged (40–50 years old) women.
(25–35 years old). May contain ovarian stroma
6. May present incidentally or after 2. More often symptomatic in
retroperitoneal hemorrhage adults.
(Wunderlich’s syndrome). Preg- 3. By definition all are multilocular
nancy increases the risk of spon- and therefore appear as Bosniak
taneous hemorrhage. III or IV cysts.
7. Diagnosed radiographically with 4. No reliable clinical or radio-
fat on CT, MRI or US. The graphic means of distinguishing
amount of fat in an AML varies MLCN from cystic RCC.
and in 14% there may be no fat. 5. Managed surgically with NSS
8. Occasionally extrarenal occur- whenever clinically feasible.
rences have been reported in
lymph nodes or even into the
renal vein.
9. Uniformly benign clinical
course.

456 edUCational reVieW manUal in UroloGy


o Leiomyoma o Hypercalcemia may occur in up to
1. Slow-growing benign renal 13% of patients with RCC and can be
tumor arising from the capsule or due to either bony involvement or the
peripelvic tissues. Rarely arises production of parathyroid hormone-
from the renal vein. like peptides. Signs of hypercalcemia
2. No reliable clinical or radio- include nausea, anorexia, fatigue and
graphic means of distinguishing decreased deep tendon reflexes. Man-
from incidental RCC, although agement includes hydration, diuretics
clearly capsular location should and selective use of bisphosphonates
raise suspicion. and steroids.
o Juxtaglomerular tumors (reninomas) o Hypertension can be due to local
1. A benign hemangiopericytoma compression of the renal artery,
derived from the specialized jux- arteriovenous fistula or
taglomerular cells. overproduction of renin.
2. Clinical presentation dominated o Polycythemia can be due to
by hypersecretion of renin with overproduction of erythropoietin.
hypertension, hypokalemia, o Nonmetastatic hepatic dysfunction
polydipsia, polyuria, myalgias (Stauffer’s syndrome) has been
and headaches. reported in 3%–20% of RCC cases
3. Female predominance, 3rd to 4th and is associated with an increase in
decade of life. serum alkaline phosphatase, pro-
4. Often at or near the corti- thrombin time and/or elevated biliru-
comedullary junction. Often rela- bin. It may also be due to elevations in
tively small and amenable to interleukin-6.
NSS. o In general, paraneoplastic syndromes
5. Cured by excision. associated with RCC require nephrec-
6. Should exclude renal artery tomy.
stenosis as cause of hyperrenine-
mia.
o Metanephric adenoma
1. Benign, occasionally
symptomatic, with peak
incidence in the 5th decade.
2. Small basophilic cells.
3. A few reported cases of associ-
ated malignant stromal elements.
4. No reliable clinical or radio-
graphic means of distinguishing
from incidental RCC. Therefore
excision, preferably by NSS, is
indicated.

• Paraneoplastic syndromes
o Found in 20% of patients with RCC.
o Renal tumors are capable of overpro-
ducing 1,25-dihydroxycholecalcif-
erol, renin, erythropoietin, and/or vari-
ous prostaglandins which may cause a
myriad of constitutional symptoms
(see Table 11).

Chapter 15: renal parenChymal and Upper Urinary traCt Urothelial neoplasms 457
2. Upper Urinary Tract
Urothelial Carcinoma

a. Epidemiology and Basic Biology b. Pathology

Less common than bladder urothelial carcino- Most upper tract urinary cancers are urothelial.
mas and more often associated with familial A small percentage are squamous or adenocar-
syndromes. cinomas.
Account for 5%–7% of all renal tumors and 5% The pathology of upper tract urothelial cancers
of all urothelial tumors. follows the WHO classification for bladder
Highest incidence in the Balkan states (may cancers (see Table 12).
account for up to 40% of all renal cancers). As with bladder tumors, 55%–75% of renal
Peak incidence in the 70–79 year old age group pelvic tumors are low grade and low stage and
(mean 65 years old). 85% are papillary.
Approximately 60% of upper tract urothelial In the ureter, invasion is more common than in
tumors are in the renal pelvis while 40% are in the bladder.
the ureters. Upper tract urothelial carcinomas are more
Men are affected twice as often, with Cau- common in the lower ureter (70%) than mid
casians affected twice as often as African ureter (25%) or upper ureter (5%).
Americans. Patients with upper tract urothelial cancer are at
Risk of developing upper tract urothelial carci- risk for developing bladder cancer. The magni-
noma in patients with a history of bladder can- tude of this risk is likely associated with grade,
cer is associated with the stage and grade of the stage, multifocality, CIS, and other co-risk fac-
bladder cancer and is highest among patients tors such as smoke exposure.
with CIS in the bladder. This may reflect
upward spread of the disease into the distal c. Evaluation, Diagnosis and Imaging
ureter or simply a high-risk urothelial pheno-
type. The most common symptom is hematuria
Etiology and risk factors (micro or gross).
o Balkan Nephropathy – a degenerative Flank pain may imply obstruction.
interstitial nephropathy. Interestingly, Diagnosis includes:
bladder cancer rates are not affected. 1. Full history and exam.
o Smoking – as with bladder cancer, this is 2. Laboratory evaluation includes liver and
the most important modifiable risk factor, renal function tests, urinalysis and cytolo-
increasing the risk 3-fold. Risk appears gies
dose related. Smoking seems to increase a. Cytology will identify most CIS and
the risk of ureteral tumors higher than it high-grade tumors but is often falsely
does renal pelvic tumors. negative with low-grade tumors.
o Coffee consumption may increase the risk. b. Upper tract washings may be more
o Analgesic abuse. sensitive than voided urine but like-
o Occupational exposure to coal, asphalt wise may be contaminated from the
or tar, petroleum, aniline dyes and lower urinary tract.
other amines. c. Interpretation of cytologies may be
o Hereditary – Lynch Syndrome II is associ- hampered by contrast materials.
ated with colon tumors and upper tract 3. Radiographic evaluation
urothelial carcinoma. An autosomal-dom- a. Includes the use of IVP, CT urograms
inant defect in the DNA mismatch repair and retrograde pyelograms
genes hMLH1 and hMSH2. i. The differential diagnosis of a
filling defect on these studies is
listed in Table 13

458 edUCational reVieW manUal in UroloGy


4. Endoscopic evaluation 2. Radical nephroureterectomy and
a. Ureteroscopy or nephroscopy nephron-sparing techniques are also
i. When combining radiographic available for upper tract urothelial tumors.
evaluations and endoscopy, the Nephron-sparing approaches should be
diagnostic accuracy of an upper done for low-grade, low-stage lesions
urinary tract defect is approxi- (open or endoscopic) and for relative or
mately 85%–90%. absolute indications as with RCC. NSS
ii. Histologic correlation between options include:
ureteroscopic biopsy and final a. Segmental parenchymal resection.
pathology is 78%–90%; however, b. Open pyelotomy with tumor excision
it is often difficult to get a good and ablation. This is an uncommon
specimen given the spatial limita- means of treatment and generally
tions of small instruments. There- requires a large extrarenal pelvis and
fore, correlation with grade and a papillary low-grade tumor.
stage is poor and requires correla- c. Segmental ureteral resection with
tion of endoscopic, cytologic and reconstruction. The type of recon-
radiographic features. struction depends on the location and
extent of the tumor and includes:
d. Staging and Prognosis i. Primary end-to-end ureteral
anastomosis.
Staging of upper urinary tract TCC follows ii. Direct bladder reimplantation.
TNM Staging guidelines (see Table 14). iii. Reimplantation with bladder
Upper tract urothelial carcinomas are generally hitch or flap.
associated with a poor prognosis with up to iv. Ureteral substitution (ie, ileal
19% of patients presenting with metastatic dis- ureter).
ease. Prognosis relates to grade and stage and 3. Radical nephroureterectomy with exci-
appears better for renal pelvic tumors than for sion of a bladder cuff to include the intra-
ureteral tumors. In the ureter, invasion is more mural portion of the ureter is considered
common than in the bladder. This may be the gold standard for large, high-grade,
related to a more well-developed muscle layer invasive or multifocal tumors. Ipsilateral
and the presence of the nearby renal adrenalectomy is not considered neces-
parenchyma in the upper urinary tract. sary.
• Upper urinary tract urothelial cancers may a. Regional lymphadenectomy is usu-
spread by direct extension, or by lymphatic or ally performed for high-risk tumors as
vascular invasion. a staging measure. The therapeutic
benefit of an extensive lymphadenec-
e. Management tomy has not yet been determined but
may ultimately be shown to be of sim-
Laparoscopic/robotic and open approaches ilar value as new data demonstrate in
1. As with RCC, treatment for upper tract bladder cancer.
urothelial cancer can be performed in an 4. Laparoscopic approaches for upper uri-
open or laparoscopic technique. In choos- nary tract urothelial carcinomas follow
ing the approach, clinical and anatomic the same basic principles as discussed in
factors must be considered as well as the RCC. Approaches include:
comfort level and experience of the sur- a. Pure laparoscopic trans or
geon. retroperitoneal surgery.
b. Hand-assisted techniques.

Chapter 15: renal parenChymal and Upper Urinary traCt Urothelial neoplasms 459
c. Management of the distal and intra- deeper penetration (5–6 mm).
mural portion of the ureter should not Settings are 5–15 watts for 2
be compromised. Options include: seconds.
i. Open excision through a low 2. Tumor can be resected with a
extraction port. ureteral resectoscope. Extra
ii. Aggressive transurethral resec- care is needed in the mid-to-
tion of the ureteral orifice. This upper ureter.
should be reserved for proximal, iii. Antegrade percutaneous manage-
low-grade tumors. ment has the advantage of the ability
iii. Intussusception (stripping) tech- to use larger instruments that can
nique – after laparoscopic remove a larger volume of tumor.
nephrectomy, a previously Staging and grading is ordinarily bet-
placed ureteral catheter is used ter as well, given the amount of
to intussuscept the ureter into the tumor that can be removed. Addi-
bladder where it is excised. tionally, lower pole tumors are more
iv. Transvesical laparoscopic liga- easily accessed and topical therapies
tion and detachment – performed can more easily be applied. It is best
via 2 transvesical ports. The reserved for larger, more extrarenal
ureter is tented up and looped. A pelvises.
Collins knife is then used to cir- 1. Steps include:
cumscribe the orifice. a. Establishment of a
v. Totally laparoscopic excision – nephroscopy tract.
requires laparoscopic detrussor b. Biopsy and definitive ther-
incision with dissection of the apy.
intramural portion and resection. c. Second look within 4–14
Endoscopic days to allow for adequate
1. Ureteroscopy, pyeloscopy and healing with rebiopsy and
nephroscopy fulguration of the base.
a. Best preserved for low-grade, unifo- 2. Results of endoscopic manage-
cal lesions of the ureter, renal pelvis or ment:
upper pole calyx. a. For retrograde management,
b. Steps for retrograde ureteroscopic the overall recurrence rates
management include: are approximately 30% and
i. Endoscopic evaluation and depend on stage, grade, size
collection of cytologies. and focality.
ii. Biopsy and definitive treatment. b. Complications of
1. Tumor may be debulked with ureteroscopy and their man-
a forceps or basket. The base agement are similar to endo-
is then treated with fulgura- scopic management of
tion or laser. Holmium: Yag stones.
laser is safer in the ureter c. Series for antegrade manage-
because of its shallower pen- ment are small. Most low-
etration (<0.5 mm). Settings grade, smaller, unifocal
for this laser are energy of lesions do well. Higher
0.6 to 1 Joule with a fre- grade, invasive lesions do
quency of 8–10 hertz. Nd: worse. Nephroscopy tract
Yag may be better in the seeding appears uncommon.
pelvis. It works by coagula-
tive necrosis and has a

460 edUCational reVieW manUal in UroloGy


Topical and systemic therapies:
1. Instillation therapy.
a. Can be performed antegrade through
a nephroscopy tract, retrograde (20
cm through a 4fr ureteral catheter) or
by reflux up an indwelling ureteral
stent.
b. Basic principles include:
i. Rule out infection first.
ii. Low pressure instillation.
c. Most series are small, retrospective,
highly selected and not controlled.
d. The same agents used in bladder can-
cer have been used in the upper uri-
nary tracts including BCG, mito-
mycin and thiotepa.
e. The accumulated experience appears
encouraging in properly selected
patients.
f. Most common complication of upper
tract instillation is sepsis.
2. Systemic therapy
a. Small, highly selected series evaluat-
ing adjuvant radiation suggest it may
decrease local recurrence but not sys-
temic disease.
b. Both induction (neoadjuvant) and
adjuvant therapies are reasonable.
Neoadjuvant chemotherapy may be a
reasonable option for patients with
bulky regional adenopathy.
c. There are no prospective controlled
trials evaluating chemotherapy for
upper urinary tract urothelial cancers.
Most of the data are extrapolations
from bladder cancer series.

Chapter 15: renal parenChymal and Upper Urinary traCt Urothelial neoplasms 461
Table 1

Familial RCC Syndromesa

Syndrome Gene/Protein Suppressor/Oncogene Manifestation

Von hippel-lindau 3p25/pVhl tumor suppressor Clear cell rCC


(Vhl) Cystic rCC
retinal angiomas
Cns hemangioblastomas
pheochromocytoma

hereditary papillary 7q34/cmet oncogene type 1 papillary rCC


rCC (hprCC) no extra renal
manifestations

Birt-hogg-dubé 17p11.2/folliculin tumor suppressor Chromophobe rCC


(Bhd) oncocytoma/oncocytosis
Cutaneous fibrofolliculomas
lung cysts pneumothorax

hereditary leiomyoma 1q42/fumurate tumor suppressor Cutaneous leiomyomas


rCC (hlrCC) hydratase Uterine fibroids
type 2 papillary rCC

a
All are autosomal dominant

Table 2

Incidence and Age of Manifestations of VHL

Organ Lesion % Incidence Mean Age of Onset (yrs)

eye retinal angioma 25-60 25

Cns hemangioblastomas 10-72 33

adrenal pheochromocytoma 10-20 30

pancreas islet cell tumors 12 36


Cysts 20-70 36

epididymis Cystadenomas 25-60 Unknown

ear endolymphatic sac 10 22


tumors

462 edUCational reVieW manUal in UroloGy


Table 3

Differential Diagnosis of More Common Adult Renal Tumors

Benign Renal Tumors

renal capsular tumors Fibroma


leoimyoma
lipoma

renal parenchymal (epithelial) tumors oncocytoma


adenoma
Cystic nephroma
metanephric adenoma

Vascular tumors hemangiomas


hamartoma
lymphangioma
renal artery aneurysm
aV malformation

pseudotumors hypertrophic Column of Bertin

mesenchymal tumor leiomyoma


reninoma (JG tumor)
angiomyolipoma

Malignant Renal Tumors

tumors of the renal pelvis and ureter papilloma


Urothelial carcinoma
squamous carcinoma
adenocarcinoma

renal parenchymal tumors Conventional carcinoma (clear cell)


papillary carcinoma
Chromophobe carcinoma
medullary carcinoma
Collecting duct carcinoma
mucinous tubular/spindle cell
translocation tumors

renal sarcomas leiomyosarcoma


liposarcoma
angiosarcoma
hemangiopericytoma
malignant fibrous histocytoma
rhabdosarcoma

other renal tumors Carcinoid


lymphoma/leukemia
metastases
Wilms’ tumor

Inflammatory Renal Tumors

renal abscess

Focal pyelonephritis

Xanthogranulomatous pyelonephritis

tuberculosis

Chapter 15: renal parenChymal and Upper Urinary traCt Urothelial neoplasms 463
Table 4 Table 4 (continuted)

AJCC TNM Staging RCC AJCC TNM Stage Stage Grouping for RCC
(v7)

Primary Tumor Stage (T)

t1a – tumor ≤4 cm stage i t1 n0 m0

t1b – tumor 4 < x ≤7 cm stage ii t2 n0 n0

t2a – tumor 7–10 cm, limited to kidney stage iii t1-t2 n1 m0


t3 n0-1 m0
t2b – tumor >10 cm, limited to kidney
stage iV t4 n0-1 m0
t3a – perirenal fat and/or renal sinus fat and/or any any m1
invasion of renal vein or segmental branches
(muscle containing) www.cancernomograms.com

t3b – involves renal vein or iVC below diaphragm

t3c – iVC above diaphragm; invasion of iVC wall

t4 – tumor invades beyond Gerota’s fascia or


contiguous extension involving the ipsilateral
adrenal

Regional Lymph Nodes (N)

nx – regional nodes cannot be assessed

n0 – no regional nodal involvement

n1 – metastasis any regional ln

Distant Metastases (M)

mx – distant metastases cannot be assessed

m0 – no distant metastases

m1 – distant metastases

www.cancernomograms.com

464 edUCational reVieW manUal in UroloGy


Table 5

Clues for the Differential Diagnosis of the Radiographic Renal Mass

Fever, flank pain, pyuria think inflammatory (pyelonephritis or abscess)

problematic stones — think of XGp

spontaneous rp bleed — think of underlying rCC

speckled Ca++ — think of possible rCC

look for fat by mri, Us or Ct and consider aml, liposarcoma

For upper pole renal lesions always look for the adrenal

always assess perirenal soft tissues

For calcified rounded hilar lesion — think about a renal artery aneurysm

Cystic lesions in middle-aged female — think of cystic nephromas

Be your own best radiologist

Table 6

Bosniak CT Classification of Renal Cystic Lesions

Bosniak Class Cancer Risk Rx Comments

i (simple cyst) 0% none unless no enhancement


symptomatic smooth, empty
no follow-up
needed

ii may be hyperdense <10% none or follow Few septa


(protein/blood) (Bosniak iiF) thin linear
calcification
no enhancement

iii (indeterminate) 50% remove thick irregular


wall or septa
thick calcifications
moderate septa
may enhance
mild to moderate
heterogeneity

iV (Cystic rCC) 80%-≥90% remove enhancing nodules

Chapter 15: renal parenChymal and Upper Urinary traCt Urothelial neoplasms 465
Table 7

Risk Stratification for RCC

Table 8

Patterns of Recurrent RCC

Site of Recurrence Median Time to Recur (years)

lung (38%) 1.6

Bone (18%) 1.5

liver (13%) 1.7

local (includes adrenal) 10% 1.7

Cns (8%) 2.5

466 edUCational reVieW manUal in UroloGy


Table 9

Growth and Progression of RCC Under Active Surveillance 2010


(18 non-redundant studies)

N Range Median Range Median Median Growth Range Median F/U # Progressed
Age (years) Size (cm) Rate (cm/year) Duration (months) to MRCC

880 54–80 0.9–6.0 0.08–0.58 20–41 17 (1.9%)

overall mean follow-up = 34 months


35% of lesions exhibited net ZERO radiographic growth (none progressed)

Table 10a

Low Risk NCCN Surveillance Protocols

www.cancernomograms.com

Chapter 15: renal parenChymal and Upper Urinary traCt Urothelial neoplasms 467
Table 10b

Intermediate Risk NCCN Surveillance Protocols

www.cancernomograms.com

Table 10c

High Risk NCCN Surveillance Protocols

www.cancernomograms.com

468 edUCational reVieW manUal in UroloGy


Table 11

Paraneoplastic Syndromes in RCC

Cachexia/fever 20%–30%
nephropathy (ig formation) 27%
htn (renin) 25%
hypercalcemia 20%
- metastatic
- nonmetastatic (pth-like)
anemia (cytokin myelosuppression 20%–40%
hyperglycemia 10%–20%
stauffer’s*(? il-6) 3%–20%
erythrocytosis (epo) 1%–8%
amyloidosis 3%–5%

*Nonmetastatic hepatic dysfunction

Table 12

WHO Pathologic Classification of Upper Tract Urothelial Cancers

Papillary Urothelial Lesion Flat Urothelial Lesions

papillary hyperplasia Flat urothelial hyperplasia

Urothelial papilloma Flat urothelial atypia

papillary urothelial neoplasm of lmp Urothelial atypia of unknown significance

low grade urothelial carcinoma dysplasia

high grade urothelial carcinoma Carcinoma in situ

Chapter 15: renal parenChymal and Upper Urinary traCt Urothelial neoplasms 469
Table 13

Differential Diagnosis of an Upper Tract Filling Defect

1. radiolucent stone 7. Urothelial cancer

2. Blood clot 8. Ureteritis/pyelitis cystica

3. renal papillae 9. tuberculosis

4. Fungus ball 10. Benign or traumatic ureteral stricture

5. extrinsic vascular compression 11. endometriosis

6. renal parenchymal tumor

Get a good history and look at the films yourself!

Table 14

AJCC Staging for Upper Urinary Tract Urothelial Cancers (2010)

Primary Tumor Stage (T)

tx – primary tumor cannot be staged

t0 – no evidence of primary tumor

ta – noninvasive

tis – Carcinoma-in-situ

t1 – invades lamina propria

t2 – invades muscularis

t3 – invades sinus/peri-ureteral fat or renal parenchyma

t4 – invades adjacent organs

Nodal Staging

n1 – single node ≤ 2 cm

n2 – nodes 2< x ≤ 5 cm

n3 – nodes > 5 cm

470 edUCational reVieW manUal in UroloGy


3. Selected Readings

Renal Cell Carcinoma Reviews Uzzo RG, Novick AC. von Hippel Lindau syn-
drome: Clinical and molecular considerations for the
Campbell SC, Novick AC, Bukowski RM. Renal urologist. AUA Update Series. 1999;18:137-144.
Tumors. In: Wein AJ, Kavoussi LR, Novick AC,
Partin AW, Peters CA. Campbell-Walsh Urology. 9th Sudarshan S, Linehan WM. Genetic basis of cancer
ed. Philadelphia, PA: WB Saunders;2007:1567. of the kidney. Semin Oncol. 2006;33(5):544-451.

Cohen HT, McGovern FJ. Renal-cell carcinoma. Choyke PL, Glenn GM, Walther MM, et al. The nat-
N Engl J Med. 2005;353:2477-2490. ural history of renal lesions in von Hippel-Lindau
disease: a serial CT study in 28 patients. AJR Am J
Motzer RJ, Bander NH, Nanus DM. Renal-cell car- Roentgenol. 1992;159: 1229.
cinoma. N Engl J Med. 1996;335:865-875.
Duffey BG, Choyke PL, Glenn G, et al. The relation-
Drucker BJ. Renal cell carcinoma: current status and ship between renal tumor size and metastases in
future prospects. Cancer Treat Rev. 2005;31:536- patients with von Hippel-Lindau disease. J Urol.
545. 2004;172:63.

Epidemiology of RCC Pathology, Evaluation, Imaging


and Prognosis of RCC
Hock LM, Lynch J, Balaji KC: Increasing incidence
of all stages of kidney cancer in the last 2 decades in Frank I, Blute ML, Cheville JC, et al. Solid renal
the United States: an analysis of surveillance, epi- tumors: an analysis of pathologicalal features related
demiology and end results program data. J Urol. to tumor size. J Urol. 2003;170:2217.
2002;167: 57.
Frank I, Blute ML, Leibovich BC, Cheville JC,
Jemal A, Murray T, Ward E, et al. Cancer statistics, Lohse CM, Zincke H. Independent validation of the
2005. CA Cancer J Clin. 2005;55:10-30. 2002 American Joint Committee on cancer primary
tumor classification for renal cell carcinoma using a
Hollingsworth JM, Miller DC, Daignault S, et al: large, single institution cohort. J Urol. 2005;173:
Rising incidence of small renal masses: a need to 1889.
reassess treatment effect. J Natl Cancer Inst.
2006;98:1331-1334. Lam JS, Shvarts O, Leppert JT, et al. Renal cell car-
cinoma 2005: new frontiers in staging, prognostica-
Molecular Biology and Kidney tion and targeted molecular therapy. J Urol. 2005;
Cancer Syndromes 173:1853-1862.

Linehan WM, Lerman MI, Zbar B. Identification of Fuhrman SA, Lasky LC, Limas C. Prognostic signif-
the von Hippel-Lindau (VHL) gene. Its role in renal icance of morphologic parameters in renal cell carci-
cancer. JAMA. 1995;273:564-570. noma. Am J Surg Path. 1982;6:655-663.

Maher ER, Kaelin WG. von Hippel-Lindau disease. Lam JS, Schvarts O, Leppert JT, et al. Postoperative
Medicine (Baltimore). 1997;76:381-391. surveillance protocol for patients with localized and
locally advanced renal cell carcinoma based on a val-
Seizinger BR, Rouleau GA, Ozelius LJ, et al. Von idated prognostic nomogram and risk group stratifi-
Hippel-Lindau disease maps to the region of chro- cation system. J Urol. 2005;174:466-472.
mosome 3 associated with renal cell carcinoma.
Nature. 1988;332:268-269. Zisman A, Pantuck AJ, Dorey F, et al. Improved
prognostication of renal cell carcinoma using an
integrated staging system. J Clin Oncol. 2001;
19:1649-1657.

Chapter 15: renal parenChymal and Upper Urinary traCt Urothelial neoplasms 471
Kattan MW, Reuter V, Motzer RJ, et al. A postopera- Kutikov A, Egleston BL, Wong YN, Uzzo RG. Eval-
tive prognostic nomogram for renal cell carcinoma. uating overall survival and competing risks of death
J Urol. 2001;166:63-67. in patients with localized renal cell carcinoma using
a comprehensive nomogram. J Clin Oncol. 2010;28:
Frank I, Blute ML, Cheville JC, et al. An outcome 311-317.
prediction model for patients with clear cell renal
cell carcinoma treated with radical nephrectomy Rothman J, Egleston B, Wong YN, et al.
based on tumor stage, size, grade and necrosis: The Histopathologic characteristics of localized renal
SSIGN score. J Urol. 2002;168:2395-2400. cell carcinoma correlate with tumor size: a SEER
analysis. J Urol. 2009;181:29-34.
Lane BR, Kattan MW, Novick AC. Prediction mod-
els of renal cell carcinoma. AUA Update Series. Crispen PL, Boorjian SA, Viterbo R, et al. Natural
2006;7:57-67. history, growth kinetics and outcomes of untreated
clinically localized renal tumors under active
Motzer RJ, Bacik J, Schwartz LH, et al. Prognostic surveillance. Cancer. 2009;115 (13):2844-2852.
factors for survival in previously treated patients
with metastatic renal cell carcinoma. J Clin Oncol. Kunkle DA, Egleston BL, Uzzo RG. Excise, ablate
2004;22:454-463. or observe: the small renal mass dilemma – a meta-
analysis and review. J Urol. 2008;179:1227-1234.
Management of RCC
Smaldone MC, Kutikov A, Egleston BL, et al. Small
Biopsy renal masses progressing to metastases under active
surveillance: a systematic review and pooled analy-
Lane BR, Samplaski MK, Herts BR, et al. Renal sis. Cancer. 2012;118(4):997-1006.
mass biopsy--a renaissance? J Urol. 2008;179:20.
Surgery
Sanchez-Ortiz RF, Madsen LT, Bermejo CE, et al. A
renal mass in the setting of a nonrenal malignancy: Campbell SC, Novick AC, Belldegrun A, et al.
when is a renal tumor biopsy appropriate? Cancer. Guideline for management of the clinical T1 renal
2004;101:2195. mass. J Urol. 2009;182:1271-1279.

Neuzillet Y, Lechevallier E, Andre M, et al. Follow- Uzzo RG, Novick AC. Nephron sparing surgery for
up of renal oncocytoma diagnosed by percutaneous renal tumors: indications, techniques and outcomes.
tumor biopsy. Urology. 2005;66:1181. J Urol. 2001;166:6-18.

Active surveillance Lane BR, Russo P, Uzzo RG, et al. Comparison of


warm and cold ischemia during open partial
Volpe A, Panzarella T, Rendon RA, et al. The natural nephrectomy for tumor in 660 solitary kidneys
history of incidentally detected small renal masses. reveals predominant role of nonmodifiable factors in
Cancer. 2004;100:738. determining ultimate renal function. J Urol. 2011;
185:421-427.
Chawla SN, Crispen PL, Hanlon AL, et al. The natu-
ral history of observed enhancing renal masses: Kutikov A, Piotrowski ZJ, Canter DJ, et al. Routine
meta-analysis and review of the world literature. adrenalectomy is unnecessary during surgery for
J Urol. 2006;175:425-431. large and/or upper pole renal tumor when the adrenal
is radiographically normal. J Urol. 2011; 185:1198-
1203.

472 edUCational reVieW manUal in UroloGy


Toujer K, Jacqmin D, Kavoussi LR, et al. The Metastatic disease
expanding role of partial nephrectomy: A critical
analysis of indications, results, and complications. Kim HL, Belldegrun AS, Freitas DG, et al. Paraneo-
Eur Urol. 2010;57:214-222. plastic signs and symptoms of renal cell carcinoma:
implications for prognosis. J Urol. 2003;170:1742-
Van Poppel H, Da Pozzo L, Albrecht W, et al. A 1746.
prospective, randomised EORTC Intergroup Phase 3
Study comparing the oncologic outcome of elective Rabbani F, Herr HW, Almahmeed T, Russo P. Tem-
nephron-sparing surgery and radical nephrectomy poral change in risk of metachronous contralateral
for low-stage renal cell carcinoma. Eur Urol. renal cell carcinoma: influence of tumor characteris-
2011;59:543-552. tics and demographic factors. J Clin Oncol.
2002;20:2370-2375.
Blom J, Van Poppel H, Mare´chal JM, et al. Radical
nephrectomy with and without lymph-node dissec- Jocham D, Richter A, Hoffman L, et al. Adjuvant
tion: final results of EORTC randomized Phase 3 autologous renal tumour cell vaccine and risk of
Trial 30881. Eur Urol. 2009;55:28-34. tumour progression in patients with renal-cell carci-
noma after radical nephrectomy: phase III, random-
Laparoscopy/robotics ized controlled trial. Lancet. 2004;363:594-599.

Gill IS, Kavoussi LR, Lane BR, et al. Comparison of Kunkle DA, Haas NB, Uzzo RG. Adjuvant therapy
1,800 laparoscopic and open partial nephrectomies for high-risk renal cell carcinoma patients. Curr Urol
for single renal tumors. J Urol. 2007;178: 41. Rep. 2007;8:19-30.

Scoll BJ, Uzzo RG, Chen DYT, et al. Robot assisted Flanigan RC, Salmon SE, Blumenstein BA, et al.
partial nephrectomy: A large single institutional Nephrectomy followed by interferon alfa-2b com-
experience. Urology. 2010;75(6):1328-1334. pared with interferon alfa-2b alone for metastatic
renal cell cancer. N Engl J Med. 2001;345:1655-
Reyes JM, Smaldone MC, Uzzo RG, Viterbo R. Cur- 1659.
rent status of robot-assisted partial nephrectomy.
Curr Urol Rep. 2012;13(1):24-37. Motzer RJ, Rini BI, Bukowski RM, et al. Sunitinib in
patients with metastatic renal cell carcinoma. JAMA.
Ablation 2006;295:2516-2524.

Kunkle DA, Egleston BL, Uzzo RG. Cryoablation Motzer RJ, Bukowski RM. Targeted therapy for
vs. radiofrequency ablation of the small renal mass: metastatic renal cell carcinoma. J Clin Oncol. 2006;
A meta-analysis of published literature. Cancer. 24:5601.
2008;113:2671-2680.
Motzer RJ, Hutson TE, Tomczak P, et al. Phase III
Matin SF, Ahrar K, Cadeddu JA, et al. Residual and randomized trial of sunitinib versus interferon-alfa
recurrent disease following renal energy ablative (IFN-α) in metastatic renal cell carcinoma (mRCC).
therapy: a multi-institutional study. J Urol. 2006; N Engl J Med. 2007;356:115-124.
176:1973-1977.
Coppin C, Porzsolt F, Awa A, Kumpf J, Coldman A,
Hegarty NJ, Gill IS, Desai MM, et al. Probe-ablative Wilt T. Immunotherapy for advanced renal cell can-
nephron-sparing surgery: cryoablation versus cer. Cochrane Database Syst Rev. 2005;(1):
radiofrequency ablation. Urology. 2006;68:7-13. CD001425.

Chapter 15: renal parenChymal and Upper Urinary traCt Urothelial neoplasms 473
4. Questions

Hudes G, Carducci M, Tomczak P, et al. Tem- 1. Which of the following hereditary renal
sirolimus, interferon alfa, or both for advanced tumor syndromes is incorrectly paired with
renal-cell carcinoma. N Engl J Med. 2007;356:2271- its appropriate gene?
2281.
A. Von Hippel Lindau – 3p
Escudier B, Bellmunt J, Négrier S, et al. Phase III
trial of bevacizumab plus interferon alfa-2a in B. Birt Hogg Dube – 17p
patients with metastatic renal cell carcinoma
(AVOREN): final analysis of overall survival. J Clin C. Hereditary Papillary RCC – 7p
Oncol. 2010;28:2144-2150.
D. Hereditary Leiomyoma RCC – 1q
Upper Urinary Tract Urothelial Carcinoma
E. Tuberous Sclerosis – 9q
Flanigan RC. Urothelial tumors of the upper urinary
tract. In: Wein AJ, Kavoussi LR, Novick AC, Partin
AW, Peters CA, eds. Campbell-Walsh Urology. 9th
ed. Philadelphia, PA: WB Saunders; 2007:1638 PC. 2. Which of the following is true regarding the
VHL gene and its pathway
Sagalowski AI, Jarrett TM. Management of urothe-
lial tumors of the renal pelvis and ureter. In: Wein A. Mutation of VHL is primarily epigenetic
AJ, Kavoussi LR, Novick AC, Partin AW, Peters (ie not a sequence mutation)
CA, eds. Campbell-Walsh Urology. 9th ed. Philadel-
phia, PA: WB Saunders; 2007:1653. B. The VHL gene is an oncogene

Brien JC, Shariat SF, Herman MP, et al. Preoperative C. The VHL protein is physiologically over-
hydronephrosis, ureteroscopic biopsy grade and uri- expressed during times of excess oxygen
nary cytology can improve prediction of advanced tension
upper-tract urothelial carcinoma. J Urol. 2010;184:
69-73. D. HIF proteins are overexpressed as a func-
tion of mutated VHL gene/protein

E. The VHL protein is a transcriptional fac-


tor

3. Which of the following is true regarding the


Bosniak classification of renal cysts?

A. The Bosniak classification of renal cysts


is an ultrasound based system

B. Hyperdense cysts are classified as


Bosniak III

C. Simple renal cysts (Bosniak I) are con-


firmed only on contrast based cross sec-
tional imaging

474 edUCational reVieW manUal in UroloGy


D. A Bosniak IIF has a risk of occult cancer 6. Which is true regarding adrenalectomy dur-
in excess of 65% ing surgery for RCC?

E. Calcium within the wall of a cyst can A. Routine removal is safe and recom-
occur in Bosniak II, III or IV lesions mended

B. The adrenal gland is commonly involved


in pT2 lesions and should be removed in
4. Which is true regarding Nephrogenic Sys- all lesions >7cm
temic Fibrosis?
C. Involvement of the adrenal is now staged
A. It typically occurs within 24 hours of as T4
gadolinium administration
D. Partial adrenalectomy is never acceptable
B. It causes a fibromyalgia type syndrome
E. Involvement of the contralateral adrenal
C. It occurs most commonly in patients with is the only indication to preserve the ipsi-
an estimated GFR between 30 and 60 lateral adrenal
cc/min

D. It may be fatal
7. Which of the following is true about parane-
E. If is does not happen after a single dose of oplastic syndromes in RCC
gadolinium it is unlikely to occur with
future doses A. IL-8 is thought to cause hepatic dysfunc-
tion (Stauffer’s syndrome)

B. Hyperlipidemia is a common paraneo-


5. Which is true about infrarenal thrombus in plastic syndrome
cases of IVC involvement?
C. Hypertension associated with RCC is
A. Typically the thrombus below the renal always aldosterone mediated
veins is bland and need not be fully
resected but may require anticoagulation D. Hypercalcemia may be associated with
or caval interruption PTH production

B. The majority of thrombus must be E. Hand and foot syndrome is common


assumed malignant until proven other-
wise

C. Tumor thrombus has been known to


release IL6 and cause paraneoplastic syn-
dromes

D. Malignant infrarenal tumor thrombus


typically stops at the femoral veins

E. Infrarenal thrombus is a poor prognostic


sign

Chapter 15: renal parenChymal and Upper Urinary traCt Urothelial neoplasms 475
8. Which of the following is true of sunitinib? Answers:

A. It is an mTOR inhibitor 1. C.
The genetics of renal tumor syndromes have increas-
B. It is an antibody to VEGF-R ingly been unraveled using large familial pedigrees,
linkage analysis and ultimately isolation and
C. It directly inhibits HIF sequencing of the responsible gene. Of those listed,
C is incorrect – HPRCC is associated with the cMet
D. It is mediated via blockade of tyrosine gene – an oncogene located on the long arm of 7
kinases (7q31). HPRCC is the only hereditary syndrome
with no extra-renal manifestations
E. It is associated with a 60-80% overall
response rate 2. D.
VHL follows an autosomal dominant inheritance
pattern. It is a tumor suppressor gene thatfollows
Knudsen’s two hit hypothesis, meaning both alleles
9. Regarding the adverse events for targeted must be mutated for it to function abnormally. The
therapies, which is paired correctly? VHL protein has been shown to regulate HIF tran-
scription factors which are normally only overex-
A. mTOR inhibitors – hand and foot syn- pressed during hypoxia. However, mutant VHL
drome leads to over expression of HIF under normoxia

B. VEGFR antibodies – stomatitis 3. E.


The Bosniak classification is a CT based system for
C. TKIs – hyperlipidemia categorizing renal cysts. It requires pre and post
contrast images. That said, a simple cyst has defini-
D. TKIs – left ventricular dysfunction tive characteristics on US (no internal echos, good
through transmission with posterior wall enhance-
E. VEGFR antibodies – hypothyroidism ment). Hyperdense (hemorrhagic or proteinaceous)
cysts are Bosniak II. Bosniak IIF cysts do not meet
10. Which of the following is true of cisplatin strict criteria for either II and III and are generally
based chemotherapy for upper tract urothe- followed (hence the term F). Their risk of cancer is
lial carcinoma considered lower than a true Bosniak III. Thin linear
calcium can occur in BII cysts. Chunky or thick cal-
A. There are level 1 data to support its use in cium is a more worrisome sign (BIII or IV)
the neoadjuvant setting
4. D.
B. There are level 1 data to support its use in NSF is a scleroderma like reaction in patients receiv-
the adjuvant setting ing gadolinium in the setting of severe renal impair-
ment (eGFR<30). It can occur up to 3mo or more
C. Its primary mode of action is to inhibit after exposure and it may be related to cumulative
DNA covalent bond and cross linking exposure, although it has been reported following a
single doseand can becan be fatal
D. Its primary mode of action is to inhibit
microtubules

E. There is little activity when used as a sin-


gle agent in urothelial carcinoma

476 edUCational reVieW manUal in UroloGy


5. A.
Infrarenal tumor thrombus is typically bland. Clues
on an MRI may be seen including lack of enhance-
ment and flow around the clot. It need not be fully
resected but a strategy to prevent emboli must be
employed including caval interruption and/or antico-
agulation.

6. C.
Adrenalectomy is reserved for large upper pole renal
tumors although more recent data suggest that even
in this circumstance routine adrenalectomy may be
unnecessary. Adrenal involvement is a poor prog-
nostic sign and is now considered T4 disease

7. D.
Stauffer’s syndrome is thought to be cytokine medi-
ated (IL-6). Hand and foot syndrome and hyperlipi-
demia are side effects of targeted therapy. Hyperten-
sion associated with RCC is primarily renin medi-
ated. Hypercalcemia associated with RCC may be
due to bony metastases or a PTH paraneoplastic syn-
drome

8. D.
Sunitinib and other tyrosine kinase inhibitors have
altered the therapeutic landscape for mRCC. They
work by blocking the message of the receptor when
bound by ligand which decreases cell survival mech-
anisms. Overall response rates are 30-40%

9. D.
Systemic therapies for RCC are associated with a
large number of potential adverse events. Of those
above, TKIs are associated with LV dysfunction and
ejection fraction is often measured prior to initiating
therapy.

10. C.
Cisplatin is one of the most potent chemotherapeutic
agents for urothelial carcinoma. There are currently
no level 1 data for its use in upper tract UCC. Its
mechanism of action is the inhibition of DNA cross
linking

Chapter 15: renal parenChymal and Upper Urinary traCt Urothelial neoplasms 477
478 edUCational reVieW manUal in UroloGy
Chapter 16:
Prostate Cancer
Judd W. Moul, MD, FACS 1, Andrew J. Armstrong, MD, SCM 2, Joseph Lattanzi, MD

Reprinted by permission of UBM Medica, LLC.


Moul JW, Armstrong AJ, Lattanzi, J. Prostate Cancer.
In: Haller D, Wagman LD, Camphausen KA, Hoskins WJ, eds.
Cancer Management: A Multidisciplinary Approach.
14th Edition; 2012. UBM Medica. All rights reserved.

1
Division of Urologic Surgery, Contents
Duke University Medical Center
2
Divisions of Medical Oncology and Urology, 1. Epidemiology
Duke University Medical Center
3
2. Etiology and Risk Factors
Department of Radiation Oncology,
Southern Ocean County Hospital,
Meridian Health System
3. Signs and Symptoms

4. Screening and Diagnosis

5. Pathology

6. Prognosis And Natural History

7. Treatment Of Localized Prostate Cancer

8. Suggested Reading

9. Abbreviations In This Chapter

10. Tables

CHAPTER 16: PRosTATE CAnCER 479


1. Epidemiology

Prostate cancer is the most common non-skin can- Age


cer and the second leading cause of cancer mortality
in American men. Despite the fact that this cancer The risk of developing prostate cancer begins to
was diagnosed in an estimated 217,730 American increase at age 50 years in white men who have no
men in the year 2010 and led to the death of approxi- family history of the disease and at age 40 years in
mately 32,030 men, there is no universally agreed- black men and those who have a first-degree relative
upon strategic plan for its diagnosis and manage- (father, brother) with prostate cancer. Risk increases
ment. The estimated number of deaths increased with age, but, unlike other cancers, prostate cancer
from last year. However, the death rate per 100,000 has no "peak" age or modal distribution. There has
people declined 2.4% per year from 2000 to 2006, been a downward "age migration" in the prostate-
and the denominator (the older population) grew, so specific antigen (PSA) era such that the median age
the overall rate of death is lower. at diagnosis is now approximately 60 years.

Race

The highest incidence of prostate cancer in the


world is found in American black men, who have
approximately a 9.8% lifetime risk of developing
this cancer. This rate is slightly higher than the 8%
lifetime risk for American white men. Black men
have an incidence of prostate cancer that is 1.6 times
that of white men.

The Japanese and mainland Chinese populations


have the lowest rates of prostate cancer. Interest-
ingly, although Japanese immigrants to the United
States have a higher incidence of prostate cancer
than Japanese people living in Japan, their rate is
still about half that of American whites.

Socioeconomic status appears to be unrelated to the


risk of prostate cancer, and the explanation for racial
variability is unknown. However, an interplay of
diet, hormonal factors, and genetics likely accounts
for the variability.

Geography

The incidence of prostate cancer is highest in Scan-


dinavian countries (22 cases per 100,000 popula-
tion) and lowest in Asia (5 per 100,000). Risk may
be inversely related to ultraviolet light exposure, as
the incidence increases the farther one lives from
the equator. However, studies show extremely high
rates of prostate cancer in populations of African
heritage, such as Jamaicans.

480 EDUCATIonAL REVIEW MAnUAL In URoLoGY


2. Etiology and Risk Factors

Family history
Data are emerging about the potential role of a
Men who have a first-degree relative with prostate
novel retrovirus, termed xenotropic murine
cancer have approximately a twofold increased
leukemia virusrelated virus (XMRV). Initial studies
risk of developing prostate cancer during their life-
found that this virus expression in the prostate
time. An individual who has two first-degree rela-
tives with prostate cancer has a ninefold increase was linked only to patients with an uncommon
in lifetime risk. predisposing genetic variant of hereditary
prostate cancer due to deficiencies in interferon
True hereditary prostate cancer occurs in a small response (RNASEL polymorphism), but recent
number of men and tends to develop at an early age reports have identified viral DNA or protein in spo-
(< 55 years old). radic higher-grade tumors and have even linked
detection of this virus (controversially) to chronic
Dietary fat fatigue-like syndromes. Further validation of this
work is required before a viral etiology to aggres-
Although early studies suggested a link between sive prostate cancer is established.
dietary fat and prostate cancer risk, more recent Schlaberg R et al: Proc Natl Acad Sci U S A 106:16351-
studies have failed to confirm these observations. 16356, 2009.
Thus, the relationship between dietary fat and
prostate cancer risk remains unclear. Using animal
models, one study pointed to high levels of simple
carbohydrates being a culprit in promoting prostate In addition, findings suggest that cruciferous or
cancer growth. brassica family vegetables may reduce the risk of
advanced prostate cancer. This family includes
Studies indicate that progression of prostate cancer, broccoli, cauliflower, cole slaw, and sauerkraut.
which is likely to be more clinically relevant, has Interestingly, the intake of brussels sprouts, spinach,
different risk factors from those associated with its and mustard greens did not appear to be protective,
initiation/incidence and that some of these risk fac- and the consumption of fruit was not associated
tors are likely modifiable. Findings from the Health with the incidence or progression of prostate cancer.
Professionals Follow-up study have, however,
demonstrated different dietary risk factors for the Vasectomy
incidence compared with progression of prostate
cancer. For example, African-American race, a pos- Several large epidemiologic studies suggest that
itive family history, low consumption of tomato vasectomy may increase the relative risk of prostate
products, and high consumption of alpha-linolenic cancer by as much as 1.85. However, these same
acid have been associated with higher risks of inci- studies do not report an increased risk of dying from
dent prostate cancer. However, height, body mass prostate cancer associated with vasectomy but do
index, low physical activity, smoking, low con- indicate a statistically increased risk of dying from
sumption of tomato sauce, high calcium and alpha- lung cancer. These findings argue against an associ-
linolenic acid intake, African-American race, and ation between vasectomy and prostate cancer. Cur-
positive family history have all been associated with rently, this association is unproved and does not
more advanced cancer. constitute grounds for fundamental changes in the
use of vasectomy.

Sexual activity/sexually transmitted disease

A large prospective study of more than 29,000 men


demonstrated an association between high ejacula-
tory frequency (more than 21 ejaculations/month)

CHAPTER 16: PRosTATE CAnCER 481


and a decreased risk of prostate cancer, with a life- Prevention
time relative risk of 0.67. However, there may be
several confounding factors associated with high Active research on the chemoprevention of prostate
sexual activity, such as differences in prostate can- cancer is ongoing. Two prospective randomized tri-
cer screening or lifestyle. There was no associated als have demonstrated a 20% to 25% reduction in
increased risk for men in the lowest ejaculatory the risk of prostate cancer among men who were
frequency category. randomized to receive either finasteride or dutas-
teride (Avodart) daily vs those men on the placebo
Inflammation may underlie the findings associated arm. Finasteride or dutasteride chemopreventive
with a relatively higher risk of prostate cancer in agents have not been universally accepted, how-
men seen in sexually transmitted disease (STD) ever, because of concerns over the relative merits of
clinics, but it may also be related to screening bias. prevention of low-grade disease, with little effect on
Several cohort studies and one meta-analysis have high-grade tumors. In addition, concerns over side
demonstrated a protective role for the daily intake of effects such as impotence, as well as reductions in
aspirin and the risk of prostate cancer. In addition, PSA levels with these therapies that may make can-
the lipidlowering and anti-inflammatory statin com- cer detection more challenging, have limited the
pounds have been associated with a reduction in the generalized use of these drugs and thus an individu-
risk of high-grade tumors. These findings require alized risk/benefit discussion about use of these
prospective validation in randomized trials. agents as preventive measures is recommended.
Finally, randomized trials using selenium and vita-
min E have failed to demonstrate a benefit of these
Andriole and colleagues reported on along-term agents to reduce prostate cancer risk. Ongoing stud-
prostate cancer prevention study with dutasteride ies will examine vitamin D and omega-3 fatty acid
(Avodart, a dual 5-alpha-reductase inhibitor) in supplementation as preventive strategies in cancer,
men with an elevated PSA level and a negative ini- including prostate cancer.
tial biopsy. They demonstrated a 22.8% relative
risk reduction overall in prostate cancer incidence
(5.1% absolute risk), although the for-cause rate of
biopsies (ie, not protocol-specified) was not differ-
ent between the two arms. There was an apparent
greater reduction in this trial of low-grade (Glea-
son < 7) tumors, and no major effect in preventing
higher-grade tumors, with a higher percentage of
Gleason 8-10 tumors detected in the dutasteride
arm in years 3 and 4. Dutasteride is known to
reduce PSA levels by more than twofold, and its
use in prevention should take into account consid-
eration of risk and uncertainty around the bene-
fits, appropriate surveillance strategies while
being given a dihydrotestosterone inhibitor, and
side effects including erectile/sexual dysfunction
Andriole GL et al: N Engl J Med 362:1192-1202, 2010.

482 EDUCATIonAL REVIEW MAnUAL In URoLoGY


3. Signs and Symptoms 4. Screening and Diagnosis

Early-stage disease Prostate cancer screening with PSA levels and digi-
tal rectal examination (DRE) has resulted in not
Men with organ-confined prostate cancer often are only an increase in prostate cancer detection but
completely asymptomatic, given the predominant also a stage shift. More cancers are now being
posterior peripheral zone location of prostate adeno- detected at earlier stages, when they are potentially
carcinomas. Men with a large component of benign curable. Prior to screening efforts, most prostate
prostatic hyperplasia often present with bladder out- cancers were detected when they produced local
let obstruction unrelated to prostate cancer. symptoms or distant metastases, at which point
treatment for cure often was impossible.
Locally advanced disease
DRE
Bladder outlet obstruction is the most common sign
of locally advanced prostate cancer. A few men with Prostate biopsy prompted by abnormal findings on
locally advanced disease present with hematuria, DRE, such as nodularity or induration of the prostate,
urinary tract infections, and irritative voiding symp- leads to a diagnosis of prostate cancer in only 15% to
toms secondary to bladder outlet obstruction. 25% of cases. This rate compares with a prostate can-
cer prevalence of < 5% among men of similar age
Advanced disease without an abnormal DRE. Although neither accurate
nor sensitive for prostate cancer detection, abnormal
Rarely, men with bulky lymph node metastasis may DRE is associated with a fivefold increased risk of
present with bilateral lower extremity edema. Men cancer present at the time of screening.
with bony metastasis often present with bone pain
and, uncommonly, with lower-extremity weakness PSA
or paralysis from spinal cord compression.
PSA is a serine protease produced by the prostatic
epithelium and secreted in the seminal fluid in large
quantities. The level of PSA in serum is increased
by inflammation of the prostate, urinary retention,
prostatic infection, benign prostatic hyperplasia,
prostate cancer, and prostatic manipulation. The
optimal threshold to recommend prostatic biopsy
has come under increasing scrutiny. The overall
sensitivity for PSA levels is approximately 50% to
70% depending on the threshold used, but it is not as
specific and does not allow for differentiation
between indolent and aggressive disease.

Using a baseline PSA value to risk-stratify young


men for their future risk of prostate cancer is
increasingly recognized as clinically useful by both
the NCCN and AUA guidelines. Tang et al studied
over 9,500 men aged 50 or younger showing that a
baseline PSA of 1.5 to 2.4 ng/mL increased the rela-
tive risk for prostate cancer by 9.3 for white men
and sixfold to sevenfold for black men. Overall, a
PSA > 1.5 was a powerful predictor of subsequent
prostate cancer
Tang P et al: J Urol 183:946-950, 2010.

CHAPTER 16: PRosTATE CAnCER 483


Two large randomized multisite trials examined the have a total PSA level > 4.0 ng/mL, a percent-free
role of PSA screening in US and European popula- PSA less than 25% may suggest a 50% to 60% prob-
tions over time. In the PLCO trial, nearly 77,000 US ability of prostate cancer. In men who have had a
men were randomized to annual PSA and DRE prior negative biopsy but who have a persistently
screening or to standard practice (which included elevated PSA level > 4.0 ng/mL, a percent-free PSA
PSA screening). After 7 years of follow-up, prostate < 10% should prompt a repeat biopsy.
cancer was more commonly detected in screened
men, but fatal disease was not detectably different. Current screening recommendations
In contrast, in the European study, named ERSPC,
182,000 men in various countries were assigned to There remains significant controversy as to the wis-
PSA screening at various intervals or to no screen- dom and effectiveness of PSA screening for the gen-
ing. Contamination seemed to be less common in eral male population. This is exemplified in recent
the control group in this study, and at 9 years' updates to both the AUA and ACS guidelines for
median follow-up, a 20% reduction in prostate can- 2010. The AUA took a more "proactive" stance by
cer mortality was noted. In this study, nearly twice recommending that all men consider having a base-
as many screened men were diagnosed with prostate line PSA test at age 40. This guideline is useful to
cancer. The number needed to screen in this study to risk-stratify men to future testing frequency. If at
save one prostate cancer death was 1,410, and the age 40 the PSA level is less than 1.0 ng/mL, the man
number needed to treat with local therapy was 48. can be reassured that he is at low risk and will be
Data from the PCPT illustrate that there is no "nor- asked to return at age 45 for consideration of addi-
mal" level for PSA. Indeed, there is a continuum of tional screening tests.
risk that exists, even at low levels of PSA. For
example, among men in the placebo arm of the On the other hand, the ACS took a more cautious
study, 11% of men with a serum PSA level < 1 stance with its 2010 update. The ACS is now de-
ng/mL had prostate cancer at the end-of-study emphasizing mass screening programs, while
biopsy. The proportion of men with prostate cancer emphasizing a discussion of the pros and cons of
rose to 30% among those with PSA levels between testing with PSA levels based on individualized
3.1 and 4.0 ng/mL, which is still within the "nor- risk. The NCCN has also adapted an individualized
mal" range. Furthermore, with a median follow-up risk assessment and informed discussion with
of 9 years, the cancer-specific survival rate was patients about the pros and cons of screening, to
improved by 20% in the screening group vs con- begin in the 40s for high-risk men, and at 50 for men
trols. However, they report an overdetection of of average risk. Readers should refer to acs.org or
small, nonlethal cancers. nccn.org for updates of these guidelines.

An additional potentially more worthwhile Using data from the Shared Equal Access Regional
approach for PSA screening may be to use the rate Cancer Hospital, the Duke Prostate Center, and
of rise in PSA (PSA velocity) in combination with Johns Hopkins Hospital, investigators concluded
the absolute PSA value. This approach has been that higher body mass index was significantly
shown to be useful in the form of age-adjusted PSA associated with higher plasma volume and lower
velocity, but accepted guidelines are still controver- PSA concentrations in men undergoing radical
sial, and the independent predictive utility of this prostatectomy. Hemodilution may therefore be
measure remains to be demonstrated. responsible for the lower serum PSA concentra-
tions among obese men with prostate cancer.
Another commonly employed test for patients with While obese men may be less likely to be diag-
a PSA level < 10 ng/mL is the percent-free PSA nosed with incident prostate cancer, they are, how-
level. There is an inverse relationship between the ever, more likely to have aggressive disease at pre-
percent-free PSA level and the risk of a cancer diag- sentation, and more likely to suffer relapse and
nosis. Most urologists utilize a cutoff of 10% to prostate cancer-specific mortality.
prompt a recommendation for a repeat biopsy. In
men who have never had a prostate biopsy but who

484 EDUCATIonAL REVIEW MAnUAL In URoLoGY


Biopsy sary. Currently, this test is being further optimized
and validated in a large series of patients. Additional
When indicated, prostate biopsy is usually per- studies are investigating the role of a novel fusion
formed as an office procedure by transrectal ultra- protein (TMPRSS2-ETS) commonly found in
sonographic guidance using an automated 18-gauge prostate tumors, both as a diagnostic and prognostic
biopsy gun. The procedure is performed with, at marker. This fusion gene is androgen regulated,
most, local anesthesia and carries a risk of signifi- directs oncogenic signals, and has been found in
cant infection of only 1 in 200 cases. Additional side over 50% of localized prostate cancers. Other mark-
effects of hematuria and hematochezia are common ers will need to be employed to detect the 50% of
for 2 to 3 days following the biopsy. Hematospermia tumors that lack this fusion protein before this test is
may last for up to 4 to 6 weeks. Since about the year likely to be useful as a screening tool.
2000, prostate biopsy includes laterally directed
extended core protocols employing 8 to 20 biopsy
cores per procedure. Multiple studies have demon-
strated that the addition of the lateral cores improves
the accuracy of biopsy.

If the biopsy result is negative, these men are typi-


cally followed conservatively with serial PSA levels
and DRE repeated annually. Repeat biopsy is per-
formed only when PSA levels rise at abnormal rates
(> 0.8ng/mL/year) or if DRE findings show new
nodularity or induration. Men in whom high-grade
prostatic intraepithelial neoplasia or atypical small
acinar proliferation (ASAP) found on biopsy usu-
ally should undergo repeat biopsy, since one-third to
one-half will be found to have prostate cancer.
Recently, the recommendation of repeat biopsy for
PIN alone has been relaxed, such that repeat biopsy
may not be recommended and a more personalized
approach to follow-up is taken.

A new urine test called PCA-3, has become more


widely available. This test, performed on voided
urine after an "attentive" DRE, is based on reverse
transcriptase-polymerase chain reaction assay for a
prostatespecific gene (DD3). It is becoming useful
not as a primary screening test, but to dictate the
need for repeat prostate biopsy in men with persis-
tently elevated PSA levels.

In addition, researchers at Johns Hopkins Univer-


sity have developed a blood-based test called
EPCA-2, which detects a nuclear matrix protein
linked to prostate cancer. Elevated levels of EPCA-
2 were found to be more sensitive and specific for
prostate cancer, even in men with normal PSA lev-
els and benign prostatic hypertrophy, and predicted
extracapsular disease as well. Further validation of
this test in larger cohorts of screened men is neces-

CHAPTER 16: PRosTATE CAnCER 485


5. Pathology 6. Prognosis And Natural History

Staging systems
Adenocarcinomas make up the vast majority of
prostate carcinomas. A total of 70% of prostate ade- The most widely used and universally accepted
nocarcinomas occur in the peripheral zone, 20% in staging system for prostate cancer is the TNM sys-
the transitional zone, and approximately 10% in the tem (Table 1). In the TNM system, T1 and T2
central zone. tumors are confined to the gland, whereas T3 and
T4 tumors have local extension.
Other tumor types
In 2010, the AJCC updated prostate cancer staging
Other tumor types are relatively rare and include recommendations in its 7th edition of the AJCC
ductal adenocarcinoma, which occurs in the major Cancer Staging Manual. These guidelines incorpo-
ducts and often projects into the urethra; and muci- rate a more risk-based approach that utilizes the
nous adenocarcinoma, which secretes abundant Gleason grading system and current PSA value in
mucin and does not arise from the major ducts. the staging system, which brings this system more
Transitional carcinoma of the prostate occurs in line with the risk-adapted approaches described
within the ducts and, to a lesser extent, in the pro- in this chapter. Additionally, microscopic bladder
static acini. Typically, primary transitional carcino- neck invasion as a form of extracapsular extension
mas are aggressive cancers that have a poor prog- was incorporated into T3 disease rather than as T4,
nosis. Similarly, neuroendocrine (small-cell) given the more favorable outcomes of this subgroup
tumors are rare and aggressive, have a poor progno- of men. This scoring system is shown in Table 2.
sis, and typically require aggressive management. Use of either this staging system or the
Other rare types include foamy carcinoma, muci- NCCN/D'Amico approach, or a nomogram-based
nous adenocarcinoma, large-cell neuroendocrine risk assessment will provide a more accurate prog-
tumors, and signet ring tumors. nostic classification system for prostate cancer at
initial diagnosis.
Histologic grade
Risk-adapted staging
The grading system developed by Gleason from
data accumulated by the Veterans Administration The development of the "Partin Tables" in 1993 ush-
Cooperative Urologic Research Group appears to ered in a new era of combining clinical stage, Glea-
provide the best prognostic information in addition son score, and PSA level to predict pathologic stage
to clinical stage and is the predominant grading sys- after radical prostatectomy. More recently, this has
tem in widespread use. led to the D'Amico et al risk groupings for newly
diagnosed men with clinically localized disease
Metastatic spread (Table 3). Patients are divided into three risk groups
(low, intermediate, or high) of occult micrometas-
Adenocarcinoma of the prostate may spread locally tases and relapse after initial local therapy. Although
through direct extension into periprostatic fat or via not perfect, this system is currently in widespread
the ejaculatory ducts into seminal vesicles; lym- use and allows a framework for multimodal and
phatically to regional lymph nodes, including the multidisciplinary treatment strategies based on risk
hypogastric and obturator lymph nodes; and grouping. Kattan et al have developed preoperative
hematogenously to bone. The most common sites of and postoperative nomograms as clinical tools to
bony metastases are predict the risk of recurrence after radical prostatec-
tomy. Although these nomograms are imperfect,
the lumbosacral spine (probably related to venous they may be useful for estimating risk and planning
drainage of the prostate through Batson's plexus) therapy as well as for stratifying and selecting
and the axial skeleton, but any bone, including the patients for clinical trials. In addition, Stephenson et
skull and ribs, can be involved. Rare sites of al have developed a fairly robust postsurgical nomo-
metastatic spread include the liver and lungs. gram of over 12,000 men that is able to predict with
> 80% accuracy 15-year prostate cancer-specific
mortality. In this model, Gleason sum, PSA level,

486 EDUCATIonAL REVIEW MAnUAL In URoLoGY


and clinical stage were the most important factors in D'Amico et al combined a number of national
predicting long-term outcome, whereas body mass datasets to report 10-year cancer-specific mortality
index and PSA velocity were not able to add to the rates for men undergoing radical prostatectomy or
predictive accuracy of the model. external-beam radiotherapy (EBRT) by this risk
grouping and age at diagnosis. These 10-year mor-
Prognosis tality graphs are useful to counsel contemporary-era
men contemplating surgery or radiation therapy.
The optimal management of patients with prostate Given recent advances in the treatment of metastatic
cancer varies widely and is highly dependent upon a disease, identifying men at high risk for metastatic
patient's age, overall health, and tumor risk assess- disease following local therapy is important, as
ment. The natural history of the disease process can these agents are incorporated earlier in the disease
be heterogeneous, ranging from an incidental find- course.
ing unlikely to result in cancer-specific mortality to
very aggressive, resulting in early widespread
metastatic disease and death. Therefore, treating Some studies have incorporated baseline func-
physicians should carefully consider the value of tion into the quality-of-life assessment and out-
curative therapy with potential toxicity in the con- comes associated w ith radical surgery or radia-
text of a patient's comorbidities and life expectancy. tion therapy. This allow s for a more precise and
detailed individual assessment of the long-term
urinary, sexual, and gastrointestinal side effects
According to the 2010 Guidelines, the NCCN
of local therapies
defines as a "very low" risk group men with
Chen RC et al: J Clin Oncol 27: 3916-3922, 2009.
prostate cancer who have low volume using the
Epstein criteria (T1c stage, Gleason < 7, PSA level <
10 ng/mL, fewer than three positive biopsy cores, <
50% cancer in each core, and PSA density < 0.15
ng/mL/g). These patients have a very low risk of
prostate cancer death within 10 to 20 years and
could be considered good candidates for active
surveillance. These criteria are imperfect (eg, as dis-
cussed in a 2008 nomogram analysis reported in
Cancer by FK Chun and colleagues), and current
efforts to improve upon them using nomograms
may better help to select men who can safely defer
initial aggressive therapy.

Among patients with clinically localized prostate can-


cer treated conservatively (observation or hormonal
therapy alone), those with a low Gleason score (2-4)
have a small risk of dying of their cancer within 15
years (4% to 7%). However, those with poorly differ-
entiated tumors (Gleason score 8 to 10) have a greater
risk of dying of prostate cancer than of any other
cause, even when the cancer is diagnosed in the
eighth decade of life. Indeed, a man diagnosed before
the age of 60 with a clinically localized, Gleason score
8 to 10 prostate cancer has an 87% risk of dying of the
disease within 15 years if untreated (Table 4).

CHAPTER 16: PRosTATE CAnCER 487


7. Treatment Of Localized Prostate
Cancer

There are several treatment options for localized Approximately 25% to 30% of men will progress
prostate cancer, including radical prostatectomy, during this period and require definitive therapy,
EBRT, brachytherapy (interstitial radiation/seeds), and this time, it is not known whether deferred
and cryotherapy. Multiple treatment series with active therapy results in inferior outcomes for these
each modality have documented the validity of the men compared with immediate therapy.
risk-stratification model based on clinical palpation
stage, Gleason score, and serum PSA level. More Radical prostatectomy
recently, it has been suggested that biopsy quantifi- Radical prostatectomy can be performed retropubi-
cation may also be an important factor. Specifically, cally through a lower midline incision—an
counting the number of involved needle biopsy approach that may include pelvic lymph node dis-
cores or the percentage of each core involved by section. Robotic-assisted laparoscopic prostatec-
cancer may be prognostic. Low-risk patients experi- tomy (RALP) is now the most popular form of radi-
ence a favorable 85% to 90% freedom from recur- cal prostatectomy in the United States. Radical per-
rence, compared with approximately 75% and 35% ineal prostatectomy is uncommon but remains a
to 50% for the intermediate- and high-risk patients, viable option.
respectively. Although no randomized studies have
been performed, contemporary series, which strat- Although the morbidity of radical prostatectomy
ify patients by the risk model, demonstrate remark- was a major concern in the past, improvements were
ably similar outcomes independent of the treatment made during the 1980s. Among the various treat-
modality. For this reason, treatment recommenda- ment options for prostate cancer, only radical prosta-
tions should be individualized based on patient pref- tectomy has been demonstrated to confer a survival
erence, life expectancy, and discussion of potential advantage over no treatment. After a median of 8.2
toxicities. years of follow-up, Axelson et al found a 35% reduc-
tion in the risk of death and of metastases among
Treatment of clinically localized disease men randomized to undergo radical prostatectomy
(T1, T2) compared with those randomized to undergo watch-
ful waiting. The benefits of surgery appear to plateau
Active Surveillance at 10 years. The hazards of anesthesia, risk of blood
Active surveillance in very low- to low-risk patients loss, and hospital stay have all been minimized.
who have a limited life expectancy (< 10 to 15 Nationwide, Medicare data suggest that surgical out-
years) also is emerging as an important initial treat- comes are significantly better at those centers per-
ment modality. Ongoing studies (CALGB/NCIC forming > 40 prostatectomies per year than at other
START study) are examining the necessity of hospitals with a lower surgical volume.
immediate vs deferred active definitive therapy.
However, at this time, the majority of men in the Transfusion is usually unnecessary, and treatment-
United States receive initial radical surgery or radia- related mortality is < 0.05% at leading prostate can-
tion as treatment of localized prostate cancer. cer centers. The average hospital stay of a man
undergoing radical prostatectomy is now approxi-
This is fast becoming an initial treatment selection mately 1 to 2 days at leading referral centers in the
of low and very low-risk men (Gleason score of 6 or United States; several institutions routinely dis-
less, low volume, PSA level < 10, ng/mL T1c dis- charge patients within 24 hours. Although urinary
ease). Most studies have suggested a less than 5% to incontinence is common in the first few months
10% prostate cancer-specific mortality rate for men after prostatectomy, most men recover urinary con-
in this category who choose initial deferred therapy, trol; at some leading centers, 90% to 98% of men
particularly those men with slow PSA doubling report few or no long-term urinary problems.
times (> 3 years). In these men, nonprostate cancer-
specific mortality far outweighs prostate cancer- Nerve-sparing radical prostatectomy is appropriate
specific mortality, illustrating the need to assess for men with small-volume disease. It offers those
age, comorbidities, and life expectancy in the initial men with good potency prior to surgery the proba-
treatment decisions of a man with prostate cancer. bility of recovering that function following the

488 EDUCATIonAL REVIEW MAnUAL In URoLoGY


operation. By permitting better visualization of San- learning curve (with attendant perioperative mor-
torini's dorsal venous plexus, the apical prostate, the bidity) prior to meeting the outcome standards set
urethra, and the striated urethral sphincter, the by the open technique.
nerve-sparing technique also reduces blood loss and
improves recovery of urinary continence. In appro- Robot-assisted laparoscopic prostatectomy (RALP)
priately selected individuals, a nerve-sparing proce- was developed to overcome some of the difficulties
dure confers no greater risk of prostate cancer recur- of the standard laparoscopic prostatectomy (eg,
rence after considering other relevant clinical infor- intracorporeal suturing). The robotic technique
mation (PSA level, Gleason score, margin status, allows for three-dimensional (3D) visualization of
seminal vesicle involvement, and the presence of the operative field and provides for a significantly
extraprostatic spread). wider range of movements intracorporeally than do
standard laparoscopic instruments. This advance
Referral centers have reported that 50% to 90% of has prompted the assimilation of the technique into
patients who are fully potent prior to surgery the armamentarium of many urologists.
recover erections following a nerve-sparing proce-
dure, but the quality (rigidity and duration) of these Current evidence suggests that in experienced
recovered erections may be compromised compared hands, the laparoscopic and robotic techniques have
with preoperative erections. Erection recovery rates similar oncologic efficacy to that of the open proce-
can be higher than 80% in patients < 60 years of age dure. However, the length of follow-up (usually <
and lower in older men. Potency may return any- 24 months) in these studies is limited, suggesting
where from 2 to 24 months following surgery. that a measure of caution be taken when interpreting
Regardless of potency, sensation of the penis is not the results. Furthermore, recent studies suggest the
changed after this procedure, and men still experi- learning curve is prolonged, with 200 to 250 cases
ence orgasm. Nerve-sparing radical prostatectomy necessary before results can be compared with those
has not compromised cancer control outcomes in of experienced surgeons. Long-term effects of these
well-selected men with early-stage disease. Also, modalities on sexual and urinary health (as mea-
recent studies have suggested that early postopera- sured by a psychometrically valid survey) have not
tive use of sildenafil (Viagra), tadalafil (Cialis), vac- been reported, and such data are critical in the con-
uum entrapment device (VED), and/or intracaver- text of the prostatectomy patient when evaluating
nosal injection (ICI) of vasoactive medications, technical results. Although vision with robotic
such as papaverine, may facilitate the return of natu- prostatectomy is excellent, the current-generation
ral erections more quickly. Generally speaking, robotic device does not allow tactile sensation for
recovery of erectile function after radical prostatec- the operating surgeon. Furthermore, the vast major-
tomy is mediated by age (the younger, the better), ity of robotic prostatectomies are performed via a
pretreatment erectile function (the stronger, the bet- transabdominal approach, whereas the open retrop-
ter), and a nerve-sparing approach (bilateral is better ubic approach avoids the peritoneal cavity.
than unilateral, which is better than none [which is
better than wide dissection]).
Robot-assisted laparoscopic prostatectomy (RALP)
Robotic radical prostatectomy has become very popular in the United States. A
Laparoscopic prostatectomy was initially described recent systematic review by Murphy et al showed
by Schuessler in 1997 but was abandoned because the learning curve to be steep and few published
of its technical difficulty and long operative time reports have reported outcomes in a standardized
with little apparent benefit over the conventional
manner. The cost of the technology is substantial
technique. A resurgence in the technique was
and little is known of the cost-effectiveness com-
prompted by improved instrumentation and refine-
pared with the open technique
ments in the procedure itself, although the laparo-
Murphy DG et al: Eur Urol 57:735-745, 2010.
scopically naive urologist must endure a substantial

CHAPTER 16: PRosTATE CAnCER 489


Pelvic lymph node dissection Studies now indicate Radiation therapy Men with positive margins or
that regional pelvic lymph node dissection may not pathologic T3 disease following radical prostatec-
be necessary for patients with stage T1c disease if tomy are potential candidates for early adjuvant
the total Gleason score is < 7 and the PSA level is < EBRT. Some controversy exists as to the efficacy of
10.0 ng/mL, ie, low-risk individuals. Selected early postoperative therapy vs intervention once a
intermediate-risk men may also not require this biochemical failure has been documented in
staging procedure, but in high-risk men, it is still patients who achieve an undetectable PSA level fol-
considered imperative. lowing surgery.

Neoadjuvant hormonal therapy Three randomized trials have been completed


Approximately 15% to 35% of men who undergo demonstrating a benefit to early adjuvant radiation
radical prostatectomy for clinical stage T2 prostate therapy for men with positive margins, seminal
cancer will be found to have pathologic T3 disease vesicle invasion, or extracapsular extension. Typi-
following surgery. This finding led some investiga- cally, this treatment is offered after continence is
tors to evaluate the efficacy of neoadjuvant andro- restored to allow healing to take place after surgery.
gen deprivation therapy in prospective clinical tri- With a median follow-up of 12 years, the most
als. Early data from these trials suggested that mature study (SWOG 8794) by Thompson et al con-
neoadjuvant hormonal therapy led to a reduction in firmed a significant improvement in the risk of
positive surgical margins. However, these findings metastasis (43% vs 54%) and overall survival (41%
need to be considered in a technical Indeed, more vs 52%) to adjuvant radiation. The largest trial, by
recent data from prospective studies have shown no Bolla et al (EORTC 22911), included 1,000 patients
benefit of neoadjuvant therapy with regard to pro- with T3 disease randomly assigned to 60 Gy vs
gression-free survival. At present, therefore, it observation. At 5 years, progression-free survival
appears that neoadjuvant hormonal therapy does not was significantly improved (74% vs 53%), with no
improve the curative potential of radical prostatec- demonstration of an overall survival benefit. A sub-
tomy but instead is associated with morphologic sequent update suggested the benefit might be lim-
alterations that complicate the prognostic utility of ited to patients with positive surgical margins. A
standard pathology. Neoadjuvant hormonal therapy German trial of nearly 400 patients also demon-
is sometimes used for technical downsizing to facil- strated a biochemical progression-free survival ben-
itate surgical resectability. However, it is not rou- efit (72% vs 54%) to early radiation therapy in this
tinely recommended to improve cancer control. pT3 group. The benefit of therapy was observed
with or without positive margins.

A multi-institutional study of 431 men with pT3 An alternate approach to early adjuvant radiation
prostate cancer were randomized to receive adju- therapy is salvage radiation therapy for PSA recur-
vant radiation therapy or observation after prosta- rence. It remains unclear whether early salvage radi-
tectomy. Patients who were given adjuvant radia- ation based on PSA thresholds is inferior to the adju-
tion had a 28% and 29% lower risk of death and
vant approach but has the benefit of not treating all
men with T3 disease with radiation. Of note, in the
metastases, respectively
SWOG trial approximately one-third of patients ini-
Thompson IM et al: J Urol 181:956-962, 2009.
tially assigned to observation ultimately received
salvage radiation largely for PSA recurrence. In a
retrospective analysis, there was a 5-year bNED
Adjuvant therapy post prostatectomy
(biochemical no evidence of disease, or unde-
The potential indications for adjuvant therapy fol-
tectable PSA levels) advantage (77% vs 38%) to
lowing radical prostatectomy in patients with clini-
early vs salvage therapy. Based on this emerging
cal T1 or T2 malignancy include pathologic evi-
evidence and a relatively low toxicity to radiation,
dence of T3 disease, positive nodes, a rising PSA
the use of early adjuvant radiation in this high-risk
level, and positive surgical margins, among others.
group should be considered.

490 EDUCATIonAL REVIEW MAnUAL In URoLoGY


The use of salvage radiation after a PSA recurrence Recent findings have suggested that tumor grade,
in the other patient groups should be based on the time to PSA recurrence after surgery, and PSA dou-
risk of having an isolated local/regional failure. bling time predict the 5-, 10-, and 15-year risks of
Approximately 60% to 70% of patients with favor- prostate cancer mortality and can help to guide the
able disease after surgical failure (PSA level < 2.0 timing and need for androgen ablation. Although
ng/mL, a slow PSA doubling time, and a long inter- these findings do not prove that early androgen
val to failure after surgery) will experience durable ablation is more beneficial than delayed androgen
disease-free survival after salvage radiotherapy, ablation based on a PSA threshold or development
presumably due to a smaller tumor burden and a of metastatic disease, it does help to risk-stratify
lower likelihood of occult metastatic disease. patients into those most likely to derive benefit from
Stephenson et al evaluated a large number of androgen ablation early. Moreover, men with a PSA
patients with salvage EBRT and persistent or doubling time of less than 15 months may be more
increasing PSA levels after surgery from five Amer- likely to die of prostate cancer than of other compet-
ican academic institutions. Forty-five percent of ing causes, suggesting that these men should be
patients were free of disease at 4 years after salvage evaluated in controlled trials of hormonal or novel
EBRT. Patients with no adverse risk features therapeutic agents.
achieved a 4-year progression-free probability of
77%. The authors subsequently developed a nomo- Definitive radiation therapy
gram based on established risk factors to more accu- EBRT utilizes high-energy photons to destroy can-
rately identify patient-specific risks to assist in clin- cer cells by damaging cellular DNA. Traditional
ical decision-making. Patients who experience PSA EBRT utilized bony landmarks and standard-beam
failure after radical prostatectomy generally should arrangements to deliver dose to the pelvic region.
be restaged with pelvic CT, bone scan, and DRE. Technologic advances in treatment planning, driven
Patients with no evidence of metastatic disease by improved computing power and the incorpora-
should be evaluated for radiotherapy. tion of individualized patient anatomy, have led to
dramatic improvements in treatment delivery.
Hormonal therapy Significant controversy exists
within the academic community as to the timing of 3D conformal EBRT creates 3D representations of
initiating androgen deprivation following radical target structures (ie, the prostate) and designs highly
prostatectomy. Clinical trials have documented a tailored treatment portals utilizing various angles to
survival benefit only for those patients with nodal create a volume of high radiation dose that con-
involvement. forms to the target shape. The anatomic information
used to define the target is generally derived from
Treatment recommendations for postprostatec- CT images obtained while the patient is placed in an
tomy recurrence immobilization device in the precise treatment posi-
Following radical prostatectomy, it is expected that tion. With the selective delivery of dose to the target
serial PSA levels will become undetectable. Any and avoidance of the surrounding normal tissue, the
detectable PSA level (> 0.2 ng/mL) following therapeutic ratio is improved. This approach has
surgery indicates possible recurrent disease and the permitted the use of doses far higher than tolerable
need for restaging and possible salvage therapies, with traditional therapy, with fewer bowel and blad-
including radiation or hormonal therapy, experi- der complications.
mental protocols, or observation. However, some
patients can develop low levels of detectable PSA Treatment volumes in patients with low-risk disease
after prostatectomy without cancer recurrence, pre- are designed to encompass the prostate plus a mar-
sumably due to small foci of benign prostate tissue gin for daily variations. Patients with a high risk for
in situ. Although there is concern for recurrence periprostatic extension and/or regional lymph node
when the PSA level is > 0.2 ng/mL, most clinicians metastasis have historically received initial pelvic
will wait until a PSA threshold > 0.4 ng/mL is treatment of 45 to 50 Gy, followed by a coned-down
reached to assume that the rise in PSA level repre- boost to the prostate.
sents meaningful recurrence.

CHAPTER 16: PRosTATE CAnCER 491


RTOG 9413 was designed to test the addition of well tolerated, with only 3% of patients experiencing
whole pelvic radiation and the timing of androgen significant (grade 3+) acute gastrointestinal/geni-
deprivation in the treatment of high-risk patients tourinary (GI/GU) morbidity and a 6% rate of signif-
(lymph node-positive potential > 15% or locally icant late toxicity. A comparison trial is being con-
advanced [Gleason score ≥ 6 and > stage cT2c dis- ducted by the RTOG (P0126); it will accrue 1,520
ease] cancers). At a median follow-up of 59.5 cases and provide information regarding any benefi-
months, Roach et al noted an improved 4-year pro- cial effect on mortality with higher radiation doses.
gression-free survival (60%) among those receiving Although no standard ≥ 75 Gy with 3D conformal
whole pelvic radiotherapy in conjunction with EBRT techniques appear to be well tolerated and
neoadjuvant androgen deprivation, compared with improve biochemical response rates.
other treatment arms (44% to 50%). In a subsequent
update, the value of whole pelvic fields was limited Intensity-modulated radiation therapy (IMRT)
to only patients receiving hormonal therapy, and the is becoming a widely used treatment for prostate
benefit became nonsignificant with a trend toward cancer. This refinement of conformal therapy
improved outcomes. Furthermore, a European ran- employs high nonuniform beam intensity profiles
domized trial (GETUG-01) did not show a benefit and dynamic multileaf collimation to create even
to larger pelvic fields. more conformal dose distributions. Further
improving the therapeutic index compared with
Therefore, at this time, there is no consensus as to the 3D conformal EBRT, IMRT is associated with
use of whole pelvic or more limited prostate-only reduced toxicity, permitting further dose escala-
fields in the intermediate-/high-risk patient groups. tions previously unattainable.

EBRT dose The previous standard radiation dose IMRT was pioneered in several major centers, and
with conventional therapy was 70 Gy given over 7 Memorial Sloan-Kettering Cancer Center has
weeks; however, more recent work has suggested a reported a series of 772 patients treated with doses
positive dose response, particularly in the intermedi- between 81 and 86.4 Gy. With a median follow-up
ateand high-risk patient populations. Multiple sin- of 24 months, the side-effect profile was improved,
gle-institution experiences have demonstrated that despite these higher doses, with less than 1% of
3D conformal EBRT techniques with doses of 75 Gy patients experiencing late grade 3+ GI/GU toxicity.
and higher can be delivered with minimal toxicities. The early PSA relapse-free survival rates for favor-
able-intermediate-and unfavorable-risk groups
A randomized trial from the MD Anderson Cancer were 92%, 86%, and 81%, respectively. Although
Center compared 70 Gy given conventionally with IMRT is quickly becoming the standard of care at
78 Gy delivered with a conformal boost. With a most institutions, some caution should be exercised.
median follow-up of 8.7 years, it showed an The precision of dose delivery and the complexity
advantage in 10 freedom from failure for the of treatment planning demand a strong commitment
higher-dose arm in patients with a PSA level > 10 by both physicians and physics personnel to ensure
ng/mL (78% vs 39%). high-quality IMRT.

Kupelian et al presented pooled data for nearly Proton therapy Technically a form of EBRT, pro-
5,000 patients from 9 institutions over a narrow ton therapy has been utilized in clinical practice for
time range (1994-1995) to remove treatment tech- more than 10 years. It offers a potential advantage
nique, stage migration, and lead-time bias. They over photon-based IMRT by exploiting superior
demonstrated favorable biochemical control out- dose distributions of the Bragg peak effect. The
comes for doses higher than 72 Gy in all risk groups. routine implementation of this technology has been
hampered by the staggering costs of building and
The RTOG has completed a dose-escalation trial to maintaining a facility. The largest experience
assess toxicity with 3D conformal EBRT. In this involving 1,277 patients at the Loma Linda proton
multiinstitutional trial, 78 Gy (prescribed as a mini- facility was reported in 2004 and demonstrated
mum to the tumor volume in 2-Gy fractions) was "comparable control rates with minimal toxicity"

492 EDUCATIonAL REVIEW MAnUAL In URoLoGY


compared with other local therapies. Although The survival benefits of androgen suppression ther-
there are theoretical benefits and ongoing trials apy (AST) for patients with intermediate-risk dis-
evaluating the possibility of further safe dose esca- ease have been uncertain. A single-institution
lation with protons, to date there is little clinical prospective trial by D'Amico et al randomized
evidence to support a significant benefit over patients with a PSA level > 10 ng/mL, a Gleason
IMRT. With several new centers now under con- score ≥ 7, or radiographic evidence of extraprostatic
struction or online, and a published economic eval- disease to receive EBRT (70 Gy) alone or the same
uation questioning its cost-effectiveness, the future EBRT with 6 months of AST. After a median fol-
of eventual widespread application of proton ther- low-up of 4.5 years, patients treated with combined
apy remains unclear. EBRT and AST were found to have improved pro-
gression-free, prostate cancerspecific, and overall
Stereotactic body radiotherapy (SBRT) is being survival (P = .04). Although hormone therapy for 3
investigated as a method to treat early-stage prostate years has been shown to be beneficial in locally
cancers. Utilizing several highly focused fractions, advanced cases, this trial in men with more local-
this method exploits the alpha/beta ratio typical of ized disease showed a benefit to a shorter duration
slowly growing malignancies. The technique, of hormone therapy. However, it is not known
which employs high-dose (approximately 700 cGy) whether high-dose radiotherapy will obviate the
for several fractions (typically 5), can be delivered need for AST in this group of patients. The survival
by several specialized methods, including linear benefit in this study was largely confined to those
accelerator-based models, tomotherapy, and men with few cardiovascular comorbidities, illus-
CyberKnife. A pooled cohort of 41 patients with a trating the importance of patient selection for AST.
median follow-up of 5 years demonstrated both the
efficacy and safety of this approach utilizing the Recent secondary findings from a large randomized
CyberKnife. This methodology represents a major study (RTOG 9202) suggest that men with high-
paradigmatic shift over the typical 8-week course of grade tumors (Gleason score 8-10) and high-risk
IMRT. This approach is gaining favor with patients (T2c-T4) localized disease benefit from long-term
intrigued by the convenience of a short course of androgen ablation (2 years) compared with short-
therapy. It is important to recognize that while term androgen ablation (4 months), based on
results from several ongoing stduies appear promis- improved prostate cancer-specific and overall sur-
ing, further data maturation in a large patient popu- vival. A caveat to these findings has been the
lation will likely be necessary to promote this as a increased incidence or acceleration of incident car-
standard technique. diovascular death in men older than 65 years of age
starting androgen ablation compared with those
Androgen ablation with EBRT Two potential ben- men who did not receive androgen ablation. These
efits of the use of transient androgen ablation prior findings suggest that cardiac evaluation should be
to EBRT have been identified. First, there may be considered in those men older than age 65 with car-
some synergy between the apoptotic response diovascular risk factors prior to undergoing andro-
induced by androgen deprivation and radiotherapy gen ablation. RTOG 94-08 was presented at
that may increase local tumor control. Second, ASTRO in April 2010. This study examined a short
androgen deprivation results in an average 20% course (4 months) of androgen deprivation (flu-
decrease in prostate volume. This volume reduction tamide plus either goserelin [Zoladex]or leuprolide
not only may reduce the number of target cells, and [Lupron]) with external radiotherapy (66.6 or 68.4
thereby improve tumor control, but also may shrink Gy) in 1,979 men. For intermediate-risk men, the 8-
the prostate and, thus, diminish the volume of rec- year survival rate was 66% in men who received
tum and bladder irradiated during conformal ther- radiation therapy alone and 72% in men who also
apy. Complete androgen blockade can be achieved received hormones. There was no benefit observed
with luteinizing hormone-releasing hormone to hormonal therapy in low-risk men.
(LHRH) agonists plus an oral antiandrogen or
LHRH antagonist therapy.

CHAPTER 16: PRosTATE CAnCER 493


Interstitial radiotherapy In the 1970s, the use of In addition to disease risk factors, certain patient
permanently placed radioactive iodine implants selection factors are important in considering
produced initial results as good as those obtained implants. A large prostate size (> 60 cc) may make
with other available radiotherapy techniques and the procedure more challenging, both from the per-
posed a small risk of impotence and other morbid- spective of increased prostate gland swelling due to
ity when compared with conventional EBRT and the increased number of needles and the difficulty
radical prostatectomy. of the pubic bone obstructing needle placement.
Patients with outlet obstruction symptoms IPSS
However, ultimate control rates were unaccept- (International Prostate Symptom Score) > 15 have
able. The technique used (freehand placement of an increased risk of requiring catheterization fol-
seeds during laparotomy) was found to distribute lowing implantation. Patients who have undergone
the radioactive seeds unevenly throughout the prior transurethral resection of prostate (TURP)
gland; cold regions may have contributed to the have been reported to have an increased risk of
relatively poor outcome. incontinence; recognizing this risk and placing
seeds farther from the defect may help to minimize
The advent of improved imaging and seed placement this risk. Therefore, with proper counseling,
techniques coupled with better patient selection has patients with small TURP defects may still be con-
resulted in vast improvements in cancer control. The sidered implant candidates.
perception of fewer side effects in a single outpatient
treatment has also contributed to some popularity of For patients with intermediate- or high-risk disease,
this treatment modality. Transrectal ultrasonography implants may be combined with EBRT. There is
is now utilized to guide seed placement from a sound logic in combining high-dose therapy to the
transperineal approach, which has corrected the prostate with an implant and moderate doses of
problem of poor seed placement in experienced EBRT to the regional tissues to sterilize
hands. Two radioactive seed isotopes have been micrometastatic disease. In this situation, an
used: iodine (I-125), with a half-life of 60 days, and implant (110 Gy of I-131 or 100 Gy of Pd-104) usu-
palladium (Pd-103), with a shorter half-life (17 days) ally either precedes or follows 20 to 45 Gy delivered
and subsequent higher dose rate. The advantage of to the pelvis. Some reports have suggested an
the brachytherapy technique is that substantial dose increase in rectal toxicity with this approach; how-
can be delivered to the prostate with minimal effect ever, this is likely due to the poor quality of the
on the surrounding tissue. implant. At least one study from a leading implant
center suggested no significant increase in severe
Although concentrating dose with brachytherapy early or late GI/GU morbidity with combination
represents a potential advantage over EBRT, it also therapy. The value of supplemental EBRT needs to
highlights the need for appropriate patient selection. be evaluated in comparison to full-dose EBRT in
Significant dose falloff 2 to 3 mm beyond place- terms of long-term morbidity and cancer control.
ment of the seeds within the gland limits the appli-
cation of seed monotherapy in patients with poten- High-dose-rate (HDR) devices Besides permanent
tial periprostatic or regional disease extension. implants, which deliver low-dose-rate (LDR) radio-
Large studies from several leading institutions have therapy, brachytherapy for prostate cancer has been
now matured and confirm the long-term effective- delivered using temporary HDR devices, usually in
ness and safety of this approach in low-risk popula- patients with locally advanced disease. In this tech-
tions. Favorable results have also been reported in nique, a high dose (minimum, approximately 5 Gy)
selected intermediate-risk patients. Typical is delivered to the prostate over ≤ 1 hour by remotely
monotherapy doses of 145 Gy for I-125 and 125 Gy inserting a highly radioactive source into catheters
for Pd-103 are utilized. To date, the data do not sup- placed into the prostate using ultrasonographic guid-
port the use of either isotope over the other. ance while the patient is under anesthesia. Several
treatments are given on separate occasions, and
EBRT is used for approximately 5 weeks as well.

494 EDUCATIonAL REVIEW MAnUAL In URoLoGY


More reports are accumulating on the application of Moreover, patients with a rising PSA level after irra-
HDR brachytherapy to prostate cancer. Various dose- diation may be a heterogeneous group, including
fractionation combinations of HDR with or without patients with truly localized failure as well as those
combined pelvic EBRT have been employed, with a with metastatic disease. Also, certain patients will
dose-response relationship apparent in biochemical have a slowly rising PSA level after irradiation and
control. Although the follow-up is short and no may not require additional treatment. In patients
prospective randomized trials evaluating this who do not receive androgen ablation, the 5-year
approach have yet been published, it appears that actuarial risk of distant metastasis from the time that
HDR prostate brachytherapy in combination with the PSA level begins to rise is ~50%. A rapidly
pelvic EBRT may be effective. The long-term conse- emerging key concept in rising PSA levels is PSA
quences for normal tissue of delivering large doses velocity, or more specifically PSA doubling time.
per fraction using this technique are unclear. Multiple recent studies have found that a PSA dou-
bling time < 10 to 12 months predicts early clinical
Medications and devices to manage impotence relapse if biochemical recurrence is untreated. In
after prostatectomy, EBRT, or brachytherapy addition, recent studies have documented the real
Treatment for postprostatectomy impotence phenomenon of postradiotherapy PSA bounce,
includes the phosphodiesterase inhibitors sildenafil, which is defined as a rise above the baseline PSA fol-
vardenafil (Levitra), and tadalafil, prostaglandin E1, lowing the initiation of radiotherapy. This may occur
administered as a urethral suppository (Muse); in 20% to 40% of men depending on the threshold of
intercavernosal injection (Caverject, Edex); or PSA rise and is not known to have prognostic signifi-
VEDs that are useful for improving erections in men cance. Thus, PSA rises should be confirmed over
who have poor erectile function after prostatectomy, time to ensure that they are durable rather than tran-
radiation therapy, or brachytherapy. These therapies sient prior to initiating salvage systemic therapy.
are effective in 15% to 40% of men with postprosta-
tectomy impotence and in 50% to 75% of men with Treatment recommendations for recurrence
postradiotherapy erectile dysfunction. Insertion of a post irradiation
penile prosthesis is typically offered to patients only In general, men who have clear evidence of a rising
after unsuccessful trials with the previously men- PSA level 2 years after definitive radiotherapy for
tioned less invasive interventions. localized prostate cancer should be advised about
the options of hormonal therapy (see next section on
Detection and treatment of "Treatment of locally advanced disease [T3, T4]"),
recurrence salvage surgery, salvage cryotherapy, observation,
or experimental therapy.
Significance and definition of a rising PSA level
post irradiation If patients have minimal comorbidity, good life
The use of PSA levels following definitive therapy expectancy, and only local evidence of disease
(either radiotherapy or radical prostatectomy) can recurrence, salvage surgery is an option but should
detect early recurrences that may be amenable to be preceded by a bone scan, CT scan, cystoscopy,
salvage treatment. A rising PSA profile following and extensive counseling because urinary difficul-
radiotherapy is unequivocal evidence of the pres- ties after salvage prostatectomy are substantial and
ence of a residual prostatic neoplasm. However, the highly prevalent.
definition of a rising PSA level after radiation ther-
apy varies in the literature. A 1996 consensus con- Factors that determine success of salvage surgery
ference recommended that PSA failure be consid- after radiation therapy include low (< 4 to 10
ered to have occurred after three consecutive PSA ng/mL) preoperative PSA level, low pathologic
level rises, with the rate of failure defined as stage (T3a or less), and prior type of radiation ther-
halfway between the first rise and the previous PSA apy (brachytherapy, IMRT being favorable). How-
level. More recently, this definition has been ever, no randomized trials have been conducted in
replaced by an absolute PSA rise of 2 ng/mL above this setting to provide level I evidence favoring
the posttreatment nadir PSA level. surgery over other modes of treatment.

CHAPTER 16: PRosTATE CAnCER 495


Treatment of locally advanced disease analysis of RTOG 85-31 by Horwitz et al, which
(T3, T4) employed early indefinite androgen deprivation,
demonstrated that patients with locally advanced
The treatment of patients with locally advanced disease (T3N0) had improved cause-specific failure
prostate cancer is centered on a multimodality and and distant metastatic failure compared with EBRT
multidisciplinary approach, including radiation ther- alone. Furthermore, a comparison to RTOG 86-10,
apy (EBRT with or without HDR interstitial ther- which studied similar patients treated with only 4
apy), androgen ablation plus EBRT, or radical months of hormonal therapy, favored the long-term
prostatectomy with or without androgen deprivation. approach. The EORTC trial randomized 415
patients and demonstrated a 15% overall survival
EBRT with and without HDR interstitial therapy benefit to 3 years of combined therapy vs radiation
For patients with locally extensive prostate cancer, therapy alone. Finally, recent randomized studies by
local failure remains a potential problem after Widmark et al and the National Cancer Institute of
EBRT This problem has prompted investigations Canada have confirmed that external beam radio-
into alternative means to intensify therapy. therapy with 2-3 years of hormonal therapy is nec-
essary for locally advanced or high-risk prostate
One strategy has been to deliver large fractions of cancer as compared to hormonal therapy alone, due
radiotherapy using HDR interstitial techniques in to improvements in local control as well as systemic
combination with EBRT. The large interstitial frac- control, and this combination should be considered
tions, which may be on the order of 5 Gy, deliver a a standard initial therapy for high-risk men.
high dose to the prostate but spare normal tissues,
due to the rapid dose falloff outside the implanted Radical prostatectomy with or without adjuvant
volume. Early experience with this strategy is therapy
encouraging, but long-term data on outcome, partic- Surgical monotherapy can be considered a reason-
ularly in patients with locally extensive disease, and able option for patients with locally advanced
on morbidity are awaited. prostate cancer. Stage T3 disease can be successfully
treated with low morbidity and significant reductions
Patients with locally advanced prostate cancer prob- in risk of local recurrence, with clinical overstaging
ably are not good candidates for permanent prostate (up to 26%) reported by Yamada et al. Well-differen-
implants. Patients with stage T3/T4 tumors are at tiated and moderately differentiated cancers have
high risk for gross extraprostatic involvement, and cancer-specific survival rates of 76% at 10 years,
this localized therapy may not offer adequate dosi- comparable to those of other treatment modalities.
metric coverage of extraprostatic disease.
The Mayo Clinic has one of the largest radical
As mentioned in the previous section, there may be prostatectomy series for T3 disease, consisting of
a synergistic effect between hormonal therapy given more than 1,000 patients. In this population, of
in conjunction with radiation therapy. In addition to whom 34% received adjuvant therapy, 15-year can-
enhancing apoptosis and producing local cytoreduc- cer-specific survival and local recurrence rates were
tion, the use of early androgen deprivation may pos- 77% and 21%, respectively. In an ECOG clinical
sibly delay or even prevent the development of trial, 98 men who were found to have nodal metas-
metastatic disease. tases following radical prostatectomy and pelvic
lymphadenectomy were randomized to receive
The current body of evidence from three large ran- immediate androgen deprivation or be followed until
domized trials (RTOG 92-02, RTOG 85-10, and clinical disease progression. At a median follow-up
EORTC 22863) suggests that immediate long-term of 7 years, 18 of 51 men in the observation group had
androgen deprivation in conjunction with EBRT died, compared with only 4 of 47 in the treatment
improves outcomes among men with locally group (P = .02). In one interesting series, da Pozzo
advanced or high-risk (Gleason score ≥ 8) prostate found that adjuvant radiation therapy with androgen
cancer compared with radiation therapy alone. An deprivation therapy was associated with a survival
benefit (biochemical and prostate cancer specific) in

496 EDUCATIonAL REVIEW MAnUAL In URoLoGY


node-positive men treated with radical prostatec- metastatic prostate cancer include pain, fatigue,
tomy initially, even after adjustment for known con- skeletal fractures, spinal cord compression, urinary
founders. This finding suggests that local tumor con- outlet obstruction, and failure to thrive. First-line
trol may prevent distant failure in this disease. hormonal therapy for men with metastatic prostate
cancer delays these complications.
Treatment of node-positive disease
Whether any local treatment adds to the overall sur- Rising PSA level A large series of more than 2,000
vival duration in patients with known nodal patients treated with radical prostatectomy at Johns
involvement is debatable. Until recently, the stan- Hopkins University demonstrated that approxi-
dard of care had been to perform frozen-section mately 17% of cases recurred, with only 5.8% being
pathologic analysis on pelvic lymph nodes at the local disease. In the remaining patients, disease
time of radical prostatectomy, prior to removal of recurred initially with either a rise in PSA levels
the prostate. If this analysis revealed micrometas- alone (9.7%) or evidence of clinical metastases
tases, radical prostatectomy was thought to be con- (1.7%).
traindicated. Although retrospective in nature,
recent data from several American centers, includ- Outcomes for men with only a rising PSA level can
ing one large study from the Mayo Clinic, have vary greatly. Time to PSA recurrence (< 2 vs > 2
shown a survival benefit in men who undergo radi- years), PSA doubling time (< 9 vs > 9 months), and
cal prostatectomy despite the presence of Gleason score (8-10 vs 5-7) are among the impor-
micrometastases to regional pelvic lymph nodes. tant factors for predicting the development of
These men tend to do better and survive longer metastatic disease and survival. There is a wide
when started on early hormonal therapy, either with variation in the PSA value seen at the onset of bone
orchiectomy or an LHRH analog. metastases; one study documented an average PSA
of 33 at this onset; however, about 25% of men will
Radiation therapy There are also compelling data develop skeletal metastases with a PSA of less than
that long-term survival is achievable in these 10, and another 25% will not develop metastases
patients with combination radiation and hormonal even with a PSA of 50-90. Thus, PSA alone does not
therapy. Data from the MD Anderson Cancer Center necessarily predict the onset of metastatic diseases
indicate a benefit to pelvic/prostate radiation ther- and other factors such as PSA doubling time, life
apy plus immediate hormonal manipulation com- expectancy, comorbidities, and patient concern
pared with hormones alone. A subset analysis of often dictate when hormonal therapy is initiated.
patients with node-positive disease from RTOG 85-
31 revealed immediate hormonal therapy plus radia- The major developments for hormonal therapy in
tion therapy resulted in 5- and 9-year cause-specific advanced prostate cancer were achieved prior to
survival rates of 84% and 76%, respectively. There- routine PSA testing and were often complicated by
fore, aggressive locoregional therapy appears to be problematic study design. There are no prospective
effective in this cohort of patients with nodal data that confirm a benefit to early hormonal ther-
involvement. apy for men with a rising PSA alone. However, for
patients with a rising PSA level who are at high risk
Treatment of advanced systemic disease for the development of metastases, some physicians
agree that early hormonal therapy is likely to benefit
Defining advanced disease this group of patients as well as those with radio-
Metastatic prostate cancer This is a heterogeneous logic evidence of metastatic disease. Ongoing ran-
group of patients that ranges from those with patho- domized phase III studies are also testing the role of
logically detected locoregional nodal metastases at docetaxel (Taxotere) with androgen deprivation
the time of radical prostatectomy to those with therapy in this setting, but the role of chemotherapy
widespread systemic disease. The most common in the nonmetastatic setting remains experimental.
sites of metastatic disease are the bone and pelvis
and abdominal lymph nodes. Other less common
sites include the liver and lungs. Complications of

CHAPTER 16: PRosTATE CAnCER 497


First-line therapies for advanced disease The flare phenomenon can also be avoided through
The standard first-line treatment of advanced prostate the use of gonadotropin-releasing hormone (GnRH)
cancer, regardless of whether local treatment has antagonists. This class of drugs leads to immediate
been applied, is to ablate the action of androgens by suppression of androgen production without the ini-
medical or surgical means. For the majority of tial testosterone surge that results with GnRH ago-
patients, androgen ablation can result in a decline in nist therapy. Abarelix (Plenaxis) is a GnRH antago-
PSA level, palliation of disease-related symptoms, nist that was approved in 2003. However, the manu-
and regression of metastatic disease on imaging. facturer halted US sales to new patients in 2005. A
second agent, degarelix (Firmagon), was approved
Bilateral orchiectomy The advantages of orchiec- by the FDA in December 2008. As a receptor antag-
tomy over other means of castration include an onist, degarelix reversibly binds to the GnRH recep-
immediate decline in testosterone levels and ease of tors in the pituitary gland, immediately suppressing
compliance for patients. Given these advantages, the secretion of the LH, follicle-stimulating hormone
however, many men still opt for medical castration, (FSH), and, subsequently, testosterone levels. Cur-
with the potential advantage of intermittent hor- rently available in a monthly formulation and not
monal therapy. In addition, the psychological associated with anaphylaxis, degarelix is expected to
impact of orchiectomy can be significant. Nonethe- be available in a 3-month depot formulation by 2011.
less, bilateral orchiectomy may be appropriate and
cost-effective for a select group of patients. Antiandrogens Antiandrogens function to block
the binding of dihydrotestosterone (DHT) to the
LHRH analogs LHRH agonists, such as leuprolide androgen receptor, blocking the translocation of the
and goserelin, interfere with the normal pulsatile DHT-androgen receptor complex into the nuclei of
secretion of luteinizing hormone (LH) from the cells. There are two general classes: steroidal and
pituitary gland, resulting in an eventual decline in nonsteroidal. Steroidal antiandrogens include
serum testosterone levels. The effect is reversible cyproterone and megestrol. The most commonly
with cessation of therapy. Because LH is initially used antiandrogens include the nonsteroidal agents
increased with LHRH agonists, testosterone levels flutamide, bicalutamide, and nilutamide. These
increase initially as well. This finding can result in a agents differ slightly in their affinity for the andro-
transient rise in PSA levels and potential growth of gen receptor and their side-effect profiles. For
metastatic sites. Because of this initial "flare example, nilutamide has been associated with inter-
response" with LHRH agonists, consideration stitial lung disease and visual adaptation (light-
should be given to the administration of an antian- dark) disturbances, whereas flutamide is associated
drogen prior to the LHRH, especially in patients with diarrhea. Antiandrogens as a category have not
who are at risk for complications from the disease been as highly associated with cardiovascular risk
(such as spinal cord compression, worsening pain, outcomes, but this may be due to the low numbers of
or urinary outlet obstruction; Table 5). Side effects patients treated with antiandrogen monotherapy.
of androgen-deprivation therapy include hot Consideration of prophylactic breast radiation prior
flashes, metabolic-type syndrome and weight gain, to the use of prolonged antiandrogen monotherapy
loss of libido, loss of peripheral hair growth, should be considered, given the high risk (> 50%) of
gynecomastia, an elevated risk of diabetes, loss of developing gynecomastia. Tamoxifen has also been
bone mineral density and an increased risk of frac- shown to prevent gynecomastia in these men but is
ture, and finally an increased risk of cardiovascular associated with other adverse events such as deep
complications (including myocardial infarction, venous thrombosis/pulmonary embolism. Typi-
stroke, sudden cardiac death, deep venous thrombo- cally, antiandrogens are used in combination with
sis, and angina). The overall risk/benefit profile of surgical or medical castration. Some trials compar-
androgendeprivation therapy thus depends on an ing antiandrogens alone with LHRH analogs have
individual man's preexisting cardiovascular risk and shown similar efficacy, but more recent trials in
comorbidities and should be individually tailored metastatic disease suggest that monotherapy with
based on this risk assessment. antiandrogens may be inferior in terms of time to
disease progression and possibly survival. For

498 EDUCATIonAL REVIEW MAnUAL In URoLoGY


example, a randomized trial of monotherapy with GnRH agonists in terms of overall survival (15% to
high-dose bicalutamide (150 mg daily) compared 20% relative risk reduction). This finding led ASCO
with flutamide plus goserelin demonstrated that to advise an informed discussion of the risks and
patients treated with. benefits of CAB.

bicalutamide monotherapy had fewer side effects,


such as loss of libido or erectile dysfunction, and Several phase III trials evaluating the role of inter-
trended toward improved quality of life. However, mittent androgen deprivation are ongoing. A
in patients with radiographic evidence of metas- phase III European trial investigating the use of
tases, bicalutamide monotherapy was associated intermittent hormone therapy in patients with
with a small 6-week decrease in survival (hazard locally advanced or metastatic prostate cancer has
ratio = 1.3). Despite this finding, for men who are
been reported. In this trial, 766 patients were reg-
intolerant to the side effects of LHRH analogs,
istered, and 626 patients whose PSA level
monotherapy with antiandrogens can be considered
decreased to < 4 ng/mL or to 80% below the initial
after careful discussion with patients.
value after 3 months of induction treatment were
Antiandrogen monotherapy with flutamide or randomized to receive continuous vs intermittent
bicalutamide is also sometimes used to treat PSA hormone therapy. The primary outcome of disease
recurrence. progression was not statistically different
between groups. There was also no significant dif-
Used alone or in combination with a 5-alpha reduc- ference in overall survival. Among the 314 patients
tase inhibitor (finasteride or dutasteride), this on intermittent therapy, 50% were off therapy for
approach is associated with fewer side effects than at least 52 weeks following initial LHRH induction.
traditional AST, but the approach is not considered Patients whose PSA level dropped below 2 ng/mL
standard and its efficacy relative to primary gonadal spent a median of 82% of their time receiving no
suppression is not established. A significant down-
therapy
side is nipple tenderness or gynecomastia, but this
Calais da Silva FE et al: Eur Urol 55:1269-1277, 2009.
"peripheral blockade" approach may preserve
potency and libido. Prophylactic breast irradiation
may prevent this complication.
Many investigators believe that the advantages
observed in trials that include an LHRH antagonist
CAB Complete androgen blockade, or CAB, refers
exist because of the "flare phenomenon," which
to the elimination of testicular androgens in combi-
occurs with LHRH agonists alone and may be lost
nation with blockade of adrenal androgens, gener-
with a short period of treatment with antiandrogens
ally with an LHRH analog and an antiandrogen
during the expected flare period. ASCO has pub-
agent. The use of CAB is somewhat controversial.
lished guidelines for the initial management of
Several randomized trials comparing LHRH ago-
androgensensitive (recurrent, metastatic) prostate
nists alone vs CAB have demonstrated a survival
cancer. Based on the literature, immediate androgen
benefit with CAB. However, in one of the largest tri-
deprivation was associated with a moderate
als conducted by the United States Intergroup, more
decrease in prostate cancer mortality and a moder-
than 1,300 men were randomized to undergo
ate increase in other causes of mortality. Currently,
orchiectomy vs orchiectomy plus flutamide. There
there is no definitive evidence favoring the early ini-
was no significant advantage to CAB in terms of
tiation of androgen deprivation in this population.
time to disease progression or overall survival.
Some investigators believe that bicalutamide, a
Diethylstilbestrol (DES) Estrogen administration,
more potent agent, may be associated with greater
in the form of DES, also produces chemical castra-
survival when used as part of CAB. Recent meta-
tion. DES inhibits prostate growth, primarily
analyses suggest a small but incremental benefit of
through the inhibition of the hypothalamic-pituitary-
noncyproterone antiandrogens in combination with
gonadal axis, which blocks testicular synthesis of

CHAPTER 16: PRosTATE CAnCER 499


testosterone and thus lowers plasma testosterone dose prednisone is combined with abiraterone to
levels. Since doses higher than 3 mg/day cause sig- prevent adrenal insufficiency as well as to reduce
nificant cardiovascular mortality, DES has fallen out the mineralocorticoid excess (hypertension, fluid
of favor as a first-line therapy to induce castration. retention, hypokalemia) seen with abiraterone. The
use of prednisone also improves upon the overall
Ketoconazole (Nizoral) Ketoconazole is an antifun- efficacy of this agent. Additional CYP17 inhibitory
gal agent that can inhibit adrenal and testicular agents are in development such as TAK700. Cur-
steroid synthesis at higher doses, leading to a decline rently, abiraterone acetate is only FDA-approved
in adrenal and testicular androgens. It has the benefit for use in the post-docetaxel setting for men with
of a rapid decline in testosterone, which can be use- metastatic castration-resistant prostate cancer.
ful for patients who present emergently with a com-
plication of newly diagnosed advanced disease. Early vs late treatment Whether to treat patients
Ketoconazole is started at a dose of 200 mg three early with hormonal therapy or wait until patients
times daily and is increased to a total dose of 400 mg become symptomatic has been tested in a large
three times daily. Ketoconazole is associated with European trial conducted by the MRC. Men were
significant side effects (such as fatigue, nausea, and randomized to receive immediate hormonal therapy
vomiting) and drug interactions, however, and must (orchiectomy or an LHRH analog) vs delayed ther-
be given with supplemental hydrocortisone to avoid apy, which was initiated with symptomatic disease
symptoms of adrenal insufficiency. Because of the progression. Men who were treated with early ther-
side effects, its use is more common in the second- apy were less likely to experience urinary obstruc-
line setting, where responses can be expected in 20% tive symptoms requiring intervention, pathologic
to 40% of patients following disease progression fractures, and spinal cord compression than those
with complete androgen blockade. treated in the delayed arm (Table 6). The survival
benefit was less clear, however, because many of
Abiraterone acetate (Zytiga). Recent evidence the men in the delayed arm died before they
suggests that androgen production is not limited to received any hormonal therapy. This study was also
the testicles and adrenal glands, but that prostate complicated by the initiation of PSA monitoring
cancer and the surrounding microenvironment can during the study period. Many patients and physi-
contribute to androgen synthesis in an cians currently are not comfortable delaying therapy
autocrine/paracrine manner. This is mediated by until the onset of symptoms while the PSA level is
overexpression of key androgen synthetic enzymes rising; this fact limits the applicability of its findings
such as cytochrome P450 17-hydroxylase/lyase in the modern era.
(CYP17) and other enzymes, overexpression of
androgen transport molecules that allow cancer Other smaller trials have examined this issue as
cells to import circulating androgen precursors, and well. A Cochrane Database review was conducted
reductions in androgen metabolizing enzymes that in 2002; it demonstrated an increase in progression-
degrade androgens. Given these findings, combined free survival and a small, but significant, improve-
with additional findings of androgen receptor (AR) ment in survival with early hormonal therapy. A ran-
mutations, amplifications, and splice variants that domized EORTC study (30891) of early vs deferred
can lead to constitutive or overactive AR signaling, androgen deprivation therapy for men with local-
there have been significant recent efforts to inhibit ized prostate cancer not amenable to local treatment
AR signaling in tumors that were previously did not show a prostate cancer-specific survival
deemed hormone-refractory but are now classified advantage to the immediate use of androgen depri-
as castration-resistant, a more biologically correct vation therapy in these men. Balancing comorbidity
term. One such CYP17 inhibitor is abiraterone and the risk of cardiovascular complications is an
acetate, an oral agent with potent CYP17 inhibitory important consideration in the timing of androgen
activity that leads to a dramatic reduction in sys- deprivation therapy in this population, given the
temic androgen levels. Feedback upregulation of competing causes of mortality.
ACTH can be seen with this agent alone; thus low-

500 EDUCATIonAL REVIEW MAnUAL In URoLoGY


One setting in which early adjuvant hormonal ther-
apy has been associated with a survival benefit is in Sipuleucel-T became the first autologous cellular
the postprostatectomy setting in men found to have therapy approved for use in patients with solid
pathologic lymph node-positive disease. Messing et
tumors in 2010. This therapy involves an initial
al published an adjuvant study that evaluated imme-
leukapheresis whereby a small fraction of leuko-
diate hormonal therapy vs delayed treatment upon
cytes are removed by a pheresis procedure, typi-
detection of distant metastases or symptomatic
recurrence in men who had undergone radical cally through the American Red Cross, and sent to
prostatectomy and lymph node dissection and were Dendreon for modulation. During a proprietary
found to have nodal metastases. At a median fol- next step, the white blood cells are then pulsed
low-up of 11.9 years (range, 9.7-14.5 for surviving with an antigen cassette composed of prostatic
patients), men assigned immediate androgen depri- acid phosphatase (PAP) fused to granulocyte-
vation therapy had a significant improvement in macrophage colonystimulating factor (GM-CSF),
overall survival, prostate cancer-specific survival, an immune adjuvant. After 3 days, these newly
and progression-free survival. primed CD54 positive cells, which have been stim-
ulated and expanded ex vivo, are shipped back to
Immunotherapy There is currently level 1 evi- the patient and infused in a local treatment facil-
dence to support the use of sipuleucel-T (Provenge)
ity. This process is then repeated 2 more times dur-
immunotherapy vaccination as a therapy for men
ing a one-month procedure. Side effects have
with presymptomatic metastatic castration-resistant
included infusional reactions (fever, headache,
prostate cancer, based on several phase II studies
and one large phase III study. This phase III study perioral numbness, chills, muscle aches, and back
(the IMPACT trial) demonstrated a 4.1 month sur- pain) that are typically mild and transient (1-3
vival advantage over sham vaccination. Thus, autol- days) and can be treated with acetaminophen or
ogous dendritic cell therapy vaccination (three NSAIDs. Other side effects may include the need
infused doses over 4 weeks following initial leuka- for a central line for leukapheresis in about 25% of
pheresis), utilizing a prostatic acid phosphatase- men, which may require heparin and may increase
GMCSF fusion protein to stimulate immune cells, the risk of infection. In a double-blind, placebocon-
has become a standard of care prior to docetaxel in trolled, multicenter phase III trial, 512 patients
men with metastatic asymptomatic to minimally were randomly assigned in a 2:1 ratio to receive
symptomatic castration-resistant prostate cancer. either sipuleucel-T (341 patients) or placebo (171
Men with pain requiring narcotics, with visceral
patients). The median survival was 4.1 months
metastases, and with a life expectancy < 6 months
longer in the sipuleucel-T group than in the
are not eligible for this therapy. Of note, vaccination
placebo group (25.8 months vs 21.7 months). The
did not lead to PSA declines, tumor responses,
improved palliation, or delayed tumor progression estimated probability of survival 36 months after
by our current measures, and thus this therapy randomization was 31.7% in the sipuleucel-T
should be regarded as adjunctive to other current group and 23.0% in the placebo group
therapies that are more cytoreductive and palliative. (Kantoff PW et al: N Engl J Med 363:411-422, 2010).

Intermittent androgen deprivation Androgen depri-


vation is associated with several short-term and long-
term adverse effects. These side effects make treat-
ment breaks provided by intermittent androgen depri-
vation an attractive option. Additionally, results from
preclinical studies suggest that hormonal resistance
may be delayed with intermittent androgen depriva-
tion. These potential advantages have led to significant
interest among patients and caregivers in intermittent

CHAPTER 16: PRosTATE CAnCER 501


androgen deprivation. Over the past decade, several from the MRC trial and the Cochrane Database
phase II-III studies of intermittent androgen depriva- review, understanding that this information is extrapo-
tion have demonstrated feasibility and safety with sug- lated from data obtained prior to PSA testing and from
gestion of improved quality of life without negative patients with clinical and radiographic metastases.
effects on time to disease progression or survival.
More recently, the phase III SWOG/NCIC/UK coop- Second-line hormonal therapies
erative group trial of continuous vs intermittent andro- It is important to realize that there has been a shift in
gen-deprivation therapy in men with M0 PSA recur- terminology from "androgen-independent" or "hor-
rent prostate cancer was reported. In this trial, 690 mone-refractory" prostate cancer to a newer term:
patients randomized to the intermittent therapy arm castration-resistant prostate cancer. This change
were found to have improved quality of life with less reflects an understanding that prostate cancer often
sexual dysfunction, hot flashes, and improved physical remains dependent on androgenic signaling, even in
function and fatigue with no significant difference in the presence of castrate levels of testosterone. Tumors
time to disease progression or overall survival. The may produce their own androgens through autocrine
final results of several ongoing phase III trials in signaling, amplify low levels of testosterone ligand
metastatic disease and PSA-only relapse will better signaling through androgen receptor (AR) mutations
define the role of intermittent androgen deprivation. or duplications, and may have activation of the AR
With the currently available information, intermittent through other ligands. Thus, progression of disease
androgen deprivation may be considered in most despite castration does not necessarily imply resis-
patient settings as a standard of care. tance to all hormonal strategies, as exemplified by the
response to antiandrogens, ketoconazole, and newer
Treatment recommendations Just as for localized second-generation agents such as MDV3100 or abi-
disease, initial treatment for advanced prostate can- raterone acetate. Thus, true hormone-refractory dis-
cer must be individualized. A patient who presents ease may refer to disease that has progressed despite
with a rising PSA level only after local treatment therapies employing all known hormonal strategies.
and a slow PSA doubling time, a prolonged time to
PSA recurrence, and a low initial Gleason score Outcomes with initial androgen ablation can vary
may not require immediate therapy, especially if from responses that last from months to years; they
there are other more likely significant comorbidi- also vary as a function of the Gleason grade, pre-
ties. However, a patient with multiple metastatic treatment PSA velocity, and extent of disease at the
sites will need immediate treatment, generally with time of initiating treatment. Once PSA levels begin
orchiectomy, LHRH agonists or antagonists, or to rise with androgen ablation, the disease is often
CAB initially followed by monotherapy with an referred to as "hormone-refractory." This term is
LHRH analog, to prevent the sequelae of metastatic actually a misnomer, because preclinical data sug-
disease, such as fracture, spinal cord compression, gest that tumors may become hypersensitive to
and ureteral obstruction. Degarelix or ketoconazole androgens, resulting in worsened disease if andro-
may be considered as initial systemic therapies for gen ablation is removed entirely. Moreover, many
patients presenting with spinal cord compression as patients have disease that remains sensitive to fur-
the first sign of prostate cancer, as well as combined ther hormonal manipulations, such as second-line
androgen blockage. Palliative radiation in this set- antiandrogens, steroids, or ketoconazole.
ting should also be considered. Given the overall
limitations of hormone therapy, all appropriate An example of this sensitivity to hormonal manipu-
patients should be offered access to clinical trials. lation is exemplified in the antiandrogen withdrawal
response. Up to one-third of patients with a rising
For patients with a rising PSA level who are at high PSA level while receiving treatment with an antian-
risk for the development of metastases, a discussion drogen will have a decline in PSA level and or clini-
regarding the potential advantages to early treatment cal regression with antiandrogen withdrawal. The
and an explanation of the lack of randomized prospec- mechanism of this response has not been fully eluci-
tive data are warranted. Some investigators favor dated but supports the hypothesis that the androgen
early treatment for these patients based on the data receptor remains important in progressive disease.

502 EDUCATIonAL REVIEW MAnUAL In URoLoGY


Although second-line hormonal therapy has demon- with castration-resistant metastatic prostate cancer
strated benefit in terms of PSA levels and response, treated with docetaxel-based chemotherapy (Table
there are no data to demonstrate a survival advan- 8). Investigators from the SWOG 9916 trial random-
tage with second-line hormonal therapy. Its role has ized patients to receive mitoxantrone plus pred-
further come into question with data that support the nisone vs docetaxel (60 mg/m2) plus estramustine
use of docetaxel chemotherapy for men with (Emcyt) and dexamethasone every 3 weeks. Patients
metastatic androgen-independent prostate cancer to in the docetaxel arm had a significant improvement
improve survival. The survival advantage of doc- in survival by 2 to 3 months. Currently, estramustine
etaxel is not limited by the number of prior hor- is no longer used in combination with docetaxel in
monal therapies. However, as exemplified by recent the front-line setting given the lack of additional effi-
novel antiandrogens (MDV3100) and cacy and the certain added toxicity.
adrenal/autocrine synthesis-inhibiting agents (abi-
raterone acetate), men with castrationresistant dis- A second international randomized phase III trial
ease remain sensitive to agents targeting the andro- (TAX327) by Tannock et al showed a similar sur-
gen receptor, and this remains a rich area of clinical vival benefit of 3 months with docetaxel (75 mg/m2)
investigation. In phase I/II trials, the novel oral plus prednisone given every 3 weeks compared with
antiandrogen MDV3100 demonstrated striking mitoxantrone and prednisone. These trials were the
PSA declines (50% to 70% achieved a > 30% first to demonstrate a survival benefit with
decline) and partial tumor responses both in the pre- chemotherapy in advanced prostate cancer and have
docetaxel and postdocetaxel settings, with sparked numerous studies involving docetaxel in
responses that were durable in many men over 6 combination with newer agents. Toxicities of doc-
months. The drug can prevent androgen-induced etaxel include myelosuppression and peripheral neu-
nuclear translocation and has shown tumoricidal ropathy, both of which can be dose-limiting. Addi-
activity even in bicalutamide-resistant model sys- tional toxicities include constipation, tearing due to
tems. Abiraterone acetate with prednisone has docetaxel deposition in tear ducts, onycholysis, and
demonstrated similar outcomes in these settings as fluid retention (peripheral or pulmonary edema,
well, showing that hormonal sensitivity may remain pleural effusion). Weekly docetaxel with prednisone
even among docetaxel-resistant men. Phase III trials (30 mg/m2) for 5 of 6 weeks provided an intermedi-
of these agents are completed or ongoing (Table 7). ate level of control and survival that was not statisti-
cally different from that with mitoxantrone
For patients with a rising PSA level only, timing of (Novantrone) and prednisone, despite favorable
chemotherapy is even less clear. The ECOG PSA declines and tumor and pain responses. Weekly
attempted a trial comparing ketoconazole/hydrocor- docetaxel is less well tolerated, and early treatment
tisone with docetaxel in patients with a rising PSA discontinuation likely limits this schedule's utility.
level but no evidence of metastatic disease after hor-
monal therapy, but the trial was closed early due to A 5-year update of this study has confirmed a 3-
lack of accrual. Ongoing randomized studies of month survival advantage to every-3-week doc-
androgen-deprivation therapy with and without etaxel in the overall study population. Additional
chemotherapy should address this question within benefits of docetaxel over mitoxantrone included
the coming few years. Until prospective data are superior PSA responses, pain and quality-of-life
available, physicians will need to counsel patients responses, greater durability of response, and
carefully on the different options and timing of improved radiographic responses. In addition,
those options available at the time of disease pro- nearly 18% of men experienced normalized PSA
gression, including second-line hormonal manipu- levels with every-3-week docetaxel, as opposed to
lation, chemotherapy, and especially clinical trials. 8% of men treated with mitoxantrone. Men with
normalized PSA levels lived on average 33 months,
Chemotherapy for castration-resistant disease compared with 16 months for men without normal-
Docetaxel The role of chemotherapy changed signif- ized PSA levels—nearly a twofold difference. Cur-
icantly in 2004 with the results of two large random- rent studies indicate that a 30% or greater decline in
ized trials demonstrating a survival benefit for men the serum PSA level within 3 months of treatment

CHAPTER 16: PRosTATE CAnCER 503


initiation may be the best predictor of overall sur- cells can be further characterized in research settings
vival of all current surrogate markers in this disease for molecular profiling, indicating their potential to
and may be used to assist in prognostication after help guide systemic therapy in the future.
treatment initiation. Although not a credentialed
surrogate for FDA approval, this level of PSA Mitoxantrone plus prednisone Mitoxantrone (12
decline was shown in both pivotal studies to be mg/m2) plus prednisone has been approved for use
highly associated with survival as compared to in advanced prostate cancer based on improvement
other outcomes and PSA metrics, including the tra- in palliation of pain and quality of life over pred-
ditional confirmed > 50% decline in PSA level and nisone alone despite no improvement in overall sur-
pain responses. The timing of docetaxel initiation is vival. Toxicities of mitoxantrone include a cumula-
controversial, given its known toxicities and the tive cardiotoxicity, typically after 10 to 12 cycles of
many active hormonal and now immunologic thera- therapy. Thus pretreatment ejection fraction assess-
pies that can be administered prior to docetaxel. ment is recommended in all men as well as serial
However, it has been demonstrated that the absolute ejection fraction assessments every 4 to 5 cycles or
survival advantage of docetaxel is greatest (3 to 4 based on new onset of cardiac symptoms. Given the
months) in men with minimal symptoms, whereas recent approval of cabazitaxel (Jevtana; see discus-
men who have impaired performance status or pain sion later in this chapter), use of mitoxantrone has
due to cancer are not able to tolerate 10 full cycles generally been limited to third-line therapy, or for
well and often have a reduction in the overall sur- palliative therapy in men who are not candidates for
vival benefit (1 to 2 month advantage). Thus, the microtubule-targeting therapies because of neu-
early timely use of docetaxel in men with clear ropathy, for example.
prostate cancer progression based on PSA or radio-
graphic disease is warranted for palliation and pre- Bisphosphonates Bone metastases from prostate
vention of pain onset as well as improved survival. cancer are associated with increased bone formation
around tumor deposits, resulting in characteristic
Several nomograms currently exist for men with osteoblastic metastases. However, concomitant
metastatic castration-resistant prostate cancer that with the osteoblastic activity is a marked increase in
can predict overall survival (Halabi, MSKCC, Arm- bone resorption and osteolysis, which can be inhib-
strong-TAX327). These studies indicate that the ited by bisphosphonates.
presence of visceral metastatic disease, anemia, per-
formance status, PSA level, PSA doubling time, ele- Studies of bisphosphonates in prostate cancer have
vated alkaline phosphatase, low albumin, Gleason demonstrated mixed results. A combined analysis of
score, the presence of significant pain, the type of two multicentered randomized controlled trials
progression and lactate dehydrogenase level are comparing pamidronate with placebo in men with
highly predictive of survival and can be used to pre- androgenindependent progressive prostate cancer
dict prognosis. In addition, circulatory tumor cells demonstrated no benefit in terms of skeleton-related
are FDA-approved as prognostic markers of survival events or palliation of symptoms. A phase III trial
in men receiving docetaxel. These cells can be enu- with an oral bisphosphonate, clodronate (Bonefos),
merated in 7.5 mL of whole blood either prior to sys- demonstrated no difference in either symptomatic
temic chemotherapy or while the patient is receiving bone metastases or prostate cancer-related deaths
chemotherapy, and the number of circulating tumor when compared with placebo.
cells (CTCs) in whole blood correlates strongly with
overall survival. Although CTCs are not creden- A phase III trial demonstrated a reduction in skele-
tialed at this time as surrogates and thus have not ton-related events for men with castration-resistant
been used or studied to guide therapeutic choices in metastatic prostate cancer with a more potent bis-
men with castrationresistant prostate cancer, the phosphonate, zoledronic acid (Zometa). However, it
ability to detect and characterize these cells holds is important to note that this trial did not show an
promise as a predictive and intermediate biomarker improvement in quality of life with zoledronic acid,
to aid in personalized medicine approaches. These nor did it demonstrate a reduction in the develop-

504 EDUCATIonAL REVIEW MAnUAL In URoLoGY


ment of new metastases. Zoledronic acid did Second-line therapy after docetaxel
demonstrate a delay in the need for radiation to Cabazitaxel (Jevtana) with prednisone In 2010, it
bone, pathologic fracture onset, and bone pain, pro- was demonstrated that a novel synthetic taxane,
viding some evidence of clinical benefit over time. cabazitaxel, demonstrated improved overall sur-
At this time, the recommendation for zoledronic vival as compared with mitoxantrone in men with
acid in prostate cancer is limited to men with castra- metastatic castration-resistant prostate cancer
tion-resistant metastatic prostate cancer, pending whose disease had progressed on docetaxel
ongoing randomized studies in the hormone-sensi- chemotherapy. The overall survival benefit was 2
tive population. It is important to recognize the limi- months (from 12.7 to 15.1 months, HR = 0.70, P <
tations of this therapy and to understand that there is .0001), and improvements in PSA response, tumor
no defined role for its use in men with androgen- response, and progression-free survival were
dependent prostate cancer. Although these men are notable. Each regimen was similarly palliative, but
at higher risk for osteoporosis, less potent oral bis- the duration of response favored cabazitaxel.
phosphonates may be a more reasonable approach, Cabazitaxel is given intravenously at 25 mg/m2
given the long-term side effects associated with every 3 weeks with prednisone dosed at 5 mg orally
zoledronic acid (renal insufficiency and osteonecro- twice daily. The risk of neutropenia and sepsis due
sis of the mandible). to myelosuppression in this heavily pretreated
group of men is high (7% to 8%) and prophylactic
Denosumab (XGeva) A novel approach to the pre- GM-CSF should strongly be considered. Other toxi-
vention of bone loss and skeletal events in recurrent cities include neuropathy, fatigue, diarrhea, nausea,
prostate cancer involves the use of an inhibitor anti- and vomiting. Continuation of testosterone suppres-
body to block the RANKL (receptor activator of sion was required. Cabazitaxel has demonstrated
nuclear factor-κB ligand), a molecule involved in activity in docetaxelresistant cell lines and was
osteoclast-mediated bone resorption and remodel- approved by the FDA in 2010 for second-line treat-
ing. This antibody, termed denosumab (Xgeva), has ment of men with chemorefractory castration-resis-
demonstrated an ability to improve bone mineral tant prostate cancer.
density in men undergoing androgen deprivation
theapy, as well as an improved capability in prevent- Abiraterone acetate (Zytiga) with prednisone On
ing pathologic fractures, the need for radiation or April 28, 2011, the FDA approved the androgen
surgery to bone, or spinal cord compression (so- synthesis inhibitor abiraterone acetate in combina-
called skeletal-related events) and restoring bone tion with low-dose prednisone for the treatment of
density as compared with zoledronic acid in head-to- men with metastatic castration-resistant prostate
head studies. While osteonecrosis is also a side effect cancer (mCRPC) who have received prior
with this compound (1%-4% risk over time), its sub- chemotherapy containing docetaxel. Autocrine
cutaneous administration and lack of kidney toxicity and/or paracrine androgen synthesis is known to be
have led to an approved and now standard role for enhanced in the tumor microenvironment during
this agent for the prevention of skeletal events in castration-resistant progression in many men, and
men with castration-resistant prostate cancer. abiraterone acetate inhibits a key step in testos-
terone/dihydrotestosterone precursor synthesis,
Further studies of the role of denosumab in the pre- notably the cytochrome P450 c17 (lyase, hydroxy-
vention of metastasis and hormone therapy-induced lase) enzyme. Blockade of this enzyme is known to
clinical fractures will help to guide the rational use reduce testosterone production from the adrenal
of this agent, which is given by subcutaneous injec- gland and is thought to reduce intratumoral synthe-
tion. Side effects of denosumab include hypocal- sis as well, mediated by a reduction in androgen
cemia and hypophosphatemia and osteonecrosis of receptor activity.
the jaw, similar to or slightly greater than that of
zoledronic acid.

CHAPTER 16: PRosTATE CAnCER 505


improvement in OS in patients receiving abi-
In 2010, the first agent to demonstrate an raterone acetate compared to those on the placebo-
improvement in survival (cabazitaxel) was
containing arm (HR = 0.646; 95% CI: 0.543–0.768;
P < 0.0001). The median OS was 14.8 vs 10.9
approved based on an improvement in overall sur-
months in the abiraterone and placebo arm, respec-
vival compared with mitoxantrone. This agent is a
tively. Abiraterone acetate was also shown to result
novel taxane that is not a substrate for the P-glyco-
in improved pain palliation and had a significant
protein drug efflux pump, and has shown antitu- delay in the time to pain progression compared to
mor activity in docetaxel-resistant cell lines and prednisone alone. Improvements in progression-
model systems. Its activity even in docetaxel- free survival (5.6 months vs 3.6 months) and PSA
refractory patients has led to it becoming a stan- responses (29% vs 6%) were also noted. All sub-
dard option in this setting. In a randomized, open- groups based on baseline characteristics (age, per-
label, phase III study of 755 patients (377 in the formance status, PSA, pain intensity, visceral
mitoxantrone group, 378 in the cabazitaxel metastases) demonstrated improvements in out-
group), an overall survival benefit was shown in comes over prednisone alone.
favor of cabazitaxel.
Median overall survival was 15.1 months (95% CI: 14.1–16.3) The most common adverse reactions seen with abi-
versus 12.7 months (11.6–13.7). This result corresponds to a raterone/prednisone (> 5%) were joint swelling or
30% reduction in relative risk of death (HR = 0.70; 95% CI: discomfort, hypokalemia, edema, muscle discom-
0.59–0.83; P < .0001; de Bono JS, et al: Lancet 376:1147- fort, hot flush, diarrhea, urinary tract infection,
1154, 2010. cough, hypertension, arrhythmia, urinary frequency,
nocturia, dyspepsia and upper respiratory tract
infection. The most common adverse drug reactions
The FDA approval is based on the results of a random- resulting in drug discontinuation were increased
ized, placebo-controlled, multicenter trial in 1,195 aspartate aminotransferase and/or alanine amino-
patients with mCRPC previously treated with doc- transferase levels, urosepsis, and cardiac failure
etaxel-containing regimens . Patients were randomly (each in < 1% of patients taking abiraterone). The
allocated (2:1) to receive either abiraterone acetate most common electrolyte imbalances in patients
orally at a dose of 1000 mg once daily (N=797) or receiving abiraterone were hypokalemia (28%) and
placebo once daily (N=398). Patients in both arms hypophosphatemia (24%). Following interruption
(abiraterone acetate and placebo) received prednisone of daily corticosteroids and/or with concurrent
5 mg orally twice daily because of the ability of pred- infection or stress, adrenocortical insufficiency (<
nisone to improve both the safety and efficacy profile 1%) has been reported in clinical trials in patients
of abiraterone acetate as well as to serve as an efficacy receiving abiraterone acetate at the recommended
control group, given the modest but known efficacy of dose in combination with prednisone. It is recom-
prednisone alone in this population. Treatment contin- mended that close monitoring of serum electrolytes
ued until disease progression (defined as a 25% and liver enzymes be conducted during therapy with
increase in PSA level over the patient's baseline/nadir abiraterone. Abiraterone should be taken in a fasting
together with protocol-defined radiographic progres- state due to the higher levels of drug exposure when
sion and symptomatic or clinical progression), unac- taken with food.
ceptable toxicity, initiation of new treatment, or with-
drawal. Patients with prior ketoconazole treatment for Thus, following docetaxel chemotherapy for
prostate cancer were excluded, although abiraterone metastatic CRPC, the hormonally active androgen
acetate is known to have some benefit in these synthesis inhibitor abiraterone acetate has demon-
patients, although probably to a lower degree. strated clinical benefit and thus represents a new
standard of care in this setting. The NCCN Prostate
A pre-specified interim overall survival (OS) analy- Cancer Guidelines also acknowledges that some
sis was performed when 552 events had occurred. men with metastatic CRPC are not candidates for
This analysis demonstrated a statistically significant chemotherapy (docetaxel or mitoxantrone) because

506 EDUCATIonAL REVIEW MAnUAL In URoLoGY


of comorbidities, peripheral neuropathy, or con- Newer therapies
cerns over tolerability and risk/benefit. In these Several newer therapies for men with prostate cancer
men, abiraterone acetate with prednisone may be an (some as single agents and some with chemother-
appropriate therapy, given its survival and palliative apy) are under investigation. Since docetaxel is the
benefits and reasonable toxicity profile. However, only approved chemotherapy in the front-line setting
its routine use in the pre-docetaxel setting should be to date associated with a survival advantage in
discouraged until level 1 evidence from an ongoing patients with metastatic castrate-resistant prostate
randomized study of abiraterone acetate and pred- cancer, it has become the "backbone" upon which
nisone vs prednisone alone in this setting has been novel therapies and response modifiers are added in
reported. This trial has completed accrual, and ini- an attempt to improve patient outcomes. For exam-
tial results are expected soon. ple, the addition of bevacizumab (Avastin) to doc-
etaxel did not improve overall survival in men with
metastatic castration-resistant prostate cancer. Thus,
In 2011, abiraterone demonstrated improvements docetaxel-prednisone remains the front-line treat-
in overall survival in the post-docetaxel setting, ment of choice for men with metastatic castration-
providing evidence that hormone-refractory resistant prostate cancer who have failed to respond
prostate cancer often remains sensitive to addi- to prior hormonal therapy.
tional hormonal manipulations, particularly those
Based on promising results in phase II testing, sev-
that reduce the autocrine/paracrine/adrenal
eral phase III trials building upon docetaxel were
androgen axis. All subgroups of patients in this
opened (Table 9). Whether building upon doc-
trial benefited; currently no biomarker exists that
etaxel with targeted therapy will be of benefit is
can predict whether abiraterone or cabazitaxel is still unknown pending the results of the remaining
the most appropriate next therapy after doc- phase III trials. Phase III trials of other therapies
etaxel. Palliative responses and survival are similar both before or after docetaxel, including novel
with each agent and it is likely that both agents agents targeting the androgen receptor
will be used in these men upon progression. Inter- (MDV3100, TAK700 for example), immune thera-
estingly, bone scan healing flares are commonly pies (lenalidomide, tasquinimod, prostvac), and
seen early on with these agents, despite pain and bone microenvironment agents (dasatinib, zibo-
PSA reductions, indicating the need to obtain con- tentan) are awaited. Table 7 presents ongoing
firmatory bone scans that show additional new phase III studies of novel agents in the postdoc-
lesions before stopping therapy in the absence of
etaxel setting. Finally, many of these agents are
being tested in men with asymptomatic CRPC or
other evidence of clinical progression. In a multina-
minimally symptomatic men with metastatic
tional, randomized, double-blind, placebo-con-
CRPC (ie, the window prior to docetaxel use), with
trolled study, 1195 patients who had previously
the aim of delaying chemotherapy use/need.
received docetaxel were randomly assigned (in a
2:1 ratio) to receive 5 mg of prednisone twice daily Sipuleucel-T vaccine was approved by the FDA for
with either 1000 mg of abiraterone acetate (797 advanced prostate cancer in April 2010. In addition,
patients) or placebo (398 patients). After a median current trials are investigating the use of systemic
follow-up of 12.8 months, overall survival was agents, including docetaxel, in locally advanced or
longer in the abiraterone acetate– prednisone PSA-recurrent disease, either prior to or adjuvant
group than in the placeboprednisone group following surgery or radiation therapy. Accrual to
4.8 months vs. 10.9 months; HR = 0.65; 95% CI: 0.54–0.77; these trials has been a priority (in RTOG 0521 [now
P < 0.001; de Bono JS, et al: N Engl J Med 364:1995-2005, completed], VA 553, and CALGB 90203), as they
2011. are investigating the role of systemic therapy to pre-
vent disease recurrence, similar to the widely
accepted use of systemic therapy for other tumor
types such as breast and colorectal cancers. RTOG

CHAPTER 16: PRosTATE CAnCER 507


is exploring the role of docetaxel in addition to long- For patients with more extensive bone involvement
term androgen-deprivation therapy for men with causing pain that may be difficult to address with
locally advanced high-risk prostate cancer who localized EBRT, alternatives include wide-field irra-
have completed IMRT of the prostate. CALGB diation (ie, hemibody irradiation) or systemic
90203 randomized high-risk men (Gleason score > administration of radioactive bone-seeking isotopes
8 or nomogram-defined high risk) to immediate rad- that can deliver therapeutic doses to skeletal
ical prostatectomy vs docetaxel for 6 cycles with metastatic disease. Radioactive isotopes used in this
androgen deprivation therapy followed by radical fashion include strontium-89 chloride (Metastron)
prostatectomy. The use of neoadjuvant or adjuvant and samarium SM 153 lexidronam (Quadramet). An
docetaxel for high-risk men will be guided through ongoing phase III study of radium-223 (alpharadin)
evidence provided by these trials. recently demonstrated an improvement in overall
survival over radiation therapy alone with best sup-
Recent guidelines by the PCWG2 have updated portive care (11 to 14 months), providing evidence
methods for categorizing disease states in men with that this radioisotopic alpha emitter can provide clin-
castration-resistant prostate cancer (node only, PSA ical benefit in men with symptomatic bone metas-
only, locally advanced, bone metastatic visceral dis- tases. Its use will likely be in the post-chemotherapy
ease). In addition, the requirements for confirmation phase for men with diffuse bony pain.
bone scans and changes in the reporting and assess-
ment of disease progression have been updated to be
more consistent with our understanding of prostate
cancer biology bone scan flares with effective ther-
apy, and to prevent the unnecessary early abandon-
ment of potentially active agents based on PSA
changes alone. For example, healing of bone lesions
may cause an apparent "worsening" of disease on
bone scan despite declines in PSA levels and
improvement in pain, likely due to osteoblastic
activity in regressing tumors. Physicians are
encouraged to evaluate these potential flare cases
with a confirmatory bone scan 6 or more weeks
later; if the patient develops no additional new
lesions or disease progression, he should be main-
tained on therapy. Many older

therapies may have been unnecessarily stopped pre-


maturely due to this misclassification of bone scan
progression using older criteria.

Radiation therapy for palliating bone metastasis


Radiotherapy is effective in controlling local pain
associated with skeletal prostate metastasis. In gen-
eral, a treatment regimen of 30 Gy over 10 treat-
ments results in rapid and durable local symptom
control and a reduced dependence on analgesics.
Single-dose palliative radiation therapy may pro-
vide equal palliation as well.

508 EDUCATIonAL REVIEW MAnUAL In URoLoGY


8. Suggested Reading

Andriole GL, Crawford ED, Grubb RL 3rd, et al; de Bono JS, Oudard S, Ozguroglu M, et al: Pred-
PLCO Project Team: Mortality results from a ran- nisone plus cabazitaxel or mitoxantrone for
domized prostate-cancer screening trial. N Engl J metastatic castration-resistant prostate cancer pro-
Med 360:1351-1354, 2009. gressing after docetaxel treatment: A randomised
open-label trial. Lancet 376:1147-1154, 2010.
Armstrong AJ, Garrett-Mayer E, de Wit R, et al:
Prediction of survival following first-line de Bono JS, Scher HI, Montgomery RB, et al: Cir-
chemotherapy in men with castration-resistant culating tumor cells predict survival benefit from
metastatic prostate cancer. Clin Cancer Res 16:203- treatment in metastatic castration-resistant prostate
211, 2010. cancer. Clin Cancer Res 14:6302-6309, 2008.

Armstrong AJ, Garrett-Mayer ES, Yang YC, et al: A Droz JP, Balducci L, Bolla M, et al: Management of
contemporary prognostic nomogram for men with prostate cancer in older men: Recommendations of
hormone-refractory metastatic prostate cancer: A a working group of the International Society of
TAX327 study analysis. Clin Cancer Res 13:6396- Geriatric Oncology. BJU Int 106:462-469, 2010. 25
6403, 2007.
Freedland SJ, Humphreys EB, Mangold LA, et al:
Armstrong AJ, Tannock IF, de Wit R, et al: The Death in patients with recurrent prostate cancer
development of risk groups in men with metastatic after radical prostatectomy: Prostate-specific anti-
castration-resistant prostate cancer based on risk gen doubling time subgroups and their associated
factors for PSA decline and survival. Eur J Cancer contributions to all-cause mortality. J Clin Oncol
46:517-525, 2010. 25:1765-1771, 2007.

Attard G, Reid AH, A'Hern R, et al: Selective inhi- Freedland SJ, Mavropoulos J, Wang A, et al: Carbo-
bition of CYP17 with abiraterone acetate is highly hydrate restriction, prostate cancer growth, and the
active in the treatment of castration-resistant insulin-like growth factor axis. Prostate 68:11-19,
prostate cancer. J Clin Oncol 27:3732-3748, 2009. 2008.

Bañez LL, Hamilton RJ, Partin AW, et al: Obesity- Giovannucci E, Liu Y, Platz EA, et al: Risk factors
related plasma hemodilution and PSA concentration for prostate cancer incidence and progression in the
among men with prostate cancer. JAMA 298:2275- health professionals follow-up study. Int J Cancer
2280, 2007. 121:1571-1578, 2007.

Barry MJ: Screening for prostate cancer—The con- Jemal A, Siegel R, Xu J, Ward E: Cancer statistics,
troversy that refuses to die. N Engl J Med 360:1351- 2010. CA Cancer J Clin 60:277-300, 2010.
1354, 2009.
Kantoff PW, Higano CS, Shore ND, et al: Sipuleu-
D'Amico AV, Chen MH, Renshaw AA, et al: Andro- cel-T immunotherapy for castration-resistant
gen suppression and radiation vs radiation alone for prostate cancer. N Engl J Med: 363:411-422, 2010.
prostate cancer: A randomized trial. JAMA 299:289-
295, 2008. Kawachi MH, Bahnson RR, Barry M, et al: NCCN
clinical practice guidelines in oncology: Prostate
D'Amico AV, Denham JW, Crook J, et al: Influence cancer early detection. J Natl Comp Canc Netw
of androgen suppression therapy for prostate cancer 8:240-262, 2010.
on the frequency and timing of fatal myocardial
infarctions. J Clin Oncol 25:2420-2425, 2007. Keating NL, O'Malley AJ, Freedland SJ, et al: Dia-
betes and cardiovascular disease during androgen
de Bono JS, Logothetis C, Molina A, et al: Abi- deprivation therapy: Observational study of veter-
raterone and increased survival in metastatic prostate ans with prostate cancer. J Natl Cancer Inst 102:39-
cancer. N Engl J Med 364:1995-2005, 2011. 46, 2010.

CHAPTER 16: PRosTATE CAnCER 509


Kirsh VA, Peters U, Mayne ST, et al: Prospective Sanda MG, Dunn RL, Michalski J, et al: Quality of
study of fruit and vegetable intake and risk of life and satisfaction with outcome among prostate-
prostate cancer. J Natl Cancer Inst 99:1200-1209, cancer survivors. N Engl J Med 358:1250-1261,
2007. 2008.

Klotz L, Zhang L, Lam A, et al: Clinical results of Scher HI, Halabi S, Tannock I, et al: Design and end
long-term follow-up of a large, active surveillance points of clinical trials for patients with progressive
cohort with localized prostate cancer. J Clin Oncol prostate cancer and castrate levels of testosterone:
28:126-131, 2010. Recommendations of the Prostate Cancer Clinical
Trials Working Group. J Clin Oncol 26:1148-1159,
Loblaw DA, Virgo KS, Nam R, et al: Initial hor- 2008.
monal management of androgen-sensitive
metastatic, recurrent, or progressive prostate can- Schröder FH, Hugosson J, Roobol MJ, et al: ERSPC
cer: 2006 Update of an American Society of Clini- Investigators: Screening and prostate-cancer mor-
cal Oncology practice guideline. J Clin Oncol tality in a randomized European study. N Engl J
25:1596-1605, 2007. Med 360:1320-1328, 2009.

Lu-Yao GL, Albertsen PC, Moore DF, et al: Sur- Schröder FH, Roach M 3rd, Scardino P: Clinical
vival following primary androgen deprivation ther- decisions: Management of prostate cancer. N Engl J
apy among men with localized prostate cancer. 26 Med 359:2605-2609, 2008.
JAMA 300:173-181, 2008.
Stephenson AJ, Kattan MW, Eastham JA, et al:
Lu-Yao GL, Albertsen PC, Moore DF, et al: Out- Prostate cancer-specific mortality after radical
comes of localized prostate cancer following con- prostatectomy for patients treated in the prostate-
servative management. JAMA 302:1202-1209, specific antigen era. J Clin Oncol 27:4300-4305,
2009. 2009.

Messing EM, Manola J, Yao J, et al: Immediate ver- Thompson IM, Ankerst DP, Chi C, et al: Assessing
sus deferred androgen deprivation treatment in prostate cancer risk: Results from the Prostate Can-
patients with node-positive prostate cancer after cer Prevention Trial. J Natl Cancer Inst 98:529-534,
radical prostatectomy and pelvic lymphadenec- 2006.
tomy. Lancet Oncol 7:472-479, 2006.
Thompson I, Trasher JB, Aus G, et al: AUA Prostate
Mostaghel EA, Nelson PS: Intracrine androgen Cancer Clinical Guideline Update Panel: Guideline
metabolism in prostate cancer progression: Mecha- for the management of clinically localized prostate
nisms of castration resistance and therapeutic impli- cancer: 2007 update. J Urol 177:2106-2131, 2007.
cations. Best Pract Res Clin Enocrinol Metab
22:242-258, 2008. Tran C, Ouk S, Clegg NJ, et al: Development of a
second-generation antiandrogen for treatment of
Moul JW, Wu H, Sun L, et al: Early versus delayed advanced prostate cancer. Science 324:787-790,
hormonal therapy for prostate specific antigen only 2009.
recurrence of prostate cancer after radical prostatec-
tomy. J Urol 179(5s):S53-S59, 2008. Trock BJ, Han M, Freedland SJ, et al: Prostate can-
cer-specific survival following salvage radiotherapy
Ryan CJ, Smith MR, Fong L, et al: Phase I clinical vs observation in men with biochemical recurrence
trial of the CYP17 inhibitor abiraterone acetate after radical prostatectomy. JAMA 299:2760-2769,
demonstrating clinical activity in patients with cas- 2008.
tration-resistant prostate cancer who received prior
ketoconazole therapy. J Clin Oncol 28:1481-1488,
2010.

510 EDUCATIonAL REVIEW MAnUAL In URoLoGY


9. Abbreviations In This Chapter

Walsh PC, DeWeese TL, Eisenberger MA: Clinical ACS = American Cancer Society;
practice: Localized prostate cancer. N Engl J Med
357:2696-2705, 2007. ASCO = American Society of Clinical Oncology;

Wong YN, Mitra N, Hudes G, et al: Survival associ- ASTRO = American Society for Therapeutic Radi-
ated with treatment vs observation of localized ology and Oncology;
prostate cancer in elderly men. JAMA 296:2683-
2693, 2006. AUA = American Urological Association;

Zheng SL, Sun J, Wiklund F, et al: Cumulative asso- CALGB = Cancer and Leukemia Group B;
ciation of five genetic variants with prostate cancer.
N Engl J Med 358:910-919, 2008. ECOG = Eastern Cooperative Oncology Group;

EORTC = European

Organisation for Research and Treatment


of Cancer;

ERSPC = European Randomized Screening Study


for Prostate Cancer;

MRC = Medical Research Council;

MSKCC = Memorial Sloan-Kettering Cancer


Center;

NCCN = National Comprehensive Cancer


Network;

NCIC = National Cancer Institute of Canada;

PCPT = Prostate Cancer Prevention Trial;

PCWG = Prostate Cancer Working Group;

PLCO = Prostate, Lung,

Colorectal, and Ovarian Cancer Screening;

RTOG = Radiation Therapy Oncology Group;

SWOG = Southwest Oncology Group

CHAPTER 16: PRosTATE CAnCER 511


10. Tables

Table 1: 2010 TNM staging system of prostate cancer

Localized disease

Tx Primary tumor cannot be assessed

T0 no evidence of primary tumor

T1 Clinically inapparent tumor neither palpable nor visible by imaging

T1a Tumor incidental histologic finding in ≤ 5% of resected tissue

T1b Tumor incidental histologic finding in > 5% of resected tissue

T1c Tumor identified by needle biopsy (eg, because of elevated PsA level)

T2 Tumor confined within prostate

T2a Tumor involves one-half of one lobe or less

T2b Tumor involves more than one-half of one lobe but not both lobes

T2c Tumor involves both lobes

Local extension

T3a Extracapsular extension (unilateral or bilateral)

T3b Tumor invades seminal vesicle(s)

T4 Bladder invasion, fixed to pelvic side wall, or invasion of adjacent structures

Metastatic disease

n1 Positive regional lymph nodes

M1 Distant metastasis

From Edge SB, Byrd DR, Compton CC, et al (eds): AJCC Cancer Staging Manual, 7th ed. New York, Springer, 2010.

512 EDUCATIonAL REVIEW MAnUAL In URoLoGY


Table 2: TNM staging system of prostate cancer, 2010 updatesa

Anatomic Stage/Prognostic Groups

Group T N M PSA Gleason

stage I T1a-c n0 M0 PsA < 10 Gleason ≤ 6

T2a n0 M0 PsA < 10 Gleason ≤ 6

T1–2a n0 M0 PsA X Gleason X

stage IIA T1a–c n0 M0 PsA < 20 Gleason 7

T1a–c n0 M0 PsA ≥ 10 < 20 Gleason ≤ 6

T2a n0 M0 PsA < 20 Gleason ≤ 7

T2b n0 M0 PsA < 20 Gleason ≤ 7

T2b n0 M0 PsA X Gleason X

stage IIB T2c n0 M0 Any PsA Any Gleason

T1–2 n0 M0 PsA ≥ 20 Any Gleason

T1–2 n0 M0 Any PsA Gleason ≥ 8

stage III T3a-b n0 M0 Any PsA Any Gleason

stage IV T4 n0 M0 Any PsA Any Gleason

Any T n0 M0 Any PsA Any Gleason

Any T Any n M1 Any PsA Any Gleason

From Edge SB, Byrd DR, Compton CC, et al (eds): AJCC Cancer Staging Manual, 7th ed. New York, Springer, 2010.
aWhen either PSA or Gleason is not available, grouping should be determined by T stage and/or either PSA or Gleason
as available.

CHAPTER 16: PRosTATE CAnCER 513


Table 3: D’Amico et al risk stratification for clinically localized prostate cancer

Low risk Diagnostic PsA < 10.0 ng/mL and highest biopsy Gleason score ≤ 6 and clinical stage
T1c or T2a

Intermediate risk Diagnostic PsA ≥ 10 but < 20 ng/mL or highest biopsy Gleason score = 7 or clinical stage
T2b

High risk Diagnostic PsA ≥ 20 ng/mL or highest biopsy Gleason score ≥ 8 or clinical stage T2c/T3

PSA = prostate-specific antigen

Table 4: Risk of dying of clinically localized prostate cancer


without definitive locoregional therapy

Age

Gleason score 55–59 60–64 65–69 70–74

2–4 4% 5% 6% 7%

5 6% 8% 10% 11%

6 18% 23% 27% 30%

7 70% 62% 53% 42%

8–10 87% 81% 72% 60%

Adapted from Albertsen PC, Hanley JA, Fine J: JAMA 293:2095−2101, 2005.

514 EDUCATIonAL REVIEW MAnUAL In URoLoGY


Table 5: Hormonal approaches to the treatment of advanced prostate cancer

Method Mechanism of action side effects

surgical castration Removal of testicular androgens Hot flashes (50%), psychological


effects

Diethylstilbestrol Inhibition of gonadotropin Gynecomastia, cardiovascular risks


(DEs) secretion at high doses

LHRH analogs Inhibition of gonadotropin Hot flashes (50%), less gynecomastia


(agonists and antagonists) secretion than with DEs, fatigue
(goserelin, leuprolide,
degarelix)

Antiandrogens Blockade of binding of Abnormal liver function studies,


(bicalutamide, flutamide, dihydrotestosterone to its diarrhea (10%)
nilutamide) receptor

Ketoconazole Adrenal androgen synthesis LFT abnormalities, nausea, drug


inhibitor, possible autocrine/ interactions
paracrine androgen inhibition

Abiraterone acetate Adrenal androgen synthesis LFT abnormalities, fluid retention,


inhibitor, possible autocrine/ hypokalemia, hypertension, fatigue
paracrine androgen synthesis
inhibition

Glucocorticoids Inhibition of androgen receptor Weight gain, immunosuppression,


activity, adrenal androgen ulcers, bone density loss/fracture,
synthesis inhibition, independent hyperglycemia
cytotoxic effects

LHRH = luteinizing hormone-releasing hormone; LFT = liver function test

CHAPTER 16: PRosTATE CAnCER 515


Table 6: The incidence of complications from advanced prostate cancer with immediate vs
deferred androgen deprivation (M1 disease)

Complication Immediate Deferred

Pathologic fracture 2.3% 4.5%

spinal cord compression 2.0% 5.0%

Ureteral obstruction 7.0% 11.8%

Extraskeletal metastases 7.9% 11.8%

Table 7: Selected ongoing phase III trials in the postdocetaxel castration-resistant


prostate cancer setting

Trial Experimental agent Type of therapy Status

Palliative radiation ± Ipilimumab Immunomodulatory ongoing


ipilimumab (CTLA-4 blockade)

MDV3100 vs placebo MDV3100 novel antiandrogen ongoing

Prednisone ± TAK-700 TAK-700 Adrenal/autocrine ongoing


synthesis inhibitor

Prednisone ± sunitinib sunitinib oral VEGF/PDGF inhibitor negative

516 EDUCATIonAL REVIEW MAnUAL In URoLoGY


Table 8: Standard chemotherapy regimens for prostate cancer

Drug/combination Dose and schedule

Mitoxantrone/prednisone1 (Repeat cycle every 21 days)

Mitoxantrone 12 mg/m2 IV every 3 weeks

Prednisone 5 mg orally twice daily started on day 1

Cabazitaxel/prednisone 2

Cabazitaxel 25 mg/m2 IV every 3 weeks for up to 10 cycles

Prednisone 5 mg orally twice daily started on day 1

Docetaxel/prednisone 3

Docetaxel* 75 mg/m2 IV every 3 weeks

Prednisone 5 mg orally twice daily started on day 1

1 Tannock IF, Osoba D, Stockler MR, et al: J Clin Oncol 14:1756–1764, 1996.
2 de Bono JS, Oudard S, Ozguroglu M, et al: Lancet 376:1147-1154, 2010.
3 Tannock IF, de Wit R, Berry WR, et al: N Engl J Med 351:1502–1512, 2004.
* Note: Standard docetaxel and cabazitaxel premedications (including dexamethasone for docetaxel and dexamethasone with
antihistamines for cabazitaxel) should be given prior to the administration of docetaxel. GnRH agonist or other forms of castra-
tion are continued through chemotherapy. General antiemetic prophylaxis with dexamethasone should be routinely considered.
Table prepared by Ishmael Jaiyesimi, DO.

CHAPTER 16: PROSTATE CANCER 517


Table 9: Phase III docetaxel combination trials in
castration-resistant metastatic prostate cancer

Experimental
Trial agent Type of therapy Phase Status

Docetaxel/prednisone± Bevacizumab Antiangiogenesis III Negative


bevacizumab

Docetaxel/prednisone Aflibercept Antiangiogeneis III Completed


+ aflibercept (VEGF-trap)

Docetaxel/prednisone Atrasentan ET-A receptor III Negative


± atrasentan antagonist

Docetaxel ± custirsen Custirsen Anti-clusterin RNA III Ongoing


(OGX-011) inhibitor

Docetaxel/prednisone± Dasatinib Antimetastatic and III Completed,


dasatinib bonetargeting agent pending
(src kinase)

Docetaxel/prednisone± ZD4054 ET-A receptor III Completed,


ZD4054 (zibotentan) antagonist pending

Docetaxel/prednisone ± Lenalidomide Immunomodulatory III Ongoing


lenalidomide agent

518 EDUCATIONAL REVIEW MANUAL IN UROLOGY


Chapter 17:
Non-Muscle Invasive
Bladder Cancer
Michael A. O’Donnell, MD

Contents

1. Epidemiology 11. Surveillance Plan Based on


Risk Classification
2. Histology
12. Workup for Positive Cytology
3. Molecular Characterization
13. Treatment of Intravesical Therapy
4. Clinical Presentation and Initial Failures
Workup
14. Methods of Chemoprevention
5. Cystoscopic Evaluation and
Initial Resection 15. Recommended Reading

6. Role of Urinary Marker Tests in 16. Questions


Initial Diagnosis

7. Pathological Staging and Grading

8. Risk Factors Associated With


Recurrence and Progression

9. Intravesical Therapy

10. Indications for Immediate


Cystectomy for Apparent
Non-Muscle Invasive Disease

CHAPTER 17: NON-MUSCLE INVASIVE BLADDER CANCER 519


1. Epidemiology

* Quitting reduces risks but never down


to non-smoker
Basic Statistics

• Median age 70 years old but rare in women


<40 years old • Risk decreases to 2.7 after 1–3 years
and 1.1 after 15 years
• Male:female ratio 3:1
* Unclear if second-hand smoke is a contributor
• 73, 510 new cases (incidence) in 2012; prevalence
~ 9X incidence • Environmental/Occupational Exposure

• 14,880 deaths estimated for 2012 * Responsible for ~20% of cases

• 4th most common cancer in men; 12th most * First occupationally-identified cancer (Rehn
common in women 1895 in aniline dye industry)

• Ethnic predominance Caucasians > African Amer- * Exposure to chemicals in following indus-
icans > Hispanics > Asians BUT higher death rate tries: aluminum, dye, paint, petroleum, rub-
in women and in African Americans ber, textile, printing

* 5-year survival rates are 84% Caucasian * Higher risk professions: truck drivers, hair-
males, 71% African American men, 76% dressers, dry cleaners, paper manufacturers,
Caucasian women, 51% African American rope/twine makers, dental technicians,
women apparel manufacturers, plumbers

• Presentation: 75% non-muscle invasive; 20% • Highest relative risk were rubber workers
muscle-invasive; 5% metastatic and bus drivers, with RR 1.29

• Analgesic abuse: phenacetin (removed in 1987) –


NOT acetaminophen
Risk Factors

• Cigarette smoking
• Pelvic radiation: e.g., cervical/anal/prostate cancer
* Smoke is a potent source of urothelial
carcinogens, especially biphenylamines • Cyclophosphamide: 4- to 9-fold increased risk,
worse with chronic low-dose use
* Dose response relationship to development of
bladder cancer (especially >40 pack years) • Chronic cystitis: indwelling catheters,
bilharziasis (for squamous cell carcinoma >
• Risk increasing in linear fashion with urothelial carcinoma)
pack years, without a “risk plateau”
• Miscellaneous: arsenic, bracken fern, Balkan
* Increases hazard ratio to develop bladder nephropathy, Aristolochia fangchi (Chinese
cancer by 2-to-6-fold weight loss herb); associated with bilateral upper
tract cancers
* Responsible for ~60% males cases;
~30% female cases

* 15–20-year latency time

520 EDUCATIONAL REVIEW MANUAL IN UROLOGY


2. Histology

• 90%-95% transitional cell carcinoma (TCC) in


United States and Europe
Genetics

• Positive family history in 8% of cases


• ~5% squamous cell carcinoma (SCC) EXCEPT
• Relative risk increases ~2-fold with family history
but non-hereditary * Increased incidence in spinal cord injury
patients or those with chronic infection or for-
• Rare familial syndrome: (Lynch syndrome II) with eign bodies
defect in DNA mismatch repair
* Up to 80% of bladder tumors are SCC in
• No specific gene locus yet identified endemic regions of Schistosoma hematobium
infection (Egypt, Sudan, Saudi Arabia,
• Polymorphisms in detoxification mechanisms Africa)
contribute to higher susceptibility to environmen-
tal carcinogens • 0.5%–2% adenocarcinoma

* Smokers: Slow acetylators have 40% higher * Usually arise from urachus or trigonal region
risk than fast acetylators
* Especially prevalent in cases of prior bladder
* Glutathione S-transferase M1 null (~2-fold extrophy
increased risk; tobacco independent)
* Majority have associated history of long-term
* These polymorphisms are present in 27% inflammation or infection
of Caucasians, 15% of African Americans,
and 3% Asian males – may partially explain * Nonurachal adenocarcinomas of the bladder
ethnic differences in bladder cancer must be distinguished from extension of
colorectal primary, usually by colonoscopy

* Most common type associated with


ureterosigmoidostomy

* Signet cell variant associated with very


aggressive behavior

• Transitional cell carcinoma with mixed features


such as squamous or glandular differentiation

* Treatment same as for TCC

• Other variant cancer histology of clinical


significance (<1% incidence). Usually invasive
at time of presentation

* Small cell or neuroendocrine cancer


• May be pure or mixed with
high-grade TCC
• Stains positive with neuroendocrine
markers, such as neuron-specific enolase,
synaptophysin or chromogranin

CHAPTER 17: NON-MUSCLE INVASIVE BLADDER CANCER 521


• Unlike other forms of bladder cancer, • Benign tumors
primary treatment is systemic
chemotherapy (VP-16 [Etoposide] * Squamous metaplasia (non-keratinizing)—
plus cisplatinum or carboplatin) common in 50% females, 10% males

* Micropapillary cancer * Nephrogenic adenoma


• Thought to be a highly aggressive
variant of TCC • Metaplastic urothelial response to
• Aggressive surgery indicated for trauma, infection, radiation, or chronic
nonmetastatic disease immunosuppression
• Superficial forms resistant to BCG ther-
apy (primary cystectomy advocated) • Key histological feature – lined by
• Muscle-invasive and advanced cuboidal epithelium
forms relatively resistant to
systemic chemotherapy • Associated with hematuria, dysuria
and frequency
* Carcinosarcoma
• Very aggressive cancer with mixed • Treated by TURBT and conservative
epithelial and mesenchymal malignant cystoscopic follow-up
elements
* Cystitis cystica and follicularis—inflamma-
* Sarcomatoid carcinoma tory lesions of bladder
• Sometimes mistaken for carcinosarcoma
because of sarcoma-like appearance but * Pseudosarcoma—spindle cell tumor
proven by immunostaining to contain associated with prior surgery
only epithelial elements
* Malacoplakia—chronic reaction to ongoing
* Metastatic cancers—may include renal cell, urinary tract infection
ovarian, melanoma, etc

• Potential premalignant lesions

* Leukoplakia—keratinizing squamous
metaplasia; precursor for squamous cell
carcinoma ~20%

* Cystitis glandularis—glandular metaplasia of


von Brunn’s nests from chronic inflamma-
tion; possible precursor to adenocarcinoma.

* Inverted papilloma—not malignant but


associated with TCC elsewhere

* NOTE: mild-moderate dysplasia is most


often reactive in nature and not associated
with malignancy, while severe dysplasia is
synonymous with carcinoma in situ (CIS)

522 EDUCATIONAL REVIEW MANUAL IN UROLOGY


3. Molecular Characterization 4. Clinical Presentation and
Initial Workup

Transitional Cell Carcinoma Signs and Symptoms

• Most common chromosomal abnormality is loss of Hematuria


large segments of chromosome 9, especially 9q in • Found in 80%–90% of cases; (70%–80% gross
low-grade TCC hematuria)

• 2 proposed pathways: proliferative (loss of 9q) • Typically visible, intermittent, painless and pre-
for majority of PUNLMP and low-grade cancers sent throughout urination
(FGFR3 mutation) vs dysplastic (p53 mutation)
for high-grade cancers • Bladder cancer is the most common cause of gross
hematuria in patients >50 years old
• Frequent aneuploidy of chr 3, 5, 7, 17; (Note:
tetraploidy is not considered abnormal—found in • Often initially ignored by patients and physicians
umbrella cells) alike (especially female patients)

* UroVysion FISH test detect aneuploidy in • Differential diagnosis: UTI, calculi, renal cancer,
chr 3, 7, 17 and loss of 9p21 (CDKN2) glomerulonephritis, trauma, BPH, others

• Locus defects in p53, retinoblastoma (Rb), H-ras, * Consider clinical context (age, gender,
p16 (CDKN2), p21 (WAF1) and p27 (Kip1) risk factors) and details of hematuria

• Markers of aggressivity include: * All noninfectious gross hematuria of


unknown etiology requires urologic work-up
* HIGH p53, Ki-67 (MIB-1), matrix metallo- (upper tract study, cytology and cystoscopy)
proteinase (MMP) immunostaining
(Note:mutant p53 shows up as increased • Irritative voiding symptoms (dysuria, urgency)
staining due to reduced degradation)
* Found in 20%–30% of cases, ~50% without
* HIGH Urokinase-type plasminogen hematuria
activator (uPA) serum level
* Maybe misattributed to (interstitial) cystitis
* LOW Rb, E-cadherin, and p27 and prostatitis
immunostaining
* More common with high-grade aggressive
Squamous Cell Carcinoma disease, especially CIS and SCC

• Frequent chromosomal alterations of 9p • Special signs and symptoms

• Mutation of CDKN2 (p16) 3X more frequent * Mucosuria—passage of mucus sometimes


than in TCC seen with adenocarcinoma (also glandular
metaplasia)
Adenocarcinoma

• Poorly defined at present

CHAPTER 17: NON-MUSCLE INVASIVE BLADDER CANCER 523


* These include BTA-Trak, BTA-Stat,
NMP22, BladderChek, UroVysion FISH,
Physical Exam

• Rarely abnormal in early disease states and Immunocyt

• Conditions associated with advanced disease Imaging Studies

* Palpable pelvic mass and/or flank pain • Intravenous pyelogram (IVP) or CT urogram
should be performed prior to cystoscopy and
* Abnormal digital rectal exam may reveal biopsy (TUR) to evaluate for obstruction and pos-
extension into prostate sibility of upper tract TCC (2% in early disease; up
to 20% in advanced disease)
* Bloody urethral discharge or mass may indi-
cate urethral or prostatic involvement • Retrograde pyelograms sometimes necessary if
upper tract studies are inconclusive of elevated
* Inguinal and distant lymphadenopathy creatinine or dye allergy

* Weight loss and cachexia • Chest x-ray to rule out metastatic disease

• Exam under anesthesia (EUA) important clinical • Bone scan not routinely performed unless high
staging step clinical suspicion of bone metastasis

* Fixed mass after transurethral resection • Renal and/or bladder ultrasound not required in
(TUR) = clinical (c) T4 majority of cases

* Palpable mass after TUR = cT3 • MRI may substitute for CT scan for clinical
staging

* Caution: MRI with contrast risky in patients


Laboratory Evaluation

• Urinalysis with microscopic exam with elevated creatinine due to chance of


Nephrogenic Systemic Sclerosis
• Urine culture with sensitivity

• Urine cytology—especially helpful if indicative


of high-grade disease

* A positive cytology showing high-grade


dysplastic cells should be considered cancer
until proven otherwise

* A positive cytology showing clusters of


low-grade papillary clumps may be
inaccurate, due to instrumentation or stones

• Electrolytes, BUN and creatinine

• Newer FDA-approved, urine-based tests support


diagnosis but do not stand alone to make diagnosis

524 EDUCATIONAL REVIEW MANUAL IN UROLOGY


5. Cystoscopic Evaluation and
Initial Resection

• Cystoscopy is currently the gold standard for blad- • Understaging rate is up to 50% for high-grade
der cancer diagnosis stage T1 disease if detrusor muscle (muscularis
propria) is absent. CAUTION: wisps of muscle
* No substitutes are adequate including urine within the submucosa (muscularis mucosa) can be
marker tests or radiologic studies mistaken for true detrusor muscle

* Office flexible cystoscopy initially for most * Understaging rate is up to 50% for high-grade
cases of hematuria after upper tract studies stage T1 disease if detrusor muscle (muscu-
laris propria) is absent. CAUTION: wisps of
* If imaging or cytology highly suggestive of muscle within the submucosa (muscularis
bladder cancer, then office cystoscopy can be mucosa) can be mistaken for true detrusor
deferred in lieu of cystoscopy with biopsy or muscle
TUR under anesthesia
• Even with true detrusor muscle present and not
• Visual features of aggressive vs nonaggressive involved by cancer there is a 10%–20% understag-
lesions ing rate along with a 30%–50% chance of leaving
residual tumor behind in T1 lesions
* Small papillary lesions on a stalk are accu-
rately classified visually >90% of time as • Strongly recommended that all high-grade T1
low-grade, noninvasive bladder tumors lesions undergo a repeat TUR within 2–6 weeks of
initial resection (AUA recommendation)
* Large tumors with a broad base or nodular
configuration and areas of necrosis are more * Reduces understaging rate to <10% if
likely to be high-grade and invasive (at least confirms no muscle invasion
stage T1)
* Improves response to intravesical
* Red velvety flat lesions are typical of CIS immunotherapy with BCG and
especially in the setting of a highly suspicious intravesical chemotherapy with
or positive urine cytology mitomycin if done routinely

• TUR for complete removal of all suspicious * Provides prognostic information for later
lesions (AUA Standard) decision making
• If no disease or only stage Ta residual,
* Avoid excess cautery for accurate histological then progression rare
assessment • If no disease or only stage Ta residual,
then progression rare <15%
* Cold cup resection alone with fulguration of • If residual T1, then progression ~75%
base and periphery is appropriate for small by 2 years
papillary tumors
• Special circumstance—tumor in diverticulum
* Laser treatment (Holmium, YAG, KTP) is not * Ta disease may be safely removed with cau-
appropriate at first tumor setting but OK in tion if technically feasible
those with history of prior low-grade lesions * T1 disease, especially if high-grade, may be
understaged and easy to perforate; consider
* Detrusor muscle is required for all stage T1 partial cystectomy/diverticulectomy if iso-
lesions for accurate staging to rule out muscle lated and technically feasible
invasive disease (AUA Standard)

CHAPTER 17: NON-MUSCLE INVASIVE BLADDER CANCER 525


• Complications of TUR • Random biopsies

* Bleeding (early or delayed) * Not appropriate if mucosa is normal and


tumor appears papillary and noninvasive with
* Bladder perforation negative cytology

• Often associated with obturator reflex * Appropriate when cytology is positive yet no
(can be mitigated by neuromuscular tumor or tumors appear papillary and nonin-
blocking agents or local anesthetic block) vasive

* Need to distinguish intraperitoneal (often * Appropriate when visually aggressive tumors


dome) from extraperitoneal – consider intra- are present
operative cystogram
* Appropriate when contemplating partial cys-
• Most extraperitoneal perforations can be tectomy or other bladder-sparing approaches
managed conservatively to rule out coincident CIS

• Some intraperitoneal perforations can be


managed conservatively

* Small, not associated with infected


urine—conservative

* Large, associated with potential peri-


tonitis, or bowel injury – repair (can
be done laparoscopically)

* Important points in acute manage-


ment

• If small and late in TUR, can fin-


ish resection (switch from water
to glycine, reduce pressure).
Check for TUR syndrome

• If large, associated with bleeding


or early in TUR, back out and
return at another time

• If associated with bowel injury,


repair immediately

• Directed biopsies of suspicious lesions is always


appropriate

526 EDUCATIONAL REVIEW MANUAL IN UROLOGY


6. Role of Urinary Marker Tests 7. Pathological Staging
in Initial Diagnosis and Grading

• Available FDA-approved markers (Table 1) • TNM system for staging (Table 2)

* Fluid-based * Non-muscle-invasive groups include:


• Stage Ta—mucosally confined
• BTA-Trak (quantitative) and BTA-Stat® • Stage Tis (CIS)—carcinoma in situ
(qualitative—point of care) * Flat lesion confined to the mucosa
* Always high-grade
• NMP22 (quantitative) and BladderChekR * Thought to be precursor for invasive
(qualitative—point of care) disease
* Nonamenable to surgical resection
* Cell-based due to multifocal nature and ten-
dency to spread widely along sur-
• ImmunoCyt™—immune assisted cytol- face, sometimes invisibly
ogy * Same as high-grade dysplasia
• Stage T1—invasive into lamina propria
• FISH—fluorescent in situ hybridization (submucosa)
(aneuploidy of chromosomes 3, 7, 17; * Usefulness of further substaging to
loss of 9p21 [CDKN2] locus) early or deep invasion (T1a, T1b) is
still controversial due to lack of con-
• Cytology—current standard sistent landmarks and inconsistent
prognostic utility.
* All new marker tests suffer from reduced * Substaging does not predict response
specificity to intravesical therapy
* Associated lymphovascular invasion
• Cytology has a specificity of ~95%, (LVI) is a poor prognostic sign
meaning that a frankly positive cytology • Increased chance of under-
for high-grade dysplastic cells should be staging (up to 88% in one
assumed to indicate cancer until proven series)
otherwise • Increased chance of occult
lymph node metastasis
• False-positive tests are common with new • Decreased survival
markers • Advocated by some to justify
* Early cystectomy
* Urine-based assays less accurate * Neoadjuvant (upfront)
with hematuria, infection or inflam- chemotherapy prior to cys-
mation present (e.g., from stones) tectomy

* Some tests (FISH) pick up positives • Grading system of urothelial neoplasms


months before they are cystoscopi-
cally visible (so-called anticipatory * Concordance rates for grade between differ-
positive) ent pathologists only about 50%–60% using
the three-tier grade 1, 2, 3 system in effect
* All new marker tests have improved sensitivity since 1973
over cytology
* As a result, a new classification system with
• BUT still miss 20%–40% of cases more rigorous standards was proposed in
(more sensitive for higher grade cancers) 1998 by the World Health Organization
(WHO)/International Society of Urologic
• NONE completely rule out cancer Pathology (ISUP) (Figure 1)

CHAPTER 17: NON-MUSCLE INVASIVE BLADDER CANCER 527


* Urologic community is currently in a state of * New classification system results in greater
flux between acceptance of 1998 vs 1973 sys- number of high-grade lesions but retains
tems, in part due to long historical familiarity prognostic utility for progression (Table 3)
and prognostic utility of older 1973 system

Table 1

Comparative Aspects of FDA-approved Urinary Markers

Marker PoC Description Sn 1° Sn 2° Sp 1° Sp 2° Comment

Hb-dipstick X Urine hemoglobin 52 40 82 87 Often first screen;


dipstick nonspecific

Cytology Lab reference 48 35 94 94 Sensitive up to


standard 70% with grade 3

Cytometry Flow DNA 60 60 80 82 Equaled to


histogram for ploidy cytology in direct
comparison

BTA-Stat® X Human complement 70 58 75 73 False-positive with


factor-H related protein inflammation or
blood

BTA-Trak Same-quantitative 69 71 65 66 False-positive with


ELISA inflammation or
blood

BladderChek X Nuclear matrix 56 50 86 87 False-positive with


protein in mitotic inflammation
spindle

NMP22 Same-quantitative 73 71 80 73 False-positive


ELISA with inflammation

ImmunoCyt Panel of 3 monoclonal 83 67 80 75 Interobserver


Abs applied to same variability
cytology sample

FISH 4 probes for 84 79 95 70 Not affected


polysomy of Chr by BCG or inflamma-
3, 7, 17 or loss tion; anticipatory
of 9p21 positives

528 EDUCATIONAL REVIEW MANUAL IN UROLOGY


Table 2

TNM Staging System for Bladder Cancer

Mucosa Stage 0a Ta N0 M0
Stage 0is Tis

Submucosa Stage I T1 N0 M0

Muscularis Stage II T2a N0 M0


T2b

Adventitia Stage III T3a N0 M0


T3b

Prostatic invasion Stage IV T4a


Outside bladder T4b N0 M0
(pelvic sidewall)
Any T N1-N3 M0
Any T Any N M1

Figure 1

Relationship of 1973 WHO to 1998 WHO/ISUP

From Lerner SP, Konety BR, Skinner EC.


AUA 2006 Annual Meeting Course 60PG syllabus, Non-muscle invasive bladder cancer: risk-adapted treatment.

CHAPTER 17: NON-MUSCLE INVASIVE BLADDER CANCER 529


Table 3

Relationship of 1973 WHO and 1998 WHO/ISUP Grade to Progression

Progression
WHO Cases % 45 mo 90 mo
1973

Papilloma 5.2 0 0

G1 31.3 11 11

G2 59 11 24

G3 4.5 60 60

Progression
WHO Cases % 45 mo 90 mo
ISUP-1998

Papilloma 2.2 0 0

LMP 21.6 8 8

Low grade 54.5 10 13

High grade 21.6 23 51

From Samaratunga H, Epstein JL. Urology. 2002;60(2):315-319.

530 EDUCATIONAL REVIEW MANUAL IN UROLOGY


8. Risk Factors Associated with
Recurrence and Progression

• Grade • Risk Classification Scheme

* Recurrence at 2 years: papilloma (~20%) *Low risk (~50% of initial cases)


< PUNLMP (~40%) < LG TCC (60%–70%)
<HG TCC (~80%) • Low-grade (grade 1–2) solitary,
primary Ta papillary tumors
* Progression at 4/8 years: papilloma
(~0%/0%) < PUNLMP (~8%/8%) < LG TCC • Progression rate <5% at 5 years)
(10%/13%) < HG TCC (23%/51%)
* Intermediate risk (~35% of initial cases)
• Stage
• Recurrent Ta/T1 low-grade
* Recurrence—minor influence Ta vs T1
• Multifocal Ta/T1 low-grade
* Progression—significantly higher for T1
or CIS (5%–7% per year) • Progression rate <10% at 5 years

• Prior recurrence rate (primary, recurring ≤1 per * High-risk (~15% of initial cases)
year, recurring >1 per year)
• Any high-grade Ta or T1
* Primarily affects recurrence
• Any CIS
• Multifocality (single, 2–7 tumors, >8 tumors)
• Progression rate 25%-50% at 5 years
* Primarily affects recurrence (highest for multifocal T1G3 + CIS)

• Tumor size (<3 cm, >3 cm)

* Primarily affects recurrence

* Some affect on progression in multifocal


high-grade cases

• Concomitant CIS

* Primarily affects progression especially


for T1 disease

• Tumor presence at initial 3-month cystoscopy

* Primarily affects recurrence (strong predictor)

CHAPTER 17: NON-MUSCLE INVASIVE BLADDER CANCER 531


9. Intravesical Therapy

• Sufficient as sole therapy for low risk


solitary tumors
Treatment Options

• Single-dose immediate postoperative (post-TUR)


intravesical chemotherapy • Insufficient as sole therapy for intermedi-
ate or high-risk or multifocal tumors (sup-
* Appropriate agents: plement with additional chemotherapy or
immunotherapy)
• Mitomycin: bioreductive alkylating agent
• Multidose delayed intravesical chemotherapy
• Doxorubicin (adriamycin) or epirubicin
(rarely used in US): topoisomerase * Appropriate agents: mitomycin, doxorubicin
inhibitor and DNA intercalator (adriamycin), epirubicin, thiotepa. (Gemc-
itabine and docetaxel under investigation.)
• Thiotepa: alkylating agent (low molecu-
lar weight 189 permits absorption) * Appropriate setting: started 1–4 weeks after
TUR for intermediate risk patients or high-
* Inappropriate agents: BCG risk patients with contraindication to BCG
(DANGER: sepsis!), interferon (ineffective)
* Contraindications: known allergy or intoler-
* Appropriate setting: after resection of any sus- ance, concurrent infection
pected non-muscle-invasive papillary tumor, sin-
gle or multiple * Technique: instill drug for 1–2 hours weekly
for 6–8 weeks
* Contraindications: suspected bladder perforation,
excessive bleeding, known allergy * Optimization techniques for maximum bene-
fit of mitomycin ~ doubles disease-free rate
* Technique: instillation of drug for 30–60 minutes
dwell time; administered within 6 hours of TUR • Reduce urine output by relative
for maximum benefit (less effective after 24 dehydration
hours)
• Completely empty bladder prior to
* Toxicity: instillation

• <5% incidence of moderate-severe cysti- • Increase concentration (40-mg in


tis with appropriate use 20 cc water)

• With perforation: potential severe local • Alkalinize urine with oral bicarbonate to
tissue reaction or peritonitis (not thiotepa) reduce acid degradation (for mitomycin)
or transient myelosuppression (especially
thiotepa) * Maintenance chemotherapy (usually monthly)
recently supported for mitomycin by 2007 AUA
* Expected benefit (Class I medical evidence of meta-analysis
effectiveness)
* Toxicity:
• 12%–15% absolute reduction in recur-
rence rate out to 3+ years • Chemical cystitis (frequency, urgency, hema-
turia) in 10%–40% of patients
• 25%–50% relative reduction in recur-
rence * Mitomycin > adriamycin > epirubicin >
thiotepa

532 EDUCATIONAL REVIEW MANUAL IN UROLOGY


* 5%–10% delay or dropout due to • Recent gross hematuria (within 24 hours);
intolerance not microhematuria

• Drug-specific • Unresolved urinary tract infection

* Mitomycin: hypersensitivity geni- • Significant immunosuppression/HIV


tal/palmar skin rash (~10%), bladder con- (mild immunosuppression with inhaled or
tracture (~2%) low-dose steroids or methotrexate
allowed with close monitoring)
* Adriamycin: allergic reaction (~1%)
• Active autoimmune disease (rheumatoid
* Thiotepa: myelosuppression (~13%), arthritis, lupus, scleroderma, etc.) espe-
dose and frequency dependent; requires cially if on Remicade® (TNF receptor
weekly complete blood count monitoring antagonist)

* Expected benefit • Known allergy

• 12%–15% absolute reduction in recurrence • NOT reflux, remote history of treated


rate out to 3+ years TB, positive PPD, prosthetic devices
(small risk)
• Soft evidence that mitomycin is superior to
adriamycin and thiotepa * Technique: instill drug for 1–2 hours weekly
for 6 weeks
• Appropriate treatment for intermediate risk
papillary tumors with or without prior imme- * Maintenance therapy
diate postoperative dose (especially multifo-
cal cases) • Usually 3-week miniseries at 3 months, then
every 6 months for up to 3 years (SWOG pro-
* Limitations tocol)

• Failure of 1 chemo agent increases failure • At least 1 year of maintenance is necessary to


likelihood of another chemo agent achieve full benefit (superiority over
chemotherapy and reduction in risk of early
• Limited activity (not first-line) for CIS but progression)
appropriate if BCG contraindicated
* Expected benefit
• Ineffective against BCG failures as single
agent therapy • Average absolute 30% reduction in recurrence
rate from TURBT alone
• No evidence of effect on tumor progression
• Superior to chemotherapy (except perhaps in
• Multidose immunotherapy with BCG low-risk group)

* Appropriate setting: started 2–6 weeks after • Favored in higher risk groups by risk/benefit
TUR for intermediate or high-risk patients ratio

* Contraindications: • Treatment of choice for CIS and stage T1


high-grade disease
• Traumatic catheterization (bleed-
ing/pain/instrumentation) – ABORT!

CHAPTER 17: NON-MUSCLE INVASIVE BLADDER CANCER 533


• Shown to reduce progression risk by ~30% • Flu-like symptoms

• Recent study showed survival benefit vs * Fever, chills, myalgias, arthralgias


epirubicin (up to 25%)

* Mechanism of action: * High fever >39.5 C° or recurring


after 24 hours may be sign of serious
• Associated with T-helper type 1 immune BCG infection
response (increased IL-2 and interferon-
gamma) • Infection/inflammation related

• Opposed by T-helper type 2 response * Severe in ~ 5% total


(increased IL-4 and IL-10)
* Usually require 3–6 months of
• Likely involvement of neutrophils, T-cells anti-TB medication (BCG is resistant
and NK cells to most conventional antibiotics
including cephalosporins)
* Toxicity:
* Single agent isoniazid—INH (300 mg
• Cystitis (frequency, urgency, hematuria) daily) for moderate severity cases

* Common (>50% of patients) * Double or triple therapy with INH,


> chemotherapy rifampin (600 mg daily), and ethambutol
(1200 mg daily) for severe cases
* ~10% delay or dropout due to intolerance
* Vitamin B6 (pyridoxine) supplementa-
* Techniques to improve tolerance include: tion recommended

• Traditional symptom-relieving drugs * Cycloserine now recognized as


(anticholinergics, pyridium—try INEFFECTIVE
FIRST)
* Pyrazinamide (para-aminosalicylic
• BCG dose reduction acid) ineffective

• Spacing treatments out 2 weeks apart * Fluoroquinolones have activity as


third-line agents
• Decreasing dwell time to as little as
30 minutes • Oral or systemic steroids useful adjunctive to
diminish inflammatory side effects (fever, sweats)
• Adding quinolone antibiotic such as and/or hypersensitivity side effects (arthralgias,
ofloxacin 8 and 20 hours after BCG myalgias) in severe cases

• Isoniazid X 3 days will NOT reduce * Organ-specific manifestations


side effects or incidence of serious
BCG infection • Prostatitis (~40% asymptomatic,
<5% symptomatic)

* May result in rise in PSA or abnormal


DRE (resolves on own)

534 EDUCATIONAL REVIEW MANUAL IN UROLOGY


* Treat only if symptomatic (with anti-TB * Intermediate-risk group
meds)
• Single-dose immediate chemotherapy
• Epididymitis (<2%) after TUR

• Distant organs (<1%) • Additional 6-week course of chemother-


apy, especially if multifocal
* Lung, liver, kidney, bone
• 6-week course of BCG with maintenance
* Sometimes miliary pattern (lung) in place of chemotherapy or if prior fail-
ure of 6-week course of chemotherapy
• Sepsis (<0.5%)
* High-risk group
* Can be fatal
• Single-dose immediate chemotherapy
* Usually result of traumatic after TUR (for papillary tumor)
catheterization
• Additional 6-week course of BCG, plus
* Rapid onset with hypotension and cardio- maintenance for minimum of 1 year if
vascular collapse achieve disease-free status

* Early use of steroids (in addition to


antibiotics) can be life-saving by
blocking inflammatory cascade

* Hypersensitivity/allergy related (rare <5%)

• Rash (2%-5%)

• Reiter’s syndrome (arthritis, uveitis,


conjunctivitis)

• Polyarthritis

• Usually requires BCG discontinuation

• Intravesical treatment by risk group classification


(supported by both 2007 AUA and 2008 EAU
Guidelines Commissions)

* Low-risk group
• Single dose immediate (within 6 hours)
chemotherapy after TUR
• TUR alone and wait for 3-month
cystoscopy result (less optimal)

CHAPTER 17: NON-MUSCLE INVASIVE BLADDER CANCER 535


10. Indications for Immediate
Cystectomy for Apparent
Non-Muscle Invasive Dis-

* Recurrent non-stromal prostatic involvement


ease

• Strong indications with TCC

* Unfavorable variant histology • Impact of delay

• Nested or plasmacytoid variants * Patients developing muscle invasion while on


surveillance do no better and perhaps worse
• Micropapillary TCC than those presenting with de novo muscle
invasion
• Small cell carcinoma (upfront systemic
chemotherapy indicated before cystec- * High-risk disease presents 6 months after
tomy) BCG has higher risk for progression

• Squamous cell carcinoma (potential * Delay of cystectomy more than 2 years after
for partial cystectomy if limited to initial presentation for high-risk disease asso-
diverticulum) ciated with reduced cancer-specific survival

• Adenocarcinoma (often understaged)

* Lymphovascular invasion in stage T1


(associated with high risk of clinical
understaging; consider systemic
neoadjuvant chemotherapy)

* Bulky incompletely resectable, multifocal,


stage T1 high-grade

* Recurrence of stage T1 high-grade or CIS


after BCG failure X2

* Prostatic stromal invasion with TCC

* High-grade T1 disease in a bladder diverticu-


lum (often understaged)

* Dysfunctional bladder (bladder cripple) with


recurrent disease even if low-grade

• Relative indications

* Stage T1 high-grade still present after


repeat TUR

* Recurrence of stage T1 high-grade after BCG


failure X1

* Multifocal stage T1 high-grade with CIS at


primary presentation

536 EDUCATIONAL REVIEW MANUAL IN UROLOGY


11. Surveillance Plan Based on
Risk Classification

• Upper tract studies * Random biopsies if cytology becomes suspi-


cious or positive (required)
* None required for low risk if initial
work-up is negative • Ancillary urine markers (NMP22, FISH, BTA)

* Every 2 years for multifocal low-grade * In general, reduced sensitivity vs initial


or frequent recurrences in bladder diagnosis. May miss up to 50%

* Every 1–2 years for high-risk groups * Not a substitute for cystoscopy

* Whenever cytology becomes positive * False (and anticipatory) positive rate


without visible lesion in bladder is a problem

• Cystoscopy * Positive ancillary test by itself is NOT


an indication for treatment
* Low-risk:
* Positive ancillary test does increase risk
• If initial 3-month cystoscopy negative, of future recurrence and may be useful
safe to wait until 12 months for next in increasing rigor of surveillance
cystoscopy, then 1–2x per year
• Office-based treatment of recurrent tumors
• If no recurrence by 5 years, then chance
of future recurrence <5% * Laser or electrofulguration appropriate
for papillary noninvasive appearing
* Intermediate- and high-risk: tumors in patients with prior documented
low-intermediate risk disease
• Every 3 months for 2 years, then every (non–high-grade, noninvasive)
6 months for 2 years, then annually
* Delay in treatment of small papillary
• Cytology noninvasive appearing tumors until larger
in those with prior low-intermediate
* Low-risk: not necessary if initial cytology (non–high-grade noninvasive) disease is
is negative safe (average growth rate ~2 mm per month)

* Intermediate- and high-risk: recommended * Office biopsy if uncertain


at time of cystoscopy

• Biopsies

* Directed biopsies for any suspicious lesions


on cystoscopy

* Random biopsies optional initially to evaluate


treatment response for high-grade disease,
especially CIS

CHAPTER 17: NON-MUSCLE INVASIVE BLADDER CANCER 537


12. Workup for Positive Cytology

• Reliability * Check prostatic urethra

* 95% accurate if unequivocally positive for • Cold cup or TUR urethral strip biopsy
high-grade dysplastic cells (between 5:00–7:00 most useful)

* False-positives more common for papillary • Incidence >20% in setting of prior bladder
clusters, especially in presence of stones or CIS previously treated with BCG even with
instrumentation complete response (median time 1 year)

* Ileal loop or neobladder cytology accurate • Deeper tissue TURP helpful to rule out
if positive but must inform cytopathologist prostatic stromal invasion (submit superficial
that specimen from bowel segment to avoid and deep resections as separate specimens)
false-positive
• Transrectal ultrasound biopsy most useful if
• Localizing source of positive cytology palpable nodule on DRE

* Check bladder first • Lesser forms of prostatic TCC – CIS, ductal


or acinar TCC may respond to BCG with
• Biopsy any suspicious lesions or without formal TURP (not required
but recommended)
• Random biopsies even if no
visible lesions * GYN source should be checked in women with
negative work-up
* Check upper tracts
* Rare prostate cancer source (~3%) usually with
• Upper tract studies (IVP, CT urogram, Gleason score ≥7
MRI urogram, retrograde pyelogram)
* Failure to identify source of positive cytology
• Ureteral wash specimens with barbotaged despite above work-up will eventually yield
saline (done before retrograde) bladder source in ~ 80% of cases

• Ureteroscopy for radiographically * Positive cytology should generally be treated,


suspicious lesions or localizing cytology once localized, with BCG if otherwise appropriate
(not indicated for routine evaluation)

• Incidence of upper tract disease >20% in


setting of prior bladder CIS previously
treated with BCG even with complete
response (median time 5 years)

• Note: positive upper tract cytology unre-


liable in presence of visible bladder
tumors or biopsy positive CIS due to con-
tamination

• May treat with topical BCG via stents or


antegrade instillation

538 EDUCATIONAL REVIEW MANUAL IN UROLOGY


13. Treatment of Intravesical 14. Methods of
Therapy Failures Chemoprevention

• Intravesical chemotherapy failures will respond to


BCG similarly to previously untreated patients • Scant evidence that high-dose antioxidant
vitamins (A, B1, 2, 3, 5, 6, 12; C, D, E, folate, zinc)
• BCG failures will seldom respond to single agent can reduce recurrence of low-grade noninvasive
conventional intravesical chemotherapy papillary cancers after BCG treatment

• BCG failures X1 will often respond to a second * Recent randomized study did NOT find
course of BCG (30%–50% response) these vitamins helpful vs BCG alone for
BCG-naïve patients
• BCG failures X2 respond very poorly to another
course of BCG (<20% response) * Potential harm of high vitamin A
• Hepatotoxicity for doses approaching
• In general, patients with high-risk disease should 40,000 IU
be offered cystectomy after BCG failure X2 (AUA • Higher rates of lung cancer in current
guideline recommendation) smokers receiving vitamin A supplemen-
tation
• Patients refusing cystectomy or with medical con-
traindications to cystectomy or those with lower • No convincing evidence for DFMO or vitamin A
risk disease may be offered alternative therapy analogues, such as fenretinide

* Low-dose BCG plus interferon (40%–50% • Smoking cessation recommended—higher recur-


2-year disease-free status; best for BCG rence and progression rate for continued smokers
failures X1)
• Higher intake of fruit and vegetables associated
* Interferon alone with decreased risk
(<15% 2-year disease-free status)
• Higher intake of fat and cholesterol associated
* Investigational chemotherapy (gemcitabine, with increased risk
docetaxel, combination/sequential therapy)
• Higher intake of nonchlorinated water >6 cups/day
* Photodynamic therapy (rarely available, associated with decreased risk
bladder contractures common)
• Oral lactobacilli has been linked to improved
* Radiation therapy (not appropriate for CIS, response rates to intravesical chemotherapy
low-grade, or papillary disease; possibly
useful for select cases of stage T1 high-grade
disease)

* Other investigational agents ideally under


auspices of clinical trials

CHAPTER 17: NON-MUSCLE INVASIVE BLADDER CANCER 539


15. Recommended Reading

1. Kirkali Z, Chan T, Manoharan M, et al. 10. Collado A, Chéchile GE, Salvador J,


Bladder cancer: epidemiology, staging and Vicente J. Early complications of endo-
grading, and diagnosis. Urology. 2005;66 scopic treatment for superficial bladder
(suppl 1):4-34. tumors. J Urol. 2000;164:1529-1532.

2. García-Closas M, Malats N, Silverman D, 11. Oosterlinck W. Guidelines on diagnosis


et al. NAT2 slow acetylation, GSTM1 null and treatment of superficial bladder cancer.
genotype, and risk of bladder cancer: Minerva Urol Nefrol. 2004;56:65-72.
results from the Spanish Bladder Cancer
Study and meta-analyses. Lancet. 12. Bassi P, De Marco V, De Lisa A, et al. Non-
2005;366:649-59. invasive diagnostic tests for bladder can-
cer: a review of the literature. Urol Int.
3. Sørensen FB, Sasaki M, Fukuzawa S, 2005;75:193-200.
Yamabe H, Olsen S, Yoshida O. Qualita-
tive and quantitative histopathology in 13. van Rhijn BW, van der Poel HG, van der
transitional cell carcinomas of the urinary Kwast TH. Urine markers for bladder can-
bladder. An international investigation of cer surveillance: a systematic review. Eur
intra- and interobserver reproducibility. Urol. 2005;47:736-748
Lab Invest. 1994;70:242-254.
14. Platz CE, Cohen MB, Jones MP, Olson DB,
4. Eble JN, Young RH. Carcinoma of the uri- Lynch CF. Is microstaging of early inva-
nary bladder: a review of its diverse mor- sive cancer of the urinary bladder possible
phology. Semin Diagn Pathol. 1997;14:98- or useful? Mod Pathol. 1996;9:1035-1039.
108.
15. Smits G, Schaafsma E, Kiemeney L, Caris
5. Mitra AP, Datar RH, Cote RJ. Molecular C, Debruyne F, Witjes JA. Microstaging of
pathways in invasive bladder cancer: new pT1 transitional cell carcinoma of the blad-
insights into mechanisms, progression, and der: identification of subgroups with dis-
target identification. J Clin Oncol. tinct risks of progression. Urology. 1998;
2006;24:5552-5564. 52:1009-1013.

6. Lee R, Droller MJ. The natural history of 16. Lopez JI, Angulo JC. The prognostic sig-
bladder cancer. Implications for therapy. nificance of vascular invasion in stage T1
Urol Clin North Am. 2000;27:1-13. bladder cancer. Histopathology. 1995;27:
27-33.
7. Zhang J, Gerst S, Lefkowitz RA, Bach A.
Imaging of bladder cancer. Radiol Clin 17. Lee CT, Montie JE, Zhang YX, Dunn RL,
North Am. 2007;45:183-205. Wood DP. Lymphovascular invasion is an
independent predictor of survival in cT1
8. Jakse G, Algaba F, Malmström PU, Ooster- bladder cancer. J Urol. 2005;173 (suppl):
linck W. A second-look TUR in T1 transi- 911A.
tional cell carcinoma: why? Eur Urol.
2004;45:539-546. 18. Samaratunga H, Makarov DV, Epstein JI.
Comparison of WHO/ISUP and WHO
9. Golijanin D, Yossepowitch O, Beck SD, classification of noninvasive papillary
Sogani P, Dalbagni G. Carcinoma in a urothelial neoplasms for risk of progres-
bladder diverticulum: presentation and sion. Urology. 2002;60:315-319.
treatment outcome. J Urol. 2003;170:
1761-1764.

540 EDUCATIONAL REVIEW MANUAL IN UROLOGY


19. Millán-Rodríguez F, Chéchile-Toniolo G, 26. Au JL, Badalament RA,Wientjes MG, et
Salvador-Bayarri J, Palou J, Vicente- al. Methods to improve efficacy of intrav-
Rodríguez J. Multivariate analysis of the esical mitomycin C: results of a random-
prognostic factors of primary superficial ized phase III trial. J Natl Cancer Inst.
bladder cancer. J Urol. 2000;163:73-78. 2001;
93:597-604.
20. Sylvester RJ, van der Meijden AP, Ooster-
linck W, et al. Predicting recurrence and 27. Thrasher JB, Crawford ED. Complications
progression in individual patients with of intravesical chemotherapy. Urol Clin
stage Ta T1 bladder cancer using EORTC North Am. 1992;19:529-539.
risk tables: a combined analysis of 2596
patients from seven EORTC trials. Eur 28. Sylvester RJ, van der Meijden AP, Lamm
Urol. 2006;49:466-475. DL. Intravesical bacillus Calmette-Guerin
reduces the risk of progression in patients
21. Holmäng S, Hedelin H, Anderström C, with superficial bladder cancer: a meta-
Johansson SL. The relationship among analysis of the published results of ran-
multiple recurrences, progression and domized clinical trials. J Urol. 2002;168:
prognosis of patients with stages Ta and T1 1964-1970.
transitional cell cancer of the bladder fol-
lowed for at least 20 years. J Urol. 29. Böhle A, Jocham D, Bock PR. Intravesical
1995;153:1823-1827. bacillus Calmette-Guerin versus mito-
mycin C for superficial bladder cancer: a
22. Fitzpatrick JM, West AB, Butler MR, Lane formal meta-analysis of comparative stud-
V, O'Flynn JD. Superficial bladder tumors ies on recurrence and toxicity. J Urol.
(stage pTa, grades 1 and 2): the importance 2003;169:90-95.
of recurrence pattern following initial
resection. J Urol. 1986;135:920-922. 30. Sylvester RJ, van der Meijden AP, Witjes
JA, Kurth K. Bacillus calmette-guerin ver-
23. O’Donnell MA. Practical applications of sus chemotherapy for the intravesical treat-
intravesical chemotherapy and ment of patients with carcinoma in situ of
immunotherapy in high-risk patients with the bladder: a meta-analysis of the pub-
superficial bladder cancer. Urol Clin North lished results of randomized clinical trials.
Am. 2005; 32:121-131. J Urol. 2005;174:86-91.

24. Lamm DL, Riggs DR, Taynelis CL, Nseyo 31. Lamm DL, Blumenstein BA, Crissman JD,
UO. Apparent failure of current intravesi- et al. Maintenance bacillus Calmette-
cal chemotherapy prophylaxis to influence Guerin immunotherapy for recurrent Ta,
the long-term course of superficial transi- T1 and carcinoma in situ transitional cell
tional cell carcinoma of the bladder. J Urol. carcinoma of the bladder: a randomized
1995; 153:1444-1450. Southwest Oncology Group Study. J Urol.
2000;163:1124-1129.
25. Sylvester RJ, Oosterlinck W, van der Meij-
den AP. A single immediate postoperative 32. Saint F, Patard JJ, Maille P, et al. Prognos-
instillation of chemotherapy decreases the tic value of a T helper 1 urinary cytokine
risk of recurrence in patients with stage Ta response after intravesical bacillus Cal-
T1 bladder cancer: a meta-analysis of pub- mette-Guerin treatment for superficial
lished results of randomized clinical trials. bladder cancer. J Urol. 2002;167:364-367.
J Urol. 2004;171:2186-2190.

CHAPTER 17: NON-MUSCLE INVASIVE BLADDER CANCER 541


33. Martínez-Piñeiro JA, Martínez-Piñeiro L, 40. Stein JP. Indications for early cystectomy.
Solsona E, et al. Has a 3-fold decreased Urology. 2003;62:591-595.
dose of bacillus Calmette-Guerin the same
efficacy against recurrences and progres- 41. Nieder AM, Brausi M, Lamm D, et al.
sion of T1G3 and Tis bladder tumors than Management of stage T1 tumors of the
the standard dose? Results of a prospective bladder: International Consensus Panel.
randomized trial. J Urol. 2005;174:1242- Urology. 2005;66 (Suppl 1):108-125.
1247.
42. Herr HW, Badalament RA, Amato DA,
34. Andius P, Fehrling M, Holmäng S. Intrav- Laudone VP, Fair WR, Whitmore WF Jr.
esical bacillus Calmette-Guèrin therapy: Superficial bladder cancer treated with
experience with a reduced dwell-time in bacillus Calmette-Guerin: a multivariate
patients with pronounced side-effects. BJU analysis of factors affecting tumor progres-
Int. 2005;96:1290-1293. sion. J Urol. 1989;141:22-29.

35. Bassi P, Spinadin R, Carando R, Balta G, 43. Kamat AM, Gee JR, Dinney CP, et al. The
Pagano F. Modified induction course: a case for early cystectomy in the treatment
solution to side-effects? Eur Urol. 2000;37 of nonmuscle invasive micropapillary
suppl 1:31-32. bladder carcinoma. J Urol. 2006;175:881-
885.
36. Colombel M, Saint F, Chopin D, Malavaud
B, Nicolas L, Rischmann P. The effect of 44. Siefker-Radtke AO, Dinney CP, Abrahams
ofloxacin on bacillus calmette-guerin NA, et al. Evidence supporting preopera-
induced toxicity in patients with superficial tive chemotherapy for small cell carcinoma
bladder cancer: results of a randomized, of the bladder: a retrospective review of the
prospective, double-blind, placebo con- M. D. Anderson cancer experience. J Urol.
trolled, multicenter study. J Urol. 2004;172:481-484.
2006;176:935-939.
45. Morgan JD, Bowsher W, Griffiths DF,
37. Lamm DL, Steg A, Boccon-Gibod L, et al. Matthews PN. Rationalisation of follow-up
Complications of Bacillus Calmette- in patients with non-invasive bladder
Guerin immunotherapy: review of 2602 tumours. A preliminary report. Br J Urol.
patients and comparison of chemotherapy 1991; 67:158-161.
complications. Prog Clin Biol Res.
1989;310:335-355. 46. Bradford TJ, Montie JE, Hafez KS. The
role of imaging in the surveillance of uro-
38. BBabjuk M, Oosterlinck W, Sylvester R, et logic malignancies. Urol Clin North Am.
al. EAU guidelines on non-muscle-inva- 2006;33:377-396.
sive urothelial carcinoma of the bladder.
Eur Urol. 2008;54:303-314. 47. Donat SM, North A, Dalbagni G, Herr HW.
Efficacy of office fulguration for recurrent
39. Hall MC, Chang SS, Dalbagni G, et al. low grade papillary bladder tumors less
Guideline for the management of nonmus- than 0.5 cm. J Urol. 2004;171:636-639.
cle invasive bladder cancer (stages Ta, T1,
and Tis): 2007 update. J Urol. 2007; 48. Soloway MS, Bruck DS, Kim SS. Expec-
178:2314-2330. tant management of small, recurrent, non-
invasive papillary bladder tumors. J Urol.
2003,170:438-441.

542 EDUCATIONAL REVIEW MANUAL IN UROLOGY


49. Schwalb MD, Herr HW, Sogani PC, Russo 56. Lamm DL, Riggs DR, Shriver JS,
P, Sheinfeld J, Fair WR. Positive urinary vanGilder PF, Rach JF, DeHaven JI. Mega-
cytology following a complete response to dose vitamins in bladder cancer: a double-
intravesical bacillus Calmette-Guerin ther- blind clinical trial. J Urol. 1994;151:21-6.
apy: pattern of recurrence. J Urol. 1994;
152:382-387. 57. Nepple KG, Lightfoot AJ, Rosevear HM,
O’Donnell MA, Lamm DL; Bladder Can-
50. Lightfoot AJ, Rosevear HM, O'Donnell cer Genitourinary Oncology Study Group.
MA. Recognition and treatment of BCG Bacillus Calmette-Guérin with or without
failure in bladder cancer. Scientific World interferon α-2b and megadose versus rec-
Journal. 2011;11:602-613. ommended daily allowance vitamins dur-
ing induction and maintenance intravesi-
51. Catalona WJ, Hudson MA, Gillen DP, cal treatment of nonmuscle invasive blad-
Andriole GL, Ratliff TL. Risks and bene- der cancer. J Urol. 2010;184:1915-1919.
fits of repeated courses of intravesical
bacillus Calmette-Guerin therapy for 58. Moyad MA. Bladder cancer prevention.
superficial bladder cancer. J Urol. 1987; Part I: what do I tell my patients about
137:220-224. lifestyle changes and dietary supplements?
Curr Opin Urol. 2003;13:363-378.
52. Steinberg G, Bahnson R, Brosman S, Mid-
dleton R, Wajsman Z, Wehle M. Efficacy 59. Moyad MA. Bladder cancer recurrence:
and safety of valrubicin for the treatment of Part II. What do I tell my patients about
Bacillus Calmette-Guerin refractory carci- lifestyle changes and dietary supplements?
noma in situ of the bladder. The Valrubicin Curr Opin Urol. 2003;13:379-383.
Study Group. J Urol. 2000;163:761-767.
60. Naito S, Koga H, Yamaguchi A, et al. Pre-
53. Joudi FN, Smith BJ, O’Donnell MA, et al. vention of recurrence with epirubicin and
Final results from a national multicenter lactobacillus casei after transurethral resec-
phase II trial of combination bacillus tion of bladder cancer. J Urol. 2008;
Guérin plus interferon alpha-2B for reduc- 179:485-490.
ing recurrence of superficial bladder can-
cer. Urol Oncol. 2006;24:344-348.

54. Akcetin Z, Todorov J, Tüzel E, et al.


Radiochemotherapy after transurethral
resection is an effective treatment method
in T1G3 bladder cancer. Anticancer Res.
2005;25:1623-1628.

55. Fleshner N, Garland J, Moadel A, et al.


Influence of smoking status on the disease-
related outcomes of patients with tobacco-
associated superficial transitional cell car-
cinoma of the bladder. Cancer. 1999;86:
2337-2345.

CHAPTER 17: NON-MUSCLE INVASIVE BLADDER CANCER 543


16. Questions

1. A 43-year-old woman whose 55-year-old B. This is a variant form of transitional cell


brother was recently diagnosed with blad- carcinoma associated with a high proba-
der cancer seeks advice from you regarding bility of muscle invasion, lymph node
her own risk. Her history is notable for involvement or distant metastasis. Radi-
pelvic radiation 9 years earlier for cervical cal cystectomy is indicated with possi-
cancer. She has smoked 1.5 packs per day ble adjuvant chemotherapy depending
since age 20, drinks 4–5 cups of artificially on the findings.
sweetened coffee per day, and works as a
hair stylist specializing in colorization. She C. This is a variant form of bladder carci-
had frequent UTIs as a child. Which of the noma with a strong predisposition to
following would constitute legitimate risk metastasize and should be treated with
factors for the development of bladder can- cisplatin-based multiagent chemother-
cer in her case? apy.

A. Female gender, pelvic radiation, D. While not malignant, this lesion is


smoking strongly associated with recurrence. A
repeat BCG induction course with fur-
B. Pelvic irradiation, smoking, coffee ther maintenance therapy should be
consumption, prior UTIs started.

C. Family history, pelvic radiation, E. This is a benign inflammatory lesion.


smoking, artificial sweetener use Only routine periodic cystoscopy is
required.
D. Pelvic radiation, smoking, occupation
3. Which of the following immunohisto-
E. Female gender, family history, prior cer- chemistry profiles for a bladder tumor
vical cancer would be associated with the most aggres-
sive tendencies?
2. Two years after resection of a stage T1
high-grade lesion with glandular differenti- A. High p53 and Rb staining, low Ki67
ation from the right lateral wall and BCG and E-cadherin
therapy with maintenance X1 year, a suspi-
cious lesion is found in a similar location B. High p53 and Ki67 staining, absent
and resected. Histology reveals a nephro- Rb and E-cadherin
genic adenoma. Which of the following
statements best reflects the clinical signifi- C. High Rb and E-cadherin, low p53
cance and implication of this result? and Ki67

A. This is a premalignant lesion that is D. High Ki67 and Rb, low p53 and
strongly associated with the subsequent E-cadherin
development of adenocarcinoma. The
lesion should be re-resected then closely E. Low p53, Rb, E-cadherin and Ki-67
followed cystoscopically with periodic
biopsies.

544 EDUCATIONAL REVIEW MANUAL IN UROLOGY


4. A 65-year-old otherwise healthy man with C. Transurethral resection with bilateral
gross hematuria and negative CT urogram ureteral wash cytologies.
has a 1.5-cm lesion found on cystoscopy
that is subsequently resected to reveal stage D. Transurethral resection with administra-
T1, high-grade TCC with deep invasion of tion of BCG within the first few hours of
the submucosa but no involvement of the surgery provided there is no significant
scant muscle present. The most appropriate bleeding or perforation recognized.
next step would be:
E. Office biopsy of the lesion under local
A. Bring patient back for circumferential anesthesia to determine grade as a guide
biopsies around the tumor resection bed for future management.
as well as random bladder biopsies to
determine suitability for a partial cystec- 6. Which of the following is true regarding
tomy. papillary neoplasms of low malignant
potential (PUNLMP)?
B. Begin a course of BCG therapy 3 weeks
later with plans to cystoscope and possi- A. They encompass some formerly
bly biopsy 6 weeks after completing the described papillomas and grade 1 transi-
BCG therapy. tional cell carcinomas.

C. Strongly advise the patient to consider B. Re-resection required with deep biop-
radical cystectomy at this point given sies of tumor base and immediate instil-
the aggressive nature of the disease. lation of mitomycin.

D. Re-resect the patient within the next C. Begin a full course of BCG therapy 3
4 weeks to determine whether there weeks later with plans for subsequent
is residual disease or deeper disease maintenance therapy to reduce progres-
present. sion risk.

E. No further treatment at present. Use the D. Begin a 6-week course of mitomycin


results of the 3-month cystoscopy to with no definitive plans for maintenance
determine whether further treatment is at this time.
indicated.
E. Return to operating room for more com-
5. A 69-year-old otherwise healthy woman plete staging evaluation including ran-
has a 2.5-cm tumor identified on cys- dom bladder biopsies, upper tract
toscopy for work-up of recurrent painless washes and prostatic urethral biopsies.
gross hematuria. An IVP is negative and
voided cytology is negative for tumor cells.
The appropriate management plan would
be:

A. Transurethral resection under anesthesia


with random bladder biopsies.

B. Transurethral resection with administra-


tion of mitomycin within the first few
hours of surgery provided there is no
significant bleeding or perforation rec-
ognized.

CHAPTER 17: NON-MUSCLE INVASIVE BLADDER CANCER 545


7. Which of the following intravesical agents 9. A patient calls 6 hours after receiving his
is unlikely to cause local tissue inflamma- fourth scheduled treatment with BCG com-
tion or necrosis if administered in the pres- plaining of frequency, urgency, light red
ence of an unsuspected bladder perfora- urine without clots and a temperature of
tion? 38.8° C with some chills. The most appro-
priate response to this situation would
A. Thiotepa involve:

B. BCG A. Reassurance that these symptoms are


common with BCG treatment and that
C. Mitomycin they will most likely clear up on their
own.
D. Doxorubicin
B. Inform the patient that he should imme-
E. Epirubicin diately report to the emergency room for
evaluation and potential admission for
8. A 64-year-old man is diagnosed with pri- 24-hour observation.
mary stage Ta low-grade bladder cancer in
3 separate locations, the largest one of C. Call in prescriptions for an anticholiner-
which (3.5 cm) is suggestive of early lam- gic medication and 7-day course of
ina propria invasion. Detrusor muscle pre- ciprofloxacin to start immediately with
sent and not involved. He currently instructions to call back if temperature
requires Remicade for severe rheumatoid exceeds 39.5° C, relapses within next 48
arthritis. Because a small bladder perfora- hours or is associated with worsening
tion was noted at the time of his initial TUR symptoms.
he did not receive any immediate postoper-
ative intravesical chemotherapy. What D. Call in a prescription for isoniazid to
would be the best treatment now based on take for next 7 days and then for 3 days
risk/benefit considerations? preceding each subsequent BCG treat-
ment.
A. Observation only with cystoscopic
surveillance at 3 months. E. Admit to hospital for possible BCG
sepsis with treatment, including
B. Repeat resection of tumor site within the triple antituberculosis therapy
next 6 weeks. (isoniazid, rifampin, ethambutol)
and steroid therapy.
C. Initiate a 6-week course of intravesical
chemotherapy within a few weeks of
TUR.

D. Provide a single dose of perioperative


chemotherapy alone.

E. Start more aggressive BCG therapy


3–6 weeks from initial TUR

546 EDUCATIONAL REVIEW MANUAL IN UROLOGY


10. A 72-year-old woman undergoes TUR of a
4-cm bladder tumor with immediate instil-
Answers

lation of mitomycin C. Pathology report 1. D.


returns 5 days later, revealing poorly differ- Female gender is associated with a lower risk.
entiated transitional cell carcinoma invasive Coffee consumption, artificial sweetener use,
into the lamina propria with focal areas of occasional UTIs and prior cervical cancer are
micropapillary disease. Detrusor muscle is unrelated to risk. Besides pelvic radiation, smoking
present and not involved. Preoperative CT and occupational exposure to permanent hair dyes,
urogram was negative for any apparent dis- family history is the only other legitimate risk factor
ease outside the bladder. Treatment at this in this case.
point should involve:
2. E.
A. Re-resection of the tumor site, which, Nephrogenic adenomas are benign lesions associ-
if negative, should be followed ated with prior trauma, surgery, chronic inflamma-
expectantly with quarterly cystoscopy tion or infection. They do not predispose to further
and cytology. cancers. Answer A. would be appropriate for cysti-
tis glandularis, answer B. would be appropriate for
B. Re-resection of tumor site, which, if the micropapillary variant and answer C. would be
negative, should be followed by a full appropriate for the small cell variant of bladder can-
6-week course of BCG therapy with cer.
full cystoscopic restaging at 3 months.
3. B.
C. Re-resection of the tumor site, which, if High p53 staining is associated with p53 mutation, a
positive for residual T1 disease, should process that inhibits self-destructive apoptosis of
prompt an immediate cystectomy. cancer. Ki67 is a marker of proliferation. High val-
ues indicate a faster growth rate. Rb functions as a
D. Immediate cystectomy presuming cell cycle brake. Loss of Rb promotes cancer cell
remainder of metastatic work-up is proliferation. E-cadherin is a cell surface molecule
negative. that keeps cells fixed in place. Loss of E-cadherin is
associated with increased invasive and metastatic
E. Neoadjuvant systemic chemotherapy X potential. The least aggressive profile for compari-
3-4 cycles, then planned cystectomy. son would be answer C.

4. D.
Essentially all stage T1 high-grade cancers should
be re-resected because of the high chance of both
residual disease or unsuspected muscle invasion.
While BCG therapy is appropriate for completely
resected and accurately staged T1 high-grade can-
cer, if the tumor is understaged and actually T2 this
3-month delay (also found in answer E.) will be
potentially harmful. Radical cystectomy or partial
cystectomy is premature unless there are extenuat-
ing circumstances (multifocal or bulky T1, associ-
ated lymphovascular invasion for radical; tumor in
diverticulum for partial).

CHAPTER 17: NON-MUSCLE INVASIVE BLADDER CANCER 547


5. B. 9. C.
A single dose of intravesical chemotherapy deliv- It is unclear at this point whether the patient is hav-
ered within 6 hours of TUR will significantly ing a normal but exaggerated transient immune
decrease the risk of recurrence for low-risk bladder reaction to BCG, iatrogenic standard bacterial UTI
cancer at minimal risk to the patient. Choice D., giv- or early signs of serious BCG infection. Treatment
ing BCG, is contraindicated for high chance of of irritative voiding symptoms is appropriate but
inducing BCG sepsis. Neither random biopsies (A.) fever >38.5° C but <39.5° C should prompt addi-
nor ureteral cytologies (C.) are indicated in the set- tional treatment. Since fluoroquinolone antibiotics
ting of low-grade disease with negative voided are active against most standard bacterial UTIs and
cytology. An office bladder biopsy (E.) in this new BCG they are a reasonable first choice until the
bladder cancer patient adds nothing to the manage- patient declares himself through either higher fever
ment, since TUR remains the most definitive initial (≥39.5°), relapsing fever after 48 hours or worsen-
diagnostic and therapeutic maneuver. ing constitutional symptoms. At that point, the
patient should report for medical evaluation, possi-
6. A. ble in-hospital evaluation and institution of more
PUNLMPs now include some of the older classified specific antituberculosis drug therapy (B.).
papillomas and low-grade Ta cancers. They do recur Extended outpatient isoniazid monotherapy is
with moderate frequency but rarely progress. Re- appropriate for milder forms of BCG infection,
resection, use of BCG or mitomycin, and more although it will not prevent the onset of fever and
extensive restaging is simply not indicated. sepsis if administered for 3 days around the time of
BCG instillation (D.). Triple-drug antibiotic therapy
7. A. and steroids would be appropriate for BCG sepsis
Although it will cause transient myelosuppression, with hemodynamic compromise (E.).
as a non-vesicant agent, Thiotepa will not lead to a
severe local tissue reaction if extravasated. Mito- 10. D.
mycin and the anthracyclines, doxorubicin (adri- Micropapillary disease is poorly responsive to con-
amycin) and epirubicin, are all vesicant drugs with a servative measures (A.) or to BCG therapy (B.),
high potential for local tissue damage. BCG can resulting in reduced survival. Any delay in therapy
also cause a localized or systemic infection in such a (including re-resection) is potentially dangerous
circumstance. (C.). Neoadjuvant chemotherapy (E.) has not been
shown to be of clear benefit in clinically localized
8. C. disease for this variant form of bladder cancer.
This patient has intermediate-risk bladder cancer
based on both multifocality and size >3 cm. While
periodic surveillance is required, neither simple
observation (A.) nor single dose perioperative
chemotherapy (D.) is sufficient as recurrence risk
exceeds 60% with a modest progression rate of
5%–15%. A 6-week course of mitomycin (C.) will
reduce the relative chance of recurrence by over
30%. Although perhaps even more efficacious,
BCG (E.) is contraindicated in patients taking the
TNF receptor blocker Remicade because of the ele-
vated risk of BCG infection. Re-resection (B.) is not
unreasonable but not required in the setting of low-
grade disease, equivocal early lamina propria inva-
sion and clearly negative detrusor involvement.

548 EDUCATIONAL REVIEW MANUAL IN UROLOGY


Chapter 18:
Bladder Cancer
Seth P. Lerner, MD, FACS

Contents

1. Natural History and Staging

2. Treatment

3. Alternatives to Radical
Cystectomy

4. Integrated Treatment
Strategies

5. Bladder Cancer Metastases

6. Urinary Diversion

7. References

8. Questions

CHAPTER 18: BLADDER CANCER 549


1. Natural History and Staging

Bladder cancer is the 4th most common cancer in catheters and recurrent infection. Adenocarcino-
men and the 10th in women. Significant progress mas of the bladder are uncommon but occur in the
has been made in the management of invasive can- bladder base/trigone area and patients with exstro-
cer and we have learned much about multimodal phy are at higher risk. Adenocarcinomas that occur
therapy. The urologist plays a central role in inte- in the bladder dome most likely originate in the
grating surgery, chemotherapy and radiation ther- urachus. Surgical treatment is partial cystectomy
apy in order to achieve the goal of long-term cancer and the biology is similar to colorectal adenocarci-
control. This chapter will focus on the evaluation noma. Small-cell neuroendocrine cancer is a rare
and treatment of patients with muscle invasive and variant and may be the predominant histology
metastatic bladder cancer. Since urothelial carci- or mixed with urothelial carcinoma. This pheno-
noma is the most common histology, much of the type is easily distinguished byimmunohisto-
talk is focused on this entity. I will mention certain chemical detection of synaptophysin and chro-
aspects of the treatment of other histologies. I will mogranin. Treatment requires neoadjuvant
also cover the most pertinent aspects of urinary chemotherapy (cisplatin and etoposide) fol-
diversion. Specific references are provided with lowed by radical cystectomy or radiation ther-
some of the material in order to provide more in- apy. Small cell may also be associated with para-
depth review and a bibliography is provided at the neoplastic syndromes, including ectopic ACTH
end as well. production, hypercalcemia and hypophos-
phatemia.
Natural History
There have been other unusual variants reported,
The majority (80%) of patients with muscle-inva- including lymphoepithelioma-like cancers, which
sive cancer present de novo as their first manifesta- have a better prognosis than high-grade invasive
tion of bladder cancer. The remaining 15%–20% TCCs, though treatment is similar stage for stage.
progress from non–muscle-invasive cancer after The micropapillary variant resembles ovarian
treatment with intravesical therapy. Deaths due to papillary serous cancer and is very aggressive. It
bladder cancer invariably occur as a result of tends toward high grade, high stage and is fre-
distant metastases present at the time of locore- quently associated with lymphatic and vascular
gional therapy. Progression of cancer after defini- invasion.1 Radical cystectomy is the treatment of
tive locoregional therapy commonly occurs within choice even with non–muscle-invasive stage.
the first 2 years after treatment. Late recurrences are Whether this variant is less chemosensitive is
more common after perioperative systemic unclear, and consideration should be given to
chemotherapy and often occur in unusual sites, neoadjuvant chemotherapy similar to treatment for
including the central nervous system, bowel serosa the more common urothelial carcinoma histology.
and peritoneum. One must consider muscle-inva- Rhabdomyosarcoma is seen both in children and
sive bladder cancer as a systemic disease and design adults and leiomyosarcoma is seen in adults. Pri-
treatment strategies around the integration of treat- mary lymphoma of the bladder is rare but the blad-
ment targeting the locoregional disease (bladder and der may be involved in up to 10% of cases of sys-
pelvic lymph nodes) and occult visceral metastatic temic lymphoma.
disease according to pathologic risk factors.
Clinical and Pathologic Staging (Table 1)
Histology
The cT stage denotes the clinical stage, which is
Primary cancers of the bladder arising from the determined prior to definitive therapy. The depth of
urothelium and transitional cell (now referred to as invasion based on a combination of the TURBT
urothelial carcinoma) are by far the most common specimen and the bimanual exam under anesthesia
cell type. Squamous cell predominates in Middle determine the clinical stage. Pathologic stage (pT) is
Eastern countries where bilharziasis is endemic. It derived from the cystectomy specimen. Clinical
also occurs more frequently in women in Western understaging is more common than overstaging
countries and in the setting of chronic indwelling (which does occur). The 1997 TNM staging schema

550 EDUCATIONAL REVIEW MANUAL IN UROLOGY


introduced a modification from prior staging Staging Pitfalls
schemes. Prior to 1997, T2 was inner half of muscle
and T3a was outer half. European Association of Urology (EAU) and AUA
Guidelines require muscularis propria in the TURBT
T3b represented all levels of perivesical fat invasion. specimen for accurate staging. Without this, under-
In the current 2010 system, T2a and T2b represent staging of T1 disease is common and outcomes for
inner and outer half of muscle, respectively, and T3 T1 cancer are worse when there is no muscularis
is split into T3a (microscopic) and T3b (macro- propria in the specimen.
scopic, or visible on gross inspection of the cystec-
tomy specimen). T4a prostate requires established T1 vs. T2
stromal invasion, which can occur via the urethra, as There are fine muscle bundles called muscularis
direct extension via the bladder neck or posteriorly mucosa present within the lamina propria (Fig-
into the seminal vesicles or periprostatic tissue. T1 ure 3a). There are blood vessels in this area as well.
prostate is no longer considered T4a. CIS of prostatic These muscle bundles may be confused with the
urethra or ducts does not upstage, as outcome is muscularis propria, leading to overstaging of a T1
determined by the primary bladder cancer stage.2 tumor as T2. In addition, recent data suggest that up
Node staging has been changed in the 7th edition of to 40% of patients initially diagnosed as T1 who
American Joint Committee on Cancer (AJCC) staging undergo cystectomy are pathologic T2. The man-
manual as of 2010. Common iliac node metastases are agement of patients with high-grade T1 tumors is
no longer staged as M1 and are staged N3. challenging. It is imperative to accurately stage
these patients prior to embarking on intravesical
Prognosis following radical cystectomy is directly therapy. Re-resection for patients with T1G3 tumors
related to the depth of invasion and the pT stage. is the standard of care in order to determine the
Lymph node metastasis is the most significant appropriate therapy.
pathologic risk factor for progression. The N stage
in the revised TNM system reflects both the number Indications for initial cystectomy for T1 disease
of nodes involved and the anatomic extent of lymph include:
node metastases. (See Figures 1 and 2.)

Figure 1 Figure 2

Disease-Specific Survival According to pT Disease-Specific Survival According to pN


Stage at Cystectomy Stage at Cystectomy

CHAPTER 18: BLADDER CANCER 551


Figure 3b

Lamina propria and deep portions of the muscularis propria

Fat can be observed in lamina propria and deep portions of the muscularis propria.

Figure 3a Table 1

Muscularis mucosa of the lamina propria TNM Staging Bladder Cancer


(AJCC 7th Edition ’09)

T1 – lamina propria invasion

T2a – inner half muscularis propria

T2b – outer half muscularis propria

T3a – perivesical fat; microscopic

T3b – perivesical fat; macroscopic

T4a – invades prostatic stroma (T1 not included) , semi-


nal vesicles, vagina, uterus, or rectum; mobile, no fixa-
tion
The muscularis mucosa of the lamina propria can
be identified in up to 80% of bladder specimens T4b - pelvic sidewall,abdominal wall; fixed
and is recognized by fine muscle bundles and large
vessels.

552 EDUCATIONAL REVIEW MANUAL IN UROLOGY


• Size >3 cm 4. Determine the status of the bladder neck
(female) and prostatic urethra (male).
• Micropapillary histology
TURBT—Goals in Staging the Primary Tumor
• Lymphovascular invasion
• Establish histology
• Recurrent high-grade T1 cancer
• Assess for lymphatic/vascular invasion— a risk
T2 vs. T3 factor for metastases
Fat can be observed in the lamina propria and
deep portions of the muscularis propria and • Depth of penetration
should not be confused with perivesical fat inva-
sion (Figure 3b). Stage T3 requires cancer in fat • A complete TURBT is not required when cystec-
in the absence of muscle bundles. tomy is anticipated. A complete TURBT may be
beneficial in patients treated with radiation therapy
Staging the Bladder and Urethra
• Directed biopsies to establish status of mucosa
There are 4 pieces of critical information to obtain regarding CIS
in order to plan appropriate therapy:
• Stage bladder neck/urethra
1. Does the local tumor cause upper tract obstruc-
tion and is there a concomitant upper tract * In men, a negative prostatic urethra biopsy is
tumor which also needs treatment? associated with a negative apical urethra mar-
Hydronephrosis is invariably associated with gin, obviating the need for intraoperative
muscle invasion, unless there is a tumor within frozen section
the ureter obstructing the upper urinary tract.
From a clinical staging standpoint, • Biopsies of the prostatic urethra should be
hydronephrosis associated with a muscle-inva- done in all patients, prior to radical cystec-
sive cancer most often indicates clinical T3 tomy, as a staging tool for determination
stage. of prostatic involvement. Using a resecto-
scope, take a full loop from the mid-
2. What is the T-stage of the tumor assessed by prostate to the distal verumontanum at 5
bimanual exam under anesthesia? and 7 o’clock adjacent to the verumon-
tanum. This is where there is the highest
T2a — nonpalpable concentration of prostatic ducts and where
CIS is most likely to be found. The full
T2b — induration; no mass thickness chip allows the pathologist to
see the interface between the urethra
T3 — 3D mass, mobile mucosa, the prostatic ducts and stroma.
This method is the most accurate means of
T4a — invades adjacent structures staging the prostatic urethra

T4b — fixed; not mobile * Bladder neck biopsies are a surrogate for ure-
thra involvement in women when considering
3. Determine stage and depth of invasion with orthotopic diversion
TURBT and status of the remaining bladder
urothelium with directed biopsies.

CHAPTER 18: BLADDER CANCER 553


Goals of Additional Staging * Elevated liver function studies

• Further assess primary bladder tumor * Rationale: preoperative histologic confirma-


tion of pelvic nodal or liver metastases alters
* Local extent treatment

* Involvement of adjacent organs Optional


• Computed tomography chest (obtain with the fol-
• Assess regional and distant lymph nodes – com- lowing)
puted tomography or MRI
* Abnormal chest x-ray or history suggests lung
• Determine presence of visceral metastases metastases
directed to the most common sites: lung, liver and
bone – computed tomography, MRI, bone scintig- * T4 or N+ disease by clinical staging
raphy
• Bone scintigraphy
Incidence of Pelvic Node Metastases
* Elevated alkaline phosphatase; bone pain
The incidence of pelvic node metastases (%) is
directly related to the depth of invasion of the pri- * T4 or N+ disease by clinical staging
mary tumor and the presence of lymphovascular
invasion.3 • MRI

P1 P2a P2b P3 P4 * Contrast allergy or to resolve equivocal bone


5 series 2–7 8–14 19–24 26–46 36–50 scan

The most common sites of visceral metastases are • PET/CT—May identify patients with node metas-
the lung, liver and bone. Following chemotherapy, tases prior to cystectomy.4 However, there is no
unusual sites including CNS and the peritoneum defined role to date for routine staging for nodal or
may occur but are generally not a focus of staging at visceral metastases. This can be helpful in resolv-
initial presentation. ing nodal or visceral metastatic disease when these
findings will determine the use of chemotherapy or
Staging Evaluation— Regional and Metastatic surgery. False positives do occur so this should be
Disease used only in selected patients with a high index of
suspicion based on cross-sectional imaging of
Routine occult metastatic disease.
• Chest x-ray
Several commercially available serum biomarkers
• Laboratory (CA-125, CA-19.9) have been reported to be ele-
vated in patients with advanced disease and may
* Electrolytes, BUN, creatinine, CBC provide a marker that correlates with disease and
response to therapy, but this is not standard.
* Liver function tests, alkaline phosphatase

• Computed tomography (CT) abdomen and pelvis

* T3 and T4 primary tumors

554 EDUCATIONAL REVIEW MANUAL IN UROLOGY


2. Treatment

Goals An extended node dissection that starts at the ori-


The treatment strategy for invasive bladder cancer gin of the inferior mesenteric artery or the aortic
should achieve complete eradication of the locore- bifurcation, including the common iliac and pre-
gional disease, including the primary tumor and sacral lymph nodes, provides accurate staging and
regional lymph nodes. Treatment should also may increase the total lymph node count and
include occult regional node and visceral metastases improve survival. In patients with T3 or T4
when the risk of this occurring is high enough using tumors, the nodes proximal to the common iliac
neoadjuvant or adjuvant chemotherapy. Patients bifurcation are involved in up to one-half of
should have volitional control of urination via the patients with node metastases.7 There is no proven
retained urethra or by a continent catheterizable survival benefit to an extended node dissection and
stoma in properly selected patients. Treatment- Phase III trials are underway in order to address
related morbidity should be minimized wherever this important question.
possible.
Surgical margins
Extent of Surgery
Radical cystectomy provides excellent local control True positive soft tissue margins occur in approxi-
of the primary tumor. In men, it should include the mately 6% of patients and are an independent predic-
bladder, prostate and seminal vesicles. In women, it tor of decreased cancer-specific survival for patients
should include the bladder, ovaries, uterus/cervix with pT3 and pT4 tumors. Patients with residual inva-
and anterior vagina. In sexually active women, sive cancer with positive margins after neoadjuvant
vaginal preservation and or reconstruction must be chemotherapy have a particularly poor survival prob-
discussed preoperatively. The rate of involvement ability.
of the uterus, cervix and ovaries is uncommon.5
Preservation of the vagina and uterus provides bet- Cystectomy should not be performed when:
ter support for a neobladder and the pelvic floor.6 A
posterior-based invasive cancer in women is a rela- 1. Lymph node metastases are unresectable due to
tive contraindication to urethral preservation in bulk or proximal extent above the common iliac
women and mandates careful attention to the margin vessels
on the vagina and includes at least a small strip of
anterior vaginal wall in order to maintain a negative 2. There is evidence of extensive periureteral disease
surgical margin.
3. The bladder is fixed to the pelvic sidewall
Much is being written about selected use of prostate
and or seminal vesicle–sparing cystectomy. This 4. Tumor is invading the rectum
should be considered investigational and not standard
of care for any patients with invasive cancer. The Morbidity and Mortality
prostate is involved with urothelial cancer in at least
40% of patients undergoing radical cystectomy. Ade- The overall complication rate in 1,145 patients oper-
nocarcinoma of the prostate is also found in 40%, and ated between 1995 and 2005 based on a modified
up to one half of these are pathologically clinically
significant. Clavien system was reported by surgeons from
Memorial Sloan-Kettering Cancer Center as 64%.9,10
A standard bilateral pelvic lymphadenectomy, The major complication rate is estimated at
including the external and internal iliac and obtu- 15%–25%, with gastrointestinal and genitourinary
rator lymph nodes, is required for all patients tract the most common problems. Prolonged ileums,
undergoing radical or partial cystectomy. The true bowel obstruction, urine leaks and urinary tract
perivesical nodes are embedded in the perivesical fat infections are common problems. Metabolic prob-
and can also harbor metastatic disease. lems and dehydration occur more frequently in the
elderly and are a common reason for readmission in
the early postoperative period. Most of these compli-

CHAPTER 18: BLADDER CANCER 555


Figure 4

Mapping study of location of transitional cell carcinoma in the female bladder (n = 36). All patients with cancer in the
urethra also had cancer at the bladder neck.

cations can be managed conservatively. Mortality higher positive margin rate for patients with pT4 dis-
rates dropped dramatically with improved antibi- ease compared to open surgery.12,13 In a small series,
otics, better pulmonary and cardiac care, and modern Nix et al found comparable node yields between
anesthesia. The mortality risk in contemporary cys- open and robotic, but the median node count was sig-
tectomy series is 1.6%–3.3% and is based predomi- nificantly lower than reported from centers perform-
nantly on the risk of postoperative pulmonary embo- ing extended pelvic and iliac lymphadenectomy.14
lus, myocardial infarction and sepsis.11 There are no long-term oncologic outcome data in
sufficient patients from randomized clinical trials to
Laparoscopic and Robotic-assisted substantiate the durable efficacy of laparoscopic or
Radical Cystectomy robotic-assisted radical cystectomy.15

Minimally invasive surgery has the potential to Management of the Urethra


reduce the morbidity and recovery time for radical
cystectomy. The technical hurdles can be overcome The risk of urethral TCC following cystectomy is
in expert hands. The challenge is to replicate the influenced by the presence and extent of involve-
pathologic and oncology outcomes of open radical ment of the prostatic urethra and prostatic
cystectomy and to consistently perform a thorough stroma, with stromal invasion carrying a risk of
anatomic pelvic lymphadenectomy. Using soft tissue approximately 30% (none < focal CIS < diffuse
margin status as a quality measure, a comparison of CIS < ductal/acinar involvement < stromal inva-
data from an international robotic surgery consor- sion). Men should undergo urethrectomy if CIS
tium and a multicenter study of open surgery indi- is diffuse within the prostatic urethra or ducts or
cate that robotic surgery was associated with similar if there is prostatic stroma invasion. Some feel
outcomes for patients with pT1-3 disease but a that the only indication is cancer at the apical ure-

556 EDUCATIONAL REVIEW MANUAL IN UROLOGY


thra margin. Small low-grade papillary tumors of • Cancer recurrence at the anastamosis is rare even
the urethra can be resected prior to cystectomy and with a positive margin
the urethra retained if there is no other indication for
urethrectomy. The probability of developing a • Nephrectomy is not indicated for CIS of the ureter
second primary TCC of the retained urethra is
lower with orthotopic diversion compared to • CIS of the ureter is not independently associated
cutaneous diversion.16 with a worse outcome

Cystectomy in women historically included the ure- • Incidence of upper tract tumors after orthotopic
thra. New data indicate that the distal two-thirds of neobladder is 2.4%–17%20
the urethra serves as an adequate sphincter mecha-
nism innervated by the pudendal nerve and is infre- • Monitor with IVP or CT and voided cytology if
quently involved with transitional cell cancer.17,18 CIS of lower tract or ureter
Limiting dissection to above the endopelvic fascia
will preserve the innervation. Cancer at the blad- Outcome
der neck or urethra or a T4 tumor involving the
anterior vagina are contraindications to urethra Survival following radical cystectomy is driven by
preservation and orthotopic neobladder. Urethra pathologic tumor stage and the status of the pelvic
cancer is always associated with bladder neck can- lymph nodes (Table 2).21 Within each stage cate-
cer, so biopsy findings of the bladder neck are an gory, positive nodes have a negative impact on sur-
excellent surrogate for preoperative staging of the vival (Table 3).
female urethra (Figure 4).
The combination of P3 or P4 and positive lymph
Nerve-sparing Cystectomy nodes is associated with a particularly bad progno-
sis and is an indication for adjuvant chemotherapy.
Nerve-sparing cystectomy is appropriate in patients
who are potent, sexually active and have clinically Prostatic Involvement
organ-confined cancer and no evidence of local
extension intraoperatively. There is no negative CIS involving the prostatic urethra, ducts or
impact on local recurrence and approximately 40% acini (no stromal invasion) does not adversely
can achieve erections satisfactory for intercourse. affect prognosis following cystectomy, as the out-
The results are age-dependent, similar to that come is driven by the primary stage of the blad-
observed for nerve-sparing radical prostatectomy. der tumor.2 As mentioned, prostatic CIS and lamina
Nerve-sparing may improve continence in patients propria invasion of the superficial prostatic stroma
with a neobladder but this may also simply reflect (AJCC 7th edition, 2009) do not change the stage,
better dissection at the apex of the prostate and which is determined by the stage of the bladder
preservation of the urethra sphincter complex.19 tumor.

Carcinoma In Situ (CIS) of the Ureter Prostatic stromal invasion occurs via prostatic
• Atypia, dysplasia does not require any action urethra (noncontiguous) or as direct invasion (con-
tiguous) from the primary bladder tumor via the
• Attempt should be made to achieve a negative bladder neck or posteriorly via penetration of the
margin on the ureter, but do not compromise on periprostatic tissues and/or seminal vesicles. This is
ureteral length true T4 disease and carries an adverse prognosis.
Some studies suggest that direct invasion from the
• Incidence of upper tract tumors after orthotopic primary bladder tumor is associated with a worse
neobladder is 2.4%–17%20 prognosis compared to invasion via noncontiguous
involvement of the prostatic urethra.22 There is a cat-
• Monitor with IVP or CT and voided cytology if egory of lamina propria invasion of the prostate that
CIS of lower tract or ureter

CHAPTER 18: BLADDER CANCER 557


Table 2

Survival Following Radical Cystectomy According to


Pathologic Tumor Stage and Node Status

Stage % with Positive Nodes 5-year Survival: Node Negative 5-year Survival: Node Positive

pT2 6%-31% 64%-83% 34%-66%

pT3 27%-64% 17%-46% 11%-27%

pT4 45%-59% 0%-33% 11%-27%

Lerner SP, Skinner DG. Radical cystectomy for bladder cancer. In: Comprehensive Textbook of Genitourinary Oncology. 2nd
ed. Philadelphia, PA: Lippincott Williams & Wilkins; 2000:425-447.

Table 3

Survival and Recurrence-Free Probability According to Number of Positive Lymph Nodes (LN)

No. of Pos. N Recurrence-Free Probability Survival

LN 5y ± SE 5y ± SE

0 127 76 ± 4.8% 68 ± 5%

>1 14 43 ± 11% 51 ± 11%

>5 10 40 ± 10% 42 ± 17%

>6 8 13 ± 12% 33 ± 18%

>10 5 20 ± 18% 20 ± 18%

Reprinted from European Urology, Volume 48, Issue 2, August 2005, pp 202-206, Vale CL. Neoadjuvant Chemotherapy in
Invasive Bladder Cancer: Update of a Systematic Review and Meta-Analysis of Individual Patient Data: Advanced Bladder Can-
cer (ABC) Meta-analysis Collaboration. With permission from Elsevier.

is contiguous with prostatic CIS that is included in • Number of positive lymph nodes
T4 stage but carries a prognosis similar to prostatic
CIS only.23 When these patients are excluded from • Percent of nodes involved with cancer
T4 the difference in outcome between invasive phe-
notypes disappears. • Pathologic stage of the primary tumor

All of the following may impact survival: • Extranodal extension


• Number of lymph nodes removed
In node-negative patients, an inadequate node dis-
* Important in both node-negative and node- section will likely miss node metastases, leading to
positive patients understaging of the cancer. In node-positive

558 EDUCATIONAL REVIEW MANUAL IN UROLOGY


patients, the number of positive nodes has a direct Gross hematuria may occur as a result of pelvic
impact on prognosis. A recent term of “lymph node recurrence invading a neobladder. The most com-
density” has been introduced that reflects the num- mon distant sites are lung, liver and bone, and a
ber of positive nodes divided by the total number of chest x-ray; lab work directed to bone marrow, liver
nodes removed.24 The pathologic stage also influ- function and bone metastases (alkaline phos-
ences the outcome in node-positive patients. Extra- phatase) should be performed at each visit. There
nodal extension of cancer is a poor prognostic fea- are no data to support or refute a rationale for CT or
ture. MRI, though many perform these at annual follow-
up visits for the first few years. Central nervous
Impact of Age system metastases may occur late (after 3 years)
• Advance age (≥80) is associated with higher inci- in patients who have received chemotherapy.
dence of stage pT3/T425 Bowel obstruction may also occur late as a manifes-
tation of intraperitoneal metastases.
• Age is an independent risk factor for cancer-spe-
cific survival after adjusting for the effects of Urethra recurrences are new tumors—so-called sec-
pathologic grade, pathologic stage, lymphovascu- ond primary tumors. Urethral wash cytology is sen-
lar invasion and lymph node metastases26 sitive for identifying urethra TCC and should be
performed at least annually in patients with any CIS
• Higher recurrence rates may be due to lower use of in the cystectomy specimen (or precystectomy biop-
perioperative chemotherapy in elderly patients sies). Upper tract tumors are uncommon following
cystectomy, though the risk can be as high as 25% at
Outcomes in Women 10 years in patients with CIS.29 In these patients, an
IVP or CT with intravenous contrast and urine
• Women are more likely to be diagnosed with cytology should be performed annually. Predictors
advanced stage disease compared to men in cystectomy patients also include urethra TCC and
patients with a positive ureter margin should be
• Women may have a longer interval from symp- monitored as well.
toms to diagnosis than men
Indications for Secondary Urethrectomy
• Women have a 30% (African American) to 50%
(Caucasian) higher risk of death compared to • Positive urethral wash cytology and biopsy confir-
men27 mation of urethral cancer

• Women with muscle-invasive cancer have a lower • Bloody urethral discharge mandates urethroscopy
long-term survival probability compared to men and biopsy
(median survival 6.8 years vs 8.3 years for men)28
* If cancer confirmed, usually invasive and dif-
Monitoring for Recurrence ficult to cure

Monitoring is focused on early detection of asymp- • Urethrectomy should include urethral meatus
tomatic recurrence and imaging directed at abnor-
malities suggested in the history and physical exam. * Local therapies, including 5FU, are generally
Most recurrences occur within the first 3 years ineffective
after cystectomy in patients who have not been
treated with systemic chemotherapy. Local pelvic
recurrence may be detectable by an abnormal DRE
or bimanual. Patient with an orthotopic diversion
may present with urinary retention as the first indi-
cation of local pelvic recurrence. Sciatic pain sug-
gests local recurrence or pelvic bone metastases.

CHAPTER 18: BLADDER CANCER 559


3. Alternatives to Radical
Cystectomy

Radical TURBT standard, however, is radical cystectomy in patients


• Initial occurrence of bladder cancer who experience a PR).

• No CIS Several recent studies suggest that partial cystec-


tomy is overutilized, particularly in non-academic
• Size ≤3 cm settings.32,33 Long-term cancer control can be
achieved in a majority of patients when strict crite-
• T2 (no palpable mass)—T3 tumors are not ria are applied to patient selection.34-36 Preoperative
amenable to complete TUR and T4 tumors are not radiotherapy and/or intraoperative intravesical
eligible, as by definition they are invading sur- chemotherapy have been reported in many series in
rounding structures an effort to minimize the risk of wound implanta-
tion, but there is no evidence to support their routine
• Not in dome or high posterior wall due to risk of use. Life-long follow up is required in order to man-
bowel injury age the risk of recurrence of both non–muscle-inva-
sive and invasive bladder cancer.
• Re-resect at 3 months to assure complete
removal of the tumor. If tumor persists at this In the unique situation of urachal adenocarci-
time, then radical cystectomy or radiation should noma, partial cystectomy is the preferred surgi-
be considered cal method for local control. This should be done
using a closed technique in order to avoid tumor
• Disadvantages spill of this highly implantable tumor, with en bloc
resection of posterior rectus sheath, urachus and
* No pathologic node staging dome of bladder.

* Lifelong surveillance—up to 25% require External Beam Radiotherapy


cystectomy External beam irradiation has a very long track
record in the treatment of patients with invasive
When these criteria are followed in highly selected urothelial carcinoma of the bladder. In the United
patients, the long-term survival is comparable to States, it has generally been used for patients who
radical cystectomy.30,31 are not surgical candidates. In Europe, it has
enjoyed a strong role for decades as a primary treat-
Partial Cystectomy ment option. As monotherapy, it is best used for
The indications for partial cystectomy are similar to patients with significant pulmonary or coronary
radical TURBT with one exception – the location. artery disease for whom the risks of surgery are pro-
Tumors should be in a location suitable to blad- hibitive and for whom multiagent cytotoxic
der preservation— usually on the dome and chemotherapy is not an option. The doses range
away from the ureteral orifices. Ureter reimplan- from 55-65Gy delivered in daily fractionation. Poor
tation may be appropriate when partial cystectomy prognostic features include the presence of
is done for a tumor in a bladder diverticulum. Bilat- hydronephrosis, locally advanced tumors (stages
eral pelvic lymphadenectomy should be performed T3b and T4) and patients who are anemic.
at the time of surgery for pathologic staging of the
nodes. Long-term relapse rates are high despite The 5-year survival probabilities are influenced by
these strict criteria. This option has a definite role in the clinical stage of the tumor (5-year survival: T2-
patients who have had an objective response to 26 = 59%; T3-10 = 52%). Patients with bulkier
neoadjuvant chemotherapy. The tumor site can be tumors do not fare as well and maximal debulking
resected in patients with a complete response (CR) of the primary tumor with TURBT should be per-
in order to pathologically assure complete tumor formed when feasible. CIS is resistant to radiation,
removal. This is also possible in patients with a par- and may be present at the time of initial diagnosis or
tial response to chemotherapy and where the tumor develop subsequent to radiation, and so the bladder
location is amenable to partial cystectomy (the gold must be monitored throughout the rest of the patient’s

560 EDUCATIONAL REVIEW MANUAL IN UROLOGY


4. Integrated Treatment
Strategies

lifetime. Patients may require additional intravesical Radical Cystectomy and Preoperative Radiation
therapy to control CIS. Today, radiation is rarely • 6 randomized trials: TCC (4); Bilharzial SCCa (2)
given as monotherapy. At a minimum, a radiosensi-
tizing chemotherapy drug should be administered • Meta-analysis
along with the radiation. The most commonly used
drugs are cisplatin, 5-FU and docetaxel, and it is * Odds ratio 0.94 (95% CI 0.57–1.55)
estimated that response rates to radiation are
increased by 10%–15%. * Potential survival benefit and better local con-
trol with bilharzial cancer

* No role in contemporary radical cystectomy


for TCC

• No role for postoperative radiation—toxicity too


high

• In the case of documented local recurrence, pelvic


irradiation can be performed safely

Neoadjuvant Chemotherapy
Rationale. Neoadjuvant chemotherapy is utilized to
treat micrometastatic disease present at the time of
cystectomy in order to reduce the risk of, or delay
the emergence of, measurable metastatic disease
and improve long-term survival. The benefit is the
primary tumor is in place and response to
chemotherapy can be objectively assessed. It is pos-
sible that an otherwise unresectable tumor (T4b or
M1 nodes—proximal to the common iliac bifurca-
tion) can be downstaged and become resectable. The
disadvantages are related to errors in clinical stag-
ing; the potential for overtreating patients who
would otherwise be cured with surgery alone and the
potential that ineffective chemotherapy may delay
definitive local therapy.

There have been several randomized trials of neoad-


juvant chemotherapy where chemotherapy followed
by cystectomy or radiation have been compared to
surgery or radiation alone (Table 4). The eligibility
criteria for these trials included patients with T2-T4
tumors and N+ disease. Both the MRC/EORTC and
the SWOG studies showed improved survival for
the patients who received neoadjuvant chemother-
apy.37,38 In the SWOG neoadjuvant chemotherapy
trial, patients received 3 cycles of M-VAC followed
by cystectomy or cystectomy alone.38 There was an
approximate 2.5-year improvement in median
survival and the 5-year survival probability was
improved by 15%.

CHAPTER 18: BLADDER CANCER 561


Figure 5

Benefit of Neoadjuvant Multi-Agent Regimens vs. Non-Benefit of Single-Agent Therapy

Cisplatin alone

Cisplatin combo

• 5% overall survival advantage with neoadjuvant chemotherapy


• 9% survival benefit with cisplatin-based combination chemotherapy

Vale, et al Eur Urol 48:202, 2005

562 EDUCATIONAL REVIEW MANUAL IN UROLOGY


Table 4

Neoadjuvant Chemotherapy Randomized Trials

Benefit?

Regimen Progression Survival

CUETO Cisplatin No No

NORDIC Cisplatin/Dox + NS Yes (T3,T4a)

XRT

MRC/EORTC CMV No Yes

SWOG M-VAC Yes Yes

GUONE M-VAC ? No

Egypt Carbo/MTX/Vbl ? Yes

Nordic II Cisplatin/Mtx No No

Dox = Doxorubicin; XRT = external beam irradiation; NS = not significant; CMV = cisplatin, methotrexate, vinblastine;
M-VAC = methotrexate, vinblastine, adriamycin, cisplatin; Carbo = carboplatin; MTX = methotrexate

Table 5

Adjuvant Chemotherapy Randomized Trials

Regimen Progression Survival

Skinner, et al (1988) CISCA Yes No

Stockle, et al (1992) M-VA(E)C Yes Not eval.

Studer, et al (1994) Cisplatin Not eval. No

Freiha, et al (1996) CMV Yes No

Bono, et al (1989) Cisplatin + No No


MTX

Cognetti, et al (2008) GC No No

SOGUG, et al (2010) PCG Yes Yes

CISCA = cisplatin, cyclophosphamide, adriamycin; M-VA(E)C = methotrexate, vinblastine, adriamycin (epirubicin), cisplatin;
CMV = cisplatin, methotrexate, vinblastine; MTX = methotrexate; GC = gemcitabine, cisplatin; PCG = paclitaxel, gemcitabine,
cisplatin

CHAPTER 18: BLADDER CANCER 563


The only chemotherapy regimens that have shown Figure 6
this benefit are M-VAC and CMV (see discussion of
metastatic disease later for more details on these Strategy for Integrating Radical TURBT, Radi-
regimens). A recent meta-analysis using individ- ation and Chemotherapy (trimodal therapy)
ual patient data found a 5% overall survival
advantage for neoadjuvant chemotherapy and a
9% advantage for multiagent platinum-based
regimens but none for single agent chemother-
apy (Figure 5).39

In summary, the benefit of neoadjuvant chemother-


apy for patients with non–organ-confined tumors is
generally accepted. The complication rate of cystec-
tomy is not materially affected by neoadjuvant
chemotherapy.38,40

However, there are still unresolved issues. Better


staging modalities that provide more accurate clini-
cal staging will provide more precise patient selec-
tion. As new agents are introduced into the treat- 7 randomized trials have been published to date
ment of patients with metastatic disease, their role (Table 5). There is clear evidence that adjuvant
in a neoadjuvant setting has yet to be evaluated. chemotherapy had a positive impact on time to pro-
Many medical oncologists are using gression but this did not translate to a long-term sur-
gemcitabine/cisplatin, but randomized clinical trials vival benefit. Adjuvant chemotherapy can be
testing this regimen in the perioperative setting have administered safely following cystectomy. Most tri-
not been reported. The molecular determinants that als allow treatment up to 12 weeks following
predict response to treatment are a subject of intense surgery, which should provide more than adequate
research and will provide more precise targeted time for recovery. None of the studies, however,
therapy options in the future. Since the randomized was performed in such a way as to be definitive,
trials to date have not been powered for subset anal- primarily due to the fact that the 3 studies using
ysis, there remains a question as to the relative bene- multiagent chemotherapy did not have the statis-
fit for patients with or without nodal metastases. tical power to detect such improvement. A recent
report from the Spanish adjuvant trial, which used 4
Adjuvant Chemotherapy cycles of paclitaxel, gemcitabine and cisplatin and
Rationale. The rationale for adjuvant chemother- closed early due to slow accrual, suggested a sur-
apy (treatment given to treat a risk of recurrence fol- vival benefit to adjuvant chemotherapy (HR 0.378).
lowing definitive local therapy) is based on patho-
logic staging of the primary tumor and lymph Many feel it is appropriate, based on the trials show-
nodes. Chemotherapy is therefore given to the ing a benefit to neoadjuvant chemotherapy and
patients most at risk for having residual surgery, to offer adjuvant chemotherapy in the case
micrometastatic disease and local therapy is given of patients with non–organ-confined cancers after
upfront when the patient is in the best physical con- complete pathologic staging of the cystectomy
dition. Chemotherapy is given when there is mini- specimen and pelvic lymph nodes. A trial at M.D.
mal (micrometastatic) disease burden. The disad- Anderson demonstrated similar results when
vantage is there is no measurable disease which chemotherapy was administered as all neoadjuvant
allows objective measurement of response and there
is the potential that the patient may not recover fully
from surgery in a timely fashion to receive
chemotherapy.

564 EDUCATIONAL REVIEW MANUAL IN UROLOGY


5. Molecular Biology

vs a split between neo and adjuvant.41 Unfortu- Transitional cell carcinoma is a multitude of dis-
nately, several new randomized adjuvant eases with variable biologic manifestations. Molec-
chemotherapy trials were closed early due to slow ular alterations facilitate a fuller understanding of
accrual, so we may not have level I evidence to sup- the biology of the disease, identify potential targets
port this approach and therefore use our knowledge for therapy and correlate response to treatments.
of the natural history of invasive bladder cancer to P53 plays a significant role in the development of
guide decision-making for now. CIS and high-grade invasive cancer. The retinoblas-
toma tumor suppressor gene (RB) is also important.
Trimodal Therapy The effects of these 2 genes are modified by p21 and
Trimodal therapy starts with as thorough a TURBT p14/16, respectively. Mutations of the fibroblast
as possible (Figure 6). Patients respond to radiother- growth factor receptor 3 (FGFR3) gene have
apy best with minimal bulk of tumor. Chemotherapy recently been implicated in the development of
is then integrated with definitive radiation (for Ta papillary disease.
doses, see description of external beam radiation
earlier in this chapter). Patients who have a partial P53 is a tumor suppressor gene and a key cell cycle
response (PR) go on to cystectomy and patients who regulator. Its normal function is control of the G1/S
have a complete response (CR) receive additional transition. It mediates its activity through the induc-
radiation and chemotherapy. The chemotherapy tion of p21. p53 is induced in response to DNA
serves to enhance the response to the radiation and damage induced by chemotherapy or radiation,
to treat micrometastatic disease. 5-year survival leading to cell cycle arrest and induction of apop-
probabilities are 40%–50% and approximately one- tosis.
fourth to one-third of patients require cystectomy.
• Located on short arm of chromosome 17 (17p)
Figure 7
• Gene product is a key protein involved in cell-
Positive p53 immunohistochemistry
cycle regulation

• Inhibits phase-specific cell-cycle progression


(G1-S)

• Mediates its control through transcriptional activa-


tion of p21 WAF1/CIP1

• DNA damage ↑ → p53 wild type ↑ → p21→ cell


cycle arrest and induction of apoptosis

Role in Bladder Cancer


• P53 altered in half of invasive bladder cancers

• P53 immunohistochemistry is a good surrogate for


p53 gene mutation

Recent studies suggest that it adds prognostic value


to mutation analysis.
Positive p53 immunohistochemistry on a high-grade
invasive TCC with intense nuclear immunoreactivity indi- • Mutated p53 protein has a prolonged half-life
cating altered p53 protein. compared to wild type (Figure 7)

* A positive nuclear stain indicates mutant


protein

CHAPTER 18: BLADDER CANCER 565


6. Bladder Cancer Metastases

* ≥10% of cells positively correlate with the • <5% of patients initially present with metastatic
p53 gene mutation bladder cancer beyond the pelvic lymph nodes

The original study from USC demonstrated the • 50% of patients with muscle-invasive bladder can-
effect p53 alterations have on prognosis was con- cer will progress to metastatic disease despite
fined to patients with organ-confined node-negative curative local therapy
cancer. The study served as the basis for a clinical
trial that attempted to validate this observation • The median duration of survival following the
prospectively and determine whether M-VAC adju- diagnosis of metastatic bladder cancer is 2 years
vant chemotherapy can improve the survival in p53-
positive patients.42 This trial closed early and p53 Liver, lung and bone are the most common sites
did not predict outcome nor did chemotherapy clat- after pelvic node metastasis. CNS metastases are
ter prognosis in patients with altered p53 status. more common after chemotherapy, as this
appears to be a privileged site.
Retinoblastoma (Rb) tumor suppressor gene
• Located on long arm of chromosome 13 (13q) • Autopsy study of 367 patients; pT2-pT444

• Rb is altered in up to 50% of invasive bladder • Sites


cancers
Regional nodes (90%) Pleura (16%)
• Inactivation of the RB gene is more frequent in
high-grade and advanced stage TCC Liver (47%) Kidney (14%)

• Alteration of RB expression has been associated Lung (45%) Adrenal gland (14%)
with disease progression and decreased survival
Bone (32%) Intestine (13%)
Immunohistochemical analysis is the most reliable
means of determining RB status. A negative or a Peritoneum (19%)
strong intense staining of more than 50% of the cells
is evidence of the altered state. A heterogeneous The best results with chemotherapy are achieved
stain is the normal phenotype. P16 is a cyclin- in patients who are asymptomatic and have
dependent kinase inhibitor, and it competes for the lymph node–only metastases, with approxi-
binding of D-cyclins by CDK 4 and 6, which ulti- mately one-third of patients surviving 3 years.
mately have an impact on RB function. When it is Patients with symptoms and/or visceral sites of
bound to the CDKs, P16 inhibits the phosphoryla- metastases fare much worse. These data are for tran-
tion of RB, which thereby inactivates RB, prevent- sitional cell cancer or TCC with mixed histology.
ing progression through the cell cycle.43 Chemotherapy for patients with primary squa-
mous cell or adenocarcinoma is not standardized
When both p53 and RB pathways are normal, the and generally felt to not be as effective.
risk of progression and death after cystectomy is
minimal. When one or the other is altered, the risk is M-VAC is the gold standard combination
higher. When both are altered, the risk is highest. chemotherapy regimen. Patients with a neoblad-
These biomarker alterations add additional indepen- der or continent cutaneous diversion should have
dent prognostics in addition to pathologic tumor a catheter placed during hydration with cisplatin
stage and lymph node status. in order to avoid overdistension of the reservoir
and with the methotrexate in order to avoid
absorption of methotrexate across the bowel and
subsequent high serum methotrexate levels. M-
VAC is administered according to the following
schedule and cycles every 28 days. Chemotherapy

566 EDUCATIONAL REVIEW MANUAL IN UROLOGY


for measurable metastatic disease is usually given GC had higher incidence of grade 3 and 4 anemia
for 6 cycles (neoadjuvant or adjuvant therapy is and thrombocytopenia, whereas M-VAC had a
given for 3–4 cycles). higher incidence of grade 3 and 4 neutropenia.

Methotrexate (30 mg/m2) Days 1, 15, 22 M-VAC is still a standard of care for metastatic
bladder cancer. GC is used increasingly, as it is eas-
Vinblastine (3 mg/m2) Days 2, 15, 22 ier to administer and there is a perception of equiva-
lency and less toxicity. The median survival is
Adriamycin (30 mg/m2) Day 2 12–15 months and performance status and visceral
metastases influence response to chemotherapy.
Cisplatin (70 mg/m2) Day 2
Taxanes
The original long-term follow-up study from • The taxanes paclitaxel (Taxol®) and docetaxol
Memorial Sloan-Kettering demonstrated the (Taxotere®) are active in this disease
requirement for a complete response to chemother-
apy, with or without surgery, in order to achieve a *Work by disrupting microtubule assembly
survival benefit.45 One-half of the complete during S phase
responses (CR) achieved in 39% of patients were
durable, resulting in an estimated 20% cure rate. • Taxol/carboplatin has been studied extensively
Failure to achieve a CR is not compatible with
long-term survival. Consideration should be given * Complete response rates vary from 0%–40%
to resection of solitary metastatic sites that persist
after chemotherapy. * This is widely felt to be an inferior regimen
as front-line treatment
CMV is another regimen commonly used. The dif-
ference between M-VAC and CMV is that CMV * May have a role as salvage therapy
uses a higher dose of cisplatin (100 mg/m2) and adri-
amycin is not used. Though no clinical trials have • Taxanes are being added to other regimens, includ-
ever compared the 2 regimens, they are generally ing a 3-drug regimen of gemcitabine, cisplatin and
felt to be equivalent in efficacy. Comparisons of paclitaxel which is being compared to gemcitabine
multiagent therapy to single-agent cisplatin clearly and cisplatin in a Phase III EORTC trial
show the inferiority of single-agent cisplatin.

Several randomized trials have led to the following


conclusions in metastatic or unresectable locally
advanced disease (Table 6):

1.) Combination chemotherapy (with M-VAC) is


superior to single-agent cisplatin;

2.) M-VAC is superior to CISCA (cisplatin,


cyclophosphamide, adriamycin); 3.) Dose inten-
sification (HD = high dose) and use of growth
factors has not resulted in improved survival; 4.)
The recent industry-sponsored trial of gemc-
itabine and cisplatin (GC) suggested that GC is
equivalent to M-VAC for the treatment of
metastatic disease with less toxicity (Figure 6).

CHAPTER 18: BLADDER CANCER 567


Figure 6

Metastatic bladder cancer: M-VAC vs. gemcitabine/cisplatin – overall survival

With permission. von der Maase H, et al. Metastatic bladder cancer: M-VAC vs. gemcitabine/cisplatin – overall survival.
J Clin Oncol. 2005;23:4602-4608.

Table 6

Randomized Clinical Trials in Metastatic or Unresectable Locally Advanced


Urothelial Cancer

nRRMDS
n RR MDS Results

Intergroup M-VAC 126 39% 12.5 M-VAC> DDP


DDP (cisplatin) 120 12% 8.2

MDACC M-VAC 65 65% 11.1 M-VAC > CISCA


CISCA 55 46% 8.3

EORTC HD-MVAC 134 62% 14.5 HD-MVAC


M-VAC 129 50% 14.1 ± > M-VAC

Lilly GC 164 49% 13.8 M-VAC ± = GC


M-VAC 151 46% 14.8

EORTC GC 315 46% 12.8 GC=PCG


PCG 312 57% 15.7

HD = high dose RR = recurrence rate MDS =median disease-free survival


Sternberg, Lerner, 2010

568 EDUCATIONAL REVIEW MANUAL IN UROLOGY


7. Urinary Diversion

The urologist planning cystectomy should review capacity and decreasing intraluminal pressures. Min-
the options of a conduit, continent cutaneous diver- imizing the risk of electrolyte absorption reduces the
sion requiring intermittent catheterization, and risk of chronic acidosis and bone resorption.
orthotopic neobladder. Particular techniques uti-
lized reflect training experience and bowel segment Continent Cutaneous Diversion
preferences. Patient factors and underlying health Right colon reservoirs rely on a detubularized seg-
influence this choice as well. Options and consider- ment that includes the cecum, ascending colon, hep-
ations include: atic flexure and proximal transverse colon based on
the ileocolic and right colic arteries with collaterals
• Incontinent from the marginal artery of Drummond. The seg-
ment is usually 30 cm long and is opened between
* Conduit the 2 anterior tinea to the base of the appendix,
which is excised or utilized as the catheterizable
• Ileum, transverse colon channel, if it has an adequate lumen, by laying it in a
trough created in the bed of an anterior tinea. The
• Continent continence mechanism of these reservoirs relies
on the ileocecal valve, which is plicated in the
* Ureterosigmoidostomy case of the Indiana47 and Florida48 pouches, or
intussuscepted and stapled in the case of the
* Orthotopic Mainz Pouch (which can also be utilized for an
orthotopic neobladder).49 The Kock pouch uti-
* Catheterizable stoma lizes ileum and an intussuscepted and stapled
nipple valve is created for both the afferent
• Considerations (ureter) and efferent limbs (catheterizable chan-
nel).50 The continence and antireflux mechanisms of
* Choice of bowel segment the T-pouch are created with a serosal-lined extralu-
minal valve.51
* Patient selection
Other Options for a Continent Catheterizable
* Surgical technique Stoma
The Mitrofanoff principle was originally
* Functional outcomes described for the appendix, which is tunneled in
an antireflux fashion into the native bladder or
* Metabolic consequences laid in a trough created along a colon tinea.
Today, the principle is applied in a variety of tapered
Neils Kock, a Swedish general surgeon, described intestinal segments that can be utilized for a
principles of urinary diversion that have withstood catheterizable stoma. Monti (from Brazil) has
the test of time.46 Capacity is assured by detubular- described a technique of utilizing short segments of
ization, which disrupts the circular smooth mus- ileum that are detubularized and rolled into a tube
cle, thereby decreasing wall tension and folding, for construction of an efferent catheterizable chan-
which decreases the intraluminal pressure nel using the Mitrofanoff principle.52 The
according to LaPlace’s law (pressure = wall ten- Benchekroun valve is a 14-cm segment of ileum
sion/radius). Separation of the urinary and fecal that is folded inward along its entire length and can
streams virtually eliminates the risk of cancer in be used with a variety of reservoirs.53 The long-term
the intestinal segment. The patient should achieve durability has been questioned recently, with a high
social continence. The need to prevent reflux is fre- rate of stenosis observed.
quently debated. It is highly unlikely that reflux of
colonized urine in a low-pressure system is harmful.
However, antireflux mechanisms may promote
“stretching” of the reservoir, thereby increasing

CHAPTER 18: BLADDER CANCER 569


Patient Selection The right lower quadrant is the most common stoma
• Life expectancy >1 year site and should be selected in consultation with the
enterostomal therapist. The umbilicus has the
• Strong self-image and desire to be “bag free” advantage of concealability and it is simple to
perform. The umbilicus is the shortest distance
• Manual dexterity to traverse, which makes it advantageous in
obese patients. Quality of life studies indicate that
* Lifelong requirement for intermittent the continence rates are higher than for orthotopic
catheterization diversion.

• Normal or nearly normal renal function Orthotopic Urinary Diversion


Lilien and Camey were perhaps the first to popu-
* ClCr ≥50 mL/min larize orthotopic bladder replacement, demon-
strating that continence could be achieved first
* Cr ≤2.0 mg/dL with non-detubularized ileum (Camey I) and
then detubularized ileum (Camey II).54 The “hemi
• Normal or nearly normal bowel function Kock” pouch adapts the Goodwin cup patch tech-
nique for a low-pressure reservoir and an intussus-
* Some recommend colonoscopy prior to cepted ileal nipple valve to prevent reflux. The
surgery if a colon reservoir will be used. Studer pouch relies on the same Goodwin cup patch
Patients with signs or symptoms suggesting technique, then a long (15–20 cm) afferent limb
colon disease or patients at increased risk for which is refluxing. The ileal neobladder creates a
colon cancer should definitely have this done low pressure reservoir by detubularizing a long seg-
ment of ileum and reconfiguring to a W or M con-
In general, patients should be offered the choice of figuration. The ureters are reimplanted in an end-to
continent diversion in all settings. However, it may side fashion or in a serosal-lined extraluminal
not be appropriate for patients who are having cys- trough. The T pouch, developed at USC, adapts this
tectomy as palliation. concept of a serosal-lined trough to create a non-
refluxing afferent limb that does not require use of
staples.

Table 7

Sites of Absorption of Various Substances Thoughout the GI Tract

Stomach Duodenum Jejunum Ileum Terminal Ileum Colon

Carbohydrates X X X X
Protein X X X (X)
Calcium (X) (X)
Iron (X) (X)
Water-soluble X X
vitamins
Vitamin B12 ((X)) X
Bile acids (X) X
Sodium, chloride, ware X X X

X-absorption, (X)-limited absorption and ((X))-very limited absorption occurs. Only absorption of bile acids and vitamin B12 are
restricted to a single area, namely the ileum, and other absorptive tasks may when needed be taken over by
different parts of the bowel.

570 EDUCATIONAL REVIEW MANUAL IN UROLOGY


The ileocolic neobladder utilizes detubularized Patient Selection for Orthotopic Diversion
cecum, ascending colon and terminal ileum, and the The patient must have normal sphincter function
ureters are implanted end-to-side in the colon seg- and this can be assessed by a careful history and
ment. Investigators at Johns Hopkins reported a 12- physical exam. The retained urethra should not
year experience in 96 patients. Bladder neck con- be at significant risk for developing cancer (see
tracture was the most common complication. Ols- invasive bladder cancer section). The patient should
son has popularized the use of absorbable staples to understand the rehabilitation period during which
construct a detubularized right colon reservoir.55 they will require pads and be highly motivated to
The sigmoid colon is an excellent option, especially work through this period in order to achieve social
when the sigmoid is redundant, as it easily reaches continence. Elderly patients or patients with locally
into the pelvis. The ureters can be implanted in an advanced tumors (P3b, P4 or N+) or those who have
end-to-side or non-refluxing tinea reimplant fash- had prior pelvic irradiation are not routinely
ion. excluded. The complication rate is not materially
affected by neoadjuvant chemotherapy.
Advantages of Small Bowel
• Lower pressures than colon Orthotopic Diversion in Women
Women with normal sphincter function are excel-
• Reliably reaches the urethra lent candidates for orthotopic reconstruction.
Patients who may require urethra or bladder neck
• Facilitates taking ureters high (eg, XRT, CIS) suspension can also be considered but may be at
higher risk for requiring intermittent catheteriza-
• Low incidence of ureteral stricture (4%) tion. Any evidence of cancer at the bladder neck
or urethra is an absolute contraindication. T3 or
• Urine is frequently sterile T4 tumors of the posterior bladder are relative
contraindications, as it may not be possible to get
Ureteral Implantation an adequate margin distally. The distal two-
The end to side ureterointestinal anastamosis is thirds of the urethra is critical to normal sphinc-
reliable with a low stricture rate of approxi- ter function and must be preserved. Dissection
mately 4%. It was described by Wayland Leadbet- below the endopelvic fascia must be avoided in
ter for use with an ileal conduit and with the original order to avoid damage to the pudendal nerve,
description of the conduit by Bricker. It can be used which innervates this portion of the urethra.28
in virtually any setting where a ureter is implanted Approximately 20% of patients will require inter-
into bowel. LeDuc described a nonrefluxing mittent catheterization (some refer to this as “hyper-
ureter reimplant that is associated with a higher continence”). This phenomenon is much more com-
stricture rate. When implanting a ureter into colon, mon in women than in men.
the end-to-side works quite well as reflux is not a
major concern. Many surgeons prefer a tinea reim- Daytime continence is achieved in a high percent-
plant that is non-refluxing. The major advantage is age of patients regardless of age or gender. During
that it can be performed without opening the bowel. sleep, older patients have a higher rate of inconti-
The Wallace technique brings the spatulated nence. As the bladder fills and intraluminal pres-
ends of both ureters together in a single implant. sures rise, resting urethra pressure does not increase
The major disadvantage is that both ureters are at to compensate for the rise in neobladder pressure,
risk if a stricture occurs, or any other type of distal resulting in incontinence. Most patients need to void
obstruction. More recently, Ghoneim described a once or twice at night in order to reduce the risk of
method of reimplantation into an ileal neobladder incontinence. Urinary retention in men is uncom-
where the ureter is laid into an extraluminal serosal- mon and must raise the possibility of local pelvic
lined trough that is non-refluxing. Discussion at the recurrence. Urethral anastamotic strictures are also
AUA in 2005 concluded that there was no role for uncommon in both men and women. The following
an antireflux anastamosis with an orthotopic data are from the author’s experience in a consecu-
neobladder. tive group of patients:

CHAPTER 18: BLADDER CANCER 571


Daytime 59/69 (86%) • Enterostomal therapist

<65 37/42 (88%) * Consult preoperatively for counseling and


marking stoma site for best results
>65 22/27 (81%)
Physiology
Night 37/68 (54%)
Stomach
<65 26/41 (63%) The stomach is used more in the pediatric popula-
tion, especially in children with renal insufficiency.
>65 11/27 (41%) The stomach absorbs carbohydrates and makes
intrinsic factor, which is important for absorp-
Conduit Urinary Diversion tion of B12 by the terminal ileum. The metabolic
• Advantages consequences include B12 malabsorption and alka-
losis, which is associated with hypokalemia and
* Simple to perform hypochloremia. Reduced hydrochloric acid secre-
tion results in loss of feedback on gastrin secretion,
* Easy to manage—high satisfaction rate which may produce peptic ulcers and hematuria, the
so-called hematuria dysuria syndrome.
• Bowel segment
Jejunum
* Terminal ileum—no prior irradiation Jejunum should not be used except as a last resort.
The jejunum secretes sodium and chloride and
* Transverse colon—prior pelvic irradiation absorbs hydrogen ions and potassium. The pri-
mary pathophysiology is dehydration that is
* Avoid a long loop which leads to stasis of related to large water shifts and the subsequent
urine stimulation of aldosterone. This facilitates sodium
reabsorption in the distal tubule and collecting duct
Ileal Conduit in exchange for potassium and protons. What fol-
• Patient selection lows is potassium diuresis with low urine sodium
concentration perpetuating the problem. Exchange
* No motivation to be free of an appliance of urinary potassium for hydrogen results in acido-
sis.56 The metabolic abnormality consists of
* Manual dexterity metabolic acidosis accompanied by hyper-
kalemia, hyponatremia and loss of chloride. The
* Elderly patient who needs something simple patient may complain of nausea, emesis and fatigue.
The treatment is hydration and sodium chloride.
* Palliative surgery
Ileum and Colon
• Stoma—usually placed in the right lower quadrant • Secretes sodium and bicarbonate
in consultation with an enterostomal therapist
after examining the patient in the lying, sitting and • Absorbs ammonium, hydrogen chloride
standing positions
* Ammonium is the principle mechanism of
* Turnbull loop ileostomy—eliminates risk of acidosis
stomal stenosis
• Hyperchloremic metabolic acidosis
* Rosebud end stoma—creates a nipple that
minimizes urine contact with skin * Total body potassium deficit

572 EDUCATIONAL REVIEW MANUAL IN UROLOGY


* Hypocalcemia, hypomagnesemia, hypera- Other Complications
monemia • Cholelithiasis

• Symptoms * Decreased bile acids causes lithogenic over-


saturation of bile
* Fatigue, anorexia, diarrhea
* Incidence: 9% with resection 30–50 cm
• Treatment ileum; 35% with resection 100 cm

* IV hydration plus bicarbonate * No documented increased risk in urinary


diversion
* Potassium supplementation
• Renal calculi
Table 7 describes sites of absorption throughout the
GI tract. The reference is a comprehensive review of * Decreased bile acids; fat malabsorption
GI physiology pertinent to interposition in the uri- and binding of intestinal calcium; increased
nary tract. oxalate

Complications * Chronic bacteriuria and struvite stones


An inevitable consequence of orthotopic or conti-
nent cutaneous diversion is a reduction in the * Dehydration
absorptive capacity of the bowel. Most patients will
ultimately adapt to this but some do experience a • Reservoir calculi—usually form on foreign body
change in bowel function. The loss of the ileocecal
valve in the case of right colon reservoirs may con- Small stones can be treated with standard endo-
tribute to loose stools or diarrhea. Diarrhea is usu- scopic techniques. Repeated manipulation of a
ally due to fat malabsorption and bile salts irrita- continent cutaneous stoma should be avoided,
tion of the colon. Metamucil works by adding bulk and so large stones are best treated with a percu-
to the stool and this is what I try first. Subsequent taneous or open technique.
management is antimotility drugs which are usually
successful. Cholestyramine will absorb excess bile Secondary Cancer
salts and is usually effective in patients that do not • Rare
respond to antimotility agents. B12 deficiency is a
consequence of malabsorption related to the loss • 0% in over 2000 patients
of the distal terminal ileum or use of a large seg-
ment of distal ileum. Other risk factors include * 1 patient with recurrent lymphoma (Kock
radiation exposure to the terminal ileum. Loss of Pouch)
B12 is insidious and takes several years, though it
has been reported earlier.57 Annual monitoring of • Colonoscopy pre-op for at-risk patients prior to
B12 levels and replacement if deficiency is identi- colon reservoir procedures
fied is recommended. Treatment should be initiated
prior to symptoms. Parenteral B12 shots are reliable • Recommend pouch endoscopy annually starting at
and there are now oral formulations that can be 3 years post-op
administered as well.58 Osteoporosis can occur as a
result of acidosis causing depletion of bone phos- • There are case reports of typical adenocarcinoma
phate stores. Patients with renal insufficiency are at of the colon forming in an Indiana Pouch and I
higher risk. have treated 2 patients for this recently–9 and 10
years postcystectomy.

CHAPTER 18: BLADDER CANCER 573


8. References

• Highest risk in ureterosigmoidostomy59 1. Kamat AM, Gee JR, Dinney CP, et al. The
case for early cystectomy in the treatment
* Contact of feces and urine40 of nonmuscle invasive micropapillary
bladder carcinoma. J Urol. 2006;175:881-
* After 10 years: annual colonoscopy—risk 885.
increases dramatically to up to 25-50 times
the normal population 2. Esrig D, Freeman JA, Elmajian DA, et al.
Transitional cell carcinoma involving the
prostate with a proposed staging classifica-
tion for stromal invasion. J Urol.
1996;156(3):1071-1076.

3. Lerner S, Sternberg C. Management of


metastatic and invasive bladder cancer. In:
Wein A, Partin, AW, Novick, AC, Peters,
CA, Kavoussi, LR, ed. Campbell-Walsh
Urology. 10th ed. Philadelphia, PA: Else-
vier; 2011.

4. Kibel AS, Dehdashti F, Katz MD, et al.


Prospective study of [18F] Fluorodeox-
yglucose positron emission
tomography/computed tomography for
staging of muscle-invasive bladder carci-
noma. J Clin Oncol. 2009;27:4314-4320.

5. Chang SS, Cole E, Smith JA Jr, Cookson


MS. Pathological findings of gynecologic
organs obtained at female radical cystec-
tomy. J Urol. 2002;168(1):147-149.

6. Ali-El-Dein B, Gomha M, Ghoneim MA.


Critical evaluation of the problem of
chronic urinary retention after orthotopic
bladder substitution in women. J Urol.
2002;168(2):587-592.

7. Vazina A, Dugi D, Shariat SF, Evans J,


Link R, Lerner SP. Stage specific lymph
node metastasis mapping in radical cystec-
tomy specimens. J Urol. 2004;171(5):
1830-1834.

574 EDUCATIONAL REVIEW MANUAL IN UROLOGY


8. Herr HW, Donat SM. Outcome of patients 15. Chade DC, Laudone VP, Bochner BH,
with grossly node positive bladder cancer Parra RO. Oncological outcomes after radi-
after pelvic lymph node dissection and rad- cal cystectomy for bladder cancer: open
ical cystectomy. J Urol. 2001;165(1):62- versus minimally invasive approaches.
64. J Urol. 2010;183(3):862-869.

9. Donat SM, Shabsigh A, Savage C, et al. 16. Stein JP, Clark P, Miranda G, Cai J,
Potential impact of postoperative early Groshen S, Skinner DG. Urethral tumor
complications on the timing of adjuvant recurrence following cystectomy and uri-
chemotherapy in patients undergoing radi- nary diversion: clinical and pathological
cal cystectomy: a high-volume tertiary can- characteristics in 768 male patients. J Urol.
cer center experience. Eur Urol. 2005;173(4):1163-1168.
2009;55(1):177-185.
17. Stein JP, Esrig D, Freeman JA, et al.
10. Shabsigh A, Korets R, Vora KC, et al. Prospective pathologic analysis of female
Defining early morbidity of radical cystec- cystectomy specimens: risk factors for
tomy for patients with bladder cancer using orthotopic diversion in women. Urology.
a standardized reporting methodology. Eur 1998;51(6):951-955.
Urol. 2009;55(1):164-174.
18. Stenzl A, Draxl H, Posch B, Colleselli K,
11. Lowrance WT, Rumohr JA, Chang SS, Falk M, Bartsch G. The risk of urethral
Clark PE, Smith JA, Jr., Cookson MS. Con- tumors in female bladder cancer: can the
temporary open radical cystectomy: analy- urethra be used for orthotopic reconstruc-
sis of perioperative outcomes. J Urol. tion of the lower urinary tract? J Urol.
2008;179(4):1313-1318. 1995;153:950-955.

12. Hellenthal NJ, Hussain A, Andrews PE, et 19. Kessler TM, Burkhard FC, Perimenis P, et
al. Surgical margin status after robot al. Attempted nerve sparing surgery and
assisted radical cystectomy: results from age have a significant effect on urinary
the International Robotic Cystectomy Con- continence and erectile function after radi-
sortium. J Urol.2010;184(1):87-91. cal cystoprostatectomy and ileal orthotopic
bladder substitution. J Urol.
13. Novara G, Svatek RS, Karakiewicz PI, et 2004;172:1323-1327.
al. Soft tissue surgical margin status is a
powerful predictor of outcomes after radi- 20. Stenzl A, Bartsch G, Rogatsch H. The rem-
cal cystectomy: a multicenter study of nant urothelium after reconstructive blad-
more than 4,400 patients. J Urol. der surgery. Eur Urol. 2002;41(2):124-131.
2010;183(6):2165-2170.
21. Stein JP, Lieskovsky G, Cote R, et al. Radi-
14. Nix J, Smith A, Kurpad R, Nielsen ME, cal cystectomy in the treatment of invasive
Wallen EM, Pruthi RS. Prospective ran- bladder cancer: long-term results in 1,054
domized controlled trial of robotic versus patients. J Clin Oncol. 2001;19(3):666-
open radical cystectomy for bladder can- 675.
cer: perioperative and pathologic results.
Eur Urol. 2010;57(2):196-201.

CHAPTER 18: BLADDER CANCER 575


22. Njinou Ngninkeu B, Lorge F, Moulin P, 30. Herr HW. Transurethral resection of mus-
Jamart J, Van Cangh PJ. Transitional cell cle-invasive bladder cancer: 10-year out-
carcinoma involving the prostate: a clinico- come. J Clin Oncol. 2001;19(1):89-93.
pathological retrospective study of 76
cases. J Urol. 2003;169(1):149-152. 31. Solsona E, Iborra I, Collado A, Rubio-
Briones J, Casanova J, Calatrava A. Feasi-
23. Shen SS, Lerner SP, Muezzinoglu B, bility of radical transurethral resection as
Truong LD, Amiel G, Wheeler TM. Pro- monotherapy for selected patients with
static involvement by transitional cell car- muscle invasive bladder cancer. J Urol.
cinoma in patients with bladder cancer and 2010;184(2):475-480.
its prognostic significance. Hum Pathol.
2006;37(6):726-734. 32. Fedeli U, Fedewa SA, Ward EM. Treat-
ment of muscle invasive bladder cancer:
24. Stein JP, Cai J, Groshen S, Skinner DG. evidence from the National Cancer
Risk factors for patients with pelvic lymph Database, 2003 to 2007. J Urol.
node metastases following radical cystec- 2010;185(1):72-78.
tomy with en bloc pelvic lymphadenec-
tomy: concept of lymph node density. 33. Hollenbeck BK, Taub DA, Dunn RL, Wei
J Urol. 2003;170(1):35-41. JT. Quality of care: partial cystectomy for
bladder cancer--a case of inappropriate
25. Clark PE, Stein JP, Groshen SG, et al. Radi- use? J Urol. 2005;174(3):1050-1054.
cal cystectomy in the elderly: Comparison
of survival between younger and older 34. Holzbeierlein JM, Lopez-Corona E,
patients. Cancer. 2005;103(3):546-552. Bochner BH, et al. Partial cystectomy: a
contemporary review of the Memorial
26. Nielsen ME, Shariat SF, Karakiewicz PI, et Sloan-Kettering Cancer Center experience
al. Advanced age is associated with poorer and recommendations for patient selection.
bladder cancer-specific survival in patients J Urol. 2004;172(3):878-881.
treated with radical cystectomy. Eur Urol.
2007;51(3):699-706. 35. Kassouf W, Swanson D, Kamat AM, et al.
Partial cystectomy for muscle invasive
27. Madeb R, Messing EM. Gender, racial and urothelial carcinoma of the bladder: a con-
age differences in bladder cancer incidence temporary review of the M. D. Anderson
and mortality. Urol Oncol. 2004;22(2):86- Cancer Center experience. J Urol.
92. 2006;175(6):2058-2062.

28. Schilling D, Horstmann M, Nagele U, 36. Capitanio U, Isbarn H, Shariat SF, et al.
Sievert KD, Stenzl A. Cystectomy in Partial cystectomy does not undermine
women. BJU Int. 2008;102:1289-1295. cancer control in appropriately selected
patients with urothelial carcinoma of the
29. Herr HW, Cookson MS, Soloway SM. bladder: a population-based matched anal-
Upper tract tumors in patients with primary ysist. Urology. 2009;74(4):858-864.
bladder cancer followed for 15 years.
J Urol. 1996;156(4):1286-1287. 37. Neoadjuvant cisplatin, methotrexate, and
vinblastine chemotherapy for muscle-inva-
sive bladder cancer: a randomised con-
trolled trial. International collaboration of
trialists. Lancet. 1999;354(9178):533-540.

576 EDUCATIONAL REVIEW MANUAL IN UROLOGY


38. Grossman HB, Natale RB, Tangen CM, et 45. Sternberg CN, Yagoda A, Scher HI, et al.
al. Neoadjuvant chemotherapy plus cystec- Methotrexate, vinblastine, doxorubicin, and
tomy compared with cystectomy alone for cisplatin for advanced transitional cell car-
locally advanced bladder cancer. N Engl J cinoma of the urothelium. Efficacy and pat-
Med. 2003;349(9):859-866. terns of response and relapse. Cancer.
1989;64(12):2448-2458.
39. Neoadjuvant chemotherapy in invasive
bladder cancer: update of a systematic 46. Faxen A, Kock NG, Sundin T. Long-term
review and meta-analysis of individual functional results after ileocystoplasty.
patient data advanced bladder cancer Scand J Urol Nephrol. 1973;7(2):127-130.
(ABC) meta-analysis collaboration. Eur
Urol. 2005;48(2):202-206. 47. Rowland RG. Present experience with the
Indiana pouch. World J Urol. 1996;14(2):
40. Hall MC, Swanson DA, Dinney CP. Com- 92-98.
plications of radical cystectomy: impact of
the timing of perioperative chemotherapy. 48. Webster C, Bukkapatnam R, Seigne JD, et
Urology. 1996;47(6):826-830. al. Continent colonic urinary reservoir
(Florida pouch): long-term surgical compli-
41. Millikan R, Dinney C, Swanson D, et al. cations (greater than 11 years). J Urol.
Integrated therapy for locally advanced 2003;169(1):174-176.
bladder cancer: final report of a random-
ized trial of cystectomy plus adjuvant M- 49. Lampel A, Fisch M, Stein R, et al. Conti-
VAC versus cystectomy with both preoper- nent diversion with the Mainz pouch. World
ative and postoperative M-VAC. J Clin J Urol. 1996;14(2):85-91.
Oncol. 2001;19(20):4005-4013.
50. Skinner DG, Lieskovsky G, Boyd S. Conti-
42. Esrig D, Spruck CH, 3rd, Nichols PW, et al. nent urinary diversion. J Urol.
p53 nuclear protein accumulation corre- 1989;141(6):1323-1327.
lates with mutations in the p53 gene, tumor
grade, and stage in bladder cancer. Am J 51. Stein JP, Skinner DG. The craft of urologic
Pathol. 1993;143(5):1389-1397. surgery: the T pouch. Urol Clin North Am.
2003;30(3):647-661, xi.
43. Xu HJ, Cairns P, Hu SX, Knowles MA,
Benedict WF. Loss of RB protein expres- 52. Monti PR, Lara RC, Dutra MA, de Carvalho
sion in primary bladder cancer correlates JR. New techniques for construction of
with loss of heterozygosity at the RB locus efferent conduits based on the Mitrofanoff
and tumor progression. Int J Cancer. principle. Urology. 1997;49(1):112-115.
1993;53(5):781-784.
53. Benchekroun A, Essakalli N, Faik M, Mar-
44. Wallmeroth A, Wagner U, Moch H, Gasser zouk M, Hachimi M, Abakka T. Continent
TC, Sauter G, Mihatsch MJ. Patterns of urostomy with hydraulic ileal valve in 136
metastasis in muscle-invasive bladder can- patients: 13 years of experience. J Urol.
cer (pT2-4): An autopsy study on 367 1989;142(1):46-51.
patients. Urol Int. 1999;62(2):69-75.

CHAPTER 18: BLADDER CANCER 577


9. Questions

54. Lilien OM, Camey M. 25-year experience 1. A 52-year-old female with a remote history of
with replacement of the human bladder CIS and BCG treatment has a new T1G3
(Camey procedure). 1984. J Urol. tumor completely resected. There is scant
2002;167:1161-1168. muscularis propria in the TURBT specimen.
The next step is:
55. Kirsch AJ, Hensle TW, Olsson CA. Rapid
construction of right colon pouch: initial A. Re-induce with BCG plus maintenance
clinical experience. Urology. BCG
1994;43(2):228-231.
B. BCG plus Interferon alpha
56. Mills RD, Studer UE. Metabolic conse-
quences of continent urinary diversion. C. Intravesical chemotherapy with Valrubicin
J Urol. 1999;161(4):1057-1066. or Gemcitabine

57. Steiner MS, Morton RA, Marshall FF. Vita- D. Re-resection with muscularis propria in
min B12 deficiency in patients with ileo- the specimen
colic neobladders. J Urol. 1993;149(2):255-
257. E. Radical cystectomy

58. Lane LA, Rojas-Fernandez C. Treatment of 2. A 55-year-old female is found to have a 3-cm
vitamin b(12)-deficiency anemia: oral ver- nodular lesion in the bladder dome, which on
sus parenteral therapy. Ann Pharmacother. histology is determined to be small cell carci-
2002;36(7-8):1268-1272. noma, deeply invasive into the lamina propria
but with negative muscularis propria involve-
59. Crissey MM, Steele GD, Gittes RF. Rat ment. Random biopsies and urine cytology
model for carcinogenesis in ureterosigmoi- are negative. CT scan of the thorax, abdomen
dostomy. Science. 1980;207(4435):1079- and pelvis are normal. The best initial treat-
1080. ment is:

A. Intravesical BCG with later maintenance

B. Repeat TURBT within 6 weeks to assess


for residual disease or understaging

C. Partial cystectomy

D. Radical cystectomy

E. Neoadjuvant systemic chemotherapy

578 EDUCATIONAL REVIEW MANUAL IN UROLOGY


3. A 42-year-old potent male is diagnosed with a 5. A 65-year-old male undergoes 3 cycles of
4-cm micropapillary TCC that extensively neoadjuvant M-VAC chemotherapy for
invades the lamina propria. Muscularis pro- T3bNxM0 TCC. After completing
pria is present and not involved. Lymphovas- chemotherapy, there is no tumor on cys-
cular invasion is identified. The next step is: toscopy. The next step should be:a) Obser-
vation with cystoscopy in 3 months
A. Restaging TURBT and intravesical BCG if
muscle invasion is absent A. Observation with cystoscopy in 3 months

B. Partial cystectomy followed by radiation B. BCG weekly for 6 weeks


therapy
C. Bladder biopsies
C. Neoadjuvant cisplatin-based chemotherapy
followed by radical cystectomy D. Radiation therapy

D. Nerve-sparing radical cystectomy E. Radical cystectomy

E. Cisplatin-based chemotherapy and radia- 6. Which of the following immunohistochem-


tion therapy istry profiles in bladder cancer is associated
with the most aggressive tendencies?
4. An orthotopic neobladder in a woman under-
going anterior pelvic exenteration for muscle- A. High p53 and Rb staining, low Ki67 and
invasive bladder cancer is contraindicated in E-cadherin
the setting of:
B. High p53 and Ki67 staining, absent Rb and
A. Age older than 75 E-cadherin

B. Nodal metastases C. High Rb and E-cadherin, low p53 and


Ki67
C. Recurrent UTI
D. High Ki67 and Rb, low p53 and E-cad-
D. Bilateral hydronephrosis herin

E. Tumor invading the anterior vaginal wall E. Low p53, Rb, E-cadherin and Ki-67

CHAPTER 18: BLADDER CANCER 579


7. A 53-year-old female with a T2NXM0 blad- 9. A 62-year-old male has T3b invasive bladder
der TCC undergoes a radical cystectomy and TCC. TUR biopsy of prostatic urethra shows
continent diversion. Final pathology shows single focus of CIS. He does not want an
pT3a N1 with a single microscopic positive external appliance. At the time of cystectomy,
lymph node in the perivesical fat. The next he should have:
step that may be considered is:
A. Frozen section of apical urethra margin
A. PET/CT scan and, if negative, an orthotopic neobladder

B. Adjuvant chemotherapy B. Urethrectomy and continent cutaneous


diversion
C. Adjuvant radiotherapy
C. Urethrectomy and Ileal conduit
D. Combined radiotherapy and
chemotherapy D. Preoperative radiation therapy followed by
cystectomy
E. More extensive lymphadenectomy
E. Bladder salvage with chemoradiation ther-
8. A 48-year-old otherwise healthy male has a apy
CT scan and TURBT which reveal a
T4aNXM0 TCC of the bladder. The next step 10. A 58-year-old male former smoker with a past
should be: history of cystectomy for a T3bN1 invasive
bladder cancer has a CT scan 5 years later
A. Neoadjuvant chemotherapy followed by which reveals a spiculated 1-cm pulmonary
radical cystectomy lesion. The next step should be:

B. Radical cystectomy followed by adjuvant A. Cisplatin chemotherapy


chemotherapy
B. Systemic cisplatin-based combination
C. Radical cystectomy, then check tumor p53 chemotherapy
status and, if altered, give chemotherapy
C. Preoperative radiotherapy followed by
D. Preoperative radiotherapy followed by rad- removal of the lung lesion
ical cystectomy
D. Radiation therapy to the lung lesion alone
E. Neoadjuvant chemotherapy with restaging
bladder biopsies and surveillance if no clin- E. Evaluation for possible lung primary
ical evidence of cancer carcinoma

580 EDUCATIONAL REVIEW MANUAL IN UROLOGY


11. In a man with good daytime continence fol- Answers
lowing radical cystectomy and orthotopic
neobladder, nocturnal incontinence is due to: 1. D.
Re-resection into muscularis propria, which is
A. Damage to the urinary rhabdosphincter required for accurate staging in T1 cancers. EAU
and AUA Guidelines are consistent regarding the
B. Neobladder hypercontractility need for routine re-resection of all T1G3 tumors.
Understaging rate of T1 tumors is as high as 40%
C. Inadequate compliance of the neobladder and is highest when there is no muscularis propria in
the specimen. Treating with intravesical
D. Loss of afferent input from the detrusor to immunotherapy or chemotherapy prior to accurate
the central nervous system staging risks missing T2 disease, for which BCG is
inadequate therapy and survival probabilities are
E. Damage to the inferior hypogastric nerve significantly reduced compared to cystectomy for
plexus <T2 disease. Though radical cystectomy may be
appropriate therapy, salvage intravesical
12. A 62-year-old male with bilateral immunotherapy therapy with BCG along or BCG +
hydronephrosis 4 years after a radical cystec- interferon could be considered if the re-resection
tomy and ileal conduit catheterizes the stoma that includes muscularis propria shows no worse
and obtains 100cc urine. The most likely cause than completely resected T1 disease. However,
is: upstaging to T2 is associated with a significantly
increased risk for cancer-specific mortality.
A. Stoma stenosis
2. E.
B. Chronic reflux Neoadjuvant chemotherapy. Small cell carcinoma is
a rare variant of urothelial cancer that can comprise
C. Ureteral obstruction due to cancer 100% or a fraction of the tumor. Its biologic behav-
ior appears similar to small cell lung carcinoma with
D. Ureter obstruction due to fibrosis a high propensity for metastatic disease. Consensus
opinion favors neoadjuvant chemotherapy with
E. Antiperistaltic orientation of the conduit etoposide and cisplatin or enrollment on a clinical
trial testing novel agents followed by either radical
cystectomy or radiotherapy.

3. A.
Kamat et al reported 44 patients with micropapillary
non–muscle-invasive bladder cancer. 67% of 27
patients treated with BCG progressed to muscle-
invasive cancer, including 22% with metastatic dis-
ease. 30 patients underwent cystectomy and only
19% remained alive with their bladder in place. On
this basis, the current recommendation is to proceed
directly to cystectomy. This patient should be offered
nerve-sparing surgery, which is not associated with
an increased risk of local pelvic recurrence.

CHAPTER 18: BLADDER CANCER 581


4. E. 9. A.
The distal two-thirds of the female urethra may serve A single focus of CIS of the prostatic urethra does
as an adequate sphincter mechanism, provided the not increase the risk of a second primary urothelial
risk of cancer in the retained urethra is low. Anterior tumor of the retained urethra and is therefore not an
vaginal wall involvement by a posterior-based blad- indication for urethrectomy. Furthermore, the prob-
der tumor or bladder neck or urethra involvement is ability of developing a second primary TCC of the
a contraindication to urethra-sparing and orthotopic retained urethra is lower with orthotopic diversion
bladder replacement. compared to cutaneous diversion.

5. E. 10. E.
The pathologic pT0 rate is only 50% in patients with Patients with a history of smoking may also develop
an apparent clinical complete response to neoadju- secondary tumors, such as lung cancer. In this case,
vant chemotherapy. There is a highly select group of a single lesion may represent a primary tumor which
patients who may survive long-term with could be completely resected.
chemotherapy only and should be reserved for
patients who are not medically fit or refuse cystec- 11. D.
tomy. The loss of afferent input and passive urethral resis-
tance result in the inability to raise urethra resting
6. B. pressure during filling of the neobladder. As
High p53 and low Rb suggest altered expression of neobladder pressures rise with filling, this will over-
these gene products, often due to mutation and asso- come urethra resting pressure resulting in inconti-
ciated with increased risk for progression after cys- nence. Patients may be able to preempt leakage by
tectomy. Ki67 is a proliferation marker and setting an alarm once or twice at night to void.
increased expression is associated with aggressive
pathologic features and decreased long-term sur- 12. A.
vival. E-Cadherin is a cell adhesion molecule and Stomal stenosis results in stasis within the conduit
decreased expression is associated with increased and increased pressures, resulting in bilateral
risk of node metastasis and decreased survival. hydronephrosis.

7. B.
Adjuvant cisplatin-based combination chemother-
apy can be considered. Randomized trials have thus
far not been definitive in overall survival on this sub-
ject. A meta-analysis suggests a 9% absolute benefit
in overall survival, but the trials represent small
numbers of patients, often closed early or due to poor
accrual and not all of the patients in adjuvant
chemotherapy trials are represented.

8. A.
The risk of occult nodal metastases is as high as
50%. Neoadjuvant chemotherapy with cisplatin-
based combination chemotherapy has demonstrated
a 9% absolute benefit in overall survival in a meta-
analysis utilizing individual patient data on 3,005
patients from 11 randomized trials treated with
neoadjuvant chemotherapy. NCCN guidelines (V
2.2011) recommend neoadjuvant chemotherapy
with gemcitabine and cisplatin or M-VAC.

582 EDUCATIONAL REVIEW MANUAL IN UROLOGY


Chapter 19:
Penile and Urethral
Cancer
Kenneth W. Angermeier, MD

Contents

1. Premalignant Penile Lesions

2. HPV-Related Lesions

3. Penile Squamous Cell Carcinoma

4. Uncommon Penile Cancers

5. Male Urethral Cancer

6. Female Urethral Cancer

7. Further Reading

8. Questions

CHAPTER 19: PENILE AND URETHRAL CANCER 583


1. Premalignant Penile Lesions

Cutaneous Horn Figure 1

• Solid protuberance resulting from cornification


of the epithelium over a preexisting skin lesion
Chronic lichen sclerosis with deformity of
the urethral meatus

• Associated with human papillomavirus


(HPV) type 16

• Consider excision for accurate histologic


evaluation

Lichen Sclerosis (LS)

• Also known as balanitis xerotica obliterans (BXO)


(Figure 1)

• Symptoms: pain; pruritis; bleeding; decreased


urinary stream

• White patch on the prepuce or glans that often


involves the urethral meatus

• May be associated with glanular or coronal


fissures, advancement of foreskin onto the glans,
meatal stenosis and/or urethral stricture

• Biopsy recommended to confirm diagnosis

• Previous reports have documented an association


with squamous cell carcinoma Reprinted with permission of the Cleveland Clinic

• Estimated risk of malignant transformation 4%–8%

• Treatment: topical steroid cream (betamethasone


or clobetasol); circumcision; possible need for
meatotomy or urethral reconstruction

Leukoplakia

• Whitish plaques often involving the urethral


meatus

• Biopsy necessary to confirm diagnosis and rule


out malignant change

• Associated with squamous cell carcinoma in situ


and verrucous carcinoma

• Treatment: surgical excision; laser; circumcision


to eliminate chronic inflammation

584 EDUCATIONAL REVIEW MANUAL IN UROLOGY


2. HPV-Related Lesions

Bowenoid Papulosis Figure 2

• Pigmented papules of the penile shaft in


younger men
Condyloma acuminata

• Associated with HPV-16

• Biopsy histologically consistent with squamous


cell carcinoma (SCC) in situ, but clinical course is
invariably benign

• Treatment: electrodessication; 5-fluorouracil


(FU) cream; laser; surgical excision

Condyloma Acuminatum

• Papillomatous genital lesions generally consid-


ered to be benign (Figure 2)

• Associated with HPV-6 and -11 most typically

• Histologically characterized by the koilocyte—a


cell with an atypical nucleus surrounded by an
empty cavity

• Subclinical flat “acetowhite” lesions may be


detected by applying 5% acetic acid to the
Reprinted with permission of the Cleveland Clinic

genitalia

• Treatment: 5% imiquimod cream; topical


podophyllin; intralesional interferon; laser; elec-
trocautery; surgical excision

• Associated in some cases with development of


SCC of the penis

Verrucous Carcinoma
(Buschke-Lowenstein tumor)

• Large condyloma that displaces and invades


adjacent tissues

• Associated with HPV-6 and 11

• Histologically no evidence of malignant change


and does not metastasize

• Treatment: cryosurgery; surgical excision with


close follow-up for recurrence

CHAPTER 19: PENILE AND URETHRAL CANCER 585


3. Penile Squamous
Cell Carcinoma

• Neonatal circumcision virtually eliminates risk of


developing invasive SCC (not so for circumcision
Carcinoma in Situ

• Erythroplasia of Queyrat: SCC in situ on the glans performed later in life)


or prepuce; red, velvety, well-marginated lesion
(Figure 3) • Associated most commonly with HPV-16, but also
with HPV-18, -31, -33

• Additional risk factors: poor hygiene, smoking,


phimosis, number of sexual partners, LS
Figure 3

• If untreated, most patients will die of their disease


within 2 years
SCC in situ of the glans penis extending into
the distal urethra

Presentation

• Primary tumor may be a nonhealing ulcer, papule,


indurated mass or exophytic growth

Figure 4

Large invasive penile SCC

Reprinted with permission of the Cleveland Clinic

• Bowen’s disease: SCC in situ located on penile


shaft

• May progress to invasive carcinoma in 10% of


cases

• Treatment: 5-FU cream; laser; surgical excision


including circumcision

Invasive SCC

• Accounts for 0.4%–0.6% of cancers among men


in the United States and Europe

• Primarily occurs in the preputial cavity of


uncircumcised older men

Reprinted with permission of the Cleveland Clinic

586 EDUCATIONAL REVIEW MANUAL IN UROLOGY


• Most common location is on the glans penis T1b Tumor invades subepithelial connective
(46%), followed by the prepuce (21%) tissue and either has lymphovascular
invasion or is poorly differentiated
• May be associated with penile bleeding or
purulent discharge T2 Tumor invades corpus spongiosum or
corpus cavernosum
• Rarely may present as an inguinal mass or drain-
ing abscess due to nodal metastases if the primary T3 Tumor invades urethra
tumor is obscured by phimosis
T4 Tumor invades other adjacent structures

Regional Lymph Nodes (N)


Diagnosis

• Delay in diagnosis is common N0 No palpable or visibly enlarged inguinal


lymph nodes
• Biopsy is necessary to confirm the diagnosis and
assess depth of invasion, tumor grade and pres- N1 Palpable mobile unilateral inguinal
ence of vascular invasion lymph node

• Basaloid and sarcomatoid histological variants are N2 Palpable mobile multiple or bilateral
more aggressive inguinal lymph nodes

• At diagnosis, thorough palpation of inguinal N3 Palpable fixed inguinal nodal mass


regions for lymphadenopathy is essential or pelvic lymphadenopathy, unilateral
or bilateral
• Computed tomography (CT) may be helpful in the
assessment of inguinal nodes in an obese patient Distant Metastasis (M)
or those with a history of inguinal surgery, and of M0 No distant metastasis
the pelvic nodes when the inguinal nodes are
palpable M1 Distant metastasis, or lymph node
metastasis outside the true pelvis
• Magnetic resonance imaging (MRI) is the best
study to obtain additional information about the • The most important factor determining survival is
extent of the primary tumor in some cases when the extent of lymph node metastases
considering organ-sparing surgery
• Factors associated with ultimate cure following
surgical treatment of lymphadenopathy include
unilateral nodal involvement, ≤2 positive inguinal
Staging and Prognosis

AJCC Staging for Penile Cancer nodes, no extranodal tumor extension and no
pelvic metastases
Primary Tumor (T)
TX Primary tumor cannot be assessed Surgical Treatment—Primary Tumor

T0 No evidence of primary tumor Laser Therapy


• Advantage of eliminating tumor with preservation
Tis Carcinoma in situ of surrounding tissues and penile function

Ta Noninvasive verrucous carcinoma • Best suited for Tis, small Ta, T1 tumors and for
patients with manageable T2 tumors who refuse
T1a Tumor invades subepithelial connective more aggressive surgical treatment
tissue without lymphovascular invasion
and is not poorly differentiated
CHAPTER 19: PENILE AND URETHRAL CANCER 587
• CO2 laser primarily suitable for Tis Figure 5b

• Try to achieve a 3–5 mm margin around the tumor Outer preputial flap outlined

• Local recurrence rate approximately 20%

Mohs Micrographic Surgery


• Excision of tissue in thin layers with immediate
microscopic examination and tumor mapping to
eliminate malignancy with maximal preservation
of local tissues

• Requires significant technical support and


surgeon experience

• Useful for Tis or small superficially invasive


Reprinted with permission of the Cleveland Clinic

tumors

• Local recurrence rates 6%


Figure 5c

Conservative Surgical Excision


• Recent studies have indicated that a 2 cm surgical
Tumor excised and circumcision performed

margin may not always be required for effective


local control of SCC penis

• Excisional biopsy with primary closure

• Circumcision

• Glans tumor excision with defect closure using an


outer preputial flap or a split-thickness skin graft
(STSG)
Figures 5a-d through 6: Reprinted with permission of the Cleveland Clinic
Surgical glans defect covered
with outer preputial flap
Figure 5a Figure 5d

Superficial glans tumor Glans defect filled with outer preputial flap

Reprinted with permission of the Cleveland Clinic Reprinted with permission of the Cleveland Clinic

588 EDUCATIONAL REVIEW MANUAL IN UROLOGY


Figure 6 Figure 7b

STSG to glans penis following excision of Corpora transected and urethra spatulated
extensive SCC in situ

Reprinted with permission of the Cleveland Clinic

Figure 7c

Glans defect filled with outer preputial flap

Reprinted with permission of the Cleveland Clinic

Figures 7a-7d: Partial penectomy


Reprinted with permission of the Cleveland Clinic

Figure 7a Figure 7d

Incision with ligation and division of dorsal Final closure with creation of urethrostomy
penile vessels

Reprinted with permission of the Cleveland Clinic Reprinted with permission of the Cleveland Clinic

CHAPTER 19: PENILE AND URETHRAL CANCER 589


• Glansectomy: complete removal of the glans
penis with preservation of the corporal bodies and
Figures 8a-8e: Total penectomy

possible application of STSG to the corporal tips

• Intraoperative frozen sections are critical to help


confirm complete tumor excision
Figure 8a

• Recently reported local recurrence rates 8%–11%


Incision

• Requires careful long-term follow-up

Partial Penectomy
• Most commonly performed surgical procedure for
the treatment of the primary tumor in patients
with invasive SCC

• Amputation of the penis at least 2 cm proximal to


the tumor (Figures 7a-7c)

• Additional goal is to preserve the ability to void in


a standing position

• Provides excellent local control with a recurrence


rate of 0%–8%

• Adequate sexual function postoperatively: 20%

Total Penectomy
• Indicated for penile tumors whose size or location
would not allow an adequate surgical margin and
preservation of upright voiding (Figures 8a-8e)

• Amputation of the penis at the base at the level of


the suspensory ligament
Reprinted with permission of the Cleveland Clinic

• Creation of perineal urethrostomy

• Excellent local control rates have been reported

590 EDUCATIONAL REVIEW MANUAL IN UROLOGY


Figure 8b
Figure 8d
Transection of the corpora near the level of
the pubis
Transposition of the urethra through
a perineal incision

Reprinted with permission of the Cleveland Clinic


Reprinted with permission of the Cleveland Clinic

Figure 8e
Figure 8c

Completion of perineal urethrostomy


Mobilization of the remaining urethra off of
the proximal corporal bodies

Reprinted with permission of the Cleveland Clinic Reprinted with permission of the Cleveland Clinic

CHAPTER 19: PENILE AND URETHRAL CANCER 591


• Deep nodes
* Within the femoral canal medial to the femoral
Management of Regional Lymph Nodes

Anatomy vein, and between it and the lacunar ligament


• SCC spreads initially to the regional lymph nodes
before the occurrence of distant metastases * Most cephalad node in this group is the lymph
node of Cloquet
• First echelon drainage is to the inguinal lymph
nodes • Second echelon drainage is from the deep inguinal
nodes to the ipsilateral external iliac nodes
• Metastasis from the primary tumor to the superfi-
cial nodes may occur bilaterally Figure 10

• Superficial nodes: Deep inguinal lymph nodes

* 4–25 nodes in the deep membranous layer of the


superficial fascia of the thigh

Figure 9

Superficial inguinal lymph nodes and


branches of the saphenous vein

SEV

Lymph node of Cloquet

SEPV

SCIV

MCV
LCV
Reprinted with permission of the Cleveland Clinic

Observation
• In the setting of palpably negative groins, observa-
tion of the inguinal regions is reasonable for the
following tumors due to the low risk of metastatic
SEV, superficial epigastric; SEPV, superficial external
disease:

* Tis, Ta
pudendal; MCV, medical cutaneous; LCV, lateral
cutaneous; SCIV, superficial circumflex iliac

* T1, grade I or II and no lymphovascular invasion


Reprinted with permission of the Cleveland Clinic

592 EDUCATIONAL REVIEW MANUAL IN UROLOGY


Antibiotics Modified Inguinal Lymphadenectomy
• Traditional teaching has been administration of • Key aspects:
antibiotics for 4–6 weeks in the setting of palpable
inguinal lymph nodes to resolve possible inflam- * Shorter skin incision
mation sparing the patient an operation
* Exclusion of the area lateral to the femoral
• This approach is reasonable in patients who are at artery and caudal to the fossa ovalis
low risk of having metastatic disease
* Preservation of the saphenous vein
• However, when lymphadenopathy is marked and
metastatic disease is strongly suspected, antibi- * Elimination of sartorius transposition
otics will help decrease the inflammatory compo-
nent but surgical treatment should not be unduly • Removal of the superficial nodes within the
delayed. If needed, diagnosis can be confirmed by described area and the deep nodes medial to the
fine needle aspiration cytology of the enlarged femoral vein to the level of the inguinal ligament
node

Sentinel Node Biopsy (SNB)


• Proposed by Cabanas in 1977
Figure 11

• Impetus for a conservative approach to the


Modified inguinal lymphadenectomy

inguinal nodes was the significant morbidity asso-


ciated with inguinofemoral lymphadenectomy

• A finger is inserted through a small incision paral-


lel to the inguinal crease and toward the pubic
tubercle, where the lymph node is palpated and
excised

• This technique was ultimately found to result in a


significant number of false negative exams and
has been abandoned
Femoral artery Inguinal
ligament

• However, recent improvement of the false nega-


tive rate of SNB has been noted with technical
modifications including: lymphoscintigraphic
imaging of the sentinel node, lymphatic tracking
Saphenous

using injection of blue dye around the primary


vein

tumor, inguinal ultrasonography with fine needle


aspiration of abnormal nodes, intraoperative pal-
pation of the wound and extended pathologic
analysis of the excised specimen. This technique
is referred to as dynamic sentinel lymph node
biopsy and has been used successfully at selected
high volume centers
Lymph node packet is medial to the femoral artery
and includes superficial and deep inguinal nodes

Reprinted with permission of the Cleveland Clinic

CHAPTER 19: PENILE AND URETHRAL CANCER 593


• Indications: * Superiorly by a line drawn from the superior
margin of the external inguinal ring to the ante-
* Patients with a primary tumor that places them rior superior iliac spine (ASIS)
at increased risk for inguinal metastases and
clinically negative groins on examination * Laterally by a line drawn from the ASIS inferi-
should undergo surgical assessment of the orly for 20 cm
inguinal lymph nodes
* Medially by a line drawn from the pubic tuber-
• Stage T2 or greater cle for 15 cm

• Stage T1—grade III SCC * Inferiorly by the inferior boundary of the


femoral triangle (Figure 12)
• Lymphovascular invasion

* Palpably negative groin with metastatic disease


involving the contralateral side Figure 12

• However, if metastatic disease develops in an


inguinal region unilaterally at a later date after ini-
Areas for removal of superficial and deep

tial treatment of the primary tumor, contralateral


inguinal lymph nodes

inguinal lymphadenectomy in the absence of pal-


pable disease is not necessary

• During the procedure, if frozen section examina-


tion demonstrates metastatic disease, the proce-
dure is converted to a standard extended
inguinofemoral lymphadenectomy

• Ipsilateral pelvic nodes are virtually never posi-


tive in the lymphadenectomy when the inguinal
nodes are negative

• Complications (usually self-limited): seroma


(25%), lymphorrhea (10%), wound infection or
necrosis (0%–9%), temporary lower extremity
edema (20%)

Radical Ilioinguinal Lymphadenectomy


• Indicated in patients with resectable metastatic
adenopathy and may be curative when the disease
is limited to the inguinal nodes.

• Aggressive lymphadenectomy is associated with


A: Possible incisions

potential cure in 30%–60% of cases


for: 1) inguinofemoral
lymph node

• Removal of superficial and deep inguinal lymph


dissection (LND);

nodes within an area outlined:


2) Unilateral pelvic LND; 3) Bilateral pelvic LND
B: Single incision approach for ilioinguinal LND

Reprinted with permission of the Cleveland Clinic

594 EDUCATIONAL REVIEW MANUAL IN UROLOGY


• Possible incisions (Figures13a-13b) • Detach sartorius muscle from ASIS and rotate to
cover exposed femoral vessels (Figures 14a-14b)
• Fascia lata is opened longitudinally over the
sartorius muscle at the lateral boundary of • Pelvic lymphadenectomy is generally indicated in
the dissection the setting of positive ipsilateral inguinal nodes,
and should include the distal common iliac, exter-
• Care is taken to avoid the femoral nerve located nal iliac and obturator nodes
lateral to the femoral artery and below the
iliacus fascia • Contemporary complication rates: Minor
40%–50%; Major (marked lymphedema, flap
• Saphenous vein is usually ligated and resected necrosis, symptomatic lymphocele) 5%–21%
with the specimen, but may be preserved in
selected cases of low volume metastatic disease

Figures 13a-13b: Dissections for radical inguinofemoral LND

Figure 13a Figure 13b

Initial dissection for radical inguinofemoral Inferior dissection during radical


LND with exposure of superior border inguinofemoral LND with removal of lymph
defined by the external oblique fascia node packet from the inferior border of the
femoral triangle

Anterior superior
iliac spine

Inguinal ligament

Sartorius
muscle

Femoral nerve
(under iliacua fascia)

Femoral canal

Reprinted with permission of the Cleveland Clinic Reprinted with permission of the Cleveland Clinic

CHAPTER 19: PENILE AND URETHRAL CANCER 595


• Palliative ilioinguinal lymphadenectomy is rea- • Complications: urethral fistula, urethral stricture,
sonable in selected patients with surgically incur- necrosis, pain, edema
able disease to prevent complications of infection,
chronic abscess with foul-smelling drainage or • Radiation therapy to the inguinal areas is not as
life-threatening femoral hemorrhage effective as surgical therapy, and may result in sig-
nificant complications of skin ulceration or break-
Radiation Therapy down and lymphedema

• Very little role in treatment of penile SCC Chemotherapy

• May be considered for treatment of selected pri- • Indicated for unresectable locoregional tumor or
mary tumors: distant metastatic disease

* Small, superficial, noninvasive lesions on the • Sites of distant metastases: lung, bone, liver
glans penis
• Most commonly used agents: cisplatin,
* Patients who refuse surgery bleomycin, methotrexate

• External beam radiotherapy or brachytherapy • Responses to multidrug regimens are partial and
have been used generally of short duration

• Local control rates are inferior to surgical therapy, • In patients with unresectable locoregional disease,
and a significant number of patients will require chemotherapy may allow subsequent tumor resec-
subsequent penectomy tion in some cases.

Figures 14a-14b: Left saphenous-


sparing inguinofemoral LND
Figure 14a Figure 14b

Adductor longus = AL; Femoral vein = V; Reprinted with permission of the Cleveland Clinic
Femoral artery = A; Sartorius = S; Saphenous vein = SV
Reprinted with permission of the Cleveland Clinic

596 EDUCATIONAL REVIEW MANUAL IN UROLOGY


4. Uncommon Penile Cancers

Melanoma Metastatic Disease

• Brown or bluish black papule or ulceration, most • Most commonly from cancer of the bladder,
typically on the glans penis prostate or rectum

• Surgery is treatment of choice • Most frequently presents as priapism (Figures


15a-15b)
* Foreskin lesion—circumcision
• Tumor usually infiltrates one or both corpora cav-
* Glans penis—partial penectomy ernosa; mass lesions may occur but are less com-
mon
* Penile shaft—partial or total penectomy
• Associated with advanced disease, and subse-
• Prognosis depends on tumor stage and extent of quent survival is often limited
lymph node metastasis

Sarcomas Figure 15a

• Very rare Priapism related to corporal infiltration by

Epithelioid Sarcoma
hormone refractory prostate cancer

• May mimic Peyronie’s disease clinically with cor-


responding delay in diagnosis

Kaposi’s Sarcoma
• Tumor of the reticuloendothelial system

• Raised, painful bluish papule or ulcer

• Often associated with human herpesvirus 8 and


AIDS

Others
• Angiosarcoma, leiomyosarcoma, malignant
fibrous histiocytoma, osteosarcoma,

Treatment
• Aggressive local tumor excision with partial or
total penectomy if needed

• Wide surgical margins are necessary as local


recurrences are typical of sarcomas

• Withdrawal of immunosuppression or radiother-


apy may be effective in some cases of Kaposi’s
Reprinted with permission of the Cleveland Clinic

sarcoma

CHAPTER 19: PENILE AND URETHRAL CANCER 597


5. Male Urethral Cancer

Figure 15b General

Resolution of priapism following external • Rare and usually presents in the 5th decade of life

• Etiologic factors:
beam radiotherapy

* Chronic inflammation due to urethral stricture,


urethritis, sexually transmitted disease

* Probably HPV-16

• Onset is usually insidious and a high index of sus-


picion is needed

• Presenting symptoms:

* Urethral bleeding

* Palpable mass

* Obstructive voiding symptoms (Figure 16)

Figure 16

Cystoscopic view of SCC of the


bulbous urethra
Reprinted with permission of the Cleveland Clinic

Reprinted with permission of the Cleveland Clinic

598 EDUCATIONAL REVIEW MANUAL IN UROLOGY


Anatomy and Pathology Evaluation and Staging

• Tumor location: • Cystoscopy with biopsy

* Bulbomembranous urethra • Bimanual examination under anesthesia


(posterior urethra): 60%
• Urine cytology is not very reliable for diagnosis of
* Penile urethra (anterior urethra): 30% primary urethral carcinoma, with sensitivity being
greatest in men with TCC
* Prostatic urethra: 10%
• CT scan to evaluate for metastatic disease
• Tumor pathology:
• MRI is helpful in detecting invasion of the corpora
* Bulbomembranous urethra: cavernosa (Figure 17)
SCC—80%;
transitional cell carcinoma (TCC)—10%;
adenocarcinoma —10%

* Penile urethra—SCC 90%; TCC 10%


Figure 17

* Prostatic urethra—TCC 90%; SCC 10%


MRI of patient with a locally extensive SCC
of the bulbous urethra

• Spread is by direct extension to adjacent struc-


tures including corpus spongiosum and peri-
urethral tissues

• Lymphatic spread:

* Anterior urethra—superficial and


deep inguinal nodes

* Posterior urethra—pelvic nodes

• Palpable inguinal nodes occur in 20% of patients


and always represent metastatic disease

Reprinted with permission of the Cleveland Clinic

CHAPTER 19: PENILE AND URETHRAL CANCER 599


TNM Staging System for Urethral Cancer Surgical Treatment

Primary Tumor (T) • Anterior urethral cancer is more amenable to sur-


Tx Primary tumor cannot be assessed gical control and the prognosis is better when
compared to posterior urethral cancer
T0 No evidence of primary tumor
Penile Urethra
Ta Noninvasive papillary, polypoid or verru- • Superficial, low grade tumors: transurethral
cous carcinoma resection, local excision, distal urethrectomy

Tis Carcinoma in situ • Infiltrating tumors (Figure 18)

T1 Tumor invades subepithelial connective * Distal half of the penis: partial penectomy
tissue
• Ilioinguinal lymphadenectomy is indicated in the
T2 Tumor invades corpus spongiosum, presence of palpable inguinal adenopathy in the
prostate, or periurethral muscle absence of pelvic or distant metastatic disease

T3 Tumor invades corpus cavernosum, beyond • As opposed to penile cancer, prophylactic or early
prostate capsule, anterior vagina, bladder inguinal lymphadenectomy in the absence of pal-
neck pable adenopathy has not been shown to be of
benefit
T4 Tumor invades other adjacent organs
Bulbomembranous Urethra
Regional Lymph Nodes (N) • Poor survival figures have been reported for all
Nx Regional lymph nodes cannot be assessed forms of treatment, but it appears that radical exci-
sion offers the best chance for local and long-term
N0 No regional lymph node metastasis disease control

N1 Metastasis in a single lymph node, 2 cm or • Early, small lesions have been successfully treated
less in size with transurethral resection or segmental urethral
excision, but tumors appropriate for this are very
N2 Metastasis in a single lymph node 2–5 cm in rare
size or multiple nodes with none > 5 cm
• Standard surgical treatment for invasive posterior
N3 Metastasis in a lymph node > 5 cm urethral cancer is total penectomy with cysto-
prostatectomy, and may include resection of the
Distant Metastasis (M) inferior aspect of the pubic rami (Figure 19)
MX Presence of distant metastasis cannot be
assessed • Proximal penile shaft: total penectomy

M0 No distant metastasis • Large pelvic floor defects may require a rectus


abdominis muscle flap or myocutaneous flap for
M1 Distant metastasis closure

600 EDUCATIONAL REVIEW MANUAL IN UROLOGY


* Post-cystectomy monitoring: Voided urine
cytology
Radiotherapy and Chemotherapy

• Radiation as monotherapy is generally reserved


for patients with early-stage lesions of the anterior * Symptoms: hematuria, urethral discharge or
urethra who refuse surgery mass

• Combination of radiation and chemotherapy has * Treatment: total urethrectomy with excision of
shown some success in a small number of patients a cuff of pouch adjacent to the anastomosis and
with localized and metastatic urethral cancer cutaneous diversion often using a portion of the
existing neobladder
• Patients with advanced stage or metastatic disease
are most commonly treated with a multimodal
approach

Urethral Cancer Recurrence After


Cystoprostatectomy

• Approximately 40% of urethral recurrences are


diagnosed within 1 year after cystoprostatectomy
with a median time to diagnosis of 18 months

• Cutaneous diversion

* Incidence of urethral recurrence: 2.1%–11.1%

* Prostatic stromal invasion by TCC increases


risk of urethral recurrence

* Post-cystectomy monitoring: urethral wash


cytology

* Symptoms: urethral bleeding, discharge or


mass

* CT and/or MRI may be needed to assess for


metastatic disease or local tumor extension

* Treatment: total urethrectomy including the ure-


thral meatus (Figures 20a-20d)

• Orthotopic diversion

* Incidence of urethral recurrence: 0.5%–4%

* Negative frozen section of distal prostatic ure-


thral margin at the time of cystoprostatectomy
results in low incidence of recurrence

CHAPTER 19: PENILE AND URETHRAL CANCER 601


Figure 18

Penile mass due to TCC of the penile urethra following previous cystoprostatectomy for
TCC of the bladder

Reprinted with permission of the Cleveland Clinic

Figure 19

Surgical specimen from total penectomy, urethrectomy, cystoprostatectomy and inferior


pubectomy for a large bulbous urethral SCC

Reprinted with permission of the Cleveland Clinic

602 EDUCATIONAL REVIEW MANUAL IN UROLOGY


Figures 20a-20d: Secondary urethrectomy following previous cystoprostatectomy

Figure 20a Figure 20c

Perineal incision Distal urethral dissection which then con-


nects to the proximal dissection at the level
of the distal shaft

Reprinted with permission of the Cleveland Clinic

Reprinted with permission of the Cleveland Clinic


Figure 20b

Division of bulbospongiosus muscle to Figure 20d


expose the bulb of the corpus spongiosum,
and initial dissection of the urethra off the Sagittal view demonstrating posterior bulb
corporal bodies dissection and location of the bulbar
arteries

Bulbar a.

Reprinted with permission of the Cleveland Clinic Reprinted with permission of the Cleveland Clinic

CHAPTER 19: PENILE AND URETHRAL CANCER 603


6. Female Urethral Cancer

General * CT to evaluate for metastatic disease

• Rare: 0.02% of all female cancers * MRI is the best test for imaging local urethral
anatomy and tumor extent
• Most commonly diagnosed in the 5th and 6th
decades of life Treatment

• Etiology: leukoplakia, chronic irritation, • Most studies have not detected any significant dif-
caruncles, polyps, HPV ferences in survival based on histologic tumor
subtype
• 5% arise within a diverticulum
• Distal urethral carcinoma tends to be low stage
• Symptoms: obstructive voiding, dysuria, urinary and is associated with improved survival com-
frequency, urethral bleeding or mass pared to proximal tumors

• Most significant prognostic factor for local con-


trol and survival is the anatomic location and
Anatomy and Pathology

• Divisions of female urethra extent of the primary tumor

* Anterior—Distal third Distal Tumors

* Posterior—Proximal two-thirds Surgery


• Circumferential distal urethrectomy
• The distal third of the urethra may be excised (70%–90% cure rate)
without compromising continence
• Complications: meatal stenosis, incontinence
• Lymphatic drainage
* Ilioinguinal lymphadenectomy only when
* Anterior urethra—superficial and deep inguinal palpable disease and no distant metastases
nodes
Radiotherapy
* Posterior urethra—iliac and obturator nodes • May use external beam, brachytherapy or combi-
nation of both
• Pathology
• 5-year survival rates equivalent to surgery in some
* SCC 50%–70% series

* Adenocarcinoma 25% (increased incidence • Complications: incontinence, stricture,


within urethral diverticula) fistula, infection

* TCC 10%

• Evaluation

* Cystoscopy with biopsy

* Bimanual examination under anesthesia

604 EDUCATIONAL REVIEW MANUAL IN UROLOGY


7. Further Reading

1. Agrawal A, Pai D, Ananthakrishnan N, Smile


SR, Ratnakar C.. The histological extent of the
Proximal Tumors

• More likely to be high stage and invade the bladder local spread of carcinoma of the penis and its
and vagina therapeutic implications. BJU Int. 2000;85:
299-301.
Surgery
• Anterior exenteration with wide vaginal excision 2. Bandieramonte G, Lepera P, Marchesini R,
and pelvic lymphadenectomy Andreola S, Pizzocaro G.. Laser microsurgery
for superficial lesions of the penis. J Urol.
• Suboptimal cure rates with surgery alone have led 1987;138:315-319.
to recommendations for multimodality therapy in
most cases 3. Bevan-Thomas R, Slaton JW, Pettaway CA.
Contemporary morbidity from lymphadenec-
Multimodality Therapy tomy for penile squamous cell carcinoma: the
• Radiotherapy + chemotherapy (5-FU and M.D. Anderson Cancer Center Experience.
mitomycin C for SCC; MVAC for TCC) J Urol. 2002;167:1638-1642.

• Radiotherapy + anterior exenteration 4. Bissada NK, Yakout HH, Fahmy WE, et al.
Multi-institutional long-term experience with
conservative surgery for invasive penile carci-
noma. J Urol. 2003;169:500-502.

5. Clark PE, Stein JP, Groshen SG, et al. The


management of urethral transitional cell carci-
noma after radical cystectomy for invasive
bladder cancer. J Urol. 2004;172:1342-1347.

6. Coblentz TR, Theodorescu D. Morbidity of


modified prophylactic inguinal lymphadenec-
tomy for squamous cell carcinoma of the
penis. J Urol. 2002;168:1386-1389.

7. Colberg JW, Andriole GL, Catalona WJ.


Long-term follow-up of men undergoing mod-
ified inguinal lymphadenectomy for carci-
noma of the penis. Br J Urol. 1997;79:54-57.

8. Culkin DJ, Beer TM. Advanced penile carci-


noma. J Urol. 2003; 170:359-365.

9. Dalbagni G, Zhang ZF, Lacombe L, Herr HW.


Male urethral carcinoma: analysis of treatment
outcome. Urology. 1999;53:1126-1132.

10. Dalbagni G, Donat SM, Eschwège P, Herr


HW, Zelefsky MJ. Results of high dose rate
brachytherapy, anterior pelvic exenteration
and external beam radiotherapy for carcinoma
of the female urethra. J Urol. 2001;166:1759-
1761.

CHAPTER 19: PENILE AND URETHRAL CANCER 605


11. Dimarco DS, Dimarco CS, Zincke H, et al. 20. Malloy TR, Wein AJ, Carpiniello VL. Carci-
Surgical treatment for local control of female noma of penis treated with neodymium YAG
urethral carcinoma. Urol Oncol. 2004;22: laser. Urology. 1988;31:26-29.
404-409.
21. McDougal WS, Kirchner FK Jr, Edwards RH,
12. Fleming ID, Cooper JS, Henson, DE, et al. Killion LT. Treatment of carcinoma of the
Penis. AJCC Cancer Staging Manual. Ameri- penis: the case for primary lymphadenectomy.
can Joint Committee on Cancer. 5th ed. J Urol. 1986;136:38-41.
Philadelphia, PA: Lippincott-Raven, 1997:
215-217. 22. McDougal WS. Carcinoma of the penis:
improved survival by early regional lym-
13. Forman JD, Lichter AS. The role of radiation phadenectomy based on the histological grade
therapy in the management of carcinoma of and depth of invasion of the primary lesion.
the male and female urethra. Urol Clin North J Urol. 1995; 154:1364-1366.
Am. 1992;19:383-389.
23. Mohs FE, Snow SN, Larson PO. Mohs micro-
14. Haas GP, Blumenstein BA, Gagliano RG, et al. graphic surgery for penile tumors. Urol Clin
Cisplatin, methotrexate and bleomycin for the North Am. 1992;19:291-304.
treatment of carcinoma of the penis: a South-
west Oncology Group Study. J Urol. 24. Oberfield RA, Zinman LN, Leibenhaut M,
1999;161:1823-1825. Girschovich L, Silverman ML. Management
of invasive squamous cell carcinoma of the
15. Hardeman SW, Soloway MS. Urethral recur- bulbomembranous male urethra with co-ordi-
rence following radical cystectomy. J Urol. nated chemo-radiotherapy and genital preser-
1990;144:666-669. vation. Br J Urol. 1996;78:573-578.

16. Hoffman MA, Renshaw AA, Loughlin KR. 25. Parra RO. Accurate staging of carcinoma of
Squamous cell carcinoma of the penis and the penis in men with nonpalpable inguinal
microscopic pathologic margins: how much lymph nodes by modified inguinal lym-
margin is needed for local cure? Cancer. phadenectomy. J Urol. 1996;155:560-563.
1999;85:1565-1568.
26. Pietrzak P, Corbishley C, Watkin N. Organ-
17. Horenblas S, van Tinteren H, Delemarre JF, sparing surgery for invasive penile cancer:
Boon TA, Moonen LM, Lustig V. Squamous early follow-up data. BJU Int. 2004;94:
cell carcinoma of the penis. II. Treatment of 1253-1257.
the primary tumor. J Urol. 1992;147:1533-
1538. 27. Slaton JW, Morgenstern N, Levy DA, et al.
Tumor stage, vascular invasion and the per-
18. Horenblas S, Jansen L, Meinhardt W, Hoef- centage of poorly differentiated cancer: inde-
nagel CA, de Jong D, Nieweg OE. Detection pendent prognosticators for inguinal lymph
of occult metastasis in squamous cell carci- node metastasis in penile squamous cancer.
noma of the penis using a dynamic sentinel J Urol. 2001;165:1138-1142.
node procedure. J Urol. 2000;163:100-104.
28. Ubrig B, Waldner M, Fallahi M, Roth S.
19. Licht MR, Klein EA, Bukowski R, Montie JE, Preputial flap for primary closure after exci-
Saxton JP. Combination radiation and sion of tumors on the glans penis. Urology.
chemotherapy for the treatment of squamous 2001;58:274-276.
cell carcinoma of the male and female urethra.
J Urol. 1995;153:1918-1920. 29. Zeidman EJ, Desmond P, Thompson IM. Sur-
gical treatment of carcinoma of the male ure-
thra. Urol Clin North Am. 1992;19:359-372.
606 EDUCATIONAL REVIEW MANUAL IN UROLOGY
8. Questions

4. The following statements pertaining to conser-


vative surgical excision for penile carcinoma
Penile Urethral Cancer

1. Where do penile cancers most commonly arise? are true, except for which one?

A. Shaft A. Glansectomy and circumcision to remove


the entire contents of the preputial cavity
B. Prepuce
B. Large defects after glans tumor excision may
C. Frenulum be covered with a flap of outer preputial skin

D. Glans C. Frozen section biopsies are usually not


needed during these procedures
E. None of the above
D. Careful postoperative long-term surveillance
is necessary
2. Which of the following is not a risk factor for
invasive SCC? E. Circumcision alone may be sufficient to treat
certain preputial tumors
A. Phimosis

B. Number of sexual partners 5. Which of the following statements about partial


penectomy is most accurate?
C. Smoking
A. It provides for normal sexual function in over
D. Poor hygiene 70% of men

E. Alcohol intake B. It is performed less often than total penec-


tomy

3. What is the most important prognostic factor for C. Postoperative voiding is through a perineal
survival in patients with penile cancer? urethrostomy

A. Primary tumor stage D. It results in local recurrence rates of less than


10%
B. Extent of lymph node metastasis
E. It requires division of the penis at least
C. Presence of vascular invasion 3–4 cm proximal to the tumor

D. Primary tumor grade

E. Medical comorbidities

CHAPTER 19: PENILE AND URETHRAL CANCER 607


6. Which of the following statements regarding D. The thigh incision is shorter than that used
the progression of penile cancer is true? for standard ilioinguinal lymphadenectomy

A. Metastasis initially involves the superficial E. Both superficial and deep inguinal nodes are
inguinal nodes included in the surgical specimen

B. Metastatic spread from the primary tumor is


usually unilateral 9. Which of the following statements regarding
ilioinguinal lymphadenectomy is true?
C. Metastasis initially is hematogenous to the
lung, liver or bone A. Rotation of the gracilis muscle is performed
to cover the femoral vessels
D. Metastasis initially involves the deep
inguinal nodes B. It is done only for palliation

E. Crossover from the inguinal nodes to the con- C. Complications are few and minor in nature
tralateral pelvic nodes is common
D. The saphenous vein may be preserved in the
setting of low-volume metastatic disease
7. Observation of the inguinal regions is reason-
able when there is no palpable adenopathy and E. Pelvic node dissection is necessary even if
the primary tumor demonstrates all of the fol- the unilateral inguinal nodes are negative
lowing, except?

A. Tis 10. What is the most frequent site of urethral cancer


in the male?
B. T1, grade II
A. Fossa navicularis
C. Vascular invasion
B. Prostatic urethra
D. Ta
C. Pendulous urethra
E. T1, grade I
D. Penoscrotal urethra

8. All of the following statements pertaining to E. Bulbomembranous urethra


modified inguinal lymphadenectomy are true,
except for:
11. Which of the following statements concerning
A. The saphenous vein is preserved distal urethral cancer in the male is true?

B. It is indicated for management of palpable A. Most common histologic type is transitional


inguinal lymphadenopathy cell carcinoma

C. The dissection excludes regions lateral to the B. Penectomy is usually indicated for tumors
femoral artery infiltrating the corpus spongiosum

608 EDUCATIONAL REVIEW MANUAL IN UROLOGY


C. Prognosis is worse than for bulbomembra-
nous urethral cancer
Answers

1. D.
D. In the absence of palpable inguinal nodes, Penile cancers occur most commonly on the glans
early inguinal lymphadenectomy is indicated penis (48%) followed by the prepuce (21%).

E. Conservative surgical therapy is never effective 2. E.


There is no evidence linking penile cancer to alco-
hol intake.
12. What is the most common histologic type of
proximal urethral cancer in women? 3. B.
The presence and extent of inguinal lymph node
A. Squamous cell carcinoma metastasis are the most important prognostic factors
for survival in patients with SCC penis.
B. Transitional cell carcinoma
4. C.
C. Adenocarcinoma Frozen section biopsies are often a critical compo-
nent of conservative surgery for penile cancer to
D. Melanoma help ensure complete tumor excision.

E. Sarcoma 5. D.
Partial penectomy results in a local recurrence rate
of 0%–8%. It provides for adequate sexual function
13. What is the most significant prognostic factor in a low percentage of men, is performed more com-
for local control and survival in female urethral monly than total penectomy, does not result in a
cancer? perineal urethrostomy and traditionally is done with
a 2-cm tumor margin.
A. Age at presentation
6. A.
B. Histologic type Metastasis initially occurs to the superficial inguinal
nodes, and this may be unilateral or bilateral. Pro-
C. Anatomic location and extent of primary gression is subsequently to the deep inguinal nodes
tumor and then the pelvic nodes. Distant metastasis occurs
late. Pelvic nodes will not be positive if the ipsilat-
D. Hematuria eral inguinal nodes are negative.

E. Presence of urethral diverticulum 7. C.


Vascular invasion in the primary tumor is an indica-
tion for modified inguinal lymphadenectomy in the
setting of clinically negative groins.

8. B.
In the setting of palpable adenopathy, more exten-
sive complete ilioinguinal lymphadenectomy is
indicated.

CHAPTER 19: PENILE AND URETHRAL CANCER 609


9. D.
In the setting of low-volume metastatic disease, the
saphenous vein may be preserved in order to try to
decrease the risk of postoperative complications.
The sartorius muscle is used to cover the femoral
vessels, and the procedure may carry a chance of
cure in 30%–60% of cases when pelvic nodes are
not involved.

10. E.
In males, urethral cancer occurs most commonly in
the bulbomembranous urethra (60%), followed by
the penile urethra (30%) and prostatic urethra
(10%).

11. B.
Penectomy is indicated for tumors infiltrating the
corpus spongiosum. The most common histologic
type is SCC and the prognosis is better than that for
bulbomembranous cancers. Early or prophylactic
has not been shown to be advantageous in urethral
cancer. Some cases of early or superficial distal ure-
thral cancer may be managed effectively with con-
servative surgical therapy.

12. A.
SCC is the most common histologic type of proxi-
mal urethral cancer in women.

13. C.
The most significant prognostic factor for local con-
trol and survival in women with urethral cancer is
the location and extent of the primary tumor.

610 EDUCATIONAL REVIEW MANUAL IN UROLOGY


Chapter 20:
Testicular Cancer
Lucas R. Wiegand, MD
Jose J. Correa, MD
Wade J. Sexton, MD

Contents

1. Histologic Classification

2. Epidemiology, Risk Factors, Presentation


and Diagnosis

3. Clinical Staging

4. Management of the Primary Tumor

5. Retroperitoneal Lymph Node Dissection


(RPLND)

6. Management of Nonseminoma

7. Management of Seminoma

8. Post treatment Surveillance of Germ Cell


Tumors and Late Relapse

9. Sex Cord/Gonadal Stromal Tumors

10. Lymphoid, Hematopoietic and Metastatic


Tumors

11. Further Reading

12. Questions

CHAPTER 20: TESTICULAR CANCER 611


1. Histologic Classification

Management recommendations for the patient with seminomas and account for the bHCG (beta-human
newly diagnosed testicular cancer are principally chorionic gonadotropin) production by some of the
dependent upon an accurate histologic description tumors.
of the primary tumor. Radiographic staging studies
and serum tumor markers further define the options Seminoma With High Mitotic Index
available to the patient and provide an initial Originally classified as an anaplastic seminoma,
glimpse of a patient’s long-term prognosis. now this seminoma variant is referred to as a semi-
noma with a high mitotic index. Stage for stage, this
Table 1 demonstrates the classification of germ cell tumor has the same prognosis as a classic semi-
and non-germ cell tumors. Germ cell tumors noma, although there are several features that sug-
account for 90%–95% of all testicular tumors and gest a more aggressive and more lethal behavior.
are composed of 5 basic cell types: seminoma, These variants show increased mitotic activity aver-
embryonal carcinoma, yolk sac tumor, chorio- aging 3 mitoses per high-power field, more cellular
carcinoma and teratoma. A broad classification is atypia and more nuclear pleomorphism. The tumors
recognized between seminomas and nonseminomas have a higher rate of local invasion, an increased
and is important for determining the management of rate of metastatic spread and a higher rate of bHCG
locoregional disease and distant metastases. Over production. Morphologically, these tumors closely
half of all primary tumors consist of >1 cell type and resemble embryonal carcinoma and account for
are known as mixed germ cell tumors. Germ cells 5%–10% of all seminomas.
tumors arise from pluripotential cells, which
explains the different germ cell elements in the pri- Spermatocytic Seminoma
mary tumor or its metastatic sites. Adequate tissue Spermatocytic seminomas account for <5% of all
sampling and appropriate use of tumor markers seminomas. They occur most commonly in men
establishes the diagnosis with near certainty. over the age of 40 years and approximately 5% of
patients have bilateral disease. Unlike other germ
Non-germ cell tumors account for 5%–10% of tes- cell tumors that can involve the ovary, retroperi-
ticular tumors and broadly include the sex cord and toneum or mediastinum, spermatocytic seminomas
gonadal stromal tumors (ie, Leydig cell and Sertoli only involve the testicle. Grossly, the tumors are
cell tumors), lymphoid and hematopoietic tumors, large with a yellowish, soft, mucoid appearance to
and metastatic tumors from other primary neo- the cut surface. The cells vary in size have deeply
plasms. pigmented cytoplasm and closely resemble differ-
ent phases of maturing spermatogonia. The nuclei
of most cells have a characteristic filamentous chro-
matin distribution. Once the diagnosis is confirmed
Seminoma

Classic Seminoma following radical orchiectomy, no further treatment


Seminomas comprise the most common type of is warranted as these tumors behave in a benign
germ cell tumor (approximately 45%) and the manner. Only rarely are spermatocytic seminomas
most common histology found in men over the age associated with sarcomas. The sarcomas are typi-
of 35. The classic seminoma accounts for 80%–85% cally undifferentiated, although on occasion there
of seminomatous germ cell tumors. The testicle is may be differentiated forms such as rhabdomyosar-
usually enlarged and, when cut, has a bulging coma. The sarcoma elements metastasize exten-
homogeneous gray-white appearance. Microscopi- sively.
cally, there are relatively large cells with clear cyto-
plasm arranged in uniform sheets, lobules or
columns. The supporting stroma show varying
amounts of lymphocytic infiltrate or granulomatous
reaction, which is thought to represent a host
response to the tumor. There are occasional mitoses
and little variation in the nuclear arrangement. Syn-
cytiotrophoblastic elements occur in 20%–25% of

612 EDUCATIONAL REVIEW MANUAL IN UROLOGY


Table 1

Histologic Classification of Testicular Germ Cell Tumorsa

Precurser lesion - Intratublular germ cell neoplasia (carcinoma in situ)

Tumors of one histologic type (pure)


Seminoma
Variet - Seminoma with syncytrophoblastic cells
Spermatocytic seminoma
Variant - spermatocytic seminoma with sarcoma
Embryonal carcinoma
Yolk sac tumor
Trophoblastic tumors
Choriocarcinoma
Choriocarcinoma with other cell types
Placental site trophoblastic tumor
Teratoma
Dermoid cyst
Monodermal teratoma
Teratoma with somatic type malignancies

Tumors of more than one histologic type (mixed)


Mixed embryonal carcinoma and teratoma (teratocarcinoma)
Mixed teratoma and seminoma
Choriocarcinoma and teratoma/embryonal carcinoma
Others

Adapted from Sesterhenn IA, Davis CJ. Pathology of germ cell tumors of the testis.
a

Cancer Control. 2004;11:374-387.

CHAPTER 20: TESTICULAR CANCER 613


central fibrovascular core covered by a layer of
cuboidal tumor cells.
Nonseminomas

Embryonal Carcinoma
Pure embryonal carcinoma comprises only 3%–4% Choriocarcinoma
of germ cell tumors but embryonal cells are present Pure choriocarcinomas account for <1% of germ cell
in variable amounts in up to 40% of tumors with neoplasms. Clinically, the primary tumors are often
mixed histologic types. Embryonal histologies are very small and may not be palpable. The actual size
most common during the 3rd decade of life and are of the tumor depends on the extent of local hemor-
less common in adolescent males between 15 and 20 rhage. Patients typically present with symptoms due
years old. These germ cell elements have not been to extensive metastatic disease. Traditionally, there
described in prepubertal testes. Pure embryonal are 2 distinct cell types that must be present to satisfy
carcinomas may be associated with elevated levels the definition of a choriocarcinoma—syncytiotro-
of serum AFP. Syncytiotrophoblastic cells are phoblasts and cytotrophoblasts. However,
scattered throughout the tumor and are responsible monophasic choriocarcinomas consisting of only
for bHCG elevation in patients with embryonal his- cytotrophoblastic or intermediate trophoblastic cells
tologies. Grossly, the tumors are typically smaller have been reported. The syncytiotrophoblasts pro-
with a poorly defined capsule. The cut surface duce bHCG and the cells may be large, multinucle-
appears grayish-white with areas of hemorrhage and ated or mononuclear. There is abundant and often
necrosis. Microscopically, there are malignant vacuolated eosinophilic cytoplasm, and the nuclei
appearing epithelioid cells arranged in glands or are large and hyperchromatic. The cytotrophoblasts
tubules. Cell borders are indistinct and there is fre- have pale staining cytoplasm and distinct cell bor-
quent nuclear overlap. The cytoplasm is pale, ders. Cytotrophoblast nuclei have 1 or 2 nucleoli.
eosinophilic or vacuolated, and the nuclei have a
see-through appearance with prominent nucleoli Teratoma
and numerous mitoses. Lymphovascular invasion Teratomas constitute approximately 35% of germ cell
and involvement of epididymal and paratesticular tumors in infants and children. In adults, teratomas
structures are common. These findings are of prog- are found in only 2%–7% of testis tumors in pure
nostic importance. form, but 45%–50% of mixed germ cell tumors con-
tain teratoma. Almost always, these tumors contain
Yolk Sac Tumor >1 germ cell layer in different stages of maturation.
The yolk sac tumor is the predominant tumor in Mature elements look like benign tissues derived
infants and children. In adults, pure yolk sac from normal endoderm, ectoderm and mesoderm.
tumors account for a very small percentage of testic- Immature elements consist of undifferentiated primi-
ular neoplasms (2%). Yolk sac elements are present tive tissues from the 3 germ cell layers. Teratomas in
in up to 40% of mixed germ cell testicular neo- children are benign as they are diploid, lack chromo-
plasms and are responsible for the production of somal imbalances and do not show gains of isochro-
AFP. Grossly, the tumor appears soft, homoge- mosome (12p). In postpubertal patients, teratomas are
neous, grey-yellow and greasy. It is poorly encapsu- hypotriploid, demonstrate chromosomal imbal-
lated. Microscopically, there are at least 10 different ances—including a gain of isochromosome (12p)—
patterns of yolk sac elements, which can make it dif- and have the potential for metastatic spread. Since
ficult to distinguish particularly in mixed patterns. mature and immature teratomas have the same
Epithelioid cells form glandular or ductal structures genetic changes and biologic potential specific for
and are arranged in columns, papillary projections prepubertal and postpubertal patients, they are no
or solid islands. The individual cells are small and longer subclassified into mature and immature cate-
range from cuboidal and columnar to flat. The cells gories.
have poorly defined borders and a vacuolated cyto-
plasm containing glycogen and fat. The nuclei are of Grossly, teratomas are often large and inhomoge-
variable sizes and there are frequent mitoses. The neous. The cut surface reveals cystic regions contain-
endodermal sinus pattern is characterized by ing lakes of mucinous or gelatinous material inter-
Schiller-Duval bodies—papillary structures with a spersed among areas of solid tissue. There is a hap-

614 EDUCATIONAL REVIEW MANUAL IN UROLOGY


hazard arrangement of enteric or salivary glands, res- that some germ cell elements regress or burn out in
piratory epithelium, fat, smooth or striated muscle, the primary specimen. Mixed tumors that contain
cartilage, bone, glial or neuroectodermal tissues. seminoma are managed as nonseminomas.
Although the tumor appears benign, lymphovascular
invasion may be present which supports its Intratubular Germ Cell Neoplasia:
metastatic potential. Pure teratoma or teratoma found Carcinoma in Situ
in mixed germ cell tumors is resistant to both radia- Testicular carcinoma in situ (CIS) is thought to
tion and chemotherapy. Rarely, teratomas will harbor be the precursor of all invasive germ cell tumors
somatic-type malignancies such as adenocarcinoma except spermatocytic seminoma in adults and the
and rhabdomyosarcoma, identical to cancers found pediatric testicular tumors (yolk-sac and mature
at other organ sites. Some patients with metastatic teratoma). This supposition is supported by the
teratomas that undergo malignant degeneration of a presence of CIS in nearly 100% of testicular
single cell type respond to chemotherapy regimens parenchyma adjacent to the primary invasive tumor.
directed towards the specific malignant cell Histologically, the malignant germ cells are large
observed. Unfortunately, the prognosis for patients with abundant pale cytoplasm, irregularly outlined
with malignant transformation (particularly at distant nuclei with 1 or 2 prominent nucleoli, and located at
sites) is poor. the periphery of the seminiferous tubules between
normal appearing Sertoli cells. Testicular CIS has
Mixed Germ Cell Tumors been studied carefully in Scandinavian countries,
Approximately 60% of germ cell tumors contain >1 where the reported incidence of CIS is approxi-
histologic pattern. Of these, 59% containseminoma, mately 1%. This correlates well with the lifetime
41% contain yolk sac tumor, and 47% contain risk for the development of invasive testis cancer in
embryonal carcinoma and teratoma. Syncytiotro- this at-risk population of men. Reports indicate that
phoblast cells are found in 42% of mixed germ cell 50% of patients with CIS found on testicular biopsy
tumors, although pure choriocarcinoma typically developed invasive tumors within 5 years of diag-
spreads to distant sites unaltered. The most frequent nosis if left untreated. The overall incidence of tes-
combination is embryonal carcinoma, yolk sac ticular CIS in the general population is low. How-
tumor, teratoma and syncytiotrophoblasts, with or ever, there are groups of patients at risk for the
without seminoma. A polyembryoma consists of a development of CIS, including: patients with a his-
mixture of yolk sac tumor and embryonal carci- tory of contralateral testicular cancer (5%–6%),
noma forming embryoid bodies, which are ovoid cryptorchidism (3%–5%), an atrophic contralateral
structures consisting of a cavity surrounded by testis with unilateral testis cancer (30%), an extrag-
loose mesenchyme associated with syncytiotro- onadal germ cell tumor (40%) and intersex or sex-
phoblasts and occasionally teratoma. Each cell type ual ambiguity (25%–100%). The data are contro-
has invasive and metastatic capabilities. versial regarding the presence of infertility and any
increased risk for testicular CIS. In patients with
Burned-out Germ Cell Tumors known invasive germ cell tumors, the finding of
Few patients with metastatic cancer lack clinical contralateral testicular microlithiasis on ultrasound
evidence of a primary testicular tumor. The patho- increases the risk of harboring carcinoma in situ in
logic examination of the orchiectomy specimen that testicle. However, testicular microlithiasis was
occasionally reveals an area of scar or hemorrhage. not felt to be associated with CIS or invasive testic-
This finding is most common in patients with pure ular germ cell tumors in asymptomatic populations
choriocarcinoma and has been proposed in patients with no risk factors for CIS or testicular cancer,
with pure teratoma and discordant pathology at unless associated with testicular inhomogeneity.
metastatic sites. Less than10% of patients with pure Patients at risk for testicular CIS and potentially for
seminoma have clinical evidence of nonseminoma- invasive germ cell tumors should be offered either
tous metastases at distant sites. However, careful surveillance or testicular biopsy, following a
30%–45% of patients who die from apparent discussion of the potential risks and benefits of
metastatic pure seminoma actually have nonsemi- diagnosis and therapy.
nomatous elements. These findings support the fact

CHAPTER 20: TESTICULAR CANCER 615


2. Epidemiology, Risk Factors,
Presentation and Diagnosis

prognosis for patients with a second germ cell


tumor is excellent. Fortunately, the long-term dis-
Incidence

In 2010, there was a projected incidence of 8,480 ease-free survival in patients with bilateral germ
new testicular cancer cases and 350 deaths related to cell tumors approaches 90%. Careful surveillance
testicular cancer within the United States. Testicular of the contralateral testicle, including monthly self-
cancer is the most common solid tumor among testicular examinations, is mandatory.
American men aged 20–34 and the 2nd most com-
mon solid tumor in men 35–40 years. The lifetime
probability of developing testicular cancer is approx-
Risk Factors

imately 1 in 300, and the risk of dying from the dis- Table 2 lists factors reported to increase one’s risk
ease is about 1 in 5,000. The age-adjusted incidence for the development of testicular cancer. Several rea-
of testis cancer varies worldwide and is highest in sons are postulated to explain why tumorigenesis
Scandinavian countries (Norway and Denmark— might occur in cryptorchid patients. These reasons
9.3 cases per 100,000 men), intermediate in the include: abnormal germ cell morphology, increased
United States (5.4 cases per 100,000 men) and lowest temperature exposure, interference with the testicu-
in Africa and Asia. African American males have a lar blood supply, endocrine dysfunction and gonadal
rate of 0.9 per 100,000 men, which is believed to be dysgenesis. Interestingly, in cryptorchid patients
10 times higher than the incidence rate in Africa. who develop testicular malignancies, 5%–10% of
the germ cell tumors actually develop in the normal
descended contralateral testicle. This finding is con-
sistent with testicular biopsy data indicating an ele-
Age and Histology

Seminoma is the most common histologic type of vated risk for CIS in the contralateral testicle of
germ cell tumor with a peak incidence between 35 patients with testicular maldescent. Orchiopexy
and 39 years. Most spermatocytic seminomas and facilitates appropriate clinical surveillance of the
malignant testicular lymphomas occur in men over involved gonad. Until recently, no data indicated
the age of 50. Generally, nonseminomatous tumors that orchiopexy prevented the development of CIS
occur in younger patients in the 20–30-year age and invasive germ cell tumors. A systematic litera-
group. Pure yolk sac tumor and pure teratoma are ture review and meta-analysis by Walsh and col-
the predominant histologic subtypes in pediatric leagues suggests that a prepubertal orchiopexy may
groups. Both yolk sac tumor and teratoma are pre- decrease the risk of testicular cancer. This study
sent in older patients, typically as germ cell elements showed an average 3-fold increased risk of tumori-
in mixed nonseminomatous tumors. Teratoma is a genesis in men when orchiopexy was delayed
benign tumor in prepubertal patients but has (10–11 years or later) or was not performed, com-
metastatic potential in adolescents and older men. pared to patients who underwent surgery earlier. A
separate meta-analysis which included another large
cohort study confirmed Walsh's findings. There
were potential confounding factors, as the early
Bilateral Testicular Cancers

Bilateral testicular germ cell tumors occur in orchiopexy group may have included patients who
1%–4% of patients. The synchronous presentation might have experienced spontaneous testicular
of bilateral tumors is less common (17%–30% of descent, making them a subgroup of
patients) than the metachronous development of a low-risk patients that may have influenced the
second contralateral tumor (70%–83%). Most results of the meta-analyses.
metachronous tumors occur at a median of 5 years
from diagnosis of the incident tumor. Seminoma is Testicular trauma does not cause testicular
the predominant histology in synchronous and cancer. The traumatic event likely leads to an
metachronous tumors. Although patients with a pri- increased awareness of testicular pathology and
mary testicular tumor have an elevated risk for con- indirectly facilitates tumor detection. Hormonal
tralateral CIS or the development of a second con- fluctuations and estrogen exposure are postulated to
tralateral invasive germ cell tumor, contralateral cause changes in germ cells leading to the develop-
testis biopsies are not recommended routinely as the ment of testicular cancer. Hormonal imbalances may

616 EDUCATIONAL REVIEW MANUAL IN UROLOGY


play a causative role in cancer development, partic- hydroceles, spermatoceles, epididymo-orchitis and
ularly in patients with testicular atrophy. Patients trauma-related pathology. These diagnoses, as well
might experience decreased feedback inhibition for as others such as testicular torsion, are part of the
gonadotropin secretion, which could be the driving differential diagnosis in patients with a testicular
force for malignant transformation of an already mass. However, a mass should be considered a tes-
damaged germinal epithelium. ticular cancer until proven otherwise. A scrotal
ultrasound is an important adjunctive study avail-
In a recent case-control study of 369 men with tes- able in equivocal cases. If local signs and symptoms
ticular germ cell tumors between 18 and 44 years of are attributed to an infectious process, such as epi-
age, compared to 979 age-matched controls, Daling didymitis, a normal physical exam must be docu-
and colleagues found a correlation between mari- mented during immediate follow-up. Unfortunately,
juana use and testicular germ cell tumors, particu- patients with symptoms due to metastatic disease on
larly nonseminomas. The frequency of use (daily or occasion are subjected to unnecessary procedures,
weekly) vs. marijuana use less than once per week biopsies or abdominal explorations, prior to the
also increased the risk of developing a testicular recognition of a clinically obvious testicular mass.
tumor. Confirmatory studies are necessary to deter- Occasionally, patients may be found to have abdom-
mine if marijuana use causes malignant transforma- inal metastases suggestive of an extragonadal germ
tion of the germinal epithelium. cell tumor. A scrotal ultrasound should be per-
formed to rule out the testicle as the source of a pri-
mary germ cell tumor. Impalpable tumors or evi-
dence for small, burned-out tumors may be visible
Presentation and Diagnosis

On average, there is a mean delay of 5 months from with ultrasonography. In the case of findings on
the onset of initial symptoms to the diagnosis of tes- ultrasonography (such as calcifications, scar or
ticular cancer. The responsibility for this diagnostic hypo/hyperechoic foci), unilateral atrophy, unde-
delay is both patient- and physician-related. Patients scended testis or unilateral metastatic pattern, it is
are reluctant to seek medical attention for testicular recommended to perform an orchiectomy to prevent
pathology due to ignorance, embarrassment, fear, tumor recurrence from a site of residual
denial or inadequate access to medical care. Physi- disease.
cians misdiagnose testicular tumors as hernias,
The most common presenting signs and symptoms
in patients diagnosed with testis cancer are listed in
Table 3.
Table 2

Seminomas tend to expand within the testis as a


painless, rubbery mass, whereas nonseminoma-
Possible Risk Factors for Testicular Cancer

tous tumors often have more discrete and irregular


characteristics. However, these distinctions are not
Risk Factor Risk Increase a

always evident. Patients with nonseminomas are


more likely to complain of testicular pain. The pre-
Cryptorchidism 3 - 14X

senting complaint in patients with metastatic dis-


ease might include: back pain due to bulky lym-
Metachronous testis cancer 2 - 4X

phadenopathy; dyspnea, cough or hemoptysis


from numerous pulmonary lesions; gastrointestinal
Positive family history 5X

disturbances, such as anorexia, nausea, emesis and


possibly even hemorrhage due to bowel involve-
Diethylstilbestrol exposure 2.5 - 5X

ment; neck mass secondary to supraclavicular


lymph node involvement; bone pain due to skeletal
Gonadal dysgenesis 50X

metastases; and even lower extremity edema from


occlusion of the inferior vena cava.
Androgen insensitivity syndrome 15X

a
= Increase in relative risk compared to

CHAPTER 20: TESTICULAR CANCER 617


The impact of the delay in the diagnosis has been vein, followed by the paracaval and the preaortic
addressed by a recent European population-based regions. In many cases, lymphatic drainage crosses
study that included 542 patients. The study con- over from right to left; therefore, left paraaortic
firmed a delay in diagnosing seminoma of 4.9–6.1 lymph node involvement occurs commonly in
months compared to 2.8–4.0 months for nonsemi- patients with right testicular primaries.
nomas. The delay in diagnosis led to a statistically
significant impact on the 5-year survival rate only Primary testicular tumors with epididymal or cord
in the patients with nonseminoma. This study involvement may spread to pelvic or inguinal lymph
stresses the importance of early detection and man- node basins. Iliac or inguinal node involvement
agement. must be considered for cryptorchid patients, patients
with scrotal wall involvement and patients who have
had prior inguinal surgery or procedures that would
disrupt the normal lymphatic flow from the primary
Patterns and Mechanisms of Spread

Testicular germ cell tumors have highly predictable tumor. Patients with advanced retroperitoneal
patterns of spread. The tunica albuginea of the testis lymph node metastases may have pelvic or inguinal
acts as a natural boundary for most testicular metastases from retrograde lymphatic spread.
tumors, preventing involvement of the epididymis Suprahilar or retrocrural lymph node involve-
and the testicular cord. Tumors confined to the testi- ment is less common for low-volume retroperi-
cle and the tunica albuginea usually spread directly toneal disease. In patients with advanced retroperi-
to the retroperitoneal lymph nodes through lym- toneal lymphatic metastases, both of these locations
phatic channels that course with the testicular cord are more common sites of nodal involvement and
and gonadal veins before fanning out medially into require careful scrutiny on preoperative radio-
the retroperitoneum after crossing over the ureter. graphic imaging studies and during retroperitoneal
The primary landing zone for left-sided tumors is lymph node dissection (RPLND).
the left paraaortic region bounded by the left renal
vein, ureter and the aorta above the inferior mesen- Lymphatic drainage above the retroperitoneum
teric artery. Less common locations of primary is to the cisterna chyli, thoracic duct and, usually,
spread from left-sided tumors include the preaortic to the left supraclavicular lymph nodes. Extran-
and the interaortocaval regions. The primary land- odal visceral and distant metastases result from
ing zone for right-sided testicular tumors is the either direct hematogenous dissemination (common
interaortocaval region beneath the left renal with choriocarcinoma) or through lymphati-
covenous communications. The most direct com-
munication is the junction of the thoracic duct and
the subclavian vein, although in the setting of bulky
Table 3

retroperitoneal lymph node metastases, small lym-


phaticovascular communications develop in the
Most Common Presenting Signs and

retroperitoneum, allowing for subsequent vascular


Symptoms and Their Frequency

tumor emboli and distant progression or visceral


involvement of testicular cancer. Rarely, distant pro-
gression occurs in the absence of retroperitoneal
Painless mass or swelling 40%- 55%

lymph node involvement. Lymphatic channels may


bypass the retroperitoneum altogether and commu-
Pain ± mass or swelling 25%- 50%

nicate directly with the cisterna chili or the thoracic


duct. This pattern of lymphatic drainage and the
Sign or symptom related to mets 5%- 20%

possibility of direct hematogenous spread accounts


for the small percentage of patients who relapse
History of trauma 3%- 13%

(most commonly in the lungs) following a negative


RPLND for clinical stage I disease.
Gynecomastia or breast tenderness 1%- 5%

Incidental finding <5%

618 EDUCATIONAL REVIEW MANUAL IN UROLOGY


3. Clinical Staging

Accurate staging of testicular germ cell tumors is is elevated in patients with histologically pure semi-
critical to the process of treatment decision making. noma, the primary specimens should be reevaluated
Staging requires a combination of serum tumor to exclude the presence of other germ cell elements,
markers, radiographic studies, and precise interpre- or areas of scar that could be representative of
tation of the histologic findings and tumor type(s) burned-out germ cell elements other than semi-
following radical orchiectomy. In 1997 (updated in noma. Patients with a histologically pure seminoma
2002), the American Joint Committee on Cancer and an elevated AFP are managed like other patients
(AJCC) adopted a staging system applicable to broadly classified as having nonseminomas.
patients with seminomas and nonseminomas
(Tables 4 and 5). The system is unique in that tumor Human Chorionic Gonadotropin (HCG)
marker levels have been incorporated into the stag- HCG is a 38 kilodalton protein secreted by placental
ing criteria emphasizing the importance of serum syncytiotrophoblasts to maintain the viability of the
tumor markers in determining prognosis. corpus luteum. In healthy men, only minute
amounts of bHCG are detected (<5 mIU/mL). There
are 2 subunits of HCG. The alpha subunit is
homologous with the pituitary glycoprotein hor-
Tumor Markers

Tumor markers are important for testicular cancer mones (luteinizing hormone, follicle-stimulating
diagnosis, staging, determining prognosis, monitor- hormone, thyroid-stimulating hormone). The beta
ing treatment response and for detecting relapse. subunit of HCG (bHCG) is 70% homologous with
There are 2 main classes of tumor markers: oncofe- the luteinizing hormone (LH) beta chain but it has a
toproteins (AFP and bHCG), which are associated unique amino acid c-terminal extension that allows
with embryologic development, and cellular the production of specific antibodies used to target
enzymes lactate dehydrogenase (LDH) and pla- the purified bHCG subunit in radioimmunoassay
cental alkaline phosphatase (PLAP). PLAP is a techniques. The serum half-life of bHCG is 24–36
marker of advanced disease (particularly for semi- hours. Like AFP, bHCG can be elevated in both
noma) and is utilized rarely for clinical purposes. benign and malignant conditions. Given the cross
reactivity with LH, it is important to remember that
Alpha-fetoprotein (AFP) conditions resulting in an elevated LH (ie, hypogo-
AFP is a 70 kilodalton single-chain glycoprotein nadism) can falsely elevate bHCG levels. An intra-
and it is the predominant serum-binding protein in muscular injection with 200 mg of testosterone can
the fetus. AFP is synthesized in the fetal yolk sac, help to distinguish cancer-related elevations of
liver and intestine. Peak gestational levels occur bHCG from cross-reaction with LH. Other condi-
during weeks 12–14 and then begin to fall after 16 tions leading to elevated levels of bHCG include
weeks, until the AFP level is minimally detectable marijuana use and cancers of the liver, pancreas,
after the first year of life. Beyond 1 year, the AFP stomach, lung, breast, kidney or bladder. bHCG
level should be <10 ng/ml. The serum half-life of levels are elevated in every patient with choriocar-
AFP is 5–7 days. After the first 6 weeks of postnatal cinoma, 40%–60% of patients with embryonal cell
life, AFP elevations can be detected in several carcinoma, and 10%–25% of patients with pure
benign and malignant conditions other than germ seminoma due to syncytiotrophoblastic elements.
cell tumors including: normal pregnancy; ataxia
telangiectasia; hereditary tyrosinemia; liver toxicity Lactate Dehydrogenase (LDH)
related to chemotherapy, anesthetics, antiepilep- LDH is a 134 kilodalton cellular enzyme produced
tics, hepatitis and alcoholism; and cancer of the by muscle, liver, and numerous other organs and
lung, stomach, pancreas and liver (seen in 70% of may be falsely elevated with hemolysis. Elevation
patients with hepatocellular carcinomas). AFP may of LDH is nonspecific and must be taken into con-
be elevated in patients with pure embryonal cell sideration with other tumor markers and staging
carcinoma, teratocarcinoma, yolk sac tumors studies. However, evidence points to the utility of
and in patients with other mixed germ cell tumors. LDH as a marker of tumor volume or a marker of
AFP levels are normal in patients with pure advanced disease—particularly for bulky semi-
choriocarcinoma and pure seminoma. If the AFP noma. LDH elevation has a direct relationship with

CHAPTER 20: TESTICULAR CANCER 619


Table 4

2002 AJCC TNMS Staging System for Testicular Cancer

Primary Tumor (T)

pTx Primary tumor cannot be assessed (radical orchiectomy has not been performed)

pT0 No evidence of primary tumor (eg, scar)

pTis Intratubular germ cell neoplasia / carcinoma in situ

pT1 Tumor limited to testis and epididymis; no lymphovascualr invasion; may invade tunica
albuginea but not tunica vaginalis

pT2 Tumor limited to testis and epididymis; lymphovascualr invasion


present or there is invasion of tunica vaginalis

pT3 Direct cord invasion ± lymphovascular invasion

pT4 Direct scrotal invasion ± lymphovascular invasion

Regional Lymph Nodes (N) – Clinical

Nx Regional lymph nodes cannot be assessed

N0 No regional lymph node mets

N1 Lymph node mass, single or multiple nodes, ≤2 cm in greatest dimension

N2 Lymph node mass, single or multiple nodes, 2-5 cm in greatest dimension

N3 Lymph node mass >5 cm

Regional Lymph Nodes (N) – Pathologic

pNx Regional lymph nodes cannot be assessed

pN0 No regional lymph node mets

pN1 Lymph node mass ≤2 cm; or ≤5 positive nodes, none >2 cm in dimension

pN2 Lymph node mass 2-5 cm; or >5 nodes positive, none >5 cm; or (+) extranodal extension

pN3 Lymph node mass >5 cm

(continued)

620 EDUCATIONAL REVIEW MANUAL IN UROLOGY


Table 4 (continued)

2002 AJCC TNMS Staging System for Testicular Cancer

Distant Metastasis (M)

Mx Distant mets cannot be assessed

M0 No distant mets

M1 Distant mets

M1a Nonregional nodal or pulmonary mets

M1b Distant mets other than nonregional nodal or pulmonary

Serum Tumor Markers (S)

Sx Markers not available or performed

S0 Markers within normal limits

S1 LDH <1.5 x normal and


hCG (mIU/mL) <5000 and
AFP (ng/mL) <1,000

S2 LDH 1.5 - 10 x normal or


hCG (mIU/mL) 5,000 - 50,000 or
AFP (ng/mL) 1,000 - 10,000

S3 LDH >10 x normal or


hCG (mIU/mL) >50,000 or
AFP (ng/mL) >10,000

Data according to 2002 American Joint Committee on Cancer (AJCC). AJCC Cancer Staging Manual. 6th ed. New York, ,
NY: Springer-Verlag; 2002.

CHAPTER 20: TESTICULAR CANCER 621


disease burden and with the stage of a patient’s
malignancy. Along with AFP and bHCG, LDH is a
Table 5

component of the TNMS staging system and the


degree of elevation carries prognostic significance.
TNMS Stage Grouping for Testis Cancer

Tumor Marker Elevation


A significant percentage of patients with nonsemi-
Stage

0 pTis N0 M0 S0 nomatous germ cell tumors have an elevation of


either bHCG or AFP (or both markers) prior to
IA pT1 N0 M0 S0 undergoing radical orchiectomy (Table 6). The
degree of marker elevation in these patients with
IB pT2-4 N0 M0 S0 nonseminomas helps to stratify patients into good,
intermediate and poor risk groups according to the
IS pT1-4 N0 M0 S1-3 International Germ Cell Consensus Classification.
Smaller numbers of patients with pure seminomas
IIA any T N1 M0 S0/1 have elevated bHCG or LDH (Table 6). Typically, if
bHCG levels in seminoma patients are abnormal,
IIB any T N2 M0 S0/1 the serum levels will be <100 mIU/mL
200 mIU/mL. Only on rare occasions does a patient
IIC any T N3 M0 S0/1 with a pure seminoma have an elevated bHCG
beyond 500 mIU/mL. Contrary to the prognostic
IIIA any T any N M1a S0/1 significance of diagnostic marker levels in patients
with nonseminomas, the degree of HCG elevation
IIIB any T N1-3 M0 S2 in patients with seminomas prior to orchiectomy has
no apparent effect on patient outcome or prognosis.
any T any N M1a S2 Low level marker elevation following radical
orchiectomy for clinical stage I patients can be due
IIIC any T N1-3 M0 S3 to any of the aforementioned benign or malignant
conditions associated with elevated bHCG, AFP or
any T any N M1a S3 LDH. In general, however, persistent marker eleva-
tion indicates the presence of systemic disease and
any T any N M1b any S patients should be considered to have occult metas-
tases.

Monitoring Treatment Response


Data according to 2002 American Joint Committee on
Cancer (AJCC). AJCC Cancer Staging Manual. 6th ed.

Serum tumor markers are very useful for monitor-


New York, NY: Springer-Verlag; 2002.

ing treatment response and disease relapse. Markers


often provide the first evidence of relapse (including
patients with normal markers at presentation) while
on surveillance for clinical stage I disease, or fol-
lowing therapy for known or suspected metastatic
disease. As demonstrated in Table 6, not every
patient with a germ cell malignancy will present
with elevated markers. However, the initial value of
a marker provides an early indication of prognosis,
and generally, the degree of marker elevation corre-
lates with tumor burden. Markers should decline
according to their known kinetics following radical
orchiectomy (in the absence of occult metastases),
and following radiation or chemotherapy when

622 EDUCATIONAL REVIEW MANUAL IN UROLOGY


patients are treated for systemic disease. A rate of chemotherapy have viable nonteratoma germ cell
marker normalization that is less than the expected tumor confirmed histologically following
rate of decline, or persistent marker elevation postchemotherapy retroperitoneal lymph node dis-
despite therapy, indicates that there is continued section (RPLND). Based on risk factors for
marker production from viable germ cell elements micrometastases, 20%–50% of patients with normal
due to incomplete resection, tumor persistence, drug markers and clinical stage I diseases are proven to
resistance or relapse. have metastatic germ cell tumor at the time of their
primary RPLND. Persistent marker elevation is
Two-thirds of patients with bulky metastatic disease indicative of systemic disease. A patient, whose
will have a marker surge after initiating chemother- markers remain elevated following radical
apy, which causes an early prolongation of the orchiectomy or induction chemotherapy, should be
marker half-life. After the marker surge, the rate of treated with primary chemotherapy or second-
normalization is proportional to the amount of line/salvage chemotherapy, respectively. Patients
viable germ cell tumor. However, normal tumor with persistently elevated markers prior to primary
markers do not signify disease absence. Approxi- RPLND or postchemotherapy RPLND almost
mately 10% of patients with normal markers after always relapse.

Table 6

Frequency of Raised Markers at Testis Cancer Diagnosis

Marker Number Percentage (%) Range (%)

Stage I seminoma

bHCG 31/298 10 5 - 40
LDH 5/18 27 0 - 33

Metastatic seminoma

bHCG 38/146 26 13 - 61
LDH 41/75 55 33 - 70

Stage I nonseminoma

AFP 103/196 53 48 - 75
bHCG 55/196 28 24 - 46
AFP or bHCG 115/196 57 24 - 75

Metastatic AFP 439/716 61 59 - 74


nonseminoma bHCG 421/760 55 40 - 68
AFP or bHCG 414/515 80 70 - 92

bHCG = beta human chorionic gonadotropin; AFP = alpha-fetoprotein


Adapted from Rustin GJ, et al. Consensus statement on circulating tumour markers and staging
patients with germ cell tumours. Prog Clin Biol Res. 1990;357:277-284.

CHAPTER 20: TESTICULAR CANCER 623


Radiographic Studies approached through a standard inguinal incision, the
vascular pedicle (testicular cord) should be con-
Scrotal Ultrasound trolled, and intraoperative ultrasound utilized to con-
Ultrasonography is an extension of the physical firm tumor location. Intraoperative frozen sections
examination and can confirm the presence of a tes- of the tumor and the surrounding testicular
ticular tumor, which usually has the appearance of a parenchyma are required to determine if testis-spar-
hypoechoic parenchymal mass. While a scrotal ing surgery is feasible or if radical orchiectomy is
ultrasound is not required for the diagnosis of a tes- indicated. Infrequently, false-negative intraoperative
ticular cancer, the study has several important appli- frozen sections occur. Patients should be informed
cations. A scrotal ultrasound is useful to exclude of this risk and the possibility that a subsequent radi-
testicular tumors in patients with hydroceles or tes- cal orchiectomy may be required.
ticular inflammatory conditions (epididymitis,
orchitis) where an adequate physical examination is Abdominal Imaging
often significantly limited or the diagnosis of a tes- Abdominal computed tomography (CT). CT is the
ticular mass is in question. An ultrasound is a modality of choice for staging the retroperitoneum
requirement for any patient suspected to have an (including the paraaortic, interaortocaval, paracaval,
extragonadal germ cell tumor (EGCT). The scrotal retrocrural, suprahilar, presacral and pelvic lymph
ultrasound might reveal a small impalpable tumor node regions) and evaluating the abdominal viscera.
or a suspicious area potentially representative of a Arterial and venous phases of the CT permit the pre-
burned-out germ cell tumor. It is extremely impor- cise definition of aberrant and accessory vascular
tant to establish the testicle as the site of origin, as structures (ie, accessory renal arteries). A detailed
the prognosis is significantly worse for patients with understanding of a patient’s vascular anatomy is
primary mediastinal EGCTs compared to patients important for patients who are candidates for an
with testicular primaries or primary retroperitoneal RPLND. However, the CT does not distinguish
EGCTs. benign lesions (fibrosis) from teratoma and viable
nonteratomatous germ cell elements such as embry-
Scrotal ultrasonography is utilized increasingly dur- onal cell carcinoma. For clinical stage I disease, the
ing the workup of infertility, orchalgia, scrotal CT scan is unable to detect micrometastases, which
swelling, varicoceles, and patient- or physician- is the reason treatment decisions following radical
detected scrotal pathology. An ultrasound per- orchiectomy are based upon tumor histology (tissue
formed for these indications occasionally reveals an types and relative percentage of germ cell elements)
incidentally detected scrotal mass. Some tumors are and other risk factors (tumor markers and pathologic
small and palpable on physical examination, but not characteristics) that help to predict the presence or
large enough to have been noticed by the patient. absence of occult metastatic disease.
Other incidentally detected testicular lesions are
impalpable. In patients with no retroperitoneal Magnetic resonance imaging (MRI). MRI has no
pathology and normal tumor markers, many inci- distinct advantage over CT for abdominal imaging.
dentally detected, impalpable lesions have benign Like the CT scan, the MRI does not distinguish
histologies. It has been suggested that, given the benign from malignant lesions. The MRI is useful
lower possibility of malignancy in impalpable for evaluating patients suspected to have obstructed
lesions <1 cm and without demonstrable internal vessels, particularly the inferior vena cava (IVC).
blood flow, these masses may be followed with Complete IVC obstruction due to tumor infiltration
serial ultrasound, delaying intervention for those or venous thrombosis infrequently occurs in
lesions that exhibit growth. Germ cell tumors are patients with bulky retroperitoneal lymphadenopa-
confirmed pathologically in smaller numbers of thy. Preoperative knowledge of this scenario and
patients. Patients with small testicular tumors are whether the patient has established collateral
candidates for testis-sparing surgery. If organ venous return helps the surgeon to determine if IVC
preservation is considered, the lesions must be resection is an option and whether an interposition
graft would be required.

624 EDUCATIONAL REVIEW MANUAL IN UROLOGY


Positron emission tomography (PET). PET has a Chest Imaging
limited role in the staging and evaluation of patients In clinical stage I disease, the risk of chest involve-
with metastatic testicular cancer. PET studies detect ment is <5%. Many urologists believe that an AP
viable malignant germ cells, but the study does not and lateral chest x-ray is adequate to image the chest
distinguish teratoma from fibrosis. Therefore, in in patients with no evidence of abdominal metas-
patients with nonseminomatous germ cell tumors, tases. Practically, many patients undergo simultane-
the recommendation for surveillance, RPLND or ous CT of the chest and abdomen. The disadvantage
adjuvant chemotherapy is based on clinical staging with this approach is the common finding of indeter-
and risk factors for metastases in patients with clini- minate subcentimeter pulmonary lesions. Over
cal stage I disease and low-volume clinical stage II time, many of these lesions do not change in size
disease. Likewise, nonseminomatous patients with and are interpreted to be benign. Rarely does a chest
postchemotherapy residual masses and normal CT alter management for patients with clinical stage
tumor markers require surgical consolidation regard- I disease, although when an abnormality is demon-
less of the PET scan results as these tumors have a strated, dual therapy might be avoided by the
high likelihood of containing teratoma or residual administration of induction chemotherapy. How-
viable germ cell tumor. On the contrary, there is a ever, the risk of chest involvement is approximately
potential role for PET scan in predicting residual 40% in the setting of an abnormal abdominal CT,
tumor following chemotherapy in patients with clin- under which circumstances more sensitive chest
ical stage II or III seminomas. Traditionally, surgery imaging with CT is required.
was recommended for patients based on the size of
the residual mass and the probability of finding
viable germ cells. Following chemotherapy,
13%–27% of residual tumors ≥3 cm harbored viable
seminoma compared to 0%–3% of residual tumors
<3 cm. The PET scan helps to clarify which patients
would benefit from an RPLND. Currently, PET scan
is recommended to evaluate residual masses of
larger than 3 cm in size in those who were initially
managed with chemotherapy for Stage II or Stage III
seminoma. The positive predictive value (PPV) has
been reported to be as low as 25%–28% in recent
studies evaluating residual masses of <3 cm in diam-
eter. However, when applied to residual masses of
>3 cm in diameter, the PPV improves to 70%–100%.
It is plausible that the false-positive PET scans are
due in part to the desmoplastic reaction and inflam-
mation associated with treated, nonviable semi-
noma. The study should be obtained at least 4–6
weeks following the completion of chemotherapy to
potentially limit false-positive results due to inflam-
mation. The negative predictive value (NPV) ranges
from 86%–100% when accounting for residual
lesions of all sizes and is even more accurate when
applied only to residual masses of >3 cm in diameter.
Thus, in the setting of a residual mass, the results of
a PET scan can significantly influence disease
management and spare some patients a technically
challenging and sometimes incomplete surgical
resection.

CHAPTER 20: TESTICULAR CANCER 625


4. Management of the
Primary Tumor

function is generally preserved throughout treat-


ment as demonstrated by testosterone and luteiniz-
Fertility and Sperm Banking

Baseline subfertility is a common finding amongst ing hormone (LH) levels.


patients diagnosed with testicular cancer. Approxi-
mately 75% of patients present with decreased base-
line spermatogenesis at the time of diagnosis. While
Radical Orchiectomy

the proposed rationale for subfertility include a his- A radical orchiectomy is usually the initial step in
tory of cryptorchidism, tumoral hormone produc- the management of a patient with a testicular tumor
tion, the presence of antisperm antibodies, and gen- that is suspicious for a germ cell neoplasm. A pre-
eralized stress associated with illness, it is the his- cise histological interpretation of the germ cell
torical presence of cryptorchidism that has been type(s) and amount(s), as well as a description of the
shown to be associated with both testicular cancer primary T stage, is critical to determining subse-
and infertility. Furthermore, treatment for testis can- quent options for disease management.
cer includes irradiation, RPLND, and the adminis-
tration of chemotherapeutic agents, all of which are Some patients with advanced disease are very symp-
can impact fertility. After discussing the risk of tomatic at presentation. A radical orchiectomy can
infertility associated with each individual treatment be temporarily delayed when tumor markers and
plan, fertility preservation methods such as sperm clinical staging are diagnostic of a germ cell malig-
banking, testicular tissue freezing, testicular sperm nancy, particularly a nonseminoma. It is important
extraction, and radiation shielding of the gonads to initiate induction chemotherapy to ensure that the
during treatment, should all be discussed with the patient experiences no delay in receiving systemic
patient regardless of age. treatment. Following chemotherapy, a radical
orchiectomy is performed as a single procedure or
In those men receiving retroperitoneal radiation for concurrent with an RPLND. Removing the primary
the management of seminoma, long-term fertility tumor is always recommended as viable germ cell
rates, as assessed by achievement of paternity, is tumor and teratoma are found in 8%–25% and
~60%–85% in couples attempting to conceive. Men 32%–42% of patients, respectively.
with carcinoma in situ of the contralateral testis
treated with 20 Gy of radiation are left permanently A radical orchiectomy provides excellent local con-
sterile. While the literature is sparse with respect to trol for almost every patient managed according to
fertility after receipt of carboplatin for stage I semi- sound oncologic principles and cures many patients
noma, it has been reported that 100% patients have with clinical stage I testis cancer. The procedure
sperm present on semen analysis and that 68% are involves the complete excision of the testicle, its
normospermic 4 years following chemotherapy surrounding tunics, and the entire spermatic cord
administration. just proximal to the level of the internal inguinal
ring. A 5–7-cm incision is made parallel to the
Treatment paradigms and the related fertility con- inguinal ligament approximately 2 cm above the
cerns are different when managing NSGCTs. pubic tubercle and carried down to the external
Instead of the detrimental effects of radiation, dis- oblique aponeurosis. The incision can be extended
cussions should include the 0%–7% chance for loss over the upper scrotum for patients with very large
of antegrade ejaculation associated with nerve tumors. The fascia is incised along the direction of
sparing primary RPLND and a dose-dependent its fibers down through the external ring. The ilioin-
reversible impairment of spermatogenesis in those guinal nerve is identified and preserved. The cord is
managed with cisplatin-based chemotherapy. While completely mobilized and elevated with a Penrose
carboplatin is less gonadal toxic than cisplatin- drain. Dissection proceeds towards the pubic tuber-
based regimens, even patients receiving up to 4 cle careful to incorporate all of the cremasteric fibers
cycles of BEP chemotherapy have been shown to and perforating cremasteric vessels, which must be
have greater than an 85%–90% chance of having a secured. The cord is wrapped with a Penrose drain
normal postchemotherapy sperm count by 1–1.5 and clamped as a tourniquet just below the internal
years posttreatment. The endocrine or Leydig cell ring. With manual pressure and eversion of the ipsi-

626 EDUCATIONAL REVIEW MANUAL IN UROLOGY


lateral hemiscrotum, the testicle and the tumor are There is a role for testis-sparing surgery in patients
delivered into the operating field. The gubernacular with incidentally detected, impalpable, small intra-
attachments are released and the radical orchiec- parenchymal tumors and a normal contralateral tes-
tomy is completed by dissecting the cord 1–2 cm ticle. Most of these patients have normal markers
into the internal inguinal ring and separately ligat- and no other clinical or radiographic evidence of dis-
ing and dividing the vas deferens and the cord ves- ease. Several reports indicate that 50%–80% of
sels. Silk ties or suture ligatures should be utilized in small lesions (particularly <10 mm greatest diame-
order to identify the gonadal vein stump if the ter) have benign histologies (Leydig cell, Sertoli cell
patient undergoes an RPLND. Scrotal or retroperi- and other benign forms). Seminoma is the most
toneal hemorrhage is the most common complica- common germ cell histology. The principles of man-
tion following a radical orchiectomy. Close atten- agement are identical to those described for testis
tion to hemostasis and careful ligation and division sparing procedures in patients with solitary testicles
of the cord structures must be ensured. Following or bilateral tumors.
irrigation, the wound is closed by approximating the
external oblique fascia, Scarpa’s fascia and the skin. Scrotal Violation

The traditional surgical approach to a radical


orchiectomy involves an inguinal incision and high
Partial Orchiectomy

Patients with a tumor in a solitary testicle and ligation of the spermatic cord. However, scrotal vio-
patients with bilateral testicular tumors are consid- lations (scrotal orchiectomy, incisional biopsy, fine
ered for testis-sparing surgery. Advantages of organ needle aspiration) occur in 4%–17% of patients with
preservation include the prevention of androgen testicular malignancies. Scrotal violation is associ-
deprivation and the subsequent need for lifelong ated with an increased risk of local recurrence par-
hormonal supplementation, and the preservation of ticularly when tumor is found in a scrotal wall speci-
fertility in smaller numbers of patients. Several men. The traditional recommendation for managing
guidelines must be respected when treating patients patients with scrotal violation includes a hemiscro-
with an organ-sparing approach: patients should tectomy, which is a disfiguring procedure for some
have normal preoperative serum levels of luteiniz- patients. Although debatable, many urologists
ing hormone and testosterone; a partial orchiectomy believe that scrotal violation excludes otherwise
should be approached through an inguinal incision acceptable patients from surveillance protocols.
and oncologic principles of dissection for a radical
orchiectomy must be followed; the operative field Recent reports question the need for a formal hemis-
must be draped to prevent tumor contamination; the crotectomy in the patient with a scrotal violation.
tumor is localized with intraoperative ultrasonogra- Scrotal violation, increases the risk of a local recur-
phy; cold ischemia should be utilized during tumor rence from <1%, up to 3%–6%. In a smaller series of
excision; and tumors should be small (ie, <20–25- patients, there were no local recurrences with scrotal
mm diameter). Frozen section for margins and biop- violation, provided that surgical margins were nega-
sies of the surrounding tumor bed should be accom- tive and the spermatic cord was uninvolved by
plished to ensure complete excision of the tumor tumor. Furthermore, no significant differences in
and to determine the presence of intratesticular distant recurrence rates or survival have been
germ cell neoplasia or carcinoma in situ (CIS); adju- demonstrated in patients with scrotal violation.
vant local irradiation should be considered for
patients with CIS in the parenchymal bed; patients Sheinfeld et al summarized the following manage-
must be highly motivated and compliant with strict ment principles for patients with scrotal violation:
follow-up guidelines. Local recurrences are very
rare for patients with CIS managed with postopera- 1. In patients with low-stage seminoma, the radia-
tive radiation. However, local recurrences are possi- tion portals should extend to the ipsilateral groin
ble in patients followed without adjuvant radiation and scrotum. Side effects include an increased risk
and thus some patients require a delayed orchiec- of infertility.
tomy.

CHAPTER 20: TESTICULAR CANCER 627


2. Low-stage nonseminoma patients undergoing evaluation. In the presence of a normal contralateral
RPLND should have simultaneous wide excision testicle, most would agree that a radical orchiec-
of the scrotal scar and the remaining testicular tomy is the treatment of choice for patients with
cord. CIS. Hypogonadal patients who require hormonal
supplementation and patients with compromised
3. Patients who received at least induction fertility also should be considered for a radical
chemotherapy should have the scrotal scar and orchiectomy. It is important to remember that many
the remaining cord structures excised at the time patients with invasive germ cell tumors or CIS are
of postchemotherapy RPLND, although local subfertile at the time of diagnosis.
relapse is rare following systemic chemotherapy.
There are other management options besides radical
4. In patients with a complete response following orchiectomy for patients with solitary testicles and
systemic induction chemotherapy with plans for CIS, or the uncommon patient diagnosed with CIS
continued observation rather than RPLND, no and a normal contralateral testicle. Options include
further inguinal surgery is required as the risk for observation, radiation and chemotherapy. Short-
local relapse is rare. term observation may be a reasonable approach for
the patient seeking to father children in light of the
5. Formal hemiscrotectomy and extensive groin protracted natural history of CIS. However, he must
dissection are no longer recommended. be counseled regarding the risks and benefits of this
approach including the development of an invasive
germ cell tumor that could result in the need for a
radical orchiectomy, or lead to anorchia. The bene-
Intratubular Germ Cell Neoplasia:

fits of radiation include cancer cure as well as the


Carcinoma In Situ

Several factors must be taken into consideration preservation of endocrine function. However, all
when deciding upon treatment for patients with tes- men who receive testicular radiation become infer-
ticular carcinoma in situ including age, fertility con- tile. Testicular radiation causes a Sertoli cell–only
cerns, status of the contralateral testicle, the pres- pattern on subsequent testicular biopsies. Lower
ence of unilateral vs. bilateral testicular CIS, the dosages of radiation (14 Gy) have been compared to
presence of testicular atrophy, and patient compli- higher dosages (up to 20 Gy standard) to determine
ance. Certainly, the major concern with carcinoma the effects on carcinoma in situ and Leydig cell
in situ is the possibility of developing an invasive function. Regardless of the radiation dose, some
germ cell tumor. It has been reported that 50% of patients still require testosterone supplementation
patients with intratesticular germ cell neoplasia over time and testosterone replacement does not
(CIS) will develop an invasive neoplasm within 5 appear to be radiation dose dependent. Furthermore,
years if left untreated. Tumor markers are not ele- relapse of CIS has been reported following irradia-
vated in patients with CIS unless an invasive neo- tion with 14 Gy. The 20 Gy dose remains the stan-
plasm is present. dard dose for testicular CIS, although 16–18 Gy
dosages are likely as efficacious as higher dosages.
Testicular biopsy remains the gold standard for the Chemotherapy is not recommended as primary
diagnosis of testicular CIS. In the United States, treatment for testicular CIS, unless it is planned as
contralateral testicular biopsies are not performed adjuvant therapy for high-risk patients or for
routinely for patients with invasive germ cell patients with metastatic disease. In patients with
known CIS prior to chemotherapy, the estimated
tumors. Biopsies are more often considered or rec- cumulative risk of CIS 5 and 10 years after treat-
ommended if patients have contralateral testicular ment is 21% and 42%. These patients must be fol-
atrophy, intersex disorders, extragonadal germ cell lowed closely with biopsies. Radiation therapy is
tumors, cryptorchidism and as part of an infertility offered for recurrence.

628 EDUCATIONAL REVIEW MANUAL IN UROLOGY


5. Retroperitoneal Lymph Node
Dissection (RPLND)

There are many rationales for performing an Open RPLND


RPLND for testis cancer: 1.) The retroperitoneal An open RPLND is usually performed through a
lymph nodes are usually the first site and often the midline abdominal incision (although thoracoab-
only site of metastatic spread from testicular can- dominal approaches have been utilized as well with
cers; 2.) RPLND alone cures 50%–90% of patients the advantage of easier access to the ipsilateral
with node-positive, low-volume nonseminoma suprahilar region). An incision extends from the
(pN1 or pN2 disease); 3.) Up to 25% of patients xiphicostal junction to just below the midpoint
with pN1 or pN2 disease harbor teratoma in their between the umbilicus and the pubic symphysis.
nodal specimens following primary RPLND which The falciform ligament is divided to prevent injury
is resistant to both chemotherapy and radiation; 4.) to the hepatic capsule during elevation of the chest
Relapse rates following a negative RPLND are wall. Self-retaining retractors are positioned. The
<5%–10%, are rarely in the field of surgical dissec- degree of bowel mobilization depends on the extent
tion, and usually are due to pulmonary metastases or of surgical dissection. A peritoneal incision is made
elevated markers; 5.) Current radiographic modali- medial to the inferior mesenteric vein along the root
ties understage roughly 15%–50% of clinical stage I of the small bowel mesentery, and carried from the
patients depending on risk factors for micrometas- cecum to the ligament of Treitz up towards the
tases; 6.) Albeit uncommon, late relapses following splenic flexure of the colon. The 4th portion of the
primary chemotherapy usually occur in the duodenum is mobilized superiorly. For bulky
retroperitoneum, have a poorer prognosis and are adenopathy, or for added exposure, the inferior
usually chemorefractory; and 7) Untreated mesenteric vein can be suture-ligated and divided.
retroperitoneal metastases are almost always fatal. The posterior peritoneal incision is carried around
Based on these and other observations, RPLND is a the cecum and the ascending colon is mobilized
critical component in the management of patients along the paracolic gutter to the foramen of
with testicular germ cell tumors. Winslow. The duodenum is Kocherized off of the
underlying vena cava, and the large and small bowel
is detached from the underlying retroperitoneum,
and great vessels to the level of the right and left
Technique and Exposure

Prior to the surgical procedure, patients should be renal hilum. The superior mesenteric artery is iden-
counseled regarding options for sperm banking tified above the left renal vein and care must be
even if nerve-sparing procedures are planned. A taken to avoid unnecessary direct compression of
mechanical bowel preparation is self-administered the vessel during bowel retraction. Likewise, the
the day prior to surgery. The surgeon should have an pancreas is carefully padded and retracted to avoid
intimate knowledge of the patient’s clinical stage to injury. The omentum, transverse colon, right colon
include tumor distribution, overall tumor burden and small bowel are placed on the anterior chest
and his vascular anatomy. Vascular anomalies are wall surrounded by moistened, warmed, laparotomy
not uncommon. Up to 20% of patients have acces- pads. The bowel should be inspected frequently
sory renal arteries. Occasionally, a patient will have throughout the case to ensure adequate vascular per-
a retroaortic renal vein that could be mistaken for a fusion. The descending colon and the splenic flex-
lumbar vein. Up-to-date tumor markers should be ure are mobilized, such that the medial aspect of the
obtained as most (80%) patients with elevated underlying left kidney, the left renal hilum and the
markers fail a primary RPLND. Induction gonadal vein are completely visualized. The inferior
chemotherapy is recommended for patients with mesenteric artery is usually left intact, although
new or persistent marker elevation following division is sometimes required based on the volume
orchiectomy. Very few patients with elevated mark- of tumor. The common iliac arteries are visualized
ers following second-line and salvage chemother- and dissected just beyond their bifurcation.
apy regimens undergo an attempted desperation
RPLND. Unfortunately, the outcome for this subset Anatomical boundaries and landmarks are carefully
of patients is poor. identified prior to proceeding with the surgical dis-
section. The superior boundary of dissection
includes the right and left renal artery and renal

CHAPTER 20: TESTICULAR CANCER 629


vein. In low-stage disease, retrocrural or suprahilar control of the lymphatic vessels results in postoper-
involvement is uncommon. With advanced disease, ative chylous leaks in a small percentage of
these regions should be carefully scrutinized prior patients—more common following postchemother-
to surgery with abdominal imaging and then pal- apy RPLNDs.
pated intraoperatively. Lateral boundaries of dissec-
tion include the ureters and the ipsilateral gonadal Robot-assisted and Pure Laparoscopic RPLND
veins. The inferior boundary courses along the com- Compared to an open RPLND, a laparoscopic
mon iliac vessels to their bifurcation distally. The RPLND offers patients several potential advantages
posterior boundaries are the anterior spinous liga- including: decreased blood loss, decreased peri-
ment and the psoas muscle fascia, which are identi- operative morbidity, decreased length of hospi-
fied during the course of the lymphadenectomy. The talization, quicker convalescence and favorable
lymphatic tissue is dissected away from the medial cosmetic results. Recently reported, robot-assisted
aspect of the ureters. The gonadal vein is divided at laparoscopic RPLND offers the additional advan-
the level of the renal vein (left side primary) or the tages of 3-dimensional vision, improved instrument
inferior vena cava (right side primary), and dis- dexterity and reduction of tremor. In a recent
sected into the ipsilateral internal inguinal ring for review of laparoscopic RPLND series, including
complete excision. If nerve sparing is planned, a studies comparing open and laparoscopic
prospective nerve dissection is initiated once the approaches, the operating time was longer for
surgical boundaries are established (although care laparoscopic procedures (204 vs 186 min), but the
should be taken inferiorly along the common iliacs complication rate (15.6% vs 33%), reintervention
to avoid injury to nerve trunks beyond the hypogas- rate (1.4% vs 6.6%) and hospital stay (3.3 vs 6.6
tric plexus). The hypogastric plexus and the post- days) favored the laparoscopic technique (P<0.05).
ganglionic sympathetic fibers are prospectively In experienced hands, the principles of dissection
identified, meticulously dissected and gently ele- during the lymphadenectomy should be identical to
vated with vessel loops. As the dissection proceeds open techniques (ie, templates of dissection, disen-
and as aorta and IVC are mobilized, the sympathetic gagement of the IVC and aorta form the posterior
ganglia and the sympathetic chains are more easily abdominal wall, nerve sparing) although some
identified and the sympathetic structures are pre- investigators report limiting the dissection anterior
served to maintain emission and antegrade ejacula- to the lumbar vessels. Median local and distant can-
tion. The left side postganglionic fibers course over cer control rates are comparable between laparo-
top of the aorta and should be dissected prior to ini- scopic RPLND and open RPLND with few in-field
tiating the split and roll maneuver. The lymphatic relapses and few distant recurrences. The number of
tissue is split along the anterior surfaces of the infe- lymph nodes resected is reportedly higher with open
rior vena cava and the aorta, the renal veins and the techniques compared to laparoscopic procedures,
iliac arteries. The IVC and the aorta are rolled later- and these findings need to be carefully considered
ally and medially identifying lumbar arteries and when addressing the therapeutic nature of a lap
veins, which are ligated and divided to disengage RPLND. Presently, however, most patients with
the great vessels from the posterior abdominal wall. positive retroperitoneal nodes are treated with adju-
During the lymphadenectomy, great care is taken to vant chemotherapy regardless of their risk for dis-
carefully dissect and expose the renal arteries, ease relapse. Thus, it is difficult to determine the
which are at the superior extent of the dissection. therapeutic potential of laparoscopic RPLND as a
Accessory renal arteries should be considered. sole treatment option for patients with pathologic
Lymphatics require precise control with surgical node-positive disease.
clips or ties, particularly at the peripheral limits of
dissection. Particular attention to lymphostasis is
required inside the right crus of the diaphragm
Primary Landing Zones and

under the right renal artery where lymphatics course


Template Dissections

to join the cisternae chyli. Failure to achieve good Carefully documented anatomic studies and
detailed mapping studies of RPLND pathology
specimens clarify the patterns of spread from the

630 EDUCATIONAL REVIEW MANUAL IN UROLOGY


primary testicular tumor and help to define the most should be given towards performing a full bilateral
common sites of lymph node involvement. For left RPLND for patients with multiple microscopic pos-
sided tumors, the first echelon of lymph node itive nodes and certainly for gross lymphadenopa-
drainage is the infrahilar left paraaortic and preaor- thy identified during primary RPLND to maximize
tic regions followed by the interaortocaval lymph the oncologic efficacy of the lymph node dissection.
nodes. Left to right crossover to the paracaval
region is uncommon for low-stage or low-volume
disease. Practically, there is very little nodal tissue
Emission, Ejaculation and

in the precaval and the paracaval location during a


Nerve Preservation

primary RPLND. If the patient has positive nodes Normal antegrade ejaculation involves the precise
during a left modified template dissection, the para- coordination of 3 events. During orgasm, the blad-
caval tissue can easily be resected during mobiliza- der neck closes and there is seminal emission, fol-
tion of the inferior vena cava, which is required for lowed by ejaculation. Sympathetic nerves originate
prospective nerve sparing, and complete excision of from the T12 to L4 spinal cord levels. The nerves
the interaortocaval nodes. Lymphatics from right- enter the paravertebral sympathetic ganglia, then
sided testicular tumors drain primarily to the postganglionic nerves course medially to form a
interaortocaval lymph nodes followed by the pre- plexus of nerves (hypogastric plexus) overlying the
caval and the preaortic regions. Intraoperative node aorta, concentrated between the inferior mesenteric
mapping studies in patients undergoing primary artery takeoff and the aortic bifurcation (Figure 1).
RPLND reveal that right to left crossover to the The sympathetic fibers travel from the hypogastric
paraaortic nodes occurs in roughly 20% of patients plexus via the pelvic plexus to innervate the vas def-
with pathologic node-positive disease. Thus, the erens, seminal vesicles, bladder neck and prostate.
preaortic and paraaortic nodes should be removed Ejaculation occurs via neural input from sacral and
for right modified template dissections. Table 7 lumbar spinal cord levels.
demonstrates the retroperitoneal regions that must
be dissected at a minimum to maximize the onco- Pudendal somatic innervation from S2 to S4 facili-
logic efficacy of a primary RPLND. Consideration tates relaxation of the external urethral sphincter

Table 7

Primary Landing Zones and Templates for Modified Primary RPLND

Predominant Minimum Dissection


Landing Zones in Modified Template

Left side template Para-aortica Para-aortic


Preaortic Preaortic
Interaortocaval Inter-aortocaval
Ipsilateral common iliac
Precaval

Right side template Inter-aortocavala Interaortocaval


Precaval Precaval
Preaortic Preaortic (above IMA)
Paracaval
Para-aortic (above IMA)
Ipsilateral common iliac

RPLND = retroperitoneal lymph node dissection; IMA = inferior mesenteric artery; a= primary landing zone

CHAPTER 20: TESTICULAR CANCER 631


Figure 1

Sympathetic nerves originating from the T12 to L4 spinal cord levels

and contraction of the bulbourethral and perineal mise the therapeutic aspects of the surgical proce-
muscles. Injury to the sympathetic ganglia, the dure.
postganglionic nerves and the hypogastric plexus
during the course of an RPLND causes failure of
emission and retrograde ejaculation. Traditionally,
Complications of RPLND

bilateral infrahilar RPLNDs were performed for A primary RPLND is generally very well tolerated.
low-stage disease. Rates of ejaculatory dysfunc- Minor complications (5%–15% of patients) and
tion were prohibitively high with bilateral major complications (5%–7% of patients) are
non–nerve-sparing techniques. Prospective nerve uncommon and occur more predominantly in
sparing techniques and modified template dissec- patients undergoing bilateral RPLND compared to
tions were described and performed, which template dissections (Table 8). The development of
decreased the rates of ejaculatory dysfunction to a small bowel obstruction requiring exploration and
<5% while maintaining oncologic efficacy. Post- lysis of adhesions occurs in ≤1% of patients. Emis-
ganglionic nerves and the hypogastric plexus are sion and antegrade ejaculation is preserved in
prospectively identified and meticulously dis- 93%–99% of patients undergoing nerve-sparing
sected, coursing over the common iliac arteries procedures. Most minor complications are related to
and above the distal aorta. Dissection proceeds wound infections and atelectasis, and usually do not
proximally toward the ganglia and the sympathetic prolong hospitalization.
trunks. Prospective nerve dissections are possible
with primary or postchemotherapy RPLNDs. The rate of complications related to postchemother-
However, nerve preservation should not compro- apy surgery is 20%–25%. Pulmonary complications

632 EDUCATIONAL REVIEW MANUAL IN UROLOGY


Table 8

Complications of Retroperitoneal Lymph Node Dissection

Primary RPLND

Minor Major Delayed

Wound infection DVT/PE Death (rarely reported)a

Pneumonia/atelectasis Bowel injury DVT/PE

Prolonged ileus Pancreatitis Hydronephrosis

UTI Unplanned nephrectomy Ventral hernia

Lymphocele Chylous ascites

Bleeding requiring transfusion Bowel obstruction (1%) Ejaculatory dysfunction


(0%-7%) in nerve-
sparing cases

Vascular injury

Postchemotherapy RPLND

Wound infection Pulmonary (prolonged ventilation, Death (1%)a


bleomycin toxicity, pneumonia, PE)

Pneumonia / atelectasis DVT/PE DVT/PE

Prolonged ileus Bowel obstruction (2%) Gastric ulcer

UTI Chylous ascites (2%) Hydronephrosis

Lymphocele Ureteral injury Ventral hernia

Bleeding requiring transfusion Pancreatitis Ejaculatory dysfunction

Renal infarction

Unplanned nephrectomy or organ resection

Bowel injury

Gastrointestinal bleeding

Hemorrhage

UTI= urinary tract infection; DVT= deep vein thrombosis, PE= pulmonary embolus
a
= immediate or delayed

CHAPTER 20: TESTICULAR CANCER 633


6. Management of Nonseminoma

account for the majority of minor and major problems The management of nonseminomatous germ cell
in the postoperative setting and are experienced by tumors (NSGCT) is approached very systematically
8%–10% of patients overall (Table 8). Bleomycin is (Figures 2 and 3).With careful risk stratification
used routinely as part of a standard chemotherapy pro- based on the precise interpretation of the orchiec-
tocol. A side effect of bleomycin is pulmonary inter- tomy specimen and the clinical staging studies, as
stitial fibrosis, which causes a restrictive pattern lung well as the careful integration of surgery and
disease and a probable oxygen diffusion deficiency. chemotherapy, most patients with NSGCT have
Both during surgery and in the perioperative setting, it very favorable outcomes.
is important to judiciously manage intravenous fluids
and balance crystalloid and colloid administration to
prevent pulmonary edema and increasing inspired
Clinical Stage 1 and Low-Volume

oxygen demands. Aggressive pulmonary physiother-


Clinical Stage 2 NSGCT

apy is mandatory. There are 3 management options available to


patients with clinical stage I disease and normal
Renovascular injuries and gastrointestinal complica- markers (cT1-4,N0,M0,S0; normal markers prior to
tions account for the majority of the remaining com- orchiectomy or normalization of markers following
plications in postchemotherapy patients. Injury to the radical orchiectomy). These options include surveil-
renal vessels may result in the development of hyper- lance or primary treatment with either chemotherapy
tension and/or renal atrophy, or may result in the need or RPLND. A risk-adapted management approach is
for an unplanned nephrectomy. Small bowel obstruc- based on the accurate interpretation of the patient’s
tions often are successfully managed with nasogastric primary pathology, as well as his staging studies.
tube decompression. Exploration is reserved for Each management option should be discussed with
patients who exhibit signs or symptoms of toxicity or the patient as part of his counseling following radical
who fail to respond to conservative measures. Causes orchiectomy. Patients should be counseled regarding
of a prolonged ileus include: mechanical obstruction, sperm banking prior to therapy.
pancreatitis, urinomas, retroperitoneal or mesenteric
hematomas, bowel ischemia and bowel infarction. Stage IS NSGCT
Patients with clinical stage IS disease have persis-
Chylous ascites occurs in 2%–7% of patients overall, tent serum tumor marker elevation or new marker
and can be associated with IVC resection, suprahilar elevation following orchiectomy and require full
dissections and hepatic resections. Careful lymphatic induction chemotherapy (BEP x 3 or EP x 4). Ele-
stasis must be ensured throughout the course of dis- vated markers imply that the patient has systemic
section, particularly inside the right diaphragmatic disease and he is unlikely to be cured by primary
crus where lymphatics course to the cisternae chyli. RPLND alone. Up to 80% of patients with elevated
Most patients with chylous leaks experience abdomi- markers prior to RPLND will relapse. In a recent
nal distention, prolonged ileus or pleural effusions case series of 24 patients with clinical stage IS
(more common with retrocrural dissections). The NSGCT, 7 underwent primary RPLND and 17
physical exam reveals a fluid wave. A paracentesis is underwent primary chemotherapy, with 3 electing
both diagnostic and therapeutic, and may be required postchemotherapy RPLND. At a median follow-up
on multiple occasions. The fluid is typically milky, of 35 months, 4 of the 14 patients who had primary
demonstrating high-fat and high-protein content. Up chemotherapy without RPLND relapsed in the
to 80% of patients are managed successfully with retroperitoneum. 6 of 7 patients who underwent pri-
combinations of conservative measures including mary RPLND had viable GCT and 4 who did not
repeated paracentesis, diuretics, total parenteral nutri- receive adjuvant chemotherapy relapsed, suggest-
tion and low-fat diets with medium-chain triglyc- ing that primary chemotherapy is preferred to pri-
erides. Smaller numbers of patients require perito- mary RPLND. Contrary to this was a report of 24
neovenous shunts, which unfortunately are associated patients with clinical stage IS NSGCT who
with high surgical revision rates. underwent primary RPLND. Positive retroperi-
toneal lymph nodes were identified in 9 patients—5
of whom received chemotherapy postoperatively.

634 EDUCATIONAL REVIEW MANUAL IN UROLOGY


in patients with clinical stage I NSGCT. Patients
with normal markers, LVI and high percentage EC
Table 9

have at least a 40%–50% risk of relapse due to


micrometastases. The presence of 1 risk factor
Reported Risk Factors for Micrometastases

(either LVI or majority EC) decreases the risk of


and Relapse in Clinical Stage I Seminoma

relapse to 20%–40%, whereas the absence of both


and NSGCT

risk factors decreases the risk of relapse to <20%.


Over 70% of patients with clinical stage I pure EC
have occult metastases detected at RPLND.
Seminoma

Surveillance
Elevated serum tumor markers
Lymphatic or vascular invasion
Primary tumor size >4-6 cm Surveillance following radical orchiectomy is an
Rete testis invasion option available to many patients who would other-
wise be overtreated and subjected to an unnecessary
RPLND or to unnecessary chemotherapy. The ratio-
nale for considering surveillance is based on the fol-
NSGCT

Elevated serum tumor marker lowing observations: current radiographic imaging


Majority embryonal cell carcinoma (>50%) studies have improved diagnostic accuracy, radical
Lymphatic or vascular invasion orchiectomy alone cures the majority of patients
MIB-1 staining with clinical stage I tumors and early relapses can be
Absence of yolk sac elements cured with cisplatin-based chemotherapy. Surveil-
Higher T stage (T2-T4) lance studies consistently show that 25%–35% of
Large tumor size (>4-6 cm) patients (regardless of risk factors) will relapse
while on observation. The majority of relapses
occur within 1 year (median time to relapse approxi-
mately 5–6 months). Relapses beyond the 2nd year
At a median follow-up of 2.9 years, no patient had are rare, although late relapses have been reported
relapsed in the retroperitoneum. and long-term follow-up is mandatory. Generally,
60% of relapses involve the retroperitoneal lymph
Risk Factors for Occult Metastases nodes (+ other sites), 30%–45% of patients have
in Clinical Stage I NSGCT normal markers at relapse, and 20% of patients will
Risk factors for the presence of occult micrometas- have a marker-only or a lung-only relapse. The type
tases for clinical stage I seminoma and clinical stage of relapse directs subsequent therapy (Figure 2).
I NSGCT are listed in Table 9. The most important Patients detected to have a low-volume retroperi-
and the most consistent risk factors for relapse in toneal relapse with normal markers (stage IIA)
patients with NSGCTs include elevated serum should be considered for a primary RPLND.
tumor markers, the presence of lymphovascular Patients with stage IIB disease or other patterns of
invasion (LVI) and the presence of majority embry- relapse should be considered for standard induction
onal cell carcinoma (EC). MIB-1 staining has been chemotherapy. The overall survival for patients
evaluated as a risk factor for occult metastases as entered onto surveillance protocols ranges from
well. MIB-1 is a monoclonal antibody developed 96%–100%. The optimal candidate for surveillance
for application in paraffin-embedded tissues, and it has cT1,N0,M0,S0 disease (normal markers, nor-
is used to measure the immunohistochemical mal radiographic studies), a low % EC and no LVI.
expression of Ki-67 (a nuclear antigen found in all The patient should be highly motivated, compliant
cycling human cells and a marker for cellular prolif- and have consistent access to surveillance proto-
eration). The results of MIB-1 staining are mixed. cols.
MIB-1 staining was found to be a promising marker
for predicting the presence of occult metastases in Primary Chemotherapy
some studies, whereas MIB-1 staining in other In order to avoid the side effects and the potential
reports did not correlate with the pathological stage morbidity of an RPLND, several studies have exam-

CHAPTER 20: TESTICULAR CANCER 635


Stages of Seminoma
Figure 2

636 EDUCATIONAL REVIEW MANUAL IN UROLOGY


Figure 3

Stages of low-stage NSGCT

CHAPTER 20: TESTICULAR CANCER


637
ined the role of primary chemotherapy in high-risk RPLND is diagnostic; it is therapeutic as a sole
patients (clinical stage IB). Relapse rates following treatment modality in 50%–90% of patients with
2 cycles of cisplatin-based chemotherapy range pathologic N1 and N2metastatic cancer; and it
from 2%–4%. Patients with clinical stage IS disease secures the retroperitoneum from disease recur-
and patients with clinical stage IIA and IIB disease rence, as in-field relapses are rare when the RPLND
who elect primary chemotherapy require standard is performed by experienced surgeons. Pathologic
induction dosages (3 courses BEP or 4 courses EP). stage I patients have a relapse rate of 5%–10% fol-
Primary chemotherapy is utilized more frequently lowing a negative RPLND. Usually, the relapse
in Europe compared to the United States. The disad- occurs out of the field of dissection (lung, marker or
vantages of primary chemotherapy include both combination). Nerve-sparing dissections are
short- and long-term chemo-related toxicity: renal accomplished with tremendous success in almost
insufficiency, neuropathy, ototoxicity, myelosup- every patient undergoing a primary modified tem-
pression, alopecia, mucositis, infertility, pul- plate or bilateral RPLND. A bilateral RPLND
monary fibrosis and Raynaud’s phenomenon. In should be considered for patients determined to
addition, BEP, particularly etoposide, is associated have multiple microscopically positive nodes dur-
with increased rates of cardiovascular disease (HR ing surgery, and for patients who are found to have
= 5.7) and atherosclerotic disease (HR = 4.7). previously undiagnosed gross lymphadenopathy.
Lower cumulative exposure to chemotherapy The morbidity of a primary RPLND is very low and
agents may have less short- and long-term toxicity, associated mortality is negligible.
but a safe lower limit of exposure has not been
established. A primary RPLND is an option for clinical stage IIA
and stage IIB patients with normal markers. The
A large prospective, randomized trial was under- surgeon should have a low threshold to perform a
taken in Norway and Sweden of patients with clini- bilateral nerve-sparing dissection in this setting.
cal stage I nonseminoma testis cancer Stage IIB patients are candidates for a primary
(SWENOTECA) to compare chemotherapy and RPLND if only lymph node metastases are present
surveillance, depending on the presence of vascular and located within the primary landing zone. If
invasion in the primary tumor. Patients with vascu- there are multifocal metastases, lymph node metas-
lar invasion (VASC) were randomized to 1 or 2 tases outside the primary landing zones, or if the
cycles of BEP, while patients without VASC were patient is symptomatic, primary induction
randomized to 1 cycle of BEP or surveillance. After chemotherapy is recommended.
1 course of BEP, with median follow-up of 4.7
years, only 3.2% of VASC-positive and 1.3% of A recent randomized phase III trial conducted by
VASC-negative patients relapsed. On surveillance, The German Testicular Cancer Study Group com-
14.5% of VASC-negative patients relapsed. The pared primary RPLND to 1 course of primary
group currently recommends 1 course of BEP for chemotherapy (BEP) following orchiectomy for
VASC-positive patients and surveillance or 1 course clinical stage I testis cancer. A total of 382 patients
of BEP for VASC-negative patients. were randomized into each treatment arm and the
mean follow-up after surgery or chemotherapy was
Retroperitoneal Lymph Node Dissection 4.7 years. There was a 7.59% difference in the 2-
(RPLND) year recurrence free survival rate favoring
In the United States, a primary RPLND remains the chemotherapy (2 recurrences with chemo, 13 recur-
favored management option for many patients with rences with RPLND). 7 of the surgical patients
clinical stage I disease or low-volume clinical stage recurred solely within the retroperitoneum or within
II disease (stage IIA and some stage IIB) and normal the inguinoscrotal region. Study sites included both
serum markers. In patients with clinical stage I dis- specialized centers with experience in the surgical
ease, candidates for a primary RPLND include management of testicular cancer as well as commu-
intermediate to high-risk patients (based on risk fac- nity hospitals with low-volume RPLND experience.
tors for occult micrometastases) with normal serum In addition, longer patient follow-up is required to
tumor markers, and poorly compliant patients. An determine if there will be more recurrences in the

638 EDUCATIONAL REVIEW MANUAL IN UROLOGY


chemotherapy arm as the mean time to delayed (Table 10, Figure 3). Patients with advanced stage
relapse following primary chemotherapy is approxi- NSGCT are initially treated with platinum-based
mately 6 years. chemotherapy. Induction chemotherapy in good risk
patients consists of 3 courses BEP (bleomycin,
Adjuvant Chemotherapy etoposide, cisplatin) or 4 courses EP (etoposide, cis-
Following Primary RPLND platin) based on the results of randomized clinical
The decision to recommend chemotherapy follow- trials showing equivalent response rates and sur-
ing an RPLND is based upon the pathologic stage, vival. Intermediate-risk and poor-risk patients
the completeness of resection, the status of the require 4 courses of BEP as induction therapy. Poor-
tumor markers and patient compliance. The ran- risk patients often present with bulky lym-
domized, prospective Testicular Cancer Intergroup phadenopathy, severe symptoms and malnourish-
Study revealed that 2 courses of cisplatin-based ment, making treatment delivery even more chal-
adjuvant chemotherapy usually prevents relapse in lenging.
patients with pathological Stage II testicular cancer
treated with orchiectomy and retroperitoneal lymph However, chemotherapy should be started as soon
node dissection. Survival was similar in patients as possible. While response and remission rates
with pathologic stage II disease randomized to approach 60%–70% with front-line therapy, many
observation and treated with standard induction poor-risk patients relapse or die from disease. Sec-
chemotherapy at the time of relapse. Relapse is ond-line therapy consists of conventional
uncommon for patients with pathologic N1 testis chemotherapy (TIP-paclitaxel, ifosfamide, cis-
cancer (≤5 positive lymph nodes, all nodes ≤2 cm in platin; or VeIP-vinblastine, ifosfamide, cisplatin) or
size and no extranodal extension) and studies reveal high-dose chemotherapy (with or without autolo-
that up to 90% of patients with N1 disease are cured gous bone marrow transplantation or stem cell sup-
following RPLND alone. Thus, observation is the port). In patients who receive ifosfamide, mesna
standard of care for completely resected, compliant should be used before and after each dose of ifos-
patients with normal markers following RPLND. famide to prevent hemorrhagic cystitis.
However, patients with pathologic N2 disease (>5
positive nodes, any node >2 cm, extranodal exten- There is controversy regarding the management of
sion) have a higher risk of relapse with observation patients with a complete serologic response and a
(~50%). 2 cycles of adjuvant chemotherapy (BEP x complete radiographic response to primary
2 or EP x 2) are recommended for this group of chemotherapy. Reports of routinely performed
patients following RPLND. Patients with elevated postchemotherapy RPLND after complete response
markers postop, remaining unresected retroperi- (CR) show an incidence of 15%–20% for teratoma.
toneal tumor or distant disease, as well as patients This has led to a recommendation for PC-RPLND in
with pathologic N3 disease following RPLND, all patients with a CR. However, recent retrospec-
require induction chemotherapy (BEP x 3 or tive studies support observation in this setting. With
EP x 4). a median follow-up of 15.5 years, 1 analysis of 141
patients shows an overall recurrence-free survival
and cancer-specific survival of 90% and 97%,
respectively. Relapse was dependent on IGCCG
Advanced NSGCT

risk (RFS/CSS of 95%/99% for good risk and


(Clinical Stage IIB-IIC, Clinical Stage III)

Primary Chemotherapy 73%/91% for intermediate or poor risk). Overall,


The prognosis of patients with advanced NSGCT 12 patients (9%) recurred, 6 (4%) in the retroperi-
has been classified according to a validated model toneum. Viable GCT was the most common histol-
published by the International Germ Cell Cancer ogy detected, whereas no patient had pure teratoma.
Collaborative Group (1997). The classification
stratifies patients into good, intermediate and poor Postchemotherapy Residual Mass
risk groups based on the levels of tumor markers Surgery plays a critical role in the management of
(AFB, -HCG, LDH) and the poor risk prognostic patients with advanced NSGCT. Following induc-
factors of metastases to organs other than the lungs tion chemotherapy, 10%–15% of patients are found

CHAPTER 20: TESTICULAR CANCER 639


Table 10

International Germ Cell Consensus Classification

Seminoma

Risk Level Good Intermediate No Patients Classified


as Poor Prognosis

Primary site Any primary site Any primary site


AND AND
Extent of mets No nonpulmonary visceral mets Nonpulmonary visceral mets
AND AND
Marker status Normal AFP Normal AFP
Any hCG Any hCG
Any LDH Any LDH

Patient (%) 90% 10%

5 year PFS 82% 67%

5 year OS 86% 72%

Nonseminoma

Primary site Testis/RP primary Testis/RP primary Mediastinal primary


AND AND OR
Extent of mets No nonpulmonary No nonpulmonary Non-pulmonary
visceral mets visceral mets visceral mets
AND AND OR
Marker status Good Markers Intermediate markers Poor markers
LDH <1.5 x nl; LDH >1.5 – 10x nl or LDH >10x nl or
hCG <5000; hCG >5000 – 50,000 or hCG >50,000 or
AFP <1000 AFP >1000 – 10,000 AFP >10,000

Patient (%) 56% 28% 16%

5 year PFS 89% 75% 41%

5 year OS 92% 80% 48%

PFS = progression free survival; OS = overall survival


Data from International Germ Cell Cancer Collaborative Group. International Germ Cell Consensus
Classification: a prognostic factor-based staging system for metastatic germ cell cancers.
J Clin Oncol. 1997;15(2):594-603.

640 EDUCATIONAL REVIEW MANUAL IN UROLOGY


to have viable tumor in their retroperitoneum and up standard for patients with completely resected ter-
to 40% of patients will have residual teratoma. atoma or fibrosis, whereas patients with completely
There is some controversy regarding patient selec- resected viable germ cell tumor in the surgical spec-
tion for a postchemotherapy RPLND. Generally, imen following induction chemotherapy derive a
patients with a residual mass and normal tumor survival benefit from 2 additional courses of
markers following chemotherapy should undergo a chemotherapy.
postchemotherapy RPLND. However, the definition
of a normal CT scan and a residual mass varies There are several rationales for resecting residual
among many centers of excellence. Definitions of a teratoma. First, teratoma may grow, invade or
normal CT include no residual mass, residual lymph obstruct vital organs and become unresectable
nodes <1 cm or residual lymph nodes <1.5–2 cm. (growing teratoma syndrome). Second, teratoma is
Currently, there are no widely accepted clinical radioresistant and chemoresistant. Finally, 2%–8%
parameters or radiographic studies that differentiate of teratomas undergo malignant transformation to
residual viable germ cell elements (embryonal cell, non-germ cell elements such as sarcoma and adeno-
yolk sac, choriocarcinoma, seminoma) from ter- carcinoma. These tumors do not respond to standard
atoma or fibrosis in the residual mass. In addition, cisplatin-based chemotherapy regimens.
the absence of teratoma in the primary tumor does Chemotherapy tailored to the malignant histology is
not preclude its presence in the retroperitoneum. recommended, although surgical resection remains
PET scans may suggest the presence of viable non- the mainstay of therapy. The prognosis for patients
teratomatous germ cell tumor, but its utility in with teratoma and malignant degeneration is signifi-
NSGCT patients is limited by its inability to differ- cantly compromised.
entiate teratoma from fibrosis. Most centers recom-
mend observation for patients who experience a Most patients that relapse with testicular cancer do
complete response to chemotherapy and reserve so within the first 2 years. However, long-term fol-
surgery for a detectable residual mass regardless of low-up is necessary for all patients with testicular
dimension, although there are advocates of per- cancer secondary to the possibility of late relapses.
forming postchemotherapy RPLNDs for all patients A late relapse is defined as a tumor recurrence
who had clinical node-positive disease before beyond 2 years after primary therapy and occurs in
chemotherapy, regardless of the postchemotherapy 2%–3% of patients following a complete response.
CT findings. The later approach is based on the Patients at all stages of disease are at risk. The
reported incidence of residual teratoma in up to 15% median time to a late relapse is 7 years (range 2–32
of patients with normal CT scans prior to years) and the most common site of recurrence is in
postchemotherapy RPLND. Although the biologi- the retroperitoneum or in the chest. Patients are
cal potential of teratoma in this setting is debatable, managed best with surgical resection, as chemother-
the presence of unresected teratoma could account apy alone is rarely effective. Viable germ cell tumor
for some of the few late relapses in patients who and teratoma are found in 80% and 20% of surgical
experience complete responses with induction specimens respectively. Unfortunately, the long-
chemotherapy. The overall relapse rate in patients term disease-free survival in patients who suffer a
undergoing a complete RPLND and who were late relapse is poor (reported to be approximately
found to have teratoma or fibrosis in the resected 25%).
specimen is 5%–10%. A recent report found that, in
patients with only teratoma in the resected specimen Patients with persistently elevated markers follow-
following initial postchemotherapy RPLND, the 10- ing induction chemotherapy require second-line or
year probability of freedom from recurrence was salvage chemotherapy with very few exceptions (ie,
80%. The size of the residual mass and IGCCCG mild false elevations of markers due to liver toxic-
risk classification were significant predictors of dis- ity, hypogonadism, etc). In patients who respond to
ease recurrence. In the patients that recurred, 50% second-line and other salvage chemotherapy regi-
recurred with viable germ cell tumor and 50% mens, the importance of a complete resection can-
recurred with teratoma (including some with malig- not be understated. The histologic characteristics of
nant transformation). Nevertheless, observation is resected metastatic sites after salvage chemotherapy

CHAPTER 20: TESTICULAR CANCER 641


are significantly different from patients undergoing expense of cancer control and complete tumor exci-
RPLND after induction chemotherapy. Viable non- sion. Every effort should be made to completely
teratomatous germ cell tumor, teratoma and fibrosis resect all retroperitoneal disease during the initial
are found in 50%, 40% and 10% of patients, respec- RPLND, as a complete resection significantly influ-
tively. Postoperative chemotherapy does not ences relapse and survival. Overall, relapse rates
improve disease-specific survival for patients with following an initial RPLND (regardless of risk
viable germ cell elements following salvage group) approach 20%. However, following repeat
chemotherapies (an important distinction from RPLNDs, relapse rates range from 35%–50% and
patients with viable germ cell elements in the com- disease-free survival is negatively impacted.
pletely resected specimen following standard induc-
tion chemotherapy). Furthermore, only select Resection of Nonretroperitoneal Sites
patients benefit from an RPLND in the setting of Small numbers of patients require resection of
marker elevation or disease progression after abdominal viscera or distant metastatic sites. Con-
aggressive systemic chemotherapy (desperation sistent with findings in the postchemotherapy
RPLND). Generally, a desperation RPLND is dis- retroperitoneal mass, the size of the mass at distant
couraged when markers remain elevated after sal- sites does not correlate with the pathological find-
vage regimens, as resection is often incomplete due ings (ie, the presence or absence of viable tumor or
to associated bulky retroperitoneal and visceral dis- teratoma). Likewise, the potential for histologic dis-
ease. However, 20%–30% of patients who are com- cordance between the resected retroperitoneal resid-
pletely resected in a desperation attempt may expe- ual mass and the tumor excised from distant sites is
rience long-term disease-free survival. well established and reported to be approximately
35%. Simultaneous excision of retroperitoneal and
The oncologic principles and the techniques of a pulmonary or mediastinal disease is an option, par-
postchemotherapy RPLND are similar to those dis- ticularly if all disease can be excised through a sin-
cussed previously, with the following caveats: gle incision.

1. A full bilateral retroperitoneal node dissection is


the standard of care. There is a variable distribu-
tion of metastases outside the boundary of a
modified dissection following chemotherapy,
and it is difficult to differentiate tumor from
fibrosis intraoperatively.

2. There is no defined role for only excision or


biopsy of the residual nonseminomatous mass.

3. Chemotherapy results in tremendous fibrosis,


making vascular dissection significantly more
difficult.

4. Retrocrural and suprahilar dissections are


required more often.

5. Complete surgical resection often involves adja-


cent organ resections (kidney, adrenal, bowel,
liver, etc) and vascular resections, particularly
for bulky disease.

6. Prospective nerve-sparing procedures are feasi-


ble depending on the bulk of tumor, but not at the

642 EDUCATIONAL REVIEW MANUAL IN UROLOGY


7. Management of Seminoma

Approximately 70% of patients with pure semi- separate surveillance trials of patients with clinical
noma present with stage I disease. Metastases to the stage I seminoma. If both risk factors were present,
retroperitoneum or to pulmonary and visceral sites the 5-year relapse rate was 31%, compared to 16%
are less common and occur in 20% and 10% of with 1 risk factor and 12% with no risk factors. In
patients, respectively. The disease characteristics of other studies, the presence or absence of LVI was
seminoma differ significantly from patients with identified as an important prognostic factor for
NSGCTs (70% of whom have metastatic disease at relapse.
presentation). According to the International Germ
Cell Consensus Classification, 90% of patients with The majority of relapses in patients with clinical
seminoma are classified as having a good progno- stage I seminoma occur in the paraaortic and in the
sis. Contrary to patients with NSGCTs, no patient interaortocaval lymph nodes (80%–90%), with a
with a pure seminoma is classified as having a poor median time to relapse between 12 and 18 months
prognosis (Table 10). Nevertheless, metastatic following radical orchiectomy. Long-term follow-
seminoma can be fatal. At the time of death, most up is critical, as late relapses beyond 5 years have
patients have liver and lung involvement. Addition- been reported. Most patients that relapse are treated
ally, 50% of patients have bone metastases and 25% with radiation therapy and, of the 10%–20% of
of patients have brain metastases. Many of those patients who experience a second relapse, almost
dying of disease actually have nonseminomatous all are salvaged with systemic chemotherapy. The
germ cell elements in metastatic sites. overall survival of patients who elect surveillance
is 98%–100%, which is similar to that reported for
Stage I Seminoma patients treated with primary adjuvant radiation.
The major disadvantage with surveillance for stage
There are 3 management options for patients with I seminoma is the fact that a reliable serum tumor
stage I seminoma; surveillance, chemotherapy and marker does not exist for the disease. Thus, surveil-
radiation (Figure 4). lance protocols depend heavily on radiographic
imaging studies. The optimal candidate for surveil-
Surveillance lance is the reliable patient with a pure seminoma,
Surveillance has emerged as an acceptable option normal markers, a tumor size <4 cm, no rete testis
for many patients with clinical stage I testicular invasion and no evidence for lymphovascular inva-
seminoma. This option is based on the results of sion.
several prospective trials that reveal cancer-specific
survival rates ranging from 98%–100% in patients There are concerns regarding the overtreatment of
entered onto surveillance protocols following radi- patients with clinical stage I seminoma and the
cal orchiectomy. In addition, while the short-term unnecessary exposure to potentially harmful long-
side effects of low-dose radiation therapy are mini- term side effects of adjuvant chemotherapy and
mal, there are still concerns regarding infertility, adjuvant radiation therapy. Thus, at the present
bowel toxicity and the possible association of radia- time, many centers of excellence recommend
tion therapy with secondary malignancies. It has surveillance in acceptable candidates and, impor-
been reported that the relative risk of secondary tantly, in those patients who will be compliant with
malignancy with surveillance is 15.2 compared to a recommended follow-up schedules. Compliance
single post-RPLND CT scan. Regardless of risk with surveillance strategies can be problematic. A
factors, approximately 15%–20% of patients community-based study revealed that up to 30% of
relapse on surveillance, indicating the presence of patients electing surveillance did not receive imag-
occult micrometastases at the time of diagnosis. ing or serum markers in the first year after diagno-
Similar to patients with NSGCT, there are several sis. Thus, patient candidates for surveillance need
pathologic variables that predict which patients to be chosen carefully.
have the greatest risk for harboring occult disease
(Table 9). 2 risk factors (rete testis invasion and
tumor size >4 cm) were examined for their prognos-
tic capabilities in a pooled multivariate analysis of 3

CHAPTER 20: TESTICULAR CANCER 643


Figure 4

Stages of high-stage NSGCT

644 EDUCATIONAL REVIEW MANUAL IN UROLOGY


Primary Chemotherapy However, by 3 months posttherapy, there was no
Single-agent carboplatin has been investigated as difference between the groups. The median follow-
adjuvant therapy in patients with clinical stage I up was 61 months, and 10 and 11 relapses occurred
seminoma. Prospective nonrandomized trials show in the 30 Gy and 20 Gy groups, respectively. In
that 2 courses of carboplatin in high-risk patients are summary, no significant difference was found in
effective in preventing relapse while systemic toxic- terms of recurrence but patients in the 20 Gy group
ity appears acceptable. In a study of 226 patients experienced less morbidity with an earlier return to
followed prospectively after 2 cycles of carboplatin, work.
only 2 had recurrence in the retroperitoneum (0.8%)
and 5 in the contralateral testis (2.2%) with a mean The effectiveness of radiation therapy for clinical
follow-up of 64 months. Furthermore, recently stage I seminoma is well documented. Relapse rates
updated results from the prospective, randomized range between 0%–7% following adjuvant radia-
Medical Research Council trial of radiation com- tion, and overall 5–10-year survival rates are
pared to single-dose carboplatin shows similar 96%–100%. Relapse usually occurs within 3 years
relapse rates for each treatment option. At 5 years, of treatment, is symptomatic and is typically outside
94.7% of patients in the chemotherapy arm were of the field of radiation. Contraindications to radia-
recurrence-free, compared to 96% in the radiation tion therapy include prior abdominal radiation, a
arm (no significant difference). These data reveal history of inflammatory bowel disease and the pres-
that single-agent carboplatin appears equal in effi- ence of a horseshoe or pelvic kidney. Under these
cacy to radiation therapy for clinical stage I semi- circumstances, patients can be treated with adjuvant
noma. chemotherapy or enter a surveillance program.

Radiation
Radiation has been the treatment of choice for most
Stage IIA and IIB Seminoma

patients with clinical stage I seminoma following Patients with stage IIA and IIB seminoma are tradi-
radical orchiectomy. Traditionally, 35 Gy was tionally treated with adjuvant radiation. The dose of
administered to the paraaortics and the ipsilateral radiation is 35–40 Gy and the radiation treatment
iliacs in a hockey-stick fashion. Prophylactic medi- fields include the ipsilateral external iliac, the bilat-
astinal and left supraclavicular radiation was eral common iliacs, the paracaval and the paraaortic
offered to many patients as well. However, supradi- nodes (high enough to cover the cisterna chyli). The
aphragmatic (mediastinal and supraclavicular) radi- radiation portal extends from the T10 to T11 region
ation is no longer recommended, as it compromises superiorly, to L4 inferiorly. Radiation to the kidney
a patient’s ability to receive subsequent therapy and is avoided due to sensitivity of the renal
it potentially increases the patient’s risk of develop- parenchyma to the radiation. Thus, chemotherapy
ing delayed radiation-related complications and (Epx4) is preferred in clinical stage IIB patients that
secondary malignancies. Currently, most centers have lymphadenopathy particularly close to the
recommend 25 Gy only to the paraaortic nodes, renal hilum. In patients that have a retained sper-
which has equivalent recurrence rates and overall matic cord or scrotal contamination, the treatment
survival rates compared to radiation directed fields can be widened to incorporate the operative
towards both the paraaortics and the ipsilateral ili- site. The contralateral testicle should be shielded.
acs. Additionally, gastrointestinal and hematologic
side effects, and adverse effects on spermatogene- The 5-year survival rate for clinical stage IIA and
sis, are less with paraaortic radiation only. The only IIB patients combined approaches 90%. Many
caveat is that patients require pelvic CTs as part of patients who relapse can be salvaged with cisplatin
their follow-up protocol. In a randomized controlled based chemotherapy. Patients with clinical IIC
trial, Jones and colleagues compared radiation ther- seminoma (N3, node >5 cm) are best managed with
apy for 625 patients with stage I seminoma using 20 systemic chemotherapy as relapse rates with radia-
Gy vs 30 Gy. Patients treated with 30 Gy had signifi- tion monotherapy exceed 50%. Patients with prior
cantly more side effects and work-related limita- abdominal radiation, a history of inflammatory
tions 1 month following the completion of therapy. bowel disease, renal hilum nodes, or with a horse-

CHAPTER 20: TESTICULAR CANCER 645


shoe or pelvic kidney should be treated with sys- possibility of detecting activity related to the tumor-
temic chemotherapy as well. associated desmoplastic reaction in response to
chemotherapy. However, in the setting of a residual
mass, the results of a PET scan can influence disease
management (Figure 4). The patient with a residual
Stage IIC and Stage III Seminoma

Cisplatin-based regimens are highly effective for mass ≥3 cm and a negative PET can be spared a
patients with bulky disseminated testicular semi- technically challenging and sometimes incomplete
noma. Response rates exceed 90% and up to 90% of surgical resection. Careful follow-up is mandated to
responders remain disease-free for prolonged peri- detect disease relapse in the absence of surgical
ods of time. Many patients who progress or who resection or biopsies.
relapse following induction chemotherapy can be
salvaged with standard second-line chemotherapy
regimens. A difficult aspect of management
involves the seminoma patient with the
postchemotherapy residual mass (Figure 4). Most
would agree that observation is warranted for resid-
ual masses <3 cm in size, as viable tumor is rarely
present in the pathological specimen (0%–3%). In
addition, data indicate that 50% of residual masses
resolve or disappear during surveillance (median
12.5 months). However, in patients with a residual
mass ≥3 cm in size, identifiable tumor is found in
13%–27% of patients undergoing postchemother-
apy biopsies or an attempted RPLND. The challeng-
ing aspect of care in a patient with a residual mass
who would otherwise be recommended to undergo
surgical excision is influenced by the extreme diffi-
culty encountered during surgical dissection.
Patients with bulky seminoma exposed to
chemotherapy have an intense postchemotherapy
desmoplastic reaction that involves the retroperi-
toneal lymph nodes occupied by seminoma. An
RPLND or an excision of the mass is often incom-
plete due to the peritumoral fibrosis (which is much
denser than the fibrosis encountered in NSGT
postchemotherapy masses). In addition, patients
with bulky seminoma only rarely have teratoma in
the postchemotherapy residual mass (likely repre-
sentative of unrecognized NSGCT components in
the primary testicular tumor). In order to avoid
unnecessary surgery and higher rates of surgical
morbidity in this patient population, PET/CT has
been studied to determine its predictive capabilities
in defining the presence or absence of viable tumor.
Recent studies examining the utility of PET for
advanced seminoma demonstrate negative predic-
tive values of 100% in patients with residual masses
following chemotherapy. The positive predictive
value of PET varies and may be influenced by the
timing of the study following chemotherapy and the

646 EDUCATIONAL REVIEW MANUAL IN UROLOGY


8. Surveillance of Germ Cell 9. Sex Cord/Gonadal Stromal
Tumors and Late Relapse Tumors

Posttreatment surveillance strategies are designed Sex cord stromal tumors compromise approxi-
to detect disease recurrence at its early stages with mately 5% of testicular neoplasms in adults, with a
the hopes of optimizing the outcomes of salvage higher prevalence in the pediatric population
therapies. Thus, the intervals at which surveillance (10%–30% of all testicular neoplasms). These
studies are conducted should coincide with known tumors originate from cells in the sex cords (Sertoli
stage-specific recurrence patterns. Most patients cell and granulosa cell tumors) or from the testis
that relapse with testicular cancer do so within the stroma (Leydig cell tumors). In general, patients
first 2 years. However, long-term follow-up is nec- with sex cord/stromal tumors present with a testicu-
essary for all patients with testicular cancer sec- lar mass or with signs and symptoms related to the
ondary to the possibility of late relapses. A late secretion of testosterone or estrogen. Most tumors
relapse is defined as a tumor recurrence beyond 2 are benign, although 10%–15% of Leydig cell
years after primary therapy and occurs in 2%–3% tumors and 10%–20% of Sertoli cell tumors are
of patients following a complete response. Patients considered malignant. Factors associated with
at all stages of disease are at risk. The median time malignant behavior include larger tumor size, high
to a late relapse is 7 years (range 2–32 years) and mitotic rate, tumor necrosis, angiolymphatic inva-
the most common site of recurrence is in the sion, positive margins and extratesticular extension.
retroperitoneum or in the chest. Patients are man-
aged best with surgical resection, as chemotherapy
alone is rarely effective. Viable GCT and teratoma
Leydig cell tumors

are found in 80% and 20% of surgical specimens, Leydig cell tumors account for 1%–3% of all testic-
respectively. Unfortunately, the long-term disease- ular tumors and they are the most common of the
free survival in patients who suffer a late relapse is non-germ cell tumors. Patient presentation occurs at
poor (reported to be approximately 25%). any age, but most commonly between 20 and 60
years. The 20% of Leydig cell tumors that occur in
prepubertal patients are considered benign. While
most patients present with a palpable mass, 30%
present with signs and symptoms such as viriliza-
tion, gynecomastia or decreased libido related to
androgen or estrogen secretion. Because some
patients present with endocrine-related symptoms
before a palpable mass is recognized, it is important
to have a high index of suspicion for a testicular
tumor (ie, gynecomastia of unknown origin) and
include an ultrasound examination of the scrotum
and testicles during the workup of endocrine-associ-
ated complaints.

Characteristically, these tumors are small, yellow-


brown and well-circumscribed. Microscopy reveal-
spolyhedral cells with round eccentric nuclei,
eosinophilic cytoplasm and occasional characteris-
tic inclusions (Reinke’s crystals). The histopatho-
logical features that predict malignant behavior
include large tumor size (>5 cm), lymphatic or vas-
cular invasion, increased mitotic activity and
involvement of the spermatic cord. The tumors can
metastasize to the retroperitoneum, lung, liver and
bone.

CHAPTER 20: TESTICULAR CANCER 647


Radical inguinal orchiectomy is the treatment of
choice. CT of the abdomen and pelvis should be
Gonadoblastoma

performed to rule out metastases if histopathologi- Gonadoblastomas occur almost exclusively in


cal features suggest malignant potential. The role of patients with intersex disorders (complete androgen
RPLND in the management of malignant Leydig insensitivity, male pseudohermaphroditism and
cell tumors or in the setting of metastatic disease is Turner syndrome) and gonadal dysgenesis
unclear, yet has been offered to some patients either (45,X/46,XY). These rare tumors account for
primarily or following chemotherapy for retroperi- <0.5% of all testis neoplasms. 80% of the patients
toneal metastases. Response to chemotherapy and are phenotypically female and 20% are phenotypi-
radiotherapy in the metastatic setting has been lim- cally male.
ited. Surveillance (including CT) is recommended
every 6 months for 2 years for patients without Radical orchiectomy is the treatment of choice and a
malignant features and more frequently for those contralateral prophylactic orchiectomy is recom-
with potential malignant tumor. mended when gonadal dysgenesis is present
because of the high incidence of contralateral dis-
ease (50%). Rarely, the gonadoblastoma develops
into a malignant germinoma (seminoma) that most
Sertoli cell tumors

Sertoli cell tumors account for <1% of testicular often is successfully managed with systemic
tumors. While patients present at any age, the pre- chemotherapy.
sentation and diagnosis usually occurs before age
20 and up to one-third of patients present with
gynecomastia. Most tumors are benign, although
10% have malignant features and can metastasize
(retroperitoneal nodes, lung and bone). Tumors usu-
ally are well-circumscribed, white or yellow, and
firm with focal cystic areas. Histologically, the
tumors have tubular formations lined by elongated
Sertoli cells with moderate to abundant cytoplasm,
pale to intense eosinophilic quality, and large cyto-
plasmic vacuoles in half of the cases. In some areas,
the tumor is solid and can be confused with semi-
noma.

Radical orchiectomy is the treatment of choice. Suc-


cessful management with RPLND has been
reported in patients with retroperitoneal involve-
ment. The value of chemotherapy and radiation
therapy for malignant disease is uncertain.

The large-cell calcifying Sertoli cell tumor subtype


that occurs most commonly in pediatric patients can
be associated with Peutz-Jeghers syndrome (gas-
trointestinal polyposis and mucocutaneous pigmen-
tation) and with Carney syndrome (pituitary adeno-
mas, mucocutaneous pigmentation and cardiac
myxomas).

648 EDUCATIONAL REVIEW MANUAL IN UROLOGY


10. Lymphoid, Hematopoietic and 11. Further Reading
Metastatic Tumors

Lymphoma Book Chapters and Websites

In the testicle, lymphoma can occur as a primary Allaf ME, Kavoussii LR. Laparoscopic Retroperi-
tumor, as the initial manifestation of clinically toneal Lymphadenectomy for Testicular Tumors In:
occult systemic disease or as the site of recurrent Wein AJ, Kavousii LR, Novick AC, Partin AW,
disease. Lymphoma is the most common testicular Peters CA, eds. Campbell’s Urology. Philadelphia,
neoplasm in patients over 60 years old. Either PA: Saunders; 2011: 893-900.
metachronous or synchronous bilateral testicular
involvement occurs in 38% of patients. Most Eble JN, Sauter G, Epstein JI, Sesterhenn IA, eds.
patients experience diffuse, painless enlargement of World Health Organization Classification of
the testicle and 30% of patients have systemic con- tumors. Pathology and Genetics of Tumors of the
stitutional symptoms, including weight loss, Urinary System and Male Genital Organs. Lyon,
anorexia and fever. Radical orchiectomy is diagnos- MA: IARC Press; 2004.
tic, although the prognosis is poor for most affected
patients with <50% 5-year disease-free survival. Sheinfeld J, Bosl GJ. Surgery of Testicular Tumors
Patients with lymphoma confined to the testis have In: Wein AJ, Kavousii LR, Novick AC, Partin AW,
a more favorable outcome. Peters CA, eds. Campbell’s Urology. Philadelphia,
PA: Saunders; 2011: 871-892

Stephenson, AJ, Gilligan, TD.. Neoplasms of the


Leukemia of the testis

The testis is a common site of recurrence of Testis. In: Wein AJ, Kavousii LR, Novick AC,
leukemia, mainly in children (acute lymphocytic Partin AW, Peters CA, eds. Campbell’s Urology.
leukemia). Primary leukemia of the testis is rare. Philadelphia, PA: Saunders; 2011: 837-870
Bilateral testicular involvement occurs in half of the
cases. Biopsy is diagnostic, and the treatment of
choice is testicular irradiation combined with
Histologic Classification

chemotherapy. Berthelsen JG, Skakkebaek NE, Sorensen BC, Mor-


gensen P. Screening for carcinoma in situ of the con-
tralateral testis in patients with germinal testicular
cancer. Br Med J. 1982;285(6536):1683-1686.
Metastatic tumors

Metastatic tumors to the testis are rare aside from


hematopoietic conditions such as lymphoma or Burke AP, Mostofi FK. Spermatocytic seminoma: a
leukemia. The most common primary tumor types clinicopathologic study of 79 cases. J Urol Pathol.
reported to metastasize to the testis include: 1993;1:21-32.
prostate, lung, gastrointestinal tract, melanoma and
kidney. Most cases are found incidentally at the time Donadio AC, Motzer RJ, Bajorin DF, et al.
of autopsy. Chemotherapy for teratoma with malignant trans-
formation. J Clin Oncol. 2003;21(23):4285-4291.

Elzinga-Tinke JE, Sirre ME, Looijenga LH, van


Casteren N, Wildhagen MF, Dohle GR. The predic-
tive value of testicular ultrasound abnormalities for
carcinoma in situ of the testis in men at risk for tes-
ticular cancer. Int J Androl. 2010;33(4):597-603.

Floyd C, Ayala AG, Logothetis CJ, Silva EG. Sper-


matocytic seminoma with associated sarcoma of the
testis. Cancer. 1988;61(2):409-414.

CHAPTER 20: TESTICULAR CANCER 649


Giwercman A, Thomsen JK, Hertz J, et al. Preva-
lence of carcinoma in situ of the testis in 207
Epidemiology, Risk Factors,

Oligospermic men from infertile couples: prospec-


Presentation and Diagnosis

tive study of testicular biopsies. BMJ. Angulo JC, González J, Rodríguez N, et al. Clinico-
1997;315(7114):989-991. pathological study of regressed testicular tumors
(apparent extragonadal germ cell neoplasms). J
Hoei-Hansen CE, Rajpert-DeMeyts E, Daugaard G, Urol. 2009;182(5):2303-2310.
Skakkebaek NE. Carcinoma in situ testis, the pro-
genitor of testicular germ cell tumours: a clinical Che M, Tamboli P, Ro JY, et al. Bilateral testicular
review. Ann Oncol. 2005;16(6):863-868. germ cell tumors: twenty-year experience at M. D.
Anderson Cancer Center. Cancer. 2002;95(6):
Holm M, Hoei-Hansen CE, Raipert-DeMeyts E, 1228-1233.
Skakkebaek NE. Increased risk of carcinoma in situ
in patients with testicular germ cell cancer with Daling JR, Doody DR, Sun X, et al. Association of
ultrasonic microlithiasis in the contralateral testicle. marijuana use and the incidence of testicular germ
J Urol. 2003;170:1163-1167. cell tumors. Cancer. 2009;115(6):1215-1223.

Mostofi FK. Proceedings: Testicular tumors. Epi- Hentrich M, Weber N, Bergsdorf T, Leidl B, Harten-
demiologic, etiologic, and pathologic features. Can- stein R, Gerl A. Management and outcome of bilat-
cer. 1973;32(5):1186-1201. eral testicular germ cell tumors: Twenty-five year
experience in Munich. Acta Oncol. 2005;44(6):529-
Motzer RJ, Amsterdam A, Prieto V, et al. Teratoma 536.
with malignant degeneration: diverse malignant his-
tologiesarising in men with germ cell tumors. J Holzbeierlein JM, Sogani PC, Sheinfeld J. Histol-
Urol. 1998;159(1):133-138. ogy and clinical outcomes in patients with bilateral
testicular germ cell tumors: the Memorial Sloan
Peterson AC, Bauman JM, Light DE, McMann LP, Kettering Cancer Center experience 1950 to 2001. J
Costabile RA. The prevalence of testicular Urol. 2003;169(6):2122-2125.
microlithiasis in an asymptomatic population of
men 18 to 35 years old. J Urol. 2001;166(6):2061- Huyghe E, Muller A, Mieusset R, Bujan L, et al.
2064. Impact of diagnostic delay in testis cancer: results of
a large population-based study. Eur Urol.
Sesterhenn IA, Davis CJ. Pathology of germ cell 2007;52(6):1710-1716.
tumors of the testis. Cancer Control.
2004;11(6):374-387. Pettersson A, Richiardi L, Nordenskjold A, Kaijser
M, Akre O. Age at surgery for undescended testis
Skakkebaek NE, Berthelsen JG, Muller J. Carci- and risk of testicular cancer. N Engl J Med.
noma-in-situ of the undescended testis. Urol Clin 2007;356(18):1835-1841.
North Am. 1982;9(3):377-385.
Siegel R, Naishadham D, Jemal A. Cancer statistics,
Tickoo SK, Hutchinson B, Bacik J, et al. Testicular 2012. CA Cancer J Clin. 2012;62(1):10-29.
seminoma: a clinicopathologic and immunohisto-
chemical study of 105 cases with special reference Tuazon E, Banks K, Koh CJ, et al. Prepubertal
to seminomas with atypical features. Int J Surg orchiopexy for cryptorchidism may be associated
Pathol. 2002;10(1):23-32. with lower risk of testicular cancer. J Urol.
2008;180(2):783-785

650 EDUCATIONAL REVIEW MANUAL IN UROLOGY


Walsh TJ, Dall’Era MA, Croughan MS, Carroll PR, Muller T, Gozzi C, Akkad T, Pallwein L, Bartsch G,
Turek PJ. Prepubertal orchiopexy for cryp- Steiner H. Management of incidental impalpable
torchidism maybe associated with lower risk of tes- Intratesticular masses of ≤5mmin diameter. BJU Int.
ticular cancer. J Urol. 2007;178:1440-1446. 2006;98(5):1001-1004.

Powell TM, Tarter TH. Management of nonpalpable


incidental testicular masses. J Urol. 2006;176
Clinical Staging

Becherer A, De Santis M, Karanikas G, et al. FDG (1):96-98.


PET is superior to CT in the prediction of viable
tumour in postchemotherapy seminoma residuals. Puc HS, Heelan R, Mazumdar M, et al. Manage-
Eur J Radiol. 2005;54(2):284-288. ment of residual mass in advanced seminoma:
results and recommendations from the Memorial
Bruns F, RaubM, Schaefer U, Micke O. No predic- Sloan-Kettering Cancer Center. J Clin Oncol.
tive value of beta-hCG in patients with stage I semi- 1996;14(2):454-460.
noma—results of a long-term follow-up study after
adjuvant radiotherapy. Anticancer Res. 2005;25 Richie JP, Steele G. Neoplasms of the testis. In:
(3A):1543-1546. Walsh PC, Retik AB, Vaughan ED, Wein AJ, eds.
Campbell’s Urology. Philadelphia, PA: Saunders;
Carmignani L, Gadda F, Gazzano G, et al. High 2002:2876-2919.
incidence of benign testicular neoplasms diagnosed
by ultrasound. J Urol. 2003;170(5):1783-1786. Schwartz BF, Auman R, Peretsman SJ, et al. Prog-
nostic value of BHCG and local tumor invasion in
Connolly SS, D’Arcy FT, Gough N, McCarthy P, stage I seminoma of the testis. J Surg Oncol.
Bredin HC, Corcoran MO. Carefully selected intrat- 1996;61(2):131-133.
esticular lesions can be safely managed with serial
ultrasonography. BJU Int. 2006;98(5):1005-1007. Sheynkin YR, Sukkarieh T, LipkeM, Cohen HL,
Schulsinger DA. Management of nonpalpable tes-
De Santis M, Becherer A, Bokemeyer C, et al. 2- ticular tumors. Urology. 2004;63(6):1163-1167.
18fluoro-deoxy-D-glucose positron emission
tomography is a reliable predictor for viable tumor Sturgeon CM, Duffy MJ, Stenman UH, et al.
in postchemotherapy seminoma: an update of the National Academy of Clinical Biochemistry labora-
prospective multicentric SEMPET trial. J Clin tory medicine practice guidelines for use of tumor
Oncol. 2004;22(6):1034-1039. markers in testicular, prostate, colorectal, breast,
and ovarian cancers. Clin Chem. 2008;54(12):e11-
Flechon A, Bompas E, Biron P, Droz JP. Manage- e79.
ment of postchemotherapy residual masses in
advanced seminoma. J Urol. 2002;168(5):1975- Toren PJ, Roberts M, Lecker I, Grober ED, Jarvi K,
1979. Lo KC. Small incidentally discovered testicular
masses in infertile men – is active surveillance the
International Germ Cell Cancer Collaborative new standard of care? J Urol. 2010;183(4):1373-
Group. International Germ cell Consensus Classifi- 1377.
cation: a prognostic factor-based staging system for
metastaticgerm cellcancers. J Clin Oncol.
1997;15(2):594-603.
Management of the Primary Tumor

Abouassaly R, Fossa SD, Giwercman A, et al.


Johns PL, Lawrentschuk N, Ballok Z, et al. 18F-flu- Sequelae of treatment in long-term survivors of
orodeoxyglucose positron emission tomography in testis cancer. Eur Urol. 2011;60(3):516-526.
evaluation of germ cell tumor after chemotherapy.
Urology. 2004;64(6):1202-1207.

CHAPTER 20: TESTICULAR CANCER 651


Aki FT, Bilen CY, Tekin MI, Ozen H. Is scrotal vio- Huyghe E, Matsuda T, Daudin M, et al. Fertility
lation per se a risk factor for local relapse and after testicular cancer treatments: results of a large
metastases in stage I nonseminomatous testicular multicenter study. Cancer. 2004;100(4):732-737.
cancer? Urology. 2000;56(3):459-462.
Leibovitch I, Baniel J, Foster RS, Donohue JP. The
Berthelsen JG, Skakkebaek NE, Sorensen BC, Mor- clinical implications of procedural deviations dur-
gensen P. Screening for carcinoma in situ of the con- ing orchiectomy for nonseminomatous testis cancer.
tralateral testis in patients with germinal testicular J Urol. 1995;154(3):935-939.
cancer. Br Med J. 1982;285(6536):1683-1686.
Leibovitch I, Little JS Jr, Foster RS, Rowland RG,
Bohlen D, Burkhard FC, Mills R, Sonntag RW, Bihrle R, Donohue JP. Delayed orchiectomy after
Studer UE. Fertility and sexual function following chemotherapy for metastatic nonseminomatous
orchiectomy and 2 cycles of chemotherapy for stage germ cell tumors. J Urol. 1996;155(3):952-954.
I high risk nonseminomatous germ cell cancer.
J Urol. 2001;165(2):441-444. Muller T, Gozzi C, Akkad T, Pallwein L, Bartsch G,
Steiner H. Management of incidental impalpable
Capelouto CC, Clark PE, Ransil BJ, Loughlin KR. intratesticular masses of ≤5 mm in diameter. BJU
A review of scrotal violation in testicular cancer: is Int. 2006;98(5):1001-1004.
adjuvant local therapy necessary? J Urol.
1995;153:981-985. Nalesnik JG, Sabanegh ES Jr, Eng TY, Buchholz
TA. Fertility in men after treatment for stage 1 and
Carmignani L, Gadda F, Gazzano G, Nerva F, et al. 2A seminoma. Am J Clin Oncol. 2004;27(6):584-
High incidence of benign testicular neoplasms diag- 588
nosed by ultrasound. J Urol. 2003;170(5):1783-
1786. Ondrus D, Hornak M, Breza J, et al. Delayed
orchiectomy after chemotherapy in patients with
Christensen TB, Daugaard G, Geertsen PF, von der advanced testicular cancer. Int Urol Nephrol.
Masse H. Effect of chemotherapy on carcinoma in 2001;32(4):665-667.
situ of the testis. Ann Oncol. 1998;9(6):657-660.
Pectasides D, Pectasides M, Farmakis D, et al. Tes-
Connolly SS, D’Arcy FT, Gough N, McCarthy P, ticular function in patients with testicular cancer
Bredin HC, Corcoran MO. Carefully selected intrat- treated with bleomycin-etoposide-carboplatin
esticular lesions can be safely managed with serial (BEC(90)) combination chemotherapy. Eur Urol.
ultrasonography. BJU Int. 2006;98(5):1005-1007. 2004;45(2):187-193.

Heidenreich A,Weissbach L, Holt LW, Albers P, Petersen PM, Giwercman A, Daugaard G, et al.
Kliesch S, Kohrmann KU, Dieckmann KP; German Effect of graded testicular doses of radiotherapy in
Testicular Cancer Study Group. Organ sparing patients treated for carcinoma-in-situ in the testis.
surgery for malignant germ cell tumor of the testis. J Clin Oncol. 2002;20(6):1537-1543.
J Urol. 2001;166(6):2161-2165.
Powell TM, Tarter TH. Management of nonpalpable
Hinz S, Schrader M, Kempkensteffen C, et al. The incidental testicular masses. J Urol. 2006;176(1):
role of positron emission tomography in the evalua- 96-98.
tion of residual masses after chemotherapy for
advanced stage seminoma. J Urol. 2008;179(3): Reiter WJ, Kratzik C, Brodowicz T, et al. Sperm
936-940. analysis and serum follicle-stimulating hormone
levels before and after adjuvant single-agent carbo-
platin therapy for clinical stage I seminoma. Urol-
ogy. Jul 1998;52(1):117-119.

652 EDUCATIONAL REVIEW MANUAL IN UROLOGY


Sheinfeld J, McKiernan J, Bosl GJ. Surgery of tes- Eggener SE, Carver BS, Sharp DS, Motzer RJ, Bosl
ticular tumors. In: Walsh PC, Retik AB, Vaughan GJ, Sheinfeld J. Incidence of disease outside modi-
ED, Wein AJ, eds. Campbell’s Urology. Philadel- fied retroperitoneal lymph node dissection tem-
phia, PA: Saunders; 2002:2920-2944. plates in clinical Stage I or IIA nonseminomatous
germ cell testicular cancer. J Urol.
Skakkebaek NE, Berthelsen JG, Muller J. Carci- 2007;177(3):937-943.
noma-in-situ of the undescended testis. Urol Clin
North Am. 1982;9(3):377-385. Evans JG, Speiss PE, Kamat AM, et al. Chylous
ascites after postchemotherapy retroperitoneal
Steiner H, Holtl L, Maneschg C, et al. Frozen sec- lymph node dissection: review of the M.D. Ander-
tion analysis-guided organ-sparing approach in tes- son experience. J Urol. 2006;176:1463-1467.
ticular tumors: technique, feasibility, and long-term
results. Urology. 2003;62(3):508-513. Janetschek G, Hobisch A, Peschel R, Hittmair A,
Bartsch G. Laparoscopic retroperitoneal lymph
node dissection for clinical stage I nonseminoma-
tous testicular carcinoma: long-term outcome. J
Retroperitoneal Lymph Node Dissection

Urol. 2000;163(6):1793-1796.
(RPLND)

Abdel-Aziz KF, Anderson JK, Svatek R, Margulis


V, Sagalowski AI, Cadeddu JA. Laparoscopic and Poulakis V, Skriapas K, de Vries R, et al. Laparo-
open retroperitoneal lymph-node dissection for scopic retroperitoneal lymph node dissection: does
clinical stage I nonseminomatous germ-cell testis it still have a role in the management of clinical
tumors. J Endourol. 2006;20(9):627-631. stage I nonseminomatous testis cancer? A European
perspective. Eur Urol. 2008;54(5):1004-1015.
Albers P, Seiner R, Krege S, et al. Randomized
phase III trial comparing retroperitoneal lymph Rassweiler JJ, Scheitlin W, Heidenreich A, Laguna
node dissection with one course of bleomycin and MP, Janetschek G. Laparoscopic retroperitoneal
etoposide plus cisplatin chemotherapy in the adju- lymph node dissection: does it still have a role in the
vant treatment of clinical stage I Nonseminomatous management of clinical stage I nonseminomatous
testicular germ cell tumors: AUO trial AH 01/94 by testis cancer? A European perspective. Eur Urol.
the German Testicular Cancer Study Group. J Clin 2008;54(5):1004-1015.
Oncol. 2008;26(18):2966-2972.
Ray B, Hajdu SI, Whitmore WF Jr. Proceedings:
Albqami N, Janetschek G. Laparoscopic retroperi- Distribution of retroperitoneal lymph node metas-
toneal lymph-node dissection in the management of tases in testicular germinal tumors. Cancer.
clinical stage I and II testicular cancer. J Endourol. 1974;33(2):340-348.
2005;19(6):683-692.
Sheinfeld J, McKiernan J, Bosl GJ. Surgery of tes-
Donohue JP, Zachary JM, Maynard BR. Distribu- ticular tumors. In: Walsh PC, Retik AB, Vaughan
tion of nodal metastases in nonseminomatous testis ED, Wein AJ, eds. Campbell’s Urology. Philadel-
cancer. J Urol. 1982;128(2):315-320. phia, PA: Saunders; 2002:2920-2944.

Donohue JP, Foster RS, Rowland RG, Bihrle R, Steiner H, Peschel R, Janetschek G, et al. Long-term
Jones J, Geier G. Nerve-sparing retroperitoneal results of laparoscopic retroperitoneal lymph node
lymphadenectomy with preservation of ejaculation. dissection: a single-center 10-year experience.
J Urol. 1990;144:287-291. Urology. 2004;63(3):550-555.

CHAPTER 20: TESTICULAR CANCER 653


Stephenson AJ, Bosl GJ, Motzer RJ, et al. Donohue JP, Leibovitch I, Foster RS, Baniel J,
Retroperitoneal lymph node dissection for nonsemi- Tognoni P. Integration of surgery and systemic ther-
nomatous germ cell testicular cancer: impact of apy: results and principles of integration. Semin
patient selection factors on outcome. J Clin Oncol. Urol Oncol. 1998;16(2):65-71.
2005;23(12):2781-2788
Ehrlich Y, Brames MJ, Beck SD, Foster RS, Ein-
Weinstein M. Lymphatic drainage of the testes. horn LH. Long-term follow-up of Cisplatin combi-
Atlas Urol Clin North Am. 1999;7:1-7. nation chemotherapy in patients with disseminated
nonseminomatous germ cell tumors: is a post-
Weissbach L, Boedefeld EA. Localization of soli- chemotherapy retroperitoneal lymph node dissec-
tary and multiple metastases in stage II nonsemino- tion needed after complete remission? J Clin Oncol.
matous testis tumor as basis for a modified staging 2010;28(4):531-536.
lymph node dissection in stage I. J Urol. 1987;
138(1):77-82. Fox EP, Weathers TD, Williams SD, et al. Outcome
analysis for patients with persistent nonteratoma-
tous germ cell tumor in postchemotherapy retroperi-
toneal lymph node dissections. J Clin Oncol.
Management of Nonseminoma

Amato RJ, Ro JY, Ayala AG, Swanson DA. Risk 1993;11:1294-1299.


adapted treatment for patients with clinical stage I
nonseminomatous germ cell tumor of the testis. Gels ME, Hoekstra HJ, Sleijfer DT, et al. Thoraco-
Urology. 2004;63(1):144-148. tomy for postchemotherapy resection of pulmonary
residual tumor mass in patients with nonseminoma-
Baniel J, Foster RS, Gonin R, Messemer JE, Dono- tous testicular germ cell tumors: aggressive surgical
hue JP, Einhorn LH. Late relapse of testicular can- resection is justified. Chest. 1997;112:967-973.
cer. J Clin Oncol. 1995;13(5):1170-1176.
Haugnes HS, Wethal T, Aass N, et al. Cardiovascu-
Brenner PC, Herr HW, Morse MJ, et al. Simultane- lar risk factors and morbidity in long-term survivors
ous retroperitoneal, thoracic, and cervical resection of testicular cancer: a 20-year follow-up study.
of post chemotherapy residual masses in patients J Clin Oncol. 2010;28(30):4649-4657.
with metastatic nonseminomatous germ cell tumors
of the testis. J Clin Oncol. 1996;14:1765-1769. Heidenreich A, Schenkmann NS, Sesterhenn IA, et
al. Immunohistochemical expression of Ki-67 to
Carver BS, Shayegan B, Serio A, Motzer RJ, Bosl predict lymph node involvement in clinical stage I
GJ, Sheinfeld J. Long-term clinical outcome after nonseminomatous germ cell tumors. J Urol.
postchemotherapy retroperitoneal lymph node dis- 1997;158(2):620-625.
section in men with residual teratoma. J Clin Oncol.
2007;25(9):1033-1037. Heidenreich A, Sesterhenn IA, Mostofi FK, Moul
JW. Prognostic risk factors that identify patients
Cullen MH, Stenning SP, Parkinson MC, et al. with clinical stage I nonseminomatous germ cell
Short-course adjuvant chemotherapy in high-risk tumors at low risk and high risk for metastasis.
stage I nonseminomatous germ cell tumors of the Cancer. 1998;83(5):1002-1011.
testis: a Medical Research Council report. J Clin
Oncol. 1996;14(4):1106-1113. Logothetis CJ, Samuels ML, Trindade A, Johnson
DE. The growing teratoma syndrome. Cancer.
Dash A, Carver BS, Stasi J, et al. The indication for 1982;50(8):1629-1635.
post-chemotherapy lymph node dissection in clini-
cal stage IS nonseminomatous germ cell tumor. Motzer RJ, Amsterdam A, Prieto V, et al. Teratoma
Cancer. 2008;112(4):800-805. with malignant degeneration: diverse malignant his-
tologiesarising in men with germ cell tumors.
J Urol. 1998;159(1):133-138.

654 EDUCATIONAL REVIEW MANUAL IN UROLOGY


Pohar KS, Rabbani F, Bosl GJ, Motzer RJ, Bajorin Dossmann M, Zagars G. Postorchiectomy radio-
D, Sheinfeld J. Results of retroperitoneal lymph therapy for stages I and II testicular seminoma. Int J
node dissection for clinical stage I and II pure Radiat Oncol Biol Phys. 1993;26:382-390.
embryonal carcinoma of the testis. J Urol.
2003;170(4 Pt 1):1155-1158. Flechon A, Bompas E, Biron P, Droz JP. Manage-
ment of postchemotherapy residual masses in
SextonWJ, Wood CG, Kim R, Pisters LL. Repeat advanced seminoma. J Urol. 2002;168(5):1975-
retroperitoneal lymph node dissection for metastatic 1979.
testis cancer. J Urol. 2003;169(4):1353-1356.
Fossa SD, Horwich A, Russell JM, et al. Optimal
Sogani PC, Perrotti M, Herr HW, Fair WR, Thaler planning target volume for stage I testicular semi-
HT, Bosl G. Clinical stage I testis cancer: long-term noma: A Medical Research Council randomized
outcome of patients on surveillance. J Urol. trial. Medical Research Council Testicular Tumor
1998;159(3):855-858. Working Group. J Clin Oncol. 1999;17(4):1146.

Stephenson AJ, Bosl GJ, Motzer RJ, et al. Jones WG, Fossa SD, Mead GM, et al. Randomized
Retroperitoneal lymph node dissection for nonsemi- trial of 30 versus 20 Gy in the adjuvant treatment of
nomatous germ cell testicular cancer: impact of stage I Testicular Seminoma: a report on Medical
patient selection factors on outcome. J Clin Oncol. Research Council Trial TE18, European Organiza-
2005;23(12):2781-2788. tion for the Research and Treatment of Cancer Trial
30942. J Clin Oncol. 2005;23:1200-1208.
Tandstad T, Dahl O, Cohn-Cedermark G, et al. Risk-
adapted treatment in clinical stage I nonseminoma- International Germ Cell Cancer Collaborative
tous germ cell testicular cancer: the SWENOTECA Group. International Germ cell Consensus Classifi-
management program. J Clin Oncol. 2009;27(13): cation: a prognostic factor-based staging system for
2122-2128. metastatic germ cell cancers. J Clin Oncol.
1997;15(2):594-603.
Vergouwe Y, Steyerberg EW, Eijkemans MJ, Albers
P, Habbema JD. Predictors of occult metastasis in Oliver RT, Mead GM, Rustin GJ, et al. Randomized
clinical stage I nonseminoma: a systematic review. trial of carboplatin versus radiotherapy for Stage I
J Clin Oncol. 2003;21(22):4092-4099. seminoma: mature results on relapse and contralat-
eral testis cancer rates in MRC TE19/EORTC
Williams SB, Steele GS, Richie JP. Primary 30982 Study. J Clin Oncol. 2011;29(8):957-962.
retroperitoneal lymph node dissection in patients
with clinical stage IS testis cancer. J Urol. 2009; Puc HS, Heelan R, Mazumdar M, Herr H, et al.
182(6):2716-2720. Management of residual mass in advanced semi-
noma: results and recommendations from the
Williams SD, Stablein DM, Einhorn LH, et al. Memorial Sloan-Kettering Cancer Center. J Clin
Immediate adjuvant chemotherapy versus observa- Oncol. 1996;14(2):454-460.
tion with treatment at relapse in pathological stage
II testicular cancer. N Engl J Med. Travis LB, Fossa SD, Schonfeld SJ, et al. Second
1987;317(23):1433-1438. cancers among 40,576 testicular cancer patients:
focus on long-term survivors. J Natl Cancer Inst.
Management of Seminoma 2005;97(18):1354-1365.

Aparicio J, Germa JR, Garcia delMuro X, Maroto P, von der Masse H, Specht L, Jacobsen A, et al.
Arranz JA, Saenz A. Risk-adapted management for Surveillance following orchidectomy for stage I
patients with clinical stage I seminoma: the Second seminoma of the testis. Eur J Cancer. 1993;
Spanish Germ Cell Cancer Cooperative Group 29A(14):1931-1934.
study. J Clin Oncol. 2005;23(34):8717-8723.

CHAPTER 20: TESTICULAR CANCER 655


12. Questions

Warde P, GospodarowiczMK, Panzarella T, et al.


Stage I testicular seminoma: results of adjuvant
Testicular Cancer

radiation and surveillance. J Clin Oncol. 1995; 1. Factors associated with malignant sex
13:2255-2262. cord/gonadal stromal tumors include all of the
following except:
Warde P, Jewett MA. Surveillance for stage I testic-
ular seminoma. Is it a good option? Urol Clin North A. larger tumor size
Am. 1998;25(3):425-433.
B. high mitotic rate
Warde P, Specht L, Horwich A, et al. Prognostic fac-
tors for relapse in stage I seminoma managed by C. tumor necrosis,
surveillance: a pooled analysis. J Clin Oncol.
2002;20(22):4448-4452. D. rete testis invasion

Yu HY, Madison RA, Setodji CM, Saigal CS. Qual- E. extratesticular extension
ity of surveillance for stage I testis cancer in the
community. J Clin Oncol. 2009;27(26):4327-4332. 2. The most common primary testicular neoplasm
in men over the age of 60 is:

A. Classic seminoma
Sex Cord / Gonadal Stromal Tumors

Conkey DS, Howard GC, Grigor KM, et al. Testicu-


lar sex cord-stromal tumours: the Edinburgh experi- B. Lymphoma
ence 1988-2002, and review of the literature. Clin
Oncol. 2005;17:322-327. C. Spermatocytic seminoma

Dilworth J, Farrow G, Oesterlin J. Non-germ cell D. Mixed nonseminoma


tumors of the testis. Urology. 1991;37:399-412.
Featherstone JM, Fernando HS, Theaker JM, Sim- E. Sertoli cell tumor
monds PD, Hayes MC, Mead GM. Sex cord stromal
testicular tumors: a clinical series--uniformly stage I 3. All of the following statements are true regard-
disease. J Urol. 2009;181(5):2090-2096. ing the patterns and mechanisms of metastatic
spread of primary testis cancers except:
Grem JL, Robins HI, Wilson KS, et al. Metastatic
Leydig cell tumor of the testis: report of three cases A. “Skip” metastases occur away from the
and review of the literature. Cancer. 1986;58:2116- retroperitoneal lymph nodes in 25% of
2119. patients.

Ulbright TM, Amin MB, Young RH. Tumors of the B. The primary landing zone for left-sided
testis, adnexa, spermatic cord and scrotum. Atlas of testis tumors is in the left paraaortic location
Tumor Pathology, farc 25, ser 3.Washington, DC:
Armed Forces Institute of Pathology; 1999: 1-290. C. Lymphatic drainage crosses over from right
to left. Therefore, left paraaortic lymph node
involvement occurs commonly in patients
with right testicular primaries

D. Lymphatic drainage above the retroperi-


toneum is to the cisterna chyli, thoracic duct,
and usually to the left supraclavicular lymph
nodes

656 EDUCATIONAL REVIEW MANUAL IN UROLOGY


4. The proper management for a patient with a D. Resection of residual masses in the
0.8-cm nonseminoma with negative margins, retroperitoneum and in the mediastinum
no evidence for carcinoma in situ and a normal should never be performed in the same oper-
contralateral testicle following partial orchiec- ative setting due to cardiopulmonary toxic-
tomy is: ity associated with systemic chemotherapy

A. Adjuvant chemotherapy E. Malignant transformation of teratoma is


identified in approximately 3% of resected
B. Adjuvant radiation residual lymph node specimens

C. Completion orchiectomy 7. Which of the following statements regarding


patients with clinical stage I nonseminoma are
D. Contralateral testicular biopsies true?

E. Observation A. Patients with clinical stage IS disease are


distinguished by lymphovascular invasion
5. The best recommendation for a compliant in their primary tumor specimen
patient with a 2-cm classic seminoma with no
evidence for lymphovascular invasion or rete B. Most patients with clinical stage IS non-
testis invasion following orchiectomy is: seminoma are treated with single-agent cis-
platin x 1 course due to an elevated risk of
A. Adjuvant radiation to the paraaortics recurrence following primary RPLND

B. Adjuvant radiation to the paraaortics and to C. Primary RPLND alone for clinical stage IB
the ipsilateral pelvic lymph nodes nonseminoma cures 50%–90% of patients
with pathologic stage II (node-positive) dis-
C. Single agent carboplatin for 1 cycle ease

D. Immediate adjuvant BEP x 1 cycle D. BEP chemotherapy x 3 courses or EP


chemotherapy x 4 courses is an accepted
E. Observation treatment option for patients with clinical
stage IB nonseminoma and normal tumor
6. Which of the following statements are false markers
regarding postchemotherapy residual masses in
patients with clinical stage III metastatic non- 8. Which of the following statements are true
seminoma? regarding bHCG as a testicular cancer tumor
marker:
A. A retroperitoneal residual lymph node mass
>1 cm in size is an indication for a A. False-positive bHCG elevation might be due
postchemotherapy RPLND to other cancers (ie, bladder cancer) as well
as marijuana use
B. Historically, the likelihood of identifying
viable nonteratomatous germ cell elements B. bHCG levels should fall by 50% per week if
in the resected lymph node specimen follow- all of the tumor has been removed with the
ing induction chemotherapy is approxi- radical orchiectomy specimen
mately 10%
C. The beta subunit of HCG is 70% homolo-
C. Histologic discordance is not uncommon gous with pituitary LH and, due to cross
between the resected lymph node specimens reactivity, might cause false elevation of
and other visceral sites of metastatic disease HCG in some patients with hypogonadism
(ie, liver)
CHAPTER 20: TESTICULAR CANCER 657
D. Both A and C 3. A.
Distant progression occurs in the absence of
E. All of the above retroperitoneal lymph node involvement. Lym-
phatic channels may bypass the retroperitoneum
9. Following a primary RPLND for nonsemi- and communicate directly with the cisterna chili or
noma, the risk of an in-field relapse in the the thoracic duct. This pattern of lymphatic drainage
retroperitoneum is reported to be: and the possibility of direct hematogenous spread
accounts for the small percentage of patients who
A. 1%–2% relapse (most commonly in the lungs) following a
negative RPLND for clinical stage I disease.
B. 10%–15%
4. E.
C. 20%–25% Local recurrence is possible following partial
orchiectomy – particularly in patients with CIS
D. >30% detected in parenchymal biopsies adjacent to tumor.
Recurrence is diminished in such patients with adju-
10. A 35-year-old patient has a pure seminoma in vant testicular radiation. In this case, observation is
the left radical orchiectomy specimen, a 5.5-cm the best option as biopsies were negative.
left paraaortic lymph node mass and normal
tumor markers. His chest CT is normal. The 5. E.
best management strategy for this patient Choices A, B and C are options for the described
includes: patient. However, the significant majority of
patients are cured with orchiectomy alone. There
A. Concurrent radiosensitizing cisplatin and 40 are increasing data addressing the risks of long-term
Gy radiation to the paraaortics, the left supr- side effects with both chemotherapy and radiation.
aclavicular lymph nodes and the ipsilateral Most academic centers now favor observation for
pelvic lymph nodes this low-risk patient.

B. Single-agent carboplatin x 2 cycles 6. D.


Simultaneous excision of all sites of residual tumor
C. Induction BEP x 3 or EP x 4 is an accepted option if all of the disease can be
resected through a single incision. While
D. Primary RPLND as his markers are normal chemotherapy is associated with a higher hazard of
cardiovascular disease, prior chemotherapy is not a
Answers: contraindication for simultaneous resection.

1. D. 7. C.
Invasion of the rete testis has been reported as a risk Patients with clinical stage IS nonseminoma have
factor for micrometastatic disease in patients with persistent elevation of their tumor markers follow-
clinical stage I testicular seminoma. ing radical orchiectomy and no evidence of radio-
graphically detected metastases. For stage IS
2. B. patients, primary RPLND is associated with an ele-
While it is possible to see all of the listed primary vated risk of recurrence following surgery,
tumors, lymphoma would be the most common his- approaching 80% in some series. Thus, for most
tology in this age group of men. patients, standard induction chemotherapy is rec-
ommended consisting of 3 cycles BEP or 4 cycles
EP. While primary chemotherapy is an option for
patients with clinical stage IB nonseminoma, BEP x
1–2 cycles is advocated rather than a standard
induction regimen.

658 EDUCATIONAL REVIEW MANUAL IN UROLOGY


8. D.
The half-life of bHCG is 24–48 hours. If the entire
tumor has been removed, elevated marker levels
should normalize by 5–7 days.

9. A.
The risk of an in-field relapse is very low in reports
of primary RPLNDs from centers of excellence.
5%–10% of patients that undergo a negative pri-
mary RPLND will relapse out of the field—most
commonly in the lungs.

10. C.
Radiation to the supraclavicular lymph nodes is no
longer advocated. There is no role for radiosensitiz-
ing chemotherapy and single-agent carboplatin is
not utilized in patients with clinical stage II semi-
noma. The patient has pure seminoma. Primary
RPLND is not an option in this setting.

CHAPTER 20: TESTICULAR CANCER 659


660 EDUCATIONAL REVIEW MANUAL IN UROLOGY
Chapter 21:
Benign Prostatic
Hyperplasia
and Bladder Calculi
James C. Ulchaker, MD, FACS
Alexis E. Te, MD

Contents

1. Epidemiology, Pathophysiology and Natural


History of BPH

2. Evaluation of BPH and LUTS

3. Medical Therapy

4. Interventional Therapies

5. LUTS, Sexual Function and Dysfunction


with Medical and Surgical Treatment

6. Metabolic Syndrome, Lower Urinary Tract


Symptoms and BPH

7. Etiology, Pathogenesis and Presentation


of Bladder Calculi

8. Recommended Reading and References

CHAPTER 21: BENIGN PROSTATIC HYPERPLASIA AND BLADDER CALCULI 661


1. Epidemiology, Pathophysiology
and Natural History of BPH

Benign prostatic hyperplasia (BPH) is one of the 2 risk factors (16%) and 3 risk factors (37%). Men
most common diseases of elderly men. It often is over age 70 years with 3 risk factors have 11 times
associated with lower urinary tract symptoms higher risk than men aged 40 years.5 Complications
(LUTS) that interfere with normal daily activities of BPH progression include worsening of symp-
and sleep patterns. The prevalence of BPH is depen- toms, such as increase in bother and decrease in
dent upon age, with initial development usually quality of life. Clinical progression can also result in
after age 40. By age 60, the prevalence of BPH is retention, hematuria, urinary tract infection (UTI),
>50%, and by age 85 it is as high as 90%. The bladder stones and renal failure, which may require
prevalence of bothersome symptoms also increases surgical intervention.8 Treatment of BPH is based
with age. About half of all men who have a histo- on relieving bladder outlet resistance to increase
logic diagnosis have moderate to severe LUTS.1 urinary flow rate.

Long-term data from population-based studies have The pathophysiology of BPH and resulting lower
recently become available, yet the risks of compli- urinary tract symptoms of BPH are intertwined. The
cations and morbidities from untreated BPH remain basic pathophysiology of benign prostatic hyperpla-
uncertain. It is unclear whether a patient with a spe- sia at the cellular level that leads to overall gland
cific symptom complex will develop complete uri- enlargement has not been well elucidated but is
nary retention over a certain period of time. Many clearly dependent on hormonal factors and other
patients are bothered by LUTS with varying degrees related growth factors. The growth and maintenance
of bother, even with the same frequency and sever- of the prostate gland is dependent on dihydrotestos-
ity of symptoms. Generally, patient perception of terone and is affected by changes in hormonal envi-
BPH severity, as well as the degree of interference ronment from aging. Such changes occur in testos-
with the patient’s lifestyle, should be the primary terone, estrogen and other growth factors which
consideration in choosing appropriate therapy. influence apoptosis and prostate growth. These
changes may also affect prostate tissue composition
BPH is defined by histologic evidence of stromog- of stromoglandular epithelial hyperplasia as well as
landular hyperplasia. BPH is not mutually inclusive alpha-adrenergic tone and innervation. The overall
with LUTS, which is defined by the presence of irri- impact of these factors results in overall gland
tative and/or obstructive symptoms, or with bladder enlargement. However, the prostate gland can be
outlet obstruction (BOO), which is defined by the anatomically divided into several zones: the periph-
presence of urodynamic obstruction. The natural eral zone, the central zone and the transition zone.
history of BPH shows that symptoms worsen in
55% of patients, remain stable in 30% of patients The changes in prostate size and muscle tone,
and improve in 15% of patients.2 Prostate size mainly in the central and transition zone which sur-
increases with age at a rate of 0.6 mL/year.3 Men round the urethra, affect urinary flow through the
with prostate volume >40 cc are 3 times as likely to prostatic urethra, narrowing the flow and creating
have elevated symptoms, twice as likely to be both- bladder outflow obstruction. This functional urody-
ered by symptoms and twice as likely to experience namic obstruction produces compensative changes
interference with normal daily activities.4 Men with in the bladder that increase bladder pressure to over-
prostate size >30 cc have a 3-fold risk for acute uri- come the outlet resistance. This results in a progres-
nary retention (AUR).5 The estimated prostate sive detrusor hypertrophy with resulting decrease in
growth rates increase by 1.6% per year across all compliance and capacity over time. Detrusor over-
ages.6 Higher baseline prostate volume is associ- activity can also occur as a result of this process.
ated with higher rates of prostate growth. Overall, these changes produce symptoms of
frequency, urgency, nocturia, slow urinary stream
The risk factors for prostatectomy are a change in and incomplete emptying that can potentially
size and force of stream, a sensation of incomplete progress to urinary retention.9
voiding and enlarged prostate on digital rectal exam
(DRE).7 The risk of AUR/prostate surgery increases
with the number of risk factors—1 risk factor (9%),

662 EDUCATIONAL REVIEW MANUAL IN UROLOGY


2. Evaluation of BPH and LUTS

Current medical therapies for BPH include alpha-


adrenergic blockers, which reduce sympathetic
Brief Summary of the AUA BPH Guidelines

tone, and 5-alpha reductase (5AR) inhibitors, which In 2010, the American Urological Association
reduce volume, a phosphodiesterase inhibitor (AUA) updated the BPH recommendations of the
whose mechanism of action remains under investi- clinical practice guidelines. The new guidelines
gation, and antimuscarinic medications, which make minimal modifications to the recommended
improve LUTS by detrussor relaxation. Ther- diagnostic methods for detecting and assessing the
motherapies, which include needle ablation and severity of BPH, based on expert clinical judgment.
microwave therapy, reduce volume and open chan- The guidelines update existing statements on patient
nels. Surgery, such as transurethral resection of the management. Extensive literature searches were
prostate (TURP), laser therapies and open prostate- conducted, and critical reviews and syntheses were
ctomy, remove volume and open channels. All of used to evaluate empirical evidence and significant
these treatments seek to improve urinary flow and outcomes of all currently available treatment
symptoms by reducing bladder outlet obstruction. approaches.

The new AUA guidelines provide an algorithm


(Figure 1) as a framework for diagnosis and treat-
ment of BPH. Individual patients may present dif-
ferently, so clinicians should exercise clinical judg-
ment and act in the patient’s best interest.10

Noninvasive Evaluation

The treatment of bladder outflow obstruction can be


improved by using urodynamic parameters. How-
ever, all rationally designed studies for BPH should
incorporate a relevant medical history focusing on
the nature and duration of symptoms, previous sur-
gical procedures, sexual function and current medi-
cations. A focused physical examination should be
performed of the supra pubic area, including a DRE.
An AUA symptom score and bother score, urinaly-
sis, free urinary flow rate, post-void residual (PVR),
PSA, when appropriate, and voiding diary should
also be performed. Free uroflowmetry is an objec-
tive efficacy parameter consistently utilized in eval-
uation of BPH therapies.

It may predict response to therapy, especially


surgery, using a maximum flow rate (Qmax) of
<10ml/sec, which is consistent with obstruction, vs
a Qmax >15 mL/sec and LUTS, which is consistent
with a non-BPH diagnosis. There are test-retest
variability issues, and there is no established flow
rate cut-off value, and no predictive value for medi-
cal therapeutic response.

PVR is an excellent indication of bladder emptying.


It may prompt additional testing, including renal
ultrasound and creatinine levels. There is consider-

CHAPTER 21: BENIGN PROSTATIC HYPERPLASIA AND BLADDER CALCULI 663


Figure 1

American Urological Association BPH treatment guidelines

Basic management of lower urinary tract symptoms (LUTS) in men (adapted with permission from Abrams 2009).
AUA SI, American Urological Association Symptom Index; DRE, digital rectal exam; PSA, prostate specific antigen.

664 EDUCATIONAL REVIEW MANUAL IN UROLOGY


able within-patient variability, which limits the pre- Endoscopic evaluation of the lower urinary tract is
dictive value of PVR. There is no proven correlation not recommended in an otherwise healthy male with
between PVR and the development of UTIs in an initial evaluation consistent with BOO, unless
patients with obstruction. the treatment alternatives being contemplated
depend on the anatomical configuration of the
No definitive volume guidelines exist. Clinicians prostate, or hematuria is present.
may monitor patients with watchful waiting or use
medical therapy for patients with high PVR. There Are flow rates, PVRs, TRUS, prostate biopsy and
is an increased risk of overall progression, symptom urodynamics necessary? Some points clinicians
progression and AUR in higher PVR groups.11 need to consider when making a decision about
BPH evaluations include: reproducibility; correla-
tion with symptoms severity and improvement;
studies to refine technique and usefulness; the need
Role of Pressure Flow Studies

Urodynamics have a utility in assessing bladder for surgical intervention or diagnosis; and monitor-
function and confirming bladder outlet obstruction ing for efficacy and morbidity risk. Are these evalu-
in men with LUTS. BPH associated with BOO may ations necessary? They definitely are useful and
cause LUTS. Patients with BPH have a greater risk integral to BPH management.12
of developing urinary retention. More than 10% of
men in their 70s will experience AUR. While it
makes sense that those with severe BOO are at high-
est risk for retention, clinical utility of pressure flow
studies in the management of LUTS and BPH is
controversial and currently not recommended for
routine use.

However, pressure flow studies are the best modality


available to diagnose obstruction. They provide an
understanding between BPH, LUTS and BOO, are
useful in monitoring therapy and assessing morbidity,
and are good for counseling patients about postopera-
tive expectations. Basic pressure flow studies are per-
formed by measuring simultaneously bladder pres-
sure with a catheter placed transurethrally, abdominal
pressure with a catheter placed transrectally, and a
urinary flow meter. Generally, bladder outlet obstruc-
tion is defined by simultaneous high bladder pressure
and low urinary flow rate. Several nomograms are
utilized to grade obstruction, such as the Abrams-
Griffiths nomogram. More invasive procedures, such
as transrectal ultrasound (TRUS) and prostate biopsy,
can confirm prostate size, confirm prostate anatomy
for invasive therapy, assess the presence of intravesi-
cal component, assess the presence and size of the
middle lobe, assess the risk of progression and assess
the presence of prostate cancer.

CHAPTER 21: BENIGN PROSTATIC HYPERPLASIA AND BLADDER CALCULI 665


3. Medical Therapy

BPH treatment is symptom driven. The goals of relieving symptoms, but may be too late or not
treatment are to relieve LUTS, treat BOO, and to enough to relieve symptoms or impact morbidity
treat and prevent morbidities, including retention, risk leading to more invasive therapeutic options.
UTIs, obstructive uropathy and stones. The basis for
treatment is decreasing bladder outlet resis-
tance/obstruction. While symptom severity does not
The Old vs New Alpha-Adrenergic Blockers

correlate well with urodynamic obstruction severity, The original alpha-adrenergic blockers, such as pra-
removing the obstruction often results in symptom zosin, terazosin and doxazosin, were initially FDA-
relief. Urodynamic obstruction severity correlates approved for the indication of hypertension. The
well with morbidities, such as retention, obstructive original formulations required dose titration for
uropathy, hematuria, infection and stones. BPH indications due to the risk of orthostatic
hypotension, dizziness and fatigue. Advancements
Factors to consider in the treatment of BPH include: in uroselectivity via receptor selectivity (alpha 1A
the symptom severity based on AUA guidelines; and D) and sustained release formulations influenc-
risk reduction in morbidity and progression; the ing the pharmacokinetic release profile have pro-
patient’s overall health and age/life expectancy; gressed the concept of uroselectivity. At least 3 dis-
expected durability of symptom relief; treatment crete alpha 1 adrenergic subtypes have been identi-
side effects; whether the treatment is reversible fied.13,14 In general, alpha 1A receptor blockers affect
(medications) or permanent (surgery); and the the prostate, alpha 1B blockers affect vascular tone,
patient’s needs and expectations. and alpha 1D blockers affect the bladder. This con-
cept led to the development of uroselective drugs
Before initiating any therapy, one must first discuss with minimal vascular side effect profiles and effec-
behavioral modifications that may contribute to tive once-a-day formulations that do not require
voiding dysfunction, such as concomitant drugs, titration, such as tamsulosin and alfuzosin. In the
regulation of fluid intake (especially in the current AUA BPH guidelines, at the FDA-approved
evening), lifestyle changes and dietary advice dosage, efficacy is presumed to be equivalent
(especially the avoidance of excess alcohol, caffeine among all FDA-approved BPH alpha-adrenergic
and highly seasoned or irritative foods). blockers: terazosin, doxazosin, tamsulosin and alfu-
zosin. Silodosin was approved by the US Food and
Drug Administration but there were no relevant
published articles in the peer-reviewed literature
Medical Therapy Mechanism of Action

Medical therapy is designed to either relax the prior to the cut-off date for the literature search.
prostate gland with alpha blockers or to shrink the Silodosin may have an advantage over other alpha
gland with 5AR inhibitors. Alpha blockers have a blockers in that the onset of improvement in symp-
dynamic component by relaxing the prostate toms and urinary flow appears to be more rapid with
smooth muscle and bladder neck, decreasing overall excellent cardiovascular tolerability. The downside
prostatic tone, and improving urinary flow rate and is a higher incidence of retrograde ejaculation.15
therefore symptoms. 5AR inhibitors affect the static The differences between the various alpha-adrener-
component by decreasing prostate size, improving gic blockers appear to lie in their tolerability and
urinary flow rate and improving symptoms. side effect profiles (Table 1).
Although more gradual than alpha blockers, 5AR
inhibitors not only shrink the prostate but also stunt Recommendation: Men with LUTS secondary to
prostate growth, and therefore affect the natural his- BPH for whom alpha-blocker therapy is offered
tory of BPH and delay progression of disease. Both should be asked about planned cataract surgery.
types of drugs can decrease detrussor pressure by 10 Men with planned cataract surgery should avoid the
cm H2O and cause a mild decrease in BOO. Symp- initiation of alpha blockers until their cataract
tom relief is provided via increased Qmax and surgery is completed. Recommendation: In men
decreased PVR, as well as influencing receptor fac- with no planned cataract surgery, there are insuffi-
tors (e.g. innervation, such as alpha 1D and cient data to recommend withholding or discontinu-
antimuscarinic). A little bit goes a long way to

666 EDUCATIONAL REVIEW MANUAL IN UROLOGY


Table 1

Alpha-Adrenergic Blockers for BPH

Agent Receptor Plasma Extended Duration of Inhibitor


Selectivity t 1/2 Release Action Mechanism
Formulation

Phenoxybenzamine ␣1, ␣2 24 hr 72-96 hr Irreversible

Prazosin ␣1 2-3 hr 4-6 hr Reversible

Terazosin ␣1 2-4 hr Up to 18 hr Reversible

Doxazosin ␣1 22 hr yes Up to 24 hr Reversible

Tamsulosin ␣1 14-5 hr yes Up to 24 hr Reversible

Alfuzosin ␣1 3-10 hr yes Up to 24 hr Reversible

ing alpha blockers for bother some LUTS secondary and doxazosin. There are insufficient exposure data
to BPH. to estimate the risk of IFIS with alfuzosin.

The dose or duration of alpha-blocker treatment that


influences the risk of IFIS is unclear. Whether stop-
Intraoperative Floppy Iris Syndrome

Intraoperative floppy iris syndrome (IFIS) was first ping alpha-blocker treatment at any time before
described by Chang and Campbell in 2005 as a triad surgery mitigates the risk of IFIS is unclear. If expe-
of progressive intraoperative miosis despite preop- rienced ophthalmologists are aware of preoperative
erative dilation, billowing of a flaccid iris, and iris alpha blocker use, pre- and intraoperative precau-
prolapse toward the incision site during phacoemul- tions can be taken to reduce the risk of IFIS compli-
sification for cataracts.10 Operative complications in cations and attain excellent visual outcomes, though
some cases included posterior capsule rupture with it remains unclear if the residual risk and outcomes
vitreous loss and postoperative intraocular pressure are any worse than among patients without IFIS.
spikes, though visual acuity outcomes appeared pre-
served. The original report linked this condition
with the preoperative use of tamsulosin; iris dilator
Prevention and Treatment Role of 5-Alpha

smooth muscle inhibition has been suggested as a


Reductase Inhibitors

potential mechanism.10 Prostate growth and maintenance is dependent on


male androgens. Dihydrotestosterone is the main
The risk of IFIS was substantial among men taking active male hormone affecting prostate growth and
tamsulosin, ranging from about 43%–90% in 10 ret- development. Testosterone is converted to dihy-
rospective and prospective studies (sometimes the drotestosterone by 5-alpha reductase (5AR). Two
denominator for these risks was patients, and some- 5AR isoenzymes have been identified that convert
times eyes). The risk of IFIS appears to be lower testosterone to dihydrotestosterone (Table 2). The
with older, generic alpha-blockers such as terazosin first 5-alpha reductase blocker is finasteride, which
is only a type 2 5AR inhibitor. Type 2 actions are

CHAPTER 21: BENIGN PROSTATIC HYPERPLASIA AND BLADDER CALCULI 667


of diminished ejaculation and libido, and erectile
dysfunction. They are primarily indicated in symp-
Table 2

tomatic patients with prostate size >30 mL or PSA


>1.4 ng/mL.18,19
5-Alpha Reductase Inhibitors

Finasteride MTOPS: The Role of Combination Therapy

Inhibits type 2 5AR The large MTOPS clinical trial found that a combi-
nation of the 2 drugs finasteride and doxazosin was
Reduces Serum DHT levels by about 70% significantly more effective than either drug alone
for preventing progression of BPH, especially in
Reduces intraprostatic DHT levels by about 80% men at high risk for disease progression.18,19
Finasteride, a 5AR inhibitor, and doxazosin, an
Reduces serum PSA by about 50% alpha-1 receptor blocker, together reduced the
overall risk of BP progression by 66% compared to
6-8 hour half-life placebo. The combined drugs also provided the
greatest symptom relief and improvement in urinary
flow rate. Doxazosin alone reduced overall risk of
progression by 39% and finasteride alone by 34%
Dutasteride

Inhibits both type 1 and type 2 5AR compared to placebo. The combination treatment
and finasteride alone also significantly reduced the
Reduces serum DHT levels by more than 90% risk of invasive therapy by 67% and 64%, respec-
tively. (Doxazosin did not reduce the long-term risk
Reduces intraprostatic DHT levels by more than 90% of invasive therapy.) Most invasive treatments in
MTOPS were transurethral or open surgeries to
Reduces serum PSA by about 50% remove prostate tissue. Other invasive therapies
included transurethral incision, microwave or laser
5 week serum half-life therapy, and prostatic stents.

Ekman P. Drug Saf. 1998;18:161-170. Ekman P. Scand J MTOPS found that combination therapy was espe-
Urol Nephrol. 1999;(suppl 203):15-20. Bartsch G, et al. Eur cially effective in men with prostates > 40cc or
serum PSAs above 4 ng/mL.19 The study shows that
Urol. 2000;37:367-380. Bartsch G, et al. World J Urol.

combination therapy offers dramatically greater and


2002;19:413-425.

mainly in the prostate gland and account for about longer lasting relief from symptoms and, over time,
70% of dihydrotestosterone production. The other that 5 AR inhibitors shrink the prostate and actually
and more recently marketed 5AR inhibitor is dutas- prevent growth so that fewer men are at higher risk
teride. This drug is a dual inhibitor, blocking both for progression of symptoms, developing retention,
type 1 and type 2 5-alpha reductase. With this drug’s and proceeding to surgery. This study is a bench-
current formulation and long half-life, dihy- mark for identifying men at risk for BPH progres-
drotestosterone is lowered by >90%. Both drugs sion, and who may benefit most from combination
have demonstrated efficacy and durability in long- therapy. The study helps alleviate the concern that
term responders and, more importantly, have been medical treatment might initially relieve symptoms
demonstrated to decrease progression not only of but mask problems related to continuing prostate
symptoms, but also towards retention and surgery. growth (Table 3).
Both drugs have demonstrated efficacy and durabil-
ity in long-term responders and, more importantly, The large CombAT study is a multicenter, random-
have been demonstrated to decrease progression not ized, double-blind, parallel group study, that evalu-
only of symptoms, but also towards retention and- ated whether the combination of dutasteride and
surgery. Both drugs will lower the serum PSA by tamsulosin was more effective than either
approximately 50%, and may have the side effects monotherapy alone for improving symptoms and

668 EDUCATIONAL REVIEW MANUAL IN UROLOGY


long-term outcomes in men with moderate to severe
LUTS and BPO. In men with a prostate volume of
Complementary Therapies: Phytotherapies

≥30 mL, combined therapy with dutasteride plus


or Nutriceuticals?

tamsulosin provided better long-term (up to 4 years) More and more men with symptoms of BPH are
control of both storage and voiding LUTS com- treating themselves with phytotherapies or nutriceu-
pared with tamsulosin monotherapy. Combined ticals sold over the counter. Two of the most com-
therapy was better than dutasteride monotherapy in monly used phytotherapies are Serenoa repens (saw
men with prostate volumes of ≥30 to <58 mL, but palmetto) and Pygeum africanum. Little is known
not in men with a prostate volume of ≥58 mL.20 about the long-term effects of these agents, and
most American physicians are reluctant to discuss
In October 2011, tadalafil, a long-acting PDE-5 or recommend them to patients because only a mod-
inhibitor, was FDA approved for the treatment of est number of published reports have appeared in
both BPH and erectile dysfunction. This once-daily the peer-reviewed medical literature. The available
therapy resulted in improvement in total IPSS score. literature supports the hypothesis that these com-
This treatment is not recommended in combination pounds may have some beneficial effects on BPH
with alpha blockers.21 symptoms, but there are no statistically significant
reports of rigorously conducted clinical trials on the

Table 3

Summary of Large North American Studies that


Evaluated 5-Alpha Reductase Monotherapy

Finasteride VA Dutasteride
Study Group1 Cooperative2 PLESS3 MTOPS4 Multicenter5

Year published 1992 1996 1998 2002 2002

Follow-up (years) 1 1 4 5 2

Number of patients 895 1229 3040 3047 4325

Mean Prostate Volume 59 36.2 54 31.2 55

Change in AUASS 1.7 NS 2 1 2.2

Change in Qmax 1.8 NS 1.7 0.8 1.6

RR in AUR NR NR 57 66 57

RR in BPH surgery NR NR 55 62 48

NS = not significant; NR = not reported


1. Gromley GJ, et al. N Engl J Med. 1992;327:1185-1191.
2. Lepor H, et al. N Engl J Med .1996;335:533-539.
3. McConnell JD, et al. N Engl J Med. 1998;338:557-563.
4. McConnell J, et al. N Engl J Med. 2003;349:2387-98.
5. Roehrborn, et al. Urology. 2002;60:434-441.

CHAPTER 21: BENIGN PROSTATIC HYPERPLASIA AND BLADDER CALCULI 669


long-term effects (both beneficial and adverse) and
on patient-reported outcomes.
Table 4

To fill the gap in this information, the National Insti-


Invasive Treatment Options for BPH

tutes of Health organized a randomized, double-


blind clinical trial called Complementary and Alter-
native Medicine for Urological Symptoms
Minimally Invasive Thermotherapy

(CAMUS). The objective of the study was to ana-


lyze Serenoa repens (saw palmetto) impact clini-
Nonsite specific therapies
TUMT® (transurethral microwave thermotherapy)
cally on clinical BPH. The results concluded that 4 commercial variations
increasing doses of saw palmetto fruit extract did
not reduce lower urinary tract symptoms more than WITTM (Water-Induced Thermotherapy)
placebo.22
Debulking Minimally Invasive Surgery
Role of Anticholinergic Therapy with LUTS
TUVP (transurethral electrovaporization )
There is a high prevalence of overactive bladder
(OAB) in men, and the symptoms are often bother- PVP (photoselective laser vaporization)
some. OAB and BPH often coexist. Irritative symp-
toms can persist after BPH treatment. Men with HoLEP (holmium laser enucleation)
LUTS/BPH are generally treated with alpha block-
ers, but may also have OAB. Men treated for BOO HoLAPTM (holmium laser ablation)
and BPH with alpha blockers and surgical therapy
can continue to have persistent OAB symptoms. Saline bipolar TURP

Antimuscarinic therapy has shown statistically sig- TUIP (transurethral incision of prostate)
nificant effectiveness on OAB symptoms in men.
Men with LUTS and confirmed BOO are generally
initially treated first for BPH/ LUTS. An antimus-
Site-specific Therapies

carinic can be added as appropriate if OAB symp- Electrical radiofrequency ablation


toms persist. Several published studies demonstrate TUNATM (transurethral needle ablation)
antimuscarinic agents such as tolterodine can be uti-
lized effectively for symptom treatment of men with Laser
no increased risk for AUR.23–26 Although tolterodine ILC (interstitial laser coagulation)
has been the single agent recently evaluated in stud-
ies specifically for the symptomatic treatment of
men, many agents have historically been utilized in
Major Surgery

the symptomatic treatment of men, including agents TURP


such as oxybutynin. Many OAB trials have also
included men in their treatment population and Open prostatectomy
many studies are underway that evaluate the symp-
tomatic treatment of men with many other available
antimuscarinic agents approved for the indication of
OAB. However, monitoring of PVRs is probably a
wise precaution when utilizing antimuscarinic
agents in men at risk for retention.23,27-29

670 EDUCATIONAL REVIEW MANUAL IN UROLOGY


4. Interventional Therapies

The rationale for interventional therapies for the Office-based analgesia generally does not include
treatment of BPH to improve lower urinary tract intravenous sedation, but if applied should include
symptoms or treat urinary retention is based on monitored anesthesia support. A variety of oral anx-
relieving bladder outlet obstruction due to the iolytic agents and analgesics are available, as well
prostate growth impingement on the prostatic ure- as local infiltrating anesthesia agents. Awareness of
thra. Traditionally, for decades, the standard of care toxicity dosage is important as well as monitoring of
was to offer either transurethral resection of the patients after the procedure is completed. Car-
prostate (TURP) or open prostatectomy to reduce diopulmonary arrest support equipment is neces-
prostatic bladder outlet obstruction. These proce- sary, and one should be aware of vasovagal events.
dures are associated with well-known quality of life Vital signs monitoring is also important as well as
complications, such as impotence and incontinence, proper consideration of anesthesia surgical assess-
as well as life-threatening complications, such as ment risk when treating patients in the outpatient
bleeding with a well-defined but low mortality rate. setting.
As a result, a plethora of alternative options have
developed, from medical therapy to minimally inva-
sive therapy to traditional surgical therapy. The
Thermotherapies

options and alternatives are many (Table 4). Invasive management of BPH has changed over the
past 15 years, with the number of cases of
transurethral resection of the prostate (TURP)
decreasing from more than 250,000 per year to
Office Analgesia

Physicians use their own discretion in choosing an about 100,000 per year. Minimally invasive ther-
analgesic. Risks increase as the physician moves motherapy has become an option to surgical, as well
from 2% intraurethral Xylocaine® jelly as, medical therapy. It offers acceptable efficacy
(transurethral instillation) to non-steroidal anti- with tolerability and a low threat of adverse events,
inflammatory drugs (NSAIDs) and analgesics in and expands the treatment population.
combination with sedative/hypnotics (IM/PO);
perineal or transrectal pro-static block; IV sedation
(for example with morphine and/or versed); and
Treatment Principles of Thermotherapies

spinal or epidural anesthesia. Light general anesthe- Dependent on the temperature that is reached in the
sia and general anesthesia are the most risky in the tissue, a clinician can either coagulate or vaporize
office setting. tissue. A photothermal effect is induced when the
tissue temperature stays below 100º C, and tissue
Many office-based analgesia therapies are gets coagulated. Photoablative (vaporization)
available: effects occur when the tissue temperature reaches
past the boiling point of 100º C, and tissue is vapor-
• Preop medications—Ativan®, Vicoprofen®, ized. The total energy delivered during lesion for-
Celebrex®, Ditropan XL® mation (measured in joules) is determined by the
relationship between the power delivered (watts)
• Periop medications—Intravesical chilled lido- and time exposure (seconds). Minimally invasive
caine/marcaine solution, Ultracet®, hyoscyamine, thermotherapy generally has a therapeutic effect
intraurethral lidocaine using a coagulative necrotic approach. The process
of targeted heating above 50º C but below 100º C
• Postop medications—Alpha blockers, such as allows a gradual process of edema, inflammation
Flomax®, Uroxatral®, Vioxx®, ibuprofen or other followed by coagulation necrosis, and gradual atrophy.
NSAIDs Conceptually, alpha-adrenergic receptor destruction
is also thought to influence symptom relief.
• Also available are local anesthetic blocks, prostate
blocks and pudendal nerve blocks

CHAPTER 21: BENIGN PROSTATIC HYPERPLASIA AND BLADDER CALCULI 671


preserves urethral mucosal tissue and decreases
morbidity. High tissue temperatures produce a large
Current Technologies and Their Differences

TUNA. In 1996, the FDA approved the minimally volume of necrosis in a short time. The coolant
invasive transurethral needle ablation (TUNA) sys- makes the procedure more comfortable for the
tem for the treatment of BPH. Needle electrodes patient and protects adjacent healthy tissue. With
deliver low level radiofrequency energy precisely low temperature, non-cooled treatment, the hottest
into the target tissue. Shields help to protect the ure- point is at the urethra and diminishes rapidly.
thra from thermal damage. The radiofrequency
energy thermally devascularizes and denervates the The Targis Cooled ThermoTherapyTM (Urologix)
target tissue, creating necrotic lesions. The radiofre- was one of the first anesthesia-free, minimally
quency energy decreases prostate size by 10%–15% invasive BPH treatments designed for in-office use.
by heat and dehydration. The heat energy produced It uses proprietary microwave technology to pre-
and the resulting thermal effect is determined by cisely deliver targeted high energy deep to the dis-
the amount of tissue contact, length of the needle and eased tissue while leaving other nontargeted areas
wattage. intact. The low temperature coolant protects the ure-
thra during energy delivery, enhancing patient com-
TUMT. The use of microwaves in the prostate was fort and minimizing recovery times.
introduced in the 1990s with transurethral
microwave thermotherapy (TUMT). TUMT uses This system has been reported to have high durabil-
microwave heating to create temperatures greater ity in cases 5 years after treatment. It can be per-
than 45º C, which is the minimum cytotoxic level formed in an office or outpatient setting in about 1
of tissue. In order to create the high intraprostatic hour. The urologist places the microwave delivery
temperature, water-cooling is often used at the ure- system, which is on the tip of the flexible catheter,
thra to maintain patient comfort levels. It is this into the bladder and inflates a standard urological
combination that creates an effective, yet mini- balloon for positioning. The system’s patented
mally invasive treatment. dipole antenna is then positioned in the prostatic
urethra. The coolant emitted from the catheter
A handful of TUMT systems are available, each with simultaneously protects the urethra. The Prolieve®
different technical characteristics based on System (Boston Scientific) provides water cooled
microwave antenna design, cooling design, duration balloon dilation TUMT. The treatment begins with
of treatment and bladder balloon design. The goal of intraurethral lidocaine 10 minutes prior to catheter
treatment is to denervate and reduce the prostate placement, then proceeds to inflation of a 46F
volume. This is accomplished by tissue coagulation dilatation/prostatic compression balloon of the
via heat. transurethral catheter. This system features a quick
power ramp up to thermotherapy temperatures.
The temperature in the prostate depends on 3 Thermotherapy is delivered for about 45 minutes at
parameters: the microwave power (watts), which a maximum of 50 watts as heated fluid is circulated
heats the tissue; the blood flow, which cools down in the prostate balloon. During a 5-minute cool
the heated tissue; and heat conduction, which redis- down period, the prostate balloon remains inflated
tributes the heat. Of these, the microwave power and the bladder is filled with 180 cc. The catheter is
and the blood flow are the most important factors to removed, and a voiding trial is begun.
obtain therapeutic temperatures.
The CoreTherm® (ProstaLund) provides feedback
Both cooled and non-cooled systems are in use. treatment with a temperature monitoring probe that
Next-generation microwave systems provide sig- is inserted into the prostate from the catheter. The
nificantly more benefit to the patient, without the temperature probe contains multiple temperature
limitations of early microwave systems and intersti- transducers to map the temperature distribution in
tial therapies. Cooling allows generation of greater the prostate along the probe. The microwave power
intraprostatic tissue temperatures (50º C–80º C), and the treatment time are varied according to these

672 EDUCATIONAL REVIEW MANUAL IN UROLOGY


readings. Intraprostatic temperature monitoring
allows control by the treating physician, who can
Electrosurgical Techniques

estimate the prostate volume treated. The standard TURP. Successful treatment of BPH
is measured by a decrease of symptom score,
The TMX 2000TM (Thermatrx) provides the lowest increase in peak flow rate and Qmax, and decrease of
energy but is not a cooled system. The antenna bother score. Absolute indications for surgery
directs heat and energy to the appropriate target include refractory urinary retention, recurrent UTI,
(periurethral tissue) but does not direct heat to sensi- recurrent gross hematuria, renal insufficiency, blad-
tive tissues (bladder neck and external sphincter). der stones due to BPH and large bladder diverticu-
Low energy and low power are required to achieve lum due to BPH.
tissue necrosis without cooling, on average 6–7
watts. A known and reproducible dose of heat is The gold standard surgical treatment for BPH since
applied. Actual periurethral tissue temperature is the 1930s has been TURP, which has an 80%–85%
continuously measured. Multiple treatment coil success rate. This surgical procedure may lead to
lengths ensure the optimal dose of heat is applied to bleeding, dilutional hyponatremia (TUR) syndrome
the appropriate tissue of every prostate. The TMX and anesthetic risk. About 10%–15% of patients
2000 can treat prostatic lengths (bladder neck to need retreatment at 10 years.
veru) of 2.5 cm and longer.
During the TURP procedure, the surgeon uses a
resectoscope’s wire loop to remove the obstructing
tissue one piece at a time. The pieces of tissue are
Analysis and Review of

carried by the fluid into the bladder and then flushed


Thermotherapy Efficacy and Safety

Microwave technologies are not created equal. The out at the end of the operation. The standard TURP
paradigms for treatment vary. There are differences is an electrosurgical procedure that is performed in
in power (maximum power: Thermatrx 23 watts, an aqueous environment. Basically, electrical cur-
Prolieve 50 watts, Cooled ThermoTherapy 75 watts, rent is carried through an insulated sheath to a wire
CoreTherm 80 watts), delivery, urethral cooling, the loop that on contact with tissues creates electrosur-
efficacy and safety profile, and in the published gical heat that enables the wire to cut the tissue. It is
peer-review literature. Thermatrx has low power important that the liquid medium be nonionic to
and no urethral cooling. CoreTherm has high power maintain high current density in the wire loop for
and intraprostatic temperature monitoring. Targis efficient electrosurgical cutting of target tissue. His-
has high power and urethral cooling. At low blood torically, the irrigating solution first utilized was
flow rates, the CoreTherm adds safety by reducing sterile distilled water. However, a well-known com-
the microwave power and avoiding a possible plication related to water irrigation fluid absorption
overtreatment. At high blood flow, the CoreTherm via open venous channels during resection (dilu-
adds efficacy by increasing the microwave power. tional hyponatremia) can occur in addition to bleed-
Finally, the available efficacy data of themothera- ing related to the open venous channels. The
pies has been criticized for a number of issues, absorption of sterile water into the bloodstream pro-
including a concern over the quality of comparative duces not only hemodynamic shifts, but also
controls, plethora of positive data from single center osmotic and electrolyte shifts between tissue and
studies rather than multicenter studies, issues of cellular compartments resulting in seizures. With
sham vs true effects within the efficacy parameters, sterile water, hemolysis also occurred resulting in
appropriate direct comparator issues, technology renal failure due to myoglobinemia. Eventually,
changes occurring faster than clinical experience nonionic normo-osmotic irrigation fluids were
accumulated, no significant effect on PSA or developed, such as glycine irrigation, which are uti-
prostate volume, high or unreported retreatment lized today. However, these fluids were still non-
rates, and inability to predict which patients would ionic (hyponatremic), and dilutional hyponatremia
benefit.30-34 is still a risk. Management of this “TUR syndrome”

CHAPTER 21: BENIGN PROSTATIC HYPERPLASIA AND BLADDER CALCULI 673


generally consists of diuresis of free water with a Advancements in minimally invasive techniques
diuretic and careful management of fluid electrolyte have allowed these procedures to be performed
status with IV electrolyte solutions replacement both laparoscopically, robotically, and most
with frequent serum electrolyte monitoring. A recently through single port technology. However,
regional anesthetic such as a spinal is generally pre- its limitations include an intense learning and sig-
ferred for a TURP because impairment of cognitive nificant expertise in the field of advanced laparo-
function is an early and key diagnostic finding of scopic techniques.37-39
significant dilutional hyponatremia.
Electrovaporization and bipolar saline TUR. An
Of interest, TURP is not recommended for large electrosurgical modification of TURP combines 2
prostates, since prolonged resections have a signifi- electrosurgical effects into 1 action. Vaporization is
cant increased risk for dilutional hyponatremia as utilized in cutting, and desiccation is utilized in
well as bleeding requiring transfusion. In fact, mod- coagulation. The clinician uses a spiked, fluted or
ern day TURP has a higher mortality rate in the grooved roller bar to direct current through the
United States than open prostatectomy. As such, resectoscope.
TURP is considered invasive along with open
prostatectomy. This well-known complication is Five reported series of 240 patients have compared
generally avoided with the advent of bipolar TURP electrovaporization to standard TURP. Both proce-
as well as other energy modalities, such as laser dures led to 55% improvement in AUA Symptom
therapy, which does not require a nonionic irrigation Score, more than 100% peak urinary flow, no incon-
solution. tinence, more than 75% retrograde ejaculation and
no mortalities. Electrovaporization led to fewer
Currently, in the United States, 48% of all surgical transfusions and catheterizations and shorter hospi-
procedures for BPH are TURP. TURP leads to large tal stays.
changes in Qmax (+125%) and PVR (+74%). The
probability of surgical complications or UTI is The advantages of electrovaporization include an
about 15% and probability of bleeding is 2%. There efficacy that approaches TURP, similar improve-
is no definitive evidence that TURP causes delayed ment in IPSS, Qmax about 1 mL/sec less, decreased
mortality. morbidity, less catheterization time, less hospital
time and earlier return to work. Disadvantages
If surgery is indicated, TURP or a TURP-like proce- include limitation on gland size and operative time.
dure is still the best option. Morbidities are tech- Modern design modifications of monopolar elec-
nique-and technology-related, which are getting trovaporization include various new optimal probe
better every year. If obstruction is the issue, this is designs as well as various electrovaporative thick
the best pathway to treatment.35 loops. Equally important are microprocessor-
controlled generators that optimize current deliv-
Open prostatectomy. Standard TURP has its limita- ered against variable electrical tissue resistance to
tions that make it less than ideal for large prostate optimize electrosurgical effects.
glands. A TURP for a large gland requiring a long
resection time is a higher risk for bleeding requiring Finally, there are new designs that incorporate bipo-
transfusion and for TUR syndrome. Additionally, lar technology that eliminate the use of glycine and
transurethral resection in a large prostate is techni- allow the advantageous use of normal saline irriga-
cally challenging since instrumentation length may tion, which reduces the risk of dilutional hypona-
be inadequate, and concurrent large bladder stone tremia. Several competing technologies have varia-
removal or diverticula repair may warrant an open tions in the optimal electrode design, optimal gener-
procedure. Therefore, open prostatectomy contin- ator characteristics and competing bipolar technol-
ues to have a role in the management of bladder out- ogy designs. Available randomized peer review data
let obstruction in large prostates.36 suggest that the technique of surgical debulking is
similar to that of standard TURP, and short-term
efficacy data by several authors suggest outcomes

674 EDUCATIONAL REVIEW MANUAL IN UROLOGY


similar to TURP. However, the bipolar technology The advantages of the ILC were that it was a mini-
has an overall superior safety profile due to the mally invasive, office-based procedure, which
increased hemostatic qualities of the electrovapor- required minimal anesthesia and had modest clini-
ization technique, and the utilization of normal cal outcomes, with no significant sexual side effects
saline irrigation with the decreased risk of dilutional and minimal morbidity. Its disadvantages included a
hyponatremia.40-44 long catheterization, no immediate symptomatic
relief, high incidence of UTI and an unsatisfactory
Laser Therapies in BPH retreatment rate.

Principles of laser tissue interactions. In the late Limited randomized, controlled trials are available,
1980s, lasers became a novel way to open a wider but several published series suggest moderate
channel and improve voiding dynamics for BPH. improvement in AUA Symptom Score and peak
This cutting edge technology has been demon- flow rates with minimal long-term side effects com-
strated to be minimally invasive and safe. Efficacy, pared to TURP.45-47
both short-term and long-term, varies by the type of
laser utilized, expertise of the surgeon, effective The high-power, 532 nm wavelength KTP photose-
debulking of tissue and general technique applied. lective laser vaporization prostatectomy (PVP).
Individual techniques may vary greatly, but the 2 Photoselective laser vaporization prostatectomy
main tissue effects are coagulation and vaporiza- with the KTP laser is the newest advance in laser
tion. Coagulation results in necrosis, gradual atro- technology and has gained fast popularity due to
phy and sloughing of necrotic tissue, (i.e., a debulk- ease of use and powerful vaporization efficiency to
ing of the prostate). The laser also causes tissue and its remove prostate tissue. The 532 nm wavelength
water to vaporize and results in an instantaneous is fully transmitted through water so that there is no
debulking of prostatic tissue. energy absorption in the irrigant and the laser is
selectively absorbed by hemoglobin, producing
Interstitial laser coagulation (ILC). Interstitial laser efficient prostate tissue removal with hemostasis.
coagulation of the prostate is a minimally invasive The coagulation zone depth is 1-2 mm with a side
coagulative technique using an 830 nm diode laser firing optical fiber in non-contact mode. The end-
system. It is technically a thermotherapy utilizing a point is a TURP-like cavity. Prospective, multicen-
site-specific approach and a laser energy source. ter studies and single center studies report efficacy
The only available commercial system was the and durability to 3 years with high-risk patients and
Indigo Dye Laser System by Ethicon Endosurgery. patients with large glands being effectively treated.
It is no longer being commercially produced. It was Recent advancements have produced a more power-
a readily transportable, low-power, diode laser ful 532 nm wavelength laser (120W) utilizing a
device that used a 15–20 watt variable power diode laser source.46–49
source.
Holmium enucleation and ablation techniques.
ILC was an office-based technique that used local The holmium: YAG laser delivers energy in a pul-
anesthesia. The fiber was introduced via a satile manner using a thermomechanical mecha-
transurethral approach directly into the prostate. nism of action. The laser emits a beam of 2150 nm,
The laser transmitted energy through the fiber to a which has high absorption in water as well as
2-cm long, light-diffusing tip that was passed via a hemoglobin. This creates a high temperature vapor
cystoscope. The laser application time was 90 sec- bubble that explodes and has a thermomechanical
onds. An ellipsoid lesion of tissue coagulated vaporization impact that is hemostatic on its target.
around the axis of the fiber. The affected area had a
diameter of 1.5–2 cm and a length of 2 cm HoLEP. Holmium laser enucleation of the prostate
corresponding to the length of the energy-diffusing (HoLEP) produces results that are similar to TURP
fiber tip. with fewer complications (less intraoperative bleed-
ing). In this procedure, a holmium laser is used to
incise the prostate tissue and the end of the endo-

CHAPTER 21: BENIGN PROSTATIC HYPERPLASIA AND BLADDER CALCULI 675


5. LUTS, Sexual Function, and
Dysfunction with Medical and
Surgical Treatment

scope is then utilized to push and enucleate prostatic The impact of medical therapies on sexual function
tissue into the bladder. The enucleated tissue is then is less than surgical treatments because it is
removed with a morcellator. The benefits of HoLEP reversible. Surgical therapies may cause retrograde
over traditional surgery include a shorter hospital ejaculation. Retrograde ejaculation may occur with
stay, shorter catheterization time, and shorter recov- minimally invasive thermotherapy (about
ery time. However, this procedure is technically 5%–10%), PVP (30%–40%), transurethral incision
difficult. of the prostate (20%–30%), TURP (about 80%-
90%) and open prostatectomy (about 80%–90%).
HoLAP. Holmium laser ablation of the prostate
(HoLAP) uses a laser to vaporize obstructive pro-
static tissue in a side fire approach like the 532-mm
wavelength laser but is limited to small glands. The
decision whether to use HoLAP or HoLEP is based
primarily on the size of the prostate. Ablation usu-
ally is performed when the prostate is smaller than
40 cc. HoLAP offers the same advantages as PVP
when compared to TURP. Patients who undergo
HoLAP usually do not require overnight hospital-
ization and in most cases, the catheter is removed
the same day or the morning following the proce-
dure. There is only 1 published single series with
long-term follow-up. More data is needed before
any significant conclusions are made.53-57

Miscellaneous Devices: Other technologies at


wavelengths such as 980 (Evolve) and 2150
(Thulium) exist, however, little to no peer-reviewed
data exists for these technologies.

Urethral Stents. This is an area which has been


investigated for many years. The early stents had
significant discomfort and perineal pain, and thus,
fell out of favor. Newer stents have optimistic
early results.

676 EDUCATIONAL REVIEW MANUAL IN UROLOGY


6. Metabolic Syndrome, Lower 7. Etiology, Pathogenesis and
Urinary Tract Symptoms and BPH Presentation of Bladder Calculi

Recent studies have suggested a link between Bladder calculi account for 5% of urinary calculi
hypertension, symptomatic BPH and sexual dys- and usually occur because of bladder outlet obstruc-
function. 58 This relationship has directed tion, neurogenic voiding dysfunction, infection or
researchers and clinicians to study a relationship foreign bodies. Supersaturation of the urine in the
between lower urinary tract symptoms (LUTS) and bladder may subsequently develop and heteroge-
the presence of a metabolic syndrome.59 In fact, neous nucleation around a nidus may occur. In addi-
recent epidemiologic findings support a relationship tion, aggregation results in crystal growth and stone
by linking obesity, hypertension, cardiovascular formation.69 Nutritional deficiencies combined with
disease with BPH, lower urinary tract symptoms a low protein and high carbohydrate diet have been
and sexual dysunction.60,61 These findings have been implicated in the pathogenesis of pediatric bladder
supported by a hypothesis demonstrated in animal lithiasis, in Africa and the Middle East.70 Further-
studies that a link exists between autonomic ner- more, dehydration, diarrhea, fever and infection
vous system overactivity and the pathophysiologi- decrease urine production and enhance crystalliza-
cal development of urinary symptoms from progres- tion.71 The composition of bladder calculi are influ-
sive bladder dysfunction, due to prostatic hyperpla- enced by both the pH and the degree of saturation of
sia and prostatic bladder outlet obstruction.62-64 urine and consist of calcium oxalate, uric acid or
Sympathetic overactivity appears to be part of the struvite if infection is present.72
metabolic syndrome and development of LUTS sec-
ondary to BPH. Metabolic syndrome has been Diet, voiding dysfunction and uncorrected anatomic
described as a collection of abnormalities, including abnormalities, such as posterior urethral valves and
obesity, dyslipidemia, hypertension, impaired glu- vesicoureteral reflux, predispose children to bladder
cose metabolism, elevated C reactive protein stones. In men, several factors, such as urethral
(chronic inflammation) and autonomic sympathetic stricture, BPH, bladder diverticulum and neuro-
overactivity with insulin resistance, such as corre- genic bladder, are all associated with bladder calcu-
lated, hypothesized, underlying pathogenic mecha- lus formation. In women, the majority of vesical
nisms.65-68 calculi are secondary to prolapse, female pelvic
surgery, neurogenic bladder or foreign bodies.
There are numerous reports concerning bladder
migration of intrauterine devices and intravaginal
accessories, including pessaries, diaphragms and
cerclages. In both genders, spinal cord injury and
indwelling Foley catheters can predispose one to
bladder stone formation.

The typical symptoms of a bladder stone are inter-


mittent and painful voiding with terminal hematuria.
The pain may be referred to the tip of the penis,
along the course of the second and third sacral nerve,
or to the scrotum. In children, the pain may be
referred to the perineum through the third and fourth
sacral nerves. Lastly, >50% of bladder calculi are not
discernable on plain x-rays. The most accurate mean
of diagnosis remains cystoscopy.73

Treatment of Bladder Calculi

If conservative treatment is desired in a child, their


diet can be altered by the addition of phosphorous,
protein, vitamins and magnesium, which can
significantly decrease the incidence of vesical lithi-

CHAPTER 21: BENIGN PROSTATIC HYPERPLASIA AND BLADDER CALCULI 677


8. References

1. Guideline on the Management of Benign


asis, especially in endemic areas. In adults, irriga- Prostatic Hyperplasia. Diagnosis and
tion of the bladder with an alkaline solution or Treatment Recommendations. Chapter 1;
Renacidin has been reported to be beneficial for p1. American Urological Association;
some stones. 2003 (updated 2006).
http://www.auanet.org/content/clinical-prac-
Open surgery remains the main treatment of bladder tice-guidelines/clinical-guidelines/archived-
calculi in children. In adults, the classical treatment guidelines/bph-archive.cfm
for bladder calculi is endoscopic transurethral disin-
tegration with mechanical cystolithotripsy, 2. Kirby RS, Fitzpatrick P. Benign Prostatic
lithalopaxy ultrasound, electrohydraulic lithotripsy Hyperplasia. Health Press; 1995.
(EHL), Swiss Lithoclast and holmium: (YAG laser
lithotripsy). Traditionally, treatment of large 3. Girman CJ, Jacobsen SJ, Rhodes T, Guess HA,
(>4 cm) vesical calculi has been via open surgery. Roberts RO, Lieber MM. Association of
Extracorporeal shock wave lithotripsy should not be health related quality of life and benign pro-
considered as a first-line therapy for the vast major- static enlargement. Eur Urol. 1999;35(4):
ity of adults with bladder lithiasis. Percutaneous pro- 277-284.
cedures are rarely performed, but numerous modali-
ties are currently in development. Elimination of all 4. Jacobsen SJ, Jacobson DJ, Girman CJ, et al.
stone material is essential during any of these proce- Natural history of prostatism: risk factors for
dures, or the remaining stone material will serve as a acute urinary retention. J Urol. 1997;158(2):
nidus for future recurrent stone formation. 481-487.

5. Kirby RS. The natural history of benign pro-


static hyperplasia: what have we learned in the
last decade? Urology. 2000;56(5 suppl 1):3-6.

6. Rhodes T, Girman CJ, Jacobsen SJ, Roberts


RO, Guess HA, Lieber MM. Longitudinal
prostate growth rates during 5 years in ran-
domly selected community men 40 to 79 years
old. J Urol. 1999;161(4):1174-1179.

7. Arrighi HM, Guess HA, Metter EJ, Fozard JL.


Symptoms and signs of prostatism as risk fac-
tors for prostatectomy. Prostate. 1990;16(3):
253-261.

8. Roehrborn CG, McConnell JD. Etiology,


pathophysiology, epidemiology, and natural
history of benign prostatic hyperplasia. In:
Walsh PC, et al. Campbell’s Urology. 8th ed.
Philadelphia, PA: WB Saunders Co;
2002:1297-1336.

9. Veltri R, Rodriguez R. The molecular biology,


endocrinology, and physiology of the prostate
and seminal vesicals. In: Wein, Kavoussi,
Novick, Partin, Peters, eds. Campbell-Walsh
Urology. 9th ed. Philadelphia, PA: Elsevier;
2007:2677-2726.

678 EDUCATIONAL REVIEW MANUAL IN UROLOGY


10. McVary KT, Roehrborn CG, Avins AL, et al. 18. Wysowski DK, Farinas E. Finasteride in
Update on AUA guideline on the management benign prostatic hyperplasia. N Engl J Med.
of benign prostatic hyperplasia. J Urol. 2004;350(13):1359-1361; author reply
2011;185(5):1793-1803. 1359-1361.

11. Roehrborn CG. Definition of at-risk patients: 19. McConnell JD. The long-term effects of doxa-
baseline variables. BJU Int. 2006;97 (suppl zosin finasteride, finas teride, and the combi-
2):7-11; discussion 21-22. nation on the clinical progression of benign
prostatic hyperplasia. N Engl J Med.
12. Te AE, Ikeguchi EF, Choi J, Kaplan SA. Uro- 2003;349:2387-2398.
dynamics and benign prostatic hyperplasia. In:
Kirby RS, McConnell JD, Fitzpatrick JM, 20. Montorsi F, Roehrborn C, Garcia-Penit J, et al.
Roehrborn CG, Boyle P, eds. Textbook of The effects of dutasteride or tamsulosin alone
Benign Prostatic Hyperplasia. 2nd ed. Oxford, and in combination on storage and voiding
England: Isis Medical Media; 2002. symptoms in men with lower urinary tract
symptoms (LUTS) and benign prostatic hyper-
13. Foglar R, Shibata K, Horie K, Hirasawa A, plasia (BPH): 4-year data from the Combina-
Tsujimoto G. Use of recombinant alpha 1- tion of Avodart and Tamsulosin (CombAT)
adrenoceptors to characterize subtype study. BJU Int. 2011; 107(9):1426-1431.
selectivity of drugs for the treatment of
prostatic hypertrophy. Eur J Pharmacol. 21. Egerdie RB, Auerbach S, Roehrborn CG, et al.
1995;288(2):201-207. Tadalafil 2.5 or 5 mg administered once daily
for 12 weeks in men with both erectile dys-
14. Hatano A, Takahashi H, Tamaki M, function and signs and symptoms of benign
Komeyama T, Koizumi T, Takeda M. Pharma- prostatic hyperplasia: results of a randomized
cological evidence of distinct alpha 1-adreno- placebo-controlled, double-blind study. J Sex
eptor subtypes mediating the contraction of Med. 2012;9:271-281.
human prostatic urethra and peripheral artery.
Br J Pharmacol. 1994;113(3):723-728. 22. Barry MJ, Meleth S, Lee JY, et al. Effect of
increasing doses of saw palmetto extract on
15. Marks LS, Gittelman MC, Hill LA, Volinn W, lower urinary tract symptoms: a randomized
Hoel G. Silodosin in the treatment of the signs trial. JAMA. 2011;306(12):1344-1351.
and symptoms of benign prostatic hyperplasia:
a 9-month, open-label extension study. Urol- 23. Jaffe WI, Te AE. Overactive bladder in the
ogy. 2009; 74(6):1318-1322. male patient: epidemiology, etiology, evalua-
tion, and treatment. Curr Urol Rep.
16. Roehrborn CG, Bruskewitz R, Nickel JC, et al. 2005;6(6):410-418.
Proscar Long Term Efficacy and Safety Study
Group.Sustained decrease in incidence of 24. Kaplan SA, Walmsley K, Te AE. Tolterodine
acute uri nary retention and surgery with finas- extended release attenuates lower urinary tract
teride for 6 years in men with benign prostatic symptoms in men with benign prostatic hyper-
hyperplasia. J Urol. 2004;171(3):1194-1198. plasia. J Urol. 2005;174(6):2273-2275; dis-
cussion 2275-2276.
17. Roehrborn CG, Marks LS, Fenter T, et al.
Efficacy and safety of dutasteride in the four- 25. Jumadilova Z, Zyczynski T, Paul B,
year treatment of men with benign prostatic Narayanan S. Urinary incontinence in the
hyperplasia. Urology. 2004;63(4):709-715. nursing home: resident characteristics and
prevalence of drug treatment. Am J Manag
Care. 2005;11(4 suppl):S112-S120.

CHAPTER 21: BENIGN PROSTATIC HYPERPLASIA AND BLADDER CALCULI 679


26. Lee JY, Kim HW, Lee SJ, Koh JS, Suh HJ, 35. Santarosa RP, Te AE, Kaplan SA.
Chancellor MB. Comparison of doxazosin Transurethral resection, incision and ablation
with or without tolterodine in men with symp- of the prostate. In: Graham SD, ed. Glenn’s
tomatic bladder outlet obstruction and an over- Urologic Surgery. 5th ed. Philadelphia, PA:
active bladder. BJU Int. 2004;94(6):817-820. Lippincott, Williams & Wilkins; 1998:
921- 931.
27. Corcos J, Casey R, Patrick A, et al. Adouble
blind randomized dose-response study com- 36. Sandhu JS, Te AE. Open prostatectomy. In:
paring daily doses of 5, 10 and 15 mg con- Kaplan SA, Cabelin M, eds. Atlas of Urologic
trolled-release oxybutynin: balancing efficacy Clinics of North America. Philadelphia, PA:
with severity of dry mouth. BJU Int. WB Saunders; 2002.
2006;97(3):520-527.
37. Sotelo R, Spaliviero A, Garcia-Segui A, et al.
28. Dmochowski RR, Nitti V, Staskin D, Luber K, Laparoscopic retropubic simple prostatec-
Appell R, Davila GW. Transdermal oxybu- tomy. J Urol. 2005 Mar;173(3):757-760.
tynin in the treatment of adults with overactive
bladder: combined results of two randomized 38. Sotelo R, Clavijo R, Carmona O, Garcia A, et
clinical trials. World J Urol. 2005;23(4): 263- al. Robotic simple prostatectomy. J Urol.
270. 2008;179(2):513-515.

29. Gonzalez RR, Te AE. Overactive bladder and 39. Desai MM, Aron M, Canes D, et al. Single-
men: indications for anticholinergics. Curr port transvesical simple prostatectomy:
Urol Rep. 2003;4(6):429-435. Initial Clinical Report. Urology. 2008;
72(5):960-965.
30. Gonzâlez RR, Te AE. How do transurethral
needle ablation of the prostate and 40. Cabelin MA, Te AE, Kaplan SA. Transurethral
transurethral microwave thermotherapy vaporization of the prostate: current tech-
compare with transurethral prostatectomy? niques. Curr Urol Rep. 2000;1(2):116-123.
Curr Urol Rep. 2003;4(4):297-306.
41. Hon NH, Brathwaite D, Hussain Z, et al. A
31. Shabbir M, Kirby RS. Fact or fiction: what do prospective, randomized trial comparing con-
the benign prostatic hyperplasia data tell us? ventional transurethral prostate resection with
Curr Urol Rep. 2005;6(4):243-250. 32. plasmakinetic vaporization of the prostate:
Mattiasson A, Wagrell L, Schelin S, et al. Five- physiological changes, early complications
year follow-up of feedback microwave ther- and long-term followup. J Urol. 2006;
motherapy versus TURP for clinical BPH: a 176(1):205-209.
prospective randomized multicenter study.
Urology. 2007;69(1):91-96. 42. Seckiner I, Yesilli C, Akduman B, Altan K,
Mungan NA. A prospective randomized study
33. Te AE. Editorial comment on five-year follow- for comparing bipolar PlasmaKinetic® resec-
up of feedback microwave thermotherapy ver- tion of the prostate with standard TURP. Urol
sus TURP for clinical BPH: a prospective ran Int. 2006;76(2):139-143.
domized multicenter study. Urology.
2007;69(1):96-97. 43. de Sio M, Autorino R, Quarto G, et al. Gyrus
bipolar versus standard monopolar
34. Walmsley K, Kaplan SA. Transurethral transurethral resection of the prostate: a ran-
microwave thermotherapy for benign prostate domized prospective trial. Urology.
hyperplasia: separating truth from marketing 2006;67(1):69-72.
hype. J Urol. 2004;172(4 Pt 1):1249-1255.

680 EDUCATIONAL REVIEW MANUAL IN UROLOGY


44. Tefekli A, Muslumanoglu AY, Baykal M, Bin- 51. Te AE. The Next Generation in Laser Treat-
bay M, Tas A, Altunrende F. Ahybrid tech- ments and the Role of the GreenLight High-
nique using bipolar energy in transurethral Performance System Laser. Rev Urol.
prostate surgery: a prospective, randomized 2006;8(suppl3):S24-S30.
comparison. J Urol. 2005;174(4 Pt 1):
1339-1343. 52. Bachmann A, Muir GH, Collins EJ, et al. 180-
W XPS GreenLight laser therapy for benign
45. Terada N, Arai Y, Okubo K, et al. Interstitial prostate hyperplasia: early safety, efficacy, and
laser coagulation for management of benign perioperative outcome after 201 procedures.
prostatic hyperplasia: long-term follow-up. Eur Urol. 2012; 61(3):600-607.
Int J Urol. 2004;11(11):978-982.
53. Gupta N, Sivaramakrishna, Kumar R, Dogra
46. Kursh ED, Concepcion R, Chan S, Hudson P, PN, Seth A. Comparison of standard
Ratner M, Eyre R. Interstitial laser coagulation transurethral resection, transurethral vapour
versus transurethral prostate resection for resection and holmium laser enucleation of the
treating benign prostatic obstruction: prostate for managing benign prostatic hyper-
a randomized trial with2-year follow-up. plasia of >40 g. BJU Int. 2006;97(1):85-89.
Urology. 2003;61(3):573-578.
54. Elzayat EA, Elhilali MM. Holmium laser enu-
47. Nørby B, Nielsen HV, Frimodt Møller PC. cleation of the prostate (HoLEP): the endouro-
Transurethral interstitial laser coagulation of logic alternative to open prostatectomy. Eur
the prostate and transurethral microwave ther- Urol. 2006;49(1):87-91.
motherapy vs transurethral resection or inci-
sion of the prostate: results of a randomized, 55. Tan AH, Gilling PJ. Lasers in the treatment of
controlled study in patients with symptomatic benign prostatic hyperplasia: an update. Curr
benign prostatic hyperplasia. BJU Int. Opin Urol. 2005;15(1):55-58.
2002;90(9):853-862.
56. Tan AH, Gilling PJ, Kennett KM, Fletcher H,
48. Lee R, Gonzalez RR, Te AE. The evolution of Fraundorfer MR. Long-term results of high-
photoselective vaporization prostatectomy power holmium laser vaporization (ablation)
(PVP): advancing the surgical treatment of of the prostate. BJU Int. 2003;92(7): 707-709.
benign prostatic hyperplasia. World J Urol.
2006;24(4):405-409. 57. Tooher R, Sutherland P, Costello A, Gilling P,
Rees G, Maddern G. A systematic review of
49. Te AE, Malloy TR, Stein BS, Ulchaker JC, holmium laser prostatectomy for benign
Nseyo UO, Hai MA. Impact of prostate- prostatic hyperplasia. J Urol. 2004;171(5):
specific antigen level and prostate volume as 1773-1781.
predictors of efficacy in photoselective vapor-
ization prostatectomy: analysis and results of 58. McVary KT, Rademaker A, Lloyd GL, Gann P.
an ongoing prospective multicentre study at 3 Autonomic nervous system over activity in
years. BJU Int. 2006;97(6):1229-1233. men with lower urinary tract symptoms sec-
ondary to benign prostatic hyperplasia. J Urol.
50. Te AE, Malloy TR, Stein BS, et al. Photoselec- 2005;174:1327-1433.
tive vaporization of the prostate for the treat
ment of benign prostatic hyperplasia: 12- 59. Kasturi S, Russell S, McVary KT. Metabolic
month results from the first United States syndrome and lower urinary tract symptoms
multicenter prospective trial. J Urol. 2004; secondary to benign prostatic hyperplasia.
172(4 Pt 1):1404-1408. Curr Urol Rep. 2006;7(4):288-292.

CHAPTER 21: BENIGN PROSTATIC HYPERPLASIA AND BLADDER CALCULI 681


60. Hammarsten J, Högstedt B. Clinical, anthro- 68. Reaven GM, Lithell H, Landsberg L. Hyper-
pometric, metabolic and insulin profile of men tension and associated metabolic abnormali-
with fast annual growth rates of benign pro- ties-- the role of insulin resistance and the
static hyper plasia. Blood Press. 1999;8:29- sympathoadrenal system. N Engl J Med.
36. 1996;334:374-381.

61. Meigs JB, Mohr B, Barry MJ, Collins MM, 69. Papatsoris AG, Varkarakis I, Dellis A, Delive-
McKinlay JB. Risk factors for clinical benign liotis C. Bladder lithiasis: from open surgery to
prostatic hyperplasia in a community-based lithotripsy. Urol Res. 2006;34:163-167.
population of healthy aging men. J Clin Epi-
demiol. 2001;54:935-944. 70. Ali SH, Rifat UN. Etiological and clinical
patterns of childhood urolithiasis in Iraq.
62. McVary KT. Growth of the rat prostate gland is Pediatr Nephrol. 2005;20:1453-1457.
facilitated by the autonomic nervous system.
Biol Reprod. 1994;51:99-107. 71. Schwartz BF, Stoller ML. The Vesical calcu-
lus. Urol Clin North Am. 2000;27:333-346.
63. Golomb E, Rosenzweig N, Eilam R,
Abramovici A.. Spontaneous hyperplasia of 72. Menon M, Resnick MI. Urinary lithiasis: etiol-
the ventral lobe of the prostate in aging, geneti- ogy, diagnosis, and medical management.
cally hypertensive rats. J Androl. 2000;21: In: Walsh PC, Retik AB, Vaughan ED, Wein
58- 64. AJ, eds. Campbell’s Urology. 8th ed. Philadel-
phia, PA: WB Saunders; 2002:3229-3305.
64. Steers WD, Clemow DB, Persson K, Sherer
TB, Andersson KE, Tuttle JB. The sponta- 73. Drach GW. Urinary lithiasis: etiology, diagno-
neously hypertensive rat: insight into the sis, and medical management. In: Walsh PC,
pathogenesis of irritative symptoms in benign Retik AB, Stamey TA, Vaughan ED, eds.
prostatic hyperplasia and young anxious Campbells Urology. 6th ed. Philadelphia, PA:
males. Exp Physiol. 1999; 84:137-147. WB Saunders:;1992:2140.

65. Haffner SM. The metabolic syndrome: inflam-


mation, diabetes mellitus, and cardiovascular
disease. Am J Cardiol. 2006;97:3A-11A.

66. Hammarsten J, Högstedt B, Holthuis N, Mell-


ström D. Components of the metabolic syn-
drome-risk factors for the development of
benign prostatic hyperplasia. Prostate Cancer
Prostatic Dis. 1998;1:157-162.

67. Rohrmann S, De Marzo AM, Smit E, Giovan-


nucci E, Platz EA. Serum C-reactive protein
concentration and lower urinary tract symp-
toms in older men in the Third National Health
and Nutrition Examination Survey (NHANES
III). Prostate. 2005;62:27-33.

682 EDUCATIONAL REVIEW MANUAL IN UROLOGY


Chapter 22:
Surgical Disease of
the Adrenal Gland
Jay T. Bishoff, MD

Contents

1. Objectives

2. Embryology

3. Anatomy

4. Physiology

5. Specific Adrenal Cortex Steroid Description

6. Adrenal Medulla

7. The Incidental Adrenal Tumors

8. Imaging of the Incidental Adrenal Tumor

9. Pheochromocytoma

10. Primary Hyperaldosteronism


(Conn’s Syndrome)

11. Cushing’s Syndrome – Glucocorticoid Excess

12. Adrenal Insufficiency (Addison’s Disease)

13. Adrenal Cortical Carcinoma

14. Metastatic Disease to the Adrenal Gland

15. Principles of Adrenalectomy

16. Complications of Adrenalectomy

17. References

18. Questions

CHAPTER 22:SURGICAL DISEASE OF THE ADRENAL GLAND 683


1. Objectives 2. Embryology

• The objective of this chapter is to help urologists • The adrenal cortex derives from mesoderm
pass board and recertification examinations
• The medulla develops from neuroectoderm
• Given the complexity of surgical adrenal disease, a
review of this topic can be overwhelming • Heterotopic adrenal tissue is usually associated
with the kidney and its development, but in the
• Recognizing this fact, I have provided a chapter in case of renal agenesis or solitary kidney, the
outline form, to be a concise and rapid overview of adrenal gland—on the affected side—is found in
adrenal disorders the normal position

• On any given examination, there will only be a few


questions covering the topic of the adrenal gland

• The questions at the end of the chapter are pro-


vided to cover commonly tested adrenal topics

684 EDUCATIONAL REVIEW MANUAL IN UROLOGY


3. Anatomy

Size Venous Drainage


• The average size of the adrenal gland is: • Single main adrenal vein is the most important sur-
gical structure
* Length: 5 cm
• Right adrenal gland drains directly into the poste-
* Width: 3 cm rior aspect of the inferior vena cava. The right-
sided adrenal vein is usually very short and easily
* Depth: 1 cm torn during dissection

* Weight: 5 gm • Left adrenal gland drains directly into the renal


vein. The insertion is usually overlying the aorta
• The fetal adrenal gland weighs as much as 10 and is usually medial to the insertion of the left
grams, almost twice the size of the adult gland gonadal vein on the renal vein. The left side can
have a significant-sized phrenic branch as well
• The fetal cortex is much larger because it is
believed to play a role in embryogenesis and Nerve Supply
homeostasis. • The adrenal medulla is innervated by sympathetic
branches, which trigger the release of the
• After birth, the fetal cortex regresses during the medullary hormones:
first 6 weeks of life * Epinephrine
* Norepinephrine
• Ratio of adrenal to kidney weight is:
* Neonate 1:3 • The adrenal cortex has no known innervations
* Adult 1:30
Lymphatic Drainage
Location • The adrenal gland drains to:
• The adrenal gland is located in the retroperi- * Lateral aortic nodes
toneum. It is triangular in shape and is contained in * Nodal chains running from the renal artery to
its own subcompartment within Gerota’s fascia diaphragm

Blood Supply • Adrenal carcinoma requires an extensive lym-


• Arterial supply phadenectomy at the time of adrenalectomy
* Receives 7 cc/grams per minute
* There are 3 commonly described arterial
sources of flow:
1. Inferior phrenic artery is the
main arterial supply
2. Aorta
3. Renal artery

• The adrenal may have gonadal branches, which


are seen in 60% of fetal dissections

• Arterial vessels are rarely seen during surgery, but


the surgeon must know of their location and use
hemostatic instrumentation to prevent bleeding

CHAPTER 22:SURGICAL DISEASE OF THE ADRENAL GLAND 685


4. Physiology

Hormone Production Adrenal Cortex


• The adrenal cortical steroids have a common pre- • Developed from mesoderm
cursor with the conversion of cholesterol to preg-
nenolone. • Contains 3 distinct zones easily remembered from
the pneumonic “GFR”
Hormone Release
• Corticotrophin-releasing hormone (CRH) is syn- • Each zone secretes adrenal steroids derived from
thesized in the hypothalamus and arrives at the the basic structure of cholesterol changed to preg-
pituitary through the portal blood system nenolone

• CRH results in the release of ACTH from the pitu- • Zona glomerulosa (outer) production of aldos-
itary gland and ACTH stimulates cortisol release terone
from the adrenal gland
• Zona fasciculata (middle) production of cortisol
• Adrenal androgen production is also influenced by
ACTH as well as other factors (DHES, DHEAS) • Zona reticularis (inner) production of sex steroids
(see Figures 1 and 2)
• However, the primary control of aldosterone
secretion from the zona glomerulosa is angiotensin
II, not ACTH

Figure 1

Adrenal cortex hormone production: steroid production chart

686 EDUCATIONAL REVIEW MANUAL IN UROLOGY


Figure 2

Hypothalamic – pituitary – adrenal axis

CHAPTER 22:SURGICAL DISEASE OF THE ADRENAL GLAND 687


5. Specific Adrenal Cortex Steroid
Description

Aldosterone Cortisol
• Produced in the zona glomerulosa • ACTH and cortisol have diurnal variation; they are
normally high in the morning and low in the
• Primary stimulus for aldosterone release is evening

• Both ACTH and cortisol inhibit CRH release by


angiotensin II

• Other stimuli for release of aldosterone include: feedback


* ACTH
* Decreased serum sodium Androgens
* Elevated serum potassium • Sex steroid secretion by the adrenal gland is stimu-
* The juxtaglomerular apparatus located in the lated by ACTH but not gonadotropins
afferent arterioles of the kidney senses
decreased renal perfusion and, as a result, • Sex steroid secretion by the adrenal gland is of
releases renin. minor physiologic importance, except in disease
states
• Renin from the kidney cleaves a single bond on
angiotensinogen-releasing angiotensin I. * I.e., elevated concentrations in women with
hirsutism from adrenal tumors
• ACE (angiotensin-converting enzyme) splits 2
amino acids off of angiotensin I to make • The exact control mechanisms for adrenal andro-
angiotensin II. gens have not been determined

* Note: In the past it was believed that ACE


from the lungs was the major site of conver-
sion of AI to AII. More recently, it has been
discovered that ACE is also found in the kid-
ney, duodenum, ileum and the uterus. Current
thoughts are that the peripheral sites play an
equal role to the lung in conversion of AI to
AII

• Angiotensin II directly stimulates the adrenal


glomerulosa cells to release aldosterone

• Aldosterone activates Na-K pump in nephron:


which results in the retention of Na and water
(water follows Na)

Glucocorticoids
• Glucocorticoids are responsible for a vast array of
different physiologic functions in the body:
* Salt retention
* Skeletal and cardiac contractions
* Protein catabolism
* Inhibits bone formation
* Inhibits collagen synthesis
* Increases vascular contractility
* Anti-inflammation
* Anti-immune activity
* Maintains normal GFR

688 EDUCATIONAL REVIEW MANUAL IN UROLOGY


6. Adrenal Medulla

• Distinct from cortex embryologically; derived • Metabolism:


from neuroectoderm * Catechol-O-methyltransferase and
monoamine oxidase break down the cate-
• Secretes catecolamines sympathetic stimulation cholamines to produce the break-down prod-
ucts:
• Stored in secretory cells
• Primary: Urine vanillylmandelic
• Preganglionic neurons trigger release acid (VMA)

Catecholamines • Secondary: Metanephrine and


• Large chromaffin cells secrete mainly normetanephrine
epinephrine, but also secrete norepinephrine
and dopamine

• Precursors to catecholamines are dietary tyro-


sine and phenylalanine

•Site of production of catecholamines: adrenal,


CNS, adrenergic nerve terminals

• Healthy humans circulating catecholamines


* Norepinephrine 73%
* Epinephrine 14%
* Dopamine 13%

• Phenylethanolamine N-methyltransferase
(PNMT) enzyme from the medulla converts
through methylation of norepinephrine, which
gives you epinephrine

• PNMT is located in the adrenal medulla. There-


fore, if a patient has an excess of both epinephrine
and norepinephrine, the tumor is in the adrenal
gland and not in other sites of chromaffin tissue

• Excess norepinephrine and epinephrine source is


adrenal pathology

Epinephrine/Norepinephrine
• Catecholamine secretion from neural sympathetic
nerves

• Half-life is only 20 seconds

CHAPTER 22:SURGICAL DISEASE OF THE ADRENAL GLAND 689


7. The Incidental Adrenal Tumors

• CT, MRI, US are more commonly performed than • Malignancy


in the past * Adrenal metastasis 3%
* Adrenal cortical cancer 4%
• Body imaging has become a new addition to the
physical examination Nature of Mass
• Medical history and size are important characteris-
• 0.5%–5% of abdominal CTs show abnormal tics
adrenal glands
• Adrenal metastasis
• Incidental adrenal finding * 6% have no history of cancer
*Found at autopsy in 2%–8% * 50% have a positive history of cancer

• Carcinoma almost never seen at autopsy • Adrenal cortical carcinoma


* 85% of adrenal tumors are nonfunctional and * Only 2% of all masses are cortical carcinoma
benign * However, 65% of masses >6 cm are cortical
* Significant cost of evaluation unless stream- carcinoma
lined
* Balance of costs and benefits Other (Rarer) Possibilities to be Included in Your
Differential Diagnosis
• Definition of an incidental mass • Myelolipoma/lipoma has CT findings that are sim-
* Mass >1 cm ilar to that of renal angiomyolipoma (fat) (see Fig-
* Discovered on examination for nonadrenal ure 3)
purpose
* Absence of signs or symptoms of adrenal dis- If the adrenal gland has fat density material, it is by
order definition a benign myelolipoma

• Questions to be asked: • Adrenal cyst/hematoma benign cysts can be fol-


* Is the mass functional? lowed and may be true cysts or pseudocysts
* Are there any physical signs or symptoms? • Ganglioneuroma
* Is there biochemical evidence of activity? • Neuroblastoma
1. Pheo screen
2. Potassium level • Findings after workup:
3. Glucocorticoid screen *If the patient has no overt signs or symptoms
* Is the mass malignant? of endocrine activity, positive findings discov-
* Is there any history of another malignancy? ered during a metabolic after workup are rare:
* Is imaging suggestive of malignancy? • Cushing’s 5%
• Pheochromocytoma 2%
Nature of Incidentally Found Adrenal Mass • Aldosteronoma <1%
N = 2005

• In a review of 2005, incidentally discovered


This point is very important! If the patient has a

adrenal masses, as well as percentages for different


known primary cancer, the adrenal mass will be

findings were:
metastatic from the primary site 50% of the time.
Therefore, consideration of and workup for other

• Nonfunctioning adenoma 82%


cancers is warranted in the patient with an inci-
dentally found adrenal mass.

• Functioning adenoma
* Cushing’s syndrome 5%
* Pheochromocytoma 5%
* Aldosteronoma 1%

690 EDUCATIONAL REVIEW MANUAL IN UROLOGY


Figure 3 • CXR

Bilateral myelolipomas • Stool for occult blood

• Mammogram (in women)

The Limited Endocrine Evaluation is Very Useful

• Pheochromocytoma
to Determine the Presence of:

• Aldosterone-producing adenoma

• Cortisol-producing adenoma

Extent of Endocrine Evaluation Remains Contro-


versial
• Most limited evaluation consists of 3 simple tests:

• Serum potassium if hypertensive R/O aldos-


Bilateral myelolipomas seen in a 64 y/o male with
teronoma
large left-sided and smaller right-sided masses,
demonstrating fat content on CT imagining consistent
• Unlikely if not hypertensive, but can be difficult to
with myelolipoma
diagnosis if on diuretics, K replacement and other
antihypertensive medications
Subclinical Cushing’s Syndrome
• This is a real entity and often misunderstood by • Plasma metanephrines: most sensitive test for
urologists as not clinically significant pheochromocytoma

• Total 24° cortisol production is often normal in • 24-hour urine cortisol


these patients, but they still have significant
pathology • The low-dose dexamethasone suppression test is
most sensitive for subclinical Cushing’s syn-
• Subtle changes often occur such as the loss of diur- drome, but 24-hour urine cortisol is a good start
nal variation of cortisol secretion
Diagnosis
• Prognosis is variable with 3 different possibilities: Endocrine Evaluation
* Normalization • The indications for a complete endocrine evalua-
* Persistence tion include:
* Progression to full clinical Cushing’s 1. Findings on examination and H&P sugges-
tive of an excess of specific hormone
• Subclinical Cushing’s is an indication for 2. Positive findings on limited evaluation
adrenalectomy and should be considered in the dif- screening tests
ferential diagnosis of the incidental adrenal mass
Hypertension Caused by an Adrenal Source is
Incidental Adrenal Mass Initial Evaluation Rare
• History and physical exam • 15% of adult population is hypertensive

• Look for signs of hormonal syndromes • Only 1% is adrenal in origin


1. 0.5% primary hyperaldosteronism
• Search for occult malignancy 2. 0.2% pheochromocytoma
3. 0.2% Cushing’s syndrome
CHAPTER 22:SURGICAL DISEASE OF THE ADRENAL GLAND 691
8. Imaging of the Incidental
Adrenal Tumor

• Most incidental adrenalomas are nonfunctional MRI


• BOTH carcinoma and pheochromocytomas are
• If not a metastasis, then usually a nonfunctioning
adenoma
hyperintense on T2 images (i.e., they “light up” as
they go from T1 to T2)

• Rarely adrenal cortical carcinoma • Intensity of the adrenal gland on T2 relative to liver
<0.8 suggests cortical adenoma (see Figures 4a and
• Uncertain progression of nonfunctional adenoma 4b)
complicates issue
• Chemical-shift MRI
• Strategies and criteria for radiographic imaging
continues to evolve, but some general guidelines • Signal from cortical adenomas drops out in
are available to help in the decision for imaging opposed phase

Diagnosis CT Adenoma Characteristics


• Sharp margins
Imaging
• It is rare for a nonfunctional adenoma to become • Smooth and homogenous
functional
• Lipid-rich
• Most series report no progression of small adeno-
mas to cancer • Most have density <10 HU on noncontrast images

• 3 series do report the occurrence of adrenal corti- • Values for cancer and pheochromocytoma are
cal carcinomas in small tumors. much higher.

• Kloos et al, 1997: Found 3 cancers in patients • Density reduces at least 60% on initial contrast den-
with initial adrenal tumors ranging in size sity at scan delayed 15 minutes
from 3–5 cm who did not have follow-up.

• Linos et al, 1997: Reported on 3 patients with


tumors measuring 2.6–2.9 cm that were found
to be carcinoma.
Figure 4a

• Zaluaka et al, 1998: Reported on a series of


Benign adenoma hypointense on T1 MRI

311 incidental adrenal tumors and found 22


adrenal carcinomas (7%) ranging in size from
3.2–20 cm.

• The small adrenal carcinoma is rare but reported.


For examination purposes, it is probably safe to
use 5 cm as the cut-off for carcinoma. For func-
tional tumors, the surgeon should always be con-
cerned about the presence of carcinoma and treat it
accordingly during surgery

692 EDUCATIONAL REVIEW MANUAL IN UROLOGY


Figure 4b Figure 5a

Hypointense on T2 MRI Adrenal metastasis often appear


heterogeneous on T1 images

Imaging Criteria

Study Indicating Adenoma Figure 5b

Sens Spec
* CT density < 10 HU 71% 98%
Adrenal metastasis often is hyperintense

† CT washout > 60% 100% 100%


on T2 MRI

* MRI T2 intensity < 0.8 80% 80%


compared to liver

* Loss of signal on
chemical shift MRI 95% 100%
NP-59 scintigraphy 93% 100%

* All evaluating for fat content


† Becoming more widely accepted

Nuclear Scintigraphy
• NP-59 (131I-6b-iodomethyl-norcholesterol) taken
up by adrenal cortex (i.e., adenoma) not space occu-
pying lesion (cancer, pheochromocytoma)

• MIBG taken up preferentially by pheochromocy-


toma (see Figures 5a, 5b and 6)

CHAPTER 22:SURGICAL DISEASE OF THE ADRENAL GLAND 693


Surveillance Recommendations for the Incidental
Adrenal Mass
Figure 6

Classic findings of benign adenoma: • Older consensus recommendation:

• CT at 6 months
Homogenous, smooth, distinct margins,
lipid-rich and nonenhancing on contrast CT

• Annual endocrine evaluation for 4 years

• If the mass shows growth or develops detectable


endocrine function, then remove

• Newer emerging recommendations:


* If mass stable on scans at 3 and 12 months and
there is no function, then routine follow-up is
not required

Imaging Size Criteria


• 90% adrenal cortical carcinomas are usually >6 cm

• It is rare for an adenoma to reach this size

• However, CT underestimates actual size by as much


as 20%

• This is why many surgeons recommend surgical


removal for tumors ≥5 cm

• As shown in the previous studies, there is no cut-off


with perfect accuracy

• For test purposes, it is recommended to remove


adrenal glands ≥5 cm and treat them as adrenal corti-
cal carcinomas

Summary of Management Recommendations


• Hormonally active mass → surgical removal

• Hormonally inactive mass

• If imaging suggests cancer → surgical removal

• If >5 cm at any age → surgical removal

• If <3 cm and nonfunctional → observe

694 EDUCATIONAL REVIEW MANUAL IN UROLOGY


9. Pheochromocytoma

Incidence and Presentation


• The clinical symptoms are directly caused by release
Figure 7a

of epinephrine and norepinephrine T1 weighted images of the bladder showing

• Hypertension is present in 90% of cases and may


large mass

be paroxysmal, sustained or both

• Patients may also have orthostatic hypotension due


to low plasma volume

• 30% of pheochromocytomas are found at autopsy


and cause of death is often due to cardiovascular dis-
ease

• These tumors can arise anywhere in paraganglion


system

• Can arise from the bladder. If in the bladder, it is


often associated with micturition syncope

Common Clinical Findings of Pheochromocytoma


• Classic triad:
1. Episodic headache
2. Tachycardia
Figure 7b

3. Diaphoresis T2 weighted image shows enhancement of

• Most common in young to middle-aged adults with


the bladder pheochromocytoma

unsustained HTN with superimposed paroxysms, but


10% are normotensive (see Figures 7a and 7b)
Other commonly reported signs and symptoms:
* Sweating
* Pallor
* Flushing
* Palpitations
* Tachycardia
* Abdominal pain
* Chest pain
* Weakness
* Nausea
* Emesis
* Anorexia
* Psychosis

• Diagnosis occasionally occurs during pregnancy and


mimics preeclampsia and toxemia

• The maternal and infant mortality is 40%

CHAPTER 22:SURGICAL DISEASE OF THE ADRENAL GLAND 695


• Small tumors may actually be more symptomatic • Norepinephrine
than larger tumors, since large tumors are able to
bind more of the catecholamines and metabolize • Epinephrine
them inside the tumor
• Dopamine
• Cardiomyopathy
* Unexpected cardiovascular response during anes- • Levels that may be measured diagnostically
thesia is usually due to catecholamine-induced car-
diomyopathy • Two different tests
* This entity is felt to be caused when catecholamine
release results in small areas of myocardial necrosis, • Breakdown products of the primary catecholamines
which leads to inflammation and fibrosis * VMA
* It is estimated that the mortality rate can be as high * Metanephrines
as 80% during surgery if cardiomyopathy is
untreated Diagnosis
* It is recommended that all pheochromocytoma
patients, regardless of age, have a complete cardiac First Level
work-up before surgery • Plasma free metanephrines—the most sensitive test!

Pheochromocytoma and the Rule of 10s • 99% of patients have elevated plasma metanephrines
• Bilateral in 10%
• Next most sensitive test is 24° urinary cate-
• Familial (nonsporadic) in 10% cholamines which, if 2x normal, is considered diag-
nostic
• Pediatric in 10%
• Most specific test:
• Malignant in 10% * 24° urinary metanephrines
* Total metanephrines
• Normotensive in 10% * Vanillylmandelic acid

• Extraadrenal in 10% Second Level


• Clonidine suppression test (0.3 mg orally, test 3
• Multiple in 10% hours later). Essential hypertensive patients will
show a reduction in catecholamines by approxi-
• Childhood presentation breaks rules! mately 50%, but in pheochromocytoma there will be
no suppression
• 25% are bilateral, multiple or extraadrenal
• Glucagon stimulation test (2 mg IV and then 3 min-
Syndromes Associated With Pheochromocytoma utes later measure catecholamines). Pheochromocy-
• Von Hippel-Lindau disease toma patients increase catecholamines dramatically.
Normal patients show no change
• Von Recklinghausen’s neurofibromatosis

• Multiple endocrine neoplasia

• Associated with MEN II but not MEN I

Pathophysiology of Pheochromocytoma
• Excessive secretion of catecholamines

696 EDUCATIONAL REVIEW MANUAL IN UROLOGY


Management Preoperative Management
• There is some recent controversy over the early
• Surgical excision is mainstay alpha blockade of patients before surgery. However,
it is still widely recommended that patients be man-
• Evaluate for extraadrenal, multiple and bilateral sites aged medically to control hypertension with a long-
acting alpha blocker, such as phenoxybenzamine
• Primarily search for extraadrenal sites is radio- hydrochloride, before surgical resection
graphic
• Initial alpha blockade may be followed by a beta
• Extent of disease staged preoperatively blocker if needed to control blood pressure and heart
rate
• Unless metastatic disease apparent, cannot tell if it
will be malignant (see Table 1) • It is accepted that beta blockers should not be

Radiographic Localization
used before alpha blockade treatment, since beta
blockade alone can cause increased vascular

Imaging Sensitivity Specificity


resistance and high blood pressure due to unop-
posed alpha adrenergic tone

CT 98% 70% Orthostatic Hypotension


MRI * 100% 67% • Liberalize water and salt intake
MIBG † 86%-100% 85%-99%
• IV hydration preoperative
* Hyperintense on T2
† scintigraphy with 123 or 131I • Invasive monitoring intraoperative
-meta-iodobenzylguanidine lights up even if not
apparent on MRI • Careful postoperative observation

Table 1

MEN Syndromes

MEN I (“Wermer’s”) MEN IIA (“Sipple’s”) MEN IIB

Parathyroid adenoma/carcinoma Medullary thyroid carcinoma Medullary thyroid carcinoma

Pituitary hyperplasia/adenoma Pheochromocytoma Pheochromocytoma

Pancreatic islet cell hyperplasia Parathyroid Mucosal and GI neuroma


hyperplasia/adenoma

Marfanoid features

CHAPTER 22:SURGICAL DISEASE OF THE ADRENAL GLAND 697


Preoperative Preparation Required Postoperative Management
• Mortality up to 80% in untreated patients with car- • Hypotension is most common after surgery and is
diomyopathy usually due to hypovolemia, which resolves once the
vasoconstrictive effects of catecholamines are no
• Consultation recommended: endocrinologist and longer present and vascular tone is diminished
anesthesiologist
• Hypotension may occur immediately after division
• Preoperative goals: of the adrenal vein
* Normalize blood pressure
* Expand plasma volume Surgical Principles
* Prevent cardiac arrhythmias • Ligate adrenal vein first!
* Prevent intraop hypertension
* Minimize postop hypotension • It prevents intraoperative catecholamine surge, by
minimizing manipulation of the adrenal gland before
Preoperative Preparation the vein is ligated, thus the dogma: “Dissect the
• Stepwise plan: pharmaceuticals patient away from the tumor”

• Consider starting with long-acting calcium channel Surgical Outcomes


blocker • Excision does not always lead to long-term cure;
even in patients with a benign tumor
• Classic initial therapy is nonselective, long-acting
phenoxybenzamine • Benign recurrence rate is 5%

• Start 5 mg BID, ↑ 10 mg QOD PRN to 100 mg BID; • Malignant recurrence rate 10%
beware postoperative hypotension
• Hypertension may persist due to vascular disease,
• If refractory arrhythmias (i.e., sinus tachycardia), age or genetic predisposition
add a beta blocker
• 80% 20-year cause-specific survival
• Use only after alpha blockade (exacerbate HTN if
unopposed) • Long-term follow-up required
* Propranolol (20–40 mg TID)
* Rarely will you need to add metyrosine (alpha-
methyl-paratyrosine) in severe cases to block
the rate of catecholamine synthesis.

Intraoperative Management
• Pre-op anesthesia consultation

• Low threshold to postpone surgery if not prepared

• Certain popular agents are usually avoided:


* Halothane, propofol, morphine sulfate, pan-
curonium

• To control intraoperative hypertension:


* Phentolamine, nitroprusside, esmolol

698 EDUCATIONAL REVIEW MANUAL IN UROLOGY


10. Primary Hyperaldosteronism
(Conn’s Syndrome)

• Conn’s syndrome was first described as a clinical Pathophysiology


syndrome in 1955 • Primary hyperaldosteronism is the autonomous
(without stimulation) secretion of aldosterone from
• Clinical presentation classically described in 1955: the adrenal gland resulting in:
* Hypertension * Salt retention
* Hypokalemia * Hypertension
* Hypernatremia * Potassium wasting
* Alkalosis * May also be associated with the autonomous
* Periodic paralysis overproduction of cortisol

Modern Clinical Presentation • Secondary hyperaldosteronism results from reno-


• Hypokalemia vascular hypertension. The decreased renal blood
flow causes increased aldosterone production sec-
• Hypertension ondary to overproduction of renin

• Depressed renin Diagnosis

• High urine and plasma aldosterone First Level

Typical Clinical Features • Random Serum K <3.0 mEq/dL


• Refractory hypertension
• Plasma renin activity (PRA) is suppressed <2 ng/mL
• Hypokalemia
• 25% of essential hypertensives have depressed PRA,
• Profound hypokalemic response to diuretics but have normal serum potassium

• Often asymptomatic • Plasma aldosterone >15 ng/dL

• May also manifest: muscle weakness, tetany, • Aldosterone: PRA >20:1


headache, polydipsia
• Confirmation can be made by placing the patient on
• Patients often will have adapted a low-salt diet a high-sodium diet for 3 days (>200 mEq/day) and
which helps to decrease potassium wasting, since measuring 24-hour urine aldosterone. This is diag-
less Na available for K exchange, and thus symp- nostic if:
toms seem to improve. * Urine aldosterone >14 mcg/24°
* Urine K >30 mEq/24°
Primary Hyperaldosteronism (Conn’s syndrome)
• A very rare cause of hypertension in ~0.5% • Renal Vein Sampling
* Adenoma not always the cause
• Most commonly seen in Caucasians, 30–60 years * Series of 4/28 (14%) vein sample positive on
of age the opposite side of the nodule on imaging
* New recommendations for renal vein sampling
• Pathology:
* 70% aldosteronoma • Series of 12 patients
* 30% bilateral adrenal hyperplasia Positive CT findings showing small nodule
* Bilateral adrenal hyperplasia mostly in men Positive Screening tests

CHAPTER 22:SURGICAL DISEASE OF THE ADRENAL GLAND 699


11. Cushing’s Syndrome –
Glucocorticoid Excess

• 11 patients Vein sampling Cushing’s syndrome simply describes the symp-


73% positive from CT defined side toms from glucocorticoid excess, but does not dis-
18% positive from opposite side tinguish one cause from another cause. This needs
9% positive from both sides! to be distinguished from Cushing’s disease.

• New Recommendations: Cushing’s syndrome is summarized in Table 2.


Aldo/Renin Ratio > 20
Serum Aldo > 15 • Incidence: 1 per 100,000–500,000
• If age > 40 or bilateral, imagining findings needs
sampling • ACTH-dependent causes are the most common
• If age < 40 years old (82%)
• Localize with CT or MRI
• If 1 cm or greater • Also consider exogenous sources from therapeutic
• No sampling needed; remove steroids

Management
• Bilateral adrenal hyperplasia
Table 2

* Medical therapy indicated with spironolactone,


which is an aldosterone antagonist
Clinical Picture of Hypercortisolism or

* Side effects of spironolactone:


Cushing’s Syndrome

• Normalizes serum K+
• Can result in painful gynecomastia
• HCTZ normalizes blood pressure
Obesity 90%

• Unilateral adrenal adenoma


* Adrenalectomy following control of HTN and
Hypertension 80%

hypokalemia (amiloride, triamterene, spirono-


lactone)
Diabetes 80%

• Partial adrenalectomy has been reported and usually


requires intraoperative ultrasound assessment of the
Central obesity 80%

adrenal gland Weakness 80%

Result of Adrenalectomy
• 35% cured of hypertension
Muscle atrophy 70%

• 56% improved hypertension with need for fewer or


Hirsutism 70%

milder medications Skin striae 70%

• All but 9% have been cured or seen improvement Moon facies 60%

Easy bruising 50%

Acne 50%

Psychological changes 50%

700 EDUCATIONAL REVIEW MANUAL IN UROLOGY


ACTH-dependent Causes of Cushing’s Syndrome A 3-Step Process
• Cushing’s disease = pituitary hypersecretion of • 1: Establish presence of excess glucocorticoids
ACTH – 70% of cases
• 2: Distinguish between ACTH-independent and
• Ectopic ACTH – 12% of cases ACTH-dependent causes

• Most common sources of ectopic • 3: Determine specific etiology


ACTH production: *ACTH-dependent
* Carcinoma lung 52% • Cushing’s disease
* Carcinoma pancreas 11% • Ectopic ACTH secretion
* Thymoma 11% *ACTH-independent
• Adrenal adenoma, carcinoma or bilateral
ACTH-independent Causes (18%) hyperplasia
• Adrenal adenoma – 8%
Step #1 Hypercortisolism?
• Adrenal carcinoma – 6% • 24° urine free cortisol is the best initial diagnostic
test: >100 mg/24 is diagnostic
• Bilateral adrenal hyperplasia – 4%
• If equivocal: low-dose dexamethasone test
Pathophysiology * 1 mg at 11 pm, obtain plasma cortisol
• ACTH-dependent = primary excessive ACTH at 8 am
secretion from the pituitary gland. Again this is * Normal suppresses plasma cortisol to
Cushing’s disease and is the most common ACTH- <5 ng/mL
dependent cause * If plasma cortisol is not suppressed = Cush-
ing’s syndrome
• ACTH may also come from ectopic sources,
which also results in an increase in cortisol pro- • Can also use salivary cortisol concentration.
duction from the adrenal gland
Step #2 Relation to ACTH?
• ACTH-independent = primary excessive cortisol • Measure late afternoon ACTH >50 pg/mL =
secretion from an adrenal adenoma, adrenal carci- ACTH-dependent Cushing’s because ACTH
noma or bilateral adrenal hyperplasia inhibits excess is causing overproduction of glucocorti-
production of ACTH coids from the adrenal gland

Diagnosis • <5 pg/mL =ACTH-independent Cushing’s


because excess glucocorticoids from the adrenal
Is this Pseudo-Cushing’s? gland are causing suppression of ACTH
• Rule out exogenous sources, glucocorticoids
Step #3 Specific Entity?
• Topical creams, lotions • ACTH-dependent Cushing’s

• Review of oral medications • High-dose dexamethasone test


* 8 mg at 11 pm
• Nonendocrine causes of hypercortisolism * Obtain plasma cortisol at 8 am
* Major depression
* Alcoholism • <50% reduction = ectopic ACTH

• 50% reduction = pituitary tumor and diagnostic of


Cushing’s disease because these patients have a rel-
ative resistance to negative feedback

CHAPTER 22:SURGICAL DISEASE OF THE ADRENAL GLAND 701


12. Adrenal Insufficiency
(Addison’s Disease)

• (No reduction = ACTH-independent) • Rare but potentially fatal if not recognized and
treated
• ACTH-independent Cushing’s imaging (CT) dis-
tinguishes: • Incidence: 1 per 4,500–6,250 hospitalized patients
* Adenoma
* Cancer • 3rd-5th decades of life is the most common age
* Bilateral hyperplasia group

Management Primary Causes of Insufficiency


Adenoma/Cancer • Autoimmune adrenalitis
• Unilateral adrenalectomy
• Infection (TB, HIV most common)
• May require preoperative reduction of cortisol
secretion, if markedly elevated (see ectopic ACTH • Adrenal hemorrhage
secretion)
• The most common cause in industrialized counties
• Excessive cortisol: Suppresses the other adrenal is autoimmune destruction of the adrenal gland.
gland and will require intraoperative steroid sup- Diagnosis is made from finding 21-hydroxylase
port and a postoperative steroid taper may be adrenal antibodies
needed while the contralateral gland recovers
function. • The most common cause in nonindustrialized
countries is TB
Bilateral Adrenal Hyperplasia
• Bilateral adrenalectomy Secondary Causes
• Metastatic disease
• After bilateral adrenalectomy, 10%–20% get Nel-
son’s syndrome which is ACTH hypersecretion by • Surgical removal of the adrenal gland
chromophobe adenoma resulting in:
* Headache • Pituitary disease
* Deep pigmentation
* Visual disturbances • Sarcoidosis

Ectopic ACTH Secretion and Pituitary Tumor • Exogenous steroid use Pathophysiology
• Ectopic ACTH secretion and Clinical
* Resection of tumor if possible
* Medical therapy with metyrapone, ketocona- • Deficiency of both glucocorticoid and mineralo-
zole or aminoglutethimide corticoid

Pituitary Tumor • Acute:


* Transsphenoidal hypophysectomy * Fever
* For failure of pituitary surgery: * Severe hypotension
• Bilateral adrenalectomy or radiation ther- * Nausea/vomiting
apy * Lethargy
* Postoperative “crash” without apparent surgi-
cal complication

702 EDUCATIONAL REVIEW MANUAL IN UROLOGY


Pathophysiology and Clinical Presentation
• Deficiency of both glucocorticoid and mineralo-
Figure 8a

corticoid Adrenal mass is seen on MRI T1 weighted

• Chronic presentation:
image with low signal intensity similar to

* Fatigue
liver

* Weight loss
* Anorexia
* Nausea and vomiting
* Abdominal pain

Diagnosis
• Clinical scenario, especially postoperative

• Labs: Hyponatremia, hyperkalemia, azotemia,


hypoglycemia eosinophilia in 15%–20%

• The most common lab findings are:


* Hyponatremia
* Hyperkalemia

Screening Test
• ACTH stimulation 0.25 mg cosyntropin IV

• Measure plasma cortisol at 60 minutes Figure 8b

• Cortisol should be >18 ug/dL if normal T2 weighted images shows enhanced sig-

Management
nal intensity of the adrenal mass consistent

• Adrenal crisis
with adrenal cortical carcinoma

* IV access, blood for chemistry, ACTH and


cortisol
*Don’t wait for result before treating
* 2–3 liters D5 NS quickly
* 4 mg dexamethasone IV
* IV cortisol will interfere with the diagnosis
later during hospitalization

• Subsequent

• Confirm diagnosis

• Determine and treat cause

• Maintenance therapy
* Hydrocortisone 30 mg/day
* Fluorohydrocortisone 0.05–0.1 mcg/day (see
Figures 8a and 8b)

CHAPTER 22:SURGICAL DISEASE OF THE ADRENAL GLAND 703


13. Adrenal Cortical Carcinoma

• This a rare but very aggressive tumor Diagnosis


Endocrine Evaluation
• Incidence: 1 per 1.7 million people
• 80% are functional
• It is the model of cancers that invade into adjacent
structures • Often secretes multiple hormones:
* Glucocorticoids 72%
• Suspicion of adrenal carcinoma should prompt * Androgens 43%
consultation and team approach with other sur- * Precursors 22%
geons, vascular, or/and surgical oncology * Estradiol 8%
* Aldosterone 4%
• Accounts for 0.02% of cancers and 0.2% of cancer
deaths • Endocrine function is useful in the diagnosis

• Can be difficult to diagnosis • Finding of multiple hormones suggestive of carci-


noma
• Weiss microscopic criteria for diagnosis:
* High mitotic rate and nuclear grade • Carcinoma more often secretes sex steroids
* Atypical mitosis (Plasma DHEA, 24° urine 17-ketosteroids) than
* Eosinophilic cytoplasm do adenomas
* Diffuse architecture
* Necrosis • Endocrine function is useful as a marker
* Microscopic invasion
Radiography
Clinical Findings • Cross-sectional imaging localization and staging
• Constitutional
• Weight loss, malaise, fever • CT or MRI can be helpful

Endocrine Evaluation Shows That 80% are • Over 90% are >6 cm
Functional Tumors
• Irregular, inhomogeneous
Most Common Clinical Findings of Cushing’s
• Sex steroid excess • Can be distinguished from adrenal adenomas

• Oligomenorrhea • NP-59 adrenal scintigraphy

• Virilization/feminization • CT ± washout curve

• Can present as incidentaloma • MRI are bright or enhancing on T2 weighted


images (see Figures 8a and Figure 8b)

Adrenal Carcinoma Staging


• Common sites of metastases:
* Lung
* Liver
* Lymph nodes

704 EDUCATIONAL REVIEW MANUAL IN UROLOGY


14. Metastatic Disease to the
Adrenal Gland

• Staging studies: • More common than adrenal cortical carcinoma


* CXR 40%–50% of metastases from:
* CBC, LFT, LDH, basic metabolic panel * Melanoma
* CT and venogram vs MRI * Breast
* Lung
Surgical Management * Kidney
• Surgery is mainstay of treatment
• 8%–38% of patients with extraadrenal malignancy
will have adrenal metastases at autopsy
• 5-year survival for clinically localized
disease 35%
• With known primary cancer
*Adrenal mass is metastasis in 32%–73%
• 5-year survival for pathologically confined
disease 50% Diagnosis
• In setting of widespread metastases (most com-
• Frequently invades adjacent structures! mon), diagnosis obvious

• Radical en-bloc adrenalectomy with regional lym- • If no other metastases, do work-up as for adrenal
phadenectomy and total or subtotal excision of incidentaloma (see below), even with known pri-
adjacent organs (kidney, spleen, liver, colon, pan- mary cancer
creas)
• Findings suggestive of metastases
• May require cavotomy, bypass or other extensive
reconstruction • No endocrine abnormalities

Medical Management • Irregular/inhomogeneous on CT, bright on T2


• Metastatic or recurrent disease images of MRI, cold spot on NP-59 scintigraphy

• Adrenal metastasis is the best indication for FNA


• Mitotane (o,p’-DDD, a cogener of DDT)
of adrenal gland
* Hormonal response in 75%
* Response of tumor mass in 35% • FNA cannot reliably distinguish adrenal adenoma
* No improvement in survival vs adrenal cortical carcinoma
* All have significant toxicity
* Trials of adjunctive use of mitotane • Must rule out pheochromocytoma biochemically
before attempting FNA

• MRI appearance alone is not sufficient, since car-


cinoma, metastasis and pheochromocytomas are
all enhancing or bright on T2 weighted images

Management
• Solitary metastasis: May be benefit to adrenalec-
tomy

• Decision in concert with oncologist

• Avoid direct gland manipulation


* Hemorrhage
* Tumor spillage

• Take generous periadrenal tissue, but resection


into other organs not usually necessary (see Fig-
ures 9 and 10)
CHAPTER 22:SURGICAL DISEASE OF THE ADRENAL GLAND 705
15. Principles of Adrenalectomy

• The laparoscopic approach is the standard of care Thoracoabdominal Approach


for 90% of adrenal tumors • Through bed of 9th or 10th rib

• Large tumors >10 cm present unique challenges • Diaphragmatic incision, with excellent extra- and
and can be carcinoma intraperitoneal exposure

• These large tumors may be best approached • Large adrenal cancers


through open incisions
Laparoscopic Adrenalectomy
In testing situations, you must be prepared to dis-
cuss most of the different surgical options for organ • Standard of care today
removal. Since bleeding is the most common com-
plication encountered during adrenalectomy, you • Two different approaches:
must be prepared to discuss how you would convert * Transperitoneal
from laparoscopic surgery to open surgery and con- • Most common
trol significant bleeding from the vena cava on the • Most familiar anatomy
right side and the renal vein and aorta on the left
side. Two sponge sticks and vascular clamps should * Retroperitoneal
be readily available for emergencies. • Faster
• Especially on left, once the surgeon has
• Early division of adrenal vein: Most important some experience
step is pheochromocytoma
Options for Adrenalectomy
• “Dissect patient away from gland” • Pheochromocytoma
* Variable in size
• Traction on adjacent tissues
* Laparoscopy suitable according to the sur-
• For open surgery: geon’s experience
* Divide superior and lateral attachments
* Allow gland to be retracted into operative • Ligation of the adrenal vein can be easier with
field laparoscopy due to improved visualization com-
pared to open surgery
Open Surgical Adrenalectomy
• Approach determined by: • Aldosteronoma
* Adrenal pathology
* Size * Usually small (<3 cm)
*Associated conditions * Laparoscopy good choice
* Patient body habitus
* Medical fitness
* Surgeon experience

Anterior (Transperitoneal) Approach


• Chevron, hemi-chevron or subcostal (not midline)

• Large masses, moderate-sized cancers, or extraa-


drenal involvement

Posterior Approach
• Rib-resecting or lumbotomy

• Small benign masses, bilateral adrenalectomy

• Lumbotomy causes less pain and seems easy, but


can be difficult to access without some prior expe-
rience!

706 EDUCATIONAL REVIEW MANUAL IN UROLOGY


16. Complications of
Adrenalectomy

Figure 9 • Bleeding is the most common complication

• Prevention—Find correct landmarks: left-sided


dissection
Chevron incision allows excellent exposure

* Renal vein
to both adrenal glands and can be useful

* Upper pole kidney


when large tumors have the possibility of

* Adrenal vein is more medial


involving adjacent organs

* Adrenal is lateral to splenic artery

•Avoid hunting through fat!

• Harmonic Scalpel®/LigaSure AtlasTM best for


hemostasis

Other Reported Complications


During Adrenalectomy:
• Bleeding: most common

• Endocrine imbalance

Figure 10 • Liver or spleen laceration

Surgical approach to the adrenal gland • Pneumothorax

• Trocar placement
through the dorsal lumbotomy incisions

• Diaphragm injury

CHAPTER 22:SURGICAL DISEASE OF THE ADRENAL GLAND 707


17. References 18. Questions

Bauer SB. Anomalies of the kidney and uretero- 1. In a patient with Cushing’s syndrome due to
pelvic junction. In: Walsh PC, Retik AB, Vaughan adrenal adenoma, the changes in hormone
ED Jr, Wein AJ, eds. Campbell’s Urology. 7th ed. secretion following a high dose dexamethasone
Philadelphia, PA: WB Saunders; 1998:1708-1756. suppression test are best represented by:

Bravo EL. Primary aldosteronism: new approaches A. ACTH: ↑ Urinary free cortisol: ↓
to diagnosis and management. Cleve Clin J Med.
1993;60:379-386. B. ACTH: ↑ Urinary free cortisol: ↑

Goeres R, Knappe G, Gerl H, Ventz M, Stahl F. C. ACTH: ↔ Urinary free cortisol: ↔


Diagnosis of Cushing’s syndrome: reevaluation of
midnight plasma cortisol vs. urinary free cortisol D. ACTH: ↓ Urinary free cortisol: ↓
and low-dose dexamethasone suppression test in a
large patient group. J Endocrinol Invest. 1999:22: E. ACTH: ↓ Urinary free cortisol: ↑
241-249.

Goldfarb DA. Contemporary evaluation and man-


agement of Cushing’s syndrome. World J Urol. 2. Which of the following is the most sensitive
1999;17:22-25. biochemical test for confirming the diagnosis of
pheochromocytoma?
Hilton S, Schwartz LH. Radiologic imaging of geni-
tourinary neoplasms. In: Raghavan D, Scher HI, A. Plasma free metanephrines
Leibel SA, Lange PH, eds. Principles and Practice
of Genitourinary Oncology. Philadelphia, PA: Lip- B. Plasma catecholamines
pincott-Raven; 1997:33-42.
C. Urinary metanephrines
Lenders JW, Pacak K, Walther MM, et al. Biochem-
ical diagnosis of pheochromocytoma: which test is D. Urinary vanillylmandelic acid
best? JAMA. 2002;287:1427-1434.
E. Urinary catecholamines
Vaughan ED Jr, Blumenfeld JD. The adrenals. In:
Walsh PC, Retik AB, Vaughan ED Jr, Wein AJ, eds.
Campbell’s Urology. 7th ed. Philadelphia, PA: WB
Saunders; 1998:2915. 3. Adrenal hemorrhage is most frequently associ-
ated with:
Vaughan ED Jr, Sosa RE. Renovascular hyperten-
sion and other renal diseases. In: Walsh PC, Retik A. Heparin-induced thrombocytopenia
AB, Vaughan ED Jr, Wein AD, eds. Campbell’s
Urology. 7th ed. Philadelphia, PA: WB Saunders; B. Trauma
1998:423-459.
C. Warfarin therapy
Vella A, Nippoldt TB, Morris JC 3rd. Adrenal hem-
orrhage: a 25-year experience at the Mayo Clinic. D. Sepsis
Mayo Clin Proc. 2001;76:161-168.
E. Adrenal adenoma

708 EDUCATIONAL REVIEW MANUAL IN UROLOGY


4. A 45-year-old hypertensive man has an elevated D. Suppression to less than half the baseline in
24-hour urinary aldosterone after a period of patients with adrenal carcinoma
salt loading. CT scan of the adrenals is normal.
The best study for localization of a surgically E. Increased urinary secretion in patients with
curable lesion is: adrenal carcinoma

A. MRI scans

B. Adrenal venography 7. In a patient with an absent right kidney and a


left pelvic kidney, the right adrenal is:
C. MIBG scan
A. Absent and the left is adjacent to the upper
D. Adrenal venous sampling pole of the kidney

E. Iodocholesterol scan B. Absent and the left is in the normal anatomic


position

C. In the normal anatomic position and the left


5. A 55-year-old obese man with epigastric dis- is adjacent to the upper pole of the kidney
comfort is noted to have a 5-cm right adrenal
mass on CT scan. The mass measures -40 D. In the normal anatomic position and the left
Hounsfield units. The next step is: is in the normal anatomic position

A. Observation E. In the normal anatomic position and the left


is absent
B. 24-hour urine for metanephrines

C. MRI scans
8. A 40-year-old woman undergoes bilateral
D. Right adrenalectomy adrenalectomy for Cushing’s disease with com-
plete resolution of her symptoms. Replacement
E. Dexamethasone suppression test therapy with cortisone and fludrocortisone is
instituted. Three years later, she complains of
visual disturbances and is noted to have skin
hyperpigmentation. The most likely explana-
6. In the diagnostic evaluation of excess cortisol tion is:
secretion, the administration of 2 mg of dexam-
ethasone QID (high dose) for 2 days results in: A. Addison’s disease

A. No suppression of urinary corticosteroid B. Pituitary adenoma


secretion in normal patients
C. Excessive cortisone replacement
B. Suppression to less than half the baseline in
patients with adrenal hyperplasia D. Excessive ACTH production

C. Suppression to less than the baseline in E. Ectopic melanocyte-stimulating hormone


patients with benign cortical adenoma

CHAPTER 22:SURGICAL DISEASE OF THE ADRENAL GLAND 709


9. Angiotensin II causes blood pressure elevation 3. A.
by its effect on: Adrenal hemorrhage is an uncommon condition that
is infrequently diagnosed while patients are alive. It
A. Peripheral vascular tone and cardiac rate has classically been associated with meningococcal
septicemia. Recent series, however, have demon-
B. Peripheral vascular tone and blood volume strated it is most often associated with heparin-
induced thrombocytopenia. Sepsis, stress and war-
C. Blood volume and cardiac output farin therapy are rarer causes.

D. Blood volume and renin substrate 4. D.


Aldosterone secreting adenomas are unilateral and
E. The juxtaglomerular apparatus small in size (1-2 cm). They may be smaller than the
resolution of CT scanning. In this setting, adrenal
venous sampling for aldosterone levels is the most
10. Hypertension in Cushing’s syndrome is sensitive assessment to identify a unilateral ade-
primarily related to: noma. An MRI scan offers no advantage over CT
scan in this setting. Adrenal venography can be
A. Elevated plasma catecholamines complicated by adrenal infarction and is rarely used.

B. Elevated plasma aldosterone 5. A.


A mass with Hounsfield units between -30/ -140 is
C. Retention of water and salt characteristic of an adrenal myelolipoma. These
tumors are benign and are composed of lipid and
D. ACTH-stimulated renin myeloid tissue. No additional therapy or evaluation
is required. (Hilton S, Schwartz LH. Radiologic
E. Elevated angiotensin II imaging of genitourinary neoplasms. In: Raghavan
D, Scher HI, Leibel SA, Lange PH, eds. Principles
and Practice of Genitourinary Oncology. Philadel-
phia, Pa: Lippincott-Raven; 1997:33-42.)

6. B.
Answers

1. C. The dexamethasone suppression test is known to


Patients with adrenal Cushing’s syndrome have have a false positive rate of 20%. However, false
autonomous adrenal production of cortisol which negative tests are rare. Dexamethasone administra-
suppresses ACTH. Exogenous administration of tion should result in suppression of plasma cortisol
either low-dose or high-dose dexamethasone is values (50% of pretreatment value), urine 17-
unable to alter the autonomous adrenal production. hydroxysteroids and urine-free cortisol in patients
Therefore, it results in no change in either ACTH or with Cushing’s syndrome. Pituitary imaging with
urinary-free cortisol production. CT or MRI will disclose an adenoma in 50% of such
cases. Cortisol levels fail to fall to these levels in
2. A. patients with adrenal carcinoma or ectopic ACTH
A number of biochemical tests are available to secretion.
exclude or confirm the diagnosis of pheochromocy-
toma. Plasma-free metanephrines testing is the
most sensitive test (99%). Other acceptable clinical
tests with lower sensitivities include urinary frac-
tionated metanephrines (97%), plasma cate-
cholamines (84%), urinary catecholamines (86%),
urinary total metanephrines (77%), and urinary
vanillymandelic acid (64%).

710 EDUCATIONAL REVIEW MANUAL IN UROLOGY


7. D.
The adrenal gland is in the normal anatomic posi-
tion, no matter whether the kidney is absent or
ectopic. (Bauer SB: Anomalies of the kidney and
ureteropelvic junction. In: Walsh PC, Retik AB,
Vaughan ED Jr, Wein AJ, eds. Campbell’s Urology.
7th ed. Philadelphia, Pa: WB Saunders Co; 1998:
1708-1756.)

8. D.
Ten percent to twenty percent of patients who have
bilateral adrenalectomy for Cushing’s syndrome
later develop pituitary tumors which are almost
always chromophobe adenomas (Nelson’s syn-
drome). Progressive hyperpigmentation due to
melanocyte stimulating hormone release by corti-
cotropic releasing hormone, headaches, and visual
disturbances, are due to the expanding adenoma that
is shown by skull x-rays or CT scans of the sella tur-
cica. Pituitary basophilic adenoma that was initially
postulated by Cushing as causing the syndrome
named for him has, in fact, rarely been a factor.

9. B.
Renin is a proteolytic enzyme secreted in the juxta-
glomerular apparatus of the kidney and is physio-
logically inert. Antiotensinogen is a plasma globu-
lin substrate of hepatic origin, upon which renin acts
to produce angiotensin I. Angiotensin I is a
decapeptide which is also physiologically inert.
Angiotensin I is acted upon by converting enzyme
to produce the octapeptide angiotensin II.
Angiotensin II is the first effector hormone of the
renin system, and is the only substance directly
responsible for elevation in blood pressure in
patients with renovascular hypertension. It acts
upon the smooth muscle of the peripheral vascula-
ture to cause vasoconstriction, and also stimulates
the zona glomerulosa to produce aldosterone which
causes sodium retention in the distal tubule and thus
produces volume expansion.

10. C.
Cushing’s syndrome is due to glucocorticoid excess
while Cushing’s disease is a pituitary adenoma.
Approximately 80% of patients with Cushing’s syn-
drome have hypertension at the time of presenta-
tion. Glucocorticoids have weak mineralocorticoid
effects, resulting in retention of salt and water.

CHAPTER 22:SURGICAL DISEASE OF THE ADRENAL GLAND 711


712 EDUCATIONAL REVIEW MANUAL IN UROLOGY
Chapter 23A:
Sexually Transmitted
Diseases
Deborah R. Erickson, MD

Contents

1. HIV/AIDS: General Information

2. HIV/AIDS Treatment

3. Urologic/Renal Manifestations of AIDS

4. Aspects of AIDS Relevant to Urologic Practice

5. Protection of Health Care Workers


From HIV/AIDS

6. Other STDs

7. Human Papilloma Virus (HPV)

8. References

9. Questions

2010 MMWR STD Guidelines:

A. Genital Ulcers

B. Penicillin Allergy

C. Urethritis

D. Gonococcal Infections

E. Epididymitis

F. Genital Warts

CHAPTER 23A: SEXUALLY TRANSMITTED DISEASES 713


1. HIV/AIDS: General Information

A. Cause: human immunodeficiency virus (HIV)1,2 b. Chronic herpes simplex ulcers (>1
month)
1. HIV-1 is the most common cause of AIDS c. Disseminated or extrapulmonary Histo-
plasmosis, Coccidioidomycosis or
2. HIV-2: a related virus, causes some AIDS Mycobacterium infection
cases, but these are rare in the developed d. Kaposi’s sarcoma
world e. Pneumocystis carinii pneumonia
f. Toxoplasmosis of the brain
B. Stages of HIV infection g. Invasive cervical cancer (HPV pro-
gresses rapidly in HIV-infected women)
1. Initial or primary infection may involve an h. Non-Hodgkin's lymphoma
acute illness “acute retroviral syndrome” 2–4
weeks after exposure. Patients are unlikely to C. Diagnostic tests1,4,6,7
see a urologist for this
a. Symptoms resemble mononucleosis: 1. Most patients develop anti-HIV antibodies
fever, malaise, adenopathy, sore throat, 3–6 weeks after infection. ELISA and West-
skin rash1-4 ern blot, which are directed towards these
b. Anti-HIV antibodies are usually not pre- antibodies, may be negative if done too early
sent this early, so ELISA, Western blot
and point-of-care tests may be negative. 2. Positive ELISA should be confirmed with a
HIV viral load assay (amount of HIV more specific test (Western blot or HIV-1
RNA in bloodstream) can make the diag- RNA assay).4 Caveat: it takes more time for a
nosis as early as 11–12 days after infec- patient to become seropositive by Western
tion.2 blot than by ELISA.1 Therefore, if ELISA is
positive and Western blot is negative, there
2. Asymptomatic carriage of virus, with vari- are 2 possibilities:
able duration1 a. No HIV infection
a. Typical progressors (60%–70% of b. Recent onset of HIV infection (to con-
patients): 10–11 years firm; do HIV-1 RNA assay)4
b. Rapid progressors (10%–20% of
patients): <5 years 4. There are several FDA-approved rapid point-of-
c. Slow progressors (5%–15% of patients): service tests, using either blood or saliva.7-9 Sensi-
>15 years. Subsets include: tivity is high but false-negative results can occur,
1) Long-term nonprogressors (<2% of for example:
patients overall) remain asymp- a. Like standard tests, these tests measure
tomatic and have low viral load anti-HIV antibodies, so are negative if
2) Elite controllers have undetectable done too early
viral load b. Some, including the saliva test, are
directed towards a single viral protein so
3. Eventual loss of cell-mediated immunity; AIDS is are negative if the infecting virus has a
the end stage mutated form10

4. AIDS case definition: an HIV+ person with no D. Basic pathophysiology (targets of current anti-
other known cause of cell-mediated immune defi- HIV drugs are in italics)11
ciency, who has a total CD4 T-cell count below
200 cells/microliter, and/or any one of 25 differ- 1. HIV virus is a retrovirus, which means its
ent “AIDS-defining”diseases.2 Some well-known genetic material is RNA
diseases on the list include:
a. Candidiasis (bronchi, trachea, lungs or
esophagus)

714 EDUCATIONAL REVIEW MANUAL IN UROLOGY


2. HIV/AIDS Treatment

2. Certain inflammatory cells (especially T lym- A. At present, HIV cannot be completely eradi-
phocytes, monocytes and macrophages) have cated, for 2 main reasons1:
the CD4 receptor, which is essential for pro-
cessing most types of foreign antigens 1. Viral sanctuaries, where HIV virus can
remain latent and sheltered, or where drugs
3. HIV virus binds to the CD4 receptor and do not penetrate, such as lymphoid tissue and
invades the cell the central nervous system
a. To invade the cell, the virus must bind not
only to the CD4 receptor, but also to a 2. High genetic variability in virus, allowing
chemokine receptor (most viral types use escape from the immune response and devel-
the receptor CCR5)5,11 opment of drug resistance
b. A naturally occurring deletion of 32 base
pairs in the gene for CCR5 is present in B. Highly active antiretroviral therapy (HAART)
up to 10%–20% of people of Northern drug regimens
European descent. The mutant CCR5
never reaches the cell membrane, there- 1. Improve immune function and decrease the
fore: rates of opportunistic infections and some
1) Homozygotes (1%–2% of Cau- AIDS-related malignancies. With HAART,
casians) are almost completely average life expectancy is >20 years12
resistant to HIV infection
2) Heterozygotes may become 2. Currently, the goal of treatment is to keep the
infected but they have slower dis- amount of HIV RNA in the blood (viral load)
ease progression undetectable (<50 copies/mL). If HAART is
c. Other genetic differences also affect host stopped, viral load and CD4 cell counts
response to HIV5 return to pretreatment levels, so treatment
must be lifelong.
4. Viral RNA is transcribed to DNA by the enzyme
reverse transcriptase C. Classes of anti-HIV drugs13,14

5. Viral DNA is integrated into host cell genome 1. Reverse transcriptase inhibitors stop conver-
sion of viral RNA to DNA. There are 2 types:
6. Host DNA replicates, making new viral RNA a. Nucleoside analogs or NRTIs (eg,
zidovudine, emtricitabine) (Tenofovir is
7. RNA is translated to protein precursors technically a nucleotide analog, not a
nucleoside analog)
8. Viral protease cleaves precursors, leaving new 1) Most are renally cleared and do not
viral proteins affect cytochrome P450 enzymes,
so have fewer drug interactions than
9. New virus particles are assembled and shed from protease inhibitors or non-nucleo-
host cells side reverse transcriptase inhibitors
NNRTIs15
10. Loss of CD4+-carrying cells leads to immune 2) Main toxicity: inhibit host mito-
deficiency chondrial DNA polymerase, leading
to lactic acidosis, pancreatitis, sub-
11. Indices of disease severity: cutaneous lipoatrophy and periph-
a. Low CD4+ cell count eral neuropathy (the former 2 may
b. Viral load, which also is useful for monitor- be life-threatening)11
ing treatment response b. Non-nucleoside analogs (eg, efavirenz).
Main toxicities are skin reactions and
liver toxicity11

CHAPTER 23A: SEXUALLY TRANSMITTED DISEASES 715


Table 1

Doses of Nucleoside Analog Reverse Transcriptase Inhibitors

Generic Name Brand Name Usual Oral Dose

Zidovudinea Retrovir® 300 mg po bid

Didanosine Videx® 200 mg po bid

Stavudine Zerit® 40 mg po bid

Lamivudine Epivir® 300 mg po bid

Abacavirb Ziagen® 300 mg po bid

Emtricitabine Emtriva® 200 mg po q day

Tenofovirc Viread® 300 mg po q day

Emtricitabine/tenofovir Truvada® 200 mg/300 mg q day

Zidovudine/ lamivudine Combivir 300/150 mg bid

Lamivudine/abacavir Epzicom 300/600 mg q day

Zidovudine/ lamivudine/abacavir Trizivir 300/150/300 mg bid

Emtricitabine/tenofovir/efavirenz Atriplad 200/300/600 a day


a
Patent expired in 2005; generic forms are now available.
b
Severe hypersensitivity reactions can be fatal. Fortunately, a genetic test can identify who is susceptible (HLA-B*5701)
and this screening is recommended before starting the drug (http://us.gsk.com/products/assets/us_ziagen.pdf).
c
Technically a nucleotide analog, not a nucleoside analog.
d
This can be given alone because it is one of the recommended combinations: 2 nucleoside analog RT inhibitors (NRTI)
and 1 non-nucleoside analog RT inhibitor (NNRTI).

Table 2

Non-nucleoside Analog Reverse


Transcriptase Inhibitors

Generic Name Brand Name Usual Dose

Nevirapine Viramune® 200 mg PO BID

Delavirdinea Rescriptor® 400 mg PO TID

Efavirenz Sustiva® 600 mg PO QHS

Etravirineb Intelence 200 mg PO BID

a
Now rarely used because the other drugs in this table are

b
superior.
A second-generation drug, it can be effective for HIV
strains resistant to efavirenz or nevirapine.

716 EDUCATIONAL REVIEW MANUAL IN UROLOGY


2. Protease inhibitors stop precursors from c. Ritonavir is now recommended in all PI
being cleaved into viral proteins (examples: regimens as a booster to inhibit the
indinavir, nelfinavir) metabolism (and thus increase the serum
a. Main toxicities: visceral fat accumula- concentrations of) other anti-AIDS
tion, metabolic problems, such as insulin drugs14
resistance and hyperlipidemia d. PDE-5 inhibitors are metabolized by the
b. Most are potent inhibitors of cytochrome same enzyme system, so dose reductions
P450 enzymes, so they prolong the serum are recommended. Also, it is better to use
half-life of other drugs that use the same a drug with a short half-life (eg, silde-
metabolism. However, the effects vary nafil) to decrease the risk of toxicity. Spe-
and are not generalizable (for example, cific dose recommendations are as fol-
paradoxically, tipranavir induces the lows17:
enzymes).16 Usually, a urologist’s best 1) Sildenafil: 25 mg to start,
option is to consult with a pharmacist maximum 50
2) Vardenafil: 5 mg to start,
maximum 10

Table 3

Doses of Commonly Used Protease Inhibitors

Generic Name Brand Name Usual Oral Dosea

Ritonavir Norvir® n/ab

Indinavir Crixivan® 800 mg PO TID

Nelfinavir Viracept® 750 mg PO TID

Lopinavir/ritonavir Kaletra® 400mg/100 mg BID

Saquinavir Fortovase® 1200 mg PO TID

Saquinavir Invirase® 1000 mg PO BID

Atazanavir Reyataz® 400 mg po q day

Tipranavir Aptivus® 500 BID†

Fosamprenavir Levixa® 1400 mg PO BID

Darunavir Prezista® 600 mg BID

a
Doses vary, depending on whether or not ritonavir is added. Since ritonavir is now recommended with all protease
inhibitors, the doses here are the ones that apply to ritonavir coadministration. This is not always specified in the prescrib-
ing information, so consultation with a pharmacist is recommended

b
Now rarely used as an anti-HIV drug per se, but is combined with other drugs to decrease their metabolism by inhibiting
cytochrome P450 enzymes.

CHAPTER 23A: SEXUALLY TRANSMITTED DISEASES 717


3) Tadalafil on demand: 5 mg to start, E. After initial treatment, patients can get immune
maximum 10 reconstitution syndrome, in which the restored
4) Tadalafil daily: no more than 2.5 mg immune response is excessive.19 Two main types:
e. Many other drugs are metabolized by the
same enzymes, and require lower doses 1. Response to pathogens that were present
for patients on protease inhibitors. Some before treatment (symptomatic or asymp-
of the ones commonly used by urologists tomatic): viruses, mycobacteria, fungi, etc
are ketoconazole, itraconazole and ery-
thromycin 2. Appearance of autoimmune disease (eg,
Graves disease, sarcoid)
3. New drug: enfuvirtide (Fuzeon) inhibits
fusion of the HIV virus with CD4 cells.18 It
must be given subcutaneously and is cur-
rently used for patients failing conventional
anti-HIV therapy

4. New drug: maraviroc (Selzentry) CCR5


receptor blocker13,14
a. Oral drug: dose depends on whether the
drugs given along with it are CYP3A
inhibitors or inducers
b. Need to confirm the viral type is CCR5-
tropic
c. Patients failing conventional therapy, or
as an alternate third agent in initial treat-
ment (see below)13
d. Black box warning: liver toxicity, may be
preceded by systemic allergic reaction

5. New drug: raltegravir (Isentress) integrase


inhibitor; keeps viral DNA from being inte-
grated into host cell genome13,14
a. Oral drug: dose depends on concomitant
medicines
b. Originally for patients failing conven-
tional therapy, but now one of the recom-
mended third agents (see below)

D. 2010 initial treatment guideline13 is 2 NRTIs (rec-


ommended tenofovir/emtricitabine) plus one of
these:

1. Efavirenz (NNRTI). The fixed-dose combi-


nation of tenofovir/emtricitabine/efavirenz
has become the standard-of-care comparator
regimen in most clinical trials13

2. Ritonavir-boosted protease inhibitor

3. Raltegravir

718 EDUCATIONAL REVIEW MANUAL IN UROLOGY


3. Urologic/Renal Manifestations
of AIDS1,3,17

A. Almost any opportunistic infection can affect the e. Penile KS treatment is palliative; options
urogenital tract or genital skin.1,3,17 The most include HAART, local excision of small
common ones include: lesions, radiotherapy for large local
lesions or systemic chemotherapy if
1. Tuberculosis metastatic

2. Aspergillus, Candida and other fungi 2. Non-Hodgkin’s lymphoma is the second


most common AIDS malignancy. The testes
3. Toxoplasma gondii are commonly involved, but kidneys or
ureters also may be involved
4. Cytomegalovirus
3. Invasive cervical cancer is one of the AIDS-
5. Genital herpes defining conditions listed in definitions
above
6. Genital warts
4. HIV+ patients have a higher incidence of
7. Molluscum contagiosum is usually self-limit- testis cancer (seminomas and nonsemino-
ing in healthy patients, but in AIDS patients mas). The usual recommendation is to use the
the lesions can be larger, more numerous and standard treatment for the tumor type and
persistent. Treatments are the same as for stage, if the patient can tolerate it1,3
genital warts.2
5. Renal cell carcinoma is about 8 times more
B. AIDS patients are more likely to get bacterial common in the HIV-infected population3,17
UTIs and they require longer course of antibiotic
treatment 6. Bowen’s disease (carcinoma in situ) of the
genitalia is more common in HIV+ patients.
C. With any infection, AIDS patients have a more Squamous carcinomas have a more aggres-
severe presentation, so are more likely to develop sive course in HIV+ patients
renal or prostatic abscesses or Fournier’s gan-
grene 7. It is debated whether HIV patients have an
increased incidence of prostate cancer3,12,17
D. Hemorrhagic cystitis may occur with viral infec-
tions, including BK virus, JC virus, adenovirus F. Renal dysfunction is common in AIDS
and cytomegalovirus20,21
1. Renal dysfunction has multiple causes
E. AIDS patients have increased incidences of sev- including dehydration, sepsis, obstruction,
eral types of malignancies. The ones most rele- thrombotic microangiopathy, immune com-
vant to urologists are: plex-mediated glomerulonephritis and
1. Kaposi’s sarcoma (KS) (most common AIDS nephrotoxic drugs2,22
malignancy)
a. Arises from lymphatic endothelium3 2. HIV-associated nephropathy (HIVAN)1,3,22,23
b. Caused by human herpes virus-817 a. Direct infection of glomerular podocytes
c. Usually presents as a painless, pigmented and renal tubular cells with HIV
(pink, red or blue) subcutaneous nodule b. Usual findings on histopathology:
d. Most common location is the lower 1) Focal segmental glomerular
extremities, but may occur on the geni- sclerosis
talia. Penile KS can progress to gan- 2) Tubulointerstitial disease is always
grene. Up to 2% of AIDS patients ini- present and often appears out of
tially present with penile KS proportion to the degree of glomeru-
lar injury23

CHAPTER 23A: SEXUALLY TRANSMITTED DISEASES 719


4. Aspects of AIDS Relevant
to Urologic Practice

c. 12x more common in African Americans A. Indinavir can form stones that are not consis-
than Caucasians, 3rd leading cause of tently seen on unenhanced CT scan1,3,17
ESRD in African Americans ages 20–64
(after diabetes and hypertension) 1. These usually resolve with conservative
d. Usual presentation: severe proteinuria, treatment (stop drug temporarily, hydration)
renal insufficiency, kidneys have
increased echogenicity and may be 2. There is no formal literature on alpha block-
enlarged or normal size3,23 ers, but they may help in this situation the
same as for other types of stones
G. Both the central and peripheral nervous systems
can be affected, either by the HIV virus itself or 3. The bottom line on manipulating urine pH:
by opportunistic infections, such as toxoplasmo- a. Indinavir is most soluble at pH less than
sis and CMV. Thus, any type of neurogenic void- 5.3,25 Therefore, alkalinizing the urine is
ing dysfunction can occur definitely the wrong plan
b. In theory, acidifying the urine would help
1. The most common problem is urinary dissolve the crystals. In practice, it is dif-
retention ficult to get the urine pH below 5.5.
Therefore, the usual treatment is hydra-
2. With combined bladder dysfunction, bowel tion and stopping the drug, without spe-
dysfunction, and back or sciatic pain, con- cific attempts to acidify the urine
sider CMV polyradiculopathy. Diagnosis
made by lumbar puncture; may be reversible 4. Reasons for intervention include persistent
if detected early17 fever, intractable pain or solitary kidney

H. Hypogonadism17,24 5. Often the stones are actually masses of crys-


tals that disperse easily, so temporary stenting
1. AIDS patients have testicular atrophy for (plus hydration and stopping the drug) is usu-
many reasons, including the HIV virus itself, ally sufficient
chronic illness/cachexia, drug side effects
and, in the HAART era, the aging process 6. Indinavir crystals may precipitate in the renal
tubules and cause acute renal failure, without
2. Hypogonadism causes not only decreased any definite stones.25 Treatment is the same as
libido but also depression, low energy and for indinavir stones
muscle wasting
7. Other drugs besides indinavir can also form
3. The Endocrine Society Clinical Practice stones, including other protease inhibitors17
Guidelines recommend short-term (3–6 and efavirenz (an NNRTI)26
months) testosterone replacement for HIV-
infected men with low testosterone levels and B. AIDS patients also form the usual types of stones
weight loss24
1. In one series of 14 patients taking indinavir
4. Patients on testosterone therapy should be who had stones, only 4 patients’ stones con-
followed with PSA, DRE and blood counts tained indinavir27
(testosterone may cause polycythemia)
2. There are several reasons for stone formation
I. Adrenal insufficiency can be due to either CNS in AIDS patients. For example, malnutrition
infection or adrenal infection/necrosis. HIV and diarrhea lead to dehydration, urine acidi-
patients having stressful procedures should be fication and hypocitraturia. A complete
monitored for this metabolic evaluation is indicated, including
blood tests and 24-urine studies3

720 EDUCATIONAL REVIEW MANUAL IN UROLOGY


C. As described above in the section on treatment, 2. Possible reasons:
some protease inhibitors increase the levels of a. Inner foreskin is rich in Langerhans cells
other drugs by inhibiting cytochrome P-450 and other HIV target cells near the
enzymes. Especially relevant to urology are: epithelial surface, where there is no or
minimal keratin protection
1. PDE-5 inhibitors (sildenafil, vardenafil, b. Inner foreskin is susceptible to tears and
tadalafil) abrasions
c. In circumcised men, the penis is thought
2. Some antibiotics (eg, ketoconazole, itracona- to have a greater keratin barrier
zole, erythromycin)
H. Being HIV+ does not require that a surgeon stop
D. Trimethoprim-sulfamethoxazole can cause rhab- operating35; also see the American College of
domyolysis in AIDS patients, any time within 2 Surgeons’ Web site at http://www.facs.org/
weeks of starting the drug28 fellows_info/statements/st-13.html

E. HIV and prostate cancer I. Screening recommendations

1. Life expectancy after starting HAART is usu- 1. Recommendations from the US Preventive
ally >20 years. Therefore, prostate cancer Services Task Force are on their Web site
screening should be done according to the (http://www.uspreventiveservicestaskforce.o
AUA recommendations3 rg/uspstf/uspshivi.htm) and in reference 36.
In summary, pregnant women and people in
2. With localized cancer, treatment outcomes high-risk groups for HIV should be screened.
appear similar for men with vs without HIV.12 High-risk groups include:
If surgery is chosen, a laparoscopic approach a. Patients with other sexually transmitted
involves less exposure to the operating diseases
team3,29 b. Past or present IV drug use
c. Men who have had sex with men
3. Effects of HIV drugs on cultured prostate after 1975
cancer cells (the clinical relevance of these in d. Unprotected sex with multiple partners
vitro effects are still undetermined). e. People whose past or present partners
a. Nelfinavir: growth arrest, apoptosis30 are/were HIV-infected, IV drug users or
b. Saquinavir: apoptosis, radiosensitization, men who had sex with men
inhibits proteasome function31 f. People who exchange sex for money or
drugs (or their partners)
F. HIV-positive individuals are candidates for kid- g. Recipients of blood transfusions between
ney transplantation if they meet certain criteria. 1978 and 1985
They have a higher incidence of rejection than h. Patients who request HIV testing
non–HIV-infected patients, probably due to the i. People in high-risk settings, such as STD
complex drug interactions with immunosuppres- clinics, homeless shelters, correctional
sant drugs and anti-HIV agents32,33 facilities, TB clinics and adolescent
health clinics, that have a high prevalence
G. HIV and circumcision34 of STDs

1. In 3 randomized trials in subSaharan Africa,


male circumcision protected men against
HIV infection during vaginal sex with
women. Infection rates for circumcised men
were about 60% lower than for the uncircum-
cised control group

CHAPTER 23A: SEXUALLY TRANSMITTED DISEASES 721


2. CDC guidelines are online 4. Disadvantages of CDC guidelines: legal and
(http://www.cdc.gov/mmwr/preview/mmwr ethical concerns.39,40
html/rr5514a1.htm) and found in reference
37. CDC guidelines are different from above For example:
because they also recommend:
a. In health care settings, routinely screen a. An ethical principle is that the test must
all patients aged 13–64 years (note: this benefit the patient. A positive HIV test
applies to patients in health care settings, must lead to accessible HIV care39
not the entire community population)
b. Screen all patients initiating treatment for b. A positive HIV test may result in harm,
TB such as domestic violence, loss of hous-
c. Screen after notifying the patient that an ing, employment or insurance, etc.
HIV test will be performed unless he/she Therefore, even if state law does not
declines (opt-out screening). Specific require informed consent, an uninformed
signed consent for HIV testing should not patient who experiences these outcomes
be required. General informed consent might claim medical malpractice.39
for medical care should be sufficient to
encompass informed consent for HIV c. In some states, CDC recommendations
tests go against state laws.41
d. HIV test results should be provided in the A good Web site to check for your state is
same manner as results of other diagnos- http://www.nccc.ucsf.edu/consultation_l
tic or screening tests ibrary/state_hiv_testing_laws/
e. Prevention counseling should not be a
required part of HIV screening programs
in healthcare settings. Such counseling is
strongly encouraged for high-risk per-
sons in settings in which risk behaviors
are assessed routinely (eg, STD clinics)
but should not have to be linked to HIV
testing

3. Rationale and advantages of the CDC guide-


lines, which are supported by the American
College of Physicians and HIV Medicine
Association38
a. Many HIV-infected people do not think
of themselves as at-risk, so do not seek
testing
b. Many patients are infected for years
before diagnosis, and they may unknow-
ingly transmit the disease to others
c. Early identification prolongs the patient’s
life, decreases risk of transmission to oth-
ers, and is cost-effective at prevalence
rates as low as 0.05%–0.2%

722 EDUCATIONAL REVIEW MANUAL IN UROLOGY


5. Protection of Health Care
Workers From HIV/AIDS

A. Though not part of HIV/AIDS, this is a good spot 2 layers of glove removes most or all of
to discuss hepatitis transmission35 the patient’s blood

1. All health care workers should be immunized C. Postexposure prophylaxis (PEP) for health care
for hepatitis B workers3,35,42-44

2. With a sharps injury, hepatitis may be a 1. The risk of becoming HIV+ after a single
greater cause for concern than HIV sharps injury is low, even from a known HIV-
positive source
a. Efficiency of transmission is higher for a. Overall risk from a single percutaneous
hepatitis B and C viruses than HIV injury involving a known HIV-positive
source: 0.3%
b. Prevalences in USA: Hepatitis C=3–4 b. Risk is much greater for injection (hol-
million, hepatitis B and HIV=about 1 low) needles than for knives or solid nee-
million dles

B. Good news: no new documented cases of occu- 2. PEP reduces the risk even further (but does
pational exposure in the last 5 years, likely due to not decrease it to zero)
universal precautions, postexposure prophylaxis
and reduction of viral load in HIV-positive 3. A health care worker who has a sharps injury
patients by HAART42 should be evaluated IMMEDIATELY at an
employee health service or emergency
C. Universal precautions department. If PEP is going to be used, it
should begin within 2–4 hours of exposure. If
1. All patients are considered potentially unsure, better to give first dose and then await
infected results of testing or counseling42

2. New thoughts on what substances are consid- 4. USA National Clinicians’ Post-Exposure
ered infectious for HIV transmission42 Prophylaxis Hotline, PEPline, call 888-448-
a. Infectious—blood, tissue, semen, vaginal 4911.
secretions, pus and the following fluids:
cerebrospinal, amniotic, pericardial, peri- 5. The usual prophylaxis is 2 or 3 different
toneal, pleural and synovial. antiviral medications (depending on the level
b. Not considered infectious unless visibly of risk) taken for 4 weeks
bloody—urine, feces, nasal secretions,
saliva, gastric fluid, sputum, tears, sweat 6. Current CDC guidelines (basic summary; see
and vomit. references 35, 42-44 or Web sites
www.cdc.gov/mmwr/PDF/rr/rr5409.pdf or
3. Care with passing sharps (use a Mayo stand http://www.cdc.gov/mmwr/preview/mmwrht
as a way-station) ml/rr5409a1.htm
a. If source is known to be HIV-negative, no
4. Do not recap needles (even clean ones cause PEP needed for any type of exposure
injury) b. If source is known to be HIV+, PEP rec-
ommended for percutaneous injuries, or
5. Much evidence that double gloving is benefi- exposure of source blood to health care
cial!35 Example mechanisms include: worker’s mucous membranes or nonin-
a. Decreases skin injury from large tact skin
monofilament suture c. If source status unknown, generally, no
b. If the surgeon’s skin is penetrated, the PEP recommended, but consider PEP if
simple action of the blade going through the source has HIV risk factors

CHAPTER 23A: SEXUALLY TRANSMITTED DISEASES 723


6. Other STDs 7. Human Papilloma Virus (HPV)

For the other STDs, practical information for clini- A. Cause of cervical and anogenital cancers (also
cians is well described in the Campbell’s Urology some head and neck cancers) and genital warts
chapter45 and the CDC report.4 The portions of the
CDC report most relevant to urology are included in 1. Types 16 and 18 cause about 70% of HPV-
the handout. The entire report can be found online related cancers, but there are other oncogenic
http://www.cdc.gov/std/treatment/2010/STD-Treat- types besides 16 and 1848
ment-2010-RR5912.pdf A few additional details
are: 2. Types 6 and 11 cause about 90% of cases of
genital warts
A. Point-of-care (POC) tests for STDs8
B. Two HPV vaccines are FDA-approved. Both
1. HIV tests were discussed earlier approval indications emphasize people NOT pre-
viously exposed. Indications and target HPV
2. Syphilis tests8,46,47 types are shown, but both offer cross coverage
a. As of this writing, none are approved for for some other types
use in the US, but several are used in
other countries 1. Gardasil
b. Not mentioned in 2010 CDC STD guide- a. Types 6,11,16,18
lines. b. FDA-approved to prevent
c. Most are treponemal, so are positive for 1) Cervical/vulvar/vaginal cancer or
life and cannot distinguish current from genital warts caused by HPV types
past infection. One new test detects both 6, 11, 16, 18 in females aged 9–26
treponemal and nontreponemal antibod- 2) Genital warts caused by HPV types
ies47 6 and 11 in males aged 9–26
3) Anal cancer and associated precan-
3. POC tests for herpes simplex measure anti- cerous lesions due to HPV types 6,
bodies in the blood. Some of these are FDA- 11, 16, 18 in people aged 9–26
approved and available. They are sensitive
and specific with 2 caveats: 2. Cervarix
a. May be negative early in the disease, a. Types 16,18
before antibodies have formed b. Approved to prevent cervical cancer
b. Usually positive for life, so cannot distin- caused by HPV types 16 and 18 in
guish current from past infection females aged 10–25

4. POC tests for gonorrhea and chlamydia are C. CDC recommendations (http://www.cdc.gov/
not as sensitive as standard tests mmwr/preview/mmwrhtml/mm5901a5.htm)

C. Expedited partner therapy 1. Routine vaccination of females aged 11–12


years with 3 doses of either Gardasil or
1. Treating sex partners of patients diagnosed Cevarix
with chlamydia or gonorrhea by giving pre- a. OK to vaccinate at age 9–10 years
scriptions or medications to the patient to b. Catch-up vaccination recommended for
take to his/her partner without the health care females aged 13–26 not previously vac-
provider first examining the partner cinated, or if they did not complete the
3-dose series.
2. Allowed in some states, but not all. CDC has
a Web site (http://www.cdc.gov/std/ept) that 2. Gardasil may be given to males aged 9–26
describes expedited partner therapy and has a years to reduce risk of acquiring genital warts
link to the rules in each state (note: this is a permissive statement, not a
recommendation for vaccination)

724 EDUCATIONAL REVIEW MANUAL IN UROLOGY


C. Mechanisms of treatments for genital warts49-51 4. 5-armed trial of provider-applied treatments55
a. Treatment arms were:
1. Podofilox (podophyllotoxin) (patient- 1) Trichloroacetic acid
applied) and podophyllin resin (physician- 2) Podophyllin 25%
applied): antimitotic, cytotoxic 3) Cryotherapy
4) Cryotherapy + podophyllin 25%
2. Imiquimod: 5) Trichloroacetic acid +
a. No direct antiviral activity podophyllin 25%
b. Works by increasing cytokines such as b. Arms 1 and 2 had worse results
TNF-α and interferon, and stimulating than 3, 4 or 5
cell mediated immunity c. Arms 3, 4 and 5 had similar
c. Caveat: may induce autoimmune skin results; 4 was slightly better
disease such as lichen sclerosus52

3. Trichloracetic acid: coagulates proteins in


wart tissue

4. Sinecatechins 15% ointment49


a. Mixture of 8 catechins derived from
green tea
b. Reduce expression HPV gene products
E6 and E7, which lead to HPV-induced
cell growth and neoplasia. May also
inhibit proinflammatory enzymes and
proteases
c. FDA approved for external genital and
perianal warts in immunocompetent
patients 18 years or older

5. Cryosurgery, laser, excision: mechanical


destruction or removal

D. Which treatment for warts is best?

1. If not bothersome to patient, OK to forgo


treatment; may resolve spontaneously4

2. A nice algorithm for decision-making among


the myriad treatments is too long to repro-
duce here but found in reference50

3. Imiquimod alone equal to or better than abla-


tion alone.53,54 Imiquimod + ablation better
than ablation alone53

CHAPTER 23A: SEXUALLY TRANSMITTED DISEASES 725


8. References

1. Krieger JN. Urologic implications of AIDS 11. Greene WC, Peterlin BM. Charting HIV’s
and HIV infection. In: Wein AJ, Kavoussi remarkable voyage through the cell: Basic
LR, Novick AC, Partin AW, Peters CA, eds. science as a passport to future therapy.
Campbell-Walsh Urology. 9th ed. Philadel- Nature Medicine. 2002;8:673.
phia, PA: Saunders Elsevier; 2007:386-404.
12. Silberstein J, Downs T, Lakin C, et al. HIV
2. Chu C, Selwyn PA. Diagnosis and initial and prostate cancer: a systematic review of
management of acute HIV infection. Am the literature. Prostate Cancer Prostatic
Fam Physician. 2010;81:1239. Dis. 2009;12:6.

3. Lebovitch S, Mydlo JH. HIV-AIDS: uro- 13. Thompson MA, Aberg JA, Cahn P, et al.
logic considerations. Urol Clin North Am. Antiretroviral treatment of adult HIV infec-
2008;35(1):59. tion: 2010 recommendations of the Interna-
tional AIDS Society-USA panel. JAMA.
4. Centers for Disease Control and Preven- 2010;304:321.
tion. Sexually Transmitted Diseases Treat-
ment Guidelines, 2010. MMWR. 14. Wilson LE, Gallant JE. HIV/AIDS: the
2010;59(No. RR-12). management of treatment-experienced HIV
infected patients: new drugs and drug com-
5. Piacentini L, Biasin M, Fenizia C, et al. binations. Clin Infect Dis. 2009;48:214.
Genetic correlates of protection against
HIV infection: the ally within. J Intern 15. Back DJ, et al. The pharmacology of
Med. 2009;265:110. antiretroviral nucleoside and nucleotide
reverse transcriptase inhibitors. J Acquir
6. Iweala O. HIV diagnostic tests: an Immune Defic Syndr. 2005;39:S1.
overview. Contraception. 2004;70:141.
16. Croom KF, Keam SJ. Tipranavir. Drugs.
7. Aberg JA, Kaplan JE, Libman H ,et al. Pri- 2005;65:1669.
mary care guidelines for the management of
persons infected with human immunodefi- 17. Shindel AW, Akhavan A, Sharlip ID. Uro-
ciency virus: 2009 update by the HIV logic aspects of HIV infection. Med Clin N
Medicine Association of the Infectious Dis- Am. 2011;95:129.
eases Society of America. Clin Infect Dis.
2009;49:651. 18. Oldfield V, Keating GM, Plosker G. Enfu-
virtide: a review of its use in the manage-
8. Greer L, Wendel GD Jr. Rapid diagnostic ment of HIV infection. Drugs.
methods in sexually transmitted infections. 2005;65:1139.
Infect Dis Clin North Am. 2008;22(4):601.
19. Beatty GW. Immune reconstitution inflam-
9. Delaney KP, Branson BM, Uniyal A et al. matory syndrome. Emerg Med Clin N Am.
Evaluation of the performance characteris- 2010;28:393.
tics of 6 rapid HIV antibody tests. Clin
Infect Dis. 2011;52:257. 20. Studmeister A. Cytomegalovirus-induced
hemorrhagic cystitis in AIDS patient
10. Brown P, Merline JR et al. Repeatedly false treated successfully with valganciclovir.
negative rapid HIV test results in a patient AIDS. 2002;16:1437.
with undiagnosed advanced AIDS. Ann
Intern Med. 2008;149:71.

726 EDUCATIONAL REVIEW MANUAL IN UROLOGY


21. Ghez D, Oksenhendler E, Scieux C, Las- 30. Yang Y, et al. HIV-1 protease inhibitor
soued K. Haemorrhagic cystitis associated induces growth arrest and apoptosis of
with adenovirus in a patient with AIDS human prostate cancer LNCaP cells in vitro
treated for a non-Hodgkin’s lymphoma. Am and in vivo in conjunction with blockade of
J Hematol. 2000;63:32. androgen receptor STAT3 and AKT signal.
Cancer Sci. 2005;96:425.
22. Roling J, Schmid H, et al. HIV-associated
renal diseases and highly active antiretrovi- 31. Pajonk F, Himmelsbach J, Riess K, et al.
ral therapy-induced nephropathy. Clin The human immunodeficiency virus
Infect Dis. 2006;42:1488. (HIV)-1 protease inhibitor saquinavir
inhibits proteasome function and causes
23. Wyatt CM, Klotman PE, D’Agati VD. HIV apoptosis and radiosensitization in non-
associated nephropathy: clinical presenta- HIV-associated human cancer cells. Cancer
tion, pathology, and epidemiology in the Res. 2002;62:5230.
era of antiretroviral therapy. Semin
Nephrol. 2008;28:513. 32. Locke JE, Segev DL. Renal transplantation
in HIV-positive recipients. Curr Infect Dis
24. Bhasin S, Cunningham GR, Hayes FJ, et al. Rep. 2010; 12:71.
Testosterone therapy in adult men with
androgen deficiency syndromes: an 33. Stock PG, Barin B, Murphy B et al.: Out-
endocrine society clinical practice guide- comes of kidney transplantation in HIV-
line. J Clin Endocrinol Metab. infected recipients. N Engl J Med. 2010;
2006;91:1995. 363:2004.

25. Kalaitzis C, Passadakis P, et al. Urological 34. Brooks RA, Etzel M, Klosinski LE ,et al.
management of indinavir-associated acute Male circumcision and HIV prevention:
renal failure in HIV-positive patients. Int looking to the future. AIDS Behav. 2010;
Urol Nephrol. 2007;39:743. 14:1203.

26. Wirth GJ, Teuscher J, Graf JD et al. 35. Mohebati A, David JM, Fry DE. Current
Efavirenz induced urolithiasis. Urol Res. risks of occupational blood-borne viral
2006;34:288. infection. Surg Infect. 2010;11:325.

27. Nadler RB, Rubenstein JN, Eggener SE, et 36. U.S. Preventive Services Task Force:
al. The etiology of urolithiasis in HIV Screening for HIV. Recommendation state-
infected patients. J Urol. 2003;169:475. ment. Ann Intern Med. 2005;143:3242.
Campos-Outcalt D. Clarifying the US Pre-
28. Walker S, Norwood J, Thornton C, et al. ventive Services Task Force’s 2005 recom-
Trimethoprim-sulfamethoxazole associ- mendations. J Fam Pract. 2006;55:425.
ated rhabdomyolysis in a patient with
AIDS: case report and review of the litera- 37. Branson BM, Handsfield HH, Lampe MA,
ture. Am J Med Sci. 2006;331:339. et al. Revised recommendations for HIV
testing of adults, adolescents, and pregnant
29. Levinson A, et al. Approach to manage- women in health care settings. MMWR
ment of clinically localized prostate cancer Recomm Rep. 2006;55(RR-14):1-17.
in patients with human immunodeficiency
virus. Urology. 2005;65:91.

CHAPTER 23A: SEXUALLY TRANSMITTED DISEASES 727


38. Qaseem A, Snow V, Shekelle P et al. 47. Castro AR, Esfandiari J, Kumar S, et al.
Screening for HIV in health care settings: a Novel point-of-care test for simultaneous
guidance statement from the American detection of nontreponemal and treponemal
College of Physicians and HIV Medicine antibodies in patients with syphilis. J Clin
Association. Ann Intern Med. 2009;150: Microbiol. 2010;48:4615.
125.
48. Campo MS, Roden RB. Papillomavirus
39. Hanssens C. Legal and ethical implications prophylactic vaccines: established suc-
of opt-out HIV testing. Clin Infect Dis. cesses, new approaches. J Virol. 2010;
2007;45:S232. 84:1214.

40. Bartlett JG, Branson BM, et al. Opt-out 49. Mayeaux EJ Jr, Dunton C. Modern man-
testing for human immunodeficiency virus agement of external genital warts. J Low
in the United States: progress and chal- Genit Tract Dis. 2008;12:185.
lenges. JAMA. 2008;300:945.
50. Kodner CM, Nasraty S. Management of
41. Mahajan AP, Stemple L, Shapiro MF, et al. genital warts. Am Fam Phys. 2004;70:2335.
Consistency of state statutes with the Cen-
ters for Disease Control & Prevention HIV 51. Scheinfeld N, Lehman DS. An evidence
testing recommendations for health care based review of medical and surgical treat-
settings. Ann Intern Med. 2009;150: 263. ments of genital warts. Dermatol Online J.
2006;12: 5.
42. Goldschmidt RH. Occupational postexpo-
sure prophylaxis for HIV: the PEPline per- 52. O'Mahony C, Yesudian PD, Stanley M.
spective. Top HIV Med. 2010;18:174. Imiquimod use in the genital area and
development of lichen sclerosus and lichen
43. Panlilio AL, Cardo DM, et al. Updated U.S. planus. Int J STD AIDS. 2010;21:219.
Public Health Service guidelines for the
management of occupational exposures to 53. Schofer H, Van Ophoven A, Henke U, et al.
HIV and recommendations for postexpo- Randomized, comparative trial on the sus-
sure prophylaxis. MMWR Recomm Rep. tained efficacy of topical imiquimod 5%
2005;54(RR-9):1-17. cream versus conventional ablative meth-
ods in external anogenital warts. Eur J Der-
44. Chin RL. Postexposure prophylaxis for matol. 2006;16:642.
HIV. Emerg Med Clin N Am. 2010;28:421.
54. Stefanaki C, et al. Comparison of cryother-
45. Frenkel T, Potts J. Sexually transmitted dis- apy to imiquimod 5% in the treatment of
eases. In: Wein AJ, Kavoussi LR, Novick anogenital warts. Int J STD AIDS.
AC, Partin AW, Peters CA, eds. Campbell- 2008;19:441.
Walsh Urology. 9th ed. Philadelphia, PA:
Saunders Elsevier; 2007:371-385. 55. Sherrard J, Riddell L. Comparison of the
effectiveness of commonly used clinic-
46. Sena AC, White BL, Sparling PF. Novel based treatments for external genital warts.
Treponema pallidum serologic tests: a Int J STD AIDS. 2007;18:365.
paradigm shift in syphilis screening for the
21st century. Clin Infect Dis. 2010;51:700.

728 EDUCATIONAL REVIEW MANUAL IN UROLOGY


9. Questions

1. Which statement about erectile dysfunction 3. Which of the following STDs is least com-
and AIDS is true? mon among men in the United States?

A. Protease inhibitors prolong the half- A. Gonorrhea


life of PDE-5 inhibitors and increase
the risk of toxicity. B. Granuloma inguinale

B. Sexually active AIDS patients should C. Chancroid


carry postexposure prophylaxis to pro-
vide to their partners. D. Herpes simplex

C. AIDS patients should not receive treat- E. Human papilloma virus


ment for erectile dysfunction because
they might infect their partners.

D. Testosterone replacement is con- 4. At 5:00 PM, a urology resident cuts his fin-
traindicated due to the high risk of ger with a scalpel while doing emergency
prostate cancer. surgery on a patient of unknown HIV sta-
tus. Which statement is true?
E. Radical prostatectomy is contraindi-
cated due to the high risk of erectile A. The resident should go to employee
dysfunction. health the next morning for evaluation,
counseling and possible post-exposure
prophylaxis.

2. Comparing HPV vaccines, Gardasil: B. The patient’s blood cannot be tested


for HIV until the patient has recovered
A. Targets fewer HPV types than from anesthesia, received counseling
Cervarix. and given informed consent.

B. Is preferred over Cervarix in the CDC C. The resident should be more worried
recommendations. about acquiring hepatitis than HIV.

C. Is more effective than Cervarix for D. Double-gloving does not change the
patients previously exposed to HPV. resident’s risk of becoming infected
with HIV.
D. Requires fewer injections than
Cervarix. E. Urine and blood are equally likely to
transmit HIV.
E. Is the only HPV vaccine FDA-
approved for males.

CHAPTER 23A: SEXUALLY TRANSMITTED DISEASES 729


5. All of the following STDs have readily
available, reliable diagnostic tests, except:

A. Syphilis

B. Gonorrhea

C. Chlamydia urethritis

D. Chancroid

E. Herpes simplex

Answers

1. A.

2. E.

3. B.

4. C

5. D.

730 EDUCATIONAL REVIEW MANUAL IN UROLOGY


2010 MMWR STD Guidelines

The following is excerpted from Morbidity and Mor-


tality Weekly Report. Recommendations and
A. Genital Ulcers

Reports. December 17, 2010; Volume 59: Number


RR-12
Diseases Characterized by Genital,
Anal, or Perianal Ulcers

In the United States, most young, sexually active


patients who have genital, anal, or perianal ulcers
have either genital herpes or syphilis. The frequency
of each condition differs by geographic area and
population; however, genital herpes is the most
prevalent of these diseases. More than one etiologic
agent (e.g., herpes and syphilis) can be present in a
genital, anal, or perianal ulcer. Less common infec-
tious causes of genital, anal, or perianal ulcers
include chancroid and donovanosis. HSV, syphilis,
and chancroid have been associated with an
increased risk for HIV transmission, and genital,
anal, or perianal lesions might be associated with
conditions that are not sexually transmitted (e.g.,
yeast, trauma, carcinoma, aphthae, fixed drug erup-
tion, and psoriasis).

A diagnosis based only on the patient’s medical his-


tory and physical examination frequently is inaccu-
rate. Therefore, all patients who have genital, anal,
or perianal ulcers should be evaluated with a sero-
logic test for syphilis and a diagnostic evaluation for
genital herpes; in settings where chancroid is preva-
lent, a test for Haemophilus ducreyi should also be
performed. Specific tests for evaluation of genital,
anal, or perianal ulcers include 1) syphilis serology
and darkfield examination; 2) culture for HSV or
PCR testing for HSV; and 3) serologic testing for
type-specific HSV antibody.

No FDA-cleared PCR test to diagnose either herpes


or syphilis is available in the United States; how-
ever, such testing can be performed by clinical labo-
ratories that have developed their own tests and
have conducted a Clinical Laboratory Improvement
Amendment (CLIA) verification study. Type-spe-
cific serology for HSV-2 might be helpful in identi-
fying persons with genital herpes (see Genital Her-
pes, Type-Specific Serologic Tests). In addition,
biopsy of genital, anal, or perianal ulcers can help
identify the cause of ulcers that are unusual or that
do not respond to initial therapy. HIV testing should
be performed on all persons with genital, anal, or
perianal ulcers who are not known to have HIV
infection (see Diagnostic Considerations, sections

CHAPTER 23A: SEXUALLY TRANSMITTED DISEASES 731


on Syphilis, Chancroid, and Genital Herpes Sim-
plex Virus).
Treatment

Successful treatment for chancroid cures the infec-


Health-care providers frequently must treat patients tion, resolves the clinical symptoms, and prevents
before test results are available, because early treat- transmission to others. In advanced cases, scarring
ment decreases the possibility of ongoing transmis- can result, despite successful therapy.
sion and because successful treatment of genital
herpes depends on prompt initiation of therapy. The Azithromycin and ceftriaxone offer the advantage
clinician should empirically treat for the diagnosis of single-dose therapy. Worldwide, several isolates
considered most likely on the basis of clinical pre- with intermediate resistance to either ciprofloxacin
sentation and epidemiologic circumstances (includ- or erythromycin have been reported. However,
ing travel history); even after complete diagnostic because cultures are not routinely performed, data
evaluation, at least 25% of patients who have geni- are limited regarding the current prevalence of
tal ulcers have no laboratory-confirmed diagnosis. antimicrobial resistance.

Chancroid

The prevalence of chancroid has declined in the


United States (93). When infection does occur, it is
usually associated with sporadic outbreaks. World-
wide, chancroid appears to have declined as well,
although infection might still occur in some regions
of Africa and the Caribbean. Chancroid, as well as
genital herpes and syphilis, is a risk factor in the
transmission of HIV infection (144).
Other Management Considerations

Men who are uncircumcised and patients with HIV


A definitive diagnosis of chancroid requires the
infection do not respond as well to treatment as per-
identification of H. ducreyi on special culture media
sons who are circumcised or HIV-negative. Patients
that is not widely available from commercial
should be tested for HIV infection at the time chan-
sources; even when these media are used, sensitivity
croid is diagnosed. If the initial test results were
is <80% (145). No FDA-cleared PCR test for H.
negative, a serologic test for syphilis and HIV infec-
ducreyi is available in the United States, but such
tion should be performed 3 months after the diagno-
testing can be performed by clinical laboratories
sis of chancroid.
that have developed their own PCR test and have
conducted a CLIA verification study.

The combination of a painful genital ulcer and ten-


Follow-Up

Patients should be re-examined 3–7 days after initi-


der suppurative inguinal adenopathy suggests the
ation of therapy. If treatment is successful, ulcers
diagnosis of chancroid (146). A probable diagnosis
usually improve symptomatically within 3 days and
of chancroid, for both clinical and surveillance pur-
objectively within 7 days after therapy. If no clinical
poses, can be made if all of the following criteria are
improvement is evident, the clinician must consider
met: 1) the patient has one or more painful genital
whether 1) the diagnosis is correct, 2) the patient is
ulcers; 2) the patient has no evidence of T. pallidum
coinfected with another STD, 3) the patient is
infection by darkfield examination of ulcer exudate
infected with HIV, 4) the treatment was not used as
or by a serologic test for syphilis performed at least
instructed, or 5) the H. ducreyi strain causing the
7 days after onset of ulcers; 3) the clinical presenta-
infection is resistant to the prescribed antimicrobial.
tion, appearance of genital ulcers and, if present,
The time required for complete healing depends on
regional lymphadenopathy are typical for chan-
the size of the ulcer; large ulcers might require >2
croid; and 4) a test for HSV performed on the ulcer
weeks. In addition, healing is slower for some uncir-
exudate is negative.

732 EDUCATIONAL REVIEW MANUAL IN UROLOGY


cumcised men who have ulcers under the foreskin. Most persons infected with HSV-2 have not been
Clinical resolution of fluctuant lymphadenopathy is diagnosed with genital herpes. Many such persons
slower than that of ulcers and might require needle have mild or unrecognized infections but shed virus
aspiration or incision and drainage, despite other- intermittently in the genital tract. As a result, the
wise successful therapy. Although needle aspiration majority of genital herpes infections are transmitted
of buboes is a simpler procedure, incision and by persons unaware that they have the infection or
drainage might be preferred because of reduced who are asymptomatic when transmission occurs.
need for subsequent drainage procedures. Management of genital HSV should address the
chronic nature of the disease and go beyond the
Management of Sex Partners treatment of acute episodes of genital ulcers.

Regardless of whether symptoms of the disease are


present, sex partners of patients who have chancroid
Diagnosis of HSV Infection

should be examined and treated if they had sexual The clinical diagnosis of genital herpes is both non-
contact with the patient during the 10 days preced- sensitive and nonspecific. The classical painful
ing the patient’s onset of symptoms. multiple vesicular or ulcerative lesions are absent in
many infected persons. HSV-1 is causing an
increasing proportion of first episodes of anogenital
herpes in some populations (e.g., young women and
Special Considerations

Pregnancy MSM) and might now account for most of these


Ciprofloxacin is contraindicated during pregnancy infections (148,149). Recurrences and subclinical
and lactation. No adverse effects of chancroid on shedding are much less frequent for genital HSV-1
pregnancy outcome have been reported. infection than for genital HSV-2 infection
(150,151). A patient’s prognosis and the type of
HIV Infection counseling needed depends on the type of genital
HIV-infected patients who have chancroid should herpes (HSV-1 or HSV-2) causing the infection;
be monitored closely because, as a group, they are therefore, the clinical diagnosis of genital herpes
more likely to experience treatment failure and to should be confirmed by laboratory testing (152).
have ulcers that heal more slowly. HIV-infected Both virologic and type-specific serologic tests for
patients might require repeated or longer courses of HSV should be available in clinical settings that
therapy than those recommended for HIV-negative provide care for persons diagnosed with or at risk
patients, and treatment failures can occur with any for STDs.
regimen. Because data are limited concerning the
therapeutic efficacy of the recommended ceftriax-
one and azithromycin regimens in HIV-infected
Virologic Tests

patients, these regimens should be used for such Cell culture and PCR are the preferred HSV tests for
patients only if follow-up can be ensured. persons who seek medical treatment for genital
ulcers or other mucocutaneous lesions. The sensi-
tivity of viral culture is low, especially for recurrent
lesions, and declines rapidly as lesions begin to
Genital HSV Infections

Genital herpes is a chronic, life-long viral infection. heal. PCR assays for HSV DNA are more sensitive
Two types of HSV have been identified as causing and are increasingly used in many settings
genital herpes: HSV-1 and HSV-2. Most cases of (153,154). PCR is the test of choice for detecting
recurrent genital herpes are caused by HSV-2, and at HSV in spinal fluid for diagnosis of HSV infection
least 50 million persons in the United States are of the central nervous system (CNS). Viral culture
infected with this type of genital herpes (147). How- isolates should be typed to determine which type of
ever, an increasing proportion of anogenital herpetic HSV is causing the infection. Failure to detect HSV
infections in some populations has been attributed by culture or PCR does not indicate an absence of
to HSV-1 infection. HSV infection, because viral shedding is intermit-
tent. The use of cytologic detection of cellular

CHAPTER 23A: SEXUALLY TRANSMITTED DISEASES 733


changes of HSV infection is an insensitive and non- does not distinguish anogenital from orolabial or
specific method of diagnosis, both for genital cutaneous infection, and regardless of site of infec-
lesions (i.e., Tzanck preparation) and for cervical tion, these persons remain at risk for acquiring
Pap smears and therefore should not be relied upon. HSV-2.

Type-specific HSV serologic assays might be useful


in the following scenarios: 1) recurrent genital
Type-Specific Serologic Tests

Both type-specific and nontype-specific antibodies symptoms or atypical symptoms with negative HSV
to HSV develop during the first several weeks after cultures; 2) a clinical diagnosis of genital herpes
infection and persist indefinitely. Accurate type- without laboratory confirmation; or 3) a partner
specific HSV serologic assays are based on the with genital herpes. HSV serologic testing should
HSV-specific glycoprotein G2 (HSV-2) and glyco- be considered for persons presenting for an STD
protein G1 (HSV-1). Such assays first became com- evaluation (especially for those persons with multi-
mercially available in 1999, but older assays that do ple sex partners), persons with HIV infection, and
not accurately distinguish HSV-1 from HSV-2 anti- MSM at increased risk for HIV acquisition. Screen-
body (despite claims to the contrary) remain on the ing for HSV-1 and HSV-2 in the general population
market (155); providers should specifically request is not indicated.
serologic type-specific glycoprotein G (gG)-based
assays when serology is performed for their patients
(156–158).
Management of Genital Herpes

Antiviral chemotherapy offers clinical benefits to


Both laboratory-based assays and point-of-care most symptomatic patients and is the mainstay of
tests that provide results for HSV-2 antibodies from management. Counseling regarding the natural his-
capillary blood or serum during a clinic visit are tory of genital herpes, sexual and perinatal transmis-
available. The sensitivities of these glycoprotein G sion, and methods to reduce transmission is integral
type-specific tests for the detection of HSV-2 anti- to clinical management.
body vary from 80%–98%, and false-negative
results might be more frequent at early stages of Systemic antiviral drugs can partially control the
infection. The specificities of these assays are signs and symptoms of herpes episodes when used
≥96%. False-positive results can occur, especially in to treat first clinical and recurrent episodes, or when
patients with a low likelihood of HSV infection. used as daily suppressive therapy. However, these
Repeat or confirmatory testing might be indicated in drugs neither eradicate latent virus nor affect the
some settings, especially if recent acquisition of risk, frequency, or severity of recurrences after the
genital herpes is suspected. IgM testing for HSV is drug is discontinued. Randomized trials have indi-
not useful, because the IgM tests are not type-spe- cated that three antiviral medications provide clini-
cific and might be positive during recurrent cal benefit for genital herpes: acyclovir, valacy-
episodes of herpes (159). clovir, and famciclovir (160–168). Valacyclovir is
the valine ester of acyclovir and has enhanced
Because nearly all HSV-2 infections are sexually absorption after oral administration. Famciclovir
acquired, the presence of type-specific HSV-2 anti- also has high oral bioavailability. Topical therapy
body implies anogenital infection. In this instance, with antiviral drugs offers minimal clinical benefit,
education and counseling appropriate for persons and its use is discouraged.
with genital herpes should be provided. The pres-
ence of HSV-1 antibody alone is more difficult to
interpret. Most persons with HSV-1 antibody have
First Clinical Episode of Genital Herpes

oral HSV infection acquired during childhood, Newly acquired genital herpes can cause a pro-
which might be asymptomatic. However, acquisi- longed clinical illness with severe genital ulcera-
tion of genital HSV-1 appears to be increasing, and tions and neurologic involvement. Even persons
genital HSV-1 also can be asymptomatic (147–149). with first-episode herpes who have mild clinical
Lack of symptoms in an HSV-1 seropositive person manifestations initially can develop severe or pro-

734 EDUCATIONAL REVIEW MANUAL IN UROLOGY


longed symptoms. Therefore, all patients with first rences who receive suppressive therapy rather than
episodes of genital herpes should receive antiviral episodic treatment.
therapy.
The frequency of recurrent genital herpes outbreaks
diminishes over time in many patients, and the
patient’s psychological adjustment to the disease
might change. Therefore, periodically during sup-
pressive treatment (e.g., once a year), providers
should discuss the need to continue therapy with the
patient.

Treatment with valacyclovir 500 mg daily decreases


the rate of HSV-2 transmission in discordant, het-
erosexual couples in which the source partner has a
history of genital HSV-2 infection (170). Such cou-
ples should be encouraged to consider suppressive
antiviral therapy as part of a strategy to prevent
Established HSV-2 Infection

Almost all persons with symptomatic first-episode transmission, in addition to consistent condom use
genital HSV-2 infection subsequently experience and avoidance of sexual activity during recurrences.
recurrent episodes of genital lesions; recurrences Suppressive antiviral therapy also is likely to reduce
are less frequent after initial genital HSV-1 infec- transmission when used by persons who have multi-
tion. Intermittent asymptomatic shedding occurs in ple partners (including MSM) and by those who are
persons with genital HSV-2 infection, even in those HSV-2 seropositive without a history of genital her-
with longstanding or clinically silent infection. pes.
Antiviral therapy for recurrent genital herpes can
be administered either as suppressive therapy to Acyclovir, famciclovir, and valacyclovir appear
reduce the frequency of recurrences or episodically equally effective for episodic treatment of genital
to ameliorate or shorten the duration of lesions. herpes, but famciclovir appears somewhat less
Some persons, including those with mild or infre- effective for suppression of viral shedding
quent recurrent outbreaks, benefit from antiviral (163–167,173). Ease of administration and cost also
therapy; therefore, options for treatment should be are important considerations for prolonged treat-
discussed. Many persons might prefer suppressive ment.
therapy, which has the additional advantage of
decreasing the risk for genital HSV-2 transmission
to susceptible partners (169,170).

Suppressive Therapy for Recurrent


Genital Herpes

Suppressive therapy reduces the frequency of geni-


tal herpes recurrences by 70%–80% in patients
who have frequent recurrences (166–169); many
persons receiving such therapy report having expe-
rienced no symptomatic outbreaks. Treatment also
is effective in patients with less frequent recur-
rences. Safety and efficacy have been documented
among patients receiving daily therapy with acy-
clovir for as long as 6 years and with valacyclovir
or famciclovir for 1 year (171,172). Quality of life
is improved in many patients with frequent recur-

CHAPTER 23A: SEXUALLY TRANSMITTED DISEASES 735


sides. Multiple resources, including websites
(http://www.ashastd.org) and printed materials, are
Episodic Therapy for Recurrent

available to assist patients, their partners, and clini-


Genital Herpes

Effective episodic treatment of recurrent herpes cians who become involved in counseling.
requires initiation of therapy within 1 day of lesion
onset or during the prodrome that precedes some Although the psychological effect of a serologic
outbreaks. The patient should be provided with a diagnosis of HSV-2 infection in a person with
supply of drug or a prescription for the medication asymptomatic or unrecognized genital herpes
with instructions to initiate treatment immediately appears minimal and transient (176), some HSV-
when symptoms begin. infected persons might express anxiety concerning
genital herpes that does not reflect the actual clinical
severity of their disease; the psychological effect of
HSV infection frequently is substantial. Common
concerns regarding genital herpes include the sever-
ity of initial clinical manifestations, recurrent
episodes, sexual relationships and transmission to
sex partners, and ability to bear healthy children.
The misconception that HSV causes cancer should
be dispelled.

The following recommendations apply to counsel-


ing of persons with genital HSV infection:

• Persons who have genital herpes should be edu-


cated concerning the natural history of the disease,
with emphasis on the potential for recurrent
episodes, asymptomatic viral shedding, and the
attendant risks of sexual transmission.
Severe Disease

Intravenous (IV) acyclovir therapy should be pro-


vided for patients who have severe HSV disease or • Persons experiencing a first episode of genital her-
complications that necessitate hospitalization (e.g., pes should be advised that suppressive therapy is
disseminated infection, pneumonitis, or hepatitis) or available and effective in preventing symptomatic
CNS complications (e.g., meningoencephalitis). recurrent episodes and that episodic therapy often
The recommended regimen is acyclovir 5–10 mg/kg is useful in shortening the duration of recurrent
IV every 8 hours for 2–7 days or until clinical episodes.
improvement is observed, followed by oral antiviral
therapy to complete at least 10 days of total therapy. • All persons with genital HSV infection should be
Acyclovir dose adjustment is recommended for encouraged to inform their current sex partners
impaired renal function. that they have genital herpes and to inform future
partners before initiating a sexual relationship.

• Sexual transmission of HSV can occur during


Counseling

Counseling of infected persons and their sex part- asymptomatic periods. Asymptomatic viral shed-
ners is critical to the management of genital herpes. ding is more frequent in genital HSV-2 infection
The goals of counseling include 1) helping patients than genital HSV-1 infection and is most frequent
cope with the infection and 2) preventing sexual and during the first 12 months after acquiring HSV-2.
perinatal transmission (174,175). Although initial
counseling can be provided at the first visit, many
patients benefit from learning about the chronic
aspects of the disease after the acute illness sub-

736 EDUCATIONAL REVIEW MANUAL IN UROLOGY


• All persons with genital herpes should remain • When exposed to HIV, HSV-2 seropositive per-
abstinent from sexual activity with uninfected sons are at increased risk for HIV acquisition.
partners when lesions or prodromal symptoms are Patients should be informed that suppressive
present. antiviral therapy does not reduce the increased risk
for HIV acquisition associated with HSV-2 infec-
• The risk for HSV-2 sexual transmission can be tion (177,178).
decreased by the daily use of valacyclovir by the
infected person. Episodic therapy does not reduce
the risk for transmission and its use should be dis-
Management of Sex Partners

couraged for this purpose among persons whose The sex partners of patients who have genital herpes
partners might be at risk for HSV-2 acquisition. can benefit from evaluation and counseling. Symp-
tomatic sex partners should be evaluated and treated
• Infected persons should be informed that male in the same manner as patients who have genital
latex condoms, when used consistently and cor- lesions. Asymptomatic sex partners of patients who
rectly, might reduce the risk for genital herpes have genital herpes should be questioned concern-
transmission (21–23). ing histories of genital lesions and offered type-spe-
cific serologic testing for HSV infection.
• Sex partners of infected persons should be advised
that they might be infected even if they have no
symptoms. Type-specific serologic testing of the
Special Considerations

asymptomatic partners of persons with genital her-


pes is recommended to determine whether such
Allergy, Intolerance, and

partners are already HSV seropositive or whether


Adverse Reactions

risk for acquiring HSV exists. Allergic and other adverse reactions to acyclovir,
valacyclovir, and famciclovir are rare. Desensitiza-
• The risk for neonatal HSV infection should be tion to acyclovir has been described (179).
explained to all persons, including men. Pregnant
women and women of childbearing age who have
genital herpes should inform their providers who
HIV Infection

care for them during pregnancy and those who will Immunocompromised patients can have prolonged
care for their newborn infant about their infection. or severe episodes of genital, perianal, or oral her-
Pregnant women who are not known to be infected pes. Lesions caused by HSV are common among
with HSV-2 should be advised to abstain from HIV-infected patients and might be severe, painful,
intercourse with men who have genital herpes dur- and atypical. HSV shedding is increased in HIV-
ing the third trimester of pregnancy. Similarly, infected persons. Whereas antiretroviral therapy
pregnant women who are not known to be infected reduces the severity and frequency of symptomatic
with HSV-1 should be counseled to avoid genital genital herpes, frequent subclinical shedding still
exposure to HSV-1 during the third trimester (e.g., occurs (180). Clinical manifestations of genital her-
oral sex with a partner with oral herpes and vaginal pes might worsen during immune reconstitution
intercourse with a partner with genital HSV-1 after initiation of antiretroviral therapy.
infection).
Suppressive or episodic therapy with oral antiviral
• Asymptomatic persons diagnosed with HSV-2 agents is effective in decreasing the clinical mani-
infection by type-specific serologic testing should festations of HSV among HIV-positive persons
receive the same counseling messages as persons (181–183). The extent to which suppressive antivi-
with symptomatic infection. In addition, such per- ral therapy will decrease HSV transmission from
sons should be educated about the clinical mani- this population is unknown. HSV type-specific
festations of genital herpes. serologies can be offered to HIV-positive persons

CHAPTER 23A: SEXUALLY TRANSMITTED DISEASES 737


during their initial evaluation if infection status is daily suppressive antiviral therapy were less likely
unknown, and suppressive antiviral therapy can be to develop acyclovir-resistant HSV compared with
considered in those who have HSV-2 infection. those who received episodic therapy with outbreaks
(185).

Genital Herpes in Pregnancy

Most mothers of infants who acquire neonatal her-


pes lack histories of clinically evident genital herpes
(186). The risk for transmission to the neonate from
an infected mother is high (30%–50%) among
women who acquire genital herpes near the time of
delivery and low (<1%) among women with histo-
ries of recurrent herpes at term or who acquire geni-
tal HSV during the first half of pregnancy (187).
However, because recurrent genital herpes is much
more common than initial HSV infection during
pregnancy, the proportion of neonatal HSV infec-
tions acquired from mothers with recurrent herpes is
Acyclovir, valacyclovir, and famciclovir are safe for
substantial. Prevention of neonatal herpes depends
use in immunocompromised patients in the doses
both on preventing acquisition of genital HSV
recommended for treatment of genital herpes. For
infection during late pregnancy and avoiding expo-
severe HSV disease, initiating therapy with acy-
sure of the infant to herpetic lesions during delivery.
clovir 5–10 mg/kg IV every 8 hours might be neces-
Because the risk for herpes is high in infants of
sary.
women who acquire genital HSV during late preg-
nancy, these women should be managed in consulta-
If lesions persist or recur in a patient receiving
tion with an infectious disease specialist.
antiviral treatment, HSV resistance should be sus-
pected and a viral isolate should be obtained for sen-
Women without known genital herpes should be
sitivity testing (184). Such persons should be man-
counseled to abstain from intercourse during the
aged in consultation with an HIV specialist, and
third trimester with partners known or suspected of
alternate therapy should be administered. All acy-
having genital herpes. In addition, pregnant women
clovir-resistant strains are resistant to valacyclovir,
without known orolabial herpes should be advised
and the majority are resistant to famciclovir. Foscar-
to abstain from receptive oral sex during the third
net, 40 mg/kg IV every 8 hours until clinical resolu-
trimester with partners known or suspected to have
tion is attained, is frequently effective for treatment
orolabial herpes. Some specialists believe that type-
of acyclovir-resistant genital herpes. Intravenous
specific serologic tests are useful to identify preg-
cidofovir 5 mg/kg once weekly might also be effec-
nant women at risk for HSV infection and to guide
tive. Imiquimod is a topical alternative, as is topical
counseling regarding the risk for acquiring genital
cidofovir gel 1%, which is not commercially avail-
herpes during pregnancy and that such testing
able and must be compounded at a pharmacy. These
should be offered to uninfected women whose sex
topical preparations should be applied to the lesions
partner has HSV infection. However, the effective-
once daily for 5 consecutive days.
ness of antiviral therapy to decrease the risk for
HSV transmission to pregnant women by infected
Clinical management of antiviral resistance remains
partners has not been studied.
challenging among HIV-infected patients, and other
preventative approaches might be necessary. How-
All pregnant women should be asked whether they
ever, experience with another group of immuno-
have a history of genital herpes. At the onset of
compromised persons (hematopoietic stem-cell
labor, all women should be questioned carefully
recipients) demonstrated that persons receiving
about symptoms of genital herpes, including pro-

738 EDUCATIONAL REVIEW MANUAL IN UROLOGY


dromal symptoms, and all women should be exam- clovir. The recommended regimen for infants
ined carefully for herpetic lesions. Women without treated for known or suspected neonatal herpes is
symptoms or signs of genital herpes or its prodrome acyclovir 20 mg/kg IV every 8 hours for 21 days for
can deliver vaginally. Although cesarean section disseminated and CNS disease or for 14 days for
does not completely eliminate the risk for HSV disease limited to the skin and mucous membranes.
transmission to the infant, women with recurrent
genital herpetic lesions at the onset of labor should
deliver by cesarean section to prevent neonatal HSV
Granuloma Inguinale (Donovanosis)

infection. Granuloma inguinale is a genital ulcerative disease


caused by the intracellular gram-negative bacterium
The safety of systemic acyclovir, valacyclovir, and Klebsiella granulomatis (formerly known as
famciclovir therapy in pregnant women has not Calymmatobacterium granulomatis). The disease
been definitively established. Available data do not occurs rarely in the United States, although it is
indicate an increased risk for major birth defects endemic in some tropical and developing areas,
compared with the general population in women including India; Papua, New Guinea; the
treated with acyclovir during the first trimester Caribbean; central Australia; and southern Africa
(188) — findings that provide assurance to women (192,193). Clinically, the disease is commonly char-
who have had prenatal exposure to acyclovir. How- acterized as painless, slowly progressive ulcerative
ever, data regarding prenatal exposure to valacy- lesions on the genitals or perineum without regional
clovir and famciclovir are too limited to provide lymphadenopathy; subcutaneous granulomas (pseu-
useful information on pregnancy outcomes. Acy- doboboes) might also occur. The lesions are highly
clovir can be administered orally to pregnant vascular (i.e., beefy red appearance) and bleed eas-
women with first episode genital herpes or severe ily on contact. The clinical presentation also can
recurrent herpes and should be administered IV to include hypertrophic, necrotic, or sclerotic variants.
pregnant women with severe HSV infection. Acy- Extragenital infection can occur with extension of
clovir treatment late in pregnancy reduces the fre- infection to the pelvis, or it can disseminate to
quency of cesarean sections among women who intraabdominal organs, bones, or the mouth. The
have recurrent genital herpes by diminishing the lesions also can develop secondary bacterial infec-
frequency of recurrences at term (189–191); the tion and can coexist with other sexually transmitted
effect of antiviral therapy late in pregnancy on the pathogens.
incidence of neonatal herpes is not known. No data
support the use of antiviral therapy among HSV The causative organism is difficult to culture, and
seropositive women without a history of genital her- diagnosis requires visualization of dark-staining
pes. Donovan bodies on tissue crush preparation or
biopsy. No FDA-cleared molecular tests for the
detection of K. granulomatis DNA exist, but such an
assay might be useful when undertaken by laborato-
Neonatal Herpes

Infants exposed to HSV during birth, as docu- ries that have conducted a CLIA verification study.
mented by maternal virologic testing or presumed
by observation of maternal lesions, should be fol-
lowed carefully in consultation with a pediatric
Treatment

infectious disease specialist. Surveillance cultures Several antimicrobial regimens have been effective,
of mucosal surfaces to detect HSV infection might but only a limited number of controlled trials have
be considered before the development of clinical been published (192). Treatment has been shown to
signs of neonatal herpes. In addition, administration halt progression of lesions, and healing typically
of acyclovir might be considered for infants born to proceeds inward from the ulcer margins; prolonged
women who acquired HSV near term because the therapy is usually required to permit granulation
risk for neonatal herpes is high for these infants. All and reepithelialization of the ulcers. Relapse can
infants who have neonatal herpes should be occur 6–18 months after apparently effective ther-
promptly evaluated and treated with systemic acy- apy.

CHAPTER 23A: SEXUALLY TRANSMITTED DISEASES 739


HIV Infection

Persons with both granuloma inguinale and HIV


infection should receive the same regimens as those
who are HIV negative; however, the addition of a
parenteral aminoglycoside (e.g., gentamicin) can
also be considered.

Lymphogranuloma Venereum

Lymphogranuloma venereum (LGV) is caused by


C. trachomatis serovars L1, L2, or L3 (194). The
most common clinical manifestation of LGV among
heterosexuals is tender inguinal and/or femoral lym-
phadenopathy that is typically unilateral. A self-lim-
The addition of an aminoglycoside (e.g., gentamicin ited genital ulcer or papule sometimes occurs at the
1 mg/kg IV every 8 hours) to these regimens can be site of inoculation. However, by the time patients
considered if improvement is not evident within the seek care, the lesions have often disappeared. Rectal
first few days of therapy. exposure in women or MSM can result in proctocol-
itis, including mucoid and/or hemorrhagic rectal
discharge, anal pain, constipation, fever, and/or
tenesmus (195,196). LGV is an invasive, systemic
Follow-Up

Patients should be followed clinically until signs infection, and if it is not treated early, LGV procto-
and symptoms have resolved. colitis can lead to chronic, colorectal fistulas and
strictures. Genital and colorectal LGV lesions can
also develop secondary bacterial infection or can be
coinfected with other sexually and nonsexually
Management of Sex Partners

Persons who have had sexual contact with a patient transmitted pathogens.
who has granuloma inguinale within the 60 days
before onset of the patient’s symptoms should be Diagnosis is based on clinical suspicion, epidemio-
examined and offered therapy. However, the value logic information, and the exclusion of other etiolo-
of empiric therapy in the absence of clinical signs gies for proctocolitis, inguinal lymphadenopathy, or
and symptoms has not been established. genital or rectal ulcers. C. trachomatis testing also
should be conducted, if available.

Genital and lymph node specimens (i.e., lesion


Special Considerations

swab or bubo aspirate) can be tested for C. tra-


chomatis by culture, direct immunofluorescence, or
Pregnancy

Pregnancy is a relative contraindication to the use of nucleic acid detection. NAATs for C. trachomatis
sulfonamides. Pregnant and lactating women should are not FDA-cleared for testing rectal specimens,
be treated with the erythromycin regimen, and con- although some laboratories have performed the
sideration should be given to the addition of a par- CLIA validation studies that are needed to provide
enteral aminoglycoside (e.g., gentamicin). results for clinical management. Additional molecu-
Azithromycin might prove useful for treating granu- lar procedures (e.g., PCR-based genotyping) can be
loma inguinale during pregnancy, but published used to differentiate LGV from non-LGV C. tra-
data are lacking. Doxycycline and ciprofloxacin are chomatis, but these are not widely available.
contraindicated in pregnant women.
Chlamydia serology (complement fixation titers
>1:64) can support the diagnosis of LGV in the
appropriate clinical context. Comparative data

740 EDUCATIONAL REVIEW MANUAL IN UROLOGY


between types of serologic tests are lacking, and the gm orally single dose or doxycycline 100 mg orally
diagnostic utility of serologic methods other than twice a day for 7 days).
complement fixation and some microimmunofluo-
rescence procedures has not been established. Sero-
logic test interpretation for LGV is not standardized,
Special Considerations

tests have not been validated for clinical proctitis


presentations, and C. trachomatis serovar-specific
Pregnancy

serologic tests are not widely available. Pregnant and lactating women should be treated
with erythromycin. Azithromycin might prove use-
In the absence of specific LGV diagnostic testing, ful for treatment of LGV in pregnancy, but no pub-
patients with a clinical syndrome consistent with lished data are available regarding its safety and
LGV, including proctocolitis or genital ulcer disease efficacy. Doxycycline is contraindicated in pregnant
with lymphadenopathy, should be treated for LGV women.
as described in this report.
HIV Infection

Persons with both LGV and HIV infection should


Treatment

Treatment cures infection and prevents ongoing tis- receive the same regimens as those who are HIV
sue damage, although tissue reaction to the infection negative. Prolonged therapy might be required, and
can result in scarring. Buboes might require aspira- delay in resolution of symptoms might occur.
tion through intact skin or incision and drainage to
prevent the formation of inguinal/femoral ulcera-
tions. Doxycycline is the preferred treatment.
Syphilis

Syphilis is a systemic disease caused by Treponema


pallidum. On the basis of clinical findings, the dis-
ease has been divided into a series of overlapping
stages, which are used to help guide treatment and
follow-up. Persons who have syphilis might seek
treatment for signs or symptoms of primary infec-
tion (i.e., ulcer or chancre at the infection site), sec-
Although clinical data are lacking, azithromycin 1 g ondary infection (i.e., manifestations that include,
orally once weekly for 3 weeks is probably effective but are not limited to, skin rash, mucocutaneous
based on its chlamydial antimicrobial activity. Fluo- lesions, and lymphadenopathy), neurologic infec-
roquinolone-based treatments might also be effec- tion (i.e., cranial nerve dysfunction, meningitis,
tive, but extended treatment intervals are likely stroke, acute or chronic altered mental status, loss of
required. vibration sense, and auditory or ophthalmic abnor-
malities, which might occur through the natural his-
tory of untreated infection), or tertiary infection
(i.e., cardiac or gummatous lesions). Latent infec-
Follow-Up

Patients should be followed clinically until signs tions (i.e., those lacking clinical manifestations) are
and symptoms have resolved. detected by serologic testing. Latent syphilis
acquired within the preceding year is referred to as
early latent syphilis; all other cases of latent syphilis
are either late latent syphilis or latent syphilis of
Management of Sex Partners

Persons who have had sexual contact with a patient unknown duration. Treatment for both late latent
who has LGV within the 60 days before onset of the syphilis and tertiary syphilis might require a longer
patient’s symptoms should be examined, tested for duration of therapy because organisms might be
urethral or cervical chlamydial infection, and dividing more slowly; however, the validity of this
treated with a chlamydia regimen (azithromycin 1 concept has not been assessed.

CHAPTER 23A: SEXUALLY TRANSMITTED DISEASES 741


disease activity. However, 15%–25% of patients
treated during the primary stage revert to being
Diagnostic Considerations

Darkfield examinations and tests to detect T. pal- serologically nonreactive after 2–3 years (200). Tre-
lidum in lesion exudate or tissue are the definitive ponemal test antibody titers should not be used to
methods for diagnosing early syphilis (197). assess treatment response.
Although no T. pallidum detection tests are com-
mercially available, some laboratories provide Some clinical laboratories and blood banks have
locally developed PCR tests for the detection of T. begun to screen samples using treponemal tests,
pallidum. A presumptive diagnosis of syphilis is typically by EIA or chemiluminescence immunoas-
possible with the use of two types of serologic tests: says (201,202). This strategy will identify both per-
1) nontreponemal tests (e.g., Venereal Disease sons with previous treatment for syphilis and per-
Research Laboratory [VDRL] and RPR) and 2) tre- sons with untreated or incompletely treated syphilis.
ponemal tests (e.g., fluorescent treponemal anti- The positive predictive value for syphilis associated
body absorbed [FTA-ABS] tests, the T. pallidum with a treponemal screening test result might be
passive particle agglutination [TP-PA] assay, vari- lower among populations with a low prevalence of
ous EIAs, and chemiluminescence immunoassays). syphilis.
The use of only one type of serologic test is insuffi-
cient for diagnosis, because each type of test has Persons with a positive treponemal screening test
limitations, including the possibility of false-posi- should have a standard nontreponemal test with titer
tive test results in persons without syphilis. False- performed reflexively by the laboratory to guide
positive nontreponemal test results can be associ- patient management decisions. If the nontrepone-
ated with various medical conditions unrelated to mal test is negative, then the laboratory should per-
syphilis, including autoimmune conditions, older form a different treponemal test (preferably one
age, and injection-drug use (198,199); therefore, based on different antigens than the original test) to
persons with a reactive nontreponemal test should confirm the results of the initial test. If a second tre-
receive a treponemal test to confirm the diagnosis of ponemal test is positive, persons with a history of
syphilis. previous treatment will require no further manage-
ment unless sexual history suggests likelihood of re-
Nontreponemal test antibody titers may correlate exposure. Those without a history of treatment for
with disease activity, and results should be reported syphilis should be offered treatment. Unless history
quantitatively. A fourfold change in titer, equivalent or results of a physical examination suggest a recent
to a change of two dilutions (e.g., from 1:16 to 1:4 infection, previously untreated persons should be
or from 1:8 to 1:32), is considered necessary to treated for late latent syphilis. If the second trepone-
demonstrate a clinically significant difference mal test is negative, further evaluation or treatment
between two nontreponemal test results that were is not indicated.
obtained using the same serologic test. Sequential
serologic tests in individual patients should be per- For most HIV-infected persons, serologic tests are
formed using the same testing method (e.g., VDRL accurate and reliable for the diagnosis of syphilis
or RPR), preferably by the same laboratory. The and for following a patient’s response to treatment.
VDRL and RPR are equally valid assays, but quan- However, atypical syphilis serologic test results
titative results from the two tests cannot be com- (i.e., unusually high, unusually low, or fluctuating
pared directly because RPR titers frequently are titers) can occur in HIV-infected persons. When
slightly higher than VDRL titers. Nontreponemal serologic tests do not correspond with clinical find-
test titers usually decline after treatment and might ings suggestive of early syphilis, use of other tests
become nonreactive with time; however, in some (e.g., biopsy and darkfield microscopy) should be
persons, nontreponemal antibodies can persist for a considered.
long period of time — a response referred to as the
“serofast reaction.” Most patients who have reactive Clinical signs of neurosyphilis (i.e., cranial nerve
treponemal tests will have reactive tests for the dysfunction, meningitis, stroke, acute or chronic
remainder of their lives, regardless of treatment or altered mental status, loss of vibration sense, and

742 EDUCATIONAL REVIEW MANUAL IN UROLOGY


auditory or ophthalmic abnormalities) warrant fur- cillin (Bicillin C-R) instead of the standard benza-
ther investigation and treatment for neurosyphilis. thine penicillin product (Bicillin L-A) widely used
Laboratory testing is helpful in supporting the diag- in the United States. Practitioners, pharmacists, and
nosis of neurosyphilis; however, no single test can purchasing agents should be aware of the similar
be used to diagnose neurosyphilis in all instances. names of these two products to avoid using the inap-
Cerebrospinal fluid (CSF) laboratory abnormalities propriate combination therapy agent for treating
are common in persons with early syphilis. The syphilis (206).
VDRL in cerebrospinal fluid (CSF-VDRL), which
is highly specific but insensitive, is the standard The effectiveness of penicillin for the treatment of
serologic test for CSF. When reactive in the absence syphilis was well established through clinical expe-
of substantial contamination of CSF with blood, it is rience even before the value of randomized con-
considered diagnostic of neurosyphilis; however in trolled clinical trials was recognized. Therefore,
early syphilis, it can be of unknown prognostic sig- nearly all the recommendations for the treatment of
nificance (203). Most other tests are both insensitive syphilis are based not only on clinical trials and
and nonspecific and must be interpreted in relation observational studies, but approximately 50 years of
to other test results and the clinical assessment. clinical experience.
Therefore, the laboratory diagnosis of neurosyphilis
usually depends on various combinations of reac- Parenteral penicillin G is the only therapy with doc-
tive serologic test results, CSF cell count or protein, umented efficacy for syphilis during pregnancy.
and a reactive CSF-VDRL with or without clinical Pregnant women with syphilis in any stage who
manifestations. Among persons with HIV infection, report penicillin allergy should be desensitized and
the CSF leukocyte count usually is elevated (>5 treated with penicillin (see Management of Patients
white blood cell count [WBC]/mm3); using a higher Who Have a History of Penicillin Allergy).
cutoff (>20 WBC/ mm3) might improve the speci-
ficity of neurosyphilis diagnosis (204). The CSF- The Jarisch-Herxheimer reaction is an acute febrile
VDRL might be nonreactive even when neu- reaction frequently accompanied by headache,
rosyphilis is present; therefore, additional evalua- myalgia, fever, and other symptoms that usually
tion using FTA-ABS testing on CSF can be consid- occur within the first 24 hours after the initiation of
ered. The CSF FTA-ABS test is less specific for any therapy for syphilis. Patients should be
neurosyphilis than the CSF-VDRL but is highly informed about this possible adverse reaction. The
sensitive; neurosyphilis is highly unlikely with a Jarisch-Herxheimer reaction occurs most frequently
negative CSF FTA-ABS test (205). among patients who have early syphilis, presum-
ably because bacterial burdens are higher during
these stages. Antipyretics can be used to manage
symptoms, but they have not been proven to prevent
Treatment

Penicillin G, administered parenterally, is the pre- this reaction. The Jarisch-Herxheimer reaction
ferred drug for treating all stages of syphilis. The might induce early labor or cause fetal distress in
preparation used (i.e., benzathine, aqueous pro- pregnant women, but this should not prevent or
caine, or aqueous crystalline), the dosage, and the delay therapy (see Syphilis During Pregnancy).
length of treatment depend on the stage and clinical
manifestations of the disease. Selection of the
appropriate penicillin preparation is important,
Management of Sex Partners

because T. pallidum can reside in sequestered sites Sexual transmission of T. pallidum is thought to
(e.g., the CNS and aqueous humor) that are poorly occur only when mucocutaneous syphilitic lesions
accessed by some forms of penicillin. Combinations are present. Although such manifestations are
of benzathine penicillin, procaine penicillin, and uncommon after the first year of infection, persons
oral penicillin preparations are not considered exposed sexually to a patient who has syphilis in
appropriate for the treatment of syphilis. Reports any stage should be evaluated clinically and sero-
have indicated that practitioners have inadvertently logically and treated with a recommended regimen,
prescribed combination benzathine-procaine peni- according to the following recommendations:

CHAPTER 23A: SEXUALLY TRANSMITTED DISEASES 743


• Persons who were exposed within the 90 days pre-
ceding the diagnosis of primary, secondary, or
early latent syphilis in a sex partner might be
infected even if seronegative; therefore, such per-
sons should be treated presumptively.
Available data demonstrate that additional doses of
• Persons who were exposed >90 days before the benzathine penicillin G, amoxicillin, or other antibi-
diagnosis of primary, secondary, or early latent otics in early syphilis (primary, secondary, and early
syphilis in a sex partner should be treated pre- latent) do not enhance efficacy, regardless of HIV
sumptively if serologic test results are not avail- status.
able immediately and the opportunity for follow-
up is uncertain. Recommended Regimen for Infants and

• For purposes of partner notification and presump-


Children

tive treatment of exposed sex partners, patients Infants and children aged ≥1 month diagnosed with
with syphilis of unknown duration who have high syphilis should have a CSF examination to detect
nontreponemal serologic test titers (i.e., >1:32) asymptomatic neurosyphilis, and birth and maternal
can be assumed to have early syphilis. For the pur- medical records should be reviewed to assess
pose of determining a treatment regimen, however, whether such children have congenital or acquired
serologic titers should not be used to differentiate syphilis (see Congenital Syphilis). Children with
early from late latent syphilis (see Latent Syphilis, acquired primary or secondary syphilis should be
Treatment). evaluated (e.g., through consultation with child-pro-
tection services) (see Sexual Assault or Abuse of
• Long-term sex partners of patients who have latent Children) and treated by using the following pedi-
syphilis should be evaluated clinically and sero- atric regimen.
logically for syphilis and treated on the basis of the
evaluation findings.

Sexual partners of infected patients should be con-


sidered at risk and provided treatment if they have
had sexual contact with the patient within 3 months
Other Management Considerations

plus the duration of symptoms for patients diag- All persons who have syphilis should be tested for
nosed with primary syphilis, 6 months plus duration HIV infection. In geographic areas in which the
of symptoms for those with secondary syphilis, and prevalence of HIV is high, persons who have pri-
1 year for patients with early latent syphilis. mary syphilis should be retested for HIV after 3
months if the first HIV test result was negative.

Patients who have syphilis and symptoms or signs


Primary and Secondary Syphilis

suggesting neurologic disease (e.g., meningitis and


hearing loss) or ophthalmic disease (e.g., uveitis, iri-
Treatment

Parenteral penicillin G has been used effectively for tis, neuroretinitis, and optic neuritis) should have an
more than 50 years to achieve clinical resolution evaluation that includes CSF analysis, ocular slit-
(i.e., the healing of lesions and prevention of sexual lamp ophthalmologic examination, and otologic
transmission) and to prevent late sequelae. How- examination. Treatment should be guided by the
ever, no comparative trials have been adequately results of this evaluation.
conducted to guide the selection of an optimal peni-
cillin regimen (i.e., the dose, duration, and prepara- Invasion of CSF by T. pallidum accompanied by
tion). Substantially fewer data are available for non- CSF laboratory abnormalities is common among
penicillin regimens. adults who have primary or secondary syphilis
(203). Therefore, in the absence of clinical neuro-

744 EDUCATIONAL REVIEW MANUAL IN UROLOGY


logic findings, no evidence exists to support varia- unrecognized CNS infection, CSF examination can
tion from the recommended treatment regimen for be considered in such situations.
early syphilis. Symptomatic neurosyphilis develops
in only a limited number of persons after treatment For retreatment, weekly injections of benzathine
with the penicillin regimens recommended for pri- penicillin G 2.4 million units IM for 3 weeks is rec-
mary and secondary syphilis. Therefore, unless clin- ommended, unless CSF examination indicates that
ical signs or symptoms of neurologic or ophthalmic neurosyphilis is present (see Neurosyphilis). In rare
involvement are present or treatment failure is doc- instances, serologic titers do not decline despite a
umented, routine CSF analysis is not recommended negative CSF examination and a repeated course of
for persons who have primary or secondary syphilis. therapy. In these circumstances, the need for addi-
tional therapy or repeated CSF examinations is
Follow-Up unclear, but is not generally recommended.

Treatment failure can occur with any regimen.


However, assessing response to treatment fre-
Management of Sex Partners

quently is difficult, and definitive criteria for cure or See General Principles, Management of Sex Part-
failure have not been established. In addition, non- ners.
treponemal test titers might decline more slowly for
persons who previously have had syphilis (207).
Clinical and serologic evaluation should be per-
Special Considerations

formed 6 months and 12 months after treatment;


more frequent evaluation might be prudent if fol-
Penicillin Allergy

low-up is uncertain. Data to support the use of alternatives to penicillin


in the treatment of early syphilis are limited. How-
Patients who have signs or symptoms that persist or ever, several therapies might be effective in non-
recur or who have a sustained fourfold increase in pregnant, penicillin-allergic patients who have pri-
nontreponemal test titer (i.e., compared with the mary or secondary syphilis. Doxycycline 100 mg
maximum or baseline titer at the time of treatment) orally twice daily for 14 days (209,210) and tetracy-
probably failed treatment or were reinfected. These cline (500 mg four times daily for 14 days) are regi-
patients should be retreated and reevaluated for HIV mens that have been used for many years. Compli-
infection. Because treatment failure usually cannot ance is likely to be better with doxycycline than
be reliably distinguished from reinfection with T. tetracycline, because tetracycline can cause gas-
pallidum, a CSF analysis also should be performed. trointestinal side effects. Although limited clinical
studies, along with biologic and pharmacologic evi-
Although failure of nontreponemal test titers to dence, suggest that ceftriaxone (1 g daily either IM
decline fourfold within 6–12 months after therapy or IV for 10–14 days) is effective for treating early
for primary or secondary syphilis might be indica- syphilis, the optimal dose and duration of ceftriax-
tive of treatment failure, clinical trial data have one therapy have not been defined (211).
demonstrated that >15% of patients with early Azithromycin as a single 2-g oral dose is effective
syphilis treated with the recommended therapy will for treating early syphilis (212–214). However, T.
not achieve the two dilution decline in nontrepone- pallidum chromosomal mutations associated with
mal titer used to define response at 1 year after treat- azithromycin resistance and treatment failures have
ment (208). Persons whose titers do not decline been documented in several geographical areas in
should be reevaluated for HIV infection. Optimal the United States (215–217). As such, the use of
management of such patients is unclear. At a mini- azithromycin should be used with caution only
mum, these patients should receive additional clini- when treatment with penicillin or doxycycline is not
cal and serologic follow-up. If additional follow-up feasible. Azithromycin should not be used in MSM
cannot be ensured, retreatment is recommended. or pregnant women. Close follow-up of persons
Because treatment failure might be the result of receiving any alternative therapies is essential.

CHAPTER 23A: SEXUALLY TRANSMITTED DISEASES 745


Persons with a penicillin allergy whose compliance who have syphilis should be tested for HIV infec-
with therapy or follow-up cannot be ensured should tion.
be desensitized and treated with benzathine peni-
cillin. Skin testing for penicillin allergy might be
useful in some circumstances in which the reagents
Treatment

and expertise are available to perform the test ade- Because latent syphilis is not transmitted sexually,
quately (see Management of Patients Who Have a the objective of treating patients with this stage of
History of Penicillin Allergy). disease is to prevent complications. Although clini-
cal experience supports the effectiveness of peni-
cillin in achieving this goal, limited evidence is
available to guide choice of specific regimens.
Pregnancy

Pregnant patients who are allergic to penicillin


should be desensitized and treated with penicillin The following regimens are recommended for peni-
(see Management of Patients Who Have a History cillin nonallergic patients who have normal CSF
of Penicillin Allergy and Syphilis During Preg- examinations (if performed).
nancy).

HIV Infection

See Syphilis Among HIV-Infected Persons.

Latent Syphilis

Latent syphilis is defined as syphilis characterized


by seroreactivity without other evidence of disease.
Available data demonstrate no enhanced efficacy of
Patients who have latent syphilis and who acquired
additional doses of penicillin G, amoxicillin, or
syphilis during the preceding year are classified as
other antibiotics in early syphilis, regardless of HIV
having early latent syphilis. Patients’ conditions can
status.
be diagnosed as early latent syphilis if, during the
year preceding the evaluation, they had 1) a docu-
Infants and children aged ≥1 month who have been
mented seroconversion or fourfold or greater
diagnosed with syphilis should have a CSF exami-
increase in titer of a nontreponemal test; 2) unequiv-
nation to exclude neurosyphilis. In addition, birth
ocal symptoms of primary or secondary syphilis; or
and maternal medical records should be reviewed to
3) a sex partner documented to have primary, sec-
assess whether children have congenital or acquired
ondary, or early latent syphilis. In addition, for per-
syphilis (see Congenital Syphilis). Older children
sons whose only possible exposure occurred during
with acquired latent syphilis should be evaluated as
the previous 12 months, reactive nontreponemal
described for adults and treated using the following
and treponemal tests are indicative of early latent
pediatric regimens (see Sexual Assault or Abuse of
syphilis. In the absence of these conditions, an
Children). These regimens are for penicillin nonal-
asymptomatic person should be considered to have
lergic children who have acquired syphilis and who
late latent syphilis or syphilis of unknown duration.
have normal CSF examination results.
Nontreponemal serologic titers usually are higher
during early latent syphilis than late latent syphilis.
However, early latent syphilis cannot be reliably
distinguished from late latent syphilis solely on the
basis of nontreponemal titers. All patients with
latent syphilis should have careful examination of
all accessible mucosal surfaces (i.e., the oral cavity,
perianal area, perineum and vagina in women, and
underneath the foreskin in uncircumcised men) to
evaluate for internal mucosal lesions. All patients

746 EDUCATIONAL REVIEW MANUAL IN UROLOGY


Other Management Considerations Special Considerations

Patients diagnosed with latent syphilis who demon-


strate any of the following criteria should have a
Penicillin Allergy

prompt CSF examination: The effectiveness of alternatives to penicillin in the


treatment of latent syphilis has not been well docu-
• Neurologic (e.g., auditory disease, cranial nerve mented. Nonpregnant patients allergic to penicillin
dysfunction, acute or chronic meningitis, stroke, who have clearly defined early latent syphilis
acute or chronic altered mental status, and loss of should respond to therapies recommended as alter-
vibration sense) or ophthalmic signs or symptoms natives to penicillin for the treatment of primary and
(e.g., iritis and uveitis); secondary syphilis (see Primary and Secondary
Syphilis, Treatment). The only acceptable alterna-
• evidence of active tertiary syphilis (e.g., aortitis tives for the treatment of late latent syphilis or latent
and gumma); or syphilis of unknown duration are doxycycline (100
mg orally twice daily) or tetracycline (500 mg orally
• serologic treatment failure. four times daily), both for 28 days. These therapies
should be used only in conjunction with close sero-
If a patient misses a dose of penicillin in a course of logic and clinical follow-up. Based on biologic
weekly therapy for late syphilis, the appropriate plausibility and pharmacologic properties, ceftriax-
course of action is unclear. Pharmacologic consider- one might be effective for treating late latent
ations suggest that an interval of 10–14 days syphilis or syphilis of unknown duration. However,
between doses of benzathine penicillin for late the optimal dose and duration of ceftriaxone therapy
syphilis or latent syphilis of unknown duration have not been defined, and treatment decisions
might be acceptable before restarting the sequence should be discussed in consultation with a special-
of injections. Missed doses are not acceptable for ist. Some patients who are allergic to penicillin also
pregnant patients receiving therapy for late latent might be allergic to ceftriaxone; in these circum-
syphilis. Pregnant women who miss any dose of stances, use of an alternative agent might be
therapy must repeat the full course of therapy. required. The efficacy of these alternative regimens
in HIV-infected persons has not been well studied.
Follow-Up

Quantitative nontreponemal serologic tests should


Pregnancy

be repeated at 6, 12, and 24 months. A CSF exami- Pregnant patients who are allergic to penicillin
nation should be performed if 1) titers increase four- should be desensitized and treated with penicillin
fold, 2) an initially high titer (≥1:32) fails to decline (see Management of Patients Who Have a History
at least fourfold (i.e., two dilutions) within 12–24 of Penicillin Allergy and Syphilis During Preg-
months of therapy, or 3) signs or symptoms nancy).
attributable to syphilis develop. In such circum-
stances, even if the CSF examination is negative,
retreatment for latent syphilis should be initiated. In
HIV Infection

rare instances, despite a negative CSF examination See Syphilis Among HIV-Infected Persons.
and a repeated course of therapy, serologic titers
might fail to decline. In these circumstances, the
need for additional therapy or repeated CSF exami-
Tertiary Syphilis

nations is unclear. Tertiary syphilis refers to gumma and cardiovascu-


lar syphilis but not to all neurosyphilis. Patients who
are not allergic to penicillin and have no evidence of
neurosyphilis should be treated with the following
Management of Sex Partners

See General Principles, Management of Sex Part- regimen.


ners.

CHAPTER 23A: SEXUALLY TRANSMITTED DISEASES 747


Neurosyphilis

Treatment

CNS involvement can occur during any stage of


syphilis. However, CSF laboratory abnormalities
Other Management Considerations

Patients who have symptomatic late syphilis should are common in persons with early syphilis, even in
be given a CSF examination before therapy is initi- the absence of clinical neurological findings. No
ated. Some providers treat all patients who have car- evidence exists to support variation from recom-
diovascular syphilis with a neurosyphilis regimen. mended treatment for early syphilis for patients
These patients should be managed in consultation found to have such abnormalities. If clinical evi-
with an infectious disease specialist. dence of neurologic involvement is observed (e.g.,
cognitive dysfunction, motor or sensory deficits,
ophthalmic or auditory symptoms, cranial nerve
palsies, and symptoms or signs of meningitis), a
Follow-Up

Limited information is available concerning clinical CSF examination should be performed.


response and follow-up of patients who have ter-
tiary syphilis. Syphilitic uveitis or other ocular manifestations fre-
quently are associated with neurosyphilis and
should be managed according to the treatment rec-
ommendations for neurosyphilis. Patients who have
Management of Sex Partners

See General Principles, Management of Sex Part- neurosyphilis or syphilitic eye disease (e.g., uveitis,
ners. neuroretinitis, and optic neuritis) should be treated
with the recommended regimen for neurosyphilis;
those with eye disease should be managed in collab-
oration with an ophthalmologist. A CSF examina-
Special Considerations

tion should be performed for all patients with


syphilitic eye disease to identify those with abnor-
Penicillin Allergy

Patients allergic to penicillin should be treated in malities; patients found to have abnormal CSF test
consultation with an infectious disease specialist. results should be provided follow-up CSF examina-
tions to assess treatment response.
Pregnancy

Pregnant patients who are allergic to penicillin


should be desensitized and treated with penicillin
(see Management of Patients Who Have a History
of Penicillin Allergy and Syphilis During Preg- The durations of the recommended and alternative
nancy). regimens for neurosyphilis are shorter than the dura-
tion of the regimen used for late syphilis in the
absence of neurosyphilis. Therefore, benzathine
penicillin, 2.4 million units IM once per week for up
HIV Infection

See Syphilis Among HIV-Infected Persons. to 3 weeks, can be considered after completion of
these neurosyphilis treatment regimens to provide a
comparable total duration of therapy.

748 EDUCATIONAL REVIEW MANUAL IN UROLOGY


patient with neurosyphilis, skin testing should be
performed (if available) to confirm penicillin
Other Management Considerations

Other considerations in the management of patients allergy and, if necessary, desensitization in consul-
who have neurosyphilis are as follows: tation with a specialist.

• All persons who have syphilis should be tested for


HIV.
Pregnancy

Pregnant patients who are allergic to penicillin


• Although systemic steroids are used frequently as should be desensitized and treated with penicillin
adjunctive therapy for otologic syphilis, such (see Syphilis During Pregnancy).
drugs have not been proven to be beneficial.
HIV Infection

See Syphilis Among HIV-Infected Persons.


Follow-Up

If CSF pleocytosis was present initially, a CSF


examination should be repeated every 6 months
until the cell count is normal. Follow-up CSF exam-
Syphilis Among HIV-Infected Persons

inations also can be used to evaluate changes in the


CSF-VDRL or CSF protein after therapy; however,
Diagnostic Considerations

changes in these two parameters occur more slowly Although they are uncommon, unusual serologic
than cell counts, and persistent abnormalities might responses have been observed among HIV-infected
be less important (219,220). The leukocyte count is persons who have syphilis. Most reports have
a sensitive measure of the effectiveness of therapy. involved serologic titers that were higher than
If the cell count has not decreased after 6 months or expected, but false-negative serologic test results
if the CSF cell count or protein is not normal after 2 and delayed appearance of seroreactivity also have
years, retreatment should be considered. been reported (223). Regardless, both treponemal
and nontreponemal serologic tests for syphilis can
Limited data suggest that in immunocompetent per- be interpreted in the usual manner for most patients
sons and HIV-infected persons on highly active who are coinfected with T. pallidum and HIV.
antiretroviral therapy, normalization of the serum
RPR titer predicts normalization of CSF parameters When clinical findings are suggestive of syphilis
(220). but serologic tests are nonreactive or their interpre-
tation is unclear, alternative tests (e.g., biopsy of a
lesion, darkfield examination, and PCR of lesion
material) might be useful for diagnosis. Neu-
Management of Sex Partners

See General Principles, Management of Sex Part- rosyphilis should be considered in the differential
ners. diagnosis of neurologic disease in HIV-infected per-
sons.
Special Considerations
Treatment

Compared with HIV-negative patients, HIV-posi-


Penicillin Allergy

Limited data suggest that ceftriaxone 2 g daily tive patients who have early syphilis might be at
either IM or IV for 10–14 days can be used as an increased risk for neurologic complications (224)
alternative treatment for patients with neurosyphilis and might have higher rates of serologic treatment
(221,222). However, the possibility of cross-reac- failure with currently recommended regimens. The
tivity between ceftriaxone and penicillin exists. magnitude of these risks is not defined precisely, but
Other regimens have not been adequately evaluated is likely small. No treatment regimens for syphilis
for treatment of neurosyphilis. Therefore, if concern have been demonstrated to be more effective in pre-
exists regarding the safety of ceftriaxone for a venting neurosyphilis in HIV-infected patients than

CHAPTER 23A: SEXUALLY TRANSMITTED DISEASES 749


the syphilis regimens recommended for HIV-nega- HIV-infected persons who meet the criteria for treat-
tive patients (208). Careful follow-up after therapy ment failure (i.e., signs or symptoms that persist or
is essential. recur or persons who have a sustained fourfold
increase in nontreponemal test titer) should be man-
aged in the same manner as HIV-negative patients
(i.e., a CSF examination and retreatment). CSF
Primary and Secondary Syphilis Among

examination and retreatment also should be strongly


HIV-Infected Persons

considered for persons whose nontreponemal test


titers do not decrease fourfold within 6–12 months of
Treatment

Treatment of primary and secondary syphilis among therapy. If CSF examination is normal, treatment
HIV-infected persons is benzathine penicillin G, 2.4 with benzathine penicillin G administered as 2.4 mil-
million units IM in a single dose. lion units IM each at weekly intervals for 3 weeks is
recommended.
Available data demonstrate that additional doses of
benzathine penicillin G, amoxicillin, or other antibi-
otics in early syphilis do not result in enhanced effi-
Management of Sex Partners

cacy, regardless of HIV status (208). See General Principles, Management of Sex Part-
ners.
Other Management Considerations

Most HIV-infected persons respond appropriately to


Special Considerations

standard benzathine penicillin for primary and sec- Penicillin Allergy. HIV-infected, penicillin-allergic
ondary syphilis. CSF abnormalities (e.g., mononu- patients who have primary or secondary syphilis
clear pleocytosis and elevated protein levels) are should be managed according to the recommenda-
common in HIV-infected persons, even in those tions for penicillin-allergic, HIV-negative patients.
without neurologic symptoms, although the clinical Patients with penicillin allergy whose compliance
and prognostic significance of such CSF abnormali- with therapy or follow-up cannot be ensured should
ties with primary and secondary syphilis is be desensitized and treated with penicillin (see Man-
unknown. Several studies have demonstrated that agement of Patients Who Have a History of Peni-
among persons infected with both HIV and syphilis, cillin Allergy). The use of alternatives to penicillin
clinical and CSF abnormalities consistent with neu- has not been well studied in HIV-infected patients.
rosyphilis are associated with a CD4 count of ≤350 These therapies should be used only in conjunction
cells/mL and/or an RPR titer of ≥1:32 with close serologic and clinical follow-up.
(204,225,226); however, unless neurologic symp-
toms are present, CSF examination in this setting
has not been associated with improved clinical out-
Latent Syphilis Among HIV-Infected Persons

comes. Treatment

The use of antiretroviral therapy as per current HIV-infected persons with latent syphilis should be
guidelines might improve clinical outcomes in HIV- treated according to the stage-specific recommenda-
infected persons with syphilis (220,227,228). tions for HIV-negative persons.

• Treatment of early latent syphilis among HIV-


infected persons is benzathine penicillin G, 2.4 mil-
Follow-Up

HIV-infected persons should be evaluated clinically lion units IM in a single dose.


and serologically for treatment failure at 3, 6, 9, 12,
and 24 months after therapy. • Treatment of late latent syphilis or syphilis of
unknown duration among HIV-infected persons is
benzathine penicillin G, at weekly doses of 2.4 mil-
lion units for 3 weeks.

750 EDUCATIONAL REVIEW MANUAL IN UROLOGY


Other Management Considerations Neurosyphilis Among HIV-Infected Persons

All HIV-infected persons with syphilis and neuro-


logic symptoms should undergo immediate CSF
Treatment

examination. Some studies have demonstrated that HIV-infected patients with neurosyphilis should be
clinical and CSF abnormalities consistent with neu- treated according to the recommendations for HIV-
rosyphilis are most likely in HIV-infected persons negative patients with neurosyphilis (see Neu-
who have been diagnosed with syphilis and have a rosyphilis).
CD4 count of ≤350 cells/ml and/or an RPR titer of
≥1:32 (204,225,226); however unless neurologic
symptoms are present, CSF examination in this set-
Follow-Up

ting has not been associated with improved clinical If CSF pleocytosis was present initially, a CSF
outcomes. examination should be repeated every 6 months
until the cell count is normal. Follow-up CSF exam-
inations also can be used to gauge response after
therapy. Limited data suggest that changes in CSF
Follow-Up

Patients should be evaluated clinically and serologi- parameters might occur more slowly in HIV-
cally at 6, 12, 18, and 24 months after therapy. If, at infected patients, especially those with more
any time, clinical symptoms develop or nontrepone- advanced immunosuppression (219,227). If the cell
mal titers rise fourfold, a repeat CSF examination count has not decreased after 6 months or if the CSF
should be performed and treatment administered is not normal after 2 years, retreatment should be
accordingly. If during 12–24 months the nontre- considered.
ponemal titer does not decline fourfold, CSF exami-
nation should be strongly considered and treatment
administered accordingly.
Management of Sex Partners

See General Principles, Management of Sex Part-


Management of Sex Partners ners.

See General Principles, Management of Sex Part-


ners.
Special Considerations

Penicillin Allergy. HIV-infected, penicillin-allergic


patients who have neurosyphilis should be managed
according to the recommendations for penicillin-
Special Considerations

Penicillin Allergy. The efficacy of alternative non- allergic, HIV-negative patients with neurosyphilis.
penicillin regimens in HIV-infected persons has not Several small observational studies conducted in
been well studied. Patients with penicillin allergy HIV-infected patients with neurosyphilis suggest
whose compliance with therapy or follow-up cannot that ceftriaxone 1–2 g IV daily for 10-14 days might
be ensured should be desensitized and treated with be effective as an alternate agent (218,229,230).
penicillin (see Management of Patients Who Have a
History of Penicillin Allergy). These therapies
should be used only in conjunction with close sero-
Syphilis During Pregnancy

logic and clinical follow-up. Limited clinical stud- All women should be screened serologically for
ies, along with biologic and pharmacologic evi- syphilis early in pregnancy. Most states mandate
dence, suggest that ceftriaxone might be effective screening at the first prenatal visit for all women
(229,230). However, the optimal dose and duration (231); antepartum screening by nontreponemal anti-
of ceftriaxone therapy have not been defined. body testing is typical, but in some settings, tre-
ponemal antibody testing is being used. Pregnant
women with reactive treponemal screening tests
should have confirmatory testing with nontrepone-
mal tests with titers. In populations in which use of

CHAPTER 23A: SEXUALLY TRANSMITTED DISEASES 751


prenatal care is not optimal, RPR test screening and greater risk for fetal treatment failure (231); such
treatment (if the RPR test is reactive) should be per- cases should be managed in consultation with
formed at the time that pregnancy is confirmed obstetric specialists. Evidence is insufficient to rec-
(232). For communities and populations in which ommend specific regimens for these situations.
the prevalence of syphilis is high and for patients at
high risk, serologic testing should be performed Women treated for syphilis during the second half
twice during the third trimester (ideally at 28–32 of pregnancy are at risk for premature labor and/or
weeks’ gestation) and at delivery. Any woman who fetal distress if the treatment precipitates the
delivers a stillborn infant after 20 weeks’ gestation Jarisch-Herxheimer reaction (236). These women
should be tested for syphilis. No infant should leave should be advised to seek obstetric attention after
the hospital without the maternal serologic status treatment if they notice any fever, contractions, or
having been determined at least once during preg- decrease in fetal movements. Stillbirth is a rare
nancy. complication of treatment, but concern for this com-
plication should not delay necessary treatment. All
patients who have syphilis should be offered testing
for HIV infection.
Diagnostic Considerations

Seropositive pregnant women should be considered


infected unless an adequate treatment history is doc-
umented clearly in the medical records and sequen-
Follow-Up

tial serologic antibody titers have declined. Serofast Coordinated prenatal care and treatment are vital.
low antibody titers might not require treatment; Serologic titers should be repeated at 28–32 weeks’
however, persistent higher titer antibody tests might gestation and at delivery as recommended for the
indicate reinfection, and treatment might be disease stage. Providers should ensure that the clini-
required. cal and antibody responses are appropriate for the
patient’s stage of disease, although most women
will deliver before their serologic response to treat-
ment can be assessed definitively. Inadequate
Treatment

Penicillin is effective for preventing maternal trans- maternal treatment is likely if delivery occurs
mission to the fetus and for treating fetal infection within 30 days of therapy, if clinical signs of infec-
(233). Evidence is insufficient to determine optimal, tion are present at delivery, or if the maternal anti-
recommended penicillin regimens (234). body titer at delivery is fourfold higher than the pre-
treatment titer. Serologic titers can be checked
monthly in women at high risk for reinfection or in
geographic areas in which the prevalence of syphilis
is high

Other Management Considerations Management of Sex Partners

Some evidence suggests that additional therapy can See General Principles, Management of Sex Part-
be beneficial for pregnant women in some settings ners.
(e.g., a second dose of benzathine penicillin 2.4 mil-
lion units IM administered 1 week after the initial
dose for women who have primary, secondary, or
Special Considerations

early latent syphilis) (235). When syphilis is diag-


nosed during the second half of pregnancy, manage-
Penicillin Allergy

ment should include a sonographic fetal evaluation For treatment of syphilis during pregnancy, no
for congenital syphilis, but this evaluation should proven alternatives to penicillin exist. Pregnant
not delay therapy. Sonographic signs of fetal or pla- women who have a history of penicillin allergy
cental syphilis (i.e., hepatomegaly, ascites, hydrops, should be desensitized and treated with penicillin.
fetal anemia, or a thickened placenta) indicate a Oral step-wise penicillin dose challenge or skin test-

752 EDUCATIONAL REVIEW MANUAL IN UROLOGY


ing might be helpful in identifying women at risk for
acute allergic reactions (see Management of
B. Penicillin Allergy

Patients Who Have a History of Penicillin Allergy). Management of Persons Who Have a

Tetracycline and doxycycline usually are not used


History of Penicillin Allergy

during pregnancy. Erythromycin and azithromycin No proven alternatives to penicillin are available for
should not be used, because neither reliably cures treating neurosyphilis, congenital syphilis, or
maternal infection or treats an infected fetus (234). syphilis in pregnant women. Penicillin also is rec-
Data are insufficient to recommend ceftriaxone for ommended for use, whenever possible, in HIV-
treatment of maternal infection and prevention of infected patients. Of the adult U.S. population,
congenital syphilis. 3%–10% have experienced an immunoglobulin E
(IgE)-mediated allergic response to penicillin
(238,239), such as urticaria, angioedema, or ana-
phylaxis (i.e., upper airway obstruction, bron-
HIV Infection

Placental inflammation from congenital infection chospasm, or hypotension). Readministration of


might increase the risk for perinatal transmission of penicillin to these patients can cause severe, imme-
HIV. All HIV-infected women should be evaluated diate reactions. Because anaphylactic reactions to
for syphilis and receive treatment as recommended. penicillin can be fatal, every effort should be made
Data are insufficient to recommend a specific regi- to avoid administering penicillin to penicillin-aller-
men for HIV-infected pregnant women (see gic patients, unless they undergo acute desensitiza-
Syphilis Among HIV-Infected Patients). tion to eliminate anaphylactic sensitivity.

Although an estimated 10% of persons who report a


history of severe allergic reactions to penicillin con-
tinue to remain allergic their entire lives, with the
passage of time, most persons who have had a
severe reaction to penicillin stop expressing peni-
cillin-specific IgE (238,239). These persons can
then be treated safely with penicillin. Penicillin skin
testing with the major and minor determinants of
penicillin can reliably identify persons at high risk
for penicillin reactions (238,239). Although these
reagents are easily generated and have been avail-
able for more than 30 years, only benzylpenicilloyl
poly-L-lysine (Pre-Pen [i.e., the major determi-
nant]) and penicillin G have been available com-
mercially. These two tests identify an estimated
90%–97% of the currently allergic patients. How-
ever, because skin testing without the minor deter-
minants would still miss 3%–10% of allergic
patients and because serious or fatal reactions can
occur among these minor-determinant–positive
patients, caution should be exercised when the full
battery of skin-test reagents is not available (Box 2).
Manufacturers are working to ensure better avail-
ability of the Pre-Pen skin test reagent as well as an
accompanying minor determinant mixture.

CHAPTER 23A: SEXUALLY TRANSMITTED DISEASES 753


Recommendations Procedures

If the full battery of skin-test reagents is available, Dilute the antigens either 100-fold for preliminary
including both major and minor determinants (see testing (if the patient has had a life-threatening reac-
Penicillin Allergy Skin Testing), patients who report tion to penicillin) or 10-fold (if the patient has had
a history of penicillin reaction and who are skin-test another type of immediate, generalized reaction to
negative can receive conventional penicillin ther- penicillin within the preceding year).
apy. Skin-test–positive patients should be desensi-
tized before initiating treatment. Epicutaneous (Prick) Tests

If the full battery of skin-test reagents, including the Duplicate drops of skin-test reagent are placed on
minor determinants, is not available, the patient the volar surface of the forearm. The underlying
should be skin tested using benzylpenicilloyl poly- epidermis is pierced with a 26-gauge needle without
L-lysine (i.e., the major determinant) and penicillin drawing blood. An epicutaneous test is positive if
G. Patients who have positive test results should be the average wheal diameter after 15 minutes is ≥4
desensitized. One approach suggests that persons mm larger than that of negative controls; otherwise,
with a history of allergy who have negative test the test is negative. The histamine controls should
results should be regarded as possibly allergic and be positive to ensure that results are not falsely neg-
desensitized. Another approach in those with nega- ative because of the effect of antihistaminic drugs.
tive skin-test results involves test-dosing gradually
with oral penicillin in a monitored setting in which
treatment for anaphylactic reaction can be provided.
Intradermal Test

If epicutaneous tests are negative, duplicate 0.02-


If the major determinant (Pre-Pen) is not available mL intradermal injections of negative control and
for skin testing, all patients with a history suggest- antigen solutions are made into the volar surface of
ing IgE-mediated reactions to penicillin (e.g., ana- the forearm by using a 26- or 27-gauge needle on a
phylaxis, angioedema, bronchospasm, or urticaria) syringe. The margins of the wheals induced by the
should be desensitized in a hospital setting. In injections should be marked with a ball point pen.
patients with reactions not likely to be IgE-medi- An intradermal test is positive if the average wheal
ated, outpatient-monitored test doses can be consid- diameter 15 minutes after injection is >2 mm larger
ered. than the initial wheal size and also is >2 mm larger
than the negative controls. Otherwise, the tests are
Penicillin Allergy Skin Testing negative.

Patients at high risk for anaphylaxis, including those


who 1) have a history of penicillin-related anaphy-
Desensitization

laxis, asthma, or other diseases that would make Patients who have a positive skin test to one of the
anaphylaxis more dangerous or 2) are being treated penicillin determinants can be desensitized (Table
with beta-adrenergic blocking agents, should be 1). This is a straightforward, relatively safe proce-
tested with 100-fold dilutions of the full-strength dure that can be performed orally or IV. Although
skin-test reagents before being tested with full- the two approaches have not been compared, oral
strength reagents. In these situations, patients desensitization is regarded as safer and easier to per-
should be tested in a monitored setting in which form. Patients should be desensitized in a hospital
treatment for an anaphylactic reaction is available. setting because serious IgE-mediated allergic reac-
If possible, the patient should not have taken anti- tions can occur. Desensitization usually can be com-
histamines recently (e.g., chlorpheniramine maleate pleted in approximately 4–12 hours, after which
or fexafenadine during the preceding 24 hours, time the first dose of penicillin is administered.
diphenhydramine HCl during the preceding 4 days, After desensitization, patients must be maintained
or hydroxyzine or phenathiazines during the preced- on penicillin continuously for the duration of the
ing 3 weeks). course of therapy.

754 EDUCATIONAL REVIEW MANUAL IN UROLOGY


C. Urethritis

Diseases Characterized by Urethritis


and Cervicitis

Urethritis

Urethritis, as characterized by urethral inflamma-


tion, can result from infectious and noninfectious
conditions. Symptoms, if present, include discharge
of mucopurulent or purulent material, dysuria, or
urethral pruritis. Asymptomatic infections are com-
mon. Although N. gonorrhoeae and C. trachomatis
are well established as clinically important infec-
tious causes of urethritis, Mycoplasma genitalium
has also been associated with urethritis (240–243).
If clinic-based diagnostic tools (e.g., Gram-stain
microscopy, first void urine with microscopy, and
leukocyte esterase) are not available, patients
should be treated with drug regimens effective
against both gonorrhea and chlamydia. Further test-
ing to determine the specific etiology is recom-
mended because both chlamydia and gonorrhea are
reportable to health departments and a specific diag-
nosis might improve partner notification and treat-
ment. Culture, nucleic acid hybridization tests, and
NAATs are available for the detection of both N.
gonorrhoeae and C. trachomatis. Culture and
hybridization tests require urethral swab specimens,
whereas NAATs can be performed on urine speci-
mens. Because of their higher sensitivity, NAATs
are preferred for the detection of C. trachomatis
(197).

Etiology

Several organisms can cause infectious urethritis.


The presence of Gram-negative intracellular diplo-
cocci (GNID) on urethral smear is indicative of gon-
orrhea infection, which is frequently accompanied
by chlamydial infection. Nongonoccocal urethritis
(NGU), which is diagnosed when examination find-
ings or microscopy indicate inflammation without
GNID, is caused by C. trachomatis in 15%–40% of
cases; however, prevalence varies by age group,
with a lower burden of disease occurring among
older men (244). Complications of NGU among
males infected with C. trachomatis include epi-
didymitis and Reiter’s syndrome. Documentation of

CHAPTER 23A: SEXUALLY TRANSMITTED DISEASES 755


chlamydial infection is essential because of the need
for partner referral for evaluation and treatment.

In most cases of nonchlamydial NGU, no pathogen


can be detected. M. genitalium, which appears to be
sexually transmitted, is associated with both symp-
toms of urethritis and urethral inflammation and
accounts for 15%–25% of NGU cases in the United
States (240–243). T. vaginalis, HSV, and adenovirus
also can cause NGU, but data supporting other
Mycoplasma species and Ureaplasma as etiologic
agents are inconsistent (244–247). Diagnostic and
treatment procedures for these organisms are
reserved for situations in which these infections are
suspected (e.g., contact with trichomoniasis, genital
lesions, or severe dysuria and meatitis, which might
suggest genital herpes) or when NGU is not respon-
sive to therapy. Enteric bacteria have been identified
as an uncommon cause of NGU and might be associ-
ated with insertive anal intercourse (244).
• Positive leukocyte esterase test on first-void urine or
microscopic examination of first-void urine sedi-
ment demonstrating ≥10 WBC per high-power field.
Confirmed Urethritis

Clinicians should attempt to obtain objective evi-


If none of these criteria are present, testing for N. gon-
dence of urethral inflammation. However, if clinic-
orrhoeae and C. trachomatis using NAATs might
based diagnostic tools (e.g., Gram-stain
identify additional infections (248). If the results
microscopy) are not available, patients should be
demonstrate infection with either of these pathogens,
treated with drug regimens effective against both
the appropriate treatment should be given and sex
gonorrhea and chlamydia.
partners referred for evaluation and treatment. If none
of these criteria are present, empiric treatment of
Urethritis can be documented on the basis of any of
symptomatic males is recommended only for men at
the following signs or laboratory tests:
high risk for infection who are unlikely to return for a
follow-up evaluation. Such patients should be treated
• Mucopurulent or purulent discharge on examina-
with drug regimens effective against gonorrhea and
tion.
chlamydia. Partners of patients treated empirically
should be evaluated and treated, if indicated.
• Gram stain of urethral secretions demonstrating ≥5
WBC per oil immersion field. The Gram stain is
the preferred rapid diagnostic test for evaluating
urethritis and is highly sensitive and specific for
Nongonococcal Urethritis

documenting both urethritis and the presence or


absence of gonococcal infection. Gonococcal
Diagnosis

All patients who have confirmed or suspected urethri-


infection is established by documenting the pres-
tis should be tested for gonorrhea and chlamydia.
ence of WBC containing GNID.
Testing for chlamydia is strongly recommended
because of the increased utility and availability of
highly sensitive and specific testing methods (e.g.,
NAATs) and because a specific diagnosis might
enhance partner notification and improve compliance
with treatment, especially in the exposed partner.

756 EDUCATIONAL REVIEW MANUAL IN UROLOGY


symptoms, pain during or after ejaculation, or new-
onset premature ejaculation lasting for >3 months.
Treatment

Treatment should be initiated as soon as possible


after diagnosis. Azithromycin and doxycycline are Unless a patient’s symptoms persist or therapeutic
highly effective for chlamydial urethritis; however, noncompliance or reinfection is suspected by the
infections with M. genitalium respond better to provider, a test-of-cure (i.e., repeat testing 3–4
azithromycin (249,250). Single-dose regimens have weeks after completing therapy) is not recom-
the advantage of improved compliance and directly mended for persons with documented chlamydia or
observed treatment. To maximize compliance with gonococcal infections who have received treatment
recommended therapies, medications should be dis- with recommended or alterative regimens. How-
pensed on-site in the clinic, and the first dose should ever, because men with documented chlamydial or
be directly observed. gonococcal infections have a high rate of reinfec-
tion within 6 months after treatment (251,252),
repeat testing of all men diagnosed with chlamydia
or gonorrhea is recommended 3–6 months after
treatment, regardless of whether patients believe
that their sex partners were treated (251).

Partner Referral

A specific diagnosis might facilitate partner referral.


Therefore, testing for gonorrhea and chlamydia is
encouraged. Because a substantial proportion of
female partners of males with nonchlamydial NGU
are infected with chlamydia, partner management is
recommended for males with NGU regardless of
To minimize transmission, men treated for NGU whether a specific etiology is identified. All sex
should be instructed to abstain from sexual inter- partners within the preceding 60 days should be
course for 7 days after single-dose therapy or until referred for evaluation, testing, and empiric treat-
completion of a 7-day regimen, provided their ment with a drug regimen effective against chlamy-
symptoms have resolved. To minimize the risk for dia. Expedited partner treatment and patient referral
reinfection, men should be instructed to abstain are alternative approaches to treating partners (71).
from sexual intercourse until all of their sex partners
are treated. Recurrent and Persistent Urethritis

Persons who have been diagnosed with a new STD Objective signs of urethritis should be present
should receive testing for other infections, including before the initiation of antimicrobial therapy. In per-
syphilis and HIV. sons who have persistent symptoms after treatment
without objective signs of urethritis, the value of
extending the duration of antimicrobials has not
been demonstrated. Persons who have persistent or
Follow-Up

Patients should be instructed to return for evaluation recurrent urethritis can be retreated with the initial
if symptoms persist or recur after completion of regimen if they did not comply with the treatment
therapy. Symptoms alone, without documentation of regimen or if they were reexposed to an untreated
signs or laboratory evidence of urethral inflamma- sex partner. Persistent urethritis after doxycycline
tion, are not a sufficient basis for retreatment. treatment might be caused by doxycycline-resistant
Providers should be alert to the possibility of U. urealyticum or M. genitalium. T. vaginalis is also
chronic prostatitis/chronic pelvic pain syndrome in known to cause urethritis in men; a urethral swab,
male patients experiencing persistent pain (perineal, first void urine, or semen for culture or a NAAT
penile, or pelvic), discomfort, irritative voiding (PCR or TMA) on a urethral swab or urine can be

CHAPTER 23A: SEXUALLY TRANSMITTED DISEASES 757


performed. If compliant with the initial regimen and
re-exposure can be excluded, the following regimen
Special Considerations

is recommended while awaiting the results of the


diagnostic tests.
HIV Infection

Gonococcal urethritis, chlamydial urethritis, and


nongonococcal, nonchlamydial urethritis might
facilitate HIV transmission. Patients who have
NGU and also are infected with HIV should receive
the same treatment regimen as those who are HIV
negative.

Studies involving a limited number of patients who


experienced NGU treatment failures have demon-
strated that Moxifloxacin 400 mg orally once daily
for 7 days is highly effective against M. genitalium
(253,254). Men with a low probability of T. vagi-
nalis (e.g., MSM) are unlikely to benefit from the
addition of metronidazole or tinidazole.

Urologic examinations usually do not reveal a spe-


cific etiology for urethritis. A four-glass Meares-
Stamey lower-urinary-tract localization procedure
(or four-glass test) might be helpful in localizing
pathogens to the prostate (255). A substantial pro-
portion of men with chronic nonbacterial prostati-
tis/chronic pelvic pain syndrome have evidence of
urethral inflammation without any identifiable
microbial pathogens. Estimates vary considerably
depending on the source and sensitivity of the assay,
but one study demonstrated that in 50% of men with
this syndrome, ≥5 WBCs per high-power field were
detected in expressed prostatic secretions (256).
Referral to a urologist should be considered for men
who experience pain for more than 3 months within
a 6-month period.

If men require treatment with a new antibiotic regi-


men for persistent urethritis and a sexually transmit-
ted agent is the suspected cause, all partners in the
past 60 days before the initial diagnosis and any
interim partners should be referred for evaluation
and appropriate treatment.

758 EDUCATIONAL REVIEW MANUAL IN UROLOGY


stain of endocervical specimens, pharyngeal, or rec-
tal specimens also are not sufficient to detect infec-
D. Gonococcal Infections

tion, and therefore are not recommended. Specific


testing for N. gonorrhoeae is recommended because
Gonococcal Infections in

of the increased utility and availability of highly


Adolescents and Adults

In the United States, an estimated 700,000 new N. sensitive and specific testing methods and because a
gonorrhoeae infections occur each year (93,293). specific diagnosis might enhance partner notifica-
Gonorrhea is the second most commonly reported tion.
bacterial STD. The majority of urethral infections
caused by N. gonorrhoeae among men produce Specific diagnosis of infection with N. gonorrhoeae
symptoms that cause them to seek curative treat- can be performed by testing endocervical, vaginal,
ment soon enough to prevent serious sequelae, but urethral (men only), or urine specimens. Culture,
treatment might not be soon enough to prevent nucleic acid hybridization tests, and NAATs are
transmission to others. Among women, gonococcal available for the detection of genitourinary infection
infections might not produce recognizable symp- with N. gonorrhoeae (197). Culture and nucleic acid
toms until complications (e.g., PID) have occurred. hybridization tests require female endocervical or
PID can result in tubal scarring that can lead to male urethral swab specimens. NAATs allow testing
infertility or ectopic pregnancy. of the widest variety of specimen types including
endocervical swabs, vaginal swabs, urethral swabs
The prevalence of gonorrhea varies widely among (men), and urine (from both men and women), and
communities and populations; health-care providers they are FDA-cleared for use. However, product
should consider local gonorrhea epidemiology inserts for each NAAT vendor must be carefully
when making screening decisions. Although examined, because specimen types that are FDA-
widespread screening is not recommended because cleared for use vary by test. NAAT tests are not
gonococcal infections among women are frequently FDA-cleared for use in the rectum, pharynx, and
asymptomatic, targeted screening of young women conjunctiva; however, some public and private lab-
(i.e., those aged <25 years) at increased risk for oratories have established performance specifica-
infection is a primary component of gonorrhea con- tions for using NAAT with rectal and pharyngeal
trol in the United States. For sexually active women, swab specimens, thereby allowing results to be used
including those who are pregnant, USPSTF (82) for clinical management. Laboratories that establish
recommends that clinicians provide gonorrhea performance specifications for the use of NAATs
screening only to those at increased risk for infec- with nongenital specimens must ensure that speci-
tion (e.g., women with previous gonorrhea infec- ficity is not compromised by cross-reaction with
tion, other STDs, new or multiple sex partners, and nongonococcal Neisseria species. The sensitivity of
inconsistent condom use; those who engage in com- NAATs for the detection of N. gonorrhoeae in geni-
mercial sex work and drug use; women in certain tal and nongenital anatomic sites is superior to cul-
demographic groups; and those living in communi- ture but varies by NAAT type (197,278–281).
ties with a high prevalence of disease). USPSTF
does not recommend screening for gonorrhea in Because nonculture tests cannot provide antimicro-
men and women who are at low risk for infection bial susceptibility results, in cases of suspected or
(82). documented treatment failure, clinicians should per-
form both culture and antimicrobial susceptibility
Diagnostic Considerations testing.

Because of its high specificity (>99%) and sensitiv- All persons found to have who have gonorrhea also
ity (>95%), a Gram stain of a male urethral speci- should be tested for other STDs, including chlamy-
men that demonstrates polymorphonuclear leuko- dia, syphilis, and HIV.
cytes with intracellular Gram-negative diplococci
can be considered diagnostic for infection with N.
gonorrhoeae in symptomatic men. However,
because of lower sensitivity, a negative Gram stain
should not be considered sufficient for ruling out
infection in asymptomatic men. In addition, Gram

CHAPTER 23A: SEXUALLY TRANSMITTED DISEASES 759


in Hawaii in 2001 (305). To ensure appropriate
antibiotic therapy, clinicians should ask patients
Dual Therapy for Gonococcal and

testing positive for gonorrhea about recent travel to


Chlamydial Infections

Patients infected with N. gonorrhoeae frequently and sexual activity in these countries.
are coinfected with C. trachomatis; this finding has
led to the recommendation that patients treated for Decreased susceptibility of N. gonorrhoeae to
gonococcal infection also be treated routinely with a cephalosporins and other antimicrobials is expected
regimen that is effective against uncomplicated gen- to continue to spread; therefore, state and local
ital C. trachomatis infection (294). Because most surveillance for antimicrobial resistance is crucial
gonococci in the United States are susceptible to for guiding local therapy recommendations (297).
doxycycline and azithromycin, routine cotreatment GISP, which samples approximately 3% of all U.S.
might also hinder the development of antimicrobial- men who have gonococcal infections, is a mainstay
resistant N. gonorrhoeae. Limited data suggest that of surveillance. However, surveillance by clinicians
dual treatment with azithromycin might enhance also is critical. Clinicians who diagnose N. gonor-
treatment efficacy for pharyngeal infection when rhoeae infection in a patient with suspected
using oral cephalosporins (295,296). cephalosporin treatment failure should perform cul-
ture and susceptibility testing of relevant clinical
specimens, consult a specialist for guidance in clin-
ical management, and report the case to CDC
Antimicrobial-Resistant N. gonorrhoeae

Gonorrhea treatment is complicated by the ability of through state and local public health authorities.
N. gonorrhoeae to develop resistance to antimicro- Health departments should prioritize partner notifi-
bial therapies (297). Quinolone-resistant N. gonor- cation and contact tracing of patients with N. gonor-
rhoeae strains are now widely disseminated rhoeae infection thought to be associated with
throughout the United States and the world (298). cephalosporin treatment failure or associated with
As of April 2007, quinolones are no longer recom- patients whose isolates demonstrate decreased sus-
mended in the United States for the treatment of ceptibility to cephalosporin.
gonorrhea and associated conditions, such as PID
(299). Consequently, only one class of antimicro-
bials, the cephalosporins, is recommended and
Uncomplicated Gonococcal Infections of

available for the treatment of gonorrhea in the


the Cervix, Urethra, and Rectum

United States. The CDC website


(http://www.cdc.gov/std/gisp) and state health
departments can provide the most current informa-
tion.

The proportion of isolates in CDC’s Gonococcal


Isolate Surveillance Project (GISP) demonstrating
decreased susceptibility to ceftriaxone or cefixime
has remained very low over time; during
1987–2008, only four isolates were found to have
decreased susceptibility to ceftriaxone, and 48 iso-
lates had decreased susceptibility to cefixime. In To maximize compliance with recommended thera-
2008, no isolates demonstrated decreased suscepti- pies, medications for gonococcal infections should
bility to ceftriaxone; cefixime was not part of test be dispensed on site. Ceftriaxone in a single injec-
panel during that year (93). Although only two tion of 250 mg provides sustained, high bactericidal
cases of suspected treatment failure with ceftriax- levels in the blood. Extensive clinical experience
one have been reported (300), approximately 50 indicates that ceftriaxone is safe and effective for
patients are thought to have failed oral the treatment of uncomplicated gonorrhea at all
cephalosporin treatment (301–304). anatomic sites, curing 99.2% of uncomplicated uro-
genital and anorectal and 98.9% of pharyngeal
Most of the treatment failures resulting from use of infections in published clinical trials (306,307). A
oral cephalosporins have been reported from Asian 250-mg dose of ceftriaxone is now recommended
countries, although one possible case was reported over a 125-mg dose given the 1) increasingly wide

760 EDUCATIONAL REVIEW MANUAL IN UROLOGY


geographic distribution of isolates demonstrating a urogenital and rectal cure rate of 96.6% (95% CI =
decreased susceptibility to cephalosporins in vitro, 93.9%), but the efficacy of cefpodoxime 400 mg
2) reports of ceftriaxone treatment failures, 3) orally at the pharyngeal site was poor (70.3%, 95%
improved efficacy of ceftriaxone 250 mg in pharyn- CI = 53.0%) (Hall, unpublished data, 2010). Gono-
geal infection (which is often unrecognized), and 4) coccal strains with decreased susceptibility to oral
the utility of having a simple and consistent recom- cephalosporins have been reported in the United
mendation for treatment regardless of the anatomic
States (308). With a cure rate of 96.5% (95% CI =
site involved.
93.6%–98.3%) for urogenital and rectal infection,
cefpodoxime proxetil 200 mg orally meets the crite-
A 400-mg oral dose of cefixime does not provide as
ria for an alternative regimen; however, its use is not
high, nor as sustained, a bactericidal level as that
advised because of concerns about the pharmacody-
provided by the 250-mg dose of ceftriaxone. In pub-
namics of cefpodoxime using this dose. Efficacy in
lished clinical trials, the 400-mg dose cured 97.5%
treating pharyngeal infection with cefpodoxime 200
of uncomplicated urogenital and anorectal (95% CI
mg is unsatisfactory (78.9%; 95% CI =
= 95.4%–99.8%) and 92.3% of pharyngeal gono-
54.5%–94%), as with cefpodoxime at the 400-mg
coccal infections (95% CI = 74.9%–99.1%)
dose.
(306,307). Although cefixime can be administered
orally, this advantage is offset by the limited effi-
Treatment with cefuroxime axetil 1 g orally meets
cacy of cefixime (as well as other oral
the criteria for minimum efficacy as an alternative
cephalosporins) for treating gonococcal infections
regimen for urogenital and rectal infection (95.9%;
of the pharynx. Providers should inquire about oral
95% CI = 94.3%–97.2%), but the pharmacodynam-
sexual exposure and if reported, treat these patients
ics of cefuroxime axetil 1 g orally are less favorable
with ceftriaxone because of this drug’s well docu-
than those of cefpodoxime 400 mg, cefixime 400
mented efficacy in treating pharyngeal infection.
mg, or ceftriaxone 125 mg (309). The efficacy of
cefuroxime axetil 1 g orally in treating pharyngeal
Single-dose injectible cephalosporin regimens
infection is poor (56.9%; 95% CI = 42.2%–70.7%).
(other than ceftriaxone 250 mg IM) that are safe and
highly effective against uncomplicated urogenital
Spectinomycin, which is useful in persons who can-
and anorectal gonococcal infections include cefti-
not tolerate cephalosporins, is expensive, must be
zoxime (500 mg, administered IM), cefoxitin (2 g,
injected, and is not available in the United States
administered IM with probenecid 1 g orally), and
(updates available at: www.cdc.gov/std/treatment)
cefotaxime (500 mg, administered IM). None of the
(310). However, it has been effective in published
injectible cephalosporins offer any advantage over
clinical trials, curing 98.2% of uncomplicated uro-
ceftriaxone for urogenital infection, and efficacy for
genital and anorectal gonococcal infections. Specti-
pharyngeal infection is less certain (306,307).
nomycin has poor efficacy against pharyngeal
infection (51.8%; 95% CI = 38.7%–64.9%) (306).

Azithromycin 2 g orally is effective against uncom-


Alternative Regimens

Several other antimicrobials are active against N.


plicated gonococcal infection (99.2%; 95% CI =
gonorrhoeae, but none have substantial advantages
97.3%–99.9%), but concerns over the ease with
over the recommended regimens, and they should
which N. gonorrhoeae can develop resistance to
not be used if pharyngeal infection is suspected.
macrolides should restrict its use to limited circum-
Some evidence suggests that cefpodoxime 400-mg
stances. Although azithromycin 1 g meets alterna-
orally can be considered an alternative in the treat-
tive regimen criteria (97.6%; 95% CI =
ment of uncomplicated urogenital gonorrhea; this
95.7%–98.9%), it is not recommended because sev-
regimen meets the minimum efficacy criteria for
eral studies have documented treatment failures,
alternative regimens for urogenital infection
and concerns about possible rapid emergence of
(demonstrated efficacy of ≥95% in clinical trials
antimicrobial resistance with the 1-g dose of
with lower 95% CI of >90%) (307). In one clinical
azithromycin are even greater than with the 2-g dose
trial, cefpodoxime 400 mg orally was found to have
(311–313). N. gonorrhoeae in the United States is

CHAPTER 23A: SEXUALLY TRANSMITTED DISEASES 761


not adequately susceptible to penicillins, tetracy- gonorrhea to be retested 3 months after treatment. If
clines, and older macrolides (e.g., erythromycin) for patients do not seek medical care for retesting in 3
these antimicrobials to be recommended. months, providers are encouraged to test these
patients whenever they next seek medical care
within the following 12 months, regardless of
whether the patients believe that their sex partners
Uncomplicated Gonococcal Infections of

were treated. Retesting is distinct from test-of-cure


the Pharynx

Most gonococcal infections of the pharynx are to detect therapeutic failure, which is not recom-
asymptomatic and can be relatively common in mended.
some populations (103,278,279,314). Gonococcal
infections of the pharynx are more difficult to eradi-
cate than infections at urogenital and anorectal sites
Management of Sex Partners

(315). Few antimicrobial regimens, including those Effective clinical management of patients with
involving oral cephalosporins, can reliably cure treatable STDs requires treatment of the patients’
>90% of gonococcal pharyngeal infections recent sex partners to prevent reinfection and curtail
(306,307). Providers should ask their patients about further transmission. Patients should be instructed
oral sexual exposure; if reported, patients should be to refer their sex partners for evaluation and treat-
treated with a regimen with acceptable efficacy ment. Sex partners of patients with N. gonorrhoeae
against pharyngeal infection. Chlamydial coinfec- infection whose last sexual contact with the patient
tion of the pharynx is unusual; however, because was within 60 days before onset of symptoms or
coinfection at genital sites sometimes occurs, treat- diagnosis of infection in the patient should be evalu-
ment for both gonorrhea and chlamydia is recom- ated and treated for N. gonorrhoeae and C. tra-
mended. chomatis infections. If a patient’s last sexual inter-
course was >60 days before onset of symptoms or
diagnosis, the patient’s most recent sex partner
should be treated. Patients should be instructed to
abstain from sexual intercourse until therapy is
completed and until they and their sex partners no
longer have symptoms.

For heterosexual patients with gonorrhea whose


partners’ treatment cannot be ensured or is unlikely,
Follow-Up

Patients diagnosed with uncomplicated gonorrhea delivery of antibiotic therapy for gonorrhea (as well
who are treated with any of the recommended or as for chlamydia) by the patients to their partners
alternative regimens do not need a test-of-cure (i.e., can be considered (see Partner Management). Use
repeat testing 3-4 weeks after completing therapy). of this approach (68,71) should always be accompa-
Patients who have symptoms that persist after treat- nied by efforts to educate partners about symptoms
ment should be evaluated by culture for N. gonor- and to encourage partners to seek clinical evalua-
rhoeae, and any gonococci isolated should be tested tion. For male patients informing female partners,
for antimicrobial susceptibility. Persistent urethritis, educational materials should include information
cervicitis, or proctitis also might be caused by C. about the importance of seeking medical evaluation
trachomatis or other organisms. for PID (especially if symptomatic). Possible under-
treatment of PID in female partners and possible
N. gonorrhoeae infection is prevalent among missed opportunities to diagnose other STDs are of
patients who have been diagnosed with and treated concern and have not been evaluated in comparison
for gonorrhea in the preceding several months with patient-delivered therapy and partner referral.
(64,251,252,267). Most infections result from rein- This approach should not be considered a routine
fection rather than treatment failure, indicating a partner management strategy in MSM because of
need for improved patient education and referral of
sex partners. Clinicians should advise patients with

762 EDUCATIONAL REVIEW MANUAL IN UROLOGY


the high risk for coexisting undiagnosed STDs or
HIV infection.
Suspected Cephalosporin Treatment
Failure or Resistance

Suspected treatment failure has been reported


among persons receiving oral and injectable
Special Considerations

cephalosporins (300–304). Therefore, clinicians of


patients with suspected treatment failure or persons
Allergy, Intolerance, and

infected with a strain found to demonstrate in vitro


Adverse Reactions

Reactions to first generation cephalosporins occur resistance should consult an infectious disease spe-
in approximately 5%–10% of persons with a history cialist, conduct culture and susceptibility testing of
of penicillin allergy and occur less frequently with relevant clinical specimens, retreat with at least 250
third-generation cephalosporins (239). In those per- mg of ceftriaxone IM or IV, ensure partner treat-
sons with a history of penicillin allergy, the use of ment, and report the situation to CDC through state
cephalosporins should be contraindicated only in and local public health authorities.
those with a history of a severe reaction to penicillin
(e.g., anaphylaxis, Stevens Johnson syndrome, and
toxic epidermal necrolysis) (316).
Gonococcal Conjunctivitis

In the only published study of the treatment of gono-


Because data are limited regarding alternative regi- coccal conjunctivitis among U.S. adults, all 12
mens for treating gonorrhea among persons who study participants responded to a single 1-g IM
have severe cephalosporin allergy, providers treat- injection of ceftriaxone (317).
ing such patients should consult infectious disease
specialists. Azithromycin 2 g orally is effective
against uncomplicated gonococcal infection, but
because of concerns over emerging antimicrobial
resistance to macrolides, its use should be limited. Consider lavage of the infected eye with saline solu-
Cephalosporin treatment following desensitization tion once. Persons treated for gonococcal conjunc-
is impractical in most clinical settings. tivitis should be treated presumptively for concur-
rent C. trachomatis infection.
Pregnancy

As with other patients, pregnant women infected


Management of Sex Partners

with N. gonorrhoeae should be treated with a rec- Patients should be instructed to refer their sex part-
ommended or alternate cephalosporin. Because ners for evaluation and treatment (see Gonococcal
spectinomycin is not available in the United States, Infections, Management of Sex Partners).
azithromycin 2 g orally can be considered for
women who cannot tolerate a cephalosporin. Either
azithromycin or amoxicillin is recommended for
Disseminated Gonococcal Infection (DGI)

treatment of presumptive or diagnosed C. tra- DGI frequently results in petechial or pustular acral
chomatis infection during pregnancy (see Chlamy- skin lesions, asymmetrical arthralgia, tenosynovitis,
dial Infections). or septic arthritis. The infection is complicated
occasionally by perihepatitis and rarely by endo-
carditis or meningitis. Some strains of N. gonor-
rhoeae that cause DGI can cause minimal genital
HIV Infection

Patients who have gonococcal infection and also are inflammation. No recent studies have been pub-
infected with HIV should receive the same treat- lished on the treatment of DGI.
ment regimen as those who are HIV negative.

CHAPTER 23A: SEXUALLY TRANSMITTED DISEASES 763


Treatment Management of Sex Partners

Hospitalization is recommended for initial therapy, Patients should be instructed to refer their sex part-
especially for patients who might not comply with ners for evaluation and treatment (see Gonococcal
treatment, for those in whom diagnosis is uncertain, Infection, Management of Sex Partners).
and for those who have purulent synovial effusions
or other complications. Examination for clinical
evidence of endocarditis and meningitis should be
performed. Persons treated for DGI should be
treated presumptively for concurrent C. trachomatis
infection.

All of the preceding regimens should be continued


for 24–48 hours after improvement begins, at which
time therapy can be switched to cefixime 400 mg
orally twice daily to complete at least 1 week of
antimicrobial therapy. No treatment failures have
been reported with the recommended regimens.

Management of Sex Partners

Gonococcal infection frequently is asymptomatic in


sex partners of patients who have DGI. As with
uncomplicated gonococcal infections, patients
should be instructed to refer their sex partners for
evaluation and treatment (see Gonococcal Infection,
Management of Sex Partners).

Gonococcal Meningitis and Endocarditis

Therapy for meningitis should be continued for


10–14 days; therapy for endocarditis should be con-
tinued for at least 4 weeks. Treatment of compli-
cated DGI should be undertaken in consultation
with an infectious disease specialist.

764 EDUCATIONAL REVIEW MANUAL IN UROLOGY


E. Epididymitis Diagnostic Considerations

Acute epididymitis is a clinical syndrome consisting Men who have acute epididymitis typically have
of pain, swelling, and inflammation of the epi- unilateral testicular pain and tenderness; hydrocele
didymis that lasts <6 weeks (402). Chronic epi- and palpable swelling of the epididymis usually are
didymitis is characterized by a ≥6 week history of present. Although the inflammation and swelling
symptoms of discomfort and/or pain in the scrotum, usually begin in the tail of the epididymis, they can
testicle, or epididymis. In most cases of acute epi- spread to involve the rest of the epididymis and tes-
didymitis, the testis is also involved in the process ticle. The spermatic cord is usually tender and
— a condition referred to as epididymo-orchitis. swollen. Testicular torsion, a surgical emergency,
Chronic epididymitis has been subcategorized into should be considered in all cases, but it occurs more
inflammatory chronic epididymitis, obstructive frequently among adolescents and in men without
chronic epididymitis, and chronic epididymalgia evidence of inflammation or infection. Emergency
(403). testing for torsion might be indicated when the onset
of pain is sudden, pain is severe, or the test results
Among sexually active men aged <35 years, acute available during the initial examination do not sup-
epididymitis is most frequently caused by C. tra- port a diagnosis of urethritis or urinary-tract infec-
chomatis or N. gonorrhoeae. Acute epididymitis tion. If the diagnosis is questionable, a urologist
caused by sexually transmitted enteric organisms should be consulted immediately because testicular
(e.g., Escherichia coli and Pseudomonas spp.) also viability might be compromised. Radionuclide
occurs among men who are the insertive partner scanning of the scrotum is the most accurate radio-
during anal intercourse. Sexually transmitted acute logic method of diagnosis, but it is not routinely
epididymitis usually is accompanied by urethritis, available. Although ultrasound is primarily used for
which frequently is asymptomatic. ruling out torsion of the spermatic cord in cases of
acute scrotum swelling, it will often demonstrate
In men aged >35 years, sexually transmitted epi- epididymal hyperemia and swelling in men with
didymitis is uncommon, whereas bacteriuria sec- epididymitis. However, differentiation between tes-
ondary to obstructive urinary disease (e.g., benign ticular torsion and epididymitis must be made on the
prostatic hyperplasia) is more common. In this older basis of clinical evaluation, because partial sper-
population, nonsexually transmitted epididymitis is matic cord torsion can mimic epididymitis on scro-
associated with urinary tract instrumentation or tal ultrasound. Ultrasound provides minimal utility
surgery, systemic disease, and immunosuppression. for men with a clinical presentation consistent with
epididymitis; a negative ultrasound does not alter
Chronic infectious epididymitis is most frequently physician management of clinical epididymitis.
seen in conditions associated with granulomatous Ultrasound, therefore, should be reserved for
reaction; Mycobacterium tuberculosis (TB) is the patients with scrotal pain who cannot be diagnosed
most common granulomatous disease affecting the accurately by physical examination, history, and
epididymis. Up to 25% of patients can have bilateral objective laboratory findings.
disease, with ultrasound demonstrating an enlarged
hyperemic epididymis with multiple cysts and calci-
fications. Tuberculous epididymitis should be sus-
pected in all patients with a known history of or
recent exposure to TB or in patients whose clinical
status worsens despite appropriate antibiotic treat-
ment.

CHAPTER 23A: SEXUALLY TRANSMITTED DISEASES 765


The evaluation of men for epididymitis should (e.g., MSM who report insertive anal intercourse),
include one of the following: ceftriaxone with a fluoroquinolone are recom-
mended.
• Gram stain of urethral secretions demonstrating ≥5
WBC per oil immersion field. Gram stain is the
preferred rapid diagnostic test for evaluating ure-
thritis because it is highly sensitive and specific for
documenting both urethritis and the presence or
absence of gonococcal infection. Gonococcal
infection is established by documenting the pres-
ence of WBC containing intracellular Gram-nega-
tive diplococci on urethral Gram stain.
Although most patients can be treated on an out-
• Positive leukocyte esterase test on first-void urine patient basis, hospitalization should be considered
or microscopic examination of first-void urine sed- when severe pain suggests other diagnoses (e.g., tor-
iment demonstrating ≥10 WBC per high power sion, testicular infarction, or abscess) or when
field. patients are unable or unlikely to comply with an
antimicrobial regimen. Because high fever is uncom-
Culture, nucleic acid hybridization tests, and NAATs mon and indicates a complicated infection, these
are available for the detection of both N. gonor- patients should be admitted for further evaluation.
rhoeae and C. trachomatis. Culture and nucleic acid
hybridization tests require urethral swab specimens,
whereas amplification tests can be performed on
Follow-Up

urine or urethral specimens. Because of their higher Patients should be instructed to return to their health-
sensitivity, amplification tests are preferred for the care providers if their symptoms fail to improve
detection of C. trachomatis. Depending on the risk, within 48 hours of the initiation of treatment. Signs
patients whose conditions are associated with and symptoms of epididymitis that do not subside
acquiring an STD should receive testing for other within 3 days requires re-evaluation of the diagnosis
STDs. and therapy. Swelling and tenderness that persist
after completion of antimicrobial therapy should be
evaluated comprehensively. Differential diagnoses
include tumor, abscess, infarction, testicular cancer,
Treatment

Empiric therapy is indicated before laboratory test TB, and fungal epididymitis.
results are available. The goals of treatment of acute
epididymitis caused by C. trachomatis or N. gonor-
rhoeae are 1) microbiologic cure of infection, 2)
Management of Sex Partners

improvement of signs and symptoms, 3) prevention Patients who have acute epididymitis that is con-
of transmission to others, and 4) a decrease in poten- firmed or suspected to be caused by N. gonorrhoeae
tial complications (e.g., infertility or chronic pain). or C. trachomatis should be instructed to refer sex
As an adjunct to therapy, bed rest, scrotal elevation, partners for evaluation and treatment if their contact
and analgesics are recommended until fever and with the index patient was within the 60 days preced-
local inflammation have subsided. Because empiric ing onset of their own symptoms.
therapy is often initiated before laboratory tests are
available, all patients should receive ceftriaxone plus Patients should be instructed to abstain from sexual
doxycycline for the initial therapy of epididymitis. intercourse until they and their sex partners have
Additional therapy can include a fluoroquinolone if been adequately treated (i.e., until therapy is com-
acute epididymitis is not found to be caused by gon- pleted and patient and partners no longer have symp-
orrhea by NAAT or if the infection is most likely toms).
caused by enteric organisms. For men who are at risk
for both sexually transmitted and enteric organisms

766 EDUCATIONAL REVIEW MANUAL IN UROLOGY


Special Considerations Treatment

Treatment is directed to the macroscopic (i.e., genital


warts) or pathologic (i.e, precancerous) lesions
HIV Infection

Patients who have uncomplicated acute epididymitis caused by infection. Subclinical genital HPV infec-
and also are infected with HIV should receive the tion typically clears spontaneously, and therefore
same treatment regimen as those who are HIV nega- specific antiviral therapy is not recommended to
tive. Other etiologic agents have been implicated in eradicate HPV infection. In the absence of lesions,
acute epididymitis in HIV infection including CMV, treatment is not recommended for subclinical genital
salmonella, toxoplasmosis, Ureaplasma ure- HPV infection whether it is diagnosed by col-
alyticum, Corynebacterium sp., Mycoplasma sp., and poscopy, acetic acid application, or by laboratory
Mima polymorpha. Fungi and mycobacteria are also tests for HPV DNA. Treatment also is not recom-
more likely to cause acute epididymitis in immuno- mended for cervical intraepithelial neoplasia 1
suppressed men than in immunocompetent men. (CIN1).

Human Papillomavirus (HPV) Infection Prevention

More than 100 types of HPV exist, more than 40 of Two HPV vaccines are licensed in the United States:
which can infect the genital area. Most HPV infec- a bivalent vaccine (Cervarix) containing HPV types
tions are asymptomatic, unrecognized, or subclini- 16 and 18 and a quadrivalent vaccine (Gardasil) vac-
cal. Oncogenic, or high-risk HPV types (e.g., HPV cine containing HPV types 6, 11, 16, and 18. Both
types 16 and 18), are the cause of cervical cancers. vaccines offer protection against the HPV types that
These HPV types are also associated with other cause 70% of cervical cancers (i.e., types 16 and 18),
anogenital cancers in men and women, including and the quadrivalent HPV vaccine also protects
penile, vulvar, vaginal, and anal cancer, as well a against the types that cause 90% of genital warts
subset of oropharyngeal cancers (404). Nononco- (i.e., types 6 and 11). Either vaccine can be adminis-
genic, or low-risk HPV types (e.g., HPV types 6 and tered to girls aged 11–12 years and can be adminis-
11), are the cause of genital warts and recurrent res- tered to those as young as 9 years of age (15,16); girls
piratory papillomatosis. Asymptomatic genital HPV and women ages 13–26 years who have not started or
infection is common and usually self-limited; it is completed the vaccine series also should receive the
estimated that more than 50% of sexually active per- vaccine. HPV vaccine is indicated for girls in this
sons become infected at least once in their lifetime age group, because benefit is greatest if it is adminis-
(405). Persistent oncogenic HPV infection is the tered before the onset of sexual activity. The quadri-
strongest risk factor for development of precancers valent (Gardasil) HPV vaccine can also be used in
and cancers. males aged 9–26 years to prevent genital warts (17).
Administering the vaccine to boys before the onset
of sexual activity is optimal. Both HPV vaccines are
administered as a 3-dose series of IM injections over
HPV Tests

HPV tests are available for women aged >30 years a 6-month period, with the second and third doses
undergoing cervical cancer screening. These tests given 1–2 and then 6 months after the first dose. Ide-
should not be used for men, for women <20 years of ally, the same vaccine product should be used for the
age, or as a general test for STDs. These HPV tests entire 3-dose series. HPV vaccine is available for eli-
detect viral nucleic acid (i.e., DNA or RNA) or cap- gible children and adolescents aged <19 years
sid protein. Four tests have been approved by the through the Vaccines for Children (VFC) program
FDA for use in the United States: the HC II High- (available by calling CDC INFO [800-232-4636]).
Risk HPV test (Qiagen), HC II Low-Risk HPV test
(Qiagen), Cervista HPV 16/18 test, and Cervista Women who have received HPV vaccine should con-
HPV High-Risk test (Hologics). tinue routine cervical cancer screening because 30%
of cervical cancers are caused by HPV types other
than 16 or 18. In the United States, the vaccines are

CHAPTER 23A: SEXUALLY TRANSMITTED DISEASES 767


not licensed or recommended for use in women >26
years of age. No published data are available on the
F. Genital Warts

effectiveness, programmatic requirements, or cost- Of genital warts, 90% are caused by HPV 6 or 11.
effectiveness of administering the HPV vaccine in HPV types 6 or 11 are commonly found before, or at
STD clinic settings. the time of, detection of genital warts (406). HPV
types 16, 18, 31, 33, and 35 are found occasionally in
visible genital warts (usually as coinfections with
HPV 6 or 11) and can be associated with foci of high-
grade intraepithelial neoplasia, particularly in per-
sons who are infected with HIV infection. In addition
to warts on genital areas, HPV types 6 and 11 have
been associated with conjunctival, nasal, oral, and
laryngeal warts.

Genital warts are usually asymptomatic, but depend-


ing on the size and anatomic location, they can be
painful or pruritic. Genital warts are usually flat,
papular, or pedunculated growths on the genital
mucosa. Genital warts occur commonly at certain
anatomic sites, including around the introitus in
women, under the foreskin of the uncircumcised
penis, and on the shaft of the circumcised penis. Gen-
ital warts can also occur at multiple sites in the
anogenital epithelium or within the anogenital tract
(e.g., cervix, vagina, urethra, perineum, perianal
skin, and scrotum). Intra-anal warts are observed pre-
dominantly in persons who have had receptive anal
intercourse, but they can also occur in men and
women who do not have a history of anal sexual con-
tact.

Diagnosis of genital warts is usually clinical, made


by visual inspection. Genital warts can be confirmed
by biopsy, which might be indicated if 1) the diagno-
sis is uncertain; 2) the lesions do not respond to stan-
dard therapy; 3) the disease worsens during therapy;
4) the lesion is atypical; 5) the patient has comprised
immunity; or 6) the warts are pigmented, indurated,
fixed, bleeding, or ulcerated. Genital warts are usu-
ally asymptomatic, but depending on the size and
anatomic location, they might be painful or pruritic.
The use of HPV DNA testing for genital wart diagno-
sis is not recommended, because test results would
not alter clinical management of the condition.

The application of 3%–5% acetic acid, which causes


skin color to turn white, has been used by some
providers to detect HPV-infected genital mucosa.
However, acetic acid application is not a specific test
for HPV infection. Therefore, the routine use of this

768 EDUCATIONAL REVIEW MANUAL IN UROLOGY


procedure for screening to detect mucosal changes apy. The response to treatment and any side effects
attributed to HPV infection is not recommended. should be evaluated throughout the course of therapy.

Complications occur rarely when treatment is admin-


istered properly. Patients should be warned that per-
Treatment

The primary reason for treating genital warts is the sistent hypopigmentation or hyperpigmentation
amelioration of symptoms (including relieving cos- occurs commonly with ablative modalities and has
metic concerns) and ultimately, removal of the warts. also been described with immune modulating thera-
In most patients, treatment can induce wart-free peri- pies (imiquimod). Depressed or hypertrophic scars
ods. If left untreated, visible genital warts can resolve are uncommon but can occur, especially if the patient
on their own, remain unchanged, or increase in size has had insufficient time to heal between treatments.
or number. Available therapies for genital warts Rarely, treatment can result in disabling chronic pain
likely reduce, but probably do not eradicate, HPV syndromes (e.g., vulvodynia and hyperesthesia of the
infectivity. Whether the reduction in HPV viral DNA treatment site) or, in the case of anal warts, painful
resulting from treatment reduces future transmission defecation or fistulas. A limited number of case
remains unclear. No evidence indicates that the pres- reports of severe systemic effects resulting from
ence of genital warts or their treatment is associated treatment with podophyllin resin and interferon have
with the development of cervical cancer. been documented.

Treatment regimens are classified into patient-


applied and provider-applied modalities. Patient-
Regimens

Treatment of genital warts should be guided by the applied modalities are preferred by some patients
preference of the patient, available resources, and the because they can be administered in the privacy of
experience of the health-care provider. No definitive the patient’s home. To ensure that patient-applied
evidence suggests that any of the available treat- modalities are effective, patients must comply with
ments are superior to any other, and no single treat- the treatment regimen and must be capable of identi-
ment is ideal for all patients or all warts. The use of fying and reaching all genital warts. Follow-up visits
locally developed and monitored treatment algo- are not required for persons using patient-applied
rithms has been associated with improved clinical therapy. However, follow-up visits after several
outcomes and should be encouraged. Because of weeks of therapy enable providers to answer any
uncertainty regarding the effect of treatment on questions patients might have about the use of the
future transmission of HPV and the possibility of medication and any side effects they have experi-
spontaneous resolution, an acceptable alternative for enced; follow-up visits also facilitate the assessment
some persons is to forego treatment and wait for of a patient’s response to treatment.
spontaneous resolution.

Factors that influence selection of treatment include


wart size, wart number, anatomic site of the wart,
wart morphology, patient preference, cost of treat-
ment, convenience, adverse effects, and provider
experience. Factors that might affect response to
therapy include the presence of immunosuppression
and compliance with therapy, which can consist of
either a single treatment or complete course of treat-
ment. In general, warts located on moist surfaces or
in intertriginous areas respond best to topical treat-
ment. The treatment modality should be changed if a
patient has not improved substantially after a com-
plete course of treatment or if side effects are severe.
Most genital warts respond within 3 months of ther-

CHAPTER 23A: SEXUALLY TRANSMITTED DISEASES 769


Podofilox is an antimitotic drug that destroys warts, not been established. The safety of sinecatechins dur-
is relatively inexpensive, easy to use, safe, and self- ing pregnancy also is unknown.
applied. Podofilox solution should be applied with a
cotton swab, or podofilox gel with a finger, to visible Cryotherapy destroys warts by thermal-induced
genital warts twice a day for 3 days, followed by 4 cytolysis. Health-care providers must be trained on
days of no therapy. This cycle can be repeated, as the proper use of this therapy because over- and
necessary, for up to four cycles. The total wart area undertreatment can result in complications or low
treated should not exceed 10 cm2, and the total vol- efficacy. Pain after application of the liquid nitrogen,
ume of podofilox should be limited to 0.5 mL per day. followed by necrosis and sometimes blistering, is
If possible, the health-care provider should apply the common. Local anesthesia (topical or injected) might
initial treatment to demonstrate the proper applica- facilitate therapy if warts are present in many areas or
tion technique and identify which warts should be if the area of warts is large.
treated. Mild to moderate pain or local irritation
might develop after treatment. The safety of pod- Pedophyllin resin 10%–25% should be applied to
ofilox during pregnancy has not been established. each wart and allowed to air-dry before the treated
area comes into contact with clothing; overapplica-
Imiquimod is a topically active immune enhancer tion or failure to air dry can result in local irritation
that stimulates production of interferon and other caused by spread of the compound to adjacent areas.
cytokines. Imiquimod cream should be applied once The treatment can be repeated weekly, if necessary.
daily at bedtime, three times a week for up to 16 To avoid the possibility of complications associated
weeks (407). The treatment area should be washed with systemic absorption and toxicity, two guidelines
with soap and water 6–10 hours after the application. should be followed: 1) application should be limited
Local inflammatory reactions, including redness, to <0.5 mL of podophyllin or an area of <10 cm2 of
irritation, induration, ulceration/erosions, and vesi- warts per session and 2) the area to which treatment is
cles, are common with the use of imiquimod, and administered should not contain any open lesions or
hypopigmentation has also been described (408). wounds. The preparation should be thoroughly
Imiquimod might weaken condoms and vaginal washed off 1–4 hours after application to reduce local
diaphragms. The safety of imiquimod during preg- irritation. The safety of podophyllin during preg-
nancy has not been established. nancy has not been established. Podophyllin resin
preparations differ in the concentration of active
Sinecatechin ointment, a green-tea extract with an components and contaminants. The shelf life and sta-
active product (catechins), should be applied three bility of podophyllin preparations are unknown.
times daily (0.5-cm strand of ointment to each wart)
using a finger to ensure coverage with a thin layer of Both TCA and BCA are caustic agents that destroy
ointment until complete clearance of warts. This warts by chemical coagulation of proteins. Although
product should not be continued for longer than 16 these preparations are widely used, they have not
weeks (409–411). The medication should not be been investigated thoroughly. TCA solutions have a
washed off after use. Sexual (i.e., genital, anal, or low viscosity comparable with that of water and can
oral) contact should be avoided while the ointment is spread rapidly if applied excessively; therefore, they
on the skin. The most common side effects of sinecat- can damage adjacent tissues. A small amount should
echins 15% are erythema, pruritis/burning, pain, be applied only to the warts and allowed to dry before
ulceration, edema, induration, and vesicular rash. the patient sits or stands, at which time a white frost-
This medication may weaken condoms and ing develops. If pain is intense, the acid can be neu-
diaphragms. No clinical data are available regarding tralized with soap or sodium bicarbonate. If an excess
the efficacy or safety of sinecatechins compared with amount of acid is applied, the treated area should be
other available anogenital wart treatment modalities. powdered with talc, sodium bicarbonate (i.e., baking
The medication is not recommended for HIV- soda), or liquid soap preparations to remove unre-
infected persons, immunocompromised persons, or acted acid. This treatment can be repeated weekly, if
persons with clinical genital herpes because the necessary.
safety and efficacy of therapy in these settings has

770 EDUCATIONAL REVIEW MANUAL IN UROLOGY


Surgical therapy has the advantage of usually elimi-
nating warts at a single visit. However, such therapy
requires substantial clinical training, additional
equipment, and a longer office visit. After local anes-
thesia is applied, the visible genital warts can be
physically destroyed by electrocautery, in which case
no additional hemostasis is required. Care must be
taken to control the depth of electrocautery to prevent
scarring. Alternatively, the warts can be removed
either by tangential excision with a pair of fine scis-
sors or a scalpel, by laser, or by curettage. Because
most warts are exophytic, this procedure can be
accomplished with a resulting wound that only
extends into the upper dermis. Hemostasis can be
achieved with an electrocautery unit or a chemical
styptic (e.g., an aluminum chloride solution). Sutur-
ing is neither required nor indicated in most cases if
surgical removal is performed properly. Surgical
therapy is most beneficial for patients who have a
large number or area of genital warts. Both carbon
dioxide laser and surgery might be useful in the man-
agement of extensive warts or intraurethral warts,
particularly for those persons who have not
responded to other treatments.

Because all available treatments have shortcomings,


some clinics employ combination therapy (simulta- Intra-anal warts should be managed in consultation
neous use of two or more modalities on the same wart with a specialist. Many persons with warts on the
at the same time). Data are limited regarding the effi- anal mucosa also have warts on the rectal mucosa, so
cacy or risk of complications associated with use of persons with anal and/or intra-anal warts might bene-
such combinations. fit from an inspection of the rectal mucosa by digital
examination, standard anoscopy, or high-resolution
Alternative Regimens anoscopy.

Alternative regimens include treatment options that


might be associated with more side effects and/or
Counseling

less data on efficacy. Alternative regimens include The following key counseling messages should be
intralesional interferon, photodynamic therapy, and conveyed to all patients diagnosed with HPV infec-
topical cidofovir. tion:

• Genital HPV infection is very common. Many types


of HPV are passed on through genital contact, most
often during vaginal and anal sexual contact. HPV
can also be spread by oral sexual contact.

• Most sexually active adults will get HPV at some


point in their lives, though most will never know it
because HPV infection usually has no signs or
symptoms.

CHAPTER 23A: SEXUALLY TRANSMITTED DISEASES 771


• In most cases, HPV infection clears spontaneously, – adolescent females; or
without causing any health problems. Nevertheless,
some infections do progress to genital warts, pre- – for health conditions other than cervical
cancers, and cancers. cancer.

• The types of HPV that cause genital warts are dif- • Two HPV vaccines are available, both of which
ferent from the types that can cause anogenital can- offer protection against the HPV types that cause
cers. 70% of cervical cancers (i.e., types 16 and 18); the
quadrivalent vaccine (Gardasil) also protects
• Within an ongoing sexual relationship, both part- against the types that cause 90% of genital warts
ners are usually infected at the time one person is (i.e., types 6 and 11). These vaccines are most effec-
diagnosed with HPV infection, even though signs of tive when all doses are administered before sexual
infection might not be apparent. contact. Either vaccine is recommended for 11- and
12-year-old girls and for females aged 13–26 years
• A diagnosis of HPV in one sex partner is not indica- who did not receive or complete the vaccine series
tive of sexual infidelity in the other partner. when they were younger. The quadrivalent HPV
vaccine can be used in males aged 9–26 years to
• Treatments are available for the conditions caused prevent genital warts.
by HPV (e.g., genital warts), but not for the virus
itself. The following are specific counseling messages for
those persons diagnosed with genital warts and their
• HPV does not affect a woman’s fertility or ability to partners:
carry a pregnancy to term.
• Genital warts are not life threatening. If left
• Correct and consistent male condom use might untreated, genital warts might go away, stay the
lower the chances of giving or getting genital HPV, same, or grow in size or number. Except in very rare
but such use is not fully protective, because HPV and unusual cases, genital warts will not turn into
can infect areas that are not covered by a condom. cancer.

• Sexually active persons can lower their chances of • It is difficult to determine how or when a person
getting HPV by limiting their number of partners. became infected with HPV; genital warts can be
However, HPV is common and often goes unrecog- transmitted to others even when no visible signs of
nized; persons with only one lifetime sex partner warts are present, even after warts are treated.
can have the infection. For this reason, the only
definitive method to avoid giving and getting HPV • It is not known how long a person remains conta-
infection and genital warts is to abstain from sexual gious after warts are treated. It is also unclear
activity. whether informing subsequent sex partners about a
past diagnosis of genital warts is beneficial to the
• Tests for HPV are now available to help providers health of those partners.
screen for cervical cancer in certain women. These
tests are not useful for screening adolescent females • Genital warts commonly recur after treatment,
for cervical cancer, nor are they useful for screening especially in the first 3 months.
for other HPV-related cancers or genital warts in
men or women. HPV tests should not be used to • Women should get regular Pap tests as recom-
screen: mended, regardless of vaccination or genital wart
history. Women with genital warts do not need to
– men; get Pap tests more often than recommended.

– partners of women with HPV; • HPV testing is unnecessary in sexual partners of


persons with genital warts.

772 EDUCATIONAL REVIEW MANUAL IN UROLOGY


• If one sex partner has genital warts, both sex part-
ners benefit from getting screened for other STDs.
HIV Infection

Persons who are HIV-infected are more likely to


• Persons with genital warts should inform current develop genital warts then persons who are not HIV-
sex partner(s) because the warts can be transmitted infected (413); moreover, lesions are more recalci-
to other partners. In addition, they should refrain trant to treatment due to depressed cell-mediated
from sexual activity until the warts are gone or immunity. No data suggest that treatment modalities
removed. for external genital warts should be different for HIV-
infected persons. However, persons who are
• Correct and consistent male condom use can lower immunosuppressed because of HIV or other reasons
the chances of giving or getting genital warts, but might have larger or more numerous warts, might not
such use is not fully protective because HPV can respond as well as immunocompetent persons to
infect areas that are not covered by a condom. therapy for genital warts, and might have more fre-
quent recurrences after treatment (414–416). Squa-
• The Gardasil vaccine, which has been approved for mous cell carcinomas arising in or resembling genital
use in males and females aged 9–26 years, protects warts might occur more frequently among immuno-
against the HPV types that cause 90% of genital suppressed persons, therefore requiring biopsy for
warts (i.e., types 6 and 11). confirmation of diagnosis for suspicious cases.
Because of the increased incidence of anal cancer in
HIV-infected MSM, screening for anal intraepithelial
neoplasia by cytology can be considered (417). How-
Special Considerations

ever, evidence is limited concerning the natural his-


tory of anal intraepithelial neoplasias, the reliability
Pregnancy

Imiquimod, sinecatechins, podophyllin, and pod- of screening methods, the safety and response to
ofilox should not be used during pregnancy. Genital treatments, and the programmatic considerations that
warts can proliferate and become friable during preg- would support this screening approach.
nancy. Although removal of warts during pregnancy
can be considered, resolution might be incomplete or Squamous Cell Carcinoma in Situ
poor until pregnancy is complete. Rarely, HPV types
6 and 11 can cause respiratory papillomatosis in Persons in whom squamous cell carcinoma in situ of
infants and children, although the route of transmis- the genitalia is diagnosed should be referred to a spe-
sion (i.e., transplacental, perinatal, or postnatal) is cialist for treatment. Ablative modalities usually are
not completely understood. Whether cesarean sec- effective, but careful follow-up is essential for patient
tion prevents respiratory papillomatosis in infants management.
and children also is unclear (412); therefore, cesarean
delivery should not be performed solely to prevent
transmission of HPV infection to the newborn.
Cesarean delivery is indicated for women with geni-
tal warts if the pelvic outlet is obstructed or if vaginal
delivery would result in excessive bleeding. Pregnant
women with genital warts should be counseled con-
cerning the low risk for warts on the larynx (recurrent
respiratory papillomatosis) in their infants or chil-
dren.

CHAPTER 23A: SEXUALLY TRANSMITTED DISEASES 773


774 EDUCATIONAL REVIEW MANUAL IN UROLOGY
Chapter 23B:
Urinary Tract Infections
John N. Krieger, MD

Contents

1. Definitions and Classifications of UTIs

2. Epidemiology of UTIs

3. Pathogenesis of UTIs

4. Antibiotics

5. Antimicrobial Prophylaxis in Urology

6. Prostatitis Syndromes

7. Unusual Infections

8. Malakoplakia

9. Candida UTIs

10. Tuberculosis

11. Summary

CHAPTER 23B: URINARY TRACT INFECTIONS 775


1. Definitions and Classification
of UTIs

UTI is defined as colonization of the urine with bac-


teria plus inflammation and invasion of the urinary
Purpose

The focus of this chapter is to assist in preparing for structures. Because the epithelial surfaces of the uri-
American Board of Urology Certification and Main- nary tract are contiguous infection at any point
tenance of Certification examination questions. In places the entire system at risk. There is a wide clin-
addition, this chapter will provide focus and guid- ical spectrum.
ance for management of important problems
encountered in clinical practice. There is no empha- The optimal clinical approach to UTI starts with
sis on the most current research findings or contro- accurate classification as a critical first step. Many
versies. UTI classification schemes have been proposed.
The most practical classification divides UTIs into 2
categories, complicated UTIs or uncomplicated
UTIs. This is helpful because this is an important
Organization

We will start with some important definitions and clinical distinction. Anatomic evaluation is seldom
the clinical classification of urinary tract infections indicated for patients with uncomplicated UTIs. In
(UTIs). Then we will briefly consider the epidemiol- contrast, patients with complicated UTIs are often a
ogy and pathogenesis of UTIs. Considerable time is diagnostic and treatment challenge. Anatomic eval-
spent on antibiotics because this area is the source of uation may be critical for planning effective therapy
many test questions. Current guidelines and best in patients with complicated UTIs.
practices for preventing and managing infections
are considered because a guidelines-based curricu- A. Complicated UTIs are defined as infections in
lum and testing policy has been stated as an impor- patients with: structural or functional voiding
tant goal of the American Board of Urology. We will abnormalities, obstruction, neurological dis-
outline diagnosis and current management of pro- eases or urinary stones. In addition, patients
statitis syndromes. Finally, we will consider a num- with diseases predisposed to kidney infection
ber of unusual infections that are seldom seen in are classified in the complicated category, such
practice, but are often seen on examinations. as: diabetes mellitus, sickle cell disease, poly-
cystic renal disease and renal transplants. UTIs
in older women are often classified as compli-
cated.

In complicated UTIs host factors are critical.


These patients may or MAY NOT have local
urinary symptoms. Complicated UTIs are often
caused by resistant organisms and are often
seen in urology.

B. Uncomplicated UTIs occur in patients who


have none of the complicating factors outlined
above. In contrast, the goals for managing
patients with uncomplicated UTIs are to limit
unnecessary evaluation and to understand
where our patients fit in the overall picture.
There is a significant amount of literature on
uncomplicated UTIs, especially in adult
women, and it is easy to write test questions on
this subject.

776 EDUCATIONAL REVIEW MANUAL IN UROLOGY


2. Epidemiology of UTIs

Uncomplicated UTIs occur mainly in healthy UTIs are common in all clinical settings. Among
women with normal anatomy and intact void- US outpatients, UTIs account for more than
ing. In this group, established risk factors for 650,000 physician visits each year, including more
uncomplicated UTIs include diaphragm-sper- than 270,000 visits to urologists. Of these patients,
micide use and urinary tract instrumentation. 68% are women. In hospitalized patients, UTIs rep-
Uncomplicated UTIs occasionally occur in resent the most common nosocomial infection and
male infants (associated with lack of circumci- the most common cause of bacteremia. Throughout
sion) and in adolescent and adult males (associ- most of life, rates of bacteriuria and symptomatic
ated with sexual intercourse). UTIs are higher in females than in males. The
exceptions are at the extremes. In infancy and in old
Uncomplicated UTIs have excellent prognosis age, symptomatic UTI rates are higher in males.
for preservation of renal function. This is During the first 3 months of infancy the male-
despite the considerable morbidity of recurrent female ratio for UTIs is 3:1. Circumcision reduces
symptomatic infections and a small risk of UTI rates in male infants by about 90%. By the time
pyelonephritis. The bacterial pathogens causing children reach school age the male-female ratio has
uncomplicated UTIs are generally one species changed dramatically to about 1:30. In girls, UTI
with E. coli causing 80%–90% of infections and prevalence is approximately 1.2% and the incidence
S saprophyticus causing 10%–20% of infec- is approximately 0.4% per year. Thus, 5%–6% of
tions (primarily young, sexually active women). school girls have an episode of bacteriuria. In boys,
Other species include the usual Gram-negative UTI prevalence is markedly lower, only about
pathogens, primarily Klebsiella, Enterobacter 0.04%.
and Proteus spp. Patients with uncomplicated
UTIs respond rapidly to appropriate therapy. Adult women represent the most common patients
with UTIs. In this population UTIs cause consider-
C. Recurrent UTIs have traditionally been con- able morbidity. Approximately 25%–30% of
sidered as either reinfections or persistent infec- 20–40-year-old women have a history of having
tions. With reinfections, each new episode is a been treated for UTI. Overall, the prevalence of bac-
new event (different bacteria). Patients have teriuria in women is about 3.5% in survey studies.
negative cultures between episodes. The bacte- Bacteriuria rates increase about 1% per decade, with
ria originates from outside the urinary tract. rates of about 10% in women over age 70. Despite
the morbidity and small risk of pyelonephritis, these
In contrast to reinfections, patients with bacte- UTIs rarely cause significant renal damage.
rial persistence have multiple UTIs with same
bacterial type. These bacteria originate from
within the urinary tract (e.g., an infected stone
Figure 1

or prostate). Recently, the distinction between


persistent and recurrent infections has come
into question due to documentation of bacterial
pods in the bladder wall (primarily in animal
Prostatism

models).
“Pyelitis” of
“Honeymoon Pregnancy
Infancy Pre School Cystitis”

CHAPTER 23B: URINARY TRACT INFECTIONS 777


3. Pathogenesis of UTIs

Factors important in the pathogenesis of infectious In postmenopausal women risk for recurrent UTI is
diseases are usually considered as either host factors determined by hormonal factors (i.e., lack of estro-
or pathogen factors. gen), a history of UTI in the premenopausal period
and genetic factors (e.g., nonsecretor status as con-
A. Host factors sidered above). After menopause, anatomic factors
also become important. These include presence of
1. Routes of infection. Most UTIs occur by incontinence, cystocele and elevated post-void
the ascending route, meaning fecal flora that residual urine.
colonize the perineum, followed by ascent
to the urethra, then the bladder, ureter and In older, institutionalized women established risk
kidney. Hematogenous infections occur less factors for recurrent UTI include: catheterization,
commonly. A classic example is urogenital incontinence, history of antimicrobial exposure, and
tuberculosis. Another example is renal car- impaired functional status.
buncle, where hematogenous dissemination
of a pyogenic skin infection results in seed- B. Pathogen factors. Because E. coli is by far the
ing of a renal infection. On occasion, infec- most common cause of UTIs, most work has
tion of the urinary tract results from a con- focused on this organism. E. coli UTI isolates
tiguous structure. For example, patients with may be classified in different phylogenetic
diverticulitis or inflammatory bowel disease groups. It is clear that fecal and cystitis isolates
may develop fistulas to the bladder. Infec- differ from invasive isolates in terms of their
tion by the lymphatic route is also possible phylogenetic grouping and virulence-associ-
in theory. ated characteristics. Isolates causing UTI are
usually classified as uropathogenic E. coli
2. UTI risk factors. Risk factors for UTI are (UPEC). The most virulent isolates are termed
best defined in adult women. In this popula- extraintestinal pathogenic E. coli (ExPEC).
tion, the Lewis blood group non-secretor Multiple virulence-associated factors character-
phenotype (Le [a+b-] and Le[a-b-]) repre- istic of highly virulent ExPEC are needed to
sents a genetic risk factor for UTI. Carbohy- colonize and infect previously healthy patients
drate structures determining blood group are who have no urological risk factors for UTI.
also related to urothelial receptor density for
E. coli adhesins. Vaginal factors also appear The current concept is that virulence of certain
to influence UTI risk, such as alkalinization phylogenetic groups of E. coli does not result
of pH, antibiotic-induced alterations of nor- from particular antigenic structures themselves
mal flora and use of diaphragm-spermicide but from associated traits within certain E. coli
products. lineages. These traits are termed uropathogenic
properties. Examples include: serum resistance,
3. Recurrent UTI risk. It is possible to con- siderophores (iron-scavenging systems), car-
sider a woman’s risk for recurrent UTIs in boxyesterase phenotype, presence of toxins—
terms of four factors: genetic background, such as hemolysin (coded for by the cytotoxin-
behaviors, hormonal status and functional hly gene cluster), colibactin, cytolethal distend-
status. ing toxins, and cytotoxic necrotizing factor—
and factors that mediate adherence. Adherence
In premenopausal women established risk factors to epithelial surfaces has been studied in some
for recurrent UTIs include historical factors, such as detail. Important factors are listed below.
having a mother with UTIs or a history of UTIs in
childhood. Behaviors are also important, especially C. Factors That Mediate Bacterial Adherence
sexual intercourse and spermicide exposure.
1. Fimbrial adhesins (pili, F-antigens)
a) Mannose-sensitive (MS) adhesins: com-
mon Type 1 pili bind to mannose residues
on host cells
778 EDUCATIONAL REVIEW MANUAL IN UROLOGY
b) Mannose-resistant (MR) adhesins transmissible diseases. Clinical microbiologists
continue to evaluate pure log-phase cultures in
a. Gal-Gal: receptor for P blood group nutrient-rich media. This is a powerful tool to
pili study acute bacterial diseases.
b. PAP operon
c. Gal a (1-4) Gal (digalactoside in neu- Unfortunately, this classic approach examines
tral glucophospholipids on epithelial only planktonic bacteria that are the free-float-
cells and RBCs) ing, single cell phenotype. Environmental
c) X adhesins: heterogeneous microbiologists have long recognized that
2. Afimbrial adhesins (afa) planktonic bacteria represent an uncommon,
usually transient, minor population in most nat-
D. Clonal hypothesis ural environments. Planktonic bacteria also
plays a limited role in chronic infections,
Medical microbiologists currently favor the including UTIs, and chronic and device-associ-
clonal hypothesis to explain the observations ated infections in urology. For example,
above. This hypothesis states that uropathogens catheter-associated UTIs are characterized by
belong to a small group of genetically related presence of a biofilm of adherent organisms that
groups (clones). Strains causing pyelonephritis is central to pathogenesis.
or sepsis in patients with anatomically normal
urinary tracts require multiple virulence factors F. Biofilm-associated Infections
to enable these clones to colonize and invade. In
contrast, E. coli from women with cystitis tend Presence of biofilms explains why use of
to have single or a few virulence factors while antimicrobial agents to sterilize the urine during
fecal E. coli are less likely to have virulence- prolonged catheterization usually results in
associated characteristics. bladder colonization with resistant organisms.
Biofilms are thin layers of microorganisms that
The clonal concept of virulence can be under- adhere to the surface of organic or inorganic
stood in terms of the E. coli genetic structure. structures. Biofilm-associated bacteria repre-
E. coli pathogenicity is correlated with genes sent the predominant bacterial phenotype in
encoding virulence factors. These genes are natural habitats, both pathogenic and environ-
organized on large DNA blocks called mental. In urology, this means that biofilms are
pathogenicity islands. Pathogenicity islands can critical in chronic and device-associated infec-
be on a variety of locations within the E. coli tions.
cells, such as plasmids, bacteriophages or the
bacterial chromosome. The precise location of Important phenotypic changes occur in biofilm.
the pathogenicity islands is not especially Bacteria excretes protective polymers. The
important. What is critical is the organization of biofilm incorporates environmental molecules
multiple virulence-associated factors as a to bind together the microbial community to a
pathogenicity island facilitates dissemination surface and to each other. This is a complex
of these pathogenicity islands between distinct multicellular community with cell-to-cell sig-
E. coli strains and, on occasion, even between naling via secreted pheromones, quorum-sens-
bacterial species. ing mechanisms, and other molecules. Signal-
ing can occur between species and even across
E. Changing Paradigm of UTI microbiological kingdoms (e.g., biofilm-associ-
ated bacteria may communicate with fungi).
Clinical microbiology labs currently evaluate These coordinated activities mean that biofilm-
most medical infections based on the model associated infections represent a formidable
developed by Robert Koch more than 100 years opponent for the host.
ago. Koch worked with bacterial strains in pure
culture. This work led to the germ theory of

CHAPTER 23B: URINARY TRACT INFECTIONS 779


4. Antibiotics

Because antibiotics represent one of the most easily B. New IV Antibiotics for MRSA + VRE
tested areas in UTI, we will devote considerable
attention to this topic. During the last several Quinupristin-dalfopristin (Synercid)
decades there has been very little new antibiotic Tigecycline (Tygacil)
development for urology. This reflects problems Daptomycin (Cubicin)-most data
with regulatory hurdles, commercial viability and Linezolid (Zyvox)-most data
the fact that the easy compounds have been found or
formulated. Below, we briefly outlined information Linezolid. Linezolid works by inhibiting initiation
that is most likely to be tested, rather than providing of protein synthesis (ribosomal). The spectrum is
a comprehensive summary of each agent. generally Gram-positive bacteria, including most
MRSA and VRE. The chief advantage is excellent
oral bioavailability. The main disadvantages are
myelosuppression (monitor blood counts) and
Figure 2

monoamine oxidase inhibition (diet and medication


restrictions).

Daptomycin. This is a cyclic lipopeptide that works


by binding bacterial membranes to depolarize mem-
brane potential with inhibition of DNA, RNA and
protein synthesis. The spectrum includes most
Gram-positive bacteria (including MRSA + VRE).
Daptomycin must be given intravenously.

Ceftaroline is the newest cephalosporin to receive


FDA approval. This drug is approved for skin struc-
ture infections and community-acquired pneumo-
nia. Ceftaroline has a limited antimicrobial spec-
trum. It is not active against important Gram-nega-
tives, but it has broad Gram-positive activity includ-
ing MRSA. In addition, ceftaroline is effective
against strains resistant to other agents such as gly-
copeptides, clindamycin, trimethoprim-sul-
A. New agents for Staphylococcus aureus famethoxazole, daptomycin and linezolid.

There are a few new antibiotics that may be used C. Agents commonly used by urologists
by urologists. The most important new drugs are
for treating S. aureus. Methicillin-susceptible Because any antibiotic can cause allergy, hypersen-
S. aureus evolved to develop resistance to com- sitivity, rash, GI upset, yeast vaginitis or pseu-
monly used drugs, becoming methicillin-resis- domembranous colitis, these potential problems are
tant S. aureus (MRSA) and vancomycin-resis- assumed as adverse effects for each drug. The
tant S. aureus (VRSA). This has occurred for antibacterial spectrum for each agent is the usual
both community-acquired and health care- spectrum.
acquired infections, although the S. aureus
strains remain microbiologically distinct. Gly- Ampicillin and amoxicillin. These beta-lactam
copeptides, particularly vancomycin, considered drugs work by inhibiting bacterial cell wall synthe-
drugs of choice for MRSA. Unfortunately, sis. The spectrum includes Streptococci, Enterococ-
MRSA led to increased vancomycin use with cus, Proteus and E. coli. Disadvantages of these
development of S. aureus phenotypes with drugs are that they are likely to disturb normal vagi-
decreased susceptibility.

780 EDUCATIONAL REVIEW MANUAL IN UROLOGY


nal flora and they may cause acute interstitial include: a high rate of resistant E. coli (many areas
nephritis. In many areas, including my hometown of >20%), interaction with warfarin (increase PT),
Seattle, 40% of E. coli are now resistant. adverse effects on fetal development and hemato-
logic problems, especially in patients with AIDS or
Ampicillin plus clavulanate. This combination G6PD deficiency.
includes clavulanate to inhibit bacterial beta-lacta-
mases. The advantage is that there is less resistance. Nitrofurantoin. Nitrofurantoin inhibits several
Disadvantages of this combination are that it is bacterial enzymes. Despite over 50 years of use,
more likely to disturb normal vaginal flora, there are bacterial resistance remains low. This drug is most
frequent GI side effects, and the combination is effective against E. coli and S. saprophyticus. The
expensive. major advantage is that nitrofurantoin has minimal
effect on fecal and vaginal flora. Disadvantages
Extended-spectrum penicillins. These beta-lac- include: neurotoxicity (avoid in patients with neuro-
tam drugs inhibit bacterial cell wall synthesis. Indi- logical disorders), pulmonary fibrosis and intersti-
vidual agents are usually combined with a beta-lac- tial pneumonitis. Because nitrofurantoin achieves
tamase inhibitor. Because the antimicrobial spectra high levels in urine but NOT in tissues longer treat-
are variable, it is advisable to look up individual ment courses are necessary (e.g., 7 days rather than
agents. Disadvantages are similar to those outlined the usual 3-day treatment course for uncomplicated
above for ampicillin and amoxicillin. In addition, cystitis).
there is a high sodium load with ticarcillin (e.g., 1.2
to 1.8 g/day). Tetracyclines. Tetracyclines exhibit a wide antibac-
terial spectrum (including several sexually transmit-
Cephalosporins. The cephalosporins are also beta- ted diseases) by inhibiting bacterial protein synthe-
lactam drugs that inhibit bacterial cell wall synthe- sis (ribosomal). Disadvantages include: tooth and
sis. The antimicrobial spectra are typically consid- bone toxicity in pregnant women and children, pho-
ered by generation. First-generation cephalosporins tosensitivity, increased BUN and a high level of
are generally effective against: Streptococci, resistance.
Staphylococci (not MRSA) and some Gram-nega-
tive rods. Second-generation cephalosporins are Tigecycline is a newer tetracycline. This minocy-
effective against Streptococci, some Gram-negative cline modification avoids 2 resistance mechanisms,
rods, and some anaerobes. The third-generation ribosomal protection and antibiotic efflux. Tigecy-
cephalosporins are effective against most Gram- cline has a broad spectrum including MRSA and
negative rods, including Pseudomonads. Disadvan- VRE. It must be administered intravenously.
tages are similar to those described above for the
penicillins. It is often stated that about 10% of peni- Fluoroquinolones. The fluoroquinolones have
cillin-allergic patients cross-react to been the mainstay of urological antibiotics. The
cephalosporins, but this appears to be an overesti- mechanism of action is inhibition of bacterial DNA
mate. gyrase, the enzyme that causes supercoiling of
DNA. The spectrum includes most Gram-negative
Trimethoprim-sulfamethoxazole. The mechanism rods including Pseudomonads. The spectrum of
of action for this combination drug is inhibition of some fluoroquinolone drugs also includes Strepto-
sequential steps of bacterial folate metabolism. cocci. A major advantage is that fluoroquinolones
Recent studies suggest that the majority of the have good penetration into the prostate.
antimicrobial efficacy is provided by trimethoprim
while the majority of allergic reactions occur Disadvantages of fluoroquinolones. Increasing
against sulfamethoxazole. The spectrum includes resistance means that these drugs are no longer rec-
Streptococci, Staphylococci, and most Gram-nega- ommended for N. gonorrhoeae. Other important
tive rods, except Pseudomonads. The major advan- disadvantages of the fluoroquinolones include: car-
tage is that trimethoprim-sulfamethoxazole has lim- tilage toxicity (avoid in pregnancy), drug interac-
ited side effects on the fecal flora. Disadvantages tions, the possibility of a false-positive urine opiate

CHAPTER 23B: URINARY TRACT INFECTIONS 781


test, and that these drugs lower seizure threshold. D. Antibiotics in pregnancy
There is an important new black box warning: fluo- (CRITICAL FOR EXAMINATIONS)
roquinolones increase the risk of tendonitis and ten-
don rupture, especially in patients who are older, Generally considered safe. A few compounds are
who are on steroids, or who have organ transplants. considered generally safe to use in treating pregnant
Fluoroquinolones also exacerbate muscle weakness women (category B). These drugs include: peni-
in patients with myasthenia gravis (avoid in patients cillins (i.e., ampicillin, amoxicillin and penicillin
with a history of myasthenia). V), cephalosporins (i.e., cephalexin, cefaclor and
ceftriaxone) and clindamycin.
Aminoglycosides. Aminoglycosides work by
inhibiting bacterial ribosomal protein synthesis. The Antibiotics to avoid during the third trimester. A
antimicrobial spectrum is primarily against most number of agents should be avoided during the third
Gram-negative rods, including Pseudomonads. Dis- trimester of pregnancy, because of a high rate of
advantages include ear and kidney toxicity, neuro- complications. These agents include the sulfon-
muscular blockade with high levels (avoid in amides (which cause hemolytic anemia in G6PD
patients with myasthenia gravis), and that these deficient patients and neonatal jaundice) and nitro-
drugs are considered pregnancy category D. furantoin (which causes hemolytic anemia due to
immature enzymes; also avoid in patients with
Aminoglycosides: once-daily dosing. The ratio- G6PD deficiency).
nale for single-daily dosing is that proximal renal
tubule uptake is saturable and that ototoxicity is pro- Antibiotics to avoid in pregnancy (Memorize).
portional to total daily aminoglycoside dose. Agents that should be avoided in pregnant women
Results of clinical series document that once-daily include: fluoroquinolones (that are toxic to develop-
dosing results in less renal toxicity, similar ear toxi- ing cartilage), chloramphenicol (causes “gray baby
city, and similar or better bacterial killing. The stan- syndrome”), trimethoprim (causes folate antago-
dard single-daily gentamicin dose is 7 mg/kg, fol- nism, anemia and fetal malformations), ery-
lowed by adjustments using a nomogram. thromycin (causes maternal cholestatic jaundice),
and the tetracyclines (toxic to fetal bones and teeth,
Aztreonam. We will consider aztreonam with the and cause maternal hepatotoxicity).
aminoglycosides because of its aminoglycoside-
like antimicrobial spectrum. However, this drug has Generally considered safe (Category B)
a beta-lactam-type mechanism of action since it
inhibits bacterial cell wall synthesis (like the peni- 1. Ampicillin, amoxicillin, penicillin V
cillins and cephalosporins). The antimicrobial spec- 2. Cephalexin, cefaclor, ceftriaxone
trum includes most Gram-negative rods, including 3. Clindamycin
Pseudomonads. Unlike other beta-lactams, how-
ever, aztreonam provides no Gram-positive antimi- Avoid during third trimester
crobial coverage. The major advantages are that
aztreonam provides an aminoglycoside-type spec- 1. Sulfonamides
trum but not nephrotoxic. There is <1% cross-reac- 2. Hemolytic anemia in G6PD
tivity for penicillin-allergic patients. deficient patients
3. Neonatal jaundice
Vancomycin. Vancomycin works by inhibiting bac- 4. Nitrofurantoin
terial cell wall synthesis, but at a different point than 5. Hemolytic anemia due to
the beta-lactam drugs. The antimicrobial spectrum immature enzymes
includes Gram-positive bacteria, as well as, MRSA 6. Avoid with G6PD deficient patients
(some resistant strains). Disadvantages include the
risks of “red-man syndrome” (histamine effect),
nephro- and ototoxicity (especially if combined
with other nephro- or ototoxic drugs).

782 EDUCATIONAL REVIEW MANUAL IN UROLOGY


5. Antimicrobial Prophylaxis
in Urology

Avoid during Pregnancy (Memorize) A. Standard


1. Antimicrobial administration
1. Fluoroquinolones
2. Cartilage development B. Guidelines
3. Chloramphenicol 1. Asymptomatic bacteriuria
4. “Gray baby syndrome” 2. Genitourinary procedures
5. Trimethoprim 3. Preventing endocarditis
6. Folate antagonism: anemia and
malformations C. Best Practices
7. Erythromycin 1. Patients with joint replacements
8. Maternal cholestatic jaundice 2. Prostate biopsy
9. Tetracyclines 3. GU prostheses
10. Fetal bones/teeth
11. Maternal liver A. One Standard. To this author’s knowledge there
is only 1 standard in this area. If you chose to give
antibiotic prophylaxis for genitourinary tract proce-
dures this should be administered within 1 hour of
the incision. (In Europe, this standard is within 30
minutes.) Administration of vancomycin should
occur within 2 hours of the incision. If prophylactic
antibiotics are not to be administered, the clinician
must document that the patient is not eligible.

B. Guidelines. Three guidelines should be consid-


ered: treatment of asymptomatic bacteriuria, pro-
phylaxis for genitourinary tract procedures and pre-
vention of endocarditis.

This author recommends the Infectious Diseases


Society of America (IDSA) guidelines because of
the rigor with which these were constructed. The
IDSA experts conducted a detailed review of all
prospective randomized controlled trials. These
were assessed individually if the trials were large
enough with meta-analysis for comparable agents.
The IDSA-US Public Health Service Grading was
then used to rank the recommendations. This rank-
ing considers both the level of scientific evidence
from the systemic review of literature (graded from
I [highest] to III [lowest]) and also the strength of
the recommendation from expert clinical judgment
(graded from A: always offer; to E : never offer).

CHAPTER 23B: URINARY TRACT INFECTIONS 783


1. Asymptomatic Bacteriuria. Asymp- f. Treatment decreases bacteremia
tomatic bacteriuria is exceedingly common and sepsis
in urological populations. 1. Variable regimens
2. Irrespective of bacteriuria
Asymptomatic Bacteriuria Prevalence
2. Other GU Procedures
a. Healthy adult women: 2%–5%
Screening and treatment of asymptomatic
b. Pregnant women: 2%–11% bacteriuria if mucosal bleeding anticipated
(A-III).
c. Diabetic women: 7%–9%
Important questions remain unanswered.
d. Elderly nursing home residents: These include the need to define the optimal
5%–50% time for preoperative screening. How long
before the procedure should antibiotics be
e. Spinal cord injury: 50% initiated? How long should we continue
treatment? Should we administer a single
f. Chronic indwelling catheter: 100% preoperative antibiotic dose or continue
treatment for the duration of catheteriza-
Screening of a population is only indicated tion?
if the test results indicate the need for treat-
ment. Despite the high prevalence of bac- 3. Preventing Endocarditis
teriuria in urological patients, routine
screening and treatment is indicated in only The best recommendations are from the
2 populations: pregnant women and certain American Heart Association and have been
urological patients who are about to undergo endorsed by a number of other organizations
invasive procedures. (American Dental Association, IDSA, Pedi-
atric Infectious Diseases Society). These
Urological Procedures: TURP recommendations were based on a very rig-
orous, systematic process and represent a
a. Recommend screening for and dramatic and courageous change from the
treatment of asymptomatic bacteriuria earlier recommendations.
before TURP (A-I)
The critical insight is that infective endo-
b. Culture results prior to procedure carditis is much more likely to originate
(A-III) from random bacteremias associated with
daily activities. In contrast, infective endo-
c. Therapy initiated shortly before carditis rarely occurs from genitourinary
procedure (A-II) tract procedures. An extremely small num-
ber of infective endocarditis cases might be
d. Do not continue unless catheter prevented by prophylaxis. Thus, prophy-
remains in place (B-II) laxis for genitourinary tract procedures is
NOT recommended based solely on an
e. Prospective, randomized increased endocarditis risk.
clinical trials

784 EDUCATIONAL REVIEW MANUAL IN UROLOGY


C. Best Practices C. Higher-risk procedures

A good definition of best practices was provided by a. Stone manipulation


the AUA Guidelines Committee, “Assessment of (includes ESWL)
the literature by the AUA Practice Guidelines Com-
mittee suggested that insufficient information was b. Transmural incision into
available to derive a guideline statement…”. This the urinary tract
group has provided important best practice recom-
mendations for 3 urological patient populations: c. Endoscopic procedures
patients with joint replacements, men undergoing of upper tract
prostate biopsy and patients having surgery for
placement of genitourinary tract prostheses. d. Procedure that includes
bowel segments
1. Urological Patients With Total
Joint Replacements e. Transrectal prostate biopsy

A. Prophylaxis NOT indicated for most f. Entry into the urinary tract
healthy urological patients with (excludes catheterization)
if colonization likely
a. Placement of pins, plates
and/or screws i. Indwelling catheter or
intermittent catheterization
b. Total joint replacements
ii. Indwelling ureteral stent
B. Consider prophylaxis in certain
populations iii. Urinary retention

a. At-risk patients undergoing iv. History of recent or recurrent


higher risk procedures UTIs or prostatitis

b. First 2 years after joint replacement v. Urinary diversion

c. Immunocompromised or D. Suggested prophylactic regimens


immunosuppressed
a. Single systemic quinolone dose
d. Comorbidities
i. Ciprofloxacin 500 mg
i. Previous prosthetic joint
infections ii. Levofloxacin 500 mg

ii. Malnourishment iii. Ofloxacin 400 mg

iii. Hemophilia b. Ampicillin 2 g IV


(or vancomycin 1 g) IV
iv. HIV infection
i. PLUS gentamicin
v. Diabetes mellitus 1.5 mg/kg IV

vi. Malignancy c. Urine should be sterile

CHAPTER 23B: URINARY TRACT INFECTIONS 785


6. Prostatitis Syndromes

2. Men Undergoing Transrectal Prostate Biopsy There are 4 prostatitis syndromes: acute bacterial,
chronic bacterial, chronic prostatitis/chronic pelvic
pain syndrome and asymptomatic inflammatory
prostatitis. We will briefly outline diagnosis and
Organisms Intestine

Patients All clinical management for each condition.

A. Acute Bacterial Prostatitis (Category I).


Acute bacterial prostatitis is characterized
First choice Fluoroquinolone

by an acute, often systemic, illness with


acute UTI symptoms.
Alternatives Aminoglycoside + Metronidazole
or Clindamycin

Etiology. Important etiological bacteria


include: E. coli 65%–80%, other Gram-neg-
Duration <24 hours

ative rods (Pseudomonas, Enterobacter,


Klebsiella, etc) and Enterococcus. The usual
3. Prophylaxis for Implanted route of infection is ascending with reflux of
Urological Prostheses bacteria into the prostatic ducts. Risk factors
include: UTI, presence of a condom or
indwelling catheter, and transurethral
surgery.
Organisms GU tract and skin

Evaluation. Patients usually present with a


Patients All

history acute onset of perineal pain and


obstructive and irritative voiding symptoms.
First choice Aminoglycoside + 1st/2nd generation

Systemic symptoms, such as fever, chills,


Cephalosporin or Vancomycin

malaise and nausea are often present. Physi-


cal exam will often reveal a boggy, hot, very
Alternatives Ampicillin/Sulbactam

tender prostate. It is important to avoid


Ticarcillin/Clavulanate

prostate massage, as this may precipitate


Piperacillin/Tazobactam

bacteremia. It is important to obtain a urinal-


ysis and culture before initiating therapy.
Duration <24 hours

Treatment. In acute bacterial prostatitis,


most antibiotics enter the prostate
parenchyma, unlike chronic bacterial pro-
statitis. For patients who require parenteral
therapy, good choices include the combina-
tion of a beta-lactam drug (i.e., ampicillin or
a first-generation cephalosporin) plus an
aminoglycoside, a second or third genera-
tion cephalosporin, or fluoroquinolone. In
patients who can take oral therapy (or after
resolution of the acute phase in patients
requiring parenteral treatment) a reasonable
choice is a 2–6-week course of trimethoprim
or a fluoroquinolone.

786 EDUCATIONAL REVIEW MANUAL IN UROLOGY


Complications include development of pro- be normal size or enlarged, boggy or not,
static abscess and urinary retention. Pres- tender or nontender. Selected patients
ence of an abscess should be considered in should also have a uroflow study, post-void
patients who do not improve substantially residual urine determination, and cytology.
after 36 hours of therapy. Transrectal ultra-
sound is the optimal diagnostic study. Treatment. The treatment approach is
Transurethral drainage is the standard treat- markedly different for the 2 chronic prostati-
ment. Placement of a small transurethral tis syndromes.
catheter or a suprapubic cystostomy (my
preference) may be necessary for patients C. Chronic Bacterial Prostatitis. Antibiotics
who cannot void. are the cornerstones of therapy. Generally,
patients should receive at least 4–12 weeks
B. Chronic Prostatitis Syndromes. There are of an appropriate agent, directed by culture
2 chronic prostatitis syndromes, chronic results. Usually, this is a fluoroquinolone,
bacterial prostatitis (category II) and chronic trimethoprim-sulfamethoxazole or
prostatitis/chronic pelvic pain syndrome trimethoprim. Surgery should be avoided
(CP/CPPS, category III). unless there are very clear indications.

Evaluation. It is important to distinguish CP/CPPS. These patients present with


chronic bacterial prostatitis from CP/CPPS genitourinary pain and other symptoms but
because these conditions require different have no bacterial pathogens diagnosed by
treatment strategies. This distinction standard methods. Evaluation has been out-
requires careful clinical evaluation employ- lined above. There are 2 CP/CPPS subtypes.
ing either the 4-glass test or the 2-glass test. Category III A (inflammatory CP/CPPS) is
characterized by presence of leukocytes in
The 4-glass test includes culture and the EPS, VB3 or semen. Category III B
microscopy of the following 4 samples (all 4 (noninflammatory CP/CPPS) is character-
get microscopy and culture): VB (1st 10 ml ized by absence of leukocytes in any of
voided urine, urethral), VB2 (midstream these specimens.
urine (bladder)). EPS (expressed prostatic
secretions), and VB3 (1st 10 ml after mas- It is important to recognize that all CP/CPPS
sage, incorporating residual EPS). The 2- treatment approaches are currently off-label
glass test is a somewhat less accurate, abbre- indications. Further, current thinking is that
viated version with 2 samples (both get CP/CPPS is a heterogeneous syndrome that
microscopy and culture): the VB2 and VB3. will likely require multiple treatment
Patients with chronic bacterial prostatitis approaches. To date, clinical trials have not
have at least a 10-fold increase in the con- distinguished III A from III B, or found sim-
centration in the EPS or VB3 of the same ilar outcomes for both subtypes.
bacterial species that caused recurrent UTIs.
A recent systemic review and network meta-
In addition to careful microbiology, patients analysis identified 262 CP/CPPS treatment
should have a history (including the studies. Of these 23 (9%) were eligible for
National Institutes of Health Chronic Pro- inclusion. The most promising therapies
statitis Symptom Index [NIH CPSI] or the compared to placebo were alpha-blockers,
Genitourinary Pain Index [GUPI]). They which had the most studies. There was evi-
should also have a physical examination, dence of publication bias. Other promising
including palpation of the prostate and other therapies were anti-inflammatory medica-
potential pain areas (perineum, groin, coc- tions that had a lesser but measurable
cyx, sphincter, pelvic side walls). It is improvement on selected outcomes. The
important to recognize that the prostate may combination of alpha-blockers with

CHAPTER 23B: URINARY TRACT INFECTIONS 787


7. Unusual Infections

antibiotics was felt to offer the greatest This section considers “unusual infections” includ-
potential impact on clinical symptoms. ing: special types of pyelonephritis, malakoplakia,
However, all controlled studies of antibiotic fungal UTIs, genitourinary and renal tuberculosis.
therapy in CP/CPPS fail to show a differ-
ence compared with placebo. Special pyelonephritis types. There are 2 “special”
categories of pyelonephritis that are seldom seen in
D. Asymptomatic Inflammatory Prostatitis practice, but are routinely encountered on examina-
(Category IV). Patients with category IV tions, emphysematous pyelonephritis and xan-
prostatitis have white blood cells or bacteria thogranulomatous pyelonephritis.
in their EPS, VB3, semen or histologic spec-
imens of prostate biopsy tissue. These
patients do not require treatment unless you
Emphysematous Pyelonephritis

are planning surgery, consider the infection Etiology. Emphysematous pyelonephritis is an


or inflammation as contributing to an ele- acute necrotizing parenchymal and perirenal infec-
vated PSA, or if the infection or inflamma- tion caused by gas-forming organisms. E. coli is the
tion is thought to be contributing to infertil- most common pathogen. Klebsiella spp., Proteus
ity. spp., and other bacteria are encountered less often.

Most patients are diabetic. Presence of renal calculi


is also a predisposing factor, especially in patients
with Proteus UTIs. The gas arises from fermenta-
tion of glucose in tissue to produce CO2. Bacteria
may also ferment necrotic tissue.

Diagnosis and treatment. Because overall mortal-


ity is 20%–40%, prompt diagnosis and treatment is
imperative. On occasion, gas may be visualized on
plain film or abdominal ultrasound. CT scan is pre-
ferred. In addition to showing presence of gas, the
CT may demonstrate presence of an abscess or
hydronephrosis.

Treatment includes broad-spectrum antibiotics,


fluid resuscitation and surgical intervention in most
cases. Most often the surgical procedures include
nephrectomy or percutaneous drainage.

Xanthogranulomatous Pyelonephritis (XGP)

Etiology. XGP is a rare, severe chronic infectious


inflammatory renal mass with accumulation of
lipid-laden foamy macrophages (xanthoma cells).

Diagnosis and treatment. XGP may be diagnosed


on occasion based on radiographic findings or on
pathologic examination of a kidney removed
because of a presumed renal tumor. Pathology
reveals lipid-laden macrophages may resemble
clear cell carcinoma on frozen section.

788 EDUCATIONAL REVIEW MANUAL IN UROLOGY


8. Malakoplakia

Etiology. Malakoplakia was originally described in


the bladder. Malakoplakia can also involve the
Xanthogranulomatous

upper urinary tract, GI system, skin, lung or other


Pyelonephritis Findings

a. Classic radiographic triad organs. The main etiologic theory is that this condi-
tion results from bacterial infection, commonly
i. Unilateral large kidney E. coli. Macrophages do not digest bacteria result-
(diffuse or mass effect) ing in formation of large macrophages, termed von
Hansemann cells, containing bacterial fragments,
ii. Renal pelvis stone termed Michaelis-Gutman bodies (that look like
(may lack hydronephrosis due to targets).
surrounding fibrosis)
Diagnosis and treatment. Malakoplakia is more
iii. Nonfunctioning or poorly common in women older than 50 years of age who
functioning kidney are debilitated or immunosuppressed. Characteris-
tically, they present with the combination of irrita-
b) CT may show cystic areas of necrosis and tive voiding symptoms and hematuria. Cystoscopy
abscess reveals multiple plaques and/or mass lesions.

i. Bear paw sign Treatment is primarily long-term antibiotics to kill


intracellular bacteria. The best choices are usually
Nephrectomy is the usual treatment of XGP. sulfonamides with or without trimethoprim, doxy-
Surgery can prove difficult because of severe cycline, rifampin or a fluoroquinolone. Surgical
inflammation. Try to remove the entire inflamma- excision may be necessary if lesions do not respond
tory mass if possible. to medical management.

CHAPTER 23B: URINARY TRACT INFECTIONS 789


9. Candida UTIs

Although Candida UTIs are not unusual organisms, The clinical presentation of symptomatic
consideration of this subject fits best in this section Candida UTI is indistinguishable from bac-
because treatment is very different from the man- terial UTI. Further, patients with asymp-
agement of bacterial UTIs. tomatic Candida UTI may be unable to
communicate or unable to mount an inflam-
A. Epidemiology. Candida UTIs are increas- matory response. Cultures and microscopy
ing in frequency. Yeasts are cultured from are essential for diagnosis.
urine from <1% of normal adults. Among
hospitalized patients Candida spp. accounts D. Microscopy + Cultures
for 5%–10% of positive cultures. Candida
UTI rates are especially high in burn units 1. Gram stain of centrifuged urine
and neonatal ICUs. The microbiology of
fungal UTIs has changed with non-albicans a. C. albicans and most species
species now accounting for >50% of Can-
dida UTIs. b. Both yeasts and budding
pseudohyphae
B. Pathogenesis. Culture of yeasts from urine
has been associated with increased mortal- 2. C. glabrata: no pseudohyphae
ity, especially in ICU patients. However, this
increased mortality is often not directly a. Potentially highly resistant
related to Candida UTI. Apparently, Can-
dida UTI is a marker for serious underlying 3. CFU/ml NOT helpful for
illness. UTI diagnosis

Fungal UTI risk factors: indwelling E. Diagnostic Imaging


catheters, previous antibiotic treatment, dia-
betes, presence as an impaired host 1. Selected symptomatic and
(steroids, malignancy, malnutrition, etc), or critically ill patients
an abnormal genitourinary tract, (congeni-
tal, neurogenic, diversion, etc). 2. Key for diagnosis of:

Candida UTI may occur by 2 routes: ante- a. Abscesses


grade infection or retrograde infection.
Antegrade infection via the blood stream is b. Obstruction
especially common in neonates. Retrograde
infection, or ascent through the urinary tract c. Fungus balls
is especially common in patients with: uro-
logical abnormalities, catheters or immuno- 3. Ultrasound
suppression.
a. Highly portable for patients
C. Diagnosis. The critical point in evaluation in ICU
of a patient with Candida is the urine to dis-
tinguish among 3 possibilities: a contami- b. TRUS excellent for lower
nated sample, colonization of the urinary urinary tract
tract, or infection of the upper or lower uri-
nary tract. First, repeat the urine culture. Do 4. CT scan most sensitive
not dismiss finding out of hand or begin
empiric treatment

790 EDUCATIONAL REVIEW MANUAL IN UROLOGY


F. Candida UTI drugs. The first clinical issue 5-Fluorocytosine is converted to 5-fluo-
is to treat predisposing conditions or urolog- rouracil by an enzyme (cytosine deaminase)
ical abnormalities, if possible. Then choose in some fungi, minimal or absent in mam-
an appropriate antifungal. malian cells. The main advantage is that this
drug may be administered orally. The pri-
Nystatin is an effective topical antifungal mary disadvantage is that 5-fluorocytosine
that works by binding to sterols in the cell is associated with high resistance rates.
membranes. Some Candida spp. (not albi-
cans) develop resistance. These strains are Echinocandins (caspofungin, etc). These
also resistant to amphotericin B. drugs work by inhibiting synthesis of beta-
1,3-glucan, an essential component of Can-
Amphotericin B is very effective and con- dida cell wall. They have efficacy similar to
sidered the gold standard. Amphotericin B amphotericin B for invasive Candida infec-
binds sterol (ergosterol) in fungal cell mem- tions, but are usually not effective for other
brane and cholesterol in mammalian cells. fungal species. These drugs have few side
Unfortunately, amphotericin B has substan- effects (other than histamine release in 2%),
tial side effects. Adverse effects (less with but must be administered intravenously.
liposomal formulations) include: fever, rig-
ors (steroids or NSAIDS may prevent; usu- G. Candida UTI: Who to treat? Often the
ally no need to stop treatment), depletion of most difficult decision is whether or not to
K, Mg, Ca and Na, kidney and liver toxicity, initiate antifungal therapy. Below we briefly
and phlebitis. outline our current recommendations.

Imidazoles and triazoles inhibit fungal P- 1. Asymptomatic patients


450 enzymes to decrease synthesis of
steroids for cell membranes. Topical agents a. Most patients improve if treat
include clotrimazole, miconazole, etc. Flu- predisposing conditions
conazole may be administered PO or IV.
(Some Candida spp. are resistant.) b. Antifungal therapy has NO
Voriconazole is a new triazole. demonstrated benefit in the
absence of renal infection
Ketoconazole has a similar mechanism of
action to other agents in this category. It 2. Symptomatic patients
inhibits mammalian P-450 enzymes and
adrenal androgens (rarely corticosteroids). a. Most patients respond to
Thus, treatment with ketoconazole may PO fluconazole
change drug metabolism leading to high lev-
els of terfenadine, astemizole, cisapride, b. Concentrated in urine (contrast
cyclosporine and phenytoin. Because keto- to other antifungal azoles)
conazole achieves low urine levels, it repre-
sents a poor choice for treatment of kidney c. Most Candida species sensitive
or bladder infections.
d. Well tolerated
Because ketoconazole requires stomach
acid for absorption, patients will have low e. Inexpensive
levels if they are also on proton pump
inhibitors, H2 blockers, or if they have
achlorhydria. Ketoconazole has significant
potential for liver toxicity, which is unusual
but can be fatal.

CHAPTER 23B: URINARY TRACT INFECTIONS 791


10. Tuberculosis

H. Refractory Candida Cystitis A. Epidemiology. Most patients have AIDS or


are foreign-born.
1. Consider PO flucytosine
B. Pathogenesis. Most cases are caused by
a. Concentrated in urine Mycobacterium tuberculosis. Humans are
the only host. Infection is acquired by
b. Potential bone marrow toxicity inhalation. Bacteria multiply in lungs, then
disseminate through the body for about 4
c. Resistance weeks. The cellular immune response con-
tains infection in most cases (PPD test
2. Amphotericin B ≥1 IV dose becomes positive). Most competent hosts
show no clinical disease. Active disease
a. Prolonged urinary excretion occurs when the host is compromised by
factors such as: AIDS, steroids, other
b. Potent antifungal activity immunosuppression, debilitation, trauma,
diabetes, etc.
c. Liposomal forms
C. Genitourinary tuberculosis. Urological
i. Less toxic and less infections are usually considered as either
effective in urinary tract renal or genital tuberculosis. Renal (geni-
tourinary) occurs when the kidneys are colo-
3. Amphotericin B bladder irrigation nized during the initial hematogenous dis-
semination. Renal foci are activated when
a. 50 mg/1 L water at 1 L/day host compromised. The standard urological
teaching is that ureteral and bladder tubercu-
b. High relapse rate losis always originates from renal tuberculo-
sis.
c. Patients with upper
tract infections D. Genital (epididymal) is said to originate
from the initial hematogenous spread of
d. Recurrent cystitis with infection (debated). The most common site
fluconazole-resistant organism is the globus minor because it has the great-
est blood supply. Testicular tuberculosis
4. Surgical procedures originates from local spread from the epi-
didymis. The critical clinical point is to be
a. Prostatic abscess: sure to check for upper tract involvement in
transurethral drainage any patient with genital tuberculosis.

b. Epididymo-orchitis: E. Progression. Infection of PMNs and


orchiectomy macrophages, progresses to formation of
caseating granulomas. Subsequent forma-
c. Renal infections tion of scar tissue results in: calyceal stric-
ture, necrosis (calyceal destruction) and cal-
i. Abscess: cifications (common sign of renal tuberculo-
drainage or infection sis). Decreased blood flow may cause
hypertension. Ureteral strictures occur most
ii. Fungus balls: drainage, commonly at the ureterovesical junction. In
removal, irrigation the bladder, changes originate at the ureteral
orifices (stricture or reflux).

792 EDUCATIONAL REVIEW MANUAL IN UROLOGY


F. Diagnosis. Symptoms are often minimal H. Treatment
and vague, such as: frequent painless mic-
turition, malaise, weight loss, lethargy, low- Most patients respond to antimicrobial ther-
grade fevers, painful epididymal or scrotal apy. Rare cases require surgical procedures
swelling, or hematospermia.
1. Antituberculosis antibiotic therapy
Urine studies are critical for diagnosis. Ster-
ile pyuria is classic, but this is present in a. 6 months (9 if slow to respond)
only 80%. Three to five early morning urine
samples for acid-fast bacterial culture are i. First two months 4 drugs:
often required. More rapid PCR-based test- isoniazid (INH),
ing may soon be available. rifampin, pyrazinamide,
ethambutol
Other diagnostic tests include the PPD skin
test. However, a positive test does not indi- ii. Next 4 months: INH and
cate active infection but only previous infec- rifampin if M tuberculo-
tion or BCG vaccination. A positive test is sis isolate is subsequently
not specific for diagnosis of genitourinary found to be sensitive to
tuberculosis. Cystoscopy is rarely indicated. both
Bladder biopsy is not indicated in patients
with active tuberculosis. b. CDC guidelines
www.cdc.gov/mmwr
G. Genitourinary Tuberculosis Radiology
2. Tuberculosis antibiotics
1. Plain films: calcifications
a. First-line (reviewed)
2. CT scan preferred
i. INH
a. Good view of mass or
nonvisualized kidney ii. Rifampin

b. Shows other areas besides iii. Pyrazinamide


the GU tract
iv. Ethambutol
KEY FOR EXAMINATIONS
b. Second-line
1. Irregular “moth-eaten” calyces
2. Hydrocalyx, hydronephrosis, i. Cycloserine, ethion-
hydroureter amide, streptomycin,
3. Superiorly displaced UPJ amikacin, kanamycin,
(“hiked-up pelvis”) capreomycin, para-amino
4. Nonvisualization of calyx from salicylic acid, lev-
infundibular stenosis or entire kidney ofloxacin
(autonephrectomy)
5. Ureteral scars Isoniazid (INH) works by inhibiting synthesis of
a. Most common site: distal ureter mycolic acids (cell wall). INH kills rapidly dividing
b. Multiple scars almost pathog- bacteria. Adverse reactions include: liver toxicity
nomonic (higher if also drinking alcohol), neurotoxicity (give
pyridoxine supplement), and INH may increase lev-
els of phenytoin and carbamazepine.

CHAPTER 23B: URINARY TRACT INFECTIONS 793


11. Summary

Rifampin inhibits bacterial RNA polymerase. This focused review has considered the UTI issues
Rifampin is effective against both dividing and that are most likely to appear on board certification
semi-dormant bacteria. Serious adverse reactions and maintenance of certification examinations. We
are uncommon, but all patients get orange discol- started with critical definitions and a practical UTI
oration of body fluids (orange underpants). classification. Attention was then devoted to UTI
epidemiology and pathogenesis. Considerable time
Drug interactions are common because rifampin was spent on antibiotics, guidelines and best prac-
induces cytochrome P450 enzymes. Thus, levels of tices for prophylaxis, because these areas often
any drugs are reduced to the subtherapeutic range, appear on examinations. Finally, we considered
including anti-HIV protease inhibitors and non- diagnosis and management of prostatitis syndromes
nucleoside reverse transcriptase inhibitors. and unusual infections that are seen more often on
examinations than in the usual office setting.
Pyrazinamide has an unknown mechanism of
action. It is effective against both dormant and semi-
dormant bacteria. Although serious reactions are
uncommon, 40% of patients develop arthralgias
(most respond to NSAIDs). Because pyrazinamide
treatment increases uric acid levels, gout is a con-
traindication. The European guidelines recommend
that patients on pyrazinamide also receive a xan-
thine oxidase inhibitor.

Ethambutol has an unknown mechanism of action.


It is effective against actively dividing bacteria. The
major adverse reaction is retrobulbar neuritis.
Therefore, it is important to follow visual acuity and
red-green color discrimination.

Surgery is required in a few patients with genitouri-


nary tuberculosis. Patients with genitourinary tuber-
culosis may require nephrectomy for persistent
symptoms, extensive disease with hypertension or
UPJ obstruction or coexisting renal carcinoma. The
role of partial nephrectomy is controversial.

For patients with ureteral strictures, an indwelling


stent represents the best initial treatment. Steroids
may be indicated for patients who do not improve
after several weeks. Reimplantation of the ureter,
stenting plus medical management may be required
if not successful. A psoas hitch or Boari flap may be
required to facilitate the ureteroneocystostomy. An
occasional patient with a severely contracted blad-
der may require augmentation cystoplasty after
intensive medical therapy.

Rare patients may require surgery for genital tuber-


culosis. Epididymectomy may be needed if a lesion
fails drug therapy. Patients may also require
orchiectomy for lesions suspicious for testis cancer.

794 EDUCATIONAL REVIEW MANUAL IN UROLOGY


Chapter 24:
Renal and
Ureteral Trauma
Allen F. Morey, MD, FACS

Contents

1. Renal Trauma

2. Ureteral Trauma

3. References

4. Questions

CHAPTER 24: RENAL AND URETERAL TRAUMA 795


1. Renal Trauma

• Traumatic thrombosis of the renal artery


(Figure 2) or its branches occurs because the
Diagnosis of Renal Injury

Mechanism of Injury media and adventitia of the renal artery are more
Blunt trauma accounts for >90% of renal injuries elastic than the intima. An intimal tear produces
(usually motor vehicle collisions, vehicle pedestrian thrombosis andocclusion with complete or seg-
accidents, sports injuries), and these are usually mental renal ischemia. No therapy is warranted
contusions or minor lacerations amenable to nonop- acutely if there is a normal contralateral kidney.
erative management.
Major renal injuries are usually associated with
• Blunt trauma—Gross hematuria or microhema- other major injuries of the chest and/or abdomen
turia plus shock warrants renal imaging. (average ≥3).

• Microhematuria without shock, after blunt trauma, Children


is indicative of renal contusion and does not war- • More susceptible to renal trauma than adults: less
rant immediate imaging in the absence of clinical cushioning by perirenal fat, underdeveloped rib
indicators. cage, less protection by muscles of flank and
abdomen. Abnormal kidneys, such as those with
• Penetrating trauma (GSW or stab wound to congenital hydronephrosis due to UPJ obstruction,
abdomen, chest or pelvis) is associated with a are more prone to major injury in the context of
much higher degree of renal damage than blunt seemingly trivial trauma.
injuries (see Figure 1)—any degree of hematuria
(micro or gross) warrants renal imaging. • Hypotension not reliable in children: outpouring
of catecholamines can maintain BP in spite of sig-
Clinical Indicators of Major Renal Injury nificant blood loss.

• Rib or vertebral body fractures • Consider UPJ disruption (more prevalent in chil-
dren because of their increased flexibility and
• Direct blow to flank renal mobility; see Table 1)—remember to look at
delayed images to check for intact ureter.
• Abdominal mass or tenderness

• Flank pain or ecchymosis

Amount of hematuria does not correlate to


Figure 1

severity of renal injury


Left renal stab wound with grade 3 injury

• Minor, completely benign renal injuries


successfully managed nonoperatively

(contusion) often present with gross hematuria.

• While deep renal lacerations do tend to


produce gross hematuria, major renal injury
(or renovascular injury) may occur in the absence
of hematuria as well. UPJ disruption or arterial
thrombosis may also occur without hematuria, but
these injuries are almost always due to rapid decel-
eration injury or multisystem trauma, which
should independently warrant abdominal imaging.

796 EDUCATIONAL REVIEW MANUAL IN UROLOGY


Table 1

Suspected Renal Injury?

Indications for Imaging

Blunt Trauma: Adults

Gross hematuria

Microhematuria (5+ RBC/HPF) plus hypotension (systolic BP <90 mm Hg)

Rapid deceleration

Blunt Trauma: Children

Gross hematuria or significant microhematuria (50+ RBC/HPF)

Multisystem trauma

Penetrating Trauma

All with any degree of hematuria after penetrating abdominal, flank or thoracic injury.

Clinical suspicion based on entrance or exit wound (regardless of hematuria)

When is Immediate Imaging Not Required (Significant Renal Injury <2%)?

Adults with isolated microhematuria (no evidence of shock)

Children with isolated insignificant microhematuria (<50 RBC/HPF) without signs of multisystem trauma

Contrasted Renal CT: Imaging Study of Choice • Perirenal hematoma size >3.5 cm is associated
• 2-phase study with initial vascular/cortical phase with the need for intervention (angioembolization
roughly 30 seconds after IV contrast infusion, sec- or surgery).
ond (delayed) phase scan at 10 minutes to docu-
ment the findings of perirenal or ureteral contrast • Beware that rapid sequence helical CT scans may
extravasation. miss urinary extravasation without delayed cuts.
Repeat CT imaging for grade 4 injuries is war-
• Advantages: excellent anatomic detail, depth and ranted after 48 hours due to the potential for uri-
location of renal laceration; magnitude of perirenal noma.
hematoma or urinoma; associated abdominal
injuries; presence and location of contralateral kid- • IVP has extremely low yield and rarely alters
ney; appropriate for blunt or penetrating trauma in patient management in blunt trauma.
stable patients (Figure 1).
• Ultrasound not reliable for imaging renal trauma.

CHAPTER 24: RENAL AND URETERAL TRAUMA 797


• If immediate laparotomy is otherwise unnecessary,
a conservative urological approach is selected ini-
Figure 2

tially. Observation alone is appropriate for all


minor renal injuries (Grade 1–3), which will
Traumatic thrombosis of left renal artery

nearly always resolve spontaneously.


results in renal ischemia

• The presence of an expanding or pulsatile


hematoma in an unstable patient should warrant
immediate renal exploration. If laparotomy is
undertaken, relative indications for renal explo-
ration include major renal injuries (Grade 4+) or
when renal imaging is absent or inconclusive.

• Increased perirenal hematoma size (>3.5 cm),


presence of intravascular contrast extravasation
into hematoma, and presence of medial/complex
renal lacerations are radiographic signs on CT
which are associated with need for intervention
(such as therapeutic angioembolization) to control
Patient remained stable without intervention

ongoing renal bleeding.


• Single Shot IVP. Single shot IVP is best used to
Cortical Rim Sign
guide exploration of penetrating injuries (i.e., soli-
The cortical rim sign is a radiographic finding asso-
tary or ectopic kidney) or to confirm nonoperative
ciated with acute renal arterial thrombosis, (grade 4
management of blunt injuries. Unstable patients
injury) due to peripheral perfusion from capsular
selected for immediate operative intervention (and
collateral vessels and found at least 8 hours after the
thus unable to have a CT scan) should undergo one-
time of injury. A conservative approach is warranted
shot IVP. A bolus IV injection of 2 cc/kg contrast fol-
since renal salvage is unlikely after prolonged
lowed by single 10-minute film is helpful in staging
ischemia of this duration.
patients with multiple penetrating abdominal
wounds or in the presence of unexpected retroperi-
Forniceal Rupture
toneal hematoma discovered during trauma laparo-
Forniceal rupture is a benign finding after blunt
tomy. If single shot IVP film reveals an obscured
trauma which may masquerade as a major renal
renal outline, this suggests injury.
injury because it produces medial urinary extravasa-
tion on CT scan (see Figure 3). In forniceal rupture,
no therapy is warranted if renal laceration is absent,
perirenal hematoma is absent and the ureter is well
Injury Classification and Interventions

The American Association for the Surgery of


visualized.
Trauma Renal Injury Scaling System
The American Association for the Surgery of
Therapeutic Angioembolization
Trauma renal injury scaling system is used widely
Therapeutic angioembolization is now recognized
with well-established prognostic value. Injury clas-
as safe and effective, as well as, the most common
sification is accomplished by either abdominal CT
method of intervention for management of renal
or direct renal exploration (see Table 2). The odds
injuries This strategy is generally employed in sta-
of renal surgery are increased by 15-fold for every
ble patients who demonstrate persistent bleeding
increase in grade.
from a segmental arterial injury. More significant
injuries require operative intervention to prevent
life-threatening hemorrhage. Some patients may
need further intervention, such as surgery or stent
placement.

798 EDUCATIONAL REVIEW MANUAL IN UROLOGY


Renal Exploration After Trauma Is Undertaken
Selectively
Table 2

Goals of renal exploration are either: a) treatment of


major renal injuries; or b) evaluation of suspected
AAST Renal Injury Grading Scale

renal injury when detailed imaging is unavailable.


Unnecessary or prolonged renal exploration must
be avoided in the critically ill patient due to risk of
Grade 1

releasing perirenal tamponade. The only absolute


indication for renal exploration is hemodynamic
Contusion or subcapsular hematoma (no laceration)

instability from renal injury with associated find-


ings such as large, expanding or pulsatile retroperi-
Grade 2

toneal hematoma. Operative assistance with a vas-


cular or trauma surgeon is advised in the setting of
Cortical laceration <1 cm deep, no extravasation

Grade 3 life-threatening renal vascular injuries.

GU Management of Renal Injury


GU management of renal injury is influenced, but
Cortical laceration >1 cm, no extravasation

not dictated, by the decision of trauma surgeons to


explore or observe associated abdominal injuries. It
Grade 4

is perfectly acceptable to decline exploration of an


injured kidney even when the abdomen is open if
Laceration: >1cm deep, into collecting system, or

radiographic features are indicative of an injury that


Vascular: Thrombosed renal artery or segmental

may be safely observed. Unnecessary renal explo-


vein injury

Grade 5 ration may result in unnecessary nephrectomy.

Associated/Predictive Features of Failed


Nonoperative Management
Laceration: Shattered kidney, or

• Increasing AAST Organ Injury Scale grade


Vascular: Renal pedicle avulsion

• Nephrectomy is performed in 4% of blunt and


21% of penetrating injuries
Figure 3

• Gunshot wounds often produce extensive tissue


Forniceal rupture and renal contusion in

destruction via blast effect; clinical course is often


solitary right kidney after blunt trauma

unpredictable and usually warrants exploration


unless imaging suggests renal injury is minor.
Gunshot wounds are 6 times more likely to result
in nephrectomy than stab wounds.

• Transfusion requirement

• Presence of shock at admission

• Nonrenal abdominal operation or major


abdominal vascular injury

• Stab wounds of the abdomen and flank are


increasingly being successfully managed nonop-
eratively (roughly 50%), particularly if the
entrance wound is posterior to the anterior axillary
line.
CHAPTER 24: RENAL AND URETERAL TRAUMA 799
Renal Exploration and Reconstruction Renal vascular injuries require prompt and efficient
Via Midline Transabdominal Incision treatment. In the event of traumatic renal artery
Repair of major visceral and vascular injuries other occlusion, arterial reconstruction may be performed
than those involving the kidney should usually be via end-to-end anastomosis, autotransplantation or
performed first. If renal hemorrhage is massive and various graft techniques—reconstruction is impera-
life-threatening, the kidney must be explored ini- tive in the setting of vascular occlusion of a solitary
tially and all other injuries should be explored sec- kidney. These maneuvers must be performed within
ondarily. 3 hours of the time of injury to be successful.
Observation alone is appropriate for cases having a
Early Vascular Control Proximal to Injury normal contralateral kidney and no evidence of
Early vascular control proximal to injury has been renal hemorrhage. Renal vein injuries may be man-
associated with increased rates of renal salvage. aged by simple ligation or direct repair. Cooling of
Access to the renal vascular pedicle is best obtained the kidney during vascular reconstruction is not
through the posterior parietal peritoneum, which is appropriate during the trauma setting. In the event
incised over the aorta, just medial to the inferior of penetrating injury to the renal pedicle, immediate
mesenteric vein. The aorta is traced superiorly until nephrectomy is often the best solution in the pres-
the left renal vein is identified crossing anteriorly. ence of a normal contralateral kidney.
The left renal vein is then elevated, exposing both
underlying renal arteries. Vessel loops are placed Nephrectomy is performed as a life-saving maneu-
around the vessels supplying the injured kidney to ver undertaken in the event of irreparable renal lac-
facilitate occlusion, should the need arise. eration or pedicle injury in the setting of hemody-
namic instability. As an alternative to immediate
Once control of the vessels is obtained, the colon is nephrectomy, if the patient is hypothermic, acidotic
reflected medially to expose the kidney. Should a and coagulopathic, a damage control approach may
great deal of blood loss be encountered, immediate be employed with initial packing of the renal fossa
vascular control may be obtained by occlusion of and subsequent pack removal, irrigation and repair
the ipsilateral renal artery. Warm ischemia time after correction of metabolic disturbances in
should be limited to 30 minutes. Once the kidney is selected cases.
exposed, the hematoma is swept away and the entire
surface and collecting system of the kidney are
inspected systematically. Renal lacerations are
Complications of Renal Trauma

debrided and oversewn with absorbable capsular Urinary Extravasation After Blunt Trauma
sutures. Large segmental vessels are suture ligated Urinary extravasation after blunt trauma usually
and the exposed collecting system is closed primar- resolves spontaneously and warrants no specific
ily with fine, absorbable sutures. Hemostasis is immediate therapy; if persistent after 3+ days,
facilitated by the application of absorbable collagen ureteral stent placement with a Foley catheter may
or gelatin sponge bolsters. be considered, especially in the presence of a large
collection, ileus, fever and rising WBC count. Per-
Once the bleeding has been controlled, the open cutaneous drainage of urinoma may be helpful.
margins of the kidney are reapproximated and the
renal capsule closed. If polar lacerations produce • Presence of >25% devitalized renal tissue with
large areas of devitalized tissue, heminephrectomy extravasation has a higher risk of complications
is the best course of action. The area of reconstruc- and has been noted as a relative indication for
tion is covered by perirenal fat or a pedicle flap of exploration, although perirenal abscess or urinoma
omentum in order to insure against urinary leakage. are now usually well managed by percutaneous
A drain is placed in the dependent portion of the drainage.
wound.

800 EDUCATIONAL REVIEW MANUAL IN UROLOGY


2. Ureteral Trauma

Persistent/Delayed Retroperitoneal Bleeding


Delayed retroperitoneal bleeding may occur within
Injury Detection (see Table 3)

several weeks of an injury and become life-threat- Diagnosis of Ureteral Injury


ening. Arteriovenous fistulas present with delayed Ureteral injury may be difficult to diagnose, since
onset of significant gross hematuria; audible bruit hematuria and retroperitoneal hematoma may be
may be noted on abdominal auscultation. Penetrat- absent (in about one-third of cases). Most of these
ing injuries account for most cases of renal AV fis- injuries occur in the context of penetrating abdomi-
tula. nal injury or in conjunction with major abdominal
or pelvic surgery.
• Arteriography: Use indicated in stable patients
with delayed or persistent bleeding from a renal • Although no single test reliably excludes a ureteral
laceration or stab wound (and no other indication laceration, a CT with 10-minute delayed films is
for laparotomy), in conjunction with selective likely to demonstrate extravasation (see Figure 4).
embolization of segmental vessels in stable If IVP is obtained, delayed films are necessary to
patients with minimal devitalized tissue (<20%). establish the diagnosis; IVP is notorious for its
Not appropriate for unstable patients or those with lack of accuracy in detecting ureteral lacerations
extensive devitalized tissue. Percutaneous angio- acutely. Retrograde pyelography is excellent but
graphic embolization is the initial treatment of often impractical in the setting of acute trauma.
choice for symptomatic AV fistulas (80% effec-
tive); surgical correction may be required for • Missed or delayed diagnosis carries with it sub-
larger fistulas. stantial morbidity and mortality. A high index of
suspicion is required to detect contusions, espe-
Hypertension After Renal Injury cially if there is a bullet path near the ureter. A pen-
Although cortical scars often develop after high etrating injury to common iliac vessels or bladder
grade renal injuries, hypertension after renal injury should arouse suspicion of ureteral injury.
is rare, but may occur either acutely as a result of
external compression (Page kidney) or may be renin • Direct inspection during laparotomy is ideal when
mediated as a late complication. Medical manage- ureteral injury is suspected. Antegrade
ment is preferable and usually successful. If medical (intrapelvic) or retrograde (intraureteral) injection
control proves difficult, surgical correction or of methylene blue or indigo carmine with gentle
nephrectomy are indicated. Nuclear renal scans are ureteral occlusion may be used to distend the
useful as a means of documenting recovery of func- ureter and identify any leaks. If normal efflux is
tion after trauma. seen from the ureteral orifice during cystotomy,
ureteral injury is unlikely.

UPJ Disruption
Radiographic hallmarks of diagnosis on CT are a
medial and periureteral pattern of contrast extrava-
sation with nonvisualization of ipsilateral distal
ureter on delayed films.

CHAPTER 24: RENAL AND URETERAL TRAUMA 801


iliac artery. For delayed repairs, a Gibson incision
may be easily extended up or down to allow renal or
Table 3

Ureteral Injuries at a Glance bladder mobilization as needed.

Suggested Types of Ureteral


Stricture/Injury Repair
Ureteroureterostomy. Minimal debridement; direct
1. High index of suspicion warranted—often not
associated with hematuria

spatulated repair over ureteral stent is preferred for


cases with short defect in abdominal ureter. The
2. Rarely isolated injury, nearly always iatrogenic or

goal of primary repair is tension-free, watertight and


due to penetrating trauma

end-to-end anastomosis. Fine, absorbable sutures


(such as 5-0 PDS) are best. Generally avoided
3. Often missed by IVP or rapid sequence CT—

below iliac vessels in lieu of ureteral reimplantation,


remember delayed films!

due to less robust ureteral blood supply distally.


Usually performed in the acute setting of iatrogenic
4. Direct inspection during laparotomy is best when

or penetrating traumatic injury with short (<1 cm)


ureteral injury is suspected

defect. When combined with downward


nephropexy, this may bridge gaps of up to 3–4 cm.
Management

Immediate diagnosis and repair is best, whenever


possible, whether in the setting of iatrogenic liga- Psoas Hitch and Reimplant. Recommended proce-
tion, laceration or in the setting of external trauma. dure for most extensive mid and distal
Delayed diagnosis often occurs, however, and thus injuries.Bladder is mobilized aggressively and
requires individualized treatment based on patient anchored posteriorly over iliac vessels to psoas
condition and length of delay. minor tendon on medial aspect of muscle. (Beware
of genitofemoral nerve laterally.)
• <5 days, consider immediate repair
Requires normal bladder capacity and elasticity;
• ≥5 days since injury, sepsis, debilitated patient. psoas hitch alone should easily reach to level of L-5.
Best to drain urinoma percutaneously and place a May be combined with downward nephropexy and
nephrostomy tube or ureteral stent in conjunction bladder flap to reach higher levels of the ureter, even
with delayed elective repair up to the renal pelvis. A wide-caliber, tension-free,
stented and refluxing ureteral reimplant anastomo-
Ureteral Contusion sis is preferred Bladder hitch should be done first,
Blast injury of ureter from gunshot wounds may then reimplant. Direct ureteral reimplant may be
appear intact during initial laparotomy, but may be performed without psoas hitch when distal ureter is
prone to delayed leak when significant contusion injured near the bladder.
occurs. If ischemia of ureteral wall appears pro-
nounced, consider focal defect resection with Transureteroureterostomy. Best reserved only for
stented primary ureteroureterostomy. Because long ureteral defects in the mid and lower ureter;
endoscopic stent placement is often impractical dur- complicated by severe scar, abscess, extravasation
ing trauma laparotomy, empiric ureteral stent place- or delayed diagnosis. Should not be used in patients
ment through a small cystotomy is prudent when with history of stone disease or advanced pelvic
ureteral contusion is noted after trauma. malignancy. Additional relative contraindications
include urothelial malignancy, retroperitoneal fibro-
Ureteral Exploration sis, chronic pyelonephritis and abdominopelvic
Midline incision is the most versatile, allowing for XRT.
all types of repair from ureteroureterostomy to auto-
transplantation to nephrectomy. This minimizes
ureteral mobilization, which will preserve blood
supply. It is easily located crossing the common

802 EDUCATIONAL REVIEW MANUAL IN UROLOGY


Ureterocalicostomy. Recommended after failed
UPJ repair where proximal renal pelvis (intrarenal)
Urethral Reconstruction Case Examples : 1

cannot be engaged. Ureter is sewn to dilated lower


pole calyx after resection of overlying parenchyma.
Damage control management of ureteral

Consider spiral flap repair using renal pelvis in


transection from GSW in unstable patient

cases having redundant extrarenal pelvis.

Ileal Interposition. Useful for delayed reconstruc-


tion of extensive ureteral defects, especially in pres-
ence of small capacity bladder when Boari bladder
flap is not appropriate. As a general rule, it is best to
use other urinary tract structures for ureteral recon-
struction preferentially over bowel segments due to
potential risks of mucous production, infection,
malignancy and bowel complications. Candidates
for ileal ureter should have normal renal function
and no evidence of bladder outlet obstruction.

Damage Control. In unstable patients, delayed


Long ureteral stent sewn into upper ureter and

reconstruction is often advised after initial stabiliza-


brought out to skin; repaired via direct ureter-

tion by ureteral ligation over feeding tube or a long


oureterostomy next day after resuscitation in ICU

(90 cm) single J stent brought out to skin. Delayed


ureteral reconstruction may then be performed
when the patient is stable.

Autotransplant. Rarely needed, but is an alternative


Urethral Reconstruction Case Examples: 2

to nephrectomy in the event of extensive uretera


injuries, such as ureteral avulsion consequent to
Psoas Hitch Ureteral Reimplant

ureteroscopic extraction of impacted upper tract


stone.
2a. Pre-op nephrostogram:
mid-ureteral stricture

CHAPTER 24: RENAL AND URETERAL TRAUMA 803


Urethral Reconstruction Case Examples: 2 Urethral Reconstruction Case Examples: 4

Psoas Hitch Ureteral Reimplant Transureteroureterostomy

2b. VCUG s/p psoas hitch reimplant 4a. Pre-op mid-ureteral stricture

Urethral Reconstruction Case Examples: 3 Urethral Reconstruction Case Examples: 4

Pre-op nephrostogram: Right ureteral Transureteroureterostomy


stricture at L3 level repaired via downward
nephropexy + Boari bladder flap 4b. Antegrade nephrostogram s/p TUU

804 EDUCATIONAL REVIEW MANUAL IN UROLOGY


Urethral Reconstruction Case Examples: 5 Urethral Reconstruction Case Examples: 6

UPJ stricture s/p 2 robotic pyeloplasties— Ileal Ureter


repaired via ureterocalcycostomy
6b. Post-op nephrostogram showing patent
ileal ureter

Urethral Reconstruction Case Examples: 6

Ileal Ureter

6a. Pre-op nephrostogram showing exten-


sive right ureteral stricture

CHAPTER 24: RENAL AND URETERAL TRAUMA 805


3. References

1. McAninch JW, Carroll PR, Klosterman PW. 11. Brandes S, Coburn M, Armenakas N, McAn-
Renal reconstruction after injury. J Urol. inch JW. Diagnosis and management of
1991;145:932-937. ureteric injury: An evidence-based analysis.
BJU Intl. 2004;94:277-283.
2 Miller KS, McAninch JW. Radiographic
assessment of renal trauma: Our 15-year expe- 12. Elliott SP, McAninch JW. Ureteral injuries
rience. J Urol. 1995;154:352. from external violence: The 25-year experi-
ence at San Francisco General Hospital.
3. Morey AF, Bruce JE, McAninch JW. Efficacy J Urol. 2003;170:1213-1216.
of radiographic imaging in pediatric blunt
renal trauma. J Urol. 1996;156:2014-2018. 13. Wright JL, Nathens AB, Rivara FP,Wessels H.
Renal and extrarenal predictors of nephrec-
4. Boone TB, Gilling PJ, Husmann DA. Uretero- tomy from the national trauma data bank.
pelvic junction disruption following blunt J Urol. 2006;175:970-975.
abdominal trauma. J Urol. 1993;150:33-36.
14. Buckley JC, McAninch JW. Selective manage-
5. Perez-Brayfield MR, Keane TE, Krishnan A, ment of isolated and nonisolated grade IV
Lafontaine P, Feliciano DV, Clarke HS. Gun- renal injuries. J Urol. 2006;176:2498-2502.
shot wounds to the ureter: A 40-year experi-
ence at Grady Memorial Hospital. J Urol. 15. Dugi DD 3rd, Morey AF, Gupta A, Nuss GR,
2001;166:119-121. Sheu GL, Pruitt JP. American Association for
the Surgery of Trauma grade 4 renal injury
6. Brown SL, Hoffman DM, Spirnak JP. Limita- substratification into grades 4a (low risk) and
tions of routine spiral computerized tomogra- grade 4b (high risk). J Urol. 2010;
phy in the evaluation of blunt renal trauma. 183(2):592-597.
J Urol. 1998;160:1979-1981.
16. Mauck RJ, Hudak SJ, Terlecki RP, Morey AF.
7. Armenakas NA, Duckett CP, McAninch JW. Central Role of Boari Bladder Flap and Down-
Indications for nonoperative management of ward Nephropexy in Upper Ureteral Recon-
renal stab wounds. J Urol. 1999;161:768-771. struction. J Urol. 2011;186(4):1345-1349.

8. Morey AF, McAninch JW, Tiller BK, Duckett


CP, Carroll PR. Single shot intraoperative
excretory urography for the immmediate eval-
uation of renal trauma. J Urol. 1999;161:1088-
1092.

9. Santucci RA, McAninch JW, Safir M, Mario


LA, Service S, Segal MR. Validation of the
American Association for the Surgery of
Trauma organ injury severity scale for the kid-
ney. J Trauma. 2001;50:195-200.

10. Al-ali M, Haddad LF. The late treatment of 63


overlooked or complicated ureteral missile
injuries: The promise of nephrostomy and role
of autotransplantation. J Urol. 1996;156:1918-
1921.

806 EDUCATIONAL REVIEW MANUAL IN UROLOGY


4. Questions

1. A1-cm segment of left ureter is lacerated com- 4. A stab wound victim is found to have a grade 2
pletely just above the iliac vessels during an laceration of the lateral left kidney on abdomi-
elective left colectomy. The injury is recog- nal CT with a small perirenal hematoma.
nized intraoperatively, the tissues appear Laparotomy is performed due to bleeding from
viable and the patient is stable. The best choice a concomitant splenic injury which is easily
for management is: controlled. The patient has received 2 units of
blood and is now stable. What is the best
A. Transureteroureterostomy over a stent course of action for the urologist?

B. Ureteroneocystostomy A. Immediate nephrectomy

C. Ileal ureter interposition B. Intraoperative one-shot IVP

D. Ureteroureterostomy over a stent C. Cystoscopy with retrograde pyelogram

E. Ureteral ligation with percutaneous D. Observation


nephrostomy and delayed repair
E. Renal angiography with super selective
2. What is the best technique to evaluate micro- embolization
hematuria in a stable patient with a transtho-
racic gunshot wound? 5. A victim of a posterior stab wound is found to
have an isolated medial, lower pole, grade 4
A. Abdominal sonography right renal laceration on abdominal CT. No
other abdominal injuries were identified and
B. Intravenous pyelography he has a moderately large (4 cm) perirenal
hematoma surrounding the renal injury and
C. Intraoperative single shot IVP an otherwise viable kidney. The patient has
received 3 units of blood and now appears
D. Immediate abdominal CT without stable. What is the next best course of action?
IV contrast
A. Immediate nephrectomy
E. Immediate CT with intravenous contrast
B. Intraoperative one-shot IVP
3. What is the best technique to evaluate micro-
hematuria in an unstable patient with multiple C. Cystoscopy with retrograde pyelogram
upper abdominal gunshot wounds? and possible stent placement

A. Abdominal sonography D. Percutaneous nephrostomy tube placement

B. Intravenous pyelography E. Renal angiography with superselective


embolization
C. Intraoperative single shot IVP

D. Immediate abdominal CT without


IV contrast

E. Immediate CT with intravenous contrast

CHAPTER 24: RENAL AND URETERAL TRAUMA 807


6. Contraindications for transureteroureteros- 8. 8. Ileal ureter interposition is best indicated for
tomy (TUU) include which of the following? which of the following?

A. Neurogenic bladder A. Extensive lower ureteral injuries

B. Obesity B. Extensive upper ureteral injuries

C. History of urolithiasis C. Patients with bladder outlet obstruction

D. History of abdominal aortic aneurysm D. Patients with obstructed ureter due to


advanced pelvic malignancy
E. History of urethral stricture
E. Patients with complete disruption of
7. A victim of an abdominal gunshot wound midureter due to gunshot wound
presently in and is found to have an iliac vein
laceration and multiple small bowel injuries.
He undergoes immediate vascular repair and
his bowel injuries are stapled to prevent ongo-
ing contamination. In addition, he has acquired
Answers

multiple transfusions, persistent acidosis, as 1. D


well as hypothermia. A complete transaction
of the upper ureter has been noted. Appropriate 2. E
initial urologic management includes which of
the following? 3. C

A. Transureteroureterostomy over a stent 4. D

B. Ureteroneocystostomy with psoas hitch 5. E


and Boari bladder flap
6. C
C. Ileal ureter interposition
7. E
D. Ureteroureterostomy over a stent
with renal mobilization and 8. B
downward nephropexy

E. Ureteral ligation with long single-J stent


placement and delayed repair

808 EDUCATIONAL REVIEW MANUAL IN UROLOGY


Chapter 25:
Bladder, Urethra
and Genital Trauma
Richard A. Santucci, MD, FACS

Contents

1. Bladder Trauma

2. Posterior Urethral Trauma

3. Anterior Urethral Trauma

4. Penile Trauma

5. Testes Trauma

6. Genital Skin Loss

7. References

8. Questions

CHAPTER 25: BLADDER, URETHRA AND GENITAL TRAUMA 809


Introduction 1. Bladder Trauma

It is the goal of the urologist in a trauma situation to


successfully diagnose and treat genitourinary
Injury Grade

injuries. However, it must be remembered (in prac- As with all other organs, the American Association
tice as well as on the written exams) that the patien- for the Surgery of Trauma (AAST) has developed a
t’s level of hemodynamic stability, the need to 5-point injury severity scale. Table 1 lists the AAST
search for associated injuries such as spinal cord organ injury severity scale for the bladder.
injury, and the need to mesh urologic stud-
ies/treatments together with other required treat-
ments (such as emergency laparotomy) demand a
Blunt Bladder Injuries

coordinated effort between all members of the • Rare: <2% of abdominal injuries require surgery.1
trauma team. Rarity is owed to the protected position of the
bladder deep in the bony pelvis1

• Severe associated injuries are seen.2 Mortality


occurs frequently (12%–22%)1, 3, 4

• Most commonly after such high-energy situations


as automobile vs pedestrian accidents5

• Occurs in association with urethral rupture in


10%–29% of patients6

• Overwhelmingly associated with pelvic fracture,


occurring in 6%–10%4, 5 of pelvic fracture cases,
specifically with fracture of the pubic arch5

Table 1

American Association for the Surgery of Trauma (AAST) Five-point Injury Severity Scale

Gradea

I Hematoma Contusion, intramural hematoma


Laceration Partial thickness

II Laceration Extraperitoneal bladder wall laceration <2 cm

III Laceration Extraperitoneal (≥2 cm) or intraperitoneal (<2cm) bladder wall laceration

IV Laceration Intraperitoneal bladder wall laceration ≥2 cm

V Laceration Intraperitoneal or extraperitoneal bladder wall laceration extending into the


bladder neck or ureteral orifice (trigone)

a
Advance one grade for multiple lesions up to grade III.
From Moore, et al.70

810 EDUCATIONAL REVIEW MANUAL IN UROLOGY


The major diagnostic goals in these patients are to: Figure 1

1. Determine if urethral injury is present. Plain film cystogram showing

2. Determine if bladder rupture is present, and


intraperitoneal bladder rupture

classify it as an intraperitoneal (requires explo-


ration and repair) or extraperitoneal (can usu-
ally be managed by bladder drainage alone)
injury.

3. Determine if associated renal injuries are pre-


sent and if these injuries require surgical explo-
ration.

Penetrating Bladder Injury

• Wartime incidence 15%-20%7

• Civilian incidence is closer to 2%

• Commonly have associated major abdominal


injuries (35%) and often are in shock (22%)8

• Mortality in this population is high (12%), usually


from acute exsanguination secondary to major
pelvic blood vessel transection8

Diagnosis
1. Suspicion
• Usually, bladder injury will be implied by
the trajectory of the knife or missile
wound, and all patients with hematuria at
risk for bladder involvement must have
formal cystography or intraoperative 3. Hematuria
exploration to rule out bladder involve- • Most (>95%) will have gross hematuria.1, 2
ment Usually associated with more severe
injury (rupture)
2. Physical signs
• After blunt trauma, lower abdominal pain, • VERY rare to have no hematuria. Minority
tenderness and bruising can be present. (5%) have microscopic hematuria. Micro-
Often difficult to differentiate from the hematuria associated with less severe
sequela of pelvic fracture injury (such as contusion)2

• Some intraperitoneal ruptures are discov- 4. Plain cystography


ered because a urethral catheter does not • Nearly 100% accurate when drainage
return urine films obtained1

• Delayed diagnosis heralded by fever, • Complete filling a must (at least 350 cc of
absence of voiding, peritoneal irritation 30% contrast, or stop for pain)
and elevated blood urea nitrogen (BUN)

CHAPTER 25: BLADDER, URETHRA AND GENITAL TRAUMA 811


• Using only 250 cc has been associated
with false negatives.4 (Test Question)
Figure 3

• Figure 1 shows intraperitoneal bladder


CT image of intraperitoneal bladder rupture

rupture as seen on plain cystography

5. CT cystography
• Preferred (patient is often already in
the CT room)

• Use retrograde filling just like plain film


cystogram (clamping the Foley and letting
IV contrast filter into the bladder antero-
grade via the kidneys is associated with
missed injuries and is not acceptable)9, 10

• Must dilute contrast to 2%–4%


(dilute 6:1 with saline)11

• 1 in 10 will be positive (blunt injury)11 Management


1. Extraperitoneal
• Figures 2 and 3 show bladder ruptures as • Found alone in 62% of cases, in combina-
seen on CT cystography tion with intraperitoneal rupture in 12%1

• Mechanism thought to be direct laceration


Figure 2

with pelvic bone4, although this has been


seen in only 35%of patients in some series1
CT image of extraperitoneal
bladder rupture

• Basically, can treat nonoperatively with


catheter drainage only.4, 12
Caution: complications can still be high
with nonoperative approach (up to 26%).13

• There are several cases where open repair


is suggested: (Test Question)

* Bone fragment projecting into


the bladder (which is unlikely to
heal, rare)14

* Open pelvic fracture (high risk of


infection, vesicocutaneous fistula)15

* Rectal perforation (high risk of


infection)15

812 EDUCATIONAL REVIEW MANUAL IN UROLOGY


* Poor drainage of bladder secondary to
blood clots in the first 48 hours
Figure 4

(theoretically increases infectious/


nonhealing complications)13,16
Open injured bladders, inspect and repair
extraperitoneal ruptures in one layer from

* Patient undergoing laparotomy for


the inside

other reasons (such as open exploration


by general surgery for intraabdominal
injuries) increases risk of vesicocuta-
neous fistula13

* Planned anterior approach open reduc-


tion of pelvic fracture by orthopedic
team. Keeps urine leak off orthopedic
hardware

* Open repair is associated with signifi-


cantly less complications15

• Early complications: 5% in
repaired vs 12% unrepaired

• Late complications: 5% in
repaired vs 21% unrepaired

• Technique Figure 5

* Open bladder and inspect/repair from


the inside out, as shown in Figure 4.
Close extraperitoneal bladder ruptures

(Test Question)
with 2 layers

* Be mindful of associated urethral, blad-


der neck, prostate and rectal injuries

* Extraperitoneal: close from the inside


in 1 layer (don’t approach extraperi-
toneal injuries extravesically)

* Extraperitoneal, 2-layer closure,


as seen in Figure 5

* Drain

* ± Suprapubic tube (no real evidence


that its required—can be left out)

CHAPTER 25: BLADDER, URETHRA AND GENITAL TRAUMA 813


2. Posterior Urethral Trauma

2. Intraperitoneal Etiology

• 25% of all bladder injuries; seen together • 4%–14% of pelvic fracture cases19
with extraperitoneal ruptures in another
12%1 • Associated with bilateral pubic rami fractures
(straddle fractures) especially with diastasis of the
• It is generally accepted that intraperitoneal sacroiliac joint (Malgaigne fracture) (odds ratio =
ruptures occur after blunt trauma because 24)20,21
rapidly rising intraperitoneal pressure
causes the bladder to burst3,5,17 • Mostly males, <2% are females22-24

• Must be repaired: • Girls younger than 17 have a significantly


increased risk when compared to older females
* Often larger than suggested on (greater compressibility of the pelvic bones)25
cystogram (mean 6 cm): unlikely
to heal spontaneously • Associated bladder rupture in 10%–17%.4,6 May
make the diagnosis of associated bladder injuries
* Continued urine leak causes chemical difficult: a catheter often cannot be placed acutely
peritonitis in 2–3 days for cystogram, and extravasated contrast from the
urethral injury can mask further extravasation
Post-Op from an extraperitoneal bladder rupture6
• Catheter. In addition to a urethral Foley
catheter, it is traditional to leave a large-bore • Urethral-rectal fistulas can occur in up to 8%26
suprapubic catheter in after repair. However, no
clinical advantage to adding a suprapubic tube18 • Rarely occur without pelvic fracture, without
pelvic fracture,27, 28 usually as a result of direct
• Complications rare perineal blunt trauma

• Acute, self-limited urinary frequency is com-


monly seen, but usually improves by 2 months.
Clinical Diagnosis

Persistent frequency rare (2%)1 • Exam: blood at meatus (50%).6, 21,28 The amount
of meatal bleeding does not correlate with the
severity of injury19

• High riding prostate (34%).28 False positives are


common: pelvic hematoma can obscure the
prostate.20 Often the exam is not done, or not
recorded29

• Clinically: triad of blood at the urethral meatus,


inability to urinate, and the presence of a palpably
full bladder

• Often discovered only when a urethral catheter


cannot be placed, or when it is misplaced into
pelvic hematoma

• Perineal hematoma may be seen if sufficient time


has passed from the time of injury, as depicted in
Figure 620

814 EDUCATIONAL REVIEW MANUAL IN UROLOGY


Figure 6 • AAST organ injury severity scale is in Table 2

Complete vs Partial Rupture


• Accurate diagnosis of exact nature of urethral
Urethral injury can cause a (late)

injury is difficult
perineal hematoma

• There may be no radiologic differences on urethro-


gram between partial and complete transections31

• Don’t worry too much about whether it’s partial or


complete!

Management

Primary open realignment (i.e., cutting the


patient open and sewing the urethra back

• Never do it
together)

• Increased impotence, incontinence,32 stricture for-


mation33 and operative blood loss

Primary endoscopic realignment


(i.e., placing a catheter either blindly or with

• Allows healing without stricture in 50%–65%.22,3–36


the aid of cystoscopes)

Most strictures that do occur are milder22,28,37,38


Urethrography

• Use 16F Foley catheter, place it 1–2 cm into the


fossa navicularis, inflate the balloon with a small • Patients not aligned (SP tube only) nearly always
amount of water (1–2 cc) until a snug fit30 (96%) develop a urethral stricture requiring poste-
rior urethroplasty.37 In a series of patients treated
• Alternatively, use Brodney or Knudson clamp with early urethral stenting, only about 50%
require open urethroplasty28,34
• Use 30% diatrizoate meglumine (Hypaque-M) or
similar, injected gently in small 10-cc increments • Even in those who ultimately need urethroplasty,
using a 60-cc catheter tip syringe. Lateral decubi- catheter placement may align the 2 ends of the ure-
tus view static films are acceptable, but simpler to thra making surgery easier, or even shorten the
perform under fluoroscopy ultimate stricture39

• Immediate (as is practical), certainly within 72


hours suggested.40 Some successful 11–19 days
Classification of Injury

• Colapinto scheme (old, but still referred to)19 after injury41-45

* Type I: Urethral stretch injury • Does not aggravate rates of impotence, an ejacula-
tion or incontinence28, 34, 36, 46
* Type II: Urethral disruption proximal to the
genitourinary diaphragm • No traction (increases the risk of bladder neck
injury and subsequent incontinence)31, 36
* Type III: Urethral disruption both proximal to
and distal to the genitourinary diaphragm

CHAPTER 25: BLADDER, URETHRA AND GENITAL TRAUMA 815


Table 2

Urethra Injury Scale

Gradea Injury Type Description of Injury

I Contusion Blood at urethral meatus; urethrography normal

II Stretch injury Elongation of urethra without extravasation on urethrography

III Partial disruption Extravasation of urethrography contrast at injury site with


visualization in the bladder

IV Complete disruption Extravasation of urethrography contrast at injury site without


visualization in the bladder; <2 cm of urethra separation

V Complete disruption Complete transection with ≥2 cm urethral separation, or exten-


sion into the prostate or vagina

a
Advance 1 grade for bilateral injuries up to grade III
From Moore, et al.70

• Methods to pass catheter: • Suprapubic tube is left in after the urethral catheter
is removed and capped, as many patients will
* First, try a single, gentle Foley attempt by the develop symptomatic urethral stricture after
urologist catheter removal. If the patient voids satisfactorily
via the urethra, the suprapubic tube is removed
* Second, consider bedside flexible retrograde ≥14 days later
cystoscopy. Pass wire retrograde under direct
cystoscopic vision, then pass Council catheter Placement of Foley by Urologist
over wire28 • Make 1 gentle attempt to insert a urethral
Foley catheter in cases of suspected partial
* Third. Go to OR. First attempt rigid retrograde urethral disruption
cystoscopy (if patient can be placed in litho-
tomy) or attempt anterograde flexible cys- • No evidence that you can turn an incomplete
toscopy through the suprapubic tube site. Often, transection into a complete one29, 34
this is possible when retrograde cystoscopy has
failed. Place guidewire through the distraction • Works even with radiographic evidence for
injury. Consider coming back to try again in 1–3 complete tear34, 37, 38
days if you fail
• Confirm placement by CT or cystogram if unsure
* Fourth, cease attempt and insert a suprapubic
tube instead Suprapubic Tube

• Catheterize for 6 weeks, then cystourethrogram22, 28 • Can do open cystotomy if unable to get a bedside
or pericatheter RUG48 to prove its healed urethral catheter in, or if percutaneous suprapubic
tube is impractical or contraindicated
• Some have catheterized for only 21 days with
good results (probably too short)49

816 EDUCATIONAL REVIEW MANUAL IN UROLOGY


• Open cystotomy has some benefits: facilitates Delayed Open Reconstruction
bladder inspection, might allow urethral catheteri-
zation under direct vision in some cases • Wait 3 months (some do wait longer, but no
evidence is better)
• No evidence that suprapubic tube causes pelvic
fracture hardware infection, although orthopedists • Difficult surgery. Consider transferring to a center
worry about it22, 50 with experience in the operation35, 52

• Absolute indication for open exploration: associ- • Combined retrograde urethrogram and simultane-
ated rectal injuries. Repair rectum, irrigation and ous cystogram ("up-and-down-o-gram" as seen in
placement of presacral drains. Area of urethral dis- Figure 7) is mandatory
location need not be explored or even exposed51
• Pubectomy required historically 10%–30% of the
• Some have also advocated open exploration in time, but increased familiarity with the perineal
cases of a high-riding or “pie in the sky” bladder, approach makes it rare now. Only used in:
and in cases of associated bladder neck tear;32
however, these are not valid indications for * Really long strictures
surgery and will heal in a majority of cases
* Some recurrent strictures
• Some have also advocated open exploration in
cases of a high-riding or “pie in the sky” bladder, * Very complex strictures
and in cases of associated bladder neck tear;32 how- (urethrorectal fistulae, etc)
ever, these are not valid indications for surgery and
will heal in the majority of cases Delayed Endoscopic Treatment
("Cut-to-the-Light")
• Requires an average of 1.5–3.428, 36 procedures in
Figure 7 those reports that actually found it helpful

• May be associated with unacceptable complication


rates, including accidentally coring a false passage
Combined cystogram and retrograde ure-

that inadvertently bypasses the bladder neck26


throgram showing stricture after posterior
urethral distraction injury

• Avoid in cases of long strictures or significant dis-


location of the prostate from the anatomic position

• Failure rate is significant54-56

• May be more beneficial to patients who cannot tol-


erate open urethroplasty second to comorbidity57

• You should probably refer these patients instead to


a center of excellence for urethral reconstruction
for definitive (open) reconstruction

CHAPTER 25: BLADDER, URETHRA AND GENITAL TRAUMA 817


3. Anterior Urethral Trauma

Complications Etiology

Impotence • 10% of all urethral injuries63


• Impotence rates of at least 13%–30%, likely due
to original pelvic trauma22, 28, 34, 36 • 5%–25% of all GU injuries in wartime series7

• Patients with severe (very lengthy) distraction • Often isolated injuries64


runs higher (48%–72%)52, 58
• The majority of disruptions occur after
• Another 32% report poorer quality erections after straddle injury65
injury, although not fully impotent58
• Remainder from gunshot wounds66
• Some will recover erectile function as long as a
year or 2 after injury26, 31 Diagnosis

• Etiology. Usually reported as injury to the penile • High index of suspicion required
parasympathetic nerves,26 but studies have also
found evidence for arterial insufficiency59, 60 • Urethrography should be performed in
suspected cases67, 68
Incontinence
• Many posterior urethral distraction injury patients • May be associated with swelling/ecchymosis
have damage or denervation of external (striated) under Buck’s fascia, causing classic eggplant
sphincter deformity of penile shaft only, as seen in Figure 8

• In these cases, continence is provided by the blad- • If Buck’s fascia is also ruptured, then hematoma
der neck61 gets contained by Colles’ fascia. In this case,
swelling of the entire scrotum and a small perirec-
• Patients with open bladder neck have a signifi- tal butterfly hematoma may be evident
cantly higher rate (53%) of incontinence when (Test Question)
compared to patients with a closed bladder neck62

• Yet, total incontinence overall is rare (2%–4%)28, 58


Initial Management

• Anterior urethral injuries are rare, and the litera-


• Mild incontinence in 6%58 ture is not clear on the best way to treat them

• Up to 10% have areflexic bladder from the • Literature prefers open anastomotic repair over
original injury58 simple catheter alignment, or (alternatively)
suprapubic tube without any attempt at repair.67, 69
12% failure rate overall. This may be one of the few
unknowns in GU trauma. Repair primarily and
place a urethral Foley? Repair primarily and place a
suprapubic tube? Just place a Foley? Just place
suprapubic tube? Stand by for research to answer
the question (Test Question)

818 EDUCATIONAL REVIEW MANUAL IN UROLOGY


4. Penile Trauma

The AAST organ injury severity scale for the penis


Tensionless is found in Table 3.
Principles of repair:

Minimal debridement
Spatulated
Watertight
Penile Amputation

Stented (Foley) vs drain with SPT only • Self-mutilation most common etiology71

• Debride only minimally64, 67 • Most (87%) are actively psychotic

• Reimplant if possible. Formalize the amputation if


not. (Test Question)
Figure 8

• Preserve amputated penis. Keep in wet sterile


“Eggplant deformity” after penile fracture

gauze, in a sterile baggy. Put baggy on ice

• <24 cold ischemia time to be viable

• Many do well: 18%–25% strictures, fistula rate71, 72

Penile Gunshot

• Rare73

• Urethrography vs intraoperative inspection for


urethral injury is suggested. 50% have urethral
involvement73-75

• Look for associated injuries (rectum, testes, thigh):


present 80% of the time. 75% will be hemodynam-
ically unstable76

• Repair most primarily,77 but massive injuries such


Classic “eggplant deformity” after penile fracture

as from a close range shotgun wound are treated


where blood (and possibly urine) is contained by

with wet dressing in a staged fashion73, 78


Buck’s fascia.

• Superficial penetration or trivial wound?


Nonoperative Rx74, 78, 79

• Repair corporal injures with buried nonabsorbable74


or absorbable (preferred)79 sutures

• Associated anterior urethral injuries should be


closed primarily with a watertight, spatulated,
catheter-stented technique using absorbable
suture74

• Posterior urethral injuries should be managed in a


staged fashion, with suprapubic catheterization74
and placement of a urethral catheter (if possible)

CHAPTER 25: BLADDER, URETHRA AND GENITAL TRAUMA 819


• Physical exam. Penile swelling and ecchymosis
are nearly always present. A palpable corporal
Table 3

defect may be appreciable,86 but this can be


obscured by swelling and hematoma
Penis Injury Scale

* The rolling clot sign, where the clot lying near


the fracture site is palpable as a firm, immobile,
Gradea Description of Injury

discrete swelling over which the penile skin can


be rolled. This sign appears to be reliably asso-
I Cutaneous laceration/contusion

II Buck’s fascia (cavernosum) laceration ciated with the location of cavernosal rupture83, 90

* Extruded blood combined with tissue swelling


without tissue loss

can create a hugely deformed ecchymotic penis,


which is referred to as an eggplant deformity.86
III Cutaneous avulsion

This sign is highly diagnostic83


Laceration through glans/meatus
Cavemosal or urethral defect <2 cm

* If Buck’s fascia is ruptured, blood and urine will


extravasate around Colles’ fascia, giving a char-
IV Partial penectomy

acteristic scrotal swelling and a butterfly sign in


Cavarnosal or urethral defect ≥2 cm

V Total penectomy the perineum91

Associated Injury
• Associated with urethral injury 0%–38% of the
Advance 1 grade for multiple injuries up to grade V From
a

• Complications. True incidence unknown (poor time. (The wide range in associated urethral injury
Moore, et al.70

follow-up) but most get erection back 1–6 weeks is likely because penile fractures in some Middle
after repair73, 75 and do not develop stricture75 Eastern areas are often due to low-energy injuries,
such as manual bending of the penis with the hand,
while those in the West are most often due to high-
energy injuries, such as those that occur during
Penile Fracture

• As infrequent as 1/175,000 hospital admissions in vigorous intercourse or falls, or both)83


the United States,80 but significantly more com-
mon in some centers abroad81-83 • Urethrography must be performed in cases of sus-
pected urethral injury (blood at meatus, inability to
• Usually from some blow to the erect penis: falling void, gross hematuria),83, 92 although urethral bleed-
on the penis,84 rough masturbation,84, 85 being struck ing without associated urethral injury has been
in the penis83 and even rolling over in bed82, 84 reported after penile fracture94

• Size of cavernosal rupture varies between • Difficulty voiding or gross hematuria is present in
0.5–5 cm.83, 84 Bilateral injuries can also occur86 20% of cases86

Diagnosis • Rare cases of false-negative urethrography in


• History is often constant: a blow to the erect penis, patients with penile fracture and urethral injury
accompanied by a snapping or popping sound, have been reported,94 so direct inspection of the
penile pain, and immediate detumescence,87 urethra during exploration is a good practice
although these classic findings are not universally
seen88 • Ultrasound has been used to aid in the diagnosis of
penile fracture,86 although the clinical picture is
• Because of embarrassment, delay in presentation usually so clear that such imaging techniques are
to the hospital is common84, 89 not necessary

820 EDUCATIONAL REVIEW MANUAL IN UROLOGY


• MRI has been used to image the injured corpora
cavernosa.88 Necessary?
Penis Caught in Zipper Injury

• First step: cut the zipper slider with an orthopedic


• Cavernosography is a sensitive tool84 but seldom pin cutter or similar metal snips. (Test Question)
adds anything to the usually unambiguous presen-
tation.83 Most only use cavernosography if the • Alternatively: local anesthetic, lubrication of the
diagnosis is somehow in doubt,87, 95 which it seldom zipper with mineral oil, and a single manual
appears to be93 attempt at opening the zipper101, 102

• 1% of cases actually have rupture of the dorsal • Second step: zipper teeth should be individually
vein only, and no corporal rupture is present pulled apart from each other using hemostats103

Management
• Surgical repair recommended.82, 85, 86, 88, 89, 92, 93, 96-98

• Circumcising incision82, 83, 92

• Absorbable sutures82, 85, 92

• Place a Foley to aid in identification of the urethra


and prevent urethral injury during repair.83, 93 The
urethral catheter may be removed when penile
swelling decreases enough to allow micturition

• Avoid compression dressing after repair

• No need for medications, such as diazepam, amyl


nitrate or diethylstilbestrol, to eliminate erections
after surgery.81, 99 Erections are good for you

Outcomes/Complications
• A certain number of patients become impotent, due
to traumatic corporeal veno-occlusive dysfunction,
persistent venous leak or arterial insufficiency100

• Unrepaired penile fractures can result in functional


inability to have intercourse, often because of
angulation at the site of injury80

• Patients who undergo early repair of penile frac-


ture have a low complication rate.81, 85 Mild curva-
ture was present in <5% of cases, and most often
this complication occurred in patients who did not
arrive at the hospital immediately after injury93

CHAPTER 25: BLADDER, URETHRA AND GENITAL TRAUMA 821


5. Testes Trauma

• Rare, as the tough tunica albuginea usually pro- • There are no specific symptoms;107 however,
tects104 history of trauma in association with scrotal pain,
ecchymosis, and (at times) hematocele are often
• The AAST organ injury severity scale for the found. Associated epididymal ruptures can occur,
testes is found in Table 4 but are extremely rare107

• Testes injuries which do not require operative


Table 4

exploration should be managed with analgesics,


scrotal elevation to promote veno-lymphatic
Testis Injury Scale

drainage, and ice to control pain and swelling112

• Ruptured? Operate. Think it may be ruptured?


Operate. (Test Question)
Gradea Description of Injury

I Contusion/hematoma

Penetrating
II Subclinical laceration of tunica albuginea

• Civilian male victims of gunshot wounds will


suffer injury to the scrotum approximately 2%
III Laceration of tunica albuginea with

of the time76
<50% parenchymal loss

• Assault is the most common, but self-inflicted


IV Major laceration of tunica albuginea

gunshots incurred by patients placing a loaded


with ≥50% parenchymal loss

V Total testicular destruction or avulsion


handgun in pants pocket do occur

• Penetrating trauma causes bilateral injury with an


incidence nearly 15 times that of blunt injury105
a
Advance one grade for multiple injuries up to grade V
From Moore, et al.70

• Use physical examination primarily and ultra-


sound secondarily to rule out testicular injury.
Blunt

• Testicular rupture after a direct blow to the testicle Equivocal cases should be explored
is as high as 50%105
• Most cases ultimately require exploration:
• Most unilateral, both testes in 1.5% of cases 106 92%–97%75, 113

• Most common etiology is assault107 and sports • Testicular salvage is only possible in 35%–65%73,
injuries,108,109 but motor vehicle accidents account
75, 114
of testicular (or cord) gunshot wounds, but
for a significant minority (10%) every attempt to save those cases with ≥40% of
spared parenchyma should be made (as seen in
• Exact mechanism of testicular injury is unknown, Figures 9a and 9b)
but hypothesized that the testicle is driven against
and trapped against the thigh or bony pelvis. Once
trapped, the force can transmit to the testicle and
Clinical Findings

cause rupture108, 110 • Associated injuries are common (83%),73 mostly


surrounding structures: thigh (75%), penis (37%),
• Rarely, testicular injury is self-inflicted (50% of perineum (25%), urethra (18%)114
reported cases of genital mutilation involve the tes-
ticles)111 • A significant minority have an associated injury to
the femoral vessels with attendant significant
• Examination may be limited by pain. Use 1% bleeding—vigilance for this injury is warranted73
lidocaine spermatic cord block

822 EDUCATIONAL REVIEW MANUAL IN UROLOGY


• Bilateral injury is present in as many as 31%
(penetrating)106
Figure 9a

• Transection of the vas is not uncommon after blunt


Close range, low-velocity testicular

trauma to the testes, occurring in 10% of patients


gunshot wound

in 1 series73

Imaging

Sonography
• A useful adjunct, but in no case should a normal
sonogram dissuade exploration of a grossly
abnormal testicle on physical exam

• Perform immediate exploration without obtaining


a sonogram in those cases where the physical
examination clearly shows significant testicular
trauma: large hematocele, gross testicular rupture
or testicular dislocation

• Abnormal sonographic findings: heterogeneity of


the parenchyma may indicate intratesticular
hematoma115 and loss of continuity of the tunica
albuginea, which may indicate tunica rupture115-117
or large hematocele117, 118

• Testicular rupture can be difficult to diagnose with


Figure 9b

ultrasound, and in 1 series sonography incorrectly


diagnosed this entity more commonly than it cor-
Same testicle is preserved after minimal

rectly diagnosed it119


parenchymal debridement and suture clo-
sure of the capsule

• Added benefit of ultrasound in some testis trauma


cases: diagnose unexpected associated conditions,
such as testicular torsion or testicular tumors asso-
ciated with a history of trauma

Radionucleotide Scans
• Useful to evaluate the painful testicle not associ-
ated with previous trauma,120 but usually not indi-
cated for trauma121

Surgical Management

• Explore and repair all cases of significant hemato-


cele,122 intratesticular hematoma or frank rupture
of the tunica albuginea

• Lack of evidence for tunica rupture on ultrasound


is not sufficient reason to delay exploration as
tunica fracture lines are rarely detected123

CHAPTER 25: BLADDER, URETHRA AND GENITAL TRAUMA 823


• Do not delay—increases orchiectomy rate
between 3–8-fold105, 107
Nonoperative Management

• Nonoperative therapy of significant testicular


• It is said that even small tears can ultimately result injuries was used historically,105, 110 but now is out
in testicular loss if ignored: intratesticular swelling of favor
results in extrusion of seminiferous tubules, forc-
ing the friable tissue out of the breached capsule • Results in higher orchiectomy rates. Often, patient
like toothpaste out of a tube.108 However, new data deterioration forces delayed scrotal exploration
that n=7 adolescent boys with known testes rup-
ture conservatively treated without surgery did • Results in longer hospital stays and period of
well may change the thinking on this in the future debility than prompt surgical therapy110

• Midline raphe incision • Only appropriate for insignificant scrotal injuries


not resulting in testicular hematoma or rupture
• At exploration, necrotic testicular tissue should be
debrided and the capsule closed with running • Specifically, patients with intratesticular
absorbable suture. In some cases, loss of capsule hematoma do poorly without operation. 40%
will require removal of intratesticular tissue to develop testicular infection and/or testicular
allow closure. (Test Question) necrosis, often requiring orchidectomy124

• In blunt injury, significant intratesticular


hematoma should be drained, as increased intrates-
Outcome

ticular pressure from these collections can result in • After repair of bilateral rupture, preservation of
testicular atrophy later105 adequate sperm production has been docu-
mented76,125
• Leave drains when the operative field cannot be
made absolutely hemostatic • A small percentage of patients with testicular
injury will have complications, such as wound
• Antibiotics, such as cefazolin are continued until infection;78 the majority do well76
drains are removed, although there are no random-
ized trials supporting this approach

• Bilateral injury? Debride conservatively and


preserve as much functioning testicular tissue as
possible106

• Injuries to the vas should be tied off with nonab-


sorbable suture for reconstruction, if necessary, at
a later time73

• Postoperatively, gentle pressure dressings can be


applied, usually by fluffed gauze held in place by
an athletic supporter

824 EDUCATIONAL REVIEW MANUAL IN UROLOGY


6. Genital Skin Loss

Etiology Table 5

Penetrating
• Not common
Scrotum Injury Scale

• Can often be closed primarily even with >50%


skin loss, owing to scrotal skin distensibility
Gradea Description of Injury

Burns
I Contusion

• Burns involve the perineum 1% of cases II Laceration <25% of scrotal diameter

• Often full-thickness, as penile skin is thin126 III Laceration ≥25% of scrotal diameter

• Treat like other burns, with early resection of burn


eschar and coverage with split-thickness skin
IV Avulsion <50%

grafts, when possible V Avulsion ≥50%

Constricting Bands
• Can result from prescription venous return con-
From Moore et al.70
a

striction devices used to prolong erection128 or


sadomasochistic sex play
Surgical Management of
Penile Skin Wounds

• Rarely results in significant skin loss, although the • The algorithm for penile skin wounds is found in
more common injury involves direct pressure Figure 10
necrosis under the band (which usually heals well)
• Foreskin flaps can be used to cover distal penile
Avulsion skin loss, such as would occur with burns to the
• Resultant from traction by mechanical devices, shaft of the penis126
such as farm machinery, industrial machinery or
by suction devices (vacuum cleaners) • Scrotal rotation flaps for more proximal defects.
Risk of unacceptable cosmetic result because the
• Subdermal penile tissue is loose areolar tissue, so scrotum is hair-bearing skin
it is often torn free without damage to the underly-
ing structures • Local flaps, such as from the abdomen and thigh
can also be used, but they have a less acceptable
The AAST Organ Injury Severity Scale for the scro- cosmetic result than STSG128
tum is found in Table 5.
• Skin coverage using the avulsed skin, which can
sometimes be retrieved from the scene of the acci-
dent, should be avoided, as some authors have
Initial Management

• Wet gauze dressings, which are changed stated that it inevitably becomes necrotic and
frequently (every nursing shift, or 8 hours), and infected112
urinary drainage. Silvadene dressings preferred
for burns. (Test Question) • Treatment of choice for penile skin coverage is
thick (0.15 inch), unmeshed, split-thickness skin
• Daily inspection by the surgical team is mandatory grafts from a thigh donor site

• Coexisting infection? Plan intervals of operative


debridement, up to daily, until active infection is
controlled

CHAPTER 25: BLADDER, URETHRA AND GENITAL TRAUMA 825


Figure 10

Algorithm for the treatment of penile skin wounds

Superficial Limited Areas Large Areas of


Skin of Full- Full-Thickness
Loss Thickness Loss Loss

Distal Proximal Potent Impotent

Wet Gauze
Dressings or STSG Foreskin flap STSG Scrotal flap
Silvadene

Nonmeshed Nonmeshed
STSG STSG
(If impotent or
inadequate
skin is not
available,
consider
meshed STSG)

* Remove subcoronal skin, even to the point of


excising unaffected subcoronal skin. Lym-
Figure 11

phatic obstruction of this skin, if not excised,


will result in circumferential lymphedema126
OpSite dressing

*Grafts will never regain normal sensation,129


although sexual function is often preserved
because of intact sensation in the skin of the
glans

*Meshed skin grafts should not be used on the


penis in potent patients, except in extreme
cases where there is a paucity of donor skin in
cases of large-area burns. Meshed grafts have
a tendency towards contracture128 and are cos-
metically less acceptable than unmeshed skin
grafts

• Treat donor site with large semi-occlusive dress-


Large OpSite dressing to cover skin graft donor site,

ing, such as Op-Site® or TegadermTM. Speeds heal-


and small drain to remove exudate while occlusive

ing (mean 9 days to reepithelialization), lowers


dressing is in place.

rates of infection (3%–5%), decreases subjective


pain scores and costs less,130 as in Figure 10

• Wound exudates under the dressing are removed


with a tiny (7F) silicone wound drain (TLS®
Surgical Tube Drainage System; Stryker)

826 EDUCATIONAL REVIEW MANUAL IN UROLOGY


* The dressing is removed if infection is sus- • Consider wound vac. Decreases pain of dressing
pected131 changes, removes infectious exudate and can dra-
matically decrease the size of the defect over time.
* If healing is not complete after dressing Air leak because of the convoluted perineal surface
removal, the residual wounds can be treated can be a problem: use huge amounts of Ioban to
with petroleum-impregnated gauze until com- cover the area to avoid
pletely healed132
• Split-thickness skin grafts, meshed 2:1, give excel-
lent cosmetic result when necessary. The meshing
allows exudates to escape from the interstices, thus
Postoperative Penile Skin

improving graft take and decreases the size of the


Graft Wound Management

• Use diluted, sprayed, fibrin glue to glue down the donor site wound. Some create a pouch and place
skin graft (much more reliable skin take) the testicles inside. Author’s preference is placing
graft over the testes
• The wound is inspected, “deblebbed” and
redressed at 24 hours. Bed rest X for 72 hours • Third option: thigh flaps. Particularly useful if the
testes have already been buried in the thighs after
• Intercourse is avoided for at least another 6 weeks traumatic or surgical scrotal removal. The thigh
until the graft is completely healed flaps can be left as a permanent solution or rotated
into a neo-scrotum if desired
• Allow patient activity and self-dressing of wound
on POD #4. Soft, dry gauze dressings are contin-
ued for another 3 weeks

• Intercourse is avoided for at least another 6 weeks


until the graft is completely healed

Surgical Management: Scrotum


• The algorithm for management of scrotal skin loss
is found in Figure 12

• Scrotal skin loss involving less than 60% of the


skin can often be closed acutely/primarily

• In more significant loss, the testes may either be


placed in thigh pouches or treated with wet dress-
ings until the time for reconstruction. (Thigh
pouches have been classically used to protect the
testes until definitive reconstruction can be com-
pleted;133 others have used saline dressings for up to
3 weeks in small numbers of patients with good
results)78

• Consider primary closure. After 2–3 weeks of


dressing changes, some even with significant scro-
tal skin loss can have primary closure and avoid the
need for scrotal reconstruction78

CHAPTER 25: BLADDER, URETHRA AND GENITAL TRAUMA 827


Figure 12

Algorithm for management of scrotal skin loss

Partial scrotal loss Total scrotal loss

Clean Infected Clean Infected

Immediate
Primary Saline gauze Immediate Saline gauze
thigh
Closure dressing 2-3 weeks STSG dressing 2-3 weeks
pouches

Primary closure Thigh Thigh Thigh


STSG STSG STSG flaps
if possible flaps flaps

828 EDUCATIONAL REVIEW MANUAL IN UROLOGY


7. References

1. Carroll PR, McAninch JW. Major bladder 12. Hayes EE, Sandler CM, Corriere JN Jr. Man-
trauma: mechanisms of injury and a unified agement of the ruptured bladder secondary to
method of diagnosis and repair. J Urol. blunt abdominal trauma. J Urol. 1983;
1984;132: 254-257. 129:946-948.

2. Cass AS. The multiple injured patient with 13. Kotkin L, Koch MO. Morbidity associated
bladder trauma. J Trauma. 1984;24:731-734. with nonoperative management of extraperi-
toneal bladder injuries. J Trauma. 1995;
3. Corriere JN Jr, Sandler CM. Management of 38:895-898.
the ruptured bladder: seven years of experi-
ence with 111 cases. J Trauma. 1986;26:830- 14. Corriere JN Jr, Sandler CM. Mechanisms of
833. injury, patterns of extravasation and manage-
ment of extraperitoneal bladder rupture due to
4. Cass AS, Luxenberg M. Features of 164 blad- blunt trauma. J Urol. 1988;139:43-44.
der ruptures [see comments]. J Urol.
1987;138:743-745. 15. Cass AS, Luxenberg M. Management of
extraperitoneal ruptures of bladder caused by
5. Hochberg E, Stone NN. Bladder rupture asso- external trauma. Urology. 1989;33:179-183.
ciated with pelvic fracture due to blunt trauma.
Urology. 1993;41:531-533. 16. Skinner EC, Parisky YR, Skinner DG. Man-
agement of complex urologic injuries. Surg
6. Cass AS, Gleich P, Smith C. Simultaneous Clin North Am. 1996;76:861-878.
bladder and prostatomembranous urethral rup-
ture from external trauma. J Urol. 17. Peters PC. Intraperitoneal rupture of the blad-
1984;132:907-908. der. Urol Clin North Am. 1989;16:279-282.

7. Selikowitz SM. Penetrating high-velocity gen- 18. Volpe MA, Pachter EM, Scalea TM, Macchia
itourinary injuries. Part I. Statistics mecha- RJ, Mydlo JH. Is there a difference in outcome
nisms, and renal wounds. Urology. when treating traumatic intraperitoneal blad-
1977;9:371-376. der rupture with or without a suprapubic tube?
J Urol. 1999;161:1103-1105.
8. Duncan AO, Phillips TF, Scalea TM, Maltz
SB, Atweh NA, Sclafani SJ. Management of 19. Colapinto V, McCallum RW. Injury to the
transpelvic gunshot wounds. J Trauma. male posterior urethra in fractured pelvis: a
1989;29:1335-1340. new classification. J Urol. 1977;118:575-580.

9. Mee SL, McAninch JW, Federle MP. Comput- 20. Koraitim MM, Marzouk ME, Atta MA, Orabi
erized tomography in bladder rupture: diag- SS. Risk factors and mechanism of urethral
nostic limitations. J Urol. 1987;137:207-209. injury in pelvic fractures. Br J Urol.
1996;77:876-880.
10. Haas CA, Brown SL, Spirnak JP. Limitations
of routine spiral computerized tomography in 21. Lowe MA, Mason JT, Luna GK, Maier RV,
the evaluation of bladder trauma. J Urol. 1999; Copass MK, Berger RE. Risk factors for ure-
162:51-52. thral injuries in men with traumatic pelvic
fractures. J Urol. 1988;140:506-507.
11. Peng MY, Parisky YR, Cornwell EE 3rd,
Radin R, Bragin S. CT cystography versus
conventional cystography in evaluation of
bladder injury. AJR Am J Roentgenol.
1999;173:1269-1272.

CHAPTER 25: BLADDER, URETHRA AND GENITAL TRAUMA 829


22. Routt ML, Simonian PT, Defalco AJ, Miller J, 32. Webster GD, Mathes GL, Selli C.
Clarke T. Internal fixation in pelvic fractures Prostatomembranous urethral injuries: a
and primary repairs of associated genitouri- review of the literature and a rational approach
nary disruptions: a team approach [published to their management. J Urol. 1983;130:898-
erratum appears in J Trauma. 1997 902.
May;42(5):981] [see comments]. J Trauma.
1996;40:784-790. 33. Coffield KS, Weems WL. Experience with
management of posterior urethral injury asso-
23. Hemal AK, Dorairajan LN, Gupta NP. Post- ciated with pelvic fracture. J Urol. 1977;
traumatic complete and partial loss of urethra 117:722-724.
with pelvic fracture in girls: an appraisal of
management. J Urol. 2000;163:282-287. 34. Kotkin L, Koch MO. Impotence and inconti-
nence after immediate realignment of poste-
24. Persky L. Childhood urethral trauma. Urology. rior urethral trauma: result of injury or man-
1978;11:603-606. agement? J Urol. 1996;155:1600-1603.

25. Hemal AK, Singh I, Chahal R, Gupta NP. Core 35. Morehouse DD, Mackinnon KJ.Management
through internal urethrotomy in the manage- of prostatomembranous urethral disrup-
ment of post-traumatic isolated bladder neck tion:13-year experience. J Urol. 1980;
and prostatic urethral strictures in adults. A 123:173-174.
report of 4 cases. Int Urol Nephrol.
1999;31:703-708. 36. Asci R, Sarikaya S, Büyükalpelli R, Saylik A,
Yilmaz AF, Yildiz S. Voiding and sexual dys-
26. Turner-Warwick R. Prevention of complica- functions after pelvic fracture urethral injuries
tions resulting from pelvic fracture urethral treated with either initial cystostomy and
injuries—and from their surgical manage- delayed urethroplasty or immediate primary
ment. Urol Clin North Am. 1989;16:335-358. urethral realignment. Scand J Urol Nephrol.
1999;33:228-233.
27. Cass AS. Urethral injury in the multiple-
injured patient. J Trauma. 1984;24:901-906. 37. Herschorn S, Thijssen A, Radomski SB. The
value of immediate or early catheterization of
28. Elliott DS, Barrett DM. Long-term followup the traumatized posterior urethra [see com-
and evaluation of primary realignment of pos- ments]. J Urol. 1992;148:1428-1431.
terior urethral disruptions [see comments].
J Urol. 1997;157:814-816. 38. Al-Ali IH, Husain I. Disrupting injuries of the
membranous urethra—the case for early
29. Jackson DH, Williams JL. Urethral injury: a surgery and catheter splinting. Br J Urol.
retrospective study. Br J Urol. 1974;46:665- 1983;55:716-720.
676.
39. Devine CJ Jr, Jordan GH, Devine PC. Primary
30. Sandler CM, Corriere JN Jr. Urethrography in realignment of the disrupted prostatomembra-
the diagnosis of acute urethral injuries. Urol nous urethra. Urol Clin North Am.
Clin North Am. 1989;16:283-289. 1989;16:291-295.

31. Morehouse DD, Belitsky P, Mackinnon K. 40. Rehman J, Samadi D, Ricciardi R Jr, Kreutzer
Rupture of the posterior urethra. J Urol. E. Early endoscopic realignment as primary
1972;107:255-258. therapy for complete posterior urethral disrup-
tions. J Endourol. 1998;12:283-289.

830 EDUCATIONAL REVIEW MANUAL IN UROLOGY


41. Porter JR, Takayama TK, Defalco AJ. Trau- 51. Koraitim MM. Pelvic fracture urethral
matic posterior urethral injury and early injuries: the unresolved controversy. J Urol.
realignment using magnetic urethral catheters. 1999;161:1433-1441.
J Urol. 1997;158:425-430.
52. Dhabuwala CB, Hamid S, Katsikas DM,
42. Towler JM, Eisen SM. A new technique for the Pierce JM Jr. Impotence following delayed
management of urethral injuries. Br J Urol. repair of prostatomembranous urethral disrup-
1987;60:162-166. tion. J Urol. 1990;144:677-678.

43. Londergan TA, Gundersen LH, van Every MJ. 53. Dixon CM, Hricak H, McAninch JW. Mag-
Early fluoroscopic realignment for traumatic netic resonance imaging of traumatic posterior
urethral injuries. Urology. 1997;49:101-103. urethral defects and pelvic crush injuries.
J Urol. 1992;148:1162-1165.
44. Cohen JK, Berg G, Carl GH, Diamond DD.
Primary endoscopic realignment following 54. Levine LA, Eckrich PC. Endoscopic repair of
posterior urethral disruption. J Urol. the completely disrupted urethra. J Endourol.
1991;146:1548-1550. 1993;7:225-227.

45. Turner-Warwick R. A personal view of the 55. Gupta NP, Gill IS. Core-through optical inter-
immediate management of pelvic fracture ure- nal urethrotomy in management of impassable
thral injuries. Urol Clin North Am. 1977;4:81- traumatic posterior urethral strictures. J Urol.
93. 1986;136:1018-1021.

46. Follis HW, Koch MO, McDougal WS. Imme- 56. el-Abd SA. Endoscopic treatment of posttrau-
diate management of prostatomembranous matic urethral obliteration: experience in 396
urethral disruptions. J Urol. 1992;147: 1259- patients. J Urol. 1995;153:67-71.
1262.
57. Chiou RK, Gonzalez R, Ortlip S, Fraley EE.
47. Gheiler EL, Frontera JR. Immediate primary Endoscopic treatment of posterior urethral
realignment of prostatomembranous urethral obliteration: long-term followup and compari-
disruptions using endourologic techniques. son with transpubic urethroplasty. J Urol.
Urology. 1997;49:596-599. 1988;140:508-511.

48. Jepson BR, Boullier JA, Moore RG, Parra RO. 58. Corriere JN Jr, Rudy DC, Benson GS. Voiding
Traumatic posterior urethral injury and early and erectile function after delayed one-stage
primary endoscopic realignment: evaluation repair of posterior urethral disruptions in 50
of long-term follow-up. Urology. men with a fractured pelvis. J Trauma.
1999;53:1205-1210. 1994;37:587-589; discussion 589-590.

49. Patterson DE, Barrett DM, Myers RP, 59. Armenakas NA, McAninch JW, Lue TF,
DeWeerd JH, Hall BB, Benson RC Jr. Primary Dixon CM, Hricak H. Posttraumatic impo-
realignment of posterior urethral injuries. tence: magnetic resonance imaging and duplex
J Urol. 1983;129:513-516. ultrasound in diagnosis and management.
J Urol. 1993;149:1272-1275.
50. Patterson BM.. Pelvic ring injury and associ-
ated urologic trauma: an orthopaedic perspec- 60. Matthews LA, Herbener TE, Seftel AD. Impo-
tive. Sem Urol. 1995;13:25-33. tence associated with blunt pelvic and perineal
trauma: penile revascularization as a treatment
option. Semin Urol. 1995;13:66-72.

CHAPTER 25: BLADDER, URETHRA AND GENITAL TRAUMA 831


61. Morey AF, McAninch JW. Reconstruction of 72. Lowe MA, Chapman W, Berger RE. Repair of
traumatic posterior urethral strictures. Tech a traumatically amputated penis with return of
Urol. 1997;3:103-107. erectile function [see comments]. J Urol.
1991;145:1267-1270.
62. Iselin CE, Webster GD. The significance of
the open bladder neck associated with pelvic 73. Gomez RG, Castanheira AC, McAninch JW.
fracture urethral distraction defects. J Urol. Gunshot wounds to the male external geni-
1999;162:347-351. talia. J Urol. 1993;150:1147-1149.

63. Mitchell JP. Injuries to the urethra. Br J Urol. 74. Miles BJ, Poffenberger RJ, Farah RN, Moore
1968;40:649-670. S. Management of penile gunshot wounds.
Urology. 1990;36:318-321.
64. Kiracofe HL, Pfister RR, Peterson NE. Man-
agement of non-penetrating distal urethral 75. Cline KJ, Mata JA, Venable DD, Eastham JA.
trauma. J Urol. 1975;114:57-62. Penetrating trauma to the male external geni-
talia. J Trauma. 1998;44:492-494.
65. Pierce JM Jr. Disruptions of the anterior ure-
thra. Urol Clin North Am. 1989;16:329-334. 76. Brandes SB, Buckman RF, Chelsky MJ,
Hanno PM. External genitalia gunshot
66. Pontes JE, Pierce JM Jr. Anterior urethral wounds: a ten-year experience with fifty-six
injuries: four years of experience at the Detroit cases. J Trauma. 1995;39:266-271; discussion
General Hospital. J Urol. 1978;120:563-564. 271-262.

67. Husmann DA, Boone TB, Wilson WT. Man- 77. Selikowitz SM. Penetrating high-velocity gen-
agement of low velocity gunshot wounds to itourinary injuries. Part II: Ureteral, lower
the anterior urethra: the role of primary repair tract, and genital wounds. Urology.
versus urinary diversion alone. J Urol. 1977;9:493-499.
1993;150:70-72.
78. Bertini JE Jr, Corriere JN Jr. The etiology and
68. Devine CJ Jr, Devine PC, Horton CE. Anterior management of genital injuries. J Trauma.
urethral injury: etiology, diagnosis, and initial 1988;28:1278-1281.
management. Urol Clin North Am.
1977;4:125-131. 79. Monga M, Moreno T, Hellstrom WJG. A strat-
egy for success: managing gunshot wounds to
69. Lucey DT, Smith MJ, Koontz WW Jr. Modern the male genitalia. Contemp Urology. 1995;
trends in the management of urologic trauma. 58-63.
J Urol. 1972;107:641-646.
80. Farah RN, Stiles R Jr, Cerny JC. Surgical treat-
70. Moore EE, Malangoni MA, Cogbill TH, Peter- ment of deformity and coital difficulty in
son NE, Champion HR, Jurkovich GJ, et al. healed traumatic rupture of the corpora caver-
Organ injury scaling VII: cervical vascular, nosa. J Urol. 1978;120:118-120.
peripheral vascular, adrenal, penis, testis, and
scrotum. J Trauma. 1996;41:523-524. 81. el-Sherif AE, Dauleh M, Allowneh N, Vijayan
P. Management of fracture of the penis in
71. Aboseif S, Gomez R, McAninch JW. Genital Qatar. Br J Urol. 1991;68:622-625.
self-mutilation. J Urol. 1993;150:1143-1146.
82. Al Saleh BM, Ansari ER, Al Ali IH, Tell JY,
Saheb A. Fractures of the penis seen in Abu
Dhabi. J Urol. 1985;134:274-275.

832 EDUCATIONAL REVIEW MANUAL IN UROLOGY


83. Zargooshi J. Penile fracture in Kermanshah, 94. Mydlo JH, Hayyeri M, Macchia RJ. Urethrog-
Iran: report of 172 cases. J Urol. 2000; raphy and cavernosography imaging in a small
164:364-366. series of penile fractures: a comparison with
surgical findings. Urology. 1998;51:616-619.
84. Karadeniz T, Topsakal M, Ariman A, Erton H,
Basak D. Penile fracture: differential diagno- 95. Dever DP, Saraf PG, Catanese RP, Feinstein
sis, management and outcome. Br J Urol. MJ, Davis RS. Penile fracture: operative man-
1996;77:279-281. agement and cavernosography. Urology.
1983;22:394-396.
85. Nicolaisen GS, Melamud A,Williams RD,
McAninch JW. Rupture of the corpus caver- 96. Meares EM, Jr. Traumatic rupture of the cor-
nosum: surgical management. J Urol. 1983; pus cavernosum. J Urol. 1971;105:407-408.
130:917-919.
97. Kalash SS, Young JD Jr. Fracture of penis:
86. Cendron M,Whitmore KE, Carpiniello V, controversy of surgical versus conservative
Kurzweil SJ, Hanno PM, Snyder HM, et al. treatment. Urology. 1984;24:21-24.
Traumatic rupture of the corpus cavernosum:
evaluation and management. J Urol. 98. Tan LB, Chiang CP, Huang CH, Chou YH,
1990;144:987-991. Wang CJ. Traumatic rupture of the corpus cav-
ernosum. Br J Urol. 1991;68:626-628.
87 Orvis BR, McAninch JW. Penile rupture. Urol
Clin North Am. 1989;16:369-375. 99. Tan LB, Chiang CP, Huang CH, Chou YH,
Wang CJ. Surgical treatment of the rupture
88. Fedel M, Venz S, Andreessen R, Sudhoff F, bladder: 22 years reviewed. J Formos Med
Loening SA. The value of magnetic resonance Assoc. 1990;89:986-991.
imaging in the diagnosis of suspected penile
fracture with atypical clinical findings. J Urol. 100. Penson DF, Seftel AD, Krane RJ, Frohrib D,
1996;155:1924-1927. Goldstein I. The hemodynamic pathophysiol-
ogy of impotence following blunt trauma to
89. Agrawal SK, Morgan BE, Shafique M, the erect penis. J Urol. 1992;148:1171-1180.
Shazely M. Experience with penile fractures in
Saudi Arabia. Br J Urol. 1991;67:644-646. 101. Cass AS, Gleich P, Smith C. Male genital
injuries from external trauma. Br J Urol.
90. Naraynsingh V, Ragu GC. Fracture of the 1985;57:467-470.
penis. Br J Surg. 1985;72:305-306.
102. Kanegaye JT, Schonfeld N.. Penile zipper
91. Gottenger EE, Wagner JR. Penile fracture with entrapment: a simple and less threatening
complete urethral disruption. J Trauma. approach using mineral oil. Pediatr Emerg
2000;49:339-341. Care.1993; 90-91.

92. Fergany AF, Angermeier KW, Montague DK. 103. Nolan JF, Stillwell TJ, Sands JP Jr. Acute
Review of Cleveland Clinic experience with management of the zipper-entrapped penis.
penile fracture. Urology. 1999;54:352-355. J Emerg Med. 1990;305-307.

93. Asgari MA, Hosseini SY, Safarinejad MR, 104. Wesson MB. Traumatism of the testicle; report
Samadzadeh B, Bardideh AR. Penile frac- of a case of traumatic rupture of a solitary tes-
tures: evaluation, therapeutic approaches and ticle. Urol Cutaneous Rev. 1946;50:16-19.
long-term results. J Urol. 1996;155:148-149.

CHAPTER 25: BLADDER, URETHRA AND GENITAL TRAUMA 833


105. Cass AS, Luxenberg M. Testicular injuries. 117. Herbener TE. Ultrasound in the assessment of
Urology. 1991;37:528-530. the acute scrotum. J Clin Ultrasound. 1996;
24:405-421.
106. Cass AS, Ferrara L, Wolpert J, Lee J. Bilateral
testicular injury from external trauma. J Urol. 118. Cunningham JJ. Sonographic findings in clin-
1988;140:1435-1436. ically unsuspected acute and chronic scrotal
hematoceles. AJR Am J Roentgenol. 1983;
107. Schuster G. Traumatic rupture of the testicle 140:749-752.
and a review of the literature. J Urol. 1982;
127:1194-1196. 119. Corrales JG, Corbel L, Cipolla B, Staerman F,
Darnault P, Guille F, et al. Accuracy of ultra-
108. MacDermott JP, Gray BK, Stewart PA. Trau- sound diagnosis after blunt testicular trauma
matic rupture of the testis. Br J Urol. 1988; [see comments]. J Urol. 1993;
62:179-181. 150:1834-1836.

109. Vaccaro JA, Davis R, Belville WD, Kiesling 120. Chen DC, Holder LE, Melloul M. Radionu-
VJ. Traumatic hematocele: association with clide scrotal imaging: further experience with
rupture of the testicle. J Urol. 1986;136:1217- 210 new patients. Part 2: Results and discus-
1218. sion. J Nucl Med. 1983;24:841-853.

110. Cass AS. Testicular trauma. J Urol. 1983; 121. Barloon TJ,Weissman AM, Kahn D. Diagnos-
129:299-300. tic imaging of patients with acute scrotal pain.
Am Fam Physician. 1996;53:1734-1750.
111. Romilly CS, Isaac MT. Male genital self-muti-
lation. Br J Hosp Med. 1996;55:427-431. 122. Altarac S. Management of 53 cases of testicu-
lar trauma. Eur Urol. 1994;25:119-123.
112. Culp DA. Genital injuries: etiology and initial
management. Urol Clin North Am. 1977; 123. McAninch JW, Kahn RI, Jeffrey RB, Laing
4:143-156. FC, Krieger MJ. Major traumatic and septic
genital injuries. J Trauma. 1984;24:291-298.
113. Monga M, Moreno T, Hellstrom WJ. Gunshot
wounds to the male genitalia. J Trauma. 1995; 124. Cass AS, Luxenberg M. Value of early opera-
38:855-858. tion in blunt testicular contusion with hemato-
cele. J Urol. 1988;139:746-747.
114. Bickel A, Mata J, Hochstein LM, Landreneau
MD, Aultman DF, Culkin DJ. Bowel injury as 125. Pohl DR, Johnson DE, Robison JR. Bilateral
a result of penetrating scrotal trauma: review testicular rupture: report of a case. J Urol.
of associated injuries. J Urol. 1990;143: 1017- 1968;99:772-773.
1018.
126. Horton CE, Dean JA. Reconstruction of trau-
115. Fournier GR Jr, Laing FC, Jeffrey RB, McAn- matically acquired defects of the phallus.
inch JW. High resolution scrotal ultrasonogra- World J Surg. 1990;14:757-762.
phy: a highly sensitive but nonspecific diag-
nostic technique. J Urol. 1985;134:490-493. 127. Meinhardt W, Kropman RF, Lycklama à Nije-
holt AA, Zwartendijk J. Skin necrosis caused
116. Wessells H, McAninch JW. Testicular trauma. by the use of negative pressure device for erec-
Urology. 1996;47:750. tile impotence. J Urol. 1990;144: 983.

834 EDUCATIONAL REVIEW MANUAL IN UROLOGY


8. Questions

128. Wheeler JS Jr, Steinberg J. Penile skin avul- All questions are taken from AUA practice exams.
sion managed by scrotal flaps. Surgical
Rounds. 1997;426-430. A. Bladder Trauma

129. Horton CE, McCraw JB, Devine CJ Jr, 1. Following an automobile accident, a 30-year-
Devine PC. Secondary reconstruction of the old comatose man has a blood pressure of
genital area. Urol Clin North Am. 1977;4: 110/70 mmHg, plus of 80/min, CVP of 12 cm
133-141. H2O and a urinary output of 40 ml/hour.
There is gross blood in the urine. Nasotra-
130. Revis DR Jr, Seagle MB. Skin grafts, Split- cheal intubation has been performed. The
thickness. eMedicine; 2000. first x-ray obtained should be:

131. Barnett A, Berkowitz RL, Mills R, Vistnes A. Skull


LM. Comparison of synthetic adhesive mois-
ture vapor permeable and fine mesh gauze B. Cervical spine
dressings for split-thickness skin graft donor
sites. Am J Surg. 1983;145:379-381. C. Chest

132. Genecov DG, Schneider AM, Morykwas MJ, D. IVP


Parker D, White WL, Argenta LC. A con-
trolled subatmospheric pressure dressing E. Cystogram
increases the rate of skin graft donor site re-
epithelialization. Ann Plastic Surg.
1998;40:219-225.
2. A 26-year-old woman has a pelvic fracture,
133. McDougal WS. Scrotal reconstruction using collapsed lung and a severe closed head
thigh pedicle flaps. J Urol. 1983;129:757-759. injury following an automobile accident. A
retrograde cystogram reveals an extraperi-
134. Cubillos J, Reda EF, Gitlin J, Zelkovic P, toneal bladder rupture. The next step in man-
Palmer LS. A conservative approach to testicu- agement is:
lar rupture in adolescent boys. J Urol. 2010;
184(4 Suppl):1733-1738. A. Catheter drainage

B. Immediate surgical repair

C. Diagnostic peritoneal lavage

D. Abdominal and pelvic CT scan

E. Suprapubic cystotomy

CHAPTER 25: BLADDER, URETHRA AND GENITAL TRAUMA 835


3. An intoxicated 45-year-old man with a his- 5. A 43-year-old woman sustains a single gun-
tory of chronic alcoholism is evaluated in the shot wound to the abdomen. You are con-
emergency room. Physical examination sulted at the time of emergency laparotomy
reveals no abnormalities other than ecchymo- for an obvious bullet hole in the dome of the
sis over the lower abdomen. The blood pres- bladder. You should:
sure is 160/80 mmHg, pulse 70, respirations
20, temperature 37.5 C and the CVP is 10 mm A. Open the bladder anteriorly and inspect
H2O. A plain film of the abdomen shows a the inside of the bladder
ground glass appearance. Initial blood stud-
ies reveal: HCT 32%, WBC 15,800/cu mm, B. Perform an intraoperative cystogram
Na 122 mEq/L, K 6.0 mEq/L, Cl 109 mEq/L,
CO2 13 mEq/L, BUN 80 mg/dL and creati- C. Debride the bullet hole and close it in 2
nine 4.3 mg/dL. A Foley catheter is placed layers
but there is no urine output. The next step in
management should be: D. Perform an intraoperative IVP

A. Kayexalate® and furosomide E. Place a ureteral stent

B. Cystogram

C. Renogram B. Urethral Trauma

D. Noncontrast CT scan 1. A 25-year-old pedestrian is struck by an auto-


mobile. On arrival in the emergency room, a
E. Tap the abdomen plain film of the pelvis reveals a left superior
and inferior pubic ramus fracture as well as a
fracture of the sacroiliac joint. Examination
of the patient reveals a suprapubic mass. No
4. A 24-year-old man is struck by a car and sus- blood is noted at the meatus and the prostate
tains multiple injuries including a pelvic frac- is in the normal position on digital examina-
ture. He has blood at the meatus and a retro- tion. The most appropriate initial diagnostic
grade urethrogram is normal. A catheter is test is:
passed and the bladder is filled with 200 cc of
contrast. A full and post-drainage film are A. IVP
normal. The next step should be:
B. Retrograde urethrogram
A. Evaluation of the upper tracts by CT scan
C. Cystogram
B. Repeat the cystogram
D. Pelvic CT scan
C. Leave catheter and irrigate as needed to
clear clots E. Peritoneal lavage

D. Flexible cystoscopy to exclude a urethral


or bladder injury

E. Intravenous urogram with tomograms

836 EDUCATIONAL REVIEW MANUAL IN UROLOGY


2. A 26-year-old uncircumcised man is shot in
the penis with a low velocity bullet from a .22
C. Genital Trauma

caliber handgun. He has voided a small 1. A 22-year-old man sustains a severe burn of
amount of grossly bloody urine and is now in his genitalia. There is marked bullous edema
urinary retention. A urethrogram shows dis- and eschar formation of the entire penis and
ruption of 1 cm of the penile urethra with much of the scrotum. He has had a Foley
extravasation of contrast material. The best catheter in his urethra to monitor urine out-
next step is debridement of the wound and: put. The most appropriate initial manage-
ment is:
A. Suprapubic tube
A. Radical eschar debridement
B. Patch graft urethroplasty
B. Split-thickness skin grafts as
C. Urethral catheter soon as possible

D. Island flap urethroplasty C. Antibiotic therapy and topical


cleansing with water
E. End-to-end reanastomosis
D. Remove the Foley and insert
a suprapubic tube

3. A 14-year-old young man has a straddle E. Observe until the wound


injury to the perineum. Physical examination begins to granulate
reveals ecchymosis limited to the penis and
scrotum. The fascia that contains the
extravasated blood is:
2. The preferred management of
A. Buck’s ruptured testis is:

B. Dartos A. Orchiectomy

C. Colles’ B. Closure of the tunica albuginea

D. External spermatic C. Orchiectomy and prosthesis infection

E. Transversalis D. Bed rest, scrotal elevation and ice packs

E. Incision and drainage of scrotum

CHAPTER 25: BLADDER, URETHRA AND GENITAL TRAUMA 837


3. A 23-year-old man suffers severe scrotal, 5. At the time of a newborn circumcision, the
penile and buttock burns. Four days later the distal one-half of the glans penis is ampu-
scrotal skin appears necrotic and malodorous tated, including the urethra. The prepuce and
with the testes visible. After giving antibi- glans have been kept in iced saline for 4
otics and performing local debridement, the hours. The best management is:
next step in management is to:
A. Primary anastomosis
A. Perform local wound care and delayed
reconstruction B. Graft of preputial skin for coverage

B. Perform split thickness skin grafts to C. Discard glans tip and allow secondary
cover testes healing

C. Place testes in subfascial thigh pouches D. Discard the glans tip and re-configure
remaining glans
D. Create lateral subcutaneous flaps to
cover the scrotum E. Primary anastomosis with microvascular
reconstruction
E. Place testes under subpubic subcuta-
neous space

6. A 16-year-old uncircumcised young man is


shot in the penis with a low velocity bullet
4. A 3-year-old boy is seen because his foreskin from a .22 caliber handgun. He has voided a
is caught in his zipper. The best treatment is: small amount of grossly bloody urine and is
now in urinary retention. A urethrogram
A. Circumcision shows disruption of 1 cm of the penile urethra
and extravasation of contrast material. The
B. Manipulation of the zipper under general best next step is debridement of the wound
anesthesia and:

C. Manipulation of the zipper under local A. Suprapubic tube


anesthesia
B. Patch graft urethroplasty
D. Divide the median bar of the zipper with
a bone cutter C. Urethral catheter

E. Excision of the piece of penile skin caught D. Island flap urethroplasty

E. End-to-end anastomosis

838 EDUCATIONAL REVIEW MANUAL IN UROLOGY


3. The most definitive study to rule out trau-
matic bladder rupture is:
2005 SESAP 5/150 Questions

1. A 13-year-old boy falls from a tree and devel-


ops gross hematuria. CT scan shows a left A. Pelvic CT scan
renal laceration extending into the collecting
system with significant urinary extravasa- B. Cystoscopy
tion. Contrast is seen in the distal ureter.
Three weeks following the injury, he devel- C. Pelvic ultrasound
ops a low-grade fever, an ileus and a tender
distended abdomen. CT scan shows a large D. CT cystogram
left urinoma. The next step is:
E. IVP
A. Placement of a urethral catheter

B. Percutaneous nephrostomy drainage

C. Open surgical drainage and renorrhaphy


2004 SESAP 3/150 Questions

1. A 20-year-old man sustains a circumferential


D. Percutaneous drainage of the urinoma avulsion of the skin of the midshaft of the
penis. There is intact skin on both the proxi-
E. Placement of a ureteral stent mal and distal aspects of the penile shaft,
with a denuded area 4 cm in length. The most
appropriate treatment is:

2. A 4-year-old boy fell from a second-story A. Primary approximation of the skin


window. On examination, his vital signs are
stable but he has right flank and upper quad- B. Split-thickness skin graft to the denuded
rant abdominal tenderness and fullness. He area
does not have peritoneal signs. Urinalysis is
negative. The next step is: C. Split-thickness skin graft and removal of
distal penile skin
A. Observation
D. Full-thickness skin graft to the denuded
B. Abdominal paracentesis area

C. Abdominal and renal ultrasound E. Scrotal rotational flap covering the


denuded area
D. IVP

E. Abdominal CT scan

CHAPTER 25: BLADDER, URETHRA AND GENITAL TRAUMA 839


2. A 23-year-old man notes a cracking noise and
subsequent penile pain during intercourse,
2003 SESAP 5/150 Questions

followed by progressive penile swelling and 1. A 42-year-old man is undergoing laparotomy


ecchymosis. He is initially embarrassed to for intraabdominal injuries and bladder rup-
seek medical attention despite persistent ture. Bleeding is noted in the perivesical area.
penile pain. Thirty-six hours after the trau- After repair of bladder rupture, pelvic pres-
matic event, he is a febrile with stable vital sure does not stop the persistent oozing. Mul-
signs. A retrograde urethrogram is normal. tiple blood transfusions are given and his
The next step is: core temperature is 35.5° C. The next step is:

A. Reassurance and cold compresses A. Intraoperative arteriography

B. Cavernosal-spongiosal shunt B. Ligation of the hypogastric arteries

C. Surgical exploration C. Intravenous aminocaproic acid

D. Foley catheter splinting D. Close the abdomen and place patient in a


MAST suit
E. Corporeal aspiration and Foley catheter
drainage E. Pack pelvis and close abdomen

3. A 10-year-old boy has a perineal “butterfly”


hematoma following a straddle injury. This
2002 SESAP 5/150 Questions

suggests rupture of the: 1. The optimal tissue for early coverage of the
perineum following an avulsion skin injury is
A. Tunica albuginea a(n):

B. Corpus spongiosum A. Island skin flap

C. Corpus cavernosum B. Musculocutaneous flap

D. Posterior urethra C. Full-thickness skin graft

E. Colles’ fascia D. Split-thickness skin graft

E. Dermal graft

840 EDUCATIONAL REVIEW MANUAL IN UROLOGY


2. A 23-year-old woman suffers a complex
pelvic fracture in a motor vehicle accident. A
Answers

cystogram reveals limited extraperitoneal


extravasation of contrast at the bladder neck.
Bladder Trauma

The bladder is compressed by a pelvic 1. B.


hematoma and an anterior vaginal laceration
is also present. No other injuries are noted 2. A.
and the patient is hemodynamically stable.
Treatment should be: 3. B.

A. Urethral catheter drainage 4. B.

B. Percutaneous suprapubic cystostomy 5. A.

C. Open bladder repair Urethral Trauma

D. Suprapubic cystostomy and perivesical 1. B.


drainage
2. E.
E. Repair of vaginal and bladder laceration
3. A.

Genital Trauma

1. B.
2001 SESAP 5/150 Questions

1. A 12-year-old prepubertal boy has severe


right scrotal pain one day after being kicked 2. B.
in the groin. There is a blue area over the
superior portion of the testis, but the exami- 3. A.
nation is difficult due to a hydrocele. Urinaly-
sis is normal. The next step is: 4. D.

A. Immediate exploration 5. A.

B. Scrotal ultrasound with Doppler 6. E.

C. Scrotal nuclear scan 2005 SESAP 5/150

D. Manual detorsion, exploration 1. D.

E. Observation, anti-inflammatory 2. E.
medications
3. D.

CHAPTER 25: BLADDER, URETHRA AND GENITAL TRAUMA 841


2003 SESAP 3/150

1. C.

2. C.

3. B.

2003 SESAP 5/150

1. E.

2002 SESAP 1/150

1. D.

2. E.

2001 SESAP 5/150

1. D.

842 EDUCATIONAL REVIEW MANUAL IN UROLOGY


Chapter 26:
Physiology and
Complications of
Laparoscopy
Jeffrey A. Cadeddu, MD

Contents

1. Physiologic Changes of Pneumoperitoneum

2. Complications

3. Special Circumstances

4. Complications of Kidney Tumor Ablation

5. Further Reading

6. Questions

CHAPTER 26: PHYSIOLOGY AND COMPLICATIONS OF LAPAROSCOPY 843


1. Physiologic Changes of
Pneumoperitoneum

Initially, laparoscopy was utilized for minor and


rapid gynecological interventions, but after 2
Respiratory Changes

decades, it is now used to treat many urological Hypercarbia


conditions. Laparoscopic and robotic-assisted Carbon dioxide (CO2), used for peritoneal insuffla-
laparoscopic interventions are now lengthy, com- tion, is noncombustible and is highly soluble in
plex and are often utilized to treat high-risk blood. However, it may be rapidly absorbed through
patients who present with multiple comorbid condi- the peritoneum and induce hypercarbia and acido-
tions. In addition to common surgical complica- sis. The absorption is increased with high intraab-
tions, such as bleeding and infection, laparoscopy dominal pressures (IAP) and prolonged procedures.
may result in a unique set of physiological distur- The compensatory renal excretion of H+ is too slow
bances and complications. Many of these can cause to correct the low bicarbonate levels in the extracel-
significant morbidity if not recognized and treated lular fluid, and acidosis results. The deep head-
promptly. This chapter will review the physiologi- down position (Trendelenburg), often utilized in
cal changes associated with abdominal laparoscopic/robotic prostate and bladder surgery,
laparoscopy and will discuss common complica- causes a significant rise in the intracranial pressure
tions of laparoscopic urological surgery emphasiz- and may worsen CO2 accumulation and hypercar-
ing their recognition and management. bia. The alternatives to CO2 insufflation include
nitrous oxide and inert gases such as helium, argon
and xenon. Nitrous oxide, however, supports com-
bustion and can cause distention of the bowel, while
inert gases are not readily soluble in blood and risk
an embolus.

Changes in Ventilatory Dynamics


High IAP decreases diaphragmatic motion, elevat-
ing intrathoracic pressure and limiting expansion of
the lungs. The functional residual capacity and pul-
monary dynamic compliance are decreased and
peak inspiratory pressures are elevated. That in turn
leads to increasing ventilation-perfusion mismatch
and shunting, which may result in hypoxemia, espe-
cially in patients with compromised pulmonary
reserves. High peak and plateau inspiratory pres-
sures can also cause ventilator-induced lung injury.

Pulmonary Complications

Hypoxemia
Adequate ventilation and oxygenation can limit
hypoxemia, but in patients with COPD, conversion
to open surgery may be required.

Atelectasis
Presents with low-grade fever in the first 24–48
hours following the procedure and is treated with
the intensive use of incentive spirometers, deep
breathing and coughing.

844 EDUCATIONAL REVIEW MANUAL IN UROLOGY


Subcutaneous Emphysema
Subcutaneous emphysema results from CO2 pass-
Table 1

ing into subcutaneous tissues and retroperitoneum


and presents with skin crepitus upon palpation. It is
Respiratory Effects of Pneumoperitoneum

often self-limited, but massive subcutaneous


emphysema involving the face and neck can
progress to pneumomediastinum and result in respi-
Parameter Change

Peak inspiratory pressure ? ratory compromise requiring intubation.

Gas Embolism
This is the most serious pulmonary complication
that can result in rapid cardiovascular collapse and
Chest wall mechanical resistance ?

death. It is extremely rare and is often caused by


direct insufflation of a vessel with a Veress needle
or rapid absorption of large amount of CO2 through
Pulmonary compliance ?

an injured vessel due to raised IAP. It presents with


sudden hypotension, cyanosis, increasing concen-
tration of end-tidal CO2 and a “mill wheel” mur-
Pulmonary dead space ?

mur over the right ventricle. If a gas embolism is


suspected the following interventions must be initi-
ated immediately to prevent additional emboli
Functional residual capacity ?

from going into the right ventricle outflow tract


Vital capacity ? and pulmonary vasculature:

• Release pneumoperitoneum

• Place the patient in the left lateral decubitus,


Shunting ?

head-down position

• Increase minute ventilation and administer


Ventilation-perfusion mismatch ?

100% O2

• Place a central venous line in an attempt to


aspirate the gas
Pneumothorax/Pneumomediastinum/
Pneumopericardium • Cardiopulmonary resuscitation may be necessary
Results from the barotrauma sustained during ele-
vated inspiratory pressures. The diagnosis is made
with a chest x-ray, and a chest drain may be neces-
Hemodynamic Changes

sary for resolution. Capnothorax is a CO2 accumu- Decrease in Cardiac Preload


lation in the intrapleural space. It is suspected when Compression of the inferior vena cava secondary to
the mean airway pressure suddenly rises, oxygen increased IAP decreases venous return and there-
saturation declines. Capnothorax often resolves fore reduces preload. Low volume status can exac-
spontaneously as CO2 is rapidly reabsorbed once erbate this effect.
the pneumoperitoneum is released. Chest tubes are
usually not necessary and treatment with oxygen is Increase in Cardiac Afterload
often sufficient (see Table 1). Hypercarbia enhances sympathetic activity and may
stimulate the renin-angiotensin cascade with resul-
tant vasoconstriction and increase in systemic vas-
cular resistance (SVR).

CHAPTER 26: PHYSIOLOGY AND COMPLICATIONS OF LAPAROSCOPY 845


Decrease in Cardiac Output
Decrease in cardiac preload and rise in SVR may
Table 2

decrease cardiac output despite tachycardia and


increased cardiac contractility arising from
Physiologic Effects of Pneumoperitonium

increased sympathetic tone.

Arrhythmias
Parameter Change

Rapid insufflation and stretching of peritoneum can


induce vagal-stimulated cardiovascular response
Heart rate ? ?

with significant bradyarrhythmias such as bradycar-


dia, bigeminy, unifocal and multifocal ventricular
ectopy, nodal rhythm, AV dissociation and asystole.
Mean arterial pressure ?

Most arrhythmias will respond to release of pneu-


moperitoneum and increase in minute ventilation
(see Table 2). Pharmacologic intervention by anes-
Systemic vascular resistance ?

thesia (e.g., atropine) may be necessary.


Venous return ?
Renal Changes

Oliguria
Oliguria during laparoscopy results from several
Central venous pressure ?

physiological derangements induced by intraperi-


toneal insufflation. Several distinct mechanisms,
such as direct compression of renal parenchyma and
Cardiac output ? ?

vasculature, decreased cardiac output, increased


activity of renin-angiotensin system and increase in
antidiuretic hormone release, are postulated to
Glomerular filtration rate ?

cause decreased urine production during abdominal


insufflation. Using warm CO2 for insufflation may
help with local renal vasodilation and limit the dele-
Urine output ?

terious effects of pneumoperitoneum on urine pro-


duction. However, the best prevention strategy is
maintenance of adequate intravascular volume
Intracranial pressure ?

throughout the procedure.

846 EDUCATIONAL REVIEW MANUAL IN UROLOGY


2. Complications

and rising creatinine. The mainstay of therapy is


vigorous hydration. Urine alkalinization has been
Complications of Patient Positioning

Laparoscopic procedures require longer surgical shown to be beneficial in animal studies, but clear
time and often rely on gravity to facilitate intraab- evidence of clinical efficacy has not been estab-
dominal organ retraction. Patients frequently spend lished. The best prevention strategy is careful
a significant time in a modified flank or padding of all pressure points, minimizing the use
Trendelenburg position and, therefore, proper posi- of kidney rest and limiting operative times
tioning and padding is critical to avoid neuromuscu-
lar injuries. Very rare cases of perioperative visual loss
(ischemic optic neuropathy) have been reported and
• The nerves most prone to injury are brachial attributed to the prolonged steep Trendelenburg
plexus, femoral, sciatic and lateral popliteal position associated with robotic pelvic surgeries.
Relative hypotension and an acute rise in intraor-
• To avoid injury to the brachial plexus the upper bital pressure due to patient position have been pos-
side arm should be supported by a pillow or a tulated to disrupt ocular perfusion and result in per-
folded blanket and care must be taken to ensure manent vision loss. Risk factors, including pre-
that the arm is not overly abducted or externally existing ocular diseases, have not been identified.
rotated to prevent the humeral head from applying
pressure on the brachial plexus Complications of Access

• The downside leg must be padded and gently • Intraperitoneal access and establishment of pneu-
flexed as the upside leg must be supported by pil- moperitoneum is most commonly achieved by 2
lows and elevated to minimize adduction at the hip distinct techniques: the Veress closed technique
and limit stretching of the sciatic nerve and the Hasson open technique

• Patients must be secured to the table with either • The closed technique consists of placing the
padded straps or padded tape at the level of the Veress insufflation needle into the abdomen (most
ankles, hips and shoulders to prevent falling and commonly through the umbilicus) blindly fol-
shifting during rotation of the table lowed by insufflation and insertion of the first tro-
car using an optical trocar system
• Neuromuscular injuries usually present with
paresthesias and weakness in the affected extrem- • The Hasson method consists of creating an open-
ity. Mild injuries will respond to physical therapy ing in the peritoneum under direct vision, a blunt-
and will frequently resolve over several weeks; tip trocar is introduced and pneumoperitoneum is
however, severe stretch injuries may result in per- established
manent neurological dysfunction
• Both approaches are relatively safe, with the
• Rhabdomyolysis is a serious complication of closed technique having a slightly higher inci-
improper positioning and involves muscle dence of vascular injury
ischemia following prolonged compression. Rhab-
domyolysis may result in acute renal failure sec- • Correct placement of Veress needle is confirmed
ondary to myoglobinuria and tubular obstruction with the following maneuvers:
by myoglobin casts. The risk factors for develop-
ment of rhabdomyolysis include morbid obesity, * Two “pops” should be felt as the needle
exaggerated intraoperative position, hypovolemia traverses the fascia and the peritoneum
with intraoperative hypotension, operative time >5
hours, diabetes, compromised renal function and * Aspiration of blood or bowel contents
hypertension. The condition should be suspected indicates improper position
in patients presenting postoperatively with exces-
sive muscular pain, oliguria, dark urine

CHAPTER 26: PHYSIOLOGY AND COMPLICATIONS OF LAPAROSCOPY 847


* Injection of 5–10 cc of saline and inability to
aspirate back indicates that the needle is in the
Intraoperative Complications

correct position Vascular Injuries


Vascular injuries are the most commonly reported
* “Drop test”: a bleb of saline left in the hub of complication of urologic laparoscopy and can result
the needle should fall freely down the needle in minor bleeding or severe life-threatening hemor-
rhage. Preventive measures include good exposure,
* Initial low pressure reading (<5 mm Hg) upon careful dissection and precise ligation of blood vessels,
insufflation confirms proper needle position clear visualization of the tip of dissecting instru-
ments and care not to manipulate hemostatic clips.

• Bleeding resulting from injury to small vessels


Vascular Injuries Due to Access

Common iliac artery can be injured by blind Veress (<4 mm) can be controlled with the use of Liga-
needle insertion, as the bifurcation of the great ves- Sure™ or Harmonic® scalpel devices. Hem-o-
sels is located approximately at the level of the lock® or metal clips can be used to ligate bleeding
umbilicus. If blood is aspirated initially upon Veress vessels unless the clips will interfere with applica-
needle placement, the needle must be withdrawn tion of a vascular stapler. Minor bleeding from the
slightly and the new position retested prior to insuf- renal vein and even vena cava can be controlled by
flation. Management of vascular injuries due to Ver- increasing the pneumoperitoneum to 20 mm Hg
ess needle depends on amount of bleeding. Many and applying direct pressure over the defect.
are small and at most will require a laparoscopic Placement of additional trocars may sometimes be
suture repair. Major vascular injuries, however, may necessary to provide retraction or to place a suc-
require immediate open conversion and formal tion/irrigator device. Once the injury is clearly
repair. Injuries to epigastric vessels occur during visualized, the adjunctive maneuvers may include
trocar placement through the lateral portion of the application of FloSeal and oxidized cellulose.
rectus muscle. This injury is diagnosed by observ- Intracorporeal suturing can be utilized as well, and
ing bleeding from the trocar site on the inside of the use of barbed sutures and/or Lapra-Ty® clips can
anterior abdominal wall. Management involves save valuable time by eliminating the need for
placement of full-thickness suture ligature through knot tying. Major vascular injuries resulting in
the fascia and the peritoneum under laparoscopic severe hemorrhage, expanding hematoma or
guidance using a fascial closure device. hemodynamic instability require immediate open
conversion and formal repair
Visceral Injuries Due to Access

An injury to colon, stomach or bladder is suspected


Injuries to Abdominal Viscera

when colored or malodorous fluid is aspirated Injuries to liver and spleen often occur when the
through the Veress needle. These organs must be instrument is outside the field of view. To prevent
carefully examined once pneumoperitoneum is these complications, it is important to follow the
established to see whether a formal repair is neces- instruments with the camera during insertion, espe-
sary. Injuries to these organs secondary to Veress cially when a novice surgeon is operating. Minor
needle placement are usually self-limited and rarely tears can be FloSeal-controlled with application of
require intervention. However, even a small perfo- oxidized cellulose, fibrin glue and gelatin thrombin
ration in the gall bladder may result in a bile leak matrix (Baxter, Inc., Deerfield, IL). Argon beam
and peritonitis and therefore a cholecystectomy is coagulation is very useful as well. Major lacerations
usually necessary. Veress needle injuries to the liver with significant bleeding may require conversion to
or spleen can usually be controlled by direct pres- open surgery and consultation with a general sur-
sure or application of hemostatic agents such as geon. Bleeding secondary to large splenic tears is
FloSeal® or fibrin glue. Argon beam coagulation is very difficult to control and a splenectomy may be
usually adequate to control bleeding from these necessary.
small punctures as well.

848 EDUCATIONAL REVIEW MANUAL IN UROLOGY


• Distal pancreas can be injured during left-sided • CT of the abdomen and pelvis with oral contrast
nephrectomy and adrenalectomy. Cautery must be has very high sensitivity for diagnosis of a bowel
avoided in close proximity to the tail of the pancreas, injury. If CT is indicative of a bowel perforation,
and if retraction of the pancreas is necessary, blunt an immediate exploration with bowel repair and/or
wide surface retractors must be utilized. If pancre- resection is necessary
atic injury is recognized intraoperatively, a general
surgery consultation must be obtained to establish • Rectal injury is a rare complication of robotic radi-
whether distal pancreatectomy is necessary cal prostatectomy. If recognized intraoperatively
and gross fecal spillage is absent, the perforation
can be closed primarily in 2 layers. Closed suction
drain should be placed near the repair and the
Gastrointestinal Injuries

• Unrecognized bowel injuries during abdominal patient’s antibiotic coverage must be broadened
laparoscopy can cause significant morbidity and for several days. However, gross fecal contamina-
even result in a fatality. The bowel is often injured tion, or inability to complete a primary repair
during applications of monopolar electrocautery require open conversion and a diverting colostomy
and during blind passage of instruments. Breaks in
insulation coating and poor visualization of • Occasionally, when developing a plane between
cautery contact points can result in thermal injuries the colonic mesentery and Gerota’s fascia, inad-
to bowel “off screen” which remain unrecognized. vertent incision in the mesentery creates a suffi-
This can occur with robotic instruments as well. cient opening for small bowel herniation. To pre-
Thermal spread from monopolar cautery can vent an internal hernia, which may lead to small
extend several centimeters from the operative site bowel obstruction, all mesenteric windows must
resulting in sufficient temperature elevations to be closed with clips or intracorporeal suturing. If
cause tissue destruction internal hernia is suspected, a CT is helpful in
establishing the diagnosis
• Patients with unrecognized bowel injuries often
present 1–2 days postoperatively and lack the typi- • Trocar site hernias causing small bowel obstruc-
cal signs and symptoms of peritonitis. If the diag- tion are rare complications of abdominal
nosis is missed, patients can succumb to sepsis and laparoscopy. In adults, 5 millimeter and 12 mil-
cardio-pulmonary collapse within 96 hours of limeter dilating trocars generally do not require
surgery. Clinicians must have a low threshold for formal fascial closure; however, the fascia of
ordering abdominal CT in patients presenting in 12 mm bladed trocar sites, especially in the mid-
the following fashion: line, should be closed using an absorbable suture,
and a fascial closure device. Trocar site hernia is
* Low-grade temperature suspected when a patient presents with a bulge at
the trocar site with or without signs of small bowel
* Abdominal distention and diarrhea with obstruction. CT is usually diagnostic. Manage-
persistent bowel sounds and absence of ment consists of open or laparoscopic exploration
peritoneal signs and reduction of the hernia. Bowel resection may
be necessary if the incarcerated loop appears non-
* Severe trocar site pain without purulence or viable
erythema

* Nausea and vomiting are uncommon


Diaphragmatic Injuries

• Diaphragm is usually injured by electrocautery


* Leucopenia with a left shift during laparoscopic renal and adrenal surgery.

CHAPTER 26: PHYSIOLOGY AND COMPLICATIONS OF LAPAROSCOPY 849


3. Special Circumstances

Diaphragmatic injuries can range from subtle—pre-


senting with increasing airway pressures and hyper-
Extraperitoneoscopy

carbia—to severe, causing a rapid cardiovascular • Hypercarbia secondary to CO2 absorption


collapse secondary to tension pneumothorax. Ten- is more pronounced, with extraperitoneal
sion pneumothorax must be treated immediately by laparoscopy, as compared to the intraperitoneal
release of pneumoperitoneum and decompression approach, especially in the presence of subcuta-
of the chest with a needle or tube thoracostomy. If neous emphysema
pleurotomy is identified intraoperatively, but the
patient remains stable, the injury can be suture • Hemodynamic changes are similar to those of
repaired at the end of the procedure once all the air intraperitoneal approach; however, decrease in
is expelled from the pleural cavity renal vein flow, oliguria and the rise in mean arte-
rial pressure are less pronounced
Ureteral Injuries

• Ureteral injuries usually result from electrocautery


Hand-Assisted Laparoscopy

burns that frequently remain unrecognized intra- • Analgesic requirements, corrected 2-week abdom-
operatively. Patients present with increased output inal/flank pain scores, and duration of convales-
from the drains, pain secondary to urinoma forma- cence are not affected by the presence of the hand-
tion or urinary ascites. Diagnosis is made with ret- port incision
rograde pyelography or intravenous urography.
Minor injuries with minimal extravasation can be • Physiologic derangements and complications of
managed with retrograde or antegrade stent inser- hand-assisted laparoscopy are similar to those of
tion or percutaneous nephrostomy drainage. Large the standard laparoscopic approach. However, the
injuries with significant tissue loss may require incidence of postoperative ileus and delayed
open reconstruction. If ureteral burn is recognized bowel function seems to be higher
intraoperatively, the affected segment must be
widely excised and ureteral continuity reestab- • The unique complications of hand-assisted
lished. Location of the injury dictates the method laparoscopy include herniation and dehiscence
of repair that should be utilized involving the hand port incision, although
both are rare

Laparoscopy in Neonates and Infants

• Young patients absorb proportionally more CO2


than older individuals and have a higher risk for
developing hypercarbia, which in turn could stim-
ulate the sympathetic nervous system and sensitize
the myocardium to the arrhythmogenic effects of
catecholamines

• Cardiovascular physiology of infants differs from


that of adults. Their blood pressure and systemic
vascular resistance are usually lower, where heart
rate, O2 consumption and cardiac output are
higher. High IAP in neonates and young children
may increase pulmonary vascular resistance
resulting in increased right-to-left cardiac shunting
and decreased cardiac index

850 EDUCATIONAL REVIEW MANUAL IN UROLOGY


4. Complications of Kidney
Tumor Ablation

• It is prudent to preheat CO2 to maintain normoth- • Both laparoscopic and percutaneous kidney tumor
ermia, especially during high flow insufflation and ablation are increasingly popular and viable alter-
in small babies. Cases of hypothermia due to CO2 natives to surgery for treatment of small renal
insufflation have been reported in neonates tumors. Complications associated with cryoabla-
tion and radiofrequency ablation techniques are
Laparoscopy During Pregnancy increasing and require familiarity by the urologist

• Insufflation of an abdomen containing a gravid • Minor Complications Include:


uterus can result in physiological changes that * Elevated serum creatinine
exceed those normally seen during laparoscopy.
Diaphragmatic expansion is significantly reduced * Hematuria
resulting in exaggerated changes in peak airway
pressure, functional residual capacity and ventila- * Minor hemorrhage (small perinephric
tion-perfusion mismatching hematoma)

• The primary concern regarding the safety of * Pain or paresthesia at the probe site
laparoscopic surgery during pregnancy is the
physiologic consequences of the CO2 insufflation • Major Complications Include:
to the fetus. Elevation of maternal partial pressure * Bowel injury — e.g., nephrocolic fistula
of arterial CO2 could potentially impair fetal CO2
excretion across the placenta and could exacerbate * Ileus
fetal acidosis. Perioperative monitoring of arterial
blood gases as well as perioperative fetal and uter- * Open conversion
ine monitoring is recommended even for healthy
parturients * Significant hemorrhage (large or expanding
hematoma—more common with cryoablation
Robotic Surgery of tumors >3 cm)

Physiologic derangements and complications of * Urinary extravasation—collecting system


robotic laparoscopy are similar to those of the stan- injury (reported with cryoablation and RFA)
dard laparoscopic approach. Robotic ports should
be placed a minimum of 8 cm apart to avoid clash- * UPJ obstruction (reported with cryoablation
ing of arms and poor access to the surgical field and RFA)

Lack of tactile feedback may increase risk of injury


to critical structures. All moving instruments should
Conclusions

be kept within the visual field to avoid inadvertent Practical knowledge of physiological changes and
injury. potential complications of laparoscopy, as well as
their prevention, timely diagnosis and proper man-
Tissue-crushing injuries may be sustained if the agement, is essential to get patients safely through
robotic arm position maintains pressure on an the complex and often prolonged urologic laparo-
exposed body area (eg, leg/foot during robotic scopic procedures. Thorough preoperative assess-
prostatectomy, abdominal wall especially in obese ment, careful perioperative monitoring, meticulous
patient). technique and good communication between sur-
geons and anesthesiologists are critical to minimiz-
ing surgical complications and improving patient
outcomes.

CHAPTER 26: PHYSIOLOGY AND COMPLICATIONS OF LAPAROSCOPY 851


5. Further Reading

5. Henny CP, Hofland J. Laparoscopic surgery:


pitfalls due to anesthesia, positioning, and
Book Chapters

1. Lee BR, Jarrett TW. Prevention and manage- pneumoperitoneum. Surg Endosc. 2005;19:
ment of laparoscopic complications. In: Smith 1163-1171.
AD, Badllani G, Bagley D, et al, eds. Smith’s
Textbook of Endourology. London, England: 6. Matsumoto ED, Cadeddu JA. Laparoscopic
BC Decker Inc; 2007:475-484. access: one goal, many choices. AUA Update
Series. 2006:25(Lesson 5).
2. Dunn MD, Monk TG. Anesthetic considera-
tions. In: Smith AD, Badllani G, Bagley D, 7. O’Rourke N, Kodali BS. Laparoscopic
et al, eds. Smith’s Textbook of Endourology. surgery during pregnancy. Curr Opin Anaes-
Hamilton, London, England: BC Decker Inc; thesiol. 2006;19: 254-259.
2007:385-394.
8. Truchon R. Anaesthetic considerations for
3. Pearle MS, Cadeddu JA. Physiologic effects laparoscopic surgery in neonates and infants:
of pneumoperitoneum. In: Smith AD, Badllani a practical review. Best Pract Res Clin Anaes-
G, Bagley D, et al, eds. Smith’s Textbook of thesiol. 2004;18:343-355.
Endourology. London, England: BC Decker
Inc; 2007:395-405. 9. Venkatesh R, Landman J, Sundaram C, Clay-
man RV. Prevention, recognition, and manage-
4. Eichel L, McDougall EM, Clayman RV. ment of laparoscopic complications of uro-
Basics of laparoscopic urologic surgery. In: logic surgery. AUA Update Series. 2003;22
Wein AJ, Kavoussi LR, Novick AC, Partin (Lesson 40).
AW, Peters CA, eds. Campbell-Walsh
Urology. Philadelphia, PA: Saunders Elsevier;
2007:171-220.
Original Articles

1. Reisiger KE, Landman J, Kibel A, Clayman


RV. Laparoscopic renal surgery and the risk of
rhabdomyolysis: diagnosis and treatment.
Review Articles

1. Conachar ID, Soomro NA, Rix D. Anaesthesia Urology. 2005; 66(suppl 5A):29-35.
for laparoscopic urological surgery. Brit J
Anaesth. 2004;6:859-64. 2. Bishoff JT, Allaf ME, Kirkels W, Moore RG,
Kavoussi LR, Schroder F. Laparoscopic bowel
2. Bird VG, Winfield HN. Laparoscopy in injury: incidence and clinical presentation. J
urology: Physiologic considerations. Urol Urol. 1999;161: 887-890.
Board Rev Man. 2002;10:1-21.
3. Vallancien G, Cathelineau X, Baumert H,
3. Gerges FJ, Kanazi GE, Jabbour-Khoury SI. Doublet JD, Guillonneau B. Complications of
Anesthesia for laparoscopy: a review. J Clin transperitoneal laparoscopic surgery in urol-
Anesth. 2006;18: 67-78. ogy: review of 1,311 procedures at a single
center. J Urol. 2002;168:23-26.
4. Hedican SP. Complications of hand-assisted
laparoscopic urologic surgery. J Endourol. 4. McDougall EM, Monk TG, Wolf JS Jr, et al.
2004;18:387-396. The effect of prolonged pneumoperitoneum on
renal function in animal model. J Am Coll
Surg. 1996;182: 317-328.

852 EDUCATIONAL REVIEW MANUAL IN UROLOGY


6. Questions

5. Strup SE, Hubosky SG, Trabulsi EJ, McGinnis 1. All of the following are properties of nitrous
DE, Diamind SM, Gomella LG. Complica- oxide insufflation, except:
tions of hand-assisted laparoscopic nephrec-
tomy: a review of 118 consecutive cases at a A. It doesn’t cause hypercarbia
single institution. J Urol. 2002;167(suppl):
1168. B. It is readily soluble in blood

6. Desai MM, Strzempkowski B, Matin SF, et al. C. It may cause bowel distention
Prospective randomized comparison of
transperitoneal versus retroperitoneal laparo- D. It doesn’t support combustion
scopic radical nephrectomy. J Urol. 2005:
173:38-41. E. It does not irritate the diaphragm or
peritoneum
7. Guillonneau B, Rozet F, Cathelineau X, et al.
Perioperative complications of laparoscopic
radical prostatectomy: the Montsouris 3-year 2. Which of the following are signs of a gas
experience. J Urol. 2002;167:51-56. embolism?

8. Cadeddu JA, Regan F, Kavoussi LR, Moore A. Sudden hypotension


RG. The role of computerized tomography in
the evaluation of complications after laparo- B. Cyanosis
scopic urological surgery. J Urol.
1997;158:1349-1352. C. Increasing end-tidal CO2

9. Chiu AW, Azadzoi KM, Hatzichristou DG, D. “Mill wheel” murmur


Siroky MB, Krane RJ, Babayan RK. Effects of
intra-abdominal pressure on renal tissue perfu- E. All of the above
sion during laparoscopy.
J Endourol. 1994;8:99-103.
3. All of the following maneuvers must be per-
10. Bergesio R, Habre W, Lanteri C, Sly P. formed immediately if one suspects a gas
Changes in respiratory mechanics during embolism, except:
abdominal laparoscopic surgery in children.
Anaesth Intensive Care. 1999;27:245-248. A. Release pneumoperitoneum

11. Steinbrook RA, Bhavani-Shankar K. Hemo- B. Increase minute ventilation and administer
dynamics during laparoscopic surgery in preg- 100% oxygen
nancy. Anesth Analg. 2001;93:1570-1571.
C. Place patient in a head-down, right lateral
12. Shankar KB, Mushlin PS. Arterial to decubitus position
end-tidal gradients in pregnant subjects.
Anesthesiology. 1997;87:1596-1598. D. Place a central venous line and attempt to
aspirate the gas
13. Johnson DB, Solomon SB, Su LM, et al.
Defining the complications of cryoablation E. Initiate CPR as indicated
and radio frequency ablation of small renal
tumors: a multi-institutional review. J Urol.
2004;172:874-877.

CHAPTER 26: PHYSIOLOGY AND COMPLICATIONS OF LAPAROSCOPY 853


4. All of the following mechanisms account for 7. The nerves most prone to injury secondary to
oliguria during laparoscopy, except: positioning for laparoscopy are all of the
following, except:
A. Direct compression of the renal parenchyma
A. Brachial plexus
B. Compression of renal vasculature
B. Lateral popliteal
C. Decrease in cardiac output
C. Sciatic
D. Compression of the ureter
D. Femoral
E. Increased activity of renin-angiotensin
access and antidiuretic hormone release E. None of the above

5. Which of the following is not true about diag- 8. All of the following confirm correct placement
nosis and treatment of perioperative rhabdo- of the Veress needle, except:
myolysis?
A. 2 pops are heard as the needle traverses the
A. It may present with muscular pain, oliguria fascia and the peritoneum
and dark urine
B. Inability to aspirate back 5–10cc of saline
B. Morbid obesity is a risk factor for develop- injected through the needle
ing rhabdomyolysis
C. Low pressure reading at initiation of
C. It may result in acute renal failure insufflation

D. Vigorous hydration is a mainstay of therapy D. A decrease in pressure with elevation of the


abdominal wall
E. Urinary alkalinization has been shown to be
beneficial in human studies E. None of the above

6. Laparoscopic bowel injury is associated with 9. Which of the following are true of the
all of the following, except: laparoscopic diaphragmatic injuries?

A. Peritoneal signs A. They are usually due to electrocautery

B. High fever B. They always result in a tension pneumotho-


rax and cardiovascular collapse
C. Severe nausea and vomiting
C. They cannot be repaired laparoscopically
D. High white blood cell count
D. When recognized, they must be repaired
E. All of the above immediately

E. All of the above

854 EDUCATIONAL REVIEW MANUAL IN UROLOGY


10. Regarding trocar site hernias, all of the follow-
ing are true, except:
Answers

1. D.
A. To prevent hernias the fascia of 12- mm radi- Nitrous oxide supports combustion.
ally dilating trocar sites must always be
closed 2. E.
All choices are correct and are common presenting
B. They can present as a tender or non-tender signs of gas embolus.
bulge at the trocar site
3. C.
C. They can cause small bowel obstruction Patient should be placed in a head-down, left lateral
decubitus position.
D. They can be repaired laparoscopically
4. D.
E. All of the above Ureteral compression has not been shown to be a
mechanism of oliguria of pneumoperitoneum.

5. E.
Alkalinization was found to be beneficial only in
animal studies.

6. E.
None of the choices are part of common presenta-
tion of laparoscopic bowel injury.

7. E.
All of the listed nerves are commonly affected.

8. E.
All of the choices indicate correct placement of the
Veress needle.

9. A.
Diphragmatic injuries are usually secondary to
electrocautery.

10. A.
Fascia of dilating trocar sites doesn’t need to be
closed. The actual diameter of the fascial opening is
about one-half of the trocar size.

CHAPTER 26: PHYSIOLOGY AND COMPLICATIONS OF LAPAROSCOPY 855


856 EDUCATIONAL REVIEW MANUAL IN UROLOGY
Chapter 27:
Evaluation and
Treatment of
Male Factor Infertility
Paul J. Turek MD, FACS, FRSM

Contents

1. Introduction

2. Reproductive Anatomy and Physiology

3. Clinical Evaluation of the Subfertile Male

4. Classification of Abnormalities

5. References

6. Questions

CHAPTER 27: EVALUATION AND TREATMENT OF MALE FACTOR INFERTILITY 857


1. Introduction 2. Reproductive Anatomy and
Physiology

The Problem Embryology

1.) Approximately 25% of couples will become 1.) Genital organs are observed during the 5th ges-
pregnant after 1 month of trying to conceive. tational week and include an indifferent gonad,
However, only 85% of couples will be preg- a mesonephric duct and the Müllerian ducts.
nant after 1 year.
2.) The indifferent gonad forms from a thickening
2.) Approximately 15% of couples will therefore in the urogenital ridge near the mesonephros;
have difficulty conceiving. germ cells migrate from the yolk sac and popu-
late the urogenital ridge. These cells are pri-
3.) During the infertility evaluation, a male factor mordial germ cells and are very closely related
alone may be found in approximately 30% of to embryonic stem cells.
couples, and both a male and female factor in an
additional 20%, such that a male factor may be 3.) Sexual differentiation of the embryo stems from
involved in the infertility problem in approxi- the presence or absence of sex determining
mately 50% of cases. region-Y (SRY) on the Y chromosome. The
SRY gene product is a protein that harbors a
high mobility group box (HMG) sequence, a
DNA-binding motif that kinks DNA. This
DNA-bending effect alters gene expression,
leading to formation of a testis and subsequently
to the male phenotype. Notably, XY individuals
who lack the SRY gene on the Y chromosome
are phenotypic females. Other genes are also
involved in determining gonadal sex, including
WT1, SOX-9, Wnt4 and DAX-1, and their
expression patterns may account for phenotypic
sex reversal cases (Figure 1).

4.) Induced to form a testis, the gonad develops


clustered cords of germ cells that converge to
form the rete testis at the hilum of the testis.
These are testis stem cells and serve as the
renewing source of germ cells for spermatogen-
esis throughout life. To further underscore their
stem cell-like character, adult human testicular
stem cells can be coaxed into embryonic-like
stem cells.

5.) During the 8th week of gestation, testosterone


is made by differentiating Leydig cells and then
declines in production after the 12th week, dur-
ing which external genital development occurs.

6.) In response to testosterone, the mesonephric


duct forms the ureter in both sexes and region-
ally specializes to form the vas deferens, epi-
didymis, ejaculatory ducts and seminal vesi-
cle in the male, joining with the testis in the form
of ductuli efferentes testis.

858 EDUCATIONAL REVIEW MANUAL IN UROLOGY


Figure 1

Genes involved in the early development cascade that determines gonadal sex. Abnormal expression of Wnt-4
or DAX-1 may account for cases of sex reversal in which SRY is not involved.

Abnormal development of mesonephric duct differential growth of the gubernaculum testis.


structures is relatively common, clinically pre- Descent into the scrotum is usually completed by
senting as infertility with azoospermia due to birth, although descent can still occur during the 1st
congenital absence of the vas deferens year of life. Dihydrotestosterone (DHT), a testos-
(CAVD), a form fruste of cystic fibrosis that is terone metabolite, masculinizes the genital anlage
associated with CFTR gene mutations but not to form the external genitalia, including the penis,
necessarily with disease symptoms. scrotum and prostate.

7.) The Müllerian duct develops into fallopian


tubes, the uterus and the upper portion of the
Gross Anatomy

vagina in the female; in the male, this develop- 1.) The male reproductive system includes the fol-
ment is inhibited by a Müllerian-inhibiting lowing components: the testes and seminiferous
substance (MIS) produced by the primitive tubules, efferent ductules and rete testis, epi-
testis (Figure 2). Except for the appendix testis didymides, vasa deferentia, ejaculatory ducts,
and prostatic utricle, regression is otherwise seminal vesicles, prostate, penis and urethra.
complete. If MIS is not secreted, an intersex con-
dition termed Persistent Müllerian Duct Syn- 2.) To understand infertility, any consideration of
drome can result. This familial X-linked or auto- anatomy must also include the hypothala-
somal-dominant disorder presents as a male mic/pituitary/gonadal axis.
patient with normal male external and internal
genitalia, but also a fully developed uterus and Reproductive Hormonal Axis (Figure 3)
fallopian tubes due to incomplete involution of
Müllerian derivatives. Clinically, there may be a 1.) Components
hernia that contains a uterus and fallopian tubes
(hernia uteri inguinalis) or cryptorchidism. a. Extrahypothalamic central nervous system

8.) Later in gestation, Leydig cells also synthesize b. Hypothalamus


Insulin-Like-3 to promote trans abdominal
testis migration that begins testis descent into c. Pituitary
the scrotum. The testis descends as a result of

CHAPTER 27: EVALUATION AND TREATMENT OF MALE FACTOR INFERTILITY 859


Figure 2

Pathway of male gonadal development. Secretion of MIS (Müllerian-inhibiting substance) from Sertoli cells
within the early male gonad governs germ cell allocation and Müllerian duct inhibition. Testosterone leads to
Wolffian duct development.

d. Testes Extrahypothalamic Central Nervous

e. Steroid-sensitive organs
System

1.) The extrahypothalamic central nervous system


2.) Functions is responsible for a variety of stimulatory and
inhibitory influences on fertility, including
a. Normal male sexual development stress and catecholamines.

b. Maintenance of secondary sexual 2.) In humans, the effects of stress of both a physi-
characteristics cal and/or emotional nature are mediated
through this system, but the mechanisms are
c. Male sexual behavior unknown. In studies of acute physical stress in
military recruits, testosterone LH decrease.
d. Sperm production and maturation

860 EDUCATIONAL REVIEW MANUAL IN UROLOGY


Figure 3

The hypothalamic-pituitary-gonadal hormone axis. Both inhibitory and stimulatory feedback loops within the
axis. Testosterone balance occurs in a negative feedback loop. GnRH – gonadotropin-releasing hormone; PRL
- prolactin; T – testosterone; LH – luteinizing hormone; FSH – follicle-stimulating hormone.

1.) The effect of GnRH is the production and


release of LH and FSH from the anterior pitu-
Hypothalamus—Gonadotropin-Releasing

itary gland.
Hormone (GnRH)

1.) The hypothalamus is the center of integration


for neuronal and humoral messages in the brain. 2.) Both LH and FSH are glycopeptides with 2
The anterior and ventromedial nuclei are most molecular chains. They share a common alpha
important for male fertility. chain; specificity is determined by a unique beta
chain.
2.) The hypothalamus is responsible for production
of GnRH, the primary releasing substance 3.) LH and FSH are secreted episodically. LH is
involved in male sexual function. rapidly metabolized, causing wide swings in
concentration in the bloodstream. When more
accurate LH levels are needed, pooled blood
samples are used. FSH is more slowly

CHAPTER 27: EVALUATION AND TREATMENT OF MALE FACTOR INFERTILITY 861


metabolized, resulting in a more constant serum transport processes across Sertoli cells. Further
levels. compartmentalization is established as Sertoli
cells divide developing germ cells into 2 areas,
4.) The testes are the primary target for LH and the basal compartment for immature germ
FSH. No other target organs for these hormones cells and stem cells and the adluminal com-
have been found. partment for germ cells undergoing differenti-
ation and maturation. This complex barrier pro-
vides an immunologically privileged site for
mature spermatozoa, as these haploid cells
The Testes

1.) Microscopic anatomy harbor unique and specific antigens that are not
otherwise recognized as self by the body’s
a. Seminiferous tubules comprise the bulk immune system.
(80%) of the testis and are responsible for
sperm production. Thus, testis atrophy sug- 3.) Seminiferous tubules—spermatogenesis
gests a low sperm count.
a. LH, FSH and testosterone are all required
b. The interstitium between the seminiferous for normal spermatogenesis.
tubules contains blood vessels, lymphatics
and Leydig cells. b. Sertoli cells, lining 250m of seminiferous
tubules in the average testis, regulate the
2.) Seminiferous tubule structural organization complex process of spermatogenesis.

a. The tubules are long ducts lined by Sertoli c. A variety of germ cell types exist, including
cells that engulf and nurture the developing spermatogonia, primary spermatocytes,
germ cells. secondary spermatocytes, spermatids and
spermatozoa. 14 germinal cell subtypes are
b. Sertoli cells contain membrane receptors that recognized histologically and are associated
bind FSH, resulting in increased intracellular with 6 distinct stages of spermatogenesis.
cAMP and subsequent cytoskeletal reorgani- Early-stage spermatogonia are actually adult
zation for protein synthesis. testis stem cells and are the source of the con-
stant self-renewal germ cell lineage. Sper-
c. The primary secretion products from Sertoli matids and spermatozoa have a haploid com-
cells include MIS in the fetus, androgen- plement of chromosomes.
binding protein (ABP), transferrin and
inhibin (a nonsteroidal glycoprotein) in the d. The process of spermatogenesis takes about
adult. 60 days to complete. The average daily out-
put is 125 million spermatozoa, which
d. Sertoli cells regulate of the tubule microenvi- declines with age. A normal man makes 1200
ronment. They govern fluid secretions, sperm for every heartbeat.
phagocytosis, steroid metabolism (in part),
sperm production and sperm movement dur- e. Spermiogenesis is the maturation process
ing development. of a spermatid to a spermatozoon. This
includes nuclear condensation and pro-
e. Sertoli cells are also responsible for the grammed repackaging of DNA from histones
blood-testis barrier. In addition, a selective to protamines, acrosome formation, residual
physiologic barrier is created by active body separation from the sperm and tail for-
mation. It is one of the most complex mor-
phological changes undertaken by a mam-
malian cell.

862 EDUCATIONAL REVIEW MANUAL IN UROLOGY


4.) Interstitium—Leydig cells c. Estradiol is produced within the testicle
and the liver upon conversion from testos-
a. Leydig cells contain membrane receptors that terone (5-alpha reductase). It is found in
bind LH, resulting again in cAMP production, smaller amounts within the blood stream
protein kinase activation and protein phos- (testosterone:estrogen ratio is typically 10:1)
phorylation. but is potent in action. The site of regulation is
also at the level of the pituitary and hypotha-
b. LH stimulation results in the conversion of lamus.
cholesterol to testosterone in a steroidogenic
pathway. 3.) FSH regulation

c. Testosterone diffuses into the plasma a. Testosterone and estradiol are the major mod-
(endocrine function) or into the seminiferous ulators of pituitary FSH secretion. This is why
tubule lumen (paracrine function). In the exogenous testosterone supplements suppress
plasma, testosterone is bound (98%) to sex spermatogenesis.
hormone–binding globulin (SHBG) or
albumin. Within the seminiferous tubules, b. In man, Sertoli cells produce inhibin, a 2-sub-
testosterone is bound to ABP. unit hormone in the transforming growth fac-
tor family, which has an inhibitory effect on
d. Depending on the target tissue, testosterone pituitary FSH output. In contrast, activin, a
may be active itself or may be reduced to glycoprotein formed as a homodimer of either
dihydrotestosterone (DHT) by the enzyme inhibin chain, has a stimulatory effect on pitu-
5-alpha reductase. itary FSH. Neither inhibin nor activin affects
pituitary LH release.
e. Testosterone is responsible in part for sexual
differentiation, spermatogenesis,
gonadotropin regulation, sexual maturation
Testicular Transport

and behavior. 1.) Human sperm production, unlike most mam-


malian species, occurs in a patchwork pattern
and not in a wave-like manner within the semi-
niferous tubules. This, in combination with con-
Feedback Mechanisms

1.) GnRH, LH and FSH drive the production of stantly renewing cycles of spermatogenesis,
testosterone and spermatozoa as noted above. insures that sperm production is continuous
There are also feedback mechanisms that regu- in nature.
late the production and release of these sub-
stances. 2.) Movement of the spermatozoa from the testis to
the epididymis is controlled by: fluid pressure
2.) LH regulation within the seminiferous tubule, myoepithelial
contractions of the seminiferous tubules and
a.) Testosterone and estradiol are the major neg- tunica albuginea, and cilia within the efferent
ative feedback substances that control the ductules.
formation and release of LH.

b. Testosterone therefore regulates its own


production and release by acting on the pitu-
itary and in hypothalamus. Thus, exogenous
testosterone supplements will suppress both
endogenous testosterone (through decreased
LH) and sperm production (through decreased
FSH).

CHAPTER 27: EVALUATION AND TREATMENT OF MALE FACTOR INFERTILITY 863


Epididymal Functions Semen Composition

1.) Transport and storage 1.) The bulk of seminal fluid originates from the
accessory ducts, with the spermatozoa adding a
a. The spermatozoa traverse the length of the small (<10%) amount to the total volume.
epididymis in approximately 12 days. This
process is governed by regular slow contrac- 2.) Prostatic fluid
tions of the muscular wall in a fashion similar
to intestinal peristalsis. a. The prostatic fluid is usually found in the
first part of the ejaculate and contributes
b. Approximately 700 million sperm are approximately 10%-20% of the total vol-
stored within the epididymides and vasa ume. This fluid is acidic (pH <6.5).
deferentia. Approximately 60% of sperm are
stored within the tail (cauda) of the epi- b. Specific prostate products include liquefac-
didymides. Sperm become progressively tion factors like prostate-specific antigen
more motile as they traverse the epididymal (PSA), zinc, citric acid, acid, phosphatase and
tubules, an important fact in harvesting sperm spermine. The latter substance, when oxi-
from the epididymis for in vitro fertilization dized to aldehydes, produces the characteris-
(IVF) with intracytoplasmic sperm injection tic odor of semen.
(ICSI).
c. PSA, a 33-kd molecular weight serum pro-
c. At emission, coordinated contractions of the tease in the family of glandular kallikreins,
tails of the epididymides and the vasa defer- serves to liquefy the coagulum of human
entia occur, mediated by the sympathetic ner- semen after 5–20 minutes following ejacula-
vous system, propelling sperm into the pro- tion.
static urethra. During ejaculation, the
somatic nervous system stimulates rhythmic 3.) Seminal vesicle fluid
contractions of periurethral and pelvic floor
muscles propel the sperm through the urethra. a. The seminal vesicle fluid is usually found in
the second part of the ejaculate and con-
2.) Sperm maturation tributes approximately two-thirds of the
total volume. This fluid is basic (pH >7.0)
a. Motility and fertilizing capacity are gained
during transport through the epididymis. b. Specific substances secreted by the seminal
vesicles include coagulation factors,
b. Sperm capacitation takes place after the prostaglandins and fructose. Fructose is
sperm have been ejaculated and are in contact measured on a semen analysis to investigate
with the female reproductive tract. Fertilizing the diagnosis of seminal vesicle agenesis or
capacity lasts approximately 48 hours within ejaculatory duct obstruction.
the female internal genitalia, important for
counseling patients on the optimum frequency
of sexual intercourse during ovulation.

864 EDUCATIONAL REVIEW MANUAL IN UROLOGY


3. Clinical Evaluation of the
Subfertile Male

Fertility History (Figure 4) 2.) Previous partner history.

a. In past relationships, the number of pregnan-


1.) Present partner history: cies achieved should be determined.

a. Duration of infertility. b. If conceptions occurred in past relationships,


but not in the current one, then the infertility
b. Contraceptive methods and time used. is considered primary.

c. Length of time trying to conceive. 3.) Sexual history

d. Number of pregnancies, including miscar- a. Frequency of intercourse and masturbation.


riages and abortions, which are indicative of Overly frequent (daily) or infrequent (>48
the potential to conceive. hours apart around the time of ovulation) of
either action can adversely affect conception.
Figure 4

Algorithm for diagnostic evaluation of male factor infertility. Adapted with permission from: Turek- P. Nat Clin
Pract Urol. 2: 1-13.

CHAPTER 27: EVALUATION AND TREATMENT OF MALE FACTOR INFERTILITY 865


b. Libido, potency and sexual technique. Prob- d. Trauma or torsion. Either condition may
lems in these areas are often overlooked by injure the duct system or result in ischemic
clinicians. Situational erectile dysfunction damage to the seminiferous tubules.
due to stress is common among couples trying
to conceive and is easily treated with phos- e. Exposure to chemicals (Table 1). A variety of
phodiesterase inhibitors. drugs and industrial compounds may be asso-
ciated with abnormal semen analyses. Indus-
c. Ejaculation. Ejaculation needs to occur deep trial solvents are the most concerning.
within the vagina. Severe problems with pre- Chemotherapy agents are also gonadotoxic.
mature ejaculation, chordee or severe
hypospadias may prevent proper deposition f. Exposure to heat. Prolonged exposure to high
of sperm. temperatures may adversely affect spermato-
genesis. Hot tubs and hot baths on a regular
d. Dyspareunia and the use of lubrication. basis may have significance in this regard.
Inadequate natural vaginal lubrication can
produce painful intercourse for either partner. g. Exposure to radiation. Even small amounts
Most lubricants are spermicidal. The use of of ionizing radiation, particularly that used
vegetable oil–based substances are safe for for medical radiation therapy or even diag-
sperm viability. nostic radiology, may destroy sperm-forming
cells. Spermatogonia are particularly
e. The ovulatory cycle. It is important that the radiosensitive.
couple understand when ovulation occurs and
have timed intercourse during this time. Evi-
dence suggests that having more sex before
rather than after ovulation improves preg-
nancy rates.
Table 1

4.) Genitourinary history


Drugs and Chemicals with Potential
Adverse Fertility Effects

a. Testicular descent. Bilateral cryptorchidism


is associated with impaired spermatogenesis
and fertility. With unilateral maldescent, there
Alcohol

is a slight decrease in fertility potential.


Chemotherapy
Arsenic

b. Sexual development and onset of puberty.


Aspirin (large doses)

Delayed puberty can indicate syndromes


Caffeine

(eg, Kallmann) or chromosomal issues (eg,


Cimetidine

XXY).
Colchicine
Dibromochloropropane (pesticide)

c. Infections. Venereal, nonvenereal, mumps (at


DES

puberty or later), recent febrile illness or other


Heavy metals: lead, cadmium, mercury

infectious problems that directly involve the


Monoamine oxidase inhibitors

genitalia or urogenital duct system may be


Marijuana

associated with a significant scarring and sub-


Medroxyprogesterone

sequent fertility problems. Viral infections


Nicotine

and other febrile illnesses not specifically


Nitrofurantoins

involving the genitalia may also decrease


Phenytoin

spermatogenesis and temporarily lower


Spironolactone

sperm counts.
Sulfasalazine
Testosterone

866 EDUCATIONAL REVIEW MANUAL IN UROLOGY


a. a. Medical illnesses. Medical problems such
as diabetes and hypertension, or their phar-
Table 2

macologic treatments, may adversely affect


erectile and ejaculatory function and hence
Semen Analysis: Minimal Standards of

fertility.
Adequacy

Seen on at least 2 occasions:

b. Abnormal sex steroid metabolism may be


associated with various liver and renal dis-
Ejaculate volume 1.5-5.0 ml

eases and interfere with spermatogenesis.


Sperm density >15 million/ml

Rare medical illnesses like Young syndrome


Motility >40% motile

(immotile cilia syndrome) and Kartagener


Forward progression >2.0 (scale 0-4)

syndrome (ciliary defect with situs inversus)


Morphology >4% normal morphology

can adversely affect fertility.


by Strict criteria

2.) Surgical history


And:

a. Inguinal herniorrhaphy, particularly when


No significant sperm agglutination

performed on a young child and when per-


No significant pyospermia

formed bilaterally, may be associated with


No hyperviscosity

injury to the vas deferens in 1%–2% of


patients.
5.) Previous infertility evaluation
b. Surgery on the ureter, bladder, bladder neck
a. Patient. Previous semen analyses and medical or urethra may result in problems with emis-
or surgical treatment are important for prog- sion and/or ejaculation.
nosis and the diagnosis of infertility etiology.
c. Retroperitoneal surgery and major pelvic
b. Wife/partner. It is wise to understand the fer- procedures may result in failure of emission
tility potential of the female partner. Obvi- and/or problems with retrograde ejaculation.
ously, conception involves 2 partners and if Men with testis cancer are cured with greatm
an irreversible problem exists in the female success. Those with nonseminomatous
partner, pregnancy is not likely. The evalua- tumors treated with retroperitoneal lym-
tion of each partner should be carried out phadenectomy may have fertility problems
simultaneously; the male evaluation should due to absent ejaculation from either failure
be completed prior to any invasive proce- of emission or from retrograde ejaculation. 2
dures on the female partner. At least 1 year of or 3 adjuvant cycles of chemotherapy given in
maternal reproductive potential should lieu of retroperitoneal lymphadenectomy do
exist for the best outcomes after male not have a significant impact on fertility.
infertility treatment.
3.) Current and past medications

a. A variety of drugs and chemotherapeutic


General Medical History

1.) While most conditions causing male infertility agents may adversely affect sperm production
do not lead to long-term medical consequences, and/or function. The adverse affects are usu-
1%-2% of men undergoing evaluation for ally reversible upon discontinuation of the
infertility will have a significant underlying medication (Table 1).
medical condition.

CHAPTER 27: EVALUATION AND TREATMENT OF MALE FACTOR INFERTILITY 867


4.) Occupation and habits b. Testes. The location, size and consistency of
the testes should be noted. The testes should
a. Occupation and stress. The effects of daily be in a dependent part of the scrotum. Testic-
stress on fertility are poorly quantified. ular size is particularly important, as the bulk
Severe, acute stress has been associated of the testicle (80%) relates to sperm produc-
with decreases in testosterone, ejaculate tion. Note the length and width of the testicle
volume and sperm count. Encouraging reg- or quantify the volume by comparison to
ular exercise, yoga or acupuncture may help. plastic models of known volumes. The nor-
mal testicle in the adult is >4 cm in length
b. The active ingredients in cigarettes, mari- and >2.5 cm in width (20 mL). Each testicle
juana, coffee, tea, alcohol and some naturo- should have a firm consistency. Ethnic varia-
pathic herbs have all been demonstrated in lab- tions in testis size are well described.
oratory studies to be gonadotoxic. The suscep-
tibility of individuals to these substances varies c. Epididymides. The epididymides should be
widely and is difficult to quantify. examined for size and consistency. The
obstructed epididymis feels enlarged and
5.) Family history firm. The epididymis that is scarred from
either trauma or chronic infection may be
a. Sibling fertility status may help to identify hard and irregular. Part of the epididymis may
familial conditions like cystic fibrosis, be missing in association with congenital
genetic infertility due to Y chromosome dele- absence of the vas deferens.
tions and congenital adrenal hyperplasia.
d. Vasa deferentia. Each vas deferens should be
b. In utero exposure to diethylstilbestrol (DES) palpable as a distinct, firm, cord-like struc-
may result in testicular, epididymal and ture in the scrotum. In 1%-2% of infertile
penile anatomic abnormalities. Despite men, 1 or both vasa are not palpable in
the finding of impaired semen quality in the scrotum, a condition termed congenital
DES-exposed men, proof of decreased fer- absence of the vas deferens (CAVD).
tility is not obvious in extensive follow-up
studies. e. Spermatic cords. Each should be evaluated
for size and consistency with the patient in
upright and supine positions. The patient
should perform a Valsalva maneuver while
Physical Examination

1.) General examination. The general examination standing to accentuate differences in blood
evaluates the patient’s body habitus and sec- volume within the cord, characteristic of a
ondary sex characteristics. In particular, the pat- varicocele. The internal spermatic veins fill
tern of hair distribution and gynecomastia can while standing and this filling may be
be evidence of general endocrine disorders. increased with this maneuver. In the supine
position, these veins drain more easily and
2.) Genitalia. The genitalia are the most important hence the varicocele is not as easily palpable.
aspect of the examination. This should be per- Persistent fullness in the spermatic cord upon
formed in a systematic fashion, taking care to reclining is suggestive of cord lipoma and
evaluate the areas listed below: not varicocele. Other abnormalities involving
the spermatic cord include hydrocele and
a. Penis. The size of the penis and location of the spermatocele.
meatus are important in assuring the delivery
of spermatozoa deep within the vagina at
ejaculation.

868 EDUCATIONAL REVIEW MANUAL IN UROLOGY


f. Inguinal region. The inguinal canals are pal- c. Viscosity. This parameter refers to the
pated for evidence of hernias. In addition, the fluid consistency of the semen after
inguinal regions should be inspected to assess coagulation and liquefaction have
if previous surgery in this area may have occurred. Viscosity is normal if it is pos-
injured the vasa deferentia or testicular blood sible to pour the semen in a drop by drop
supply. fashion.

3.) Rectal examination. A general rectal examina- d. pH. The pH of semen normally ranges
tion can detect lower gastrointestinal pathology from 7.2–8.0. A low pH implies absence
and palpate the prostate and seminal vesicles. or blockage of the seminal vesicles, as
The prostate should be small and benign in con- the ejaculate consists entirely of acidic
sistency, without tenderness or inflammation. prostatic fluid.
The seminal vesicles are generally not palpable
under normal conditions but may be palpable e. Fructose. As noted above, fructose is
with ejaculatory duct obstruction. produced by the seminal vesicles. Fruc-
tose is checked in low volume (<1.5 mL)
azoospermia. Absence of fructose in
semen suggests absence of the vasa def-
Semen Analyses

1.) 1.) Collection. Generally, 2 semen analyses are erentia and seminal vesicles, ejaculatory
needed to establish a baseline. If a significant duct obstruction or dysfunction of the
discrepancy exists, a 3rd or perhaps even a 4th seminal vesicles.
specimen may be needed. Each semen analysis
is collected after 2-3 days of sexual abstinence.
The specimen is generally collected by mastur-
Other Laboratory Tests

bation into a clean, dry container and examined 1.) Urinalysis. This is useful to rule out infection of
within 1 hour. If the specimen is collected at the lower genitourinary tract and associated
home, it should be kept near body temperature glandular structures.
during transportation to the laboratory (shirt
pocket). Lubricants should be avoided during 2.) Endocrine evaluation. 99% of endocrine con-
specimen collection. If needed, silicone condom ditions can be detected through the initial
devices are available. measurement of serum FSH and testosterone.
Prolactin and LH can be included if the initial
2.) Minimal standards of adequacy. While there is evaluation is abnormal. Hormonal testing can
no absolute measure of fertility on semen analy- differentiate between hypogonadism due to
sis, minimal standards of semen adequacy have hypothalamic or pituitary disease and primary
been defined with expert consensus by the testicular failure (Table 3).
World Health Organization (Table 2).
3.) Other tests regularly recommended in the past are
3. Additional physical parameters. Other semen now unnecessary on a routine basis. These include
properties examined on a routine semen analysis testing thyroid and adrenal function (<0.5% of
include: cases). Prolactin should be determined if a low
testosterone is found or if the patient has gyneco-
a. Coagulation. This occurs immediately mastia, severe headaches or visual disturbances.
after ejaculation. Prolactin-secreting pituitary tumors produce high
levels of circulating prolactin, and these lesions
b. Liquefaction. This occurs 5–30 min fol- tend to reduce LH and FSH levels by altering reg-
lowing ejaculation. Prostatic-specific ulation or by mass effect.
antigen (PSA) is the serine protease
responsible for this process.

CHAPTER 27: EVALUATION AND TREATMENT OF MALE FACTOR INFERTILITY 869


Table 3

Common Endocrine Abnormalities

Condition Testosterone FSH LH Prolactin

Normal Normal Normal Normal Normal

Primary Testicular Low High High Normal


Failure

Hypogonadotropic Low Low Low Normal


Hypogonadism

Hyperprolactinemia Low Low Low High

Androgen Resistance High High High Normal

4.) Genetic testing. Karyotype analysis is critical testicular health, because shape characteris-
for all men with azoospermia and severe tics are determined during spermatogenesis.
oligospermia (<5 million sperm/mL) who are Sperm morphology complements other infor-
planning IVF/ICSI. In addition, specific dele- mation to estimate the chances of fertility.
tions in the Y-chromosome are found in approxi-
mately 15% of men with azoospermia and b. Sperm chromatin assay. The structure of
5%–8% of men with severe oligospermia. Sev- sperm chromatin (the DNA-associated pro-
eral regions of the long arm of the Y-chromo- teins) is independent of semen quality and can
some have been classified as Azoospermic Fac- be measured by COMET and TUNEL assays
tor (AZF) regions. Most deletions are found in and by flow cytometry after acid treatment
the AZFc region in a specific gene complex and staining of sperm with acridine orange.
termed DAZ (Deleted in Azoospermia). Dele- These tests assess the degree of DNA frag-
tions in these regions appear specific for infer- mentation that occurs after chemically stress-
tility. Deletions of AZFa and AZFb have a much ing the sperm DNA-chromatin complex, and
poorer prognosis for sperm retrieval from the can indirectly reflect the quality of sperm
testis than patients with AZFc deletions. DNA integrity. Abnormally fragmented
sperm DNA rarely occurs in fertile men,
5.) Tests of sperm function but can be found in 5% of infertile men
with normal semen analyses and 25% of
a. Sperm morphology. Sperm cytology is infertile men with abnormal semen analy-
another measure of semen quality. By assess- ses. This test can detect infertility that is
ing the dimensions and shape characteristics missed on a conventional semen analysis.
of the sperm head, midpiece and tail, sperm Often reversible, causes of DNA fragmenta-
can be classified as normal or not. In the tion include tobacco use, medical disease,
strictest classification system (Kruger mor- hyperthermia, air pollution, infections and
phology), only 14% of sperm are normal. In varicocele.
fact, this number correlates with the success of
egg fertilization in vitro (IVF) and thus is
ascribed clinical significance. In addition,
sperm morphology is an indicator of

870 EDUCATIONAL REVIEW MANUAL IN UROLOGY


4. Classification of Abnormalities

6. Antisperm antibodies. A link between infertil-


ity and antisperm antibodies has been recognized
General Information

for years. The effect of antibodies is to disturb 1.) A variety of classification schemes exist to cate-
either sperm transport through the female gorize fertility problems. One of the most useful
reproductive tract or sperm-egg interaction. is based on the findings on semen analyses with
Enzyme-linked immunosorbent assay (ELISA) initial classification of problems into 1 of 4 cate-
and immunobead binding assays are 2 commonly gories: 1.) all parameters normal; 2.) azoosper-
used tests to detect the presence of antibodies on mia; 3.) a single abnormal parameter and 4.)
sperm. multiple abnormal parameters. The decision to
treat a semen abnormality to improve fertility
7. White blood cells. Reproductive tract infections depends in part on maternal reproductive poten-
are a treatable cause of infertility and are often tial and also on how correctable the problem is,
heralded by the presence of excessive white as outlined in Figure 5.
blood cells in the ejaculate. Special stains are
needed to distinguish white cells from immature 2.) Distribution of semen abnormalities among
germ cells, the latter of which are not pathologic. infertile men:

Radiologic procedures Azoospermia 8%

1.) Transrectal Ultrasound (TRUS). TRUS is indi- Single abnormal parameter 37%
cated in infertile men with low-volume ejaculates
(<1.5 mL). Ejaculatory duct obstruction Multiple abnormal parameters 55%
(EDO) may be identified by seminal vesicle
dilatation (>1.5 cm in diameter); seminal vesicle Among the single abnormal parameters, isolated
hypoplasia or absence is also easily diagnosed. abnormalities in motility account for the majority
Frequently, the causes of EDO, including stones, of cases.
scar, cysts or a persistent utricle, can be diagnosed
by TRUS. In addition, seminal vesicle fluid can
be sampled to confirm the presence of sperm in
All Parameters Normal

patients with suspected obstruction and seminal 1.) Further evaluation of the female partner is rec-
vesiculography or chromotubation (antegrade ommended. This may include assessment of
injection of diluted indigo carmine monitored by menstrual history, cycle day 3 FSH and estra-
flexible cystoscopy) can be performed to demon- diol levels, an antral follicle count by ultra-
strate obstruction. sound and a hysterosalpingogram to assess
uterine anatomy.
2.) Vasography. Classically, intraoperative, trans
scrotal vasography is used to detect abdominal 2.) If the female partner is normal, sperm function
vas deferens, seminal vesicle and ejaculatory tests may be useful. Most couples with unex-
duct patency prior to definitive surgery for plained infertility will go on to be treated with
obstruction. It is usually performed at the same intrauterine insemination (IUI) or IVF.
setting as reconstruction, as there is a 5%–10%
chance of vasal scarring after the procedure. Azoospermia

3. Scrotal ultrasound. A scrotal ultrasound is indi- 1.) When no sperm are found on semen analysis,
cated when the testes are not easily palpable due the specimen should be centrifuged to confirm
to coexistent hydrocele, to confirm the origin and the absence of any sperm (pellet analysis). In
character (solid vs cystic) of intrascrotal masses, addition, a collection error and/or retrograde
and for confirming the presence of a clinically ejaculation must be ruled out as causes of
suspicious varicocele. There is level I evidence to azoospermia.
suggest that finding and treating subclinical
varicoceles does not improve fertility.
CHAPTER 27: EVALUATION AND TREATMENT OF MALE FACTOR INFERTILITY 871
Figure 5

Algorithm for treatment of male infertility

Algorithm for treatment of male infertility. Used with permission from P. Turek. Nat Clin Pract Urol. 2005;2:1-13.

Table 4

Techniques for Testis Biopsy

Technique Advantages Disadvantages Harvest

Fine needle aspiration Office procedure/local Minimal tissue – Yes/no sperm Y

Needle core biopsy Office/good tissue prep Painful/decrease control ?

Open biopsy “window” Office procedure/ Painful/some Y


adequate tissue testicular injury

872 EDUCATIONAL REVIEW MANUAL IN UROLOGY


2.) The results of gonadotropin levels determine important in the azoospermic patient with normal
what additional evaluation and treatment is nec- hormones and normal-sized testes with fructose in
essary. the ejaculate. The microscopic examination of the
biopsy will indicate whether spermatogenesis is
a. LH, FSH and testosterone levels differentiate progressing normally. With the advent of IVF and
primary testicular failure from secondary ICSI, sperm found on testicular biopsy can be cry-
testicular failure caused by either pituitary opreserved for future use (Table 4).
or hypothalamic dysfunction (Table 3).
d. Alternatively, nonsurgical fine needle aspi-
b. A serum FSH >3 times normal along with ration (FNA) “mapping” of the testis under
atrophic testicles on physical examination is a local anesthesia can provide diagnostic infor-
medical biopsy and indicates severe testicular mation on 1.) whether sperm are being pro-
failure. This does not mean that there is no duced in the testis, 2.) the location of sperm
sperm in the testis, as 50% of men have tes- and 3.) the relative quantity of sperm. This
ticular sperm on more extensive evalua- can inform men with testicular failure who
tion. However, this finding obviates the need often show patchy or focal areas of sperm
for a surgical biopsy to rule out obstructive production, about the chances of successful
conditions. Sperm found in these men can be subsequent sperm retrieval for IVF-ICSI.
used with IVF/ICSI by testicular sperm
extraction techniques (TESE). e. If the testicular biopsy or FNA mapping indi-
cates complete spermatogenesis, scrotal
3.) After ruling out a major endocrine abnormality, exploration and vasography are indicated. A
the major differential diagnosis is ductal vasogram is performed by injecting contrast
obstruction or testicular failure. material through 1 vas deferens to determine
patency. If patency is demonstrated, then
a. Negative fructose test. 3 possibilities exist in exploration of the epididymis is undertaken to
the azoospermic patient with normal hormone localize the site of obstruction. Fluid within
studies and a negative fructose test. These are the vas deferens lumen at vasotomy should
congenital bilateral absence of the vas defer- be examined microscopically to determine if
ens (CAVD), bilateral ejaculatory duct sperm are present or absent. A microscopic
obstruction and, rarely, seminal vesicle dys- vasoepididymostomy is necessary to correct
function similar to bladder myopathy. The epididymal obstruction. If no spermatozoa
treatment of CAVD is direct sperm aspiration are detected in the tubules of the epididymis
from the epididymis. The sperm are then pro- during this exploration, then intratesticular
cessed and used in combination with IVF and ductal obstruction may be the cause of
intracytoplasmic injection (ICSI) techniques. azoospermia. Since these cases are difficult to
Ejaculatory duct obstruction is managed by correct, IVF and ICSI with testicular sperm is
transurethral resection. recommended.

b. Positive fructose test. A positive fructose test 4.) Vasectomy is 1 cause of azoospermia amenable
rules out complete obstruction of the ejacula- to surgical repair with microsurgery.
tory ducts and severe dysfunction of the semi- It is well established that 1.) surgeon experience
nal vesicles but does not give an indication of and 2.) use of optical magnification, improve
the patency of the ductal system. Thus, a pos- outcomes after vasectomy reversal. In experi-
itive fructose test in azoospermia does not enced hands, a return of sperm to the ejacu-
differentiate between proximal ductal late should be possible in 90%-95% of cases
obstruction and testicular failure. of vasovasostomy, with pregnancy rates from
35%-60% depending on partner fertility
c. Testicular biopsy. A testicular biopsy is potential. In older vasectomies, it is often nec-
essary to perform a vasoepididymostomy

CHAPTER 27: EVALUATION AND TREATMENT OF MALE FACTOR INFERTILITY 873


for secondary epididymal obstruction that is fullness of the spermatic cord when the
acquired due to prolonged obstruction after patient moves from the standing to the supine
vasectomy. position. The majority (90%) of varicoceles
occur on the left side, but bilateral varicoceles
are more common than previously believed.
Repair of subclinical varicoceles (ie, those
Multiple Abnormal Parameters on the

detected by ultrasound but not by palpation)


Semen Analysis

1.) Diffuse abnormalities of all or many of the semi- is ineffective.


nal parameters are the most common pathologic
pattern identified (55%). Determination of the c. The exact mechanism through which the varic-
LH, FSH and testosterone levels can rule out ocele exerts a detrimental effect is unclear; the
endocrine causes. leading theory is that it causes increased
intrascrotal temperature through retrograde
2.) Stress, infections and other environmental fac- venous blood flow. There is a characteristic
tors, such as wet heat, drugs and toxin exposures decrease in normal sperm morphology, with an
may produce a transient abnormality of all semi- increase in immature and tapered sperm forms.
nal parameters. Therefore, when other specific There may also be decreased sperm motility
factors cannot be identified, it may be beneficial and varying degrees of oligospermia.
to follow these patients for 3 months to assess
for self-correction of the abnormality. If sponta- d. Treatment of varicocele involves surgical lig-
neous correction has not occurred, nonspecific ation or transvenous angiographic identifica-
therapy can be instituted as discussed below or, tion and embolization of the involved internal
more commonly, couples can be offered assisted spermatic veins. The usual surgical approach
reproductive technology (IUI, IVF). is retroperitoneal, inguinal or ligation of
the internal spermatic and collateral veins.
3. Varicoceles Laparoscopy may be useful in cases of bilat-
eral varicoceles. Newer interventional angio-
a. A varicocele is defined as dilated, varicose graphic techniques and catheters allow for
internal spermatic veins producing fullness, selective catheterization of the internal sper-
dilatation and poor drainage of the matic veins. For venous occlusion, a variety
pampiniform plexus. A varicocele is found of techniques are used, including sclerosing
in 15% of males in the general population but solutions, balloons and stainless steel coils.
is found in 35%-40% of men presenting
with primary infertility and results in abnor- e. Results of varicocele ligation from several
mal semen parameters. case series indicate approximately a 70%
improvement in semen quality associated
b. Varicoceles are classified according to size with a 40% pregnancy rate. With initial
as either large (grade III), medium (grade sperm counts >10 million sperm per milliliter,
II) or small (grade I). Large varicoceles can the improvement in pregnancy rates is signifi-
be seen as a bag of worms under the scrotal cantly better than if the initial counts are
skin. Medium varicoceles are readily lower. Both surgical and angiographic tech-
detected by palpation, especially on stand- niques for occlusion of varicoceles have a
ing. Small varicoceles can only be identified small failure rate with persistence and/or
by feeling an impulse in the scrotum with the recurrence of the varicocele noted during
Valsalva maneuver or a difference in the size follow up. Pregnancies, when they occur,
and are generally noted a mean of 7-8 months
after varicocelectomy.

874 EDUCATIONAL REVIEW MANUAL IN UROLOGY


b. Sperm motility and forward progression may
also be adversely affected by the presence of
Isolated Abnormal Parameter on Semen

antisperm antibodies that agglutinate or


Analysis

1.) Abnormal semen volume immobilize the spermatozoa. Special sperm


tests are available to determine the levels of
a. Large ejaculate volume. A volume >5.5 mL antisperm antibodies in semen. Several treat-
may dilute the spermatozoa and cause poor ments are available for antibodies, including
cervical placement of seminal fluid during IUI, IVF, and ICSI and treatment of the male
intercourse. Mechanical concentration of the with systemic steroids for 6–9 months. The
spermatozoa and intrauterine insemination latter treatment has been associated with
may be employed. decreased antibody levels but not necessarily
increased natural pregnancy rates.
b. Absent or low ejaculate volume. Once a col-
lection error has been ruled out, it is essential 4.) Oligospermia
to consider retrograde ejaculation, absence
or blockage of the accessory sex glands, or a. Decreased sperm numbers as an isolated
endocrine dysfunction (low testosterone) as problem may be secondary to endocrine dys-
causes of low ejaculate volume. Retrograde function, genetic or idiopathic conditions.
ejaculation is confirmed by the finding of Occasionally the absolute number of sperm is
large quantities of spermatozoa in the poste- relatively normal, but the number of sperm
jaculatory urine sample (>15 sperm/HPF). per milliliter may appear to be low due to a
Sympathomimetic drugs with alpha-adren- large ejaculate volume. With sperm densi-
ergic activity can reverse this problem in ties <5 X 106 /mL, a karyotype analysis and
one-third of patients, generally those without specific deletions in the Y chromosome
scar tissue at the bladder neck from prior should be evaluated.
surgery. In others, it may be necessary to
obtain, wash and inseminate (IUI) sperm col- b. In general, there are 2 treatments for
lected from the postejaculate urine sample. oligospermia. One attempts to stimulate the
Endocrine abnormalities and infections are testes with a variety of drugs to increase the
treated with the appropriate hormones and output of spermatozoa (see section on idio-
antibiotics. pathic infertility). Alternatively, with
advances in assisted reproduction, spermato-
2. Hyperviscosity. Problems with hyperviscous zoa can be concentrated for use with IUI. In
semen are rare and may reflect enzymatic imbal- addition, IVF and ICSI are available when
ance in the semen. Mechanical disruption of the simpler approaches fail.
sample in the laboratory to decrease viscosity,
followed by IUI may be useful. 5.) Abnormal morphology. An isolated problem
with morphology is unusual. This may result
3. Decreased motility and forward progression from gonadotoxic exposures (eg, tobacco), sys-
(asthenospermia) temic illness, fevers, varicocele or idiopathic
causes.
a. This is the most common isolated abnormality
found on semen analysis and can be due to
endocrine dysfunction, infection of accessory
Empirical Treatment of Idiopathic Infertility

glands, varicocele, epididymal dysfunction, 1.) Second only to the large proportion of patients with
genetics, reactive oxygen species (oxidants) or varicoceles, idiopathic infertility is the second
environmental exposures. Specific therapy is largest group of infertility patients. Essentially, idio-
available for some of these problems. Empirical pathic infertility refers to men who have an abnor-
treatment with antioxidants (vitamins A, C and mal parameter or parameters on the semen analyses
E, zinc and folate) may be tried. with a normal history, physical examination and

CHAPTER 27: EVALUATION AND TREATMENT OF MALE FACTOR INFERTILITY 875


screening hormone analysis. The etiology of the potential to raise LH and FSH levels, but its
abnormal semen quality is unclear and reflects our administration is complex and costly.
incomplete knowledge about the genetics of sper-
matogenesis. Many of these cases are likely to have b. Clomiphene citrate is an antiestrogen that
genetic etiologies. has been used nonspecifically for idiopathic
infertility for 40 years. As a selective estro-
2.) Nonpharmacologic treatments. A variety of gen receptor modulator (SERM), this med-
nonpharmacologic treatments have been sug- ication blocks the negative feedback of estra-
gested, but their efficacy has not been rigorously diol on the hypothalamic-pituitary axis,
demonstrated. These include: resulting in increased LH and FSH levels.
Overall improvement in the semen analysis is
a. Vitamins/diet. Specific vitamins and changes noted in approximately 50% of patients and
in dietary habits have not been reliably asso- pregnancies occur in 25%–30% of patients in
ciated with improved semen quality and fer- uncontrolled studies. Men with low to normal
tility. Antioxidant therapy has been shown to testosterone and FSH levels, indicative of
increase motility of in vitro isolated sperm, mild central hypogonadism, may respond
but treatment of men with unexplained severe best to this therapy.
asthenospermia with high-dose vitamin E and
C did not improve motility in a large, random- c. Tamoxifen is also an antiestrogen. This
ized study. However, this therapy is likely to agent, however, lacks the estrogenic activities
specifically help tobacco smokers to reduce of clomiphene and may provide equal effi-
the oxidative effects on sperm. A recent cacy.
Cochrane meta-analysis of 34 studies also
suggested that antioxidants improved preg- d. Aromatase inhibitors block the conversion
nancy rates and live delivery rates with of testosterone to estrogens in men. They may
assisted reproduction. be useful in oligospermic or azoospermic
men with a testosterone:estrogen ratio of
b. Jockey shorts to boxer shorts. A recent study <10:1 to increase sperm yield from the ejacu-
has refuted the idiom that a switch from late or from testis sperm extraction proce-
jockey to boxer shorts improves spermatoge- dures.
nesis and semen parameters.
e. L-Carnitine is found in high concentrations
c. Antibiotics for occult infection. in the normal epididymis and is postulated to
be important for sperm motility. Oral L-Car-
d. Varicocelectomy for the occult or subclinical nitine is currently available from several pop-
varicocele. There are 3 randomized, con- ular fertility supplements and there is early
trolled clinical trials suggesting no benefit to evidence that some men with low motility
varicocelectomy in cases of nonpalpable, will show improvement, especially those with
ultrasound-detected varicoceles. varicoceles.

3.) Drug therapy for idiopathic infertility Assisted reproductive technology (ART)

a. The use of human menopausal 1.) ART attempts to improve the conception by
gonadotropin (hMG) or recombinant FSH, bypassing many or all of the barriers associated
human chorionic gonadotropin (hCG, with normal fertilization. The simplest tech-
essentially LH), or the combination of FSH niques involve sperm processing and insemina-
and hCG does not result in significant tion of the female; the more sophisticated ones
improvements in sperm counts or fertility involve manipulation of the sperm and ova
with idiopathic infertility. GnRH therapy, extracorporeally. Fertilization with these proce-
when given in a pulsatile fashion, has the dures can occur in vitro or in vivo.

876 EDUCATIONAL REVIEW MANUAL IN UROLOGY


2.) Semen processing is used as an isolated proce- induce the maturation of multiple oocytes
dure or in conjunction with oocyte processing. which are then harvested ultrasonographically,
Sperm washing, swim-ups, sedimentations, just prior to ovulation. Processed semen and
and gradient centrifugations are commonly recovered oocytes are mixed in vitro. If fertiliza-
employed to remove seminal plasma and leuko- tion occurs, embryos are placed back into the
cytes and to select for and concentrate highly uterus transcervically. Usually only 20%–30%
motile sperm. Theoretically, with these proce- percent of replaced embryos survive and
dures, fewer sperm than normal are needed since become clinical pregnancies. Of those couples
they are placed higher within the female repro- who fail the first IVF attempt, 10%–18% percent
ductive tract than the ejaculate during inter- will fertilize on the second cycle. Depending on
course. the cause of infertility, and the number of
embryos replaced, pregnancy rates can approach
3.) Intrauterine insemination (IUI) is used to treat 40%–50% per IVF attempt.
male factor infertility. A small catheter is used to
inject processed sperm through the cervix and 5.) Micromanipulation. Sperm and oocyte micro-
into the uterine cavity. By bypassing the cervical manipulation by intracytoplasmic sperm
mucus, more motile sperm can progress to the injection (ICSI) has become the mainstay of
fallopian tubes where normal fertilization addressing the poor fertilizing capacity of
occurs. Indications for IUI include male factor sperm often seen in male infertility. By inject-
infertility due to mechanical or anatomical prob- ing a single sperm directly into an oocyte, multi-
lems (eg, erectile dysfunction, hypospadias), ple barriers of fertilization are bypassed and suc-
low semen quality and/or cervical mucus prob- cess rates (when compared to standard IVF) are
lems. Usually, more than 5 million motile greatly improved. Pregnancy rates of 40%–50%
sperm are required for IUI. Pregnancy rates have been obtained using this technique even in
are variable with these techniques, but aver- the most severe cases of male infertility.
age 30%–35% in couples who try 3–4 times.
a. ICSI allows only ejaculated sperm but sperm
4.) IVF, first introduced in 1978, was developed to retrieved from the male reproductive tract to
manage fallopian tube obstruction. It is now produce pregnancies. Vasal, epididymal and
used routinely for male factor infertility. Female even testicular sperm, from both
partners undergo ovarian hyperstimulation to obstructed and nonobstructed men, are
now routinely used with IVF and ICSI.
Table 5

Techniques for Sperm Harvest in Assisted Reproduction

Technique Advantages Disadvantages

Percutaneous epididymal Office/minimally invasive Damage to epididymis


sperm aspiration (PESA) large numbers of sperm

Testicular sperm extraction Office/easily repeatable Lower numbers of sperm


(TESE)

Microsurgical epididymal Large numbers/controlled Anesthetic required/invasive


Sperm aspiration (MESA)

Testicular microdissection Search for active spermatogenesis Anesthetic/lengthy/invasive


In Klinefelter’s/NOA

CHAPTER 27: EVALUATION AND TREATMENT OF MALE FACTOR INFERTILITY 877


b. Consequently, sperm retrieval techniques concerns regarding possible developmental
(Table 5) have been developed to harvest issues. There is a significant increase in sex
sperm from male reproductive tract organs. In chromosomal abnormalities among ICSI off-
cases of ejaculatory failure, vasal sperm spring (0.8%-0.2% naturally) that is likely
aspiration (VASA) is possible. For men with attributable to the chromosomal status of the
epididymal obstruction including those with fathers rather than to the technique itself. As
vasectomy, epididymal sperm aspiration by previously mentioned, 5%–8% of men with
microscope (MESA) or percutaneously severe oligospermia have deletions in the AZF
(PESA) is possible. For obstructed men, region of the Y chromosome. It has been shown
testis sperm needle aspiration (TESA) or that these genetic deletions are passed to the
biopsy (TESE) is also indicated. For men male offspring produced by these fathers
with nonobstructive azoospermia or testis through IVF/ICSI.
failure, TESE or microdissection TESE are
generally needed. There is a measurable risk c. More recently, concerns have arisen regard-
of testis atrophy and hormone failure after ing whether rare imprinting diseases such
microdissection TESE. as Beckwith-Wiedemann Syndrome and
Angleman Syndrome are increased along
6.) IVF with ICSI should not be performed without with autism in children conceived with ICSI.
karyotype analysis on both partners. Despite For these reasons, genetic counseling is rec-
its great success, there remain concerns with ommended for all couples considering
ICSI. IVF/ICSI.

a. Multiple gestations occur in 35% of 7.) Preimplantation genetic diagnosis (PGD) is


IVF/ICSI pregnancies compared to now possible by sampling a single cell from an
1%–2% of naturally conceived pregnan- 8-cell developing embryo in vitro and perform-
cies. Multiple gestations are associated with ing either fluorescent in situ hybridization
lower birth weights, higher overall complica- (FISH) or single-cell polymerase chain reaction
tion rates and increased learning disabilities (PCR) to examine for aneuploidy, chromoso-
relative to singletons. In addition, the deliv- mal translocations or even specific diseases
ery costs associated with managing multiple, caused by point mutations. Current indications
often premature, gestations are at least 10-20 for PGD include lethal disease prevention and
times higher than that associated with the aneuploidy screening in advanced maternal age
singleton deliveries. There is also strong evi- couples.
dence to suggest that the incidence of low
birth weight singletons is also increased after
ICSI. Among respectable ART programs,
there is a trend to transfer as few embryos
back as possible to reduce the multiple gesta-
tion rate.

b. ICSI bypasses many mechanisms of natu-


ral selection, such that many men who
would not be able to conceive under nor-
mal circumstances are now able to father
children. Although overall rates of major
birth defects (3.3%) associated with IVF
and ICSI may be similar to intercourse, there
is concern about elevated risks of specific
malformations, including hypospadias, with
this technology. In addition, there are

878 EDUCATIONAL REVIEW MANUAL IN UROLOGY


5. References 6. Questions

Goldstein M, ed. Surgery of Male Infertility. 1. Embryologically, the vas deferens and body of
Philadelphia, PA: WB Saunders Co.;1995. the epididymis are derived from what develop-
mental structure?
Jarow JP, Sharlip ID, Belker AM, et al for the Male
Infertility Best Practice Policy Committee of the A. Mullerian ducts
American Urological Association Inc. J Urol. 2002;
167:2138-2144. B. Wolffian ducts

Sigman M, Howards SS. Male infertility. In: Walsh C. Urogenital ridge


PC. Retik A. Vaughan ED Jr, Wein A, eds. Camp-
bell’s Urology. 7th ed. Philadelphia, PA:Saunders; D. Gubernaculum testis
1998:1287-1330.
E. Metenephros
Sigman M, Jarow JP. Endocrine evaluation of infer-
tile men. Urology. 1997; 50(5):659-664.
2. The vast majority of the fluid in the male ejac-
Turek PJ. Male Infertility. In: Tanagho EA, McAn- ulate is derived from the:
inch JC, eds. Smith’s Urology. 15th ed. Stam-
ford,CT: Appleton and Lange; 2003. A. Epididymides

Turek PJ. Physiology of Male Reproduction. In: B. Ejaculatory ducts


Wein AJ, Kavoussi L, Partin A, Peters P, eds. Camp-
bell’s Urology. 10th ed. Philadelphia, PA: Saunders; C. Seminal vesicles
2011: 591-617.
D. Testicles
Turek PJ, Reijo Pera RA. Current and Future
Genetic Screening for Male Infertility. Urol Clin E. Vas deferens
North Am. 2002;29:767-792.

Turek PJ. Practical approach to the diagnosis and 3. The most common and correctable identifiable
management of male infertility. Nature Clin Pract problem causing male infertility is:
Urol. 2005;2:1-13.
A. Infection

B. Obstruction

C. Gonadotoxin exposure

D. Varicocele

E. Genetic

CHAPTER 27: EVALUATION AND TREATMENT OF MALE FACTOR INFERTILITY 879


4. A 25-year-old bodybuilder eschews the merits D. CFTR (delta f 508)
of natural bodybuilding and cycles and stacks
injectable anabolic steroids regularly to maxi- E. 45XO
mize muscle bulk. His fertility potential would
be expected to be:
7. Low serum testosterone, LH and FSH charac-
A. Normal, because exogenous testosterone terize what type of hypogonadism?
does not impair production of endogenous
testosterone A. Hypergonadotropic hypogonadism

B. Low, because exogenous testosterone stim- B. Young syndrome


ulates pituitary production of FSH and LH
C. Hypoprolactinism
C. Low, because exogenous testosterone
inhibits pituitary production of FSH and D. Kallman syndrome
LH
E. Klinefelter syndrome
D. Low, because exogenous testosterone is not
as potent as endogenous testosterone at
nurturing spermatogenesis 8. How do elevated levels of prolactin influence
testosterone production?
E. Normal, because intratesticular testos-
terone concentrations are 50x higher than A. Inhibit GnRH and LH
serum levels, whether or not the blood con-
tains exogenous testosterone B. Indirectly inhibit Sertoli cells

C. Directly inhibit Leydig cells


5. The role of PSA in the ejaculate is:
D. Upregulate inhibin
A. To coagulate the ejaculate
E. Downregulate activin
B. To serve as a marker for prostate cancer

C. To serve as a liquefaction factor 9. What testicular hormone is the major feedback


inhibitor of LH secretion?
D. To give semen its characteristic odor
A. Testosterone
E. To agglutinate the ejaculate
B. Inhibin

6. What genetic mutation is involved with con- C. Activin


genital absence of the vas deferens (CAVD)?
D. Prolactin
A. Klinefelter syndrome (47,XXY)
E. Sertolin
B. Reifenstein syndrome

C. 3p1

880 EDUCATIONAL REVIEW MANUAL IN UROLOGY


10. What is the initial evaluation for a man with 1 13. Hormonal screening of infertility patients
semen analysis that shows low-volume ejacu- below what sperm concentration will pick up
late? most endocrinopathies?

A. Semen pH A. <1 million/mL

B. Postejaculate urinalysis B. <5 million/mL

C. Semen fructose C. <10 million/mL

D. TRUS D. <20 million/mL

E. Repeat semen analysis E. <60 million/mL

14. Vasography is routinely performed by all of


11. What is the normal size for adult human testes? the following methods EXCEPT which one?

A. 5 mL A. Scrotal

B. 10 mL B. Transperineal

C. 20 mL C. Transrectal

D. 30 mL D. Transurethral

E. 40 mL E. Transabdominal

12. In the absence of genital tract infection, round 15. Why is performing varicocelectomy at the
cells in the semen analysis are most likely what same time as vasectomy reversal discouraged?
kind of cell?
A. Venous congestion
A. Squamous epithelial cells
B. Arterial compromise
B. Immature germ cells
C. Increased risk of sperm granuloma
C. Prostatic epithelial cells
D. All of the above
D. Leydig cells
E. None of the above
E. Sertoli cells

CHAPTER 27: EVALUATION AND TREATMENT OF MALE FACTOR INFERTILITY 881


16. A man has the following semen analysis pro- D. Epididymal sperm
file after vasectomy reversal. This pattern is
typical of what problem? E. Testis sperm

Timepoint Volume Concentration


Motility after surgery 19. What patency rates are achievable from con-
temporary series of microscopic vasovasos-
6 weeks 3.0 45 million/mL 45% tomies?
3 months 3.5 50 million/mL 15%
6 months 2.5 20 million/mL 0% A. 90%–99%

A. Antisperm antibodies B. 80%–90%

B. Testicular injury C. 50%–60%

C. Stricture formation at anastomosis D. 40%–50%

D. Primary hypogonadism E. 25%–35%

E. Ejaculatory duct obstruction


20. Which fluid characteristic(s) from the testis
vas deferens predict the best success for vasec-
17. What percentage of oligospermic infertile tomy reversal?
males will have genetic microdeletions of the
Y chromosome? A. Clear with no sperm

A. 1% B. Creamy with sperm fragments

B. 5% C. Creamy with no sperm

C. 25% D. Cloudy with motile sperm

D. 50% E. Cloudy with fragmented sperm

E. 75%

18. Which of the following has the highest impact


Answers

on pregnancy rates with in vitro fertilization 1. B.


(IVF)? Mullerian ducts regress in the male. The indifferent
gonad migrates to the urogenital ridge to become
A. Oligospermia—male factor infertility the testicle. The gubernaculums testis is responsible
for pulling the testis into the scrotum during devel-
B. Tubal obstruction—female factor opment.
infertility
2. C.
C. Female age At least 65%–70% of ejaculate volume is derived
from the seminal vesicles, with the remainder from
the vas deferens (with sperm) and prostatic secre-

882 EDUCATIONAL REVIEW MANUAL IN UROLOGY


tions. Periurethral glands may also contribute a 14. E.
small amount of fluid to the normal ejaculate. Vasography is not routinely done transabdominally

3. D. 15. A.
Infection and obstruction occur in 5%–10% of male With varicocele repair, all venous drainage from the
infertility. The prevalence of gonadotoxin exposure testis is ligated except for the vasal veins. With
is not well known. Varicocele occurs in 40% of vasectomy reversal, injury to the vasal veins may
infertile men. occur.

4. C. 16. C.
Because of negative feedback inhibition that main- Anastomotic strictures after vasectomy reversal
tains homeostatic balance in the pituitary-gonadal typically cause a decrease in sperm motility fol-
axis, excess testosterone of any type will cause ante- lowed by a decrease in sperm concentration over
rior pituitary production of LH and FSH to fall. This time.
results in azoospermia in most of men on anabolic
steroids, but the effect will vary based on the dose, 17. B.
frequency and duration of the cycles and stacking 5%. The range is 3%–8%.
regimen
18. C.
5. E.
PSA is a serine protease that enzymatically breaks 19. A.
down the seminal coagulum after ejaculation.
20. D.
6. D.
CFTR mutations, most commonly delta 508, are
involved with CAVD

7. D.
Kallmann syndrome, a form of hypogonadotropic
hypogonadism

8. A.
Hyperprolactinemia causes hypogonadotropic
hypogonadism

9. A.
Testosterone.

10. E.
Low-volume ejaculates are commonly due to col-
lection error.

11. C.

12. B.

13. C.
99% of endocrinopathies will be detected if screen-
ing is done in men with <10 million sperm/mL

CHAPTER 27: EVALUATION AND TREATMENT OF MALE FACTOR INFERTILITY 883


884 EDUCATIONAL REVIEW MANUAL IN UROLOGY
Chapter 28:
Erectile Dysfunction,
Peyronie’s Disease
and Priapism
Mikkel Fode, MD
Susanne A. Quallich, BSN, CUNP
Jens Sønksen, MD
Dana A. Ohl, MD

Contents

I. Erectile Dysfunction

1. Normal Sexual Function

2. Erectile Dysfunction (ED):


General Considerations

3. Evaluation of a Man Presenting With ED

4. ED Treatments

II. Peyronie’s Disease

1. Epidemiology and Etiology of PD

2. Clinical Course of PD

3. Office Evaluation and Medical Management

4. Surgical Therapy

III. Priapism

1. Ischemic vs Nonischemic Priapism

2. Approach to Priapism

3. Questions

4. Further Reading

CHAPTER 28: ERECTILE DYSFUNCTION, PEYRONIE’S DISEASE AND PRIAPISM 885


886 EDUCATIONAL REVIEW MANUAL IN UROLOGY
I. Erectile Dysfunction

1. Normal Sexual Function

2. Erectile Dysfunction (ED):


General Considerations

3. Evaluation of a Man Presenting With ED

4. ED Treatments

CHAPTER 28: ERECTILE DYSFUNCTION, PEYRONIE’S DISEASE AND PRIAPISM 887


1. Normal Sexual Function

thetic and arise from T10-L3. They travel out of the


sympathetic chain, through the superior hypogastric
Neuroanatomic considerations

Sexual activity is initiated in the central nervous plexus and then through the pelvis to the ejaculatory
system. The brain is the most poorly understood organs, including the vas deferens, seminal vesicles,
component of the sexual response, but is of great prostate and bladder neck. The locations of the
importance. Some important areas that are known to
contribute to sexual response include the medial
sympathetic chain just lateral to the aorta and of

preoptic nucleus and paraventricular nucleus, which


the hypogastric plexus anterior to the aorta make

reside near the hypothalamus. The exact function


these structures prone to surgical injury from aor-

and interaction of these structures are not known. for testis cancer. Indeed, prior to nerve-sparing pro-
tic surgery or retroperitoneal lymphadenectomy

The effects of neurochemicals in the brain are gen- cedures for testis cancer, a radical lymph node dis-
erally understood as follows: serotonin and nore- section in this area nearly always caused absence of
pinephrine inhibit libido, erectile response and abil- seminal emission or (less commonly) retrograde
ity to climax; dopamine, oxytocin, melanocortins ejaculation. Aortic surgery still carries with it a sig-
and nitric oxide (NO), conversely, generally nificant risk of ejaculatory dysfunction.
improve all of these functions. The effects of antide-
pressants that may increase CNS serotonin or nore-
pinephrine levels thus commonly cause sexual dys-
Penile anatomy and vascular supply

function. The most common sexual adverse effect


(Figure 1)

The penile deep structures consist of 3 erectile bod-


ies, 2 corpora cavernosa and a single corpus spon-
of the frequently prescribed selective serotonin

although decreased libido and isolated erectile dys- giosum. Surrounding the entire package of erectile
reuptake inhibitors (SSRIs) is anorgasmia,

function may also occur. bodies is Buck’s fascia. All 3 bodies contain erectile
tissue. Practically speaking, in normal anatomic cir-
Descending neural pathways controlling erection cumstances, there is free flow of blood between the
exit the brain and travel in the spinal cord to the corpora cavernosa and they function together as a
level of S2-4, where they exit as peripheral unit. That is why injection of a drug into only 1 side
parasympathetic nerves. These penile erection can effect erection on both sides. The 2 units may
effector nerves are perilously close to the prostate not function together in cases of anatomic problems,
and rectum as they travel through the pelvis, making such as repaired diphallus from exstro-
them prone to injury from surgical extirpative pro- phy/epispadias syndrome. The corpora cavernosa
cedures. The final cavernous nerves enter the cor- are responsible for the primary structural integrity
pora cavernosa just below the prostate at the diver- of the erection, due to their ability to become rigid
gence of the crura. Activation of these parasympa- when filled with blood. The rigidity is due to limita-
tion of the swelling that can occur due to the tough
nature of the containing tissue layer, the tunica
thetic nerves causes vascular changes that lead to

albuginea.
erection.

Conversely, the detumescence mechanism is


The corpus spongiosum surrounds the urethra and
The nerves responsible for this function, as well as distally enlarges to become the glans penis.
under control of the sympathetic nervous system.

for the basic sympathetic tone resulting in baseline Although the corpus spongiosum becomes
flaccidity, arise from T10-L3. After leaving the engorged during sexual stimulation, it does not con-
spinal cord and sympathetic chain, these nerves typ- tain a tough fascial covering and thus cannot
ically travel out of the superior hypogastric plexus become rigid. The lack of a tough fascial covering
in close proximity to the cavernous nerves and enter prevents blood trapping due to lack of veno-occlu-
the penile tissue in the same location. sive function (see below). The inability to trap
blood in the tissue can be exploited in cases of pri-
Interestingly, ejaculation is a completely different apism by intentionally shunting blood from the cor-
neurological event from penile erection. The pus cavernosum to the spongiosum so that it may
nerves controlling seminal emission are also sympa- escape into the systemic circulation.

888 EDUCATIONAL REVIEW MANUAL IN UROLOGY


Microscopically, it is difficult to differentiate the tis- the penis) and the deep penile artery (syn., caver-
sue within the different erectile bodies. In this tis- nosal artery, profunda artery), supplying the corpus
sue, blood flows thorough lacunar spaces, which are cavernosum. The deep arteries enter each corpus
endothelial-lined and surrounded by the trabecular cavernosum and course down the center, giving off
smooth muscle. It is this smooth muscle that is acti- branches into the erectile tissue. Blood then flows
vated or relaxed by the final neural pathways lead- radially toward the tunica albuginea toward the
ing to erection and detumescence. venules that give egress.

The arterial blood supply to the penis is from the These arterial systems are present on both sides and
common iliac → internal iliac (hypogastric) → are usually both functioning. Normal blood flow on
internal pudendal → common penile artery. After one side is usually enough to keep things function-
giving off branches to the scrotum and urethral bulb, ing properly. However, there can be individual
the common penile artery bifurcates into the dorsal variation in the anatomy. In some men, only one
penile artery (which supplies the glans and skin of side provides the majority of blood, and in other, the

Figure 1

Penile anatomy and vascular supply

Internal pudendal a. Periprostatic plexus Internal pudendal v.


Deep dorsal v.
Circumflex v.
Cavernous v.

Cavernous a.
Bulbar v.
Bulbourethral a. Bulbourethral v.
Circumflex a.
Dorsal a. Subtunical venous plexus
Retrocoronal venous plexus

Internal iliac v.

External
iliac v. A. Penile arterial supply
Internal pudendal v.
B. Venous drainage

Periprostatic plexus
C. Cross section of penis showing
Saphenous v. Crural v. corpora cavernosa, corpus spongiosum
Superficial dorsal v. Cavernous v.
and their relationship to the vascular
Deep dorsal v.
supply
Circumflex v.
Subtunical plexus

Emissary v.

Bulbourethral v.

From Lue TF. Physiology of penile erection and pathophysiology of erectile dysfunction and priapism.
In: Walsh PC, ed. Campbell’s Urology. 8th ed. Philadelphia, PA: Saunders; 2002.

CHAPTER 28: ERECTILE DYSFUNCTION, PEYRONIE’S DISEASE AND PRIAPISM 889


dorsal arteries may provide intracavernosal flow via This increase in intracavernosal pressure is also
perforating vessels more distally than normal. This responsible for the veno-occlusive function of the
anatomic variation may have importance in plan- penis. In the flaccid state, the low blood flow into
ning Peyronie’s disease surgery, in which the neu- the corpus caversosum courses radially from the
rovascular bundle is elevated. center, eventually exiting by small venules that per-
forate the tunica albuginia. During the low-flow
The venous drainage of the penis is redundant and flaccid state, this egress proceeds in an unimpeded
plentiful. The largest vein is the deep dorsal, origi- fashion. When the intracavernosal pressure
nating at the glans and running deep to Buck’s fas- increases to more blood coming in at the limit of the
cia. Circumflex veins coming up from the side and a tunica capacity, these venules are compressed
few direct cavernosal perforators join the deep dor- against the underside for the tunica, resulting in
sal vein in its proximal course. After leaving the blood trapping.
penis, the deep dorsal vein perforates the urogenital
diaphragm and courses toward the apex of the The smooth muscle relaxation is controlled by well-
prostate, where it can be problematic during radical known neurochemical cascades. The major control
prostatectomy. At the cavernosal crura, large caver- system is the nitric oxide/cyclic GMP (NO/cGMP)
nosal veins also drain the corpora cavernosa. It is a system. NO is released by nitrergic end-neurons and
common misconception that veno-occlusive dys- endothelial cells in the cavernosal tissue in response
function is caused by vascular anomalies. Rather, to sexual stimulation. NO from the neurons initiates
this is a physiological problem in the tissue, rather the erection while endothelial NO allows it to reach
than anatomic. a maximum and maintains it. NO is produced by the
cleavage of l-arginine by nitric oxide synthase
(NOS). In the neurons, neuronal NOS (nNOS) is the
active enzyme and endothelial NOS (eNOS) is the
Neurovascular changes of erection

In the flaccid state, there is very little blood flow active form in the vascular lining. Production of NO
into the penis. The predominant basic neural tone is causes the conversion of GTP into cGMP by the
sympathetic. The smooth muscle surrounding the action of guanylate cyclase. Interaction of cGMP
lacunar spaces are held in a state of contraction from with protein kinase G causes calcium shifts and
this sympathetic tone, and therefore, as each unit of resultant smooth muscle relaxation.
space holds less blood, the penis as a whole contains
less blood volume and is smaller. The baseline sym- Cyclic nucleotides of all types are broken down by
pathetic tone also keeps the blood supply in a rela- phosphodiesterase (PDEs) in the body. The most
tively vasoconstricted state. important PDE in the penile tissue is PDE5, which
is present in high concentration, is specific for
The actual process of erection involves filling and cGMP and plays a central role in inactivation of the
trapping of blood in the penile tissue. Blood flow NO/ cGMP pathway. As soon as cGMP is produced,
increases 20–40-fold through the cavernosal arter- PDE5 begins breaking it down. There is a catalytic
ies. Smooth muscle relaxation of the cavernosal domain of the PDE5 molecule that breaks the com-
smooth muscle surrounding the lacunar spaces also pound down, and a regulatory domain that makes
occurs, creating a permissive increase in blood vol- the enzyme even more efficient in breaking down
ume stored in each space. Therefore, penile blood cGMP, when levels of substrate increase. Therefore,
volume increases and the penis grows. The increase as soon as the chemical reaction takes place to pro-
in penile size is checked by the limitation of stretch duce cGMP, PDE5 begins turning it off. Preventing
of the tunica albuginia surrounding the corpus cav- the breakdown of cGMP by PDE5 inhibitors form
ernosum. As this limit is reached, an increase in the basis for the drugs sildenafil, vardenafil and
blood volume pushed into the penis results in an tadalafil.
increase in pressure, since size has reached its maxi-
mum. PDE5 is clearly the predominant PDE in the penis,
and its relative importance in the penis compared to
the rest of the body is substantial, allowing an oral

890 EDUCATIONAL REVIEW MANUAL IN UROLOGY


agent to be given to get an effect in the penis without reaching climax. Following high-pressure contrac-
exaggerated systemic adverse events. However, one tion of the pelvic/periurethral muscles, ascending
must remember that PDE is present throughout the pathways meet those descending from the brain at
body. It is present in systemic and pulmonary vascu- the thoracolumbar junction at levels T10-L3. From
lar beds and the gastroesophageal sphincter, among there, sympathetic nerves exit the cord and course
other areas. PDE5 inhibitors may cause adverse via the route above, culminating in the ejaculatory
effects from such systemic inhibition, as well as organs. The vas deferens propels sperm distally, and
cross-reactivity with other PDE families. the prostate and seminal vesicles contract, expelling
the semen into the posterior urethra. At the same
Although less important than the cGMP system, a time, sympathetic nerves also cause extremely tight
parallel cAMP system is present in the penile tissue contraction of the bladder neck, preventing retro-
as well. cAMP is degraded by several cAMP spe- grade ejaculation. After a short delay, involuntary
cific PDEs (not PDE5, which is cGMP-specific). rhythmic skeletal muscle contraction of the bulbo-
When present, cAMP also causes smooth muscle cavernosus and bulbospongiosus result in the pro-
relaxation via protein kinase A and calcium flux in jectile ejaculation phase.
much the same way as cGMP. Although not neces-
sarily so important in normal physiological func- Since ejaculation is a separate neurological event
tioning, this system is exploited during intraurethral from penile erection, climax and ejaculation can
suppository or intracavernosal injection administra- occur in the absence of an erection.
tion of prostaglandin E1, as this drug initiates the
production of cAMP.

Both cyclic nucleotide systems can be manipulated


by non-specific PDE inhibitors, such as papaverine.
Injection of this drug into the penis causes pro-
longed smooth muscle relaxation by preventing the
degradation of both cGMP and cAMP. However,
oral administration cannot achieve a targeted
response in the penis without insurmountable sys-
temic adverse effects of PDE inhibition.

Following sexual activity, activation of the sympa-


thetic supply to the penis results in arterial vasocon-
striction, contraction of lacunar smooth muscle,
defeat of the veno-occlusive mechanism as intrape-
nile pressure decreases, and reversal of the changes
mentioned above. This results in return to the base-
line flaccid state.

Physiology of ejaculation

After a period of sexual stimulation, coordination of


psychic and physical stimulation results in climax
and ejaculation. The brain component of this stimu-
lation is poorly understood. Sensory input into the
reflex is mostly from the glans and is carried via the
dorsal nerves of the penis, which enter the cord at
levels S2-4. Sacral input into the reflex causes con-
traction of the periurethral muscles and propriocep-
tion from these muscles may give further to promote

CHAPTER 28: ERECTILE DYSFUNCTION, PEYRONIE’S DISEASE AND PRIAPISM 891


2. Erectile Dysfunction (ED):
General Considerations

The Selvin study also redemonstrated the link


between ED and cardiovascular disease. Sexual
Epidemiology of ED

Erectile dysfunction has been defined by several dysfunction was positively correlated with age,
consensus conferences, with the most oft-quoted hypertension and presence of preexisting cardiovas-
being the NIH Consensus Development Panel on cular disease. Diabetics have a particularly high rate
Impotence, convened in 1993. This panel suggested of problems with a crude incidence rate of >50%.
ED should be defined as: “Inability of the male to The recent study has corroborated a great deal of
prior information suggesting this inextricable link.
mit satisfactory sexual intercourse.” This and the
attain and maintain erection of the penis to per-

other panel definitions have some commonalities. Previous reports have linked ED to hypertension,
First, the term “erectile dysfunction” should be used dyslipidemia, smoking, sedentary lifestyle, dia-
instead of “impotence,” as the latter can be con- betes, obesity, depression and preexisting cardio-
strued as derogatory and is not as descriptive of the vascular disease. Surprisingly, smoking status, dia-
actual condition. The dysfunction should be recur- betes, hypertension and coronary heart disease were
rent or persistent. The ability either to obtain and/or not significantly associated with the rate of decline
maintain an erection may be affected. Finally, the in sexual function in the Olmsted County data, but
definition weighs heavily on patient satisfaction. this may represent an early occurrence of ED in pre-
It is important to remember that sexual function disposed individuals.
and expectation is not homogeneous across a patient
population, so the definition of dysfunction Men with ED are also at risk for developing mani-
must add in an element of patient expectation/ festations of cardiovascular disease when followed
satisfaction. over time, suggesting that ED represents the first
manifestation of the generalized condition. Thomp-
The Massachusetts Male Aging Study of men ages son showed this to be true in the Prostate Cancer
40–70, suggested that nearly 50% of men suffer Prevention Trial, where men presenting with no car-
some degree of erectile dysfunction. This study has diovascular disease at study entry were examined
been criticized of overstating the numbers due to over the follow-up period. Those presenting with
inclusion of men with “mild” ED. Data from the ED at study entry had a near-doubling the rate of
Olmsted County Study, following 1,827 men aged cardiovascular events. A similar association has
40–79 for 14 years, was reported in 2009 and the been found in patients with diabetes and ED and the
incidence rate of ED was found to be similar to the Massachusetts Male Aging Study showed that ED
rate found in the Massachusetts Male Aging Study. was predictive of the metabolic syndrome in men
The incidence of ED increased from 6/1,000 person- with a BMI <25. The pathophysiology is thought to
years for men in their 40s to 118/1,000 person-years be atherosclerosis, endothelial dysfunction and pos-
for men in their 70s. A recent European study by sibly reduced testosterone levels in men with
Corona et al estimates the prevalence of moderate to metabolic syndrome. A small amount of plaque in
severe ED to be 30% in males aged 40–79 with age, the 1-mm penile arteries can lead to functional
cardiovascular disease, diabetes, LUTS, obesity and impairment that may predate any functional prob-
depression as predisposing factors. However, this lem in larger arteries, such as coronaries and
study finds that only 38% of the men reporting ED carotids. Similarly, the small diameter of the penile
are concerned about it. arteries and the dependence of the erectile mecha-
nism on endothelial NO may make the penile tissue
Other studies have examined broader age ranges and more sensitive to endothelial dysfunction than other
have come up with a lower incidence. A paper pub- organs. As diabetes further develops, autonomic
lished by Selvin et al in 2007 suggested that 18.4% nerve dysfunction can contribute to the erectile dys-
of men over age 20 had ED. This was from data function.
obtained by the National Health and Nutrition
Examination Survey. The link between ED and cardiovascular disease, as
well as diabetes and the metabolic syndrome, is
confirmed in most studies on the topic, although a

892 EDUCATIONAL REVIEW MANUAL IN UROLOGY


new European study found that while men with ED compound the problem. It is also important to note
were at increased risk for cardiovascular events, ED that testosterone deficiency can be part of the
was not an age-independent predictor of this during metabolic syndrome.
a 6.5-year follow-up period.
Anatomic: Peyronie’s Disease can lead to sexual
The association between ED and CVD means that a difficulty due to deformity, decreased flexibility of
cardiovascular assessment should be done in men the penile tissue or from the development of veno-
with ED, either by referral to a specialist or general occlusive dysfunction (see more in the section on
doctor, or by the urologist Peyronie’s Disease).

Drug-induced: Many drugs can lead to erectile dys-


function, and the list is too long to print here. The
General types of ED

As one can surmise from the functional require- most commonly implicated drugs are beta-blockers
ments for sexual function, there are several general (not nebivolol) and thiazide diuretics (which may
ED variants. cause ED simply by doing what they are intended to
do—lower blood pressure) and antidepressants, due
Arteriogenic: If one cannot increase blood flow to to the CNS effects. Other antihypertensives includ-
the 20–40-fold that is necessary to fill the penis, ing ACE inhibitors, angiotensin-receptor blockers
inadequate blood supply will be the cause of ED. and calcium-channel blockers are reported either to
This can be due to atherosclerotic disease, diabetic have no effect or to actually improve erectile func-
small vessel disease, endothelial dysfunction or tion.
trauma, which may either be a major pelvic injury or
chronic compression, such as that seen in bicycle Psychogenic: Although with our current under-
riders. Arteriogenic contribution is the most com- standing of ED we believe that 90% of cases have a
mon type of ED. predominantly organic cause, there remains a large
subset of patients with a psychogenic source. Dif-
Veno-occlusive dysfunction: Blood trapping is an ferentiation of these cases is usually by history, but
essential part of the erectile process. Lack of smooth other testing may be necessary. Since these patients
muscle relaxation in the cavernosal bodies, due to should be treated with therapy, rather than organic
tissue degeneration from ischemia (as can be seen treatments with potential adverse effects, it is
after radical prostatectomy), trauma, inadequate important to identify them.
neurotransmitter release (diabetic microneuropa-
thy), or Peyronie’s Disease can cause ED, even in
the face of normal arterial inflow.

Neurogenic: Conditions such as diabetic or alco-


holic neuropathy, multiple sclerosis, spinal cord
injury, or radical prostatectomy can lead to a situa-
tion where the signal to cause vascular changes
never gets to the penile tissues.

Hormonal: Testosterone deficiency has been shown


in multiple animal models to result in decreased
production of nitric oxide synthase, NO and cGMP
production, and worsened erectile function. This
has been corroborated in human case series. It is
clear now that in addition to effects on libido, testos-
terone deficiency may result in an unfavorable
change in penile tissue function. While hypogo-
nadism may not be the primary cause of ED, it may

CHAPTER 28: ERECTILE DYSFUNCTION, PEYRONIE’S DISEASE AND PRIAPISM 893


3. Evaluation of a Man Presenting
With ED

The most typical model for evaluation of a man with The history is essential in determining whether the
ED involves medical evaluations, history, physical ED appears to be predominantly organic or psy-
examination, basic lab tests, specialized testing and, chogenic.
finally, treatment. How the ED evaluation differs,
however, is that treatment is many times introduced
prior to having a specific diagnosis. For instance, it
Components of the ED history that would support

may not be necessary to know if a patient has arte-


the diagnosis of organic ED include:

rial inflow disease, if an arterial bypass procedure • Older age


may not be indicated in such a man. An oral agent
for treatment may be given to that man to achieve • Presence of organic risk factors, such as dia-
his goals of treatment, with no specialized testing betes, hypertension or obesity
done at all.
• Gradual onset and progressive worsening

tion treatment is a widely accepted concept. There • Pelvic or retroperitoneal surgery, such as radi-
The goal-oriented approach to erectile dysfunc-

have been several consensus panels convened that cal prostatectomy


have verified this idea. One must remember that
consensus panels on the definition of ED have • Consistent dysfunction—no difference
focused on putting the definition in terms of “satis- between intercourse and masturbation
factory” sexual activity, and the goal-oriented treat-
ment approach follows along that same line. Never- • Sleep erections and morning erections are
theless, some basic evaluation should be done in all absent or significantly diminished in quality
men and specialized tests are appropriate for some.
• Orgasm present, even without penile tumes-
History cence

The extent and duration of the ED should be ascer-


tained. The degree of current function may guide
Conversely, components of the history that should

the physician in predicting what level of therapy


make the clinician suspect a psychogenic source

might be needed to circumvent the problem. For


include:

instance, if the patient states he retains 70% of his • Younger age


natural function, an oral agent will likely be suc-
cessful in treating the problem. Whether the dys- • Absence of risk factors
function is intermittent or consistent is important.
Prior treatments for the problem should be listed • Abrupt onset, especially in relation to a psy-
and the exact method of administration of such chologically traumatic event
treatments should be elucidated, as incorrect admin-
istration may by the central reason for failure of the • Inconsistent and varying dysfunction, with
previous treatment. There have been 2 large studies differences between sexual situations
that demonstrated re-education alone resulted in
changing PDE5 inhibitor failures into successes. • Sleep and morning erections present
Furthermore, patient compliance with the medica-
tions should be taken into account. Compliance has • Orgasm absent
been shown to improve with patient education and
management of expectations, prescribing individu- Individuals with psychogenic ED should be identi-
alized treatment and optimizing treatment outcomes fied in the history, so that they may receive proper
by scheduling follow-up visits. psychological treatment. In those patients that have
a confusing history, further testing can be carried
out.

894 EDUCATIONAL REVIEW MANUAL IN UROLOGY


A general medical history should be performed on risk group should be delayed in their treatment until
all men presenting with ED. Drug interactions further cardiac evaluation is done, restratifying
are possible with ED treatments, and it is important them into low or high risk (Figure 2).
to have a complete list of patient medications.
Previous surgeries, such as prostatectomy, can lead Finally, the history should include queries about
to ED. Others, such as retroperitoneal lymph node what the patient’s goals are in treatment of erectile
dissection, can lead to disorders of ejaculation. dysfunction. This may change the choice of treat-
ment plan.
Evaluation of the person’s cardiovascular risk and
risk reduction program is important. Many men
with ED may not have good general medical care,
Physical examination

and with the Internet providing a convenient avenue The degree of virilization should be noted, and if
for supporting self-diagnosis, they may present to abnormal, may make the clinician suspect hypogo-
the urologist without having seen a physician for nadism. This condition should also be suspected
many years. Assessment for obesity, hypertension, when gynecomastia or small testicular size is noted.
dyslipidemia and diabetes may be deficient. Presen- Peripheral pulses should be palpated. If abnormal,
tation for ED may represent an opportunity for there may be an increased likelihood of vasculo-
improving the general health of a patient. Many genic ED. Neurological examination can tip off
urologists will not wish to order blood tests assess- CNS disease or peripheral neuropathies that may be
ing such cardiovascular risk, but should at least the cause of the dysfunction. Detailed examination
inform the patient of the relationship between ED of the penis for hypospadias, disproportion of the
and other conditions, and facilitate the referral to a corporal bodies and plaque is performed. The diag-
general doctor for such care. Alternatively, basic nosis of Peyronie’s Disease can be suspected by his-
blood tests, such as lipid profile and fasting blood tory, but corroborative findings on physical exami-
sugar might be ordered in anticipation of a primary nation are essential. If a plaque is palpated, it is
care office visit. essential to determine if it is present deep within the
cavernosal tissue, as this may be suspicious for an
The other cardiac history that is necessary in the infiltrative process, such as a penile metastasis.
urologist’s office is whether the patient can tolerate
the physical rigors of sexual activity. There is a pos-
sibility that an ED patient has not had an erection for
Basic laboratory testing

many years, and has not been subjected to any phys- Because of the potential effects on libido and also
ical challenge close to that of sexual intercourse. If penile tissue function, a testosterone level should
he is then given a method to have an erection, he be performed in all men presenting with ED. It is
may be at risk for a cardiac event during sex. Ques- unlikely that testosterone deficiency will be the
tions regarding ability to successfully perform entire cause of sexual dysfunction, but it may be a
activities of about 3–5 MET will predict success contributing cause. Whether or not to obtain a total
with sexual activity. testosterone (low cost) or free testosterone (more
accurate) is controversial. It is probably best to
The Princeton Guidelines is a good reference that obtain the level in the morning, especially if the
allows the stratification of men into certain cardiac patient is a younger man, as there is a diurnal
risk classes (Figure 2). When a man presents with variation with peaks in the morning. In men with
ED, a clinical assessment of risk factors allows him hypogonadism on initial testing, further evaluation
to be placed into low-, intermediate- and high-risk should include a prolactin level and LH. It is also
groups. The low-risk patients can be given an ED possible to check another pituitary axis (e.g., thy-
treatment and allowed to have sexual activity. The roid) to look for global pituitary dysfunction.
high-risk patients should be counseled that sexual Finally, a TSH test may be performed based on the
activity should not even be attempted until their patient’s medical history as both hyper- and
clinical situation is changed, i.e., with cardiac medi- hypothyroidism have been associated with ED.
cations, or revascularization, etc. The intermediate-

CHAPTER 28: ERECTILE DYSFUNCTION, PEYRONIE’S DISEASE AND PRIAPISM 895


Figure 2

Princeton Guidelines algorithm for interaction between


erectile dysfunction treatment and cardiac risk

Adapted from DeBusk R, Drory Y, Goldstein I, Jackson G, et al. Management of sexual dysfunction in patients with cardio-
vascular disease: recommendations of the Princeton Consensus Panel. Am J Cardiol.
2000;86:62F-68F.

In men without good prior medical care, assessment from those with predominantly psychogenic prob-
for cardiovascular risk factors with blood testing is lems. The men with organic ED will have poor
recommended. These include lipid profile and responses noted during sleep, in concert with their
assessment for diabetes with either fasting blood organic dysfunction. Men with psychogenic prob-
glucose or hemoglobin A1C. These will uncom- lems will have a discrepancy between sleep and
monly be ordered by the urologist, but patients conscious erections. They are psychologically pre-
should be encouraged to work with a primary care venting the erectile response during conscious sex-
doctor to have these assessments done. ual encounters, but in the absence of their anxiety,
during sleep, the response will function perfectly
Specialized testing fine.

As mentioned previously, specialized tests are not NPT testing can be done in a supervised sleep labo-
necessary in all patients. They should be used selec- ratory or in an ambulatory setting. The disadvan-
tively, and in those patients for whom the results tages of the laboratory setting are cost and difficulty
will alter clinical care decisions. Some of the sleeping in a strange surrounding. The advantage of
potential tests that may be used in men with ED are the lab is the ability to monitor for rapid eye move-
as follows: ment (REM) sleep by lab personnel. Absence of
such sleep may lead to false-positive testing.

Men will normally have several erections during Many urologists utilize the RigiScan ambulatory
Nocturnal penile tumescence (NPT) monitoring:

sleep. This is related to sleep pattern and not to sex- NPT monitor. Advantages include low cost and the
ual stimulation. Because men with a psychogenic ability of patient to sleep in his own surroundings.
source of ED will continue to have such sleep pat- Another advantage is the ability of the machine to
tern-related erections, this test represents an oppor- measure penile rigidity. REM sleep is not measured
tunity to differentiate men with organic problems in the ambulatory setting and false-positive tests

896 EDUCATIONAL REVIEW MANUAL IN UROLOGY


may be encountered. There are many standards for nation of the penile brachial index. Advantages of
what constitutes a normal test, but most would con- duplex ultrasound over prior tests include ability to
sider 2–3 erectile events per night, duration of >15 visualize the artery being measured, identification
minutes per event and rigidity of 50%–60% a nor- of Peyronie’s plaques, and direct measurement of
mal test (Figure 3). blood flow in response to pharmacologic injection.

Duplex doppler ultrasonography: This has uni- Injection of a vasoactive drug into the penis is
formly replaced other gross screening tests for essential to glean useful information from the test.
penile blood flow used in the past, such as determi- As noted previously, penile blood in the flaccid state
Figure 3

NPT monitoring

A. Normal RigiScan study


showing several episodes of
prolonged tumescence during
a night of sleep

B. Abnormal RigiScan study


with short-lived episodes of
tumescence. The tip does not
have adequate rigidity

From Broderick GA, Lue TF. Evaluation and nonsurgical management of erectile dysfunction and priapism.
In: Walsh PC, ed. Campbell’s Urology. 8th ed. Philadelphia, PA: Saunders; 2002: Chapter 46.

CHAPTER 28: ERECTILE DYSFUNCTION, PEYRONIE’S DISEASE AND PRIAPISM 897


is extremely low and nearly non-measurable by considered. The arteriogram is performed under
ultrasound scanning. It is the increase in blood flow pharmacologically stimulated erection. Penile arte-
that needs to be measured to assess for arteriogenic rial supply, as well as patency of potential donor
erectile dysfunction. There are a wide number of arteries (usually inferior epigastrics) is carefully
protocols for type and dose of injected medicines, examined. Also, it is important to note the presence
including 10–40 micrograms of prostaglandin E1, or absence of generalized atherosclerosis, the pres-
30–60 mg of papaverine, and combinations of ence of which may lessen enthusiasm for a revascu-
papaverine, phentolamine and prostaglandin E1. larization procedure. This very uncommon test
should be performed by angiographers with exper-
Normal ranges vary from author to author, but peak tise in this area, to maximize the chance that useful
systolic blood >25–35 cm per second in the caver- information will be gleaned.
nosal artery following injection represents normal
arterial inflow. End-diastolic flows should be <3 cm
per second and resistive index >0.85. These suggest
normal veno-occlusive function. The patient should
achieve a normal erection.

If during such a test, the patient notes that the erec-


tion obtained is much poorer than his normal base-
line erection, the test may be a false-positive.
Epinephrine that is released in response to the penile
injection activates the detumescence mechanism
and may be the cause. A patient with sufficient anxi-
ety can also create a false-positive.

Cavernosometry/cavernosography: The test is


invasive, with 2 needles in the penis, 1 for infusion
of medications and saline, and the other for pressure
measurements. A pharmacologic agent is injected to
achieve full smooth muscle relaxation and saline
infusion is begun. Salient data obtained from the
study are necessary to obtain erection and maintain
erection and pressure degradation curves. What is
considered a normal test varies with authors. Injec-
tion of contrast at the end of the study may identify
the extremely rare patient with a focal large leak
from the corpus cavernosum that may be amenable
to surgical therapy.

In men whom veno-occlusive dysfunction is sus-


pected, this test can give more specific information,
but is rarely carried out, as specific treatment of
such dysfunction is rarely performed. When venous
ligation procedures were first introduced, this test
was more common, but such surgery has fallen out
of favor due to the poor durability of the results.

Penile arteriography: Specific arteriography of the


internal pudendal system is reserved for those indi-
viduals in whom penile revascularization is being

898 EDUCATIONAL REVIEW MANUAL IN UROLOGY


4. ED Treatments

results in inhibition of PDE throughout the body,


but due to the relative importance of the enzyme in
Sex Therapy

the penile tissue, a favorable environment in the


penile tissue of prolonged cGMP effect is seen
After identification of a psychogenic source for

treating sexual dysfunction is in order. Organic without major change in systemic homeostasis.
ED, referral to a therapist with expertise in

treatments may be used in conjunction with the This is due to direct inhibition of the breakdown
therapist’s recommendations, but they should have of cGMP produced by the NO/cGMP system
a good safety profile, so as to not put the patient at described earlier.
undue risk. It is wise to let the therapist guide intro-
duction of organic treatments, otherwise the patient Drugs available include sildenafil citrate (25, 50 and
may rely on the organic treatment and have little 100 mg) vardenafil hydrochloride (5, 10 and 20
incentive for working on the psychogenic side. Sex mg), and tadalafil (5, 10 and 20 mg for demand
therapy approaches include psychodynamic ther- dosing, and 2.5 or 5 mg for daily use). All are highly
apy, systematic desensitization, sensate focus, cou- specific for PDE5, and all are efficacious in
ples therapy, behavioral assignments, sex education, promoting penile erection capability in men with
communication and sexual skills training, and mas- ED. Vardenafil is the most potent agent in labora-
turbation exercises. Unfortunately, there is a lack of tory testing, followed by sildenafil and then
studies comparing these therapies and it is unknown tadalafil, but laboratory potency does not necessar-
which ones are most effective. ily translate into clinical effectiveness. In recent
clinical trial by Jannini et al, the 3 drugs have shown
Yohimbine equal efficacy.

Yohimbine is a supplement that has a historical Success rates for improvement of erection is
place in the treatment of ED. Its proposed role in the approximately 70%–80%, and improvement to the
treatment of ED is to increase parasympathetic and point where the drugs are suitable for monotherapy
decrease sympathetic activity. Many OTC male of ED is seen in approximately 60% of cases. The
enhancement supplements may contain some drugs are more effective in men with mild ED, and
amount of yohimbine. In high enough doses it can men with severe ED may not respond to these
lead to diaphoresis, palpitations and elevated heart agents. They are permissive agents—they do not
rate. The AUA does not recommend yohimbine for work autonomously, but require sexual stimulation
the treatment of ED, and patients with a history sug- to initiate the processes leading to nitric oxide pro-
gestive of cardiovascular disease should be cau- duction in the penis. Without such production, the
tioned against these supplements. drugs have no effect, as they do not turn on the pro-
cess—they merely stop it from turning off. PDE5
inhibitors are taken prior to sexual activity to obtain
tissue levels prior to sexual stimulation, and the
Apomorphine

Apomorphine is a nonspecific dopaminergic ago- effectiveness of each drug is dependent on pharma-


nist which works at the hypothalamic level to initi- cokinetics.
ate erection. Apomorphine shows lower efficacy
and satisfaction rates than PDE5 inhibitors but is not Sildenafil and vardenafil each achieve peak plasma
contraindicated in patients taking nitrates. Side concentrations in about 1 hour, and tadalafil peaks
effects include nausea, headache and dizziness. in 2 hours. The half lives of sildenafil and vardenafil
Apomorphine is not approved for use in the United are 4–5 hours, and for tadalafil, the half-life is 18
States. hours in young men and longer in elderly men.
Daily tadalafil presents the option for continuous
treatment for ED, equivalent to that of other organic
diseases, instead of the on-demand treatment link-
PDE5 inhibitors

This class of drug is considered first-line therapy for ing medication directly to sexual intercourse.
those men presenting with ED without a contraindi- Choice of agent must take into consideration the
cation to their administration. Administration preference and sexual pattern of the patient, without

CHAPTER 28: ERECTILE DYSFUNCTION, PEYRONIE’S DISEASE AND PRIAPISM 899


the physician making assumptions about such There have been recent reports of non-arteritic
things. The physician should be aware of correct ischemic optic neuropathy (NAION) associated
administration of the drugs as well as compliance. If with PDE5 inhibitors. Caution needs to be exercised
one drug taken correctly does not work, there is evi- in men with acute vision loss on these agents, with
dence to suggest that another might have an effect in discontinuation the safe option. However, men with
spite of the identical theoretical mechanism of vascular disease are exactly those people at risk for
action. both NAION and ED. It is likely that the associa-
tion, if any, is due to the underlying medical condi-
There are class-related side effects due to systemic tion and not a drug effect.
inhibition of PDE5. Most of these are vasodilatory
effects, with systolic and diastolic blood pressures
decreasing into the range of low single digits. Other
Advantages of PDE5 inhibitors include oral

vasodilatory side effects commonly seen are skin


administration, safety and effectiveness. Disad-

flushing, rhinitis and headache. Inhibition of PDE5


vantages include cost, systemic administration as

at the gastroesophageal sphincter may lead to acid


opposed to local effect, potential for interaction

reflux and dyspepsia.


with nitrates and poor efficacy in men with severe
ED. Cost is becoming an issue as well, as some

Although the drugs are highly specific for PDE5,


insurance companies offer extremely limited cov-

there are 2 important cross-class PDE interactions.


erage for any ED medications.

Sildenafil at clinical doses inhibits PDE6 in the New studies have suggested that daily PDE5
retina of the eye to a lesser degree than PDE5, and inhibitors can improve endothelial function and that
this results in 5%–10% of men reporting a bluish this effect may remain even after the treatment is
discoloration in their color perception. This stops as stopped. Whether this can produce lasting effects on
the drug level goes down, and no long-term prob- erectile function warrants further investigation, as
lems have been associated with this vision change. data are limited. One trial found no sustained effect
Mild QT interval prolongation has been reported after cessation of daily vardenafil in patients with
with vardenafil treatment and caution should be mild to moderate ED, compared to patients who had
taken in patients with preexisting QT prolongation. received on-demand treatment but did note signifi-
cantly higher IIEF-EF scores compared to baseline
Tadalafil at clinical doses inhibits PDE11, but the in both groups. Another retrospective study found
effect of this inhibition is not known. The back pain some sustained effect after a 4-week washout period
and muscular pain associated specifically with following daily tadalafil treatment. Larger studies
tadalafil administration is of unknown etiology. with control groups and longer follow-up are
needed.
Overall, the frequency of side effects do not differ
significantly between the 3 drugs in spite of their
different pharmacokinetic properties.
Testosterone

Testosterone treatment for ED is generally contro-


versial and should only be initiated in patients with
low testosterone. The treatment is indicated in
Because these agents inhibit the PDE5 systemi-

tion with organic nitrates. Nitrates activate the young patients with testosterone deficiency and has
cally, it is essential they NOT be given in conjunc-

NO/cGMP system in the systemic circulation and been shown to improve erectile function in this
these agents stop it from turning off. Profound drops group. Testosterone treatment seems to be less
in blood pressure may be seen. To a lesser degree, effective in older patients and in men whose testos-
alpha blockers may synergistically drop blood pres- terone deficiency was diagnosed after they pre-
sure with PDE5 inhibitors, and caution must be used sented with ED. Combination therapy with PDE5
in this drug interaction in conjunction with pub- inhibitors and testosterone can be attempted in men
lished prescribing information. with low testosterone who fail to respond to PDE5
inhibitors alone.

900 EDUCATIONAL REVIEW MANUAL IN UROLOGY


Prostate and cardiovascular risks associated with omimetic agents, if addressed in a timely fashion.
testosterone treatment are currently subject to inves- Fibrosis can occur in a small number of cases, but
tigation. At this time, testosterone treatment is con- with PGE1, the fibrosis rate is much less than with
traindicated in men with clinical evidence of off-label use of papaverine.
prostate cancer. Close follow-up is necessary,
including digital rectal examination and monitoring
of PSA, hematocrit and hepatic biomarkers.
Advantages of PGE1 injection therapy include
higher efficacy in difficult cases and its ability to
initiate erection. Disadvantages include cost,
Penile injection therapy invasiveness, penile pain, and the possibility of

It was noted quite by accident by the famous French


priapism and penile fibrosis.

surgeon, Virag, that injection of papaverine into the


penis can result in erection. Early work by Brindley
Intraurethral alprostadil

verified this finding, as his memorable presentation In hopes of decreasing invasiveness of injectable
at the Las Vegas AUA meeting will attest to. PGE1, the urethral suppository form of the drug is
Throughout the intervening years from this early available in doses from 125–1000 micrograms. The
report to the present, many vasoactive agents have efficacy is approximately 30%–40%, but in respon-
been used for ED therapy. Only alprostadil sive individuals, this system avoids the need for
(PGE1) has been approved by the FDA for this indi- penile injection. Penile pain is also a problem with
cation. Alprostadil, when introduced into the corpus this method of administration, although priapism is
cavernosum, actives the cAMP system described extremely uncommon. The initial administration
earlier, leading to calcium flux and smooth muscle should be performed under medical supervision.
relaxation. According to the AUA guidelines, com-
binations of alprostadil, papaverine and phento-
lamine can be used to increase effect or to reduce
Advantages of intraurethral alprostadil include

side effects if treatment with alprostadil alone fails.


less invasive administration compared to injection
and ability to initiate erection. Disadvantages

Alprostadil is available commercially in prepara-


include low efficacy, penile pain, a small risk of

tions up to 60 micrograms. It is stored as a powder


hypotension and cost. Combining the treatment

and needs to be reconstituted prior to use. Direct


with a vacuum constriction device or a PDE5

injection of the drug results in a suitable erection for


inhibitor may increase the efficacy.

intercourse in 60%–80% of ED cases. In distinction


to PDE5 inhibitors, alprostadil is an actual initiator
Vacuum constriction devices

of erection and does not depend on sexual activity The concept of these devices is actually quite sim-
for erection to occur (although sexual activity may ple—instead of positive arterial pressure bringing
augment the response). The initial injection proce- blood into the penis, an external negative pressure
dure should be done under medical supervision. device pulls the blood in. A constriction band is
placed at the base of the penis prior to removal of
Adverse events associated with PGE1 injection the vacuum to keep the blood there. The band is
include priapism and penile pain. The pain is pre- allowed to remain in place for up to 30 minutes.
sent in about 35%–40% of cases and may be a cause These devices can cause discoloration of the penis,
for discontinuation of the treatment. Priapism usu- decrease in temperature, discomfort with ejacula-
ally occurs at the time of the first injection, when tion, reduced sensation and loss of erection during
sensitivity to the drug in the individual patient is not intercourse. However, the adverse events are mild
yet known. Penile injection therapy should not be and limited, and the FDA has taken away the
used when conditions that predispose to priapism requirement for a prescription that was in place sev-
are present. The priapism associated with penile eral years ago, due to the track record of safety.
injection therapy is easily treated with sympath- There is a high drop-out rate in people that begin

CHAPTER 28: ERECTILE DYSFUNCTION, PEYRONIE’S DISEASE AND PRIAPISM 901


using these devices, usually due to the less-than- 2-piece inflatable devices (Figure 5) were designed
optimal sexual experience. However, there are some to meet the advantages of inflatables, as far as girth
long-term users, and for the patient who is a nonin- expansion and rigidity, while allowing flaccidity
terventionalist, a vacuum device may be a satisfac- and concealment, and at the same time also capital-
tory choice. izing on the advantages of semirigid—lack of the
need for a reservoir implantation. There is only one
2-piece inflatable available on the market. It has 2
cylinders which are filled in the center, allowing for
Advantages of vacuum constriction devices

fluid insertion just under the surface of the cylin-


include safety, noninvasiveness, rapid initiation of

ders. The fluid source is partly from a small reser-


therapy with little training and no prescription is

voir at the base of each cylinder (deep in the corporal


required. Disadvantages include poor sustenance

body) and partly from the scrotal pump used to activate


of erection, change in color or temperature of the

the device.
penis, ejaculatory pain and a high drop-out rate
among users.

Some investigators have reported very high satis-


faction rates with the two-piece inflatable, while
Penile prosthesis

Penile implants have been available since the 1970s. other are more pessimistic. While the manufacturers
Initial devices had an extremely high failure rate, have successfully developed a product with simplic-
with the expectation that reoperation would be nec- ity and inflatability, some patients find the excur-
essary in the majority of cases. The devices have sion limited, the erection not acceptable and/or the
improved substantially since then. Penile implants deflatability limited and concealment difficult.
are indicated in men who fail or reject more conser-
vative treatments for ED. Failure of other treatments 3-piece inflatable devices (Figure 6) are most com-
are most often seen in diabetics or after pelvic monly reported in series in the literature. They con-
surgery, as well as in untreated priapism. Advan- sist of 2 cylinders, a reservoir in the pelvis and a
pump in the scrotum. They are the most compli-
cated devices and require a larger surgical proce-
tages of penile prosthesis are high efficacy in all

dure to implant the device. However, they give the


patient severities, and high patient and partner

best flaccidity and the best rigidity of all the devices


satisfaction rates. Disadvantages include the need

available.
for surgery and potential complication of infec-
tion, erosion and need for reoperation.

There are 4 major types of penile prosthesis: Surgical approaches for penile implants

Malleable or semirigid devices (Figure 4) remain There are a variety of ways to place a penile pros-
firm all the time, but can usually be molded into thesis, but most surgeons utilize the infrapubic or
conformation with the clothing. They are very sim- penoscrotal approaches.
ple and have a low failure rate, but concealment is
often many times a problem. Furthermore, since In the infrapubic approach, an incision is made at
there is no excursion in girth, many patients are the base of the penis. The corporal bodies are
unhappy with the erection quality. approached dorsolaterally, and the cylinders placed
there, after dilation. The pump is placed lateral to
Mechanical devices are similar to semirigids in that the penis as low in the scrotum as possible. A fascial
they are in a fixed degree of penile girth, so erection incision is made to place the reservoir and the con-
quality is a complaint of these devices. However, nections are made to complete the implantation.
since they are arranged with stacked disks that
allow a total of more movement, they are much
Advantages of the infrapubic approach include

more malleable and easier to conceal. The flexibil-


avoidance of the urethra, and ability to directly

ity sometimes allows the penis to assume some


open the abdominal fascia for reservoir place-

rather odd shapes and occasionally be over flexible


ment. Disadvantages include longer operative

during sexual activity.


time, abdominal fascial incision pain and possibil-
ity of dorsal neurovascular bundle injury.

902 EDUCATIONAL REVIEW MANUAL IN UROLOGY


In the penoscrotal approach, an incision is made at
the penoscrotal angle over the urethra. The corporal
Figure 5

bodies are approached lateral to the urethra and the


cylinders placed there. The pump is placed in the
2-piece inflatable penile prosthesis

inferior dependent portion of the scrotum. The


reservoir is placed blindly through the internal ring
into the retropubic space and inflated after insertion.
Before this is done, complete bladder emptying
through a urethral catheter must be performed.
Reservoir displacement is seldom seen and can be
corrected through an inguinal incision. Advantages
of the penoscrotal approach include decreased
operative time, decreased postoperative pain due
to no fascial incision and better access to the cor-
poral bodies for dilation. Disadvantages include
the possibility of bladder or vascular injury during
blind reservoir placement.

American Medical Systems, Minneapolis, MN

Figure 4 Figure 6

Malleable penile prosthesis 3-piece inflatable penile


prosthesis with paired cylinders,
reservoir and scrotal pump

American Medical Systems, Minneapolis, MN

Coloplast, Copenhagen, Denmark

CHAPTER 28: ERECTILE DYSFUNCTION, PEYRONIE’S DISEASE AND PRIAPISM 903


tion, and coating of the devices with antibiotics or
bacterial resistant materials to lower the risk of
Complications of penile prosthesis surgery

infection.
include infection, erosion, and mechanical failure
necessitating reoperation. Erosion is thought to

A study by Wilson et al assessed 15-year outcomes


be due to subclinical infection in nearly all cases.

of 3-piece inflatable devices (n=2,384), finding a


After placement of a penile implant, there will be

60% revision-free implant survival rate. Future sur-


a lack of glans tumescence and the erect penis is

vival rates are expected to surpass this due to


usually shorter than in earlier erections. If the

improvements in design.
cylinders used are too short, a downward drooping
of the distal penis may be seen. This can be cor-
rected with longer cylinders. Penile sensation is
usually preserved. Penile Revascularization

Infection can be acute after placement of prosthesis, Revascularization of the penis to correct erectile dif-
or indolent, presenting well after the initial surgery. ficulty is uncommonly performed and an accurate
Acute infection tends to be with virulent organisms, estimate of the efficacy cannot be made based on the
such as S. aureus creating purulent infection. Late available literature. Criteria for patient selection
infection tends to be most commonly due to S. epi- should include a young age, recently acquired ED
dermidis. Coating of prosthetic devices has been due to focal arterial occlusive disease or significant
shown to reduce the infection rate. This should be pelvic/perineal trauma, and the absence of systemic
<1% in nondiabetic patients while it is a little higher vascular disease. The procedure appears to have an
in diabetics. especially high success rate (up to 100% in 1 trial)
when performed due to pelvic or perineal trauma.

Angiography is a prerequisite to the surgery.


One can never be faulted for removing a device to

Because ED in older men coexists with other comor-


treat infection. However, the fibrosis and shrink-

considers a salvage operation in such cases. This bidities and commonly involves some degree of
age seen following explantation dictates that one

procedure, popularized by Mulcahy, involves venous leak, revascularization is not an option. Due
removal of the infected device, followed by irriga- to the lack of data on the subject, no specific surgical
tion of a series of solutions of peroxide, betadine approach can be recommended.
and antibiotics. The operative field is re-prepped
and draped with an entire new set of instruments, as
if it were a new operation. The new device is then
Penile Rehabilitation

inserted. The rate of a successful outcome in a sal- Radical prostatectomy—even with the nerve-spar-
vage prosthesis operation is approximately 80%. ing techniques— usually causes some degree of ED.
The exact magnitude of the problem is controver-
Mechanical malfunctions may be due to cylinder or sial, as incidences vary widely in published rapports,
tubing leak, connector failure, autoinflation due to but it is indisputable that the surgery is an increas-
pressure on the reservoir or cylinder aneurysm for- ingly prevalent cause of ED because prostate cancer
mation. If mechanical malfunction is seen in a is diagnosed more often and in younger individuals
than earlier. The proposed pathophysiological
with a new one. Recent data from Henry and others mechanism behind post-prostatectomy ED is that
prosthesis, the entire device should be replaced

suggest that some modification of a salvage opera- damage to penile nerves and vasculature reduce
tion be done in all re-do implants, as the rate of sub- blood flow to the penis. This results in fibrosis and
clinical bacterial colonization is substantial. The 2 degeneration of smooth muscle tissue in the corpora
companies currently producing 3-piece inflatables cavernosa. Penile rehabilitation programs have
have made significant improvements in recent years focused on increasing blood flow to the penis fol-
to lower the failure rate. These include: Dacron but- lowing surgery to increase the rate of spontaneous
tressing and parylene coating of silicone cylinders, erections. In order to accomplish this, penile injec-
improvement of connector system, lock-out systems tion therapy, daily and on-demand PDE5 inhibitors
in the reservoir and/or pump to prevent autoinfla- and vacuum constriction devices have been

904 EDUCATIONAL REVIEW MANUAL IN UROLOGY


employed in various regimens. In 1997, Montorsi et
al showed the effect of penile injection therapy in a
small study. Unfortunately, the effects have never
been reproduced. Padma-Nathan et al has shown the
effect of sildenafil daily, but Montorsi et al were
later unable to show a lasting effect on spontaneous
erections with both daily and on-demand vardenafil
use. Likewise, vacuum constriction devices have
shown promise in some studies but there is still a
lack of compelling evidence to support the treat-
ment. In spite of the limited data, penile rehabilita-
tion programs are becoming increasingly
widespread. Current consensus states that potential
benefits should be discussed with patients, but no
specific recommendations regarding the optimal
rehabilitation regimen can be given at this time.

Future treatments

New targets of drug treatments include novel PDE5-


inhibitors, guanylate cyclase activators and com-
pounds inhibiting smooth muscle contraction. Stud-
ies are also being conducted in the areas of gene and
stem cell therapy. These treatments are, for the most
part, on the stages of laboratory and animal testing,
and focus on endothelial and vascular function as
well as on smooth muscle cells and neural tissue.
The first phase I clinical trial in humans regarding
gene transfer to restore smooth muscle function has
been conducted by Melman et al and has shown no
serious adverse events in the 11 study participants.

CHAPTER 28: ERECTILE DYSFUNCTION, PEYRONIE’S DISEASE AND PRIAPISM 905


906 EDUCATIONAL REVIEW MANUAL IN UROLOGY
II. Peyronie’s Disease

1. Epidemiology and Etiology of PD

2. Clinical Course of PD

3. Office Evaluation and Medical Management

4. Surgical Therapy

CHAPTER 28: ERECTILE DYSFUNCTION, PEYRONIE’S DISEASE AND PRIAPISM 907


1. Epidemiology and Etiology of PD

Older studies have underestimated the rate of PD in


the population. Many of the older reports were sim-
Introduction

Peyronies’s Disease (PD) was first described by ply case series or simple calculations based on num-
Francois de la Peyronie in 1743, and the disease still ber of patients seen vs number of cases of the condi-
carries his name. The disease is manifested by tion. Incidences in these studies were ranging from
development of penile plaque, pain and deformity 0.3%–1%. Dr. Charles Devine estimated that 1% of
of erection due to disproportion of the tunica albug- male physicians in his patient population suffered
inea from the plaque. There are large voids in our from the condition. Autopsy studies with evidence
understanding of this condition, and thus we are of penile plaque suggest a prevalence as high as
lacking specific treatments to reverse the condition. 22% of men.
Many patients with this condition will tolerate it
until it reaches stability, and then decide whether or More recent studies have reported a higher inci-
not surgical intervention is indicated. It represents a dence and may be more accurate due to their
frustrating condition for both the patient and treat- methodology. Sommer in 2002 reported results of a
ing physician. survey of 8,000 men with a self-reported rate of
3.2%. Mulhall looked for evidence of PD in men
attending a mass prostate cancer screening event
and found that 8.9% of men had objective findings
on evaluation. The true incidence of Peyronie’s
Disease is higher than previously thought and is
most likely between 3%–10%.

Most investigators believe that Peyronie’s Disease


results from a combination of a scar-forming ten-
dency coupled with penile trauma as the direct
inciting event.

Evidence for the genetic predisposition is present.


There is an association between Dupuytren’s con-
tracture and PD; men are prone to PD if their father
had Dupuytren’s contracture. There is an associa-
tion seen between PD and plantar fasciitis. Gene
expression studies in PD tissue and normal tunica
albuginea show differences in genes that control
scar modification.

In a man with a predisposition to PD, penile trauma


during intercourse results in an injury to the tissue.
Cytokines and fibrin in the tissue launch an initial
attempt at healing. In men with a PD tendency, the
normal migration of fibroblasts occurs, but these
cells then differentiate into myofibroblasts, and con-
tribute to abnormal healing and abnormal scar for-
mation. Scar remodeling proceeds in an abnormal
fashion in PD patients, resulting in a plaque.

908 EDUCATIONAL REVIEW MANUAL IN UROLOGY


2. Clinical Course of PD 3. Office Evaluation and Medical
Management

In the early stages of the condition, there is develop- In men presenting with PD, one should try to elicit
ment of a (sometimes) painful plaque. As the plaque a history of penile trauma during intercourse, but
remodels, pain may persist as penile deformity commonly, no history can be found. Other scar-
evolves. In nearly everyone, the pain eventually forming manifestations in the past history, family
abates and this is usually associated with plaque sta- history or on physical examination should be
bilization. This occurs in about 12–18 months. Once sought. The degree of erectile function should be
stabilization occurs, it is less likely that the situation documented and objective testing is sometimes
will worsen or improve. indicated to serve as a baseline prior to therapy. A
history of prior surgeries and treatment for ED, par-
Early reports of PD suggested a very optimistic out- ticularly intracavernosal injection therapy, should
look for spontaneous improvement or even resolu- be noted.
tion of the condition. In 1970, Williams reported
that 76% of people with PD either improved or On physical examination, one should define the
resolved their condition within 4 years. Gelbard was extent and location of the plaque. This is a chal-
less optimistic in 1990, reporting only 13% resolv- lenge in itself, as the plaque can extend distally,
ing the situation, and 50% of patients stabilizing, proximally, laterally, through the septum or some
with the rest continuing to worsen. A large recent combination of these. Ultrasound may be used to
study by Mulhall agreed with these numbers with corroborate the physical exam findings. As patients
12% improving and a little less than 50% of patients tend to overestimate the degree of curvature, a pho-
stabilizing with at least 1 year of follow-up. Thus, it tograph of the deformity in the erect state is an
appears that spontaneous improvement of Pey- excellent tool to allow the clinician to appreciate
the degree of physical impairment associated with
the condition. In-office pharmacologic erection
ronie’s Disease occurs at a relatively low rate,

also can be used to document the deformity. Accu-


probably between 10%–15%.

Erectile dysfunction is a common problem, and PD, rate measurements help aid in determining the
based on recent reports is also common. Thus, it response to therapy.
may be difficult to decipher whether there is an
association between the 2 conditions. It appears that Duplex ultrasound following intracavernosal injec-
the rate of ED in PD patients has increased. Hell- tion can detect the degree of arterial inflow disease
strom suggested that 20%–40% of men with PD will or, more importantly, the degree of veno-occlusive
suffer from erectile dysfunction. Investigators impairment. Documentation of baseline impair-
examining the etiology have found abnormal veno- ment may be helpful in addressing concerns after
occlusive function in a large number of patients, surgical therapy when patients complain of decline
suggesting that the plaque creates abnormal tissue in function.
function and pliability underneath.
Most importantly, it is important to document the
source of intracavernosal flow prior to planning
surgical therapy. During Peyronie’s plaque incision
and grafting, the neurovascular bundle is com-
monly elevated from the tunica to allow access to
the plaque. If an individual supplies his cavernosal
bodies with blood from the dorsal arteries (Figure
7), such an operation may result in interruption of
this important collateral supply, with resultant arte-
riogenic ED.

CHAPTER 28: ERECTILE DYSFUNCTION, PEYRONIE’S DISEASE AND PRIAPISM 909


Figure 7

Duplex ultrasound of a man with aberrant penile blood supply

His cavernosal arteries are supplied by perforators from the dorsal arteries. This man would
be at high risk for arteriogenic erectile dysfunction if the neurovascular bundle were elevated in a
surgical procedure.

There have been a variety of medical managements Perhaps the best studied and most promising medi-
put forth for the treatment of PD. Most reports on cal management is intralesional verapamil. Scar for-
such therapies are simple case series, where all mation has calcium-dependent processes, and the
patients have received the treatment and the success theory is that a calcium channel blocker such as ver-
rate is published. Since there is no control group in apamil may be beneficial. The largest experience is
most studies, the placebo effect cannot be quanti- in an uncontrolled series of Levine, which sug-
fied, nor can the background rate of spontaneous gested that penile deformity improved in more than
resolution. Treatments that have been suggested as 60% of cases, with worsening in only 10%—very
effective in PD through the years have included: favorable to historical thinking. There are 2 small
vitamin E, colchicines, Potaba, tamoxifen, Acetyl- placebo-controlled studies showing some benefit
L-Carnitine, coenzyme Q(10), omega-3 fatty acids, from this therapy.
as well as injectable corticosteroids, collagenases,
verapamil, and interferon-a-2a or -2b and, recently,
shock-wave therapy. Most of these treatments
remain doubtful; however, smaller randomized tri-
als have suggested that Potaba, acetyl-L-carnitine
and coenzyme Q(10) may be of benefit. Further
studies are needed on shock-wave therapy as well as
on injectable collagenases and interferon, as current
results are contradictory.

910 EDUCATIONAL REVIEW MANUAL IN UROLOGY


4. Surgical Therapy

Prior to performing surgery on PD, it must be stable In men presenting with PD with deformity and con-
for a minimum of 6 months. One does not want to comitant decline in erectile function, the rate of
miss the opportunity for spontaneous resolution but, postoperative ED is expected to be higher. Such
more importantly, if the deformity is changing, it men would be better served with implantation of a
may continue to change after surgery. Therefore, the 3-piece penile prosthesis to treat the ED and also to
penis may be straight at the end of the operation, help correct the penile deviation. In the operating
only to have a recurrence of a deformity some room, after placement of the prosthesis, a penile-
weeks later. If the patient has excellent erections molding procedure consisting of forceful shaping of
and the deformity does not preclude sexual inter- the penis with the device inflated can be performed
course, the wisdom of such surgery is suspect, as if residual curvature is present. Rarely, prosthesis
the patient may end up worse after surgery than his implantation needs to be combined with the formal
pre-surgical condition. straightening procedure described above.

The first group of surgical procedures for PD are the


penile-shortening procedures, commonly referred
to as plication procedures, or the Nesbit procedure
Figure 8

(Figure 8). The idea is that the tunica albuginea


opposite the plaque is shortened to match the short-
Penile plication procedure

ening from the disease. The resultant erection will


be straight, as both sides are restricting expansion of
the penis equally. Advantages of penile-shortening
procedures are lower incidence of postoperative
ED and lower incidence of penile numbness, since
the neurovascular bundle is usually not mobilized.
The disadvantage is that an already shortened
penis (shortened from the PD) is made even
shorter by the surgery.

Penile lengthening procedures involve plaque inci-


sion (and rarely excision) with grafting of the tunica
defect (Figure 9). Since most plaques are located
dorsally, access must be gained by elevation of the
neurovascular bundle, opening up the possibility of
post-operative glans numbness. The fact that the
intracavernosal tissue is exposed opens up the pos-
sibility of destroying some degree of tissue func-
tion, resulting in postoperative erectile dysfunction.
Graft materials used were originally dermis, fol-
lowed by synthetics, such as Dacron and Gore-Tex.
Currently, many PD surgeons have gravitated to the
cadaveric pericardium or porcine small intestinal
submucosa grafts. The advantage of penile-length-
ening procedures is reversal, to some degree, of
penile shortening. The disadvantages include a
higher rate of penile numbness and
From Gholami SS, Lue TF. Correction of penile curvature
postoperative erectile dysfunction. using the 16-dot plication technique: a review of 132
patients. J Urol. 2002;167:2066-2069.

CHAPTER 28: ERECTILE DYSFUNCTION, PEYRONIE’S DISEASE AND PRIAPISM 911


Figure 9

Grafting of the tunica defect

A.

After elevation of the neurovascular bundle, an


artificial erection test shows severe deformity

B.

The graft (in this case dermis) has been sutured into
place to fill the defect caused by plaque incision

C.
Final artificial erection test shows correction of the
deformity

From: Backhaus BO, Muller SC, Albers P. Corporoplasty


for advanced Peyronie’s disease using venous and/or der-
mis patch grafting: new surgical technique and long-term
patient satisfaction. J Urol. 2003;169:981-984.

912 EDUCATIONAL REVIEW MANUAL IN UROLOGY


III. Priapism

1. Ischemic vs Nonischemic
Priapism

2. Approach to Priapism

3. Questions

4. Further Reading

CHAPTER 28: ERECTILE DYSFUNCTION, PEYRONIE’S DISEASE AND PRIAPISM 913


1. Ischemic vs Nonischemic
Priapism

One of the immediate challenges that faces the treat-


ing physician is determining if the priapism is
Introduction

Priapism is a pathological prolonged erection unas- ischemic.


sociated with sexual stimulation. It is one of the true
urological emergencies. If this condition persists Ischemic priapism is a hypoxic, low-flow state and
without treatment, permanent penile tissue damage is due to lack of drainage of blood from the corpus
may result in the loss of erectile function. cavernosum. It is quite painful to the patient, and
leads to cavernosal damage and permanent erectile
dysfunction if not treated. The erection is usually
very firm and noncompressible. There is very little,
if any, flow on duplex ultrasonography. Blood gases
obtained from corpus cavernosum show low PO2
(<30 mm Hg), high PCO2 (>60 mm Hg), and low
pH (<7.25). Etiologies of low-flow priapism
include penile injection therapy (usually low-flow
after several hours); hematological disorders
including sickle cell anemia and leukemia; psy-
chotropic drugs; pelvic malignancies with obstruc-
tion of venous outflow; metastatic tumors of the
penis; heparin and intravenous fat emulsion. About
half of ischemic priapism cases are estimated to be
idiopathic. Stuttering priapism is the term for recur-
rent ischemic priapism and is usually associated
with either sickle cell or idiopathic anemia.

In contrast, nonischemic priapism is a high-flow


state, caused by unchecked blood flow into the cor-
pus cavernosum, but with preserved outflow. There
is continuous turnover of oxygenated blood and
lack of ischemia. Although the patient might be
troubled by the fact that he has a continuous erec-
tion, it tends to not be painful. On examination, it is
usually compressible but fills rapidly after the pres-
sure is relieved. Cavernosal blood gases show a nor-
mal arterial pattern, and duplex Doppler ultrasound
shows high continuous flows through the caver-
nosal arteries. Etiologies of nonischemic priapism
are penile trauma, with laceration of the cavernosal
artery, intracavernosal pharmacotherapy (early after
injection) and certain neurological condition, such
as acute spinal cord injury.

Ischemic priapism IS an emergency, but non-


ischemic priapism is not.

914 EDUCATIONAL REVIEW MANUAL IN UROLOGY


2. Approach to Priapism

Initial assessment includes attempting to find the


source of the problem on history. Has the patient
Figure 10

used penile injection therapy or psychotropic drugs?


Is there a history of sickle cell? Is the problem recur-
Ebbehøj shunt

rent? Examination may give clues as to whether


ischemia is present. Blood may be obtained for
blood gas determination to make a final determina-
tion. One should determine on physical exam which
compartments of the penis are erect. Usually the
corpus spongiosum is not involved and makes surgi-
cal shunting into the spongiosal space a possibility.
If the entire penis is involved, one needs to consider
the possibility of an infiltrative process, rather than
simple priapism.

Laboratory assessment includes cavernosal blood


This is one variety of distal cavernosal-spongiosal

gas determination. One should also order a com-


shunt for priapism

plete blood count to look for hematological malig-


nancies and hemoglobin electrophoresis. A toxicol-
From Bochinski DJ, Deng DY, Lue TF. The treatment of

ogy screen may give insight into drug-induced pri-


priapism—when and how? Int J Impot Res. 2003;15

apism. Color Doppler ultrasound may assist in rates for shunting procedures in rectifying the pri-
(suppl 5):S86-S90.

determining state of ischemia. apism situation are between 60%–70%.

Initial management of ischemic priapism is aimed at Erectile dysfunction occurs after many cases of pri-
treating the underlying cause, if identified, and apism and the likelihood for permanent dysfunction
clearing the cavernosum of the ischemic blood and increases as the time from the onset of the priapism
restoring oxygenated blood flow. Aspiration of increases. The rate is low in men who promptly
blood coupled with saline irrigation will rarely be respond to irrigation or injection of phenylephrine.
curative. In cases of priapism due to penile injection Those who require surgical shunting have a higher
therapy, this may work. However, after irrigation is rate of permanent dysfunction. Those responding to
attempted, injection of a sympathomimetic agent is distal shunts have a 50% rate of ED and those
the next step. The best agent is phenylephrine, 500 requiring proximal shunts retain erectile function in
only about 25% of cases. This is not to imply that
micrograms for adult patients). This can be distal shunts are less successful in rectifying the
micrograms per mL, to inject 1–2 mL (500–1,000

repeated every 3–5 minutes for an hour. Blood pres- problem, but more a reflection of longer duration of
sure should be monitored during this administra- refractory priapism, causing further damage while
tion. Phenylephrine is the drug of choice due to its increasingly complex procedures are introduced
pure alpha agonist effect and lack of secondary neu- over time to treat it. With long-lasting priapism, the
rotransmitter release. Injection of a sympath- immediate insertion of a penile implant may be con-
omimetic agent has a success rate between 40% and sidered as the risk of permanent erectile dysfunction
80%. is high and as an implant can be difficult to place
later due to extensive fibrosis of the corpora caver-
If sympathomimetic drugs do not resolve the situa- nosa.
tion, surgical shunting should be performed. The
idea is to shunt blood from the corpus cavernosum Treatment of stuttering priapism include self-
into either the corpus spongiosum or the systemic administration of phenylephrine injection in highly
circulation. Distal shunts (Winter, Ebbehøj, as compliant and reliable patients. Antiandrogens,
shown in Figure 10, or al-Ghorab) should be per- estrogens and gonadotropin-releasing hormone
formed first, followed by proximal shunts (Quack- analogs have been given successfully for prophy-
les or Grayhack, as shown in Figure 11). Success laxis, but the literature on topic is limited and there

CHAPTER 28: ERECTILE DYSFUNCTION, PEYRONIE’S DISEASE AND PRIAPISM 915


Figure 11

Quackles shunt (A.) and Grayhack shunt (B.)

A.

B.
Proximal cavernosal-spongiosal shunts, utilized if distal shunts fail to relieve the priapism

From Bochinski DJ, Deng DY, Lue TF. The treatment of priapism—when and how? Int J Impot Res. 2003;
15 (suppl 5):S86-S90.

916 EDUCATIONAL REVIEW MANUAL IN UROLOGY


are significant adverse effects associated with these ligation of the offending blood supply gives a higher
treatments. Recent basic science studies have linked rate of ED.
the occurrence of stuttering priapism to dysregula-
tion of PDE5 in penile tissue. In a study by Burnett The AUA Guidelines algorithm for the treatment of
et al, sildenafil and tadalafil were shown to effec- priapism is shown in Figure 13.
tively reduce the occurrence of priapism in 6 out of 7
patients. PDE5 inhibitors thus represent a poten- Limited studies from Burnett suggest a novel use for
tially safe and attractive treatment of stuttering pri- phosphodiesterase type 5 inhibitors in the prevention
apism of recurrent, or “stuttering,” priapism. The proposed
action is via altered signaling in the nitric oxide path-
Men with nonischemic priapism can be followed way, which is corrected by the administration of the
over time for resolution of the problem. Angioem- PDE5 medication, resulting in relaxation of the
bolization with autologous clot can give a temporary smooth muscle and subsequent correction of the pri-
cessation of blood flow to break the cycle and fix the apism. Burnett’s case reports demonstrated success
problem. Permanent ED following clot emboliza- with tadalafil 5 mg daily, and also sildenafil citrate at
tion is uncommon. Permanent angioembolization or 50 mg daily.

Figure 12

Angioembolization of a case of high-flow priapism with autologous clot

A. Initial contrast injection shows pseudoaneurysm of the cavernosal artery from trauma
B. After autologous clot is injected, there is no longer flow from the injured vessel. This patient regained his
potency after his treatment

From Park JK, Jeong YB, Han YM. Recanalization of embolized cavernosal artery: restoring potency in the patient with
high flow priapism. J Urol. 2001;165:2002-2003.

CHAPTER 28: ERECTILE DYSFUNCTION, PEYRONIE’S DISEASE AND PRIAPISM 917


Figure 13

AUA Guidelines Panel algorithm for the treatment of priapism

From: Montague DK, Jarow J, Broderick GA, et al. American Urological Association guideline on the management of
priapism. J Urol. 2003;170:1318-1324.

918 EDUCATIONAL REVIEW MANUAL IN UROLOGY


3. Questions

1. The nitric oxide/cyclic GMP (NO/cGMP) 4. All of the following characteristics in a patient
system in the penis is of the utmost importance with a history of erectile dysfunction suggest a
in the generation of penile erection. cGMP pro- primary psychogenic problem, except:
duced by this cascade is inactivated by which of
the following? A. Sudden onset
(Note: NOS = nitric oxide synthase;
PDE = phosphodiesterase) B. Young age

A. PDE 2, 3 and 4 C. Sleep erections present

B. PDE 5 D. Varying degree of dysfunction

C. Norepinephrine E. Orgasm is preserved

D. Neuronal NOS
5. Which of the following agents cause partial
E. Endothelial NOS inhibition of PDE6?

A. Sildenafil
2. Which of the following statements is the most
accurate regarding the source of the blood that B. Tadalafil
results in erection of the corpora cavernosa?
C. Vardenafil
A. The blood supply comes from the
cavernosal (deep) penile arteries D. Prostaglandin E1

B. The blood supply comes from the deep E. Apomorphine


dorsal arteries

C. The blood supply comes from both the 6. Which of the following erectile dysfunction
cavernosal and deep dorsal arteries treatments results in the highest satisfaction
rates?
D. The blood supply does not come from the
cavernosal or deep dorsal arteries A. Sildenafil

B. Intraurethral prostaglandin E1
3. Which of the following is the most common
sexual adverse effect of selective serotonin C. Penile injection therapy with prostaglandin
reuptake inhibitors (SSRIs)? E1

A. Decreased libido D. Vacuum constriction device

B. Premature ejaculation E. Inflatable penile prosthesis

C. Erectile dysfunction

D. Anorgasmia

E. Orchialgia

CHAPTER 28: ERECTILE DYSFUNCTION, PEYRONIE’S DISEASE AND PRIAPISM 919


7. The penoscrotal approach for inflatable penile B. Nocturnal penile tumescence study
prosthesis placement offers which of the fol-
lowing advantages over the infrapubic C. Duplex Doppler ultrasound of the penile
approach? vessels

A. Safer reservoir placement D. Biothesiometry

B. Preservation of glans tumescence E. Penile-brachial index

C. Less risk of penile contracture


10. Which of the following injectable agents is
D. Less risk of penile sensory loss recommended by the AUA Guidelines on pri-
apism, due to its pure alpha-adrenergic effect
E. Larger girth implant is possible and lack of secondary neurotransmitter release?

A. Epinephrine
8. A man presents with Peyronie’s Disease and
very significant penile shortening. Intercourse B. Phenylephrine
is impossible due to a 90-degree dorsal defor-
mity. He has firm erections, but is quite upset C. Metaraminol
with how short the penis has become. Length
preservation is the primary goal of the patient. D. Norepinephrine
Which of the following procedures should be
avoided to try to help achieve his goals? E. Dopamine

A. Penile plication procedure (e.g., Nesbit)

B. Penile prosthesis placement Answers

C. Penile prosthesis placement with a molding 1. B


procedure
2. A
D. Plaque excision with dermal graft
3. D
E. Plaque incision with pericardial graft
4. E

9. A 48-year-old man present with Peyronie’s Dis- 5. A


ease with a 90-degree dorsal deformity. Erec-
tions are perfectly firm by history. He under- 6. E
goes a plaque incision and grafting, with eleva-
tion of the neurovascular bundle. Sensation 7. D
returns to normal, but he is completely unable
to achieve erection following the surgery. 8. A
Which of the following preoperative tests might
have predicted this complication from surgery? 9. C

A. Serum testosterone level 10. B

920 EDUCATIONAL REVIEW MANUAL IN UROLOGY


4. Further Reading

AUA Guidelines available online: Montague DK, Jarow J, Broderick GA, et al. Ameri-
can Urological Association guideline on the man-
Management of Erectile Dysfunction: agement of priapism. J Urol. 2003;170:1318-1324.
http://www.auanet.org/content/clinical-practice-
guidelines/clinical-guidelines.cfm?sub=ed Mulhall JP, Schiff J, Guhring P. An analysis of the
natural history of Peyronie’s disease. J Urol.
Management of Priapism: 2006;175:2115-2118.
http://www.auanet.org/content/clinical-practice-
guidelines/clinical-guidelines.cfm?sub=priapism Porst H, Rajfer J, Casabé A, et al. Long-term safety
and efficacy of tadalafil 5 mg dosed once daily in
Burnett AL, Bivalacqua TJ, Champion HC, Musicki men with erectile dysfunction.J Sex Med.
B. Feasibility of the use of phosphodiesterase type 5 2008;5(9):2160-2169.
inhibitors in a pharmacologic prevention program
for recurrent priapism. J Sex Med. 2006;3(6): Selvin E, Burnett AL, Platz EA. Prevalence and risk
1077-1084. factors for erectile dysfunction in the US. Am J Med.
2007;120:151-157.
Burnett AL, Bivalacqua TJ, Champion HC, Musicki
B. Long-term oral phosphodiesterase 5 inhibitor Wilson SK, Delk JR, Salem EA, Cleves MA. Long-
therapy alleviates recurrent priapism. Urology. termsurvival of inflatable penile prostheses: single
2006;67(5):1043-1048. surgical group experience with 2,384 first-time
implants spanning two decades. J Sex Med.
Corona G, Lee DM, Forti G, et al. Age-related 2007;4:1074–1079.
changes in general and sexual health in middle-aged
and older men: results from the European Male
Ageing Study (EMAS). J Sex Med. 2010;7:1362-
1380.

Francis SH, Corbin JD. Phosphodiesterase-5 inhibi-


tion: the molecular biology of erectile function and
dysfunction. Urol Clin North Am. 2005;32:419-
429.

Gades NM, Jacobson DJ, McGree ME, et al. Longi-


tudinal evaluation of sexual function in a male
cohort: the Olmsted county study of urinary symp-
toms and health status among men. J Sex Med.
2009;6:2455-2466.

Gholami SS, Gonzalez-Cadavid NF, Lin CS, Rajfer


J, Lue TF. Peyronie’s disease: a review. J Urol.
2003;169:1234-1241.

Greenfield JM, Levine LA. Peyronie’s disease: eti-


ology, epidemiology and medical treatment. Urol
Clin North Am. 2005;32:469-478.

Lue TF. Erectile dysfunction. N Engl J Med.


2000;342:1802-1813.

CHAPTER 28: ERECTILE DYSFUNCTION, PEYRONIE’S DISEASE AND PRIAPISM 921


922 EDUCATIONAL REVIEW MANUAL IN UROLOGY
Chapter 29:
Interstitial Cystitis/
Bladder Pain Syndrome
(IC/PBS)
Michel Pontari, MD

Contents

1. Definition

2. Epidemiology

3. Etiology

4. Pathology

5. Evaluation

6. References

7. Questions

CHAPTER 28: ERECTILE DYSFUNCTION, PEYRONIE’S DISEASE AND PRIAPISM 923


1. Definition 2. Epidemiology

Chronic pelvic pain, pressure or discomfort per-


ceived to be related to the urinary bladder, accompa-
Prevalence

nied by at least 1 other urinary symptom, such as It is estimated that 3.3–7.9 million women in US
persistent urge to void or frequency1 have symptoms of IC/PBS, but only 10% of these
may have a formal diagnosis.2 Also occurs in men at
a. Present for at least 6 weeks half the prevalence of women.3

b. Pain with bladder filling and/or relieved by


bladder emptying
Associated Conditions

• High association with other chronic pain syn-


c. Must rule out other conditions/diseases such as dromes, including fibromyalgia, chronic fatigue
infection which can account for the symptoms syndrome and irritable bowel syndrome 4, 5

• Markedly higher incidence of inflammatory bowel


disease in patients with IC/PBS6

• Genetic association of thyroid disease, panic


attacks and IC/PBS7

• Psychological dysfunction including depression,


anxiety, stress and catastrophizing8

924 EDUCATIONAL REVIEW ANUAL IN UROLOGY


3. Etiology 4. Pathology

IC/PBS is considered to be a multifactor condition • There is no pathognomonic histology for IC/PBS.


with the common endpoint phenotype of pain The diagnosis is made clinically. Biopsy can be
attributed to the bladder, possibly caused by several used to rule out other conditions, such as cancer. It
factors. Proposed etiologic factors include epithelial is not uncommon for bladder biopsies in patients
dysfunction, mast cell activation and neurogenic with symptoms of IC/PBS to have normal bladder
inflammation9 biopsies without inflammation12

• May be preceded by UTI. UTI present at onset of • Glomerulations are not specific for IC/PBS. They
disease in 18%–36% of cases10 can be seen with radiation therapy, defunctional-
ized bladders, after chemotherapy, or even in nor-
• Some evidence for IC/PBS as bladder manifesta- mal bladders after hydrodistention13
tion of a central pain syndrome11

CHAPTER 29: INTERSTITIAL CYSTITIS/BLADDER PAIN SYNDROME (IC/PBS) 925


5. Evaluation

Physical examination: should include both abdomi-


nal pelvic exams. Pelvic floor should be palpated
Overview

IC/PBS is a diagnosis of exclusion. Other disorders for tender points. Brief neurologic exam should be
that must be ruled out include common conditions, performed.
such as urinary tract infection, bladder cancer and
especially carcinoma in situ, lower urinary tract
stones, urethral diverticulum or any diagnosable
Post-Void Residual Urine

condition that can cause similar symptoms. Laboratory:

1. Urinalysis and urine culture. Urine culture should


be considered even for negative urinalysis to
History

Urologic symptoms: symptom duration, number of detect lower levels of bacteria that are clinically
voids per day, location, character and severity of significant.14 Evaluate microhematuria
pain. Baseline assessment of urinary symptoms
should be obtained in order to measure subsequent 2. Consider cytology for dysuria, smoking history
treatment effects. At least a 1-day voiding diary is and/or associated microhematuria.
recommended. Self-report instruments include the
O’Leary-Sant symptom and problem questionnaire, 3. TSH with a history of panic attacks7
and the pelvic pain and urgency/frequency (PUF)
questionnaire. Pain intensity can be evaluated using Imaging:
a 10-point visual analog scale, or pain checklist
such as McGill pain questionnaire. 1. Consider CT for evaluation of abdominal pain

Non-urologic symptoms: must assess for involve- 2. Consider pelvic MRI if urethral diverticulum is
ment of other organ systems that may be present as suspected on physical exam
part of a systemic pain syndrome. Identification of
problems within these systems should prompt refer- 3. Consider MRI of lumbar and sacral spine if his-
ral to a specialist in that field. Every patient with tory or neuro exam suggest vertebral disk dis-
IC/PBS may at some point need referral to gastroen- ease
terology, neurology, rheumatology, gynecology,
pain clinics or other specialists. Cystoscopy and/or urodynamics: used when the
diagnosis is in doubt. Cystoscopy can help with
• Neurologic symptoms including migraine diagnosis of cancer, stones and diverticulum in
headaches, numbness or tingling in the legs to addition to the presence or absence of Hunner’s
suggest vertebral disk disease, and/or symptoms ulcer. There are no characteristic cystoscopic find-
suggestive of neurologic disease such as MS ings except for the Hunner’s ulcer, present in a small
percentage of patients. There also are no pathog-
• Gastroenterological symptoms: IBS symptoms nomonic findings on urodynamics. Specific indica-
such as constipation, diarrhea or both, fecal tions for urodynamics include excluding bladder
urgency, fecal incontinence, or exacerbation of the outlet obstruction in both sexes, and diagnosis of
voiding symptoms or pain with bowel movement poor bladder contraction. Urodynamics are not used
to regularly establish a diagnosis of IC/PBS.
• Rheumatology: fibromyalgia is characterized by
multiple areas of pain throughout the body. Signif- Not recommended forevaluation:
icant fatigue along with pain may be characteristic
of chronic fatigue syndrome. • Potassium sensitivity test15

• Gynecology: abnormal menstrual cycles, vaginal • Cystoscopy with hydrodistension—can be used


discharge for treatment, but not used to make a diagnosis

926 EDUCATIONAL REVIEW ANUAL IN UROLOGY


Treatment: per AUA guidelines14 Second-line treatments

First-line treatment: • Manual physical therapy techniques. May treat


Should be performed on all patients pelvic, abdominal and/or hip muscular trigger
points, lengthen muscle contractures and release
• Patient education on normal bladder function, scars. Note: pelvic floor strengthening exercises
what is known and not known about IC/PBS and (Kegel exercises) should be avoided
that no single treatment has been found to be effec-
tive for a majority of patients. Should discuss that • Multimodal pain management approaches includ-
symptom control may require trials of multiple ing pharmacologic, stress management, and physi-
therapies cal therapy if available should be initiated

• Self-care practices and behavioral modification • Second-line oral medications

* Altering urine volume status by hydration or * Amitriptyline: Start at 25 mg PO qhs and


fluid restriction titrate up to 75–100 mg as tolerated. In ran-
domized placebo-controlled trial, significant
* Trial of avoiding certain foods if they seem to effect compared to placebo was not seen until
make symptoms worse. In a study of women 50-mg dose was reached17
with IC/PBS, most frequently reported and
most bothersome foods and drinks were cof- * Cimetidine: small trials have demonstrated
fee, tea, soda, alcoholic beverages, citrus efficacy.20 300 mg PO BID or 200 mg PO TID
fruits and juices, artificial sweeteners and hot
pepper.16 Note: not every person has the same * Hydroxyzine: limited data. Doses range from
response to foods, so restrictive diets limiting 25–75 mg per day21
all these products are not needed in all patients
* Pentosan polysulfate 100 mg PO TID.
* As part of a placebo-controlled trial, 45% of Originally proposed to repair “leaky”
patients with IC/PBS assigned to a standard- bladder epithelium22
ized education and behavioral modification
program combining education on the disease, • Second-line intravesical treatments
management of stress, management of fluid
intake, food triggers and bladder training/urge * DMSO.23 Weekly instillation for 6 weeks.
suppression were moderately or markedly
improved17 * Heparin: single center showed 80% relief
in 1 month24
• Stress management practices to improve coping
techniques and manage stress-induced symptom * Lidocaine: alkalinized lidocaine was
exacerbations superior to placebo in multicenter trial25

Patients with IC/PBS demonstrate greater sympa- Third-line treatments


thetic activity in response to stress compared to
those without the condition.18 Stress has been * Cystoscopy under anesthesia with short dura-
shown to exacerbate symptoms in patients with tion, low pressure hydrodistension; if first-
IC/PBS, but does not change pain or voiding pat- and second-line treatments have been ineffec-
terns in controls19 tive

CHAPTER 29: INTERSTITIAL CYSTITIS/BLADDER PAIN SYNDROME (IC/PBS) 927


Low pressure is 60–80 cm H2O, and short duration Sixth-line treatment
is <10 minutes. If Hunner’s ulcer is found, then this
is treated directly. Can determine capacity under • Major surgery such as substitution cystoplasty, uri-
anesthesia (capacity at max pressure of distension). nary diversion with or without cystectomy
Low capacity under anesthesia may indicate fibro-
sis. Beneficial effect of hydrodistension appears to Patients must be counseled that pain relief is not
be on the order of 3 months26, 27 guaranteed. Symptoms can persist even with the
bladder out. Patients who respond best appear to be
• Fulguration if Hunner’s ulcer is present: laser or those with classic IC/PBS, i.e., those with Hunner’s
electrocautery, and/or injection of triamcinolone ulcers,33 small bladder capacity under anesthesia
into ulcer28 and absence of neuropathic pain.

Patients should be counseled that retreatment may Treatments that should not be offered: treatments
be necessary. that appear to lack efficacy and/or appear to have
unacceptable adverse events/side effects.
Fourth-line treatment
• Long-term oral antibiotic administration
• Trial of neurostimulation
• Intravesical bacillus Calmette-Guerin (BCG)
• Neuromodulation is not FDA approved for
IC/PBS, but many patients will have urgency fre- • Intravesical instillation of resiniferatoxin
quency. Success of 72% in patients with IC/PBS
reported over median follow-up of 61 months 29 • High pressure long-duration hydrodistension

Fifth-line treatment • Systemic (oral) long-term glucocorticoid adminis-


tration
• Cyclosprorine A orally
Multimodal approach to treatment of IC/PBS
• Some studies have shown continued efficacy with
follow-up out to 5 years.30 However, must be The phenotype of patients with IC/PBS usually
weighed against significant risk with CSA, used as encompasses more than simple discomfort with
immunosuppression in transplant patients, includ- bladder filling. A classification system has been
ing nephrotoxicity proposed to identify clinically relevant aspects of
patients, both men and women with pelvic pain.
• Intradetrusor Botulinum toxin A (BTX-A). Must This system has been called UPOINT as a
discuss the possibility of post treatment intermit- mnemonic for the areas described34:
tent self-catheterization
U Urinary symptoms: frequency, urgency, noc-
• Patient must be counseled that intermittent self- turia, incontinence, dysuria
catheterization may be needed after the procedure.
Reported incidence is 8%–10% requiring post-op P Psychosocial: symptoms of depression, catas-
catheterization at some point.31 Results have trophizing or problems with social interactions
reported duration of beneficial effects from Botox
injections out to 24 months32 O Organ specific: pain with bladder filling,
relieved on emptying, Hunner’s ulcers, inflam-
mation on bladder biopsy

928 EDUCATIONAL REVIEW ANUAL IN UROLOGY


6. References

I Infection: significant bacteriuria with typical 1. Hanno P, Lin A, Nordling J, et al. Bladder Pain
uropathogenic bacteria in the previous 2 years Syndrome Committee of the International Con-
associated with exacerbation of baseline symp- sultation on Incontinence. Neurourol Urodyn.
toms and return to baseline after antimicrobial 2010;29(1):191-198.
therapy
2. Berry SH, Elliott MN, Suttorp M, et al. Preva-
N Neurologic (neurogenic pain): concurrent diag- lence of symptoms of bladder pain syn-
nosis of IBS, fibromyalgia, chronic fatigue syn- drome/interstitial cystitis among adult females
drome, vulvodynia or other contain that suggest in the United States. J Urol. 2011;186(2):540-
neural upregulation 544.

T Tenderness: pelvic floor or lower abdominal 3. Link CL, Pulliam SJ, Hanno PM, et al. Preva-
tenderness, including trigger points on abdomi- lence and psychosocial correlates of symptoms
nal and pelvic examinations suggestive of painful bladder syndrome: results
from the Boston area community health survey.
The number of domains positive correlates with J Urol. 2008;180(2):599-606.
symptom severity.35 Although not completely vali-
dated, this scheme does appear to be useful to help 4. Nickel JC, Tripp DA, Pontari M, et al. Intersti-
the clinician identify potential areas to target for tial cystitis/painful bladder syndrome and asso-
therapy. Treatment is simultaneous for all domains ciated medical conditions with an emphasis on
identified, not sequential. A multimodal approach irritable bowel syndrome, fibromyalgia and
increases treatment success in chronic pain syn- chronic fatigue syndrome. J Urol. 2010;184(4):
dromes.36 1358-1363.

5. Rodríguez MA, Afari N, Buchwald DS,


National Institute of Diabetes and Digestive and
Kidney Diseases Working Group on Urological
Chronic Pelvic Pain. Evidence for overlap
between urological and nonurological unex-
plained clinical conditions. J Urol.
2009;182(5):2123-2131.

6. Alagiri M, Chottiner S, Ratner V, Slade D,


Hanno PM. Interstitial cystitis: unexplained
associations with other chronic disease and pain
syndromes. Urology. May 1997;49(5A
Suppl):52-57.

7. Weissman MM, Gross R, Fyer A, et al. Intersti-


tial cystitis and panic disorder: a potential
genetic syndrome. Arch Gen Psychiatry.
2004;61(3):273-279.

8. Nickel JC, Tripp DA, Pontari M, et al. Psy-


chosocial phenotyping in women with intersti-
tial cystitis/painful bladder syndrome: a case
control study. J Urol. Jan 2010;183(1):167-172.

CHAPTER 29: INTERSTITIAL CYSTITIS/BLADDER PAIN SYNDROME (IC/PBS) 929


9. Hanno PM. Bladder Pain Syndrome (Interstitial 19. Rothrock NE, Lutgendorf SK, Kreder KJ,
Cystitis) and Related Disorders. In: Wein AJ, Ratliff T, Zimmerman B. Stress and symptoms
Kavoussi LR, Novick AC, Partin AW, Peters in patients with interstitial cystitis: a life stress
CA, eds. Campbell-Walsh Urology. Vol 1. 10th model. Urology. 2001;57(3):422-427.
ed. Philadelphia, PA: Elsevier-Saunders; 2012.
20. Thilagarajah R, Witherow RO, Walker MM.
10. Warren JW, Brown V, Jacobs S, Horne L, Lan- Oral cimetidine gives effective symptom relief
genberg P, Greenberg P. Urinary tract infection in painful bladder disease: a prospective, ran-
and inflammation at onset of interstitial cysti- domized, double-blind placebo-controlled trial.
tis/painful bladder syndrome. Urology. BJU Intl. 2001;87(3):207-212.
2008;71(6):1085-1090.
21. Sant GR, Propert KJ, Hanno PM, et al. A pilot
11. Fitzgerald MP, Koch D, Senka J. Visceral and clinical trial of oral pentosan polysulfate and
cutaneous sensory testing in patients with oral hydroxyzine in patients with interstitial
painful bladder syndrome. Neurourol Urodyn. cystitis. J Urol. 2003;170(3):810-815.
2005;24(7):627-632.
22. Mulholland SG, Hanno P, Parsons CL, Sant GR,
12. Wyndaele JJ, Van Dyck J, Toussaint N. Cys- Staskin DR. Pentosan polysulfate sodium for
toscopy and bladder biopsies in patients with therapy of interstitial cystitis. A double-blind
bladder pain syndrome carried out following placebo-controlled clinical study. Urology.
Scand J Urol Nephrol. 2009;43(6):471-475. 1990;35:552-558.

13. Waxman JA, Sulak PJ, Kuehl TJ. Cystoscopic 23. Rössberger J, Fall M, Peeker R. Critical
findings consistent with interstitial cystitis in appraisal of dimethyl sulfoxide treatment for
normal women undergoing tubal ligation. interstitial cystitis: discomfort, side-effects and
J Urol. 1998;160(5):1663-1667. treatment outcome. Scand J Urol Nephrol.
2005;39(1):73-77.
14. Hanno PM, Burks DA, Clemens JQ, et al. AUA
guideline for the diagnosis and treatment of 24. Parsons CL. Successful downregulation of
interstitial cystitis/bladder pain syndrome. bladder sensory nerves with combination of
J Urol. 2011;185(6):2162-2170. heparin and alkalinized lidocaine in patients
with interstitial cystitis. Urology. 2005;65(1):
15. Hanno P. Potassium sensitivity test for painful 45-48.
bladder syndrome/interstitial cystitis: con.
J Urol. 2009;182(2):431-432. 25. Nickel JC, Moldwin R, Lee S, Davis EL, Henry
RA, Wyllie MG. Intravesical alkalinized lido-
16. Shorter B, Lesser M, Moldwin RM, Kushner L. caine (PSD597) offers sustained relief from
Effect of comestibles on symptoms of intersti- symptoms of interstitial cystitis and painful
tial cystitis. J Urol. 2007;178(1):145-152. bladder syndrome. BJU Int. 2009;103(7):910-
918.
17. Foster HE Jr, Hanno PM, Nickel JC, et al. Effect
of amitriptyline on symptoms in treatment 26. Cole EE, Scarpero HM, Dmochowski RR. Are
naïve patients with interstitial cystitis/painful patient symptoms predictive of the diagnostic
bladder syndrome. J Urol. 2010;183(5):1853- and/or therapeutic value of hydrodistention?
1858. Neurourol Urodyn. 2005;24(7):638-642.

18. Lutgendorf SK, Latini JM, Rothrock N, Zim-


merman MB, Kreder KJ Jr. Autonomic
response to stress in interstitial cystitis. J Urol.
2004;172(1):227-231.

930 EDUCATIONAL REVIEW ANUAL IN UROLOGY


27. Erickson DR, Kunselman AR, Bentley CM, et 35. Nickel JC, Shoskes D, Irvine-Bird K. Clinical
al. Changes in urine markers and symptoms phenotyping of women with interstitial cysti-
after bladder distention for interstitial cystitis. tis/painful bladder syndrome: a key to classifi-
J Urol. 2007;177(2):556-560. cation and potentially improved management.
J Urol. 2009;182(1):155-160.
28. Cox M, Klutke JJ, Klutke CG. Assessment of
patient outcomes following submucosal injec- 36. Shoskes DA, Nickel JC, Kattan MW. Phenotyp-
tion of triamcinolone for treatment of Hunner's ically directed multimodal therapy for chronic
ulcer subtype interstitial cystitis. Can J Urol. prostatitis/chronic pelvic pain syndrome: a
2009;16(2):4536-4540. prospective study using UPOINT. Urology.
2010;75(6):1249-1253.
29. Gajewski JB, Al-Zahrani AA. The long-term
efficacy of sacral neuromodulation in the man-
agement of intractable cases of bladder pain
syndrome: 14 years of experience in one centre.
BJU Int. 2011;107(8):1258-1264.

30. Sairanen J, Forsell T, Ruutu M. Long-term out-


come of patients with interstitial cystitis treated
with low dose cyclosporine A. J Urol. 2004;171
(6 Pt 1):2138-2141.

31. Tirumuru S, Al-Kurdi D, Latthe P. Intravesical


botulinum toxin A injections in the treatment of
painful bladder syndrome/interstitial cystitis: a
systematic review. Int Urogynecol J. 2010;
21(10):1285-1300.

32. Kuo HC, Chancellor MB. Comparison of


intravesical botulinum toxin type A injections
plus hydrodistention with hydrodistention alone
for the treatment of refractory interstitial cysti-
tis/painful bladder syndrome. BJU Int.
2009;104(5):657-661.

33. Rössberger J, Fall M, Jonsson O, Peeker R.


Long-term results of reconstructive surgery in
patients with bladder pain syndrome/interstitial
cystitis: subtyping is imperative. Urology.
2007;70(4):638-642.

34. Shoskes DA, Nickel JC, Rackley RR, Pontari


MA. Clinical phenotyping in chronic prostati-
tis/chronic pelvic pain syndrome and interstitial
cystitis: a management strategy for urologic
chronic pelvic pain syndromes. Prostate Can-
cer Prostatic Dis. 2009;12(2):177-183.

CHAPTER 29: INTERSTITIAL CYSTITIS/BLADDER PAIN SYNDROME (IC/PBS) 931


Questions

1. In the AUA IC/PBS guidelines, under sec- 4. The pathognomonic histologic changes on
ond tier treatments, it is recommended to which to make a diagnosis of IC/PBS
avoid which treatment? include:

A. Manual physical therapy A. Mast cells

B. Trigger point release B. Eosinophils

C. Pelvic floor strengthening exercises C. Missing glycosaminoglycan layer


(Kegel exercises)
D. Glomerulations
D. Amitriptyline
E. Diagnosis is not based on histology
E. Pentosan polysulfate
5. Which of the following laboratory tests
2. What is the minimum length of symptoms should be performed in all patients?
to be able to make a diagnosis of IC/PBS ?
A. Urine cytology
A. 2 weeks
B. TSH
B. 3 weeks
C. Urinalysis and urine culture
C. 4 weeks
D. BUN and creatinine
D. 6 weeks
E. C-reactive protein
E. 6 months
6. Which of the following tests is not recom-
3. Which of the following comorbid condi- mended for evaluation?
tions has been linked to IC/PBS in genetic
studies? A. Potassium sensitivity test

A. Fibromyalgia B. Cystoscopy

B. Migraine C. Urodynamics

C. Chronic Fatigue Syndrome D. CT scan

D. Sjogren’s syndrome E. Pelvic MRI

E. Panic attacks

932 EDUCATIONAL REVIEW ANUAL IN UROLOGY


7. In the AUA IC/PBS guidelines, all of the 10. Which statement most accurately reflects
following should be avoided except: the effect of cystectomy on pain relief in
patients with IC/PBS?
A. 15 minute hydrodistension
A. Pain always go away
B. Intravesical BCG
B. Pain never goes away
C. Oral corticosteroids
C. Pain only goes away with urinary
D. Oral cyclosporine diversion without cystectomy

E. Long term antibiotics D. Pain can persist after cystectomy

8. The recommended treatment for a patient E. Response is better in patients with a


with a Hunner’s ulcer who has failed first large preoperative bladder capacity
and second line therapy is:

A. Cystectomy

B. Direct treatment with fulguration of


ulcer or submucosal injection of
steroids

C. Intravesical formalin

D. High pressure hydrodistension

E. Intravesical resiniferatoxin

9. One of the potential side effects of intraves-


ical botulinum toxin-A (BTX-A) injections
that should be discussed before the proce-
dure is:

A. Risk of urinary retention and need for


self-catheterization

B. Hematuria

C. Development of Guillain-Barré
syndrome

D. Risk of developing bladder cancer

E. Bladder necrosis

CHAPTER 29: INTERSTITIAL CYSTITIS/BLADDER PAIN SYNDROME (IC/PBS) 901


Answers

1. C.
Pelvic floor strengthening exercises (Kegel exer-
cises).

2. D.
6 weeks.

3. E.
Panic attacks.

4. E.
Diagnosis is not based on histology.

5. C.
Urinalysis and urine culture.

6. A.
Potassium sensitivity test.

7. D.
Oral cyclosporine.

8. B.
Direct treatment with fulguration of ulcer or submu-
cosal injection of steroids

9. A.
Risk of urinary retention and need for self-catheteri-
zation.

10. D.
Pain can persist after cystectomy.

934 EDUCATIONAL REVIEW ANUAL IN UROLOGY


Chapter 30:
Pediatric Urological
Incontinence and
Voiding Dysfunction
Martin A. Koyle, MD, FAAP, FACS

Contents

1. The Normal Bladder

2. The Wet Child ICCS Terminology

3 Hinman’s Syndrome

4. Nocturnal Enuresis

5. Neurogenic Voiding Dysfunction

6. Management of Neurogenic Voiding


Dysfunction: Medical and Surgical

7. Issues in Augmentation

8. Late Issues

9. Conclusion

10. References

11. Questions

12. Answers

CHAPTER 30: PEDIATRIC UROLOGICAL INCONTINENCE & VOIDING DYSFUNCTION 935


1. The Normal Bladder

A. The bladder has 2 basic responsibilities C. Urodynamics (UDS) in Children


under normal circumstances: to store and
to empty. 1. UDS is an art and it is important to recognize
that in infants in particular, there is discoordi-
1. Store nated voiding resulting in elevated detrusor
pressures!
a. Formulas for normal capacity for age:
Children <1 year old: weight (kg) X 7 = a. Males <1 1 year old P det as high as 118 cm
capacity (cc) H2O and females as high as 75 cm H2O =
Children (age [years] + 2) x 30 = staccato voiding associated with transient
capacity (cc) DSD

b. Sense distension 2. Remember these UDS tips when children are


being evaluated:
c. Accommodate increasing volume without
change in pressure = remain relaxed when a. Slow fill <10% predicted cap/minute!
not voiding
b. Fluid should be body temperature!
2. Emptying
c. Catheter size can affect LPP!
a. Initiate and sustain contraction
3. BLPP >40 cm if sustained is detrimental to
b. Empty to completion the UVJ competence and ultimately to the
upper tract
B. Development of Control
a. 68% developed VUR
1. Newborns are thought to practice reflex void-
ing, which occurs when bladder capacity is b. 81% developed dilation of the upper tracts
reached

2. Central maturation facilitates storage beyond Figure 1


FBC – Functional Bladder Capacity
Bladder Pressure
3. Bladder capacity gradually increases over
time, which results in voiding that occurs:

a. Hourly- 2–4 weeks old

b. 8–10x daily- 3-year-old

c. 4–6x daily- 12-year-old

It has been shown that a a LPP >40 cm H20 is detrimental to


the UVJ competence and ultimately to the upper tracts. High
pressures are akin to functional obstruction!

936 EDUCATIONAL REVIEW MANUAL IN UROLOGY


D. Bladder/Bowel Control: Figure 2
There is a Routine Sequence
Anatomical incontinence
1. Nocturnal bowel control… followed by

2. Diurnal bowel control… followed by

3. Diurnal control of voiding… followed by

4. Nocturnal control of voiding

Girls perfect this sequence, in general, at earlier


ages than boys and potty train much faster!

E. Beyond the ABC’s: History and


Physical… ESSENTIAL!!!

Consider all causes (ABC):


Must have a high index of suspicion when child always has
1. ANATOMICAL soggy underwear, despite apparent control that anatomical
issues are involved, especially in females! (ectopic ureter,
female epispadias)
2. BEHAVIORAL
Evaluation of the Wet Child
3. CNS (NEUROGENIC)
Figure 3
4. DEVELOPMENTAL
Schematic approach to the child who is wet.
5. ENDOCRINE/ RENAL

6. FUNCTIONAL

7. GENETIC

8. HABITUAL

9. INFECTION/IRRITATION

Minimal evaluation is necessary, assuming a complete H&P


have not been suggestive of anatomical or neurogenic incon-
tinence. A U/A and, if necessary, a C&S should be done to
assure absence of infection, concentrating defects and dia-
betes..

CHAPTER 30: PEDIATRIC UROLOGICAL INCONTINENCE & VOIDING DYSFUNCTION 937


2. The Wet Child:
ICCS Terminology

A. 2 primary categories: D. Voiding Phase

1. Storage phase 1. Detrusor

2. Voiding phase a. Normal

b. Underactive (poor emptying)


B. Each subcategorized:
c. Acontractile (neurogenic state)
1. Detrusor activity
2. Urethra
2. Urethral and sphincter complex activity
a. Normal
C. Storage Phase
b. Overactive (voluntary pelvic floor, vs
1. Detrusor anatomic, ie, PUV)

a. Normal 1. Dysfunctional voiding is discoordination


of bladder and outlet/voluntary sphincter
b. Overactive
E. Functional Voiding Disorders
1. Unstable (non-neurologic)–urge due to
1. Overactive Bladder (OAB) =
detrusor instability
Urge incontinence
2. Hyperreflexic (neurologic state) 2. Small Capacity Hypertonic Bladder (AVOID
USING TERM: Detrusor Hyperreflexia)
2. Urethra
3. Underactive Bladder (UAB) (AVOID USING
a. Normal TERM: The Infrequent Voider = Lazy Blad-
der Syndrome)
b. Incompetent–leakage despite a quiet
bladder (stress, giggle) 4. Bladder-Sphincter Discoordination

a. Psychogenic Non-Neuropathic Bladder


(Hinman’s Syndrome)

Figure 4. Urodynamic findings in voiding dysfunction.

Category Filling Phase Voiding Phase

Normal No detrusor instability Steady flow; ↓ EMG


Urge before 1/2 EBC No abdonimal straining

OAB Multiple UBC; ↑ EMG Usually normal

DSD Usually normal; EBC normal ↑ EMG; staccato flow;


↑Bladder Pressure; ↑ PVR
UAB ↑Bladder size; no urge Prolinged flow;
↑ Abdominal straining; ↑ PVR
ICCS, 2006 (Modified after Bauer)

938 EDUCATIONAL REVIEW MANUAL IN UROLOGY


F. Overactive Bladder (OAB) H. Enuresis Risoria (Giggle Incontinence)

1. Sudden and unexpected experience of an 1. Involuntary wetting with laughter in young


immediate need to void female (4–8 years) with otherwise normal
elimination habits
2. Not applicable before the attainment of
bladder control or age 5 years, whichever 2. Generally improves through puberty
comes first
3. Methylphenidate (Ritalin) is treatment of
3. Lack of mature central inhibition of detrusor choice for significant cases
contraction
4. Alpha-adrenergic agents
a. Reflex opening bladder neck
a. Phenylpropanolamine
b. Urgency/incontinence
b. Ephedrine
4. Urodynamic definition
c. Ornade
a. Involuntary pressure rises 15 cm
5. Antiepileptics
5. Defenses
I. Daytime Frequency Syndrome
a. Urgency
1. Usually toilet trained male (5–8 years)
b. Squatting, Vincent’s Curtsy
2. Sudden onset frequency
c. Heel into perineum
3. Often associated urgency
d. Squeeze penis
4. Rarely if ever incontinent or UTI
6. Treat with timed voiding, bowel program;
medications are rarely necessary and should 5. Usually sleeps through the night
be used only as last resort
6. Self-limited: 2–6 months
G. Vaginal Voiding
a. No treatment necessary in most cases
1. Classic history of post-void dribbling in a
female 7. Reassurance

2. May have labial adhesions–invariably coinci- 8. Behavior modification


dental but occasionally, cause/effect?
a. Stretch out voiding interval
3. Treatment
9. Anticholinergics, rarely if long-term
a. Pull down panties below knees with
voiding

b. Sit backwards on toilet seat with voiding or


support feet

CHAPTER 30: PEDIATRIC UROLOGICAL INCONTINENCE & VOIDING DYSFUNCTION 939


J. UAB (Lazy Bladder Syndrome or Infrequent K. Constipation & the Urinary Tract
Voider)
1. 234 children with chronic constipation
1. Void 2–4 times/day
a. Daytime incontinence 29%
a. Do not void on awakening
b. Nighttime incontinence 34%
b. Do not void at school, public restrooms
c. UTI’s 11% / VUR 16%
c. First void at home after school
2. Bowel treatment program instituted and
2. Anticipation/memory of dysuria maintained

3. Presentation 3. Results:

a. Asymptomatic bacteriuria a. Constipation relieved 52%

b. Dribbling/nocturnal enuresis b. Daytime dry 89%

c. Constipation/encopresis c. Nighttime dry 63%

1. Treat constipation d. No UTIs

d. Reassurance and timed voiding, double


voiding in some

e. Alpha-adrenergic blockers in recalcitrant


cases

f. Botox?

g. DO not cysto and dilate

940 EDUCATIONAL REVIEW MANUAL IN UROLOGY


3. Hinman’s Syndrome

A. Natural reaction to tighten sphincter in order to F. Therapy of Dysfunctional Elimination/


avoid urinary or fecal incontinence Hinman’s

B. Can become pathological due to Catch 22 phe- 1. Infection control


nomenon of bladder contraction against closed
outlet a. Prophylaxis if indicated

C. Hinman – Allen Syndrome in most severe form 2. Constipation management


but a spectrum of presentations, just like poste-
rior urethral valves! 3. Behavior modification

1. Often dysfunctional environments a. Timed voiding schedule

D. Overachievers, high divorce rate b. Diary/Fluids/4 C’s

E. R/O child abuse 4. Biofeedback therapy?

Figure 5. 5. Pharmacologic management

The conundrum of Hinman’s Syndrome. a. Anticholinergics / alpha blockers-timing?

6. Botox, CIC, surgery?

G. Therapeutic Outcomes

1. No single therapeutic plan is effective

2. Goals to maintain kidneys, urinary continence

a. Disciplined and well-defined approach

b. Adherence basic principles of management

3. VUR when present is almost always sec-


ondary to high pressure; surgical failure and
Hinman’s Syndrome: A Vicious Cycle potential complications are high
What comes first: the chicken or the egg? The problems
and defense mechanisms in the most severe forms of
voiding dysfunction, often termed Hinman’s Syndrome,
often include the bladder and upper tracts. Bowel and
bladder problems along with recurrent UTI, go hand in
hand. The dilemma is that in many cases there are asso-
ciated patient as well as family-based psychosocial
problems. Each problem has the potential to potentiate
the next constituent in the sequence. The underlying
basic problem is hence uncertain.

CHAPTER 30: PEDIATRIC UROLOGICAL INCONTINENCE & VOIDING DYSFUNCTION 941


4. Nocturnal Enuresis 5. Abnormal Bladder in Children

A. Definitions: Figure 6.

1. Primary–never dry- 75% Relationship of the outlet competency to the


bladder itself.
2. Secondary–dry period >6 months- 25%

B. 25% will have diurnal problems & should not be


included

C. Fhx important–45% chance if 1 parent enuretic,


>80% if both

D. 20% of 5-year-olds with annual 15% resolu-


tion–3% college aged

E. ADHD–increases risk and poor responders to Rx

1. Treatment of PNE
In ideal circumstances, both the bladder and sphincter
F. Motivation function normally. At the other extreme is a situation
where there is competent or good sphincter but a bad
G. Alarms (conditioning) –60%–75% response bladder—one that is poorly compliant ±hypercontrac-
tile. In the latter state, the kidneys are at greatest risk of
H. Desmopressin (DDAVP/ sustaining damage.
Desmospray/Desmotabs)–70% response ini-
tially with 30%–50% relapse with cessation
A. Introduction
1. Acts at distal tubule and increases urine osm
1. Primary objective: preserve renal function
2. Give 1–2 hours before hs
a. Eliminate high pressures
3. Hyponatremia associated with nasal spray
only, not oral (may also cause decreased urine b. Prevent secondary changes bladder wall
Na, mild hyperkalemia)
c. Empty bladder regularly -----CIC
I. Imipramine–50%
d. Treat detrusor overactivity ----- drugs
1. Use in older kids only
e. Promote bowel emptying
2. Cardiac toxicity–prolongs PR and QT
intervals f. Prevent UTI

3. OD (>3.5 mg/kg) –dry as a bone, red as a beet B. Spina Bifida (Myelodysplasia)

a) Rx with physostigmine 1. Most common cause of NGB in children

J. Anticholinergics and bladder retention protocols a. 1:1000 births USA


have minimal effect over placebo

942 EDUCATIONAL REVIEW MANUAL IN UROLOGY


Figure 7. 4. Prenatal diagnosis:

The abnormal bladder of childhood. a. Alpha-FP (24 weeks, 3 SD): >70%

b. Combined with US: > 80% sensitive; 99%


specificity

5. C-section delivery reduces risk of paralysis

C. Myelomeningocele

1. At birth >95% normal upper tracts

2. Without proper management: <5 years: > 60%


have upper tract damage

3. Reflux/hydronephrosis/renal scarring

Although the urological goal of prevention of renal injury a. Urological problems:


should always be paramount in the management of the
child with neurogenic incontinence, time issues of qual- i. Incontinence >95%
ity of life become increasingly important and impact
management. ii. UTI >90%

iii. Constipation >80%

b. Incidence decreasing iv. Sexual dysfunction

i. Folic acid supplements v. Fertility problems

ii. Prenatal ultrasound—elective termina- 4. Consideration in management:


tion
a. Hydrocephalus
2. Etiology
i. Severity may reflect level of intelligence:
a. Folic acid deficiency increasing physical and intellectual dis-
abilities tend to go hand in hand
b. Teratogens
b. Mobility
c. Diabetic mother (sacral agenesis)
i. MM above L3 ultimately WC bound in
3. Family history majority: consider issues of transferring
etc, when older
a. 90% negative FHx
c. Spinal deformity
b : 5%–10% if affected sibs
i. Kyphoscoliosis predominant in T-L
defects: can preclude ability to perform
self cath

CHAPTER 30: PEDIATRIC UROLOGICAL INCONTINENCE & VOIDING DYSFUNCTION 943


D. Occult Spinal Dysraphism F. Imperforate Anus/ VACTERRL/ VATER

1. 40% have abnormal bladder / sphincter 1. Etiology:


function
a. Structural anomalies of the urinary tract
b. 55% will have DESD or detrusor hyper- common-solitary kidney in up to 40%
reflexia alone
b. Vertebral anomalies and associated
2. 1/3 with normal neurological exam have abn. neurologic deficits (45%)
UDS
c.Potential damage from surgical correction
3. UDS response with neurosurgical correction, of imperforate anus/fistula
more likely in younger children (80%–90%
compared to 17%–54%) G. Cerebral Palsy

E. Sacral Agenesis 1. 1–2:1000 births

1. Incidence: 0.09%–0.43% 2. Perinatal anoxia or infection

a. Varied spectrum with absence of part or all 3. Most achieve total urinary controls
of sacral vertebral bodies
4. Urinary incontinence
2. Caudal regression syndrome
a. Mainly due to physical limitation and
SA that includes lumbar vertebral anomalies timing to get to the toilet
and involves anorectal and GU systems
b. Detrusor hyperreflexia in 86% with under-
3. Often treated as DES/Hinman’s Syndrome lying bladder pathology

a. Incontinence of urine and feces

b. UTI

c. VUR in >90%, often with abnormal bladder


(tics, trabecs) ± upper tract changes

d. Often subtle findings:

e. Disorders of the lower extremities (S 2–4)


with flat buttocks, hair tufts, dimpling

4. Neurological findings:

a. Don’t correspond with level of bony


abnormality

b. Motor > sensory deficits

c. Variable types of neurogenic bladder dys-


function

944 EDUCATIONAL REVIEW MANUAL IN UROLOGY


6. Management of Neurogenic Voiding
Dysfunction: Medical and Surgical

A. Medical Management of Complex D. Pediatric Reconstruction: Key Points


Incontinence
1. In children: try to preserve bladder, not
1. Nonsurgical management is always first line: divert
CIC, anticholinergics, alpha and beta block-
ers, antibiotics 2. Detubularize and reconfigure bowel: avoid
hour glass!
2. Early surgery: rarely required for failure of
medical management to stabilize or improve a. Intact bowel P- 60–100 cm H2O
upper tract
3. Maintain terminal 10–20 cm distal ileum
a. Detrusor hypertonicity + anatomical or (B12 absorption – megaloblastic anemia,
physiological outflow obstruction= high risk peripheral neuropathy, optic atrophy,
dementia)
3. Later surgery: indicated for social continence
4. Bladder neck closure as last resort only
B. Neonatal vs Childhood Treatment of
Spina Bifida 5. Consider MACE and Mitrofanoff

1. Fewer bladder augmentation needed in those


treated neonatally (11% vs 27%)

2. Psychological benefits

C. Surgical Intervention

1. Last resort when medical therapy fails:

a. Botox, Augmentation ±

b. BN procedure: injection, suspension, sling,


urethral lengthening (Piipi Salle, Kropp),
AUS. Last resort is BN closure

c. Mitrofanoff- Monti-Yang ±

d. Reimplant ±

e. Malone ACE

CHAPTER 30: PEDIATRIC UROLOGICAL INCONTINENCE & VOIDING DYSFUNCTION 945


7. Issues in Augmentation

A. Ileum C. Ileocecal

1. Plus: 1. Plus

a. Availability a. Constant blood supply and excellent


muscle backing
b. Less contractions
b. Inherent antireflux mechanism of IC valve
c. Less mucus than colon
2. Minus
2. Minus
a. Removes distal ileum (B12)
a. Short mesentery
3. In NGB: loss of IC valve associated with
b. Minimal muscle backing to implant severe diarrhea in >10%

c. Vitamin B12 deficiency and diarrhea 4. Hypercontractile (10 X ileum)

d. Hyperchloremic, hypokalemic metabolic 5. Hyperchloremic, hypokalemic metabolic


acidosis! acidosis

B. Sigmoid D. Stomach

1. Plus: 1. Plus

a. Easily mobilized a. Little mucus: least UTI, stones

b. Excellent backing for implants b. Superb backing for implant

2. Minus 2. Minus

a. More mucus and unit contractions a. Hypochloremic, hypokalemic metabolic


in large bowel segments (cecum and alkalosis
sigmoid) than in ileum
b.Hematuria: dysuria syndrome in sensate
b. Higher risk of perforation? (Rink) patients

c. Hyperchloremic, hypokalemic metabolic c. Highest risk of malignancy?


acidosis
d. Requires large incision and tedious mobi-
lization (R. gastroepiploic A pedicle; avoid
antrum and injury to Vagus N)

946 EDUCATIONAL REVIEW MANUAL IN UROLOGY


8. Late Issues in Neurogenic
Patients

E. Jejunum 1. Females: 70%–80% can bear children

1. Plus 2. Males: S1 and below, likely erectile capacity; S1


above: 50% chance (good response to Viagra)
a. Good compliance
a. Poor gonadal function with infertility (primary
2. Minus testicular failure)

a. Poor muscle backing 3. Beware of spinal cord tethering:

b. Hyponatremic, hypochloremic, a. Detethering in infants: 60% normalize UDS


hyperkalemic metabolic acidosis with
hypovolemia b. Detethering older: 27% normalize

c. N, V and lethargy 4. Late Issues: Pregnancy after augmentation

F. Urinary Diversion a. Vascular pedicle shifts laterally and is not


affected
1. Vesicostomy: most common temporary
diversion in children b. Issue of bacteriuria and pregnancy

a. Allows voiding, but bladder questionable c. If BN not reconstructed, C/S not mandated
cycle
d. If C/S: ideally urologist should be present,
b. No data suggesting detrimental long-term especially if catheterizable (Mitrofanoff/
effect on bladder MACE) channels

c. Bacteriuria, stenosis, prolapse

G. Surgical Intervention Pitfalls

1. Lifelong follow-up: kidneys, metabolic,


perforation, stones, DVT, tumor, pregnancy,
cath problems

2. BN procedure alone: watch upper tracts due


to secondary damage from covert bladder
anomaly

3. Metaplasia/malignancy

4. Taking ileocecal valve: potential for


diarrhea, B12

CHAPTER 30: PEDIATRIC UROLOGICAL INCONTINENCE & VOIDING DYSFUNCTION 947


9. Conclusions 10. References

1. The wet child represents a diverse spectrum and 1. Koff SA. Relationship between dysfunctional
often is not cut and dry voiding and reflux. J Urol. 1982;148: 1703-1705.

2. Although many functional forms of incontinence 2. Neveus T, von Gontard A, Hoebeke P, et al. The
create significant anxiety, it is imperative to fol- standardization of terminology of lower urinary
low the ABC’s and exclude to correctable tract function in children and adolescents: report
anomalies from the Standardisation Committee of the Inter-
national Children’s Continence Society. J Urol.
3. Care of the neurogenic patient is lifelong and, 2006;176(1):314-324.
although prevention of renal injury is always the
primary goal, as the patient ages, quality of life 3. McGuire EJ, Woodside JR, Borden TA, Weiss
issues become significant. RM. Prognostic value of urodynamic testing in
myelomeningocele. J Urol. 1981;126:205-209.

4. Koyle MA, Pfister RR, Woo H, Lum GM, Ford


DM, Karrer FM. Management of the lower uri-
nary tract in pediatric transplantation. Semin
Urol. 1994;XII(2):74-83.

5. Koyle MA, Kaji DM, Duque M, Wild J, Galansky


SH. The Malone antegrade continence enema for
neurogenic and structural fecal incontinence and
constipation. J Urol. 1995;154:759-761.

6. Lowe JB, Furness PD 3rd, Barqawi AZ, Koyle


MA. Surgical management of the neuropathic
bladder. Seminars in Ped Surg. 2002;11(2):
120-127.

948 EDUCATIONAL REVIEW MANUAL IN UROLOGY


11. Questions and Answers

1. A 21-year-old woman underwent ileocystoplasty 4. A 7-month-old female is seen for evaluation of a


for neurogenic bladder 4 years ago. During an reported straddle injury. Which factor mandates
emergency cesarean section for dystocia, the vas- an evaluation for sexual abuse?
cular pedicle to the cystoplasty segment is
divided. The next step is: A. Recurrent UTIs

A. Revascularization of the pedicle B. Diurnal enuresis beyond age 6 years

B. Excise ileal patch; immediate ileal C. Infrequent voiding


augmentation
D. Age
C. Excise ileal patch; delayed sigmoid
augmentation E. Encopresis

D. Place suprapubic tube and drain 5. A 16-year-old male with an ileal conduit for 14
years is scheduled for a renal transplant. He had a
E. Observation with follow-up urodynamics PUV treated as an infant in Africa. CMG shows a
60-ml capacity bladder with detrusor overactivity,
2. A 14-year-old female with a neurogenic bladder is and maximum voiding pressure is 50 cm H2O.
incontinent despite an aggressive program of CIC After 5 days of bladder cycling, his bladder capac-
and antimuscarinic medications. Videourody- ity increases to 200 ml. The best management is:
namic evaluation demonstrates a flaccid, large-
capacity bladder and low urethral resistance. The A. Transplant into the existing ileal conduit
best long-term management is CIC and:
B. Bladder augmentation before transplant
A. Periurethral bulking agent
C. Transplant into native bladder
B. Bladder neck tubularization
D. Neobladder construction before transplant
C. An artificial urinary sphincter
E. Indiana pouch construction before transplant
D. A pubovaginal fascial sling
6. A 7-year-old boy with a previously treated PUV
E. A Mitrofanoff procedure and hydronephrosis has nocturnal enuresis and
some daytime wetness. A 24-hour urine collec-
3. The blood supply to the appendix that must be tion shows a volume of 1800 ml. His creatinine is
preserved during an appendicovesicostomy 1.2 mg/dL. The best treatment is:
arises from a branch of which artery?
A. Vasopressin
A. Ileal
B. Salt restriction
B. Anterior cecal
C. Postvoid catheterization
C. Posterior cecal
D. Decreased oral fluid intake
D. Ascending colic
E. Increase frequency of voiding
E. Inferior mesenteric

CHAPTER 30: PEDIATRIC UROLOGICAL INCONTINENCE & VOIDING DYSFUNCTION 949


7. A 17-year-old boy with spina bifida is scheduled 10. UDS evaluation of a healthy 7-year-old female
for bladder augmentation with AUS implantation with no history of UTI and diurnal enuresis
due to incontinence. Infection or erosion are likely will reveal:
more common if AUS implantation is performed:
A. Detrusor overactivity
A. Prior to augmentation
B. Excessive external contraction during
B. Following augmentation bladder filling.

C. Simultaneous with augmentation C. A lax bladder neck reflecting incompetence.

D. With sigmoid enterocystoplasty D. Normal ext sphincter activity during filling


and voiding
E. In an area of previous surgery
E. Flaccid, large-volume bladder
8. A 10-year-old boy with a myelomeningocele
closed at birth has failed antimuscarinic therapy
and CIC for a noncompliant neurogenic bladder.
He has undergone multiple neurosurgical and
orthopedic surgeries uneventfully. He undergoes
a mechanical bowel prep and received preopera-
tive antibiotics. 45 minutes after starting an ente-
rocystoplasty, he develops acute, severe hypoten-
sion. The most likely cause is:

A. Dehydration secondary to bowel prep

B. Sepsis

C. Latex allergy

D. Autonomic dysreflexia

E. Unrecognized iliac venous compression

9. The most important risk factor for progressive


hydronephrosis in patients with myelomeningo-
cele is:

A. High grade VUR

B. Detrusor overactivity

C. Decreased detrusor compliance

D. LPP >40 cm of H2O

E. Striated sphincter dyssynergia

950 EDUCATIONAL REVIEW MANUAL IN UROLOGY


12. Answers

1. E. 7. E.
It is very likely that the augmented segment will The artificial sphincter has become much more reli-
have neovascular supply from the native bladder and able since its original inception. It has been shown
hence the division of the pedicle may create anxiety that although seemingly counterintuitive, success of
but is unlikely to affect viability and function. implantation is not affected in relation to timing of
augmentation or choice of segment. The highest risk
2. D. patients for erosion are those who have had prior
The primary issue in this patient is an incompetent surgeries at the site implantation.
outlet. In the face of a large flaccid bladder, a proce-
dure that increases outlet resistance is ideal. CIC 8. C.
may be done via the urethra or in some cases a Latex allergy is much more common in patients
Mitrofanoff may be beneficial. where latex was used in early and repetitive surgical
procedures. It can be life threatening and must be
3. A. anticipated.
Often in exams, anatomy questions will be asked.
This answer is the ileal, which is the best answer. 9. D.
A LPP >40 cm H2O and maybe even lower is associ-
4. D. ated with the potential for upper tract deterioration.
It is unlikely that a child of this age is ambulatory
and hence straddle injury would be unlikely. 10. D.
Although dysfunctional elimination/bowel bladder Despite her issues related to bowel-bladder dysfunc-
dysfunction may be an associated problem subse- tion, she is a healthy girl and hence likely neurologi-
quently so that any of the other answers could apply cally intact. As such, UDS is likely to be normal.
as well; age of this patient is the tip off.

5. C.
In the patient with valves, although high-pressure
bladders are an early urodynamic finding in most,
with time, the bladder develops myogenic failure
and in this case demonstrates rapid increase in
capacity to acceptable levels with cycling. This
demonstrates that the bladder is safe.

6. E.
This patient likely has chronic kidney disease. Early
signs of renal insufficiency and hydronephrosis alone
can affect urinary concentration and lead to nephro-
genic diabetes insipidus. In such cases, salt and fluid
restriction are not helpful and the kidney is relatively
unresponsive to vasopressin. Hence, if able, the blad-
der should be maintained as empty as possible and
timed voiding is a better option than CIC.

CHAPTER 30: PEDIATRIC UROLOGICAL INCONTINENCE & VOIDING DYSFUNCTION 951

You might also like